You are on page 1of 1377

2022-23

EDITION

Combined Defence Services

ENTRANCE
EXAMINATION

Compiled & Edited by


Arihant ‘Expert Team’

ARIHANT PUBLICATIONS (INDIA) LIMITED


ARIHANT PUBLICATIONS (INDIA) LIMITED
All Rights Reserved

© Publisher
No part of this publication may be re-produced, stored in a retrieval system or by any means,
electronic, mechanical, photocopying, recording, scanning, web or otherwise without the written
permission of the publisher. Arihant has obtained all the information in this book from the sources
believed to be reliable and true. However, Arihant or its editors or authors or illustrators don’t take
any responsibility for the absolute accuracy of any information published and the damage or loss
suffered thereupon.
All disputes subject to Meerut (UP) jurisdiction only.

Administrative & Production Offices


Regd. Office
‘Ramchhaya’ 4577/15, Agarwal Road, Darya Ganj, New Delhi -110002
Tele: 011- 47630600, 43518550

Head Office
Kalindi, TP Nagar, Meerut (UP) - 250002
Tel: 0121-7156203, 7156204

Sales & Support Offices


Agra, Ahmedabad, Bengaluru, Bareilly, Chennai, Delhi, Guwahati,
Hyderabad, Jaipur, Jhansi, Kolkata, Lucknow, Nagpur & Pune.

ISBN 978-93-25798-01-4
PO No : TXT-XX-XXXXXXX-X-XX
Published by Arihant Publications (India) Ltd.
For further information about the books published by Arihant, log on to
www.arihantbooks.com or e-mail at info@arihantbooks.com
Follow us on
CONTENTS
CDS Solved Paper 2021 II 1-41 20. Measurements of Angles and
CDS Solved Paper 2021 I 42-82 Trigonometric Ratios 194-224
CDS Solved Paper 2020 II 3-39 21. Height and Distance 225-236
CDS Solved Paper 2020 I 40-78 22. Lines and Angles 237-247
CDS Solved Paper 2019 II 1-40 23. Triangles 248-271
CDS Solved Paper 2019 I 41-82 24. Quadrilateral and Polygon 272-286
CDS Solved Paper 2018 II 1-37 25. Circle 287-310
26. Area and Perimeter of Plane figures 311-335
MATHEMATICS 27. Surface Area and Volume of solids 336-362
1. Number System 3-19 28. Statistics 363-384
2. Sequence and Series 20-24
3. HCF and LCM of Numbers 25-31 GENERAL ENGLISH
4. Decimal Fractions 32-37 1. Spotting the Errors 387-434
5. Square Roots and Cube Roots 38-46 2. Vocabulary 435-451
6. Time and Distance 47-59 3. Synonyms 452-462
7. Time and Work 60-68 4. Antonyms 463-474
8. Percentage 69-76 5. Idioms and Phrases 475-482
9. Simple Interest 77-82 6. Sentence Completion 483-499
10. Compound Interest 83-91 7. Sentence Improvement 500-514
11. Profit and Loss 92-100 8. Ordering of Words and Sentences 515-543
12. Ratio and Proportion 101-112 9. Comprehension 544-576
13. Logarithm 113-119
14. Algebraic Operations 120-131
GENERAL SCIENCE
15. HCF and LCM of Polynomials 132-138
16. Rational Expressions 139-143
PHYSICS
Ÿ Measurement, Motion, Work,
17. Linear Equations 144-159
Energy and Power 579-586
18. Quadratic Equations and Inequalities 160-182
Ÿ Rotational Motion and Gravitation 587-589
19. Set Theory 183-193
Ÿ Properties of Matter 590-592
Ÿ Heat and Thermodynamics 593-596 Ÿ Man-Made Materials 656-658
Ÿ Oscillations and Waves 596-599 Ÿ Environment and its Pollution 658-659
Ÿ Optics 600-606 Ÿ Practice Exercise 660-681
Ÿ Electric Current 606-609
Ÿ Modern Physics 609-611 BIOLOGY
Ÿ Practice Exercise 612-631 Ÿ Cell : The Unit of Life 682-686
Ÿ Classification of Plants and Animals 686-690
CHEMISTRY Ÿ Genetics and Molecular Biology and
Evolution of Life 691-697
Ÿ Matter 632-634
Ÿ Plant Morphology and Physiology 697-704
Ÿ Atomic Structure 634-636
Ÿ Animal Physiology 704-723
Ÿ Radioactivity 636-637 Ÿ Human Health and Diseases 723-726
Ÿ Chemical Bonding and Redox Reactions 638-640 Ÿ Applied Biology 726-731
Ÿ Gas Laws and Solutions 640-641 Ÿ Practice Exercise 732-750
Ÿ Acids, Bases and Salts 642-643
Ÿ Chemical Thermodynamics and GENERAL STUDIES
Surface Chemistry 644-645 1. History 753-852
Ÿ Electrochemistry 645-647 2. Geography 853-946
Ÿ Inorganic Chemistry 647-654 3. Indian Polity 947-1005

Ÿ Organic Chemistry 654-656 4. Indian Economy 1006-1049


5. General Knowledge 1050-1092
CDS
Combined Defence Service

SOLVED PAPER 2021 (II)


PAPER I Elementary Mathematics

1. The radius of circum-circle of a difference between their 3. Let p be the area of the square X and
right angled triangle is 10 cm and diameters q be the area of the square formed on
the altitude drawn to the (a) 1 cm (b) 1.5 cm the diagonal of the square X . What is
hypotenuse is 8 cm. What is the (c) 2 cm (d) 4 cm p
area of the triangle? the value of ?
_ (c) Let the centre of the two circles q
(a) 60 cm2 (b) 80 cm2 be c1 and c 2, then c1c 2 = 9 cm
1 1
(c) 100 cm2 (d) 120 cm2 (given) (a) (b)
8 4
_ (b) Given, (c)
1
(d)
1
Radius of circumcircle of right r1 r2 3 2
angled triangle = 10 cm c1 c2
_ (d) Let length of the side of the square X
be a unit.
A
Then, p = ( a )2 = a 2 … (i)
If r1 and r2 denotes the radii. Then,
A D
πr12 + πr22 = 41π
O
⇒ r12 + r22 = 41 … (i)
m

10cm X
8c

Also r1 + r2 = 9 … (ii) a

B C Put r2 = 9 − r1 into Eq (i), we get


a C
⇒ r12 + ( 9 − r1 )2 = 41 B
In the right angled triangle, the ⇒ r12 + 81 + r12 − 18r1 = 41
hypotenuse is the diameter of the ⇒ 2 r12 − 18 r1 + 40 = 0
circumcircle and its centre is the
mid-point of the hypotenuse. ⇒ r12 − 9r1 + 20 = 0
⇒ r12 − 4r1 − 5r1 + 20 = 0 The length of the side of the bigger
i.e., OA = OC = 10 cm square
∴ Hypotenuse ⇒ r1( r1 − 4) − 5( r1 − 4) = 0
= The length of diagonal of square X
= AC = 2 × (10) = 20 cm ⇒ ( r1 − 4)( r1 − 5) = 0 = 2a
The altitude drawn to the hypotenuse ∴ r1 = 4 or 5 ∴ q = ( 2 a )2 = 2 a 2 … (ii)
= BO = 8 cm When r1 = 4 p a2
∴ Area of the right angled triangle Now, = [from Eq. (i) and Eq. (ii)]
⇒ r2 = 9 − r1 = 9 − 4 = 5 cm q 2 a2
1
= × AC × BO When r1 = 5 p 1
2 =
⇒ r2 = 9 − r1 = 9 − 5 = 4 cm q 2
1
= × 20 × 8 = 80 cm2 For, r1 = 4 cm and r2 = 5 cm
2 4. The area of a rhombus is 336 sq cm.
Hence, area of the triangle is 80 cm2. ⇒ d 1 = 2 × 4 = 8 cm
and d 2 = 2 × 5 = 10 cm If the length of one of its diagonals is
2. Two circles touch externally. The ∴ d 2 − d 1 = 10 − 8 = 2 cm
48 cm, then what is the perimeter of
sum of their areas is 41π square Similarly, for r1 = 5 cm and r2 = 4 cm,
the rhombus?
cm. If the distance between their the difference between the (a) 200 cm (b) 120 cm
centres is 9 cm, then what is diameters is 2 cm. (c) 100 cm (d) 90 cm
2 CDS Solved Paper 2021 (II)

_ (c) Given, Time interval from 10:10 am Let n number of bottles are used to empty
to 10:30 am = 20 min the bowl.
Area of rhombus ABCD = 336 cm 2

In 1 min, angle by minute hand = 6º The bowl will be completely empty, when
Length of one of its diagonal = 48 cm
In 20 min, angle by minute hand Volume of n number of cylindrical bottles
A = 20 × 6 = 120º = Volume of hemispherical bowl.
Q The area of sector of circle ⇒ n × Vone bottle = Vbowl
θ 2
= × πr 2 ⇒ n × πr22h = πr13
360º 3
O 120º 2
B D ∴Required area = × π(21)2 ⇒ n × ( 3) ( 4) = × (18)3
2
360º 3
1 22 2
= × × 21 × 21 ⇒ n × 9 × 4 = × 18 × 18 × 18
3 7 3
= 22 × 21 2 × 6 × 18 × 18
C ⇒ n=
= 462 cm2 9×4
We know, the area of rhombus
Hence, the required area is 462 cm2. = 6 × 18 = 108
1
= (d 1 × d 2 ) Hence, 108 number of bottles are used to
2 6. The length and breadth of a room empty the bowl.
Where, d 1 and d 2 are the lengths of are 21 m and 16 m respectively. If
the diagonals. the length of the longest rod that 8. A hollow spherical shell is made of a
1
Therefore, 336 = × ( 48) can be placed in the room is 29 m, metal of density 7 g/cm 3 . If its
2 then what is the height of the internal and external radii are 3 cm
× (length of other diagonal) room ? and 6 cm respectively, then what is
The length of other diagonal 22
(a) 10 m (b) 11 m the mass of the shell ? (Take π = )
336 7
= = 14 cm (c) 12 m (d) 13 m
24
_ (c) Given, length of the room, (a) 2772 g (b) 3322 g
We know that, diagonals of rhombus (c) 4433 g (d) 5544 g
bisect each other at right angle. l = 21 m
1 breadth of the room, b = 16 m _ (d) Given, density of metal of spherical
Q AO = × 14 = 7 cm and shell = 7 g / cm
3
2 and the length of the longest rod that
1 can be placed in the room = 29 m Internal radius, r1 = 3 cm
BO = × 48 = 24 cm
2 We know that, the length of the External radius, r2 = 6 cm
longest rod that can be placed in the
∴In ∆ABO, Volume of the hollow spherical shell
room is the diagonal of the room 4
AB = OA + OB
2 2 2
(cuboid). = π( r23 − r13 )
3
AB2 = 7 2 + 24 2 We know, diagonal of a cuboid 4 22
AB = 49 + 576 = 25 cm. = × × ( 63 − 33 )
= l 2 + b 2 + h2 3 7
Length of side of rhombus = 25 cm 4 22
⇒ 29 = (21)2 + (16)2 + h2 = × × (216 − 27 )
Perimeter of rhombus = 4 × 25 cm 3 7
= 100 cm. ⇒ (29)2 = 441 + 256 + h2 88
= × 189
5. The minute hand of a clock is ⇒ 841 = 697 + h2 ⇒ h2 = 144 21
21 cm long. What is the area on ⇒ h = 12 cm = 88 × 9 = 792 cm3
the face of the clock described by Hence, height of the room is 12 cm. Q Mass = Density × Volume
the minute hand between 10:10 7. A hemispherical bowl of internal ∴Mass of the shell = 7 × 792 = 5544 g
22
am and 10:30 am? (take π = ) radius 18 cm contains a liquid. Hence, mass of the shell is 5544 g.
7 The liquid is filled in small 9. A cone of height 16 cm and diameter
(a) 231 cm2 (b) 331 cm2 cylindrical bottles of internal 14 cm is mounted on a hemisphere of
(c) 462 cm2 (d) 492 cm2 radius 3 cm and internal height same diameter. What is the volume
_ (c) Given, the length of minute hand 4 cm. What is the number of of the solid thus formed?
of the clock = 21 cm bottles used to empty the bowl? 22
Time for calculating the area is 10:10 (a) 54 (b) 81
(Take π = )
7
am to 10:30 am. (c) 108 (d) 135
(a) 1540 cm3 (b) 1078 cm3
_ (c) Given, internal radius of (c) 1048 cm3 (d) 770 cm3
hemispherical bowl, r1 = 18 cm
cm

_ (a) Given, height of the cone, h = 16cm


21

Internal radius of cylindrical bottle,


θ r2 = 3 cm diameter of the cone = 14 cm
r 14
and internal height of cylindrical ∴Radius of the cone, r = = 7 cm
bottle, h = 4 cm 2
CDS Solved Paper 2021 (II) 3

Therefore, volume will remain same. i.e., l = 21 cm, b = 11 cm and h2 = 7 cm


∴Vcuboid = 3 × VCube Let n number of coins are melted to form
⇒ l × b × h = 3 × 343 a cuboid.
h ⇒ 3a × a × a = 3 × 343 In such case, volume will remain same.
⇒ a = 343 ⇒ a = 7 cm
3 ∴Volume of n coins = Volume of cuboid
Putting, a = 7 cm into Eq. (i), we get ⇒ n × Vone coin = Vcuboid
r
l ⋅ b ⋅ h2
Total surface area of cuboid ⇒ n × πr 2h1 = l × b × h2 ⇒ n =
πr 2h1
= 14 × (7 ) = 686 cm
2 2
21 × 11 × 7
And diameter of hemisphere = 14 cm ⇒n =
11. A cubical block of side 14 cm is 22
× (175
. × 175
. ) × 0.4
∴Radius of hemisphere, surmounted by a hemisphere of 7
14
r= = 7 cm radius 7 cm. What is the total 21 × 7 × 7
2 ⇒n =
surface area of the solid thus 7 7  4 
Volume of the solid formed = Volume 2 × ×  × 
22  4 4   10 
of the cone + volume of hemisphere formed? (Take π = )
7 21 × 4 × 4 × 10
1 2
= π r 2h + π r 3 n=
3 3 (a) 1330 cm 2
(b) 1306 cm 2 2 ×4
1 22 (c) 1296 cm2 (d) 1256 cm2 = 21 × 2 × 10 = 420
= × × 7 × 7 × 16
3 7 Hence, 420 silver coins must be melted to
_ (a) Given, length of side of cubical
+ ×
2 22
×7 ×7 ×7 block, a = 14 cm form a cuboid.
3 7 Radius of hemisphere, r = 7 cm 13. A tub is in the shape of a frustum of
22 × 7 × 16 2 × 22 × 7 × 7
= + Total surface area of the solid so a cone. The radii of two circular ends
3 3 formed
22 × 7
of the tub are 105 cm and 42 cm. If
= total Surface area of cube + curved
= (16 + 14) the vertical height of the tub is 16
3 Surface area of hemisphere −
154 circular base area of hemisphere. cm, what is its slant height
= × 30 = 1540 cm3
3 (a) 63.5 cm (b) 65 cm
Hence, the volume of the solid r (c) 73.5 cm (d) 75 cm
formed is 1540 cm3. _ (b) Given, dimension of the frustum are :
10. 3 cubes each of volume 343 cm 3 radius of one circular part, r1 = 42 cm
are joined end to end. What is radius of another circular part,
r2 = 105 cm
the total surface area of the
resulting cuboid? Vertical height, h = 16 cm
In ∆ABC,
(a) 343 cm2 (b) 350 cm2
(c) 686 cm2 (d) 700 cm2 a ( AC )2 = AB2 + BC 2⇒ l 2 = h2 + ( r2 − r1 )2
l 2 = (16)2 + (105 − 42 )2
_ (c) Given, volume of one cube = 6 × ( a )2 + 2 πr 2 − πr 2
= 343 cm
3 r1
= 6 × 14 2 + πr 2 A
Let the length of the side of the cube 22
be a unit. = 6 × 196 + ×7 ×7
7 l
= 1176 + 154 = 1330 cm2 r2
a 12. How many silver coins, 3.5 cm in C
B
a diameter and of thickness 4 mm,
a a a must be melted to form a cuboid of = 256 + ( 63)2 = 256 + 3969 = 4225
dimensions 21 cm × 11 cm × 7 cm? l= 4225 = 65 cm
When cubes are joined end to end,
length of the resulting cuboid will be (a) 420 (b) 210 (c) 200 (d) 168 Hence, the slant height is 65 cm.
3a but breadth and height will be the
_ (a) Given, dimensions of silver coins 14. ABC is a triangle right angled at B
same. (cylindrical shape)
In cuboid, l = 3a, b = a and h = a withAB = 8 cm and BC = 6 cm. It is
Thickness ( h1 ) = 4 mm
Total surface area of cuboid
made to revolve about its side BC.
4
= cm = 0.4 cm What is the approximate total surface
= 2( lb + bh + hl ) 10
= 2( 3a × a + a × a + a × 3a ) area of the cone so formed?
Diameter = 3.5 cm 22
= 2( 3a 2 + a 2 + 3a 2 ) 3.5 ( Take π = )
∴Radius, r = = 175
. cm 7
= 2(7 a 2 ) = 14a 2 … (i) 2
Since, cuboid is formed using And dimensions of cuboid are (a) 452 cm2 (b) 440 cm2
3 cubes. 21 cm × 11 cm × 7 cm (c) 432 cm2 (d) 420 cm2
4 CDS Solved Paper 2021 (II)

_ (a) In right angled triangle ABC 3.5cm In the options given factors are ( x + 14),
given that, ( x − 14), ( x − 6) and ( x − 7 ).
AB = 8 cm We know that ( x − a ) is a factor of the
polynomial f( x ), if f ( a ) = 0
BC = 6 cm 2 10cm
Factor ( x + 14)
C
Put x + 14 = 0 or
x = − 14 into f ( x ), we get
l 20 f ( −14) = ( −14 − 1)(14 − 2 )( −14 − 4) − 90
1
6cm = ( −15)( −16)( −18) − 90
= −4320 − 90
B 8cm A = − 4410 ≠ 0
7cm
Factor ( x − 14) :
When the triangle is revolved about Volume of the solid rod = Volume of
its side BC, Put x − 14 = 0 or x = 14 into f( x ) , we get
cylinder 1 + Volume of cylinder 2.
the height of the cone so formed, f (14) = (14 − 1) (14 − 2 ) (14 − 4) − 90
= πr12h1 + πr22h2
h = 6 cm = (13) (12 ) (10) − 90 = 1560 − 90 ≠ 0
22
= × (7 × 7 × 20 + 3.5 × 3.5 × 10) Factor ( x − 6) :
the radius of the cone so formed, 7
r = 8 cm 22 Put x − 6 = 0 or x = 6 into f( x ) , we get
= (7 × 140 + 3.5 × 35)
Therefore, total surface area of the 7 f( 6) = ( 6 − 1) ( 6 − 2 ) ( 6 − 4) − 90
cone formed = πr 2 + πrl … (i) = 5 × 4 × 2 − 90 = 40 − 90 ≠ 0
= 22(140 + 3.5 × 5)
In ∆ABC, Factor ( x − 7 ) :
= 22(140 + 17.5)
AC 2 = AB2 + BC 2 Put x − 7 = 0
= 22(157.5) = 3465 cm 3

l 2 = 82 + 62 3465 or x = 7 into f ( x ) , we get


= m3
l 2 = 100 100 × 100 × 100 f(7 ) = (7 − 1) (7 − 2 ) (7 − 4) − 90
⇒ l = 10 cm = 6 × 5 × 3 − 90
∴Mass of rod
From Eq. (i), we have 3465 = 90 − 90 = 0
= × 10000 kg
Total surface area of the cone 100 × 100 × 100 Hence, ( x − 7 ) is a factor of the given
= πr( r + l ) polynomial.
22 = 34.65 kg
= × 8 × ( 8 + 10) Hence, mass of the rod is 34.65 kg. 18. What is the square root of23 − 4 15
7
22 × 8 × 18 16. What are distinct prime factors of ?
=
7 the number 26381? (a) 6 − 3 2
=
3168
(a) 29, 17, 37 (b) 31, 17, 47 (b) 7 − 3 5
7
(c) 19, 37, 13 (d) 23, 31, 37 (c) 3 − 2 5
≈ 452 cm2 (d) 5 − 4 3
_ (d) Given number is 26381.
Hence, the required approximate
Factors of the number : _ (c) 23 − 4 15 = 3 + 20 − 4 15
total surface area of the cone formed
is 452 cm2. = ( 3 )2 + (2 5 )2 − 2 × 3 × 2 5
23 26381
= ( 3 − 2 5 )2
15. A solid rod consists of a cylinder 31 1147 ∴ 23 − 4 15
of height 20 cm and radius 7 cm.
It is surmounted by another solid 37 37 = ( 3 − 2 5 )2
cylinder of height 10 cm and 1 = 3 −2 5
radius 3.5 cm. If 1 cubic metre of
rod weights 10000 kg, what is the 26381 = 23 × 31 × 37 19. What is the remainder after dividing
22 Hence, distinct prime factors of the the number 37 1000 by 9?
mass of the rod ? (Take π = ) given number are 23, 31 and 37.
7 (a) 1 (b) 3
(a) 34.65 kg (b) 31.56 kg 17. Which one of the following is a (c) 7 (d) 9
(c) 3.465 kg (d) 3.156 kg factor of the polynomial _ (a) 37
1000
= 37 1000 − 11000 + 1
( x − 1)( x − 2 )( x − 4) − 90 ? We know that x n − a n is divisible by x − a.
_ (a) Given,
Dimensions of cylinder 1 : (a) x + 14 (b) x − 14 ∴ 37 1000 − 11000 is divisible by

Radius ( r1 ) = 7 cm (c) x − 6 (d) x − 7 ( 37 − 1 = ) 36.

Height ( h1 ) = 20 cm _ (d) Let the polynomial represented The number is divisible by 36, it is
by f( x ). divisible by 9.
Dimensions of cylinder 2 :
∴ f( x ) = ( x − 1) ( x − 2 ) ∴ 37 1000 − 11000 is divisible by 9.
Radius ( r2 ) = 3.5 cm
( x − 4) − 90 … (i) ∴Required remainder = 1
Height ( h2 ) = 10 cm
CDS Solved Paper 2021 (II) 5

20. The sum of LCM and HCF of the 22. If x is a negative real number, _ (c) Given expression is
numbers is 536 and the difference then which of the following are = 2 x 2 + xy − 3 y 2
between LCM and HCF is 296. If not correct? = 2 x 2 + 3xy − 2 xy − 3 y 2
one of the numbers is 104, then 1. There is some natural number = x (2 x + 3 y) − y(2 x + 3 y)
what is the other number? k such that kx > 0 = (2 x + 3 y) ( x − y)
(a) 420 (b) 480 (c) 484 (d) 506
2. x 2 + x > 0 always Linear factors (in x and y) are (2 x + 3 y)
and( x − y).
_ (b) Let the LCM and HCF of two
numbers be x and y respectively. 3. 2x < x < − x Their sum = 2 x + 3 y + x − y
Now, according to the question, 4. x 2 is always a rational number = 3x + 2 y
x + y = 536 … (i) Select the correct answer using Hence, sum of linear factors of the given
x − y = 296 … (ii) the code given below: expression is 3x + 2 y.
Adding Eq. (i) and Eq. (ii), we get (a) 1, 2 and 3 (b) 1, 2 and 4 24. Which one of the following
2 x = 832 ⇒ x = 416 (c) 1, 3 and 4 (d) 2, 3 and 4 equations does not have real roots?
From Eq. (i), (a) 2 x 2 + 16x + 3 = 0
_ (b) Given, x is a negative number.
416 + y = 536 (b) 2 x 2 + 10x − 1 = 0
1. ‘‘There is some natural number k
⇒ y = 536 − 416 = 120 such that kx > 0’’. (c) x 2 − 8x + 1 = 0
LCM = x = 416 When a negative number ( x ) is
(d) 4x 2 + 9x + 6 = 0
and HCF = y = 120 multiplied by a natural number ( k ),
then the resulting number will be
We know, _ (d)2 In a quadratic equation
negative. ax + bx + c = 0,
Product of LCM and HCF = Product
of two numbers. ∴ kx > 0 is not correct. The discriminant, D = b 2 − 4ac
Let the other number be P. 2. “x + x > 0 always”.
2
if b 2 − 4ac < 0
1
∴ 416 × 120 = 104 × P [Q one of the For x = − then the equation does not have real
numbers is 104] 2 roots.
2
416 × 120  1  1 A. 2 x 2 + 16x + 3 = 0
⇒P = = 480 ⇒ x2 + x = −  + − 
104  2  2 b 2 − 4ac = (16)2 − 4 (2 ) ( 3)
Hence, the other number is 480. =
1 1 1
− =− <0 = 256 − 24 = 232 > 0
4 2 4 (gives real roots)
21. 20 men are supposed to complete
∴x 2 + x > 0 is not always correct. B. 2 x 2 + 10x − 1 = 0
a work in 10 days after working
3. “2 x < x < − x ” b 2 − 4ac = (10)2 − 4 (2 ) ( − 1)
for 5 days they realise that only
one fourth of the work is done. ⇒ 2x < x = 100 + 8 = 108 > 0
How many more men they need and x < − x (gives real roots)
to employ to finish the work on ⇒ 2 × (negative number) < (same C. x 2 − 8x + 1 = 0
time? negative number) and negative
number < positive number [Q − x b 2 − 4ac = ( − 8)2 − 4(1) (1)
(a) 40 (b) 30 (c) 20 (d) 15 is positive] = 64 − 4 = 60 > 0
_ (a) Here, M1 = 20 ∴ 2 x < x < − x is always correct. (gives real roots)
1 2
4. “x is always a rational number”. D. 4x 2 + 9x + 6 = 0
D1 = 5 ⇒ W1 =
4 Let x = − (2 )1/ 4 b 2 − 4ac = ( 9)2 − 4( 4) ( 6)
Let x more men are employeed. then, x = [− (2 )
2 1/ 4 2
] = 81 − 96 = − 15 < 0
∴M 2 = 20 + x = + 2 1/ 4 × 2 = 2 1/ 2 = 2 , (gives imaginary roots or not real )
D2 = 5 which is irrational. Hence, option (d) has the quadratic
 1 3 ∴x 2 is always a rational number is equation with no real roots.
W2 = 1 −   =
 4 4 an incorrect statement. 25. The sum and the product of the roots
MD M D
We know that, 1 1 = 2 2 Hence, 1, 2 and 4 are not correct. of a quadratic equation are 7 and 12
W1 W2 respectively. If the bigger root is
23. What is the sum of the linear
⇒ M1D1W 2 = M 2D2W1 factors (in x andy) of the halved and the smaller root is
3
⇒ 20 × 5 × = (20 + x ) × 5 ×
1
expression doubled, then what is the resulting
4 4 quadratic equation?
2x 2 + xy − 3y 2 ?
⇒ 20 × 3 = (20 + x ) (a) x 2 − 6x + 12 = 0
⇒ 60 = 20 + x (a) 2 x − 3y
(b) x 2 − 8x + 12 = 0
(b) 3x − 2 y
⇒ x = 40
(c) 3x + 2 y (c) x 2 + 8x + 12 = 0
Hence, 40 more men need to be
employed to finish the work on time. (d) 2 x + 3y (d) x 2 − 10x + 12 = 0
6 CDS Solved Paper 2021 (II)

_ (b) Let roots of the quadratic 27. If α + β + γ = αβ + βγ + γα, then 30. Three runners are running in a circular
equation be a and b. Then,
what is (1 − α )(1 − β ) (1 − γ ) equal track, and they complete one round in
a+ b=7 … (i) 20, 30 and 35 minutes respectively.
to
a ⋅ b = 12 … (ii) when will they next meet at the
(a) 1 − αβγ
Put b = 7 − a into Eq.(ii), we get starting point?
(b) 1 + αβγ
a (7 − a ) = 12
(c) α 2 + β 2 + γ 2 (a) After 3 hour 30 minutes
⇒ 7 a − a 2 = 12 (b) After 4 hour 30 minutes
(d) (α − β)(β − γ)(γ − α)
⇒ a 2 − 7 a + 12 = 0 (c) After 3 hour
⇒ a − 3a − 4a + 12 = 0
2 _ (a) Given, (d) After 7 hour
⇒ a( a − 3) − 4( a − 3) = 0 α + β + γ = αβ + βγ + γα
⇒ 0 = αβ + βγ + γα − (α + β + γ )
_ (d) The time interval when they will meet
⇒ ( a − 3) ( a − 4) = 0 next at the starting point is the LCM (20,
a = 3 or 4 ⇒ αβ + βγ + γα − (α + β + γ ) = 0 30 and 35).

When, a = 3, b = 7 − 3 = 4 … (i) Thus, the prime factorisation of 20, 30


∴ (1 − α ) (1 − β ) (1 − γ ) and 35 is
When, a = 4, b = 7 − 4 = 3
= (1 − α − β + αβ ) (1 − γ ) 20 = 2 × 2 × 5 = (2 )2 × ( 5)1,
So, the smaller root is 3 and the
bigger root is 4. = 1 − α − β + αβ) 30 = 2 × 3 × 5 = (2 )1 × ( 3)1 × ( 5)1

According to the question, − γ + αγ + βγ − αβγ and 35 = 5 × 7 = ( 5)1 × (7 )1


1 = 1 − α − β − γ + αβ LCM (20, 30, 35)
New roots are × ( 4)and 2 × ( 3)i.e.,
2 + βγ + γα − αβγ = (2 )2 × ( 3)1 × ( 5)1 × (7 )1
2 and 6.
= 1 + αβ + βγ + γα = 12 × 35
The resulting quadratic equation :
− (α + β + γ ) − αβγ = 420 min
x 2 − (sum of roots) x + (product of 420
= 1 + 0 − αβγ [from Eq. (i)] = h=7h
roots) = 0 60
= 1 − αβγ
⇒ x 2 − (2 + 6. ) x + 2 × 6 = 0 Hence, all three runners will next meet
28. If log10 x + log10 x 2 after 7 h at the starting point.
⇒ x 2 − 8x + 12 = 0
Hence, the required quadratic = 2 log10 x + 1then what is the 31. What is the minimum value of
equation is x 2 − 8x + 12 = 0 value ofx cos 3 θ + sec 3 θ where 0° ≤ 90° ?
26. For which values of k , does the (a) 1 (b) 2 (a) 0 (b) 1
(c) 5 (d) 10
equation x 2 − kx + 2 = 0 have (c) 2 (d) None of these
_ (d) log10 x + log10 x = 2 log10 x + 1
2
real and distinct solutions ? _ (c) Given expression :
(a) −2 2 < k < 2 2 log10 x + 2 log10 x = 2 log10 x + 1 cos 3 θ + sec 3 θ where, 0º ≤ θ ≤ 90º
[Q log x m = mlog x ]
(b) k < −2 2 only In the interval 0º ≤ θ ≤ 90º, cosθ and sec θ
⇒ log10 x + 2 log10 x − 2 log10 x = 1 are positive.
(c) k > 2 2 only
⇒ log10 x = 1 On applying AM-GM on cos 3 θ and sec 3 θ,
(d) k < −2 2 or k > 2 2
⇒ x = (10) 1 we get
_ (d) Given, quadratic equation is ⇒ x = 10 cos 3 θ + sec 3 θ
x 2 − kx + 2 = 0. ≥ cos 3 θ ⋅ sec 3 θ
∴ The value of x is 10. 2
In a quadratic equation ⇒ cos 3 θ + sec 3 θ ≥ 2
ax 2 + bx + c = 0, 29. The LCM of two prime numbers
Hence, the minimum value of
if the discriminant, p and q is 2231, where p > q . cos 3 θ + sec 3 θ is 2.
D = b 2 − 4ac > 0, then the What is the value of p − q ?
quadratic equation will have real and (a) 67 (b) 70 32. If 14 sin 2 θ + 10 cos 2 θ = 11 where
distinct solutions. (c) 74 (d) 81 0° < θ < 90° then what is the value of
For x 2 − kx + 2 = 0 tan θ + cot θ ?
_ (c) Given, LCM of two prime
a = 1, b = − k and c = 2 numbers p and q is 2231, where 4 2
(a) (b)
∴b − 4ac = ( − k )2 − 4(1) (2 )
2 p > q. 3 3
= k2 − 8 We know, LCM of two prime (c) 3 (d) 2 3
numbers = Product of the numbers
Put b − 4ac > 0 ⇒ k − 8 > 0 _ (a) Given, 14 sin θ + 10cos θ = 11
2 2 2 2

⇒ 2231 = p × q
⇒ k − (2 2 ) > 0
2 2
Where, 0º < θ < 90º,
⇒ p × q = 2231
⇒ (k − 2 2 ) (k + 2 2 ) > 0 14 sin2 θ + 10 cos 2 θ = 11
⇒ p × q = 23 × 97
∴ k < − 2 2 or k > 2 2 ⇒ 4 sin2 θ + 10 sin2 θ +10 cos 2 θ = 11
Hence, the quadratic equation will ∴ p = 97 and q = 23 (Q p > q )
⇒ 4 sin2 θ + 10(sin2 θ + cos 2 θ) = 11
have real and distinct solutions for Now, p − q = 97 − 23 = 74
⇒ 4 sin2 θ + 10 × 1 = 11
k < − 2 2 or k > 2 2. Hence, the value of p − q is 74. [Q sin2 θ + cos 2 θ = 1]
CDS Solved Paper 2021 (II) 7

⇒ 4 sin2 θ = 1 ⇒ sin2 θ =
1 So, In ∆OPR ⇒ ∠R = 30º ∴ Maximum value
4 Q OR = OQ = 1 + 2 sin2 45° cos 2 45°− sin4 45°− cos 4 45°
1 1 1 1 1
⇒ sinθ = ± ⇒ ∠ ROQ = ∠ORQ = 1+ 2 ⋅ ⋅ − −
2 2 2 4 4
Let PQ = h
Qθ is in I-quadrant. 1 1
R = 1+ − = 1
∴sinθ cannot be negative. 2 2
1 30° 10
Therefore, sinθ = 36. From an aeroplane flying about a
2
Q river at an altitude of 1200 m, it is
⇒ θ = 30°
10 observed that the angles of
Now, tanθ + cot θ h depression of opposite points on the
30°
= tan 30°+ cot 30° two banks of a river are 30° and θ. If
30° 60°
1 1+ 3 O P
= + 3 = the width of the river is 3000 m, then
3 3 which one of the following is
PR
4 Qtan 60º =
= OP correct ?
3 10 + h
⇒ 3= (a) θ < 30° (b) 30°< θ < 45°
33. What is the OP (c)45°< θ < 60° (d) 60°< θ < 90°
10 + h
sin3 θ + cos 3 θ sin3 θ − cos 3 θ ⇒ OP = … (i) _ (c) Given, altitude of aeroplane is 1200
+ 3
sinθ + cos θ sinθ − cos θ m.
h
equal to? Qtan 30º = and the width of the river is 3000 m.
(a) 0 (b) 1 OP
So, we have,
h
(c) 2 (d) 4 ⇒ OP = = 3h … (ii) BC = 3000 m, AD = 1200 m,
(1 / 3 )
_ (c) Given expression, ∠B = 30º and ∠C = θ ,
From Eq. (i) and Eq. (ii), we get
sin3 θ + cos 3 θ sin3 θ − cos 3 θ A
+ 10 + h
sinθ + cos θ sinθ − cos θ = 3h ⇒ 10 + h = 3h 30° θ
3
sin3 θ + cos 3 θ sin3 θ − cos 3 θ
⇒ + ⇒ 10 = 2 h

1200m
sinθ + cos θ sinθ − cos θ
⇒ h = 5m
(sinθ + cos θ) (sin2 θ Height of flagstaff
+ cos 2 θ − sinθ cos θ) 30° θ
= = h + 10 = 5 + 10 = 15 m
(sinθ + cos θ) B D C
Hence, the height of the flagstaff is
3000 m
15 m.
(sinθ − cos θ) (sin2 θ
In ∆ABD,
+ cos 2 θ + sinθ cos θ) 35. What is the maximum value of
+ AD
(sinθ − cos θ) 1 + 2 sin2 θ cos 2 θ − sin4 θ − cos 4 θ tan 30º =
BD
Q a 3 + b 3 = ( a + b ) (a2  where 0°< θ < 90° ?
1 1200
 + b 2 − ab )  (a) 1 (b) 2 (c) 3 (d) 4 ⇒ =
  3 BD
and a − b = ( a − b)
3 3
 _ (a) Given expression : ⇒ BD = 1200 3
 ( a 2 + b 2 + ab ) 1 + 2 sin2 θ cos 2 θ − sin4 θ − cos 4 θ
∴ DC = BC − BD = 3000 − 1200 3
= 1 − sinθ cos θ + 1 + sinθ cos θ Where,
In ∆ACD,
[Q sin2 θ + cos 2 θ = 1] 0º < θ < 90º DC
= 1+ 1 ⇒ 1 + 2 sin2 θ cos 2 θ − sin4 θ − cos 4 θ cot θ =
AD
=2 = 1 − [sin4 θ + cos 4 θ 3000 − 1200 3
=
34. A ladder 10 m long reaches a − 2 sin2 θ cos 2 θ] 1200
point 10 m below the top of a = 1 − [(sin2 θ)2 + (cos 2 θ) 2 30 − 12 3 5 − 2 3
= =
vertical flagstaff. From foot of the − 2 sin2 θ ⋅ cos 2 θ] 12 2
ladder, the elevation of top of the = 1 − [(cos 2 θ − sin2 θ)2 ] cot θ = 2.5 − 1732
. = 077
.
flagstaff is 60°. What is the height [Q a 2 + b 2 − 2 ab = ( a − b )2 ] We know, cot θ is a decreasing function in
of flagstaff ? = 1 − [(cos 2θ)2 ] first quadrant.
(a) 12 m (b) 15 m [Q cos 2 x = cos 2 x − sin2 x ] cot 45º = 1
(c) 16 m (d) 20 m = 1 − cos 2θ = sin 2θ
2 2
and cot 60º =
1
=
3 1732
=
.
≈ 0.6
[Q sin A + cos A = 1]
2 2 3 3 3
_ (b) Let OQ be a ladder.
∴ OQ = 10 m For maximum value sin2 2θ = 1 ∴cot 45º > cot θ > cot 60º
Some part of Flagstaff QR = 10 m ⇒sin2 θ = 1 ⇒2 θ = 90° ∴θ ∈ ( 45º, 60º )
Now, QR = QO = 10 m ⇒ θ = 45° Hence, option (c) is correct.
8 CDS Solved Paper 2021 (II)

cos 2 θ − 3 cos θ + 2 2. sin4 θ + cos 4 θ = 1 + 2 sin2 θ cos 2 θ 41. If x = m sec A + n tan A and
37. If =1 LHS = (sin2 θ) 2 + (cos 2 θ) 2
sin θ 2 y = m tan A + nsecA, then what is
= (sin2 θ + cos 2 θ)2
Where 0° < θ < 90° then what is x 2 − y 2 equal to ?
− 2 sin2 θ ⋅ cos 2 θ
sin 2 θ + cos θ equal to (a) m2 − n2
[Q a + b = ( a + b )2 − 2 ab ]
2 2

5 3 7 (b) m2 + n2
(a) (b) (c) (d) 2 = 1 − 2 sin2 θ ⋅ cos 2 θ
4 2 4 ≠ R.H.S. (c) m2 + n2 − mn
cos 2 θ − 3 cos θ + 2 3. tan4 θ + tan2 θ = sec 4 θ − sec 2 θ
_ (a) Given, =1 (d) m2 − n2 + mn
sin2 θ ⇒ tan2 θ(tan2 θ + 1)
Where, 0º < θ < 90º = sec 2 θ(sec 2 θ − 1) _ (a) Given,
⇒ cos 2 θ − 3 cos θ + 2 = sin2 θ ⇒ tan2 θ ⋅ sec 2 θ = sec 2 θ ⋅ tan2 θ x = msec A + n tan A
⇒ cos 2 θ − 3 cos θ + 2 = 1 − cos 2 θ [Q1 + tan2 A = sec 2 A ] and y = m tan A + nsec A
Which is true. x 2 − y2
[Q sin2 A + cos 2 A = 1] = ( msec A + n tan A )2
Hence, 1 and 3 are the identities.
⇒ 2 cos 2 θ − 3 cos θ + 1 = 0 − ( m tan A + nsec A )2
⇒ 2 cos 2 θ − 2 cos θ − cos θ + 1 = 0 39. What is the value of = m2 sec 2 A + n2 tan2 A
⇒ 2 cos θ(cos θ − 1) − 1(cos θ − 1) = 0 sin 24 ° sin 66° − cos 24 ° cos 66° + + 2 mnsec A tan A
⇒ (cos θ − 1) (2 cos θ − 1) = 0 tan 24 ° tan 66° − cot 24 ° cot 66° ? − m2 tan2 A − n2 sec 2 A
1 (a) 0 (b) 1 (c) 2 (d) 3 − 2 mn tan A sec A
⇒ cosθ = 1 or cosθ =
2 = m2 sec 2 A + n2 tan2 A
_ (a) sin24ºsin 66º − cos 24ºcos 66º − m2 tan2 A − n2 sec 2 A
Q In the given interval θ ≠ 0 ⇒ + tan24º tan 66º − cot 24ºcot 66º
cosθ ≠ 1 = m2(sec 2 A − tan2 A )
⇒ sin( 90º − 66º )sin( 90º − 24º ) + n2 (tan2 A − sec 2 A )
1
∴cosθ = (only) − cos 24º⋅ cos 66º + tan( 90º − 66º )
2 tan( 90º − 24º ) − cot 24ºcot 66º = m2(sec 2 A − tan2 A )
− n2(sec 2 A − tan2 A )
sinθ = 1 − cos 2 θ = cos 24º⋅ cos 66º − cos 24º cos 66º
= m2 × 1 − n 2 × 1
1 3 3 + cot 24º⋅ cot 66º − cot 24ºcot 66º [Q1 + tan2 θ = sec 2 θ]
= 1− = =
4 4 2 [Qsin( 90º − θ) = cos θ = m2 − n 2
Now, sin θ + cos θ
2 and tan( 90º − θ) = cot θ] Hence, x 2 − y 2 = m2 − n2
 3
2 = 0 + 0= 0
=
1 3 1
 + = + 42. If for some θ lying between 0° and
 2  2 4 2 40. If x = p sin A cos B, 90°, tan θ = 1, then what is the value
=
3 2 5
+ = y = p sin A sin B and z = p cos A , of sin 2 θ − 2 sin θ cos θ ?
4 4 4 then what is the value of 1 1
(a) −1 (b) 0 (c) (d) −
Hence, sin2 θ + cos θ =
5
. x2 +y2 +z2 ? 2 2
4
(a) −p 2 (b) 0 _ (d) Given, tanθ = 1, where
38. Consider the following (c) p 2
(d) 2p 2
0º < θ < 90º.
1. sin 4 θ − sin 2 θ = cos 4 θ − cos 2 θ tanθ = 1 ⇒ θ = 45º
_ (c) Given,
2. sin 4 θ + cos 4 θ = 1 + 2 sin 2 θ cos 2 θ x = psin A cos B, y = psin A sin B Now, sin2 θ − 2 sinθ cos θ
and z = pcos A = sin2 45º − 2 sin 45ºcos 45º
3. tan 4 θ + tan 2 θ = sec 4 θ − sec 2 θ
x + y + z
2 2 2 = [sin 45º ]2 − 2 sin 45º⋅ cos 45º
Which of the above are identities? = ( psin A cos B) + ( psin A sin B)
2 2
 1 
2
1 1
=  − 2⋅ 2 ⋅ 2
(a) 1 and 2 only + ( pcos A )2  2
(b) 2 and 3 only = p2 sin2 A cos 2 B + p2 sin2 A sin2 B 1 1
= − 2⋅
(c) 1 and 3 only + p2 cos 2 A 2 2
(d) 1, 2 and 3 = p [sin A cos B
2 2 2
1 1
= − 1= −
_ (c) Considering the given identities + sin2 A sin2 B + cos 2 A ] 2 2
1. sin θ − sin θ = cos θ − cos θ
4 2 4 2
= p [sin A(cos 2 B + sin2 B)
2 2 1
Hence, sin2 θ − 2 sinθ cos θ = −
2
⇒ sin θ (sin θ − 1)
2 2
+ cos A ]
2

= cos 2 θ(cos 2 θ − 1) = p2[sin2 A × 1 + cos 2 A ] 2 sin 3 θ − sin θ


43. What is , (0° < θ < 90° )
⇒ sin θ ( − cos θ) = cos θ ( − sin θ)
2 2 2 2
[Q sin θ + cos θ = 1]
2 2
cos θ −`2 cos 3 θ
[Q sin2 A + cos 2 A = 1 ] = p2[sin2 A + cos 2 A ]
equal to
⇒ − sin2 θ ⋅ cos 2 θ = − sin2 θ ⋅ cos 2 θ = p2 × 1 = p2
(a) sinθ (b) cosθ
Which is true. Hence, x 2 + y 2 + z2 = P 2
(c) tanθ (d) cotθ
CDS Solved Paper 2021 (II) 9

_ (c) Given expression, = cot


A A A
+ cos − cot − cos
A
Directions (Q.Nos. 46-48) Consider the
2 sin θ − sinθ
3 2 2 2 2
, where 0º < θ < 90º following for the next three questions that
cos θ − 2 cos 3 θ [Qtan( 90º − θ) = cot θ
follow.
and sin( 90° − θ) = cos θ]
sinθ(2 sin2 θ − 1) A solid consisting of a right circular cone
= =0
cos θ(1 − 2 cos 2 θ) of radius x and height 2x standing on a
Hence, the given expression results 22
[− 1(1 − 2 sin2 θ)] zero. hemisphere of radius x (take π = )
= tanθ ⋅ 7
[− (2 cos 2 θ − 1)]
[cos 2θ] 45. In a triangle ABC, right angled at 46. The volume of the solid is equal to
= tanθ ⋅ B, AB + BC = 10(1 + 3) cm
[cos 2θ] that of a
[Qcos 2 A = 2 cos 2 A − 1 length of the hypotenuse is 20 (a) Sphere of radius x
= 1 − 2 sin2 A] cm. What is the value of (b) sphere of diameter x
= tanθ × 1 = tanθ tan A + tan C ? (c) cylinder of radius x
2 sin3 θ − sinθ 4
Hence, = tanθ (a) (d) cylinder of radius 2x
cos θ − 2 cos θ3 3
2 _ (a) Given dimensions :
(b) Cone : radius ( r1 ) = x
44. If A, B andC are int4rior angles 3
height ( h) = 2 x
of a triangle ABC, then what is (c) 3
Hemisphere : radius ( r2 ) = x
B + C B + C (d) 2 3
tan  + sin 
 2   2 
_ (a) In a right angled triangle ABC, it
 A  A is given that,
− cot   − cos   equal to ?
2 2 AB + BC = 10(1 + 3 ) , ∠B = 90º
(a) 0 and AC = 20 cm 2x
1 C
(b)
2
x
A + B + C
(c) sin  
 4 
20cm
A + B + C
(d) tan  
 4 
∴Volume of the solid = Volume of cone +
Volume of hemisphere
_ (a) Given expression,
B + C B + C 1 2
B A = πr12h + πr23
tan  + sin 
 2   2  3 3
In ∆ABC, 1 2
 A  A = π ⋅ x ⋅ (2 x ) + π ⋅ ( x )3
2
− cot   − cos   AB2 + BC 2 = AC 2 3 3
2 2
AB2 + BC 2 = 20 2 2 2 4
We know that the sum of all three = πx + πx = πx 3
3 3
AB2 + BC 2 = 400 … (i) 3 3 3
interior angles of a triangle is 180º.
i.e., in ∆ ABC, A + B + C = 180° Now, AB + BC = 10 (1 + 3) Hence, the volume of the solid is equal to
Squaring on both sides, we get that of a sphere of radius x.
A
( AB + BC )2 = [10( 1 + 3 )]2 47. What is the approximate total surface
⇒ AB + BC + 2 ⋅ AB ⋅ BC
2 2
area of the solid
= 100(1 + 3 + 2 3 ) . x 2 (b) 12.5x 2 (c) 13.3x 2 (d) 151
(a) 112 . x2
⇒ 400 + 2 AB ⋅ BC = 100( 4 + 2 3 ) _ (c) We have,
[From Eq. (i)]
Cone : r1 = x and h = 2 x
⇒ 2 AB ⋅ BC = 400 + 200 3 − 400
Hemisphere : r2 = x
B C ⇒ AB ⋅ BC = 100 3 … (ii)
A
 180º − A   180º − A  Now, tan A + tanC
= tan  + sin 
 2   2  =
BC
+
AB
 A  A AB BC
− cot   − cos  
2 2 BC 2 + AB2 l
= 2x
[Q A + B + C = 180º ] AB ⋅ BC
 A  A 400 4
= tan 90º −  + sin 90º −  = =
 2  2 100 3 3
B
O x
4
 A  A Hence, tan A + tanC =
− cot   − cos  
2 2 3
10 CDS Solved Paper 2021 (II)

Total surface area of solid  4 5 5π 3


= πx 3  3 −  = π ⋅ x 3 ⋅ = x 50. What is the approximate area of
= curved surface area of cone +  3 3 3
major segment of the circle?
curved surface area of hemisphere 5 × 314
. 157
. 3~
= x3 = x − 5 .24x 3 (a) 10.05 cm2
= πr1l + 2 πr22 3 3
(b) 10.15 cm2
= π ⋅ x ⋅ AB + 2 πx 2 … (i) Hence, the approximate volume of
. x 3.
water left in the cylinder is 524 (c) 11.05 cm2
In right angled triangle ABC,
(d) 11.15 cm2
AB2 = OA 2 + OB2 Directions (Q.Nos. 49 and 50)
⇒ AB2 = (2 x )2 + x 2 _ (d) The approximate area of major
Consider the following for next two segment of circle = Area of circle − area of
= 4x 2 + x 2 = 5x 2 question that follow. minor segment of circle
⇒ AB = 5x The chord of a circle of radius 2.1 cm = πr 2 − 2.71
Put AB = 5x into Eq (i), we get is inclined the angle of 120° on the [obtained in question (49)]
Total surface area of solid 22
centre of circle. = × 2.1 × 2.1 − 2.71
= π ⋅ x ⋅ 5x + 2 πx 2 22 7
(Take π = and 3 = 1.732)
= 314
. × 2.23 x 2 + 6.28x 2 7 = 22 × 0.3 × 2.1 − 2.71
= 7.0022 x 2 + 628
. x2 = 13.86 − 2.71
49. What is the approximate area of = 1115
. cm2
= 13.2822 x 2 ~
− 13.3x 2
minor segment of the circle ?
Hence, the approximate total Hence, the approximate area of major
surface area of the solid is 13.3x 2.
(a) 2.71 cm2 (b) 2.42 cm2 . cm2.
segment of the circle is 1115
. cm2
(c) 191 . cm2
(d) 171
48. The solid is placed upright in a Directions (Q.Nos. 51-53) Consider the
right circular cylinder full of following for the next three questions that
water such that it touches the follow.
bottom. If the internal radius of
ABC is a triangle with sides AB = 6 cm,
cylinder is x and height is 3x,
1c
m 2.1cm BC = 10 cm and CA = 8 cm. With vertices
what is the approximate volume 2. 60° 60°
of water left in the cylinder? A, B and C as centres, three circles are
drawn each touching the other two
(a) 5.04x 3 (b) 5.09x 3
externally.
. x3
(c) 514 (d) 5.24x 3
_ (a) Given, Radius ( r ) of the circle
= 2.1 cm; ∠AOB = θ = 120º 51. What is the sum of the radii of the
_ (d) We are given the dimensions as
circles?
Cone : r1 = x and h1 = 2 x Area of minor segment of circle
Hemisphere : r2 = x = Area of minor sector − Area of (a) 10.4 cm (b) 11.2 cm
triangle formed (c) 12 cm (d) 13 cm
Cylinder : r3 = x and h2 = 3x
θ 1  θ _ (c) In ∆ABC it is given that
= πr 2 − ×  2 r sin  
360º 2  2 AB = 6 cm, BC = 10 cm
θ and CA = 8 cm
× rcos  
2
θ θ θ
= πr 2 − r 2 sin  cos   A
3x 360º 2 2 r1 r1
120º 22
= × × (2.1)2 − (2.1)2 r3 r2
360º 7
 120º   120º 
sin  cos   C r3 r2 B
 2   2 
1 22 21 21
= × × ×
When the solid is placed into the 3 7 10 10
21 21 Let radius of circles A, B and C be r1,
cylinder then the water equal to the − × × sin 60ºcos 60º
volume of solid will be emptied. 10 10 r2 and r3.
462 441 3 1 We have,
Hence, water left in the cylinder = − × ×
= Volume of cylinder − Volume of
100 100 2 2 AB + BC + CA = 6 + 10 + 8
1 ⇒ ( r1 + r2 ) + ( r2 + r3 ) + ( r3 + r1 ) = 24
solid = ( 462 − 110 .25 × 1.732 )
4 100 [from the above figure]
= πr32h2 − πx 3 1
3 = ( 462 − 190.953) ⇒ 2 r1 + 2 r2 + 2 r3 = 24
[Qobtained in question (46)] 100
⇒ 2( r1 + r2 + r3 ) = 24
4 271047
.
= π ⋅ x 2 ⋅ ( 3x ) − πx 3 = ≈ 2.71 cm2 ⇒ r1 + r2 + r3 = 12 cm
3 100
Hence, sum of the radii of the circles is
4 Hence, approximate area of minor
= 3 πx − πx
3 3
12 cm.
3 segment of the circle is 2.71 cm2
CDS Solved Paper 2021 (II) 11

52. What is the length of the altitude _ (c) Q r1 + r2 + r3 = 12 Now, AB2 = AM 2 + BM 2 … (ii)
of the triangle drawn from vertex (from Q.No. 51) Also, AD 2 = AM 2 + DM 2 … (iii)
A on BC ? and r1 + r2 = 6 , r2 + r3 = 10, AB2 − AD 2 = ( AM 2 + BM 2 )
(a) 2.4 cm (b) 3 cm r1 + r3 = 8 − ( AM 2 + DM 2 )
(c) 4 cm (d) 4.8 cm ∴ r1 = 2 cm, r2 = 4 cm, r3 = 6 cm = AM + BM − AM 2 − DM 2
2 2

1 1
1. P = π r12 = × π × (2 )2 = π cm 2 = BM 2 − DM 2 = ( BM + DM ) ( BM − DM )
_ (d) In ∆ABC it is given that 4 4
AB = c = 6 cm, = (CM + DM ) ( BM − DM ) (from Eq. (i))
θ
BC = a = 10 cm 2. 9Q + 4R = 9 × 1 × π r22 + 4 = CD × BD
360°
Hence, AB2 − AD 2 = CD × BD Or
and θ
× 2 × π r32 AC 2 − AD 2 = CD × BD
CA = b = 8 cm 360°
θ [since, AB = AC ]
A = 9 × 1 × π ( 4)2
360°
θ
55. If AD = 5 cm, BD = 4 cm and CD = 6
+4 × 2 × π ( 6)2
360° cm, then what is AB equal to ?
b c
144 144 (a) 7 cm (b) 6.5 cm (c) 6 cm (d) 5.5 cm
= π θ1 + π θ2
360° 360° _ (a) Given,
144 AD = 5 cm, BD = 4 cm and CD = 6 cm
C M B = π ( θ1 + θ 2 )
a 360°
We have,
144
a+ b+c = π × 90° [Qθ1 + θ 2 = 90°] AB2 − AD 2 = CD × BD
s= 360°
2 [obtained in question (54)]
= 36 π cm2
where s is semi perimeter ⇒ AB − ( 5)2 = ( 6) × ( 4)
2
Hence, both 1 and 2 are correct.
10 + 8 + 6 ⇒ AB2 − 25 = 24
=
2 ⇒ AB2 = 24 + 25 = 49
24
Directions (Q.Nos. 54 and 55)
= = 12 cm Consider the following next two ⇒ AB = 49 = 7 cm
2
questions that follow. Hence, AB is equal to 7 cm.
∴Area of triangle
ABC is triangle in which AB = AC
= s( s − a ) ( s − b ) ( s − c ) Directions (Q.Nos. 56-58) Consider the
1 and D is any point on BC.
⇒ × BC × AM following for next three questions that
2 54. Which one of the following is follow.
= 12(12 − 10) (12 − 8) (12 − 6) correct? ABC is a triangle with AB = 16 . cm,
1 (a) AB2 − AD2 = AD × BD
⇒ × 10 × AM = 12 × 2 × 4 × 6 BC = 63. cm and CA = 65
. cm. Let P and Q
2
(b) AC 2 − AD2 = BD × CD be the mid-points of AB and BC

5 × AM = 4× 3×2 × 4×2 × 3 (c) AB2 − AD2 = 2 AD × BD respectively.
⇒ 5 × AM = 2 × 3 × 2 × 2 (d) AC 2 − AD2 = 2 BD × CD 56. What is AB 2 + 4 BQ 2 equal to ?
24
⇒ AM = _ (b) Given, (a) 41.25 cm2 (b) 42.25 cm2
5
ABC is a triangle in which (c) 43.75 cm2 (d) 44.25 cm2
⇒ AM = 4.8 cm
Hence, the length of the required AB = AC and D is any point on BC _ (b) Given,
altitude is 4.8 cm. Draw AM ⊥ BC In ∆ABC, AB = 16
. cm, BC = 6.3 cm
A and CA = 6.5 cm
53. If P, Q and R are the areas of
Q P is mid-point of AB.
sectors at A, B and C within the
∴ AP = BP … (i)
triangle respectively, then which
QQ is mid- point of BC.
of the following is/are correct
∴ BQ = QC … (ii)
1. P = πcm 2
A
2. 9Q + 4 R = 36πcm 2 1 2
B D M C
Select the correct answer using
the code given below: In ∆ABM and ∆ACM,
P
(a) 1 only AB = AC (given)
(b) 2 only ∠1 = ∠2 (90º)
(c) Both 1 and 2 and AM = AM (common)
Q
(d) Neither 1 nor 2 ⇒ ∆ABM ≅ ∆ACM
B C
∴ BM = CM [QCPCT] … (i)
12 CDS Solved Paper 2021 (II)

AB = 16
. 59. What is ∠BAP equal to ? _ (b) Let the cost of one pencil, one
⇒ AP + PB = 16
. (a) 30° (b) 40°
notebook and one eraser be x , y and z
respectively.
⇒ PB + PB = 16
. [Q AP = BP ] (c) 45° (d) 60°
Then, according to the question,
⇒ 2 PB = 16
.
_ (c) Given, 5x + 6 y + 7 z = 250 … (i)
⇒ PB = 0.8 … (iii) AB is a diameter of a circle with
and 6x + 4 y + 2 z = 180 … (ii)
BC = 6.3 centre O.
6 × Eq. (i) − 5 × Eq. (ii)
⇒ BQ + QC = 6.3 Radius OP is perpendicular to AB
and Q is any point on arc PB. 30x + 36 y + 42 z = 1500
⇒ BQ + BQ = 6.3 [Q BQ = QC ]
Since, chord PB makes ∠BOP at the 30x + 20 y + 10 z = 900
⇒ 2 BQ = 6.3
centre and ∠BAP at A. − − − −
6.3
⇒ BQ = = 315
. … (iv)
2 16 y + 32 z = 600
Now, AB2 + 4BQ 2 dividing the equation by 8, we get
= (16 . )2 = 2.56 + 4( 9.9225)
. )2 + 4( 315 2 y + 4 z = 75
O Hence, the cost of 2 notebooks and 4
= 2.56 + 39.69 = 42.25 cm2 A B
erasers is ` 75.
Hence, AB2 + 4 BQ 2 = 42.25 cm2

57. What is AQ 2 + CP 2 equal to? 62. How many zeros are there in the
2 2
Q product 1 50 × 2 49 × 3 48 ×K×501 ?
(a) AC (b) 12
. AC P (a) 262 (b) 261
(c) 1.25 AC 2 . AC 2
(d) 15 (c) 246 (d) 235
1
∴∠BAP = ∠BOP
_ (c) AQ + CP
2 2
2 _ (a) Given product :
= AB2 + BQ 2 + BP 2 + BC 2 [since, angle made by a chord at a 150 × 2 49 × 3 48 × … × 501.
AC 2 point present on the circle is half of We know one zero is created by
= AC 2 + PQ 2 = AC 2 + the angle made by chord on the multiplication of 2 and 5 i.e., one pair of 2
4
centre] and 5 in multiplication gives one zero.
= AC 2 + 025
. AC 2 = 1.25 AC 2 1
∠BAP = × 90º = 45° In the given multiplication, we will find
58. What is 4(CP 2 − AQ 2 ) equal to? 2 (2 × 5)n.
Hence, ∠BAP is equal to 45º. Here, n represents the number of zeros in
(a) 101.39 cm2 (b) 111.39 cm2
the expression.
(c) 121.39 cm2 (d) 131.39 cm2 60. What is ∠AQP equal to?
We know that, power of 2 will be more
_ (b) (a) 30° (b) 40° than power of 5.
Now, in ∆ABQ, (c) 45° (d) 60° Hence, we will find (2 × 5)n with the help of
AQ = AB + BQ = (16
2 2
. ) + ( 315
2
. ) 2 2 5 n only. Because all 5,
_ (c) Since, chord AP makes ∠AOP at n number of times will be paired with 2.
[from Eq. (iv) of question (56)] the centre and ∠AQP at Q.
∴ Number of 5 in product
= 2.56 + 9.9225
= 5 46 × 5 41 × 5 36 × 5 31 × ( 5 2 )26
= 12 .4825 cm2 … (v)
× 5 21 × 516 × 511 × 5 6 × ( 5 2 )1
In ∆BCP,
46 + 41 + 36 + 31 + 52 + 21 + 16 + 11 + 6 + 2
=5
CP 2 = BC 2 + PB2 = ( 6.3)2 + ( 0.8)2 O
A B = 5 262 = 5 n
[from Eq. (ii) of qusestion (56)]
n = 262
= 39.69 + 0.64 Hence, there will be 262 zeros in the
= 40.33 cm2 … (vi) given product.
Q
4(CP − AQ 2 )
2
P
63. If p and q ( p > q )are the roots of the
= 4 [40.33 − 12.4825] 1
= 4 [27.8475]
∴∠AQP = ∠AOP equation x 2 − 60x + 899 = 0 then
2
= 11139
. cm2 1 which one of the following is
= × 90º = 45º correct?
Hence, 4(CP 2 − AQ 2 ) equal to 2
. cm2.
11139 Hence, ∠AQP is equal to 45º. (a) p − q − 1 = 0
(b) p − 2q + 27 = 0
Directions 61. 5 pencils, 6 notebooks and
(Q.Nos. 59 and 60) (c) 2 p − q − 30 = 0
Consider the following for the next
7 erasers cost ` 250; whereas 6
(d) 3 p − 2q − 43 = 0
pencils, 4 notebooks and 2
two questions that follows.
erasers cost ` 180. What is the _ (b) Given quadratic equation,
AB is a diameter of a circle with x 2 − 60x + 899 = 0
cost of 2 notebooks and 4
centre O. Radius OP is perpendicular erasers? Roots of the equation are p and q, where
to AB. Let Q be any pont on are PB. p> q
(a) ` 90 (b) ` 75 (c) ` 60 (d) ` 40
CDS Solved Paper 2021 (II) 13

x 2 − 60x + 899 = 0  25  x 2 − 8 − ( − x 2 + 2 x + 4)
x − 1 = log10   ⇒2B =
⇒ x 2 − 31x − 29x + 899 = 0  10  x +2
⇒ x ( x − 31) − 29( x − 31) = 0 x − 1 = log10 25 − log10 10 x 2 − 8 + x 2 − 2x − 4
  m  ⇒2B =
⇒ ( x − 31) ( x − 29) = 0 Qlog  n  = log m − log n x +2
x = 31 or 29  
2 x 2 − 2 x − 12
x − 1 = log10 5 2 − 1 [Q log a a = 1] ⇒2B =
Here, p = 31 and q = 29 x +2
[Q p > q ] x = log10 5 2
x2 − x − 6
(a) p − q − 1 = 0 x = 2 log10 5 ⇒ B=
x +2
p − q − 1 = 31 − 29 − 1 [Q log a m = mlog a ]
x 2 − 3x + 2 x − 6
= 2 − 1= 1≠ 0 Hence, the value of x is 2 log10 5. =
x +2
(b) p − 2q + 27 = 0 8 48
66. If 96 − 64a 3 + − t 3 = 0, − ( x − 3) ( x + 2 )
p − 2q + 27 = 31 − 2 × 29 + 27 a a3 6 = =x −3
(x + 2 )
= 31 − 58 + 27 = 58 − 58 = 0 then what is a 2 t + 4a 3 equal to?
2x 2 − 7x + 3
Hence, (b) is the correct option. Option (c) is ,
(a) 0 (b) 1 2x − 1
64. If the roots of the equation (c) 2 (d) 3
On solving this further, we get
x 2 − 4 x − log10 N = 0 _ (c) Given, 2 x 2 − 6x − x + 3
8 48 =
96 − 64a + 3
− −t = 0
3
2x − 1
are real, then what is the a6 a3
2 x ( x − 3) − 1( x − 3)
minimum value of N? 8 48 =
⇒ 6
− 64a 3 − 3
+ 96 = t 3 (2 x − 1)
(a) 0.1 (b) 0.01 a a
(2 x − 1) ( x − 3)
(c) 0.001 (d) 0.0001 2 
3
2 
2
= =x −3
⇒  2  − ( 4a )3 − 3 2  × 4a (2 x − 1)
_ (d) Given equation, a  a 
2 Since, value of B and option (c)
x 2 − 4x − log10 N = 0 + 3 × 2 × ( 4a )2 = t 3
a represents the same value.
Roots of the equation will be real
3 2x 2 − 7x + 3
only, when the discriminant ( D ) ≥ 0. 2  Hence, B is equal to .
⇒  2 − 4a  = t 3 2x − 1
For ax 2 + bx + c = 0, D = b 2 − 4ac a 
For x 2 − 4x − log10 N = 0
⇒t =
2
− 4a ⇒ t =
2 − 4a 3 x4
68. What is
a = 1, b = − 4 and c = − log10 N a 2
a 2
( x 2 − y 2 )( x 2 − z 2 )
∴ b 2 − 4ac ≥ 0 [Q roots are real ⇒ a 2t = 2 − 4a 3
(given)] y4
⇒ a 2t + 4a 3 = 2 +
⇒ ( − 4)2 − 4(1) ( − log10 N ) ≥ 0 Hence, a 2 t + 4a 3 = 2 (y 2 − x 2 )(y 2 − z 2 )
⇒ 16 + 4 log10 N ≥ 0
x2 −8 z4
⇒ 4 log10 N ≥ − 16 67. If A + B = and − equal to?
⇒ log10 N ≥ − 4 x +2 (z 2 − x 2 )(z 2 − y 2 )
⇒ N ≥ (10)− 4 [on taking − x 2 + 2x + 4 (a) −1 (b) 0
A −B = , then what
anti-logarithm]
x +2 (c) 1 (d) x2 + y2 + z2
1
⇒ N≥ is B equal to ?
10000 _ (c) Given expression,
⇒ N ≥ 0.0001 x2 − 4 x2 − 4 x4
(a) (b)
Hence, the minimum value of N is x2 + 4 x + 4 x2 − 4 x + 4 ( x − y ) ( x 2 − z2 )
2 2

0.0001. 2x −7x + 3
2
2x + 7x − 3
2
y4
(c) (d) +
65. If 5 x − 1 = (25 2x − 1 2x − 1 ( y − x )( y 2 − z2 )
2 2
. ) log10 5 , then what is
the value of x ? _ (c) Given, z4
+
(a) 1 (b) log10 2 x −8 2 ( z − x ) ( z2 − y 2 )
2 2
A+ B= … (i)
(c) log10 5 (d) 2 log10 5 x +2 x4
=
_ (d) Given, 5 log10
( 2. 5)
= (2 .5)log10 5 − x + 2x + 4
2
( x − y ) ( x 2 − z2 )
2 2
and A − B = … (ii)
−1 log10 x +2 y4
⇒ 5x = ( 5) (2.5) −
By Eq. (i) − Eq. (ii), we get (x − y2 )
2

[Q alog( b ) = blog( a) ] ( y 2 − z2 )
( A + B) − A − B)
On comparing LHS and RHS, we get
x 2 − 8 ( − x 2 + 2 x + 4) z4
x − 1 = log10(2.5) = − +
x +2 x +2 ( x 2 − z2 ) ( y 2 − z2 )
14 CDS Solved Paper 2021 (II)

x 4 ( y 2 − z2 ) − y 4 ( x 2 − z2 ) x y The numbers are 2 × 6, 3 × 6 and 5 × 6


+ z4 ( x 2 − y 2 ) 70. If = i.e., 12, 18 and 30.
= b + c −a b − c −a
( x 2 − y 2 ) ( x 2 − z2 ) ( y 2 − z2 ) Sum of the numbers
z = 12 + 18 + 30 = 60
x (y − z ) − y x + y z
4 2 2 4 2 4 2
= = k then what is
+ z4 x 2 − z4 y 2 a +b −c Hence, sum of the numbers is 60.
=
( x 2 − y 2 ) ( x 2 − z2 ) ( y 2 − z2 ) x 2 + y 2 + z 2 − 2xy − 2yz + 2zx 72. Consider the following inequalities
x 4 ( y 2 − z2 ) − y 4 x 2 + z4 x 2 equal to ?
+ y 4 z2 − z4 y 2 a2 −b2 a −b
= (a) k 2 (a2 + b 2 + c 2 ) 1. > where a > b > 0
(x − y ) (x − z ) ( y − z )
2 2 2 2 2 2
a +b
2 2 a +b
(b) k 2 (a2 − b 2 + c 2 )
x 4 ( y 2 − z2 ) − x 2( y 4 − z4 )
(c) k 2 (a + b + c )2 a3 +b3 a2 +b2
=
+ y 2 z2( y 2 − z2 ) 2. > only when
( x 2 − y 2 )( x 2 − z2 )( y 2 − z2 ) (d) k 2 (a − b + c )2 a2 +b2 a +b
x 4 ( y 2 − z2 ) − x 2( y 2 − z2 ) a>b>0
_ (c) Given,
( y 2 + z2 ) + y 2 z2( y 2 − z2 ) Which of the above is/are correct ?
= x
=
y
( x 2 − y 2 ) ( x 2 − z2 ) ( y 2 − z2 ) b+ c − a b−c − a (a) 1 only
( y 2 − z2 )[x 4 − x 2 y 2 =
z
=k (b) 2 only
− x 2 z2 + y 2 z2 ] a+ b−c (c) Both 1 and 2
=
( x 2 − y 2 ) ( x 2 − z2 )( y 2 − z2 ) ⇒ x = k( b + c − a ), … (i) (d) Neither 1 nor 2
[x 4 − x 2 z2 − x 2 y 2 + y 2 z2 ] y = k( b − c − a ) … (ii) _ (a) Given inequalities:
=
( x 2 − y 2 ) ( x 2 − z2 ) and z = k( a + b − c ) … (iii) a2 − b 2 a−b
1. > where a > b > 0
x (x − z ) − y (x − z )
2 2 2 2 2 2 x + y + z − 2 xy − 2 yz + 2 zx
2 2 2
a2 + b 2 a+ b
=
( x 2 − y 2 ) ( x 2 − z2 ) = ( y − x − z)2 i.e., a and b are positive and a > b.
= k ( b − c − a ) − k( b + c − a ) (a − b ) (a + b ) (a − b )
( x 2 − z2 ) ( x 2 − y 2 ) ⇒ >
= − k( a + b − c )]2 ]
( x 2 − y 2 ) ( x 2 − z2 ) a2 + b 2 (a + b )
[from Eq. (i), Eq. (ii) and Eq. (iii)]
a+ b 1
=1 = k 2[b − c − a − ( b + c − a ) ⇒ >
a2 + b 2 a+ b
Hence, the given expression is equal − ( a + b − c )]2
to 1. [Q( a − b ) is a positive number]
= k [b − c − a − b − c + a − a
2
⇒ ( a + b )2 > a 2 + b 2
69. If (2ab − b ) : (6a − ab ) = 1 : 6,
2 2
− b + c] 2
⇒ a 2 + b 2 + 2 ab > a 2 + b 2
then what is (a + b ) : (a − b ) equal = k 2[b − b − b − c − c + c
⇒ 2 ab > 0
to? − a + a − a]2
which is true. [Qa > 0 and b > 0]
(a) 3 only = k 2[− b − c − a ]2
a3 + b 3
a2 + b 2
(b) 5 only = k 2( a + b + c )2 2. > only when
a2 + b 2 a+ b
(c) −3 or 3 Hence, x 2 + y 2 + z2 − 2 xy
− 2 yz + 2 xz = k 2( a + b + c )2 a> b> 0
(d) −5 or 5
Since, a and b are positive.
2 ab − b 2 1 71. If three positive numbers are in
_ (d) = ⇒ (a + b ) (a3 + b 3 )
6a − ab
2
6 the ratio 2 : 3 : 5 and the sum of
> (a2 + b 2 ) (a2 + b 2 )
⇒ 12 ab − 6b = 6a − ab 2 2 their squares is 1368, then what
⇒ a + ab + ba 3 + b 4
4 3
⇒ 6a 2 − 13ab + 6b 2 = 0 is sum of the numbers?
> a 4 + b 4 + 2 a 2b 2
⇒ 6a 2 − 9ab − 4ab + 6b 2 = 0 (a) 30 (b) 45
⇒ ab 3 + ba 3 > 2 a 2b 2
⇒ 3a(2 a − 3b ) − 2 b(2 a − 3b ) = 0 (c) 60 (d) 75
⇒ ab( b + a ) − 2 a 2b 2 > 0
2 2
⇒ (2 a − 3b ) ( 3a − 2 b ) = 0 _ (c) Given, the ratio of three positive
numbers is 2 : 3 : 5. ⇒ ab[b 2 + a 2 − 2 ab ] > 0
a 3 a 2
= , = ⇒ ab ( a − b )2 > 0
b 2 b 3 Let three numbers be 2 a, 3a and 5a.
a+ b 3+2 According to the question, ∴ If a and b are positive then the given
= inequalities forms into ab( a − b )2 > 0,
a−b 3−2 (2 a ) + ( 3a ) + ( 5a ) = 1368
2 2 2
which is true but here, it is not necessary
a+ b 2 + 3 ⇒ 4a 2 + 9a 2 + 25a 2 = 1368 that a > b.
or =
a−b 2 −3 ⇒ 38a 2 = 1368 The inequality is true for b > a > 0 also.
= 5 or − 5 ⇒ a2 =
1368
= 36 i.e., a > b > 0 is not the only condition.
a+ b 38 Hence, option (a) is correct.
Hence, is equal to − 5 or 5.
a−b ⇒ a=6
CDS Solved Paper 2021 (II) 15

73. Let work done by (3n − 1) men in From Eq. (i), we get _ (a) Given,
ay − bx = ck … (ii)
(2n + 1) days be x and work done Area of rectangular garden
by (3n + 1) men in ( 4n − 3) days be cx − az = bk … (iii) = 375m2 … (i)
y. If x : y = 6 : 11, then what is the bz − cy = ak … (iv) Sum of the lengths of three sides of
value n ? Multiplying Eq. (ii) by c, we get garden = 65 m … (ii)

(a) 6 (b) 7 acy − bcx = c 2k … (v) Let length and breadth of the garden be x
and y m.
(c) 8 (d) 9 Multiplying Eq. (iii) by b, we get
x
bcx − abz = b 2k … (vi)
_ (b) Let all the men do the same
amount of work in one day and one Multiplying Eq. (iv) by a, we get
day work of each men = 1unit; we get abz − acy = a 2k … (vii) y y
Amount of work done by ( 3n − 1) men Adding Eq. (v), Eq. (vi) and Eq. (vii),
in (2 n + 1) days we get x
= x = ( 3n − 1) (2 n + 1) units of work. acy − bcx + bcx − abz + abz
Similarly, amount of work done by − acy = k( a 2 + b 2 + c 2 ) Then,
( 3n + 1) men in ( 4n − 3) days ⇒ 0 = k( a 2 + b 2 + c 2 ) x × y = 375 m2 [from Eq. (i)] … (iii)
= y = ( 3n + 1) ( 4n − 3) units of work. Since, a, b and c are not zero. x + y + x = 65 [from Eq. (ii)]
According to the question, ∴ k=0 ⇒ 2 x + y = 65
x 6
= Put k = 0 into Eq. (ii), Eq (iii) and Eq. ⇒ y = 65 − 2 x … (iv)
y 11 (iv), we get From Eq. (iv), put y = 65 − 2 x into Eq.
( 3n − 1) (2 n + 1) 6
⇒ = *1) ay = bx ⇒ =
x y
… (viii) (iii), we get
( 3n + 1) ( 4n − 3) 11 a b x ⋅ ( 65 − 2 x ) = 375
6n 2 + 3n − 2 n − 16 *2) cx = az ⇒ =
x z
… (ix) ⇒ 65x − 2 x 2 = 375
⇒ =
12 n2 − 9n + 4n − 3 11 a c
⇒ 2 x 2 − 65x + 375 = 0
z y
6n + n − 1
2
6 *3) bz = cy ⇒ = … (x) ⇒ 2 x − 50x − 15x + 375 = 0
2
⇒ = c b
12 n2 − 5n − 3 11 x y ⇒ 2 x ( x − 25) − 15( x − 25) = 0
Eq. (viii) says = is correct.
⇒ a b ⇒ ( x − 25) (2 x − 15) = 0
66n2 + 11n − 11 = 72 n2 − 30n − 18 Now, ⇒ x = 25 or
15
⇒ 0 = 6n − 41n − 7
2
z⋅ a 2
From Eq. (ix), x = and from Eq.
⇒ 6n2 − 42 n + n − 7 = 0 c When x = 25
z⋅ b ⇒ y = 65 − 2 × 25
⇒ 6n( n − 7 ) + 1( n − 7 ) = 0 (x), y =
c
⇒ ( n − 7 ) ( 6n + 1) = 0 [from Eq. (iv))]
x + y+ z
1 Put into , we get ⇒ y = 15
n = 7, n ≠ − a+ b+c
6 Perimeter = 2( x + y) = 2(25 + 15)
z⋅ a z⋅ b = 2( 40) = 80 m
[Qnumber of men can’t be negative] + + z
Hence, the value of n is 7. ⇒ c c 15
a+ b+c When, x =
2
ay − bx cx − az bz − cy z⋅ a + z⋅ b + z⋅c
74. If = = , ⇒ y = 65 − 2 ×
15
c b a = c 2
then which of the following is/are a+ b+c
⇒ y = 50
correct ? z( a + b + c ) 1 z  15 
x y = ⋅ = Perimeter = 2( x + y) = 2  + 50
1. = c (a + b + c ) c 2 
a b Hence, both the Statements 1 and 2 = 15 + 100 = 115m
x +y +z z
2. = are correct. As per the given options, perimeter of the
a +b +c c garden is 80 m.
75. A person wishes to fence 375 m 2
Select the correct answer using rectangular garden. He has 65 m
the code given below: of barbed wire and is able to
Directions (Q.Nos. 76-79) Consider the
(a) 1 only (b) 2 only fence only three sides of the following for next four questions that
(c) Both 1 and 2 (d) Neither 1 nor 2 garden. What is the perimeter of follow.
the garden? The following pie charts show the
_ (c) Let percentage of different categories
ay − bx cx − az bz − cy (a) 80 m
= = =k ( A, B, C, D, E and F) of employees in a
c b a (b) 84 m
… (i) (c) 90 m company in the year 2019 and 2020. The
In the given condition we know that (d) 100 m total number of employees was 4000 in
a, b and c can not be zero. 2019 and 5000 in 2020.
16 CDS Solved Paper 2021 (II)

F A F A 78. What was the percentage increase of category-E


22% 20% 21% 22% employees in 2020 as compared to that in 2019?
B (a) 20% (b) 25% (c) 30% (d) 40%
6% 10% B
27% 10% E 27% _ (b) It is already solved in Q. (77).
E C 11%
15% The percentage increase of category-E employees in 2020 as
9% C compared to that in 2019 is 25%.
D D Hence, option (b) is correct.
2019 2020
79.There are two categories of employees whose strength
increased by same percentage in 2020. What are the
76. There are two categories of employees in 2019 whose two categories
total strength remained the same in 2020. What are
(a) E and F (b) C and E (c) A and E (d) A and C
these two categories ?
(a) C and B (b) E and A (c) E and F (d) C and D _ (d) It is already solved in Q. (77).
For category-A and category -C of employees strength
_ (d) Given pie chart, increased by same percentage i.e., 37.5%.
In 2019, Strengths are : In 2020, Strengths are Hence, option (d) is correct.
20 22
A: × 4000 = 800 A: × 5000 = 1100 Directions (Q.Nos. 80-83) Consider the following for the
100 100
next four questions that follow .
6 10
B: × 4000 = 240 B: × 5000 = 500 The following Bar chart gives the production of cars by
100 100
three different companies X, Y and Z in different years
10 11
C: × 4000 = 400 C: × 5000 = 550 800 X Y Z
100 100
15 9 700
D: × 4000 = 600 D: × 5000 = 450
100 100 600
Number of Cars Produced

27 27
E: × 4000 = 1080 E: × 5000 = 1350 500
100 100
400
22 21
F: × 4000 = 880 F: × 5000 = 1050 300
100 100
200
In 2019 ⇒ Strength of C and D = 400 + 600 = 1000
100
In 2020 ⇒ Strength of C and D = 550 + 450 = 1000
Hence, option (d) is correct. 0
2014 2015 2016 2017 2018
77. In which one of the following categories of employees, YEARS
the percentage change in number of employees in 2020
was maximum as compared to 2019? 80. The percentage increase in the total production of cars
from 2014 to 2018 was
(a) A (b) B (c) C (d) D
(a) 50% (b) 100% (c) 150% (d) 200%
_ (b) Percentage change in number of employees
Final value − initial value _ (b) Production of cars in 2014,
= × 100 X : 300 cars
Initial value
1100 − 800 Y : 200 cars
A : Percentage change = × 100 = 37.5%
800 Z : 500 cars
500 − 240 Total production of cars in 2014 = 300 + 200 + 500 = 1000
B : Percentage change = × 100 = 108.3%
240 Production of cars in 2018,
550 − 400
C : Percentage change = × 100 = 37.5% X : 700
400
Y : 500
450 − 600
D : Percentage change = × 100 = − 25% Z : 800
600
1350 − 1080 Total production of cars in 2018 = 700 + 500 + 800 = 2000
E : Percentage change = × 100 = 25% Percentage increase
1080
1050 − 880 Final value − initial value
F : Percentage change = × 100 = 19.3% = × 100
880 initial value
2000 − 1000 1000
Hence, the percentage change in number of employees is = × 100 = × 100 = 100%
maximum for B. 1000 1000
Hence, option (b) is correct.
CDS Solved Paper 2021 (II) 17

81. Consider the production of each company separately. In how many We can see that the percentage increase in
instances was the percentage increase in production of cars over the production of cars is minimum in 2018 as
previous year’s production greater than 20% ? compared to its previous year.
(a) 5 (b) 6 (c) 7 (d) 8 83. In one of the years the percentage
_ (a) Percentage increase in production of cars over the previous year’s production increase in production of cars was
Company X : minimum as compared to its previous
year. What was the minimum
Year 2014 to 2015 2015 to 2016 2016 to 2017 2017 to 2018 percentage ?
Percentage 400 − 300 500 − 400 600 − 500 700 − 600 1
(a) 11 %
2
(b) 11 %
increase 300 400 500 600 9 9
× 100 × 100 × 100 × 100
1
(c) 13 % (d) 20%
= 20% 3
= 33.3% = 25% = 167
. %
_ (a) It is already solved in Q. (82).
For company X, percentage increase in production is greater than 20% at two The minimum percentage increase in
instances. production of cars as compared to its
1
Company Y : previous year is 11.1% i.e., 11 %.
9
Year 2014 to 2015 2015 to 2016 2016 to 2017 2017 to 2018 Hence, option (a) is correct.
Percentage 300 − 200 400 − 300 500 − 400 500 − 500
increase 200 300 400 500 Directions (Q.Nos. 84 and 85) C onsider
× 100 × 100 × 100 × 100 the following for the next two questions that
follow.
= 50% = 33.3% = 25% = 0%
Collect all the sequences of five consectutive
For company Y, percentage increase in production is greater than 20% at three integers such that their product is equal to
instances. one of these integers. Let X be the collection
Company Z : of all possible such sequences. Let P be the
Year 2014 to 2015 2015 to 2016 2016 to 2017 2017 to 2018 smallest integer and Q be the largest integer
occuring in these sequences.
Percentag 500 − 500 600 − 500 700 − 600 800 − 700
e increase 500 500 600 700 84. How many such sequences of five
× 100 × 100 × 100 × 100 consecutive integers are possible ?
= 0% = 20% = 167
. % = 14.3% (a) One (b) Two
(c) Three (d) Five
For company Z, percentage increase in production is greater than 20% at zero
instances.
_ (d) Let five consecutive integers be in the
form ( x − 2 ), ( x − 1), x , ( x + 1) and ( x + 2 ).
Hence, in total 5 instances, the percentage increase in production of cars over Since, it is given that the product of these five
the previous year’s production greater than 20%. consecutive integers is equal to one of these
integers. This is only possible when one of the
82. In which year was the percentage increase in production of cars integer is zero.
minimum as compared to its previous year?
Hence, all possible sequences are
(a) 2015 (b) 2016 (c) 2017 (d) 2018
(i) − 4, − 3, − 2, − 1, 0
_ (d) From the given Bar-chart, we have (ii) − 3, − 2, − 1, 0, 1
Year 2014 2015 2016 2017 2018 (iii) − 2, − 1, 0, 1, 2
(iv) − 1, 0, 1, 2, 3
Production 300 + 200 400 + 300 500 + 400 600 + 500 700 + 500 and (v) 0, 1, 2, 3, 4
of Cars + 500 + 500 + 600 + 700 + 800 = 2000
Hence, number of required sequences will be
= 1000 = 1200 = 1500 = 1800 five.

Now, percentage increase in the production 85. What is the arithmetic mean of
P and Q?
Year 2014 to 2015 2015 to 2016 2016 to 2017 2017 to 2018 (a) 0
Percentag 1200 − 1000 1500 − 1200 1800 − 1500 2000 − 1800 (b) 1
e increase 1000 1200 1500 1800 (c) 2
× 100 = 20% = 20% = 20% (d) Cannot be determined due to insufficient
= 20% = 25% = 20% = 11.1% data
18 CDS Solved Paper 2021 (II)

_ (d) Given,  3
2
 3 From Eq. (ii), put y = x − 7 into Eq. (i), we
a   − 4a   + 15 = 0
P is the smallest integer. 2 2 get
9 x(x − 7 ) = 8
Q is the largest integer. ⇒ a⋅ − 2 ⋅ a ⋅ 3 + 15 = 0
4 ⇒ x 2 − 7x − 8 = 0
Now, Arithmetic mean (AM),
P+Q 9a ⇒ x 2 − 8x + x − 8 = 0
AM = ⇒ − 6a + 15 = 0
2 4 ⇒ x ( x − 8) + 1( x − 8) = 0
9a − 24a + 60 ⇒ ( x − 8) ( x + 1) = 0 ⇒ x = 8 or − 1
(i) − 4, − 3, − 2, − 1, 0 ⇒ =0
4 From Eq. (ii),
[from previous solution]
⇒ − 15a + 60 = 0 when x = 8
P = − 4, Q = 0
−4+ 0 ⇒ a=
60
=4 ⇒ y= 8−7 =1
AM = = −2 15
2 When x = − 1
The quadratic equation is ⇒ y = − 1− 7 = − 8
(ii) − 3, − 2, − 1, 0, 1
4x 2 − 16x + 15 = 0 We get( x , y) = ( 8, 1) and( x , y) = ( − 1, − 8)
P = − 3, Q = 1
− 3+ 1 −2 ⇒ 4x 2 − 10x − 6x + 15 = 0 Since, the digits of the number can’t be
AM = = = −1
2 2 ⇒ 2 x (2 x − 5) − 3(2 x − 5) = 0 negative.
(iii) − 2, − 1, 0, 1, 2 ⇒ (2 x − 5) (2 x − 3) = 0 So, we must remove second pair.
P = − 2, Q = 2
5
x = or
3 Therefore, the number is 10 × 1 + 8 = 18
−2 + 2 0 2 2 Hence, sum of the digits in the number is
AM = = =0 1 + 8 = 9.
2 2 Sum of the squares of the roots
2 2
(iv) − 1, 0, 1, 2, 3  5  3 88. A motor boat has speed 30 km/h in
=  +  
P = − 1, Q = 3 2 2
still water. It goes 60 km down
− 1+ 3 =
25
+
9 9
AM = stream and comes back in h. What
2 4 4 2
2 34 17
= =1 = = is the speed of the stream ?
2 4 2
(a) 5 km/h
(v) 0, 1, 2, 3, 4 Hence, sum of the squares of the
17 (b) 8 km/h
P = 0, Q = 4 roots is .
2 (c) 10 km/h
0+ 4
AM = =2 (d) 12 km/h
2 87. A two-digit number is such that
Arithmetic mean of P and Q have five the product of the digits is 8. If 63 _ (c) Let speed of stream be x km/h.
different values. As no particular is added to this number, the Speed of boat downstream
condition is given. digits interchange their places. = ( 30 + x ) km/h
Therefore, we cannot mark any of the What is the sum of the digits in Speed of boat upstream = ( 30 − x ) km/h
mean with this insufficient Distance travelled downstream
the number ?
information.
(a) 6 (b) 7 = 60 km
Hence, option (d) is correct.
(c) 8 (d) 9 Distance travelled upstream
86. If one of the roots of the equation = 60 km
3 _ (d) Let the digits at units and tens
ax − 4ax + 15 = 0 is , then what
2 place of the given number be x and y Time taken by boat in upstream
2 respectively. distance travelled upstream
=
isthe sum of the squares of the Thus, the number is 10 y + x . Speed upstream
roots? The product of the digits is 8. 60
=
15 17 ∴ xy = 8 … (i) ( 30 − x )
(a) (b)
2 2 After interchanging the digits, the Time taken by boat in downstream
19 21 number becomes 10x + y.
(c) (d) distance travelled downstream
2 2 =
If 63 is added to the number, the Speed downstream
_ (b) Given quadratic equation, digits interchange their places.
60
=
ax 2 − 4ax + 15 = 0 … (i) Thus,
30 + x
3 (10 y + x ) + 63 = 10x + y
One root of the equation is . 9
2 ⇒ 10x + y − 10 y − x = 63 Total time taken = h
2
3 ⇒ 9x − 9 y = 63
Put x = into Eq. (i), we get 9 60 60
2 = +
⇒ x − y=7 … (ii) 2 30 − x 30 + x
CDS Solved Paper 2021 (II) 19

9 60( 30 + x ) + 60( 30 − x ) Then, denominator = 10 − x Now, Sum = P


⇒ =
2 900 − x 2 According to the question, A = 4P
9 3600 x + 3 Time = n years
⇒ = =1
2 900 − x 2 (10 − x ) − 1
Again using, A = P  1 +
r 
n

x + 3 
1 400 ⇒ =1  100 
⇒ =
2 900 − x 2 9−x
 r 
n
⇒ 4P = P  1 + 
⇒ x + 3=9−x  100 
⇒ 900 − x = 800
2

⇒ 2x = 6 ⇒ 4 = (2 1/ 5 )n [From Eq. (i)]


⇒ x 2 = 100
⇒ x =3 ⇒ 2 2 = 2 n/ 5
⇒ x = 10 km/h
∴Numerator = 3 and denominator On comparing the powers with same
Hence, the speed of the stream is
10 km/h. = 10 − 3 = 7 base, we get
∴Difference n
2 = ⇒ n = 10
89. The present age of a father is = Denominator − Numerator 5
equal to sum of the ages of his =7 − 3= 4 Hence, the sum will become four times in
4 children. After ten years the Hence, the required difference is 4. 10 yr.
sum of the ages of the children
will be 1.6 times the age of their 91. If the system of equations 93. Between 3 and 4 O’clock, both hour
father. What is the present age of 7 x + ky = 27 and kx +`7y = 19 hand and minute hand will coincide
father? have unique solution, then which past 3 O’clock between
(a) 36 yr
one of the following is correct (a) 15-16 min (b) 16-17 min
(b) 40 yr (a) k ≠ 7 (b) k ≠ 13 (c) 17-18 min
(c) 42 yr (c) k = 7 (d) = 13 (d) 18-19 min
(d) 45 yr _ (a) Given system of equations, _ (b) In the clock
7 x + ky = 27 … (i) θ = 90 + φ
_ (b) Let the age of father be x years
and sum of the ages of his 4 children and kx + 7 y = 19 … (ii) A
be y years. We know two equation
⇒ x = y … (i) a1x + b1 y = c1 and a2x + b 2 y = c 2
After 10 years, father’s age = x + 10 will have unique solution if θ
7 k O φ B
and sum of ages of 4 children a1 b
≠ 1⇒ ≠
= y + 40 a2 b 2 k 7
C
According to the question, ⇒ 7 × 7 ≠ k × k ⇒ k2 ≠ 7 2
y + 40 = 16
. ( x + 10) ⇒ k≠±7 .
⇒ x + 40 = 16
. ( x + 10) (from Eq. (i)) Hence, k ≠ 7 Where, θ → angle travelled by minute
⇒ x + 40 = 16
. x + 16 hand
92. A sum of money compounded φ → angle travelled by hour hand
⇒ 40 − 16 = 16
. x −x
annually doubles itself in 5 yr. In We know that, 360º of minute hand = 30º
⇒ 24 = 0.6x
how many years will it become of hour hand
24
⇒x = = 40 four times of itself? φ 30º
0.6 ∴ =
(a) 10 years (b) 12 years θ 360º
Hence, the present age of the father
(c) 15 years (d) 20 years θ
is 40 yr. ⇒ φ=
12
90. The sum of nummerator and _ (a) Let the sum be P and interest rate
be r% per annum. ⇒ θ = 12 φ
denominator of a fraction is 10. If Time, n = 5 yr ⇒ 90 + φ = 12 φ [Qθ = 90 + φ]
the numerator is increased by 3 (Compounded annually) ⇒ 90 = 11φ
and denominator is decreased by A = 2P 90
1, the fraction becomes 1. What is ⇒ φ= ≈ 8.2
As we know the formula, 11
the difference between
 r 
n
∴12 φ = 12 × 82
. = 98.4
numberator and denominator of A = P1 + 
 100  360º angle means → 60 min.
the fraction?
 r 
5 96º angle means → 16 min
(a) 2 (b) 3 ⇒ 2 P = P1 + 
(c) 4 (d) 5  100  102º angle means → 17 min
5 ∴98.4º will be meant between
_ (c) Given, 
⇒ 2 = 1 +
r 
 16 to 17 min.
Sum of numerator and denominator  100 
Hence, option (b) is correct.
of fraction is 10. r
⇒ 2 1/ 5 = 1 + … (i)
Let the numerator be x. 100
20 CDS Solved Paper 2021 (II)

94. Sheela can stitch a suit in 2 days, 96. If p varies directly as q and Now, according to the question,
while Meena can stitch a suit in 500x − 100( 30 − x ) = 11400
inversely as square of r , what is
1 the percentage increase in p due ⇒ 500x − 3000 + 100x = 11400
1 days. How many days will
2 to an increase in q by 20% and a ⇒ 600x = 11400 + 3000
both take in stitching 30 suits? decrease in r by 20% ? ⇒ 600x = 14400
14400 144
(a) 32 days (a) 87.5% (b) 85% ⇒ x = = = 24
600 6
(b) 33 days (c) 82.5% (d) 80%
Hence, the person worked for 24 days.
(c) 35 days
_ (a) Given, p varies directly as q and
(d) 40 days inversely as square of r. 98. A person bought a chair and a table
_ (*) Given, Hence,
q
p = 2, Assuming
for ` 750. He sold the chair at a gain
Sheela stitches a suit in 2 days. r of 5% and the table at a gain of 20%.
1 proportional constant as 1. he gained 16% on the whole. What is
∴Sheela’s 1 day’s work = suit
2 q is increased by 20% and r is original cost of table?
3 decreased by 20%.
Meena stitches a suit in days. (a) ` 400 (b) ` 450 (c) ` 550 (d) ` 600
2 Hence, value of p becomes,
120
. q _ (c) Let cost of chair be x and cost of table
∴Meena’s 1 day’s work p= be y.
1 2 ( 8r )2
= = suit. Then, x + y = 750 … (i)
3/2 3 120q
p= 2 By selling the chair at a gain of 5% and the
Sheela and Meena’s 1 day’s work 64r
table at a gain of 20%, there is a profit of
1 2 Change in value of
= + 16% on the whole.
2 3 120q q 56 q
p= − 2= ⋅ Then, ( x + 5% of x)+ ( y + 20% of y)
3+ 4 7 64r 2 64 r 2
= = suit r = 750 + (16% of 750)
6 6 Increase in P’s percentage  5   20 
⇒ x + × x +  y + × y
Let number of days will both take in  56 q   100   100 
stitching 30 suits  ⋅ 
 64 r 2 
6 180 = × 100 = 750 +
16
× 750
= 30 × = q  100
 2
7 7 r 
⇒ 100x + 5x + 100 y + 20 y
Hence, the required number of days 56
are 35 days. = × 100 = 75000 + 12000
64
⇒ 105x + 120 y = 87000 … (ii)
95. If 2x − 3y − 7 = 0 7
= × 100 From Eq. (i), put y = 750 − x into
8
then what is the value of Eq. (ii), we get
700
= = 87.5% ⇒ 105x + 120(750 − x ) = 87000
8x 3 − 36x 2 y + 54 xy 2 8
⇒ 105x + 90000 − 120x = 87000
− 27y 3 − 340? Hence, percentage increase in p is
87.5%. ⇒ 90000 − 87000 = 120x − 105x
(a) −1 (b) 0 ⇒ 3000 = 15x
(c) 1 (d) 3 97. A person agrees to work for 30
⇒ x = 200
days, on a condition that for
_ (d) Given, 2 x − 3 y − 7 = 0 ∴ y = 750 − 200 [from Eq. (i)]
every day’s work he should
2x − 3y = 7 = 550
receive ` 500, and that for every
On taking power 3 both the sides, day’s absence from work he Hence, the original cost of the table is
we get ` 550.
should forfeit ` 100. At the end of
(2 x − 3 y)3 = 7 3
the time he received ` 11,400. 99. A person rode one third of a journey
⇒ (2 x )2 − ( 3 y)3 − 3(2 x )2( 3 y) How many days did he work?
+ 3(2 x ) ( 3 y)2 = 343
at 60 km/hr, one third at 50 km/hr
(a) 20 (b) 21 and the rest at 40 km/hr. Had the
[Q( a − b ) = a − b
3 3 3
(c) 24 (d) 25 person ridden half of the journey at
− 3a 2b − 3ab 2]
60 km/hr and the rest at 40 km/hr, he
⇒ 8x 3 − 27 y 3 − 36x 2 y _ (c) Let the person work for x days.
∴Number of days he does not work
would have taken 4 minutes longer
+ 54xy 2 − 343 = 0 to complete the journey. What
= 30 − x
⇒ 8x 3 − 36x 2 y + 54xy 2 distance did the person ride?
Salary he gets for working x days
− 27 y 3 − 340 = 3 = ` x × 500 = ` 500 x (a) 180 km (b) 210 km
Hence, the value of the given Fine for the day he does not work (c) 240 km (d) 300 km
expression is 3. = ` 100( 30 − x )
CDS Solved Paper 2021 (II) 21

4
_ (c) Let the total distance be 3d. T2 − T1 = h (given) The given problem forms the following
Distance 60 AP,
Time = 25d 37d 4
speed ⇒ − = AP : 2000, 3000, 4000, …
400 600 60 First term, a = 2000
According to the question,
75d − 74d 4
d ⇒ = Common difference,
t1 = 1200 60
60 d = 3000 − 2000 = 1000
d
d ⇒ = 4 ⇒ d = 80 Sum of n terms, S n = 170000
t2 = 20
50 n
∴ 3d = 3 × 80 = 240 km We know, S n = [2 a + ( n − 1)d ]
t3 =
d 2
40
Hence, the required distance is
n
240 km. ⇒ [2 a + ( n − 1)d ] = 170000
T1 = t 1 + t 2 + t3 2
=
d
+
d
+
d 100. A person saves ` 100 more than n
⇒ [2 × 2000 + ( n − 1) (1000)]
60 50 40 he did the previous year. If he 2
d 1 1 1 saves ` 2000 in the first year, in = 170000
=  + + 
10  6 5 4 how many years will he save ⇒ n[4000 + ( n − 1) (1000)] = 340000
d  10 + 12 + 15  ` 170000 ⇒ n[4 + ( n − 1)] = 340
=  
10  60  (a) 16 yr ⇒ n( n + 3) = 340
37d (b) 17 yr ⇒ n2 + 3n − 340 = 0
=
600 (c) 18 yr ⇒ n2 + 20n − 17 n − 340 = 0
15
. d 15
. d (d) 19 yr
T2 = + ⇒ n( n + 20) − 17( n + 20) = 0
60 40
_ (b) Given, ⇒ ( n + 20) ( n − 17 ) = 0
15
. d  1 1  15d  2 + 3
=  + =   A person saves ` 1000 more than he n ≠ − 20, n = 17
10  6 4  100  12 
did the previous year. Hence, the person saves ` 170000 in
25d
= Savings in the first year is ` 2000. 17 yr.
400
PAPER II English
Directions (Q. Nos. 1-10) Given 6. Turncoat The middle four sentences in each have
below are some idioms/phrases (a) Expert at altering coats been jumbled up and labelled as P, Q,
followed by four alternative meanings (b) Someone who deserts one group R and S. You are required to find the
to each. Choose the most appropriate to join another proper sequence of the four sentences.
answer from among (a), (b), (c) or (d). (c) A truly dishonest person
(d) Going round and round in a court 11. S1: An ideal citizen is one who
1. Forty winks of law establishes his standard in
(a) Winking forty times _ (b) ‘Turncoat’ refers to someone who everything.
(b) Sleep disorders switches to an opposite side. Hence,
(c) Long sleeping hours
S6: He can be called a thorough
option (b) is the correct answer.
(d) A short sleep during the day gentleman.
7. Say your piece
_ (d) The idiom ‘forty winks’ means a P : Because he is a true patriot.
short sleep or nap, especially during (a) Speak but don’t listen
(b) Speak in a talkative manner Q : Also, he can lay down his life for
daytime.
(c) Make your argument piece by the honour of his country.
2. Life in the raw piece R: That he can make any sacrifice
(a) Life in its natural, unembellished state (d) Express your opinion
(b) Life at its easiest
for his motherland.
_ (d) The idiom ‘say your piece’ means S : He loves his country so much so
(c) Life that is complex to state your opinions.
(d) A daredevil’s life The correct sequence should be
8. Top-notch
_ (a) The idiom ‘life in the raw’ means (a) The highest marking on a tree or a (a) S R Q P (b) R P Q S
life in its unnatural, unembellished (c) P Q R S (d) Q R S P
state. building
(b) Person or organisation that attracts _ (a) S R Q P
3. A paper tiger only the top
(a) A person or thing that appears 12. S1: There are multiple legal
(c) Of highest possible quality
threatening but is ineffectual (d) Person of integrity avenues for patients to indict
(b) To threaten someone but do no doctors, but rarely is it the other
harm _ (c) ‘Top-notch’ means that something
is of very high standard or quality. way round.
(c) Environmental protection
paperwork 9. Under the table S6: It is clear that patients have
(d) To speak to people in a roaring (a) Work under difficult circumstances multiple avenues to pursue, should
voice (b) Working undercover they feel an injustice has been
_ (a) The idiom ‘a paper tiger’ refers to (c) Working under furniture perpetrated against them.
someone who seems powerful and (d) Making or receiving payments
dangerous at the first glance, but is P : An FIR can be lodged against
surreptitiously
actually powerless and harmless. them under the section 304A of the
_ (d) The idiom ‘under the table’ means Indian Penal Code.
4. Penny-wise and pound-foolish to do something secretly because it is
(a) Wise to spend each penny carefully illegal. Hence, option (d) is the correct Q : A compensation case can be
(b) Careful about small amounts but anwer.
filed in a consumer forum.
careless about large amounts 10. At the drop of a hat R : For one occurrence, there are
(c) People who don’t understand the (a) Clumsy person who drops hats
importance of each penny multiple forums where doctors
everywhere
(d) Careful about large amounts but (b) Suddenly and without much
have to defend themselves.
careless about small amounts thought S : For instance, a complaint can be
_ (b) The idiom ‘penny-wise and (c) Do something without much made to their employer.
pound-foolish’ means to be careful pressure
about small amounts but not about (d) A happy and easygoing man The correct sequence should be
large amounts. (a) P Q R S (b) P S R Q
_ (b) The idiom ‘at the drop of a hat’ (c) R S Q P (d) S R Q P
5. Pin back your ears means immediately and without any
(a) To listen carefully to something thought. Hence, option (b) is the _ (c) R S Q P
(b) Person or organisation that pins correct answer
13. S1: Paragraphs are the building
important issues
Directions (Q. Nos. 11-20) In this blocks of any write-up.
(c) To keep yourself away from hearing
bad stuff section, each question consists of six S6: Understanding of these makes
(d) To clean your ears with a pin sentences of a passage. The first and one a good writer of paragraphs.
_ (a) The idiom ‘pin back your ears’ sixth sentences are given in the P : A paragraph need not be pages
means to listen carefully to beginning as S1 and S6. together in length.
something.
CDS Solved Paper 2021 (II) 23

Q : But actually a paragraph is a (a) S Q R P (b) P Q S R The correct sequence should be


group of at least five sentences on (c) Q S R P (d) R P S Q (a) P R S Q (b) P Q S R
the lower limit. _ (a) S Q R P (c) S R Q P (d) R S Q P
R : In reality, unity and coherence, 16. S1: Now only fifteen minutes were _ (c) S R Q P
not length, constitute a really good left. 19. S1: Different people have different
paragraph. S6: Our performance was excellent. hobbies.
S : Many people define paragraphs P : Thank God, we rubbed the word S6: With hobbies, you spend time in
in terms of their length. defeat’ writ large on our forehead. a gainful way.
The correct sequence should be Q : It was a game of life and death. P : Thus, they say, as many people,
(a) Q P R S (b) R Q P S R : Each one had concluded that we so many hobbies.
(c) S Q P R (d) Q R S P Q : In due course, that becomes
were going to lose.
_ (c) S Q P R S : It was a matter of minutes. your hobby.
14. S1: Then what is necessary with R : Whatever leisure time you get,
The correct sequence should be
regard to taming science and you use that in creative activity.
(a) P S Q R (b) P Q S R
technology is : (c) R P S Q (d) R S Q P S : Hobby is a leisure time activity.
S6: Then why do we not tame The correct sequence should be
_ (d) R S Q P
atomic energy to peaceful purposes (a) P Q R S (b) P Q S R
? 17. S1: The river Ganga is very (c) S R Q P (d) R S Q P
important for us culturally,
P : We must have the basic
spiritually, ecologically- as well as _ (c) S R Q P
knowledge of the two. 20. S1: Life in a village is ideal.
economically.
Q : It is said, ‘‘Science is a useful S6: They can no longer be
S6: Journey in the right direction is
servant but destructive master’’. considered backward.
firmly making progress, gathering
R : By the misuse of science, momentum and becoming a P : Our villages are no longer dirty.
mankind will meet its doom quite people’s movement.
soon. Q : Moreover, they are disease-free.
P : It is rich in cultural heritage, R : Because, the village people are
S : If we do not harness it for the natural splendour and biodiversity.
welfare of mankind, I am afraid, a well-educated now.
Q : People have used its water since S : Rather, they are highly clean and
total annihilation is a must.
the beginning of civilisation for tidy.
The correct sequence should be different purposes.
(a) S R Q P (b) P Q S R The correct sequence should be
R : It has the most dense cultivation
(c) R Q S P (d) Q R S P (a) S R Q P (b) P S Q R
in its basin and is critical for (c) S R P Q (d) R S Q P
_ (b) P Q S R ensuring food and water security.
15. S1: Uttarakhand is vulnerable to _ (b) P S Q R
S : We need to utilise its water for
disasters. different purposes while ensuring Directions (Q. Nos.21-30) Each
S6: The lack of ability to learn that its natural ecology is protected, question in this section consists of
lessons from disasters, and the lack aquatic life thrives, and forests sentences with an underlined word
of any accountability, ensure the along the river remain rejuvenated. followed by four words or group of
perpetuation of the situation. The correct sequence should be words. Select the option that is opposite
P : The absence of necessary (a) Q R S P (b) Q S R P in meaning to the underlined word and
monitoring, early warning systems (c) R S Q P (d) P R Q S mark your answer accordingly.
and the overall disaster _ (d) P R Q S 21. I am very particular about it.
management system add another (a) quiet (b) vague
18. S1: The Republic Day in India falls
layer of damages during the (c) precise (d) minute
on 26th January.
disasters.
S6: He attends the march past. _ (b) ‘Particular’ means specific or
Q : Major interventions act as force distinct. Hence, ‘vague’ is the most
multipliers during such disasters. P : The President of India takes the nearly opposite in meaning to
salute. particular.
R : The violation of legal and other
prudent norms, further increases Q : It is organised at the India Gate. 22. He has become paunchy.
the damages. R : People from far and wide come (a) stout (b) slim
to witness it (c) plump (d) fat
S : Climate change is increasing
these vulnerabilities. S : It is celebrated with great pomp _ (b) ‘Paunchy’ means having a large or
protruding belly. Hence, ‘slim’ is its
The correct sequence should be and show. correct antonym.
24 CDS Solved Paper 2021 (II)

23. This river originates from the _ (c) ‘Eccentric’ means unconventional 39. Life is a solo fight, and each
Ganges. or slight strange. Hence, ‘normal’ is its person makes his or her own
correct antonym.
(a) inaugurates (b) culminates journey.
(c) initiates (d) emanates (a) Noun (b) Adjective
Directions (Q. Nos. 31-40) Each of
_ (b) ‘Originates’ means to have a (c) Intensifier (d) Adverb
the following sentences has a word or
specified beginning. Hence,
phrase underlined. Read the sentences _ (b) ‘Solo’ is an adjective.
‘culminate’ meaning to reach a climax
or close with is its correct antonym. carefully and find which part of speech 40. The cat loves comfort.
the underlined word belongs to. (a) Indefinite article (b) Definite article
24. The film I saw was hilarious.
31. They wandered around aimlessly (c) Intensifier (d) Subject
(a) tragic (b) serious
(c) uproarious (d) jovial (a) Verb (b) Adjective _ (b) The is a definite article.
(c) Intensifier (d) Noun
_ (*) (a) and (b) ‘Hilarious’ means Directions (Q. Nos. 41-50) Each of
extremely amusing. Hence, ‘tragic’ _ (c) ‘Around’ is an intensifier.
and ‘serious’ are its correct antonyms. the following sentences in this section
32. We went away after they had left. has a blank space with four words or
25. On that day, pandemonium (a) Pronoun (b) Adjective group of words given. Select whichever
reigned in the hall. (c) Intensifie (d) Conjunction word or group of words you consider
(a) hullaballoo (b) uproar
(c) peace (d) accolade _ (d) Conjunction is a word that the most appropriate for the blank
connects two or more clauses or space and mark your answer.
_ (c) ‘Pandemonium’ means wild and sentences to form new sentences.
noisy disorder or confusion. Hence, Hence, ‘after’ is a conjunction. 41. The polythene bags ……
‘peace’ is its correct antonym. non-biodegradable, i.e. they
33. Public culture is associated with
26. The police detained me today cannot
extremely new civil societies .
amidst busy traffic. (a) are (b) become
(a) Adverb (b) Intensifier
(a) impeded (b) released (c) is (d) were
(c) Adjective (d) Noun clause
(c) confined (d) held _ (a) are
_ (b) Intensifiers are adverb or adjective
_ (b) ‘Detained’ means to officially seize words that are used to add force and 42. be decomposed ..........
and hold someone or something. to intensify another adjective, adverb micro-organisms into manure.
Hence, ‘released’ is its correct antonym.
or verb. Hence, ‘extremely’ is an They
27. In my state of despair, I confessed intensifier.
(a) by (b) through
everything. 34. You are paying less attention to (c) into (d) in
(a) despondency (b) determination your studies these days. _ (a) by
(c) dependant (d) elation
(a) Adverb (b) Adjective 43. remain as …… even after years.
_ (d) ‘Despair’ means the complete loss (c) Intensifier (d) Noun
Animals that started
or absence of hope. Hence, ‘elation’
meaning joy and happiness is the _ (b) ‘Less’ is an adjective. (a) they are (b) it is
most nearly opposite in meaning to 35. Why, is it really Sujata on the (c) even (d) after
despair.
phone? _ (a) they are
28. He was accused by the entire (a) Interjection (b) Adjective 44. eating waste food …… with these
community after he failed in the (c) Intensifier (d) Noun
polythene bags, ultimately
mission . _ (a) ‘Why’ is an interjection. (a) besides (b) beside
(a) vindicated (b) incriminated
36. Sit down and rest a while. (c) thorough (d) along
(c) indicted (d) arraigned
(a) Adverb (b) Adjective _ (d) along
_ (a) ‘Accused’ means charged with an (c) Intensifier (d) Noun
offence or crime. Hence, ‘vindicated’ 45. started dying …… their internal
meaning cleared of suspicion is its _ (a) The given sentence uses ‘while’ as system was getting blocked.
correct antonym. an adverb.
(a) of
29. There was much to boast about 37. Rakesh is too old to run fast. (b) though
the quality of his work. (a) Adverb (b) Conjunction (c) because
(a) bluster (b) brag (c) Intensifier (d) Noun (d) for
(c) deprecate (d) flaunt _ (c) ‘Too’ is an intensifier. _ (c) because
_ (c) ‘Boast’ means to talk with 38. For the next generation of interior 46. The Government had no
excessive self-pride Hence, alternative …… to ban these
‘deprecate’ meaning to express architects and design graduates,
disapproval of is its correct antonym. work opportunities are immense. polythene
(a) Adverb (b) Adjective (a) yet (b) but
30. He delivered an eccentric speech. (c) so (d) because
(c) Noun (d) Pronoun
(a) An odd (b) A peculiar
(c) A normal (d) An idiosyncratic _ (c) ‘Opportunities’ is a noun. _ (b) but
CDS Solved Paper 2021 (II) 25

47. bags. It is ...... duty of the 56. Don’t loiter ...... the street. 65. They fought ……… the last man in
government to look into this (a) near (b) around (c) in (d) on the army.
matter. (a) on (b) from (c) with (d) to
_ (d) on
(a) but also (b) still
57. Sit here …… me. _ (d) to
(c) the (d) yet
(a) by (b) beside 66. Keep him ........ arm’s length.
_ (c) the (c) in (d) on (a) at (b) by (c) for (d) off
48. however, it is also the duty of each
_ (b) beside _ (a) at
individual to ..........
(a) get it (b) do it
58. We mustn’t shy ..... entry-level or 67. He succeeded ...........
(c) convene it (d) see to it freelance jobs as they help us gain (a) on himself (b) by himself
an insight into the context in (c) in the long run (d) on the long run
_ (d) see to it which a company operates.
49. that we ....... use polythene bags. _ (c) ‘In the long run’ means ultimately
(a) in (b) with or eventually. Hence, it is the correct
By using these we will (c) away from (d) upon phrase to fill the blank.
(a) didn’t (b) weren’t
_ (c) away from 68. Have you ever tried ....... a coconut
(c) don’t (d) aren’t
59. Civil society and media have a tree ?
_ (c) don’t major role in making the coalition (a) climbing (b) to climbing
50. not only harm the environment work …… the people. (c) going on (d) going up
but also ourselves. Thus, say ‘No’ (a) for (b) about _ (a) Climbing
to plastic bags and contribute …… (c) with (d) over 69. He is a man .......... means.
the society.
_ (a) for (a) for (b) in (c) of (d) above
(a) by (b) to (c) into (d) in
60. He travelled …… Mr. Joshi’s car. _ (c) of
_ (b) to (a) by (b) for
70. He dislikes …… punish his friends.
(c) into (d) on
Directions (Q. Nos. 51-60) Each of (a) have to (b) having to
the following sentences in this section _ (a) by (c) for (d) regarding
has a blank space with four options. _ (b) having to
Directions (Q. Nos. 61-70) Each of
Select whichever preposition or
the following sentences in this section
determiner you consider the most Directions (Q. Nos. 71-80) Each
has a blank space and four words or
appropriate for the blank space and question in this section consists of a
group of words are given after the
mark your answer. sentence with an underlined word
sentence. Select the most appropriate
51. He succeeded …… dint of followed by four words. Select the
word or group of words for the blank
preseverance and hard work. option that is nearest in meaning to the
space and mark your answer.
underlined word and mark answer.
(a) by (b) for (c) on (d) upon 61. Honesty is ......... on his face.
71. He drowned in the flood water last
_ (a) By dint of means by means of. (a) wrote (b) written
year.
‘Hence’, ‘by’ is appropriate to fill the (c) writing (d) writes
blank. (a) swarmed (b) swam
_ (b) written (c) submerged (d) floated
52. Due to his illness, he could not
finish the work …… time. 62. He lives ......... his pen. _ (c) ‘Drowned’ means to die through
(a) by (b) with (c) on (d) off submersion in the water or to
(a) by (b) for (c) on (d) upon ‘submerge’. Hence, option (c) is the
_ (c) on _ (d) ‘Live off’ means to depend on correct answer.
something as a source of income.
53. There is no meaning …… what you Hence, ‘off’ is appropriate to fill the 72. Autumn is a rather dry season.
say. blank. (a) arid (b) humid
63. Much water has run ....... the (c) rainy (d) moist
(a) by (b) for (c) in (d) on
_ (c) in
bridge since then. _ (a) ‘Arid’ is the correct synonym for
(a) near (b) about ‘dry’.
54. These are good rules ...... live by. (c) under (d) in 73. Our differences are growing day
(a) with (b) to (c) in (d) on
_ (c) Under by day.
_ (b) to 64. The gun ........ with a loud noise. (a) confabulations (b) interferences
55. He is the man I have been looking (c) disagreements (d) discrepancies
(a) went on (b) went off
...... (c) went about (d) went around _ (c) ‘Differences’ means
disagreement, quarrel or dispute.
(a) at (b) for _ (b) ‘Went off’ means to be fired or Hence, option (c) is the correct
(c) in (d) on explode. Hence, it is appropriate to fill answer.
the blank.
_ (b) for
26 CDS Solved Paper 2021 (II)

74. The big tree hindered access of any underlined part. If you find no (c) an American?
sunlight into the house. error, your answer should be option (d). (d) No error
(a) impeded (b) shaded 81. (a) Everything is going well; _ (d) The given sentence is error free.
(c) facilitated (d) poured (b) we didn’t have 90. (a) I went to Canada
_ (a) ‘Hinder’ means to make (c) any problem. (b) a few years ago
something difficult for someone to do. (d) No error (c) for a holiday.
Hence, ‘impede’ meaning to delay or (d) No error
prevent someone is its correct _ (d) The given sentence is error free.
synonym. 82. (a) Lavanya hasn’t gone _ (b) Part (b) contains the error.
Replace ‘a few’ with ‘few’ to make the
(b) to work
75. The complexity of the issue baffled sentence grammatically correct.
(c) yesterday.
everyone. (d) No error
(a) conclusion (b) intricacy Directions (Q. Nos. 91-100) Each of
(c) grievance (d) complacency _ (a) Part (a) contains the error. the following question in this section
Replace ‘hasn’t gone’ with ‘didn’t go’
consists of a sentence, parts of which
_ (b) ‘Complexity’ refers to the state or to make the sentence error free.
quality of being intricate or have been jumbled. These parts have
complicated. Hence, ‘intricacy’ is its 83. (a) Look ! The boy over there been labelled as P, Q, R and S. Given
correct synonym. (b) wears the same sweater
(c) as you below each sentence are four sequences,
76. He drank excessive amount of (d) No error namely (a), (b), (c) and (d). You are
liquor. required to rearrange the jumbled parts
_ (b) Part (b) contains the error.
(a) inculpable (b) inordinate Replace ‘wears’ ‘with ‘is wearing’ to of the sentence and mark your answer.
(c) unreasonable (d) innocuous make the sentence errror free.
91. P: from Europe to America
_ (c) ‘Excessive’ means more than 84. (a) It begins Q : in 1992
necessary or desirable. Hence, (b) to turn dark.
‘unreasonable’ is its correct synonym. (c) Shall I switch on the lights? R : his first voyage
(d) No error S : Columbus made
77. He initiated the dialogue. _ (a) Part (a) contains the error. The correct sequence is
(a) ordered (b) interfered Replace ‘begins’ with ‘is beginning’ to
make the sentence error free. (a) S P R Q (b) S R P Q
(c) began (d) planned (c) P R S Q (d) Q R S P
_ (c) ‘Initiated’ means started. Hence, 85. (a) Raman and Mitali
‘began’ is its correct synonym. (b) have been married _ (b) S R P Q
(c) for twenty years. 92. P: Scientists have
78. She looked pale after recovering (d) No error
from Covid-19. Q : the effects of
(a) sallow (b) ruddy _ (d) The given sentence is R : warned us about
grammatically correct.
(c) glowing (d) radiant S : climate change
86. (a) I have played The correct sequence is
_ (a) ‘Pale’ means (a face) having less (b) basketball
colour than usual, typically due to (a) R P S Q (b) R S P Q
shock or illness. Hence, ‘sallow’ is its (c) for the past three hours.
(c) P R Q S (d) Q S P R
correct synonym. (d) No error
_ (c) P R Q S
79. I overrule your proposal. _ (a) Part (a) contains the error.
Replace ‘played’ with ‘been playing’ 93. P : speed is a potent cause
(a) wan (b) veto to make the sentence grammatically
(c) weaken (d) supersede correct. Q: Industrial growth at a terrific
_ (b) ‘Overrule’ means to reject or 87. (a) Jamila had a book R : in a big city
disallow by exercising one’s superior (b) in front of her S : of pollution
authority. Hence, ‘veto’ is its correct (c) but she didn’t read it.
synonym. The correct sequence is
(d) No error
(a) R S Q P (b) Q P R S
80. His behaviour shocked me. _ (d) The given sentence is error free (c) P R S Q (d) Q P S R
(a) entreated (b) pacified and grammatically correct.
(c) appalled (d) scintillated _ (d) Q P S R
88. (a) When she heard the news
_ (c) ‘Appalled’ means greatly (b) she hasn’t been 94. P : and garbage
dismayed or horrified. Hence, it is the (c) very pleased. Q : today our environment
correct synonym for shocked. (d) No error
R : filth and squalor
Directions (Q. Nos. 81-90) Each _ (b) Part (b) contains the error. S : is in constant grip of
Replace ‘hasn’t been’ with ‘wasn’t’ to
question in this section has a sentence make the sentence grammaticaly The correct sequence is
with three underlined parts labelled as correct.
(a) R P S Q (b) R S P Q
(a), (b) and (c). Read each sentence to 89. (a) Where are you coming from? (c) Q S P R (d) Q S R P
find out whether there is any error in (b) Are you
_ (d) Q S R P
CDS Solved Paper 2021 (II) 27

95. P : of corruption everywhere Directions (Q. Nos. 101-110) The (c) before ten minutes
following questions have one part of (d) every ten minutes
Q : to ease out
R : the monster
the sentence followed by four _ (d) The bus service is very good;
alternatives. Complete the sentences by there is a bus every ten minutes.
S : it is the duty of the government choosing the correct alternative. 107. We live near a busy airport; the
The correct sequence is 101. I asked two people the way to the planes fly
(a) S Q R P (b) Q R S P station (a) near our house
(c) P R S Q (d) Q R P S (b) by our house
(a) but neither of them could help me
_ (a) S Q R P (c) over our house
(b) however none of them could show
(d) around our house
96. P : made the movement me the way
(c) yet they had doubts about the _ (c) We live near a busy airport, the
Q : the poor visibility station planes fly over our house.
R : of the vehicles (d) nevertheless they couldn’t guide me 108. Dan was very quiet. He didn’t say
S : quite difficult _ (a) I asked two people the way to the a word
station but neither of them could help
The correct sequence is (a) all the evening
me.
(a) Q R P S (b) R S P Q (b) the entire evening
(c) Q R S P (d) P R S Q 102. The room was very warm (c) all the entire evening
(a) because the AC was set to (d) entire evening
_ (a) Q R P S moderate temperature _ (b) Dan was very quiet. He didn’t say
97. P : Mohan clearly (b) though the AC was set to very cold a word the entire evening.
Q : I couldn’t see (c) since the AC was not very warm in
its temperature setting
109. I don’t like stories
R : only a few yards ahead (d) since the AC was in a very low (a) which can have unhappy endings
S : though he was temperature setting (b) which had sad endings
(c) that have unhappy endings
The correct sequence is _ (b) The room was very warm though (d) which are unhappy endings
the AC was set to very cold.
(a) R Q S P (b) R S P Q
103. We could leave today or we could _ (c) I don’t like stories that have
(c) P R S Q (d) S R Q P unhappy endings.
_ (d) S R Q P
leave tomorrow
110. Not everything
(a) preferring what you want
98. P : both winter and summer (b) as you prefer one of the two (a) that happened was my fault
Q : extreme (c) depending on what you prefer (b) which happen was my fault
R : are truly (d) whichever you prefer (c) what happened was my fault
S : here in India (d) whatever happened was my fault
_ (c) We could leave to day or we could
The correct sequence is leave tomorrow, depending on what _ (a) Not everything that happened was
you prefer. my fault.
(a) R P S Q (b) S R Q P
(c) S P R Q (d) P S R Q 104. Mohan and I couldn’t get into the Directions (Q. Nos. 111-120) In
_ (c) S P R Q house because this section, you have two short
99. P : get crystallised (a) neither of us had the keys passages. After each passage, you will
(b) both of us do not have the keys
Q : the self and world view find some questions based on the
(c) either of us did not have the keys
R : and physical space where basic (d) neither of us did have the keys passage. Read the passage and answer
ideas of the questions based on it.
S : public culture is a mental _ (a) Mohan and I couldn’t get into the
house because neither of us had the Passage I
The correct sequence is keys.
As to happiness, I am not so sure.
(a) R Q P S (b) S R Q P 105. There are many good hotels, you Birds, it is true, die of hunger in large
(c) S R P Q (d) Q R S P can choose to stay in numbers during the winter, if they
_ (b) S R Q P (a) many of them are not birds of passage. But during
100. P : that he ordered (b) any one of them the summer they do not foresee this
(c) either of them catastrophe or remember how nearly
Q : the post was (d) All of them it befell them in the previous winter.
R : the first job he performed on _ (b) There are many good hotels, you With human beings the matter is
reaching can choose to stay in any one of otherwise. I doubt whether the
them. percentage of birds that will have
S : my release from jail
The correct sequence is 106. The bus service is very good; there died of hunger during the present
is a bus winter (1946-47) is as great as the
(a) R P S Q (b) R S P Q percentage of human beings that will
(c) P R S Q (d) R Q P S (a) after ten minutes
(b) in ten minutes have died from this cause in India
_ (d) R Q P S and central Europe during the same
28 CDS Solved Paper 2021 (II)

period. But every human death by _ (b) ‘Erudite’ means learned and _ (a) The general tone of the passage is
starvation is preceded by a long scholarly. Hence, ‘ill-educated is its optimistic .
correct antonym.
period of anxiety, and surrounded by 117. The willingness of consumers to
the corresponding anxiety of 115. Which one of the following is the go back to normal lifestyle
neighbours. We suffer not only the central theme of the passage ? indicates their
evils that actually befall us, but all (a) Life of the birds and the mice (a) Casual attitude
those that our intelligence tells us we (b) Starvation in India and central (b) Change of moods
have reason to fear. The curbing of Europe
(c) Desire for future plans
impulses to which we are led by (c) Progress of mankind
(d) Sense of economic security
forethought averts physical disaster (d) Disasters in 1946-47
at the cost of worry, and general lack
_ (c) ‘Progress of mankind’ is the _ (d) The passage suggests that Indian
consumers show increased
of joy. I do not think that the learned central theme of the given passage.
confidence about the future of the
men of my acquaintance, even when economy and that consumers feel
they enjoy a secure income, are as Passage II
secure about the economy. Hence,
happy as the mice that eat the More than eight months after the option (d) is the correct answer.
crumbs from their tables while the national lockdown was announced in
late March, urban India is learning
118. Which statement in the
erudite gentlemen snooze. In this
respect, therefore, I am not convinced to live with the Covid-19 pandemic. passage/phrase indicates that
that there has been any progress at In fact, indicating a positive outlook ‘Life must go on’?
all. for the future, many survey (a) They “settled into their new
respondents in a recent survey say routines”
111. The birds die of hunger in winter (b) “Salary cuts could be coming to an
they plan to return to pre-lockdown
because levels of shopping, personal end”
(a) they do not move to warmer places (c) People are “moving on with their
grooming, going to cinemas and
(b) people do not feed them lives”
socialising as pandemic fears
(c) they do not get the food of their (d) “Pent - up demand”
continues to recede.
choice
(d) they are too young to get the food The survey findings highlight that _ (c) By the line, people are ‘moving on
with their lives’ the author aims to
the suppression of consumer demand
_ (a) According to the passage, if the because of fears of job losses and
indicate that’ ‘life must go on’.
birds are not ‘birds of passage’
(migratory), they die in large numbers salary cuts could be coming to an end. 119. What does the author mean by
during winters. Increasingly, urban Indians are ‘pent-up demand’ ?
112. The birds do not foresee the showing increased confidence about (a) Desire (suppressed) to spend
catastrophe because they the future of the economy. This could money once pandemic is
be an indicator of the possible controlled
(a) cannot predict an accident
‘pent-up demand’ that several (b) Economic normalcy of consumers
(b) overlook a difficult situation
(c) cannot expect a sudden disaster
economists have been talking about - (c) Flamboyant shopping by
(d) ignore the problems a demand that could be unleashed consumers
once a vaccine is developed and (d) Criticism of the public on the
_ (*) (b) and (c) according to the distributed, or when there are signs pay-cuts
passage, birds don’t expect this
of the pandemic’s spread reducing to
catastrophic situation of the winter _ (a) By the phrase ‘pent-up demand’,
and overlook the situation even with negligible levels or vanishing totally. the author means that the desire to
the prior experience of winter. Around 65 per cent of respondents spend money in shopping for a
113. Human beings cannot be happy said they had settled into new product will increase once the
because they routines, or that they saw signs of the pandemic is brought under control.
(a) do not get time to enjoy situation improving, or they had 120. What, according to the author, is
(b) worry too much about their work come to terms with the pandemic and
the reason behind suppression of
(c) are not healthy were moving on with their lives. And
consumer demands ?
(d) worry too much about future since the survey was conducted
before the news of the successful (a) The lack of vaccine till date
_ (d) The passage mentions that trials of the Pfizer vaccine for the (b) Professional insecurity
human beings worry too much about
the future, giving uncecessary coronavirus was announced, it is (c) Motive to save up some money
forethought, Hence, they cannot be likely that the consumers are now (d) Careful and calculative attitude of
happy. even more positive in their outlook the general public
114. Which one of the following is the about the future. _ (b) The reason behind the
antonym of the word ‘erudite’ in 116. The general tone of the passage is suppression of consumer demands is
the passage? that of fear of job losses and salary cuts.
(a) Qualified (b) Ill educated Hence, option (b) is the correct
(a) Optimism (b) Pessimism
(c) Logical (d) Learned answer.
(c) Fatalism (d) Defeatism
PAPER III General Studies
1. Who among the following ancient _ (b) Khema is not a jain 8. Which one of the following is the
Indian kings was praised in Acharya.Khema was a Buddhist opportunity cost of a chosen
glowing terms in the Prayaga bhikkhuni or nun, who was one of the activity ?
top female disciples of the Buddha.
Prashasti? She is considered the first of the (a) Out of pocket cost
(a) Ashoka Buddha’s two chief female disciples, (b) Out of pocket cost plus cost
(b) Harshavardhana along with Uppalavanna. incurred by the Government
(c) Samudragupta (c) Value of all opportunities forgone
5. In India, the first major public (d) Value of next best alternative that is
(d) Bindusara
appearance of Mahatma Gandhi given up
_ (c) Samudragupta was the Ancient was in
Indian king who was praised in _ (d) In microeconomic theory, the
glowing terms in the Prayaga (a) Champaran (1917) opportunity cost of a particular activity
Prashasti.In Prayaga Prashasti the (b) Kheda (1918) option is the loss of value or benefit
poet Harishena mentions the victories (c) Inauguration of Banaras Hindu that would be incurred by engaging in
of Samudragupta. The inscription is a University (1916) that activity, relative to engaging in an
panegyric praising Samudragupta (d) Rowlatt Satyagraha (1919) alternative activity offering a higher
and lists the political and military return in value or benefit. Thus,
achievements of his reign including _ (c) Gandhi’s first major public opportunity cost of a chosen activity is
his expeditions to the South. appearance was during the
inauguration of Banaras Hindu the value of next best alternative that
2. The Self-Respect Movement was University in 1916.Banaras Hindu is given up. For example-Someone
initiated by University is an internationally reputed gives up going to see a movie to
temple of learning, situated in the holy study for a test in order to get a good
(a) B.R. Ambedkar city of Varanasi. grade. The opportunity cost is the
(b) Jyotiba Phule cost of the movie and the enjoyment
(c) E.V. Ramaswamy Naicker 6. Consider the following statements of seeing it.
(d) Jawaharlal Nehru about Patanjali’s Mahabhashya
9. Which one of the following
_ (c) The Self-Respect Movement was 1. It makes a mention of Kautilya.
statements in the context of social
initiated by E.V. Ramaswamy Naicker 2. It is a book on grammar and refers
in 1925 in Tamil Nadu. Self-Respect sector spending in India during
to historical personalities only
Movement was a dynamic social incidentally. 2014 - 19 (both States and the
movement aimed at destroying the Union Government together) is
contemporary Hindu social order in its Which of the statements given
true?
totality and creating a new, rational above is/are correct?
(a) Expenditure on' education was 5%
society without caste, religion and (a) Only 1 (b) Only 2 of GDP.
God. (c) Both 1and 2 (d) Neither 1 nor 2 (b) Expenditure on health was 4% of
3. The location of the ancient city of _ (b) Statement 2 is correct. The the social services expenditure.
Taxila (Takshshila), mentioned in Mahabhasya, attributed to Patanjali, is (c) There was a stagnation in the
a commentary on selected rules of spending on education as a
ancient Indian texts, was Sanskrit grammar from Panini’s percent of GDP.
identified by treatise, the Astadhyayi, as well as (d) Health sector spending amounted
(a) Alexander Cunningham Katyayana’s Varttika-sutra, an to 10% of the total expenditure.
(b) R.D. Banerji elaboration of Panini’s grammar. It is
(c) John Marshall dated to the 2nd century BCE. _ (c) There was a stagnation in the
spending on education as a percent
(d) Daya Ram Sahni 7. Which one of the following may of GDP in the context of special
_ (a) Alexander Cunningham identified lead to movement along the sector spending in India during
the location of the ancient city of Taxil demand curve of a commodity ? 2014-19 as the education expenditure
as mentioned in the Ancient Indian has grown only to 3.1% of GDP in
(a) Change in its price 2019 from 2.8% in 2014.
Text. Alexander Cunningham was
(b) Change in price of the other
keen admirer of heritage, sculptures, 10. According to UNDP’s Human
commodities
antiquities and numismatics. He laid
the foundation of Archaeological (c) Change in income of the consumer Development Report - 2020, in
Survey of India (ASI) in 1861.He also (d) Change in tastes and preferences which of the HDI components has
became its first Director-General. of consumers India improved in recent years?
4. Who among the following was not _ (a) The demand curve is a graphical 1. Life expectancy at birth
representation of the relationship
a Jain Acharya? between the price of a good or 2. Expected years of schooling
(a) Bhadrabahu service and the quantity demanded 3. GNI per capita
(b) Khema for a given period of time. Thus 4. Mean years of schooling
(c) Haribhadra change in a price of commodity may
lead to the movement along the Select the correct using the codes
(d) Siddhasena Divakara
demand curve of a commodity. given below:
30 CDS Solved Paper 2021 (II)

(a) 1 and 2 (b) 2 and 3 Select the correct answer using the 16. Under which Article of the
(c) 1 and 3 (d) 1,2,3 and 4 codes given below: Constitution of India has
_ (d) All 1,2,3 and 4 are correct. (a) 1,2 and 3 (b) 1 and 2, provision been made for
According to UNDP’s HDI Report, (c) 2 and 3 (d) 1 and 3 reservation of seats for women in
2020,HDI value of India has increased
to 0.645 from 0.429, registering an _ (a) Statements 1,2 and 3 are Panchayats ?
increase of over 50%. Life expectancy correct.Under Article 120 of the (a) Article 243 B (b) Article 243 C
at birth in India rose by nearly 12 Constitution, the business of the (c) Article 243 D (d) Article 243 E
years. Mean years of schooling House is to be transacted in Hindi or
witnessed an increase of 3.5 years. in English, but a member who cannot _ (c) Article 243D clause (3) after the
adequately express himself in either of 73rd constitutional amendment
The expected years of schooling also
the two languages can, with the ensures participation of women in
rose by 4.5 years. GNI per capita of
permission of the Speaker, address Panchayati Raj Institutions by
India also increased, registering a rise
the House in any of the languages mandating not less than one- third
of nearly 274%.
mentioned in the Eighth Schedule of reservation for women out of total
11. Which one of the following is true the Constitution or in his mother number of seats to be filled by direct
of a pure voluntary exchange tongue. The minutes and reports of election and number of offices of
the Parliamentary Committees are chairpersons of Panchayats.
between two parties A and B ?
invariably prepared and presented to 17. The painted illustration of the
(a) A can exploit B or vice-versa the house both in Hindi and in
(b) Both gain; it is a win-win situation English. moving of the Ashoka Pillar at
(c) If A makes profit, it must be at the Topra is found in
cost of B 14. Which one of the following is the
(a) Tarikh-i-Firuz Shahi
(d) Both can lose correct meaning of the term (b) Tarikh-i-Shahi
‘State’, as defined in Article 12 of (c) Sirat-i-Firuz Shahi
_ (b) In a pure voluntary exchange
between two parties A and B, both the Constitution of India ? (d) Akbar Nama
gain which is a win-win situation. (a) It refers only to the Government of
Voluntary exchange is the act of _ (a) Tarikh-i Firuz Shahi is the finest
India. specimen of Indo-Persian
buyers and sellers freely and willingly (b) It refers only to the Government of historiography produced during the
engaging in market transactions. India and the Parliament of India. Sultanate period in India. It contains
Voluntary exchange is a fundamental the painted illustration of the moving
(c) It refers only to the Government of
assumption made by neoclassical of the Ashokan pillar at Topra.
India, Parliament of India, and
economics which forms the basis of Situated in Pong valley of is the
Governments and Legislatures of
contemporary mainstream original home of Delhi-Topra Ashokan
economics.
each State.
(d) It refers to the Government of pillar, one of many pillars of Ashoka,
12. The provision, ‘‘the State shall India, Parliament of India, that was moved from Topra to Feroz
Governments and Legislatures of Shah Kotla in Delhi in 1356 CE by
take steps to separate the Judiciary Firuz Shah Tughlaq (1309-1388 CE).
from the Executive in the public each State, and all local or other
authorities within the Territory of 18. Which one among the following is
services of the State’’ is India.
incorporated in which part of the a temple of the Vaishnavite
Constitution of India ? _ (d) According to Article 12 of the tradition?
Constitution of India, the term ‘State’
(a) Part-IV (b) Part-V denotes the Union and State (a) Srirangam (b) Chidambaram
(c) Part-VI (d) Part-VII Governments, the Parliament and (c) Gangaikonda Cholapuram
state legislatures, and all local or (d) Thanjavur
_ (a) Article 50 contained in Part IV of other authorities within the Territory of
Constitution of India is a directive _ (a) Srirangam is a temple of
India or under the control of the Indian Vaishanvite tradition.Srirangam is the
principle of state policy . It gives a
Government. foremost of the eight self-manifested
direction to the State to keep Judiciary
separated and independent of the 15. Which one among the following is shrines (Swayam Vyakta Kshetras) of
Executive, particularly in judicial Lord Vishnu .It is also considered the
the largest State in terms of seats first and the most important of the 108
appointments.
in the Rajya Sabha? main Vishnu temples (Divyadesams).
13. Which of the following statements (a) Andhra Pradesh The Vaishnavite tradition is known for
are correct? (b) Bihar the loving devotion to an avatar of
Vishnu (often Krishna), and as such
1. Speaker may permit any Member (c) Rajasthan
has been key to the spread of the
to address the House in his/her (d) Karnataka Bhakti movement in South Asia in the
mother tongue, if he/she cannot _ (b) Among the mentioned states, 2nd millennium CE. Chidambaram,
adequately express in either Hindi Bihar with 16 Rajya Sabha seats is the Gangaikonda Cholapuram and
or English. largest state in terms of seats in the Thanjavur temples are dedicated to
Rajya Sabha. Andhra Pradesh, lord shiva.
2. Business of the Parliamentary
Committees is transacted either in Rajasthan and Karnataka has 11,10 19. The biography of Shaikh
and 12 Rajya Sabha seats
Hindi or in English. Muinuddin Chishti, authored by
respectively.
3. The minutes of the Parliamentary Uttar Pradesh with 31 seats is the Jahanara, is known as
Committees are prepared largest states in terms of Rajya Sabha (a) Munis al Arwah
invariably in Hindi or English. seats (b) Fawaid ul Fuwad
CDS Solved Paper 2021 (II) 31

(c) Sirat ul Auliya word Sangh, which means company, 26. Which one of the following States
(d) Muraqqa-e Dehli fellowship and association.In Sikh
vocabulary, the word has a special
is not prominent for plantation
_ (a) The biography of Shaikh connotation. It stands for the body of agriculture?
Muinuddin Chishti, authored by men and women who meet (a) Rajasthan (b) Assam
Jahanara, is known as Munis al religiously, especially in the presence
Arwah.Jahanara Begum was a (c) Nagaland (d) Kerala
of the Guru Granth Sahib.
Mughal princess and later became _ (a) Among the given options, the
Padshah Begum of the Mughal 23. Who amongst the following will state of Rajasthan is not prominent for
Empire from 1631 to 1658 and again be at more risk with respect to the plantation agriculture. Rajasthan is a
from 1668 until her death. She was dry state where Wheat, Jau, Jwar,
the second and the eldest surviving
earthquake hazard zone specified Bajra, Maize and Pulses are the major
child of Emperor Shah Jahan and by the Geological Survey of India? food crops. Some examples of
Mumtaz Mahal. (a) Shahji at Secunderabad plantation crops are Coffee, Cocoa,
(b) Ranbir at Indore Cotton, Tea, Sugarcane, Sisal, Oil
20. Which one of the following is not a seeds, Rubber trees etc. which are
(c) Malti at Pithoragarh
political method of the moderates (d) Maitri at Bhubaneswar found in states of Assam, Nagaland
in the National Movement? and Kerala.
(a) Overthrow of alien rule _ (c) Malti at Pithoragarh is at zone-V of
seismic zone which is the highest 27. The distance between which two
(b) Constitutional agitation seismic zone as given by Geological cities has been reduced by the Atal
(c) Slow, orderly political progress survey of India. A seismic hazard Tunnel?
(d) Mobilisation of public opinion zone describes an area with a
particular level of hazard due to (a) Bagdogra to Gangtok
_ (a) The Early Nationalists, also known earthquakes. Typically, a high seismic (b) Jammu to Srinagar
as the Moderates, were a group of
hazard zone is nearest a seismic zone (c) Manali to Leh
political leaders in India active
between 1885 and 1907. Their where there are more earthquakes, (d) Itanagar to Tawang
and a lower seismic hazard zone is
emergence marked the beginning of
farther away from a seismic zone. _ (c) The distance between Manali to
the organised national movement in Leh has been reduced by the Atal
India.Moderates believed in the policy 24. The Government wants to prepare Tunnel. Atal Tunnel in Himachal
of settlement of minor issues with the Pradesh’s Rohtang, connects Solang
government by deliberations. The
a plan for drought prone areas of Valley near Manali to Sissu in Lahaul
methods of the moderates were: (i) the country. Which one of the and Spiti district. Atal tunnel is the first
Sending petitions to the Government following regions will be step towards all-year connectivity to
officials. (ii) Sending memorandums predominantly focused? Ladakh. The tunnel has the potential
to the officers. (iii) Passing resolutions. to link Ladakh to Manali and
(a) Coastal Andhra Pradesh Chandigarh throughout the year, as it
21. The town of Chisht, from which (b) Ganga plains would bypass Rohtang Pass, which is
the Sufi Chishti Silsila derives its (c) North-eastern region snowed in through the winter months.
name, is located in (d) Kutch region
28. Which one of the following
(a) Western Turkey _ (d) Kutch region is the dry land region biosphere reserves comprises
(b) Central Afghanistan which is prone to droughts as it is
(c) Eastern Iran close to Thar desert and this islands with estuaries, beaches,
(d) Eastern Iraq government has to prepare a plan for coral reefs, salt marshes and
such drought prone region. Other mangroves?
_ (b) The town of Chisht, from which the region mentioned gets adequate
Sufi Chishti Silsila derives its name, is rainfall. (a) Sunderban biosphere reserve
located in central Afghanistan.The (b) Gulf of Mannar biosphere reserve
Chishti Order is a tariqa, an order or 25. Four persons are travelling to (c) Nilgiri biosphere reserve
school within the mystic Sufi tradition different States of India. To whom (d) Nandadevi biosphere reserve
of Sunni Islam. The Chishti Order is will you advise to protect oneself
known for its emphasis on love, _ (b) Gulf of Mannar biosphere reserve
tolerance, and openness. It began in
from blizzards? comprises islands with estuaries,
(a) The person travelling to Arunachal beaches, coral reefs, salt marshes
Chisht, a small town near Herat
Pradesh and mangroves.The Gulf of Mannar,
located in Central Afghanistan, about
running down South from
930 AD. (b) The person travelling to Assam
Rameswaram to Kanyakumari in
(c) The person travelling to Tripura
22. The rules for congregational Tamil Nadu. It is one of the
(d) The person travelling to Odisha biologically richest coastal regions in
worship (Sangat) involving
_ (a) Person travelling to Arunachal all of mainland of India. It is the first
collective recitation were Marine Biosphere Reserve in the
pradesh should be advised to protect
organised by oneself from Blizzards. Blizzards South and South-East Asia.
(a) Guru Nanak (b) Guru Angad commonly occur with temperatures
29. Which one of the following is not
(c) Guru Arjan around or below 20 degrees
(d) Guru Govind Singh Fahrenheit.These low temperatures a correct statement with reference
combined with strong winds create a to the Constitution of India?
_ (a) Guru Nanak organised rules for low wind-chill factor, which is the (a) The Constitution (Eighty-Fifth)
congregational worship involving amount of cooling someone feels
collective recitation. Sangat is a Sikh Amendment Act has inserted the
from the combination of temperature 'Right to Education under Article
term with its origin in the Sanskrit and wind speed. 21-A.
32 CDS Solved Paper 2021 (II)

(b) Article 22 talks about preventive _ (b) All 1, 2 and 3 are correct. Article recorded his impressions of
detention. 368 (1) of the Constitution of India Vijayanagara state in his Chronica
(c) Right to Constitutional Remedies is grants constituent power to make dos reis de Bisnaga (‘‘Chronicle of the
in Part-III of the Constitution. formal amendments and empowers Vijayanagar kings’’).
(d) Writ jurisdiction of High Courts is Parliament to amend the Constitution
by way of addition, variation or repeal
35. The extent of damage caused by
wider than the Supreme Court of
of any provision according to the earthquake is not influenced by
India.
procedure laid down therein, which is which one of the following ?
_ (a) 85th Amendement Act to the different from the procedure for (a) Strength of earthquake
Constitution of Indian made an ordinary legislation.Article 368 of Part
amendment In Article 16 of the (b) Population density
XX of Indian Constitution provides for
Constitution, in clause (4A), The two types of amendments. Viz, By a
(c) Type of building
Constitution (Eighty-sixth special majority of Parliament and By (d) Climate of the area
Amendment) Act, 2002 inserted
Article 21-A in the Constitution of India
a special majority of the Parliament _ (d) The extent of damage caused by
with the ratification by half of the total earthquake is not influenced climate
to provide free and compulsory states. of the area. When an earthquake
education of all children in the age strikes, the intensity of earthquake
group of six to fourteen years as a 32. Which one of the following is not shaking determines the severity of
Fundamental Right in such a manner a circumstance for proclamation of damage. In turn, the main factors
as the State may, by law, determine. Emergency by the President of affecting earthquake shaking intensity
30. Which one of the following India under Article 352 of the are earthquake depth, proximity to the
fault, population density, the
statements about the Cripps Constitution of India ? underlying soil,and building
Mission is not correct? (a) War characteristics particularly height.
(a) It was sent to India by the British (b) External aggression
Government in March 1942. (c) Internal disturbance 36. An avalanche is a type of which
(b) It proposed that the Constitution of (d) Armed rebellion one of the following disasters?
India was to be framed by an (a) Atmospheric (b) Terrestrial
_ (c) Among the given options, internal (c) Aquatic (d) Biological
elected Constituent Assembly of disturbance is not a circumstance for
the Indian people. proclamation of Emergency by the _ (b) An Avalanche is a Terrestrial type
(c) Any province which was not President of India. After 44th of disaster. An avalanche (also called
prepared to accept the Amendment Act, 1978, National a snow slide) is a rapid flow of snow
Constitution would be free to retain Emergency can only be declared on down a slope, such as a hill or
its constitutional provisions existing grounds of ‘‘External aggression or mountain.Terrestrial disasters include
at that time. war’’, also called as External earthquake, volcanic eruption,
(d) The British Government could Emergency and on the ground of landslide, avalanches, subsidence, etc.
enter into a separate Constitutional ‘‘armed rebellion’’, also called as
arrangement with any of the Internal Emergency. 37. An infection constantly
acceding provinces. maintained at a baseline level in a
33. Akbar issued a Farman in 1598
geographic area without external
_ (b) Cripps Mission proposed that a permitting in the city of Cambay
Constituent Assembly would be input is known as
(Khambat), Gujarat, the
formed to frame a new constitution for (a) Endemic (b) Pandemic
the country. This Assembly would construction of a
(c) Epidemic (d) Outbreak
have members elected by the (a) Temple (b) Church
provincial assemblies and also (c) Synagogue (d) Jain Upashraya _ (a) Endemic (of a disease) persisting
nominated by the princes. in a population or region, generally
_ (b) Akbar issue a Farman in 1598 having settled to a relatively constant
31. Which of the following statements permitting in the city of rate of occurrence. Thus, an infection
regarding constitutional Cambay(Khambat), Gujarat, the constantly maintained at a baseline
construction of a Church. Akbar level in a geographic area without
amendment is/are correct? permits the Jesuit order to build a external input is called Endemic.
1. The procedure for amendment to church in the city of Cambay, Gujarat. Endemic describes a disease that is
the Constitution is provided in The date given in the farman is: present permanently in a region or
Article 368. ‘‘written on the 25th day of the month population.
of Farvardin, year 42 of the Ilahi era.’’
2. A Bill to amend the Constitution 38. Streams and rivers coming from
can be introduced in either House 34. Domingo Paes, the medieval the mountains deposit heavy
of the Parliament. traveller, has described the city of materials of rocks and boulders in
3. The special procedure in Article (a) Madurai (b) Vijayanagara (a) Khadar (b) Bhangar
368 vests constituent powers upon (c) Arikamedu (d) Gingee (c) Bhabar (d) Terai
the ordinary legislation. _ (b) Domingo Paes was a Portuguese
Select the correct answer using the traveller who visited the Vijayanagara _ (c) Streams and rivers coming from
the mountains deposit heavy
codes given below: Empire, located in the Deccan in materials of rocks and boulders in
southern India, around 1520. He went Bhabar. Bhabar is a region located to
(a) Only 1 there as part of a group of traders
(b) 1, 2 and 3 south of the Lower Himalayas and the
from the then colony of Goa.His visit Sivalik Hills in Uttarakhand state of
(c) 2 and 3 took place during the rule of King India. It is the alluvial apron of
(d) 1 and 2 Krishna Deva Raya and Paes sediments washed down from the
CDS Solved Paper 2021 (II) 33

Sivaliks along the northern edge of 41. Who among the following was 45. Pinaka, developed in India, is a
the Indo-Gangetic Plain. In Bhabar,
streams and rivers comes from the
chosen for the Kuvempu Award (a) battle tank
mountains deposit heavy materials of 2020 ? (b) multi-barrel rocket launcher
rocks and boulders. (a) Dr. Rajendra Kishore Panda (c) anti-tank guided missile
(b) Amitav Ghosh (d) armoured utility vehicle
39. Which of the following statements
regarding the electoral rolls is/are
(c) Vikram Seth _ (b) Pinaka is a Multi Barrel rocket
(d) Chetan Bhagat ' launcher produced in India and
correct? developed by the Defence Research
1. There is one general electoral roll _ (a) Dr. Rajendra Kishore Panda was and Development Organisation for the
awarded Kuvempu Awar 2020.
for every territorial constituency. Indian Army. The system has a
Rajendra Kishore Panda is an Indian
maximum range of 40 km for Mark-I
2. No person is ineligible for poet and novelist from Odia
and 60 km for Mark-I enhanced
inclusion in the electoral roll on language. He has published 16 poetry
version and can fire a salvo of 12 HE
collections.The Kuvempu Rashtriya
grounds of religion, race, caste, Puraskar, the national award was
rockets in 44 seconds.
sex or any of them. instituted in memory of late poet 46. What is Itat e-dwiar ?
3. A citizen of India, not less than 18 laureate Kuvempu. The prestigious
(a) An Urdu magazine published from
years of age, can cast his/her vote award carries a cash award of Rs 5
Lucknow
unless disqualified under a law of lakh, a silver medal, and a citation.
(b) An e-filing portal of Income Tax
an appropriate legislature. 42. Avani Lekhara became the first Appellate Tribunal
Select the correct answer using the Indian woman to win two (c) A religious monument
codes given below: Paralympics medals at the Tokyo (d) Name of a UNESCO world
(a) Only 1 (b) 1, 2 and 3 Paralympics 2020. Which one of heritage site
(c) 2 and 3 (d) 1 and 2 the following was her discipline? _ (b) Itat e-dwar is an e-filling portal of
the Income Tax Appellate Tribunal.It is
_ (b) All statements 1,2 and 3 are (a) Badminton (b) Shooting an initiative to reduce the digital divide
correct.Right to equality provides that (c) Table Tennis among the people. The launch of the
there cannot be any ineligibility for (d) Archery e-Filing Portal 'itat e-dwar' aims to
inclusion in the electoral roll on
_ (b) Avani Lekhara is an Indian enhance the accessibility,
grounds if religion, race, caste, sex or
Paralympian and rifle shooter. She accountability and transparency in the
any of them.
won a gold medal in 10m air rifle day-to-day working of the ITAT.It is a
Requirements for registering to
standing and a bronze medal in 50m step towards the digitisation of ITAT.
vote:-You can enroll as a Voter if you:
rifle 3 positions at Tokyo 2020
l are an Indian citizen. 47. What is the range capability of
Paralympics.She became the first
l have attained the age of 18 years Indian women to win two paralympics Agni-P Ballistic Missile?
on the qualifying date i.e. 1st of medals at the Tokyo Paralympics (a) 1,000 - 2,000 km
January of the year of revision of 2020. (b) 2,000 - 3,000 km
electoral roll.
43. Bagram Air Base is located in (c) 3,000 - 4,000 km
l are ordinarily resident of the
(d) 4,000 - 5,000 km
part/polling area of the constituency (a) Turkey (b) Pakistan
where you want to be enrolled. (c) Afghanistan (d) Israel _ (a) Agni-P Ballistic Missile is a
l are not disqualified to be enrolled
canisterised missile with a range
as an elector. _ (c) Bagram Air base is located in capability between 1,000 and 2,000
Afganistan.Bagram Airfield-BAF also km. Agni-P is a new generation
40. Which one of the following known as Bagram Air Base is an advanced variant of the Agni class
Afghan military base, and formerly the (under IGMDP - Integrated Guided
statements is not correct about largest U.S. military base in Missile Development Program).
‘Totalitarianism’? Afghanistan. It is located next to the
(a) It is not akin to autocracy and ancient city of Bagram, 11 kilometres 48. Which one of the following Indian
authoritarianism. southeast of Charikar in the Parwan states has recently declared itself
(b) It usurps political freedom of the Province of Afghanistan. as the first rabies-free state?
individuals, but it doesn’t usurp 44. The Panchmuli Lake is situated (a) Punjab
personal freedoms.
near (b) Himachal Pradesh
(c) It implies abolition of civil society. (c) Tamil Nadu (d) Goa
(d) It is usually identified with a (a) Statue of Equality
one-party state. (b) Thiruvalluvar Statue _ (d) Goa has recently declared itself as
(c) Dhyan Buddha Statue the first rabies-free state in India.Goa
_ (b) Totalitarianism is a form of (d) Statue of Unity has not reported a single rabies case
government and a political system in the last three years since 2017.
that prohibits all opposition parties, _ (d) Panchmuli lake is situated near
outlaws individual opposition to the the Sardar Vallabhbhai Patel 'Statue of 49. Department of Public Enterprises
State and its claims, and exercises an Unity' in Kevadia, Gujarat.The Statue is a part of
extremely high degree of control and of Unity is a colossal statue of Indian
statesman and independence activist (a) Ministry of Finance
regulation over public and private life.
It usurps political freedom of the Vallabhbhai Patel, who was the first (b) Ministry of Heavy Industries
individuals as well as the personal deputy prime minister and home (c) PMO
freedoms. minister of independent India (d) Ministry of Commerce and Industry
34 CDS Solved Paper 2021 (II)

_ (a) Department of Public Enterprises naval exercise with the Italian UNESCO world heritage site tag.
is a part of ministry of Finance. Dholavira became the fourth site from
Navy. The exercise was conducted Gujarat and 40th from India to make
Department of Public Enterprises is
the nodal department for all the in the list, it is the first site of the ancient
Central Public Sector Enterprises (a) Ionian sea Indus Valley Civilisation (IVC) in India
(CPSEs) and formulates policy (b) Adriatic sea to get the tag.
pertaining to CPSEs. (c) Tyrrhenian sea 57. Which one of the following is the
50. Chronologically arrange the (d) Mediterranean sea
oldest football tournament of Asia?
following Indian-origin astronauts _ (d) Indian Naval Ship Tabar (a) Federation Cup
participated in two day Naval
on the basis of their flying into (b) Santosh Trophy
exercises with a frontline frigate of the
space starting with the first. Italian Navy after it entered the Port of (c) Durand Cup
1. Sunita Williams Naples as part of a deployment to the (d) Rovers Cup
Mediterranean Sea. _ (c) Durand Cup is the oldest football
2. Rakesh Sharma
tournament in Asia. It was first held in
3. Sirisha Bandla 54. Which one of the following 1888 in Annadale, Shimla. It is hosted
4. Kalpana Chawla countries has recently been by the Durand Football Tournament
Select the correct using the codes awarded a malaria-free Society.
given below. certification by the WHO 58. Who among the following is the
(a) 4-3-2-1 (b) 2-4-1-3 (a) Pakistan (b) Bhutan first Indian woman to win a medal
(c) 2-1-4-3 (d) 1-4-3-2 (c) China (d) Nepal in the Olympics ?
_ (b) The correct order of Indian origin _ (c) The World Health Organisation (a) Karnam Malleswari
(WHO) has declared China as
astronauts on the basis of their flying (b) Mirabai Chanu
into space starting with the first is ‘malaria-free’.The final decision on
(c) P.T. Vsha
Rakesh Sharma (1984), Kalpana awarding a malaria-free certification
rests with the WHO Director-General, (d) P. V. Sindhu
Chawla (1997), Sunita Williams(2006)
and Sirisha Bandla (2021). based on a recommendation by the _ (a) She is the first Indian woman to
independent Malaria Elimination win a medal at the Sydney Olympics
51. President Jovenel Moise, who was Certification Panel (MECP). in 2000. She is a retired Indian
assassinated recently, was the weightlifter. She received the Arjuna
55. Who amongst the following was Award and in 1999, she received the
President of which one of the honoured with Gandhi Peace Prize Rajiv Gandhi Khel Ratna award,
following countries? for the year 2020 ? India's highest sporting honour and
(a) Dominican Republic (a) Qaboos bin Said AI Said the civilian Padma Shri award.
(b) Haiti (b) Sheikh Mujibur Rahman 59. The Russian Federation
(c) Guatemala (d) Cuba (c) Ekal Abhiyan Trust participated in the Tokyo
_ (b)Jovenel Moïse was a Haitian (d) Sulabh International Olympics, 2020 under which of the
entrepreneur and politician who
served as the President of Haiti from _ (b) Sheikh Mujibur Rahman was following names?
honoured with Gandhi Peace Prize
2017 until his assassination in 2021. 2020 for his outstanding contributions (a) Russian Olympic Committee
He was sworn in as President in towards social, economic and political (b) Russian Federal Committee
February 2017 after winning the transformation through non-violent (c) Russian Sports Committee
November 2016 election. In 2019, and other Gandhian methods break (d) Russian Republican Committee
political unrest and calls for his Gandhi Peace Prize is an annual
resignation became a crisis. award instituted by the Government of
_ (a) Russian Federation participated in
the Tokyo Olympics 2020 under
52. Recently, which one among the India since 1995, the 125th Birth Russian Olympic Committee.Athletes
Anniversary commemoration year of didn’t compete under the Russian flag
following Ministries was formed Mahatma Gandhi. The award is open
under the Government of India? because of a punishment handed
to all persons regardless of down by the World Anti-Doping
(a) Ministry of New and Renewable nationality, race, language, caste, Agency (WADA). Originally, Russia
Energy creed or sex. had been suspended for four years of
(b) Ministry of Development of 56. Why was Dholavira in the news Olympic action, but in late 2020, that
North-Eastern Region punishment was reduced to two
(c) Ministry of Cooperation
recently? years.
(d) Ministry of Jal Shakti (a) Skeletons discovered at this site
(b) For its conservation effects 60. Which of the following
_ (c) The Ministry of Co-operation is a (c) For the discovery of gold and States/Union Territory/Region are
ministry under the Government of
India which was formed in July 2021.
precious stones at this site in special focus in the National
The ministry provides a separate (d) It received the UNESCO World Mission on Edible Oils - Oil Palm?
administrative, legal and policy Heritage Tag (a) Himachal Pradesh and Andaman
framework for strengthening the _ (d) Dholavira is an archaeological site and Nicobar Islands
cooperative movement in the country. at Khadirbet in Bhachau Taluka of (b) North-East Region and Andaman
Kutch District, in the state of Gujarat. and Nicobar Islands
53. Indian Naval Ship INS Tabar had Dholavira, the archaeological site of a (c) North-East Region and Gujarat
recently participated in a two-day Harappan-era city, received the (d) Uttar Pradesh and Gujarat
CDS Solved Paper 2021 (II) 35

_ (b) National Mission on Edible Oil-Oil _ (a) Statement 1 is correct. India uses _ (c) Contour bunding or contour
Palm (NMEO-OP) is a new Centrally only one time zone (even though it farming or Contour ploughing is the
Sponsored Scheme. It is proposed to spans across two geographical time farming practice of plowing and/or
have an additional 6.5 lakh hectares zones) across the whole nation and planting across a slope following its
for palm oil by 2025-26. It will involve all its territories, called Indian elevation contour lines.Such type of
raising the area under oil palm Standard Time (IST), which equates to farming practice is prevalent in hIlly
cultivation to 10 lakh hectares by GMT+05:30, five and half hours regions. Out of four options, Himachal
2025-26 and 16.7 lakh hectares by ahead of Greenwhich mean time. Pradesh is the states S witnessed
2029-30.The special emphasis of the such farming practice.
scheme will be in India’s 64. D had recently visited Khardung
North-Eastern (NE) states and the La, Nubra Valley and several 68. Which one of the following target
Andaman and Nicobar Islands due to Buddhist monasteries. Which one groups is the beneficiary of ‘Garib
the conducive weather conditions in of the following States / UT had Kalyan Rojgar Abhiyan’ scheme?
the regions.
she visited? (a) All rural women
61. Swami Dayanand Saraswati took (a) Himachal Pradesh(b) Uttarakhand (b) All returning migrants
inspiration from (c) Sikkim (d) Ladakh (c) All below poverty line households
(d) All new migrants to town
(a) Puranas (b) Vedas
_ (d) Khardung La, Nubra Valley and
(c) Medieval saints (d) Sufism several Buddhist monasteries are _ (b)The Government of India launched
situated in Ladakh. Ladakh is most the Garib Kalyan Rojgar Abhiyaan
_ (b) Swami Vivekananda took initiative to tackle the impact of
inspiration from Vedas. He was of the famous for breathtaking landscapes,
the crystal clear skies, the highest COVID-19 on shramik workers ( All
view that All that is called knowledge returning migrants) in India. It is a rural
is in the Vedas. Every word is sacred mountain passes, thrilling adventure
activities, Buddhist monasteries and public works scheme which was
and eternal, eternal as the soul, launched on 20th June, 2020 with an
without beginning and without festivals.
initial funding of ` 50,000 crore.
end.According to him, the three 65. Which one of the following states
essentials of Hinduism are belief in 69. Match List I with List II and select
God, in the Vedas as revelation, in the receives rainfall from both the
the correct using the codes given
doctrine of Karma and transmigration. Arabian Sea branch and the Bay of
below the lists.
62. The call for ‘renunciation of (all) Bengal branch of Monsoon ?
List I
voluntary associations with the (a) Punjab (b) Maharashtra (curve)
List II (Indication)
(c) Meghalaya (d) Tamil Nadu
(British) Government’ was given A. Lorenz curve 1. Inflation and employment
during _ (d) Tamil Nadu experiences two major
periods of rainfall: The South-West B. Phillips curve 2. Tax rates and tax revenue
(a) Non-Cooperation Movement Monsoon (Arabian sea branch) in
(b) Civil Disobedience Movement C. Engel curve 3. Inequality in distribution
Tamil Nadu starts in June and lasts till of income or wealth
(c) Quit India Movement September.
(d) Protest against partition of Bengal The North-East Monsoon (Bay of D. Laffer curve 4. Income and proportion of
Bengal Branch) in Tamil Nadu begins expenditure on food
_ (a) Non-Cooperation Movement gave in October and goes on till December.
a call for renunciation of all voluntary Codes
associations with the British 66. Inceptisols, Entisols, Vertisols and A B C D A B C D
Government.The Non-Cooperation (a) 3 4 1 2 (b) 2 1 4 3
Movement was a political campaign
Mollisols are orders of which one
of the following? (c) 3 1 4 2 (d) 2 4 1 3
launched on 4th September, 1920. by
Mahatma Gandhi to have Indians (a) Sugarcane (b) Soil _ (c) The correct matching is
revoke their cooperation from the (c) Coal (d) Copper A-3,B-1,C-4,D-2.
British Government, with the aim of The Lorenz Curve (the actual
inducing the British to grant _ (b) Inceptisols, Entisols, Vertisols and distribution of income curve), a
self-governance and full independence Mollisols are orders of Soil. Soil graphical distribution of wealth
(Purna Swaraj) to India. Taxonomy places soils into one of 12 developed by Max Lorenz in 1906,
categories known as ‘orders.’ Each of shows the proportion of income
63. Which of the following statements these orders represents a grouping of earned by any given percentage of
is/are correct with respect to Time soils with distinct characteristics and the population.
ecological significance.The 12 soil The Phillips curve is an economic
Zone in India? orders are Entisols, Inceptisols, concept developed by A. W. Phillips.
1. There is one standard time for the Andisols, Mollisols, Alfisols, He stated that inflation and
whole country. Spodosols, Ultisols, Oxisols, Gelisols, unemployment have a stable and
Histosols, Aridisols and Vertisols. inverse relationship.
2. Andaman and Nicobar Islands and
Engel curves relate the quantity of
Lakshadweep Islands have 67. S had witnessed contour bunding good consumed to income. If the
different Time Zones. and contour ploughing while good is a normal good, the Engel
3. Indian Standard Time (lST) is five visiting one of the states in India. curve is upward sloping. If the good is
and half hours behind GMT. Identify the state from the options an inferior good, the Engel curve is
Select the correct answer using the downward sloping.
given below. The Laffer curve illustrates a
codes given below. (a) Punjab (b) Haryana theoretical relationship between rates
(a) Only 1 (b) Only 2 (c) Himachal Pradesh of taxation and the resulting levels of
(c) 1 and 2 (d) 1 and 3 (d) Rajasthan the government's tax revenue.
36 CDS Solved Paper 2021 (II)

70. Which one of the following is a _ (d) The Directorate of Enforcement is 75. The Constitution of India
typical example of monopolistic a law enforcement agency and guarantees freedom of speech and
economic intelligence agency
competition? responsible for enforcing economic expression. But the freedom is
(a) Retail vegetable markets laws and fighting economic crime in subject to certain reasonable
(b) Market for soaps India. It is part of the Department of restrictions imposed by the state.
(c) Indian Railways Revenue, Ministry of Finance, These restrictions may relate to
(d) Labour market for software Government Of India.The prime
objective of the Enforcement which of the following?
engineers
Directorate is the enforcement of two 1. Defamation
_ (b) Market for soaps is a typical key Acts of the Government of India 2. Decency or morality
example of monopolistic competition namely, the Foreign Exchange
Monopolistic competition is a type of Management Act, 1999 (FEMA) and 3. Incitement to an offence
imperfect competition such that there the Prevention of Money Laundering Select the correct answer using the
are many producers competing Act, 2002 (PMLA).
against each other, but selling
codes given below.
products that are differentiated from 73. Which one of the following is not (a) Only 1 (b) 2 and 3
one another and hence are not correct? (c) 1 and 3
perfect substitutes. (a) Taxes on agricultural income is a (d) 1, 2 and 3
71. Following the Constitution (One subject under the State List. _ (d) All statements 1,2 and 3 are
Hundred Arid First Amendment) (b) Price control is a subject under the correct.The right to freedom of
Concurrent List. speech and expression is not
Act, 2016, the Parliament of India absolute and has been reasonably
(c) Insurance does not come under
enacted quite a few GST Acts in the Union List. restricted by the Constitution of India
the year 2017. Which one of the (d) ‘Forests’ is a subject under the under Article 19(2). The grounds for
following does not fall in this Concurrent List. imposing these restrictions
are:Sovereignty and Integrity of India,
category? _ (b) Price controls are restrictions set Security of the State, to maintain
(a) The Central Goods and Services in place and enforced by friendly relations with foreign states,
Tax Act governments, on the prices that can decency and morality, defamation
(b) The Integrated Goods and be charged for goods and services in and contempt of court.
Services Tax Act a market.In India, the government first
enacted price control in 2013 for the 76. Which one of the following
(c) The Goods and Services Tax
Drug Price Control order.This order amendments in the Constitution
(Compensation to States) Acts
gave local regulatory body and the of India made a Proclamation of
(d) The State Goods and Services Tax Pharmaceutical Pricing Authority the
Act power to set ceiling prices on the
Emergency immune from judicial
_ (c) The Constitution (One Hundred National List of Essential medicines. It review?
and First Amendment) Act, 2016, comes under the Union List. (a) 39th Amendment
amendment introduced a national (b) 40th Amendment
Goods and Services Tax (GST) in
74. Which one of the following
(c) 42nd Amendment
India from 1st July, 2017. It replaces statements is not correct about (d) 44th Amendment
all Indirect taxes levied on goods and the nature of India’s federal
services by the Indian Central and system? _ (*) The Thirty-eighth (38th)
State Governments. Amendment of the Constitution of
(a) There is no equality of India, officially known as The
Goods and Service Tax (GST) is representation of states in the Constitution Act, 1975, made the
levied on the supply of goods and Council of States. declaration of ‘‘The Emergency’’ final
services. Goods and Services Tax
(b) Sikkim was not initially included in and conclusive. In particular it
Law in India is a comprehensive, codified and enlarged the State’s
the Union as a full-fledged state.
multi-stage, destination-based tax power to remove fundamental rights
that is levied on every value addition.
(c) Special provisions have been laid
down in the Constitution of India from its citizens during states of
GST is a single domestic indirect tax emergency.
law for the entire country.
for Andhra Pradesh under Article
371 D. 77. Which one of the following British
72. Which one of the following is not (d) The Constitution of India enables officials was elevated to the
correct in respect of Directorate of the federal government to acquire
the strength of a unitary system position of Governor General after
Enforcement ? the Regulating Act of 1773 ?
during emergencies.
(a) It is a specialised financial
investigation agency under the _ (d)A national emergency modifies the (a) Warren Hastings
Department of Revenue, Ministry of federal system of government to a (b) Lord North
Finance. unitary one by granting Parliament the (c) Mountstuart Elphinstone
(b) It enforces the Foreign Exchange power to make laws on the 66 (d) Philip Francis
subjects of the State List (which
Management Act, 1999.
contains subjects on which the state _ (a) Among the given options,
(c) It enforces the Prevention of Governor General Warren Hastings
governments can make laws). Also,
Money Laundering Act, 2002. was elevated to the aforementioned
all state money bills are referred to the
(d) It enforces the Prohibition of Benami position after the Regulating Act of
Parliament for its approval.
Property Transaction Act, 1988. 1773. It limited his power by making
CDS Solved Paper 2021 (II) 37

the Governor-General one member of 81. Refraction of light, as it enters _ (a) Leaves of the nettle plant secrete
a five-man Supreme Council. He was methanoic acid which causes a
the first Governor of the Presidency of
from one transparent medium to
painful sting on touching.Nettle
Fort William (Bengal), the head of the another, is due to stings contain formic acid, histamine
Supreme Council of Bengal and so (a) change in temperature of the media and other chemicals. Formic acid is
the first de facto Governor-General of (b) change in the amplitude of light also known as methanoic acid
Bengal in 1772–1785. (c) change in speed of light (HCOOH).
78. Which of the following ideas was (d) internal property of light 85. Which of the following statements
preached by the Kherwar or Sapha _ (c) Light refracts whenever it travels at is/are correct ?
Har Movement of the 1870s? an angle into a substance with a
different refractive index (optical
1. All the bases are alkali.
(a) Acceptance of. the Hindu 2. All alkalis dissolve in water.
density). This change of direction is
pantheonic order
caused by a change in speed.When 3. Alkalis are soapy-to-touch, bitter
(b) Monotheism and internal social light travels from air into water, it
reform in taste and corrosive in nature .
slows down, causing it to change
(c) Philosophy of Yoga and Mimansa direction slightly. This change of Select the correct using the codes
(d) Polytheism direction is called refraction. given below.
_ (b) Monotheism and internal social 82. A bus starting from a bus-stand (a) Only 1
reform was preached by Sapha Har (b) 1 and 3
and moving with uniform
movement. Kherwar Movement was (c) 2 and 3
started apparently in 1868. It is also acceleration attains a speed of 20 (d) Only 3
known as Sapha Har Movement. This km/h in 10 minutes. What is its
movement had popularised the acceleration ? _ (c) Statement 2 and 3 are correct.
concept of One God as well as aimed Alkali is a base.It is a base that
at social reform.This movement (a) 200 km/h2 (b) 120 km/h2 dissolves in water which is very
initially taught Monotheism and (c) 100 km/h2 (d) 240 km/h2 important to remember.Not all bases
Internal social reform and but before are alkali but all alkali is base.Bases
_ (b) Acceleration is the rate of change that dissolve in water are alkali.Alkalis
its suppression it took the form of of the velocity of an object with
campaign against revenue are soapy to touch as they react with
respect to time. Accelerations are oils of our skin to form
endowment. vector quantities. soaps.Compounds that have an
79. In which one of the following Change in velocity = (final velocity − alkaline pH, such as baking soda,
initial velocity) v − v 0 = 20 − 0 = 20km/h often have a bitter flavor.Alkalis are
years was the Poona Sarvajanik
Time (t ) = 10min = 10/60 h very corrosive in nature and penetrate
Sabha established Now, Acceleration deeply.
(a) 1884 (b) 1876 (c) 1869 (d) 1870 Change in velocity
= 86. Which one of the following
_ (d) Poona Sarvajanik Sabha, was time
20 materials is present in a guard
established in 1870 at Pune by MG = = 120 km/h 2
Ranade in 1870.Poona Sarvajanik 10 tube (drying tube) that is used for
Sabha, was a sociopolitical 60 preparation of HCl gas?
organisation in British India which (a) Calcium chloride
started with the aim of working as a 83. Which one of the following is the
(b) Calcium bromide
mediating body between the correct reactivity order of metals (c) Calcium iodide
government and people of India and reacting with dilute HCl? (d) Calcium fluoride
to popularise the peasants' legal
(a) Mg >AI > Zn > Fe
rights.
(b) Mg< AI < Zn < Fe _ (a) During the preparation of
hydrogen chloride gas on a humid
80. Which one of the following was (c) Mg > Zn > Fe > AI day, the gas is usually passed
an important Pandya port, (d) Fe> Mg> AI> Zn through the guard tube containing
celebrated for its pearls in Sangam anhydrous calcium chloride. The role
_ (a) Those metals which are above in of anhydrous calcium chloride taken
Poems and Greek Accounts? the reactivity series displace hydrogen
in the guard tube is to absorb
(a) Muchiri from dilute acids as nitrogen
moisture from the gas,moisten the
(b) Korkai monoxide,dinitrogen monoxide. The
gas and absorb moisture the gas.
(c) Puhar arrangement of metals in decreasing
(d) Arikamedu order of their reactivities is called 87. Fertilizers are used to obtain higher
reactivity series or activity series of yields of crops. However, all
_ (b) Korkai was popularly known for its metals. This is a decreasing order of
nutrients are usually not available
pearl fishery and has a long history to chemical reactivity.Thus, correct
cherish finding its name in Tamil sequence is Mg>Al>Zn>Fe. in fertilizers. Which one of the
sangam literatures and also noted as following nutrients is usually not
‘Colchis’ in Periplus of the Erythrean 84. Which one of the following acids available in fertilizers?
Sea which is a maritime traders' guide is secreted by leaves of Nettle that
belong to the mid-first century and (a) Iron
‘Kolkhoi’ by Ptolemy who visited the
causes painful stings ? (b) Potassium
Southern peninsula in the second (a) Methanoic acid (b) Citric acid (c) Nitrogen
century. (c) Tartaric acid (d) Acetic acid (d) Phosphorus
38 CDS Solved Paper 2021 (II)

_ (a) Most fertilizers that are commonly internode of stem. This is usually seen (c) The member abstains from voting
used in agriculture contain the three in monocotyledonous plants. in the House without prior
basic plant nutrients: nitrogen, permission
phosphorus and potassium.Iron is a
91. The federation of which one of the
(d) The Member holds the office of the
micronutrient for plant growth which is following states has been Chairman of the Board of Directors
not available in fertilizers. Iron helps described as’ an indestructible of the National Coal Development
the plant move oxygen throughout the union of indestructible states’? Corporation Ltd
roots, leaves and other parts of the
plant, producing the green colour that (a) India (b) U.S.A. _ (c) Following are the disqualification
lets you know your plant is healthy. (c) Canada (d) Australia grounds of a Member of Parliament.
Many plants also rely on iron to The Constitution of India has provided
_ (b) The US Constitution, in all its (in Article 102) that a member of
complete the enzyme functions that provisions, looks to an ‘indestructible
keep the plant thriving. Parliament will be disqualified for
union composed of indestructible
membership if:
states’ while on the other hand India is
88. Rupa and Sachin observed an a ‘indestructible union of destructible l He holds any office of profit under
animal in their school garden. state’. In USA,States cannot be the Union or State Government
Rupa called it an insect while altered in terms of its size,name unlike (except that of a minister or any
in India where states are destructible other office exempted by
Sachin identified it as an Parliament).
provided in the Article 3 of the Indian
earthworm. Which one of the Constitution. l He is of unsound mind and stands
following characteristics confirms so declared by a court.
that it is an insect? 92. Who among the following is the l He is an undischarged insolvent.

(a) The animal had jointed legs. author of the famous essay, ‘The l He has ceased to be a citizen of
(b) Body of the animal had very little End of History and the Last Man’? India.
segmentation. (a) Francis Fukuyama (b) Daniel Bell l He is disqualified under any other
(c) Body of the animal was cylindrical. (c) Abraham Lincoln law by parliament.
(d) Body of the animal was bilaterally (d) Anthony Giddens
symmetrical,
95. Who among the following
_ (a) The End of History and the Last considered the Directive Principles
_ (a) Insects have segmented bodies, Man is a 1992 book of political
of State Policy as aiming at
which are divided into head, thotax philosophy by American political
and abdomen, three pairs of jointed scientist Francis Fukuyama which ‘furthering the goals of social
legs and external skeletons argues that with the ascendancy of inclusion' ?
(exoskeletons) made up of chitin.The Western liberal democracy after the
(a) B.N. Rau (b) Granville Austin
word ‘arthropod’ refers to all end of cold war and after the fall of
(c) K.C. Wheare
invertebrates with jointed legs. Soviet Union in 1991, mankind has
reached the end of history. (d) Rajni Kothari
Arthropods also have a hard
exoskeleton, like you might see on a
93. Which one of the following _ (b) Granville Austin considered
crab or on a beetle. Directive principles of state policy as
statements is not correct ? aiming at furthering the goals of social
89. Animals which are marine, (a) Allahabad High Court has a Bench inclusion. He was of the view that core
bilaterally symmetrical, have a at Lucknow. of the commitment to the social
coelom and a notochord, but never (b) Madhya Pradesh High Court has a revolution lies in Part III and IV, in the
form a vertebral column are placed Bench at Gwalior. Fundamental Rights and in the
Directive Principles of State Policy.
under which one of the following (c) Rajasthan High Court is located at
These are conscience of the
groups? Jodhpur.
Constitution.
(d) Guwahati High Court has no Bench.
(a) Chordata (b) Protochordata 96. D performs her train journey by
(c) Vertebrata (d) Mammalia _ (d) Guwahati High Court has the
largest number of Benches. The the shortest route from Bengaluru
_ (a) Animals in the phylum Chordata principal seat of the Guwahati High to New Delhi. Which one of the
share five key chacteristics that Court is at Guwahati in Assam.Apart
appear at some stage during their following rivers will she not cross
from the Principal Seat, the High
development: a notochord, a dorsal Court has 3 (three) outlying Benches,
while performing the journey?
hollow (tubular) nerve cord, viz, Kohima Bench for the State of (a) Narmada (b) Yamuna
pharyngeal gill arches or slits, a Nagaland (1972), Aizawl Bench for (c) Godavari (d) Krishna
post-anal tail and an endostyle/thyroid the State of Mizoram (1990) and
gland. Itanagar Bench for the State of _ (a) While traveiling from Bengaluru to
New Delhi, D will not cross Narmada
90. What is the location of intercalary Arunachal Pradesh (2000).
River. The Narmada, the largest West
meristem in plants? 94. Which among the following is not flowing river of the Peninsula, rises
near Amarkantak range of mountains
(a) Base of the leaves or internodes a condition for the disqualification
in Madhya Pradesh. Yamuna ,
(b) Stems or roots of a Member of Parliament? Godavari and Krishna River will be
(c) Tips of stems and leaves (a) Voluntary acquisition of citizenship encountered while travelling through
(d) Base of flower pedicel of a foreign country states of Karnataka
(krishna),Maharashtra (Godavari),
_ (a) Intercalary Meristems are located (b) Holding the office of the
at the internodes or the base of the Chairperson of the National Gujarat, Rajasthan,Uttar
leaves. The intercalary meristems help Commission for Women Pradesh(Yamuna) and New Delhi.
in increasing the length of the
CDS Solved Paper 2021 (II) 39

97. Identify the State on the basis of _ (b) Statement 2 is correct. India has _ (d) One end of the solenoid behaves
the largest population of adolescents as a magnetic North pole, while the
the following characteristics. other behaves as the South pole. The
in the world being home to 243 million
1. Tropic of Cancer passes through individuals aged 10-19 years and is field lines inside the solenoid are in
the state. regarded as a youthful population the form of parallel straight lines. This
with high potential.Since 1971 the indicates that the magnetic field is the
2. The state has more North-South same at all points inside the solenoid.
proportion of adolescent population
extension. That is, the field is uniform inside the
has remained around 21 per cent
3. The state has international border solenoid.
proportion of youth population
with Bangladesh and Myanmar. increased steadily from 16.5 per cent 103. A ray of light travelling from a
Select the correct using the codes in 1971 to 19.2 per cent in 2011. rarer medium to a denser medium
given below: 100. In a discussion, M from Arunachal (a) slows down and bends away from,
(a) Tripura Pradesh, J from Assam, N from the normal.
(b) Mizoram Meghalaya and S from Nagaland (b) slows down and bends towards
(c) Nagaland the normal.
are claiming that as per Census
(d) Manipur (c) speeds up and bends away from
2011, their state has the maximum the normal.
_ (b) Mizoram is a state in density of population. Identify the (d) speeds up and bends towards the
North-Eastern India, with Aizawl as its
seat of government and capital
person making the correct claim. normal.
city.Aizawl-Lunglei road, is now (a) M (b) J _ (b) When a ray of light enters from a
identified as the exact location where (c) N (d) S rarer to denser medium,light bends
the Tropic of Cancer passes in towards the normal and slows down.
Mizoram. The length of the state from _ (b) Assam ranks first among all the Since the speed of light changes as it
North to South is 277 km. At the states of North-East India in terms of
enters from a rarer to denser medium
broadest from East to West, it is 121 density of population in the region
the frequency of light does not
km.Thus, Mizoram has more with 397 person per sq. km followed
change but its wavelength changes.
North-South extension.Mizoram by Tripura (350 person per sq. km),
shares a 722-kilometre border with Meghalaya(132 person per sq. km), 104. An electric circuit is consisting of
the neighbouring countries of Manipur (122 person per sq. km), a cell, an ammeter and a nichrome
Bangladesh and Myanmar. Nagaland (119 person per sq. km),
Mizoram (51 person per sq. km) and wire of length l. If the length of
98. M wants to visit a place in a Union Arunachal Pradesh (16 person per sq. the wire is reduced to half (l/2),
Territory, which is located at 34° N Km) according to census year 2011. then the ammeter reading
and 77° E. Which one of the 101. Where should an object be placed in (a) decreases to one half
following Union Territories must front of a convex lens to get a real (b) gets doubled
he have planned to visit? (c) decreases to one-third
and enlarged image of the object?
(d) remains unchanged
(a) Andaman and Nicobar Islands (a) At twice the focal length
(b) Lakshadweep (b) At infinity _ (b) According to the Ohm’s Law
(c) Puducherry V = IR.
(c) Between the principal focus and
l I
(d) Ladakh twice the focal length but R = ρ ⇒ R ∝ l⇒R ∝
(d) Beyond twice the focal length A A
_ (b) M must have planned to visit Hence, the resistance of the wire is
Lakshadweep. The latitude of Ladakh,
Jammu and Kashmir, India is
_ (c) To form an inverted and enlarged directly proportional to the length of
image by a convex lens, the object
34.209515 degree N and the the wire.
should be placed between F and 2F
longitude is 77.615112 degree E. in front of the lens i.e. Between the In the first cast, R1 = KL1
KL
99. Consider the following statements principal focus and twice the focal In second case R 2 = 1
length. 2
with respect to the adolescent R1
∴ R2 =
population (age group of 10 to 19 A 2
years) of India, according to the
2F2 105. What is the effect of pressure of a
2011 Census. B'
2F1 B F2 human body on sand?
1. Adolescent population comprises F1
(a) Larger while standing than while
half the population of the country. (AB=Object) lying.
2. The group is regarded as youthful A' (b) Smaller while standing than while
population with high potential. (Convex lens) (A'B'=Image) lying.
.

3. In this group there are greater (c) Same while standing or lying.
number of females than males. 102. The magnetic field lines inside a (d) Larger while standing during the
Which of the statements given current carrying long solenoid are daytime and smaller during the
in the form of night time while lying.
above is/are correct?
(a) Only 1 (a) ellipse _ (a) The effect of pressure of a human
(b) Only 2 (b) parabola body on sand is larger while standing
than while lying because the pressure
(c) 1 and 2 (c) hyperbola exerted on a surface by an object
(d) 2 and 3 (d) parallel straight lines increases as the weight of the object
40 CDS Solved Paper 2021 (II)

increases or the surface area of _ (c) Graphite is a good conductor of 112. What are the constituents of alloy
contact decreases. Alternatively the electricity because in a graphite
pressure exerted decreases as the solder?
molecule, one valence electron of
weight of the object decreases or the each carbon atom remains free. Due (a) Pb and Zn
surface area of contact increases. to the free electrons in its framework, (b) Pb and Sn
106. An athlete completes one round of graphite can perform electricity. (c) Pb and Si
Therefore, graphite is said to be a (d) Pb and Co
a circular track of diameter 100 m good conductor of electricity.
in 20 s. What will be the _ (b) The composition of solder alloy is
109. What is the approximate 50 per cent lead (Pb) and 50 per cent
displacements after 1 minute and tin (Sn).It has a very low melting point
10 s, respectively? percentage of carbon in the
that is quite less than its constituting
(a) 0 m, 50 m (b) 300 m, 100 m Earth’s crust? metals (Pb and Sn). Thus, solder is
(c) 300 m , 50 m (d) 0 m, 100 m (a) 0·045% (b) 0'025% widely used in soldering and welding
(c) 0·015% (d) 0·005% of electrical wires.
_ (a) Displacement = Final position –
initial position = change in position. _ (b) Although widely distributed in 113. Which of the following limits the
After 1 minutes i.e. 60 sec, nature, carbon is not particularly number of trophic levels in a food
Displacement will be 0 m as change plentiful as it makes up only about
in position will be 0 m because athlete 0.025 per cent of Earth’s crust yet it
chain ?
will be at the Initial position. forms more compounds than all the (a) Deficient food supply
After 10 s , Displacement will be 50 m other elements combined.Carbon is a (b) Polluted air
(50 - 0 ) is the change on position of chemical element with the symbol C (c) Decrease in the available energy at
the athlete. and atomic number is 6. It is higher trophic levels
non-metallic and tetravalent element. (d) Parasitic organisms
107. Which of the following statements It is having 4 electrons available for
about universal indicator is/are covalent chemical bonds. It belongs _ (c) Energy in the form of food
to group 14 of the periodic table. decreases as it moves up trophic
true? levels in food chain because 90%
1. It is a mixture of several 110. When copper reacts with moist energy is lost as metabolic heat on
indicators. carbon dioxide (CO 2 ) in air, it each trophic level as organisms from
one trophic level are consumed by
2. It shows different colours at forms a green coating of which organisms from the next level. Trophic
different concentrations of one of the following compounds? Level Transfer Efficiency (TLTE)
hydrogen ions in solution. (a) Cupric carbonate measures the amount of energy that
3. It helps to determine the strength (b) Cuprous oxide is transferred between trophic
of given acid and base in titration. (c) Cupric oxide levels.Thus, decrease in the available
energy at higher trophic levels limits
Select the correct using the codes (d) Copper sulphate
the number of trophic levels in a food
given below. _ (a) Cupric carbonate, Cu 2(OH) 2CO 3, chain available.
(a) Only 1 (b) 1 and 2 is formed when copper(Cu) reacts 114. In pea, a pure tall plant (TT) is
(c) 2 and 3 (d) 1,2 and 3 with moist carbon dioxide (CO 2) in
air.Cupric Carbonate is used as a
crossed with a short plant (tt).
_ (d) Statements 1,2 and 3 are correct. pigment in products, paints and What will be the ratio of pure tall
A universal indicator is a pH indicator plants to short plants in the F2
made of a solution of several varnishes.Copper carbonate is used
compounds that exhibits several in artist paints to obtain desired generation?
smooth colour changes over a wide colours for which it is also known with (a) 1:1 (b) 1:3
range pH values to indicate the acidity different names such as verditer and (c) 3:1 (d) 2:1
or alkalinity of solutions.The main mountain green.Copper carbonate is
components of a universal indicator, highly demanded in fireworks and _ (a) The ratio of pure tall plants to pure
pottery glazes as pigment and short plants in F2 generation will be
in the form of a solution, are thymol
1:1 because according to the
blue, methyl red, bromothymol blue colorant.
mechanism of inheritance, when a
and phenolphthalein. This mixture is
important because each component 111. What is the name of the process pure homozygous tall plant and short
that converts sulphide ores into plant crossed together, the resulting
loses or gains protons depending offsprings of F1 generation are all
upon the acidity or alkalinity of the oxides by heating strongly in the tall.In the F2 generation, the tall and
solution being tested. presence of excess air? dwarf plants will be produced in 3 : 1
108. Which of the following carbon (a) Calcination ratio. Out of the three tall plants, one
tall plant will be homozygous tall (TT)
allotropes is/are good conductor(s) (b) Roasting·
and two tall plants will be
of electricity? (c) Smelting heterozygous tall (Tt).
(d) Incineration
1. Diamond 115. Which one of the following
2. Graphite _ (b) Roasting is a process of heating a
sulfide ore to a high temperature in statements about variations is not
3. Fullerene the presence of air. It is a step in the true?
Select the correct using the codes processing of certain deres.This (a) Variation is minimum in asexual
process is generally applied to sulfide reproduction.
given below. minerals. During roasting, the sulfide
(a) Only 1 (b) 1 and 2 (b) All variations in a species have
is converted to an oxide and sulfur is
(c) Only 2 (d) 1 and 3 equal chances of survival.
released as sulfur dioxide, a gas.
CDS Solved Paper 2021 (II) 41

(c) Changes in genetic constitution or species found in a particular area (c) Caserta, tapeworm, lice
result in variation. which interact with one-another (d) Paramecium, Amoeba, Caserta
(d) Variants can be selected by through interspecific interactions.
environmental factors. Community are named after local _ (a) Organism like Mushroom, Yeast
dominant vegetation type e.g. and Bread mould are saprophytic
_ (b) All variations in a species will not grassland community, forest fungi They break down dead organic
have equal chance of survival as community etc. compounds outside the body and
according to ‘Survival of the fittest’, absorb the simpler digested nutrients
only the species with adaptable 117. In the human body, blood flows particles. This is known as
variation which can adapt themselves through a process of double extracellular digestion.
to the environment can survive.
Selection of the variants occurs by the circulation. Which one of the 119. A sound wave has a frequencuy of
different environmental factors following statements is true in this 4 kHz and wavelength 30 cm. How
harmful. Variations that are not regard? long will it take to travel 2.4 km?
suitable or advantageous to the
(a) Oxygenated blood reaches the left (a) 2.0 s (b) 0.6 s
organism in the prevailing
environmental conditions are not
side of the heart from the lungs. (c) 1.0 s (d) 8.0 s
maintained. (b) Blood in the left side of the heart is
poor in oxygen and is brought to _ (a) Velocity (v) = frequency ( v ) ×
116. While studying vegetation of an the right side of the heart. wavelength ( w )
area, terms like ‘population’ and (c) Deoxygenated blood from the left = 4000 × 0.3 m
'community' are often used. Which side of the heart is brought to the = 1200 m/s
lungs for oxygenation. Distance given = 2400 m
one of the following statements Distance (d)
(d) Oxygenated blood from the right Thus, Time (t ) =
best describes a population ? side of the heart is sent around the Speed (v)
(a) A group of organisms of one body. = 2400 m/ 1200 s
species, living in the same area at
_ (a) Double circulation is a process by = 2.0 seconds.
the same time.
which blood passes twice through the
(b) A group of organisms of one
heart during one complete cycle.It is
120. An electric bulb is connected to a
species living in different areas oxygenated in the lungs and comes 110 V generator. The current is 0.2
during different seasons. back to the heart. From left side of A. What is the power of the bulb?
(c) A unit consisting of biotic and heart oxygenated blood is distributed
abiotic components. (a) 0.22 W
to all parts of the body. Hearts’s right
(d) A group of organisms of more than (b) 2.2 W
side collect deoxygenated blood and
one species, living in the same send it to lungs for oxygenation.
(c) 22 W
area at the same time. (d) 220 W
118. In which of the following groups
_ (d)A population refers to a group of _ C. Power = Voltage × Current
organisms of a species that
of organisms, is food material Voltage= 110 V
interbreed and live in the same place broken down outside the body and Current = 0.2 A
at the same time. It is a group of one absorbed ? . Thus, power = 110 × 0.2
particular species in a particular (a) Yeast, mushroom, bread mould = 22 watt
place. with interacts of species level. (b) Mushroom, Amoeba, green plants
Community is a group of all organism
CDS
Combined Defence Service

SOLVED PAPER 2021 (I)


PAPER I Elementary Mathematics

1. If the number 413283P759387 is _ (b) Given, 4. Consider the pairs of prime numbers
divisible by 13, then what is the 2 1000000 is divided by 7. (m, n ) between 50 and 100 such that
value of P ? 2 1000000 = 2 1 × 2 999999 = 2(2 3 )333333 m − n = 6. How many such pairs are
(a) 3 (b) 6 = 2( 8)333333 …(i)
there?
(c) 7 (d) 8 All the 8’s will be divided by 7 (a) 2 (b) 3 (c) 4 (d) 5
which will give remainder as 1 for _ (d) Given,
_ (d) Given, dividend each 8 ÷ 7 . Then all the remainders
= 413283P759387 will be multiplied and the result will
( m, n) is a pair of prime numbers
between 50 and 100 such that
and divisor = 13 be again divided by 7.
m − n = 6.
Let N = ∴ 2 1000000 = 2( 8)333333
Prime numbers between 50 and 100
n1n2n3n4n5n6n7n8n9n10 n11n12n13 333333
2( 8) is divided by 7 = 2(1) 333333
are the following :
is divided by 7
⇒ ( n11n12n13 ) – ( n8n9n11 ) + ( n5n6n7 ) 53, 59, 61, 67, 71, 73, 79, 83, 89, 97.
= 2 × 1÷7
– ( n2n3n4 ) + ( n1 ) = T Since, m − n = 6
= 2 ÷7
If T is divisible by 13 then the whole ∴Pairs will be (59, 53), (67, 61),
number N will be divisible by 13. Hence, the remainder will be 2
(73, 67), (79, 73), (89, 83).
when 2 1000000 is divided by 7.
N = 4132 83P759 387 Hence, the required number of pairs are 5.
Here, T = 387 − 759 + 83P 3. How many pairs of ( x , y ) can be
− 132 + 4
5. How many terms are there in the
chosen from the set {2, 3, 6, 8, 9}
x y following product?
= 83P − 500
such that + = 2, where x ≠ y? (a1 + a2 + a3 ) (b1 + b2 + b3 + b4 )
At P = 3 y x
(c1 + c 2 + c 3 + c 4 + c 5 )
⇒ T = 833 − 500 = 333 (a) Zero (b) One (a) 15 (b) 30 (c) 45 (d) 60
333 is not divisible by 13. (c) Two (d) Three
At P = 6 _ (d) Given product,
⇒ T = 836 − 500 = 336
_ (a) Given set, ( a1 + a2 + a3 ) ⋅ ( b1 + b 2 + b 3 + b 4 )
( x , y) ∈ {2, 3, 6, 8, 9} ⋅ (c1 + c 2 + c 3 + c 4 + c 5 )
336 is not divisible by 13.
x
+
y
= 2, where x ≠ y Number of terms in ( a1 + a2 + a3 ) = 3
At P = 7 ⇒ T = 837 − 500 = 337 y x
Number of terms in
337 is not divisible by 13.
x 2 + y2 ( b1 + b 2 + b 3 + b 4 ) = 4
At P = 8 ⇒ =2
y⋅ x Number of terms in
⇒ T = 838 − 500 = 338
⇒ x 2 + y 2 = 2 xy (c1 + c 2 + c 3 + c 4 + c 5 ) = 5
338 ÷ 13 = 26.
⇒ x + y 2 − 2 xy = 0
2
Since, all the terms are distinct.
Hence, the given number will be
⇒ ( x − y)2 = 0
divisible by 13 when P = 8. Therefore, the total number of terms in
⇒ x − y=0 the expansion
2. What is the remainder when ⇒ x = y = Product of number of terms
21000000 is divided by 7? But it is given that x ≠ y. in each component.
(a) 1 (b) 2 = 3 × 4 × 5 = 60
Hence, the required number of
(c) 4 (d) 6 pairs will be zero. Hence, option (d) is correct.
CDS Solved Paper 2021 (I) 43

6. What is the remainder when 3. a + b + c = 0 10. What is the value of x, if


27 27 − 15 27 is divided by 6? ⇒ a + b + = –c
b + b 2 − 2bx
⇒ a 3 + b 3 + 3ab( a + b ) = – c 3 =a?
(a) 0 (b) 1 (c) 3 (d) 4
⇒ a 3 + b 3 + c 3 = –3ab( a + b ) b − b 2 − 2bx
_ (a) Let N = 27 − 15
27 27
Hence, 1, 2 and 3 are correct. ab 2 ab
(a) (b)
27 27 = (21 + 6)27 (a + b) (a + 1)
3q + 2 + 3q − 2
= C 0(21)27 +
27 27
C1(21)27( 6)1 + … 8. If p = , then 2 ab ab
+ 27
C 27(21)0( 6)27 …(i) 3q + 2 − 3q − 2 (c) (d)
(a + 1)2 (a + b )2
15 27
= (21 − 6) 27
= 27
C 0(21) 27
what is the value of p − 3pq + 2 ? 2

− 27
C1(21)26( 6)1 + … − _ (c) Given,
(a) 0 (b) 1 (c) 2 (d) 3
27
C 27(21) ( 6) 0 27
…(ii) b+ b 2 − 2 bx
=a
_ (b) Given, b− b 2 − 2 bx
Subtracting Eq. (i) from Eq. (ii),
3q + 2 + 3q − 2
we get p=
3q + 2 − 3q − 2 b+ b 2 − 2 bx a
27 27
− 15 27
⇒ =
= 2 [ C1(21) ( 6) +
27 26 1 27
C 3(21) ( 6) 24 3 ( 3q + 2 + 3q − 2 ) b− b 2 − 2 bx 1
⇒p=
+…+ 27
C 27(21)0( 6)27] …(iii) ( 3q + 2 − 3q − 2 ) Using componendo-dividendo rule,
In Eq. (iii), we can see that all the ( 3q + 2 + 3q − 2 (b + b 2 − 2 bx ) + ( b − b 2 − 2 bx )
terms contains 6. × ⇒
( 3q + 2 + 3q − 2 ) (b + b 2 − 2 bx ) − ( b − b 2 − 2 bx )
Therefore, when Eq. (iii) is divided
by 6 i.e., 27 27 − 15 27 is divided by ( 3q + 2 + 3q − 2 ) 2
a+1
⇒p= =
6, the remainder will be zero. ( 3q + 2 )2 − ( 3q − 2 )2 a−1
Hence, the remainder is zero. [Q( a − b ) ( a + b ) = a 2 − b 2] 2b a+1
⇒ =
⇒ 2 b − 2 bx
2 a−1
7. If a + b + c = 0, then which of the 3q + 2 + 3q − 2
following are correct? + 2 3q + 2 3q − 2 ⇒ 2 b( a − 1) = ( a + 1) ⋅ 2 b 2 − 2 bx
p=
1. a 3 + b 3 + c 3 = 3abc 3q + 2 − ( 3q − 2 ) ⇒ b( a − 1) = ( a + 1) b 2 − 2 bx
[Q( a + b ) = a + b + 2 ab]
2 2 2
b( a − 1)
2. a 2 + b 2 + c 2 = − 2(ab + bc + ca ) ⇒ = b 2 − 2 bx
6q + 2 ( 3q )2 − (2 )2 ( a + 1)
3. a 3 + b 3 + c 3 = − 3ab(a + b ) ⇒p=
3q + 2 − 3q + 2 b 2( a − 1)2
Select the correct answer using ⇒ = b 2 − 2 bx
6q + 2 9q − 4 2 ( a + 1)2
the code given below. ⇒p=
4 b ( a − 1)2
(a) 1 and 2 only (b) 2 and 3 only ⇒ = b − 2x
3q + 9q 2 − 4 ( a + 1)2
(c) 1 and 3 only (d) 1, 2 and 3 ⇒p=
2 b( a − 1)2
…(i) ⇒2x = b −
_ (d) Given, a + b + c = 0 ( a + 1)2
⇒ 2 p − 3q = 9q − 4
2

1. a 3 + b 3 + c 3 = 3abc b( a + 1)2 − b( a − 1)2


⇒ 4 p2 + 9q 2 − 12 pq = 9q 2 − 4 ⇒2x =
We know that,
⇒ 4 p − 12 pq = − 4
2 ( a + 1)2
a 3 + b 3 + c 3 − 3abc
⇒ p − 3 pq = − 1
2
b[a + 1 + 2 a − ( a 2 + 1 − 2 a )]
2

= (a + b + c ) ⇒2x =
⇒ p − 3 pq + 2 = − 1 + 2 = 1
2
( a + 1)2
( a 2 + b 2 + c 2 − ab − bc − ca )
Hence, p2 − 3 pq + 2 = 1 b[4a ]
⇒ a 3 + b 3 + c 3 − 3abc = 0 ⇒2x =
( a + 1)2
[from Eq. (i)] 9. What is the unit digit in the
⇒ a + b + c = 3abc
3 3 3
…(ii) expansion of 67 32 ? ⇒x =
2 ab
(a) 1 (b) 3 (c) 7 (d) 9 ( a + 1)2
2. a 2 + b 2 + c 2
32
= − 2( ab + bc + ca ) _ (a) Given term is 67 . 11. The expression
We know that, Unit digit of ( 67 )32 = Unit digit of (x 3 − 1) (x 2 − 9x + 14)
(7 )32 = unit digit of (7 4 )8 simplifies to
(a + b + c ) = a + b + c 2 2 2 2
(x 2 + x + 1) (x 2 − 8x + 7 )
Now, 7 = 7 , 1
7 = 49
2
+ 2 ab + 2 bc + 2ca (a) (x − 1) (b) (x − 2 )
7 3 = 343 , 7 4 = 2401
⇒0 = a + b + c 2 2 2
4 (c) (x − 7 ) (d) (x + 2 )
Unit digit of 7 is 1.
+ 2( ab + bc + ca )
So, the unit digit of (7 4 )8 is 1. _ (b) Given expression,
[from Eq. (i)]
Hence, the unit digit in the ( x 3 − 1) ( x 2 − 9x + 14)
⇒ a2 + b 2 + c 2
expansion of ( 67 )32 is 1. ( x 2 + x + 1) ( x 2 − 8x + 7 )
= − 2( ab + bc + ca )
44 CDS Solved Paper 2021 (I)

We know, 1 x y
⇒a = Now, −
a − b = ( a − b ) ( a + b + ab )
3 3 2 2 x 2 − 4x − 3x + 12 a2 b2
( x − 1) ( x 2 + 12 + x ⋅ 1) 1 a2 b2
= = − = 1− 1= 0
( x 2 − 7 x − 2 x + 14) x − 7 x + 12
2
a 2
b2
=
( x 2 + x + 1) ( x 2 − 7 x − x + 7 ) Hence, option (a) is correct. 14. If ( x − k ) is the HCF of x 2 + ax + b
( x − 1) ( x + x + 1)
2

{ x ( x − 7 ) − 2( x − 7 )} 13. If
x y x y
+ = a + b and 2 + 2 = 2, and x 2 + cx + d , then what is the
= a b
( x 2 + x + 1) a b value of k?
{ x ( x − 7 ) − 1( x − 7 )} x y − d−b
then what is 2 − 2 equal to? (a)
d b
(b)
( x − 1) ( x − 7 ) ( x − 2 ) a b − a−c
= = (x − 2 )
c a
( x − 7 ) ( x − 1) + d−b
(a) − 2 (b) − 1 (c)
d b
(d)
Hence, the given expression (c) 0 (d) 1 c + a c+ a
results is ( x − 2 ).
_ (c) The given equations are _ (b) Given,( x − k ) is HCF of
12. What should be added to 1 1 x 2 + ax + b and x 2 + cx + d .
x + y − (a + b ) = 0
1 a b Since, ( x − k ) is the HCF.
to get
( x − 2) ( x − 4 ) and
1 1
x + 2 y−2 = 0 Therefore, ( x − k ) is a factor of
2x − 5 a2 b x 2 + ax + b and x 2 + cx + d .
? Using cross multiplication method,
( x − 5x + 6) ( x − 4 )
2
we get
Factor theorem states that ( x − p) is a
factor of f( x ) only if f( p) = 0.
1 1
(a) (b) x y 1 Using the fact that x 2 + ax + b and
(x 2 − 7 x + 12 ) (x 2 + 7 x + 12 ) 1 1 1
− (a+b) x 2 + cx + d have common factor
b a b
(c)
1
(d)
1 ( x − k ), we have
(x 2 − 7 x − 12 ) (x 2 + 7 x − 12 ) 1
−2
1 1 k 2 + ak + b = 0 …(i)
b2 a2 b2
and k 2 + ck + d = 0 …(ii)
_ (a) Let a should be added to x
=
y
1
to get 2 (a + b ) (a + b ) 2 Using Eq. (i) − Eq. (ii), we get
( x − 2 ) ( x − 4) − + − +
b b2 a2 a ( a − c )k + ( b − d ) = 0
2x − 5 1 (d − b )
= ⇒ k=
( x 2 − 5x + 6) ( x − 4) 1 1 (a − c )

1 ab 2 a 2b Hence, option (b) is correct.
Therefore, + a
( x − 2 ) ( x − 4) ⇒
x
=
y
− 2b + a + b − a − b + 2a 15. Consider the following statements.
2x − 5
= b2 a2 1. If x is directly proportional to z
( x − 5x + 6) ( x − 4)
2

=
1 and y is directly proportional to z,
(2 x − 5) a−b
⇒a = then ( x 2 − y 2 ) is directly
( x 2 − 5x + 6) ( x − 4) a2 b 2
1
proportional to z 2 .
− ⇒
x
=
y
=
1
( x − 2 ) ( x − 4) a−b a−b a−b
2. If x is inversely proportional to z
(2 x − 5) and y is inversely proportional to
⇒a = b2 a2 a 2b 2
( x − 3x − 2 x + 6) ( x − 4)
2
x 1
z, then ( xy ) is inversely
⇒ = proportional to z 2 .
1 a−b a−b
− Which of the above statement(s)
( x − 2 ) ( x − 4) b2 a2 b 2
a−b is/are correct?
(2 x − 5)
⇒a = (a) 1 only (b) 2 only
( x − 3) ( x − 2 ) ( x − 4) ⇒ x = b2
a−b (c) Both 1 and 2 (d) Neither 1 nor 2
1
− a 2b 2
( x − 2 ) ( x − 4) _ (c) Given, statements :
⇒ x = a2
(2 x − 5) − ( x − 3) 1. x ∝ z and y ∝ z
⇒a = y 1
( x − 3) ( x − 2 ) ( x − 4) ⇒ =
a−b a−b ∴ x = k1 z ⇒ x 2 = k12 z2 …(i)
2x − 5 − x + 3 2 2 2
and y = k 2 z ⇒ y 2 = k 22 z2 …(ii)
⇒a = a a b
( x − 3) ( x − 2 )( x − 4) a−b Here, k1 , k 2 are proportional constants.
(x − 2 ) ⇒ y= a2 Now, Eq. (i) − Eq. (ii), gives that,
=
( x − 3) ( x − 2 ) ( x − 4) a−b
x 2 − y 2 = z2( k12 − k 22 )
1 a2 b 2
= x 2 − y 2 = k 3 z2 [Let k 3 = k12 − k 22 ]
( x − 3) ( x − 4) ⇒ y = b2 ∴ x 2 − y 2 ∝ z2
CDS Solved Paper 2021 (I) 45

1 1
2. x ∝ and y ∝ _ (a) Given expression, 19. If x ( x − 1) ( x − 2) ( x − 3) + 1 = k 2 ,
z z
8x 4x x + 1 x −1
k4 − + − then which one of the following is a
∴ x = …(iii) 1 − x4 x2 + 1 x −1 x + 1 possible expression for k ?
z
k5 =
8x

4x (a) x 2 − 3x + 1
and y = …(iv) 1– x 4 x2 + 1
z (b) x 2 − 3x − 1
( x + 1)2 – ( x – 1)2
On multiplying Eq. (iii) and Eq. (iv), + (c) x 2 + 3x − 1
we get x2 – 1
8x 4x 4x (d) x 2 − 2 x − 1
k k
x⋅y= 4 ⋅ 5 = – +
1– x 4
x +1
2
x –12
z z _ (a) Given,
k6 [Q( a + b )2 – ( a – b )2 = 4ab ] x ( x − 1) ( x − 2 )( x − 3) + 1 = k 2
⇒x⋅ y = 2 [Let k 6 = k 4 ⋅ k 5] 2 2
...(i)
8x 4x ( x – 1 – x – 1) ⇒ x ( x – 1)( x – 2 )( x – 3) + 1 = k 2
=
z

⇒x⋅ y ∝ 2
1 1– x 4 x4 – 1 ⇒ ( x 2 – x )( x 2 – 5x + 6) + 1 = k 2
8x 8x
= +
z
⇒ x 4 – 5x 3 + 6x 2 – x 3 + 5x 2
Hence, both the statements are 1– x 4 x4 – 1
– 6x + 1 = k 2
correct. 8x 8x
= – =0 ⇒ x – 6x + 11x – 6x + 1 = k 2
4 3 2
1– x 4 1– x 4
16. What is the HCF of x 3 − 19 x + 30 ⇒ x + 9x 2 + 1 – 6x 3 – 6x + 2 x 2 = k 2
4
Hence, option (a) is correct.
and x − 5x + 6 ?
2
⇒ ( x 2 – 3x + 1)2 = k 2
18. For what integral value of x is [Q ( a + b + c )2 = a 2 + b 2 + c 2
(a) (x + 2 ) (x − 3) (b) (x − 2 ) (x + 3)
12
(c) (x + 2 ) (x − 1) (d) (x − 3) (x − 2 ) =x? +2 ab + 2 bc + 2ca]
6
7− ∴ k = x – 3x + 1
2
_ (d) Given function, 3
f( x ) = x − 19x + 30
3 7− 1
5− x 20. What is
and g ( x ) = x − 5x + 62 bc (a − b ) (a − c )
(a) 4 (b) 3 (c) 2 (d) 1
f( x ) = x 3 − 19x + 30 1
+
When, x =2 _ (c) Given, ca(b − c ) (b − a )
⇒ f(2 ) = 8 − 38 + 30 = 0 12
=x
6 1
Therefore, ( x − 2 ) is a factor of f( x ). 7 − + equal to ?
7 −
3 ab(c − a ) (c − b )
Using long division method, 5−x
x − 2 )x 3 − 19x + 30 (x 2 + 2 x − 15 12 (a) a + b + c (b) 3
⇒ =x
x − 2x
3 2 6 (c) ab + bc + ca (d) 0
7 −
− + 7( 5 − x ) − 3
_ (d) Given expression,
2 x 2 − 19x + 30 (5 − x ) 1 1
+
2 x 2 − 4x 12 bc( a − b ) ( a − c ) ca( b − c ) ( b − a )
− + ⇒ =x
6
7 − +
1
− 15x + 30 35 − 7 x − 3
ab(c − a ) (c − b )
− 15x + 30 (5 − x )
1 1
0 12 = −
⇒ =x bc( a − b ) ( a − c ) ca( b − c ) ( a − b )
∴ f( x ) = ( x − 2 ) ( x 2 + 2 x − 15) 6( 5 − x )
7 − 1
f( x ) = ( x − 2 ) ( x 2 + 5x − 3x − 15) 32 − 7 x +
ab( a − c ) ( b − c )
f( x ) = ( x − 2 ) ( x + 5) ( x − 3) …(i) 12( 32 − 7 x )
⇒ =x a( b − c ) − b( a − c ) + c( a − b )
Now, g ( x ) = x − 5x + 6
2 7( 32 − 7 x ) − 6( 5 − x ) =
abc( a − b ) ( b − c ) ( a − c )
g ( x ) = x 2 − 3x − 2 x + 6 12( 32 − 7 x )
⇒ =x ab − ac − ba + bc + ca − cb
g ( x ) = ( x − 3) ( x − 2 ) …(ii) 224 − 49x − 30 + 6x =
abc ( a − b ) ( b − c ) ( a − c )
From Eqs. (i) and (ii), we get 384 − 84x
⇒ =x 0
HCF of f( x ) and g ( x ) 194 − 43x =
abc( a − b ) ( b − c ) ( a − c )
= ( x − 3) ( x − 2 ) ⇒ 384 − 84x = 194x − 43x 2
=0
Hence, option (d) is correct. ⇒ 43x 2 − 278x + 384 = 0
Hence, the given expression is equal to
8x x +1
4x ⇒ 43x 2 − 86x − 192 x + 384 = 0
zero.
17. What is − 2 + ⇒ 43x ( x − 2 ) − 192( x − 2 ) = 0
1− x4 x +1 x −1 21. For how many real values of k is
⇒ ( x − 2 ) ( 43x − 192 ) = 0
x −1 6kx 2 + 12kx − 24 x + 16 a perfect
− equal to? x =2 Q x ≠ 192 
x +1 Q
 43  square for every integer x ?
(a) 0 (b) 1 (c) 2 (d) 4 Hence, the value of x is 2. (a) Zero (b) One (c) Two (d) Four
46 CDS Solved Paper 2021 (I)

 1 
2 q
_ (c) Let = ( x )2 −     x 2 + 2  ⇒ =6
1
f( x ) = 6kx 2 + 12 kx − 24x + 16 x   p
  x
f( x ) = ( 6k )x 2 + 12( k − 2 )x + 16 ⇒
q
=6
=  x −   x +   x 2 + 2 
1 1 1
We know, a polynomial 1
 x  x  x 
ax 2 + bx + c will form a perfect ⇒ q =6
square if D = 0. i.e., b 2 − 4ac = 0 3 5 17
= × × Therefore, p + q = 1 + 6 = 7
2 2 4
In the given polynomial, we have Hence, ( p + q ) is equal to 7.
[From Eqs. (i), (ii) and (iii)]
a = 6k,b = 12( k − 2 ) and c = 16
=
255 25. 4 x + 12x 2 − x − 3 is divisible by
3
Put b 2 − 4ac = 0
16
⇒ [12( k − 2 )]2 − 4( 6k ) (16) = 0 (a) (2 x + 1) only
1 255
Hence, x − 4 =
4
(b) (2 x − 1) only
⇒ 12 × 12( k − 2 ) − 24 × 16k = 0 2
x 16
⇒ 12( k − 2 )2 − 2 × 16k = 0 (c) Both (2 x + 1) and (2 x − 1)
⇒ 3( k 2 + 4 − 4k ) − 2 × 4k = 0
23. If the equation 4 x 2 − 2kx + 3k = 0 (d) Neither (2 x + 1) nor (2 x − 1)
⇒ 3k 2 + 12 − 12 k − 8k = 0 has equal roots, then what are _ (c) Let f( x ) = 4x + 12 x − x − 3 …(i)
3 2

⇒ 3k − 20k + 12 = 0
the values of k ?
(a) 4, 12 (b) 4, 8 We know by factor theorem that ( x − a )
⇒ 3k 2 − 18k − 2 k + 12 = 0 is a factor of f( x ) only if f( a ) = 0
⇒ 3k( k − 6) − 2( k − 6) = 0 (c) 0, 12 (d) 0, 8
Or
⇒ ( k − 6) ( 3k − 2 ) = 0 _ (c) Given equation, f( x ) will be divisible by ( x − a ) only if
k = 6 or k =
2 4x 2 − 2 kx + 3k = 0 f( a ) = 0
3 In ax 2 + bx + c = 0, condition for For the factor (2 x + 1),
Hence, for two values of k, the given equal roots is 1
Put 2 x + 1 = 0 i.e., x = − into Eq. (i),
polynomial is a perfect square. D = 0 i.e., b 2 − 4ac = 0 2
1 5 For the given equation, we get
22. If x + = , then what is 3 2
a = 4, b = − 2 k and c = 3k − 1  − 1  − 1
x 2 f   = 4  + 12  
1 Put b 2 − 4ac = 0  2   2   2 
x 4 − 4 equal to?
− 1
x ⇒ ( − 2 k )2 − 4( 4) ( 3k ) = 0 −   −3
195 255 625 ⇒ 4k 2 − 48k = 0  2 
(a) (b) (c) (d) 0
16 16 16 ⇒ k 2 − 12 k = 0  − 1  1 1
= 4⋅   + 12 ⋅   + − 3
1 5 ⇒ k( k − 12 ) = 0  8   4 2
_ (b) Given, x + =
x 2 k = 0 or k = 12 1 1
=− + 3 + − 3= 0
1 5 2 2
x + = …(i) Hence, the required values of k are
x 2 0 and 12. Therefore, f( x ) is divisible by (2 x + 1).
2 2
⇒  x + 1  =  5 For the factors (2 x − 1),
    24. If the sum as well as the product
 x 2 1
of the roots of the equation Put 2 x − 1 = 0 i.e., x = into Eq. (i),
1 25 2
⇒x + 2
+2 = px 2 − 6x + q = 0 is 6, then what we get
x2 4
is ( p + q ) equal to? 3 2
f   = 4  + 12   −   − 3
1 25 1 1 1 1
⇒ x2 + = −2
x2 4 (a) 8 (b) 7 (c) 6 (d) 5 2 2 2 2
1 17
_ (b) Given equation, + 12 ⋅   − − 3
⇒ x +
2
= 1 1 1
…(ii) = 4⋅
x2 4 px − 6x + q = 0
2
…(i) 8  4 2
1 17
⇒ x2 + −2 = −2 For ax 2 + bx + c = 0 =
1 1
+ 3 − − 3= 0
x2 4 −b 2 2
2 Sum of the roots =
⇒ x − 1 = 9 a Therefore, f( x ) is divisible by (2 x − 1)
 
 x 4 c also.
and product of the roots =
x − 1 = 3 a Hence, option (c) is correct.
⇒   …(iii)
 x 2 For Eq. (i), it is given that
26. Which one of the following fractions
1 Sum of the roots = 6
Now, x 4 − will have minimum change in its
x4 ( − 6)
⇒ − =6 value if 3 is added to both the
2
= ( x 2 )2 −  2 
1 p numerator and the denominator of
x  ⇒
6
=6 all the fractions ?
p
=  x 2 − 2   x 2 + 2 
1 1 2 3
(a) (b)
 x   x  ⇒ p=1 3 4
[Qa 2 − b 2 = ( a − b ) ( a + b )] Product of the roots = 6 (c)
4
(d)
5
5 6
CDS Solved Paper 2021 (I) 47

_ (d) Given, 3 is added to both the 28. The income of A, B and C are in 1 1 1
30. If + +
numerator and the denominator of
the ratio 7 : 9 : 12 and their 1×2 2×3 3× 4
all the fractions.
expenditures are in the ratio 1 99
2
(a) = 0.67 +…+ = , then what is the
3 8 : 9 : 15. If A’s saving is one-fourth n(n + 1) 100
2 + 3 5 of his income, then the ratio of value of n?
Now, = = 0.83 savings of A, B and C is
3+ 3 6 (a) 98 (b) 99
(a) 56 : 99 : 69 (b) 99 : 56 : 69
Difference = 0.83 − 0.67 = 016
. (c) 100 (d) 101
3 (c) 69 : 56 : 99 (d) 99 : 69 : 56
(b) = 075. _ (b) Given,
4 _ (a) Given, 1 1 1
3+ 3 6 + + +…+
Now, = = 0.86 Ratio of incomes of A : B : C = 7 : 1× 2 2 × 3 3 × 4
4+ 3 7 9 : 12
1 99
=
Difference = 0.86 − 075
. = 011
. Ratio of expenditures of
n( n + 1) 100
4 A : B : C = 8 : 9 : 15
(c) = 0.80 2 – 1 3 – 2 4 – 3 1 
5 Let income of A, B and C be 7 x , 9x ⇒ + + + ...+
 1 × 2 2 × 3 3 × 4 n( n + 1)
4+ 3 7 and 12 x respectively.
Now, = = 0.88 99
5+ 3 8 Let expenditures of A, B and C be =
8 y, 9 y and 15 y respectively. 100
Difference = 0.88 − 0.80 = 0.08 1 1 1 1 1 1
5 According to the question, ⇒ − + − + −
(d) = 0.83 1 1 2 2 3 3 4
6 7x − 8y = × 7x 1 1 99
5+ 3 8 4 +…+ − =
Now, = = 0.89 7x n ( n + 1) 100
6+ 3 9 ⇒ 7x − = 8y
4 1 1 99
⇒ − =
difference = 0.89 − 0.83 = 0.06 21x 1 ( n + 1) 100
5 ⇒ = 8y
Hence, the fraction will show the 4 ( n + 1) − 1 99
6 ⇒ =

x
=
32 ( n + 1) 100
minimum change when numerator
y 21 99
and denominator are added with 3. ⇒
n
=
Let x = 32 k , y = 21k ( n + 1) 100
27. Let the average score of a class of Savings of A = 7 x − 8 y
boys and girls in an examination ⇒ n × 100 = 99 × ( n + 1)
= k(7 × 32 − 8 × 21) ⇒ 100n = 99n + 99
be p. The ratio of boys and girls
= k(224 − 168) = 56k ⇒ 100n − 99n = 99
in the class is 3 : 1. If the average
score of the boys is ( p + 1), then Savings of B = 9x − 9 y ⇒ n = 99
what is the average score of the = k( 9 × 32 − 9 × 21) Hence, the value of n is 99.
girls? = k( 9 ( 32 − 21) = 99k
Savings of C = 12 x − 15 y
31. A trader gives successive discounts
(a) ( p − 1) of 20%, 10% and 5% respectively.
(b) a( p − 2 ) = k(12 × 32 − 15 × 21)
What is the overall discount?
(c) ( p − 3) = k( 384 − 315) = 69k
(a) 30% (b) 31.6%
(d) p Hence, the ratio of savings of A, B
and C is 56 : 99 : 69. (c) 32.8% (d) 35%
_ (c) Let the number of boys be 3a
and number of girls be a. 29. A train 200 m long passes a _ (b) Let the marked price be ` 100.
Selling price after three successive
Let the average score of the girls platform 100 m long in 10 sec.
discount
be N. What is the speed of the train? 100 − 20   100 − 10 
Sum of scores of boys = 3a( p + 1) (a) 40 m/s (b) 30 m/s =   × 
 100   100 
Sum of scores of girls = Na (c) 25 m/s (d) 20 m/s 100 − 5 
Sum of scores of the whole class ×   × 100
_ (b) Total distance covered by train  100 
= ( 3a + a ) p = 4ap in 10 sec 80 90 95
∴3a( p + 1) + Na = 4ap = × × × 100
= length of platform + length of 100 100 100
⇒ 3ap + 3a + Na = 4ap train 4 × 19 × 9
= 100 m + 200 m = 300 m =
⇒ a( 3 + N ) = 4ap − 3ap = ap 10
⇒ a( 3 + N ) = ap distance 300
∴ Speed = = = ` 68.4
⇒3 + N = p time 10
Discount = ` 100 − ` 68.4 = ` 31.6
⇒N = p− 3 = 30 m/s 316
.
Hence, the speed of the train is Discount% = × 100
Hence, the average score of the 100
girls is ( p − 3). 30 m/s.
= 316
. %
48 CDS Solved Paper 2021 (I)

32. A sum of money was invested at _ (a) Let the usual speed of the man 38. In a mixture of 80 L of a liquid and
simple interest at a certain rate be 5x m/min and usual time be y min. water, 25% of the mixture is the
for 5 yr. Had it been invested at a Distance covered by the man liquid. How much water should be
5% higher rate, it would have = 5xy m. added to the mixture so that the
4
fetched ` 500 more. What was Speed of man = × 5x liquid becomes 20% of the mixture?
5
the principal amount? (a) 15 L (b) 20 L
= 4x m/min
(a) ` 2000 (b) ` 1800 (c) 24 L (d) 25 L
∴ Distance covered by the man
(c) ` 1600 (d) ` 1200
= 4x ( y + 12 ) m _ (b) Quantity of liquid in the mixture
_ (a) Let the principal be `x. QDistance remains same = 25% of 80
Difference in SI 25
⇒ 5xy = 4x ( y + 12 ) = × 80
P × (Difference in Rate) × T 100
= 5 y = 4 y + 48
100 = 20 L
x ×5×5 y = 48 min
500 = Quantity of water in the mixture
100 36. The cost price of 100 mangoes is = 80 − 20
Hence, the principal x = ` 2000 equal to the spelling price of = 60 L
33. The difference between the 80 mangoes. What is the profit Let x litre of water is added in the
compound interest (compounded percentage? mixture.
annually) and the simple interest (a) 16% (b) 20% (c) 24% (d) 25% Quantity of liquid in the new mixture
= 20% of ( 80 + x )
on a certain sum of money at 12% _ (d) Cost price of 100 mangoes =
per annum for 2 yr is ` 72. What Selling price of 80 mangoes 20 = 20% of ( 80 + x )
Let CP of 1 mango = ` 1 20
is the principal amount? ⇒ × ( 80 + x ) = 20
100
(a) ` 6500 (b) ` 6000 CP of 80 mangoes = ` 80
⇒ 80 + x = 100
(c) ` 5500 (d) ` 5000 CP of 100 mangoes = ` 100
⇒ x = 20 L
SP of 80 mangoes = ` 100
_ (d) Let the principal be ` P. Hence, 20 L of water is added to the
Let’s a person buys and sell
Difference between compound mixture.
80 mangoes
interest and simple interest for 2 yr
SP − CP 
= ` 72 Profit% =   × 100
39. If 20 persons can clean 20 floors in 20
2  CP  days, then in how many days can 16
⇒  R  % of P = 72
  100 − 80 
= 
 100  persons clean 16 floors?
 × 100
144  80  (a) 25 days (b) 20 days
⇒ × P = 72
100 × 100 1 (c) 20 days (d) 16 days
= × 100 = 25%
72 × 100 × 100 4
⇒ P= _ (c) Here, M1 = 20, W1 = 20, D1 = 20
144 37. X sells his goods 25% cheaper and M 2 = 16, W 2 = 16, D2 = ?
Hence, the principal P = ` 5000 than Y and 25% dearer than Z. Using the formula,
How much percentage is Z’s M1 × D1 M 2 × D2
34. A train travels 600 km in 5 h and =
goods cheaper than Y ?
the next 900 km in 10 h. What is W1 W2
100
the average speed of the train? (a) % (b) 40% 20 × 20 16 × D2
3 ⇒ =
(a) 80 km/h (b) 90 km/h 200 20 16
(c) 50% (d) %
(c) 100 km/h (d) 120 km/h 3 ⇒ D2 = 20 days.

_ (c) Average speed of the train _ (b) Let the price of Y’s goods be 40. Let the work done by ( x − 1) men in
Total distance 600 + 900 ` 100. ( x + 1) days be y. Let the work done
= =
Total time 5 + 10 Price of X’s goods by ( x + 2) men in ( x − 1) days be z. If
=
1500
= 100 km/h
= 100 − 25% of 100 = ` 75 y : z = 9 : 10, then what is the value of
15 Price of Z’s goods x?
4  100  (a) 8 (b) 9 (c) 10 (d) 12
35. Walking at th of his usual =  × 75
 100 + 25 
5
4 _ (a) Here,
speed, a man is 12 min late for = × 75 = ` 60 M1 = ( x – 1), D1 = ( x + 1), W1 = y
5
his office. What is the usual time and,
∴ Percentage cheaper of Z’s
taken by him to cover that goods than Y’s goods M 2 = ( x + 2 ), D2 = ( x – 1), W 2 = Z
distance?
 100 − 60  Using the formula,
=  × 100
(a) 48 min (b) 50 min  100  M1 × D1 M 2 × D2
=
(c) 54 min (d) 60 min = 40% W1 W2
CDS Solved Paper 2021 (I) 49

( x − 1) × ( x + 1) 10
⇒ _ (c) Let the two alternate natural ( x − 3) log10 = 3 log10 2
y numbers be x and x + 2. 2
( x + 2 ) × ( x − 1) ( x − 3)[log10 10 − log10 2 ] = 3 log10 2
= According to question,
z 1 1 7 Qlog m = log m − log n
+ =  a a a

x2 − 1 y x + 2 24 n
⇒ =
x
x + 2x − x − 2
2
z x +2 + x 7 ( x − 3) [1 − log10 2 ] = 3 log10 2
=
x −1
2
9 x(x + 2 ) 24 [Q log a a = 1]
⇒ =
x + x −2
2
10 2x + 2 7 x − 3 − x log10 2 + 3 log10 2
=
x 2 + 2x 24 − 3 log10 2 = 0
⇒ 10x − 10 = 9x + 9x − 18
2 2

7 x 2 + 14x = 48x + 48 x [1 − log10 2 ] = 3


⇒ x − 9x + 8 = 0
2
3
⇒ x 2 − 8x − x + 8 = 0 7 x 2 + 14x − 48x − 48 = 0 x =
1 − log10 2
⇒ x ( x − 8) − 1( x − 8) = 0 7 x 2 − 34x − 48 = 0
( x − 8) ( x − 1) = 0 7 x − 42 x + 8x − 48 = 0
2
46. What is the least value of
⇒x = 8 [Q x ≠ 1] 7 x ( x − 6) + 8( x − 6) = 0 3 sin 2 θ + 4 cos 2 θ ?
(7 x + 8) ( x − 6) = 0
41. What is log10 31 ⋅ 25 equal to? 8
(a) 5 (b) 4
⇒ x ≠ − , so, x = 6 (c) 3 (d) 2
(a) 3 − 5log10 2 7
_ (b) Given, 3sin θ + 4cos θ
2 2
(b) 3 − 2 log10 2 Therefore, the two alternate natural
(c) 5 − 5log10 2 numbers are 6 and 8. = 3 sin2 θ + 3 cos 2 θ + cos 2 θ
(d) 5 − 3log10 2 The sum of these alternate natural = 3 + cos 2 θ
3125 numbers are = 6 + 8 = 14
_ (a) Given, log10 31.25 = log10 minimum value = 3 + 1
100 44. If n is any natural number, then [Qminimum value of cos θ = –1]
125 125 × 8
= log10 = log10 5 2n − 1 is always divisible by how =4
4 4×8
many natural numbers? π
= log10
1000 47. If sin θ cos θ = k, where 0 ≤ θ ≤ , then
32
(a) One (b) Four 2
(c) Six (d) Eight which one of the following is correct?
Qlog m = log m − log n
 a
n
a a
 _ (d) If n = 1, then 5 − 1 = 5 − 1
2n 2
(a) 0 ≤ k ≤ 1
= log10 10 − log10 2
3 5 = 24 (b) 0 ≤ k ≤ 0.5 only
= 3 log10 10 − 5 log10 2 If n = 2, then (c) 0.5 ≤ k ≤ 1 only
5 4 − 1 = ( 5 2 − 1) ( 5 2 + 1) (d) 0 < k < 1
= 3 − 5 log10 2
= 24 × 26
[Q log a a = 1] _ (b) Given that, sinθ cos θ = k, where
If n = 3, then π
42. What is the square root of 0≤ θ≤ ⇒ 0 ≤ 2θ ≤ π
5 6 − 1 = ( 5 2 − 1)( 5 4 + 5 2 + 1) 2
15 − 4 14 ? = 24( 5 4 + 5 2 + 1) It can also be written as,
Therefore, it is observed from the π π
(a) 2 2 − 7 either 0 ≤ 2θ ≤ or ≤ 2θ ≤ π
above data that 24 is always a 2 2
(b) 3 2 − 2 7 factor of 5 2n − 1 ∀ n ∈ N. π
(c) 15 − 7 ⇒ sin 0 ≤ sin2θ ≤ sin
The factors of 24 are 1, 2, 3, 4, 6, 2
(d) 5 − 3 8, 12, 24.
⇒ 0 ≤ 2 sinθ cos θ ≤ 1
_ (a) 15 − 4 14 Hence, number of natural numbers
⇒ 0 ≤ sinθ cos θ ≤
1
that always divides 5 2n − 1 are 8.
2
= 7 + 8−2 ×2 × 2 × 7
45. If 5 x − 3 = 8, then what is x equal to ? ⇒ 0≤ k≤
1
= ( 7 )2 + (2 2 )2 − 2 × 7 × 2 2 2
3 3
(a) (b) Hence, 0 ≤ k ≤ 0.5 only is the correct
= ( 7 − 2 2 )2 1 − log10 2 1 + log10 2
option.
= 7 −2 2 2 5
(c)
1 − log10 2
(d)
1 − log10 2 π
48. If p = sin 2 θ + cos 4 θ for 0 ≤ θ ≤ ,
43. The sum of the reciprocals of two 2
x −3
7 _ (a) Given, 5 =8 then consider the following
alternate natural numbers is .
24 Taking log on both sides at base 10. statements.
−3
What is the sum of the numbers? log10 5 x = log10 8 3
1. p can be less than .
(a) 12 (b) 13 ( x − 3)log10 5 = log10 2 3 4
(c) 14 (d) 16 [Q log a mn = nlog10 m] 2. p can be more than 1.
50 CDS Solved Paper 2021 (I)

1 − sinθ   1 + cos θ 
Which of the above statements _ (a) The equation, =   +  
 cos θ   sinθ 
is/are correct? x + y − 2 xy sin θ = 0
2 2 2

(a) 1 only (b) 2 only Can be written as  1 − sinθ − cos θ 


 cos θ 
(c) Both 1 and 2 (d) Neither 1 nor 2 x 2 − 2 xy sin2 θ + y 2 = 0
sinθ − sin2 θ + 1 + cos θ − sinθ
_ (d) Given that, p = sin θ + cos θ
2 4 For the real solution of the
− sinθ cos θ − cos θ − cos 2 θ
π equation, =
for 0 ≤ θ ≤ . sinθ cos θ
2 b 2 − 4ac ≥ 0
⇒ (– 2 y sin2 θ)2 − 4 × 1 × y 2 ≥ 0 −(sin2 θ + cos 2 θ) + 1 − sinθ cos θ
P = sin2 θ + cos 2 θ .cos 2 θ =
sinθ cos θ
P = sin2 θ + cos 2 θ(1 − sin2 θ) ⇒ 4 y 2[sin4 θ − 1] ≥ 0
= sin2 θ + cos 2 θ − sin2 θ cos 2 θ As, 4 y 2 ≥ 0, sin4 θ − 1 ≥ 0 = −1

(2 sinθ cos θ) 2 ⇒ sin θ ≥ 1


4
53. If cosec θ − cot θ = m , then what is
= 1−
4 Also, sin4 θ ∈ [0, 1] cosec θ equal to?
sin2 2θ ⇒ sinθ = 1 1 1
p= 1− (a) m + (b) m −
4 π
⇒ θ= m m
Q 0 ≤ sin2 2θ ≤ 1 2 m 2 m 1
(c) + (d) +
1 − sin2 2θ x 2 − 2 xy + y 2 = 0 2 m 2 2m
=− ≤ ≤0
4 4 ( x − y) ( x − y) = 0
_ (d) Given that,
1 sin2 2θ ⇒ x = y
⇒ = 1− ≤ 1− ≤ 1+ 0 cosec θ − cot θ = m …(i)
4 4
51. Consider the following As, cosec 2 θ − cot 2 θ = 1,
3
⇒ = ≤ P≤1 inequalities (cosec θ − cot θ) (cosec θ + cot θ) = 1
4
Hence, both statements are
1. sin 1º < cos 57º ⇒ cosec θ + cot θ =
1
…(ii)
incorrect. 2. cos 60º > sin 57º m
Adding Eqs. (i) and (ii),
49. What is the ratio of the greatest Which of the above is/are
1
correct? 2 cosec θ = m +
to the smallest value of m
π (a) 1 only (b) 2 only
m2 + 1
2 − 2 sin x − sin 2 x, 0 ≤ x ≤ ? (c) Both 1 and 2 (d) Neither 1 nor 2 cosec θ =
2 2m
(a) − 3 (b) − 1 _ (a) Statement 1 =
m2
+
1
(c) 1 (d) 3 sin1º < cos 57 º 2m 2m
sin1º ≤ sin( 90º − 57 º ) =
m
+
1
_ (*) Let f( x ) = 2 − 2 sin x − sin x
2

sin1º ≤ sin 33º 2 2m


π
for 0 ≤ x ≤ . As sin x is an increasing function in
2 54. Let ABC be a triangle right angled at
f( x ) = 3 − 1 − 2 sin x − sin x 2 Ist quadrant. C, then what is tan A + tan B equal to?
= 3 − [1 + 2 sin x + sin x ] 2 Statement 1 is correct. a a2 b2 c2
(a) (b) (c) (d)
= 3 − (1 + sin x )2 Statement 2 bc bc ca ab
f( x ) max = 3 − (1 + 0) 2 cos 60º > sin 57 º
_ (d) Given, ∠c = 90º
Q 0 ≤ x ≤ π  sin( 90º − 60º ) > sin 57 º
c 2 = a2 + b 2 …(i)
 2  sin 30º > sin 57 º
(Pythagoras theorem)
= 3 − 1= 2 As sin x is an increasing function in
Ist quadrant, Statement 2 is A
f( x ) min = 3 − (1 + 1)2 incorrect.
=3−4 Hence, only Statement 1 is true.
= −1 c
2 52. If p = sec θ − tan θ and b
Hence, the required ratio =
−1 q = cosec θ + cot θ, then what is
= −2 p + q ( p − 1) equal to?
(a) − 1 (b) 0
50. If the equation C a B
(c) 1 (d) 2
x 2 + y 2 − 2xy sin 2 θ = 0 contains b a2 + b 2
_ (a) Given that, p = sec θ − tanθ tan A + tan B =
a
+ =
real solution for x and y, then q = cosec θ + cot θ b a ab
(a) x = y (b) x = − y p + q ( p − 1) = (sec θ − tanθ) c2
= [Qfrom Eq. (i)]
(c) x = 2 y (d) 2x = y + (cos ec θ + cot θ) (sec θ − tanθ − 1)
ab
CDS Solved Paper 2021 (I) 51

55. Let cos α + cos β = 2 and ⇒ 2 cos 2 θ − 5 cos θ + 2 = 0 59. A pole on the ground leans at
⇒ 2 cos 2 θ − 4 cos θ
sin α + sin β = 0, where 0 ≤ α ≤ 90º, 60º with the vertical. At a point
0 ≤ β ≤ 90º.What is the value of − cosθ + 2 = 0 x metre away from the base of
cos 2α − cos 2β ? ⇒ 2 cos θ(cos θ − 2 ) the pole on the ground, two
(a) 0 − 1 ( cos θ − 2 ) = 0 halves of the pole subtend the
(b) 1 ⇒ cosθ ≠ 2 same angle. If the pole and the
(c) 2 1 point are in the same vertical
So, cosθ = [Q cos θ ∈ [0, 1]]
(d) Cannot be determined due to 2 plane, then what is the length of
insufficient data ⇒ θ = 60º the pole?
π
when, θ ∈=  0, 
(a) 2 x m (b) 3 x m
_ (a) Given, cos α + cos β = 2  2  (c) 2x m (d) 2 2 x m
and sinα + sinβ = 0 2
 3 3 _ (b)
cos α + cos β = 2 ∴ sin2 60 =   =
 α + β α − β 2  2  4
C M
⇒ 2 cos   cos   =
 2   2 
57. What is (1 + cot θ − cosec θ)
 α + β  cos  α − β 
⇒ cos     =1 (1 + tan θ + sec θ) equal to? D
 2   2  60°
(a) 4 (b) 3
…(i)
(c) 2 (d) 1 A B
sinα + sinβ = 0 x
 α + β  cos  α − β  0 _ (c) Given,
⇒ 2 sin    = (1 + cot θ − cos ec θ) (1 + tanθ + sec θ) Let the length of the pole is BC.
 2   2  ∠CBM = 60º
 1 + cos θ − 1   1 + sinθ + 1 
 α + β  cos  α − β     
⇒ sin    =0  sinθ sinθ   cos θ cos θ  ∠CBA = 90 − 60º
 2   2 
sinθ + cos θ − 1  cos θ + sinθ + 1 = 30º
…(ii) =
 sinθ   cos θ  D is mid point of pole BC.
Dividing Eq. (ii) by Eq. (i)
∠BAD = ∠CAD (given)
α + β
tan  =0 =
(sinθ + cos θ)2 − 1
In ∆ABD and ∆ACD,
 2  sinθ cos θ
α +β ∠BAD = ∠CAD (given)
⇒ =0 sin2 θ + cos 2 θ + 2 sinθ cos θ − 1 AD = AD
2 = (common)
sinθ cos θ
α +β=0 BD = CD (given)
1 + 2 sinθ cos θ − 1
Substitute α + β = 0 in Eq. (i) = ∆ABD ≅ ∆ACD
sinθ cos θ
α − β ∠ADB = ∠ADC
cos   =1 [Q sin2 θ + cos 2 θ = 1]
 2  BDC is a straight line,
=2 1
α −β ∠ADB = ∠ADC = × 180 = 90º
⇒ =0
2 58. If 6 + 8 tan θ = sec θ and 2
⇒ α −β = 0 8 − 6 tan θ = k sec θ, then what is the In ∆ABD,
cos 2α − cos 2β value of k 2 ? cos 30° =
BD
AB
= − 2 sin(α + β ) sin(α − β ) (a) 11 (b) 22
3
=0 (c) 77 (d) 99 ⇒ BD = x
2
5
56. If sec θ + cos θ = , where _ (d) Given, 6 + 8 tanθ = sec θ BC = 2 BD [as BD = DC ]
2 and 8 − 6 tanθ = k sec θ 3
=2 × x= 3x
0 ≤ θ ≤ 90º, then what is the value Squaring and Adding both equations 2
of sin 2 θ? ⇒ ( 6 + 8 tanθ)2 + ( 8 − 6 tanθ)2
60. A vertical tower standing at the
(a)
1
(b)
1 = sec 2 θ + ( k sec θ) 2 corner of a rectangular field
4 2 ⇒ 36 + 64 tan2 θ + 96 tanθ subtends angles of 60º and 45º at
3
(c) (d) 1 + 64 + 36 tan2 θ − 96 tanθ the two nearer corners. If θ is the
4
= (1 + k 2 )sec 2 θ angle that the tower subtends at
5
_ (c) Given, sec θ + cos θ = ⇒ 100 + 100 tan2 θ = (1 + k 2 ) sec 2 θ the farthest corner, then what is
2
⇒ 100(1 + tan2 θ) = (1 + k 2 ) sec 2 θ cot θ equal to?
where 0 ≤ θ ≤ 90º
⇒ 100 sec 2 θ = (1 + k 2 )sec 2 θ 1
1 5 (a) (b) 2
+ cos θ = 2
cos θ 2 ⇒ 100 = 1 + k 2
2 4
1 + cos 2 θ 5 ⇒ k 2 = 99 (c) (d)
⇒ = 3 3
cos θ 2
52 CDS Solved Paper 2021 (I)

r 2( h − 2 r ) = 0 ⇒
22
× 4x 2 × 3x = 1617
_ (c) D
60°
E
h − 2r = 0 [Q r ≠ 0] 7
1617 × 7
h 2
= = (2 : 1) ⇒ x3 =
r 1 22 × 4 × 3
343
θ 62. A solid sphere of diameter 60 mm ⇒ x3 =
A C 8
is melted to stretch into a wire of 7
length 144 cm. What is the ⇒ x =
45° 2
diameter of the wire?
Therefore, radius of the cylinder is 7 cm
(a) 0.5 cm (b) 1 cm 21
and height of the cylinder is cm.
B (c) 1.5 cm (d) 2 cm 2
Let ABCD be a rectangle. A vertical CSA of cylinder = 2 πrh
tower is standing at a point C. The
_ (b) Given, diameter of sphere
= 60 mm = 6 cm 22 21
angle made by the point B and D =2 × ×7 ×
are 45º and 60º respectively. Radius of sphere = 3 cm 7 2

Let the angle made by A i.e., Length of the wire = 144 cm = 22 × 21 ⇒ 462 cm 2
the farthest corner be θ. Let the radius of wire be r cm. 64. The difference between the outside
EC
In ∆BCE, tan 45° = As the sphere is melted to form and the inside surface area of a
BC wire, their volumes are equal. cylindrical pipe 14 cm long is 44 cm 2 .
∴Volume of wire = Volume of
EC
⇒ 1= The pipe is made of 99 cm 3 of metal.
BC sphere
⇒ BC = EC 4 If R is the outer radius and r is the
πr 2(144) = π( 3)3 inner radius of the pipe, then what is
In ∆DEC, tan 60° =
EC 3
CD 4 × 32
( R + r ) equal to?
⇒ r2 =
⇒ 3=
EC 22
CD
144
(Take π = )
9 7
⇒ r =
2
⇒ EC = 3 CD 36 (a) 9 cm (b) 7.5 cm
∴ AC is the diagonal of the 1
rectangle. ⇒ r= (c) 6 cm (d) 4.5 cm
4
AC = BC 2 + CD 2 1 _ (d) Given, Length of the pipe = 14 cm
⇒ r= cm
2 Difference between the outside and
EC 2
= EC 2 + Hence, the diameter of wire is 1 cm.
inside surface area of a cylinder pipe
3 = 44 cm2
2
= EC 63. The ratio of the radius of base to 2 π( R − r ) h = 44
3 the height of a cylinder is 2 : 3. If 22
2 × ( R − r ) × 14 = 44
In ∆ACE, cot θ =
AC the volume of the cylinder is 7
EC 1617 cm 3 , then what is the R −r=
1
…(i)
2 curved surfaced area of the 2
EC
2 Volume of the cylinder = 99 cm3
= 3 = 22
EC 3 cylinder? (Take π = )
7 π( R 2 − r 2 ) × 14 = 99
61. A cone and a hemisphere have 2 2 22
(a) 242 cm (b) 385 cm × ( R + r ) ( R − r ) × 14 = 99
equal bases and equal volumes. 2 7
(c) 462 cm (d) 770 cm2
99 × 2
What is the ratio of the height of R + r= [Qfrom Eq. (i)]
the cone to the radius of the _ (c) Let the radius of the cylinder be 22 × 2
hemisphere? 2 x cm and the height of the cylinder 9
be 3x cm. R + r= = 4.5 cm
(a) 1 : 1 (b) 2 : 1 2
2x
(c) 3 : 2 (d) 4 : 3 65. A metal solid cube of edge 24 cm is
_ (b) Given that radius and volumes of melted and made into three small
the cone and hemisphere are equal. cubes. If the edges of two small cubes
Let radius of the cone and 3x are 12 cm and 16 cm, then what is
hemisphere be r unit and height of the surface area of the third small
cone be h unit. cube?
According to question, (a) 1200 cm2
Volume of cone = Volume of Volume of cylinder = 1617 cm 3
(b) 1800 cm2
hemisphere
πr h = 1617
2
(c) 2400 cm2
1 2 2
πr h = πr 3
3 3 (d) 3600 cm2
CDS Solved Paper 2021 (I) 53

_ (c) Let the edge of third small cube 68. A cone of height 24 cm has a _ (b) If the rectangular paper is rolled
be x cm. curved surface area 550 cm 2 . along its breadth, then circumference of
As the solid cube of edge 24 cm is the base = 22 cm and height of cylinder
melted and mode into three small
What is the ratio of its radius to = 44 cm
22 ⇒ 2 πr1 = 22
cubes, their volumes are equal. slant height? (Take π = )
⇒ (12 )3 + (16)3 + x 3 = (24)3 7 22
⇒ 2 × × r1 = 22
5 5 7 7 7
x 3 = (24)3 − (12 )3 − (16)3 (a) (b) (c) (d)
12 13 25 25 7
= 4 3[6 3 − 3 3 − 4 3 ] ⇒ r1 = or 3.5 cm
2
= 64 [216 − 27 − 64] _ (c) Let the radius of cone be r cm
and slant height of the cone be l cm. Volume of cylinder = πr12 h
= 64 [125]
∴ l =r + h
2 2 2 ( y) = π( 3.5)2 × 44 cm 3
= x = 4 × 5 = 20
⇒ l = r + (24)
2 2 2 If the rectangular paper is rolled along
∴ Surface area of third small cube
⇒ l 2 = r 2 + 576 its length, then circumference of the
=6×x 2
base = 44 cm and height of cylinder
⇒ l 2 − r 2 = 576 …(i) = 22 cm
= 6 × 400
Given, curved surface area ⇒ 2 πr2 = 44 ⇒ r2 = 7 cm
= 2400 cm2
= 550 cm 2 Volume of the cylinder ( x ) = π(7 )2 × 22
66. A conical vessel whose internal πrl = 550 x π × (7 )2 × 22
radius is 5 cm and height 24 cm is 22 ∴Required Ratio = =
× r × l = 550 y π × (3.5)2 × 44
full of water. The water is 7
550 × 7 2
emptied into a cylindrical vessel rl = = = 2 :1
with internal radius 10 cm. What 22 1
is the height to which the water rl = 175 …(ii) 70. A hollow spherical shell is made up
rises? From Eqs. (i) and (ii), of a metal of density 3 g /cm 3 . If the
(a) 1 cm (b) 2 cm l −r
2 2
576 internal and external radii are 5 cm
=
(c) 3 cm (d) 4 cm lr 175 and 6 cm respectively, then what is
l r 576 22
_ (b) Let the height to which the − = the mass of the shell? (Take π = )
water level rises be h cm. r l 175
7
Volume of cylinder = Volume of Let the ratio of radius to slant
(a) 1144 g (b) 1024 g
cone height be x. i.e., r / l = x
1 576 (c) 840 g (d) 570 g
1 −x =
π(10)2 × h = π( 5)2 × 24
3 x 175 _ (a) Volume of the hollow sphere
25 × 8 175x 2 + 576x − 175 = 0 =
4
π( R 3 − r 3 )
h=
100 175x + 625x − 49x − 175 = 0
2 3
h = 2 cm 25x (7 x + 25) − 7(7 x + 25) = 0 4 22 3
= × ( 6 − 53 )
5 7
67. A metal solid cube of side 22 cm (25x − 7 )(7 x + 25) = 0
4 22
7 = × × (216 − 125)
is melted to make a cone of x = 3 7
25
height 21 cm. What is the radius − 25  4 22
 = × × 91
of the base of the cone? Qx ≠
 7  3 7
22 1144
(Take π = ) Hence, the ratio of radius to slant = cm 3
3
7 height is 7 : 25.
Mass of the shell
(a) 11 cm (b) 16.5 cm 69. A rectangular paper is 44 cm = Volume of shell × Density
(c) 22 cm (d) 27.5 cm long and 22 cm wide. Let x be the 1144
= × 3 = 1144 g
_ (c) Let the radius of cone be r cm. volume of the largest cylinder 3
As the solid cube is melted and formed by rolling the paper along
form a cone, their volume are its length and y be the volume of 71. A cloth of 3 m width is used to make
equal. the largest cylinder formed by a conical tent 12 m in diameter with
Volume of cone = Volume of cube rolling the paper along its width. a slant height of
1 2 What is the ratio of x to y? 7 m. What is the length of the cloth?
πr h = (22 )3
22
3
(Take π = )
22 (Take π = )
22 × 22 × 22 × 3 × 7 7
r2 = 7
22 × 21 (a) 21 m (b) 28 m
(a) 1 : 1 (b) 2 : 1
r = 22 cm (c) 1 : 2 (d) 3 : 2 (c) 44 m (d) 66 m
54 CDS Solved Paper 2021 (I)

The diagonals of the trapezium divides


_ (c) Let the length of the cloth be l m. 74. A wire is in the form of a circle
each other in equal proportion.
diameter = 12 m of radius 70 cm. If it is bent in the
AP BP
radius = 6 m form of a rhombus, then what is ⇒ =
PC PD
∴Area of the cloth = Curved surface 22
its side length? (Take π = ) 4
=
2x − 1
area of cone 7 4x − 4 2 x + 4
22
l×3= × 6×7 (a) 55 cm (b) 75 cm 1 2x − 1
7 =
(c) 95 cm (d) 110 cm x − 1 2x + 4
l = 22 × 2
l = 44 m _ (d) Given, radius of circle = 70 cm 2 x + 4 = (2 x − 1) ( x − 1)
Let the side of rhombus be a cm. 2x + 4 = 2x 2 − x − 2x + 1
72. A sphere of diameter 6 cm is 2 x 2 − 5x − 3 = 0
If the wire in the form of circle is
dropped into a cylindrical vessel bent in the form of rhombus, then 2 x − 6x + x − 3 = 0
2
partly filled with water. The perimeter of rhombus and the
circumference of circle would be 2 x ( x − 3) + 1( x − 3) = 0
radius of the vessel is 6 cm. If the
sphere is completely submerged equal. (2 x + 1) ( x − 3) = 0
∴Perimeter of rhombus Q x ≠ − 1
in water, then by how much will x =3  
the surface level of water be = Circumference of circle  2 
raised? 22
4× a=2 × × 70 77. ∆ABC is similar to ∆DEF. The
7
(a) 0.5 cm (b) 1 cm perimeters of ∆ABC and ∆DEF are 40
2 × 22 × 10
(c) 1.5 cm (d) 2 cm a= = 110 cm cm and 30 cm respectively. What is
4
6
_ (b) Radius of the sphere = cm the ratio of ( BC + CA ) to ( EF + FD )
2 75. If the perimeter of a semi-circular equal to?
= 3 cm park is 360 m, then what is its (a) 5 : 4 (b) 4 : 3 (c) 3 : 2 (d) 2 : 1
Let the height of water level raised 22
be h cm
area ? (Take π = ) _ (b)
7
The height of water raised in the A D
(a) 3850 m 2 (b) 7700 m 2
vessel will be due to the volume of
sphere. (c) 11550 m 2 (d) 15400 m 2
4
Therefore, π( 6)2 h = π( 3)3 _ (b) Let the radius of semi-circular B C
3 be r m. E F
4×3×3 ∴ Perimeter of semi-circular park
h= ∴ ∆ABC ~ ∆DEF
6×6 = 360 m
Perimeter of ∆ABC AB BC AC
h = 1 cm ⇒ πr + 2 r = 360 = = =
Perimeter of ∆DEF DE EF DF
r  + 2  = 360
22
73. A sector is cut from a circle of  7  (property of similar triangles)
radius 21 cm. If the length of the 40 AB BC AC
36 = = =
arc of the sector is 55 cm, then r× = 360 30 DE EF DF
7
what is the area of the sector? Let BC = 4x , EF = 3x and AC = 4 y and
r = 70 m DF = 3 y
2 2
(a) 577.5 cm (b) 612.5 cm ∴ Area of semi-circular park BC + CA 4x + 4 y
(c) 705.5 cm 2 (d) 725.5 cm 2 1 1 22 =
= πr 2 = × × 70 × 70 EF + FD 3x + 3 y
2 2 7
_ (a) Let θ be the angle obtained by 4( x + y) 4
= 7700 m2 = = = 4:3
sector on the centre of the circle. 3( x + y) 3
Length of the arc = 55 cm 76. In a trapezium ABCD, AB is
θ parallel to DC. The diagonals AC 78. Two isosceles triangles have equal
⇒ × 2 πr = 55 cm
360 and BD intersect at P. If vertical angles and their areas are in
where, r = 21 cm AP : PC = 4 : ( 4 x − 4 ) and the ratio 4.84 : 5.29. What is the ratio
θ 55 × 7 BP : PD = (2x − 1) : (2x + 4 ), then of their corresponding heights?
=
360 2 × 22 × 21 what is the value of x? (a) 11 : 23 (b) 23 : 25
θ 5 3 (c) 22 : 23 (d) 484 : 529
= (a) 4 (b) 3 (c) (d) 2
360 12 2
θ
_ (c)
∴ Area of sector = πr 2 _ (b) A B A D
360
2x–1
5 22
= × × 21 × 21
12 7 P
= 577.5 cm 2 2x+4 4x–4
D C B C E F
CDS Solved Paper 2021 (I) 55

Let ABC and DEF are the isosceles 81. The sides of a right-angled Similarly,
triangles whose vertical angles are ∆ANQ ~ ∆ABC
equal. triangle are in the ratio
⇒ ∆ABC ~ ∆DEF
x : ( x − 1) : ( x − 18). What is the AN NQ
= ⇒
2 AM
=
NQ
perimeter of the triangle? AB BC 3 AM 12
Let height of ∆ABC be h1 and NQ = 8 cm
height of ∆DEF be h2. (a) 28 units (b) 42 units
2 (c) 56 units (d) 84 units ∴ PM + QN = 4 + 8 = 12 cm
ar ( ∆ABC )  h1 
=  83. AB and CD are the diameters of a
ar ( ∆DEF )  h2  _ (c) Given, the sides of a right
angled triangle are in the ratio circle which intersect at P. Join
[Qproperty of similar triangles]
2
x : x − 1 : x − 18. AC , CB, BD and DA. If ∠PAD = 60º,
4.84  h1  h
=  ⇒ 1 =
22
= 22 : 23
Let the side are xy, ( x − 1) y and then what is ∠BPD equal to?
529
.  h2  h2 23 ( x − 18) y.
(a) 30º (b) 60º (c) 90º (d) 120º
As, xy is the longest side,
79. ABC is a triangle right angled at ( xy)2 = ( x − 1)2 y 2 + ( x − 18)2 y 2 _ (d)
D B
A and AD is perpendicular to BC. x 2 = x 2 + 1 − 2x
If BD = 8 cm and DC = 125
. cm, + x 2 + 324 − 36x
then what is AD equal to?
x − 38x + 325 = 0
2
P
(a) 7.5 cm (b) 8.5 cm x 2 − 13x − 25x + 325 = 0 60°
(c) 9 cm (d) 10 cm x ( x − 13) − 25( x − 13) = 0 A C
_ (d) C ( x − 13) ( x − 25) = 0
x = 25 [Q x ≠ 13]
12.5cm The angle made by an arc at centre is
The sides of the triangles are in twice the angle made by same arc on
ratio 25 : 24 : 7 . the circumference.
∴ Perimeter of the triangle ∠BPD = 2 × ∠BAD
8cm
= 25 + 24 + 7= 56 units. (angles are made on arc BD)
82. ABC is a triangle right angled at = 2 × 60º = 120º
A B
B. Let M and N be two points on 84. An equilateral triangle ABC and a
AD = BD × CD
2
AB such that AM = MN = NB. Let scalene triangle DBC are inscribed in
AD = 8 × 12 .5 = 100
2
P and Q be two points on AC a circle on same side of the arc. What
⇒ AD = 10 cm such that PM is parallel to QN is ∠BDC equal to?
80. The surface area of a cube is and QN is parallel to CB. If (a) 30º (b) 45º (c) 60º (d) 90º
equal to that of a sphere. If x is BC = 12 cm, then what is
the volume of the cube and y is ( PM + QN ) equal to? _ (c)
the volume of the sphere, then (a) 10 cm (b) 11 cm A
what is x 2 : y 2 equal to? (c) 12 cm (d) 13 cm D
(a) π : 6 (b) 6 : π _ (c) Given that, AM = MN = NB and
(c) π : 3 (d) 3 : π PM || QN || CB.
A
_ (a) Let the side of cube be s and B C
radius of sphere be r.
Given that, the surface area of a M P
cube is equal to that of sphere. ∠BAC = 60º
⇒ 6s = 4 πr
2 2
Q (Qproperty of equilateral triangle)
N
s 2
2 Angles made by an arc on the
= π circumference of a circle are equal.
r2 3 B 12 cm C ∠BDC = ∠BAC
Volume of cube ( x ) = s 3
In ∆AMP and ∆ABC, [angle made by arc BC]
4
Volume of sphere ( y) = πr 3 ∠AMP = ∠ABC = 60º
3
(corresponding angles)
According to question, 85. The sides of a triangle ABC are
3 ∠MAP = ∠BAC (common)
x2
(s ) 9  s2 
3 2 4 cm, 6 cm and 8 cm. With the
= =   ∆AMP ~ ∆ABC ( AA )
2 2
16 π 2  r 2  vertices of the triangle as centres,
y  4 π r3
  AM
=
MP
three circles are drawn each touching
3  AB BC
the other two externally. What is the
9 8π3 AM MP
= × = sum of the radii of the three circles?
16 π 2
27 3 AM 12
(a) 6 cm (b) 7 cm (c) 9 cm (d) 10 cm
= π:6 ⇒ MP = 4 cm
56 CDS Solved Paper 2021 (I)

_ (c) 87. If the perimeter of a right-angled 89. ABCD is a trapezium in which AB


A triangle is 30 cm and the is parallel to DC and 2AB = 3DC .
Q hypotenuse is 13 cm, then what is The diagonals AC and BD intersect
P
the area of the triangle? at O. What is the ratio of the area of
C (a) 24 cm2 (b) 27 cm2 ∆AOB to that of ∆DOC?
R B
(c) 30 cm2 (d) 36 cm2 (a) 2 : 1 (b) 3 : 2
(c) 4 : 1 (d) 9 : 4
Let the sides AB, BC and AC of _ (c) A
triangles are 4 cm, 8 cm and 6 cm
_ (d) A B

respectively and the intersecting


point of the circles are P, Q and R. O
AP = AQ, CQ = CR and BP = BR
[Qradius of circles] D C
B C
Let AP = AQ = x cm
Let ABC be a right triangle, right Given, ABCD is a trapezium and
PB = ( 4 − x ) cm angled at B. AB|| DC .
⇒ BR = 4 − x cm Let AB = h cm and BC = b cm ∠AOB = ∠DOC
CR = BC − BR AC = 13 cm (vertically opposite angles)
= 8 − (4 − x ) b + h + 13 = 30 ∠ABO = ∠BDC (alternate angles)
= ( 4 + x ) cm b + h = 17 ∆AOB ~ ∆COD ( AA )
QC = 4 + x cm ar (∆AOB)  AB 
2 2
Squaring on both sides, we get
=  3
AC = 6 cm  = 
b 2 + h2 + 2 bh = 289 ar ( ∆COD )  CD  2
AQ + QC = 6 cm 2 bh = 289 − 169 9
= = 9:4
x + 4+ x =6 [Q b 2 + h2 = AC 2 ] 4
2x = 2 2 bh = 120
90. A circle touches all the four sides of
x = 1 cm bh = 60 a quadrilateral ABCD.
AP = AQ = 1 cm 1
Area ( ABC ) = × b × h If AB = 9 cm, BC = 8 cm and
BP = BR = 4 − 1 = 3 cm 2
CD = 12 cm, then what is DA equal
CQ = CR = 4 + 1 = 5 cm 1
= × 60 = 30 cm 2 to?
∴Sum of the radii of three circles 2
(a) 14 cm (b) 13 cm
= 1 + 3 + 5 = 9 cm 88. ABC is a triangle right-angled at C. (c) 12 cm (d) 11 cm
86. Let PAB be a secant to a circle Let p be the length of the
perpendicular drawn from C on _ (b)
intersecting the circle at A and B.
Let PT be the tangent segment. If AB. If BC = 6 cm and CA = 8 cm, 12 cm
D R C
then what is the value of p?
PA = 9 cm and PT = 12 cm, then
(a) 5.4 cm (b) 5 cm
what is AB equal to? (c) 4.8 cm (d) 4.2 cm
(a) 5 cm (b) 6 cm Q 8 cm
_ (c) A S
(c) 7 cm (d) 9 cm

_ (c)
T 8 cm D A P B
9 cm

As, the tangents from an external


P points are equal in length.
A B
C 6 cm B AP = AS
Using the formula, PB = BQ
Let AB = x cm 1 1 1
⇒ = + CQ = CR
then PB = 9 + x cm CD 2 AC 2 BC 2 RD = DS
PT 2 = PA × PB 1 1 1
⇒ = + 2 Let AS = x cm
(12 )2 = 9 × ( 9 + x ) P 2 82 6
AP = x cm
9+ x =
144 1 1 1 1 64 + 36
⇒ 2 = + ⇒ 2 = BP = AB − AP
9 P 64 36 P 64 × 36
9 + x = 16 cm = ( 9 − x ) cm
64 × 36 8× 6
⇒ P2 = ⇒P = = 4.8 cm BQ = BP = ( 9 − x ) cm
x = 7 cm 100 10
CDS Solved Paper 2021 (I) 57

CQ = BC − BQ For country E,
_ (d) The first 10 composite numbers are
= 8 − (9 − x ) Production increment 4, 6, 8, 9, 10, 12, 14, 15, 16, 18
= ( x − 1) cm 60.9 − 58 4 + 6 + 8 + 9 + 10 + 12
= × 100
CR = CQ = x − 1 cm 58 + 14 + 15 + 16 + 18
Mean =
290 10
DR = DC − CR = = 5%
58 112
= 12 − ( x − 1) = = 11.2
Hence, for countries A, C , D and E, 10
= (13 − x ) cm
production increment per cent has
DS = DR = (13 − x ) cm been more than or equal to 5%. 95. The marks obtained by 5 students are
AD = AS + SD 21, 27, 19, 26, 32. Later on 5 grace
92. The following table shows the marks are added to each students.
= x + 13 − x = 13 cm
marks of 90 students in a test of What are the average marks of the
91. Consider the following data 80 marks : revised marks of the students?
with regard to production of cars Marks Number of students (a) 26 (b) 30 (c) 31 (d) 32
(in lakhs):
1-10 5 _ (b) Average marks of the student
Year 2015 Year 2016 21 + 27 + 19 + 26 + 32 125
11-20 8 = = = 25
Country A 35 38 5 5
21-30 10
Country B 45 47 After adding 5 grace marks to each
31-40 13 student, the revised average marks
Country C 88 93
41-50 18 = 25 + 5 = 30
Country D 75 79
51-60 17 96. Let p be the mean of m observations
Country E 58 60.9
61-70 12 and q be the mean of n observations,
In which of the countries, the 71-80 7 where p ≤ q . If the combined mean of
production of cars has increased (m + n ) observations is c, then which
by more than or equal to 5% in The percentage of students who one of the following is correct?
2016 over 2015? have obtained less than or equal (a) c ≤ p (b) c ≥ q
to 50% marks is (c) p ≤ c ≤ q (d) q ≤ c ≤ p
(a) B and E
(a) 30% (b) 40% (c) 45% (d) 60%
(b) A, C and D only _ (c) Given am = p and an = q
(c) A, C, D and E _ (b) Given that, total number of Sum of m observations = mp and
students = 90
(d) A, D and E only sum of n observations = nq
Number of students who scored
_ (c) For country A, less than or equals to 50% marks Average of ( m + n) observation
Production increment i.e., less than or equal to 40 marks. mp + nq
=
38 − 35 = 5 + 8 + 10 + 13 = 36 m+ n
= × 100
35
∴ Required percentage =
36
× 100 mp + nq
c=
=
300
= 8.5%
90 m+ n
35 = 40% Case 1
For country B, mp + np − np + nq
93. What is the median of the c=
Production Increment
following data? m+ n
47 − 45 
=   × 100 n(q − p)
 45  2, 3, − 1, 2, 6, 8, 9 c = p+
m+ n
200 (a) 2 (b) 3 (c) 4 (d) 5
= = 4.44% c = p + k1
45 _ (b) The data in the ascending order p≤ c [Q k1 ≥ 0] …(i)
For country C, = − 1, 2, 2, 3, 6, 8, 9 Case 2
Production Increment There are 7 terms, so mp + nq
c=
93 − 88  m+ n
=   × 100 Median = 
n + 1
 th term
 88   2  mp − mq + mq + nq
c=
500 7 + 1 m+ n
= = 5.68% =   th term
88  2  m( p − q )
c= +q
For country D, = 4 th term = 3 m+ n
Production increment [Q p ≤ q ⇒ p – q ≤ 0]
79 − 75 94. What is the arithmetic mean of c = k2 + q [Q k 2 ≥ 0]
= × 100
75 the first ten composite numbers? q≥c …(ii)
400 (a) 8.5 (b) 9.5
= = 5.33% From Eqs. (i) and (ii), we get
75 (c) 10.2 (d) 11.2
p≤ c ≤ q
58 CDS Solved Paper 2021 (I)

Direction (Q. Nos. 97-100) _ (a) The total number of tablets


The total number of tablets produced in
produced in the year 2000 is the year 2004 is
Consider the following data with 180 + 110 + 85 + 95 + 115 = 585
regard to different types (I, II, III, 160 + 80 + 70 + 90 + 75 = 475
The percentage drop in total production
IV, V) of multivitamin tablets The total number of tablets in 2004 compared to
produced in the year 2001 is
695 − 585 
2001 = 
produced in a company (in lakhs).
200 + 150 + 85 + 160 + 100 = 695  × 100
 695 
Year I II III IV V
The total number of tablets
110 11000
2000 160 80 70 90 75 produced in the year 2002 is = × 100 = %
695 695
2001 200 150 85 160 100 135 + 35 + 44 + 95 + 85 = 394
The percentage drop in total production
2002 135 35 44 95 85 The total number of tablets in 2000 compared to
produced in the year 2003 is
695 − 475 
2003 240 95 120 80 120
240 + 95 + 120 + 80 + 120 = 655 2001 =   × 100
 695 
2004 180 110 85 95 115 The total number of tablets
220
2005 210 150 100 92 110 produced in the year 2004 is = × 100
695
180 + 110 + 85 + 95 + 115 = 585
22000
97. Which product is produced least The total number of tablets = %
695
over the years 2000-2005? produced in the year 2005 is
Hence, the required ratio
(a) Type II (b) Type III 210 + 150 + 100 + 92 + 110 = 662
11000 22000 1
(c) Type IV (d) Type V The difference in total number of = %÷ %=
695 695 2
tablets produced between 2003
_ (b) Total production of each tablets and 2005 is 100. In which year, the production of
over the years 2000 − 2005.
662 − 655 = 7 Type I is more than the sum of the
Type I
The difference in total number of production of Type III and Type IV?
160 + 200 + 135 + 240 + 180 + 210 tablets produced between 2001
(a) 2001 (b) 2002
= 1125 and 2005 is
(c) 2003 (d) 2004
Type II 695 − 662 = 33
80 + 150 + 35 + 95 + 110 + 150 The difference in total number of _ (c) The number of tablets of Type I
= 620 tablets produced between 2003 produced in the year 2001 = 200
Type III and 2004 is The total number of tablets of Type III
70 + 85 + 44 + 120 + 85 + 100 655 − 585 = 70 and IV together produced in the year
2001 = 85 + 160 = 245
= 504 The difference in total number of
tablets produced between 2000 The number of tablets of Type I
Type IV
and 2002 is produced in the year 2002 = 135
90 + 160 + 95 + 80 + 95 + 92
475 − 394 = 81 The total number of tablets of Type III
= 612
and IV together produced in the year
Type V Hence, the difference in total 2002 = 44 + 95 = 139
number of tablets produced is
75 + 100 + 85 + 120 + 115 + 110 minimum between 2003 and 2005. The number of tablets of Type I
= 605 produced in the year 2003 = 240
Hence, the required product is 99. The ratio of percentage drop in The total number of tablets of Type III
type III. total production in 2004 and IV together produced in the year
compared to 2001 to that in 2000 2003 = 120 + 80 = 200
98. In which one of the following compared to 2001, is The number of tablets of Type I
pairs of years, the difference in 1 1 1 1 produced in the year 2004 = 180
total number of tablets produced (a) (b) (c) (d)
3 4 2 5 The total number of tablets of Type III
between them is minimum? and IV together produced in the year
(a) (2003, 2005) _ (c) The total number of tablets 2004 = 100 + 92 = 192
produced in the year 2000 is
Hence, for the year 2003, the
(b) (2001, 2005) 160 + 80 + 70 + 90 + 75 = 475
production of Type I is more than the
(c) (2003, 2004) The total number of tablets total production of tablet of the type III
(d) (2000, 2002) produced in the year 2001 is and Type IV together.
200 + 150 + 85 + 160 + 100 = 695
Paper I English

Directions (Q. Nos.1-10) Each question in this section has a 6. Language is an essential modes of communication
sentence with three underlined parts labelled as (a), (b) and a. b.
(c). Read each sentence to find out whether there is any error in and every language matters in communication.
any underlined part and indicate your response against the
c.
corresponding letter i.e., (a) or (b) or (c). If you find no error,
No error
your response should be indicated as (d).
d.
1. He had been doing the job of a physician
a. b. _ (b) The phrase ‘an essential modes of communication’
contains the grammatical error. To make the sentence
during the1990s. No error grammatically correct and contextually meaningful, replace
c. d. ‘modes’ with ‘mode’.

_ (c) The phrase ‘during the 1990s’ contains the grammatical error. 7. The matter was been resolved after long deliberations
The given sentence is in past perfect continuous tense which a. b.
states that the action started in the past continued up till another
time in the past. Hence, replace ‘during’ with ‘since’. among the members of the team. No error
c. d.
2. He said that he preferred coffee than tea. No error
a. b. c. d. _ (a) The phrase ‘the matter was been resolved’ contains the
grammatical error. To make the sentence grammatically
_ (c) The phrase ‘coffee than tea’ contains the grammatical error. To correct and contextually meaningful, remove ‘been’.
make the sentence grammatically correct and contextually
meaningful, replace ‘than’ with ‘to’. 8. Care has been taken to ensure safety and security
3. All that glitters are not gold. No error a. b.
a. b. c. d. of the participants in the event. No error
c. d.
_ (c) The phrase ‘are not gold’ contains the grammatical error. To
make the sentence grammatically correct and contextually _ (d) The given sentence is grammatically correct and
meaningful, replace ‘are’ with ‘is’. contextually meaningful.
4. A species is considered endangered when it is 9. Indian film industry is considered to be
a. b. a. b.
in a very high risk of extinction in the world. No error one of the symbol of cultural and social life. No error
c. d.
_ (c) The phrase ‘in a very high risk of extinction in the world’ contains c. d.
the grammatical error. To make the sentence grammatically correct
and contextually meaningful, replace ‘in’ with ‘at’.
_ (c) The phrase ‘one of the symbol of cultural and social life’
contains the grammatical error. To make the sentence
5. Our greatest glory is not in never falling grammatically correct and contextually meaningful, replace
‘symbol’ with ‘symbols’.
a. b.
but in rising every time we fell. No error 10. It is still true that the Indian economy
c. d. a. b.
is dependent on monsoon. No error
_ (c) The phrase ‘but in rising every time we fell’ contains the
grammatical error. The given sentence is in present tense. Hence, c. d.
to make the sentence grammatically correct and contextually
meaningful, replace ‘fell’ with ‘fall’.
_ (d) The given sentence is grammatically correct and
contextually meaningful.

Directions (Q. Nos. 11-20) Given _ (a) The idiom ‘be in eclipse’ means 13. Up in arms
below are some idioms/phrases ‘much less successful and important (a) Very happy
than before’. (b) Very satisfied
followed by four alternative meanings
to each. Choose the response (a), (b), 12. Ways and means (c) Very angry
(c) or (d) which is the most (a) A technique (d) Feeling fine
appropriate meaning and mark your (b) Methods of achieving something _ (c) The idiom ‘up in arms’ means
response accordingly. (c) Norms and regulations of doing ‘angry and rebellious’.
something
11. Be in eclipse 14. Big ticket
(d) Improving one’s way of doing
(a) Less successful (a) Very less (b) Very costly
(b) Feeling happy _ (b) The phrase ‘ways and means’ (c) Very easy (d) Not much
means ‘methods and resources for
(c) Very successful accomplishing something’. _ (b) The idiom ‘big ticket’ means
(d) Being defeated ‘costing a great deal and expensive’.
60 CDS Solved Paper 2021 (I)

15. Bolt from the blue Directions (Q. Nos. 21-30) In this (a) It does not encourage
section you have two short passages. disagreement
(a) An event or piece of news which
is unexpected After each passage, you will find some (b) It does not accept conformity
(b) Desirable event or news questions based on the passage. Read (c) It is not fact based, but problem
based
(c) An event which takes place as the passages and answer the
planned (d) It is only religious education.
questions based on them. You are
(d) News which has been long required to select your answers based _ (a) According to the given passage,
expected, but arrives late conventional education supports
on the content of the passage and conformity. It promotes success
_ (a) The idiom ‘bolt from the blue’ opinion of the author only. which comes with conformity and a
means ‘something important or lack of conflict. Thus, to avoid conflict
unusual that happens suddenly or PASSAGE-I it discourages any disagreements and
unexpectedly’. makes independent thinking
Conventional education makes
16. Be a law unto yourself independent thinking extremely extremely difficult.
(a) Behave in an unconventional and difficult. Conformity leads to 22. Worshipping success leads to
unpredictable way mediocrity. To be different from the (a) questioning the established
(b) Abide by law and order group or to resist environment is not practices
(c) Ask others to follow the law easy and is often risky as long as we (b) accepting the established
(d) Create law and order for others worship success. The urge to be practices
_ (a) The idiom ‘be a law unto yourself’ successful, which is the pursuit of (c) modifying the established
means ‘behaving in an independent reward whether in the materials or practices
way, ignoring laws, rules, or in the so-called spiritual sphere, the (d) neither accepting nor rejecting
conventional ways of doing things’. search for inward or outward the established practices
17. Spiff up security, the desire for comfort-this _ (b) In the given passage, it is
whole process smothers discontent, suggested that those who worship
(a) to make oneself look neat
puts an end to spontaneity and success desire to live a peaceful life in
(b) To make oneself look untidy comfort. Hence, such people try to
(c) To make oneself look arrogant breeds fear; and fear blocks the
avoid conflict and accept the
(d) o appear on the stage as a intelligent understanding of life. established practices.
baboon With increasing age, dullness of
mind and heart sets in. 23. Why do most people want to find
_ (a) The idiom ‘spiff up’ means ‘to a quiet corner in life?
make someone, something, or In seeking comfort, we generally
oneself appear more visually (a) Because they do not like to be in
find a quiet corner in life where peace
attractive, appealing, or fashionable’.
there is a minimum of conflict, and (b) Because they seek pleasure
18. Run wild then we are afraid to step out of (c) Because they seek comfort
(a) To run like a wild animal that seclusion. This fear of life, this (d) Because they do not worship
(b) o treat anyone like a wild creature fear of struggle and of a new success
(c) To feel like a wild animal experience, kills in us the spirit of
_ (c) As per the passage, people seek
(d) To grow without any control adventure; our whole upbringing comfort which they can find in a quiet
and education have made us afraid corner of life.
_ (d) The phrase ‘run wild’ means ‘grow
or develop without restraint or to be different from our neighbour,
24. Which word in the passage means
discipline’. afraid to think contrary to the
‘bias’?
established pattern of society,
19. Wind down (a) contrary
falsely respectful of authority and
(a) To relax after a period of activity (b) prejudice
tradition.
(b) To act furiously after a period of (c) uncomprehendingly
silence Fortunately, there are a few who are (d) discontent
(c) To speak out the truth to people in earnest, who are willing to
_ (b) The word ‘bias’ means ‘inclination
(d) To act on the ground examine our human problems or prejudice for or against one person
without the prejudice of the right or or group, especially in a way
_ (a) The idiom ‘wind down’ means ‘to of the left; but in the vast majority considered to be unfair’. From the
relax after stress or excitement.’
of us, there is no real spirit of given options, ‘prejudice’ means the
20. Mellow out discontent, or revolt. When we same as bias.
(a) To feel bad about other’s yield uncomprehendingly to 25. The word ‘uncomprehendingly’ in
enjoyment environment, any spirit of revolt the passage means
(b) To like and dislike people
concurrently
that may have had died sown, and (a) Achieve success
our responsibilities soon put an end (b) Earning money
(c) To enjoy oneself without doing
much to it. (c) Without knowing
(d) With complete knowledge
(d) To work hard and do much work 21. Why does conventional education
_ (c) The idiom ‘mellow out’ means ‘to make independent thinking _ (c) The meaning of the word
‘uncomprehendingly’ is ‘without any
relax and to calm down’. extremely difficult? reason or knowledge.’
CDS Solved Paper 2021 (I) 61

PASSAGE-II the opportunity to use trees is 28. ‘Single layer of trees is worthless
Noise is a common problem in both restricted a combination of shrubs as a noise barrier.’ Why?
urban and rural areas, traffic noise and tall grass can be use(d) Instead (a) Noise can’t penetrate easily
in particular. Trees and shrubs are of tarmac or gravel surfaces soft (b) Help in the growth of shrubs
not only visually attractive but also grass ground cover is preferable. It (c) Noise is sent back to the source
can make effective noise barriers. will absorb noise rather than reflect (d) Noise can penetrate easily
Therefore, when space permits we it. Rows of trees, shrubs, crotons,
_ (d) As per the given passage, trees
have to plant more trees and shrubs ground covering plants, climbers must be dense to reduce noise. Even
in areas where there is noise and their flowers not only reduce a small gap will allow the noise to
problem. By doing this noise can be noise and provide a beautiful pass easily.
reduced by six decibels over a backyard but they also help block 29. Higher the trees
distance of 30 m. wind, provide shade, and absorb
(a) higher the noise
The only thing we have to keep in traffic pollution. (b) lower the noise
mind is that the planting should be 26. What distance a good number of (c) greater the decibels
particularly dense; the trees which trees can cover in reducing noise? (d) thinner the air
have dense foliage and relatively (a) 30 m (b) 60 m (c) 3 m (d) 300 m _ (b) As mentioned in the passage, the
vertical orientation of arrangements trees should be high to reduce noise
should be plante(d) It will _ (a) As mentioned in the passage, a effectively.
good number of trees can reduce
completely screen the source of noise in an area of 30 m. 30. How do shrubs and tall grass
noise. If there is any gap in barrier reduce noise?
it will still allow noise to penetrate. 27. Trees can reduce noise only when
(a) planted in isolation (a) They absorb noise
Single layer of trees is worthless as
(b) planted with flowering shrubs (b) They reflect noise
a noise barrier. Due to
(c) they are planted dense (c) They divert noise
inter-reflection, multi-rows of trees (d) they are neutral to noise
are more effective. The planted trees (d) planted with shorter ones
must be taller. We can reduce noise _ (c) According to the passage, trees _ (a) According to the passage, trees,
can be effective in reducing noise shrubs and tall grass reduce noise
level by 1.5 decibels with each pollution by absorbing noise.
pollution only if they are planted
additional 3 feet of tree height.When dense.

Directions (Q. Nos. 31-40) Each of the following 33. Enough potassium, banana provides
questions in this section consists of a sentence, parts of P Q
which have been jumbled. These parts have been labelled from various sugars and sustained energy
as P, Q, R and S. Given below each sentence are four R S
sequences namely (a), (b), (c) and (d). You are required to (a) P Q R S (b) R S Q P
re-arrange the jumbled parts of the sentence and mark you (c) S Q R P (d) Q P S R
response accordingly.
_ (d) The correct sequence that would form a logical and
31. Consideration for other for self-gratification contextually meaningful sentence is QPSR.
P Q 34. of great trouble this is a time
freedom does not mean the opportunity
P Q
R the world to the maximum as one virus threatens
or the setting aside of
R S
S (a) P Q S R (b) R S P Q
(a) S P Q R (b) Q S P R (c) S R Q P (d) Q P S R
(c) R P Q S (d) R Q S P
_ (d) The correct sequence that would form a logical and
_ (d) The correct sequence that would form a logical and contextually meaningful sentence is QPSR.
contextually meaningful sentence is RQSP.
35. are of the opinion some archaeologists
32. of any kind to freedom and intelligence P Q
P Q and that everybody enjoyed equal status
domination or compulsion is a direct hindrance R
that Harappan society had no rulers,
R S
S
(a) R P S Q (b) R Q P S (a) S P Q R (b) Q S R P
(c) S P Q R (d) Q R S P (c) Q P S R (d) R P S Q
_ (a) The correct sequence that would form a logical and _ (c) The correct sequence that would form a logical and
contextually meaningful sentence is RPSQ.
contextually meaningful sentence is QPSR.
62 CDS Solved Paper 2021 (I)

36. in the same locality marriage within a unit (a) Q S P R (b) Q R S P


P Q (c) S Q R P (d) S P Q R
endogamy refers to or a group living _ (c) The correct sequence that would form a logical and
R S contextually meaningful sentence is SQRP.

(a) Q R S P (b) R S P Q 39. brought on by destructive pests humans have suffered


(c) Q P S R (d) R Q S P P Q
_ (d) The correct sequence that would form a logical and frustration and food losses since earliest times,
contextually meaningful sentence is RQSP. R S
37. when steam engines dominated industrialisation (a) S P Q R (b) Q R S P
(c) S Q R P (d) R S Q P
P Q
coal was the main source of energy in the initial stages, _ (c) The correct sequence that would form a logical and
contextually meaningful sentence is SQRP.
R S
(a) S P R Q (b) R S P Q 40. most spectacular gold coins some of the
(c) P Q R S (d) Q R S P P Q
the Gupta rulers in India were issued by
_ (b) The correct sequence that would form a logical and
contextually meaningful sentence is RSPQ. R S
(a) R S P Q (b) Q P S R
38. from the scientists ? is the information
(c) P Q R S (d) S P Q R
P Q
that comes how important _ (b) The correct sequence that would form a logical and
contextually meaningful sentence is QPSR.
R S

Directions (Q. Nos. 41-50 ) In this P : On the other hand in


S : It takes different forms.
section, each questions consists of six North-eastern states, winters are
mild except in the hills. The correct sequence should be
sentences of a passage. The first and
Q : Because in North India, both the (a) S R Q P (b) S Q P R
sixth sentences are given in the (c) R S P Q (d) P Q R S
beginning as SI and S6. The middle sessions have extreme weather
four sentences in each have been conditions. _ (a) The correct sequence that would
form a logical and contextually
jumbled up and labelled as P, Q, R R : School clothes during the meaningful paragraph is SRQP.
and S. You are required to find the summer are different from those
proper sequence of the four sentences during the winter. 44. S1 : The dhoti kurta is a traditional
and mark your response accordingly. S : Why do you wear lighter clothes lungi-type garment worn by
during summers and heavy men in India.
41. S1 : Man has progressed with woollen clothes during the S6 : Gandhi ji usually wore a simple
science and technology. winter in North India? shawl with his dhoti.
S6 : What was difficult and The correct sequence should be P : It is commonly worn with a
impossible is now easy and (a) S R P Q (b) R S Q P loose long tunic.
possible. (c) P Q R S (d) Q P R S Q : This, he started wearing after he
P : He is on the road to conquer abandoned his western attire of
other planets. _ (b) The correct sequence that would
form a logical and contextually pants and shirts.
Q : Now with scientific knowledge meaningful paragraph is RSQP. R : A dhoti, made of homespun
and technical know-how he has khadi cotton cloth, was the
done what was impossible.
43. S1 : Policy, like many other terms, is
a complex concept. garment favoured by Gandhi ji.
R : Hitherto he had been backward. S : The dhoti is rectangular piece of
S6 : Public policy is the outcome of the
S : He has reached the moon. struggle between government and unstitched cloth, around five
The correct sequence should be powerful groups. yards long, wrapped around the
(a) S R Q P (b) R P Q S P : It is form of government control midriff and tucked between the
(c) R Q S P (d) Q R S P usually expressed in a law, a legs.
_ (c) The correct sequence that would legislation, a regulation or an The correct sequence should be
form a logical and contextually order. (a) P S R Q
meaningful paragraph is RQSP. Q : Public policy is what a (b) R S P Q
(c) S P R Q
42. S1 : We drink more water during government chooses as
guidance for action. (d) Q R S P
summers.
S6 : There are variations in weather R : A policy may be general or _ (c) The correct sequence that would
form a logical and contextually
conditions during different specific, broad or narrow, simple
meaningful paragraph is SPRQ.
seasons. or complex, public or private,
written or unwritten.
CDS Solved Paper 2021 (I) 63

45. S1 : Languages in contact with each S6 : But values can be either P : Ayushman Bharat comprising
other cause change. inherent or conferred. twin missions is set to transform
S6 : Changes often occur first in P : For many philosophers, only the nation’s health system.
informal and casual language. humans are moral agents, being Q : This will also foster a linkage
P : The increasing use of English in capable of acting morally or across public and private
many parts of the world affects immorally. healthcare.
both local languages and Q : Environmental ethics asks about R : The policy aims at developing
English. the moral relationships between an integrated health
Q : Language contact has long been humans and the world around information system that serves
recognised as a major engine of us. the need of all stakeholder and
change. R : Ethics evaluated the improves efficiency,
relationship, rules, principles or transparency.
R : This gives rise to new, hybrid
language varieties. codes that required or forbid S : This will be carried out by
certain conduct. operationalising firstly 1.5 lakh
S : A historical example is that of
S : Value is a measure or the worth health and wellness centers
Danish and English which led to
of something. offering preventive and primary
a major shift in the vocabulary
The correct sequence should be care.
and grammar of English.
(a) R Q P S (b) P Q S R The correct sequence should be
The correct sequence should be
(c) Q S R P (d) R S P Q (a) Q S R P (b) Q R S P
(a) R S Q P (b) Q S P R
(c) R Q P S (d) S P R Q
(c) Q R S P (d) S Q R P _ (a) The correct sequence that would
form a logical and contextually _ (c) The correct sequence that would
_ (b) The correct sequence that would meaningful paragraph is RQPS. form a logical and contextually
form a logical and contextually
meaningful paragraph is RQPS.
meaningful paragraph is QSPR. 48. S1 : The Constitution of India,
introduced on January 26, 1950, 50. S1 : Ecology, history and geography
46. S1 : Urbanisation in India has
laid down certain basic determine the distribution of
become an important and
principles and values. organisms.
irreversible process.
S6 : The Constitution guaranteed all S6 : Others prefer to interpret
S6 : As per the Census of India 2011,
citizens equality before the law biogeographical history through
the urban population is
and equality of opportunity in vicariance (range-splitting)
increasing rapidly.
government employment. events.
P : The 2030 development agenda of
P : It was also to be a federation P : Some historical biogeographers
the United Nations has
with demarcation of spheres of are interested in finding
emphasised the role of
action between the Union centre-of-origins.
sustainable cities for making
cities inclusive, safe, resilient Government and the State Q : Most species distributions result
and sustainable. Governments. from a combination of all these
Q : India was to be a secular and factors.
Q : This shows that India is in the
midst of transition from a democratic republic with a R : But biogeographers tend to
predominantly rural to a parliamentary system based on specialise in ecological aspects
quasi-urban society. adult franchise. (ecological biography or
R : These include : freedom of historical aspects).
R : The process of urbanisation has
been characterised by a speech and expression, freedom S : Ecological biogeographers are
dramatic increase in the number to assemble peacefully etc. interested in the effects of
of large cities. S : It guaranteed all Indian citizens environmental factors.
S : It is an important determinant of certain Fundamental Rights. The correct sequence should be
national economic growth and The correct sequence should be (a) Q R S P (b) R Q P S
poverty reduction. (a) Q P S R (b) R S P Q (c) P Q R S (d) S P R Q
The correct sequence should be (c) P S Q R (d) S Q R P _ (a) The correct sequence that would
form a logical and contextually
(a) Q R S P (b) P S R Q _ (a) The correct sequence that would meaningful paragraph is QRSP.
(c) S R Q P (d) R Q S P form a logical and contextually
meaningful paragraph is QPSR. Directions (Q. Nos. 51-60) Each of
_ (c) The correct sequence that would
form a logical and contextually 49. S1 : The National Health Policy 2017 the following sentences in this section
meaningful paragraph is SRQP. envisages creation of a digital has a blank space and four words or
47. S1 : Ethics is a branch of philosophy health technology eco-system. group of words are given after the
concerned with the distinction S6 : Secondly, offering financial sentence. Select the most appropriate
between right and wrong protection up to 5 lakh per year word or group of words for the blank
(morals) and the ultimate worth per family for the deprived 10 space and indicate your response
of actions or things (values). crores plus households. accordingly.
64 CDS Solved Paper 2021 (I)

51. He said that he ……… when he lost 57. What is the name of the person _ (c) The underlined word ‘absurd’
all his investment in the stock ……… daughter married your means ‘utterly or obviously senseless,
illogical, or untrue and contrary to all
market. cousin last year? reason or common sense’. From the
(a) was broken (a) whose (b) who given options, ‘reasonable’ is its
(b) was broke (c) whom (d) with correct antonym.
(c) brake
(d) had been collapse
_ (a) Option (a) ‘whose’ makes the 63. Relatives of the deceased have
sentence grammatically correct and been informed about the accident.
_ (b) Option (b) ‘was broke’ makes the contextually meaningful.
sentence grammatically correct and (a) injured (b) alive
contextually meaningful.
58. The world is changing ……… that (c) dead (d) survived
the rich become richer and the
52. The man in black thought that he _ (b) The underlined word ‘deceased’
poor become poorer. means ‘dead’. From the given
was ……… selected for the team to (a) through options, ‘alive’ is its correct antonym.
play at the national level. (b) in many way 64. At last she was able to get some
(a) one of the player (c) in a way
solace as the matter has been
(b) one of the players (d) in such a way
(c) is one player resolved amicably.
(d) the player of the team
_ (d) Option (d) ‘in such a way’ makes (a) comfort
the sentence grammatically correct
and contextually meaningful. (b) relief
_ (b) Option (b) ‘one of the players’ (c) punishment
makes the sentence grammatically
correct and contextually meaningful.
59. Had I been informed beforehand, I (d) aggravation
……… it to the celebrations.
53. When she ……… her friend after a _ (d) The underlined word ‘solace’
(a) could have made means ‘to give comfort to in grief or
period of two decades, her throat (b) will have made misfortune’. From the given options,
choked. (c) would have ‘aggravation’ meaning ‘to make
(a) meets (d) ought to have made something worse’ is its correct
antonym.
(b) had been meeting _ (a) Option (a) ‘could have made’
(c) met makes the sentence grammatically 65. Twenty first century has turned
(d) has met correct and contextually meaningful. out to be a century of problems
_ (c) Option (c) ‘met’ makes the 60. Indian perspective of life informs contrary to the thinking that it
sentence grammatically correct and would be a better time.
contextually meaningful.
that leading a life of greed ……… in
unhappiness. (a) similar (b) different
54. The portrait of the lady conveys (a) will lead (b) shall be (c) divergent (d) good faith
……… it appears. (c) will result (d) would resulted _ (a) The underlined word ‘contrary’
(a) the best means ‘different or opposite’. From
(b) more than
_ (c) Option (c) ‘will result’ makes the the given options, ‘similar’ is its
sentence grammatically correct and
(c) the most correct antonym.
contextually meaningful.
(d) nothing 66. The poet said that poetry is a
Directions (Q. Nos. 61-70) Each
_ (b) Option (b) ‘more than’ makes the question in this section consists of spontaneous overflow of powerful
sentence grammatically correct and feelings.
contextually meaningful. sentences with an underlined word
followed by four words or group of (a) prepared (b) alerted
55. A biography is ……… person’s life words. Select the option that is (c) deliberate (d) well executed
history. opposite in meaning to the underlined _ (a) The underlined word
(a) about a word and mark your response ‘spontaneous’ means ‘performed or
(b) about accordingly. occurring as a result of a sudden
(c) for a impulse or inclination and without
(d) with a 61. His arguments are not valid premeditation or external stimulus.’
People consider it bombastic. From the given options, ‘prepared’ is
_ (a) Option (a) ‘about a’ makes the its correct antonym.
sentence grammatically correct and (a) outdated (b) straight forward
contextually meaningful. (c) verbose (d) not true 67. Language is an instrument for
56. She told me that she ……… her _ (b) The underlined word ‘bombastic’ asserting one’s identity, attitude
means ‘high-sounding but with little and perspective.
degree by 2023. meaning.’ From the given options,
(a) declaring (b) supporting
(a) will have completed ‘straight forward’ is its correct
antonym. (c) denying (d) propagating
(b) shall be completed
(c) will completed
62. The decision was absurd for many _ (c) The underlined word ‘asserting’
(d) would have completed means ‘to state with assurance,
of the members of the team. confidence, or force’. From the given
_ (a) Option (a) ‘will have completed’ (a) bizarre (b) meaningless options, ‘denying’ is its correct
makes the sentence grammatically antonym.
correct and contextually meaningful. (c) reasonable (d) thoughtful
CDS Solved Paper 2021 (I) 65

68. He has been exonerated as he 69. Persuasion is essential for people 70. Every habitat has some distinctive
tendered an apology. to work as a team. vegetation which defines the
(a) honoured (b) pardoned (a) Dislike (b) Discouraging ecosystem.
(c) convicted (d) felicitated (c) Convincing (d) Deter (a) unique
_ (c) The underlined word ‘exonerated’ _ (b) The underlined word ‘persuasion’ (b) common
means ‘absolve (someone) from means ‘the action or process of (c) special
blame for a fault or wrongdoing’. convincing someone or of being (d) unfamiliar
From the given options, ‘convicted’ convinced to do or believe
meaning ‘the act or process of finding something’. From the given options, _ (b) The underlined word ‘distinctive’
means ‘different’. From the given
a person guilty of a crime especially in ‘discouraging’ is its correct antonym. options, ‘common’ is its correct
a court of law’ is its correct antonym. antonym.

Directions (Q. Nos. 71-80) Each of the following sentences in this section has a blank space with four words or group
of words given below. Select whichever word or group of words you consider the most appropriate for the blank space
and indicate your response accordingly.
It sounds like ……… 71. (a) a great idea. People get instantly excited
(b) an
(c) the
(d) some
_ (a) Option (a) ‘a’ makes the sentence grammatically correct and contextually meaningful.
when they hear ……… 72. (a) a phrase ‘education for peace’ or the title
(b) the
(c) that
(d) some
_ (b) Option (b) ‘the’ makes the sentence grammatically correct and contextually meaningful.
of a course that calls itself ‘Peace Education’. About a decade ago, I proposed such
such ……… 73. (a) any course in my institute. The idea of a course
(b) the
(c) some
(d) a
_ (d) Option (d) ‘a’ makes the sentence grammatically correct and contextually meaningful.
……… 74. (a) to ‘peace education’ had occurred to me on the way back from
(b) in
(c) for
(d) on
_ (b) Option (b) ‘in’ makes the sentence grammatically correct and contextually meaningful.
one of the neighboring countries where I met students of Class IX. I could not find
………… 75. (a) any difference ………… 76. (a) with Indian students
(b) the (b) between
(c) that (c) among
(d) this (d) to
_ 75. (a) Option (a) ‘any’ makes the sentence grammatically correct and contextually meaningful.
_ 76. (b) Option (b) ‘between’ makes the sentence grammatically correct and contextually meaningful.
and their counterparts in the other country on peace ………… 77. (a) with
(b) and
(c) because
(d) but
_ (b) Option (b) ‘and’ makes the sentence grammatically correct and contextually meaningful.
66 CDS Solved Paper 2021 (I)

living together. It made me think that if one learns to live with a conflict, life grows across it like grass
……… 78. (a) or trees. The arguments and stances
(b) with
(c) for
(d) some
_ (b) Option (b) ‘with’ makes the sentence grammatically correct and contextually meaningful.
children took demonstrated ……… 79. (a) for peace comes from under-
(b) but
(c) that
(d) when
_ (c) Option (c) ‘that’ makes the sentence grammatically correct and contextually meaningful.
standing a conflict well, not necessarily ……… 80. (a) by solving it.
(b) with
(c) to
(d) for
_ (a) Option (a) ‘by’ makes the sentence grammatically correct and contextually meaningful.
Directions (Q. Nos. 81-90) Each 84. Some people think that their
question in this section consists of a strength is perpetual. 88. He appears to be very haughty, but
sentence with an underlined words (a) temporary (b) powerful he is a humble person.
followed by four words/group of (c) ever-lasting (d) all persuasive (a) tough (b) modest
words. Select the option that is _ (c) The underlined word ‘perpetual’ (c) arrogant (d) knowledgeable
nearest in meaning to the underlined means ‘continuing forever’. From the
word and mark your response given options, ‘ever-lasting’ is its _ (c) The underlined word ‘haughty’
means ‘arrogant’.
accordingly. correct synonym.
89. The newly appointed secretary is
81. Having got excited she opened up 85. One’s actions exemplify one’s
industrious.
the Pandora’s Box which led to lot of attitude and values.
(a) diligent
commotion. (a) devise
(b) knowledgeable
(b) sympathise
(a) uproar (b) peace (c) indolent
(c) asks for
(c) sound (d) furious (d) insincere
(d) demonstrate
_ (a) The underlined word ‘commotion’ _ (a) The underlined word ‘industrious’
means ‘a state of confused and noisy _ (d) The underlined word ‘exemplify’ means ‘diligent and hard-working’.
disturbance’. From the given options, means ‘to illustrate or clarify’. From
‘uproar’ meaning ‘a loud and the given options, ‘demonstrate’ is its 90. The indignant attitude of the
impassioned noise or disturbance’ is correct synonym. speaker made the groups unhappy.
its correct synonym. 86. The crux of the issue was that there (a) resentful (b) congenial
82. The inherent danger in the problem was no evidence to prove the (c) unruly (d) supportive
accused guilty of the act.
is that it would lead to many more _ (a) The underlined word ‘indignant’
problems. (a) core (b) part means ‘feeling or showing anger
(c) idea (d) tip because of something unjust or
(a) outward (b) difficult ‘resentful’.
(c) hallow (d) inbuilt _ (a) The underlined word ‘crux’ means
_ (d) The underlined word ‘inherent’
‘a main or central feature’. From the Directions (Q. Nos. 91-100) Each of
means ‘existing in something as a given options, ‘core’ is its correct the following sentences has a word or
synonym.
permanent, essential, or characteristic phrase underlined. Read the sentences
attribute’. From the given options, 87. Each child develops his/her carefully and find which part of
‘inbuilt’ is its correct synonym.
competency based on the contexts speech the underlined word is.
83. The officer was reprimanded by the and the inputs for learning. Indicate your response accordingly.
court for delaying the case. (a) capability 91. The preacher said, ‘No one is above
(a) admonished (b) appreciated (b) thinking the all powerful’.
(c) praised (d) disliked (c) knowledge (a) Adverb
_ (a) The underlined word (d) ideal (b) Place value
‘reprimanded’ means ‘a severe
_ (a) The underlined word ‘competency’ (c) Preposition
reproof or rebuke, especially a formal refer to ‘the ability to do something (d) Verb
one by a person in authority’. From successfully or efficiently’. From the
the given options, ‘admonished’ given options, ‘capability’ is its correct _ (a) As the given word ‘above’ tells us
means the same and hence, is its synonym. about position, it is an adverb of
correct synonym. position.
CDS Solved Paper 2021 (I) 67

92. The most beautiful seven monkeys 99. Ravi won the match with a huge 105. The company’s bid for the contract
of the landlord were taken for a margin. is rejected
stroll. (a) Intransitive verb (a) since it filled all the conditions
(a) Object (b) Adjective (b) Transitive verb (b) for the company fulfilled the
(c) Noun phrase (d) Noun (c) Demonstrative verb conditions
(d) Adjective (c) for the company had not been in
_ (b) As the underlined words describe the goodwill list
the noun ‘seven monkeys’, they are _ (b) The word ‘won’ is a transitive verb.
(d) since the deposit in the account
adjective. 100. There was an exultation in the is found less than required
93. Alas! He has been defeated. group
_ (c) Option (c) makes the sentence
(a) Adjective (b) Adverb (a) Adjective grammatically correct and
(c) Interjection (d) Conjunction (b) Adverb contextually meaningful.
(c) Adjectival Noun
_ (c) Interjections are words intended to (d) Noun 106. I could not sleep
express surprise or a spontaneous (a) because being very tired
feeling or reaction. They are often _ (d) The word ‘exultation’ is a noun. (b) despite being very tired
punctuated with an exclamation mark. (c) in spite of being sleepless
Hence, “Alas!” is an interjection. Directions (Q. Nos. 101-110)
Following questions have one part of (d) since being very tired
94. The man who is in the green shift is the sentence followed by four _ (b) Option (b) makes the sentence
the chief of the day. alternatives. Complete the sentences grammatically correct and
(a) Pronoun contextually meaningful.
by choosing the correct alternative.
(b) Relative pronoun 107. She married him
(c) Demonstrative pronouns 101. Sheela has been at the receiving end
(a) since his good nature
(d) Assertive noun (a) but she is very honest
(b) because his nature
(b) and she is very honest
_ (b) As the underlined word refers to (c) because of his nature
the noun and introduces a dependent (c) she had been proud of it
(d) in spite his nature
clause that connects it to an (d) she was fine with it
independent clause, it is a relative _ (c) Option (c) makes the sentence
pronoun.
_ (a) Option (a) makes the sentence grammatically correct and
grammatically correct and
contextually meaningful.
95. He has been living in the jungle for contextually meaningful.
long and he knows all the animals 108. I will come to meet you
102. Elam has no other option
here. (a) in case you want
(a) unless he wins (b) in case of time
(a) Subordinating conjunction (b) but to accept the challenge
(b) Coordinating conjunction (c) because you need
(c) though he has won challenge (d) although you want
(c) Preposition (d) he has to accept the challenge
(d) Personal pronoun _ (a) Option (a) makes the sentence
_ (b) Option (b) makes the sentence grammatically correct and
_ (b) ‘And’ is a coordinating grammatically correct and contextually meaningful.
conjunction. contextually meaningful.
96. They have been driving since early 109. If you had planned the work well
103. Although the group has been
morning. acquitted (a) you shall have completed it one
time
(a) Adjective (b) Adverb (a) they are under the watch of police (b) you could complete it on time
(c) Noun (d) Comparison (b) they are free to do anything (c) you will complete it on time
_ (b) As the word ‘since’ tells us about (c) there is no suspicion in them (d) you could have completed it on time
the time, it is an adverb of time. (d) they have not been arrested
_ (d) Option (d) makes the sentence
97. This time we woke up to the virus _ (a) Option (a) makes the sentence grammatically correct and
related diseases. grammatically correct and contextually meaningful.
contextually meaningful.
(a) Demonstrative 110. You can use my computer
(b) Article 104. Unless you fulfil the essential
(a) as long as you can’t use any
(c) Adverb qualifications external drive
(d) Noun phrase (a) you will be called for the interview (b) as long as you used any external
_ (a) The word ‘this’ is a demonstrative (b) you shall be selected for the post drive
word. (c) you will not be called for the (c) as long as you do not use any
interview external drive
98. Love your neighbour as thyself. (d) you can’t be rejected (d) as long as you shall use any
(a) Noun (b) Pronoun external drive
(c) Preposition (d) Adjective _ (c) Option (c) makes the sentence
grammatically correct and
_ (c) Option (c) makes the sentence
_ (b) The word ‘thyself’ is a reflexive contextually meaningful. grammatically correct and
pronoun. contextually meaningful.
68 CDS Solved Paper 2021 (I)

Directions (Q. Nos. 111-120) Each _ (b) Option (b) ‘between’ makes the 117. I will take care ……… myself. You
of the following sentences in this sentence grammatically correct and don’t bother about me.
contextually meaningful.
section has a blank space followed by (a) of (b) about(c) for (d) off
four options. Select whichever 114. The village possesses ……… scenic
_ (a) Option (a) ‘of’ makes the sentence
preposition or determiner you beauty. grammatically correct and
consider the most appropriate for the (a) some (b) a contextually meaningful.
blank space and indicate your (c) an (d) the
118. She is considered …………
response accordingly. _ (b) Option (b) ‘a’ makes the sentence nightingale of the college because of
grammatically correct and her melodious voice.
111. Don’t go ……… the wind, you will contextually meaningful.
face the music later. (a) a (b) an (c) some (d) the
(a) with (b) for 115. He is all out to blame everyone _ (d) Option (d) ‘the’ makes the
(c) about (d) between ……… his mistakes. sentence grammatically correct and
(a) for (b) in contextually meaningful.
_ (a) Option (a) ‘with’ makes the
sentence grammatically correct and (c) on (d) with 119. The room is a bit dark. Could you
contextually meaningful. _ (a) Option (a) ‘for’ makes the please turn ……… the light?
sentence grammatically correct and (a) on (b) in (c) off (d) away
112. He has been considered ………… contextually meaningful.
honest contestant in the
_ (a) Option (a) ‘on’ makes the
competition. 116. This is ……… unique system of sentence grammatically correct and
(a) an (b) a (c) the (d) one governance where everyone is the contextually meaningful.
government.
_ (a) Option (a) ‘an’ makes the 120. Don’t walk so fast! I can not keep
sentence grammatically correct and (a) an ……… with you.
contextually meaningful. (b) the
(c) a (a) in (b) up to
113. Don’t read ……… the lines. Read the (d) some (c) up (d) on
lines.
_ (c) Option (c) ‘a’ makes the sentence _ (c) Option (c) ‘up’ makes the
(a) in (b) between grammatically correct and sentence grammatically correct and
(c) on (d) by contextually meaningful. contextually meaningful.
Paper III General Awareness

1. Which one of the following 4. The increases in private (c) The Congress did not agree to the
statements is not correct? investment spending induced by suspension of the Civil
the increase in Government Disobedience Movement.
(a) Real GDP is calculated by valuing
spending is known as (d) The Congress agreed to take part
outputs of different years at
in the Second Round Table
common prices. (a) Crowding in (b) Deficit financing
Conference.
(b) Potential GDP is the real GDP that (c) Crowding out (d) Pumping out
the economy would produce if its _ (d) The second table conference was
resources were fully employed.
_ (a) The increase in private held in London from 7, September,
investment spending induced by the 1931 to 1st December, 1931 with the
(c) Nominal GDP is calculated by increase in government spending is participation of Gandhi and the Indian
valuing outputs of different years known as crowding in. Crowding-in National Congress.
at constant prices. effects explains that if government
spending increase, demand for Features of Gandhi-Irwin Pact
(d) Real GDP per capita is the ratio of
goods and services increase. This The Indian National Congress (INC)
real GDP divided by population.
situation causes a need for agreed to take part in the Round
_ (a) Real GDP is calculated by valuing enhancing production capacity of Table Conference.
outputs of different years at common private sector and private sector The INC would stop the civil
prices. Real Gross Domestic product investments increases. disobedience movement.
(Real GDP) is an inflation adjusted
measure that reflects the value of all 5. The asset or assets that a borrower Withdrawal of all ordinances that
goods and services produced by an curbed the activities of the
pledges in order to guarantee
economy in a given year (expressed Congress.
repayment of a loan is called as
in base-year prices) and is often Withdrawal of all prosecutions
referred to as constant price GDP, (a) Cheque (b) Collateral except those involving violent
inflation-corrected GDP or constant (c) Guarantee card (d) Bond crimes.
dollar GDP. Real GDP is calculated Release of those who were arrested
by dividing nominal GDP over a GDP _ (b) Collateral refers to an asset that a
lender accepts as security for a for taking part in the civil
deflator. disobedience movement.
loan.The collateral acts as a form of
2. The mismatch in the regional or protection for the lender. Removal of the salt tax.
occupational pattern of job 6. The percentage by which the 8. Who among the following
vacancies and the pattern of money the borrower pays back founded the Bhil Seva Mandal in
worker availability results in exceeds the money that was 1922?
(a) Structural unemployment borrowed is called as (a) Dayaram Gidumal
(b) Disguised unemployment (a) Bank rate (b) Gurusaday Dutt
(c) Altered unemployment (b) Nominal interest rate (c) Dhondo Keshav Karve
(d) Cyclical unemployment (c) Real interest rate (d) Amritlal Vithaldas Thakkar
_ (a) Structural unemployment is a (d) Terms of credit
_ (d) Bhil Seva Mandal was founded
category of unemployment caused
by differences between the skills _ (b) The percentage by which the by Amritlal Vithaldas Thakkar in 1922.
money the borrower pays back He was popularly known as Thakkar
possessed by the unemployed Bapa. He was an Indian social worker
exceeds the money that was
population and the jobs available in who worked for upliftment of tribal
borrowed is called as Nominal
the market. people in Gujarat state in India.
interest rate. It is a interest rate which
3. The situation in an economy refers to the interest rate before He became a member of the
taking inflation into account. servants of Indian Society founded
which is growing slowly along
by Gopal Krishan Gokhale in 1905.
with rapid inflation (rising price 7. Which one of the following Thakkar established a school for the
level) is called statements about the Gandhi-Irwin children of the sweepers of kurla
(a) Stagnation Pact of 1931 is correct? with the help from Ramaji Shinde, a
(b) Deflation (a) The Government would not release member of the depressed classes
mission.
(c) Stagflation those prisoners who had been
(d) Recession non-violent. 9. In 1911, who among the following
(b) The Government would not release introduced a bill in the Imperial
_ (c) Stagflation or recession-inflation those prisoners who had indulged
is a situation in which the inflation rate Legislative Council for
is high, the economic growth rate in the peaceful picketing of liquor introduction of compulsory and
slows, and unemployment remains and foreign cloth shops. free primary education in India?
steadily high.
70 CDS Solved Paper 2021 (I)

(a) Dadabhai Naoroji 12. The Amara-Nayaka system was programme initiated by the Department
of Science and Technology (DST) of the
(b) Bal Gangadhar Tilak a major political innovation of Indian government for cultivating
(c) Sir Harcourt Butler which Indian imperial rulers? knowledge-based innovations and ideas
(d) Gopal Krishna Gokhale (a) Cholas (b) Chalukyas driven by technology into fruitful
start-ups. NIDHI supports start ups from
_ (d) Gopal Krishna Gokhale (c) Guptas (d) Vijayanagar
the ideation stage to the marketing stage.
introduced a bill in the imperial
legislative council for introduction of _ (d) The Amara Nayaka system As part of the Start up India initiative
was a major political innovation of launched by the Prime Minister, DST
compulsory and free primary
Vijayanagar rulers. It was derived proposes to provide momentum and
education in India. He was a great
from the Iqta system of the Delhi scale to radically change the start up
social reformer and educationist
Sultanate. The Amara- Nayakas ecosystem in India.
who provided exemplary leadership
were military commanders. They
to India’s freedom movement. NIDHI is focused on creating a smooth
were given territories for governing
purposes by the Rayas. As part of and innovation-led entrepreneurial
10. Which one of the following ecosystem specifically by directing the
their military duties, the Amara-
statements about the Act V of youth towards it and consequently
Nayakas maintained an army. This
1843 relating to Slavery in India collection of armed forces allowed ushering in its positive effect on the
is correct? the Rayas to claim dominance over country’s development in the social and
the entire southern peninsula. economic spheres.
(a) It gave the masters the right to
wilfully keep their slaves tied to 13. KIRAN (Knowledge Involvement 15. The Saubhagya Scheme aims at
their estates. in Research Advancement universal
(b) It denied the masters the use of through Nurturing) is an (a) LPG connection
Courts to assert their claims on (b) household electrification
initiative to provide
slaves. (c) primary school education
opportunities to women
(c) The Law Courts and masters
scientists who (d) public health insurance
worked jointly in resolving the
cases of desertion. (a) had a break in their career due _ (b) Pradhan Mantri Sahaj Bijli Har Ghar
to family reason Yojana (SAUBHAGYA Scheme), was
(d) The slaves became the owners of
(b) are pursuing innovative research launched in 2017 to ensure electrification
the land. of all households in the country in rural as
in life sciences. well as urban areas.
_ (b) The Act V of 1843 relating to (c) have collaborative projects with
slavery in India. It denied the Focus on achieving Universal
masters the use of courts assert labs in foreign countries. Household Electrification in the country,
their clams on slaves. The Hon’ble (d) have leading edge projects in through last mile connectivity and
court of Directors in 1838 directed Nano-Science and Technology. providing access to electricity to all
that the Govt. of India should lose un-electrified households in rural areas
no time in passing an enactment to _ (a) The Knowledge Involvement in and poor households in urban areas.
Research Advancement through
the effect of the recommendation
Nurturing (KIRAN) Scheme is one Beneficiaries are identified using
just referred to. In 1843, Act V of
of the several pioneering initiatives Socio-Economic and Caste Census
that year was passed which carried
started by the Department of (SECC) 2011 data. However,
out the original recommendation of
Science and Technology (DST) for un-electrified households not covered
law commission. According to
promoting women in science. It is under the SECC data would also be
Section 4 of the act, which would
primarily aimed to bring gender provided electricity connections under
be penal offence if done to free man
parity in the Science and the scheme by paying ` 500.
shall be equally an offense if done
Technology sector by inducting
to any person on the pretext of his 16. Mission Indradhanush aims at
more women talent in the research
being in a condition of slavery.
and development domain through (a) reducing child deaths due to
11. Which Maratha statesman various programmes. It also pneumonia.
provides fellowship support to
signed the Treaty of Bassein on women ranging in the age group
(b) reducing the impact of rotavirus
December 31, 1802? 27 to 57 years for continuing higher (c) elimination of maternal tetanus.
(a) Baji Rao II education in Science and (d) full immunisation of children.
Technology after a break in career.
(b) Vithuji Holkar _ (d) Mission Indradhanush aims at full
immunisation of children. Mission
(c) Daulat Rao Sindhia 14. NIDHI is an umbrella scheme for Indradhanush (MI) was launched by the
(d) Madhava Rao Narayan the promotion of Ministry of Health and Family Welfare
_ (a) The Treaty of Bassein was (a) young and aspiring innovators (MoHFW) on 25th December, 2014.
essentially a Subsidiary Alliance (b) scientific research It aims of expanding immunisation
signed between the Britishers and coverage to all children across India.
Peshwa, Baji Rao II. The Treaty was
(c) primary health care
(d) primary education in rural areas It aims to cover all those children who
signed after battle of Pune. The
are either unvaccinated or are partially
company took control over the
_ (a) NIDHI (National Initiative for vaccinated against vaccine preventable
Maratha state and Peshwa under Development and Harnessing diseases. India’s Universal Immunisation
this pact. Innovations) is an umbrella Programme (UIP) provide free vaccines
CDS Solved Paper 2021 (I) 71

against 12 life threatening diseases : all-weather road to unconnected _ (d) The given characteristics in the
Tuberculosis, Diphtheria, Pertussis, habitations.The fund allocation to statement are of Black soil. Best soil
Tetanus, Polio, Hepatitis B, states has been made in subsequent for cotton cultivation, most of the
Pneumonia and Meningitis due to years commensurate with the value of Deccan is occupied by Black soil.It
Haemophilus Influenzae type b projects sanctioned to states. The has a high water retaining capacity.
(Hib), Measles, Rubella, Japanese Union Government bears 90% of the Self-ploughing is a characteristic of
Encephalitis (JE) and Rotavirus project cost in respect of projects the black soil as it develops wide
diarrhoea. (Rubella, JE and sanctioned under the scheme in cracks when dried. It is rich in: Iron,
Rotavirus vaccine in select states North-Eastern and Himalayan States, lime, calcium, potassium, aluminum
and districts). whereas for other states the Union and magnesium and is deficient in:
Government bears 60% of the cost. Nitrogen, Phosphorous and organic
17. Which one of the following Centrally Sponsored Scheme. matter. Mostly deep black to light
statements about the DDU- GKY is black colour with clayey texture.
19. India’s territorial limit extends
not correct?
towards the 22. Arabica, Robusta and Liberica are
(a) It is a skill training programme.
(a) 10 nautical miles varieties of
(b) It is for rural youth from poor
(b) 12 nautical miles (a) Coffee
families.
(c) Its objective is placement in wage (c) 14 nautical miles (b) Tea
employment. (d) 15 nautical miles (c) Sugarcane
(d) It relies entirely on skill training by _ (b) India’s territorial limit extends (d) Cotton
Government agencies. towards the 12 nautical miles. The
laws of seas are managed by the _ (a) Arabica, Robusta and Liberica
are varieties of Coffee. Arabica and
_ (d) The Ministry of Rural United Nations Convention on the
Development (MoRD) announced it Robusta are the two main types of
Law of the Sea (UNCLOS), also coffee beans which are used in
on Antyodaya Diwas in 2014.It is a called the Law of the Sea Convention.
demand-driven placement linked skill production of marketable coffee.
It is an international agreement that Coffee is a Beverage crop.
training initiative working under the establishes a legal framework for all
National Rural Livelihood Mission
(NRLM).It uses skill training and
marine and maritime activities. The 23. Aluminium is manufactured from
convention set the limit of various
placement in wage employment as a areas, measured from a carefully (a) Copper ore (b) Bauxite ore
tool to diversify income and enable defined baseline. (c) Mica ore (d) Manganese ore
sustained upward movement out of
poverty. 20. Which one of the following is a _ (b) Aluminium is manufactured from
bauxite ore. Aluminum originates
Beneficiaries: DDU-GKY is uniquely cold ocean current? from bauxite, an ore typically found in
focused on rural youth between the the topsoil of various tropical and
(a) Brazilian Current
ages of 15 and 35 years from poor subtropical regions.
families. And its objectives are (b) Gulf Stream
Adding diversity to the incomes of (c) North Equatorial Current 24. Which one of the following sea
rural poor families and Cater to the (d) California Current ports gained significance for
career aspirations of rural youth.
_ (d) Among the given options, handling iron-ore exports to Japan ?
18. Which of the following statements California current is a cold ocean (a) Kandla (b) Ennore
about the Pradhan Mantri Gram current. It is a cold water Pacific
ocean current that moves southward (c) Kochi (d) Mormugao
Sadak Yojana are correct? along the western coast of North _ (d) Marmagao Port, situated at the
1. It is part of Government of America. Brazilian, Gulf Stream and entrance of the Zuari estuary, is a
India’s poverty reduction North Equatorial are warm ocean natural harbour in Goa. It gained
strategy. currents. significance after its remodelling in
1961 to handle iron-ore exports to
2. It is a centrally sponsored 21. Identify the type of soil on the Japan.
scheme for rural development. basis of the given characteristics:
25. Which one of the following is not
3. It provides connectivity in rural 1. They are rich in lime, iron,
a function of money?
areas. magnesia and alumina.
(a) Acts as an intermediate in the
Select the correct answer using the 2. They are generally clayey, deep
exchange process
codes given below: and impermeable.
(b) Acts as a store of value
(a) 1, 2 and 3 (b) 1 and 3 3. They are mainly found in (c) Used as the unit of account
(c) 2 and 3 (d) 1 and 2 Maharashtra, Madhya Pradesh (d) Used for regulating consumption
and Gujarat.
_ (a) The Union Minister of Rural _ (d) Money is any item or verifiable
Development provided information on Select the correct answer using the record that is generally accepted as
implementation of the Pradhan Mantri codes given below. payment for goods and services and
Gram Sadak Yojana (PMGSY), in (a) Laterite soil repayment of debts, such as taxes, in
Rajya Sabha. Launched on 25th a particular country or
(b) Red and yellow soil
December, 2000. Its objective are : socio-economic context. The main
(c) Saline soil
To provide connectivity, by way of an functions of money are distinguished
(d) Black soil
72 CDS Solved Paper 2021 (I)

as: a medium of exchange, a unit of actions. Most exchange rates are free Bombay areas, as well as Assam and
account, a store of value and floating and will rite or fall based on Coorg provinces. In this system, the
sometimes, a standard of deferred supply and demand in the market peasants or cultivators were regarded
payment. Any item or verifiable record while some currencies are not free as the owners of the land. They had
that fulfils these functions can be floating and have restrictions. ownership rights, could sell,
considered as money. mortgage or gift the land.
29. Francois Bernier was physician to The taxes were directly collected by
26. The situation where the (a) Prince Murad the government from the peasants.
equilibrium level of real GDP falls (b) Princess Jahanara Here there were no middlemen as in
short of potential GDP is known as the Zamindari system.
(c) Emperor Shah Jahan
(a) Recessionary gap (d) Prince Dara Shikoh
(b) Inflationary gap
32. Which one of the following
_ (d) Francois Bernier, a French statements about the British
(c) Demand-side inflation physician, political philosopher, Indian Medical Service (IMS) is not
(d) Supply-side inflation traveller, and historian, lived in India
for 12 years. He was associated with correct ?
_ (a) A recessionary gap, or the Mughal Court as a personal (a) IMS began in 1764.
contractionary gap, refers to the
difference between actual and physician of Dara Shikoh (eldest son (b) It recruited health professionals by
potential production in an economy. It of emperor Shah Jahan) and later as means of a competitive
basically occurs when the countries a scientist with Danishmand Khan.
examination.
real GDP is lower than the GDP at its 30. Which one of the following British (c) Indians were never admitted to
full employment. IMS.
firms was taken over by
27. The excess of total expenditure of Soorajmull-Nagarmull group? (d) The IMS was at first meant to look
Government over its total receipts, after the troops.
(a) McLeod
excluding borrowings, is known as (b) Octavius Steel _ (c) The Indian Medical Service (IMS)
was a military medical service in
(a) Primary deficit (b) Fiscal deficit (c) Davenport British India, which also had some
(c) Current deficit (d) Capital deficit (d) Andrew Yule civilian functions. It served during the
_ (b) Fiscal Deficit is the difference two World Wars, and remained in
_ (a) Among the given options, existence until the independence of
between the total income of the McLeod was the British firm taken
government (total taxes and non-debt over by Soorajmull - Nagarmull India in 1947.
capital receipts) and its total group. Soorajmull Jalan and The first three surgeons to have
expenditure. A recurring high fiscal Nagarmull Bajoria both were Marwari served were John Banester on the
deficit means that the government traders in Calcutta in 1930s and Leicester, Lewis Attmer on the
has been spending beyond its owned extensive real estate. In 1940s Edward and Rober on the Francis.
means. Soorajmull-Nagarmull group moved The first indian natives to join was
into jute manufacturing. To further Soorjo Coomar Goodeve
28. Exchange rates state the value of enhance there productivity they Chuckerbutty on 24th January, 1855,
one currency in terms of other bought control of Mcleod and Co. followed by Rajendra Chandra on
currencies. Which one of the Mcleod , owned 10 jute mills, 16 tea 27th January, 1858.
following statements with respect companies and light railways.
33. When was the monopoly of China
to the exchange rate of currency is 31. Which one of the following trade lost by East India Company?
correct ? statements with regard to ryotwari (a) 1813 (b) 1833
(a) Floating exchange rates are rates settlement is not correct? (c) 1838 (d) 1860
in which the Governments interfere (a) In southern and western India, the
by buying or selling their ryotwari settlement was adopted. _ (b) In 1833, the monopoly of China
trade was lost by East India
currencies. (b) Ryotwari was in principle a direct company. In the late seventeenth
(b) Fixed exchange rates are rates set contract between the ryot and the century the East India Company
by Government decisions and state. shifted its attention in East Asia to
maintained by Government (c) It means a tax contract valid for China. Tea, silk, and porcelain were
actions. the main exports from China; silver,
usually 30 years.
(c) Under the Bretton Woods System, Bengal cotton, and, eventually, opium
(d) In principle, it strengthened the (traded indirectly) were the
the exchange rates are floated in former elite, the zamindars and company’s principal exports. The
terms of rise or fall in price of gold. weakened the peasantry. Charter Act, 1813,with the rising
(d) Under the classical gold standard, demand of english traders in share of
the exchange rates are fixed in _ (d) Ryotwari settlement strengthened Indian trade the company was
the former elite, the zamindars and
terms of price of dollar. weakened the peasantry. This system deprived with trade monopolies,
of land revenue was instituted in the however was allowed to enjoy the
_ (b) Exchange rates state the value of monopoly of trade with China and
one currency in terms of other late 18th century by Sir Thomas
Munro, Governor of Madras in 1820. trade in tea. With the Act of 1833
currencies. Fixed exchange rates are
Ryot means peasant cultivators. This there was a complete abolishment of
rates set by government decisions
was practiced in the Madras and companies trade monopoly.
and maintained by government
CDS Solved Paper 2021 (I) 73

34. National Disaster Management 37. ‘Legal Positivism’ theory was (a) Welfare approach
Authority is headed by developed by (b) Basic needs approach
(a) the Prime Minister (a) T.H. Green (b) Thomas Hobbes (c) Income approach
(b) the Home Minister (c) John Austin (d) Patrick Devlin (d) Capability approach
(c) the President _ (c) Legal positivism is one of the _ (b) Basic Needs approach of human
(d) the Health Minister leading philosophical theories of the development was initially proposed
nature of law, and is characterised by by the International Labour
_ (a) National Disaster Management two theses: (1) the existence and Organisation (OLO) in 1976 and it
Authority (NDMA) is headed by the emphasised on health, education,
content of law depends entirely on
Prime Minister. It is an Apex body of food water supply, sanitation and
social facts (e.g., facts about human
Government of India. It was housing. The basic needs approach
behaviour and intentions), and (2)
established through the Disaster is one of the major approaches to the
there is no necessary connection
Management Act enacted by the measurement of absolute poverty in
between law and morality—more
government on 23rd December, developing countries.
precisely, the existence and content
2005. NDMA mandate is to lay down
of a law do not depend on its merits 41. Which one of the following
policies for disaster management.
or demerits.
tributaries of river Indus feeds the
35. Which of the following statements 38. Who among the following said canal system of the Bhakra Nangal
regarding the ‘casting vote’ in the that Democracy means a system of Project?
Parliament is/are correct? 'Government by Consent' ? (a) Chenab (b) Setluj
1. It is cast by the speaker or a (a) John Locke (c) Ravi (d) Jhelum
person acting as such. (b) J.S. Mill _ (b) The Setluj river feeds the canal
2. It is cast in addition to voting in (c) Jeremy Bentham system of Bhakra Nangal Project. The
the first instance. (d) J.J. Rousseau Bhakra-Nangal Dam is the second
tallest dam in Asia and located in the
3. It is cast in the case of equality _ (a) The statement ‘Democracy border of Punjab and Himachal
of votes. means a system of Government by Pradesh.
4. It is always cast to maintain the consent’ was made by John Locke.
He was a British philosopher and 42. When the rivers discharge their
status quo. medical researcher. Two treatises of waters from all directions into a
Select the correct answer using the Government (1689), An Essay lake or depression, the pattern is
codes given below: concerning Human Understanding known as
(1689) are some of his notable
(a) 1, 2 and 3 (b) 1 and 3 books. (8) Trellis (b) Dendritic
(c) 2 and 4 (d) Only 3 (c) Radial (d) Centripetal
39. Milpa and Ladang are different
_ (b) A Speaker uses his/her power to _ (d) When the rivers discharge their
vote, in order to resolve a deadlock. names for waters from all directions into a lake
That is, when the House initiates a (a) shifting cultivation or depression, the pattern is known
voting procedure, the speaker does as centripetal. In this system, streams
(b) mixed farming
not cast a vote in the first instance. It flow toward a central depression.
is only when the two sides receive (c) truck farming
During wetter portions of the year,
equal number of votes that the (d) plantation agriculture these streams feed ephemeral lakes,
Speaker’s vote breaks the deadlock, which evaporate away during dry
making his/her position impartial. _ (a) Milpa and Ladang are different
names for shifting cultivation. Shifting periods. e.g. Loktak lake in Manipur.
36. Equality before the law or equal cultivation in Mexico is called Milpa
and shifting cultivation in Indonesia is 43. Which one of the following
protection of the laws within the called Ladang. Shifting cultivation is statements about the Coriolis force
territory of India is guaranteed an agricultural method in which a is not correct?
under which one of the following person uses a piece of land just a (a) It is maximum at the Poles.
Articles of the Constitution of short time later to abandon or change
(b) It is absent at the Equator.
the initial use. This method also
India? (c) It deflects the wind to the right
involves. Clearing piece of land
(a) Article 14 before the soil loses fertility, followed direction in the southern
(b) Article 15 by several years of wood harvesting hemisphere.
(c) Article 16 or farming. (d) It deflects the wind to the right
direction in the northern hemisphere.
(d) Article 22 40. Which one of the following
_ (a) According to the Constitution of approaches of human development _ (c) The rotation of the Earth about its
India Article 14 states that the State axis affects the direction of the wind
was initially proposed by the and this force is called the Coriolis
shall not deny to any person equality International Labour Organisation
before the law or the equal protection force. It is directly proportional to the
of the laws within the Territory of
(ILO) and emphasised on health, angle of latitude. It deflects the wind
India, on grounds of religion, race, education, food, water supply, to the left direction in the southern
caste, sex or place of birth. sanitation and housing? hemisphere and the right direction in
the northern hemisphere.
74 CDS Solved Paper 2021 (I)

44. Chronologically arrange the Select the correct answer using 49. Which one of the following is not
following treaties/conventions the codes given below : an objective of NITI Aayog?
signed between the Marathas and (a) 1 and 2 (b) 1, 2 and 3
(a) It provides a critical direction
the British (starting with the (c) 2 and 3 (d) 1 and 3
and strategic input for
earliest) _ (b) The Faqir- Sanyasi resistance development process.
1. Treaty of Salbai movement was organised and lead
by Manju Shah. The rebellion started (b) It functions as a ‘think tank’ in
2. Treaty of Purandar in 1750s onward and took violent turn providing key elements of
3. Convention of Wadgaon in 1773 when Warren Hastings took policy.
over the Governor- Generalship of
4. Treaty of Surat Bengal and Bihar. (c) It monitors and evaluates the
Select the correct answer using the implementation of the
codes given below:
46. In 1943, young leaders in which programmes.
one of the following districts of
(a) 4-2-3-1 (b) 4-3-2-1 (d) It offers a platform for
Maharashtra set up a parallel
(c) 1-3-2-4 (d) 1-2-3-4 resolution of inter-state
Government (prati sarkar) with
conflicts as ‘provider of first
_ (a) Treaty of Surat 6th March, volunteer corps (seva dals) and
1775, was signed between and last resort’.
village units tufan dals)?
Raghunathao, a claimant to the
(a) Pune (b) Nasik _ (d) Objectives of NITI Aayog
throne of the Peshwa and the British
1. To foster cooperative federalism
East India Company at Bombay. As (c) Nagpur (d) Satara
through structured support
per this treaty, the British got Salsette
and Bassein (Vasai) and also part _ (d) Satara (mid-1943 to 1945), “Prati initiatives and mechanisms with
Sarkar”, was organised under leaders the States on a continuous
revenues from Baruch and Surat.
like Y.B. Chavan, Nana Patil, etc. basis, recognising that strong
Treaty of Purandar 1st March States make a strong nation.
1776, signed by the British Calcutta 47. Who founded the Central Hindu 2. To develop mechanisms to
Council with Nana Phadnavis School at Benaras which was later formulate credible plans at the
representing the Peshwa. As per this developed into Benaras Hindu village level and aggregate these
treaty, Raghunath Rao was only
University? progressively at higher levels of
given a pension and was not
(a) Madan Mohan Malaviya government.
promised any support for his claim
to the Peshwa seat. But the British (b) Annie Besant 3. To ensure, on areas that are
did retain Baruch and Salsette. (c) Ishwar Chandra Vidyasagar specifically referred to it, that the
interests of national security are
Convention of Wadgaon (d) Madame H.P. Blavatsky
incorporated in economic
13th January 1779, compact
concluded after the First Maratha _ (a) Pt. Madan Mohan Malaviya strategy and policy.
founded central Hindu School at 4. To pay special attention to the
War in India (1775–82), marking the
Benaras which was later developed sections of our society that may
end of British efforts to intervene in
into Benaras Hindu University. He be at risk of not benefitting
Maratha affairs by making
was born on 25th December, 1861, in adequately from economic
Raghunath Rao Peshwa or at least
Allahabad in Uttar Pradesh. Malaviya progress.
regent for his infant great-nephew.
played a key role in the Indian
Treaty of Salbai 17th May, 1782, independence struggle against British 5. To provide advice and
which ended the First Anglo-Maratha rule and founded the Banaras Hindu encourage partnerships between
War, was signed on 17th May, 1782 University (BHU) in 1916. He is also key stakeholders and national
between the British East India remembered for his role in ending the and international like-minded
Company and the Marathas. Indian indenture system, especially in Think Tanks, as well as
Marathas promised not to allow any the Caribbean. educational and policy research
French settlements in their regions. institutions.
48. Which one among the following is 6. To offer a platform for resolution
45. Which of the following statements not a feature of centralisation? of inter-sectoral and
about the Sanyasi and Faqir inter-departmental issues in
(a) National unity (b) Uniformity
disturbances/rebellions are correct? order to accelerate the
(c) Prosperity (d) Liberty implementation of the
1. Governor-General Warren development agenda.
Hastings faced the persistent _ (d) Among the given options, liberty
is not a feature of centralisation.
Sanyasi and Faqir disturbances Centralisation is the method where 50. Article 231 of the Constitution of
in Bengal and Bihar. designing and decision making India grants power to establish a
processes within an organisation or a common High Court for two or
2. There were a number of
managerial body are implied. In the more states to
Shaivite Naga Sanyasis who central governing body, roles, and
formed into armed bands. duties are clearly defined by the (a) the Parliament
3. Majnu Shah, who led bands higher authority and decision (b) the Supreme Court
formulation is very transparent and (c) the President of India
into Bengal from 1771, was uniform.
their prominent leader. (d) the Union Cabinet
CDS Solved Paper 2021 (I) 75

_ (a) In 7th Constitutional Amendment _ (b) Rice is an important Kharif crop in 57. Alluvial soils vary in nature from
Act, 1956 , Article 231 was amended India. Aman, Aus and Bovo are rice
sandy loam to clay. They are
to enable Parliament to establish a varieties. Rice is usually sown thrice.
common High Court for two or more Aman variety is sown in the rainy generally
States. season (July-August) and harvested (a) poor in potash and rich in
in winter. This is mainly produced. phosphorus.
51. Which one among the following Aus rice sown in summer along with (b) poor in both potash and
motions cannot be made while the premonsoonal showers and
harvested in autumn. The quality of
phosphorus.
introducing an ordinary Bill in the (c) rich in both potash and phosphorus.
this rice is rough. Boro rice sown in
Parliament? (d) rich in potash and poor in
winter and harvested in summer. This
(a) That the Bill be taken into is also called spring rice. phosphorus.
consideration
(b) That the Bill be circulated for the 54. Which one of the following is a _ (d) The alluvial soil occurs mainly in
the Setluj- Ganga- Brahmaputra
purpose of eliciting public opinion type of igneous rock? Plains. They are also found in the
(c) That the Bill be referred to a Select (a) Marble (b) Halite valleys of the Narmada, Tapi, and the
Committee (c) Granite (d) Shale Eastern and Western coastal plains.
(d) That the Bill be referred to a Joint These soils are mainly derived from
_ (c) Among the given options, Granite the debris brown from the
Committee of the House without is a type of igneous rock. Igneous
the concurrence of the other rocks form when hot, molten rock Himalayas. This soil is well-drained
crystallises and solidifies. The melt and poorly drained with an immature
House
originates deep within the Earth near profile in undulating areas.
_ (d) An Ordinary Bill is introduced in active plate boundaries or hot spots, This soil has a phosphorous
either House of the parliament. This deficiency. The color of soil varies
then rises toward the surface.
bill is introduced by a minister or a from light grey to ash. This soil is
Igneous rocks are divided into two
private member. As per Articles 107 suited for Rice, maize, wheat,
groups, intrusive or extrusive,
and 108 of the Indian Constitution, an sugarcane, oilseeds, etc.
depending upon where the molten
ordinary bill is concerned with any
rock solidifies. e.g. diabase, diorite,
matter other than financial subjects.
gabbro, granite, pegmatite, and 58. Who among the following was
52. Which one among the following peridotite. popularly known as ‘Lokahitwadi’ ?
was eliminated by the 44th 55. Ozone layer, which absorbs the (a) Gopal Hari Deshmukh
Amendment Act of the ultra-violet radiation, is found in (b) Mahadev Govind Ranade
Parliament? which one of the following layers (c) Gopal Krishna Gokhale
(a) Right against Exploitation of the atmosphere? (d) Jyotiba Phule
(b) Right to Constitutional Remedies (a) Ionosphere (b) Troposphere _ (a) Gopal Hari Desmukh (1823-92),
(c) Right to Property popularly known as “Lokahitawadi”,
(c) Mesosphere (d) Stratosphere
was a product of the Western
(d) Right to Education
_ (d) The ozone layer is the common learning in India. He was a judge and
_ (c) The 44th Amendment of 1978 term for the high concentration of a member of the Governor-General’s
removed the right to property from ozone that is found in the Council in 1880. A great social
the list of fundamental rights. A new stratosphere around 15–30 km above reformer and rational thinker,
provision, Article 300-A, was added to the earth’s surface. It covers the “Lokahitawadi” urged the people to
the Constitution, which provided that entire planet and protects life on be self-reliant and seek Western
“no person shall be deprived of his Earth by absorbing harmful learning. These were, in his view,
property save by authority of law. ultraviolet-B (UV-B) radiation from tools for cultivating a rational outlook
the Sun. and for solving the country’s pressing
53. Identify the crop on the basis of problems.
the following characteristics : 56. Which one of the following
statements about primary waves 59. Who among the following socialist
1. It is a kharif crop. activists persuaded Gandhiji not to
of earthquakes is not correct ?
2. Aus, Aman and Boro are its restrict the salt march protest to
(a) They are similar to sound waves.
three different growing periods men alone?
(b) They can travel only through solid
in an agricultural year. materials. (a) Kamaladevi Chattopadhyay
3. About one-fourth of the total (c) They travel through gaseous, liquid (b) Sarojini Naidu
cropped area of India is under and solid materials. (c) Matangini Hazra
its cultivation. (d) They move faster and are the first (d) Mithuben Petit
Select the correct answer using the to arrive at the surface.
_ (a) The socialist activist Kamaladevi
codes given below: _ (b) P-waves move faster and are the Chattopadhyay had persuaded
(a) Wheat first to arrive at the surface. These are Gandhiji not to restrict the protests to
also called ‘primary waves’. The men alone. Kamaladevi was herself
(b) Rice P-waves are similar to sound waves. one of the numerous women who
(c) Pulses They travel through gaseous, liquid courted arrest by breaking the salt or
(d) Cotton and solid materials. liquor laws.
76 CDS Solved Paper 2021 (I)

60. Which one of the following _ (b) Faizi translated Lilavati which is a 66. Which mega public sector
work on mathematics into Persian
painters was not associated with undertaking is designated with
and under his supervision, the
Humayun ? Mahabharata was translated into the ‘Navratna’ status?
(a) Mir Sayyid Ali Persian language. Faizi was brother (a) Indian Oil Corporation Limited
(b) Maulana Dost Musawir of famous poet Abul Fazl. (b) Gas Authority of India Limited
(c) Maulana Yusuf 63. Who among the following was the (c) Bharat Petroleum Corporation
(d) Bihzad Chairman of the National Limited
(d) Bharat Electronics Limited
_ (d) Among the given painters, Bihzad Commission for Review of the
was not associated with Humanyun. Working of the Constitution (2000)? _ (d) Among the given options, Bharat
A Under the reign of Timurids and the Electronics limited is designated with
(a) Justice M.N. Venkatachaliah ‘Navratna’ Status Bharat Electronics
dynasty of Safavids in todays
Afghanistan and Iran. He lead an (b) Justice J.S. Verma Limited (BEL) is an Indian
entire workshop producing (c) Justice Ranganath Mishra Government owned aerospace and
manuscripts illumination. defence electronics company. It was
(d) Justice Y.K Sabharwal
founded in 1954.
61. Match List I with List II and select _ (a) Justice M. N Venkatachaliah was The Headquarters of BEL is located
the correct answer using the code the chairman of the National in Bengaluru, Karnataka.
Commission for Review of the
given below the lists :
working of the Constitution (2000). It 67. Which of the following statements
List I List II was a resolution set up by the NDA with regard to the creation of a new
(Author) (Poetical work) Government of India for suggesting
possible amendments to the state or alteration of boundaries of
A. Brindavan Das 1. Shivasankirttan Constitution of India. states is/are correct ?
B. Krishnadas 2. Chaitanyacharitamrita 64. M.M. Punchhi Commission on 1. An amendment has to be
Kaviraj moved in the Parliament under
Centre-State Relationshas
C. Mukundaram 3. Chandimangal recommended the disposal of a bill Article 368 of the Constitution
Chakravarti reserved for the consideration of of India.
D. Rameshwar 4. Chaitanyamangal the Union Executive within 2. Ratification by one half of the
Bhattacharya (a) four months (b) six months states is necessary after the
(c) eight months (d) five months
amendment is made by the
Codes Parliament.
A B CD A B CD _ (b) Bills reserved for the
consideration of the Union Executive 3. The legislation can be passed
(a) 4 2 3 1 (b) 4 3 2 1 should be disposed of within the time by a simple majority of both
(c) 1 3 2 4 (d) 1 2 3 4 period of six months. In April 2007, a Houses of Parliament.
three member commission headed
_ (*) Chandimangal is a work written by the former chief justice of India
Select the correct answer using the
by Mukundaram Chakravarti. code given below:
M.M. Punchhi was set up by the UPA
Chandimangal is an important
Government to take a fresh look at (a) 1and 3 (b) Only 2
subgenre of managalkavya, the most
relative roles and responsibilities of (c) Only 3 (d) 2 and 3
significant genre of medieval Bengali
various levels of government and
literature.
their inter-relations. This commission _ (c) Article 3 Formation of new States
Chaitanyacharitamrita was had submitted its report in 2010 to and alteration of areas, boundaries or
composed by Krishnadas Kaviraj. It then Home Minister P. Chidambaram. names of existing States: Parliament
is one of the primary biographies may by law
detailing the life and teachings of 65. Who among the following founded (a) Form a new State by separation
Chaitanya Mahaprabhu. the Marathi newspaper ‘Kesari’ ? of territory from any State or by
Chaitanya Mangala was composed (a) Lokmanya Tilak uniting two or more States or
by Lochana Dasa. It is influenced by parts of States or by uniting any
the Sanskrit Kadacha of Murari
(b) Vallabhbhai Patel territory to a part of any State;
Gupta. (c) Lala Lajpat Rai
(b) Increase the area of any State;
(d) Mahatma Gandhi
62. Which one of the following (c) Diminish the area of any State;
scholars of Akbar’s court _ (a) Kesari is a newspaper founded in (d) Alter the boundaries of any
1881 by Lokmanya Bal Gangadhar State;
translated Bhaskaracharya’s Tilak, a prominent leader of the Indian
Lilavati into Persian ? Independence movement. Bal (e) Alter the name of any State;
Gangadhar Tilak used to run his two Provided that no Bill for the
(a) Abul Fazl
newspapers, the Kesari, in Marathi purpose shall be introduced in
(b) Faizi either House of Parliament
and Maratha in English from Kesari
(c) Fathullah Shirazi except on the recommendation
Wada.
(d) Ataullah Rashidi of the President
CDS Solved Paper 2021 (I) 77

68. Which one of the following is the Codes 74. Which of the following statements
largest linguistic group of India? A B CD about the Panchayati Raj System
(a) Sino-Tibetan (b) Austric (a) 3 2 4 1 are correct ?
(c) Indo-Aryan (d) Dravidian (b) 3 4 2 1 1. All seats in a Panchayat are
(c) 1 4 2 3 filled by persons chosen by
_ (c) Indo-Aryan are the largest
lingustic group of India. People (d) 1 2 4 3 direct election.
speaking Indo Aryan language came _ (b) The correct matching of the lists is 2. The ‘Gram Sabha’ consists of
through North-western Mountains
and got settled in the North West in List I List II persons registered in the village
Punjab and later in Gangatic plains. electoral rolls.
These were called as Aryans or Indo A. Cyclones 3. Indian Ocean
3. The Chairperson of a
Aryans. They spoke Indo Iranian, Indo
B. Hurricanes 4. Atlantic Ocean Panchayat is elected in
European or Sanskrit. Origin of
Aryans is not clearly known, different C. Typhoons 2. South China Sea accordance with a law passed
scholars have different views on this by each state.
point. It is said that Aryans lived in D. Willy willies 1. Western Australia
4. All states in India have a
East of Alps (Eurasia), Central Asia,
Arctic region, Germany, and Southern 72. Which of the following is/are three-tier system of
Russia. social security scheme(s)? Panchayats.
Select the correct answer using the
69. Which one of the following water 1. Atal Pension Yojana
codes given below:
bodies separates the Andaman and 2. Pradhan Mantri Jeevan Jyoti
the Nicobar islands? (a) 1 and 4 (b) 1, 2 and 3
Bima Yojana
(c) 2 and 3 (d) 2, 3 and 4
(a) Gulf of Mannar 3. Pradhan Mantri Suraksha Bima
(b) Ten Degree Channel Yojana _ (b) A three-tier structure of the Indian
administration for rural development
(c) Eleven Degree Channel Select the correct answer using the is called Panchayati Raj. The aim of
(d) Palk Strait codes given below: the Panchayati Raj is to develop local
self-governments in districts, zones
_ (b) The Ten Degree Channel is a (a) Only 1
and villages. Balwant Rai Mehta
channel that separates the Andaman
(b) 2 and 3 Committee and Panchayati Raj, the
and Nicobar in the Bay of Bengal.
The two sets of islands together form (c) 1,2 and 3 committee was appointed in 1957, to
the Indian Union Territory of (d) 1 and 3 examine and suggest measures for
Andaman and Nicobar Islands. It also better working of the Community
separates Little Andaman and Car _ (c) All the schemes given above are Development Programme and the
social security schemes. National Extension Service. The
Nicobar.
Atal Pension Yojana was launched in committee suggested the
70. Which one of the following the year 2015 to create a universal establishment of a democratic
longitudes is the standard social security system for all Indians, decentralised local government
especially the poor, under privileged which came to be known as the
meridian of India? and the workers in unorganised Panchayati Raj.
(a) 83°30′ E (b) 82°30′ E sector.
(c) 82°30′ W (d) 83°30′ W Pradhan Mantri Jeevan Jyoti Bima
75. Which of the following statements
Yojana was launched in year 2015. It is/are correct?
_ (b) From Gujarat to Arunachal is a life insurance scheme.
Pradesh there is a time lag of two 1. The elections to the Panchayat
hours. Hence, time along the Pradhan Mantri Suraksha Bima are conducted by the State
Standard Meridian of India (82° 30’ E) Yojana was launched in year 2015. It
Election Commission.
passing through Mirzapur (in Uttar is a government backed accident
Pradesh) is taken as the standard insurance scheme. 2. The State Election
time for the whole country. Commissioner is appointed by
73. Which one of the following units the Chief Election
71. Match List I with List II and select is not a part of All India Radio’s
the correct answer using the codes Commissioner of India.
Transcription and Programme
given below the lists: Exchange Service? 3. The State Legislatures have the
power to make laws on all
List I List II (a) Central Archives
matters relating to Panchayat
(b) Sound Archives
A. Cyclones 1. Western Australia elections.
(c) Advanced Research Unit
B. Hurricanes 2. South China Sea Select the correct answer using the
(d) Transcription Unit
codes given below:
C. Typhoons 3. Indian Ocean
_ (c) Advanced Research unit is not a (a) Only 2 (b) 1 and 2
D. Willy-willies 4. Atlantic Ocean part of All India Radio’s Transcription
and Programme Exchange Service. (c) 1, 2 and 3 (d) 1 and 3
78 CDS Solved Paper 2021 (I)

_ (d) The commission is responsible 78. Which among the following process is quite similar to that of the
for superintendence, direction and President of India but the electoral
fundamental rights is/are available college that elects President is
control of the preparation of electoral
rolls and conducting elections for the to non-citizens? different from the electoral college
panchayat.The state legislature may 1. Freedom of speech responsible for the election of
make provisions with respect to all Vice-President of India.
matters relating to elections to the 2. Protection against
panchayats. self-incrimination 80. A motion of no-confidence is
3. Freedom of conscience moved against
76. Which one of the following is not (a) an individual Minister.
a correct description of the Union 4. Non-discrimination in matters
of employment (b) the Council of Ministers.
Cabinet?
(c) the Prime Minister.
(a) It is part of the Parliament. Select the correct answer using the
codes given below (d) a political party.
(b) It is responsible to the Parliament.
(c) It remains in power till it enjoys the (a) 1, 2 and 3 (b) 1.3 and 4 _ (b) A motion of no-confidence is
moved against council of Ministers.
confidence of the Parliament. (c) 2 and 3 (d) Only 2 A no-confidence motion can be
(d) A person from outside the
_ (c) Fundamental Rights (available to moved by any member of the House.
Parliament can never be appointed citizens as well foreigners) (except It can be moved only in the Lok
a member of the Cabinet. enemy aliens) Sabha and not Rajya Sabha. Rule
198 of the Rules of Procedure and
_ (b) The Council of Ministers is Equality before law – Article 14
conduct of Lok Sabha specifies the
directly responsible to the Lok Sabha Protection in respect of conviction for procedure for moving a
and can remain in office so long as it offences. – Article 20 no-confidence motion. If the
enjoys the majority support in the
Protection of personal life and liberty government is not able to prove its
House of the People. If the Lok Sabha
– Article 21 majority in the House, then the
passes a vote of no confidence
government of the day has to resign.
against the Council of Ministers, the Protection against arrest and
ministers have to resign collectively. detention in certain cases 81. Which one of the following is
77. Which one of the following – Article 22 correct about ‘Pinaka’ ?
statements about the composition Prohibition of human trafficking and (a) It is a multi-barrel rocket system.
of the Parliament is not correct? forced labour – Article 23 (b) It is a type of battle tank.
Prohibition of employment of children (c) It is an advanced submarine.
(a) Representatives of the states in
in factories – Article 24 (d) It is an indigenously developed
Rajya Sabha are elected directly
Freedom of conscience and free drone system.
by the people.
profession, practice and propagation
(b) Representatives from Union of religion – Article 25 _ (a) Pinaka is an indigenous
Territories in Lok Sabha are multi-barrel rocket launch system,
Freedom to manage religious affairs which has been developed by the
chosen by direct elections.
– Article 26 Defence Research and Development
(c) Rajya Sabha has 12 nominated Organisation (DRDO) for the Indian
members. Freedom from payment of taxes for
promotion of any religion – Article 27 Army.Its weapon system has a
(d) Lok Sabha has seats reserved for state-of-the-art guidance kit
Freedom from attending religious bolstered by an advanced navigation
SCs and STs. instruction or worship in certain and control system.
_ (a) Rajya Sabha (The Council of educational institutions – Article 28
States) is the Upper House (Second 82. Which one of the following is not
Chamber or House of Elders) and it 79. Which one of the following a member of the Quad group of
represents the States and Union statements about the nations?
Territories of the Indian Union.It is Vice-President of India is not
called the permanent House of the (a) France (b) USA
correct?
Parliament as it is never fully (c) Australia (d) Japan
dissolved. (a) He is not eligible for re-election.
The IV Schedule of the Indian (b) He must not be less than 35 years _ (a) Among the given options, France
is not a member of Quad group of
Constitution deals with the allocation of age.
nations. The Quadrilateral Security
of seats in the Rajya Sabha to the (c) He is elected by members of an Dialogue (QSD) or QUAD Group is
states and UTs. Maximum strength electoral college consisting of an informal strategic forum
of Rajya Sabha is 250 ( out of which members of both Houses of comprising India, Japan, Australia
238 members are representatives of and the United States of America. It
Parliament.
the states and UTs (elected was established in 2007. It is often
indirectly) and 12 are nominated by (d) His term of office is five years.
dubbed as an “Asian” or “mini”
the President). _ (a) Vice-President of India is Eligible North Atlantic Treaty Organisation
The representatives of states are for Re-election. There is no direct (NATO), and is viewed as a
elected by the members of state election for the Vice-President of India counterbalance to China’s military
legislative assemblies. however, he/she is indirectly elected and economic clout in the
by an Electoral College. The election Indo-Pacific region.
CDS Solved Paper 2021 (I) 79

83. The first phase of Exercise 86. ‘CARAT Bangladesh 2020’ was a 89. In the year 2020 which one of the
Malabar, 2020 was conducted at joint naval exercise between following tiger reserves of India
(a) Port Blair Bangladesh and was declared by UNESCO as a
(b) Mumbai (a) India (b) USA Biosphere Reserve?
(c) Visakhapatnam (c) Japan (d) Thailand (a) Panna Tiger Reserve
(d) Kochi _ (b) On 4th November, 2020, (b) Namdapha Tiger Reserve
Bangladesh and United States held a (c) Dudhwa Tiger Reserve
_ (c) The first phase of Exercise joint Naval Exercise CARAT. CARAT
Malabar, 2020 was conducted at (d) Pench Tiger Reserve
Visakhapatnam. is Cooperation Afloat Readiness and
Training. The exercise was held at _ (a) In 2020 Panna Tiger Reserve was
Malabar Exercise is a multilateral Chittagong. It is a series of annual declared as Biosphere reserve.
war-gaming naval exercise that was bilateral military exercise conducted Biosphere reserves are areas of
started in 1992. It began as a by the US Pacific fleet with countries terrestrial and coastal or marine
bilateral exercise between the such as Bangladesh, Cambodia, ecosystems or its amalgamation. The
navies of India and the United Brunei, Indonesia, Philippines, biosphere reserve network was
States. Japan and Australia first Malaysia, Sri Lanka, Singapore and launched in 1971 by UNESCO, two
participated in 2007, and since Thailand. The focus of the exercise is years after the initiation of MAB- Man
2014, India, the US and Japan have mainly on ASEAN. However, it is also and the biosphere program.
participated in the exercise every held with non-ASEAN members such According to UNESCO, as of July
year. In 2020 Australia joined the as Sri Lanka and Bangladesh. This 2021, there are 714 Biosphere
Malabar Exercise on India’s request year, Bangladesh was the second to reserves across 129 countries in the
in order to contain China in the hold the exercise after Brunei. CARAT world which also include 21
Indo-Pacific region Brunei was held in South China sea transboundary sites. There are 18
between 5th-9th October, 2020. biosphere reserves in India.
84. Hollywood actor Sir Sean
Connery, who died recently, was 87. Peninsula Shield Force is a military 90. The Kaleshwaram Lift Irrigation
popularly known for the portrayal arm of System, which was in the news
of the role (a) Organisation of Islamic
recently, is constructed in
(a) Rocky Cooperation (a) Kerala
(b) Captain Jack Sparrow (b) Organisation of the Petroleum (b) Gujarat
(c) Terminator Exporting Countries (c) Telangana
(d) James Bond (c) Gulf Cooperation Council (d) Maharashtra
(d) Arab League
_ (d) Hollywood actor Sir Sean _ (c) The Kaleshwaram Lift Irrigation
Connery was known for the portrayel _ (c) The Peninsula Shield Force is the Project is a multi-purpose irrigation
of the role James Bond. He was a military arm of the Gulf Cooperation project on the Godavari River in
Scottish actor. He was the first actor Council (GCC). It is intended to deter, Kaleshwaram, Bhupalpally,
to portray fictional British secret agent and respond to, military aggression Telangana, India. Currently the
in movie, James Bond. against any of the GCC member world’s largest multi-stage lift
countries: Bahrain, Kuwait, Oman, irrigation project, its farthest upstream
85. The Luhri Hydro-Electric Power Saudi Arabia, Qatar, and the United influence is at the confluence of the
Project is being constructed on the Arab Emirates. In 1984, GCC Pranhita and Godavari rivers.
river decided to create a joint military force
of 10,000 soldiers divided into two 91. Cell wall is not present in cells of
(a) Setluj (b) Beas brigades, called the Peninsula Shield (a) Bacteria
(c) Ravi (d) Chenab Force, based in Saudi Arabia near the (b) Plants
Kuwaiti and Iraqi borders.
_ (a) 210 MW Luhri Stage-I Hydro (c) Fungi
Electric Project which is being located 88. Which one of the following ships (d) Humans
on River Setluj in Shimla and Kullu
districts of Himachal Pradesh. It is was involved in ‘Mission Sagar-II’? _ (d) A cell wall is a dead, rigid and
being implemented by Setluj Jal (a) INS Shakti (b) INS Vikramaditya protective structural layer surrounding
Vidyut Nigam Limited (SJVNL) on some types of cells, just outside the
(c) INS Kesari (d) INS Airavat
Build-Own-Operate-Maintain (BOOM) cell membrane. Its major function is
basis with active support from _ (d) INS Airavat was involved in to act as pressure vessels, preventing
Government of India and the State Mission Sagar II. As part of ‘Mission over-expansion of the cell when water
Government. Government of India is Sagar-II’, the Government of India is enters and protects cell from outer
providing grants of ` 66.19 crore for providing assistance to Friendly shock and invasion of other
enabling infrastructure. JVNL has Foreign Countries to overcome unwanted materials. Cell walls are
envisaged Internal Growth Targets of natural calamities and Covid-19 absent in animals and humans, but
total installed capacity from all pandemic. Towards the same INS are present in most other eukaryotes
sources of 5000 MW by 2023, 12000 Airavat is delivering food aid for the including algae, fungi and plants and
MW by 2030 and 25000 MW by the people of Sudan. in most prokaryotes.
year 2040.
80 CDS Solved Paper 2021 (I)

92. A child receives a tall beautiful _ (b) Mitochondria and Plastids like 98. Which one of the following
Chloroplast have their own DNA and
plant as a birthday gift from his substances do silver articles react
70 s ribosomes because of which
father with a quiz. The father they are able to synthesise some of with, that makes the silver articles
asked her how she would verify their own proteins and replicate black?
whether this tall plant was the independent of the nucleus. Thus, (a) Sulphur (b) Oxygen
progeny of both the tall parents or known as semi-autonomous cell
organelles.
(c) Carbon dioxide (d) Phosphorus
one tall and one short parent
_ (a) Silver articles, when exposed to
plant. She could verify this 95. Osmosis is the process of air become black after sometime.
through movement of water molecules This is because the silver metal
(a) cross-pollination from its reacts with sulphur present in the
atmosphere and forms silver
(b) self-pollination (a) higher concentration to its lower sulphide. Thus, a layer of silver
(c) tissue culture concentration through a cell wall. sulphide is formed on the surface of
(d) negative propagation (b) lower concentration to its higher silver articles, because of which they
concentration through a fully appear dull and black.
_ (b) Self-pollination is a form of permeable membrane.
pollination in which pollen from the 99. Which one of the following metals
same plant arrives at the stigma of a (c) higher concentration to its lower
is kept immersed in Kerosene oil
flower. There are two types of concentration through a fully
self-pollination: in autogamy, pollen is permeable membrane.
to protect it and to prevent
transferred to the stigma of the same (d) higher concentration to its lower
accidental fire?
flower; in geitonogamy, pollen is (a) Calcium (b) Sodium
concentration through a
transferred from one of the flower to
semi-permeable membrane. (c) Vanadium (d) Magnesium
the stigma of another flower on the
same flowering plant, or from _ (d) Osmosis is a process by which _ (b) Sodium is kept in kerosene
microsporangium to ovule within a the molecules of a solvent pass from because it is a highly reactive metal. If
single gymnosperm. Due to a solution of low concentration to a it is kept in an open container, it will
self-pollination, if all new progenies solution of high concentration react with oxygen and water vapor
are tall than the parents of that plant through a semi-permeable present in the atmosphere producing
were both tall, but if there are few membrane. sodium oxides and sodium
short plants in new progenies than hydroxides respectively which is a
one of the parent of that plant was 96. Growing of two or more crops highly exothermic reaction.
short and other one was tall. simultaneously on the same field
in a definite pattern is known as 100. A solution contains 20 g of solute
93. A student was doing an in 180 g of solvent. If the solvent is
experiment on increasing the cell (a) mixed cropping
water, what is the concentration of
division among plants. She asked (b) inter cropping the solution in terms of mass by
her supervisor to suggest the (c) mixed farming mass percentage?
specific plant hormone for the (d) hybrid farming
(a) 11.1% (b) 22.2%
same. Had you been her _ (b) Intercropping is a multiple (c) 10% (d) 20%
supervisor, which plant hormone cropping practice that involves
would you suggest? growing two or more crops in a _ (c) Mass of solute = 20 g
definite pattern simultaneously on the Mass of solvent (water) = 180 g
(a) Abscisic acid same field.
(b) Gibberellins ∴Mass of solution
(c) Cytokinin
97. When we heat lead nitrate = 20 + 180 = 200 g
(d) Auxin
[Pb(NO 3 ) 2 ] in a boiling tube, we Concentration in mass percentage
observe the emission of brown
_ (c) Cytokinins (CK) are a class of = × 100
Mass of the solute
fumes. Which one of the following
plant hormones that promote cell Mass of solution
division or cytokinesis, in plant roots is the brown gas ? 20
= × 100 = 10%
and shoots. They are involved (a) NO (b) N 2 O 2 200
primarily in cell growth and (c) NO 3 (d) NO 2
differentiation, but also affect apical 101. Which one of the following
dominance, axillary bud growth, and _ (d) When lead nitrate [Pb(NO 3 )2 ] is elements is a non-metal but is
leaf senescence. heated strongly it breaks down to lustrous ?
form simpler substances like lead
94. Which cell organelles have their monoxide (PbO), nitrogen dioxide
(a) Carbon (b) Silicon
own DNA and Ribosomes? (NO 2 ) and oxygen (O 2 ).In this (c) Germanium (d) Iodine
(a) Golgi body and Endoplasmic decomposition reaction the _ (d) Luster is basically due to free
Reticulum colourless compound lead nitrate electron. As we move down the
(b) Mitochondria and Plastids forms a yellow compound lead group, the size of the atom increases
monoxide and brown fumes of and more shell are added, so due to
(c) Lysosome and Golgi body
nitrogen dioxide gas are evolved. screening effect, effective nuclear
(d) Vacuole and Plastids charge is less and hence electrons
CDS Solved Paper 2021 (I) 81

are loosely bound. More over the _ (c) Fission wheather, it is binary or 108. Which one of the following laws
non-metallic property decreases on multiple is a mode of asexual
going from top to the bottom of the of electromagnetism does not give
reproduction in which a unicellular
group. Therefor iodine is Lustrous. organism like Amoeba, Bacteria, etc., the direction of magnetic field ?
divides into two or multiple progenies (a) Right-hand thumb rule
102. Which one of the following metals through cell division. It results in (b) Fleming’s left-hand rule
has both malleability and ductility transformation of heavier nuclei of (c) Fleming’s right-hand rule
properties? parent into smaller nuclei of
progenies.
(d) Faraday’s law of electromagnetic
(a) Na (b) Au induction
(c) Ce (d) Hg 105. Which one of the following is an
_ (d) Faraday’s law of induction is a
_ (b) Gold (Au) is the most malleable electric conductor? basic law of electromagnetism
of all metals. It can be drawn into a (a) A plastic sheet predicting how a magnetic field will
wire of single-atom width, and then interact with an electric circuit to
stretched considerably before it
(b) Distilled water
produce an electromotive force —a
breaks. (c) Human body phenomenon known as
(d) A wooden thin sheet electromagnetic induction.
103. We are given three copper wires
of different lengths and different _ (c) Flow of charges and ions 109. An antibiotic is not useful
constitute electric current. As our
areas of cross-section. Which one body cells contain various ions like
against a virus whereas a
of the following would have sodium ion, potassium ion, chloride vaccine is. Which one of the
highest resistivity? ion, etc. which have the tendency to following is the most appropriate
conduct electricity and this makes reason for this?
(a) Copper wire of 50 cm length and
our body good conductor of
1 mm diameter. electricity.
(a) An antibiotic can break RNA only,
(b) Copper wire of 25 cm length and whereas virus has DNA.
0·5 mm diameter. 106. A non-spherical shining spoon (b) An antibiotic is a carbohydrate in
(c) Copper wire of 10 cm length and can generally be considered as a its chemical nature, whereas a
2.0 mm diameter. (a) Spherical mirror vaccine is a protein which works
(d) All the wires would have same (b) Parabolic mirror well to kill a virus.
resistivity. (c) Plane mirror (c) Only a vaccine can break the
genetic material of a virus.
_ (d) The correct answer is All the wires (d) Lens
would have same resistivity. (d) A virus does not use biochemical
_ (b) The property of a parabolic mirror pathways which can be blocked
Resistivity of material depends on is that all reflected rays pass through
the nature of the material. a single point, the focus. On the other by an antibiotic. But a vaccine can
The resistivity of a particular material hand, the concave side of the spoon boost an immune system to fight
is measured in units of Ohm-Metres acts like a concave mirror. If the the virus.
(Ω m) which is also affected by spoon is held at arm’s length, the
object is farther from the reflecting
_ (d) Antibiotics block various
temperature. biochemical pathways of a
surface than the center of curvature. pathogen, i.e. bacteria to inhibit its
104. Which of the following statements growth and kill it in process, but virus
about ‘fission’ is correct? 107. Who amongst the following is a is a non-living particle which does
pioneer in discovering the heating not have these biochemical
1. It is related with the creation of
effect of electric current? pathways and use host cells to
new individuals by means of perform their replication activities.
cell division in unicellular (a) Isaac Newton (b) Galileo Galilei
So, they can’t be blocked by
organism. (c) James P. Joule (d) J.J. Thomson antibiotics. Vaccines are designed to
2. It is related with 'the _ (c) James Prescott Joule English induce a protective immune
physicist who established that the response in the body against the
transformation of heavier various forms of energy mechanical, specific viruses represented in the
nuclei into smaller nuclei. electrical and heat are basically the vaccine. When vaccinated the
3. It is related with the creation of same and can be changed from one immune system of the body
into another. produces a specific response,
a heavier nuclei by means of consisting of specific cells and
combining two higher nuclei. In 1840 that the amount of heat per
second that develops in a wire specific antibodies that fight off the
Select the correct answer using the carrying a current is proportional to infection when exposure to the virus
codes given below: the electrical resistance of the wire occurs at a later stage.
and the square of the current.
(a) Only 1 110. ‘Sleeping sickness’ is caused by
(b) Only 2 He determined that the heat evolved
per second is equivalent to the (a) Trypanosoma
(c) 1 and 2 (b) Leishmania
electric power absorbed or the
(d) 1 and 3 power loss. (c) Plasmodium
P = I 2R (d) Paramecium
82 CDS Solved Paper 2021 (I)

_ (a) African Trypanosomiasis, also This increases the percentage of 118. The direction of acceleration in
known as “sleeping sickness”, is carbon dioxide in air. Due to this, the
supply of air is cut off and fire is uniform circular motion is
caused by protozoan parasites of the
species Trypanosoma brucei. It is extinguished. along the
transmitted by the tse-tse fly, which is (a) direction of motion.
found only in sub-Saharan Africa. 115. Which one of the following (b) tangent to the circle at the point of
sodium compounds is used for observation.
111. Which one among the following is softening hard water?
a free living animal ? (c) direction of velocity.
(a) Na 2 CO 3 (d) direction perpendicular to velocity.
(a) Liver fluke (b) Wuchereria (b) NaHCO 3
(c) Plasmodium (d) Planaria _ (d) Uniform circular motion can be
(c) NaOH described as the motion of an object
_ (d) Planarian, any of a group of (d) Na 2 SO 4 in a circle at a constant speed. As an
widely distributed, mostly free-living object moves in a circle, it is
flatworms of the class Turbellaria. _ (a) The sodium compound which is constantly changing its direction.
Rest options are patasites. used for softening hard water is Accelerating objects are objects
called sodium carbonate which is which are changing their velocity -
112. Which one of the following is the most commonly known as washing either the speed (i.e., magnitude of
structure of a cardiac muscle cell ? soda (Na 2CO 3.10H 2O). It is prepared the velocity vector) or the direction.
by heating baking soda and then An object undergoing uniform circular
(a) Cylindrical, Unbranched and recrystallising the product so motion is moving with a constant
Multinucleate obtained to get washing soda. speed. Nonetheless, it is accelerating
(b) Spinal shaped, Unbranched and due to its change in direction. The
Uninucleate
116. Calcium oxide reacts vigorously
direction of the acceleration is
(c) Spinal shaped, Branched and with water to produce slaked lime inwards.
Uninucleate whose solution is used for
V
white-washing walls. This slaked V
(d) Cylindrical, Branched and
Uninucleate lime reacts with component (A) in F
air to form a thin layer of F F
_ (d) Cardiac muscles are branched, component (B) to give a shiny
cylindrical, Involuntary and
uninucleated. It constitutes the main finish. What are the components F
tissue that mades the wall of the (A) and (B)? V V
heart. They are uninucleate as each
(a) A—O 2 ; B—CcCO 3
cell has a single nucleus. Also, these 119. The weight of an object is due to
cells are not controlled by our will and (b) A—CO 2 ; B—Ca(OH) 2
hence said to be involuntary. (c) A—O 2 ; B—Ca(OH) 2 (a) the net force acting on it.
(d) A—CO 2 ; B—CaCO 3 (b) the total of all forces acting on it
113. Which one of the following irrespective of their directions.
naturally occurring acids is found _ (d) A solution of slaked lime (c) the force that it exerts on the
[Ca(OH) 2] is used to white wash the
in abundance in tomato ? ground.
walls. Calcium hydroxide reacts with
(a) Acetic acid the carbon dioxide ( A ) in air to form (d) its inert property.
than layer of calcium carbonate
(b) Citric acid _ (c) The weight of an object is nothing
(c) Oxalic acid [(CaCO3 ) (component B)] on the walls. but the gravitational force with which
It gives a shiny finish to the walls. it is attracted towards the center of
(d) Tartaric acid
the Earth, It also define as the
_ (b) Citric acid is the most abundant
117. How many internal reflections of magnitude of the reaction force
organic acid in tomatoes with some light take place in the formation of exerted on a body by mechanisms
malic acid also present. primary rainbow? that counteract the effects of gravity:
the weight is the quantity that is
114. Which one of the following is (a) 0 (b) 1
measured by, for example, a spring
used in soda-acid fire (c) 2 (d) More than 2 scale.
extinguishers _ (b) A rainbow is regarded as one of 120. The size of particles being studied
(a) Sodium chloride the most spectacular light shows
observed on the earth. A rainbow is a in ‘nano-technology’ is about
(b) Sodium hydrogen carbonate multicoloured arc formed by light (a) 1 Å — 10 nm (b) 1 − 100 nm
(c) Calcium hydroxide striking water droplets. Rainbows are
created after rain by the reflection,
(c) 1 − 50 µ (d) 1 mm − 10 mm
(d) Acetic acid
refraction, and dispersion of light in _ (b) (b) The correct answer is 1 − 100
_ (b) Soda-acid fire extinguisher water droplets. All of these events nm. Nanoparticle, ultrafine unit with
contains a solution of sodium produce a light spectrum in the sky,
hydrogen carbonate (NaHCO 3 ) and dimensions measured in nanometres
which is known as a rainbow.In (nm; 1 nm = 10 − 9 metre).
sulfuric acid (H 2SO 4 ). The following Primary Rainbow, one total internal
reaction takes place : Nanoparticles exist in the natural
reflection and two refractions of white world and are also created as a result
2NaHCO 3 + H 2SO 4 → light by water droplets produce the of human activities.
Na 2SO 4 + 2H 2O + 2CO 2 primary rainbow.
CDS
Combined Defence Service

SOLVED PAPER 2020 (II)


PAPER I Elementary Mathematics

1. x 3 + x 2 + 16 is exactly divisible 3. If the roots of the quadratic (a) an even number and it is not a square
by x, where x is a positive equation x 2 − 4 x − log10 N = 0 of an integer
(b) an odd number and it is not a square of
integer. The number of all such are real, then what is the
an integer.
possible values of x is minimum value of N? (c) square of an even integer.
(a) 3 (b) 4 1
(a) 1 (b) (d) square of an odd integer.
(c) 5 (d) 6 10
_ (d) We have, I = a + b + c
2 2 2
1 1
(c) (d)
_ (c) We have, x + x + 16 is exactly
3 2
100 10000 a, b are consecutive integer and c = ab
divisible by x.
∴ b=a+1
x 3 + x 2 + 16 16 _ (d) Given equation
∴ = x2 + x + 2
x − 4x − log10 N = 0 has real and c = a( a + 1) = a 2 + a
x x
roots ∴I = a 2 + ( a + 1)2 + ( a 2 + a )2
16
x + x +
2
is positive integer ∴( 4)2 + 4(log10 N ) ≥ 0
x I = a2 + a2 + 2 a + 1 + a4 + a2 + 2 a3
log10 N ≥ − 4 I = a 4 + 2 a 3 + 3a 2 + 2 a + 1
∴ x is factor of 16.
N ≥ (10)− 4 I = a4 + a2 + 1 + 2 a3 + 2 a2 + 2 a
i.e., 1, 2, 4, 8, 16
1
∴ Number of all possible value of x N≥ 4 I = ( a 2 + a + 1)2
10
is 5. a 2 + a + 1 is an odd integer.
1
∴Minimum value of N = ∴ I is a square of an odd integer.
2. The number of (a, b, c ), where 10000
a, b, c are positive integers such 4. The number of different 6. If the number 23P62971335 is
that abc = 30, is solutions of the equation divisible by the smallest odd
(a) 30 (b) 27 x + y + z = 12, where each of x , y composite number, then what is the
(c) 9 (d) 8 and z is a positive integer, is value of P?
(a) 4 (b) 5
_ (b) We have, (a) 53 (b) 54
(c) 6 (d) 7
abc = 30, where a, b, c are positive (c) 55 (d) 56
integer. _ (a) Given,
_ (c) We have,
∴30 = 1 × 1 × 30 = 3 ways 23P62971335 is divisible by smallest odd
x + y + z = 12, where x , y, z is a composite number.
30 = 1 × 2 × 15 = 6 ways positive integer.
Q Smallest odd composite number is 9.
30 = 1 × 3 × 10 = 6 ways ∴Total number of different solution
∴Given number is divisible by 9.
30 = 1 × 5 × 6 = 6 ways 11 × 10
= 12 − 1C 3 − 1 = 11C 2 = = 55 For divisibility by 9, sum of its digits is
30 = 2 × 3 × 5 = 6 ways 2
multiple of 9.
∴Total number of ( a, b, c )
5. If I = a 2 + b 2 + c 2 , where a and b ∴2 + 3 + P + 6 + 2 + 9
=3+ 6+ 6+ 6+ 6
are consecutive integers and + 7 + 1 + 3 + 3 + 5 = 41 + P
= 27 ways
c = ab, then I is ∴The possible value of P is 4.
4 CDS Solved Paper 2020 (II)

_ (a) x = ( x + a − a)
n n
7. What is the remainder when the 14. The sum of all possible products
sum 1 + 2 + 3 + 4 + 5 is
5 5 5 5 5
x n = ( x − a )n + nC1( x − a )n − 1 taken two at a time out of the
divided by 4? ( a ) + nC 2( x − a )n − 2 a 2 + … + a n numbers ± 1, ± 2, ± 3, ± 4, ± 5 is
(a) 0 (b) 1 (c) 2 (d) 3 ⇒ x n − a n = ( x − a) (a) 0 (b) − 30
[( x − a ) n−1
+ C1( x − a )
n n− 2
a + …] (c) − 55 (d) 55
_ (b) Given number,
1 +2 + 3 + 4 + 5
5 5 5 5 5 Clearly, x n − a n is divisible by x − a _ (c) Given number, ± 1, ± 2, ± 3, ± 4, ± 5
for every natural number n.
1 + 2 + ( 4 − 1) + 4 + ( 4 + 1)
5 5 5 5
We know that,
1 + 2 5 + 4 5 + 4m − 1 + 4n + 1 11. If 17 2020 is divided by 18, then ( x 1 + x 2 + x 3… x n )2
what is the remainder? n n
∑xi ∑ x ix j
where m and n are integer. = 2
+2
1 + 2 + 4 + 4m + 4n = 1 + 4k
5 5 (a) 1 (b) 2 (c) 16 (d) 17 i =1 1≤i < j≤n

2 5 + 4 5 + 4m + 4n is divisible by 4. _ (a) We have, ∴(− 1 + 1 − 2 + 2 + 3 − 3


∴Remainder = 1 (17 )2020 = (18 − 1)2020 + 4 − 4 + 5 − 5)2
= 18k + ( − 1)2020 = 18k + 1 = 2(1 + 2 + 3 + 4 2 + 5 2 ) + 2( Σx i x j ),
2 2 2
8. What is the digit in the unit place
∴If 17 2020 is divided by 18, 0 = 55 + Σx i x j
of 3 99 ?
then remainder = 1 Σx i x j = − 55
(a) 1 (b) 3
(c) 7 (d) 9 12. What is the value of 15. A train of length 110 m is moving at
1 1 1
_ (c) Given number + + a uniform speed of 132 km/h. The
3 99 = 3 3 ⋅ 3 96 = 27( 3 4 )24 1+ 2 2+ 3 3+ 4 time required to cross a bridge of
= 27( 81)24 1 length 165 m is
+…+ ?
= 27( 80 + 1)24 99 + 100 (a) 6.5 s (b) 7 s
= 27( 80k + 1) (a) 1 (b) 5
(c) 7.5 s (d) 8.5 s
= 27 × 80k + 27 (c) 9 (d) 10 _ (c) Length of train = 110 m
Q27 × 80k has unit digit is 0 Length of bridge = 165 m
_ (c) Given,
and unit digit of 27 is 7. 1 1 1 Total length = 110 + 165
+ +
∴Unit digit of 3 99 = 7 1+ 2 2 + 3 3+ 4 = 275 m
9. LCM of two numbers is 28 times 1 Speed of train = 132 km / h
+…+
their HCF. The sum of the HCF 99 + 100 = 132 ×
5
m/s
18
and the LCM is 1740. If one of 2 −1 3− 2 4− 3
= + + Time required to cross the bridge
these numbers is 240, then what 2 −1 3−2 4−3
Total distance
is the other number? 100 − 99 =
(a) 420 (b) 640 +…+ Speed
100 − 99
275 × 18
(c) 820 (d) 1040 =
= 2 − 1+ 3− 2 + 4− 3 132 × 5
_ (a) Let LCM of two number be x and +…+ 100 − 99
HCF of two number be y. = 7.5 s
= 100 − 1 = 10 − 1 = 9
Given, 16. The simple interest on a certain sum
x = 28 y … (i) 13. If x m = 14 x x x , then what is is one-fourth of the sum. If the
and x + y = 1740 … (ii) number of years and the rate of
the value of m?
From Eqs. (i) and (ii), we get 1 1 3 7 annual interest are numerically
(a) (b) (c) (d)
x = 1680 and y = 60 8 4 4 4 equal, then the number of years is
We know that, (a) 2.5 (b) 3
_ (a) We have, (c) 3.5 (d) 5
LCM × HCF
x m = 14 x x x 1
= Product of two numbers _ (d) Given, SI = P
1680 × 60 14 4
∴Other number = = 420 ⇒x m
= x x⋅x 1/ 2

240 and R =T
⇒ x m = 14 x ( x 3/ 2 )1/ 2 PRT
∴ SI =
10. ( x − a ) is divisible by ( x − a ),
n n
⇒ xm =
14
x ⋅ x 3/ 4 100
where x ≠ a, for every 1 1 PT 2
⇒ P=
(a) natural number n ⇒ x m = ( x 7/ 4 )14 4 100
1
(b) even natural number n only ⇒ T 2 = 25
⇒ xm = x 8
(c) odd natural number n only ∴ T =5
1
(d) prime number n only ⇒ m= Q Number of years = 5
8
CDS Solved Paper 2020 (II) 5

17. A 60-page book has n lines per _ (b) Here, d ( n), denotes the number _ (a) Speed of flow of water = 2 km/h
page. If the number of lines were of positive divisors of a positive Width of river = 40 m
integer n.
reduced by 3 in each page, the Depth of river = 3 m
number of pages would have to be ∴d( 5) = 2 , d(11) = 2 ,
Volume of water in one minutes
increased by 10 to give the same d ( 55) = d ( 5 × 11) = 2 × 2 = 4
1
d (16) = d (2 ) = ( 4 + 1) = 5
4 = 40 × 3 × 2000 ×
writing space. What is the value 60
of n? ∴d ( 5) = d (11) 1st is true.
= 4000 m3
(a) 18 (b) 21 d ( 5) d (11) = 2 × 2 = 4
= 4000000 L
(c) 24 (d) 30 = d( 55) 2nd is true.
d ( 5) + d (11) = 2 + 2 = 4 ≠ d (16) … 23. If a television set is sold at ` x, a loss
_ (b) Total number of line in 60 page
= 60n … (i) 3 rd is false. of 28% would be incurred. If it is sold
Total number of line when number of ∴1 and 2 are correct. at ` y, a profit of 12% would be
line were reduced 3 in each page incurred. What is the ratio of y to x ?
and number of pages increased by
20. If A n = Pn + 1, where Pn is the
(a) 41 : 9 (b) 31 : 9
10 = ( n − 3) (70) … (ii) product of the first n prime
(c) 23 : 9 (d) 14 : 9
From Eqs. (i) and (ii), numbers, then consider the
60n = ( n − 3) 70 following statements : _ (d) Let the cost of television be ` 100.
If television set sold at 28% loss,
60n = 70n − 210 1. A n is always a composite
then selling price of television
n = 21 number.
( x ) = 100 − 28 = 72
18. If x men working x hours per day 2. A n + 2is always an odd number. If television sold at 12% profit
can do x units of work in x days, 3. A n + 1 is always an even then selling price of television
then y men working y hours per number. i.e., ( y) = 100 + 12 = 112
day in y days would be able to do k Which of the above statements y 112
∴ =
units of work. What is the value of is/are correct? x 72
k? (a) only 1 (b) only 2 14
=
(a) x2 y− 3 (b) x3 y− 2 (c) only 3 (d) 2 and 3 9
(c) y2 x− 3 (d) y3 x− 2 _ (d) Given An = Pn + 1, where Pn is y : x = 14 : 9
the product of first n prime number 24. By increasing the speed of his car by
_ (d) Given,
x men working x hours per day can Pn is always an even number. 15 km/h, a person covers a distance
do x units of work in x days [Q P1 = 2 ]
of 300 km by taking an hour less
∴ x3 = x ∴ A n is an odd number. than before. What was the original
⇒ x =1
2
… (i) ∴ A n + 1 is always an even number. speed of the car?
y men working y hours per day in ∴ A n + 2 is always an add number. (a) 45 km/h
y day would be able to do k units of (b) 50 km/h
work.
21. A shopkeeper sells his articles at
their cost price but uses a faulty (c) 60 km/h
∴ y3 = k (d) 75 km/h
balance which reads 1000 gm for
y3
⇒ =1 …(ii) 800 gm. What is the actual profit _ (c) Let the speed of car = x km/h
k
percentage? Speed of car after increasing 15 km/h
From Eqs. (i) and (ii),
(a) 20% (b) 25% (c) 30% (d) 40% = ( x + 15) km/h
y3
x2 = ⇒ k = x − 2 y3 _ (b) Here, 200 gm is gained in Time taken by car on original speed
k 300
800 gm. =
x
19. Let d (n ) denote the number of ∴Profit % =
Profit
× 100
CP Time taken by car after increasing speed
positive divisors of a positive 300
200 =
integer n. Which of the following = × 100 = 25%
800 x + 15
are correct?
Difference between time = 1 h
1. d (5) = d (11) 22. A river 3 m deep and 40 m wide
300 300
is flowing at the rate of 2 km/h and ∴ − =1
2. d (5) ⋅ d (11) = d (55) x x + 15
falls into the sea. What is the
3. d (5) + d (11) = d (16) amount of water in litres that will ⇒ 300( x + 15 − x ) = x ( x + 15)
Select the correct answer using fall into the sea from this river in ⇒ x 2 + 15x − 4500 = 0
the code given below: a minute? ⇒ ( x + 75) ( x − 60) = 0
(a) 1 and 3 (b) 1 and 2 (a) 4000000 L (b) 400000 L x = 60 , x ≠ − 75
(c) 2 and 3 (d) 1, 2 and 3 (c) 40000 L (d) 4000 L ∴Original speed of car = 60 km / h
6 CDS Solved Paper 2020 (II)

25. Three persons start a business 27. Ena was born 4 yr after her Decrease at rate of x %

= N  1 +
1 1 1 x   x  x
with capitals in the ratio : : . parents marriage. Her mother is  − N 1 + 
 100   100  100
3 4 5 3 yr younger than her father and
 x2 
= N  1 +
The first person withdraws half 24 yr older than Ena, who is 13 yr x   x 
 1 −  = N 1 − 
his capital after 4 months. What old. At what age did Ena’s father  100   100   10000 
is his share of profit if the get married? Which is less than N.
business fetches an annual profit (a) 25 yr (b) 24 yr
of ` 96800? 30. If ab + xy − xb = 0 and
(c) 23 yr (d) 22 yr
x c
(a) ` 32000 (b) ` 34500 _ (c) Present age of Ena = 13 yr bc + yz − cy = 0, then what is +
(c) ` 36000 (d) ` 36800 a z
Present age of her mother equal to?
_ (a) Three person start a business = (13 + 24) = 37 yr y b
with capital in the ratio (a) (b) (c) 1 (d) 0
1 1 1 Age of Ena mother’s when Ena born b y
: : = 20 : 15 : 12
3 4 5 = 37 − 13 = 24 yr _ (c) Given, ab + xy − xb = 0 and
Total share of capital in a year by Age of Ena mother get married bc + yz − cy = 0
1st person is = 24 − 4 = 20 yr ⇒ x ( y − b ) = − ab and c( b − y) = − yz
= (20 × 4)x + (10 × 8)x = 160x ∴Age of Ena father’s x −b c − y y
⇒ = and = =
2nd person = (15 × 12 )x = 180x = (20 + 3) = 23 yr. a y−b z b− y y−b
3rd person = (12 × 12 )x = 144x −b y−b
28. Mahesh is 60 yr old. Ram is 5 yr ∴
x c
+ = +
y
= =1
∴Ratio of their profits a z y−b y−b y−b
= 160 : 180 : 144
younger to Mahesh and 4 yr elder
to Raju. Babu is a younger brother 31. What is the HCF of the polynomials
Profit share of 1st person
 160 
of Raju and he is 6 yr younger. x 6 − 3x 4 + 3x 2 − 1 and
=  × 96800 What is the age difference
 160 + 180 + 144  x 3 + 3x 2 + 3x + 1 ?
between Mahesh and Babu?
=
160
× 96800 = ` 32000 (a) 18 yr (b) 15 yr (c) 13 yr (d) 11 yr (a) ( x + 1) (b) ( x + 1)2
484 (c) x + 1 2
(d) ( x + 1)3
_ (b) Here, age of Mahesh = 60 yr
26. If x varies as y, then which of the Ram is 5 yr younger to Mahesh. _ (d) Given polynomial, x − 3x + 3x − 1
6 4 2

following is/are correct? ∴Age of Ram = 60 − 5 = 55 yr = ( x 2 − 1)3 = ( x + 1)3( x − 1)3


1. x 2 + y 2 varies as x 2 − y 2 Ram is 4 yr elder to Raju and x 3 + 3x 2 + 3x + 1 = ( x + 1)3

∴Raju’s age = 55 − 4 = 51 yr HCF = ( x + 1)3


x
2. 2 varies inversely as y
y Babu’s is 6 yr younger to Raju 32. The HCF and the LCM of two
∴Babu’s age = 51 − 6 = 45 polynomials are 3x + 1 and
3. n x 2 y varies as 2n x 4 y 2 ∴Difference of age between Mahesh 30x 3 + 7 x 2 − 10x − 3 respectively. If
Select the correct answer using and Babu = 60 − 45 = 15 yr one polynomial is 6x 2 + 5x + 1, then
the code given below : 29. The number of items in a booklet what is the other polynomial?
(a) 1 and 2 (b) 2 and 3 is N. In the first year there is an (a) 15 x2 + 4 x + 3 (b) 15 x2 + 4 x − 3
(c) only 3 (d) 1, 2 and 3 increase of x% in this number and (c) 15 x2 − 4 x + 3 (d) 15 x2 − 4 x − 3
in the subsequent year there is a
_ (b) Given x varies as y _ (d) HCF and LCM of two polynomials are
decrease of x%. At the end of the
∴ x = ky 3x + 1 and 30x + 7 x − 10x − 3
3 2
two year, what will be the
1. x + y = k y + y = y ( k + 1)
2 2 2 2 2 2 2
and one polynomial is 6x 2 + 5x + 1.
number of items in the booklet?
and x − y = k y − y = y ( k − 1)
2 2 2 2 2 2 2 We know,
(a) Less than N
∴x 2 + y 2 not varies with x 2 − y 2. HCF × LCM = Product of two polynomials
(b) Equal to N
x ky k ∴Other polynomial
2. 2 = 2 = (c) More than N
y y y ( 3x + 1) ( 30x 3 + 7 x 2 − 10x − 3)
(d) It depends on the value of N =
x 6x 2 + 5x + 1
∴ Clearly, varies inversely as y.
2 _ (a) Number of items in a booklet = N
y ( 3x + 1) ( 30x 3 + 7 x 2 − 10x − 3)
1 Number of items in booklet after first =
( 3x + 1) (2 x + 1)
3. n x 2 y = ( x 2 y)n = ( k 2 y 3 )1/ n year
1 Increase at rate of x % = N + x % of N 30x 3 + 7 x 2 − 10x − 3
=
2n
x y =
4 2
( x 4 y 2 )2n 2x + 1
= N  1 +
and x 

1  100  (15x 2 − 4x − 3) (2 x + 1)
= ( x 2 y)n = ( k 2 y 3 )1/ n =
Number of items in booklet in 2x + 1
Q n x 2 y is varies as 2n x 4 y 2 . 2nd year = 15x 2 − 4x − 3
CDS Solved Paper 2020 (II) 7

33. If ( p + 2) (2q − 1) = 2pq − 10 and 35. What is the square root of 38. X , Y and Z travel from the same place
( p − 2) (2q − 1) = 2pq − 10, then 4 x 4 + 8x 3 − 4 x + 1? with uniform speeds 4 km/h, 5 km/h
what is pq equal to? (a) 2 x2 − 2 x − 1 and 6 km/h respectively. Y starts 2 h
(a) − 10 (b) − 5 (b) 2 x2 − x − 1
after X . How long after Y must Z
(c) 5 (d) 10 start in order that they overtake X at
(c) 2 x2 + 2 x + 1 the same instant?
_ (c) Given, (d) 2 x2 + 2 x − 1 3 4
( p + 2 ) (2q − 1) = 2 pq − 10 (a) h (b) h
_ (d) We have, 2 3
and ( p − 2 ) (2q − 1) = 2 pq − 10 9 11
∴2 pq + 4q − p − 2 = 2 pq − 10 4x 4 + 8x 3 − 4x + 1 (c) h (d) h
= 4x 4 + 4x 2 + 8x 3 − 4x + 1 − 4x 2 8 8
⇒ 4q − p = − 8 … (i)
= 4x 4 + 4x 2 + 1 + 2(2 x 2 )(2 x ) _ (b) Distance travelled by X in t hours = 4t
and 2 pq − 4q − p + 2 = 2 pq − 10
− 2(2 x ) − 2(2 x 2 ) Distance travelled by Y starts 2 h after X
− 4q − p = − 12
= (2 x 2 )2 + (2 x )2 + (1)2 + 2(2 x 2 ) (2 x ) = 5(t − 2 )
4q + p = 12
…(ii)
− 2(2 x ) (1) − 2(2 x 2 ) (1) Distance travelled by X and Y are same.
1
From Eqs. (i) and (ii), p = 10, q = = (2 x 2 + 2 x − 1)2 ∴ 4 t = 5 t − 10
2
1 ∴ 4x 4 + 8x 3 − 4x + 1 ⇒ t = 10 h
∴ pq = 10 × = 5
2 Let Z starts n hours after Y.
= (2 x 2 + 2 x − 1)2
∴ Distance travelled by
34. What is the value of = 2x 2 + 2x − 1 Z = 6 ( 8 − n) = 48 − 6 n
a 2 + ac a2 − c 2
− 36. The sum of the digits of a two Distance travelled by X and Z are same.
a 2 c − c 3 a 2 c + 2ac 2 + c 3 digit number is 13 and the ∴ 40 = 48 − 6 n
2c 3 4
− 2 + ? difference between the number ⇒ 6n = 8 ⇒ n = h
a −c 2 a + c and that formed by reversing the 3

(a) 0 (b) 1
digits is 27. What is the product 39. 1 − x − x n + x n + 1 , where n is a
ac 6 of the digits of the number?
(c) (d) natural number, is divisible by
a2 + c 2 a+c (a) 35 (b) 40
(a) (1 + x)2 (b) (1 − x)2
(c) 45 (d) 54
_ (d) We have, (c) 1 − 2 x − x 2
(d) 1 + 2 x − x2
a + ac
2
a −c
2 2 _ (b) Two digits number be 10x + y.
− Given, x + y = 13 … (i) _ (b) Given,
a 2c − c 3 a 2c + 2 ac 2 + c 3
1 − x − xn + xn +1
and (10x + y) − (10 y + x ) = 27
2c 3 = 1 − xn − x + xn +1
− 2 + x − y=3 …(ii)
a − c2 a+c
= 11
( − x n ) − x (1 − x n ) = (1 − x ) (1 − x n )
From Eqs. (i) and (ii),
a( a + c ) (a + c ) (a − c ) = (1 − x ) (1 − x ) (1 + x + x 2 …+ x n − 1 )
= − x = 8, y = 5
c( a + c ) ( a − c ) c( a + c ) ( a + c )
= (1 − x )2(1 + x + x 2 + …+ x n − 1 )
2c 3 ∴ xy = 8 × 5 = 40
− + ∴(1 − x )2 is a factor of1 − x − x n + x n − 1.
(a + c ) (a − c ) a + c x y z
37. If = = , then 40. A person sold an article for ` 75
a a−c b +c c +a b −a
= −
c( a − c ) c( a + c ) which cost him ` x. He finds that he
which one of the following is
2c 3 realised x% profit on his outlay. What
− + correct?
(a + c ) (a − c ) a + c is x equal to?
(a) x + y+ z=0
a( a + c ) − ( a − c ) ( a − c ) (a) 20% (b) 25%
(b) x − y− z=0
− 2c 2 + 3c( a − c ) (c) 50% (d) 100%
= (c) x + y− z=0
c( a + c ) ( a − c )
(d) x + 2 y + 3z = 0 _ (c) Given cost price of articles = ` x
a 2 + ac − a 2 − c 2 Profit % = x %
x y z
+ 2 ac − 2c 2 + 3ac − 3c 2 _ (b) Let = = =k
= b+ c c + a b−c Selling price = ` 75
c( a + c ) ( a − c )
Profit
∴ x = k( b + c ) Profit % = × 100 %
6ac − 6 c 2 CP
= y = k(c + a )
c( a + c ) ( a − c ) 75 − x
z = k( b − a ) x%= × 100 %
6c ( a − c ) x
= x − y− z=k
c( a + c ) ( a − c ) ⇒ x 2 + 100x − 7500 = 0
( b + c − c − a − b + a) = 0 ⇒ ( x + 150) ( x − 50) = 0
6
= ∴ x − y− z=0
a+c x = 50
8 CDS Solved Paper 2020 (II)

3 − 4x x +5
41. A car did a journey in t hours. _ (c) Given, 6 4 =8 _ (b) We have, cosec θ − sinθ = m and
Had the average speed been x 6 ⋅ 4 ⋅ 45
3 x sec θ − cos θ = n
=2 3
km/h greater, the journey would 64 x 1 − sin2 θ
=m
have taken y hours less. How 2 ⋅ 3 ⋅ 2 2x ⋅ 2 10
3 3 sinθ
⇒ =8
long was the journey? 2 4 x ⋅ 34 x 1 − cos 2 θ
and =n
(a) x(t − y) t y (b) x(t − y) t y −1 ⇒ 3 4 x ⋅ 2 2x = 2 10 ⋅ 3 3 cos θ

(c) x(t − y) t y −2
(d) x(t + y) t y Taking log both sides, cos 2 θ sin2 θ
= m and =n
4x log 3 + 2 x log 2 sinθ cos θ
_ (b) Let the speed of car = S km / h m 4 / 3n 2/ 3 + m 2/ 3n 4 / 3
= 10 log 2 + 3 log 3
Distance travelled by car = D (2 x − 10) log 3 0.477
= = = m2/ 3n2/ 3[m2/ 3 + n2/ 3 ]
Time taken = t hour 3 − 4x log 2 0.301 2/ 3 2/ 3
 cos 2 θ   sin2 θ 
∴ S =
D
… (i) 2 x − 10    
t ⇒ = 158
.  sinθ   cos θ 
3 − 4x
When speed of car increased  cos 4 / 3 θ sin4 / 3 θ 
 + 
⇒ 2 x − 10 = ( 3 − 4x )(1. 58)
 sin θ cos 2/ 3 θ 
x km/h. 2/ 3

∴Speed of car = (S + x ) ⇒ 2 x − 10 = 474


. − 6.32 x
 cos 2 θ + sin2 θ 
Distance be same = D ⇒ 8.32 x = 1474
. = (sinθ cos θ)2/ 3  2/ 3 
1474
.  (sinθ cos θ) 
Time = (t − y) ⇒ x = = 177
.
D 8.32 =1
∴S + x = … (ii)
t − y ∴ 1<x<2
47. If cos θ + sec θ = k, then what is the
From Eqs. (i) and (ii), 44. The Euclidean algorithm is used value of sin 2 θ − tan 2 θ?
x =
D

D to calculate the
t − y t (a) 4 − k (b) 4 − k 2
(a) square root of an integer
D(t − t + y) (b) cube root of an integer (c) k 2 − 4 (d) k 2 + 2
x =
t (t − y) (c) square of an integer _ (b) Given, cos θ + sec θ = k
−1
D = x (t − y) ty (d) HCF of two integers (cos θ + sec θ)2 = k 2
42. When a ball is allowed to fall, the _ (d) Euclidean algorithm is cos θ + sec 2 θ + 2 = k 2
2

used to calculate the HCF of two


time it takes to fall any distance 1 − sin2 θ + 1 + tan2 θ + 2 = k 2
integers.
varies as the square root of the 4 − k 2 = sin2 θ − tan2 θ
distance and it takes 4 s to fall 45. If radius of a sphere is rational, ∴ sin θ − tan2 θ = 4 − k 2
2

78.40 m. How long would it take then which of the following


is/are correct? 48. ABC is a triangle inscribed in a
to fall 122.50 m?
semicircle of diameter AB.
(a) 5 s (b) 5.5 s (c) 6 s (d) 6.5 s 1. Its surface area is rational.
What is cos( A + B ) + sin( A + B )
_ (a) Given, t = k d 2. Its volume is rational. equal to?
When, t = 4 and d = 78.40 Select the correct answer using 1
(a) 0 (b)
4 the code given below 4
∴ k= 1
78.40 (a) only 1 (c) (d) 1
2
When, d = 122.50 (b) only 2
4 (c) Both 1 and 2 _ (d) We have,
∴ t = × 122.50
78.40 (d) Neither 1 nor 2 ABC is a triangle inscribed in a
semi-circle of diameter AB.
122.50 _ (d) Radius of sphere is rational
=4 ∴∠ABC is a right angle at C.
78.40 Surface area of sphere = 4 πr 2, which
1225 is irrational C
t =4×
784 4
Volume of sphere = πr 3, which is
35 3
=4× = 5 sec also irrational.
28

43. If 6 3 − 4 x 4 x + 5 = 8 (Given 46. If cosec θ − sin θ = m and A B

log10 2 = 0301
. and log10 3 = 0.477), sec θ − cos θ = n, then what is ∴ A + B = 90º
4 2 2 4
then which one of the following cos( A + B) + sin( A + B)
is correct? m 3n 3 + m 3n 3 equal to? = cos 90º + sin 90º
(a) 0 < x < 1 (b) 1 < x < 2 (a) 0 (b) 1 =0+1
(c) 2 < x < 3 (d) 3 < x < 4 (c) mn (d) m2 n2 =1
CDS Solved Paper 2020 (II) 9

49. Consider the following 52. Consider the following statements : 55. If sin θ + cos θ = 2, then what is
statements : 1. The equation sin 6 θ + cos 6 θ + 6 sin 2 θ cos 2 θ equal
1. sin θ = x +
1
is possible for 2 sin 2 θ − cos θ + 4 = 0 is possible to?
x for all θ. 1 3
(a) (b)
some real value of x. 4 4
2. tan θ + cot θ cannot be less than 7
1 π
2. cos θ = x + is possible for 2, where 0 < θ < .
(c) 1 (d)
x 4
2
some real value of x. _ (d) Given,
Which of the above statements sinθ + cos θ = 2
Which of the above statements
is/are correct? It is possible only θ = 45º
is/are correct?
(a) only 1 ∴sin6 θ + cos 6 θ + 6 sin2 θ cos 2 θ
(a) only 1 (b) only 2
(b) only 2
(c) Both 1 and 2 (d) Neither 1 nor 2 = sin6 45º + cos 6 45º +6 sin2 45ºcos 2 45º
(c) Both 1 and 2 6 6 2 2
_ (d) We have,  1   1   1   1 
(d) Neither 1 nor 2 =  +   + 6   
1  2  2  2  2
1. sinθ = x +
x _ (b) We have, 1 1 1 1
1. 2 sin2 θ − cos θ + 4 = 0 = + + 6× ×
sinθ ∈ [− 1, 1] 8 8 2 2
1 Put θ = 90º 1 + 1 + 12 14
x + ∈ ( − ∞, − 2 ] ∪ [2, ∞ ] = = =7/4
x 2 × 1− 0 + 4 = 6 ≠ 0 8 8
∴ 1st is incorrect. Q 1st is incorrect.
56. What is the least value of
1 2. tanθ + cot θ
2. cosθ = x +
1
9 sin 2 θ + 16 cos 2 θ?
x tanθ + ≥2
cos θ ∈ [− 1, 1] tanθ (a) 0 (b) 9
1 ∴ 2nd is correct. (c) 16 (d) 25
x + ∈ [− ∞, − 2 ] ∪ [2, ∞ )
x
53. A road curve is to be laid out on _ (b) We have,
∴2nd is also incorrect. 9 sin2 θ + 16 cos 2 θ
a circle. What radius should be
50. What is the magnitude (in used if the track is to change = 9 sin2 θ + 9 cos 2 θ + 7 cos 2 θ
radian) of the interior angle of a direction by 42º in distance of 44 = 9(sin2 θ + cos 2 θ) + 7 cos 2 θ
regular pentagon? m? = 9 + 7 cos 2 θ
π 2π 3π 4π 22 ∴0 ≤ cos 2 θ ≤ 1
(a) (b) (c) (d) (Assume π = )
5 5 5 5 7 ∴Minimum value of 9 + 7 cos 2 θ = 9
_ (c) Interior angle of a regular (a) 60 m (b) 66 m 57. If cos 47 ° + sin 47º = k, then what is
pentagon
(c) 75 m (d) 80 m
=
(5 − 2 )
× π=
3π the value of cos 2 47º − sin 2 47 °?
5 5 _ (a) Given, θ = 42 º
(a) k 2 − k 2 (b) − k 2 − k 2
Length of an arc = 44
51. The difference between two θ (c) k 1 − k 2 (d) − k 1 − k 2
angles is 15º and the sum of the We know, l= × 2 πr
360
5π 360º × l _ (a) Given,
angles in radian is . The bigger ∴r=
12 θ × 2π cos 47 ° + sin 47 º = k
angle is k times the smaller angle. 360 × 44 × 7 ⇒ cos 2 47 ° + sin2 47 º
=  π = 22 
What is k equal to? 42 × 2 × 22  7  + 2 sin 47 ºcos 47 º = k 2
4 3 6 7
(a) (b) (c) (d) r = 60 m ⇒ sin2( 47 º ) = k 2 − 1
3 2 5 6
⇒ sin2(2θ) = ( k 2 − 1)2 [let 47 ° = θ]
_ (b) Let the angle be x and kx. 54. What is the maximum value of
⇒ 1 − cos 2 2θ = ( k 2 − 1)2
∴ kx − x = 15º …(i) 3 sin θ − 4?
⇒ cos 2 2θ = 1 − ( k 2 − 1)2
5π (a) − 4 (b) − 1 (c) 0 (d) 1
and kx + x =
12 ⇒ cos 2θ = 1 − ( k 4 − 2 k 2 + 1)
_ (b) We know that,
⇒ x ( k + 1) = 75º …(ii) − 1 ≤ sinθ ≤ 1 ⇒ cos2θ = 2 k 2 − k 4
From Eqs. (i) and (ii),
− 3 ≤ 3 sinθ ≤ 3
k − 1 15 = k 2 − k2
= − 3 − 4 ≤ 3 sinθ − 4 ≤ 3 − 4
k + 1 75
cos 2 θ − sin2 θ = k 2 − k 2
− 7 ≤ 3 sinθ − 4 ≤ − 1
⇒ 5k − 5 = k + 1
∴Maximum value of 3 sinθ − 4 = − 1 ∴cos 2 47 º − sin2 47 º = k 2 − k 2
⇒ 4k = 6 ⇒ k = 3 / 2
10 CDS Solved Paper 2020 (II)

58. If cosec θ − sin θ = p 3 and 61. In a quadrilateral ABCD, ∠B = 90º _ (c) Here, ABCD is cyclic quadrilateral,
the bisector of the angles A, B, C and
sec θ − cos θ = q , then what is
3
andAB + BC + CD − AD = 0 ,
2 2 2 2
D cut the circle at P, Q, R and S
the value of tan θ ? then what is ∠ACD equal to? respectively.
p q (a) 30º (b) 60º D P
(a) (b) (c) pq (d) p2q 2
q p (c) 90º (d) 120º Q C
_ (b) We have, _ (c) Given, in quadrilateral ABCD
cosec θ − sinθ = p3 and ∠B = 90º
sec θ − cos θ = q 3 and AB2 + BC 2 + CD 2 − AD 2 = 0
A S
1 − sin2 θ 1 − cos 2 θ In ∆ABC, ∠B = 90º
= p3 and = q3 R B
sinθ cos θ D
cos θ2
sin θ 2
∴PQRS is also cyclic quadrilateral
= p3 and = q3 C
sinθ cos θ ∴∠PQR + ∠RSP = 180º
cos 2 θ cos θ p3
∴ × = 3 64. ABC is an equilateral triangle. The
sinθ sin θ q
2
side BC is trisected at D such that
p3 p BC = 3 BD. What is the ratio of AD 2
⇒ (cot θ) = 3 ⇒ cot θ =
3
q q A B
to AB 2 ?
∴ tanθ =
q ∴ AB + BC = AC
2 2 2
(a) 7 : 9 (b) 1 : 3
p
∴ AB2 + BC 2 + CD 2 − AD 2 = 0 (c) 5 : 7 (d) 1 : 2
59. If 0 ≤ α, β ≤ 90º such that ⇒ AC 2 + CD 2 = AD 2
_ (a) Here, ABC is an equilateral triangle.
cos(α − β ) = 1, then what is ∴∠C = 90º ∴ AB = BC = AC
sin α − sin β + cos α − cos β equal [By converse of Pythagoras and BC = 3BD
to? theorem]
Draw an altitude AE on BC
(a) − 1 (b) 0 (c) 1 (d) 2 ∴∠ACD = 90º
∴BE = EC
_ (b) Given, 62. In a ∆ABC, AC = 12 cm,
A
cos(α − β ) = 1 , where 0 ≤ α, β ≤ 90º AB = 16 cm and AD is the
∴α − β = 0, α = β bisector of ∠A. If BD = 4 cm,
∴sinα − sinβ + cos α − cos β then what is DC equal to?
= sinα − sinα + cos α − cos α = 0 (a) 2 cm (b) 3 cm
60. Consider the following (c) 4 cm (d) 5 cm B D E C
statements. _ (b) Given, in ∆ABC
In ∆ABE,
1. The value of cos 61º + sin 29º AC = 12 cm, AB = 16 cm
AE 2 + BE 2 = AB2
cannot exceed 1. BD = 4, AD is the bisector of ∠A.
AE 2 = AB2 − BE 2 …(i)
2. The value of tan 23º − cot 67º is By angle bisector,
In ∆ADE,
less than 0. A
AE 2 = AD 2 − DE 2 …(ii)
Which of the above statements
From Eqs. (i) and (ii),
is/are correct?
16 12 AB2 − BE 2 = AD 2 − DE 2
(a) only 1 (b) only 2
(c) Both 1 and 2 (d) Neither 1 nor 2 AB2 − AD 2 = BE 2 − DE 2
B D C = ( BE + DE ) ( BE − DE )
_ (a) We have,
1. cos 61º + sin29º AB BD ⇒ AB2 − AD 2 = CD ⋅ BD
=
= cos( 90º − 29º ) + sin29º AC CD [Q BE = CE]
= sin29º + sin29º = 2 sin29º 16 4
⇒ = ∴ BE + DE = CD
12 CD
Qsin29º < sin 30º 2 BC BC
∴ CD = 3 ⇒ AB2 − AD 2 = ×
⇒ 2 sin29º < 2 sin 30º 3 3
1
⇒ 2 sin29º < 2 × < 1 63. ABCD is a cyclic quadrilateral. 2 BC 2
⇒ AB2 − AD 2 =
2 The bisectors of the angles 9
1 st is correct. A, B, C and D cut the circle at ⇒ 9 AB2 − 9 AD 2 = 2 AB2
2. tan23º − cot 67 º P, Q, R and S respectively. What is ⇒ 7 AB2 = 9 AD 2
= tan23º − cot( 90º − 23º ) ∠PQR + ∠RSP equal to? AD 2 7
⇒ =
= tan23° − tan23° = 0 (a) 90º (b) 135º AB2 9
∴ 2nd is incorrect. (c) 180º (d) 270º ∴ AD 2 : AB2 = 7 : 9
CDS Solved Paper 2020 (II) 11

65. Consider the following statements : _ (a) Here, H, C and V are respectively 69. The diagonals of a rhombus differ by
the height, curved surface area and 2 units and its perimeter exceeds the
1. The diagonals of a trapezium volume of a cone. sum of the diagonals by 6 units.
divide each other What is the area of the rhombus?
∴3 πVH 3 + 9V 2
proportionally. 2 (a) 48 sq units (b) 36 sq units
= 3 π  πr 2H  H 3 + 9 πr 2H 
1 1
2. Any line drawn parallel to the 3  3  (c) 24 sq units (d) 12 sq units
parallel sides of a trapezium
Q V = 1 πr 2H  _ (c) Let the diagonal of rhombus.
divides the non-parallel sides  
3 AC = 2 x and BD = 2 y
proportionally.
= π 2r 2H 4 + π 2r 4 H 2 D C
Which of the above statements
= π 2r 2H 2[r 2 + H 2 ] = H 2 π 2r 2l 2
is/are correct? y x
= H 2( πrl )2 = H 2C 2 [Q C = πrl ]
(a) only 1 (b) only 2 O
= C 2H 2 x y
(c) Both 1 and 2 (d) Neither 1 nor 2
_ (c) 1. Diagonals of a trapezium 67. How many solid lead balls each A B
divide each other proportionally are of diameter 2 mm can be made ∴OA = OC = x , OB = OD = y
true from a solid lead ball of radius ∴ AC − BD = 2
D C 8 cm? ⇒ x − y=1 …(i)
(a) 512 (b) 1024 DC 2 = OC 2 + OD 2 = x 2 + y 2
P Q
O (c) 256000 (d) 512000 Given perimetre of rhombus exceeds the
sum of diagonal by 6 units.
A B _ (d) Here, R = 8 cm, r = 01
. cm
∴4 DC = AC + BD + 6
Volume of sphere whose radius is
In trapezium ABCD 4
8 cm, V1 = π( 8)3 ⇒ 4 x 2 + y2 = 2 x + 2 y + 6
AB is parallel to CD 3
∴ In ∆AOB and ∆COD ⇒ 2 x 2 + y 2 = x + y + 3 ⇒ 4( x 2 + y 2 )
Volume of sphere whose radius is
∠COD = ∠AOB 4
(0.1) cm, V2 = π( 01
. )3 = x 2 + y 2 + 9 + 6x + 6 y + 2 xy
[Vertically opposite angles] 3 ⇒ 3( x 2 + y 2 ) = 6( x + y) + 9 + 2 xy
∠OCD = ∠OAB Q V1 = nV2 ⇒ 3[( x − y)2 + 2 xy] − 2 xy = 6( x + y) + 9
4 4
[Alternate interior opposite angles] ⇒ π( 8)3 = n × π( 01
. )3 ⇒ 3 + 4xy = 6( x + y) + 9 [Q x − y = 1]
3 3
∴ ∆AOB ~ ∆ COD ⇒ 2 xy = 3x + 3 y + 3 … (ii)
⇒ n = 512 × 1000
AO OB
= From Eqs. (i) and (ii), we get x = 4, y = 3
∴ n = 512000
CO OD 1
Area of rhombus = (2 x ) (2 y) = 2 xy
∴Diagonals of trapezium divide each 68. The two sides of a triangle are 2
other proportionally. 40 cm and 41 cm. If the perimeter = 2 × 4 × 3 = 24 sq units
2. In ∆DAB of the triangle is 90 cm, what is
PO is parallel to AB.
70. What is the area of a right-angled
its area? triangle, if the radius of the
DP DO
∴ = … (i) [By B.P.T] (a) 90 cm2 (b) 135 cm2 circumcircle is 5 cm and altitude
PA OB
(c) 150 cm2 (d) 180 cm2 drawn to the hypotenuse is 4 cm?
In ∆BCD,
(a) 20 cm2 (b) 18 cm2 (c) 16 cm2 (d) 10 cm2
OQ is parallel to DC. _ (d) Here, sides of triangle are 40 cm,
DO CQ 41 cm and perimetre of triangle is 90 _ (a) ABC is a right angle triangle.
∴ = …(ii) cm.
OB QB BC is hypotenuse.
∴Third sides of triangle
[By B.P.T] = 90 − ( 40 + 41) = 9 C
From Eqs. (i) and (ii), ∴40 2 + 9 2 = 1600 + 81= 1681 = 412
DP CQ
= C O
PA QB D
∴ Any line drawn parallel to the 4
41
parallel sides of a trapezium divides 9
non-parallel sides proportionally. A B

66. If H , C and V are respectively the Given, OB = OC = r = 5 cm


A 40 B ∴ BC = 2 r = 10 cm
height, curved surface area and
volume of a cone, then what is ∴∆ABC is a right angle. Altitude AD = 4 cm
1
3πVH 3 + 9V 2 equal to? Area of ∆ABC = × AB × BC 1
∴Area of ∆ABC = × AD × BC
2 2
(a) C 2 H2 (b) 2 C 2 H2 1 1
= × 40 × 9 = 180 cm2 = × 4 × 10 = 20 cm2
(c) 5 C 2 H2 (d) 7C 2 H2 2 2
12 CDS Solved Paper 2020 (II)

71. In a triangle, values of all the 73. The surface areas of two spheres _ (a) Interior angle of regular polygon
angles are integers (in degree are in the ratio 1 : 4. What is the n − 2
whose side n is   π.
measure). Which one of the ratio of their volumes?  n 
following cannot be the proportion (a) 1 : 16 (b) 1 : 12 Exterior angle of regular polygon whose
of their measures? (c) 1 : 10 (d) 1 : 8  n − 2 2
side n is π  1 −  = π 
(a) 1 : 2 : 3 (b) 3 : 4 : 5 _ (d) Let the surface areas of two  n   n
spheres is s1 and s 2 and their radii is n − 2
(c) 5 : 6 : 7 (d) 6 : 7 : 8
∴ 2 π
r1 and r2 respectively.   π = 2 
 n   n 
_ (d) In triangle, angle of triangle are ∴s1 = 4 πr12, s 2 = 4 πr22
integers. ⇒ n−2 = 4
r2
∴Sum of angle of triangle is 180º.
s1
= 12 ⇒ n=6
s 2 r2
(a) x + 2 x + 3x = 180º Hence, the polygon is hexagon.
2
6x = 180º 1  r1 
⇒ =  [Q s1 : s 2 = 1 : 4] 76. A bicycle wheel makes 5000
⇒ x = 30º is integer 4  r2 
revolutions in moving 11 km. What
(b) 3x + 4x + 5x = 180º r1 1 V 4 / 3 πr13
⇒ = ∴ 1 = is the radius of the wheel? (Assume
12 x = 180º r2 2 V2 4 / 3 πr23 22
⇒ x = 15º is integer 3 3
π= )
r 
=  1  =  
1 7
(c) 5x + 6x + 7 x = 180º 2
 r2  (a) 17.5 cm (b) 35 cm
18x = 180º 1 (c) 70 cm (d) 140 cm
=
⇒ x = 10º is integer 8
_ (b) We have,
(d) 6x + 7 x + 8x = 180º ∴ V1 : V2 = 1 : 8 A bicycle wheel makes 5000 revolution in
21x = 180º moving 11 km.
180º
74. The length, breadth and height of
x = is not integer ∴In one revolutions distance covered by
21
a brick are 20 cm, 15 cm and 10 11
cm respectively. The number of wheel of bicycle is km
∴Only option (d) is not integer. 5000
bricks required to construct a 11000 11
72. The length of a rectangle is wall with dimensions 45 m = = m
5000 5
increased by 10% and breadth is length, 0.15 m breadth and 3 m Radius of wheel of bicycle = r cm
decreased by 10%. Then, the area height is 11
of the new rectangle is ∴2 πr = × 100 cm
(a) 12450 (b) 11250 5
(a) neither increased nor decreased (c) 6750 (d) None of these 11 × 20 × 7
r=
(b) increased by 1%
_ (c) Given, l = 20 cm, b = 15 cm 2 × 22
(c) decreased by 1% = 35 cm
h = 10 cm
(d) decreased by 10%
Volume of one brick 77. The volumes of two cones are in the
_ (c) Let the length of rectangle = x m = 20 × 15 × 10 = 3000 cm3
ratio 1 : 4 and their diameters are in
and breadth of rectangle = y m Dimension of walls is 45m, 0.15 m the ratio 4 : 5. What is the ratio of
∴Area of rectangle = xy and 3 m,
their heights?
When length is increased by 10% ∴Volume of wall
= ( 45 × 015
. × 3 ) m3 (a) 25 : 64 (b) 16 : 25
110x
∴New length = = (2025
. ) m = 2025
3
. × 10 cm 6 3 (c) 9 : 16 (d) 5 : 9
100 V 1 r 4
When breadth is decreased by 10%
Number of bricks _ (a) Given, 1 = and 1 =
V2 4 r2 5
Volume of wall
90 y =
∴New breadth = 1 2
100
Volume of one brick πr1 h1
3 1
. × 10 6
2025 ∴ =
Area of new rectangle = 1 2
πr2 h2 4
9900 99 3000 3
= xy = xy
10000 100 2025 × 10 4 2
= = 6750 r  h 1
99xy 1 3 × 10 3 ⇒  1 × 1 =
Difference = xy − = xy  r2  h2 4
100 100
75. If the sum of all interior angles of 2
⇒   × 1 =
Percentage decrease in area of new 4 h 1
xy / 100 a regular polygon is twice the  5 h2 4
rectangle = × 100%
xy sum of all its exterior angles, 2
h1  5  1 25
= 1% then the polygon is ⇒ =  × =
h2  4  4 64
∴Area of new rectangle is decreased (a) Hexagon (b) Octagon
∴ h1 : h2 = 25 : 64
by 1%. (c) Nonagon (d) Decagon
CDS Solved Paper 2020 (II) 13

78. In a triangle ABC, if A 84. Areas of two squares are in the ratio
2 ∠A = 3 ∠B = 6 ∠C, then what is m 2 : n 4 . What is the ratio of their
70º
∠A + ∠C equal to? perimeters?
(a) 90º (b) 120º M N (a) m : n (b) n : m
(c) 135º (d) 150º (c) m : n2 (d) m2 : n
O
_ (b) Given, in triangle ABC _ (c) Given, ratio of area of two squares is
2 ∠A = 3 ∠B = 6 ∠C m2 : n 4 .
Q ∠A + ∠B + ∠C = 180º B C A1 m2 x 2 m2 x m
= 4 ⇒ 12 = 4 ⇒ 1 = 2
3 1 Circle touches sides AB and AC at
⇒ ∠B + ∠B + ∠B = 180º A2 n x2 n x2 n
2 2 M and N respectively. Ratio of perimeter of square
⇒ 3∠B = 180º ⇒ ∠B = 60º ∴MA and NA is tangent of circle = 4x 1 : 4x 2 = x 1 : x 2 = m : n 2
∴∠A + ∠C = 180º − ∠B ∴ ∠OMA = ∠ONA = 90º 85. AD is the median of the triangle
= 180º − 60º = 120º. ∴∠A + ∠MON = 180º ABC. If P is any point on AD, then
79. If the perimeter of a circle and a 70º + ∠MON = 180º which one of the following is
square are equal, then what is the ∠MON = 110º correct?
ratio of the area of the circle to 82. The sum of the squares of sides (a) Area of triangle PAB is greater than the
that of the square? of a right-angled triangle is 8450 area of triangle PAC.
(a) 1: π (b) 2 : π (c) 3 : π (d) 4 : π square units. What is the length (b) Area of triangle PAB is equal to area of
triangle PAC.
_ (d) Given, of its hypotenuse?
(c) Area of triangle PAB is one-fourth of the
Perimetre of a circle and square are (a) 50 units (b) 55 units
equal. area of triangle PAC.
(c) 60 units (d) 65 units (d) Area of triangle PAB is half of the area of
∴2 πr = 4x ⇒ πr = 2 x
Area of circle = πr 2
_ (d) In right angle triangle ABC, triangle PAC.
∠C = 90º
πr π 2r 2
2 _ (b) We have,
Area of square = x =   =
2 Given, AB2 + BC 2 + AC 2 = 8450 AD is median of the ∆ABC.
2  4 A P is any point on AD.
Ratio of area of circle to square A
πr 2 4
= 2 2 = = 4: π
π r π
4
P
80. The lengths of the sides of a
right- angled triangle are B C
consecutive even integers (in AB + AB = 8450 ⇒ 2 AB2 = 8450
2 2

cm). What is the product of these B D C


∴ AB = 4225 = 65 units
integers? Area of ∆ABD = Area of ∆ADC
(a) 60 (b) 120 (c) 360 (d) 480
83. A triangle and a parallelogram In ∆PBC, PD is also median of ∆PBC
have equal areas and equal bases. If
_ (d) Let the sides of a right angle ∴Area of ∆PBD = Area of ∆PCD
triangle is x − 2, x, x + 2.
the altitude of the triangle is k
Area of ∆PAB = Area of ∆ABD − Area of
times the altitude of the
∴ ( x + 2 )2 = x 2 + ( x − 2 )2 ∆PBD
parallelogram, then what is the
x 2 + 4x + 4 = x 2 + x 2 − 4x + 4 Area of ∆PAC = Area of ∆ABD − Area of
value of k? ∆PDC
x =8 1
∴Sides of triangle are 6, 8, 10.
(a) 4 (b) 2 (c) 1 (d) ∴Area of ∆PAB is equal to Area of ∆PAC.
2
Product of sides of triangle
_ (b) Let base of triangle and 86. What is the area of a segment of a
= 6 × 8 × 10 = 480. parallelogram be b and altitude of circle of radius r subtending an angle
81. A circle is inscribed in a triangle triangle and parallelogram be h1 and θ at the centre?
h2 respectively.
ABC. It touches the sides AB and 1 2
Given, area of triangle = Area of (a) r θ
AC at M and N respectively. If O 2
parallelogram and h1 = kh2
θ θ
(b) r 2  θ − 2 sin cos 
is the centre of the circle and 1
h1
∠A = 70º, then what is ∠MON ⇒ =k 2  2 2
h2
1 2 θ θ
r  θ − sin cos 
equal to? 1
∴ bh1 = bh2 (c)
(a) 90º (b) 100º (c) 110º (d) 120º 2 2  2 2
_ (c) Given, 1 θ θ
h1
=2 (d) r 2 sin cos
A circle inscribed in a triangle ABC. h2 2 2 2
14 CDS Solved Paper 2020 (II)

_ (b) Area of segment of a circle of 88. Four circular coins of equal 90. If p , q , r , s and t represent length,
radius r subtending an angle θ at the radius are placed with their
centre breadth, height, surface area and
centres coinciding with four volume of a cuboid respectively, then
vertices of a square. Each coin 1 1 1
touches two other coins. If the what is + + equal to?
uncovered area of the square is p q r
O
θ s 2t s 2s
r θ/2 θ/2 r 42 cm 2 , then what is the radius (a) (b) (c) (d)
22 t s 2t t
A B of each coin? (Assume π = )
7 _ (c) We have, p, q , r, s and t represent the
(a) 5 cm (b) 7 cm length, breadth, height, surface area and
= Area of sector − Area of ∆OAB volume of cuboid
(c) 10 cm (d) 14 cm
1 1 ∴s = 2( pq + qr + rp), t = pqr
= r 2θ − r 2 sinθ
2 2 _ (b) Let radius of coins = r s  pq + qr + rp 
= 2 
1 ∴Sides of square = 2 r  
= r 2(θ − sinθ) t pqr
2 Area of shaded region 1 1 1
= 2 + + 
θ θ
= r 2  θ − 2 sin cos  = Area of square p q r
1
2  2 2 − Area of four quadrants 1 1 1 s
⇒ + + =
Q sinθ = 2 sin θ cos θ  r r p q r 2t
 D
2 2  C
r r 91. Fifteen candidates appeared in an
87. ABC is a triangle right-angled at examination. The marks of the
C. Let P be any point on AC and r r candidates who passed in the
Q be any point on BC. Which of A B examination are 9, 6, 7, 8, 8, 9, 6, 5, 4
r r
the following statements is/are and 7. What is the median of marks of
correct? = (2 r )2 − πr 2 all the fifteen candidates?
1. AQ 2 + BP 2 = AB 2 + PQ 2 = 4r 2 − πr 2 (a) 6 (b) 6.5
2. AB = 2PQ 2
= r 4 −
22  6r 2
 =
(c) 7 (d) 7.5
 7  7
Select the correct answer using _ (a) The marks of the candidates who
6r 2 passed in examination are 9, 6, 7, 8, 8, 9, 6,
the code given below : ⇒ = 42 5, 4, 7
7
(a) only 1 (b) only 2 Rearranging marks in ascending order of
⇒ r 2 = 49
(c) Both 1 and 2 (d) Neither 1 nor 2 all the fifteen candidates
⇒ r = 7 cm
_ (a) Given, ABC is a right angled at C −, −, −, −, −, 4, 5, 6, 6, 7, 7, 8, 8, 9, 9
and P and Q any point on AC and BC 89. The radii of the flat circular Median of marks of all the fifteen
respectively.
faces of a bucket are x and 2x. If candidates
In ∆AQC the height of the bucket is 3x, (15 + 1) th observation
=
AQ 2 = QC 2 + AC 2 …(i) what is the capacity of the 2
In ∆BPC, bucket? = 8 th observation
BP 2 = BC 2 + PC 2 …(ii) 22 =6
(Assume π = )
Adding Eqs. (i) and (ii), 7 92. If the yield (in gm) of barley from 7
AQ 2 + BP 2 = QC 2 + AC 2 + BC 2 + PC 2 (a) 11x3 (b) 22 x3 (c) 44 x3 (d)55 x3 plots of size one square yard each,
A
_ (b) Given, were found to be 180, 191, 175, 111,
Radius of flat circular faces of a 154, 141 and 176, then what is the
bucket are x and 2 x and height of median of yield?
P bucket is 3x.
(a) 111 gm (b) 154 gm
Volume of bucket
(c) 175 gm (d) 176 gm
1
= πh( R 2 + r 2 + Rr )
B Q C 3 _ (c) Given data
180, 191, 175, 111, 154, 141, 176
∴Capacity of bucket
AQ 2 + BP 2 = AC 2 + BC 2 + QC 2 Rearranging in ascending order
1
+ PC 2 = AB2 + PQ 2 = π( 3x ) {(2 x )2 + ( x )2 + (2 x ) ( x ))}
3 111, 141, 154, 175, 176, 180, 191
∴Ist statement is correct. 7 + 1
= πx ( 4x 2 + x 2 + 2 x 2 ) Median =   th observations
If P and Q are mid-point of AC and = 7 πx 3  2 
BC respectively, then AB = 2 PQ
=7 ×
22 3
x = 22 x 3 = 4 th observations
∴2nd statement is incorrect. 7 = 175
CDS Solved Paper 2020 (II) 15

93. Which one of the following ∴Mean 98. The mean of five observations x,
7 + 10 + 17 + 23
measures of central tendency will + 43 + 45 + 65 + x x + 2, x + 4, x + 6, x + 8 is m. What is
be used to determine the average = the mean of the first three
8
size of the shoe sold in the shop? 210 + x observations?
=
(a) Arithmetic mean 8 (a) m (b) m − 1 (c) m − 2 (d) m − 3
(b) Geometric mean New mean = 25 + 15
. = 26.5 _ (c) Given data x, x + 2, x + 4, x + 6,
(c) Median 210 + x x + 8
∴26.5 =
(d) Mode 8 Mean = m
_ (a) Average size of the shoe sold in ⇒ 212 = 210 + x ⇒ x = 2 x+ x+2+ x+ 4
+ x + 6+ x + 8
shop by Arithmetic mean. ∴m =
96. The mean weight of 100 students 5
94. When the class intervals have in a class is 46 kg. The mean 5m = 5x + 20
equal width, the height of a weight of boys is 50 kg and that x =m−4
rectangle in a histogram of girls is 40 kg. The number of ∴Mean of x, x + 2, x + 4
represents boys exceeds the number of girls x + x +2 + x + 4
(a) Width of the class by =
3
(b) Lower class limit (a) 10 (b) 15 (c) 20 (d) 25 3x + 6
= = x + 2 = m − 4 + 2 = m−2
(c) Upper class limit _ (c) We have, 100 student in a class. 3
(d) Frequency of the class Let number of boys = x
99. What is the median of 2, 4, 6, … ,
_ (d) The height of a rectangle in a and number of girls = y 100?
histogram represents frequency of ∴ x + y = 100 …(i)
the class. (a) 48 (b) 49 (c) 50 (d) 51
Mean of weight of boys = 50 kg
95. The ages of 7 family members are _ (d) Given, data
∴Total weight of boys = 50 x kg
2, 5, 12, 18, 38, 40 and 60 yr 2, 4, 6, … , 100
Mean of weight of girls = 40 kg
respectively. After 5 yr a new Here, n = 50
∴Total weight of girls = 40 y kg (25th + 26th) Observation
member aged x year is added. If ∴Median =
Mean of weight of 100 students 2
the mean age of the family now
= 46 kg ∴25 th observation = 25 × 2 = 50
goes up by 1.5 yr, then what is
∴ Total weight of 100 students 26 th observation = 26 × 2 = 52
the value of x?
= 4600 kg 50 + 52 102
(a) 1 (b) 2 ∴ Median = = = 51
∴ 50x + 40 y = 4600 …(ii) 2 2
(c) 3 (d) 4
From Eqs. (i) and (ii),
_ (b) We have, 100. The harmonic mean and the
x = 60, y = 40
Ages of 7 family members are geometric mean of two numbers are
∴ x − y = 60 − 40
2, 5, 12, 18, 38, 40, 60 10 and 12 respectively. What is their
= 20 arithmetic mean?
Mean
2 + 5 + 12 + 18 + 38 + 40 + 60 97. What is the algebraic sum of the 25
= (a) (b) 120
7 deviations from the mean of a set 3
175 (c) 11 (d) 14.4
= of values 25, 65, 73, 75, 83, 76, 17,
7 15, 7, 14? _ (d) Given, HM = 10,
= 25 (a) − 1 (b) 0 GM = 12
Ages of 7 family members after 5 yr (c) 1 (d) 2 We know that,
7, 10, 17, 23, 43, 45, 65
_ (b) We know that, algebraic sum of AM.HM = (GM)2 ⇒ AM (10) = (12 )2
After 5 yr new member ages x years the deviation from the mean of any 144
⇒ AM = = 14.4
is add data is zero. 10
PAPER II English
Directions (Q. Nos. 1-10) In this 1. Post colonialism is _ (a) The given passage clearly states
section you have two short passages. (a) a contestation of the then existing that to contradict or fight against the
After each passage, you will find some dominant western practices male dominance, women began
(b) a contestation of western practices coming out of their homes and started
items based on the passage. Read the participating in social relations,
in colonial states
passages and answer the items based politics, law, medicine, the arts,
(c) a contestation of the superstitious
on them. You are required to select practices
popular and academic knowledge.
your answers based on the content of (d) an approval of indigenous practices 5. Which word in the passage is
the passage and opinion of the author opposite of ‘contrast’?
only. _ (a) Post colonialism as defined in the
passage is ‘that contest the previous (a) Contestations (b) Trivial
dominant western ways of seeing (c) Discredited (d) Analogy
PASSAGE - I things’. Accordingly, option (a) is the
Post colonial cultural analysis has correct answer. _ (d) The word contrast means different.
been concerned with the elaboration Its opposite is analogy which means
2. What does ‘........... she was always similarity.
of theoretical structures that contest
an object, never a subject’ mean ?
the previous dominant western PASSAGE - II
(a) Women were given respect and
ways of seeing things. A simple How wonderful is the living world!
worshipped
analogy would be with feminism, (b) Women were not given any right The wide range of the living types is
which has involved a comparable equal to men amazing. The extraordinary
kind of project: there was a time (c) Women were treated at par with habitats in which we find living
when any book you might read, any men organisms, be it cold mountains,
speech you might hear, any film (d) Women liked to be treated inferior deciduous forests, oceans, fresh
that you saw, was always told from to men water lakes; deserts or hot springs,
the point of view of male. _ (b) The given phrase refers to the leave us speechless. The beauty of a
The woman was there, but she was, inequality that was meted out against galloping horse, or a migrating bird,
always an object, never a subject. women wherein she was never given the valley of flowers or the attacking
any right as equal to men.
From what you would read, or the Hence, option (b) correctly elucidates shark evokes awe and a deep sense
films you would see, the woman was this fact and thus is the correct answer. of wonder. The ecological conflict and
always the one who was looked at. cooperation among members of a
She was never the observing eye.
3. Why was ‘she never the observing population and among populations of
For centuries it was assumed that eye’? a community or even a molecular
women were less intelligent than (a) She was beautiful, so she was traffic inside a cell make us deeply
observed by men
men and that they did not merit the reflect on - what indeed is life? This
(b) She liked to be observed by men
same degree of education. They question has two implicit questions
(c) Women were assumed to be less
were not allowed a vote in the intelligent than men within it. The first is a technical one
political system. (d) Women were assumed to be more and seeks answer to what living is as
By the same token, any kind of intelligent than men opposed to the non-living, and the
knowledge developed by women was second is the philosophical one, and
_ (b) The word ‘observing eye’ in the seeks answer to what the purpose of
regarded as non-serious, trivial, phrase refers to the qualities of wit and
gossip or alternatively as knowledge intelligence. Then the statement that life is. What is living? When we try to
that had been discredited by science, women were never the observing eye define 'living', we conventionally look
such as superstition or traditional indicates that women were assumed to for distinctive characteristics
practices of childbirth or healing. All be less intelligent than men. exhibited by living organisms.
these attitudes were part of a larger Growth, reproduction, ability to
4. The contestation to dominance of
system in which women were sense environment and mount a
the male resulted in suitable response come to our mind
dominated, exploited, and physically
(a) participation of women in social immediately as unique features of
abused by men. Slowly, but relations, politics, law, medicine,
increasingly, from the end of 18th the arts, popular and academic
living organisms. One can add a few
century, feminists began to contest knowledge more features like metabolism,
this situation. The more they (b) participation of men in social ability to self-replicate, self-organise,
contested it, the more it became relations, politics, law, medicine, interact and emergence to this list.
the arts, popular and academic
increasingly obvious that these
knowledge
6. Why are the living types amazing?
attitudes extended into the whole of (a) The extraordinary diversity of
(c) participation of women in social
the culture; social relations, politics, habitats makes it amazing
movements
law, medicine, the arts, popular and (b) The living organisms are acting as
(d) contestations with males in life
academic knowledge. leading to divorce per their interests
CDS Solved Paper 2020 (II) 17

(c) The human thinking makes the living types amazing _ (c) Preposition ‘in’ will replace ‘to’ to make the sentence
(d) The evolution of life makes it amazing grammatically correct.
_ (a) The given passage states “The wide range of the living types 13. ‘Where there is a will then there is a way’ is an old epithet.
is amazing”. This indicates that the diversity of organism and
habitats makes the living types amazing.
(a) (b) (c)
No error.
7. Why does the author say, ‘ecological conflict and cooperation’ ? (d)
(a) Because living organisms are structured this way
(b) Because ecological mechanism works with conflict and _ (b) Conjunction ‘then’ will be removed to make the sentence
grammatically correct.
cooperation
(c) Because humans want to fight and live together 14. Indian feminism grew out of the women's movements
(d) Because living organisms sometimes fight and sometimes (a)
live together of the late nineteenth century,
_ (d) The author states this to highlight the tendency of all living (b)
organisms to fight in certain occasions and to live peacefully in
others. reached full maturity in the early twentieth century.
(c)
8. Which of the following statements is true of the passage?
No error.
(a) Meaning of life could be reflected as to what living is as
opposed to the non-living and what the purpose of life is (d)
(b) Meaning of life could be reflected as to how living _ (c) Article ‘the’ will be removed to make the sentence
organisms live and non-living organisms exist grammatically correct.
(c) Meaning of life could be reflected as to where the life 15. The greatest merit of democracy is that everyone feels free
begins and where it ends
(d) Meaning of life could be reflected on how various living (a) (b)
organisms differ and can pursues his/her interest. No error.
_ (a) The given passage states that the meaning of life could be (c) (d)
reflected as to what living is as opposed to the non-living and _ (c) ‘Pursues’ will be replaced by ‘pursue’ to make the sentence
what the purpose of life is. grammatically correct.
9. Distinctive characteristics exhibited by organisms indicate 16. All stake holders of education
that (a)
(a) they are living organisms have the right to ask for accountability
(b) they are non-living organisms (b)
(c) they can be either living organisms or non-living organisms
(d) they know the purpose of life
in every aspects of its implementation. No error.
(c) (d)
_ (a) According to the given passage, the distinctive characteristics
indicate that an organism is a living being. _ (c) ‘Aspects’ will be replaced by ‘aspect’ to make the sentence
grammatically correct.
10. Which word in the passage means ‘unique’?
(a) common (b) characteristics 17. Learning many languages
(c) distinctive (d) general (a)
_ (c) The synonym of unique is distinctive. promotes linguistic, cultural and social harmonies
(b)
Directions (Q.Nos. 11-20) Each item in this section has among people speaking different languages. No error.
a sentence with three underlined parts labelled as (a), (b) (c) (d)
and (c). Read each sentence to find out whether there is
any error in any underlined part and indicate your _ (b) ‘Harmonies’ will be replaced by ‘harmony’ to make the
sentence grammatically correct.
response on the Answer Sheet against the corresponding
letter, i.e., (a) or (b) or (c). If you find no error, your 18. One should not act according to one’s
response should be indicated as (d). (a) (b)
11. He has been one the most revered member whims and fancies on public places. No error.
(a) (b) (c) (d)
of the committee of enquiry. No error. _ (c) Preposition ‘on’ will be replaced by ‘in’ to make the sentence
grammatically correct.
(c) (d)
_ (b) Preposition ‘of’ will be added after “one…the” to make the 19. Economists believe that India had taken a new turn in 1990
sentence grammatically correct. (a) (b)
12. Rahul asked me whether I was interested with the liberalisation to her economy. No error.
(a) (b) (c) (d)
to joining the group for the trip. No error. _ (c) Preposition ‘to’ will be replaced by ‘of’ to make the sentence
(c) (d) grammatically correct.
18 CDS Solved Paper 2020 (II)

20. Irrigation works have a special importance The correct sequence should be
(a) (a) S P Q R (b) Q S R P (c) P R S Q (d) R P S Q
in an agricultural countries like India, _ (a) The correct and meaningful sentence is given by SPQR.
(b) 26. the imagination of children stories can exercise.
where rainfall is unequally distributed throughout the seasons. P Q
(c) more than the stories because they tell
No error. R S
(d) The correct sequence should be
_ (b) Article ‘an’ will be removed to make the sentence (a) Q R S P (b) S P Q R
grammatically correct. (c) Q P S R (d) R S Q P

Directions (Q. Nos. 21-30) Each of the following items, in _ (c) The correct and meaningful sentence is given by QPSR.
this section consists of a sentence, parts of which have been 27. as a record of and suffering of humans
jumbled. These parts have been labelled as P, Q, R and S. P Q
Given below each sentence are four sequences namely (a), the achievements, experiments history is considered
(b), (c) and (d). You are required to re-arrange the jumbled R S
parts of the sentence and mark your response accordingly. The correct sequence should be
21. history of life evolutionary Biology is forms on earth (a) S P R Q (b) R Q S P
P Q R (c) P Q R S (d) Q R S P
the study of _ (a) The correct and meaningful sentence is given by SPRQ.
S 28. can be invented it appears has been invented that all that
The correct sequence should be P Q R S
(a) S P Q R (b) Q S P R (c) R P Q S (d) P S Q R The correct sequence should be
_ (b) The correct and meaningful sentence is given by QSPR. (a) Q S P R (b) Q R S P
(c) R S Q P (d) S P Q R
22. life is considered the origin of the history of universe
P Q R _ (a) The correct and meaningful sentence is given by QSPR.
a unique event in 29. during the last century Indian social, political and cultural life
S P Q
The correct sequence should be as a testimony of Indian cinema stands
(a) Q P S R (b) P S Q R (c) S Q P R (d) R S P Q R S
_ (a) The correct and meaningful sentence is given by QPSR. The correct sequence should be
(a) S P Q R (b) Q R S P
23. productive resources is how we manage
(c) P Q R S (d) S R Q P
P Q
and competitiveness critical to strategic growth _ (d) The correct and meaningful sentence is given by SRQP.
R S 30. of all searches for knowledge should be the beginning
The correct sequence should be P Q
(a) P Q R S (b) R S P Q (c) S R P Q (d) Q P S R an exploration into truth and experiments of life
_ (d) The correct and meaningful sentence is given by QPSR. R S
24. in service firms operations strategy The correct sequence should be
(a) R Q P S (b) S P Q R (c) R S P Q (d) Q R S P
P Q
from the corporate strategy is generally inseparable _ (a) The correct and meaningful sentence is given by RQPS.
R S
The correct sequence should be
Directions (Q. Nos. 31-39) Given below are some
idioms/phrases followed by four alternative meanings to
(a) S R Q P (b) Q P S R (c) R S P Q (d) P S Q R
each. Choose the response (a), (b), (c) or (d) which is the most
_ (b) The correct and meaningful sentence is given by QPSR. appropriate expression and mark your response in the
25. are travelling, a recent survey has revealed Answer Sheet accordingly.
P Q 31. Get the jitters
that they are worried about their safety (a) Feeling anxious (b) Feeling happy
R (c) Stammering (d) Feeling exposed
even as more and more Indians _ (a) The idiom ‘get the jitters’ means ‘to feel anxious’.
S
CDS Solved Paper 2020 (II) 19

32. French leave _ (b) The phrase ‘around the corner’ environmental aspects, it is
means ‘an event that is going to take likely that larger and larger
(a) Absent from work without asking
place soon’.
for permission in French share of automobile sector
(b) Asking for permission before 38. With Heavy Heart would be in the form of
leaving work (a) With heavy weight electric vehicles.
(c) Work for permission to get leave (b) With joy and humour
(d) Absent from work without asking S : This presents a great
(c) With sense of shame
for permission (d) With pain and regret
opportunity for the
automobile industry as the
_ (d) The idiom ‘French leave’ means ‘to _ (d) The idiom ‘With heavy heart’ means
be absent from work without informing ‘with pain and regret’.
demand for automobiles
or asking for permission’. would only increase.
39. Cost a bomb The correct sequence should be
33. Take a stand
(a) To be very arrogant
(a) To publicly express an opinion (a) S R Q P (b) R Q S P
(b) To be with rich people
about something (c) Q P S R (d) Q S R P
(c) To be very expensive
(b) To make a stand for one to sit
(d) To be stingy _ (a) The correct and meaningful order
(c) To be firm on your work of sentences is SRQP.
(d) To be part of the work _ (c) The idiom ‘cost a bomb’ means ‘to
be very expensive’. 42. S1: Central government receipts
_ (a) The idiom ‘take a stand’ means ‘to can broadly be divided into
publicly express opinions’. 40. Roll your sleeves up
non-debt and debt receipts.
34. Cut and run (a) To prepare for wrestling
(b) To prepare for hard work S6 : This is also evident from the
(a) To avoid a difficult situation by composition of non-debt
(c) To make someone work for you
leaving suddenly
(d) To work with others receipts.
(b) To avoid an event suddenly
(c) To meet some danger suddenly _ (b) The idiom ‘roll your sleeves up’ P : Debt receipts mostly consist of
(d) To ask for sudden meeting with means ‘to prepare for hard work’. market borrowing and other
someone liabilities which the
_ (a) The idiom ‘cut and run’ means ‘to
Directions (Q.Nos. 41-50) In this government is obliged to
avoid a situation by leaving suddenly’. section each item consists of six repay in the future.
sentences of a passage. The first and
35. Cut the cord Q : The non-debt receipts comprise
sixth sentences are given in the
(a) To stop needing your parents for of tax revenue, non-tax
money beginning as S1 and S6. The middle
four sentences in each have been
revenue, recovery of loans and
(b) To stop needing someone else to
jumbled up and labelled as P, Q, R and disinvestment receipts.
look after you and start acting
independently S. You are required to find the proper R : The outcomes as reflected in
(c) To be safe on your own sequence of the four sentences and the Provisional Actual figures
(d) To be a married person is lower than the budget
mark your response accordingly on the
_ (b) The idiom ‘cut the cord’ means ‘ to Answer Sheet. estimate owing to reduction in
stop needing or relying on someone to the net tax revenue.
look after you and start acting 41. S1: The country’s economy is
independently’. growing and would continue S : The Budget 2018-19 targeted
to grow at a rapid pace in the significantly high growth in
36. Cupboard love
coming years. non debt receipts of the
(a) Loving someone to get something
from the person S6 : The market share of electrical Central Government, which
(b) Loving the cupboards vehicles increases with was driven by robust growth.
(c) Innocent love increasing availability of The correct sequence should be
(d) Loving to be free of all conditions (a) S R P Q (b) R S Q P
infrastructure.
_ (a) The idiom ‘Cupboard love’ means P : It also provides us an (c) P Q R S (d) Q P R S
‘to love someone for ones own benefit
or to love someone to get something opportunity to grow as _ (d) The correct and meaningful order
of sentences is QPRS.
from the person’. manufacturer of electric
vehicles. 43. S1: Palaeontology is the study of
37. Around the corner
Q : According to NITI Aayog the remains of dead organisms
(a) A thing which is at the end of the
(2019), if India reaches an over enormous spans of time.
corner
(b) An event or thing which is going to electric vehicles sales S6 : Faunal analysis gives
happen soon penetration, emission and oil information about the animal
(c) An event that corners someone for savings can be achieved. people hunted and
his wrong R : Given the commitments that domesticated, the age of
(d) An event that happens in the India has made on the climate animal at death, and the
corner of powerful place. diseases that afflicted them.
front as a nation and on
20 CDS Solved Paper 2020 (II)

P : Bones provide a great R : This act of earthworms affects S : Who should get what? How
information. the soil, and therefore the should power and other resource
Q : The distribution of faunal plants growing in it. be distributed? Should society
remains (animal bones) at a S : Rabbit’s fleas carry the virus be based on cooperation or
site can indicate which areas which causes myxomatosis, so conflict? And so on.
were used for butchering, they can affect the size of the The correct sequence should be
cooking, eating, bone tool rabbit population. (a) R S Q P (b) P Q S R
making and refuse dumping. The correct sequence should be (c) Q S R P (d) R S P Q
R: Within this discipline, (a) R S Q P (b) P S R Q _ (a) The correct and meaningful order
molecular biology and DNA (c) Q R S P (d) S Q R P of sentences is RSQP.
studies have been used to _ (c) The correct and meaningful order 48. S1: Regular exercise makes many
understand hominid evolution. of sentences is QRSP. of the organ systems become
S : Hominid evolution answers 46. S1 : The ecosystem of water is more efficient.
the questions about what complex and many S6 : Different activities require
ancient people looked like, and environmental factors are different levels of fitness.
to plot patterns of migration. intricately linked. P : It can improve your strength;
The correct sequence should be S6 : The trees slowly transfer make your body more flexible
(a) Q P R S (b) S P Q R rainwater into the sub-soil and and less likely to suffer from
(c) R S P Q (d) P Q R S this is critical for sustaining sprain.
_ (c) The correct and meaningful order water for months after the rains. Q : It can also improve your
of sentences is RSPQ. P : Thick forests make for endurance.
44. S1 : Hormones have several excellent catchments. R : It also uses up energy and helps
functions in the body. Q : The problems we see are to prevent large amounts of
S6 : The two hormones together because we have undermined, fat building up in the body.
regulate the glucose level in these links over decades. S : Exercise can increase your
the blood. R : First, rain and snowfall are the fitness in three ways.
P : They help to maintain the only sources of water-about 99%. The correct sequence should be
balance of biological activities S : In the four months of (a) Q R S P (b) R S P Q
in the body. (c) P S Q R (d) S Q R P
monsoon, there are about 30-35
Q : Insulin is released in response
downpours and the challenge is _ (b) The correct and meaningful order
to the rapid rise in blood
to hold this water in systems of sentences is RSPQ.
glucose level.
R : On the other hand hormone that can last us over 365 days. 49. S1: On increasing the temperature
glucagon tends to increase the The correct sequence should be of solids, the kinetic energy of
glucose level in the blood. (a) Q R S P (b) P S R Q the particles increases.
S : The role of insulin in keeping (c) S R Q P (d) R Q S P S6 : The temperature at which a
the blood glucose level within _ (a) The correct and meaningful order solid melts to become a liquid
the narrow limit is an example of sentences is QRSP. at the atmospheric pressure is
of this function. 47. S1: Politics is exciting because called its melting point.
The correct sequence should be people disagree. P : A stage is reached when the solid
(a) P S R Q (b) R S P Q S6 : It is not solitary people who melts and is converted to a
(c) S R Q P (d) Q R S P make politics and a good liquid.
_ (a) The correct and meaningful order society; it is the people Q : Due to the increase in kinetic
of sentences is PSRQ. together which make good energy, the particles start
45. S1 : All living things affect the living politics and society. vibrating with greater speed.
and non-living things around P : For Aristotle politics is an R : The particles leave their fixed
them. attempt to create a good positions and start moving
S6 : This interdependability needs to society because politics is, more freely.
be understood when we, humans, above all, a social activity. S : The energy supplied by heat
consume much morethan Q : They also disagree about how overcomes the forces of
required and abuse nature. such matters should be attraction between the particles.
P : This can also affect the resolved, how collective The correct sequence should be
population of fox, if foxes decision should be made and (a) Q S R P (b) Q R S P
depend on rabbits for food. who should have a say. (c) P R S Q (d) S P R Q
R : They disagree about how they
Q : For example, earthworms, _ (a) The correct and meaningful order
make burrows and worm casts. should live. of sentences is QSRP.
CDS Solved Paper 2020 (II) 21

50. S 1 : Things are often not what 53. When I visited the villages nearby Directions (Q. Nos. 61-70) Each
they seem. the city I .......... many water bodies item in this section consists of a
S6 : This happened without you intact. sentence with an underlined word(s)
even knowing it. So imagine (a) came across (b) come across followed by four words/group of
the changes that occur to this (c) came (d) came in words. Select the option that is nearest
earth and humanity. _ (a) As the given sentence is in past in meaning to the underlined word and
P : But you are really not, because tense, ‘came across’ will be the correct mark your response on the Answer
filler for the sentence.
the Milky Way galaxy, of Sheet accordingly.
which you are a part, is 54. He has lost all his investments and 61. Emboldened by its success,
moving through space at 2.1 he is ............. .
the leader now plans to go ahead
million kilometre an hour. (a) broke (b) broken with the plan and implementation.
Q : So in roughly twenty second (c) discredited (d) defunct
(a) Encouraged (b) Disgruntled
that it would have taken you _ (a) Adjective broke will be the correct (c) Succeeded (d) Failed
to read this paragraph, you filler for the sentence.
_ (a) The given word ‘Emboldened’
have already moved thousands 55. He ............ whether he could get any means encouraged.
of kilometre. certificate for the course.
62. It is encouraging to see India’s
R : And that is without taking (a) said (b) told indigenous cinema is going
into account the effects of (c) thought of (d) asked
places.
Earth’s rotation on its own _ (d) The tense form ‘asked’ will be
axis, its orbiting around the (a) homogenous
correct filler for the sentence.
(b) classical
Sun and Sun’s journey around 56. I ............ farewell to all my course (c) home-grown
the Milky Way. mates last year. (d) Non-native language
S : As you read this sentence,
perhaps sitting in a
(a) bid (b) bade _ (c) The given word ‘Indigenous’
(c) said (d) bad means originating or occurring
comfortable chair in your naturally in a particular place; native.
study, you would probably _ (b) The word Bade is correctly used Therefore, its synonym will be
with the word ‘farewell’. home-grown.
consider yourself at rest.
The correct sequence should be 57. Very few of the texts from very 63. The ability to imagine and conceive
early Vedic period are ............. now. a common good is inconsistent
(a) Q R P S (b) R Q P S
(c) P Q R S (d) S P R Q (a) extant (b) exit with what is known as ‘pleonexia’
(c) exempt (d) redundant
_ (d) The correct and meaningful order is a major struggle for a good
of sentences is SPRQ. _ (a) The word ‘Extant’ meaning democracy to realise.
surviving will be the correct filler for the
sentence. (a) Greed to grab everything for
Directions (Q. Nos. 51-60) Each of oneself
the following sentences in this section 58. A speech is a ............. address, (b) Greed to accumulate more and
has a blank space and four words or delivered to an audience that seeks more wealth
group of words are given after the to convince, persuade, inspire or (c) Dislike for others
sentence. Select the most appropriate inform. (d) Over ambitious
word or group of words for the blank (a) formal (b) informal _ (b) The given word ‘pleonexia’ means
space and indicate your response on (c) humorous (d) political extreme greed for wealth or material
the Answer Sheet accordingly. possessions.
_ (a) The word ‘formal’ is the correct filler
51. If I ............. a good match I would for the sentence. 64. He tried to avoid saying
have got married. 59. All that .......... is not gold. something that would implicate
(a) had found (b) have found (a) glitter (b) glitters
him further.
(c) found (d) have (c) glittering (d) gliding (a) reward (b) incriminate
(c) encourage (d) incite
_ (a) As the given sentence is in past _ (b) The word ‘glitters’ is the correct filler
perfect form, ‘had found’ will be the for the sentence. _ (b) The given word ‘Implicate’ means
correct filler for the sentence. to introduce some to crime or to
60. Having been in politics for about 40 incriminate.
52. The lady has been declared as one years, the party now treats him like
of top ten ........... of the community. ............. . 65. The statutory corporate tax which
forms the major income of the
(a) more powerful members (a) a have-been
(b) most powerful members
government has not changed this
(b) a had-been
(c) most powerful member (c) a has-been
year.
(d) more powerful member (d) would have been (a) legislature (b) unlawful
(c) government (d) legal
_ (b) The correct adjectival phrase to be _ (c) The phrase ‘a has-been’ meaning a
filled in the blank is ‘most powerful person who is outdated is the correct _ (d) The given word ‘statutory’ means
members’. filler. legal.
22 CDS Solved Paper 2020 (II)

66. He has been part of the all dissident _ (a) The word ‘diffuse’ means 80. He is one of the confidants of the
activities. spread-out. Its antonym is leader and can influence the
concentrate.
(a) rebellious (b) supportive decision of the government.
(c) conformist (d) legal 73. Unrest in some pockets made the (a) opponents (b) intimate
city dwellers confine themselves at (c) close friend (d) colleague
_ (a) The given word ‘dissident’ means
rebellious. home.
_ (a) The word ‘confidants’ refers to a
67. Advocacy is one major component (a) Turbulence (b) Unease friend or a companion. Its antonym is
(c) Apprehension (d) Calm opponents.
of any new programme.
(a) promotion (b) opposition _ (d) The word ‘unrest’ means ‘a feeling
(c) critique (d) liking
of disturbance’. Its antonym is calm. Directions (Q. Nos. 81-90) Each of
74. Peace and tranquility are the following sentences has a word or
_ (a) The given word ‘advocacy’ means phrase underlined. Read the sentences
to promote someone. instruments which would boost the
development of society. carefully and find which part of speech
68. People avoided him for his high the underlined word is. Indicate your
mindedness. (a) uproar (b) calm
(c) serenity (d) sound response on the Answer Sheet
(a) toughness (b) strong principles accordingly.
(c) anger (d) whims _ (a) The word ‘tranquility’ means peace.
Its antonym is uproar which means 81. All the pilgrims rested for a while
_ (b) The given word ‘high-mindedness’ disturbance.
under the banyan tree.
refers to someone with very high and
strong principles.
75. Barring a decision of such disputes, (a) Adverb (b) Place value
other matters relating to the (c) Preposition (d) Verb
69. There is a tendency to treat social election of President or Vice-
changes as mere development in _ (c) The word ‘under’ is a preposition.
President may be regulated by law
terms of accumulation of wealth. made by Parliament. 82. The wonderful statue, of the
(a) position (b) predisposition (a) excepting (b) without leader welcomes all people to
(c) thinking (d) idea (c) including (d) excluding city.
_ (b) The given word ‘tendency’ means (a) Object (b) Adjective
_ (c) The word ‘barring’ means
an inclination towards a particular exceptions. Its antonym is including. (c) Noun phrase (d) Noun
characteristic or type of behaviour. The
word ‘predisposition’ means the same. 76. His speech was full of emotions and _ (d) The word ‘statue’ is a noun.
it was an extempore. 83. This is his pen.
70. During the ancient period poets
(a) prepared (b) ready made (a) Possessive pronoun
were patronised through various
(c) unrehearsed (d) ad lib (b) Possessive adjective
institutions. (c) Adverb
(a) supported (b) respected _ (a) The word ‘extempore’ means
spontaneous or unprepared. Its (d) Verb
(c) opposed (d) scolded antonym is prepared. _ (b) The word ‘his’ is a possessive
_ (a) The given word ‘patronised’ means 77. The teacher asked her students to adjective.
to be supported.
understand the ensuing problems 84. When people found that the jewel
Directions (Q.Nos. 71-80) Each item and address them suitably . was in records of Rahim, they' gave
in this section consists of sentences with (a) subsequent (b) consequent it to him.
an underlined word followed by four (c) retrospective (d) en suite (a) Pronoun (b) Nominative
words or group of words. Select the option _ (c) The word ‘ensuing’ means ‘as a (c) Noun (d) Adverb
result of’. Its antonym is retrospective
that is opposite In meaning to the
which deals with past.
_ (a) The word ‘it’ is a pronoun.
underlined word and mark your response 85. It is eleven O’clock now and all of
on the Answer Sheet accordingly. 78. All the allegations against the actor
us should retire to bed.
were expunged by the committee of
71. The archaic thinking leads to (a) Personal pronoun
inquiry. (b) Relative pronoun
unfounded beliefs.
(a) got rid of (b) part of (c) Impersonal pronoun
(a) antiquated
(c) accepted (d) rejected (d) Verb
(b) outmoded
(c) beyond the times _ (c) The word ‘expunged’ means to _ (c) The word ‘it’ is an impersonal
(d) modern remove. Its antonym is accepted. pronoun.

_ (d) The word ‘archaic’ means old 79. His relatives dissuaded him from 86. The flower is very beautiful.
fashioned. Its antonym is modern. giving up the job. (a) Adjective
72. Police had to resort to tear gas to (a) persuaded (b) discouraged (b) Adverb
diffuse tension among the crowd. (c) advised against (d) deter (c) Preposition
(d) Conjunction
(a) concentrate (b) scatter _ (a) The word ‘dissuaded’ means to
(c) disperse (d) strew discourage. Its antonym is persuaded _ (b) The word ‘very’ is an adverb.
which means to convince.
CDS Solved Paper 2020 (II) 23

87. This boy is stronger than Ramesh. (c) Demonstrative preposition _ (c) The word ‘fought’ is a transitive.
(d) Adjective
(a) Pronoun (b) Adjective 90. I can hardly believe it.
(c) Article (d) Adverb _ (b) The word ‘myself’ is a pronoun. (a) Adjective
_ (b) The word ‘this’ is an adjective. 89. The ants fought the wasps. (b) Preposition
(a) Intransitive verb (c) Adverb
88. I hurt myself. (b) Transitive verb (d) Verb
(a) Noun (c) Demonstrative verb
(b) Pronoun (d) Adjective _ (c) The word ‘hardly’ is an adverb.

Directions (Q.Nos. 91-110) Each of the following sentences in this section has a blank space with four words or group of
words given. Select whichever word or group of words you consider the most appropriate for the blank space and indicate
your response on the Answer Sheet accordingly.
The difficult thing about ............... 91. (a) studying the science of habits is that most people, when they hear
(b) study
(c) studies
(d) are studying
_ (a) The correct filler is studying.
about this field of research .............. 92. (a) wanting to know the secret formula for quickly changing any habit.
(b) wanted
(c) wants
(d) want
_ (d) The correct filler is want.
If scientists have discovered how ............. 93. (a) those patterns work, then it stands to reason that they
(b) this
(c) these
(d) that
_ (c) The correct filler is these.
................. 94. (a) must have also found a recipe for rapid change, right? If only it .................
(b) will
(c) could
(d) might
_ (a) The correct filler is must.
............. 95. (a) are that easy. It’s not ............. 96. (a) these formulas don’t
(b) were (b) this
(c) was (c) that
(d) will be (d) which
_ (b) The correct filler is were. _ (c) The correct filler is that.
exist. The problem is that there isn’t one formula for ............. 97. (a) changing
(b) changed
(c) having changed
(d) changes for
_ (a) The correct filler is changing.
habits. There are thousands. Individuals and habits are ............... 98. (a) full
(b) all
(c) complete
(d) most
_ (b) The correct filler is all.
different, and so the specifics of diagnosing and changing the patterns in our lives differ from person to ...................
99. (a) people and behaviour to behaviour. Giving up
(b) persons
(c) personnel
(d) person
_ (d) The correct filler is person.
cigarettes is different .............. 100. (a) from curbing overeating, which is different
(b) since
(c) to
(d) into
_ (a) The correct filler is from.
24 CDS Solved Paper 2020 (II)

from changing how you communicate with your spouse, .............. 101. (a) it
(b) this
(c) what
(d) which
_ (d) The correct filler is which.
is different from how you prioritise tasks at work. What's more, each person's habits are
............ 102. (a) broken by different cravings. As a result, this book does not
(b) given
(c) driven
(d) prescribed
_ (c) The correct filler is driven.
.............. 103. (a) contain one prescription. Rather, I hoped to deliver something
(b) contains
(c) contained
(d) containing
_ (a) The correct filler is contain.
else : a framework for understanding ............ 104. (a) how habits work and a
(b) what
(c) where
(d) whose
_ (a) The correct filler is how.
guide to experimenting with how they ............... 105. (a) might change. Some habits yield easily
(b) would
(c) will
(d) must
_ (a) The correct filler is might.
to analysis and influence. Others are ............. 106. (a) quiet complex and obstinate, and require prolonged study. And for
(b) most
(c) better
(d) more
_ (d) The correct filler is more.
others, change is a ................. 107. (a) process that never fully concludes. But that does not
(b) processing
(c) processed
(d) processes
_ (a) The correct filler is process.
............. 108. (a) means it can’t occur. Each chapter in this book explains
(b) meant
(c) meaning
(d) mean
_ (d) The correct filler is mean.
a different aspect of why habits exist and how they function. The framework
............... 109. (a) describing in this section is an attempt to distil, in
(b) described
(c) will describe
(d) description
_ (b) The correct filler is described.
.............. 110. (a) a very basic way, the tactics that researchers have found
(b) any
(c) the
(d) rather
_ (a) The correct filler is a .
for diagnosing and shaping habits within our own lives.
CDS Solved Paper 2020 (II) 25

Directions (Q. Nos. 111-120) In this _ (d) The correct spelling is Curriculum. (a) Snobbery (b) Snoberry
section a word is spelt in four different 114. Which one of the following (c) Snabbery (d) Snobbory
ways. Identify the one which is correct. alternatives has the correct _ (a) The correct spelling is Snobbery.
Choose the correct response (a), (b), (c) spelling? 118. Which one of the following
or (d) and indicate on the Answer Sheet (a) Magnificent (b) Magnificant alternatives has the correct
accordingly. (c) Magneficent (d) Magenficient spelling?
111. Which one of the following _ (a) The correct spelling is Magnificent. (a) Neurasis (b) Nuroesis
alternatives has the correct (c) Neurosis (d) Neuresis
115. Which one of the following
spelling? alternatives has the correct _ (c) The correct spelling is Neurosis.
(a) Mountaneous (b) Mountenous
spelling? 119. Which one of the following
(c) Mountaineous (d) Mountainous
(a) Felecitation (b) Felicitation alternatives has the correct
_ (d) Mountainous. (c) Falicitation (d) Felicitasion spelling?
112. Which one of the following _ (b) The correct spelling is Felicitation. (a) Dipthteria (b) Diptheria
alternatives has the correct (c) Diphtheria (d) Diphthria
116. Which one of the following
spelling?
alternatives has the correct _ (c) The correct spelling is Diphtheria.
(a) Etiquette (b) Etiquete
spelling? 120. Which one of the following
(c) Etiequtte (d) Etequtte
(a) Twelth (b) Twelfth alternatives has the correct
_ (a) The correct spelling is Etiquette. (c) Tweluth (d) Twelthe spelling?
113. Which one of the following (a) Meagre
alternatives has the correct _ (b) The correct spelling is Twelfth. (b) Megare
spelling? 117. Which one of the following (c) Meagr
(a) Curriculam (b) Curiculum alternatives has the correct (d) Megear
(c) Curiculeum (d) Curriculum spelling? _ (a) The correct spelling is Meagre.

PAPER III General Studies


1. As per the Budget Estimates of 2. As per the World Bank’s Ease of 3. Which one of the following
2019-20, the following are some of Doing Business Ranking, India’s statements with regard to the
the important sources of tax rank has improved from 142 in 2014 National Food Security Act is not
receipts for the Union Government: to 63 in 2019. During this period, in correct?
1. Corporation Tax which of the following parameters (a) The Act was enacted in the year 2013.
has India’s rank deteriorated? (b) The Act was rolled out in the year
2. Taxes on Income other than 2014.
Corporation Tax (a) Ease of starting a business
(c) The Act legally entitles 67% of the
(b) Getting electricity
3. Goods and Services Tax population to receive highly
(c) Registering property
4. Union Excise Duties subsidised food grains.
(d) Paying taxes
(d) The Act is not being implemented
Which one of the following is the
_ (c) As per the World Bank’s Ease of in all the States/Union Territories.
correct descending order of the Doing Business Report, India’s rank
foresaid tax receipts as a has improved from 142 in 2014 to 63 in _ (d) The National Food Security (NFS)
Act, 2013 aims to provide subsidised
percentage of GDP? 2019. During this period India’s rank
food grains to 67% of India’s
has deteriorated in Registering
(a) 1, 2, 3, 4 (b) 1, 3, 2, 4 population. It was rolled out in the year
Property.
(c) 3, 2, 1, 4 (d) 2, 4, 3, 1 2014. It is implemented in all the
S.No. Parameters 2014 2019 State/Union Territories.
_ (b) As per the Budget Estimates of 1. Starting a business 156 136 Salient Features of this Act
2019-20, the important sources of tax
2. Dealing with 183 27 l Beneficiaries of the PDS are entitled
receipts for the Union Government are
as follows- construction permits to 5 kg per person per month of
3. Getting electricity 134 22 cereals at the following prices: Rice
Budgeted
Sources 2019-20
at ` 3 per kg, Wheat at ` 2 per kg and
4. Registering property 115 154
(` crore) Coarse grains (millet) at ` 1 per kg.
5. Getting credit 30 25
Corporation Tax 766000 l Pregnant women, lactating mothers
6. Protecting investors 21 13 and certain categories of children
Goods and Services Tax 663343
Taxes on Income 569000 7. Paying taxes 154 115 are eligible for daily free cereals.
Union Excise Duties 300000 8. Trading across borders 122 68 l It includes the Midday Meal Scheme,

Customs 155904 9. Enforcing contracts 186 163 Integrated Child Development


Services (ICDS) scheme and the
Hence, option (b) is correct. 10. Resolving insolvency 135 52
Public Distribution System (PDS).
26 CDS Solved Paper 2020 (II)

4. Which one of the following The annual average rainfall of the 10. Which one of the following
other given places are
statements about Indian economy indicates the Tropical Savannah
Jodhpur – 363 mm
during 2019-20 is not correct? climate?
New Delhi – 617 mm
(a) There has been deceleration in (a) Aw (b) Dfc (c) Cwg (d) Am
growth rate. Bengaluru – 970 mm
(b) There has been sluggish growth in _ (a) Aw or the tropical wet and dry
7. Timber vegetation is generally not climate, also known as the Savannah
tax revenue relative to the Budget found in which of the following climate, where there is an extended
Estimates. dry season during the winter.
(c) Fiscal deficit as percentage of
regions?
(a) Subtropical region During the wet season, rainfall is less
GDP has been as per the Budget
than 1000 mm, occurring mainly in
Estimates. (b) Temperate region
the summer time.
(d) The non-tax revenue registered a (c) Alpine region
considerably higher growth. (d) Tundra region Other Climatic Types
Aw - Monsoonal Type Climate
_ (c) There has been deceleration in _ (d) Timber vegetation is generally not Dfc - Cold Humid winter with Short
growth rate in Indian Economy during found in Tundra region. The Tundra is
2019-20. There has been sluggish a treeless polar desert found in the summer.
growth in tax revenue relative to the high latitudes in the polar regions. Cwg - Monsoon Type with Dry winters.
Budget Estimates. Fiscal deficit as Tundra lands are covered with snow
percentage of GDP has been more for much of the year. 11. The largest geographical area of
than the Budget Estimates. The The Tundra regions are-Alaska, India is covered by which one of
non-tax revenue registered a Canada, Russia, Greenland, Iceland. the following types of soils?
considerable growth. The characteristics of the Tundra (a) Inceptisols (b) Entisols
5. As per the Budget Estimates of Regions are: (c) Alfisols (d) Vertisols
l Extremely cold climate
expenditure on major subsidies _ (a) The largest geographical area of
during 2019-20, the maximum l Low biotic diversity India is covered by Inceptisols, which
l Limitation of drainage is 39.74%. Inceptisols are usually the
expenditure was likely to be on weakly developed young soil though
l Simple vegetation structure
(a) urea subsidy they are more developed than entisols.
(b) petroleum subsidy The Arctic Tundra’s temperature is
– 34° C to – 6° (c) Other soils cover an area of
(c) food subsidy Entisols – 28.08%
(d) fertilizer subsidy 8. Decadal growth rate of population Albisols – 13.55%
_ (c) In 2019-20, the total expenditure in percentage was highest in India Vertisols – 8.52%
on subsidies is estimated to increase in the year
to ` 3,38,949 crore (13.3%) over the 12. Which one of the following cities
revised estimate of 2018-19. This is (a) 1991 (b) 1981 is closest to the Equator?
owing to an increase in expenditure (c) 1971 (d) 1961
(a) Mogadishu (b) Singapore
on petroleum, fertilizer, food and other _ (c) The decadal growth rate of (c) Colombo (d) Manila
interest subsidies. population in India in percentage was
Subsidies in 2019-20 (` crore) highest in India in the year 1971 which _ (b) Singapore is closest to the
was 24.80. The decadal growth of equator.
Subsidy Budget other given year is as follows. Other cities which are near to the
2019-20 equator are
l 1961-21.64%
Food subsidy 184220 l 1981-24.66% Quito (Equador)
Fertilizer subsidy 79996 l 1991-27.87%
Kampala (Uganda)
Pekanbaru (Indonesia)
Petroleum subsidy 37478 The Decadal growth of 2011 is 17.64.
Padang (Indonesia)
Other subsidies 37255 9. The Isotherm Line, which divides Maccapa (Brazil)
Total 338949
India North-South in almost two Kisumu (Kenya)
equal parts in the month of Libreville (Gabon)
Hence, option (c) is correct. January, is 13. Who among the following gave
(a) 10 °C (b) 25 °C evidence before the Joint Select
6. Which one of the following Indian
(c) 15 °C (d) 20 °C Committee on the Government of
places receives minimum rainfall
in a year? _ (d) The Isotherm Line, which divides India Bill, 1919 in favour of female
India North-South in almost two equal
(a) Jodhpur parts in the month of January is 20°C
franchise?
(b) Leh Isotherm is a line that connect points 1. Annie Besant 2. Sarojini Naidu
(c) New Delhi of equal temperature on map, so at 3. Hirabai Tata
(d) Bengaluru every point along a given isotherm Select the correct answer using the
_ (b) Leh receives minimum rainfall in a the temperature values are same. It is
used to observe the distribution of air
codes given below.
year among the given option. The
average annual rainfall is only 102 temperature over a vast area. (a) Only 1 (b) 1 and 2
mm (4.02 inches). (c) 2 and 3 (d) 1, 2 and 3
CDS Solved Paper 2020 (II) 27

_ (d) Annie Besant, Sarojini Naidu and 17. Which one among the following and statutory responsibilities, the
Hirabai Tata gave evidence before the Speaker of the Lok Sabha is assisted
was India’s first trade union in the
Joint Select Committee on the by the Secretary-General, Lok Sabha,
Government of India Bill, 1919 in proper sense of the term? functionaries of the level of the
favour of Female Franchise. (a) Bombay Labour Union Additional Secretary, Joint Secretary
The Government of India Act, 1919, (b) Ahmedabad Labour Union and other officers and staff of the
was an Act of the Parliament of the (c) Madras Labour Union Secretariat at various levels.
United Kingdom. It was passed to (d) Allahabad Labour Union The Secretary General remains in
expand participation of Indians in the office till his/her retirement at the age
Government of India. The Act
_ (c) Madras Union Labour was India of 60. He/she is answerable only to
first trade Union in the proper sense
embodied the reforms recommended the Speaker, his/her action cannot be
of term. Madras Labour Union was
in the report of the Secretary of State discussed or criticised in or outside
formed on April 27, 1918 by the
of India, Edwin Montagu, and the workers of the then Buckingham and the Lok Sabha. On behalf of the
Viceroy Lord Chelmsford. Carnatic Mills at Perambur, Chennai. President of India, he/she summons
The two prominent traders involved in members to attend session of
14. In which one of the following Parliament and authenticates Bills in
forming the union were Selvapathi
places was the Ahmadiyya Chettiyar and Ramanujulu Naidu. the absence of the Speaker.
Movement started by Mirza BP Wadia was the founding member 20. Who among the following moved
Ghulam Ahmad? of the MLU. Selvapathi and Ramanujulu
became the General Secretaries of the the motion of Secret Sitting
(a) Patna (b) Aligarh union while Thiru Vi Ka its Vice Session of the Assembly (1942)?
(c) Bhopal (d) Gurdaspur President. (a) MS Aney (b) GV Mavalankar
_ (d) The Ahmadiyya Movement was 18. Who among the following formed (c) CM Stephen (d) A. Ayyangar
launched by Mirza Ghulam Ahmad of
Qadiyan in 1889 from Gurudaspur. the Seva Samiti Boy Scouts _ (a) Madhav Shrihari Aney (MS Aney)
Association in 1914? moved the motion of Secret Sitting
He began his work as a defender of Session of the Assembly in 1942.
Islam against the Polemics (Religions (a) Hriday Nath Kunzru The objective of this Session was to
Propoganda) of the Arya Samaj and (b) SG Vaze discuss the 2nd World War situation.
the Christian missionaries. (c) Annie Besant He was member of the Viceroy’s
15. With whom did Subhas Chandra (d) Shri Ram Bajpai Executive Council from 1941-1944.
He is popularly referred as Loknayak
Bose form an alliance to destroy _ (a) Hriday Nath Kunzru and Madan Bapujj Aney. He was also Governor of
the Holwell Monument in Calcutta Mohan Malviya formed the Seva
Bihar from 12th January, 1948 to 14th
Samiti Boy Scouts Association. On
during 1939-40? June, 1952.
the line of the Worldwide Baden-
(a) The Communist Party of India Powell Organisation, which at the time 21. Which one among the following
(b) The Muslim League refused to allow Indians to join it.
statements pertaining to the
(c) The Hindu Mahasabha They were assisted by Shri Ram
President’s term of Office is not
(d) The Unionist Party Bajpai.
correct?
_ (b) Subhas Chandra Bose with an 19. Which one among the following is (a) The President holds Office for a
alliance with the Muslim League
destroyed the Holwell Monument in
not correct about the Secretary term of five years.
Calcutta during 1939-40. General of the Lok Sabha? (b) The President may be removed
(a) The Secretary General is the from the Office by way of
The Holwell Monument was
advisor of the Speaker. impeachment.
constructed to commemorate the
victims of the ‘Black Hole Tragedy (b) The Secretary General acts under (c) The President may resign before
1956. It was named after John the authority in the name of the the expiration of his/her term by
Zephaniah Holwell, one of the Speaker. writing to the Speaker of the Lok
survivor of this incident. (c) The Secretary General works under Sabha.
the Speaker with delegated (d) The President shall, not
16. Who among the following created authority. withstanding the expiration of
the first All India Trade Union (d) The Secretary General passes his/her term, continue to hold
Congress in 1920? orders in the name of the Speaker. Office until his/her successor
enters upon his/her Office.
(a) BP Wadia (b) SA Dange _ (d) The Secretary General of the Lok
(c) NM Joshi (d) BT Ranadive Sabha passes orders in the name of _ (c) “The President may resign before
the speaker is not correct statement. the expiration of his/her term by
_ (c) NM Joshi created the first All India writing to the speaker of the Lok
Trade Union Congress (AITUC) in The Secretary General of the Lok Sabha”, is not correct. The President
1920. Sabha is the Administrative Head of submits his resignation to the
The AITUC was founded on October the Lok Sabha Secretariat. He/she is Vice-President.
31, 1920 in Mumbai . appointed by the Speaker of the Lok The Indian President is the Head of
Sabha. The post of Secretary the State and he is also called the
l It is associated with the Communist
Party of India. According to General is of the rank of the Cabinet First Citizen of India. He is a part of
provisional statistics from the Secretary in the Government of India, Union Executive, provisions of which
Ministry of Labour, AITUC had who is the senior most civil servant to are dealt with Articles 52-78 including
a membership of 14.2 million in the Indian Government. Articles related to President
2013. In the discharge of his constitutional (Articles 52-62).
28 CDS Solved Paper 2020 (II)

Supreme Court shall inquire and There are five major types of Writ viz. Maharashtra and Gujarat in June
decide regarding all doubts and Habeas Corpus, Mandamus,
Prohibition, Quo Warranto and
2020?
disputes arising out of or in
connection with the election of a Certiorari. (a) Maldives (b) Bangladesh
President as per Article 71(1) of the Quo warranto means ‘by what (c) Thailand (d) Japan
Constitution. Supreme Court can warrant?’ This writ is issued to _ (b) Bangladesh had chosen the name
remove the President for the electoral enquire into legality of the claim of a ‘Nisarga’ for the cyclone which
malpractices or upon being not person or Public Office. It restrains devastated the coastline of
eligible to be Lok Sabha member the person or authority to act in an Maharashtra and Gujarat in June
under the Representation of the Office which he/she is not entitled to; 2020. The Nisarga cyclone is formed
People Act, 1951. and thus stops usurpation of Public because of the depression in the
The President may also be removed Office by anyone. This Writ is Arabian Sea. It is a tropical cyclone
before the expiry of the term through applicable to the public offices only formed because of exceptional warm
impeachment for violating the and not to Private Offices. surface ocean temperatures.
Constitution of India by the 27. Who among the following played
24. Which one of the following
Parliament of India. The process may
start in either of the two Houses of Commissions is related to the role of Shakuntala Devi in the
the Parliament. Article 338A? biopic movie based on the life of
(a) The National Commission for the famous mathematician?
22. Which of the following Articles in
Scheduled Castes (a) Madhuri Dixit (b) Rani Mukherjee
the Constitution of India are (b) The National Commission for (c) Tabu (d) Vidya Balan
exceptions to the Fundamental Scheduled Tribes
Rights enumerated in Article 14 (c) The National Commission for _ (d) Indian bollywood actress
Vidya Balan played the role of
and Article-19? Backward Classes Shakuntala Devi in the biopic movie
(a) Article 31A and Article 31C (d) The National Commission for based on the life of the famous
(b) Article 31B and Article 31D Women Mathematician.
(c) Article 12 and Article 13 _ (b) Article 338A of the Indian Shakuntala Devi (1929 -2013) was an
(d) Article 16 and Article 17 Constitution deals with the National Indian writer and mental calculator,
Commission for Scheduled Tribes. popularly known as the ‘Human
_ (a) Article 31A and Article 31C in the This article was added in the Computer’.
Constitution of the India are
Constitution through the 89th She strove to simplify numerical
exceptions to the Fundamental Rights
Constitutional Amendment Act, 2003. calculations for students. Her talent
enurmated in Article 14 and Article 19.
The Chairperson, Vice-Chairperson earned her a place in the 1982
By the 1st Constitutional Amendment
and other members of the edition of The Guinness Book of
of 1951, the Parliament added
Commission shall be appointed by World Records.
Article 31A to the Indian Constitution.
the President by Warrant under his 28. GC Murmu, who was appointed as
Article 31A of Indian Constitution was
hand and seal. The Commission shall
immune to Article-14 and 19 of Indian the Comptroller and Auditor
have the power to regulate its own
Constitution that provide for Right to General of India in August 2020,
procedure. Article 338 of the Indian
Equality and the Right to Freedom,
respectively.
Constitution deals with National was the Lieutenant Governor/
Commission for Scheduled Castes. Administrator of which one of the
Article 31 (C) of Indian Constitution
was included through the 25th 25. August 12 is celebrated as following Union Territories prior
Amendment Act of 1971 through (a) the World Environment Day to this appointment?
which the government gave primacy
(b) the World No-Tobacco Day (a) Ladakh
to some Directive Principles of State
(c) the International Day against Drug (b) Jammu and Kashmir
Policy over the Fundamental Rights.
Abuse and Illicit Trafficking (c) Chandigarh (d) Puducherry
23. Which one of the following is not (d) the International Youth Day
_ (b) Girish Chandra Murmu is
the necessary condition for the _ (d) The August 12 is celebrated as appointed as the 14th Comptroller
issue of a Writ of Quo Warranto? the International Youth Day, each year and Auditor General of India on
(a) The Office must be a Public Office. to recognise efforts of the World’s August 8, 2020. He was the first
youth in enhancing Global Society. Lieutenant Governor of the Union
(b) The Office must be created by the
It also the day aims to raise Territory of Jammu and Kashmir.
Statute or by the Constitution itself.
awareness about the problems faced He is a 1985 batch Indian
(c) The Office must not be a
by youth. The first International Youth Administrative Service Officer of
substantive one.
Day was observed on August 12, Gujarat Cadre and was Principal
(d) There has been a contravention of
2000. Secretary of Narendra Modi during
the Constitution or a Statute in
The theme of the International Youth his tenure as Chief Minister of
appropriating such person to that
Day, 2020 was ‘Youth Engagement Gujarat.
Office.
for Global Action’. 29. Which one of the following States
_ (c) “The Office must not be a
substantive one” is not the necessary 26. Which one of the following is planned to host the Khelo India
condition for the issue of a Writ of Quo countries had chosen the name Youth Games (4th Edition)?
Warranto. Orders, warrants,
‘Nisarga’ for the cyclone which (a) Kerala (b) Haryana
directions, etc. Issued under authority
are examples of Writ. devastated the coastline of (c) Gujarat (d) Manipur
CDS Solved Paper 2020 (II) 29

_ (b) On July 25, 2020, the Union Minister of Youth Affairs and Sports, Shri Kiren Rijiju 33. As per the use-based classification of
announced that Haryana will host the Fourth Edition of Khelo India Youth Games. The the Index of Industrial Production
games has been scheduled to be conducted after Tokyo Olympics.
(IIP), the maximum weight has been
The Khelo India Scheme was launched to improve sports culture in India. The
Scheme is implemented by the Ministry of Youth Affairs and Sports. The Scheme assigned to
aims to make India a good sporting nation. (a) primary goods
(b) intermediate goods
30. Which one among the following is not a Coral Reef Island? (c) consumer durables
(a) Great Barrier Reef (Australia) (b) Rainbow Reef (Fiji) (d) consumer non-durables
(c) Swaraj Island (India) (d) Kyushu Island (Japan)
_ (a) The Index of Industrial Production (IIP)
_ (d) Kyushu Isalnd (Japan) is not a Coral Reef Island. A coral island is a type of island is an index that indicates the
formed from coral detritus and associated organic material. It occur in tropical and performance of various industrial sectors
sub-tropical areas, typically as part of coral reefs which have grown to cover a far of the Indian economy. It is calculated
larger area under the sea. and published by the Central Statistical
The Great Barrier Reef is the world's largest Coral Reef System composed of over Organisation (CSO), every month. The
2900 individual reefs and 900 islands stretching for over 2,300 km over an area of maximum weight has been assigned to
approximately 344,400 km 2. The reef is located in the Coral Sea, off the Coast of primary goods.Weights of the different
Queensland, Australia. sectors under the used based
classification (2011-12 series)
The Rainbow Reef is a reef in the Somosomo Strait between the Fijian islands of
Taveuni and Vanua Levu. It is one of the most famous dive sites in the South Pacific. Number of
Sectors Weights
Swaraj Island is part of Ritchie's Archipelago, in India's Andaman Islands. It's known Groups
for its dive sites and beaches, like Elephant Beach, with its coral reefs. Primary Goods 15 34.05
31. Since 2014-15, India has consistently run trade surplus with which one Capital Goods 67 8.22
among the following countries?
(a) China (b) Saudi Arabia (c) USA (d) Germany Intermediate 110 17.22
Goods
_ (d) Since 2014-15, India has consistently run trade surplus with USA.
Infrastructure/C 29 12.34
2019-20 onstruction
Trade Status Country 2014-15 2015-16 2016-17 2017-18 2018-19
(April-Nov.) goods
Trade Surplus USA 20.63 18.55 19.90 21.27 16.86 10.91
Country Consumer 86 12.84
durables
Trade Deficit UAE 6.89 9.67 10.87 6.41 0.34 0.25
Countires Consumer 100 15.33
Non-durables
China PRP –48.48 –52.70 –51.11 –63.05 –53.57 –35.32
Saudi Arabia –16.95 –13.94 –14.86 –16.66 –22.92 –14.32 Total 407 100

Iraq –13.42 –9.83 –10.60 –16.15 –20.58 –13.98 34. Normally, there will not be a shift in
Germany –5.25 –5.00 –4.40 –4.61 –6.26 –3.09 the demand curve when
Korea RP –8.93 –9.52 –8.37 –11.90 –12.05 –7.80 (a) price of a commodity falls.
Indonesia –10.96 –10.31 –9.94 –12.48 –10.57 –6.99 (b) consumers want to buy more at any
given price.
Switzerland –21.06 –18.32 –16.27 –17.84 –16.90 –11.97 (c) average income rises.
Hong Kong 8.03 6.04 5.84 4.01 –4.99 –3.88 (d) population grows.
Singapore 2.68 0.41 2.48 2.74 –4.71 –3.15 _ (a) The demand curve is a graphical
Bilateral Trade Surplus/Deficit (Values in US $ Billion) representation of the relationship
between the price of a good or service
and the quantity demanded for a given
32. Arrange the following countries in descending order as per the Global
period of time. If the price of a
Human Development Index, 2019 : commodity falls then there will be no shift
1. Germany 2. USA 3. South Africa 4. India in the demand curve.
Select the correct answer using the codes given below. 35. A market, in which there are a large
(a) 1, 2, 3, 4 (b) 1, 3, 2, 4 number of firms, homogeneous
(c) 3, 2, 1, 4 (d) 4, 3, 2, 1 product, infinite elasticity of
_ (a) Countries in descending order as per the Global Human Development Index, 2019 demand for an individual firm and no
are- Germany (4) USA (15), South Africa (113) and India (129). In HDI 2019, Norway, control over price by firms, is termed
Switzerland, Ireland and Germany topped the HDI ranking of 189 countries and
territories, while Niger, the Central African Republic, South Sudan, and Burundi have
as
the lowest scores in the Index. The United Nations Development Programme (UNDP) (a) oligopoly
ranks countries into four tiers of human development by combining measurements of (b) imperfect competition
life expectancy, education and per-capita income into the Human Development Index (c) monopolistic competition
(HDI) in its annual Human Development Report. (d) perfect competition
30 CDS Solved Paper 2020 (II)

_ (d) Perfect competition is a market 38. Climax Mine, the largest producer 41. The largest Barrier Reef System in
form in which there are a large of Molybdenum, is located in the world is found at
number of firms, homogenous
product, infinite elasticity of demand (a) Canada (b) USA (a) East Australian Coast
for an individual firm and no control (c) Australia (d) South Africa (b) West Australian Coast
over price rise. Perfect competition (c) North Australian Coast
provides both allocative efficiency and
_ (b) The Climax mine, located in (d) South Australian Coast
Climax, Colorado, United States, is a
productive efficiency. major Molybdenum mine. _ (a) The Great Barrier Reef is the
In oligopoly, small group of large Shipments from the mine began in world's largest coral reef. It is found
sellers (oligopolists) dominate the 1915. At its highest output, the Climax off the East Australian coast. It is
market. Monopolistic competition mine was the largest Molybdenum made up of nearly 2900 coral reefs
occurs when an industry has many mine in the world, and for many years and over 600 islands. It is 3,27,800
firms offering products that are It supplied three-fourths of the world's km2 big and 2600 km long. It has
similar but not identical. Imperfect supply of Molybdenum. The mine is been listed as an important World
competition refers to a situation owned by Climax Molybdenum Heritage Site by UNESCO.
where the characteristics of an Company, a subsidiary of It is home to over 1600 species of
economic market do not fulfil all the Freeport-McMoRan.
fish, 411 species of hard coral and
necessary conditions of a perfectly Molybdenum is a chemical element 150 species of soft coral, more than
competitive market. with the symbol Mo and atomic 30 species of whales and dolphins
number 42. and six of the world’s seven species
36. Which one of the following
39. Which one among the following of marine turtles.
statements with regard to ozone is
Union Territories of India is the The Great Barrier Reef is the largest
not correct? structure made by living things. It can
(a) Ozone is found mostly at 15-55 km smallest in geographical area? be seen from outer space. The
in the atmosphere. (a) Chandigarh biggest threat to the Great Barrier
(b) Ozone is produced by gaseous (b) Puducherry Reef today is coral bleaching caused
chemical reactions. (c) Dadra and Nagar Haveli and by high sea water temperatures as a
(c) 16th November is celebrated as Daman and Diu result of global warming.
the International Day for the (d) Lakshadweep
Preservation of the Ozone Layer.
42. During the 19th century, who
(d) Ozone is a form of oxygen in which _ (d) The Union Territory of among the following wrote
Lakshadweep is smallest in area. It
three oxygen atoms are bounded has total area of 32 km 2. It is an
Satapatra Series?
together. archipelago, situated in Arabian Sea (a) MG Ranade
_ (c) September 16 was designated by (Indian Ocean). Its capital is Kavaratti. (b) BG Tilak
the United Nations General Assembly It is divided into two group of islands (c) Bankim Chandra Chatterjee
as the International Day for the i.e. Amindivi (Northern group of (d) GH Deshmukh
Preservation of the Ozone Layer. This islands) and Cannanore (Southern
designation had been made on group of Islands). The Islands are _ (d) ‘Lokahitavadi’ Gopal Hari
made up of coral reef. Deshmukh wrote Satapatra Series
December 19, 2000, in
(1848-50) in which he called for social
commemoration of the date, in 1987, The geographical areas of other given reforms, advocated indigenous
on which nations signed the territories are enterprise, but also welcomed British
Montreal Protocol on Substances that Chandigarh - 14 km 2 rule. Gopal Hari Deshmukh ( 1823 -
Deplete the Ozone layer. The theme
for 2020 World Ozone Day was Puducherry - 483 km 2 1892) was an activist, thinker, social
Dadra and Nagar Haveli and reformer and writer from Maharashtra.
‘Ozone for Life: 35 years of Ozone
Layer Protection’. Daman and Diu - 603 km 2 43. Which one of the following was
37. Sea of Azov is connected to 40. Buenos Aires and Montevideo are not a demand made by the
(a) Black Sea (b) Baltic Sea situated across the banks of Congress moderates?
(c) Mediterranean Sea (a) River Plate (b) Orinoco River (a) Universal Adult Franchise
(d) North Sea (c) Purus River (d) Madeira River (b) Repeal of the Arms Act
(c) Extension of Permanent Settlement
_ (a) The Sea of Azov is a sea in _ (a) Buenos Aires, the capital and (d) Higher Jobs for Indians in the army
Eastern Europe connected to the most populous city of Argentina,
Black Sea by the narrow (about 4 km) located on the western bank of the _ (a) Universal Adult Franchise was not
Strait of Kerch, and is sometimes River Plate. Buenos Aires is one of the a demand made by the Congress
regarded as a Northern Extension of safest cities to visit In South America. moderates.
the Black Sea. The sea is bounded in Montevideo is situated on the The early phase of the Congress
the North-West by Ukraine, in the Southern coast of the country, on the was dominated by the ‘moderates’.
south-east by Russia. The Don River North-Eastern bank of the River Plate. They believed in British rule and were
and Kuban River are the major rivers Montevideo is the capital and largest
loyal to them.
that flow into it. There is a constant city of Uruguay.
outflow of water from the Sea of Azov The city was established in 1724 by a Prominent moderate
to the Black Sea. The Sea of Azov is Spanish soldier, Bruno Mauricio de leaders-Dadabhai Naoroji, Womesh
the shallowest sea in the world, with Zabala, as a strategic move amidst Chandra Bonnerjee, G. Subramania
the depth varying between 0.9 and 14 the Spanish-Portuguese dispute over Aiyer, Sir Surendranath Banerjee and
m (2 ft 11 in and 45 ft 11 in). the platine region. Gopal Krishna Gokhale.
CDS Solved Paper 2020 (II) 31

Major Popular Demands of the (c) the Jats and Yadavs often participating members of its
Moderates (d) the Ahirs and Kurmis committees.
l Decreased land revenue tax and Tej Bahadur Sapru emerged as the
ending peasant oppression.
_ (d) The Triveni Sangh was formed in most important leader among the
1934 by the members of three
l Education of the masses and backward castes of Bihar, namely Liberals.
organising public opinion, make Yadavs (Ahirs), Koer and Kurmi. It
people aware of their rights.
48. The National Disaster
was formed to fight against the
l Indian representation in the political solidarity of ‘middle peasant Management Authority functions
Executive Council and in the Indian castes’ as well as to carve a space in under the Ministry of
Council in London. democratic politics for the lower (a) Environment, Forest and Climate
l Separation of the Executive from the
castes. Its nomenclature was derived Change
judiciary. from the confluence of three mighty (b) Home Affairs
rivers viz. the Ganga, Yamuna and the
l Decreased land revenue tax and (c) Commerce and Industry
mythical Saraswati at Allahabad. The
ending peasant oppression. (d) Finance
Sangh claimed of having atleast one
l After 1892, raised the slogan, ‘‘No
taxation without representation’’.
million dues-paying members. Its _ (b) The National Disaster
formation was countered by the Management Authority (NDMA)
l Abolishing salt tax and duty on formation of Indian National functions under the Ministry of Home
sugar. Congress's Backward class Affairs. It was established through the
l Freedom of speech and expression. Federation, which was established at Disaster Management Act enacted by
l Freedom to form associations.
the same time. the Parliament on December 23rd
2005.
l Development of modern capitalist 46. Who among the following was the
industries in India. NDMA is responsible for framing
first to accept a ministerial policies, laying down guidelines and
l End of economic drain of India by
position in the Central Provinces best practices for coordinating with
the British.
in October 1925? the State Disaster Management
l Repealing the Arms Act of 1878.
(a) BS Moonje (b) MR Jayakar Authorities (SDMAs) to ensure a
44. Who among the following (c) SB Tambe (d) BN Sasmal holistic and distributed approach to
disaster management. Prime Minister
founded the Mohammedan
_ (c) Shripad Balwant Tambe accepted Narendra Modi is the Chairman of
Anglo-Oriental Defence a ministerial position in the central the NDMA. The members enjoy the
Association (1893)? provinces in October, 1925. He was a rank of Secretary of the Union
pledger from Amravati in Berar government.
(a) Auckland Colvin
division of Central Provinces. He was
(b) Badruddin Tyabji a member of the Swaraj Party and 49. The socialist idea of Sapta Kranti
(c) Theodore Beck President of the Central Provinces
(d) Sir Syed Ahmad Khan
(Seven Revolutions) was proposed
Legislative Council. His appointment
created political interest throughout
by
_ (d) Mohammedan Anglo-Oriental (a) Ram Manohar Lohia
Defence Association was established India and the Swaraj Party.
in 1893 by Sir Syed Ahmad Khan as a Balakrishna Shivram Moonje was a (b) Jawaharlal Nehru
part of the Aligarh Movement. He leader of the Hindu Mahasabha in (c) MG Ranade
established it as he considered India. (d) Jayaprakash Narayan
competence in English and Western Makund Ramrao Jayakar was the
Sciences necessary skills for first Vice-Chancellor of the University
_ (a) The socialist idea of Sapta Kranti
(Seven Revolutions) was proposed by
maintaining Muslims' political of Poona. great socialist leader Ram Manohar
influence, especially in Northern India. Birendra Nath Sasmal was a lawyer Lohia.
Also, the growing influence and and political leader. He was known
popularity of the Congress became a His Seven Revolutions include
as Deshpran because of his work for 1. For equality between man and
cause of concern for the British. In
the country and for his efforts in the woman.
order to counter the growing influence
Swadeshi Movement.
of the Congress, the British 2. Against political, economic and
encouraged the formation of the 47. Who among the following formed race-based inequalities.
Mohammedan Anglo-Oriental the National Liberation Federation 3. For the destruction of castes.
(M.A.O.) Defence Association. 4. Against foreign domination.
(Liberal Party)?
Sir Syed Ahmed Taqvi bin Syed 5. For economic equality, planned
Muhammad Muttaqi (a) Motilal Nehru and CR Das
production and against private
(1817-1898), commonly known as Sir (b) Muhammad Ali and CR Das property.
Syed Ahmed Khan, was an Islamic (c) TB Sapru and MR Jayakar
6. Against interference in private life.
pragmatist, reformer, and (d) MR Jayakar and CR Das
7. Against arms and weapons and for
philosopher of nineteenth century. He
_ (c) The National Liberation Federation Satyagraha.
is considered as the pioneer of (Liberal Party) was formed by
Muslim nationalism in India. He provided the ideological and
Surendra Nath Banerjee and some of organisational base for OBC
45. After the First World War, the its prominent leaders were
empowerment. Much of his career
Tej Bahadur Sapru, VS Srinivasa
Triveni Sangh was formed by was devoted to combating injustice
Sastri and MR Jayakar. The Liberal
through the development of a
(a) the Jats and Gujjars Party was formed in 1910, and British
distinctly Indian version of socialism.
(b) the Rajputs and Yadavs intellectuals and British officials were
32 CDS Solved Paper 2020 (II)

50. Which one among the following is _ (d) Seats reserved for Scheduled 54. In August 2020, a blast has
not a character of a Secular State? Castes (SCs) and Scheduled Tribes taken place at Beirut killing
(STs) and the chairpersons of the
(a) It refuses theocracy. Panchayats at all levels also shall be about one hundred people and
(b) It separates religion from the State. reserved for SCs and STs in thousand wounded. The blast was
(c) A State in order to be secular must proportion to their population. Hence, caused by
be democratic. statement (d) is incorrect. The 73rd
(a) dynamite
(d) It must prevent religious conflict Constitutional Amendment Act of
1992 provided Constitutional status to (b) ammonium nitrate
and promote religious harmony.
the Panchayats. The salient features (c) RDX
_ (c) For being a Secular State, it is not of Panchayati Raj System are follows (d) mercury nitride
necessary that State must be democratic.
There are many autocratic states l Three-tier system of panchayats at _ (b) In August 2020, a blast had taken
which are secular. The characteristics village, intermediate place at Lebanon’s capital Beirut. At
of Secular State are as follows block/taluk/mandal and district. least 100 people were killed and
l levels except States with population nearly 4000 injured in this blast. The
l The term ‘Secular’ means being
below 20 lakhs (Article-243B). blast was occurred due to 2700
‘separate’ from religion, or having tonnes of ammonium nitrate stored for
no religious basis. l Panchayat area means the territorial
six years in a warehouse in the port.
l It refuses theocracy.
area of a Panchayat.
Ammonium nitrate (NH NO ) is a
l Gram Sabha includes all people in
l It must prevent religious conflict and white, crystalline chemical which is
promote religious harmony. the electoral rolls of village within a soluble in water. It is the main
Panchayat. ingredient in the manufacture of
l Religion is kept separate from the
l Legislature of a state may, by law, commercial explosives used in mining
social, political, economic and
cultural spheres of life. make provision with respect to the and constructions.
composition of Panchayat.
51. A special address by the Governor 55. What is ‘Little Boy’?
53. Which one of the following (a) The fission bomb dropped at
refers to the address delivered by
statements with regard to Hiroshima
the Governor
‘protective democracy’ is not (b) The fusion bomb dropped at
(a) when President’s Rule is called for Nagasaki
correct?
(b) when a national emergency (c) The first nuclear bomb tested by
necessitates dissolution of (a) It propounds that citizen
participation is essential in America
Legislative Assembly
democracies. (d) The first nuclear bomb tested by
(c) at the commencement of the first North Korea
session after general election and at (b) Citizens must be able to protect
the first session of each year themselves from governmental _ (a) Little Boy is a codename of the
(d) whenever he/she has concluded that encroachments. fission bomb dropped at Hiroshima (a
such is necessary (c) It is compatible with laissez-faire city in Japan) on August 6, 1945 by
US. Uranium was used in it as a fissile
capitalism.
_ (c) Article-176(1) of the Constitution of material. On August 9, 1945, US
India provides that the Governor shall (d) Political equality is understood dropped another bomb codenamed
Address State’s Assembly at the in formal terms as equal voting Fat Man, on Nagasaki. These
commencement of the first Session rights. bombing marked the end of World
after each general election to the War II, with Japan surrendering to the
Assembly and at the commencement _ (c) John Locke (1631-1704) is Allies on August 14, 1945. Thousands
regarded as the great apostle of
of the first session of each year and more died in the following years due
protective democracy.
inform the Legislature of the causes of to the exposure to radiation from the
its Summons. The basic features of protective blast and also from the black rain
The aforementioned address of the democracy are as follows that fell in the aftermath of the
Governor is referred as Special l It propound that the citizen explosions.
Address. The executive power of the participation is essential in
State is vested in the Governor. He is democracies. 56. Which one of the following Indian
appointed by the President by warrant l Both the popular sovereignty and institutes was approved by the
under his hand and seal. representative form of government Drugs Controller General of India
52. Which one of the following are legitimate. for conducting human trials of the
statements in relation to l Citizens must be able to protect Oxford-Astra Zeneca COVID-19
Panchayats is not correct? themselves from governmental vaccine candidate?
encroachments.
(a) Legislature of a State may, by law, (a) Bharat Biotech
make provisions with respect to l Political equality is understood (b) AIIMS
the composition of Panchayats. in formal terms as equal voting (c) Serum Institute of India
(b) Panchayat area means the rights. (d) National Institute of Epidemiology
territorial area of a Panchayat. l The authority is accountable to the
(c) Gram Sabha includes all persons People and in order to establish _ (c) Serum Institute of India (SII) was
approved by the Drug Controller
in the electoral rolls of village within it elections are held on regular General of India for conducting trials
a Panchayat. basis. of the Oxford-Astra Zeneca
(d) Reservation of seats for SCs and l Constitution is the source of power Coronavirus Vaccine. ChAdOx1
STs has nothing to do with for all and is the guarantor of rights nCoV-19 or AZD1222 has been
proportion of their population. and liberties. developed by the Oxford University
CDS Solved Paper 2020 (II) 33

and drug maker Astra Zeneca (Anglo- Continent Country 62. Which one of the following is
Swedish pharma major).
South Argentina, Brazil, Chile considered as the deepest point of
SII is the world’s largest vaccine America Paraguay
manufacturer by the number of
the oceans?
doses produced and sold Africa Namibia, Botswana, (a) Tonga Trench
globally. It has a manufacturing South Africa, (b) Mariana Trench
Mozambique, (c) Philippine Trench
partnership with Astra Zeneca to
Madagascar
produce the Oxford- Astra Zeneca (d) Kermadec Trench
vaccine. Australia Australia
_ (b) The Mariana Trench is considered
57. The recent explosion near OIL Uruguay is not located on Tropic of as deepest point of the oceans. It is
Capricorn. located in the western Pacific Ocean,
well in Baghjan is due to East of the Mariana Islands. The
(a) removing the spool during the 60. Match List-I with List-II and select maximum known depth is 10,984 m
blowout control operations (36,037 ft) at the southern end of a
the correct answer using the codes
(b) transfer of oil from its depot to a small slot-shaped valley its floor
pipeline given below the Lists. known as the Challenger Deep.
(c) the leakage of methyl isocyanide List-I List-II Other Important Trenches of the world
(d) the leakage of radiations from (Active Volcano) (Location)
Trench Ocean
radioactive substance A. Mount Merapi 1. Hawaii
Tonga Trench Pacific Ocean
_ (a) On May 27th, 2020, an explosion B. Sakurajima 2. Italy
Philippine Trench Pacific Ocean
was occurred near OIL well in
C. Mount Vesuvius 3. Japan Kuril-Kamchatka Pacific Ocean
Baghjan (Assam) at the time of
removing spool during the blowout D. Mauna Loa 4. Indonesia Trench
control operation. The blowout Kermadec Trench Pacific Ocean
occurred at Well No. 5 in the Oil Codes
Field, resulting in a leak of natural A B C D Izu-Bonin Trench Pacific Ocean
gas. (a) 1 2 3 4 Japan Trench Pacific Ocean
(b) 1 3 2 4
The leaking well subsequently caught (c) 4 2 3 1 Puerto Rico Trench Atlantic Ocean
fire and has caused environmental (d) 4 3 2 4 South Sandwich Atlantic Ocean
damage to the nearby Trench
Dibru-Saikhowa National Park. _ (d) Correct match are as follows
Peru-Chile Trench or Pacific Ocean
58. Who among the following is Active Volcano Location Atacama Trench
the architect of the Ram Mount Merapi Indonesia Java Trench Indian Ocean
Temple being constructed at Sakurajima Japan
Ayodhya? 63. The four planets closest to the Sun
Mount Vesuvius Italy
(a) PO Sompura
are called
Mouna Loa Hawaii (a) Terrestrial planets (b) Giant planets
(b) Chandrakant Sompura
(c) Brinda Somaya (c) Dwarf planets (d) Gas planets
61. Which one of the following is not
(d) BV Doshi _ (a) The four planet closest Sun are
a major tectonic plate? called Terrestrial planets. It includes
_ (b) Chandrakant Sompura is the (a) Saudi Arabian plate Mercury, Venus, Earth, and Mars.
architect of the Ram Temple, being
constructed at Ayodhya (UP). He is a (b) Antarctica and the surrounding Terrestrial planets are Earth-like
Ahmedabad-based architect whose oceanic plate planets made up of rocks or metals
design was approved by Shri Ram (c) India-Australia-New Zealand plate with a hard surface. Terrestrial planets
Janmabhoomi Teerth Kshetra.The (d) Pacific plate also have a molten heavy-metal core,
stone laying ceremony of the Ram and topological features such as
Temple was done by the Prime _ (a) Lithosphere is divided in different valleys, volcanoes and craters.
tectonic plates. These plates are of
Minister Narendra Modi in August two types- Major and Minor. There 7 64. Which one of the following
2020. major plates and 20 minor plates. countries does not have Tundra
59. Which one of the following Major Plates are as follows vegetation?
countries is not located on the 1. North American plate (a) Belarus (b) USA
Tropic of Capricorn? 2. South American plate (c) Russia (d) Canada
(a) Chile 3. Pacific plate
_ (a) Belarus do not have Tundra
(b) Brazil 4. Antarctica and the surrounding vegetation. The tundra is the coldest
(c) Paraguay oceanic plate biome. It also receives low amounts of
(d) Uruguay 5. Eurasia and the adjacent oceanic precipitation, making the tundra
plate similar to a desert. Scattered trees
_ (d) The Tropic of Capricorn is one of 6. Africa with the eastern Atlantic floor grow in some tundra regions. The
the five major circles of latitude tundra soil is rich in nitrogen and
plate
marked on maps of Earth. Its latitude phosphorus. Tundra is found in the
is 23°26’11.7’’ South of the Equator. 7. India-Australia-New Zealand plate
regions just below the ice caps of the
There are 10 countries, 3 continents Saudi Arabian plate is a minor plate. Arctic, extending across North America,
and 3 water bodies on it. Hence, option (a) is correct. to Europe, and Siberia in Asia.
34 CDS Solved Paper 2020 (II)

65. Which one of the following is not 68. Who among the following wrote He fought against the British forces in
the Awadh region of Uttar Pradesh. It
a fluvial landform? The Philosophy of the Bomb?
was like a manifesto in which aims
(a) Cirque (b) Gorge (a) Sukhdev were set out for what rebels were
(c) Braids (d) Canyon (b) Chandrashekhar Azad fighting for. This proclamation
(c) Bhagwati Charan Vohra appealed both Hindus and Muslims
_ (a) Cirque is a glacial landform. The (d) Bhagat Singh
landform created by the glacier are to cooperate with each other in fight
known as glacial landform. Aretes, against the Britishers.
_ (c) The Philosophy of the Bomb was
Nunatak, Esker etc. are important written by the revolutionary freedom
glacial landforms.
72. Ibn Batuta went to China as the
fighter Bhagwati Charan Vohra. He
was associated with Hindustan
envoy of which one of the
66. In the Gandhara School of Art, Socialist Republican Association. He following Delhi Sultans?
initially blue schist and green died in Lahore on May 28, 1930 while (a) Alauddin Khilji
phyllite were used. When did testing a bomb. (b) Muhammad bin Tughluq
stucco completely replace stone as 69. At which one of the following (c) Iltutmish
main material used by Gandhara (d) Firoz Shah Tughluq
Sessions of the Indian National
School sculptors? Congress was the resolution on _ (b) Muhammad bin Tughlaq
(a) 1st Century CE (AD 1324-51) had sent Ibn Batuta
Fundamental Rights and Economic (Moroccan Traveller) to China as the
(b) 2nd Century CE
Policy passed? envoy. Ibn Batuta arrived India in
(c) 3rd Century CE
(a) Tripuri Session AD 1334. He wrote about the history
(d) 5th Century CE
(b) Lahore Session of the Tughlaq dynasty.
_ (c) The Gandhara school of Art (c) Lucknow Session
flourished in the areas of Afghanistan 73. Al-Biruni’s Kitab-ul-Hind was
(d) Karachi Session
and present North-Western India. It written in which language?
flourished from 1st Century BCE to _ (d) At the Karachi Session (1931), the (a) Arabic (b) Persian
4th Century BCE. The material used Indian National Congress passed to
(c) Urdu (d) Turkish
for in the sculpture making were resolution on Fundamental Rights
green phyllite and gray blue schist (FR) and Economic policy. _ (a) Al-Biruni’s Kitab-ul-Hind was
and Stucco, which was used written in Arabic language. He came
increasingly after the 3rd Century 70. Which one of the following towns to India with Mahmud Ghazni in
BCE. This school of Art is associated was not a centre of the Revolt of 11th Century AD and lived here for
with the Greco-Roman style of Art. 1857? many years. He also learnt Sanskrit
The main theme of this art were language. In this book, he described
(a) Ayodhya (b) Agra the socio-economic, political, religious
derived from the Mahayana
(c) Delhi (d) Kanpur and economic condition of the then
Buddhism.
67. Which one of the following _ (b) Agra was not a centre of the India.
Revolt of 1857. In 1857, people of
statements about Gupta coins is country started a revolt against the 74. Which of the following statements
not correct? exploitative British government. The with regard to the privileges of the
revolt began on 10th May 1857, at Members of the Parliament are
(a) Gupta kings issued large number
Meerut in the form of sepoy mutiny. correct?
of gold coins known as Dinar.
The political economic and
(b) Chandragupta II, Kumaragupta I, socio-religious activities of British 1. Privileges would not be fettered
Skandagupta and Budhagupta government were responsible for the by the Article-19(1)(a) of the
issued silver coins. revolt. Constitution of India.
(c) The obverses of coins are carved
71. Consider the following statements : 2. Privileges must be read subject to
with the images of the kings and
on the reverse are carved deities. the Articles-20-22 and Article-32
The Azamgarh Proclamation refers of the Constitution of India.
(d) The largest number of coins issued to
by the Guptas were of copper. 3. Immunity is available in relation
1. the declaration by the rebels of to both civil and criminal
_ (d) Gupta king issued large number 1857.
of gold coins known as ‘Dinars’. prosecution.
Gupta coins were usually minted in 2. the statement by the leader of the 4. Immunity is available in relation
gold and silver. The obverses of coins underground movement in the to freedom of speech even in
were carved with the images of the Revolt of 1942. his/her private or personal capacity.
kings in the reverse with deities. Which of the statements given above
Gupta kings like Chandragupta II, Select the correct answer using the
is/are not correct? codes given below.
Kumaragupta I, Skandagupta and
Budhagupta issued silver coins. (a) Only 1
(a) 1, 2 and 4 (b) 1 and 2
Gupta kings also depicted (b) Only 2
(c) 2 and 3 (d) 1 and 4
socio-political event such as marriage (c) Both 1 and 2
of the kings and queens, Ashvmedha (d) Neither 1 nor 2 _ (b) Every Member of the Parliament
Yagya etc. enjoys certain privileges so that they
Gupta dynasty reigned from 319 to
_ (b) The Azamgarg proclamation was can carry out their duty without
declared by the Feroz Shah, interference. Privileges are of two
467 CE. It covered much of the (Grandson of the Mughal Emperor) in
Indian subcontinent. type-Collective and Individual.
1857.
CDS Solved Paper 2020 (II) 35

Individual Privileges of Member of in India as outlined the Election _ (a) BN Rau was appointed as the
Parliament include. Commission. The Commission Constitutional Adviser to the
l Immunity from assest during the
classifies parties into three main Constituent Assembly in 1946.
session of Parliament. This privilege heads: National Parties, State Parties, He created general draft of the
is available in civil case only. and Registered (unrecognised) Constitution of India. He was also
Parties. India's representative to the United
l A MP may refuse to appear in court
or present any evidence, during a Political parties in India are classified Nations Security Council from 1950 to
Parliamentary session. for the allocation of symbols. 1952.
Currently, there are 8 National BR Ambedkar was the President of
l No member is liable to any
Parties. It includes the Indian the Drafting Committee of the
proceeding in any given court for Constituent Assembly.
anything said or any vote by National Congress (INC), the
Nationalist Congress Party (NCP), Pattabhi Sitaramayya played
him/her in the Parliament or its instrumental role in the creation of
committees. the Bharatiya Janata Party (BJP), the
Communist Party of India (CPI), the Andhra Pradesh in 1953.
Privileges would not be fettered by Alladi Krishnaswamy Iyer was the
Communist Party of India, Marxists
the Article-191(a) of the Constitution. member of the Drafting Committee of
(CPI-M), the Bahujan Samaj Party
And it must be read subject to the the Constituent Assembly.
(BSP), and the Rashtriya Janata Dal
Article 20-22 and Article-32 of the
(RJD). 79. In the Indian judicial system, writs
Constitution. Privileges does not
cover freedom of speech in his/her 77. Which of the following terms are issued by
private or personal capacity. were added to the Preamble of the (a) the Supreme Court only
75. Which one of the following Constitution of India by the (b) the High Courts only
statements with regard to the Constitutional Amendment, 1976? (c) the Supreme Court and High
Courts only
appointment of the Members of 1. Socialist 2. Secular (d) the Supreme Court, High Courts and
the Parliamentary Committees is 3. Integrity 4. Fraternity Lower Courts
correct? Select the correct answer using the _ (c) The Constitution of India under
(a) The Members are only appointed codes given below. Article-32 and 226 empowers the
(b) The Members are only elected (a) 1 and 2 (b) 1, 2 and 3 Supreme Court of India High Court to
(c) The Members are only nominated. issue writs respectively. Writs are a
(c) 2 and 4 (d) 1, 3 and 4
(d) The Members are appointed or written order that commands
elected on a motion made and _ (b) The 42nd Amendment Act, 1976 constitutional remedies for Indian
adopted or nominated by the added words ‘Socialist’, ‘Secular’ and Citizens against the violation of their
‘Integrity’ in the Preamble of the Fundamental Rights. Writs are of five
Speaker of the Lok Sabha or the
Constitution of India. This Amendment types, namely, Habeas Corpus,
Chairman of the Rajya Sabha
is termed as Mini Constitution. Other Prohibition, Mandamus, Certiorari and
_ (c) Parliamentary Committees are Provision of this Amendment- Quo-Warranto. The High Courts can
constituted to perform various l Added Fundamental Duties by the issue writs not only for the
functions. Members are appointed or citizens (new Part IV A). preservation of Fundamental Rights,
elected on a motion made and but also for any other purpose.
l Made the President bound by the
adopted or nominated by the Speaker However, the Supreme Court can
advice of the cabinet.
of Lok Sabha or the Chairman of the issue writs only for the preservation of
l Provided for administrative tribunals
Rajya Sabha. It has a secretariat Fundamental Rights.
provided by the Lok Sabha/Rajya and tribunals for other matters
Sabha secretariat. Indian Constitution (Added Part-XIV A). 80. The Citizenship (Amendment) Act falls
mentions two kinds of Parliamentary l Frozen the seats in the Lok Sabha under which one of the following
Committees - Standing Committees and State Legislative Assemblies on parts of the Constitution of India?
and Ad Hoc Committees. the basis of 1971 census till 2001-
Population Controlling Measure (a) Part I (b) Part II
Standing Committees are permanent
l Made the constitutional
(c) Part IV (d) Part VI
committees and are constituted for a
fixed tenure. Ad hoc Committees are amendments beyond judicial _ (b) The Citizenship (Amendment) Act
appointed for a specific purpose and scrutiny. falls under Part II of the Constitution of
they cease to exist when they finish l Curtailed the power of judicial India. This part contains Articles-5-11.
the task assigned to them after review and writ jurisdiction of the Article Provision
submitting the report. Supreme Court and High Courts.
5 Citizenship at the commencement of
l Raised the tenure of Lok Sabha and
76. Which one of the following is not State Legislative Assemblies from
the Constitution
a classified category of political 5 to 6 years. 6 Citizenship of certain persons who
parties as outlined by the Election have migrated from Pakistan
78. Who among the following was the 7 Citizenship of certain migrants to
Commission of India?
advisor to the Constituent Pakistan
(a) National Parties
Assembly? 8 Citizenship of certain persons of Indian
(b) State Recognised Parties
(c) Regional Parties (a) BN Rau origin residing outside India
(d) Registered Unrecognised Parties (b) BR Ambedkar 9 People voluntarily acquiring citizenship
(c) Pattabhi Sitaramayya of a foreign country will not be citizens
_ (b) State Recognised Parties is not a (d) Alladi Krishnaswamy of India.
classified category of political parties
36 CDS Solved Paper 2020 (II)

Article Provision 83. The Government of India 86. If the linear momentum of a
10 Any person who is considered a citizen programme regarding ‘Stay in moving object gets doubled due to
of India under any of the provisions of India and Study in India’ is initiated by application of a force, then its
this Part shall continue to be citizens
and will also be subject (a) the Ministry of Youth Affairs and kinetic energy will
Sports (a) remain same
to any law made by the Parliament.
(b) the Ministry of Culture (b) increase by four times
11 The Parliament has the right to make (c) the Ministry of Education (c) increase by two times
any provision concerning the
(d) the Ministry of Tourism (d) increase by eight times
acquisition and termination of
citizenship and any other _ (c) A programme called ‘Study in _ (b) Let m be the mass and v be the
matter relating to citizenship. India - Stay in India’ was initiated by velocity of the moving object.
the Ministry of Education in August, Kinetic energy of object,
2020. The objective of this plan is to 1 1
81. Recently a rare kind of yellow prevent students from leaving the K = mv 2 = ( m 2v 2 )
turtle was discovered in India. The country seeking higher education 2 2m
1
State in which it was seen is abroad and also to bring back Indian K = ( mv )2 …(i)
students studying abroad. 2m
(a) Uttarakhand
For this initiative a committee has Now, linear momentum,
(b) Odisha
(c) Tamil Nadu
been constituted under the p = mv …(ii)
leadership of DK Singh to prepare Substituting Eq (ii) in Eq (i), we get
(d) Arunachal Pradesh
guidelines and measures to
p2
_ (b) A rare Yellow Turtle was strengthen both Study in India - Stay K = ⇒ K ∝ p2
discovered in India from Balasore in India. 2m
district of Odisha. The colour of turtle If linear momentum gets doubled,
was due to albinism. This turtle is 84. Which one of the following lakes
then new kinetic energy will become
known as the Indian flap shell turtle. in India has a large quantity of a
This turtle is commonly found in (2 p)2
substance found in the Moon? K′ =
Pakistan, Sri Lanka, India, Nepal, 2m
Bangladesh, and Myanmar. Albinism (a) Lonar Lake (Maharashtra)
(b) Pangong Lake (Ladakh) 4 p2
is a type of genetic disorder where it =
is little or no production of pigment in (c) Chilika Lake (Odisha) 2m
the skin, eyes, and hair or in other (d) Loktak Lake (Manipur) K ′ = 4K
species in the fur, feathers or scales.
_ (a) Lonar Lake is in Indian State of Hence, if linear momentum of a
82. ASEEM is Maharashtra and it has a large moving object gets doubled, its
quantity of a substance found in the kinetic energy will increase by four
(a) Aatmanirbhar Skilled Employee Moon. In March 2019, IIT-Bombay times.
Employer Measurement had found in research that mineral
(b) Aatmanirbhar Skilled Employee contents of Lonar lake is similar to 87. If the distance between two
Employer Mapping Moon rocks. This lake was formed objectsi s increased by two times,
(c) Aatmanirbhar Skilled Employee after a meteorite crash around 50,000 the gravitational force between them
Enterprises Medium years ago.
will
(d) Automatic Skilled Employee
_ The Lonar Crater is also a wildlife (a) remain same
Employer Mission sanctuary and a notified Geo-Heritage (b) increase by two times
_ (b) ASEEM is an acronym of monument. It is home to a variety of
(c) decrease by two times
'Aatamanirbhar Skilled Employee flora and fauna.
(d) decrease by four times
Employer Mapping' portal. It was
85. The Pragyan rover installed in
launched by the Ministry of Skill _ (d) The gravitational force between
Development and Entrepreneurship Chandrayaan-2 mission had how two objects of masses m1 and m2,
(MSDE) in July 2020 to help skilled many wheels? separated by a distance r is given by
people in fighting sustainable (a) 2 F =
Gm1m2
…(i)
livelihood opportunities. (b) 3 r2
It is Managed by the National Skill (c) 4 where, G is the universal gravitational
Development Corporation (NSDC) in (d) 6 constant.
collaboration with Bengaluru-based If the distance between the objects is
company ‘Betterplace’. _ (d) Indian Space Research increased by two times, the
Organisation (ISRO) launched
The ASEEM digital platform consists Mission Chandrayaan-2 in July 2019 gravitational force will become,
of three IT-based interfaces Gm1m2
(Mission was unsuccessful). This F′ =
Employer Portal Employer onboarding, Mission was comprised of Vikram, the (2 r )2
Demand Aggregation, candidate lander and Pragyan,the rover. Gm1m2
selection Pragyan was a six-wheeled =
4r 2
Dashboard Reports, Trends, solar-powered vehicle. It was sent to
F
analytics, and highlight gaps study the composition of the surface ⇒ F′ = [Using Eq (i)]
near the lunar landing site and 4
Candidate Application Create and
determine the abundance of various Hence, the gravitational force will
Track candidate profile, share job
elements. decrease by four times.
suggestion
CDS Solved Paper 2020 (II) 37

88. Which one of the following _ (a) All the individuals of a particular _ (a) A mixture of sodium chloride (salt)
statements about the properties of organism, such as rose plants and ammonium chloride can be
belongs to a taxonomic category separated by sublimation. This
neutrons is not correct? called species because species is a technique involves the conversion of a
(a) Neutron mass is almost equal to group of individual organisms with solid direct into vapour. On heating,
proton mass. fundamental similarities. ammonium chloride changes from
(b) Neutrons possess zero charge. solid state to ammonia vapours. Here,
93. Pearls are harvested from ammonium chloride can be sublimed
(c) Neutrons are located inside the
atomic nuclei. (a) prawn (b) pila whereas sodium chloride cannot be
(d) Neutrons revolve around the (c) tuna (d) oyster sublimed.
atomic nuclei. _ (d) Pearls are harvested from oyster. 97. Symbol of element was introduced
A pearl is an ulcer formed in the
_ (d) Statement (d) is not correct about oysters when an unknown outside
by
the properties of neutrons, whereas all (a) John Dalton
other statements are correct. particle get into the shell. It is due to
the stress that the oysters produce (b) Antoine Lavoisier
Correct Statement Electrons revolve nacre that will eventually become the (c) Jons Jacob Berzelius
around the atomic nuclei, not neutrons. pearl in a couple of years. So, (d) Robert Boyle
89. Which one of the following basically the pearl is the result of a
disease in the oyster’s body. _ (a) Symbol of element was introduced
statements with regard to the by John Dalton. Symbols are the
ultraviolet light is not correct? 94. Wings of birds and bats are representation of an element. It is
simple to use the symbol of an
(a) It is an electromagnetic wave considered analogous structures element rather than writing a whole
(b) It can travel through vacuum because they have word.
(c) It is a longitudinal wave (a) common origin and common Some symbols along with elements
(d) Its wavelength is shorter/smaller function are as follows :
than that of visible light (b) different origin and common u Hydrogen Carbon
function
_ (c) Statement in option (c) is not Phosphorus ⊕ Sulphur
correct about the ultraviolet light. (c) common origin and different  Copper L Lead
The correct statement is : function
Ultraviolet light is a transverse wave.
(d) different origin and different 98. Identify the correct pair of
function elements among the following
90. If the speed of light in air is
_ (b) Wings of birds and bats looks which are liquid at room
represented by c and the speed in a alike and perform same function of temperature and standard pressure.
medium is v, then the refractive flying. But anatomically they are different
in origin. Birds have feathers projecting (a) Bromine and Fluorine
index of the medium can be (b) Mercury and Rubidium
back from lightweight, fused arms
calculated using the formula and hand bones. Bats have flexible, (c) Bromine and Thallium
(a) v/c (b) c/v relatively short wings with membranes (d) Bromine and Mercury
(c − v ) stretched between elongated fingers.
(c) c / (2. v ) (d)
Thus, these are analogous. _ (d) Bromine and Mercury is the pair of
c element which are liquid at room
95. Apart from hyper acid secretion, temperature and standard pressure.
_ (b) Refractive index of medium
Speed of light in air Bromine is a liquid non-metal whereas
= =
c peptic ulcers are also developed mercury is a liquid metal.
Speed of light in medium v due to bacterial infection. The
causative agent is 99. Which one of the following oxides
91. Under the kingdom– Plantae,
(a) Helicobacter pylori shows both acidic and basic
which of the following individuals
(b) E. coli behaviour?
are predominantly aquatic?
(c) Streptococcus pneumoniae (a) Zinc oxide (b) Copper oxide
(a) Bryophytes (b) Algae (d) Salmonella typhimurium (c) Magnesium oxide
(c) Pteridophyta (d) Gymnosperms (d) Calcium oxide
_ (a) Helicobacter pylori is the causative
_ (b) Under the kingdom– plantae, agent of peptic ulcer. It is a sore that _ (a) Zinc oxide (ZnO) shows both
Phyllum-Algae have individuals that develops on the living of the acidic and basic behaviour. It reacts
are predominantly aquatic (both fresh oesophagus, stomach or small with both acids and bases to form salt
water and marine). intestine. Ulcer occurs when stomach and water.
These are chlorophyll bearing simple, acid damages the living of the ZnO( s ) + 2HCl( aq ) → ZnCl 2( aq ) + H 2O( l )
thalloid, autotrophic organism. They digestive tract. Common causes of Zinc Hydrochloric Zinc Water
occurs in a variety of habitats like ulcer include the bacteria H. pylori oxide acid chloride
standing water, moist stones, soils and anti- inflammatory pain relievers ZnO( s) + 2NaOH( aq ) → Na 2ZnO 2( aq ) + H 2O
and woods. including aspirin. Zinc Sodium Sodium Water
oxide hydroxide zincate
92. All the individuals of a particular 96. A mixture of sodium chloride (salt)
organism, such as rose plants, belong and ammonium chloride can be 100. Silver articles turn black when
to a taxonomic category called separated by kept in the open for longer time
(a) Species (b) Genus (a) sublimation (b) filtration
due to the formation of
(c) Family (d) Order (c) chromatography (d) distillation (a) H2 S (b) AgS (c) AgSO 4 (d) Ag 2 S
38 CDS Solved Paper 2020 (II)

_ (d) Silver articles turn black when kept _ (c) When a light ray enters into glass 107. Which one of the following
in the open for longer time due to the medium from water at angle of statements about phloem is
formation of silver sulphide (Ag 2S). incidence 0°, it will fall normally on the
Silver metal reacts with sulphur glass medium. correct?
present in the atmosphere and forms (a) Phloem transports water and
Normal
Ag 2S. Complete reaction is as follows minerals.
2Ag + S → Ag 2 S Water (b) Phloem transports photosynthetic
Glass products.
101. Which of the following lenses will (c) Phloem is a simple tissue.
bend the light rays through largest (d) Phloem gives support to the plant.
angle? If the incident ray falls normally to the
_ (d) Option (b) is correct as the phloem
(a) Lens with power + 2.0 D surface of glass medium, then there is the means by which the products of
(b) Lens with power + 2.5 D will be no bending of the light ray. It photosynthesis are transported to
(c) Lens with power – 1.5 D goes straight without any deviation. non-photosynthetic parts of the plant,
(d) Lens with power – 2.0 D ∴Angle of refraction = 0° such as the roots and to developing
leaves, nectaries, fruits and seeds.
_ (b) The power of a lens is defined as 104. Which one of the following Other statements can be corrected as
its ability to bend the light rays falling on phenomena verifies the fact that l Xylem transports water and
it. So, the lens with more power will
bend the light rays through largest light travels much faster than minerals.
angle. sound? l Phloem is a complex tissue.

∴The lens with + 2.5 D will bend the (a) Twinkling of stars in night sky l Primary function of phloem is

light rays through largest angle. (b) Lighting of a matchstick transport of sugars not support.
(c)Thunderstorm 108. Mature sclerenchyma cells have
102. A luminous object is placed at a
(d) Mirage
distance of 40 cm from a converging (a) cellulose wall and are living
lens of focal length 25 cm. The _ (c) In case of a thunderstorm, the (b) lignified wall and are living
lightning is seen first and then we (c) suberized wall and are dead
image obtained in the screen is hear the thunder after some time. (d) lignified wall and are dead
(a) erect and magnified This is because light travels with
(b) erect and smaller speed 3 × 10 8 ms −1 in air while _ (d) Mature Sclerenchyma consists of
long, narrow cells with thick and
(c) inverted and magnified sound travels at a speed of 340 ms −1
lignified cell walls having a few or
(d) inverted and smaller in air. This phenomenon proves the
numerous pits.
fact that light travels much faster than
_ (c) Given, sound.
They are usually dead and without
Object distance, u = − 40 cm protoplasts.
Focal length, f = 25 cm 105. Which one of the following 109. In human beings, the
Let v be the image distance. combinations of source and screen chromosomes that determine birth
From lens formula, would produce sharpest shadow of of a normal female child are
1 1 1 an opaque object?
= − (a) one X chromosome from mother
f v u (a) A point source and an opaque and one X chromosome from
1 1 1 1 1 screen father
⇒ = + = +
v f u 25 ( − 40) (b) An extended source and an (b) one X chromosome from mother
1 1 1 8−5 3
opaque screen and one Y chromosome from
⇒ = − = = (c) A point source and a transparent father
v 25 40 200 200
screen (c) two X chromosomes from mother
200
⇒ v = = 667. cm (d) An extended source and a and one X chromosome from
3 transparent screen father
Now, magnification produced by (d) one X chromosome and one Y
lens, _ (c) To form the sharpest shadow of
an opaque object, a point source and chromosome from father and one
v 667.
m= = = − 1.66 a transparent screen are required. X chromosome from mother
u ( − 40)
106. Which one among the following is _ (a) The chromosome number of a
Since, the magnification is negative, normal female is 44 + XX. It has one
so the image formed will be real and a non-conventional source of X-chromosomes which comes from
inverted. energy? mother and one X-chromosome
Also, the magnitude of magnification (a) Petroleum which comes from father.
is greater than unity, so the image (b) Coal Thus option ‘A’ is correct.
will be magnified. (c) Radioactive elements l Fusion of two X from mother and

(d) Solar energy one X from father will lead to birth of


103. When a light ray enters into glass superfemale (XXX).
medium from water at an angle of _ (d) Non-conventional sources of l Fusion of one X-chromosome
energy are those sources of energy
incidence 0°, what would be the which are being produced and one Y-chromosome from
angle of refraction? continuously in nature and are father and one X-chromosome
(a) 90° (b) 45° inexhaustible. Amongst the given from mother will lead to birth of a
(c) 0° options, solar energy is a child with Klienfelter syndrome
non-conventional source of energy. (XXY).
(d) The ray will not enter at all
CDS Solved Paper 2020 (II) 39

110. Antibiotic such as penicillin blocks 115. Lead nitrate on heating gives Which one of the following islands
(a) cell wall formation in bacteria (a) PbO and NO was not connected initially?
(b) RNA synthesis in bacteria (b) PbO and NO (a) Shaheed Island
(c) DNA synthesis in bacteria (c) PbO and NO (b) Swaraj Island
(d) division in bacteria (d) PbO and NO (c) Little Andaman
_ (a) Antibiotics such as penicillin kills _ (b) Lead nitrate on heating gives PbO (d) Port Blair
bacteria by inhibiting or blocking the (lead oxide) and NO (nitrogen
_ (a) In August 2020, submarine Optical
proteins which cross-link pepti- dioxide). Complete reaction is as Fibre Cable (OFC) connecting
doglycans in the cell wall. When these follows Andaman and Nicobar Islands to the
heat
bacterium divides in the presence of 2Pb(NO 3 )2( s )  → 2Pbo( s) mainland was inaugurated. Services
penicillin, it cannot fill in the ‘holes’ left Lead Nitrate Lead Oxide began from Chennai to Port Blair, Port
Ceohite salt Residue
in its cell wall. Blair to Little Andaman and Port Blair
+ 4NO 2( g ) + O 2( g ) to Swaraj Island. Hence, Shaheed
111. The radioactive isotope of Nitrogen dioxide Island was not connected initially. The
hydrogen is (brown)
project of laying 2300 km optical fibre
(a) protium (b) deuterium 116. Which one of the following is a started in December, 2018.
(c) tritium (d) hydronium decommissioned aircraft carrier? A submarine optical fibre
_ (c) Tritium is the radioactive isotope (a) INS Rajput (b) INS Chakra communications cable is a cable laid
of hydrogen. The nucleus of tritium (c) INS Khanderi (d) INS Viraat on the seabed between land-based
contains one proton and two stations to transmit tele-
neutrons. It is represented as 3H or T. _ (d) Amongst the given options, INS communication signals across
Viraat is a decommissioned aircraft
It is used as a radioactive tracer, in stretches of ocean and sea.
carrier. It was a Centaur-class aircraft
radioluminescent light sources for
watches and instruments.
carrier of the Indian Navy. India 119. In August 2020, who among the
acquired this aircraft carrier from following was administered the
112. Which one of the following is used England and commissioned into the
oath as the Prime Minister of Sri
for storing biological tissues? Indian Navy on May 12th, 1987.
Viraat was formally decommissioned Lanka for the fourth time?
(a) Liquid nitrogen (b) Liquid helium on March 6th, 2017. (a) Gotabaya Rajapaksa
(c) Liquid argon (d) Liquid bromine INS Khanderi is a Kalvari-class (b) Basil Rajapaksa
_ (a) Liquid nitrogen is used for storing submarine, built in India. INS Rajput (c) Mahinda Rajapaksa
biological tissues. This process is is a guided-missile destroyer. INS (d) Namal Rajapaksa
known as Cryo preservation. Chakra is a nuclear-powered attack
In this process, organelles, cells tissues submarine. _ (c) Mahinda Rajapaksa was
administered oath as the Prime
or other biological lonsbuets susceptible
117. Who among the following has Minister of Sri Lanka for the fourth
to damage caused by unregulate time in August 2020. He is leader of
Chemical kinetics are preserved by won the Singles Title in
Sri Lanka People’s Party (SLPP)
cooling to very low temperatures. Wimbledon Tennis Championship leader. His party registered victory in
113. Which one of the following does (Women) in the year 2019? 145 constituencies, bagging a total of
(a) Karolina Pliskova (b) Simona Halep 150 seats with its allies, a two-thirds
not form oxide on reaction with majority in the 225-member
oxygen? (c) Serena Williams (d) Naomi Osaka
Parliament. He has replaced Ranil
(a) Magnesium (b) Lead _ (b) Simona Halep (Romania) won the Wickremesinghe.Mahinda Rajapaksa
(c) Tin (d)Silver Women’s Singles title in Wimbledon was sworn in as the Prime Minister for
Tennis Championship in the year the first time on 6th April, 2004.
_ (c) Tin (Sn) does not form oxide on 2019.
reaction with oxygen. It readily form Other Winners 120. Which one of the following Indian
peroxide. Complete reaction is Ocean island nations has recently
Sn + O 2 → SnO 2 Category Winner
declared a State of environmental
Tin peroxide Men’s Singles N. Djokovic (Serbia)
emergency due to oil spill from a
All other options such as magnesium
Men’s Doubles Juan Sebastian Cabal grounded ship?
(Mg), lead (Pb) and Silver (Ag) forms
(Colombia) and
oxide. Reactions are as follows Robert Farah (a) Maldives (b) Mauritius
2Mg + O 2 → 2MgO (Colombia) (c) Madagascar (d) Sri Lanka
2Pb + O 2 → 2PbO Women’s SW. Hsieh (Taiwan) _ (a) Mauritius is an island nation in the
4Ag + O 2 → 2Ag 2O Doubles and B. Strycova Indian Ocean, which declared a state
(Czech Republic) of emergency due to oil spill in August
114. The valency of phosphorus is 2020. A japanese bulk-carrier ship MV
Mixed Doubles I. Dodig (Croatia) and Wakashio which was carrying fuel oil
(a) 2, 3 (b) 3, 4 (c) 4, 5 (d) 3, 5
L. Chan (Taiwan) had split into two parts near Blue Bay
_ (d) Valency of phosphorus is Marine Park in South-East Mauritius. It
3, 5-orbits are stable when they are caused leakage of over 1000 tonnes
fully filled or half filled. So, to attain a
118. Recently islands of Andaman and
of oil. It endangers the already
stable configuration it can take 3 Nicobar were connected with endangered coral reefs, seagrasses,
electrons or remove 5 electrons. So, mainland by Submarine Optical mangroves, the fishes and other
valency of ‘P’ is 3 and 5. Fibre Cable. aquatic fauna.
CDS
Combined Defence Service

SOLVED PAPER 2020 (I)


PAPER I Elementary Mathematics

1. The number 2 × 3 × 5 × 7 × 11 + 1 is 3. How many digits are there in (54 )10 ? 5. In a competitive examination,
(a) a prime number (Given that log10 2 = 0301
. 250 students have registered. Out
(b) not a prime, but power of a prime and log10 3 = 0.477) of these, 50 students have
(c) not a power of a prime, but a registered for Physics, 75 students
composite even number (a) 16 (b) 18
(c) 19 (d) 27 for Mathematics and 35 students
(d) not a power of a prime, but a
for both Mathematics and Physics.
_ (b) Let x = ( 54)
10
composite odd number
What is the number of students
_ (a) Given, number ⇒ log x = 10 log 54
who have registered neither for
2 × 3 × 5 × 7 × 11 + 1 = 10 log(2 × 3 3 )
Physics nor for Mathematics?
= 2310 + 1 = 2311 = 10{log 2 + 3 log 3}
(a) 90 (b) 100 (c) 150 (d) 160
Which is a prime number. = 10{ 0.301 + 3 × 0.477}
= 10{ 0.301 + 1431
. } _ (d) Total number of students have
2. Two unequal pairs of numbers registered = 250
satisfy the following conditions = 10 × 1732
. = 17.32
Students have registered for Physics
Since, the characteristic of log x is 17. = 50
(i) The product of the two numbers
in each pair is 2160. ∴ Number of digits is ( 54)10 is Students have registered for
17 + 1 = 18 Mathematics = 75
(ii) The HCF of the two numbers in
each pair is 12. 4. Which one of the following is a set And students have registered for
both Mathematics and Physics = 35
If x is the mean of the numbers in of solutions of the equation
∴Number of students who registered
the first pair and y is the mean of x x = n x x , if n is a positive for atleast one subject
the numbers in the second pair,
integer? = ( 50 − 35) + (75 − 35) + 35
then what is the mean of x and y?
(a) {1, n2 } (b) {1, n} = 15 + 40 + 35 = 90
(a) 60 (b) 72 (c) 75 (d) 78
(c) {1, n} (d) {n, n2 } ∴The number of students who have
_ (b) Let two unequal pairs of numbers registered neither for Physics nor for
_ (a) Given, x = Mathematics = 250 − 90 = 160
x n
are (12 a, 12 b) and (12c, 12d ). xx …(i)
According to the question, x1 / 2
⇒ x = (x ) x 1/ n
6. If the sum of the digits of a
12 a × 12 b = 12c × 12d = 2160
⇒ x1 / 2
=x x /n number 10n − 1, where n is a
⇒ a × b = c × d = 15 x
Here factors of 15 are 1, 3, 5, 15 ⇒ x = x/n 1/ 2 natural number, is equal to 3798,
∴ a = 1, b = 3, c = 5, d = 15 ⇒ x 1/ 2 = n then what is the value of n?
12 a + 12 b 12c + 12d ⇒ x = n2 (a) 421 (b) 422 (c) 423 (d) 424
Now, x = ,y=
2 2 On putting x = 1 in Eq. (i), we get _ (b) Given, number = 10 − 1
n

x + y For n = 1, 101 − 1 = 9
∴Mean of x and y = (1) 1
= (11 )1/ n
2 For n = 2, 10 2 − 1 = 99
12 ( a + b + c + d ) ⇒ 1= 1
= For n = 3, 10 3 − 1 = 999
4 ⇒ x = 1 also satisfies the given ∴ Sum of digits of 10 n − 1 = 9n,
= 3(1 + 3 + 5 + 15) equation. where n is a natural number.
= 3 × 24 = 72 So, solution set is {1, n2} ∴ 9n = 3798 ⇒ n = 422
CDS Solved Paper 2020 (I) 41

7. Which one of the following is the c


=
1
… (iii) ⇒ 2 x 2 − 3x − 104 = 0
largest number among 2222 2 , d 2 3 ± 9 + 4 × 2 × 104
⇒ x =
2 ×2
d
222 22 , 22 222 , 2 2222 ? =3 … (iv)
e
(a) 2 2222 (b) 22 222 (c) 222 22 (d) 2222 2 3 ± 9 + 832 3 ± 841
and
e 1
= … (v) ⇒ x = =
2 22 222 2222 4 4
_ (a) Given, 2222 , 222 , 22 ,2 f 4
3 ± 29
Taking log On multiplying Eqs. (i), (ii) and (iii), ⇒ x =
we get 4
= log(2222 )2,log(222 )22,log(22 )222,
1 1 1 3 + 29 3 − 29
log(2 )2222
a b c
× × = ×2 × = ⇒ x = ,
b c d 3 2 3 4 4
= 2 log(2222 ), 22 log(222 ), 222 log(22 ), 13
a 1 ⇒ x = 8, −
2222 log(2 ) ⇒ = … (vi)
d 3 2
= 2 log(2 × 1111), 22 log(2 × 111), From Eq. (ii), we get
On multiplying Eqs. (ii), (iii) and (iv),
222 log(2 × 11), 2222 log(2 ) we get 8
y= =1
= 2(log 2 + log 1111), b c d 1 8
× × =2 × × 3= 3
22(log 2 + log 111), c d e 2 ∴Required number = 10x + y
222(log 2 + log 11), 2222 log 2 b = 10 × 8 + 1
⇒ =3 … (vii)
= 2( 0.301 + 3), 22( 0.301 + 2 ), e = 81
222( 0.301 + 1), 2222( 0.301) On multiplying Eqs. (iii), (iv) and (v),
Since, log 2 = 0.301, we get 12. If α and β are the roots of the
log1111 = 3 (approx.) c d e 1 1 quadratic equation
× × = ×3×
log111 = 2 (approx.), d e f 2 4 x 2 + kx − 15 = 0 such that
log11 = 1 (approx.) ⇒
c 3
= … (viii) α − β = 8, then what is the positive
= 2( 3.301), 22(2.301), 222(1301
. ), f 8 value of k?
2222( 0.301) Again multiplying Eqs. (vi), (vii) and (a) 2 (b) 3
(viii), we get
= 6.602, 55.622, 288.822, 668.822 abc 1 3 3
(c) 4 (d) 5
Thus, 2 2222 is the largest number. = ×3× =
def 3 8 8 _ (a) Given quadratic equation,
8. If m is the number of prime x 2 + kx − 15 = 0
10. Which one of the following is the k
numbers between 0 and 50; and n ∴α + β = − = − k … (i)
largest divisor of 1
is the number of prime numbers 3 x + 3 x + 1 + 3 x + 2 , if x is any and αβ = − 15 … (ii)
between 50 and 100, then what is
natural number ? and also, given α − β = 8 … (iii)
(m − n ) equal to?
(a) 3 (b) 13 (c) 39 (d) 117 Since, (α − β )2 = (α + β )2 − 4αβ
(a) 4 (b) 5 (c) 6 (d) 7
x +1 x +2 ⇒ ( 8)2 = ( − k )2 − 4( − 15)
_ (c) Given, 3 + 3 + 3
x
_ (b) Prime numbers between 0 and 50 ⇒ 64 = k 2 + 60
are 2, 3, 5, 7, 11, 13, 17, 19, 23, 19, = 3 x (1 + 3 + 3 2 )
31, 37, 41, 43, 47. ⇒ k2 = 4
= 3 x (13) Where x is any
∴Total number of prime numbers ⇒ k=±2
natural number.
between 0 to 50. ∴ k =2 > 0
Put x = 1, then, 31(13) = 39
∴ m = 15
Put x = 2, then 3 2(13) = 117 13. What are the values of p and q
And Prime numbers between 50 to
100 are ∴Largest divisor of respectively, if ( x − 1) and ( x + 2)
3 x + 3 x + 1 + 3 x + 2 is 39. divide the polynomial
53, 59, 61, 67, 71, 73, 79, 83, 89, 97
∴Total number of prime numbers 11. A two-digit number is 9 more than x 3 + 4 x 2 + px + q ?
between 50 to 100 four times of the number obtained (a) 1, − 6 (b) 2, − 6
∴ n = 10 by interchanging its digits. If the (c) 1, 6 (d) 2, 6
∴ m − n = 15 − 10 = 5
product of digits in the two-digit _ (a) Given polynomial is
a 1 b c 1 d
9. If = , = 2, = , = 3 and number is 8, then what is the x 3 + 4x 2 + px + q .
b 3 c d 2 e number? Since, ( x − 1) divide the given
e 1 (a) 81 (b) 42 (c) 24 (d) 18 polynomial.
= , then what is the value of
f 4 ∴(1)3 + 4(1)2 + p(1) + q = 0
_ (a) Let two digit number = 10x + y
abc ⇒ 1+ 4 + p+ q = 0
? According to the question,
10x + y = 4 (10 y + x ) + 9 ⇒ p + q = − 5 … (i)
def
⇒ 10x + y = 40 y + 4x + 9 and ( x + 2 ) divide the polynomial
1 3 3 27
(a) (b) (c) (d)
⇒ 6x − 39 y = 9 ∴( − 2 )3 + 4( − 2 )2 + p ( − 2 ) + q = 0
4 4 8 4
⇒ 2 x − 13 y = 3 … (i) ⇒ − 8 + 16 − 2 p + q = 0
a 1
_ (c) Given, = … (i)
Also, xy = 8 ⇒ y = 8 / x … (ii) ⇒ − 2 p+ q = − 8 … (ii)
b 3
b From Eqs. (i) and (ii), we get On solving Eqs. (i) and (ii), we get
=2 … (ii) p = 1, q = − 6
c 2 x − 13( 8 / x ) = 3
42 CDS Solved Paper 2020 (I)

14. If ( x + k ) is the HCF of x 2 + 5x + 6 _ (a) Given, x + 9 y = 6xy


2 2
22. If f ( x ) is divided by ( x − α ) ( x − β ),
and x 2 + 8x + 15, then what is the ⇒ x 2 + 9 y 2 − 6xy = 0 where α ≠ β, then what is the
⇒ ( x − 3 y)2 = 0 remainder?
value of k? ( x − α )f(α ) − ( x − β )f( β )
⇒ x − 3y = 0 (a)
(a) 5 (b) 3 α −β
⇒ x = 3y
(c) 2 (d) 1 ( x − α )f(β ) − ( x − β )f(α )
⇒ y / x = 1/ 3 (b)
_ (b) Since, ( x + k ) is the HCF of ⇒ y : x =1: 3 α −β
x 2 + 5x + 6 and x 2 + 8x + 15. ( x − β )f(α ) − ( x − α )f( β )
18. If m and n are positive integers (c)
∴( − k )2 + 5( − k ) + 6 = 0 α −β
⇒ k 2 − 5k + 6 = 0 … (i) such that m n = 1331, then what is ( x − β )f ( β ) − ( x − α )f(α )
(d)
and ( − k )2 + 8( − k ) + 15 = 0 the value of (m − 1)n − 1 ? α −β
⇒ k 2 − 8k + 15 = 0 … (ii)
(a) 1 (b) 100 (c) 121 (d) 125 _ (c) Given, f( x ) is divided by
From Eqs. (i) and (ii), we get
_ (b) Given, m = 1331
n ( x − α) ( x − β).
3k − 9 = 0 ⇒ k = 3
⇒ mn = 113 ∴Let quotient is q ( x ) and remainder
15. If 5 x + 1 − 5 x − 1 = 600, then what is is ( ax + b ).
⇒ m = 11, n = 3
∴f( x ) = ( x − α )( x − β ) q ( x ) + ( ax + b )
the value of 10 2x ? ∴ ( m − 1)n − 1 = (11 − 1)3 − 1 … (i)
(a) 1 (b) 1000 = 10 2 = 100 On putting x = α, β in Eq. (i), we get
(c) 100000 (d) 1000000 1 1
19. What is + f(α ) = aα + b … (ii)
+1 −1
_ (d) Given, 5
x
− 5x = 600 … (i) m −n n −m
a −1 a −1 and f(β ) = aβ + b … (iii)
⇒ 5 x  5 −  = 600
1 equal to? From Eqs. (i) and (ii), we get
 5 (a) 1 (b) − 1 a(α − β ) = f(α) − f(β)
x  24  (c) 0 (d) 2am − n f(α ) − f(β )
⇒ 5   = 600 ⇒ a= … (iv)
 5 1
+
1 α −β
_ (b)
600 × 5 am − n − 1 an − m − 1
⇒ 5x = = 125 = 5 3 From Eqs. (ii) and (iv), we get
24 1 1  f(α ) − f(β )
= m + n
⇒ x =3 f(α ) =  α +b
 α −β 
a a
−1 −1
∴ 10 2x = 10 6 = 1000000 an am
αf(β ) − βf(α )
an am ⇒b =
16. A number divides 12288, 28200 = m + n α −β
and 44333 so as to leave the same a −a n
a − am
∴Remainder ax + b
remainder in each case. What is an am f(α )x − f(β )x αf(β ) − βf(α )
= − = +
am − an am − a
n
that number? α −β α −β
(a) 272 (b) 232 a −a
n m
a − am
n
( x − β )f(α ) − ( x − α )f(β )
= =− = −1 =
(c) 221 (d) 120 a −a
m n
an − am α −β
_ (c) Let divisor is x and remainder is r.
Then, according to the question, 20. If x = 2, y = 3 3 and z = 6 6, then 23. If the area of a square is
12288 = x × q 1 + r … (i) which one of the following is correct? 2401x 4 + 196x 2 + 4, then what is
28200 = x × q 2 + r … (ii) (a) y < x < z (b) z < x < y its side length?
and 44333 = x × q 3 + r … (iii) (c) z < y < x (d) x < y < z (a) 49 x2 + 3 x + 2 (b) 49 x2 − 3 x + 2
From Eqs. (i) and (ii), we get
_ (b) Given, x = 2 , y = 3 3 and (c) 49 x2 + 2 (d) 59 x2 + 2
(q 2 − q 1 )x = 28200 − 12288 = 15912 z=6 6
From Eqs. (ii) and (iii), we get _ (c) Given area of square
⇒ x = 2 1/ 2, y = 31/ 3 and z = 61/ 6
= 2401 x 4 + 196x 2 + 4
(q 3 − q 2 )x = 44333 − 28200 = 16133 ⇒ x = 2 6/12, y = 3 4 /12 and z = 6 2/12
… (v) = ( 49x 2 )2 + 2 × 49x 2 × 2 + (2 )2
⇒ x = ( 64)1/12, y = ( 81)1/12
From Eqs. (i) and (iii), we get = ( 49x 2 + 2 )2
and z = ( 36)1/12
(q 3 − q 1 )x = 44333 − 12288 = 32045 ∴Length of side = 49x 2 + 2
Clearly, z < x < y.
… (vi) 24. If x varies as yz, then y varies
Since Eqs. (iv), (v) and (vi) are 21. If log x = 12500
. and y = x log x , then inversely as
divisible by x. what is log y equal to? x
∴HCF of (15912,16133, 32045) = 221 (a) xz (b)
(a) 4.2500 (b) 2.5625 z
∴ x = 221 (c) 1.5625 (d) 1.2500 z 1
(c) (d)
17. If x 2 + 9y 2 = 6xy, then what is _ (c) Given, log x = 1.2500
x ( xz)
y : x equal to? and y = x log x ⇒ _ (c) Given, x varies as yz.
(a) 1 : 3 (b) 1 : 2 log y = (log x )(log x ) ∴x ∝ yz
(c) 2 : 1 (d) 3 : 1 ⇒ log y = (log x )2 x
⇒ ∝ y⇒ y ∝
1
⇒ log y = (125
. )2 = 15625
. z (z/ x)
CDS Solved Paper 2020 (I) 43

Difference of price of X and Y 300( x + 5 − x )


25. If the points P and Q represent real ⇒ =2
= 5000 − 2000 = ` 3000 x ( x + 5)
numbers 0 ⋅ 73 and 0 ⋅ 56 on the
In 2013, ⇒ 300 × 5 = 2 x 2 + 10x
number line, then what is the
price of article ⇒ 2 x + 10x − 1500 = 0
2
distance between P and Q ? 3
X = 5000  1 +
1 1 16 11 20  ⇒ x 2 + 5x − 750 = 0
(a) (b) (c) (d) 
6 5 45 90  100  ⇒ x + 30x − 25x − 750 = 0
2

3
⇒ x ( x + 30) − 25( x + 30) = 0
= 5000 × 
_ (a) Let x = 073
. = 073333
. .... 120 

 100  ⇒ ( x + 30) ( x − 25) = 0
⇒ 10x = 7.3333 .... (i)
6 6 6 ⇒ x + 30 = 0 or x − 25 = 0
⇒ 100x = 73.333 .... .... (ii) = 5000 × × ×
5 5 5 ⇒ x = − 30 or x = 25
Subtracting Eq. (i) from Eq. (ii),
= ` 8640 ⇒ x = 25 km/h [Q x ≠ − 25]
90x = 66 3
Price of article Y = 2000 1 +
66 11 10  30. If 6 men and 8 women can do a
⇒ x = = 
90 15  100  piece of work in 10 days, and
Similarly, y = 0.56 3
13 men and 24 women can do the
= 2000
110 
56 − 5 
⇒ y=  100  same work in 4 days, then what is
90 11 11 11 the ratio of daily work done by a
= 2000 × × × = ` 2662
51 17
⇒ y= = 10 10 10 man to that of a woman?
90 30 ∴Difference of price of X and Y (a) 2 : 1 (b) 1 : 2 (c) 4 : 3 (d) 3 : 4
∴Distance between P and Q = 8640 − 2662 = ` 5978
11 17 22 − 17 5 1 _ (a) Let a man’s one day work = x unit
= − = = = 28. The ratio of speeds of X andY is
15 30 30 30 6 and a woman’s one day work = y unit
5 : 6. IfY allows X a start of 70 m in a ∴6 men and 8 women one day work
26. What is the point on the xy-plane 1.2 km race, then who will win the 1
satisfying 5x + 2y = 7 xy and race and by what distance?
⇒ 8x + 8 y = … (i)
10
10x + 3y = 8xy? (a) X wins the race by 30 m and 13 men and 24 women one day
(a)  − 1,  (b)  , − 1
1 1 (b) Y wins the race by 90 m work
 6 6  (c) Y wins the race by 130 m 1
⇒ 13x + 24 y = … (ii)
 1
(c)  1,  
(d)  − , − 1
1 (d) The race finishes in a dead heat 4
 6  6 
_ (c) Speed of X = 5k On solving Eqs. (i) and (ii), we get
1 1
_ (b) Given, 5x + 2 y = 7 xy and speed of Y = 6k x = and y =
5 2 100 200
⇒ + =7 … (i) Distance travelled by
1 1
y x X = 12 . × 1000 − 70 ∴Ratio of x and y = : = 2 :1
100 200
and 10x + 3 y = 8xy = 1200 − 70
10 3 = 1130 m 31. Students of a class are made to sit
⇒ + =8 … (ii)
y x Distance travelled by Y = 1200 m in rows of equal number of chairs.
From 2 × Eq. (i) − Eq,(ii), we get 1130 226 If number of students is increased
∴Time taken by X = =
4 3 5k by 2 in each row, then the number
− = 14 − 8 k
1200 200 of rows decreases by 3. If number
x x Time taken by Y = =
1 1 6k of students is increased by 4 in
⇒ = 6⇒x = k
6 226 200 each row, then the number of
x Since, >
From Eqs. (i), we get k k rows decreases by 5. What is the
5
+ 2 × 6=7⇒
5
= 7 − 12 = − 5 ∴ Y wins the race. number of students in the class?
y y ∴ Difference of distance to win the (a) 100 (b) 105 (c) 110 (d) 120
race = 
⇒ y= −1 226 200 
 k
−  × 5k _ (d) Let the numbers of row = x
k 
27. The price of an article X increases Number of students in each row = y
= 26 × 5 = 130m
by 20% every year and price of ∴Number of students = xy
articleY increases by 10% every 29. A train takes two hours less for a According to the question,
year. In the year 2010, the price of journey of 300 km if its speed is ( x − 3) ( y + 2 ) = xy
article X was ` 5000 and price of increased by 5 km/h from its usual ⇒ xy + 2 x − 3 y − 6 = xy
articleY was ` 2000. In which year speed. What is its usual speed? ⇒ 2x − 3y = 6 … (i)
(a) 50 km/h (b) 40 km/h and ( x − 5) ( y + 4) = xy
the difference in their prices
(c) 35 km/h (d) 25 km/h ⇒ xy + 4x − 5 y − 20 = xy
exceeded ` 5000 for the first time? ⇒ 4x − 5 y = 20 … (ii)
(a) 2012 (b) 2013 (c) 2014 (d) 2015 _ (d) Let usual speed of train = x km/h On solving Eqs. (i) and (ii), we get
_ (b) In year 2010, According to the question, x = 15 and y = 8
300 300
Price of article X is ` 5000. − =2 ∴Number of students in the class
and price of article Y is ` 2000. x x + 5 = xy = 15 × 8 = 120
44 CDS Solved Paper 2020 (I)

32. A sum was put at simple interest Now, total number of male in both 37. The maximum marks in a test are
villages = 675 + 1200 = 1875
at certain rate for 2 yr. Had it been converted from 250 to 50 for the
Total number of female in both
put at 1% higher rate of interest, it purpose of an Internal Assessment.
villages = 850 + 1400 = 2250
would have fetched ` 24 more. The highest marks scored were
∴Required ratio = 1875 : 2250 = 5 : 6
What is the sum? 170 and lowest marks were 70.
(a) ` 500 (b) ` 600 34. In a class room the ratio of What is the difference between the
(c) ` 800 (d) ` 1200 number of girls to that of boys is maximum and minimum marks
3 : 4. The average height of students scored in the Internal Assessment?
_ (d) Let ` x was put at simple interest
at rate of r% per annum for 2 yr. in the class is 4.6 feet. If the average (a) 15 (b) 17 (c) 20 (d) 24
According to the question, height of the boys in the class is 4.8
x × r×2 feet, then what is the average height _ (c) Since maximum marks in a test
SI = … (i) are converted from 250 to 50 for the
100 of the girls in the class? Internal Assessment.
x × ( r + 1) × 2 (a) Less than 4.2 feet 50 1
and (SI + 24) = … (ii) ∴ =
100 (b) More than 4.2 feet but less than 250 5
From Eqs. (i) and (ii), we get 4.3 feet 1
⇒ Marks to reduced to th for the
2x (c) More than 4.3 feet but less than 5
24 =
100 4.4 feet purpose of an Internal Assessment.
24 × 100 (d) More than 4.4 feet but less than
∴ Highest marks =
150
= 34
⇒ x = 4.5 feet
2 5
⇒ x = 1200 _ (c) Let Number of girls = 3x and Lowest marks =
70
= 14
∴ Required sum = ` 1200 and number of boys = 4x 5
Total number of students ∴Required difference = 34 − 14 = 20
33. The population of two villages is
= 3x + 4x = 7 x
1525 and 2600 respectively. If the Directions (Q. Nos. 38-40) Read the
Sum of heights of all students
ratio of male to female population = 4.6 × 7 x following information and answer the
in the first village is 27 : 34 and the = 32.2 x given questions follow.
ratio of male to female population sum of heights of all the boys The following data presents count of
in the second village is 6 : 7, then = 4.8 × 4x released convicts who have served
what is the ratio of male to female = 19.2 x prison terms ( X ), those who have
population of these two villages ∴Sum of heights of all the girls received some educational or
taken together? = 32.2 x − 192. x = 13x technical training during their
(a) 33 : 41 (b) 85 : 82 ∴Average height of the girls in the term ( Y ) and those who were offered
(c) 71 : 90 (d) 5 : 6 13x 13 Company placement ( Z ) respectively,
class = = = 4.33
3x 3
_ (d) Given population of first village from six different jails A, B, C, D, E
= 1525 Hence, Average height of the girls in and F, in the year 2010.
Let male population in first village is the class more than 4.3 feet but less
than 4.4 feet. X Y Z
27 x.
A 86 45 25
And female population in first village 35. What is the median of the data 3,
= 34x 5, 9, 4, 6, 11, 18? B 1305 903 461
Then, 27 x + 34x = 1525 (a) 6 (b) 6.5 (c) 7 (d) 7.5 C 2019 940 474
⇒ 61x = 1525 ⇒ x = 25 D 1166 869 416
∴Male population in first village
_ (a) Given data 3, 5, 9, 4, 6, 11, 18.
Now, arranging in ascending order E 954 544 254
= 27 x
3, 4, 5, 6, 9, 11, 18 F 1198 464 174
= 27 × 25 = 675
∴ Median = 6
and Female population in first village
= 34x 36. In a pie-diagram there are three 38. Jails with highest and smallest
= 34 × 25 = 850 sectors. If the ratio of the angles of percentage of trained convicts are
Population of second village = 2600 the sectors is 1 : 2 : 3, then what is respectively
Let male population in second village the angle of the largest sector? (a) F and D (b) D and F
= 6y (c) C and A (d) D and A
(a) 200º (b) 180º (c) 150º (d) 120º
and female population in second _ (b) Total released convicts from
village = 7 y _ (b) Let angles of three sectors are x, point A
2 x and 3x.
Then, 6 y + 7 y = 2600 = 86 + 45 + 25 = 156
In a pie–diagram,
⇒ 13 y = 2600 ⇒ y = 200 ∴Trained convicts from jail A
x + 2 x + 3x = 360 45
∴Male population in second village = × 100 = 28.84%
⇒ 6x = 360° ⇒ x = 60° 156
= 6 y = 1200
∴ Largest angle Total released convicts from jail B
and Female population in second
village = 7 y = 1400 = 3x = 3 × 60° = 180° = 1305 + 903 + 461 = 2669
CDS Solved Paper 2020 (I) 45

Trained convicts from jail B Convicts who offered company _ (b) Given equation 5x + 9 y = 7,
903 placement from jail A = 25
= × 100 = 33.83% where − 500 < x , y < 500.
2669 Trained convicts from jail B = 903 7 − 5x
Total released convicts from jail C ∴Half of the trained convicts Now, y =
9
= 2019 + 940 + 474 = 3433 =
903
= 4515
. x y
Trained convicts from jail C 2
940 Convicts who offered company −1 12/9
= × 100 = 27.38%
3433 placement from jail B = 461 −4 3
Total released convicts from jail D Trained convicts from jail C = 940
−13 8
= 1166 + 869 + 416 = 2451 ∴Half of the trained convicts
940 −22 13
Trained convicts from jail D = = 470
869
2 −31 18
= × 100 = 35.45% Convicts who offered company
2451
placement from jail C = 474 Here, when
Total released convicts from jail E
Trained convicts from jail D = 869 x = − 4, − 13, − 22, − 31, …
= 954 + 544 + 254 = 1752
∴Half of the trained convicts = 434.5 i.e. common difference with 9, then
Trained convicts from jail E
Convicts who offered company value of y = integer value
544
= × 100 = 5105
. % placement from jail D = 416 ∴ 495 = − 495 + ( n − 1)( 9)
1752
Trained convicts of jail E = 544 495 + 495
Total released convicts from jail F ⇒n − 1=
∴Half of the trained convicts = 272 9
= 1198 + 464 + 174 = 1836 990
Convicts who offered company = = 110 ⇒ n = 111
Trained convicts from jail F placement from jail E = 254 9
464
= × 100 = 25.27% Trained convicts from jail F = 464
1836 43. Let XYZ be a 3-digit number. Let
∴Half of the trained convicts = 232
∴Jails with highest and smallest S = XYZ + YZX + ZXY . Which of
Convicts who offered company
percentage of trained convicts are the following statements is/are
placement from jail F = 174
respectively 35.45% and 25.27%. correct?
Hence, jails from which more than
39. Jail with highest placement rate of half of the trained convicts are 1. S is always divisible by 3 and
trained convicts is offered jobs are A, B and C. ( X + Y + Z ).
(a) F (b) D (c) B (d) A 41. The number of three digit 2. S is always divisible by 9.
3. S is always divisible by 37.
_ (d) Placement rate of trained convicts numbers (all digits are different)
25 Select the correct answer using the
of jail A = × 100 = 55.55 which are divisible by 7 and also
45 divisible by 7 on reversing the code given below:
Placement rate of trained convicts of order of the digits, is (a) 1 only (b) 2 only
461
jail B = × 100 = 5105
. (a) Six (b) Five (c) Four (d) Three (c) 1 and 2 (d) 1 and 3
903
Placement rate of trained convicts of _ (c) Let the three digits number = xyz _ (d) Given, S = XYZ + YZX + ZXY
jail C =
474
× 100 = 50.43 and the reverse of it will be = zyx ⇒ S = (100 X + 10Y + Z )
940
Since, both the numbers are divisible + (100Y + 10Z + X )
Placement rate of trained convicts of by 7. + (100Z + 10 X + Y )
416
jail D = × 100 = 47.87 ∴ 100x + 10 y + z = 7 m … (i) ⇒ S = 111 ( X + Y + Z )
869
and 100 z + 10 y + x = 7 n … (ii) = 3 × 37 ( X + Y + Z )
Placement rate of trained convicts of
254 From Eqs. (i) and (ii), we get ⇒ S is always divisible by 3, 37 and
jail E = × 100 = 46.69 ( X + Y + Z ).
544 ⇒ 99 ( x − z) = 7( m − n)
Placement rate of trained convicts of Here, 99 is not divisible by 7 then it is 44. In covering certain distance, the
jail F =
174
× 100 = 37.5 must that ( x − z) is a factor of 7. average speeds of X andY are in
464 ∴Possible that x = 2, z = 9; the ratio 4 : 5. If X takes 45 min
∴Jail with highest placement rate of x = 1, z = 8 more thanY to reach the destination,
trained convicts = 55.55 ∴Possible numbers are 168, 861, then what is the time taken by Y to
40. Jails from which more than half of 259 and 952.
reach the destination?
the trained convicts are offered 42. How many integral values of x (a) 135 min (b) 150 min
jobs, are and y satisfy the equation (c) 180 min (d) 225 min
(a) A, B and C (b) A, B and D 5x + 9y = 7, where − 500 < x < 500 _ (c) When the speed is same, then
(c) A, D and E (d) A, E and F and − 500 < y < 500 ? speed is inversely proportional to
time.
_ (a) Trained convicts from jail A = 45 (a) 110
(b) 111 ∴ ty : tx = 4 : 5
∴Half of the trained convicts
45 (c) 112 ∴1 unit difference = 45 min
= = 22.5 (d) None of the above ∴ 4 unit = 4 × 45 = 180 min
2
46 CDS Solved Paper 2020 (I)

45. For two observations, the sum is S 49. If ( x 2 − 1) is a factor of 53. Let 0 < θ < 90º and 100 θ = 90º. If
99
and product is P. What is the ax 4 + bx 3 + cx 2 + dx + e , then α = Σ cot nθ, then which one of
harmonic mean of these two n =1
which one of the following is correct?
observations? the following is correct?
2S S (a) a + b+ c =d + e
(a) (b) (b) a + b+ e =c + d (a) α = 1 (b) α = 0
P (2 P) (c) α > 1 (d) 0 < α < 1
(c) b + c+d =a+e
2P P
(c) (d) (d) a + c+e=b+d _ (a) Given that,
S (2S ) 99
_ (d) Given, ( x − 1) is a factor of
2
100 θ = 90° and α = Σ cot nθ
_ (c) Given that, S = x + y ax + bx + cx + dx + e
4 3 2 n−1

P = xy ⇒ ( x − 1) ( x + 1) is a factor of = cot θ.cos 2θ.cot 3θ... cot 99 θ


2 xy 2P ax 4 + bx 3 + cx 2 + dx + e = cot θ.cos 2θ.cot 3θ.... tan( 90° − 99θ)
∴Harmonic Mean = =
x + y S ⇒ x = − 1, 1 roots of = cot θ.cot 2θ.cot 3θ.... tan(100θ − 99θ)
ax 4 + bx 3 + cx 2 + dx + e = cot θ.cot 2θ.cot 3θ.... tanθ
46. If the annual income of X is 20%
more than that of Y , then the ∴ a( − 1)4 + b( − 1)3 + c( − 1)2 = cot θ. tanθ.cot 2θ. tan2θ ....
income of Y is less than that of X + d ( − 1) + e = 0 ⇒ α =1
⇒ a− b+ c −d + e = 0 π
by p%. What is the value of p? 54. If tan 6 θ = cot 2 θ, where 0 < 6 θ < ,
2 ⇒ a+c+e =b+d 2
(a) 10 (b) 16 then what is the value of sec 4 θ ?
3  1  2 4
1 50. If  x 8 + 8  = 47, what is the (a) 2 (b) 2 (c) (d)
(c) 17 (d) 20  x  3 3
3
 1  _ (a) Given, tan 6θ = cot 2θ
_ (b) Let annual income of Y = ` 100 value of  x 6 + 6  ?
 x  ⇒ tan 6θ = tan( 90° − 2θ)
Then annual income of X = ` 120
120 − 100 (a) 36 (b) 27 (c) 18 (d) 9 ⇒ 6θ = 90° − 2θ
∴ p% = × 100 ⇒ 6θ + 2θ = 90° ⇒ 8θ = 90°
120 1
_ (c) Given, x + = 47
8
20 x8 ⇒ 4θ = 45°
= × 100
120 1 ∴ sec 4θ = sec 45° = 2
⇒ x8 + + 2 = 47 + 2
20 × 5 2 x8
= = 16 55. A tree of height 15 m is broken by
6 3 2
⇒  x 4 + 4  = 49 ⇒ x 4 + 4 = 7
1 1 wind in such a way that its top
47. What is the least perfect square  x  x touches the ground and makes
1
which is divisible by 3, 4, 5, 6 ⇒ x4 + 4 + 2 = 7 + 2 = 9 an angle 30º with the ground.
and 7? x What is the height from the
⇒  x 2 + 2  = 3
1
(a) 1764 (b) 17640 ground to the point where tree is
 x 
(c) 44100 (d) 176400 broken?
3
= ( x 2 )3 +  2 
1 1
_ (c) A number which is divisible by 3, ∴ x6 + (a) 10 m (b) 7 m
4, 5, 6 and 7 should be a multiple of x 6 x  (c) 5 m (d) 3 m
3, 4, 5, 6 and 7. 3
=  x 2 + 2 
1
i.e. 2 2 × 3 × 5 × 7
_ (c) D
 x 
For least perfect square, then it
− 3( x 2 )  2   x 2 + 2 
1 1
should be a perfect square i.e. x   x  C
2 2 × 3 2 × 5 2 × 7 2 = 44100 = ( 3)3 − 3( 3) = 27 − 9 = 18
48. In a water tank there are two 51. A wheel makes 360 revolutions in
outlets. It takes 20 min to empty one minute. What is the number of 30°
the tank if both the outlets are A B
radians it turns in one second?
opened. If the first outlet is Let AD be the tree such that AD = 15 m
(a) 4π (b) 6π (c) 12 π (d) 16π
opened, the tank is emptied in And let AC = x
30 min. What is the time taken to _ (c) 60 sec = 360 revolutions
Then, CD = BC = 15 − x
empty the tank by second outlet? ∴1 sec = 6 revolutions AC
= 6 × 2 π radians = 12 π radians ∴ sin 30° =
(a) 30 min (b) 40 min BC
(c) 50 min (d) 60 min 52. What is the least value of x
=
_ (d) Let x is the time taken when both (25cosec 2 x + sec 2 x )? 15 − x
outlets are open and y is the time 1 x
taken by 1st outlet then time taken by (a) 40 (b) 36 (c) 26 (d) 24 ⇒ =
2 15 − x
_ (b) 25 cosec x + sec x
IInd outlet 2 2

xy 20 × 30 600 ⇒ 2 x = 15 − x
= = = ∴Minimum value of
y−x 30 − 20 10 ⇒ 3x = 15
25 cosec 2x + sec 2 x
∴ x = 5m
= 60 min = ( 25 + 1)2 = ( 5 + 1)2 = 36
CDS Solved Paper 2020 (I) 47

h sinθ / cos θ − cos θ / cos θ + 1 / cos θ


56. On a plane area there are two ⇒ tan27 ° = … (i) =
vertical towers separated by p sinθ / cos θ + cos θ / cos θ − 1 / cosθ
and In ∆BCD, tan 63° =
CD sinθ / cos θ + 1 / cos θ
100 feet apart. The shorter tower is −
40 feet tall. A pole of length 6 feet BC cos θ / cos θ
⇒ tan( 90° − 27 ° ) =
h tanθ − 1 + sec θ tanθ + sec θ
stands on the line joining the base = −
q tanθ + 1 − sec θ 1
of two towers so that the tip of the
⇒ cot 27 ° =
h tanθ + sec θ − 1
towers and tip of the pole are also … (ii) = − (tanθ + sec θ)
on the same line. If the distance of
q (tanθ − sec θ) + 1
From Eqs. (i) and (ii), we get (tanθ + sec θ − 1) (tanθ + sec θ)
the pole from the shorter tower is =
75 feet, then what is the height of
h h
tan27 ° cot 27 ° = . (tanθ − sec θ + 1) (tanθ + sec θ)
the taller tower (approximately)?
p q − tanθ − sec θ
⇒ h2 = pq
(a) 85 feet (b) 110 feet
⇒ h = pq (tanθ + sec θ − 1) (tanθ + sec θ)
(c) 125 feet (d) 140 feet =
58. What is the value of (tanθ 2θ − sec 2 θ) + (tanθ + sec θ)
_ (a) A
I sin 2 6º + sin 2 12º + sin 2 18º + … + − tanθ − sec θ
B (tanθ + sec θ − 1) (tanθ + sec θ)
H sin 2 84º + sin 2 90º?
C − tanθ − sec θ
G =
(a) 1 (b) 2 (c) 4 (d) 8 − 1 + (tanθ + sec θ)
_ (d) sin 6 + sin 12 ° + sin 18° + ...
2 2 2
6 feet 40 = tanθ + sec θ − tanθ − sec θ
feet
+ sin2 84° + sin2 90° =0
D = (sin2 6° + sin2 84° )
F 75 feet E 100 feet 61. What is
+ (sin2 12 ° + sin2 78° ) +
Here, (tan x + tan y ) (1 − cot x cot y ) +
(sin2 18° + sin2 72 ° )
∆ABI ~ ∆IGH (cot x + cot y ) (1 − tan x tan y )
+ K + (sin2 42 + sin2 48° ) + 1
IG HG 40 − 6 75 equal to?
∴ = ⇒ = = (sin 6° + cos 2 6° )
2
AB IB AB 100 (a) 0 (b) 1 (c) 2 (d) 4
34 × 100 + (sin2 12 ° + cos 2 12 ° )
⇒ AB = = 45.33 + (sin 18° + cos 2 18° ) + ....+
2
_ (a) (tan x + tan y) (1 − cot x cot y)
75
∴ AD = AB + BD = AB + IE (sin 42 ° + cos 2 42 ° ) + 1
2
+ (cot x + cot y) (1 − tan x tan y)
= 45.33 + 40 = 85.33 feet = (1 + 1 + 1 + ... 7 times) + 1 = tan x + tan y − cot y − cot x
= 85 feet (approx.) = 7 + 1= 8
+ cot x + cot y − tan y − tan x = 0
cos θ 1
57. The angles of elevation of the top 59. What is + equal to? sec x − tan x
1 + sin θ cot θ 62. What is equal to?
of a tower from two points at
(a) cosec θ (b) sec θ sec x + tan x
distances p and q from the base (c) sec θ + cosec θ (d) cosec θ − cot θ 1
and on the same straight line are cos θ 1 (a)
_ (b) + sin x + cos x
27º and 63º respectively. What is 1 + sinθ cot θ 1
the height of the tower? cos θ 1 (b)
= + tan x + cot x
(a) pq (b) pq 1 + sinθ cos θ / sinθ
1
pq pq cos θ sinθ (c)
(c) (d) = + sec x + tan x
2 2 1 + sinθ cos θ
1
_ (b) Let height of the tower CD be h. cos 2 θ + sinθ + sin2 θ (d)
= cosec x + cot x
D cos θ(1 + sinθ)
sec x − tan x
(cos 2 θ + sin2 θ) + sinθ _ (c)
= sec x + tan x
cos θ(1 + sinθ)
(sec x − tan x ) (sec x + tan x )
1 + sinθ =
h = = sec θ (sec x + tan x ) (sec x + tan x )
cos θ(1 + sinθ)
sin θ − cos θ + 1 sin θ + 1 =
sec 2 x − tan2 x
60. What is −
sin θ + cos θ − 1 cos θ (sec x + tan x )2
A 27° 63° C
q equal to? sec 2 x − tan2 x
B
p (a) 0 (b) 1 =
(c) 2sinθ (d) 2cosθ (sec x + tan x )2
∴In ∆ACD,
sinθ − cos θ + 1 sinθ + 1 =
1
tan27 ° =
CD
_ (a) −
sinθ + cos − 1 cos θ sec x + tan x
AC
48 CDS Solved Paper 2020 (I)

63. If θ lies in the first quadrant and ∴ sec θ + cos ec θ =


25 25
+ 3+1 AB
Now, =
63 24 7 sin105° sin 30°
cot θ = , then what is the value 175 + 600 3+1
16 = ⇒ =
x
168  3 + 1 1 / 2
of (sin θ + cos θ)? 775  
69 79 =  2 2 
(a) 1 (b) (c) (d) 2 168
65 65 ⇒ 2 2 = 2x
63 66. What is the area of the triangle
_ (c) Given, cot θ = ⇒ x = 2
16 having side lengths
y z z x x y ∴ Required area of triangle
⇒ tanθ =
16 + , + , + ? 1
= ( AB × BC ) × sin 45°
63 z x x y y z 2
C
( x + y + z)2 xyz 1
= × 2 × ( 3 + 1) ×
1
(a) (b)
xyz x+ y+ z 2 2
1
65 x y z xy + yz + zx = ( 3 + 1) cm 2

16 (c) + + (d) 2
y z x xyz
y z 68. ABCD is a plate in the shape of a
A θ _ (c) Let a = + parallelogram. EF is the line
B z x
63 z x x y parallel to DA and passing
b= + and c = +
In right angled triangle ABC, x y y z through the point of intersection
AC 2 = 63 2 + 16 2 = 3969 + 256 x y z O of the diagonals AC and BD.
⇒ AC 2 = 4225 2 + + 
a+ b+c y z x Further, E lies on DC and F lies
⇒ AC = 65 ∴ s= =
2 2 on AB. The triangular portion
16 63 79
∴sinθ + cos θ = + = x
= + +
y z DOE is cut out from the plate
65 65 65 y z x ABCD. What is the ratio of area
64. What is the value of ∴Area of triangle of remaining portion of the plate
1 − 2 sin 2 θ cos 2 θ = s( s − a ) ( s − b ) ( s − c ) to the whole?
+ 4 equal to? 5 5
sin 4 θ + cos 4 θ  x y z  x y z y z (a) (b)
  y + z + x   y + + − − 
z x z x 8 7
(a) 0 (b) 1 (c) 2 (d) 5  3 7
(c) (d)
1 − 2 sin2 θ cos 2 θ x y z z x
+ 4 =  + + − +  4 8
_ (d) y z x x y
sin4 θ + cos 4 θ _ (d) D E
C
x y 
1 − 2 sin θ cos θ
y z x
 + + − − 
2 2
= y z x y z 
(sin2 θ + cos 2 θ)2 − 2 sin2 θ cos 2 θ
+ 4 O
x y z   x   y  z 
1 − 2 sin2 θ cos 2 θ =  + +      
= + 4= 1+ 4 = 5 y z x   y   z  x  A B
1 − 2 sin2 θ cos 2 θ F
x y z
= + + We know that, the diagonal of a
65. A rectangle is 48 cm long and y z x
14 cm wide. If the diagonal makes parallelogram cut the parallelogram
in the four equal parts.
an angle θ with the longer side, then 67. If the angles of a triangle are 30º
∴ar( AOB) = ar( BOC ) = ar( DOC )
what is (secθ + cosec θ) equal to? and 45º and the included side is
( 3 + 1) cm, then what is the area of = ar( AOD )
775 725 375 325
(a) (b) (c) (d) And ar( DOE ) = ar( EOC )
168 168 84 84 the triangle? 1
= ar(COD )
_ (a) D C (a) ( 3 + 1) cm2 (b) ( 3 + 3 ) cm2 2
1
(c) ( 3 + 1) cm2 (d) 2( 3 + 1) cm2 1 1
= × ar( ABCD )
2 2 4
14 cm
_ (c) Third angle of the triangle 1
= ar( ABCD )
θ = 180° − ( 30° + 45° ) 8
A B ∴ ar (Remaining portion of the
48 cm = 180° − 75° = 105°
A plate after cutting portion DOE)
Diagonal, AC = 48 + 14
2 2
7
= ar ( ABCD )
= 2304 + 196 105° 8
= 2500 = 50 ∴The ratio of area of remaining
AC 50 25
∴ sec θ = = = portion of the plate to the whole
AB 48 24 45° 30° 7 / 8 ar ( ABCD )
B C = = 7/8
AC 50 25 (√ 3 +1) cm
and cosec θ = = = ar ( ABCD )
BC 14 7
CDS Solved Paper 2020 (I) 49

69. Two circles of radii 20 cm and 16 cm ⇒ 8 2 = ( 5 + x )2 + y 2 Then, according to the question,


intersect and the length of common ⇒ 64 = ( 5 + x )2 + ( 5 2 − x 2 ) πr 2 = x 2
[from Eq. (i)] r 1
chord is 24 cm. If d is the distance ⇒ πr = x ⇒ =
between their centres, then which ⇒ 64 = 25 + x 2 + 10x + 25 − x 2 x π
⇒ 64 = 50 + 10x 2 πr
one of the following is correct? ∴ Ratio of their perimeters =
⇒ 10x = 14 ⇒ x = 14 . 4x
(a) d < 26 cm
Now, from Eq. (i), 25 = (14 . )2 + y 2 π 1 π
(b) 26 cm < d < 27 cm = . = = π :2
(c) 27 cm < d < 28 cm ⇒ 25 = 196
. + y 2 2 π 2
(d) d > 28 cm ⇒ y = 25 − 196. 73. A circle of diameter 8 cm is
_ (b) = 4.8 placed in such a manner that it
C
∴Length of chord BC = 2 y touches two perpendicular
20 cm 16 cm = 2 × 4.8 lines. Then another smaller circle
= 9.6 cm is placed in the gap such that it
A D B 71. Two circles touch internally. The touches the lines and the circle.
sum of their areas is 136π cm 2 and What is the diameter of the
distance between their centres is smaller circle?
4 cm. What are the radii of the (a) 4(3 − 2 ) cm (b) 4(3 − 2 2 ) cm
Here, AB = d , CD = 12 cm (c) 8(3 − 2 ) cm (d) 8(3 − 2 2 ) cm
circles?
In ∆ADC, ∠ADC = 90°
(a) 11 cm, 7 cm _ (d)
∴ AC 2 = AD 2 + CD 2
(b) 10 cm, 6 cm Y
⇒ 20 2 = AD 2 + 12 2 (c) 9 cm, 5 cm
⇒ AD 2 = 400 − 144 = 256 (d) 8 cm, 4 cm
⇒ AD = 16
_ (b)
And in ∆BCD, ∠BDC = 90°
BC 2 = BD 2 + CD 2 C1
⇒ 16 2 = BD 2 + 12 2
⇒ BD 2 = 256 − 144 = 112 r2
⇒ BD = 112 = 10.58 C2
r1 r
∴ AB = AD + DB O X
R
⇒ d = 16 + 10.58 = 26.58 cm
Let the distance between the small
∴26 < d < 27 cm circle and the origin O is x and radius
70. In a circle of radius 5 cm, AB and of small circle and larger circle are
Let radius of larger circle be r1 and r and R respectively.
AC are two chords such that radius of smaller circle be r2.
AB = AC = 8 cm. What is the ∴ OC 2 = r2 + r2 = r + x
∴ r1 − r2 = 4
length of chord BC ? ⇒ r1 = 4 + r2 … (i) ⇒ r+ x = 2 r
(a) 9 cm (b) 9.2 cm and πr12 + πr22 = 136 π ⇒ x = r( 2 − 1) … (i)
(c) 9.6 cm (d) 9.8 cm And also,
⇒ π( 4 + r2 )2 + πr22 = 136 π
_ (c) A ⇒ 16 + r22 + 8r2 + r22 = 136 OC 2 = R2 + R2
⇒ 2 r22 + 8r2 = 136 − 16 = 120 = (x + 2 r + R )
⇒ r22 + 4r2 − 60 = 0 ⇒ x + 2r + R = 2R
8 cm 5 cm 8 cm ⇒ r2 + 10r2 − 6r2 − 60 = 0
2
⇒ r( 2 − 1) + 2 r + 4 = 2 ( 4)
5 cm O ⇒ r2( r2 + 10) − 6( r2 + 10) = 0 [Q2 R = 8 cm]
⇒ ( r2 + 10) ( r2 − 6) = 0
⇒ r( 2 − 1 + 2 ) = 4( 2 − 1)
B C ⇒ r2 = 6
D ⇒ r( 2 + 1) = 4( 2 − 1)
[Q r2 ≠ − 10]
4( 2 − 1) 2 −1
∴r1 = 4 + 6 = 10 cm ⇒ r= ×
2 +1 2 −1
Let OD = x cm 72. If area of a circle and a square are
4( 2 − 1)2
and BD = y cm same, then what is the ratio of ⇒ r=
2 −1
In right angle triangle BOD, their perimeters?
BO 2 = OD 2 + BD 2 (a) 2 π : 1 (b) π:1 ⇒ r = 4(2 + 1 − 2 2 )
⇒ 52 = x 2 + y2 … (i) (c) π : 2 (d) π :4 ⇒ r = 4( 3 − 2 2 )
And in right angled triangle ABD, ∴Diameter of the smaller circle = 2 r
_ (c) Let radius of circle be r and side
AB2 = AD 2 + BD 2 of square be x. = 8( 3 − 2 2 ) cm
50 CDS Solved Paper 2020 (I)

74. The thickness of a cylinder is 76. The length and breadth of a In right angle triangle ADB
BD
1 foot, the inner radius of the rectangle are in the ratio 4 : 3. sin 60° =
cylinder is 3 feet and height is Then, what is the ratio of the area of AB
3 BD
7 feet. To paint the inner surface it the triangle formed by the parts of ⇒ =
requires one litre of a particular the diagonals with a long side to the 2 AB
3
colour. How much quantity of the area of the triangle formed by the ⇒ BD = AB2
2
4
same colour is required to paint all parts of diagonals with a short side?
the surfaces of the cylinder? (a) 3 : 4 (b) 4 : 3 (c) 16 : 9 (d) 1 : 1 79. A line through the vertex A of a
7 3 8 10 parallelogram ABCD meets DC in
(a) L (b) L (c) L (d) L _ (d) Let length of rectangle be 4x and P and BC produced in Q. If P is the
3 2 3 3 breadth of rectangle be 3x.
mid-point of DC, then which of
_ (c) Given, thickness of a cylinder is D C
1 feet and inner radius of the cylinder the following is/are correct?
( r2 ) = 3 feet O 1. Area of ∆PDA is equal to that of
3x
∆PCQ.
r1 2. Area of ∆QAB is equal to twice
A 4x B that of ∆PCQ.
h=7 feet
∴Diagonal of rectangle, Select the correct answer using the
AC = ( 4x ) + ( 3x ) = 25x = 5x
2 2 2 code given below:
(a) Only 1 (b) Only 2
r2 Here, side of triangle OAB formed by (c) Both 1 and 2 (d) Neither 1 nor 2
the parts of the diagonal with long
∴Outer radius of the cylinder side are 2.5x, 2.5x and 4x. _ (a) Since, BC|| AD
( r1 ) = ( 3 + 1) = 4 feet And side of triangle OBC formed by ∴∠PAD = ∠CQP
Now, Inner surface area of cylinder the parts of the diagonal with shord CP = PD [P is the middle point of CD]
= 2 πr2h side are 2.5x, 2.5x and 3x.
and ∠CPQ = ∠APD
1 3x
= 2 π( 3)(7 ) feet 2 × 4x ×

ar( ∆OAB) 2
= 2 = 4×3 Q
= 42 π feet 2
ar( ∆OBC ) 1 × 3x × 4x 3×4
∴ 42 π feet ≡ 1 L2
2 2
1
⇒ 1 feet 2 ≡ L ⇒ ar( ∆AOB) : ar( ∆BOC ) = 1 : 1
42 π P
D C
Total surface area of the cylinder 77. Suppose a region is formed by
= 2 πr1h + 2 πr2h + 2( πr12 − πr22 ) removing a sector of 20º from a
= 2 πh( r1 + r2 ) + 2 π( r1 − r2 )( r1 + r2 ) circular region of radius 30 feet.
= 2 π × 7( 4 + 3) + 2 π( 4 − 3) ( 4 + 3) What is the area of this new region?
= 2 π( 49) + 14 π (a) 150π sq feet (b) 550π sq feet A B
= 112 π feet 2 (c) 650π sq feet (d) 850π sq feet
∴ ∆PDA ~ = ∆PCQ [by AAS]
∴Required quantity of same colour to _ (d) Area of new region ∴ ar( ∆PDA ) = ar( ∆PCQ )
θ  2 θ 
= πr 2 −  2
paint all the surface of cylinder
1 8  πr = π r  1 −  Now, ∠BQA = ∠CQP
= 112 π × L= L  360°   360° 
42 π 3 Since, the ∆PCQ is a part of the
20° 
= π( 30) 2  1 −  17 
 = π( 900)   ∆BQA.
75. A square and a rectangle have  360°   18  So, there area cannot be equal.
equal areas. If one side of the = 850 π sq feet
rectangle is of length numerically 80. How many cubic metre of earth is
78. ABCD is a parallelogram where to be dug out to dig a well of
equal to the square of the length of
the side of the square, then the AC and BD are the diagonals. If radius 1.4 m and depth 5 m?
other side of the rectangle is ∠BAD = 60º, ∠ADB = 90º, then (a) 30.2 m 3 (b) 30.4 m 3
(a) square root of the side of the what is BD 2 equal to? (c) 30.6 m 3 (d) 30.8 m 3
square 3 2 3 1 2
(a) AB (b) AB 2 (c) AB 2 (d) AB 2 _ (d) Volume of well with radius 1.4 m
(b) half the side of the square 5 4 2 3 and depth 5 m.
(c) of unit length 22
_ (b) Given, ABCD is a parallelogram . )2 × 5 =
= π × (14 × 14
. × 14
. ×5
(d) double the side of the square where, AC and BD are the diagonals 7
and ∠BAD = 60° , ∠ADB = 90° = 30.8 cubic metre
_ (c) Let side of square2be x, then one
side of rectangle be x and let other B A 81. If the diagonals of a rhombus are
side of rectangle be y. 60°
x and y, then what is its area?
Then, according to the question, xy xy
(a) (b)
x 2 = x 2 × y (area of square = area of 2 4
rectangle) (c) xy (d) x2 − y2
⇒y=1 C D
CDS Solved Paper 2020 (I) 51

_ (a) Area of rhombus In right angled ∆ABC, CQ 3 r


∴ cos 30° = = =
1 AB = 5 − ( 4.8) ⇒ AB = 14
2 2 2 x (let)
= × diagonal I × diagonal II . m OC
2 2r
1
and in right angled triangle ADE ⇒ x = … (i)
= xy 3
2 AD = 5 − (14
2
. ) = 4.8 2
A
82. The lengths of sides of a triangle ∴ The breadth of the room
BD = AB + AD = 14 . + 4.8 = 6.2 m
are 3x, 4 y, 53 z, where h
86. What is the area of the largest
3x < 4 y < 53 z . If one of the 2x
P
2x
square plate cut from a circular
angles is 90º, then what are the disk of radius one unit? O x
minimum integral values of (a) 4 sq units (b) 2 2 sq units B C
x , y and z respectively? (c) π sq units (d) 2 sq units 2x
(a) 1, 2, 3 (b) 2, 3, 4 Let P be the position of 4th ball and
_ (d) For largest square plate cut from
(c) 1, 1, 1 (d) 3, 4, 5 a circular disk OP is the distance of centre of 4th
ball from the ground.
_ (c) Lengths of sides of a triangle are
3x , 4 y, 53 z, where 3x < 4 y < 53 z In ∆POC, ∠POC = 90°
and one angle is 90°. CP 2 = OP 2 + OC 2
∴ ( 53 z )2 = ( 3x )2 + ( 4 y )2 ⇒ OP 2 = CP 2 − OC 2 = (2 r )2 − x 2
2
⇒ 25 z2/ 3 = 9x 2 + 16 y  2 r  [By using Eq. (i)]
= 4r 2 −  
⇒ Value of x , y, z = 1 which satisfied  3
the equation. 4r 2 8r 2
Diagonal of square = 4r 2 − =
83. What is the maximum number of 3 3
= Diameter of circle
circumcircles that a triangle can ⇒ OP =
2 2r
=2
2
⇒ 2 × side of square r unit
have? 3 3
= 2 × 1 = 2 unit
(a) 1 (b) 2 (c) 3 (d) Infinite 2 88. A right circular cylinder just
⇒ side of square = = 2 unit.
_ (a) A triangle has maximum one 2 encloses a sphere. If p is the
circumcircles since it touches all the
vertex of the triangle. ∴Area of the largest square plate surface area of the sphere and q is
= (side )2 = ( 2 )2 = 2 sq. units the curved surface area of the
84. If an arc of a circle of radius 6 cm cylinder, then which one of the
subtends a central angle 87. Out of 4 identical balls of radius r,
3 balls are placed on a plane such following is correct?
measuring 30º, then which one of (a) p = q
the following is an approximate that each ball touches the other
(b) p = 2q
two balls. The 4th ball is placed on
length of the arc? (c) 2 p = q
(a) 3.14 cm (b) 2.15 cm
them such that this ball touches all (d) 2 p = 3q
(c) 2.14 cm (d) 2 cm the three balls. What is the
_ (a) Let radius of sphere be r.
θ distance of centre of 4th ball from
_ (a) Length of arc = × 2 πr ∴Surface area of sphere,
360° the plane?
p = 4 πr 2 … (i)
30 2
= × 2 × π × 6 = π = 3.14 cm (a) 2 r unit
360° 3
85. A ladder 5 m long is placed in a 3+2 2
(b) r unit
room so as to reach a point 4.8 m 2
r
high on a wall and on turning the (c) unit
3−2 2
ladder over to the opposite side of
3+2 2
the wall without moving the base, (d) r unit
3 Now, height of cylinder, h = 2 r
it reaches a point 1.4 m high. What
∴Surface area of the cylinder
is the breadth of the room? _ (a) Let Q be the mid-point of BC and
since ∆ABC is an equilateral triangle. q = 2 πrh = 2 πr(2 r )
(a) 5.8 m (b) 6 m (c) 6.2 m (d) 7.5 m
= 4 πr 2 … (ii)
_ (c) Here, AC and AE are portion of From Eqs. (i) and (ii), we get p = q
ladder. A
89. ABCD is a quadrilateral such that
C
r r AD = DC = CA = 20 units,
r
BC = 12 units and ∠ABC = 90º.
O r
E m 4.8 What is the approximate area of
5m 5 Q
1.4 m B r r C the quadrilateral ABCD ?
m
(a) 269 sq units (b) 300 sq units
D A B (c) 325 sq units (d) 349 sq units
52 CDS Solved Paper 2020 (I)

_ (a) In right angled triangle ABCD, 91. What is the area of the shaded sinθ =
BC
=
3 3
=
3
AB2 = AC 2 − BC 2 region in the given figure, if the OB 6 2
= (20)2 − (12 )2 = 400 − 144 radius of each of the circles is 2 cm? ⇒ sinθ = sin 60° ⇒ θ = 60°
⇒ AB = 256 = 16 units ∴∠AOB = 2θ = 120°
20 units A B ∴Area of shaded region APBA
D C
= Area of sector
APBOA − area of ∆ABO
 120°  1
=  π( 6) − × 3 × 6 3
2
its

20 units 12 units C  360°  2


un

[Q OC = 3 units]
20

(a) 4 3 − 2 π cm 2 1
= × π × 36 − 9 3 = 12 π − 9 3
(b) 3 − π cm 2 3
A B
π = 3( 4 π − 3 3 ) sq units
∴Area of quadrilateral ABCD (c) 3 − cm 2
= ar( ∆ACD ) + ar( ∆ABC ) 2 93. In the given figure, if y = 6 and
3 1 (d) 2 π − 2 3 cm 2 x
= (side) 2 + × Base × height
z
4 2 _ (a) Side of the equilateral triangle = 5, then what is the value of x ?
=
3 1
× (20)2 + × 16 × 12 ABC = 2 × 2 = 4 cm x
4 2 ∴Area of shaded region
= 100 3 + 96 = Area of equilateral triangle ABC
= 269 sq units (approx.) − 3 × Area of sector with angle 60° y
and radius 2 cm x z
90. Let PQRS be the diameter of a
60° 
× ( 4)2 − 3 × 
3 (a) 45º (b) 30º (c) 15º (d) 10º
circle of radius 9 cm. The length =  π(2 )
2
4  360°  y z
PQ, QR and RS are equal. _ (c) Given, = 6 and = 5
1
Semi-circle is drawn with QS as = 3 × 4 − 3 × × π( 4) x x
6 ⇒ y = 6x and z = 5x
diameter (as shown in the given
= 4 3 − 2 π cm2 Now, x + y + z = 180° [straight angle]
figure). What is the ratio of the
⇒ x + 6x + 5x = 180°
shaded region to that of the 92. In the given figure, what is the
⇒ 12 x = 180 ⇒ x = 15°
unshaded region? area of the shaded region?
94. ABCD is a trapezium, where AB is
parallel to DC. If AB = 4 cm,
BC = 3 cm, CD = 7 cm and
P S
6√3 uints DA = 2 cm, then what is the area
Q R of the trapezium?
6 uints 6 uints
2 3
(a) 22 cm2 (b) 22 cm2
3 2
6√3 uints
(a) 25 : 121 (b) 5 : 13 22 2
(c) 22 3 cm2 (d) cm2
(c) 5 : 18 (d) 1 : 2 3
(a) 9( π − 3 ) sq units
_ (b) We have, PS = 2 × 9 = 18 cm _ (d) Area of trapezium, when the
(b) 3(4 π − 3 3 ) sq units lengths of parallel and non-parallel
Since, PQ = QR = RS (c) 3(3 π − 4 3 ) sq units sides are given
PS 18 a+ b
∴PQ = QR = RS = = = 6 cm (d) 9( 3 − π ) sq units = s( s − k )( s − c )( s − d )
3 3 k
∴Area of the shaded region _ (b) Diameter of circle Where k = b − a = (7 − 4) = 3 cm
= diagonal of rectangular part
1
= ( π × 92 − π × 62 ) k+c+d 3+ 2 + 3
and s = = = 4 cm
2 = ( 6 3 )2 + ( 6)2 2 2
π 45 = 108 + 36 = 144 = 12 A 4 cm B
= ( 81 − 36) = π cm2
2 2 12
∴Radius of circle == 6 cm
And area of unshaded region 2 2 cm 3 cm
h
1 In ∆OAB,
= ( π × 92 + π × 62 )
2 P
D 7 cm C
π π
= ( 81 + 36) = × 117
2 2 ∴Required area of trapezium
6√ 3 units
A B (4 + 7 )
∴Ratio of the shaded region to that = 4( 4 − 3) ( 4 − 2 ) ( 4 − 3)
θ 3
of the unshaded region 6 units
6 11 11 × 2 2
45 O
π = 4(1)(2 )(1) =
45 3 3
= 2 = = 5 : 13
117
π 117 =
22 2
cm 2
2 3
CDS Solved Paper 2020 (I) 53

95. Angles are shown in the given 97. In the given figure AB is parallel to 99. What is the approximate area of the
figure. What is the value of CD and AC is parallel to BD. If shaded region in the figure given?
∠1 + ∠2 + ∠3 + ∠4 + ∠5 + ∠6 ∠EAC = 40º, ∠FDG = 55º,
+ ∠7 + ∠8 ? ∠HAB = x °, then what is the value
3 of x?
8 cm
H
2
4 x B 6 cm 6 cm
A
8 cm
1 40°

D
F
8 E C 55° (a) 15.3 cm2 (b) 25.5 cm2
5
(c) 28.4 cm2 (d) 30.5 cm2
G
7 6 (a) 85 (b) 80 _ (d) Diameter of circle = diagonal of
(a) 240º (b) 360º (c) 75 (d) 65 rectangular region = 82 + 62
(c) 540º (d) 720º
_ (a) Given, AB||CD and AC|| BD. = 64 + 36 = 100 = 10 cm
_ (b) Sum of angles of a quadrilateral ∴∠BAC = ∠BDC = 55° 10
= 360° ∴Radius of circle = = 5 cm
Now, 2
⇒ (180° − ( ∠1 + ∠2 )) + (180°
∠HAB + ∠BAC + ∠CAE = 180° ∴Area of shaded region
− ( ∠ 3 + ∠ 4) + (180° − ( ∠ 5 + ∠ 6))
⇒ x + 55° + 40° = 180° = Area of circle − area of rectangular
+ (180° − ∠7 + ∠ 8)) = 360°
⇒ x + 95° = 180° region
⇒ 720°− ( ∠1 + ∠2 + ∠ 3 + ∠ 4 + ∠ 5 = π( 5)2 − ( 8 × 6) = 314
. × 25 − 48
⇒ x = 180° − 95°
+ ∠6 + ∠7 + ∠ 8) = 360° = 78.5 − 48 = 30.5 cm2
⇒ x = 85°
⇒ ∠1 + ∠2 + ∠ 3 + ∠ 4 + ∠ 5
98. In the given figure, there are three 100. Consider the following statements
+ ∠ 6 + ∠7 + ∠ 8 = 360°
semi-circles ABC, AEF and CDF. with reference to the given figure.
96. In the given figure PQ is parallel to The distance between A and C is B
RS, ∠AEF = 95º, ∠BHS = 110º, and 28 units and F is the mid-point of
∠ABC = x °. Then, what is the AC. What is the total area of the
A
value of x? three semi-circles?
P R O
B

D C
A
E 1. The sum of the areas of ∆AOD
95°
G
A
F
C and ∆BOC is equal to the sum of
x
the areas of ∆AOB and ∆DOC.
B
E D 2. ∠AOD = ∠BOC
H 110°
C F (a) 924 sq units (b) 824 sq units 3. AB + BC + CD + DA > AC + BD
(c) 624 sq units (d) 462 sq units Which of the above statements are
_ (d) Given, AC = 28 units correct?
Q S (a) 1 and 2 (b) 2 and 3
28
(a) 15 (b) 25 ∴ AF = = 14 units
2 (c) 1 and 3 (d) 1, 2 and 3
(c) 30 (d) 35
∴Area of three semi-circles _ (b) ∠AOD = ∠BOC
_ (b) ∠BEF = 180° − 95° = 85° ABC , AEF and CDF [vertically opposite angles]
and ∠FHG = 110° π(14)2 π(7 )2 π(7 )2 In ∆ADC , AD + DC > AC … (i)
= + +
[vertically opposite angle] 2 2 2 In ∆ABC , AB + BC > AC … (ii)
∴Since PQ|| RS π In ∆BCD, BC + CD > BD … (iii)
= (14 + 7 + 7 )
2 2 2

∴ ∠EFH + ∠GHF = 180° 2 and ∆BCD, AB + AD > BD … (iv)


π On adding Eqs. (i), (ii), (iii) and (iv),
⇒ ∠EFH + 110° = 180° = (196 + 49 + 49 )
2 we get
⇒ ∠EFH = 180 − 110° = 70° 2 ( AB + BC + CD + AD )
π
In ∆BEF, ∠B + ∠F + ∠E = 180° = (294) > 2( AC + BD )
2
⇒ x + 70° + 85° = 180° 22 1 ⇒ AB + BC + CD + AD > AC + BD
⇒ x + 155° = 180° = × × 294 It is not necessary that
7 2
⇒ x = 180°−155° ar ( ∆AOD ) + ar (∆BOC )
= 462 sq units
⇒ x = 25° = ar ( ∆AOB) + ar ( ∆DOC )
PAPER II English

Directions (Q. Nos. 1-10) In this _ (a) According to the given passage, 5. Which word in the passage means
section you have a few short passages. all civilisations have developed after ‘changeover’?
After each passage, you will find some going through a phase of agriculture.
(a) Transitioned
Hence, agriculture had been as part
items based on the passage. First, read (b) Channel
of all civilisations.
a passage and answer the items based (c) Coerce
on it. You are required to select your 2. A significant number of people (d) Hierarchies
answers based on the contents of the were sent to carry out other work _ (a) The synonym of changeover is
passage and opinion of the author only. from agriculture because transitioned. Both word represent a
(a) there were insufficient agricultural shift to a new way.
PASSAGE-I products.
PASSAGE-II
Not all agricultural societies become (b) people were needed to build
civilisations, but no civilisation can monuments, weapons, jewellery, When we pick up a newspaper, a
become one without passing through etc. book or an article, we come to our
the stage of agriculture. This is (c) there were sufficient agricultural task with certain preconceptions
because at some stage in the products. and predispositions. We expect to
development of agriculture, as (d) this enabled the development of find a specific piece of information
civilisations. or be presented with an argument or
productivity improves, not all people
would need to be engaged in _ (c) The given passage states that as an analysis of something, say, the
producing or procuring food. the agriculture production was bulk, likelihood of recession in the next
people had to time to invest their time six months or the reasons why
A significant number of people could in other activities.
children can’t read. We probably
be freed up to pursue other activities
such as building walls or monuments
3. What kind of agriculture based know a little about the book or
for new cities; making new tools, societies would emerge as article we are reading even before
weapons and jewellery; organising civilisations? we start. There was, after all, some
(a) Societies which achieved high reason why we chose to read one
long-distance trade; creating new
artistic masterpieces; coming up with productivity in agriculture had the piece of writing rather than another.
opportunity to find time for other Our expectations and
new inventions; keeping accounts; and
work. predispositions may, however, blind
perhaps constructing new public
(b) Societies which depended on us to what the article and its author
infrastructure such as irrigation canals agriculture completely moved to
that further improve the productivity of other fruitful work so as to move is actually saying. If, for example,
agriculture, thus realising even more to many places. we are used to disagreeing with the
people to do new things. (c) Societies which transitioned from author, we may see only what we
This can happen, of course, only if a one stage of agriculture to expect to see and not what is
another. actually there. Day after day in our
society that has transitioned to
(d) Societies which could not do routine pattern of life we expose
high-productivity agriculture has agriculture for lack of resources ourselves to the same newspaper,
also, at some stage in its evolution, moved to other work. the same magazine, even books by
found a way to channel the bonanza of
free time into other work fruitfully. _ (a) According to the given passage, authors with the same perspectives.
those agricultural societies where In order to reflect on our reading
In the ancient world, this often agricultural production was high and habits and improve our skills we
involved creating new ideologies and in bulk, those societies emerged as need to break out of this routine,
new hierarchies or power structures civilisations. step back and look at what we are
to coerce or otherwise convince large
4. People as groups were convinced doing when we read.
groups of people to devote their time
to the new tasks for very little reward.
to do new work through 6. According to the author, which
(a) reward, force and community one of the following statements is
1. Which one of the following persuasions. not true?
statements is true according to the (b) ideologies, hierarchies and power
(a) Reader’s preconceptions
author? structures.
influence their reading.
(a) Agriculture has always been part (c) excessive agricultural products.
(b) Readers have expectations when
of all civilisations. (d) very high rewards. they read an article or a book.
(b) Not all civilisations have undergone _ (b) The last line of the passage (c) Readers look for specific
the processes of agriculture. states that it was ‘ideologies, information in any of their
(c) Agriculture gave birth to new hierarchies and power structures’ that readings.
civilisations. convinced or forced people to devote (d) Readers assume that everything
(d) Communities discontinued their time to activities other than they read will have new
agriculture to become civilisations. agriculture. information.
CDS Solved Paper 2020 (I) 55

_ (d) As per the given passage, readers _ (d) The synonym of ‘viewpoints’ is finance would point the same way.
have certain presumptions and ideas ‘perspectives’, as both words means (c) / No error (d)
that govern what and how they would a persons opinion or point of view.
read any literary work. In fact, it also
Directions (Q. Nos. 11-20) Each _ (c) Replace ‘were’ with ‘where’ to make
points that readers look for some the given sentence grammatically and
specific pieces of information in any item in this section has a sentence with contextually meaningful.
literature. Hence, option (d) is not three underlined parts labelled as (a),
17. Evolutionary biology leave us (a) /
given in the passage. (b) and (c). Read each sentence to find
out whether there is any error in any distinctly pessimistic about the
7. Our expectations and possibility (b) / that altruism can
predispositions may, however, underlined part and indicate your
response on the Answer Sheet against arise naturally among humans. (c)
blind us because / No error (d)
the corresponding letter i.e., (a) or (b)
(a) we may not get the actual ideas
of the author. or (c). If you find no error, your _ (a) Replace ‘leave’ with ‘leaves’ to
response should be indicated as (d). make the given sentence
(b) we will get the actual ideas of the grammatically and contextually
author. 11. After mysteriously expanding for meaningful.
(c) we may disagree with the author. decades (a) / Antarctica’s sea ice
(d) we will agree with all the ideas of 18. When everything starts working
the author.
cover (b) / starting melting. (c) /
for you (a) / you will find (b) /
No error (d)
_ (a) The passage points out that things are achieve and delivered.
whenever we have preconceived _ (c) Replace ‘starting’ with ‘started’ to (c) / No error (d)
ideas or viewpoints, it blinds us or make the given sentence
does not let us understand what the grammatically and contextually _ (c) Replace ‘achieve’ with ‘achieved’
meaningful. to make the given sentence
author truly wants to say. We often
grammatically and contextually
assume the meaning to be something 12. The auction, conducted by the meaningful.
else than what the author actually
wanted to say. bank (a) / will be price based (b) /
19. If I were you (a) / I would not go
using multiple priced method. (c) /
8. One of the ways to improve our for (b) / change of job. (c) / No
No error (d)
reading habits is to error (d)
_ (c) Replace ‘method’ with ‘methods’
(a) break the routine by changing the to make the given sentence _ (c) Add ‘a’ before change to make the
time of reading. grammatically and contextually given sentence grammatically and
(b) change the types of topics we meaningful. contextually meaningful.
read. 20. At the beginning of the nineteenth
(c) break the routine of reading the 13. If the scheme would have been
same newspaper. implemented effectively (a) / all century, (a) / female literacy was
(d) stop reading for some time and affected (b) / would have extremely lowed (b) / in
then restart reading. benefitted. (c) / No error (d) comparison to male literacy. (c) /
No error (d)
_ (c) According to the given passage, _ (a) Replace ‘would have been’ with
the best way to improve our reading ‘had been’ to make the given _ (b) Replace ‘lowed’ with ‘low’ to make
habit is to break the routine of reading sentence grammatically and the given sentence grammatically and
the same literary work again and contextually meaningful. contextually meaningful.
again.
14. Government Stock offers (a) / Directions (Q. Nos. 21-30) Each of
9. Which quality does the author safety, liquidity and attractive the following items in this section
here advocate, to be a good returned (b) / for long duration. (c) consists of a sentence, parts of which
reader? / No error (d) have been jumbled. These parts have
(a) Being objective to the ideas of been labelled as P, Q, R and S. Given
the author. _ (b) Replace ‘returned’ with ‘returns’ to below each sentence are four
make the given sentence
(b) Having preconceptions and sequences, namely (a), (b), (c) and (d).
grammatically and contextually
predispositions.
meaningful. You are required to re-arrange the
(c) Having continuous routines.
15. Scrolling thorough my social jumbled parts of the sentence and
(d) Disagreeing with the author.
mark your response accordingly.
_ (a) The passage clearly states that we
media timeline, (a) / I hovered
must be open and objective to the over a video (b) / of a minor road 21. for long and (P) / the backbone of
ideas of the author to become an traffic accident. (c) / No error (d) India (Q) / will continue to be the
effective reader. same (R) / agriculture has been (S)
_ (a) Replace ‘thorough’ with ‘through’ to
10. Which word in the passage means make the given sentence (a) SPQR
‘viewpoints’? grammatically and contextually (b) SQPR
meaningful. Thorough means in detail. (c) QRSP
(a) Preconceptions
(b) Predispositions 16. The fascination with gold at least (d) QSRP
(c) Pattern (a) / seems to be a case were _ (b) The correct and meaningful order
(d) Perspectives traditional belief and (b) / modern of the parts of sentence is SQPR.
56 CDS Solved Paper 2020 (I)

22. the cry of general public (P) / _ (b) The correct and meaningful order 34. The dog days
agenda in any country (Q) / public of the parts of sentence is RQPS.
(a) Days celebrating dogs
policy making (R) / is generally 29. poetry is (P) / and ideas (Q) / (b) The bitter days
driven by (S) powerful feelings (R) / the (c) The hottest days
spontaneous overflow of (S) (d) The coldest days
(a) RQSP (b) RPSQ
(c) PSRQ (d) QRSP (a) SRQP _ (b) The idiom ‘the dog days’ means
(b) PQRS ‘The bitter days’.
_ (a) The correct and meaningful order
of the parts of sentence is RQSP. (c) RSQP 35. A banana republic
(d) PSRQ (a) A small or poor country with a
23. before it starts (P) / of the
government is (Q) The essential _ (d) The correct and meaningful order weak government.
of the parts of sentence is PSRQ. (b) A small or poor country which
power (R) / the power to manage produces banana.
conflict (S) 30. historical identity and a common
(c) A country which has been
(a) RSPQ (b) SQRP descent (P) / a group of people (Q) occupied a big country.
(c) RQSP (d) QRSP / is called an ethnic group (R) / (d) A country without any government.
who share a common culture (S)
_ (c) The correct and meaningful order _ (a) The idiom ‘a banana republic’
of the parts of sentence is RQSP. (a) QSPR means ‘a small or poor country with a
(b) QRPS weak government’.
24. a majority of the vote (P) / the (c) PSQR
party that received (Q) / of the (d) RQPS 36. The pros and cons
government (R) / must take (a) The good and bad parts of a
_ (a) The correct and meaningful order situation.
control (S) of the parts of sentence is QSPR.
(b) Like and dislike of a situation.
(a) QPSR (b) PSRQ
Directions (Q. Nos. 31-40) Given (c) A bad experience in an event.
(c) RSPQ (d) SQPR
below are some idioms/phrases (d) A good moment of an event.
_ (a) The correct and meaningful order followed by four alternative meaning
of the parts of sentence is QPSR. _ (a) The idiom ‘The pros and cons’
to each. Choose the response (a), (b), means ‘the good and bad parts of a
25. can express a view on (P) / in (c) or (d) which is the most situation’.
which the electorate (Q) / a appropriate expression and mark your 37. Prime the pump
particular issue of public policy (R) response in the Answer Sheet (a) To do something in order to make
/ a referendum is a vote (S) accordingly. something succeed.
(a) SQPR (b) RPQS 31. A paper tiger (b) To do good things to succeed in life.
(c) QRSP (d) PQRS (a) Person or organisation that (c) To do something in order to get
bad things done.
_ (a) The correct and meaningful order appears powerful, but actually is
of the parts of sentence is SQPR. not. (d) Asking people to do things to
(b) Person or organisation that acts make something succeed.
26. in modern societies (P) / or merely
suppressed (Q) / has class conflict
like a tiger. _ (a) The idiom ‘prime the pump’
(c) People who campaign for the means ‘to do something in order to
(R) / been resolved (S) protection of tigers. make something succeed’.
(a) RPSQ (b) RSPQ (d) A daredevil. 38. The green-eyed monster
(c) PRSQ (d) QRSP
_ (a) The idiom ‘a paper tiger’ means (a) Feeling of being joyous
_ (a) The correct and meaningful order ‘a person or organisation that (b) Feeling of being jealous
of the parts of sentence is RPSQ. appears to be powerful but actually is
(c) Feeling bad about happenings
not’.
27. several of our food (P) / are being (d) Feeling lucky about something
32. Lily-livered
extensively cultivated (Q) / and _ (b) The idiom ‘the green-eyed
vegetable crops (R) / hybrid (a) Brave and courageous monster’ means ‘feeling of jealousy’.
varieties of (S). (b) Not brave
(c) Comical
39. Rise to the occasion
(a) QRSP (b) SPQR (a) To celebrate a success in a
(d) Outrageous
(c) QPRS (d) SPRQ difficult situation.
_ (b) The idiom ‘lily livered’ means (b) To regret a situation which ended
_ (d) The correct and meaningful order ‘coward or not brave.
of the parts of sentence is SPRQ. in failure.
33. Eat like a bird (c) To succeed in dealing with a
28. against the officer (P) / reason for difficult situation.
(a) Eat fast
the accusation (Q) / there should (d) To motivate people to succeed in
(b) Eat very little
have been / (R) who was in-charge a difficult situation.
(c) Eat a lot
at that time (S) (d) Pretending to be eating _ (c) The idiom ‘rise to the occasion’
(a) RPSQ (b) RQPS means ‘to succeed in dealing with a
(c) PQRS (d) SPRQ
_ (b) The idiom ‘Eat like a bird’ means difficult situation’.
‘eat very little’.
CDS Solved Paper 2020 (I) 57

40. Call it a day R : It contains a slot for every item S : Progress in physics and
(a) End of the day of information coming to us. chemistry proceeded much faster
(b) Completion of work S : As E.F. Schumacher says, “When than in biology.
(c) Stop doing something we think, we do not just think; we The correct sequence should be
(d) A beautiful day think ideas.” (a) QPRS (b) PRQS
_ (b) The idiom ‘call it a day’ means ‘the The correct sequence should be (c) RPQS (d) SRPQ
completion of work’. (a) PSRQ (b) SPRQ _ (d) The correct and meaningful
Directions (Q. Nos. 41-50) In this (c) QRPS (d) RQPS paragraph is given by SRPQ.
section each item consists of six _ (c) The correct and meaningful 45. S1 : People in society need many
sentences of a passage. The first and paragraph is given by QRPS. goods and services in their
the sixth sentences are given in the everyday life including food,
beginning are S1 and S6. The middle 43. S1 : Biology is the study of life in its
entirety. clothing, shelter, transport, etc.
four sentences in each have been
jumbled up and labelled as P, Q, R S6 : Classical descriptive and clueless S6 : The teacher in the local school
and S. You are required to find out biology found a theoretical has the skills required to impart
the proper sequence of the four framework in the evolutionary education to the students.
sentences and mark your response theory of Darwin. P : A weaver may have some yarn,
accordingly on Answer Sheet. P : In later years, the focus was some cotton and other instruments
physiology and internal required for weaving cloth.
41. S1 : Chinua Achebe was born in 1930
and educated at the Government morphology or anatomy. Q : A family farm may own a plot of
College in Umuahia, Nigeria. Q : Darwinian ideas of evolution by land, some grains, farming
natural selection changed the implements, may be a pair of
S6 : Chinua Achebe has written over bullocks and also the labour
twenty books, including novels, perception completely.
R : The growth of biology as a services of the family members.
stories, essays and collections of
poetry, and won the Nobel Prize natural science during the last R : Every individual has some
for literature. 1000 years is interesting from amount of the goods and services
many points of view. that one would like to use.
P : During the Civil War in Nigeria,
he worked for the Biafran S : One feature of this growth is S : In fact, the list of goods and
government service. changing emphasis from mere services that any individual needs
description of life forms to is so large that no individual in
Q : After the War, he was appointed society, to begin with, has all the
Senior Research Fellow at the identification and classification of
all recorded living forms. things one needs.
University of Nigeria, Nsukka.
The correct sequence should be The correct sequence should be
R : He joined the Nigerian
(a) RSPQ (b) SPRQ (a) PQRS (b) RSPQ
Broadcasting Company in Lagos
(c) QRPS (d) PQRS (c) QPSR (d) SRQP
in 1954, later becoming its
Director of External Broadcasting. _ (a) The correct and meaningful _ (d) The correct and meaningful
paragraph is given by RSPQ. paragraph is given by SRQP.
S : He received a BA from London
University in 1953 and in 1956 he 44. S1 : Biology is the youngest of the 46. S1 : Farming is the main production
studied broadcasting in London at formalised disciplines of natural activity in the village.
the BBC. science. S6 : The new ways of farming need
The correct sequence should be S6 : Life expectancy of human beings less land, but much more capital.
(a) SRPQ (b) RPQS has dramatically changed over the P : These have allowed the farmers
(c) PQRS (d) QRSP years. to produce more crops from the
_ (a) The correct and meaningful P : However, the twentieth century same amount of land.
paragraph is given by SRPQ. and certainly the twenty-first Q : Over the years there have been
42. S1 : “Every person carries in his head century has demonstrated the many important changes in the
a mental model of the world-a utility of biological knowledge in way farming is practised.
subjective representation of furthering human welfare, be it in R : But in raising production, a great
external reality,” writes Alvin health sector or agriculture. deal of pressure has been put on
Toffler in Future Shock. Q : The discovery of antibiotics, and land and other natural resources.
S6 : When we begin to think we can synthetic plant-derived drugs, S : This is an important
do so only because our mind is anaesthetics have changed achievement, since land is fixed
already filled with all sorts of medical practice on one hand and and scarce.
ideas with which to think. human health on the other hand. The correct sequence should be
P : It organises our knowledge and R : Applications of physics and (a) QPSR (b) RSPQ
gives us a place from which to argue. chemistry in our daily life also (c) SRPQ (d) PRSQ
have a higher visibility than those
Q : This mental model is, he says, _ (a) The correct and meaningful
like a giant filing cabinet. of biology. paragraph is given by QPSR.
58 CDS Solved Paper 2020 (I)

47. S1 : Britain was the first country to _ (d) The correct and meaningful Directions (Q. Nos. 51-60) Each of
experience modern paragraph is given by RPSQ. the following sentences in this section
industrialisation. 49. S1 : All governments claim eternal has a blank space and four words or
S6 : This gave people a wider choice consistency and success. group of words are given after the
for ways to spend their earnings S6 : Diplomacy offers choices, and sentence. Select the most appropriate
and expanded the market for the those choices must be negotiated word or group of words for the blank
sale of goods. with other sovereign actors. space and indicate your response on
P : This meant that the kingdom had P : Choices involved uncertainty, the Answer Sheet accordingly.
common laws, a single currency risk and immediacy; those who 51. On his way to the capital, the
and a market that was not must take the choices operate in minister ……… the eminent social
fragmented by local authorities the contemporary political milieu.
and uneven taxation. worker at his residence.
Q : And yet the essence of (a) called on (b) called
Q : It had been politically stable governance is choice. (c) calling for (d) call off
since the seventeenth century, R : Nowhere is this more true than
with England, Wales and Scotland in foreign policy decision-making. _ (a) The word ‘called on’ makes the
unified under a monarchy. sentence grammatically correct and
S : Some even claim omniscience. contextually meaningful.
R : By then a large section of the
people received their income in The correct sequence should be 52. The fire brigade fought for four
the form of wages and salaries (a) SQPR (b) QSRP hours to ……… the fire in the
than in goods. (c) SRPQ (d) RSPQ building.
S : By the end of the seventeenth _ (a) The correct and meaningful (a) put in (b) put out
century, money was widely paragraph is given by SQPR. (c) put on (d) put off
used as the medium of 50. S1 : Buddhism continued to spread _ (b) The word ‘put out’ makes the
exchange. into many lands of Asia during sentence grammatically correct and
The correct sequence should be the period of 5th and 6th century. contextually meaningful.
(a) QPSR (b) PSQR S6 : He translated several scriptural 53. Ravi has proved that he can ………
(c) RSQP (d) SRQP commentaries into Pali and wrote on his promise by winning the
_ (b) The correct and meaningful a work called the Visuddimagga, match.
paragraph is given by PSQR. which soon attained the status of
(a) carry through (b) carry out
a classic work on Theravada (c) carry (d) carry off
48. S1 : For several million years, doctrine and meditation.
humans lived by hunting wild _ (b) The word ‘carry out’ makes the
animals and gathering wild plants. P : While this can be understood as a sentence grammatically correct and
part of larger processes of cultural contextually meaningful.
S6 : As a result, conditions were interaction, especially trade, a key
favourable for the growth of role was played by monks. 54. It is best to ……… politics when in
grasses such as wild barley and the classroom.
wheat. Q : We know a little bit about some
of them, but there must have been (a) keep out (b) keep on
P : This led to the development of countless men whose (c) keep off (d) keeping
farming and pastoralism as a way
of life.
commitment to the Buddhist path _ (a) The word ‘keep out’ makes the
gave them the courage and sentence grammatically correct and
Q : This change took place because determination to persevere in the contextually meaningful.
the last ice age came to an end face of the long, hard journey to
about 13,000 years ago and with 55. It shows that she has ……… many
India and back. years of service.
that warmer, wetter conditions R : Buddhism had made its way to
prevailed. (a) put in (b) put out
Sri Lanka many centuries earlier, (c) put (d) put on
R : Then, between 10,000 and 4,500 during the time of Ashoka, and a
years ago, people in different thriving Buddhist community _ (a) The word ‘put in’ makes the
parts of the world learnt to sentence grammatically correct and
soon took root. contextually meaningful.
domesticate certain plants and S : In the 5th century, the monk
animals. Buddhaghosha travelled to Sri 56. The chairperson said that the
S : The shift from foraging to Lanka. group was ………… of time.
farming was a major turning point The correct sequence should be (a) running out (b) running
in the human history. (c) running with (d) run out
(a) RQPS (b) QRPS
The correct sequence should be (c) PQRS (d) PSRQ _ (a) The word ‘running out’ makes the
(a) QSPR (b) SPQR sentence grammatically correct and
(c) PSQR (d) RPSQ _ (c) The correct and meaningful contextually meaningful.
paragraph is given by PQRS.
CDS Solved Paper 2020 (I) 59

57. If I ……… an angel, I would solve _ (b) The word incorporated means _ (a) The word ascertained means ‘to
the problems of people. ‘included’. From the given options, find out or discover’. From the given
‘integrated’ means the same and options, ‘determined’ means the
(a) am (b) were (c) was (d) have hence is the correct answer. same and hence is the correct
_ (b) The word ‘were’ makes the answer.
sentence grammatically correct and
64. His thesis makes all generic
statements which have already Directions (Q. Nos. 71-80) Each
contextually meaningful.
been proved. item in this section consists of
58. Where there is a ………, there is a sentences with an underlined word
(a) specific (b) crude
way. (c) broad (d) non-standard followed by four words or group of
(a) way (b) road (c) wing (d) will words. Select the option that is
_ (c) The word generic means ‘general opposite in meaning to the underlined
_ (d) The word ‘will’ makes the or common’. From the given options,
sentence grammatically correct and ‘broad’ means the same and hence is word and mark your response on the
contextually meaningful. the correct answer. Answer Sheet accordingly.
59. The police could not establish how 65. The captain produced yet another 71. Early medieval period was not a
the accident ……… . stellar show to make her team combination of urban and rural
(a) came off (b) came about enter the semi-finals. civilisation. It was not a period
(c) came on (d) came out (a) extraordinary (b) eclipse of urban decay as claimed by
some.
_ (b) The word ‘came about’ makes the (c) poor (d) not a great
sentence grammatically correct and (a) survival (b) waste away
contextually meaningful. _ (a) The word stellar means
‘extraordinary’. Hence, option (a) is (c) decomposition (d) spoil
60. I ………… my old friend after the correct answer. _ (a) The word decay means ‘to
twenty years. decompose or die’. From the given
66. A new show is trying to change options, ‘survival’ meaning to
(a) ran into (b) ran in the cliched depictions of women in continue to live is its antonym.
(c) run in (d) run on animation.
72. He speaks eloquently and can pull
_ (a) The word ‘ran into’ makes the (a) original (b) hackneyed
crowds.
sentence grammatically correct and (c) crony (d) artificial
contextually meaningful. (a) confusingly
_ (b) The word cliched means ‘a lack of (b) expressively
Directions (Q. Nos. 61-70) Each originality’. From the given options,
‘hackneyed’ means the same and (c) powerfully
item in this section consists of a (d) fluently
sentence with an underlined word hence is the correct answer.
followed by four words/group of 67. Not everyone finds a vocation _ (a) The word eloquently means
‘fluently or persuasively’. From the
words. Select the option that is which suits one’s aptitude. given options, ‘confusingly’ is its
nearest in meaning to the underlined (a) attitude (b) approach antonym.
word and mark your response on the (c) liking (d) occupation 73. Everyone has to fight the inertia
Answer Sheet accordingly.
_ (d) The word vocation means in the system.
61. All the developments that took ‘profession or occupation’. Hence,
option (d) is the correct answer. (a) sluggishness (b) indolence
place in the 20th century have had (c) activity (d) torpor
implications for the next century. 68. Uninterrupted rain had fatigued
_ (c) The word inertia means ‘lazy or
(a) consequences (b) interferences the commuters from the outskirts inactive’. From the given options,
(c) feedback (d) planning to the city and work suffered. ‘inactivity’ is its antonym.
_ (a) The word implications means (a) excited (b) refreshed 74. There is a need to promote
‘conclusion’. From the given options, (c) slowed (d) exhausted philanthropy in education.
‘Consequences’ means the same and
hence is the correct answer. _ (d) The word fatigued means (a) charity
‘exhausted’. (b) benevolence
62. He is such a leader that his actions 69. The leader said, “I am aghast with (c) nastiness
are contagious. the developments so far. I will take (d) likeliness
(a) complicated (b) transmittable time to understand this”. _ (c) The word philanthropy means ‘the
(c) effective (d) unthinkable desire to promote the welfare of
(a) satisfied (b) sad
others’. From the given options,
_ (b) The word contagious means (c) amused (d) horrified ‘nastiness’ is its antonym.
‘capable of being transmitted’. From
the given options, ‘transmitted’ is the _ (d) The word aghast means ‘horrified’. 75. What we lack in the current times
correct answer. 70. The cause of the accident is yet to is compassion.
63. The budget incorporated a number be ascertained, but police officials (a) empathy (b) carefulness
of tax reforms which included suspect the driver of the vehicle (c) indifference (d) hardship
higher taxes for the very rich. allegedly fell asleep.
_ (c) The word compassion means
(a) excluded (b) integrated (a) determined (b) curtained ‘kindness’. From the given options,
(c) laid down (d) removed (c) thought of (d) being known ‘indifference’ is its antonym.
60 CDS Solved Paper 2020 (I)

76. Tempestuous behaviour would not 82. The man in dark blue is the one _ (d) The given highlighted word is an
yield much in any place. who made us win the match. Interjection.

(a) relaxed (b) passionate (a) Relative clause 90. Ravi was declared as the winner in
(c) intense (d) windy (b) Interrogative pronoun the tie because he had hit the most
(c) Relative pronoun
_ (c) The word tempestuous means (d) Affirmative
number of fours and sixes.
‘characterised by strong and turbulent (a) Conjunction (b) Interjection
or conflicting emotion’. From the _ (c) The given highlighted word is a (c) Adverb (d) Cause
given options, ‘relaxed’ is its antonym. Relative Pronoun.
_ (a) The given highlighted word is a
77. Wooing everyone over an issue 83. The most beautiful actor of the Conjunction.
for support will not serve much industry was awarded today. Directions (Q. Nos. 91-100) In this
purpose. (a) Adjective (b) Numeral section a word is spelt in four
(a) discouraging (b) encouraging (c) Adverb (d) Noun different ways. Identify the one which
(c) pursuing (d) persuading
_ (a) The given highlighted word is an is correct. Choose the correct response
_ (a) The word wooing means ‘seek Adjective. (a), (b), (c) or (d) and indicate on the
the favour’ or ‘support’. From the Answer Sheet accordingly.
given options, ‘discouraging’ its 84. “What is the latest news?” asked
antonym. the Captain. 91. (a) Continuum (b) Continuem
(a) Relative pronoun (c) Contuneim (d) Continueiam
78. The highest award was bestowed
(b) Adjective
upon her for her yeoman service. _ (a) The correctly spelt word is
(c) Adverb ‘Continuum’ which means ‘a
(a) conferred (b) withdrawn (d) Adjectival clause continuing sequence’.
(c) imparted (d) imbibed
_ (b) The given highlighted word is an 92. (a) Stretegy (b) Stretagy
_ (b) The word bestowed means ‘to Adjective.
give’. From the given options, (c) Stratagy (d) Strategy
‘withdrawn’ is its antonym. 85. Noticing the change in the
_ (d) The correctly spelt word is
behaviour of the officer, the cadets ‘Strategy’ which means ‘a plan’.
79. One feels elated when someone
returned to their position.
praises one’s work. 93. (a) Commisionor (b) Commisioner
(a) Participle (c) Commissioner (d) Comissioner
(a) feels good (b) excited (b) Present continuous
(c) depressed (d) sober (c) Noun phrase _ (c) The correctly spelt word is
‘Commissioner’ which refers to ‘a
_ (c) The word elated means ‘happy (d) Noun
person appointed by a commission’.
and joyful’. From the given options,
‘depressed’ meaning ‘sorrowful and _ (a) The given highlighted word is a
Participle. 94. (a) Vacum (b) Vacuum
sad’ is its antonym. (c) Vacuem (d) Vacam
80. All business activities need not 86. When he reached the department,
the officials had left for the meeting. _ (b) The correctly spelt word is
result in profit-making. There is a ‘Vacuum’.
need to be charitable. (a) Past perfect verb
(b) Past tense 95. (a) Psephology (b) Psefoloagy
(a) lenient (b) malevolent (c) Dependent clause (c) Sephology (d) Psyphology
(c) unforeseen (d) gracious (d) Independent clause _ (a) The correctly spelt word is
_ (b) The word charitable means ‘to _ (a) The given highlighted word is a
‘Psephology’ which means ‘the
assist or help someone in need’. Past Perfect verb. statistical study of elections and
From the given options, ‘malevolent’ trends in voting’.
meaning ‘a wish to do something bad 87. He has offered her another chance.
96. (a) Neuphrology (b) Nephrology
or evil’ is its antonym. (a) Intransitive verb (b) Past tense (c) Neprology (d) Neaprology
(c) Perfect tense (d) Transitive verb
Directions (Q. Nos. 81-90) Each of _ (b) The correctly spelt word is
the following sentences has a word or _ (c) The given highlighted word is a ‘Nephrology’ which means ‘the study
phrase underlined. Read the sentences verb in perfect tense. of a specialty of medicine and
carefully and find which part of pediatric medicine that concerns with
88. The building is very ancient. study of the kidneys’.
speech the underlined word belongs (a) Transitive verb
to. Indicate your response on the (b) Intransitive verb 97. (a) Psudonym (b) Pseudonym
Answer Sheet accordingly. (c) Phrasal verb (c) Pseudanym (d) Seeudonym
(d) Auxiliary verb _ (b) The correctly spelt word is
81. He has been working in the
‘Pseudonym’ which means ‘another
Department of Foreign Affairs _ (b) The given highlighted word is an name’.
Intransitive verb.
since 2002.
89. Hurrah! What a scintillating 98. (a) Pnumonia (b) Neumonia
(a) Adverb (b) Adjective
(c) Pneumonia (d) Numania
(c) Intensifier (d) Noun beauty the landscape is !
(a) Conjunction (b) Adjective _ (c) The correctly spelt word is
_ (a) The given highlighted word is an ‘Pneumonia’ which refers to ‘a
Adverb. (c) Adverb (d) Interjection
disease of lungs’.
CDS Solved Paper 2020 (I) 61

99. (a) Resilient (b) Resilint (b) Parents had been waiting to meet 107. Which is the correct combination
(c) Risilient (d) Realisent the counsellor since the morning. of the given two sentences?
(c) Parents are waiting to meet the
_ (a) The correctly spelt word is counsellor in the morning. Priya reached the station. The bus
‘Resilient’ which means ‘strong and
tough’. (d) Parents have been waiting since left before her.
morning to meet the counsellor. (a) When Priya reached the station,
100. (a) Suplementary the bus had already left.
(b) Supplementary
_ (b) The grammatically correct
combination of the sentences is given (b) When Priya had reached the
(c) Supplementery in option (b). station, the bus already left.
(d) Supplemantory (c) Priya reached the station, when
104. Which is the correct combination the bus already left.
_ (b) The correctly spelt word is of the given two simple sentences
‘Supplementary’ which means (d) When Priya had reached the
‘completing or enhancing something’. using ‘If’ clause? Minchi should station, the bus had already left.
have worked hard. She would have _ (a) The grammatically correct
Directions (Q. Nos. 101-110) In this
section two sentences are given and cleared the test. combination of the sentences is given
(a) If Minchi had worked hard, she in option (a).
you are required to find the correct
would have cleared the test. 108. Which is the correct combination
sentence which combines both the
(b) Had not Minchi worked hard, she
sentences. Choose the correct response could not have cleared the test.
of the given two sentences?
(a), (b), (c) or (d) and indicate on the (c) If Minchi has worked hard, she He is too tired. He could not stand.
Answer Sheet accordingly. would have cleared the test. (a) He is so tired that he could
101. Which is the correct combination (d) If Minchi had worked hard, she scarcely stand.
will have cleared the test. (b) He is too tired and cannot stand.
of the given two sentences? The
_ (a) The grammatically correct (c) He will not stand and he is very
officer will return from China on tired.
combination of the sentences is given
Monday. You can meet him. in option (a). (d) He is so tired that he could not be
(a) You can meet the officer when he standing.
returned from China on Monday. 105. Which one of the following is the
(b) You can meet the officer when correct statement combining the _ (a) The grammatically correct
combination of the sentences is given
he will return from China on two statements using ‘though’? in option (a).
Monday. He has been trying his level best
(c) You can meet the officer when he to win. He could not succeed. 109. Which is the correct combination
returns from China on Monday. of the given two sentences?
(a) Though he is trying his level best
(d) The officer will meet you when The teacher entered the classroom.
to win, he could not succeed.
you return from China on
Monday. (b) He is trying his level best to win, All students stopped talking.
though he could not succeed. (a) No sooner did the teacher enter
_ (c) The grammatically correct (c) Though he has been trying his the classroom than the students
combination of the sentences is given level best to win, he could not stopped talking.
in option (c). succeed. (b) As soon as the teacher entered
102. Which is the correct combination (d) Though he had been trying his the classroom all students were
level best to win, he could not asked to stop talking.
of the given two sentences? He is succeed. (c) All students stopped talking as
hard-working. He is honest too. the teacher enters the classroom.
(a) He is not only hard-working, but
_ (d) The grammatically correct
combination of the sentences is given (d) No sooner did the students stop
also honest. in option (d). talking than the teacher entered
(b) He is only hard-working and the classroom.
honest. 106. Which is the correct combination
(c) He is hard-working but honest of the given two sentences using _ (a) The grammatically correct
combination of the sentences is given
too. ‘relative clause’? in option (a).
(d) He is not hard-working but also
honest.
Gandhiji preached peace. He is an 110. Which one of the following is the
apostle of peace. correct statement of the
_ (a) The grammatically correct (a) Gandhiji who preached peace is
combination of the sentences is given combination of the two sentences
in option (a). an apostle of peace.
given below using ‘whereas’?
(b) Gandhiji preached peace
103. Which is the correct combination because he is an apostle of Kavya is interested in reading
of the given two sentences? peace. books. Her sister shows interest in
Parents have been waiting since (c) Gandhiji who preached peace is outdoor games.
morning. They want to meet the called an apostle of peace. (a) Kavya is interested in reading
(d) Gandhiji is an apostle of peace books whereas her sister’s
counsellor. because he preached peace. interest is outdoor games.
(a) The counsellor has been waiting
_ (c) The grammatically correct (b) Kavya is interested in reading
to meet the parents since
combination of the sentences is given books whereas her sister is not
morning.
in option (c). interested in it.
62 CDS Solved Paper 2020 (I)

(c) Kavya is interested in reading (b) Navanitha exclaimed to her (d) The leader advised her followers
whereas her sister’s interest is friends what a scintillating beauty that those who sow the seeds the
outdoor games. it was. previous season would reap the
(d) Kavya is interested in reading (c) Navanitha asked her friends harvest this season.
books whereas her sister’s whether it was a scintillating
interest is to play outside. beauty.
_ (c)
(d) Navanitha exclaimed to her 118. He said to his manager,
_ (a) The grammatically correct friends that it was a scintillating
combination of the sentences is given “Could you please pass the bill this
in option (a). beauty. week?”
Directions (Q. Nos. 110-120) In this _ (b) (a) He told his manager that bill to be
passed.
section direct speech sentences are 115. The Captain said to the soldiers,
(b) He requested his manager to
given and you are required to find the “March forward and aim at the pass the bill that week.
correct indirect speech sentence of the peak of the hill today.” (c) He ordered his manager to pass
same. Choose the correct response (a), (a) The Captain requested the the bill that week.
(b), (c) or (d) and indicate on the soldiers to march forward and (d) He requested his manager to
Answer Sheet accordingly. aim at the peak of the hill that pass the bill this week.
day.
111. Rahul said to his teacher, “Madam, (b) The Captain ordered the soldiers _ (b)
what is the way to solve the to march forward and aim at the 119. The village chief said to the
question”? peak of the hill today. villagers, “All of us need to adopt
(a) Rahul asked his teacher what the (c) The Captain ordered the soldiers new regulations. We will protect
way to solve the question was. to march forward and aim at the
peak of the hill that day. our Earth forever.”
(b) Rahul told his teacher what was
the way to solve the question. (d) The Captain told the soldiers that (a) The village chief ordered the
they should march forward and villagers that all of them needed
(c) Rahul asked to his teacher what
aim at the peak of the hill that day. to adopt new regulations and
the way was to solve the
they would protect their Earth
question. _ (c) forever.
(d) Rahul told his teacher what the
way was to solve the question. 116. “Where were you last evening?” (b) The village chief told the villagers
said the lady to her maid. that all of them need to adopt
_ (c) new regulations and they will
(a) The lady asked her maid where protect their Earth forever.
112. He said to his friend, “Could you she had been the previous
evening. (c) The village chief wanted the
please close the door?” villagers needed to adopt new
(a) He requested his friend to close (b) The lady asked her maid where
regulations and they would
the door. she had been in the last evening.
protect their Earth forever.
(b) He requested his friend to please (c) The lady asked her maid where
had she been the evening before. (d) The village chief told the villagers
close the door. that all of them needed to adopt
(c) He ordered his friend to close the (d) The lady told her maid where she
new regulations and they would
door. had been to the last evening.
protect their Earth forever.
(d) He wanted his friend to close the _ (a)
door for him. _ (d)
117. “Those who sowed the seeds last
_ (a) 120. The grandfather said to the baby,
season will reap the harvest this “May you live long with all good
113. Raj said to Sheela, “The Sun rises season,” said the leader to her things of life”.
in the East.” followers.
(a) The grandfather blessed to the
(a) Raj told Sheela that the Sun rose (a) The leader said to her followers baby with long life and all good
in the East. that those who sowed the seeds things of life.
(b) Raj told Sheela that the Sun rises the previous season would reap
in the East. the harvest that season. (b) The grandfather asked the baby
(c) Raj asked Sheela that the Sun that she would live long with all
(b) The leader addressed her
rises in the East. good things of life.
followers that those who have
(d) Raj said to Sheela that the Sun sown the seeds the previous (c) The grandfather wanted the baby
has arisen in the East. season would reap the harvest to live long with all good things of
this season. life.
_ (b)
(c) The leader addressed her (d) The grandfather blessed the
114. Navanitha said to her friends, followers that those who had baby that she would live long with
“What a scintillating beauty it is !” sown the seeds the previous all good things of life.
(a) Navanitha told to her friends that season would reap the harvest
that season. _ (a)
it was a scintillating beauty.
PAPER III General Studies
1. Scattering of α-particles by a thin 4. A solution having pH equal to 7. The property of the sound waves that
gold foil suggests the presence of zero is known as determines the pitch of the sound
(a) electron in an atom (a) Highly alkaline solution is its
(b) proton in an atom (b) Highly acidic solution (a) frequency (b) amplitude
(c) positively charged nucleus at the (c) Weakly acidic solution (c) wavelength (d) intensity
centre of an atom (d) Neutral solution
_ (a) The property of the sound waves
(d) isotopes of gold that determines the pitch of the sound
_ (b) pH is a number which indicates
the acidic or basic nature of a solution. is its frequency. The pitch of a sound
_ (c) Rutherford proposed the model depends on the frequency of
which was based upon the α-particle The solutions having pH between 0 to
scattering experiment. In this 2 are strongly acidic, those with pH vibration. Greater the frequency of a
experiment, Rutherford concluded between 2 to 4 are moderately acidic, sound, the higher will be its pitch.
that the atom consists of a heavy and while others having pH between 4 to 7
8. Which one of the following is not
positively charged part at its centre, are weakly acidic. When the solutions
called the nucleus. The entire mass of having pH equal to 7 are neutral a property of the X-rays?
an atom resides in its nucleus and is whereas others which have pH (a) They are deflected by electric fields.
equal to the sum of masses of between 12 to 14 are strongly alkaline. (b) They are not deflected by magnetic
protons and neutrons, since the mass 5. Which one of the following acids fields.
of electron is negligible.
is produced in human stomach? (c) They have high penetration length
2. The elements of which of the (a) Formic acid (b) Sulphuric acid in matter.
following pairs are isobars? (c) Nitric acid (d) Hydrochloric acid (d) Their wavelength is much smaller
(a) 11H and 13 H (b) 11H and 12 H than that of visible light.
_ (d) Stomach acid or gastric juice, is a
(c) 126 C and 146 C 40
(d) 18 Ar and 40
20 Ca
digestive acid i.e., formed in the _ (a) X-rays are not deflected by
stomach and is composed of electric fields because X-rays do not
_ (d) Isobars are the atoms of different hydrochloric acid (HCl), potassium carry any charge. These waves are
elements that have same mass chloride (KCl) and sodium chloride electromagnetic radiations. X-rays
number but different atomic number. (NaCl). Hydrochloric acid (HCl) plays were discovered by Roentgen in
40
Therefore, the elements of 18 Ar and a key role in digestion of proteins and 1895. Their wavelength is of the order
40
20 Ca are isobars. enzymes. of 10 −12m to 10 −8m. They are mainly
3. Which one of the following used in detecting the fracture of
6. Match List-I with List-II and select bones, hidden bullet, needle, costly
chemical reaction is not feasible? the correct answer using the codes material etc, inside the body and also
(a) Fe + CuSO4 → FeSO4 + Cu given below the Lists used in the study of crystal structure.
(b) Zn + CuSO4 → ZnSO4 + Cu Properties of X-rays are :
List I List II
(c) Cu + PbCl 2 → CuCl 2 + Pb l They are not deflected by electric
(d) Mg + CuSO4 → MgSO4 + Cu (Compound) (Use)
and magnetic fields.
(A) Boric acid 1. Antiseptic
_ (c) l Fe + CuSO 4 → FeSO 4 + Cu l They have high penetration length

In this reaction, iron being more (B) Citric acid 2. Food in matter.
preservative
reactive than Cu, displaces Cu from l Their wavelength is much smaller
CuSO 4 solution and forms new (C) Magnesium 3. Antacid than that of visible light.
product, iron sulphate and Cu hydroxide
metal. So, this reaction is feasible.
9. Which one of the following is not
(D) Acetic acid 4. Pickle
l Zn + CuSO 4 → ZnSO 4 + Cu true about the image formed by a
Codes plane mirror?
In this reaction, zinc being more
A B C D
reactive than Cu, displaces Cu (a) It is of the same size as the subject.
(a) 1 2 3 4
from. CuSO 4 solution and forms (b) It is laterally inverted.
zinc sulphate and Cu metal. So,
(b) 1 3 2 4
(c) 4 3 2 1 (c) It is real image.
this reaction is also feasible.
(d) 4 2 3 1 (d) It is formed as far behind the mirror
l Cu + PbCl 2 → Cu Cl 2 + Pb
_ (a) Boric acid is used as an antiseptic as the object is in front.
l This reaction is not feasible
and insecticide.
because lead chloride (PbCl 2 ) is _ (c) Image formed by a plane mirror
less soluble. Thus, in this reaction, l Citric acid is used as a preservative has following properties :
Pb not displaced by PbCl 2. and flavouring agent in food and l It is always virtual and erect.
l Mg + CuSO 4 → MgSO 4 + Cu beverages such as soft drinks.
l The size of image is equal to the
l Magnesium hydroxide reduces
In this reaction, magnesium is more size of the object.
reactive than Cu. stomach acid, hence it is used as
l The image formed is as far behind the
an antacid.
It displaces Cu from CuSO 4 mirror as the object is in front of it.
l Acetic acid is used in pickle as Vinegar.
solution and form magnesium l The image is laterally inverted (i.e.,
sulphate and Cu metal. Hence, this Thus, the correct code is given in
left seems to be right and
reaction is feasible. option (a).
vice-versa)
64 CDS Solved Paper 2020 (I)

10. In a periscope, the two plane Electromagnetic Spectrum international borders, e.g. smallpox,
mirrors are kept Name Frequency Wavelength which throughout history, has killed
range (Hz) range (m) between 300-500 million people in all
(a) parallel to each other over the world.
(b) perpendicular to each other Gamma ( γ ) rays 5 × 1022 0.6 × 10 −14
to 5 × 1018 15. Which one of the following animals
(c) at an angle of 60° with each other
to10 −10 has a three-chambered heart?
(d) at an angle of 45° with each
other X-rays 3 × 10 21 10 −13 (a) Scoliodon (b) Salamander
to 1 × 1016 to 3 × 10 −8 (c) Pigeon (d) Human being
_ (a) In a periscope, the two plane
mirrors are kept parallel to each other Ultraviolet rays 8 × 1014 4 × 10 −9 _ (b) Amphibians and reptiles, such as
at an 45° angle with its surface, which (UV) to 8 × 1016 to 4 × 10 −7 salamanders, frogs, toads, lizards
helps to reflect the light rays from the and snakes, have three-chambered
−7
upper end of the periscope to lower Visible light 4 × 10 14
4 × 10 heart, one ventricle and two atria. In
end of the periscope. to 8 × 1014 to 8 × 10 −7 three-chambered heart, blood from the
Periscope is especially used in a ventricle travels to the lungs and skin
Thermal or 3 × 1011 8 × 10 −9 where it is oxygenated and also to the
submarine to see above the surface
of the sea.
Infrared rays (IR) to 4 × 1014 to 3 × 10 −3 body. In the ventricle, deoxygenated
and oxygenated blood are mixed
11. If the speed of light in air is Microwaves 3 × 10 8 10 −3 to 1 before being pumped out of the heart.
3 × 10 8 m/s, then the speed of light to 3 × 1011 Scoliodon has two-chambered heart,
3 Radiowaves 3 × 10 3 10 −3 to 10 5 while pigeon and human beings have
in a medium of refractive index to 3 × 1011
four-chambered heart.
2
16. Which one of the following is the
is 13. The unit of the force constant k of
9
correct sequence of events during
(a) 2 × 108 m/s (b) × 108 m/s a spring is sexual reproduction in plants?
4
(a) N-m (b) N/m (c) N-m2 (d) N/m 2 (a) Seedling, formation of embryo,
3
(c) × 108 m/s (d) 3 × 108 m/s pollination, fertilisation, division of
2 _ (b) The unit of force constant k of a
spring is N/m. According to Hooke’s zygote
_ (a) Given, speed of light in air law, force applied by spring, (b) Formation of embryo, seedling,
= 3 × 10 8 m/s F = − kx or, k = −
F
pollination, fertilisation, division of
3 x zygote
Refractive index, n = where, k is a constant that is a
2 (c) Pollination, fertilisation, division of
characteristic of the spring known as
Refractive index of a medium, zygote, formation of embryo,
spring constant or force constant
seedling
n=
c and x is the final elongation of the
v spring. (d) Seedling, formation of embryo,
division of zygote, pollination,
where, 14. Which one of the following is not fertilisation
c = speed of light in air and an epidemic disease?
_ (c) In flowering plants during sexual
v = speed of light in medium. (a) Cholera (b) Malaria reproduction following events occur
Using the formula, (c) Smallpox (d) Elephantiasis l Formation of Flower which is the

c sexual organ of plant.


n= _ (c) Smallpox is not an epidemic l Gametogenesis in which pollen
v disease. An epidemic is the rapid
grains are formed in anther, while
c 3 × 10 8 spread of an infectious disease to a
⇒ v = = large number of people in a given ovules are formed in ovary.
n 3/2 l Pollination in which pollen grains
population within a short period of
∴ v = 2 × 10 8 m/s time, usually two weeks or less. transferred to style of
Malaria and elephantiasis both are gynoecium/Pistil through various
Therefore, speed of light in a medium
3 vector (Mosquito) borne diseases, means like air, water, insects,
of refractive index is 2 × 10 8 m/s. thus during favourable conditions like etc.
2
flood or heavy rain, the sudden l Fertilisation Where a haploid male
12. Which one of the following types increase in numbers of mosquito will gamete fuses with another haploid
of radiation has the shortest also cause outbreak of these egg cell of embryo sac of ovule.
wavelength ? diseases. l Embryogenesis After fertilisation a

Similarly, cholera is also an epidemic diploid zygote is formed, which


(a) Radio waves develops into a multicellular
disease as it is water borne disease,
(b) Visible light thus presence of bacteria Vibrio embryo through repetitive cell
(c) Infrared (IR) cholerae in local water source like divisions.
(d) Ultraviolet (UV) pond or well will cause outbreak of l Formation of Seed and Seedling

cholera in every member of locality in After maturation the embryo


_ (d) Decreasing order of wavelength a short period of time. develops into seed. The dormant
of Electromagnetic waves are : seed after germination is known as
A pandemic is an epidemic of
Radio waves > Infrared (IR)> Visible seedling, which give rise to a new
world-wide proportions i.e. outbreaks young plant.
light> Ultraviolet (UV)
of these diseases occur across
CDS Solved Paper 2020 (I) 65

17. When air is blown from mouth starch gets hydrolysed in the oral _ (c) Copper sulphate (CuSO 4 ⋅ 5H 2O)
into a test-tube containing cavity by the action of salivary is the inorganic compound in which
amylase (at optimum pH 6.8) into a four water molecules are directly
limewater, the limewater turns disaccharide, i.e. maltose. attached to Cu atom while one is
milky. This is due to the presence Lysozyme acts as an antibacterial present as water of crystallisation.
of agent. It kills bacteria thus prevents
23. Silver articles become black after
(a) water vapour infections.
sometime when exposed to air
(b) oxygen 20. Mahatma Gandhi’s Dandi March, a because
(c) carbon dioxide great event in Indian freedom
(d) carbon monoxide (a) silver gets oxidised to silver oxide
struggle, was associated with (b) silver reacts with moist carbon
_ (c) The lime water [Ca(OH)2 ] is turned (a) iron dioxide in the air to form silver
into milky due to formation of calcium (b) sodium chloride carbonate
carbonate when the lime water reacts (c) sulphur
with carbon dioxide. The products (c) silver reacts with sulphur in the air
formed in this reaction is calcium (d) aluminium to form a coating of silver sulphide
carbonate (CaCO 3 ) and water. The _ (b) Mahatma Gandhi’s Dandi March, (d) silver reacts with nitrogen oxides
reaction is given below a great event in Indian freedom in the air to form silver nitrate
Ca(OH)2 + CO 2 → struggle was associated with sodium
Calcium hydroxide Carbon chloride. _ (c) When the silver articles react with
(Lime water) dioxide the hydrogen sulphide gas that is
CaCO 3 + H 2 O 21. Match List I with List II and select present in the air, a black coating of
Calcium Water the correct answer using the code silver sulphide (Ag 2S) is formed. Silver
Carbonate
sulphide are insoluble in all the
given below the Lists. solvents.
18. Which one of the following is the
‘energy currency’ for cellular List I (Name) List II (Formula) 2Ag( s ) + H 2S( g ) →
Silver Hydrogen
processes? (A) Bleaching 1. NaHCO 3 sulphide
Ag 2S ( s ) + H 2( g )
(a) Glucose (b) ATP powder Silver Hydrogen
sulphide gas
(c) ADP (d) Pyruvic acid (B) Baking soda 2. Na 2CO 3 ⋅ 10H 2O
24. If x the temperature of a system in
_ (b) During the process of oxidation of (C) Washing soda 3. Ca(OH)2
kelvin and y is the temperature of
food within a cell, all the energy
contained in the respiratory substrates (D) Slaked lime 4. CaOCl 2 the system in °C, then the correct
is not released free into the cell, or in Code relation between them is
a single step. Instead, it is released in
A B C D (a) x = 273 − y (b) x = 273 + y
a series of stepwise reactions
controlled by enzymes and is
(a) 4 1 2 3 (c) x = 173 + y (d) x = 173 − y
(b) 4 2 1 3
trapped as chemical energy in the
(c) 3 2 1 4 _ (b) Relation among the temperatures
form of Adenosine Triphosphate measured by Celsius scale and Kelvin
(ATP). (d) 3 1 2 4
scale.
Thus, it is said that ATP acts as the _ (a) The chemical formula of K = 273 + C
energy currency of the cell. The bleaching powder is Ca(OCl)Cl. It is
used for water treatment and as a where, K and C are the temperature
energy trapped in ATP is used in
bleaching agent. of a system in kelvin and celsius (°C)
many energy requiring biological
l Sodium bicarbonate (NaHCO 3 ) is respectively.
processes of the organisms such as
locomotion, digestion, thinking, etc. commonly known as baking soda. According to question, K = x and
During aerobic respiration, most It is widely used in baking. C = y
l Sodium Carbonate
of the ATPs are formed in Hence, x = 273 + y
mitochondria. (Na 2CO 3 ⋅ 10H 2O) is the inorganic
compound which is used as 25. The resistivity ρ of a material may
19. Which one of the following is the washing soda. be expressed in units of
first enzyme to mix with food in l Calcium hydroxide (Ca(OH)2 ) is a
(a) ohm (b) ohm/cm
the digestive tract? colorless crystal i.e., known as
slaked lime. Slaked lime is used as (c) ohm-cm (d) ohm-cm 2
(a) Trypsin (b) Cellulose
(c) Pepsin (d) Amylase
a pH–regulating agent and acid
_ (c) The resistivity ρ of a material may
neutraliser in soil and water. be expressed in units of ohm-cm.
_ (d) Amylase is the first enzyme which Hence, the correct codes are shown Resistivity of a conductor is defined
mix with food in the digestive tract. in option (a). as the resistance of a conductor of
The mastication of food in mouth is
22. The number of water molecules unit length and unit area of
the first event of digestion in
associated with copper sulphate cross-section.
alimentary canal or digestive tract.
RA
During this process, digestion starts molecule to form crystals is Resistivity, ρ =
in the oral cavity by hydrolytic action l
of enzyme salivary amylase. (a) 2
where, l is the length of the
(b) 4
The saliva secreted into oral cavity conductor, A is the area of
(c) 5
contains two enzymes, i.e. amylase cross-section of the conductor and R
and lysozyme. About 30% of the (d) 6 is the resistance of the conductor.
66 CDS Solved Paper 2020 (I)

26. The electromagnetic waves, which 29. Which one of the following cell session was historic because for the
first time in the history of freedom
are used for satellite organelles contains DNA?
struggle, the national leaders
communication, are (a) Golgi apparatus demanded complete independence.
(a) infrared radiations (b) Mitochondrion Later, 26th, January was selected as
(b) ultraviolet radiations (c) Lysosome the Republic Day of Independent
India.
(c) radio waves (d) Endoplasmic reticulum
(d) visible lights 32. Which one of the following acts
_ (b) Mitochondrion is a semi-
autonomous cell organelle. reserved seats for women in
_ (c) The electromagnetic waves,
which are used for satellite Mitochondria were first observed by Legislatures in accordance with
communication, are radio waves. Kollikar in 1850. They contain DNA as the allocation of seats for different
well as ribosomes and are able to
In the space wave propagation, the synthesise proteins. According to
communities?
radiowaves emitted from the scientific observation, the new (a) The Government of India Act,
transmitter antenna reaches the mitochondira are originated by the 1858
receiving antenna through space. growth and division of pre-existing
These radiowaves are called space (b) The Indian Councils Act, 1909
mitochondria, which can’t occur
waves. The space waves are the without the synthesis of proteins.
(c) The Government of India Act,
radiowaves of frequency range from 1919
54 MHz to 4.2 GHz. 30. Who among the following (d) The Government of India Act,
The space wave propagation is used scientists introduced the concept 1935
for television broadcast microwave of immunisation to the medical
link and satellite communication. world? _ (d) The Government of India Act,
1935 reserved seats for women in
27. Which one of the following is the (a) Edward Jenner Legislatures in accordance with the
correct sequence of organs that (b) Robert Koch allocation of seats for different
(c) Robert Hooke communities.
occur in the path of urine flow in
(d) Carl Linnaeus According to this Act, the council of
human body? states (upper house) had 156
(a) Kidney, ureter, urinary bladder, _ (a) Edward Jenner (17th May, 1749- members for British India and upto
26th January, 1823) was an English 104 members for the princely states.
urethra
physician who invented the smallpox All members from British India were to
(b) Kidney, urinary bladder, ureter, vaccine. Jenner is often called a
be directly elected, except for six
urethra pioneer of immunisation or father of
members who were to be nominated
(c) Kidney, ureter, urethra, urinary vaccination as he invented first
by the Governor-General, to secure
bladder vaccine and also proposed the
due representation for the scheduled
concept of immunisation.
(d) Urinary bladder, kidney, urethra, classes, women and minority
ureter Robert Koch investigated the communities.
anthrax disease cycle in 1876 and
_ (a) Urine is formed in the kidneys studied the bacteria that cause 33. Which one among the following
through the filtration of blood. The tuberculosis in 1882 and cholera in was demanded by the All India
urine is then passed through the two 1883. He also formulated Koch's
separate tubes connected to each
Depressed Classes Leaders’
postulates.
kidney, known as ureters which open Conference at Bombay in 1931?
to the urinary bladder, where urine is Robert Hooke looked at a sliver of
cork through a microscope lens and (a) Universal adult suffrage
stored. During urination, the urine is (b) Separate electorates for
passed from the urinary bladder first time discovered cells in 1635.
through the urethra to the outside of Carl Linnaeus is known as the untouchables
the body. ‘Father of Taxonomy’. He proposed (c) Reserved seats for the minorities
binominal nomenclature for uniform (d) A unitary state in India
28. Which of the following endocrine and unique identification of
glands is not found in pair in humans? organisms. _ (b) ‘All India Depressed Classes
Leaders’ conference at Bombay in
(a) Adrenal (b) Pituitary 31. On 31st December, 1929 in which 1931 demanded for separate
(c) Testis (d) Ovary one of the following Congress electorates for untouchables. This
demand arose because their leaders
_ (b) Pituitary is a single gland, which is Sessions was proclamation of were dissatisfied with the provisions of
not found in pair like adrenal gland, testie Purna Swaraj made? Joint electorates provided in the
and overies. It is the smallest endocrine
(a) Ahmedabad (b) Calcutta Simon Commission report. This report
gland but serves very important role in
(c) Lahore (d) Lucknow empowered Governor to authorise a
the human endocrine system. It
non–depressed class member to
directly or indirectly controls almost all
other endocrime glands of the body. It _ (c) On 31st, December, 1929, in the contest on behalf of depressed
Lahore Session of the Congress, classes.
is also known as master gland.
proclamation of Purna Swaraj made. Finally, this demand led to the
It is reddish grey in colour and is During this session, Pt. Jawaharlal issuance of MacDonald Award or
roughly oval in shape and about a size Nehru was the President of the Communal Award (1932). This award
of a pea seed. It is located in a small Congress. The resolution of Purna
provided for separate electorate for
bony cavity of the brain called sella Swaraj proclaimed 26th January,
Scheduled Castes.
tursica. 1930 as the Independence Day. This
CDS Solved Paper 2020 (I) 67

34. Who among the following was Mistral is a cold and dry strong wind 41. Which one of the following rivers
one of the founders of the Indian in Southern France that blows down joins Ganga directly?
from the North along the lower Rhone
Society of Oriental Art? river valley towards Mediterranean sea. (a) Chambal (b) Son
(a) Rabindranath Tagore Bise is a cold, vigourous and (c) Betwa (d) Ken
(b) Abanindranath Tagore persistent North or North-Easterly _ (b) Among the following rivers, Son
(c) Dwarkanath Tagore wind blowing from the alpine river directly joins river Ganga.
(d) Bankim Chandra Chattopadhyaya mountains, that affects Switzerland
Son river originates near Amarkantak
and Eastern France.
in Anuppur district of Madhya Pradesh
_ (b) Abanindranath Tagore alongwith Blizzard is a cold, violent, powdery
his brother Gaganendranath Tagore and joins Ganga river near Patna (Bihar).
founded of Indian Society of Oriental polar wind. They are prevalent in the
Chambal, Betwa and Ken rivers join
Art (ISOA) in 1907. The primary North and South polar regions like
Yamuna river in Uttar Pradesh.
objective of ISOA was to promote the Canada, the USA, Siberia, etc.
Yamuna then joins Ganga at
rich artistic heritage of India. This 38. Nyishi tribe is found mainly in Prayagraj (Uttar Pradesh).
initiative went a long way in
demolishing the myth of the cultural (a) Andaman and Nicobar 42. Which one of the following is not
supremacy of the western world. (b) Arunachal Pradesh a type of commercial agriculture?
35. Who among the following Sultans (c) Nilgiri-Kerala (a) Dairy farming (b) Grain farming
(d) Kashmir Valley (c) Livestock ranching
succeeded in finally breaking and
destroying the power of _ (b) Nyishi tribe is found mainly in (d) Intensive subsistence agriculture
Turkan-i-Chihalgani? Arunachal Pradesh. Nyishi tribe is the
largest ethnic community in Arunachal _ (b) Among the following, intensive
(a) Iltutmish subsistence agriculture is not a type
Pradesh. Nyishi tribe people are
of commercial agriculture.
(b) Balban primarily agriculturalists who practice
(c) Alauddin Khalji Jhum (Shifting) cultivation. Commercial agriculture is a cropping
The attire of Nyishis uses the creast method in which crops and livestock
(d) Muhammad bin Tughluq
of a hornbill beak. As a result, they are raised in order to sell the
_ (b) After Balban ascended the throne, have adversely affected the products in the market for
he realised that the real threat to the population of hornbill bird. Boori Boot remuneration. It is capital intensive
monarchy was from Chahalgani. agriculture in which crops are grown
Yollo is an important festival.
Thus, he broke its power. on large farms, using modern
Turkan-i-Chihalgani was a class of 39. In the field of tourism, which one technologies like farm machinery,
ruling elite of fourty powerful military of the following Indian States is fertilizers, insecticides, high yeilding
leaders. This class was created by described as ‘One State Many variety of seeds, etc. Dairy farming,
Slave dynasty ruler Iltutmish. grain farming, livestock ranching,
Worlds’? plantation agriculture, etc. are few
36. Who among the following Mongol (a) Assam (b) West Bengal examples of this type of agriculture.
leaders/commanders did not cross (c) Karnataka (d) Rajasthan Intensive subsistence agriculture is
Indus to attack India? practiced in areas where farmers
_ (c) In the field of tourism, Karnataka have small fields. The crops are
(a) Chenghiz Khan is described as ‘One State Many
Worlds’. This is because of its diverse grown to be consumed by farmers
(b) Tair Bahadur
culture and topography. themselves.
(c) Abdullah
(d) Qutlugh Khwaja Assam is known as ‘The Awesome 43. Which one of the following is not
Assam’. West Bengal is known as a land use category?
_ (a) Among the following ‘Beautiful Bengal’. Rajasthan is
leaders/commanders, Chenghiz Khan known as ‘the Incredible State of India.’ (a) Forest land
was the Mongol ruler who did not (b) Pasture land
cross the Indus to attack India. 40. The number of people per unit (c) Marginal land
Tair Bahadur invaded Punjab in area of arable land is termed as (d) Barren and wasteland
1241. Abdullah invaded Punjab with (a) agricultural density
his force in 1292. _ (d) Among the following Barren and
(b) arithmetic density wasteland is not a land use category.
Qutlugh Khwaja attacked Delhi in (c) physiological density Barren and wastelands include
1299. This led to Battle of Kili. mountains and hill slopes, deserts
(d) economic density
37. In the region of Eastern shore of and rocky areas. These areas cannot
_ (c) The number of people per unit be brought under plough except at a
Adriatic sea, a cold and dry wind area of arable land is termed as high input cost with possible low returns.
blowing down from the mountain physiological density. A higher
is known as physiological density suggests that 44. Which one of the following is not
the available agricultural land is being an objective of the MGNREGA?
(a) Mistral (b) Bora used by more people.
(c) Bise (d) Blizzard (a) Providing up to 100 days of skilled
Agricultural density is defined as
labour in a financial year
the number of farmers per unit area
_ (b) Bora is a cold and dry wind (b) Creation of productive assets
blowing down the mountain in the of farmland. Arithmetic density is
region of Eastern shore of Adriatic same as population density (Number (c) Enhancing livelihood security
sea. of people per unit area of land). (d) Ensuring empowerment to women
68 CDS Solved Paper 2020 (I)

_ (b) Creation of productive assets is earliest. The correct order of the 49. The ‘Basel Convention’ is aimed at
not an objective of the MGNREGA. It launch dates of the schemes is given
protecting human health and
is an accronym of Mahatma Gandhi below
National Rural Employment l Pradhan Mantri Gram Sadak Yojana
environment against adverse
Guarantee Act. It was enacted in the (December, 2000) effects of which of the following?
year 2005. l Deendayal Antyodaya Yojana (June, (a) Hazardous wastes
The objectives of MGNREGA include 2011) (b) Persistent Organic Pollutants
l Ensuring social protection for the l Deendayal Upadhyaya Grameen (c) Mercury
most vulnerable in rural India. Kaushalya Yojana (September, (d) Chemicals and pesticides
l Ensuring livelihood security for
2014).
poor through creation of durable l Saansad Adarsh Gram Yojana _ (a) The ‘Basel Convention’ is aimed
assets. (October, 2014) at protecting human health and
Pradhan Mantri Gram Sadak Yojana environment against adverse effects
l Strengthening drought proofing
was launched with an objective to ‘hazardous waste’. The convention is
and flood management in rural
provide single all–weather road a multilateral agreement on the
India.
connectivity to eligible unconnected control of transboundry movement of
l Aiding in empowerment of the
habitation of designated population hazardous waste and their disposal. It
marginalised communities, was adopted in 1989 in Basel
of 500+ in plain areas and 250+ in
especially women, SCs and STs, (Switzerland).
North–East, hill, tribal and desert
through the rights based
areas. The Stockholm Convention (2001)
legislation.
is related to Persistent Organic
l Strengthening decentralisation 47.Which of the following statements
Pollutants (POPs). ‘The Minamata
etc. is not correct regarding the convention (2013) on mercury is
Members of Parliament Local Area related to protection of human health
45. Which one of the following Development Scheme (MPLADS)? and environment from the adverse
effects of mercury.
statements with regard to the (a) Members of the Parliament (MPs)
functioning of the Panchayats is sanction, execute and complete The Rotterdam Convention (2004) is
related to protection of human health
not correct? works under the scheme
and environment from adverse effect
(a) Panchayats may levy, collect and (b) Nominated Members of the of chemicals and pesticides.
appropriate taxes, duties, tolls, Parliament can recommend works
for implementation anywhere in the 50. Which one of the following is the
etc.
country biggest cause of incidence of
(b) A person who has attained the
(c) The scheme is fully funded by the migration of female persons in
age of 25 years will be eligible to
Government of India India?
be a member of a Panchayat.
(c) Every Panchayat shall ordinarily (d) The annual entitlement per MP is ` 5 (a) Employment (b) Education
continue for five years from the crore (c) Marriage (d) Business
date of its first meeting. _ (a) Under MPLADS, the MPLAD _ (c) The biggest cause of incidence of
(d) A Panchayat reconstituted after division is responsible for the migration of female persons in India is
premature dissolution shall implementation of the scheme. Under marriage. According to Census Data
the scheme, each MP has the choice of 2011, out of total women migrants,
continue only for the remainder of to suggest to the District Collector for 64.9% migrated due to marriage,
the full period. works to the tune of ` 5 crore per 3.2% due to employment, 1.3% due
annum to be taken up in his/her to education and 0.3% due to
_ (b) Among the given options, only constituency. business.
option (b) is not correct. According
to Article-243 of the Indian 48.Saubhagya, a Government of India 51. Which one of the following is not
Constitution, a person who has
attained the age of 21 years (and not
scheme, relates to which of the correct about Repo rate?
25 years) will be eligible to be a following areas? (a) It is the interest rate charged by the
member of a Panchayat. Rest of the (a) Achieving universal household Central Bank on overnight loan.
statements are correct. electrification (b) It is the interest rate paid by the
(b) Providing clean cooking fuel to commercial banks on overnight
46. Which one of the following is the poor households borrowing.
earliest launched scheme of the (c) Rationalising subsidies on LPG (c) It is the interest rate agreed upon
Government of India? (d) Stopping female foeticide in the loan contract between a
commercial bank and the Central
(a) Deendayal Antyodaya Yojana _ (a) Pradhan Mantri Sahaj Bijli Har Ghar
Yojana (Also known as Saubhagya) Bank.
(b) Pradhan Mantri Gram Sadak
was launched in September, 2017. (d) It is the cost of collateral security.
Yojana
Under this scheme, free electricity
(c) Saansad Adarsh Gram Yojana connections to all households (both _ (d) Among the given options, only
(d) Deendayal Upadhyaya Grameen option (d) is incorrect. Repo Rate (or
APL and BPL) in rural areas and poor
repurchase rate) is the key policy rate
Kaushalya Yojana families in urban areas was provided.
of interest at which Central bank (or
Rural Electrification Corporation (REC)
_ (b) Among the given schemes of the has been designated as nodal agency
RBI) lends short-term money to
Government of India, Pradhan Mantri banks. It helps in managing the
for this scheme.
Gram Sadak Yojana was launched liquidity of the economy.
CDS Solved Paper 2020 (I) 69

52. The Cash Reserve Ratio refers to The Best FIFA Football Award is an 58. Amitabh Bachchan was recently
association football award presented conferred with the prestigious
(a) the share of Net Demand and annually by the sport’s governing
Time Liabilities that banks have to Dada Saheb Phalke Award. Who
body, FIFA. The first awarding
hold as liquid assets ceremony was held on 9th January,
among the following was the first
(b) the share of Net Demand and 2017 in Zurich (Switzerland). recipient of the award?
Time Liabilities that banks have to (a) Prithviraj Kapoor (b) Devika Rani
55. In September, 2019, which one of (c) Sohrab Modi (d) Naushad
hold as balances with the RBI the following travel giants
(c) the share of Net Demand and Time declared itself bankrupt? _ (b) The first Dada Saheb Phalke
Liabilities that banks have to hold Award was conferred to the actress
(a) Expedia (b) Cox and Kings Devika Rani in 1969. She is widely
as part of their cash reserves
(c) SOTC (d) Thomas Cook acknowledged as the ‘First Lady of
(d) the ratio of cash holding to
Indian Cinema’.
reserves of banks _ (d) In September, 2019, travel giant The Dada Saheb Phalke Award is
Thomas Cook declared itself
_ (b) The Cash Reserve Ratio (CRR) bankrupt. India’s highest award in cinema
refers to the share of Net Demand presented annually at the National
and Time Liabilities that banks have to Thomas Cook is a British tour
Film Awards Ceremony by the
hold as balances with the RBI. CRR is operator, one of the world’s oldest
Directorate of Film Festivals (an
set according to guidelines of the Central travel brands. One of the main
organisation under the Ministry of
Bank of a country. Currently, CRR is reasons for it going bankrupt was not
Information and Broadcasting).
4% of total deposits of banks in India. moving its market on digital platform.
CRR is also an instrument used by RBI
59. The famous Mughal painting,
to control the liquidity in the system. 56. Greta Thunberg, a teenage depicting Jahangir embracing the
environment activist who was in Safavid king Shah Abbas, was
53. In economics, if a diagram has a
news recently hails from painted by which one of the
line passing through the origin
following Mughal painters?
and has 45° angle with either axis (a) Sweden (b) Germany
and it is asserted that along the (c) The USA (d) Canada (a) Abd al-Samad (b) Abul Hasan
(c) Dasavant (d) Bishandas
line X = Y what is tacitly _ (a) Greta Thunberg, a teenage
assumed? environment activist hails from _ (b) The famous Mughal Painting,
Sweden. She addressed the 2018 UN depicting Jahangir embracing the
(a) Both variables are pure numbers. Safavid king Shah Abbas was painted
Climate Change Conference and took
(b) Both variables are in the same part in 2019 UN Climate Action by Abul Hasan in 1615 CE.
unit. Summit. Abul Hasan (1589–1630 CE) was a
(c) Both variables are in different Due to her activism, she received Mughal painter of miniatures under
units. awards like Ambassador of the reign of Jahangir. Jahangir
(d) At least one variable is a pure Conscience Award, Right Livelihood bestowed on him the title
number. Award, International Children’s Peace ‘Nadir–uz–Saman’ (Wonder of the Age).
Prize, etc in 2019. Abd al–Samad is one of the founding
_ (b) Y masters of the Mughal miniature
57. Recently the Reserve Bank of India
tradition. He headed the Mughal imperial
has imposed limitations, initially workshop during reign of Akbar.
for a period of six months, on the
Dasavant was also a painter in
withdrawal of amount by account Mughal Court during Akbar.
45°
holders of which one of the Bishandas was a Mughal painter
45° following banks? during emperor Jahangir.
X
(a) IndusInd Bank 60. Which of the following statements
It is assumed that along the line X = (b) Dhanlaxmi Bank
about ‘Mughal Mansab’ system are
Y. Then, tacitly assumed that (c) Punjab and Maharashtra
variables of X and Y are in the same correct?
Cooperative Bank
units. Because an angle of 45° (d) South Indian Bank 1. ‘Zat’ rank was an indicator of a
degree means that value of X equal Mansabdar’s position in the
the value of Y. _ (c) In September, 2019 the RBI has imperial hierarchy and the salary
imposed limitations on the withdrawal of the Mansabdar.
54. Who among the following won amount by the account holders of
the Best Men’s Player Award of Punjab and Maharashtra Cooperative
2. ‘Sawar’ rank indicated the number
Bank (PMC), initially for a period of of horsemen the Mansabdar was
FIFA Football Awards, 2019? required to maintain.
six months. This was done in the
(a) Cristiano Ronaldo light of recently exposed scam in the 3. In the seventeenth century,
(b) Virgil van Dijk bank. Mansabdars holding 1000 or above
(c) Lionel Messi PMC bank was founded in 1984 and ‘Sawar’ rank were designated as
(d) Xavi has 137 branches across seven nobles (Umara).
States. Its customers include small Codes
_ (c) Lionel Messi won the best Men’s business, housing societies and (a) 1 and 2 (b) 1 and 3
player Award of FIFA Football Awards,
2019. institutions. (c) 2 and 3 (d) 1, 2 and 3
70 CDS Solved Paper 2020 (I)

_ (d) According to the Mughal Mansab (b) It is a text ushering in linguistics as 66. Which of the following are warm
or Mans ab dari system ‘Zat’ rank was a formal science ocean currents?
an indicator of a Mansabdar’s (c) It is a text discussing
position in the imperial hierarchy and (a) Kuroshio and California Current
developments in various spheres (b) North Atlantic Drift and Brazil
the Salary.
of natural sciences Current
‘Sawar’ rank indicated the number of
horsemer the Mansabdar was
(d) It is a text showing through (c) Canaries and Benguela Current
required to maintain. illustration what should and (d) West Wind Drift and Falkland
In the 17th century, Mansabdars
should not be done Current
holding 1000 or above ‘Samar’ rank _ (d) Panchatantra is a text showing _ (b) North Atlantic Drift (North Atlantic
were designated as nobels (Umara). through illustration what should and
Ocean) and Brazil current (South
Hence, all the statements are correct. should not be done. It is a
Atlantic Ocean) are warm ocean
‘Niti-Shastra’ (or textbook of Niti)
currents.
which roughly means ‘the book on
The Mansabdari system was wise conduct of life’. 67. In India, how many States/Union
introduced by Mughal emperor Akbar
as new administrative machinery and Vishnu Sharma is considered to Territories have more than two
revenue system. be the author of the text. Original international boundaries?
text was written in Sanskrit
61. Which one of the following pairs language. (a) 1 (b) 2 (c) 3 (d) 4
is not correctly matched? 64. Geomorphic factors influencing _ (d) In India, 4 States/Union Territories
have more than two international
(a) Kuddapah-kar Rocky wastelands plant and animal distributions are boundaries. These are
(b) Nancai Wet fields (a) slope angle and relief only l Ladakh with Pakistan, Afghanistan
(c) Puncai Dry fields (b) slope aspect and relative relief and China.
(d) Tottakal Garden lands (c) slope angle, slope aspect and l Sikkim with Nepal, Bhutan and
China.
_ (a) Among the following options, only relief
option (a) is not correctly matched. l West Bengal with Nepal, Bhutan
(d) slope angle, slope aspect and
and Bangladesh.
In ancient India, Kuddapah–kar was relative relief
l Arunachal Pradesh with Bhutan,
one of the two main seasons of rice
cultivation (the other being _ (d) Geomorphic factors influencing China and Myanmar.
Samba-peshanam). Rest all the plant and animal distribution are slope
angle, slope aspect and the 68. In the Hadley cell thermal
options are correctly matched. circulation, air rises up and finally
distribution is also influenced by
62. Which of the following rulers climatic factors (like temperature, descends at
were identified through water, light, etc.) and Edopic factors
(like salinity, soil pH, mineral nutrients,
(a) intertropical convergence zone
matronymics (names derived from etc.) (b) doldrums
that of the mother)? (c) sub-tropical high-pressure cells
65. Which one of the following
(a) Mallas of Pava (d) equatorial troughs
groups of cities does not have
(b) Videhas of Mithila
Sclerophyll as its natural _ (c) In the Hadley cell thermal
(c) Yaudheyas circulation, air rises up (at equator)
(d) Satavahanas
vegetation cover? and finally descends at sub-tropical
(a) Valparaiso and Cape Town high- pressure cells.
_ (d) Satavahanas rulers were identified (b) Lisbon and Perth According to tri-cellular model of
through matronymics. For example,
the name of greatest ruler of (c) Los Angeles and Adelaide atmospheric circulation, circulation
Satavahanas was Gauthamiputra (d) Las Vegas and Queensland takes place in three cells namely:
Satakarni, which meant Gauthami’s Hadley Cell Air rises at equator (0°)
son Satakarni. _ (d) Among the following groups of and decends near sub-tropical
cities, Las Vegas and Queensland
The Satavahanas (or Andhras) were does not have Sclerophyll as its high-pressure zone (30° N and S).
ancient Indian dynasty based in natural vegetation cover. Ferrel Cell Air decends at
Deccan region. They reigned from sub-tropical high pressure zone (30°
Sclerophyll is a type of vegetation
late 2nd century BCE to early 3rd N and S ) and ascends at sub-polar
that has hard leaves, short
century CE and ruled over present low pressure zone (60° N and S).
internodes and leaf orientation
day Telangana, Maharashtra, Andhra
parallel or oblique to direct Polar Cell Air rises in sub–polar low
Pradesh and parts of Gujarat,
sunlight. This vegetation is mainly pressure zone (60° N and S) and
Madhya Pradesh and Karnataka.
found in Mediterranean type of descends at poles in both hemisphere.
63. Which one of the following climate.
69. Which one of the following soils is
statements about the famous text Thus, Sclerophyll vegetation is found
characterised by very high content
of Panchatantra is correct? in areas like Perth, Sydney and
Adelaide region of Australia, the of organic matter?
(a) It is a philosophical text reflecting Mediterranean basin, Californian (a) Vertisol (b) Histosol
the debates of the time and region, Cape province of South Africa, (c) Gelisol (d) Spodosol
refuting rival positions etc.
CDS Solved Paper 2020 (I) 71

_ (b) Histosol soils are characterised Eligibility criteria for the scheme 2002. This was done by amending the
by very high content of organic include Article-51 A of the Constitution.
matter. They contain at least 20-30% l No restrictions on family size, age This amendment further added
of organic matter by weight. Histosols or gender. Article-21A to the Constitution.
are often very difficult to cultivate l All pre-existing conditions are Article-21A states that, “the State
because of poor drainage and due to covered from day one. shall provide free and compulsory
low chemical fertility. l Covers upto 3 days of education to all children of the age of
Vertisols are rich in clay content. pre-hospitalisation and 15 days of 6 to 14 years in such manner as the
Gelisols are permafrost soils in post-hospitalisation expenses. state may, by law, determine.”
l Benefits of the scheme are portable Moreover, 86th Constitutional
which organic content is limited to
uppermost layer. across country. Amendment Act substituted new
l Public hospitals are reimbursed for article for Article-45. New article
Spodosols are acidic soils rich in healthcare services at par with states that, “The state shall
iron and aluminium. private hospitals. endeavour to provide early childhood
70. Overseas Indians can exercise 72. Which of the following are care and education for all children
franchise in an election to the Lok until they complete the age of 6 years.”
considered to be the four pillars of
Sabha under which of the human development? 74. Which one of the following
following conditions? (a) Equity, inclusion, productivity and
Articles of the Constitution of
1. They must be citizens of India. empowerment
India protects a person against
2. Their names must figure in the (b) Equity, productivity, double jeopardy?
electoral roll. empowerment and sustainability (a) Article-20 (b) Article-21
3. They must be present in India to vote. (c) Productivity, gender, inclusion (c) Article-22 (d) Article-23
Select the correct answer using the and equity
_ (a) Article-20 of the Constitution of
codes given below. (d) Labour, productivity, inclusion India protects a person against
(a) 1, 2 and 3 (b) 2 and 3 and equity double jeopardy. This means that the
Constitution prohibits the prosecution
(c) 1 and 2 (d) Only 1 _ (b) Human development is defined and punishment for the same offence
as the process of enlarging people’s more than once.
_ (a) All the statements regarding freedoms and opportunities, and
franchise of overseas Indians are Other provisions of the Article-20 are
correct. improving their well-being. This
l Protection against Ex-post Facto
concept was propounded by
According to Section 20A of the economist Mahbub-ul-haq in 1970s. legislation, i.e. an individual cannot
Representation of People Act, 1950, be convicted for actions that were
The four pillars of human committed before the enactment of
overseas Indians can exercise
development are the law.
franchise in an election to the Lok
Sabha only under the conditions Equity It refers to creating equal l Immunity from self-incrimination, i.e.

l They must be citizens of India.


access to opportunities and ensure no person accused of any offence
that it is available to all. shall be compelled to be a witness
l Their names must figure in electoral
roles. Sustainability It means durableness against himself.
l They must be present in India to
in the availability of opportunities. 75. Consider the following statements
vote. Productivity It means productivity in about Stone Age in India :
terms of human work.
71. Which one of the following is not 1. Different periods are identified on
Empowerment It means to have the basis of the type and
a feature of the Ayushman Bharat
power to make choices.
Scheme? technology of stone tools.
(a) There is no cap on family size and
73. Which one of the following was 2. There are no regional variations in
added as a Fundamental duty the type and technology of tools in
age
through the Constitution (86th different periods.
(b) The scheme includes pre-and
Amendment) Act, 2002? 3. Stone Age cultures of different
post hospitalisation expenses periods evolved uniformly in a
(c) A defined transport allowance per (a) To strive towards excellence in neat unilinear fashion all over the
hospitalisation will also be paid to individual and collective activity subcontinent.
the beneficiary (b) To provide opportunities for Which of the statements given above
(d) The scheme provides a benefit education to one’s child between is/are correct?
cover of ` 10 lakh per family the age of 6 and 14 years
(a) Only 1 (b) 1 and 2
(c) To work for the welfare of women
_ (d) Among the following options, only and children
(c) Only 3 (d) 1, 2 and 3
option (d) is incorrect.
Ayushman Bharat Scheme provides (d) To promote peace and harmony _ (a) Among the given statements, only
statement 1 is correct about Stone
a benefit cover of ` 5 lakh (and not _ (b) The Fundamental Duty “To Age in India.
`10 lakh) per family per year. provide opportunities for education to
one’s child between the age of 6 and Stone Age in India was characterised
Ayushman Bharat Scheme is world’s by different periods that were
largest health insurance scheme fully 14 years”, was added through the
Constitution (86th Amendment) Act of identified on the basis of the type
financed by the Government of India.
72 CDS Solved Paper 2020 (I)

and technology of store tools. For _ (b) Lord Palmerston believed that the In 2019, the festival was organised
instance, during palaeolithic age, Russian designs were ‘an imminent near Hemis Monastry of Ladakh.
tools were made of quartzite and peril to the security and tranquility’ of The shordol dance (folk dance of
sharpened by chipping technique; the Indian empire in 1836. Ladakh) is performed during this
Mesolithic age was characterised by Lord Palmerston was foreign festival.
use of ‘microlith’ store tools; while during secretary during the Whig Cabinet of
neolithic age tools were more Lord Melborne. It was this belief that
82. Which one of the following is
specialised (like knife, dagger, etc.) guided the policy of British India India’s official entry for the Best
and were completely or partially towards Afghanistan. This finally led International Feature Film
polished. to Afghan wars. category in the 92nd Academy
There was large regional variations in Awards?
the type and technology of tools in
79. The ‘Tattvabodhini Sabha’ was
different periods. For example, in the established by (a) Bulbul Can Sing
foothills of Himalayas, tools were (a) Devendranath Tagore in 1839 (b) Super Deluxe
majorly made up of pebbles, while in (b) Keshab Chandra Sen in 1857 (c) Gully Boy
central India basaltic rocks were main (d) And The Oscar Goes To
(c) Akshay Kumar Datta in 1850
type of stone used in making tools.
(d) Dwarakanath Tagore in 1840 _ (c) Gully Boy was selected as India’s
Stone Age in India did not evolve
official entry for the Best International
uniformly in a unilinear fashion over _ (a) The ‘Tattvabodhini Sabha’ was Feature Film category in the 92nd
the sub-continent. For instance, established by Devendranath Tagore
in 1839. It was established as Academy Awards.
stone tools in Shivalik hills date back
to about 2 to 1.2 million years, while Tattvaranjini Sabha and later renamed The Academy Award for Best
in Bori (Maharashtra) they data back as Tattvabodhini Sabha. This Sabha International Feature Film Award is
to 1 million years back. was a particular reform movement one of the Academy Awards given
organisation, aimed to popularise annually by the USA based Academy
76. From which one of the following Brahmodharma (or Brahmo faith). Its of Motion Picture Arts and Sciences
factory sites were limestone and primary objective was to propagate (AMPAS).
chert blades mass produced and the spirit of Hindu Scriptures,
including the Vedas. 83. The Global Goalkeeper Award is
sent to various Harappan given by
settlements in Sindh? 80. What was the code name given to
the first ever tri-service military (a) the Bill and Melinda Gates
(a) Sukkur and Rohri Hills Foundation
(b) Khetri in Rajasthan
exercise between India and USA?
(b) the United Nations Environment
(c) Chagai Hills (a) Lion Triumph Programme
(d) Hills of Baluchistan (b) Elephant Triumph (c) the Kellogg School of
(c) Tiger Triumph Management
_ (a) Sukkur and Rohri hills were the (d) Bison Triumph
factory sites from where limestone (d) the World Meteorological
and chert blades were mass _ (c) Tiger Triumph was the code Organisation
produced and sent to various name given to the first ever tri-service
Harappan settlements in Sindh. military exercise between India and _ (a) The Global Goalkeeper Award is
USA. This exercise was a given by the Bill and Melinda Gates
Khetri in Rajasthan was known for
Humanitarian Assistance and Disaster Foundation. In 2019, it was given to
exploitation of copper. Chagai hills
Relief (HADR) exercise. It was held in the Prime Minister of India for India’s
were known for availability of Lapis
November, 2019 off the massive progress under the Swachh
Lazuli stone during the period. Bharat Abhiyan.
Visakhapatnam and Kakinada coasts
77. The work Siyar-ul- Mutakherin, in Andhra Pradesh. The Goalkeepers, Global Goals
which describes the Battle of Other joint exercises between India Awards aim to highlight the
Plassey, 1757, was written by and USA are extraordinary stories of remarkable
(a) Salabat Jung (b) Qasim Khan individuals who are taking action to
l Military exercise : Yudh Abhyas and
give life to the global goals and help
(c) Ghulam Husain (d) Ram Mohan Roy Vajra Prahar
achieve them by 2030. In 2019,
l Air Forces Exercise : Cope India
_ (c) The work Siyar-ul-Mutakherin, l Naval Exercise (includes Japan) :
fourth edition of these awards was
which describes the Battle of Plassey, presented.
1757 was written by Ghulam Husain. Malabar.
These awards were presented in
This work was completed in 1781 CE 81. ‘Naropa’ is an annual festival of 5 categories in 2019–Progress Award
and has three volumes. (age 16-30), Changemaker Award
(a) Sikkim
78. Who believed that the Russian (age 16-30), Campaign Award (age
(b) Ladakh 16-30), Goalkeepers Voice Award
designs were ‘an imminent peril to (c) Arunachal Pradesh
the security and tranquility’ of the (any age) and the Global Goalkeeper
(d) Nagaland Award (any age).
Indian Empire in 1836?
(a) Lord Auckland _ (b) Naropa is an annual festival of The Global Goal Keeper Award aims
Ladakh. It is one of the biggest to recognise a political leader for
(b) Lord Palmerston festivals in the Himalayas that his/her commitment to global goals
(c) Lord Canning celebrates the life and legacy of the through impactful work in their
(d) Alexander Burnes Buddhist Scholar Naropa. country.
CDS Solved Paper 2020 (I) 73

84. ‘Gandhi Solar Park’ is located at _ (b) Only statements 1 and 3 regarding 91. Which one of the following Articles
Coal India Limited (CIL) are correct. was defended by Dr. BR Ambedkar
(a) New York (b) Vladivostok
(c) Thimphu (d) Houston CIL is a state owned coal mining on the plea that it would be used
corporate, which came into being in
as ‘a matter of last resort’?
_ (a) ‘Gandhi Solar Park’ is located at November, 1975. CIL’s headquarters
New York (UN headquarters). This is at Kolkata. CIL is the single largest (a) Article-352 (b) Article-359
park was inaugurated by PM coal-producing company in the (c) Article-356 (d) Article 368
Narendra Modi alongwith other world world. CIL operates through 82
leaders, commemorating the 150th mining areas spread over eight (and _ (c) Article-356 was defended by Dr.
birth anniversary of Mahatma Gandhi. BR Ambedkar on the plea that it
not 20) provincial states of India.
would be used as a matter of last
Gandhi Solar Park is a first of its kind
symbolic effort by India at the UN. It
88. Which one of the following climatic resort.
types is found in Central Spain? Article-356 imposes President’s rule
has 50 KW capacity, and a gesture
in the states, if the government of the
by India that it is willing to go beyond (a) Subarctic
state cannot carry on the governance
the talk on climate change. (b) Mediterranean dry hot summer according to the provisions of the
85. Who among the following was the (c) Subtropical Steppe Constitution.
first to arrive in Africa as traders (d) Humid continental warm summer Since, this article empowers Union
Government over State Government
that eventually led to European _ (b) Climate of Spain is mainly it weakens federalism. Hence, it was
colonisation of Africa? Mediterranean with the extremes of
temperatures being found in Central opposed by various members of the
(a) French (b) Spanish Spain (due to continentality). Constituent Assembly. As a result,
(c) Portuguese (d) Dutch Mediterranean type of climate is Dr. Ambedkar defended this
characterised by seasonal shifting of provision by stating the above
_ (c) Portuguese were the first to arrive pressure belts. In this climate, statement.
in Africa as traders that eventually led summers are hot and dry while
to European colonisation of Africa. winters are cold and wet. It is best 92. What is the ground on which the
They first reached Africa as explorers developed in the European region Supreme Court can refuse relief
under Henry the Navigator. Initially, bordering Mediterranean sea.
under Article-32?
the Europeans colonised the areas 89. Which one of the following is not
which were un-inhabited like cape (a) The aggrieved person can get
varde region.
among the principal languages of remedy from another court.
Jammu and Kashmir? (b) That disputed facts have to be
86. The college of Military Engineering (a) Urdu (b) Gujari investigated.
affiliated to Jawaharlal Nehru (c) Koshur (d) Monpa (c) That no Fundamental Right has
University is situated at been infringed.
(a) New Delhi (b) Dehradun
_ (d) Monpa is not among the principal (d) That the petitioner has not asked
languages of Jammu and Kashmir.
(c) Nainital (d) Pune Monpa language is spoken by the for the proper writ applicable to
Monpa ethnic group of Arunachal his/her case.
_ (d) The college of Military Engineering Pradesh.
affiliated to Jawaharlal Nehru University _ (c) The Supreme Court can refuse
is situated at Pune. It was established According to the department of tourism relief under 32 on the ground that no
in 1943 as School of Military of Jammu and Kashmir, major Fundamental Right has been infringed.
Engineering at Roorkee and later languages of Jammu and Kashmir Article-32 provides the right to
moved to Pune in 1948. This was done are Urdu, Hindi and Koshur. Further, constitutional remedies which means
in view of increased responsibilities the Gujari language spoken by Gurjar that a person has a right to move to
and in keeping with the higher status of tribes which are spread across North Supreme Court for getting his/her
the degree of Engineering courses India, including Jammu and Kashmir. Fundamental Rights protected.
being conducted by the school. The Supreme Court has the power to
institute is the premier technical 90. The major part of Central Asia is
issue writs, homely habeas corpus,
training institution of the Corps of dominated by which one of the mandamus, prohibition, quo
Engineers of the Indian Army. following language families? warranto and certiorari.
87. Which of the following statements (a) Indo-European (b) Sino-Tibetan
(c) Austric (d) Altaic
93. The Ministry of Heavy Industries
with regard to Coal India Limited and Public Enterprises consists of
(CIL) is/are true? _ (d) The major part of Central Asia is (a) the Department of Heavy Industry
dominated by Altaic family of
1. CIL has its headquarters at Kolkata. languages. Altaic group of language, and the Department for Promotion
2. CIL operates through 82 mining mainly includes Turkie, Mangolian and of Industry and Internal Trade
area spread over twenty provincial Tungusic family of languages. (b) the Department of Public Enterprises
States of India. Apart from Altaic, Indo-European is and the Department for Promotion
3. CIL is the single largest coal the second major family of language of Industry and Internal Trade
producing company in the world. in this region. Central Asia is the (c) the Department of Scientific and
region which stretches from Caspian Industrial Research and the
Select the correct answer using the sea in the West, China in the East, Department of Heavy Industry
codes given below. Iran and Afghanistan in the South (d) the Department of Heavy Industry
(a) Only 1 (b) 1 and 3 and Russia in the North. It consists of and the Department of Public
Kazakhstan, Kyrgyzstan, Tajikistan,
(c) 2 and 3 (d) 1, 2 and 3 Enterprises
Turkmenistan and Uzbekistan.
74 CDS Solved Paper 2020 (I)

_ (d) The Ministry of Heavy Industries l Any dispute between Indian _ (d) Rustom-2, which crashed in
and Public Enterprises consists of the Government and one or more states. Karnataka September, 2019, was an
department of Heavy Industry and the l Any dispute between Indian unmanned aerial vehicle. Aeronautical
department of Public Enterprises. Government and one or more Development Establishment (ADE), a
Department for promotion of Industry states on one side and one or more laboratory of DRDO based in
and Internal Trade is under the states on the other side. Bangalore was responsible for its
Ministry of Commerce and Industry. l Any dispute between two or more design and development. It is a
Department of Scientific and states, etc. Medium-Altitude Long Endurance
industrial research is under the 97. Who among the following is the Unmanned Aerial Vehicle
Ministry of Science and Technology. (MALE-UAV) that flies at an altitude of
Chairman of the Economic 3000-9000 m for extended durations
94. The First Delimitation Advisory Council to the Prime of time.
Commission in India was Minister (EAC-PM)?
constituted in 101. The maiden trilateral naval
(a) Ratan P. Watal (b) Bibek Debroy exercise involving India, Singapore
(a) 1949 (b) 1950 (c) 1951(d) 1952 (c) Ashima Goyal (d) Sajjid Chinoy
and Thailand was held at
_ (d) The first Delimitation Commission _ (b) Bibek Debroy is the Chairman of (a) Port Blair (b) Chennai
in India was constituted in 1952 under the Economic Advisory Council to the
the Delimitation Commission Act, Prime Minister (EAC-PM).
(c) Panaji (d) Kochi
1952.
EAC-PM is an independent body _ (a) The maiden trilateral exercise
Delimitation Commission is a constituted to give advice on involving the Republic of Singapore’s
statutory body concerned with fixing economic and related issues to the Navy (RSN), Royal Thailand Navy
the limits of territorial constituencies Government of India, specifically to (RTN) and Indian Navy was held at
in the country. Till now, Delimitation the Prime Minister. Port Blair, in Andaman sea in
Commission has been constituted September, 2019. The name of
four times in 1952, 1963, 1973 and Ratan P. Watal is the Member
Secretary of EAC-PM, while Ashima exercise was SITMEX-19
2002, respectively. (Singapore-India Thailand Maritime
Goyal and Sajjid Chinoy are part-time
95. Who among the following stated members. Exercise). The objective of exercise as
to bolster the maritime
in the Constituent Assembly that 98. Hilsa is the national fish of inter-relationship among them and
on 26th January, 1950, India was contribute significantly to enhance the
(a) Pakistan (b) India
going to enter a life of overall security in the region.
(c) Bangladesh (d) Nepal
contradictions?
102. The creation of a Federal Court in
(a) Dr. BR Ambedkar _ (c) Hilsa is the national fish of
Bangladesh. Around 70% of the total India was advocated by which
(b) Jawaharlal Nehru Hilsa fish production in the world is of the following
(c) Mahatma Gandhi from Bangladesh. Acts/Commissions?
(d) SP Mukherjee Hilsa is also the State fish of West
(a) The Government of India Act, 1919
Bengal. Masheer is the national fish
_ (a) Dr. BR Ambedkar was the one of Pakistan. Dolphin is the national (b) The Lee Commission, 1923
who stated in the Constituent
fish of India. (c) The Government of India
Assembly that on 26th January, 1950,
India was going to enter a life of Act, 1935
99. The Vijaynagar Advanced Landing
contradictions. This because, with the (d) The Indian Councils Act, 1909
enactment of Constitution, India
Ground of the Indian Air Force,
would have political equality (one which was reopened recently is _ (c) The Government of India Act,
located in 1935 advocated for the creation of a
person, one vote), but in social and
economic life, India will have Federal Court in India. This act was
(a) Jammu and Kashmir watershed moment in the evolution of
inequality.
(b) Arunachal Pradesh Indian Constitutional. This act was
96. The power of the Supreme Court to (c) Karnataka enacted during the period of
decide in the case of a dispute (d) Himachal Pradesh Governor-General, Lord Wellingto.
between two or more States is called Other important provisions of this act
_ (b) The Vijaynagar Advanced are as follows
(a) original jurisdiction Landing Ground of the Indian Air
l Establishment of federation of India
(b) inherent jurisdiction Force is located in Arunachal
with British India territories and
Pradesh. Earlier, it was declared unfit
(c) plenary jurisdiction Princely states.
for fixed wing aircraft such as
(d) advisory jurisdiction transport planes as the runway got l Elaborate safeguards and

damaged. It was reopened in protective instruments for


_ (a) The power of the Supreme Court minorities.
to decide in the case of a dispute September, 2019.
between two or more states is called 100. Rustom-2, which crashed in 103. Who founded the ‘Seva Samiti’ at
original jurisdiction.
Karnataka recently, was a/an Allahabad in 1914?
Original jurisdiction of Supreme
Court is under Article-131. It refers to (a) fighter aircraft (a) Hridayanath Kunzru
matters for which Supreme Court can (b) helicopter (b) GK Gokhale
be approached directly. Few matters (c) transport aircraft (c) Shri Ram Bajpai
covered under original jurisdiction are (d) unmanned aerial vehicle (d) TB Sapru
CDS Solved Paper 2020 (I) 75

_ (a) Seva Samiti was founded by the 106. The power to legislate on all 108. The provisions of the Constitution
Hridayanath Kunzru at Allahabad matters relating to elections to of India pertaining to the
(Prayagraj) in 1914. He had set-up
this samiti to organise social service Panchayats lies with institution of Panchayat do not
during natural disaster like flood, to (a) the Parliament of India apply to which one of the
promote education, sanitation, to (b) the State Legislatures following States?
uplift depressed class and reform (c) the State Election Commission (a) Meghalaya (b) Tripura
criminals. He was one of the most
prominent member of the servants of (d) the Election Commission of India (c) Assam (d) Goa
India society, founded by GK Gokhale _ (b) According to the 73rd
in 1905.
_ (a) Panchayati Raj is the basic unit of
Constitutional Amendment Act, 1992 administration in a system of
104. The State of Hyderabad in the the power to legislate on all matters governance. The Constitutional (73rd
relating to election to Panchayat lies Amendment) Act, 1992 came into
Deccan officially acceded to the with the State Legislature. Under the force in India on 24th April, 1993 to
Indian Union in the year Article-329, an election to a provide constitutional status to the
(a) 1948 (b) 1950 Panchayat can be called in question Panchayati Raj institutions. This act
only by an election petition which was extended to the Panchayats in
(c) 1949 (d) 1947
should be presented to such authority the tribal areas of eight states,
_ (a) After getting Independence from and in such manner as may be namely Andhra Pradesh, Gujarat,
Britisher’s in 1947, the Princely State prescribed by or under any law made Himachal Pradesh, Maharashtra,
of Hyderabad in the Deccan, declared by the State Legislature. Madhya Pradesh, Odisha and
itself as an independent province and Rajasthan from 24th December,
denied to amalgamate with India. The 107. The 11th Schedule of the 1996. Currently, the Panchayati Raj
Nizam Osman Alikhan of Hyderabad Constitution of India distributes system exists in all States of India
formed an irregular army known as powers between except Nagaland, Meghalaya,
Razakars. When talks were not Mizoram and in all Union Territories
resulted into any solution, then (a) the Union and the State
except Delhi and Jammu and
military action was taken under Legislatures Kashmir.
the code name ‘Operation Polo’ in (b) the State Legislatures and the
1948. Panchayat 109. Which one of the following rivers
Razakar’s were defeated very badly (c) the Municipal Corporation and does not drain into Black sea?
and State of Hyderabad was merged the Panchayat (a) Volga (b) Dnieper
with India. (d) the Gram Sabha and the (c) Don (d) Danube
105. The Hunter Commission (1882) Panchayat
_ (a) Volga river does not drain into
appointed to survey the State of _ (b) The 11th Schedule of the
Black sea. Black sea is a body of
education in India Constitution of India distribute powers water and marginal sea of
between the State Legislature and Atlantic Ocean between Eastern
(a) deprecated University education Europe, the Car casus and
Panchayat. There are overall
(b) overruled the Despatch on 1854 12 Schedules in the Constitution. The Western Asia. Many rivers drain into it
(c) endorsed the Despatch of 1854 Schedules are as follows such as the Danube, Dnieper,
with greater emphasis on primary Southern Bug, Dniester, Don and the
Schedule Subject
Rioni.
education 1st List of State and Union Territories
(d) criticised the grants-in-aid system The Volga river is the longest river in
2nd Salary and Emolument Europe. It is also Europe’s largest
of schooling
3rd Form of oaths and affirmation river in terms of discharge and
_ (c) The Hunter Commission was 4th Allocation of seats to State and UTs in
drainage basin. The Volga river flows
appointed by the then through Central Russia and drains
the Rajya Sabha
Governor-General of India in 1882 to into Caspian sea.
survey the state of education in India. 5th Administration and Control of
The report of Hunter Commission Scheduled Areas 110. The National Water Academy
endorsed the Despatch of 1854 with 6th Administration of Assam, Meghalaya, (NWA) is located at
greater emphasis on primary Tripura and Mizoram (a) Dehradun
education. This commission gave
7th Distribution of Power between the (b) Hyderabad
suggestion regarding secondary
education, grant-in-aid for indigenous Union and the State List (c) Bhopal
schools, encouragement of primary 8th List of Recognised Language (d) Khadakwasla
education and also emphasised
on the female education in the
9th Validation of Certain Act and _ (d) National Water Academy (NWA)
Regulation is located at Khadakwasla, Pune.
country.
10th Provision also disqualification on NWA was formerly known as
Charles Wood had sent a despatch Central Training Unit. It was formed
ground of defection
to Lord Dalhousie, the then in 1988 to impart training to the
Governor- General of India for 11th It has 29 matters. This schedule was in-service engineers of various
disseminating education in India. The added by the 73rd Amendment Act, Central/State organisations
Woods despatch is considered as 1992 involved in the development
12th Contains the Powers, Authority and and management of water
‘Magna-Carta’ of English Education
Responsibilities of Panchayat resources.
in India.
76 CDS Solved Paper 2020 (I)

111. Which one of the following is the 113. Which one of the following is the 115. According to the Census 2011, in
correct sequence of formation of latest addition to the AYUSH group India, what is the percentage of
the Commissions starting from the of healthcare system? people (approximately) considered
earliest? (a) Unani (b) Siddha to be migrants (internal), i.e., now
(a) Finance Commission, Planning (c) Sowa-Rigpa (d) Reiki settled in a place different from
Commission, Investment their previous residence?
_ (c) Sowa-Rigpa is the latest addition
Commission, Election to the AYUSH group of healthcare (a) 25% (b) 35%
Commission systems. The Union Cabinet in (c) 45% (d) 55%
(b) Election Commission, Planning November, 2019 also approved for
setting up of National Institute of _ (b) According to census data 2011,
Commission, Finance 45.36 crore Indians (35%) in India are
Sowa-Ragpa (NISR) at Leh (Ladakh). internal migrants, now settled in a
Commission, Investment
Sowa-Rigpa is a traditional system of place different from their previous
Commission medicine in the Himalayan Belt of residence. In 2001, the figure stood at
(c) Planning Commission, Election India. It originated in Tibet and 31.45 crore. Most of the migrants,
Commission, Finance popularly practised in countries around 70%, are females who migrate
Commission, Investment namely, India, Nepal, Bhutan, for marriage.
Commission Mongolia and Russia. Migrants by place of birth are those
(d) Investment Commission, Finance The other AYUSH systems are, who are enumerated at a village/town
Commission, Planning Ayurveda, Yoga, Naturopathy, at the time of census other than their
Unani, Siddha and Homeopathy. place of birth. A person is considered
Commission, Election as migrant by place of last residence,
Reiki has not been included
Commission in AYUSH group of healthcare if the place in which he is
enumerated during the census is
_ (b) In option (b) the commissions are systems.
other than his place of immediate last
arranged in the correct sequence. The
Election Commission of India was
114. Which one of the following is the residence.
formed on 25th January, 1950. The nodal agency in India for the 116. Suppose an agricultural labourer
Planning Commission was formed on United Nations Environment earns ` 400 per day in her village.
15th March, 1950. The Finance Programme?
Commission was first time formed on She gets a job to work as
22nd November, 1951. The (a) The Ministry of Environment, babysitter in a nearby town @ `
Investment Commission of India (ICI) Forest and Climate Change 700 per day. She chose to work as
was formed in December 2004. (b) The Ministry of Science and agricultural labourer. Which one
112. The formulation of policy in Technology of the following is the opportunity
respect to Intellectual Property (c) The Ministry of Earth Sciences cost of the agricultural labourer?
Rights (IPRs) is the responsibility (d) The Ministry of Home Affairs
(a) ` 1100 (b) ` 700
of _ (a) The Ministry of Environment, (c) ` 400 (d) ` 300
Forest and Climate Change
(a) the Ministry of Law and Justice _ (d) The opportunity cost is the loss of
(MoEFCC) is the nodal agency in
(b) the Department of Science and the administrative structure of the potential gain from other alternatives
Technology Central Government for the when one alternative is chosen. The
(c) the Department for Promotion of planning, promotion, coordination opportunity cost is the ‘cost’ incurred
and overseeing the implementation by not enjoying the benefit
Industry and Internal Trade associated with the best alternative
of India’s environmental and
(d) the Ministry of Human Resource choice. Opportunity cost is a key
forestry policies and
Development programmes. concept in economics and has been
described as expressing “the basic
_ (c) The Comptroller General of The Ministry also serves as the nodal relationship between scarcity and
Patents, Designs and Trade Marks agency in the country for the United choice”.
(CGPDTM) under the Department for Nations Environment Programme
Promotion of Industry and Internal (UNEP). Opportunity cost requires sacrifices. If
Trade (DPIIT) under the Ministry of The United Nations Environment there is no sacrifice involved in a
Commerce and Industry is entrusted Programme (UNEP) is the leading decision, there will be no opportunity
global environmental authority that cost. In this regard the opportunity
with the responsibility of administering
sets the global environmental costs not involving cash flows are not
the laws relating to Patents, Designs,
agenda, promotes the coherent recorded in the books of
Trade Marks and Geographic
implementation of the environmental accounts, but they are important
Indications with the Territory of India.
dimension of sustainable considerations in business decisions.
The DPIIT is also entrusted with
development within the United So, when an agricultural labourer
matters concerning the UN agency
Nations system and serves as an refuses to work as babysitter, she
(World Intellectual Property
authoritative advocate for the global has incurred an opportunity cost of
Organisation, WIPO) on Intellectual ` 300.
Property Rights (IPRs). environment.
CDS Solved Paper 2020 (I) 77

117. Match List-I with List-II and select many producers sell products that _ (c) BRICS is the acronym coined for
the correct answer using the codes are differentiated from one another an association of five major emerging
and hence are not perfect national economies: Brazil, Russia,
given below the Lists : substitutes. An oligopoly is a market India, China and South Africa. Since
List-I List-II form wherein a market or industry is 2009, the BRICS nations have met
(Market (Characteristic) dominated by a small number of annually at formal summits. Russia
structure) large sellers. hosted the 12th BRICS Summit in
July, 2020 at St Petersburg. China
A Perfect 1. Only one producer 118. Which one of the following was hosted the 9th BRICS Summit at
competition selling one commodity the host country for World Xiamen in September, 2017, while
B Monopoly 2. Few producers selling Tourism Day, 2019? The 11th BRICS Summit was
similar or almost similar convened in November 2020 at
(a) The USA Brasilia, Brazil.
products
(b) India
C Monopolistic 3. Many producers selling The 2019 BRICS Summit was
(c) Russia focused on the theme, ‘BRICS:
Competition differentiated products (d) Canada Economic Growth for an Innovative
D Oligopoly 4. Many producers selling Future’.
similar products _ (b) World Tourism Day is observed
every year on 27th September with 120. The Government of India has
Codes the objective to raise awareness
about the importance of tourism
recently constituted a civilian
A B C D
(a) 4 3 1 2 among the global community and to award in the name of Sardar
(b) 4 1 3 2 encourage its social, cultural, political Vallabhbhai Patel in the field of
(c) 2 1 3 4 and economic values. The World contribution to
(d) 2 3 1 4 Tourism Day celebrations have been
led by the United Nations World (a) unity and integrity of India
_ (b) Perfect competition is the Tourism Organisation (UNWTO) since (b) art and culture
situation prevailing in a market in 1970. (c) social work
which buyers and sellers are so
numerous and well informed that all The World Tourism Day is hosted by (d) entrepreneurship
elements of monopoly are absent and a different country every year. World
Tourism Day, 2019 was hosted by _ (a) On 20th September, 2019
the market price of a commodity is Government of India has instituted
beyond the control of individual India for the very first time at New
Sardar Vallabhbhai Patel Award. It is
buyers and sellers. Delhi. This year the World Tourism highest civilian award in the field of
Day theme was “Tourism and Jobs: contribution to the unity and integrity
A monopoly is a specific type of
A better future for all”. of India. The award seeks to
economic market structure. A
monopoly exists when a specific Djibouti and Addis Ababa hosted the recognise notable and inspiring
person or enterprise is the only World Tourism Day, 2020 on the contributions to promote the cause of
supplier of a particular good. As a theme “Tourism: Building Peace! national unity and integrity and to
result, monopolies are characterised Fostering Knowledge !” reinforce the value of a strong and
by a lack of competition within the United India. The award will be
119. BRICS Summit, 2020 will be hosted by announced on the occasion of the
market producing a good or service.
(a) India (b) China National Unity Day. i.e., the birth
Monopolistic competition is a type of anniversary of Sardar Patel on 31st
imperfect competition such that (c) Russia (d) Brazil
October.
CDS Solved Paper 2019 (II) 1

CDS
Combined Defence Service

SOLVED PAPER 2019 (II)


PAPER I Elementary Mathematics

Directions (Q. Nos. 1 and 2) Read 2. What is the mode of the frequency 3. Number of scooters of company Y
the following frequency distribution distribution of Series-II? sold by showroom E is what per
for two series of observations and (a) 26 (b) 36 (c) 46 (d) 56 cent of the number of scooters of
answer the given question in below. both companies sold by showroom
_ (c)
C?
Class Frequency Since, the highest frequency is 58.
interval Therefore, modal class is 40-50. (a) 52 (b) 54
Series-I Series-II (c) 55 (d) 56
∴ l = 40, h = 10, f1 = 58,
10 - 20 20 4
8 f0 = 4, f2 = 26 _ (c) Scooters of both companies sold
20 - 30 15 12
 f1 − f0  by showroom E = × 6400 = 768
30 - 40 10 4 ∴ Mode = l +   ×h 100
40 - 50 x 2x  2 f1 − f0 − f2 
Scooters of company X sold by
50 - 60 y y  58 − 4 
= 40 +   × 10 showroom E =
8
× 3000 = 240
Total 100 100  2 × 58 − 4 − 26  100
1. 54 Scooters of company Y sold by
What is the mean of frequency = 40 + × 10
distribution of Series-I? 86 E = 768 − 240 = 528
= 40 + 62
.
= 462
. ≈ 46 Scooters of both company is i.e., sold
(a) 33.6 (b) 35.6 (c) 37.6 (d) 39.6
by showroom
_ (c) First of all we have to find x and y C =
15
× 6400 = 960
from the given table Directions (Q. No. 3-6) Read the 100
20 + 15 + 10 + x + y = 100 following information and answer the 528
x + y = 55 …(i) ∴Required % = × 100 = 55%
given question in below. 960
4 + 8 + 4 + 2 x + y = 100
2 x + y = 84 …(ii)
Let the distribution of number of 4. Number of scooters of both the
On solving Eqs. (i) and (ii), we get
scooters of companies X and Y sold by 5 companies sold by showroom B is
showrooms (A, B, C, D and E) in a certain what percent more than the
x = 29, y = 26 year be denoted by S1 and the number of scooters of company X
Class interval Class mean xi distribution of number of scooters of only sold by showroom A?
10-20 15 company X sold by the five show-rooms in
2 1
the same year be denoted by S2. (a) 78 (b) 83
20-30 25 3 3
30-40 35 Showroom A B C D E Total 2 1
(c) 86 (d) 88
40-50 45 number of 3 3
50-60 55 scooters _ (c) Number of scooters of both the
sold companies X and Y sold by showroom
Mean of frequency distribution of B = 21% of 6400
series-I S1 (in %) 19 21 15 33 12 6400
21
Σf x 3760 S2 (in%) 24 18 20 30 8 3000 = 6400 × = 1344
∴ x = 1 i= = 37.6 100
Σf1 100
2 CDS Solved Paper 2019 (II)

Number of scooters of company X


sold by showroom A = 24% of 3000
Directions (Q. Nos. 7-10) Read the 11. Let a and b be two positive real
following information and answer the numbers such that a a +b b = 32
24
= 3000 × = 720
100
given question in below and a b + b a = 31. What is the
∴Required percent The data shows that Indian roads are 5 (a + b )
value of ?
1344 − 720 turning deadlier over the years. 7
= × 100
720 Year 2014 2015 2016 2017 (a) 5 (b) 7
624
= × 100 Number of 40957 46070 52750 48746 (c) 9
720 bikers killed (d) Cannot be determined
260 2
= = 86 % Number of 12330 15746 15746 20457
3 3 _ (a) Given, a a + b b = 32 …(i)
pedestrians
5. What is the average number of killed a b + b a = 31 …(ii)
scooters of company Y sold by the On squaring both sides Eqs. (i) and
Number of 4037 2585 2585 3559
(ii), we get
showrooms A, C and E? cyclists
1 1 killed ( a a + b b )2 = 32 2
(a) 461 (b) 431
3 3 [Q ( a + b )2 = a 2 + b 2 + 2 ab]
1 1 7. What was the average number of ∴ a + b 3 + 2 ab ab = 1024
3
…(iii)
(c) 426 (d) 416
3 3 pedestrians killed per day in the ( a b + b a )2 = 312
year 2017? a 2b + b 2a + 2 ab ab = 961
_ (a) Number of scooters of both the …(iv)
companies X and Y sold by (a) 51 (b) 53 (c) 54 (d) 56 On subtracting Eq. (iv) from Eq. (iii),
showrooms A, C and E
_ (d) Since, 2017 is not a leap year, total we get
= (19 + 15 + 12 )% of 6400 number of days in 2017 = 365 days. ( a 3 + b3 + 2 ab ab )
46
= 6400 × = 2944 ∴ Required average =
20457
= 56.05 − ( a 2b + b 2a + 2 ab ab )
100 365 = 1024 − 961
Number of scooters of company X ≈ 56 pedestrians. a 3 + b 3 − a 2b − b 2a = 63
sold by showrooms A, C and E
= (24 + 20 + 8)% of 3000 8. What is the approximate percentage a 3 − a 2b − b 2a + b 3 = 63
52 change in the pedestrains fatalities a 2( a − b ) − b 2( a − b ) = 63
= 3000 × = 1560
100 during the period 2014-17? ( a 2 − b 2 )( a − b ) = 63
∴ Number of scooters of company Y (a) 66% (b) 68% (c) 71% (d) 76% ( a + b )( a − b )( a − b ) = 63
sold by showrooms A, C and E
_ (a) Required percentage {Qa 2 − b 2 = ( a + b )( a − b )}
= 2944 − 1560 20457 − 12330 ( a + b ) ( a − b )2 = 63
= × 100
= 1384 12330 ( a + b ) and ( a − b )2 must be a
∴Required average 81270 co-prime number, by factorising 63 we
= = 65.91% ≈ 66%
1384 1 1233 get 7 and 9 that can satisfy the above
= = 461
3 3 equation.
9. What is the average number of Q a+ b=7
6. What is the difference between the bikers killed daily in road accidents
( a − b )2 = 9 ⇒ a − b = ± 3
number of scooters of both in the year 2017?
companies sold by showroom A If we take, a − b = − 3, we will get b
(a) 163 (b) 152 negative that can not be possible,
and total number of scooters of (c) 147 (d) 134 Q a+ b=7
company X sold by showrooms B
and E together? _ (d) Since, 2017 is not a leap year, total a−b=3
number of days in 2017 = 365 days
On solving this, we get
(a) 416 (b) 426 48746
(c) 432 (d) 436 ∴ Required average = a=5
365
⇒ b=2
_ (d) Number of scooters of both the = 133.55
5( a + b ) 5( 5 + 2 )
companies X and Y sold by showroom ≈ 134 bikers. Value of = =5
A = 19% of 6400 7 7
10. What is the average number of
= 6400 ×
19
= 1216 12. 1+ 3
100 cyclists killed daily in road If x = and y = x 3 , then y
accidents in 2017? 2
Total number of scooters of company
X sold by showroom B and E together (a) 10 (b) 12 (c) 19 (d) 21
satisfies which one of the following
= (18 + 8)% of 3000 equations?
_ (a) Since, 2017 is not a leap year, total
= 3000 ×
26
= 780 number of days in 2017 = 365 days (a) 8 y 2 − 20y − 1 = 0
100 3559 (b) 8 y 2 + 20y − 1 = 0
∴ Required average = = 975
. (c) 8 y 2 + 20y + 1 = 0
∴Required difference 365
= 1216 − 780 = 436 ≈ 10 cyclists. (d) 8 y 2 − 20y + 1 = 0
CDS Solved Paper 2019 (II) 3

_ (a) Given, y = x
3
…(i) _ (c)  10 
4

= 100 x   1 +  − 1
1+ 3 1. The last digit of a number depends on   100  
and x = …(ii) the unit digit, so 17 as 7 will show
2   11  4

same characteristic.
1+ 3 = 100 x    − 1
On putting x = in Eq. (i), we get We know, the cyclicity of 7 is 5.   10  
2 While solving this problem we will take
14641 − 10000 
= 100 x 
3
1 + 3  the power and divide by 5.
 
y=  10000
174 can be written as 5n + 4, where
 2 
n = 34 = 100x ×
4641
(1 + 3 )3 Hence, 17 174 = 17 ( 5 × 34 + 4 ) 10000
y=
23 Hence, 7 4 and 17 174 will have same ⇒ CI1 = 46.41 x
1 + 3 3 + 3 3(1 + 3 ) digit in the unit place, so answer is 1.
= In 2nd scheme
8 2. Let any two odd number be 5 and 3.
P = 100x , R = 20%, T = 2 yr
[Q( a + b )3 = a 3 + b 3 + 3ab( a + b )] According to the question,
 r 
T

1+ 3 3 + 3 3 + 9 ⇒ 5 2 − 3 2 = 25 − 9 = 16 CI 2 = P   1 +  − 1
y=  
8 16 which is divisible by 8.  100 
5+3 3 Let check once more with non  20 
2

y= = 100x   1 +
4 consecutive odd number be 7 and 3.  − 1
  
100 
4y = 5 + 3 3 …(iii) According to the question,
On squaring both sides in Eq. (iii), ⇒ 7 2 − 3 2 = 49 − 9 = 40  6  2

= 100x    − 1
we get 40 which is divisible by 8.   5  
( 4 y)2 = ( 5 + 3 3 )2 So, difference of the square of any two 36 − 25 
odd number is divisible by 8. = 100 x 
16 y 2 = 25 + 27 + 30 3  25 
3. Let two consecutive number be x and
[Q( a + b )2 = a 2 + b 2 + 2 ab] CI 2 = 44x
x + 2.
16 y = 52 + 30 3
2
According to the question, x ( x + 2 ) + 1 According to the question,
8 y 2 = 26 + 15 3 …(iv) = x 2 + 2 x + 1 = ( x + 1)2 Cl1 − Cl 2 = 482
On multiplying Eq. (iii) by 5, we get Thus, adding 1 to the product of two 46.41x − 44x = 482
20 y = 25 + 15 3 …(v) consecutive odd numbers makes it
2.41x = 482
perfect square.
Eq. (v) substract from Eq. (iv), 482
Hence, statements 2 and 3 are correct x =
⇒ 8 y 2 − 20 y = 1 and statement 1 is incorrect.
2.41
⇒ 8 y − 20 y − 1 = 0
2
⇒ x = 200
15. The rate of interest on two different ∴ Principal = 100 × 200
Option (a) will satisfy ‘ y’.
schemes is the same and it is 20%. = ` 20000
13. HCF of two numbers is 12. Which But in one of the schemes, the
one of the following can never be interest is compounded half yearly 16. For what value of k can the
their LCM? and in the other the interest is expression x 3 + kx 2 − 7 x + 6 be
(a) 80 (b) 60 (c) 36 (d) 24 compounded annually. Equal resolved into three linear factors?
_ (a) HCF of two numbers is 12. amounts are invested in the (a) 0 (b) 1 (c) 2 (d) 3
schemes. If the difference of the
LCM is common multiple for the two _ (a) For a particular value of k we must
numbers. returns after 2 yr is ` 482, then what get 3 different value of x.
So, the required answer is 80 because is the principal amount in each We can satisfy the option in this type of
all other are multiple of 12. scheme? question, and by solving we will get
(a) ` 10000 (b) ` 16000 that only option (a) will satisfy the
14. Consider the following statements equation.
174 (c) ` 20000 (d) ` 24000
1. Unit digit in 17 is 7. ⇒ x 3 + kx 2 − 7 x + 6 = 0
2. Difference of the squares of any _ (c) Let the principal be 100x. On putting k = 0,
In 1st scheme P = 100x ,
two odd numbers is always ⇒ x 3 + x 2 − x 2 − 7x + 6 = 0
20
divisible by 8. Rate (R) = % = 10%, ⇒ x 3 − x 2 + x 2 − x − 6x + 6 = 0
2
3. Adding 1 to the product of two ⇒ x 2( x − 1) + x ( x − 1) − 6( x − 1) = 0
Time (T ) = 4 half yearly.
consecutive odd numbers makes ⇒ ( x − 1)( x 2 + x − 6) = 0
  r 
T

it a perfect square. CI1 =  P  1 +  − P ⇒ ( x − 1)( x 2 + 3x − 2 x − 6) = 0
  
Which of the above statements
100  ⇒ ( x − 1)( x ( x + 3) − 2( x + 3)) = 0
are correct?  r 
T
 ⇒ ( x − 1)( x − 2 )( x + 3) = 0
= P 1 +  − 1
(a) 1, 2 and 3 (b) Only 1 and 2   100   We will get, x = 1, 2, and −3.
(c) Only 2 and 3 (d) Only 1 and 3
4 CDS Solved Paper 2019 (II)

17. X , Y and Z start at same point and On putting x = − 3 1800


x
⇒ x 2 + kx + 1 = 0 Net loss% = 91 × 100
same time in the same direction to run 20000x
⇒ ( −3)2 + k( −3) + 1 = 0
around a circular stadium. X 91
9 − 3k + 1 = 0 ⇒ 3k = 10
completes a round in 252 s,Y in 308 s = 9%
10
and Z in 198 s. After what time will ∴ k=
3 Alternate Method
they meet again at the starting point?
20. A lent ` 25000 to B and at the same When two article sell at same selling
(a) 26 min 18 s (b) 42 min 36 s price,one at a profit of a% another at a
(c) 45 min (d) 46 min 12 s time lent some amount to C at same loss of a%, then there is always a loss
7% simple interest. After 4 yr A  a
2
_ (d) X completes round in 252 s. received ` 11200 as interest from B of   %.
Y completes round in 308 s.  10 
and C. How much did A lend to C?
Z completes round in 198 s. Here, a = 30%
They all meet again at starting together (a) ` 20000 (b) ` 25000
Required loss percentage
after, LCM of 252, 308 and 198 (c) ` 15000 (d) ` 10000 2 2
252 = 2 × 2 × 3 × 3 × 7  a  30 
=   =   = 9%
308 = 2 × 2 × 7 × 11 _ (c) Let A lent x amount to C.  10   10 
198 = 2 × 3 × 3 × 11 According to the question,
∴ Required LCM 25000 × 7 × 4 x × 7 × 4
22. The monthly incomes of A and B are
⇒ + = 11200 in the ratio 4 : 3. Each save ` 600. If
= 2 × 2 × 3 × 3 × 7 × 11 100 100
their expenditures are in the ratio 3 :
= 2772 s = 46 min 12 s. QSI = P × R × T 
2, then what is the monthly income
18. What is the LCM of 1 , 5 , 2 , 4 ?  100 
of A?
28x
3 6 9 27 7000 + = 11200 (a) ` 1800 (b) ` 2000
5 1 100
(a) (b) (c) ` 2400 (d) ` 3600
28x = 420000
18 27
10 20 ∴ x = 15000 _ (c) Let monthly income of A and B be
(c) (d) 4x and 3x.
27 3 21. A trader sells two computers at the
1 5 2 4 According to the question,
_ (d) LCM of , , and
same price, making a profit of 30%
A’ s expenditure 3
3 6 9 7 on one and a loss of 30% on the =
LCM of numerator B’ s expenditure 2
⇒ other. What is the net loss or profit
HCF of denominator percentage on the transaction? [Q Expenditure = Income − Saving]

LCM of 1, 5, 2 and 4 20
= 4x − 600 3
(a) 6% loss (b) 6% gain =
HCF of 3, 6, 9, 27 3 3x − 600 2
(c) 9% loss (d) 9% gain
19. If the equations x 2 + 5x + 6 = 0 and 8x − 1200 = 9x − 1800
_ (c) Let the selling price of each
x 2 + kx + 1 = 0 have a common computers be 100x. x = 600

root, then what is the value of k ? Cost price of computer 1 A’s monthly income = 4 × 600
5 10 5 10  100  = ` 2400
(a) − or − (b) or = selling price ×  
2 3 2 3  100 + profit % 
5 10 5 10
23. The train fare and bus fare between
(c) or − (d) − or = 100x ×
100 1000 x
= two stations is in the ratio 3 : 4. If
2 3 2 3 130 13 the train fare increases by 20% and
_ (b) Given, equations Cost price of computer 2 bus fare increases by 30%, then
x 2 + 5x + 6 = 0 …(i)  100  what is the ratio between revised
= selling price ×  
x 2 + kx + 1 = 0 …(ii)  100 − loss %  train fare and revised bus fare?
If both equations have common roots 100 1000x 9 17
= 100x × = (a) (b)
then, 13 12
70 7
⇒ x 2 + 5x + 6 = 0 (c)
32
(d)
19
Total cost price of computers 1 and 2 43 21
⇒ x + 3x + 2 x + 6 = 0
2

1000x 1000x 20000x


⇒ x ( x + 3) + 2 ( x + 3) = 0 = + = _ (a) Let the fare of train and bus be 3x
13 7 91 and 4x.
⇒ ( x + 2 )( x + 3) = 0
⇒ x = − 2, − 3 Total selling price of both computers According to the question,
The value of x must satisfy the Eq. (ii) = 100x + 100x = 200x New fare of train = 3x ×
120
= 3.6 x
x = −2 100
On putting Net loss = Total CP − Total SP
130
⇒ x 2 + kx + 1 = 0 20000x New fare of bus = 4x × = 5.2 x
= − 200x 100
⇒ ( −2 )2 + k( −2 ) + 1 = 0 91
3.6 x 36 9
⇒ 4 − 2k + 1 = 0 20000 x − 18200x 1800 ∴ Required ratio = = =
= = x 52
. x 52 13
5
⇒ 2k = 5 ⇒ k = 91 91
2
CDS Solved Paper 2019 (II) 5

4 3
24. When N is divided by 17, the = 2 πr 3 − πr 29. Which one of the following is
3
quotient is equal to 182. The correct in respect of a right angled
6 πr 3 − 4 πr 3 2 πr 3
difference between the quotient and = = triangle?
the remainder is 175. What is the 3 3
(a) Its orthocentre lies inside the
value of N? Volume of cylinder ( V ) = 2 πr 3 triangle
V
(a) 2975 (b) 3094 (c) 3101(d) 3269 ∴ Remaining volume = (b) Its orthocentre lies outside the
3 triangle
_ (c) Let the remainder be x.
According to the question,
27. Three parallel lines x , y and z are (c) Its orthocentre lies on the triangle
cut by two transversals m and n. (d) It has no orthocentre
182 − x = 175 ⇒ x = 7
Transversal m cuts the lines x , y, z _ (c) Orthocentre of a right angle
∴Number
at P, Q , R, respectively and triangle lies on the triangle.
= Divisor × Quotient + Remainder
transversal n cuts the lines x , y, z at It is a property of a right angle triangle.
= 17 × 182 + 7
L, M, N , respectively. If PQ = 3 cm, 30. Let the bisector of the angle BAC of
= 3094 + 7 = 3101 QR = 9 cm and MN = 10.5 cm, then a triangle ABC meet BC in X .
25. A stock of food grains is enough for what is the length of LM ? Which one of the following is
240 men for 48 days. How long will (a) 3 cm (b) 3.5 cm correct?
the same stock last for 160 men? (c) 4 cm (d) 4.5 cm
(a) AB < BX (b) AB > BX
(a) 72 days (b) 64 days m n
(c) AX = CX
_ (b) (d) None of these
(c) 60 days (d) 54 days A
_ (b)
_ (a) Let total stock of food grains be 1 a P L c
x
unit.
240 men eat 1 units food grains in
y
b Q M d
= 48 days
1 man eat 1 unit food grains in z
R N
= 48 × 240 days
B C
(By unitary method) X
x || y || z ∠BXA is exterior angle of ∆AXC
∴160 men eat 1 unit food grains in
Then, by Basic proportionality ∠BXA = ∠XAC + ∠ACX
48 × 240 theorem,
= ∴∠BXA > ∠XAC ( ∠XAC = ∠BAX )
160 a c a b
= or =
= 72 days b d c d ∠BXA > ∠BAC
Here, a = PQ = 3 cm ∴ AB > BX (side opposite to greater
26. A hollow right circular cylindrical b = QR = 9 cm angle is greater)
vessel of volume V whose diameter c = LM = ?
is equal to its height, is completely d = MN = 10.5
filled with water. A heavy sphere of a c 31. What is the value of
⇒ = log10 (cos θ) + log10 (sin θ)
maximum possible volume is then b d
3 c + log10 (tan θ) + log10 (cot θ)
completely immersed in the vessel. ⇒ =
What volume of water remains in 9 10.5 + log10 ( secθ) + log10 ( cosecθ)?
the vessel? c = 3.5 = LM (a) −1 (b) 0 (c) 0.5 (d) 1
V V LM = 3.5 cm
(a) (b) _ (b) log10(cos θ) + log10(sinθ)
2 3 28. The area of a sector of a circle of + log10 (tanθ)
2V V
(c) (d) radius 4 cm is 25.6 cm 2 . What is the + log10(cot θ) + log10(sec θ)
3 4 radian measure of the arc of the + log10(cosec θ)
_ (b) Let the radius of cylinder = r sector? ⇒ log10(cos θ ⋅ sinθ)
Height of cylinder ( h) = 2 r (a) 2.3 (b) 3.2 (c) 3.3 (d) 3.4 + log10(tanθ ⋅ cot θ)
_ (b) Area of a sector = 25.6 cm
2
Maximum possible sphere can be + log10(sec θ ⋅ cosec θ)
immersed of radius r.
A×2 [Q log a b + log a c = log a b ⋅ c ]
Volume of cylinder ( V ) = πr 2h Length of arc ( l ) =
r ⇒ log10(cos θ ⋅ sinθ)
= πr 2(2 r ) = 2 πr 3 25.6 × 2
l= ⇒ l = 12.8, θ =
l + log10(sec θ ⋅ cosec θ) + log10(1)
4
Volume of sphere = πr 3 4 r ⇒log10 (cos θ ⋅ sinθ ⋅ sec θ ⋅ cosec θ) + 0
3 θ = angle in radian [Q log10 1 = 0]
After immersing sphere in the cylinder 12.8 128 16
the remain water ∴ θ= = = = 3.2 ⇒ log10(1) ⇒ 0
4 40 5
6 CDS Solved Paper 2019 (II)

32. If cos 2 x + cos x = 1, then what is the  4 16  36. The perimeter of a triangle is 22 cm.
⇒ 1 + 3 × + 9 × 
 3 9
value ofsin 12
x + 3 sin 10
x + 3 sin x 8
 64 
Through each vertex of the
 1 + 27 ×  triangle, a straight line parallel to
+ sin 6 x?  27 
the opposite side is drawn. What is
⇒ (1 + 4 + 16)(1 + 64)
(a) 1 (b) 2 (c) 4 (d) 8 the perimeter of triangle formed by
= 21 × 65 = 1365
_ (a) Given, cos x + cos x = 1
2 these lines?
Option (d) is correct.
cos x = 1 − cos 2 x (a) 33 cm (b) 44 cm(c) 66 cm
cos x = sin x 2 34. The angles of elevation of the tops (d) 88 cm
of two pillars of heights h and 2h (b) A
⇒ sin12 x + 3 sin10 x + 3 sin8 x D F
from a point P on the line joining
+ sin x 6
the feet of the two pillars are
⇒ (sin4 x )3 + 3 (sin4 )2 sin2 x
complementary. If the distances of B C
+ 3(sin2 x )2 ⋅ sin4 x + (sin2 x )3 the foot of the pillars from the point
[Q ( a + b )3 = a 3 + b 3 + 3a 2b + 3b 2a )] P are x and y respectively, then
⇒ (sin4 x + sin2 x )3
which one of the following is
[Q sin2 x = cos x , sin4 x = cos 2 x ] correct?
E
⇒ (cos x + cos x )
2 3
As, AC || BE and AB|| CE, ABEC
(a) 2h 2 = x 2 y (b) 2h 2 = x y 2
[given, cos 2 x + cos x = 1] forms a parallelogram.
(c) 2h 2 = x y (d) 2h 2 = x 2 y 2
⇒ (1)3 = 1 where, AC = BE and AB = CE
_ (c) Similarly,
Option (a) is correct.
As, AC || DB and BC || AD, ADBC
33. If 0 < θ < 90°, sin θ = 3/5 and forms a second parallelogram.
x = cot θ, then what is the value of where, AC = BD and AD = BC
1 + 3x + 9 x 2 + 27 x 3 2h With this we can conclude that B is
+ 81x 4 + 243x 5 ? h
mid-point of DE and AC is half of DE.

(a) 941 (b) 1000 (c) 1220(d) 1365 Similarly, it will be true for FD and EF
θ 90–θ also.
3
_ (d) Given, sinθ = x P y The ratio of the perimeter of new
5
According to the question, triangle to that original triangle is 2 : 1.
A
tanθ =
h
…(i) ∴ Required perimeter
x 2
2h = 22 × = 44 cm
tan( 90 − θ) = 1
3 5 y
2h 37. The sides AD, BC of a trapezium
cot θ = …(ii)
y ABCD are parallel and the
[Q tan ( 90° − θ) = cot θ] diagonals AC and BD meet at O. If
B 4 C
On multiplying Eqs. (i) and (ii), we get the area of triangle AOB is 3 cm 2
In a right angle triangle, h 2h and the area of triangle BDC is 8 cm
Perpendicular P 3 tanθ ⋅ cot θ = × 2
sinθ = , = x y , then what is the area of triangle
Hypotenuse H 5 AOD?
2 h2
Using Pythagoras theorem, 1= [tanθ.cot θ = 1]
xy (a) 8 cm 2 (b) 5 cm 2
AC = AB + BC , 5 = 3 + BC
2 2 2 2 2 2
(c) 3.6 cm 2 (d) 8 cm 2
∴ 2 h2 = x y
⇒ BC 2 = 5 2 − 3 2 ⇒ BC = 4 A D
35. What is the value of _ (d)
Base BC
cot θ = =
sin 19 ° cos 73°
Perpendicular AB + ? O
cot θ =
4
⇒ x =
4
[given, cot θ = x]
cos 71° sin 17 °
3 3 (a) 0 (b) 1 (c) 2 (d) 4
⇒ 1 + 3x + 9x + 27 x 3
2 B C
sin19° cos 73°
_ (c) Given, + ∆ABC and∆BDC
+ 81x 4 + 243x 5 cos 71° sin17 °
Lie on the same base BC and between
⇒ 1 + 3x + 9x 2 + 27 x 3 sin19° cos 73°
= + same parallel AD and BC.
(1 + 3x + 9x 2 ) cos( 90 − 19° ) sin( 90 − 73° )
∴ar( ∆ABC ) = ar( BDC )
⇒ (1 + 3x + 9x )(1 + 27 x 3 )
2
sin19° cos 73°
= + = 1 + 1= 2 Subtracting ar( ∆BOC ) both sides, we
Now, on putting x =
4 sin19° cos 73° get ar( ∆ABC ) − ar( ∆BOC )
3 Q sin( 90° − θ) = cos θ,
 cos( 90° − θ) = sinθ  ar( BDC ) − ar( BOC ),
 4  4  
2
 4 
3
 
1 + 3 × + 9 ×   1 + 27 ×    ar( ∆AOB) = ar( DOC )
 3  3    3 
  Option (c) is correct. Then, ar( ∆DOC ) = 3 cm 2
CDS Solved Paper 2019 (II) 7

So, ar ( ∆BOC ) = ar( BDC ) − ar( ∆DOC ) PQR is 20 cm, then what is the _ (b) Area of three faces of cuboid are
= 8 cm 2− 3cm 2 = 5 cm 2 length of corresponding side of the x , y and z. Let l, b and h be the length,
triangle ABC? breadth and height of cuboid.
In ∆ABO and ∆BOC,
According to the question,
1 (a) 25 cm (b) 30 cm
× AO × height x =l×b
ar ( ∆ABO ) 2 (c) 40 cm (d) 45 cm
=
ar ( ∆BOC ) 1 × CO × height y=b×h
_ (b) ∆ABC and ∆PQR are two similar
2 triangles. z=h×l
3 AO
= AB BC AC xyz = l × b × b × h × h × l
5 CO Then, = = =k
PQ QR PR = l 2 b 2 h2 = ( lbh)2
∆AOD ~ ∆BOC { AAA} ∴ AB = kPQ, BC = kQR, AC = kPR ∴xyz = V 2 (Q V = l × b × h)
2
ar( ∆AOD )  AO 
=  42. If l is the length of the median of an
ar( BOC )  CO  AB + BC + AC = K ( PQ + QR + PR )
equilateral triangle, then what is its
2 75 = 50 × k
 3 area?
=  75
 5 k= 3l2 3l2
50 (a) (b) (c) 3 l 2 (d) 2l 2
9 3 2
= ⇒ k=
3
25 2 _ (a) Median of a equilateral triangle
9
ar ( ∆AOD ) = × 5 = 18
. cm 2 AB 3
= 3
25 = a
PQ 2 2
38. A line segment AB is the diameter 3 3 3 2l
AB = × PQ = × 20 l= a ⇒ a=
of a circle with centre at O having 2 2 2 3
radius 6.5 cm. Point P is in the plane ∴ AB = 30 cm 3 2
Area of equilateral triangle = a
of the circle such that AP = x and 40. Let PQRS be a parallelogram whose 4
BP = y. In which one of the diagonals PR and QS intersect at O. 3  2l 
2
= × 
following cases the point P does not If triangle QRS is an equilateral 4  3
lie on the circle? triangle having a side oflength 10 3 4l 2 3 l2
= × =
(a) x = 65
. cm and y = 65
. cm cm, then what is the length of the 4 3 3
(b) x = 12 cm and y = 5 cm diagonal PR?
(c) x = 5 cm and y = 12 cm
43. A piece of wire is in the form of a
(a) 5 3 cm (b) 10 3 cm sector of a circle of radius 20 cm,
(d) x = 0 cm and y = 13 cm
(c) 15 3 cm (d) 20 3 cm subtending an angle 150° at the
P
_ (a) _ (b) S R centre. If it is bent in the form of a
x y circle, then what will be its radius?
19
(a) cm (b) 7 cm
A B O 3
6.5 O 6.5
(c) 8 cm (d) None of these
P Q _ (d) Radius = 20 cm
In triangle QRS, each angle is 60°. AO θ
Circle with centre O having AB as a Length of arc ( l ) = × 2 πr
diameter. is altitude of triangle QRS. Altitude in 360°
equilateral triangle =
3 150°
We know that, diameter create 90° a = × 2 × π × 20
angle on the circumference of the circle.
2 360°
3 50
∴ x 2 + y 2 = 13 2 Altitude = × 10 [Q a = 10 cm] = π
2 3
x + y = 169
2 2
= 5 3 cm Perimeter of sector
Now, by putting option we can get the
required answer, ∆QSR is similar to ∆PQS and both are = Length of are + 2 r
equilateral triangle with side 10 cm. 50
On putting option (a) = π + 40
3
x = 6.5, y = 6.5 Then, diagonal PR = 2 × Altitude
⇒ ( 6.5)2 + ( 6.5)2 = 169 According to the question,
=2 × 5 3
⇒ 42.25 + 42.25 = 169 2 πr =
50
π + 40
⇒ 84.50 ≠ 169 = 10 3 cm 3
41. The areas of three adjacent faces of 25 20
So, option (a) cannot be possible. ⇒ r= +
a cuboid are x , y and z. If V is the 3 π
39. The perimeters of two similar 25 70
triangles ABC and PQR are 75 cm volume of the cuboid, then which ⇒ r= +
3 11
and 50 cm, respectively. If the one of the following is correct?
485
length of one side of the triangle (a)V = xyz (b)V 2 = xyz ∴ r= = 14.7 [approx.]
33
(c)V 3 = xyz (d)V = (xyz)2
8 CDS Solved Paper 2019 (II)

(b) A
44. Suppose P, Q and R are the 46. An equilateral triangle and a square
mid-points of sides of a triangle of are constructed using metallic
area 128 cm 2 . If a triangle ABC is wires of equal length. What is the
5 D
drawn by joining the mid-points of ratio of area of triangle to that of
sides of triangle PQR, then what is square?
the area of triangle ABC? (a) 3 : 4 (b) 2 : 3
(a) 4 cm 2 (b) 8 cm 2 (c) 4 3 : 9 (d) 2 3 : 9 B C
10
(c) 16 cm 2 (d) 32 cm 2
_ (c) Let side of triangle be ‘x’ unit. In ∆ABC, AB = 5 cm, BC = 10 cm
_ (b) X Let the side of square be ‘a’ unit. Using Pythagoras theorem, we get
Perimeter of square = 4 × a = 4a unit AB2 + BC 2 = AC 2, 5 2 + 10 2 = AC 2
According to the question, AC = 5 5
P C Q Perimeter of triangle = Perimeter of AB × BC 5 × 10
BD = =
square AC 5 5
A B 3x = 4a 10 10 5
Y 4a BD = = × ∴BD = 2 5 cm
R Z x = 5 5 5
3
We know, by mid-point theorem, in 49. Two cylinders of equal volume
Area of triangle
∆XYZ and ∆PQR Required ratio =
Area of square have their heights in the ratio
1
ar( ∆XYZ ) = ar ( ∆PQR ) 2 : 3. What is the ratio of their radii?
4 3 2
x
1 (a) 3 : 1 (b) 3 : 2
Similarly, ar ( ∆PQR ) = ar ( ∆ABC ) = 4 2
4 a (c) 2 : 3 (d) 3 : 2
1 3 4a 4a
ar ( ∆XYZ ) = ar( ∆ABC ) × × _ (b) Let radius of the cylinders be r1 and
16 = 4 3 3
r2 and their heights be 2 x and 3x,
ar( ∆ABC ) =
128
cm 2 a2 respectively.
16 4 3
∴ ar( ∆ABC ) = 8 cm 2 = According to the question,
9
π r12 × 2 x = πr22 × 3x
45. Let two lines p and a be parallel. ∴4 3:9
Consider two points B and C on the [Q Volume = πr 2h]
47. All the four sides of a parallelogram
line p and two points D and E on r12 3
are of equal length. The diagonals ⇒ =
the line q. The line through B and E r22 2
are in the ratio 1 : 2. If the sum of
intersects the line through C and D
at A in between the two lines p and the lengths of the diagonals is 12 r1 3
⇒ =
q. If AC : AD = 4 : 9, then what is cm, then what is the area of the r2 2
the ratio of area of triangle ABC to parallelogram?
∴ r1 : r2 = 3: 2
that of traingle ADE? (a) 9 cm 2 (b) 12 cm 2
(c) 16 cm 2 (d) 25 cm 2 50. The length and breadth of a
(a) 2 :.3 (b) 4 : 9
rectangle are increased by 20% and
(c) 16 : 81 (d) 1: 2 _ (c) If all the sides of a parallelogram is 10%, respectively. What is the
equal that mean its a rhombus.
_ (c) B C percentage increase in the area of
p Let diagonal of rhombus are x and2 x.
the rectangle?
According to the question,
A x + 2 x = 12 (a) 32% (b) 30% (c) 25% (d) 15%
3x = 12 _ (a) Let length and breadth of rectangle
q x =4 be x and y.
D E 1 According to the question,
∴Area of rhombus = × d 1 × d 2
2 120
In ∆ABC and ∆ADE, New length = × x = 12. x
1 100
∠BAC = ∠DAE [opposite angle] = ×4×8
2 110
New breadth = × y = 11. y
∠ACB = ∠ADE = 16 cm 2 100
[alternate interior angle] Area of rectangle = x × y
48. ABC is a triangle right angled at B.
∠ABC = ∠AED Area of new rectangle = 1.2 x × 1.1 y
If AB = 5 cm and BC = 10 cm, then
[alternate interior angle]
what is the length of the = 1.32 x y
Q ∆ABC ≈ ∆AED perpendicular drawn from the ∴Required percentage
Ar( ∆ABC ) AC 2 4 2 vertex B to the hypotenuse? 1.32 xy − x y
= = = × 100
Ar( ∆AED ) AD 2 9 2 xy
(a) 4 cm (b) 2 5 cm
Ar( ∆ABC ) 16 0.32 x y
∴ =
4 = × 100 = 32%
(c) cm (d) 8 cm
Ar( ∆AED ) 81 5
xy
CDS Solved Paper 2019 (II) 9

Alternate method 53. A right circular cylinder has a In fig. (ii),


Here, a = 20%, b = 10% diameter of 20 cm and its curved Using Pythagoras theorem in ∆ORS,
Required percentage increase in the surface area is 1000 cm 2 . What is (OS )2 = (OR )2 + ( RS )2
area of rectangle the volume of the cylinder? a2 5a 2
a × b r 2 = a2 + ⇒ r2 =

= a + b + % (a) 4000 cm 3 (b) 4500 cm 3 4 4
 100 
(c) 5000 cm 3 (d) 5200 cm 3 4r 2 2r
20 × 10  a2 = ⇒ a=

=  20 + 10 + % Diameter 5 5
 100  _ (c) Radius of circle = 2
2  2r 
= ( 30 + 2 )% = 32% 20
= 10cm =  
 5
2 ∴ Required Ratio =
51. If the length of the hypotenuse of a Curved surface area = 1000 cm 2 ( r 2 )2
right angled triangle is 10 cm, then [Q area of square = (side) 2]
(Q h = height of cylinder)
what is the maximum area of such a 4r 2
⇒ πrh = 500 cm 2
right angled triangle? = 25
Volume of cylinder = πr 2h
(a) 100 cm 2 (b) 50 cm 2 r ×2
= πrh × r = 500 × 10 2
(c) 25 cm 2
(d) 10 cm 2 = ⇒2 :5
= 5000 cm 3 5
_ (c) (10) = x = y
2 2 2
54. A piece of wire of length 33 cm is 56. The quotient when x 4 − x 2 + 7 x + 5
For maximum value, x = y, bent into an arc of a circle of radius
Then, (10)2 = x 2 + x 2
is divided by ( x + 2) is
14 cm. What is the angle subtended
⇒ 2 x 2 = 100
ax 3 + bx 2 + cx + d . What are the
by the arc at the centre of the circle?
x = 5 2 cm values of a, b, c and d, respectively?
(a) 75° (b) 90° (c) 135° (d) 150°
1 (a) 1, − 2, 3, 1 (b) −1, 2, 3, 1
∴ Maximum area possible = ×x× y _ (c) Length of arc ( l ) = 33 cm
2 (c) 1, − 2, − 3, − 1 (d) −1, 2, − 3, − 1
Radius ( r ) = 14 cm
5 2 ×5 2 θ _ (a) Lets divide x − x + 7 x + 5 with
4 2
= l= × 2 πr
2 360° long division method,
= 25 cm 2 θ 22 x + 2 ) x 4 − x 2 + 7 x + 5 ( x 3 − 2 x 2 + 3x + 1
33 = ×2 × × 14
52. A square is drawn such that its 360° 7 x 4 + 2x 3
vertices are lying on a circle of ∴ θ = 135° − −

radius 201 mm. What is the ratio of 55. What is the ratio of the area of a − 2x 3 − x 2
area of circle to that of square? square inscribed in a semicircle of − 2 x 3 − 4x 2
+ +
(a) 11 : 7 (b) 7 : 11 radius r to the area of square 
3x 2 + 7 x
(c) 20 : 19 (d) 19 : 20 inscribed in a circle of radius r ? 3x 2 + 6x
(a) (a) 1 : 2 (b) 2 : 5 (c) 2 : 3 (d) 3 : 5 − −

D C x + 5
_ (b) x +2
A D − −

O 3
2r According to the question,
A B ax 3 + bx 2 + cx + d
= x 3 − 2 x 2 + 3x + 1
Let side fo square be x cm. B C By comparing it, we will get
Diagonal of square (i) a = 1, b = − 2, c = 3 and d = 1
= 201 × 2 = 402 cm P S
57. The sides of a triangle are 30 cm, 28
Using Pythagoras theorem,
r a cm and 16 cm, respectively. In order
x 2 + x 2 = ( 402 )2 to determine its area, the logarithm of
⇒ 2 x 2 = 402 × 402 O a R which of the quantities are required?
Q
x 2 = 201 × 402 2
(ii) (a) 37, 11, 28, 16 (b) 21, 30, 28, 7
⇒ x = 201 2 (c) 37, 21, 11, 9 (d) 37, 21, 9, 7
Let r be the radius of circle and side of
πr 2
∴ Required ratio = square ‘a’. In fig. (i), _ (d) Sides of triangle are 30, 28 and 16
x2 Diagonal of square = 2 r cm
22
× 201 × 201 Using Pythagoras theorem in ∆BCD, 30 + 28 + 16
Semi perimeter = = 37
= 7 2
201 2 × 201 2 BC + CD = BD
2 2 2

Area
11 a 2 + a 2 = (2 r )2 ⇒ 2 a 2 = 4r 2
= = 11 : 7 ( A) = 37( 37 − 30)( 37 − 28)( 37 − 16)
7 a=r 2
10 CDS Solved Paper 2019 (II)

A= 37 × 7 × 9 × 21cm2 60. Let XYZ be an equilateral triangle = 2 × [( 61764


. + 0.4771) − 4]
On taking log both sides, we get in which XY = 7 cm. If A denotes = 2 × ( 6.6535 − 4]
log A = log( 37 × 7 × 9 × 21)1/ 2 the area of the triangle, then what is = 2 × (2.6535]
1 the value of log10 A 4 ? (given that = 5.3070
= log( 37 × 7 × 9 × 21)
2 Option (a) is correct.
[Q log a b = b log a ]
log10 1050 = 30212
. and
1
= [log 37 + log 7 + log 9 + log 21] log10 35 = 15441
. ) 61. A hollow sphere of external and
2 (a) 5.3070 (b) 5.3700 internal diameters 6 cm and
[Q log( a × b ) = log a + log b ] 4 cm, respectively is melted into a
(c) 5.5635 (d) 5.6535
Logarithmic of 37, 7, 9 and 21 is cone of base diameter 8 cm. What is
required. _ (a) Given, XYZ is an equilateral triangle the height of the cone?
and XY = 7 cm
58. If log10 1995 = 33000
. , then what is X (a) 4.75 cm (b) 5.50 cm
1
(c) 6.25 cm (d) 6.75 cm
the value of (0001995
. )8 ?
1 1 _ (a) The external radius of hollow
(a) (b) sphere
100. 3475 100. 3375 Diameter 6
1 1 Y Z ( r1 ) = = = 3 cm
(c) (d) 2 2
100. 3275 100. 3735 Area ( A ) of equilateral triangle
The internal radius of hollow sphere
3
= ( XY )2
_ (b) Let x = ( 0.001995)
1/ 8
4
4 r2 = = 2 cm
On taking log10 both sides, we get 2
3 2 3
A= (7 ) = × 49 cm 2 …(i) 4
log10 x = log10 ( 0.001995)1/ 8 4 4 Volume of hollow sphere = π( r13 − r23 )
3
1 = log10 A 4 = log10( A 2 )2 = 2 log10 A 2
= log10( 0.001995) 4 19
8 2 = π ( 33 − 2 3 ) = × 4π
 49 3  3 3
[Q log a b = b log a ] = 2 log10   by Eq. (i)
 4  According to the question, volume of
log10 
1 1995 
= hollow sphere = Volume of cone
8  1000000   49 × 49 × 3 
= 2 log10   19 1
4×4 × 4 π = π ( 4)2 × h
1
= [log10 1995 − log10 10 6 ]   3 3
8 = 2[log10( 49 × 49 × 3) − log10 16]
(h = height of cone, radius of cone
1
= [log10 1995 − 6 log10 10] Q log a = log a − log b 
  = 4 cm)
8 b
19
1 1 1 = 2[log10(7 × 3) − log10 2 ]
4 4 ∴ h= = 475
. cm
= [3 ⋅ 3 − 6] = [− 2.7 ] = − [2.7 ] 4
8 8 8 = 2[log10 7 4 + log10 3 − 4 log10 2 ]
= − 0.3375 [Qlog( a × b ) = log a + log b]
62. A solid metallic cylinder of height
10 cm and radius 6 cm is melted to
On taking antilog both sides, we get 2 × 5
= 2  4 log10 7 + log10 3 − 4 log10 make two cones in the ratio of
 5 
1
x = volume 1 : 2 and of same height as
10 0. 3375 [Q log a b = b log a] 10 cm. What is the percentage
59. What is ( x − a )( x − b )( x − c ) equal [5 divided and multiply in last number] increase in the flat surface area?
to ? = 2[4 log10 7 + log10 3 − 4(log10 10 (a) 25% (b) 50% (c) 75% (d) 100%
− log10 5)]
(a) x − (a + b + c) x + (bc + ca
3 2
_ (b) Radius of cylinder ( r ) = 6 cm
= 2[4 log10 7 + 4 log10 5 + log10 3
+ ab)x − abc Height of cylinder ( h) = 10 cm
− 4 log10 10]
(b) x 3 + (a + b + c) x 2 + (bc + ca Let volume of cone be v1 and v 2 .
= 2[4 log10 35 + log10 3 − 4 log10 10]
+ ab)x + abc Ratio of volume of two cones
= 2[4 × log10 35 + log10 3 − 4] …(ii)
(c) x − (bc + ca + ab) x 2 + (a + b
3
v1 : v 2 is 1 : 2
[Qlog10 10 = 1]
+ c)x − abc If v1 is x , v 2 will be 2x.
⇒ log10 1050 = 3.0212
(d) x 3 + (bc + ca + ab) x 2 Hence, volume of cylinder
log10(105 × 10) = 3.0212
− (a + b + c)x − abc = v1 + v 2 = 3x
log10 10 + log10( 35 × 3) = 3.0212
_ (a) Let y = ( x − a)( x − b )( x − c ) Volume of cylinder = πr 2h
log10 10 + log10 35 + log10 3 = 3.0212 22
= ( x 2 − ax − bx + ab )( x − c ) = × 6 × 6 × 10
log10 3 = 3.0212 − log10 35 − log10 10 7
= x 3 − ax 2 − bx 2 + abx − cx 2 + acx
+ bcx − abc = 3.0212 − 15441
. − 1 = 0.4771 …(iii) 7920
= cm 3
= x 3 − x 2( a + b + c ) On putting the value of log10 3 in Eq. 7
(ii), we get
+ x ( ab + bc + ac ) − abc Surface area of flat surface of cylinder
= 2[4 × 15441
. + 0.4771 − 4] = 2 πr 2 = 2 × 6 × 6 × π = 72 π
After solving this we get option (a) is
correct. [Q log10 35 = 15441
. ]
CDS Solved Paper 2019 (II) 11

Now, this cylinder is recast into 2 64. A thin rod of length 24 feet is cut 66. Three copper spheres of radii
cones in the ratio 1 : 2.
into rods of equal size and joined, so 3 cm, 4 cm and 5 cm are melted to
So, volume of cone ( v1 ) = volume of as to form a skeleton cube. What is form a large sphere. What is its
x x 7920
cylinder × = × the area of one of the faces of the radius?
x + 2 x 3x 7
2640 largest cube thus constructed? (a) 12 cm (b) 10 cm(c) 8 cm(d) 6 cm
= cm 3
(a) 25 sq feet (b) 24 sq feet
7 _ (d) Sphere of radii 3 cm, 4 cm and 5
2 x 7920 (c) 9 sq feet (d) 4 sq feet cm are melted to form a large sphere
Volume of cone (V2) = ×
of radius r.
3x 7
_ (d) Length of rod = 24 feet
5280 According to the question,
= cm 3 Let the side of cube = x feet
7 4 4 4 4
Cube consist 12 edges, π 3 3 + π 4 3 + π 5 3 = πr 3
Let the radii of the 2 cones be R1 and 3 3 3 3
so, 12 x = 24 feet
R 2, respectively. ⇒ 33 + 43 + 53 = r 3
x = 2 feet
The height of both cones is 10 cm. ⇒ 27 + 64 + 125 = r 3
2640 Edge of the largest cube that can be
πR12h = constructed of 24 feet rod is 2 feet. ⇒ r 3 = 216
7 ⇒ r = 6 cm
5280 Now, the area of one of the face
and πR 2 h =
2
= 2 × 2 = 4 sq feet. 67. The volume of a hemisphere is
7
264 65. Consider a trapezium ABCD, in 155232 cm 3 . What is the radius of
πR1 =
2
7 which AB is parallel to CD and AD the hemisphere?
and πR 2 =
2 528
[Q h = 10]
is perpendicular to AB. If the (a) 40 cm (b) 42 cm
7 trapezium has an incircle which (c) 38 cm (d) 36 cm
R12 =
84
and R 22 =
168 touches AB at E and CD at F, where
_ (b) Let the radius of hemisphere be
7 7 EB = 25 cm and FC = 16 cm, then ‘r’ cm.
π R12 = 12 π and πR 22 = 24 π what is the diameter of the circle? According to the question,
Combined surface area of flat surface (a) 16 cm (b) 25 cm 2 3
πr = 155232
of both cones (c) 36 cm (d) 40 cm 3
= 12 π + 24 π = 36 π. 2 22
Change in flat surface area _ (d) ⇒ × × r 3 = 155232
3 7
72 π − 36 π A E G B
= × 100 ⇒ r 3 = 74088
72 π
⇒ r = 42 cm
36 π
= × 100 = 50%
72 π r T 68. A bucket is in the form of a
truncated cone. The diameters of
63. If one side of a right-angled triangle
the base and top of the bucket are 6
(with all sides integers) is 15 cm, C
D F
cm and 12 cm, respectively. If the
then what is the maximum
height of the bucket is 7 cm, what is
perimeter of the triangle? ABCD, is trapezium in which AB is the capacity of the bucket?
(a) 240 cm (b) 225 cm parallel to CD. Let BC touches circle
at T. (a) 535 cm 3 (b) 462 cm 3
(c) 113 cm (d) 112 cm
FC = CT = 16 cm (c) 234 cm 3 (d) 166 cm 3
_ (a) Let 2 other sides be x and y. r1
EB = BT = 25 cm [tangent of _ (b) Given,
So, x = y + 15
2 2 2
circle]
[for maximum perimeter]
BC = CT + BT = 16 + 25 = 41 h
x 2 − y 2 = 15 2
Let radius of the circle is r,
( x − y)( x + y) = 225
∆CGB, is right angled triangle,
To make perimeter maximum we need r2
to minimize the difference between From Pythagoras theorem,
sides. CB = CG + BG
2 2 2 Here, r1 = 3 cm, r2 = 6 cm, h = 7 cm

So, on comparing Here, CB = 2 r, BC = 41 Volume of bucket


1
x + y = 225 …(i) BG = BE − FC = 25 − 16 = 9 = πh( r12 + r22 + r1r2 )
3
x − y = 1 …(ii) 412 = (2 r )2 + 9 2 1 22
By solving Eqs. (i) and (ii), we get = × × 7 ( 3 2 + 6 2 + 3 × 6)
(2 r )2 = 412 − 9 2 3 7
x = 113, y = 112 4r 2 = 50 × 32 1 22
= × × 7 × 63
∴ Maximum perimeter = x + y + 15 ∴ r = 20 cm 3 7
= 113 + 112 + 15 Diameter of the circle = 2 × 20 = 40 = 22 × 21 = 462 cm 3
= 240 cm cm
12 CDS Solved Paper 2019 (II)

69. A right circular cone has height 8 1 26 x 2 + 1 26 According to the question,


x + = ⇒ =
cm. If the radius of its base is 6 cm, x 5 x 5 x − 1 = 2( y − 1) ⇒ x − 1 = 2 y − 2
then what is its total surface area? ⇒ 5( x 2 + 1) = 26x ⇒ 2y − x = 1 …(ii)
(a) 96π cm 2
(b) 69π cm 2 ⇒ 5x 2 + 5 − 26x = 0 On solving Eqs. (i) and (ii), we get
(c) 54 π cm 2 (d) 48 π cm 2 ⇒ 5x 2 − 25x − x + 5 = 0 y = 9, x = 17
⇒ 5x ( x − 5) − 1( x − 5) = 0 ∴Required difference = 17 − 9 = 8 yr
_ (a) Given,
⇒ ( 5x − 1)( x − 5) = 0
A
1
74. A person carries ` 500 and wants to
∴ x = 5, x = buy apples and oranges out of it. If
5
the cost of one apple is ` 5 and the
72. Consider the following statements : cost of one orange is ` 7, then what
l
h 1. If p is relatively prime to each of is the number of ways in which a
q and r, then p is relatively prime person can buy both apples and
oranges using total amount?
to the product qr.
B C (a) 10 (b) 14 (c) 15 (d) 17
r 2. If p divides the product qr and if
Here, r = 6 cm, h = 8 cm p dividesq, then p must divider. _ (b) Let there are x number of apples
and y number of oranges
∆ABC is right angled triangle, Which of the above statements So, 5x + 7 y = 500
Using Pythagoras theorem, is/are correct? Now, we have to check for x = 1, 2 , …
r 2 + h2 = l 2 (a) Only 1 (b) Only 2
When, x =2
62 + 82 = l 2 (c) Both 1 and 2 (d) Neither 1 nor 2
10 + 7 × y = 500, y = 70
36 + 64 = l 2 _ (a) When, x = 9, 45 + 7 y = 500, y = 65
l = 100 = 10
2 1. Let p = 4, q = 3, r = 5
Similarly, we can go ahead and find all
According to the question, Here, p is relatively prime to q and r.
the ways or we can check at which
The product of qr = 3 × 5 = 15 value of y oranges will be minimum.
Total surface area = πr 2 + πrl
In this case also p is relatively prime to For y = 5, x (s) will be minimum
= πr( r + l ) product of qr.
= π × 6( 6 + 10) 5x + 35 = 900, x = 93
2. Let p = 4, q = 8 and r = 3
= π × 6 × 16 This forms an arithmetic series, where
Here, product of qr = 8 × 3 = 24
a = 2, d = 9 − 2 = 7 and an = 93
= 96 π cm 2 p divides the product of qr but cannot
divide r. an = a + ( n − 1) d , 93 = 2 + ( n − 17
)
70. Six cubes, each with 12 cm edge are n − 1 = 13, n = 14
Let p = 4, q = 8 and r = 12
joined end to end. What is the
Product of qr = 96 There are 14 number of ways by which
surface area of resulting cuboid? a person can by apples and oranges.
p divides the product of qr and also
(a) 3000 cm 2 (b) 3600 cm 2 divide r.
2
75. Given y is inversely proportional to
(c) 3744 cm (d) 3777 cm 2 We cannot say that p will divide r. x , and x = 36 when y = 36. What is
Hence, statement 1 is correct and
_ (c) When six cubes are joined end to the value of x when y = 54 ?
end, then resulting solid is a cuboid. statement 2 is incorrect.
(a) 54 (b) 27
Length ( l ) = 6 × 12 = 72 cm 73. Radha and Rani are sisters. Five (c) 16 (d) 8
Breadth ( b ) = 12 cm years back, the age of Radha was
Height ( h) = 12 cm three times that of Rani, but one _ (c) According to the question,
1
Surface area of cuboid year back the age of Radha was two y∝
times that of Rani. What is the age x
= 2( lb + bh + lh)
k
= 2(72 × 12 + 12 × 12 + 12 × 72 )
difference between them? y=
x
= 2 × 1872 cm 2 = 3744 cm 2 (a) 8 (b) 9 (c) 10 (d) 11
x = 36, y = 36
71. If the sum of a real number and its _ (a) Let the present age of Radha be x k
Rani be y. ⇒ 36 = ⇒ k = 216
26 36
reciprocal is , then how many Five years ago,
5
Age of Radha = x − 5 Now, y = 54, k = 216
such numbers are possible? 216
Age of Rani = y − 5 54 =
(a) None (b) One x
According to the question,
(c) Two (d) Four
x − 5 = 3( y − 5) ⇒ x − 5 = 3 y − 15 ∴ x =4
_ (b) Let the number be x. 3 y − x = 10 …(i) ⇒ x = 16
1
Reciprocal of number = One year ago, 76. What is the square root of 16 + 6 7 ?
x
Age of Radha = x − 1
According to the question, (a) 4 + 7 (b) 4 − 7
Age of Rani = y − 1 (c) 3 + 7 (d) 3 − 7
CDS Solved Paper 2019 (II) 13

_ (c) Let x = 16 + 6 7 Where, p and q are quotients. _ (d)


On square rooting both sides, we get How, we can put in the values of pandq 1. 75 is not a rational number.
into the equation, starting with 0. Only root of perfect square is rational
x = 16 + 6 7 When x = 14 p + 7 number, here 75 is not a perfect
= 9+7 +2 ×3× 7 p= 0 x = 14 × 0 + 7 = 7 square.
4x 7
p=1 x = 14 × 1 + 7 = 21 2. − <−
= ( 3)2 + ( 7 )2 + 2 × 3 × 7 5 8
p=2 x = 14 × 2 + 7 = 35 etc.
= (3 + 7 )2 ⇒ −32 x < −35
When x = 15 q + 5
⇒ 32 x > 35
[Q( a + b )2 = a 2 + b 2 + 2 ab] q =0 x = 15 × 0 + 5 = 5
It is possible, if x ≥ 2.
x =3+ q =1 x = 15 × 1 + 5 = 20
7 x −2
q =2 x = 15 × 2 + 5 = 35 3. < 1 does not follow for all real
77. What is the number of digits in Here, we found the common value for
x
values, It only follow for natural
7 25 , 8 23 and 9 20 , respectively? x. Hece, the least possible value for x numbers,
[given, log10 2 = 0.301, for which both statements are true, is
For example, If x = − 1
log10 3 = 0.477, log10 7 = 0845
. ] 35.
−1 − 2
<1
(a) 21, 20, 19 (b) 20, 19, 18 79. Two taps X and Y are fixed to a −1
(c) 22, 21, 20 (d) 22, 20, 21 water tank. If only X is opened, it −3
⇒ <1
drains out the full tank of water in −1
_ (c) Let x = 7
25
20 min. If both X andY are opened, 3 < 1 (does not follow)
On taking log both sides, we get then they drain out the full tank of if x = 3
log10 x = log10 7 25 water in 15 min. If onlyY is opened, 3−2 1
< 1, < 1 (follow)
= 25 log10 7 [Q log a b = b log a] how long does it take to drain out 3 3
= 25 × 0.845 ⇒ = 21125
. the full tank of water? 4. x = 4.232323 …(i)
∴Number of digits = 21 + 1 = 22 (a) 30 min (b) 45 min On multiply by 100 both sides, we get
Let y = 8 23 (c) 60 min (d) 90 min 100x = 423 .2323 …(ii)
On subtracting Eq. (i) from Eq. (ii),
On taking log both sides, we get _ (c) X drains out full tank in 20 min. we get
log10 y = log10 8 23 X 1 min work =
1
100x − x = 423.2323 … − 4.2323……
= log10(2 ) = log10 2
3 23 69 20
X and Y drain out full tank in 15 min. 99x = 419
= 69 log10 2 [Q log a b = b log a] 419
X and Y 1 min work =
1 ⇒ x =
= 69 × 0.301 99
15
= 20769
. 419 p
Y 1 min work =
1

1 ∴ is in form and both p and q
∴Number of digits = 20 + 1 = 21 99 q
15 20
Let z = 9 20 1 are integers.
=
On taking log both sides, 60 Hence, statements 2 and 4 are
we get correct and statements 1 and 3 are
Y will take 60 min to drain out full tank.
log10 z = log10 9 20 incorrect.
= log10( 3 2 )20
80. Consider the following statements 81. A library has an average number of
= log10 3 40
= 40 log10 3 1. 75 is a rational number. 510 visitors on Sunday and 240 on
[Q log a b = b log a] 2. There exists at least a other days. What is the average
= 40 × 0.477 positive integer x such that number of visitors per day in a
4x 7 month of 30 days beginning with
= 19.08 − <− . Saturday?
Number of digits = 19 + 1 = 20 5 8
x −2 (a) 276
78. Let x be the smallest positive 3. < 1 for all real value (b) 282
integer such that when 14 divides x, x
(c) 285
the remainder is 7; and when 15 of x.
(d) 375
divides x, the remainder is 5. Which 4. 4.232323… can be expressed in
one of the following is correct? the form p /q where p and q are _ (c) Since, the months begin with
Saturday, there will be five Sunday,
(a) 20 < x < 30 (b) 30 < x < 40 integers.
Required average
(c) 40 < x < 50 (d) x > 50 Which of the above statements are
Total number of visitors
correct? =
_ (b) Dividend Number of days
= (Integer quotient) × divisor + (a) 1 and 2
remainder 510 × 5 + 240 × 25
(b) 2 and 3 =
x = 14 p + 7 (c) 3 and 4 30
8550
⇒ x = 15 q + 5 (d) 2 and 4 = = 285
30
14 CDS Solved Paper 2019 (II)

82. If 36 = 3 + 1 Selling price of both items is 990 85. A real number x is such that
, where
11 1 ∴ 110x = 990 ( x − x 2 ) is maximum. What is x
x+ ⇒ x =9
1 equal to ?
y+ 90 y = 990
z (a) − 1.5 (b) − 05
. (c) 05
. (d) 1.5
⇒ y = 11
x , y and z are natural numbers, then
CP1 of 1st item = 100 × 9 = 900
_ (c) x 2is a real number such that
what is ( x + y + z ) equal to ( x − x ) is maximum.
CP 2 of 2nd item = 100 × 11 = 1100 Let y = x − x2
(a) 6 (b) 7
Total cost price = CP1 + CP2 On differential both sides w.r.t. x, we
(c) 8 (d) 9
36 1 = 900 + 1100 get
_ (a) Given, = 3+
11 1 = 2000 dy
=
d
(x ) −
d
(x 2 )
x+
y+
1 ∴Required loss dx dx dx
2000 − 1980 = x1 − 1 − 2 x 2 − 1
z %= × 100
3 1 2000
⇒ 3+ = 3+ dy
= 1 − 2x
11 1 = 1% loss
x+ dx
1
y+ Alternate Method To be the function maximum
dy
=0
z
By comparing, we get Here, a = 10% dx

3 1 ∴ Required loss percentage 1 − 2x = 0 ⇒ 2x = 1


= 1
11 x + 1  a
2
 10 
2
∴ x = ⇒ x = 0.5
1 =   =   = 1% 2
y+  10   10 
z Alternate method : On solving the
1 11 84. It takes 11 h for a 600 km journey if given question with respect of option :
x+ =
1 3 120 km is done by train and the rest From option (a),
y+
z by car. It takes 40 min more if 200 (x − x 2 ) =
1 2 −15
. − ( −15 . )2 = −15
. − 2.25 = −375
⇒x + = 3+ km are covered by train and the rest .
1 3
y+ by car. What is the ratio of speed of From option (b),
z
By comparing, we get
the car to that of the train? ( x − x 2 ) = −0.5 − ( −0.5)2
x = 3, (a) 3 : 2 (b) 2 : 3 = 0.5 − 025
. = −075
.
1 3 (c) 3 : 4 (d) 4 : 3 From option (c),
y+ =
z 2 _ (a) Let the speed of train be x km/h ( x − x 2 ) = 0.5 − ( 0.5)2
1 1
y + = 1+ and car y km/h. = 0.5 − 025
. = 025
.
z 2
According to the question, From option (d),
By comparing, we get y = 1, z = 2
120 480
+ = 11 …(i) ( x − x 2 ) = 15
. − ( −15
. )2
Then, ( x + y + z) = 3 + 2 + 1 = 6
x y = 15
. − 2.25 = −075
.
83. A person sells two items each at Rs. 200 400 2 Hence,x = 0.5
+ = 11 …(ii)
990, one at a profit of 10% and x y 3
another at a loss of 10%. What is the 86. If 10n divides6 23 × 75 9 × 105 2 , then
To multiply Eq. (i) by 5 and Eq. (ii) by 3,
combined percentage of profit or and subtracting, we get what is the largest value of n?
loss for the two items? 600 2400 600 1200 (a) 20 (b) 22 (c) 23 (d) 28
+ − − = 55 − 35
(a) 1% loss
_ (a) Let x = 6 × 75 × 105
x y x y 23 9 2

(b) 1% profit 1200


= 20 We can also write this expression as
(c) No profit no loss y
(d) 0.5% profit x = 2 23 × 3 23 × 518 × 3 9
⇒ y = 60 km/h × 5 2 × 3 2 × 7 2= 2 23 × 3 34 × 7 2 × 5 20
_ (a) Let CP of 1st and 2nd items are On putting y = 60 in Eq. (i), we get
100x and 100 y respectively. If 10 n divides this expression, then n
120 480 must be equal to the number of zero in
According to the question, + = 11
x 60 the expression, from above
SP1 of 1st item expression we can conclude that there
120
 100 + profit %  ⇒ = 11 − 8 will be 20 number of zero ( 5 20 × 2 20 )
= CP ×   x
 100  because only 5 and 2 make zero.
x = 40 km/hr So, n = 20
110
= 100x × = 110x y 60
100 ∴ Required ratio = = 87. What is the digit in the unit’s place
x 40
SP2 of 2nd item of the number represented by
3
 100 − Loss% 
= CP ×  
= 3 98 − 3 89 ?
  2
100
= 3:2 (a) 3 (b) 6
90
= 100 y × = 90 y (c) 7 (d) 9
100
CDS Solved Paper 2019 (II) 15

_ (b) x = 3 − 3 = 3 ( 3 − 1)
98 89 89 9 x + y=7 = x2 + 2 ×
5
x +
25 25
− + 10
We know that, the cyclicity of 3 is 4. x = 6 and y = 1 [1st pair] 2 4 4
2
So, we can write above expression as If x + y = 35  5 15
= x +  +
x = 3 4 × 22 × 31( 3 2 × 4 × 3 − 1) x − y=1  2 4
It will be equal to = 3 ( 3 − 1) = 3 × 2
1 1
x = 18 and y = 17 [2nd pair] 1
To make the expression
=6 Only 2 pair are possible. x 2 + 5x + 10
Unit digit of the expression is 6. maximum, then we must minimize the
91. If (b − 6) is one root of the quadratic value of denominator.
88. The sum of the squares of four equation x 2 − 6x + b = 0, where b is To minimize denominator of expression
consecutive natural numbers is 294. an integer, then what is the 1 5
, we can take x = −
What is the sum of the numbers? maximum value of b 2 ?  5
2
15 2
(a) 38 (b) 34 (c) 30 (d) 26 x +  +
(a) 36 (b) 49 (c) 64 (d) 81  2 4
_ (b) Let the four consecutive natural _ (d) ( b − 6) is one root of expression The maximum value of expression will
numbers are x − 1, x, x + 1, x + 2 1 4
x 2 − 6x + b = 0 be =
According to the question,  5 5
2
15 15
So, x = b − 6 should satisfy the − +  +
( x − 1)2 + x 2 + ( x + 1)2 + ( x + 2 )2 equation.  2 2 4
= 294 ( b − 6)2 − 6( b − 6) + b = 0
b + 36 − 12 b − 6b + 36 + b = 0
2 94. If the ratio of the work done by
x 2 + 1 − 2x + x 2 + x 2 + 1
b 2 − 17 b + 72 = 0 ( x + 2) workers in ( x − 3) days to the
+ 2 x + x 2 + 4x + 4 = 294 b 2 − 8b − 9b + 72 = 0 work done by ( x + 4 ) workers in
4x 2 + 4x − 288 = 0 b( b − 8) − 9( b − 8) = 0 ( x − 2) days is 3 : 4, then what is the
⇒ x 2 + x − 72 = 0 ( b − 9)( b − 8) = 0
b = 8 and 9
value of x ?
x + 9x − 8x − 72 = 0
2
To get maximum value of b 2 (a) 8 (b) 10
⇒ x ( x + 9) − 8( x + 9) = 0 b = 9 ⇒ b 2 = 81 (c) 12 (d) 15
( x − 8)( x + 9) = 0
92. If a = 7 + 4 3, then what is the _ (b) Number of workers ( x + 2 ) worked
x = 8 and −9 ( x = −9)[ignore] for ( x − 3) days and ( x + 4) workers
1 worked for ( x − 2 ) days.
The numbers are, 7, 8, 9, 10 value of a + ?
Sum of numbers a According to the question,
= 7 + 8 + 9 + 10 = 34 ( x + 2 )( x − 3) 3
(a) 2 (b) 3 =
(c) 4 (d) 7 ( x + 4)( x − 2 ) 4
89. The equation x 2 + px + q = 0 has
x2 − x − 6 3
roots equal to p and q where q ≠ 0. _ (c) a = 7 + 4 3 =
x 2 + 2x − 8 4
What are the values of p and q, = 4+ 3+ 2 ×2 3
respectively? 4( x 2 − x − 6) = 3( x 2 + 2 x − 8)
= 2 + ( 3) + 2 × 2 × 3
2 2
4x 2 − 4x − 24 = 3x 2 + 6x − 24
(a) 1, − 2 (b) 1, 2
(c) − 1, 2 (d) −1, − 2 = (2 + 3) 2 x 2 − 10x = 0
[Q( a + b ) = a + b + 2 ab]
2 2 2 x 2 = 10x ∴ x = 10
_ (a) x + px + q = 0
2

a=2 + 3 95. Which one of the following is not


So, p+ q = − p
1 1 correct?
q = −2p …(i) So, =
a 2 + 3 (a) 1 is neither prime nor composite
pq = q ⇒ p = 1
1 1 2 − 3 (b) 0 is neither positive nor negative
Now, on putting p = 1in Eq. (i), we get = ×
a 2 + 3 2 − 3 (c) If p × q is even, then p and q are
q = −2 ×1
always even
q = −2 1 2− 3
= =2 − 3 (d) 2 is an irrational number
p = 1 and q = − 2 a 4− 3
1 _ (c) If p × q is even, then p and q can or
90. How many pairs of natural Now, a + = 2 + 3 + 2 − 3 = 4 cannot be even, Let illustrate with
a
numbers are there such that the example,
difference of their squares is 35? 93. What is the maximum value of the Let p = 2 and q = 4, then
1 p×q=2 × 4= 8 (even)
(a) 1 (b) 2 (c) 3 (d) 4 expression ?
x 2 + 5x + 10 Let p = 2 and q = 3, then
_ (b) Let the number be x and y. p×q=2 × 3= 6 (even)
15 15 4
x 2 − y 2 = 35 (a) (b) (c) 1 (d)
Let p = 3 and q = 4, then
4 2 15
( x − y)( x + y) = 35 1 p × q = 3 × 4 = 12 (even)
_ (d)
Thus, the factor of 35 possibles are 7, x 2 + 5x + 10 With above example we can conclude
5, 35, 1 that if any one of pand q is even we will
Take denominator x 2 + 5x + 10.
If x − y=5 get p × q even.
y = x 2 + 5x + 10
16 CDS Solved Paper 2019 (II)

96. What is the sum of all integer _ (a) Let p = x + 3x + 3x + 1


3 2
99. If X = {a, {b }, c } , Y = {{a }, b, c } and
values of n for which n 2 + 19n + 92 = ( x + 1)3 Z = {a, b, {c }},
is a perfect square? [Q( a + b )3 then ( X ∩ Y ) ∩ Z equals to
= a + b + 3a b + 3b 2a]
3 3 2
(a) 21 (b) 19 (c) 0 (d) −19 (a) {a, b, c } (b) {{a }, {b }, {c }}
Let q = x 3 + 5x 2 + 5x + 4 (c) {φ} (d) φ
_ (d) Let k = n + 19n + 92
2 2

= x 3 + 4x 2 + x 2 + 4x + x + 4
On multiplying both sides by 4, we get = x 2( x + 4) + x ( x + 4) + 1 ( x + 4)
_ (d) Given, X = { a, { b}, c}
4n + 76n + 368 = 4k
2 2 Y = {{ a}, b, c}, Z = { a, b, {c}}
= ( x + 4)( x 2 + x + 1)
⇒ (2 n) + 2 × 19 × 2 n
2 Then, X ∩ Y = {c}
Let r = x 2 + 5x + 4 ( X ∩ Y ) ∩ Z = {c} ∩ { a, b, {c}} = φ
+ 361 + 7 = 4k 2
⇒ (2 n + 19)2 = ( 4k 2 − 7 ) …(i) = x 2 + 4x + x + 4 100. Two numbers p and q are such that
Let (2 n + 19) = m = x ( x + 4) + 1( x + 4) the quadratic equation
Then, 4k 2 − 7 = m2 = ( x + 1)( x + 4) px 2 + 3x + 2q = 0 has −6 as the sum
4k 2 − m2 = 7
LCM of p, q and r and the product of the roots. What
(2 k − m)(2 k + m) = 7
= ( x + 1)3( x + 4)( x 2 + x + 1) is the value of ( p − q ) ?
[Q a − b = ( a + b )( a − b )]
2 2
(a) −1 (b) 1 (c) 2 (d) 3
Factor of 7 are 7 and 1 98. What is the value of
_ (c) px + 3x + 2q = 0
2
So, 2k + m = 7 ( x − y ) 3 + (y − z ) 3 + (z − x ) 3
2k − m = 1 3
9( x − y ) (y − z )(z − x ) Sum of roots (α + β ) = −
By solving this, we get k = 2 p
1 2q
Now, putting k = 2 in Eq. (i), we get (a) 0 (b) Product of root (α ⋅ β ) =
3
(2 n + 19) = ( 4(2 ) − 7 ) = 16 − 7
2 2 p
1
⇒ (2 n + 19)2 = 9
(c) (d) 1 −3
9 According to the question, = −6
⇒ 2 n + 19 = ± 3 p

⇒ 2 n + 19 = + 3 ⇒ n = − 8
_ (b) Let 1
( x − y)3 + ( y − z)3 + ( z − x )3 p=
p= 2
⇒ 2 n + 19 = − 3 ⇒ n = − 11 9( x − y) ( y − z) ( z − x ) 2q
Sum of integer value of n = − 8 + ( −11) ⇒ = −6
Let a = x − y, b = y − z, c = z − x p
= − 19
a + b + c = x − y + y − z + z − x = 0If −6 × p
97. What is the LCM of the polynomials ⇒ q =
a + b + c = 0, 2
x 3 + 3x 2 + 3x + 1, then a 3 + b 3 + c 3 = 3abc −3
⇒ q =
x 3 + 5x 2 + 5x + 4 and x 2 + 5x + 4 ? 3( x − y)( y − z)( z − x )
2
∴p = 1  3 1 3
(a) (x + 1) (x + 4)(x + x + 1)
3 2 9( x − y) ( y − z)( z − x ) Then, p − q = −  −  = +
2  2 2 2
(b) (x + 4)(x 2 + x + 1) 1
= 4
(c) (x + 1)(x 2 + x + 1) 3 = =2
2
(d) (x + 1)2 (x + 4)(x 2 + x + 1)
PAPER II English

Directions (Q. Nos. 1-7) In this Directions (Q. Nos. 8-17) Given 13. Be in seventh heaven
section, a word is spelled in four below are some idioms/phrases followed (a) To be extremely happy
different ways. You are to identify the by four alternative meanings to each. (b) To be extremely upset
one which is correct. Choose the Choose the response (a), (b), (c) or (d) (c) To be extremely adventurous
alternative bearing the correct spelling which is the most appropriate meaning. (d) To be extremely silent
from (a), (b), ©) and (d). 8. Dirt cheap _ (a) The idiom ‘Be in seventh heaven’
means ‘to be extremely happy’. So,
1. (a) Accommodate (a) Extremely cheap option (a) is the correct answer of given
(b) Acommodate (b) Extremely costly idioms.
(c) Accomdate (c) Very cheap person 14. Hand in glove
(d) Acomodait (d) Very cheap item
(a) Working separately
_ (a) ‘Accommodate’ is the correct _ (a) Idiom ‘Dirt cheap’ means ‘very (b) Working together
spelling and it means to make an inexpensive’. So, option (a) ‘extremely
(c) Working for someone
adjustment to suit a particular purpose. cheap’ express its correct meaning.
(d) Not willing to work
2. (a) Recommand 9. A shrinking violet
_ (b) The idiom ‘Hand in glove’ means
(b) Reccommed (a) A lean person ‘working together’. So, option (b)
(c) Recommend (b) A shy person express the correct meaning of given
(d) Reccomand (c) A happy person idiom.
(d) A sad person 15. Nip in the bud
_ (c) ‘Recommend’ is the correct
spelling. The word means to advocate _ (b) Idiom ‘A shrinking violet’ means ‘an (a) Prevent a small problem before it
or to suggest that a particular person extremely shy person’. So, option (b) becomes severe
will be suitable for a job. ‘a shy person’ express its correct
(b) Prevent the big problems
meaning.
3. (a) Argyument (b) Argument (c) Make it severe
(c) Arguement (d) Argyooment 10. Gordian knot (d) Beating the problem
(a) Undoable job
_ (b) ‘Argument’ is the correct spelling. _ (a) The idiom ‘Nip in the bud’ means
The word means a set of reasons given (b) A difficult problem ‘prevent a small problem before it
in support of an idea. (c) A different problem becomes severe’. So, option (a) express
(d) Doable job the correct meaning of given idiom.
4. (a) Decisive (b) Desisive
_ (b) Idiom ‘Gordian knot’ means ‘an 16. Like a shag on a rock
(c) Descisive (d) Desicive
extremely difficult problem’. So, option (a) Completely alone
_ (a) ‘Decisive’ is the correct spelling. (b) express the correct meaning of
(b) Completely idle
The word means able to make given idiom.
decisions quickly. (c) Complete silence
11. Fall in a heap (d) Complete happy
5. (a) Aggressive (b) Agresive (a) To be at the mercy of someone
(c) Agressive (d) Aggresive
_ (a) Idiom ‘Like a shag on a rock’ means
else ‘completely alone or isolated’. So,
(b) To be thinking about someone option (a) express the correct meaning
_ (a) ‘Aggressive’ is the correct spelling. of given idiom.
The world means behaving or done in (c) To lose control of one’s own
a determined and forceful way. feelings 17. A pearl of wisdom
6. (a) Assassination (b) Asassination (d) To be in control of one’s own
(a) An important piece of news
feelings
(c) Asasination (d) Assasination (b) An important person
_ (c) The idiom ‘Fall in a heap’ means ‘to (c) An important thing for life
_ (a) ‘Assassination’ is the correct lose control of one’s own feelings’. So, (d) An important piece of advice
spelling. The word means the act of option (c) express the correct meaning
killing a prominent person for either of given idiom. _ (d) Idiom ‘A pearl of wisdom’ means
political or other reason. ‘an important piece of advice’. So,
12. Have a conniption fit option (d) is the correct choice.
7. (a) Embarassment
(a) To be very angry
(b) Embbarasment
(c) Embrasement
(b) To be very happy Directions (Q. Nos. 18-36) Each of
(c) To be very sad the following passage in this section
(d) Embarrassment
(d) To be a jubilant person has some blank spaces with four words
_ (d) ‘Embarrassment’ is the correct
spelling. The word means a feeling of _ (a) The idiom ‘Have a conniption fit’ or groups of words given. Select
self-consciousness, shame or means ‘to become unreasonably whichever word or group of words you
awkwardness. angry or upset’. So, option (a) express
the correct meaning of given idiom. consider most appropriate for the
18 CDS Solved Paper 2019 (II)

blank space and indicate your response democratic and civil libertarian (c) theirs
on the Answer Sheet accordingly. political order and a representative (d) all
system of government ……… 26. physical environment on the one
CLOZE COMPREHENSION 1 (a) basing hand and among the organisms
The founders of the Indian Republic (b) basis of on the other hand. ……… 30.
(c) based (a) Through
……… 18. (a) had
(d) function (b) In spite of
(b) has
on free and fair elections to be (c) Though
(c) has had
conducted on the basis of universal (d) Because
(d) were
adult franchise. the term ‘ecology’ was first coined
the farsightedness and the courage
and used by the German biologist
to commit ……… 19. (a) them _ Solutions (Q. Nos. 18-26)
18. (a) Here, in the given blank, ‘had’ is Ernst Haeckel in 1869, a few
(b) themselves an appropriate choice as the given conceptual terms ……… 31. (a) are
(c) the people sentence is in Past Tense.
(b) were
(d) The course 19. (b) Here, reflexive pronoun
(c) have been
to two innovations of historical ‘themselves’ will be used as it is used
for plural noun ‘founders’. (d) have
significance in nation-buiding and
20. (a) With ‘to’, base form of verb is already proposed to reveal
social engineering : first, to ………
used. So, option (a) ‘build’ (V1) is the relationships ……… 32. (a) among
20. (a) build correct choice for the given blank.
(b) those
(b) building 21. (a) Here, ‘Libertarian’ is the correct
(c) of
(c) constructing word according to the given context
as ‘Libertarian’ means ‘a person or (d) between
(d) built
society who believes in free will’. organisms and their environment.
a democratic and civil …… 21. 22. (b) In the given blank option (b) ‘within For example, French zoologist IG
(a) libertarian a’ is the correct choice as the word Hilaire used the term ‘ethology’
(b) liberation ‘within’ is used to indicate enclosure
or containment.
……… 33. (a) for
(c) liberating
23. (a) ‘Break through’ means ‘an (b) to
(d) liberty
important development or (c) with
society among illiterate people and, achievement’ and it is best suited for (d) in
second, to undertake economic the given blank. the study of the relations of ………
development ……… 22. (a) with a 24. (b) In the given blank, preposition
‘from’ will be used. ‘From’ is used to
34. (a) the
(b) within a
show the starting point of an action. (b) a
(c) for the
25. (c) Article ‘a’ should be used before (c) live
(d) without a
the word which starts with a (d) dead
democratic political structure, consonant sound (democratic). organisms within the family and
Hitherto, in all societies in which 26. (c) The suitable word for the given society in the aggregate and in the
an economic takeoff or an early blank in ‘based’. ‘Based on
something’ means ‘to use particular community. British naturalist St.
industrial and agricultural …………
ideas or facts to make a decision’. George Jackson Mivart proposed
23. (a) breakthrough the term ‘hexicology’ with regard
(b) breakout CLOZE COMPREHENSION 2 to the study of the relations …………
(c) breaking 35. (a) for
Ecology, in a very simple term, is a
(d) investment
science that ……… 27. (a) studies (b) of
had occurred, effective democracy, (c) within
especially from the working (b) study
(d) in
people, had been extremely limited. (c) studying
(d) exploring living creatures to other organisms
On the other hand, ……… 24. and their environment as regards
the interdependent, mutually
(a) with the nature of the locality they
reactive interconnected
(b) from frequency, the temp erature and the
(c) within
relationships ……… 28. (a) among
……… 36. (a) amount
(d) for (b) between
(c) to (b) focus
the beginning, India was (b) share
(d) for
committed to ……… 25. (a) few (d) quality
(b) some
the organisms and ……… 29.
of light which suit them, and their
(c) a (a) their relations to other organisms as
(d) an (b) its
CDS Solved Paper 2019 (II) 19

enemies, rivals, or accidental and 38. The officer in charge of the 44. Many communication problems can
involuntary benefactors. operations has been impugned for be attributed directly to
_ Solutions (Q. Nos. 27-36) the excesses. misunderstanding and inaccuracies.
27. (a) Here, the general fact is given so (a) Expelled (a) Disapproved (b) Unofficial
Present Indefinite Tense will be used. (b) Rewarded (c) Ascribed (d) Tribute
Hence, option (a) ‘studies’ is the (c) Challenged
correct choice. _ (c) ‘Ascribed’ is the correct synonym of
(d) Given allowance the underlined word ‘Attributed’. Both
28. (b) Here, Preposition ‘between’ will be
words mean regard something as
used to fill the given blank. ‘Between’ _ (c) ‘Challenged’ is correct synonym of being caused by.
is used with two persons or things or ‘Impugned’. Both means to challenge
group of things. or oppose as false or lacking integrity 45. The exemptions granted to state
29. (a) Here, the suitable word is ‘their’ to impugned the defendant’s character.
institutions for acquiring informed
indicate the physical environment of 39. Cognitivist and linguists believe that consent from processing personal
the organisms.
every child is born with innate data in many cases appear to be too
30. (c) Conjunction ‘though’ is the
appropriate choice to fill the given
qualities. blanket.
blank as the rest of the sentence (a) Biological (b) Intrinsic (a) Obtain (b) Lose
appear to contradict. (c) Extrinsic (d) Unnatural (c) Giving (d) Thinking
31. (b) The given sentence is in Past
Tense, so Past form ‘were’ will be _ (b) ‘Intrinsic’ is the correct synonym of _ (a) ‘Obtain’ is the correct synonym of
the given word ‘Innate’ both words the underlined word ‘Acquiring’. Both
used to fill the given blank. mean inborn or natural. words mean to obtain or to begin to
32. (d) To fill the given blank, preposition have.
‘between’ is the suitable choice. 40. It was obligatory for the board to
‘Between’ is used with two persons or implement the rule. 46. The manner in which this exercise
things or group of things. has been undertaken leaves much to
(a) Compulsory (b) Unnecessary
33. (a) Here, preposition ‘for’ is the be desired.
(c) By chance (d) Problematic
correct choice to fill the given blank.
(a) Disliked (b) Unlikely
‘For’ is used to show the purpose of _ (a) ‘Compulsory’ is the appropriate (c) Wish for (d) Asked for
something. synonym of the given underlined word
‘Obligatory’. Both means mandatory or
34. (a) Here, article ‘the’ should be used _ (c) ‘Wish for’ is the appropriate
to make definite or specific reference required by a legal, moral or other rule. synonym of the underlined word
that has been already referred to. ‘Desire’. Both means strongly wished
41. They describe the act as a blatant for or intended.
35. (b) In the given blank preposition ‘of’ betrayal of faith.
is the suitable choice. Preposition ‘of’
is used to show the possession. (a) Loyal (b) Faithfulness Directions (Q. Nos. 47-55) In this
36. (a) With the word ‘light’, option (a) (c) Treachery (d) Honesty section, each item consists of six
‘amount’ is the suitable choice in the
given context.
_ (c) ‘Treachery’ is the appropriate sentences of a passage. The first and
synonym of the given underlined word sixth sentences are given in the
‘Betrayal’. Both words mean disloyalty
Directions (Q. Nos. 37-46) Each or to expose to an enemy by treachery. beginning as S1 and S6 . The middle
item in the following questions consists four sentences in each item have been
42. However, if it must decide, then it
of a sentence with an underlined word jumbled up and labelled as P, Q, R and
should do so on the narrowest
followed by four words. Select the S. You are required to find the proper
ground possible.
option that is nearest in meaning to the sequence of the four sentences and
(a) Widest (b) Slightly
underlined word and mark your mark you response accordingly.
(c) Smallest (d) Thick
response on your answer sheet
_ (b) ‘Slightly’ is the appropriate
47. S1 : The master always says, ‘‘Refuse
accordingly. synonym of the word ‘Narrowest’. Both to be miserable’’.
words mean limited in extent, amount
37. The properties of the family have or scope.
S 6 : His is the art of right contact in
been impounded by the order of the life.
court. 43. This is akin to a contractual P : Before you fall into self-pity and
relationship that places obligations blame games, remember that
(a) Confiscated
on the entities entrusted with responsibility comes to only
(b) Permitted
data. those who feel responsible.
(c) Sold
(d) Put on hold (a) Removed (b) Narrow
Q : Challenges are faced by the
(c) Similar (d) Unparallel
_ (a) The word ‘Confiscated’ is the strong and courageous, and if
correct synonym of the given word _ (c) ‘Similar’ is the appropriate life brings you such
‘Impounded’. Both words mean to synonym of the given underlined word
‘Akin’. Both mean corresponding or
opportunities, then turn failures
seize and take legal custody of
something. alike. into success.
20 CDS Solved Paper 2019 (II)

R : Life can be painful, but it need Q : The gulf between the ‘haves’ Q : They are commonly used in all
not be sorrowful. and the ‘have nots’ continued to types of language, informal and
S : If you wan to be happy, find increase and out of this gap formal, spoken and written.
occasions to be cheerful. between the rich and poor R : In additions, idioms often have a
The correct sequence should be sprang disputes. stronger meaning than
(a) RSPQ R : It generated new wealth but as non-idiomatic phrases.
(b) SQPR this new wealth only went to a S : One of the main problems
(c) QRSP minority, it could not solve the students have with idioms is
(d) RQSP
question of distribution.
that it is often impossible to
S : The Industrial Revolution solved guess the meaning of an idiom
_ (a) RSPQ is the correct sequence. the question of production. from the words it contains.
48. S 1 : Gandhiji reached Newcastle and The correct sequence should be The correct sequence should be
took charge of the agitation. (a) PQRS (b) SRQP
(a) RQPS (b) RSPQ
S 6 : The treatment that was meted (c) SRPQ (d) RQSP
(c) SRQP (d) QPSR
out to these brave men and
_ (b) SRQP is the correct sequence. _ (d) QPSR is the correct sequence.
women in jail included
starvation and whipping and 50. S 1 : Institutions define and play a 52. S : Each organism is adapted to its
1
being forced to work in the regulatory role with regard to environment.
mines by mounted military human behaviour.
S 6 : What can be taken in and
police. S 6 : It shows how important it is for broken down depends on the
P : During the course of the march, a nation to build institutions for body design and functioning.
Gandhiji was arrested twice, nurturing democracy.
P : There is a range of strategies by
released, arrested a third time P : Once established, institutions which the food is taken in and
and sent to jail. set a dynamic relationship with used by the organism.
Q : The employers retaliated by the members constituting them
and they mutually affect each Q : For example, whether the food
cutting off water and electricity source is stationary (such as
to the workers’ quarters, thus other.
grass) or mobile (such as deer),
forcing them to leave their Q : They shape preferences, power would allow for differences in
homes. and privilege. how the food is accessed and
R : Gandhiji decided to march this R : At the same time, institutions what is nutritive apparatus used
army of over two thousand men, themselves can be transformed by a cow or a lion.
women and children over the by the politics they produce and
border and thus see them lodged R : The form of nutrition differs
such transformation can affect depending on the type and
in Transvaal jails. social norms and behaviours. availability of food material as
S : The morale of the workers, S : They also provide a sense of
however, was very high and well as how it is obtained by an
they continued to march till they order and predictability. organism.
were prosecuted and sent to jail. The correct sequence should be S : Some organisms break down the
The correct sequence should be (a) RPQS (b) QRSP food material outside the body
(a) QRPS (b) SRQP (c) PSRQ (d) QSRP and them absorb it and others
(c) QPSR (d) RQSP _ (b) QRSP is the correct sequence. take in the whole material and
break it down inside their
_ (d) RQSP is the correct 51. S 1 : Idioms are a colourful and
sequence. bodies.
fascinating aspect of language.
The correct sequence should be
49. S 1 : One of the most important S 6 : Idioms may also suggest a
forces in the modern world, (a) RQPS (b) QPSR
particular attitude of the person
socialism was a direct result of (c) SQPR (d) QPRS
using them, for example,
the Industrial Revolution. disapproval, humour, _ (a) RQPS is the correct sequence.
S 6 : This is how socialism as a theory exasperation or admiration, so 53. S 1 : ‘‘When I was alive and had a
and practice came into being. you must use them carefully. human heart’’, answered the
P : Socialism was a direct challenge P : Your language skills will statue, ‘‘I did not know what
to capitalism and sought to put increase rapidly if you can tears were, for I lived in the
and end to such an exploitative understand idioms and use them Palace of Sans-Souci where
economic structure. confidently and correctly. sorrow is not allowed to enter.
CDS Solved Paper 2019 (II) 21

S 6 : And now that I am dead they 55. S 1 : I had spent many nights in the 58. I will meet you in the third week of
have set me up here so high that jungle looking for game, but this August.
I can see all the ugliness and all was the first time I has ever spent a (a) Pronoun (b) Verb
the misery of may city, and night lookign for a man-eater. (c) Preposition (d) Noun
though my heart is made of lead S 6 : It was in this position my men _ (c) ‘In’ is a simple preposition and
an hour later found me fast used to describe a location, time or
yet I cannot choose but weep.’’ place is used as preposition.
asleep; of the tiger I had neither
P : So I lived, and so I died.
heard nor seen anything. 59. Jasmines and roses are my favourite
Q : Round the garden ran a very flowers.
P : I bitterly regretted the impulse
lofty wall, but I never cared to ask
that had induced me to place (a) Verb (b) Preposition
what lay beyond it, everything (c) Conjunction (d) Interjection
myself at the man-eather’s
about me was so beautiful.
mercy. _ (c) ‘And’ is a conjunction as it joins two
R : My courtiers called me the subjects.
Q : The length of road immediately
Happy Prince, and happy indeed
in front of me was brilliantly lit 60. She truthfully answered the detective’s
I was, if pleasure be happiness.
by the, moon, but to right and questions.
S : In the daytime I played with my left the overhanging trees cast (a) Verb (b) Adjective
companions in the garden and in dark shadows and when the (c) Noun (d) Adverb
the evening I led the dance in the night wind agitated the
Great Hall. _ (d) Here, the word ‘truthfully’ is an
branches and the shadows adverb which is modifying the verb
The correct sequence should be moved, I saw a dozen tigers ‘answered’.
(a) QSRP (b) PQRS advancing on me. 61. Hurrah! We won the game!
(c) PRQS (d) RPQS R : As the grey dawn was lighting (a) Interjection (b) Conjunction
_ (*) SQRP is the correct sequence. up the snowy range which I was (c) Noun (d) Pronoun

54. S 1 : One day her mother, having facing, I rested my head to my _ (a) The underlined word ‘hurrah’ is an
drawn-up knees. Interjection or exclamation. In this part
made some cakes, said to her, of speech, different words show
‘‘Go, my dear, and see how your S : I lacked the courage to return to different emotions.
grandmother is doing, for I hear the village and admit I was too
frightened to carry out my 62. The son writes meaningless letters to
she has been very ill. Take her a his father.
cake, and this little pot of self-imposed task, and with
(a) Adverb (b) Verb
butter.’’ teeth chattering, as much from
(c) Pronoun (d) Adjective
S 6 : ‘‘Does she live far off?’’ said the fear as from cold, I sat out the
long night. _ (d) The word meaningless is an
wolf. ‘adjective’ that telling more about the
The correct sequence should be noun (letters).
P : He asked her where she was going.
(a) QPSR (b) SRPQ 63. The secretary himself visited the
Q : The poor child who did not (c) PRSQ (d) RPQS
know that it was dangerous to affected families.
stay and talk to a wolf, said to _ (a) QPSR is the correct sequence. (a) Verb (b) Noun
(c) Adverb (d) Pronoun
him, ‘‘I am going to see my
grandmother and carry her a Directions (Q. Nos. 56-65) Given _ (d) The word ‘himself’ is possessive
below are a few sentences. Identify pronoun, used for noun ‘secretary’.
cake and a little pot of butter
from my mother’’. the part of speech of the underlined 64. The children were walking, through
R : As she was going through the word. Choose the response (a), (b), (c), the forest.
wood, she met with a wolf, who or (d) which is the most appropriate (a) Verb (b) Adverb
answer. (c) Adjective (d) Preposition
had a very great mind to eat her
up, but he dared not, because of 56. Rita eats her dinner quicky. _ (d) Here, ‘through’ is preposition.
‘Through’ is a simple preposition and
some woodcutters working (a) Verb (b) Preposition can be used with respect to ‘by’ means
nearby in the forest. (c) Adjective (d) Adverb of or from one side to another.
S : She set out immediately to go to 65. The Presiding Officer walked slowly
_ (a) ‘Eats’ is a verb as it show action.
her grandmother, who lived in to the dais.
another village. 57. He thought the movie ended abruptly.
(a) Adverb (b) Adjective
The correct sequence should be (a) Noun (b) Adverb (c) Verb (d) Noun
(c) Verb (d) Adjective
(a) PRQS (b) SRPQ _ (a) Slowly is an ‘adverb’ that modifying
(c) PRSQ (d) RPQS _ (b) ‘Abruptly’ is an adverb, which is the verb ‘walked’.
modifying the verb ‘ended’ in the
_ (b) SRPQ is the correct sequence. sentence.
22 CDS Solved Paper 2019 (II)

Directions (Q. Nos. 66-75) Each 71. People look for plausible remedies to Directions (Q. Nos. 76-88) Each of
item in this section consists of a the problems which they do not the following items in this section
sentence with an underlined word know. consists of a sentence, the parts of
followed by four words/group of (a) Acceptable (b) Unthinkable which have been jumbled. These parts
words. Select the option that is opposite (c) Solvable (d) Believable have been labelled as P, Q, R and S.
in meaning to the underlined word and _ (b) The underlined word ‘plausible’ Given below each sentence are four
mark your response on your answer sheet means believable or probable. So, its sequence namely (a), (b), (c) and (d).
correct antonym would be
accordingly. ‘unthinkable’ or incredulous. You are required to rearrange the
66. Beauty lies is the eyes of the beholder. 72. The departing speech of the jumbled parts of the sentence and
mark your response
(a) Allure (b) Charm Chairperson ended with a plaintive
(c) Inelegance (d) Ideal accordingly.
note.
_ (c) From the given options (a) Melancholic 76. the company are often asked
‘inelegance’ is the correct antonym of (b) Gleeful (P)
underlined word ‘beauty’. It means (c) Doleful
ungraceful or ugly. the formal or informal interviews
(d) Adventurous
(Q)
67. Reading details about suicide cases _ (b) The underlined word ‘plaintive’
can push vulnerable people taking means sad or mournful.Its correct
employees who are leaving
the extreme step. antonym would be ‘gleeful’ which (R)
means full of happiness. for their opinions during
(a) Imperious (b) Impervious
(c) Helpless (d) Defenseless 73. The members have taken a (S)
unanimous decision to discord some The correct sequence should be
_ (b) The underlined word ‘vulnerable’
means exposed to the possibility of of the rulings of the Managing (a) RPSQ (b) RQPS
being attacked or harmed. Its Committee on problems relating to (c) PSQR (d) PQSR
correct antonym is ‘impervious’ which maintenance.
means unable to be affected by _ (a) RPSQ is the correct sequence.
anything. (a) Accord (b) Dissension
(c) Dispute (d) Friction 77. a hailstorm activity in the evenings
68. Standing before a judge in a courtroom (P)
can be daunting for anyone. _ (a) The underlined word ‘discord’
means disagreement between there is a possibility of
(a) Uncomfortable(b) Encouraging people. Its correct antonym would be
‘accord’ means an official agreement
(Q)
(c) Demoralising (d) Off-putting
or treaty. while there could be
_ (b) The underline word ‘daunting’ means (R)
to make (some one) feel intimidated or 74. The insolent nature of the speaker
apprehensive. Its correct antonym is had provoked the members of the heavy rain towards the weekend
‘encouraging’ which means to
house and this led to pandemonium. (S)
persuade someone to do something.
(a) Respectful The correct sequence should be
69. He has been facing a kind of (b) Autocratic (a) SQPR
intimidation by his friends for last (c) Impudent (b) QSRP
two years. (d) Thought provoking (c) QRPS
(a) Wiles (b) Conviction (d) SPRQ
_ (a) The underlined word ‘insolent’
(c) Persuasion (d) Support means showing a rude and arrogant
_ (b) QSRP is the correct sequence.
behaviour. Its correct antonym is
_ (c) The underlined, word ‘intimidation’ ‘respectful’ which means showing 78. has been below normal since last week
means to frighten or threaten
regards or respect.
someone. Its correct antonym is (P)
‘persuasion’ which means the act of 75. Incessant rains have resulted in the minimum temperature
reasoning or pleading with someone.
failure of crops during this season. (Q)
70. There are many factors that (a) Sporadic in some part of the city
constrain the philosophy of job (b) Persistent
(R)
enrichment in practice. (c) Continual
when rain and hailstorm activity
(a) Oblige (b) Pressure (d) Ceaseless
recorded
(c) Restrict (d) Support
_ (a) The underlined word ‘incessant’ (S)
means continuous or non-stop.
_ (d) The underlined word ‘constrain’ The correct sequence should be
means compel or force (someone) to Its correct antonym would be
follow a particular course of action. ‘sporadic’ which means happening (a) RSPQ (b) SPRQ
So, among the given options its only sometimes, not regular or (c) QPSR (d) PSQR
correct antonym is ‘support’. continuous.
_ (c) QPSR is the correct sequence.
CDS Solved Paper 2019 (II) 23

79. for guest teachers by the things you didn’t do twenty 88. have a great influence
(P) (R) (P)
in the Department of years from now and they often shape our personality
Biotechnology was also held (S) (Q)
(Q) (R) The correct sequence should be on our adult lives
a Selection Committee meeting (a) PRSQ (b) PRQS (R)
(S) (c) PQSR (d) SPRQ events in our childhood
The correct sequence should be _ (d) SPRQ is the correct sequence. (S)
(a) SPRQ (b) QRSP 84. man is one who can lay The correct sequence should be
(c) PRQS (d) RSPQ (a) SPRQ (b) SQRP
(P)
(c) SRQP (d) PQRS
_ (a) SPRQ is the correct sequence. a firm foundation with the bricks
Ê (a) SPRQ is the correct sequence.
80. for contractual assignment at (Q)
Cultural Centres abroad a successful Directions (Q. Nos. 89-103) Each
(P) (R) item in this section has a sentence
as Teacher of Indian Culture for others have thrown at him which has multiple parts. Find out the
two years (S) error part from the given option and
The correct sequence should be indicate your response from the options
(Q)
(a) PQSR (b) RQSP (a), (b), (c) and (d) on the answer sheet.
applications are invited in a
(c) RPQS (d) QSPR
prescribed format 89. Experience has shown that the
(R) _ (c) RPQS is the correct sequence. change-over from a closed economy
from Indian Nationals for deployment 85. what we may be to a mercantile economy has presented
(P) in human society innumerable
(S)
but not we not know problems.
The correct sequence should be (a) Experience has shown that
(a) QPRS (b) SRPQ
(Q) (b) The change over from a closed
(c) PQRS (d) RSQP we know what we are economy
(R) (S) (c) to a mercantile economy has presented
_ (d) RSQP is the correct sequence.
The correct sequence should be (d) in human society innumerable
81. while they are small and do the problems
(a) RSQP (b) QPRS
(P) (Q) (c) QRPS (d) RQPS _ (d) In the given sentence, part (d) has
great things while they are easy an error. ‘In’ should be replaced by
(R) _ (a) RSQP is the correct sequence. article ‘the’ to make the sentence
grammatically correct.
do the difficult things 86. for the ordinary not willing to risk
(S) (P) (Q)
90. A closed economy is identified as a
The correct sequence should be human community which produces
the unusual if you are all it consumes and consumed all it
(a) SRQP (b) PSQR (R) produces.
(c) SRPQ (d) QPSR you will have to settle (a) A closed economy is identified
_ (a) SRQP is the correct sequence. (b) as a human community
(S)
(c) which produces all it consumes
82. then you sure if you can’t The correct sequence should be
(d) and consumed all it produces
(P) (Q) (a) PRQS (b) SPQR
(c) RQSP (d) QSRP _ (d) Here, part (d) of the given sentence
don’t deserve me at my best handle has an error. The word ‘consumed’
me at my worst _ (c) RQSP is the correct sequence. should be replaced with ‘consumes’ as
the whole sentence is in simple present
(R) (S) 87. as mere stepping stones tense.
The correct sequence should be (P)
(a) PRQS (b) QSPR his major achievements 91. Iron is the most useful against all
metals.
(c) RQSP (d) PSRQ (Q) (a) Iron is (b) the most useful
for the next advance he regarded
_ (b) QSPR is the correct sequence. (c) against all metals
(R) (S) (d) No error
83. you will be more disappointed
The correct sequence should be _ (c) Here, part (c) of the given sentence has
(P) (a) SPQR (b) SQPR an error. The word ‘against’ should be
than by the ones you did do replaced with ‘among’ because the
(c) SPRQ (d) RPQS
sentence talks about all metals.
(Q)
_ (b) SQPR is the correct sequence. ‘Among’ is for more than two things.
24 CDS Solved Paper 2019 (II)

92. Mumbai is largest cotton centre in 97. Here knowledge of Indian languages 102. Education is the passport to the
the country. are far beyond the common. future, for tomorrow belong to those
(a) Mumbai is (a) Her knowledge who prepare for it today.
(b) largest cotton centre (b) of Indian languages (a) Education is the passport to the
(c) in the country (d) No error (c) are far beyond the common future,
_ (b) Here, part (b) has an error. Before (d) No error (b) for tomorrow belong to those
superlative word ‘largest’ we should (c) who prepare for it today
use definite article ‘the.’ _ (c) Here, part (c) has an error. Use ‘is’
in place of ‘verb’ are as the subject (d) No error
93. While every care have been taken knowledge is singular, to make
_ (b) Here, part (b) has an error,
in preparing the results, the company the given sentence grammatically
‘belongs’ should be used in place of
correct.
reserves the right to correct any ‘belong’ to make the given sentence
inadvertent errors at a later stage. 98. The care, as well as the love of a grammatically correct.
(a) While every care have been taken father, were missing in her life. 103. There come a time when you have to
(b) in preparing the results choose between turning the page
(a) The care, as well as the love
(c) the company reserves the right to
(b) of a father and closing the book.
correct
(d) any inadvertent errors at a later (c) were missing in her life (a) There come a time
stage (d) No error (b) when you have to choose
_ (c) Here, part (c) is an erroneous part. (c) between turning the page
_ (a) Here, part (a) has an error. The
subject (every care) of the given As we know that with singular subject, (d) and closing the book
sentence is singular. So, verb should singular verb should be used.
also be used in singular form. Therefore, ‘was’ should be used in _ (a) Here, part (a) has an error. Replace
place of ‘were’ as the subject of the ‘there come’ with ‘there comes’ to
Therefore, we must use ‘has been’
given sentence (care) is singular. make the given sentence
in place of ‘have been’ to make the
grammatically correct.
given sentence grammatically correct.
99. You look as if you have ran all the
94. My sister and me are planning a way home. Directions (Q.Nos. 104-120) In this
trip from Jaipur to Delhi. (a) you look as if section, you have few short passage.
(a) My sister and me are (b) you have ran After each passage, you will find some
(b) planning a trip (c) all the way home items based on the passage. First, read
(c) from Jaipur to Delhi (d) No error a passage and answer the items based
(d) No error
_ (b) Here, part (b) has an error. With on it. You are required to select your
_ (a) Here, part(a) has an error of present perfect verb have, we use third answers based on the contents of the
pronoun with noun (my sister), use ‘I’ in form of verb, so run should be used in
place of ‘ran’ to make the given passage and opinion of the author only.
place of ‘me’ to make the given
sentence grammatically correct. sentence grammatically correct.
PASSAGE 1
95. Despite the thrill of winning the 100. The real voyage of discovery consist Mankind’s experience of various
lottery last week, my neighbour not in seeking new landscapes, but in evolutionary changes from primitive
still seems happy. having new eyes. times to the present day has been
(a) The real voyage of discovery extensive and varied. However,
(a) Despite the thrill of winning
(b) consist not in seeking new man’s problems were never before as
(b) the lottery last week,
complicated as they seem to be today.
(c) my neighbour landscapes,
Man’s economic activity centres
(d) still seems happy (c) but in having new eyes
primarily around production. Labour
(d) No error
_ (b) In the given sentence, part (b) is an is said to be the primary factor of
erroneous part. Article ‘a’ should be _ (b) Here, part (b) has an error. Write production; its role, therefore, has
used in place of ‘the’ before lottery. ‘consists' in place of ‘consist’ to make been given a lot of importance. It
Because lottery is a non-specific or the given sentence correct. should be useful to have an overall
non-particular noun and with view of the economic history of man
non-specific or indefinite nouns article 101. No struggle can ever succeeded
from the nomadic times to the
‘a’ should be used. without women participating side by
modern factory system and study its
96. Children are not allowed to use the side with men. relevance to the various labour
swimming pool unless they are (a) No struggle can ever succeeded problems of today.
with an adult. (b) without women participating Initially, man passed through ‘the
(a) Children are not allowed (c) side by side with men hunting and fishing stage’. During
(b) to use the swimming pool (d) No error this period, his basic needs were
(c) unless they are with an adult adequately met by Nature. Wild
_ (a) Here, part (a) has an error. animals, birds and fruits satisfied
(d) No error Remove succeeded and write
‘succeed’ as sentence is in simple his hunger, and his thirst was
_ (d) There is no error. Sentence is present tense. quenched by the waters of springs
grammatically correct. and rivers.
CDS Solved Paper 2019 (II) 25

Caves gave him shelter and barks of _ (c) The given sentence means that the late 1950s and 1960s, and the early
trees were used as clothing. During present times pose much more 1970s saw more stringent
challenges to humans that it was in amendments in the laws to plug
this stage of man’s progress, labour
primitive times. Man faces more
problems did not exist because of the complications and challenges in
loopholes in the earlier laws.
absence of any economic, political present time. But the record of implementation has
and social systems. not been satisfactory. Around 3
106. Why does the author say that million hectares of land has been
Then came ‘the pastoral stage’, which
labour problems did not exist during ‘the declared surplus so far, which is
was marked by a certain amount of
economic activity. The nomadic and
hunting and fishing stage’? hardly 2 percent of net sown area in
migratory nature of man persisted (a) There was no nation existing at India. About 30 percent of this land
and together with his goats and that time has not yet been distributed as it is
cattle, he moved on to fresh pastures (b) There were no economic, political caught up in the litigations.
and meadows. Some conflicts would and social systems Besides, a number of Benami and
sometimes take place among (c) There was no capitalism and clandestine transactions have resulted
herd-owners, for, during this period, market in illegal possession of significant
the institution of nominal private (d) There was no labour law amounts of land above ceiling limits.
property ownership was not known. There are widespread reports of
_ (b) The author says that labour allotment of inferior, unproductive,
This stage paves the way for ‘the problem did not exist during the
agricultural stage’, during which the hunting and fishing state as society barren and wasteland to landless
was not divided and did not have any household, many of whom have been
class system began to develop. There economic, political and social system. forced to sell it off, in the absence of
was a small artisan class mostly Everybody lived in the same simple resources to make it productive.
self-employed; and there were also way.
landed proprietors or Zamindars as In many instances, lands allotted to
well as slaves. 107. “The pastoral stage was marked by a the rural poor under the ceiling laws
certain amount of economic are not in their possession. In some
During the fourth stage of these cases, Pattas were issued to the
activity.” How?
developments, the handicrafts stage’, beneficiaries, but possession of land
a number of social and economic (a) Humans started migrating and
shown in the Pattas was not given, or
changes took place which marked held goat-herds corresponding changes were not
the beginning of the labour (b) Humans started owning land made in the records of right.
problem in the world. The (c) Conflicts started as humans
The balance of power in rural India is
self-sufficient economy of the village owned goats
so heavily weighed against the
underwent a drastic change. The (d) Humans started doing agriculture
landless and the poor that
community of traders and merchants _ (a) The pastoral stage was marked by implementing land ceiling laws is
emerged. a certain amount of economic activity difficult. It is clear that without
as humans started migrating from one
104. Humanity’s evolution from primitive place to another alongwith his goats
massive mobilisation of the rural
stage to the present has been and cattle. poor and depending on democratic
governance in rural India very little
(a) static and smooth 108. Which word in the passage means can be achieved in this direction.
(b) huge and diversified
‘surfaced’? Although half of India’s population
(c) always violent
(d) always peaceful (a) Quenched (b) Emerged continues to depend on agriculture as
(c) Nomadic (d) Adequately its primary source of livelihood, 83
_ (b) According to the passage, percent of farmers operate holdings
humanity’s evolution has been very _ (b) The word ‘surfaced’ means ‘having of less than 2 hectares in size and the
huge as well as diversified. It has risen or emerged’. So, option (b)
‘emerged’ is the correct choice. average holding size is only 1.23
evolved extensively and in various field.
hectares. This is often in fragments
105. .......... “man’s problems were never PASSAGE 2 and unirrigated. There are also those
before as complicated as they seem to Ever since independence, land who are entirely landless, although
be today” means reforms have been a major agriculture is their main source of
instrument of state policy to promote livelihood.
(a) the present times are the best
times of humanity both equity and agricultural They have inadequate financial
investment. Unfortunately, progress resources to purchase and often
(b) the present times are the crucial
on land reforms has been slow, depend on leasing in small plots, on
period for humanity
reflecting the resilience of structures insecure terms, for short periods,
(c) the present times pose much more sometimes only for one season.
of power that gave rise to the problem
challenges to humans than the Hence, many face insecurity of
in the first place. The main
previous times instrument for realising more tenure and the growing threat of land
(d) the present times provide much equitable distribution of land is the alienation and pressure from
more facilities than the previous land ceiling laws. These laws were urbanisation, industrialisation and
times enacted by several states during the powerful interest.
26 CDS Solved Paper 2019 (II)

109. Why does the land reform prove to 113. According to the author, what is the can be generated with ease to provide
be slow? primary source of livelihood of snapshots of a given period of time,
(a) Because of the disparity in power majority of India’s population? trend analysis and breakdowns
structure (a) Industry (b) Forest
according to various demographics.
(b) Because of the power of the You may be interested in responses
(c) Agriculture (d) None
government
by age, sex, job categories,
_ (c) The primary source of livelihood of departments, division, functions or
(c) Because states have different laws majority of India’s population is ‘agriculture’. geography. The survey can be
(d) Because of the scarcity of land in
the country 114. “There are also those who are conducted by placing
entirely landless, although agriculture microcomputers in several locations
_ (a) As per the passage land reforms is their main source of livelihood” means convenient for employees’ use.
are very slow due to disparity in power Employees are advised where the
structure and also ‘land ceiling law’ has (a) they do not have money to buy lands
not been implemented successfully. computers will be, for how long, and
(b) they have sold off their lands to others
when the data will be collected (for
110. Which of the following statements (c) most of them are agriculture labourers
instance, daily at 5:00 p.m. for three
is/are correct? (d) they are migrant labourers from
weeks). The screens should not be
other places
1. Land ceiling laws have proved to viewable to supervisors or passers-by.
be unsatisfactory. _ (c) The given statement means that While there may be some risk that
although some farmers are totally employees will take the survey more
2. The democratic structure of the
dependent on agriculture for livelihood than once, there are comparable risks
government cannot provide
but they do not have enough money to with other methods too. Managers
solution to the problem of land buy lands. may be interested in knowing how
reforms.
they are perceived by their peers and
3. The owners of land have abundant Passage 3 subordinates. Packages are available
natural resources. that can be customised, which allow
Despite downsizings, workers’
4. Identified land for distribution has the manager to complete a
overall job satisfaction actually
not been distributed due to court improved between 1988 and 1994. self-assessment tool used to compare
cases against it. Some reasons given were improved self-perceptions to the anonymous
Select the correct answer using the work flow, better cooperation opinions of others. This comparison
codes given below between departments, and increased may assist in the development of a
(a) 1 and 4 (b) Only 1 fairness in supervision. Many firms more effective manager.
(c) 3 and 4 (d) 2 and 4 today rely on attitude surveys to
monitor how employees feel about 115. Which one of the following is not the
_ (a) Statement 1 and 4 are correct. working in their firms. reason for improved job satisfaction
Land ceiling laws have proved to be of employees?
unsatisfactory and land which has The use of employee attitude surveys
been identified for distribution has had grown since 1944 when the (a) Improved work flow
court cases or litigations so it could not National Industrial Conference (b) Better cooperation between
be distributed. Board “had difficulty finding fifty departments
companies that had conducted (c) Supervisors fairness
111. One of the reasons of selling off the opinion surveys”. Today, most (d) Increased remuneration
lands by the allottees is that the lands companies are aware of the need for
(a) unproductive and barren employees’ anonymity the impact of _ (d) No where in the passage it has
been mentioned that increased
(b) salty, not getting water both the design of the questions and remuneration is reason for improved
(c) fertile,but uncultivable their sequence, the importance of job satisfaction.
(d with the powerful people effective communication, including
knowing the purpose of the survey 116. Companies feel that it is necessary to
_ (a) Land alloted to landless household before it is taken and getting (a) maintain anonymity of the employees
was unproductive and barren, so they
sell off the unproductive land in the
feedback to the employees after it is and to have effective design and
absence of resources. completed. Computerisation of sequence of questions and effective
surveys can provide anonymity, if communication
112. Which word/group of words in the there is no audit trail to the user,
(b) maintain the fairness of the
passage means ‘lawsuit’? especially for short answers that are
managers to be part of the survey
(a) Amendments entered rather than written or typed
on an identifiable machine. (c) conduct surveys from their employees
(b) Litigations (d) maintain anonymity of the
(c) Illegal possession Survey software packages are
employees and not to have effective
(d) Fragments available that generate questions for
design and sequence of questions
a number of standard topics and can
_ (b) Litigations also means ‘Law suit’. be customised by modifying existing
and effective communication
Both words means ‘a claim or dispute
brought to a law court for adjudication’. questions or by adding questions. If _ (c) Many companies felt that it is
the survey is compu- terised, reports necessary to conduct surveys from
their employees.
CDS Solved Paper 2019 (II) 27

117. One major benefit of using survey 118. Which word in the passage means _ (a) The screens should not be
software packages is ‘tendency’? viewable to supervisors or passers by
in order to keep the surveys secret.
(a) reports can be generated easily (a) Trend (b) Breakdowns
(b) privacy of a person is exposed to (c) Convenient (d) Perceptions 120. What does the word ‘customised’
the supervisors means here?
_ (a) ‘Trend’ also means ‘tendency’. Both
(c) employees would like to take up words means ‘an inclination or likelihood’. (a) Adapted
the test on computer (b) Take as it is
(d) employer can get to know the 119. “The screens should not be viewable (c) Fixed
information immediately to supervisors or passers by.” Why? (d) Mass produced
(a) To maintain the secrecy of a person
_ (a) If companies use survey software (b) The main problem is to enable Ê (a) The word ‘customise’ also means
packages, they can generate reports
‘adapted’. Both word means ‘to
easily, can provide snapshots of a everyone to participate
modify according to a customer’s
given period of time, trend analysis and (c) The manager has to be fair enough
breakdowns according to various individual requirements’.
(d) To maintain the problems faced
demographics.
by women in job market

PAPER III General Studies


1. Which one of the following is the Home, Department of Official 4. Under which one of the following
motto of NCC? Languages and Department of States. Articles of the Constitution of
(a) Unity and Discipline Ministry of Home Affairs do not have India, a statement of estimated
Department of Legal Affairs under receipts and expenditure of the
(b) Unity and Integrity
it. It is under Ministry of Law and
(c) Unity and Command Justice. government of India has to be laid
(d) Unity and Service before the Parliament in respect of
3. Which of the following statements every financial year?
_ (a) National Cadet Corps (NCC) is is/are correct?
youth wing of armed forces with its (a) Article-110 (b) Article-111
headquarters at New Delhi (India). It is 1. India is a signatory to the United (c) Article-112 (d) Article-113
open to school and college students Nations Convention to Combat
on voluntary basis.
Desertification (UNCCD). _ (c) Article-112 in the Constitution of
India 1949 says
The discussion for motto of NCC was
2. Ministry of Home Affairs is the nodal The President shall in respect of every
started in 11th Central Advisory
Meeting (CAD) held on 11th August, ministry in the Government of India financial year cause to be laid before
1978, at that time there were many for the UNCCD. both the Houses of Parliament a
motto in mind like ‘duty and Select the correct answer using the statement of the estimated receipts
discipline’, ‘duty, unity and discipline’, and expenditure of the government
codes given below of India for that year, in this Part
‘duty and unity’. Later, at the 12th CAD
meeting ‘unity and discipline’ was (a) Only 1 (b) Only 2 referred to as the annual financial
selected as the motto of NCC. (c) Both 1 and 2 (d) Neither 1 nor 2 statement.
● Article 110 (3) of the Constitution of
2. Which one of the following _ (a) United Nations Convention to
Combat Desertification (UNCCD) is a India categorically states that if any
departments is not under the question arises whether a Bill is a
convention to mitigate the effects of
Ministry of Home Affairs? drought through national action Money Bill or not, the decision of the
(a) Department of Official Languages programs that incorporate long-term speakers of the house of the people
(b) Department of Border strategies supported by international thereon shall be final.
Management cooperation and partnership ● Article 111 of the Constitution of
(c) Department of Jammu and arrangements. India ‘Assent to Bills’ when a Bill has
Kashmir Affairs The convention was adopted in been passed by the Houses of
France (Paris) on 17th June, 1994 and Parliament, it shall be presented to
(d) Department of Legal Affairs
entered into force in December, 1996. the President and the President shall
_ (d) The Ministry of Home Affairs is a India became a signatory to UNCCD declare either that he assent to the
Ministry of the Government of India, it is Bill, or that he withholds assent there
on 14th October, 1994 and ratified it
mainly responsible for the from.
maintenance of internal security and on 17th December, 1996. Ministry of
domestic policy. The Ministry has 6 Environment, Forest and Climate ● According to Article 113 of the Indian
departments, namely– Department of Change is the nodal ministry for the Constitution estimates expenditure
Border Management, Department of convention, not Ministry of Home from the Consolidated Fund of India
Internal Security, Department of Jammu Affairs. in the Annual Financial Statement
and Kashmir Affairs, Department of Hence, option (a) is correct. are to be voted the Lok Sabha.
28 CDS Solved Paper 2019 (II)

5. The South Asian Association for expenditure at all levels of programme _ (c) Invest India was operationalised in
Regional Cooperation (SAARC) implementation. From 2014, it has been early 2010, as a joint venture company
was founded in envisaged that digitisation of accounts between the Department of Industrial
shall be achieved through PFMS. Policy and Promotion (DIPP) (35%
(a) Colombo (b) Islamabad Controller General of Accounts (CGA) equity), Federation of Indian Chambers
(c) Kathmandu (d) Dhaka is the Principal Accounting Adviser to of Commerce and Industry (FICCI)
Government of India in the (51% equity) and State Govenment of
_ (d) The South Asian Association for department of expenditure, Ministry of
Regional Cooperation (SAARC) is the India (0.5% each).
regional inter-governmental organisation Finance.
The core mandate of invest India is
in South Asia. investment promotion and facilitation.
8. Match List I with List II and select
SAARC was founded in Dhaka on 8th It provides sector-specific and
the correct answer using the codes state-specific information to a foreign
December, 1985. Its secretariat is
based in Kathmandu (Nepal). The given below the lists. investor, assists in expediting
organisation promotes development regulatory approvals and offers
List I (Institute) List II
of economic and regional integration. hand-holding services.
(Location)
Its member states include Afghanistan, Hence, both statements are correct.
A. National Institute 1. Chennai
Bangladesh, Bhutan, India, Maldives,
Nepal, Pakistan and Sri Lanka.
of Ayurveda 10. The National Dope Testing
B. national Institute of 2. Bengaluru Laboratory functions under
6. Which one of the following Homoeopathy (a) Ministry of Health and Family Welfare
countries is not a founding member C. National Institute 3. Kolkata (b) Ministry of Science and Technology
of the New Development Bank? of Unani Medicine (c) Ministry of Youth Affairs and Sports
(a) Brazil (b) Canada D. national Institute of 4. Jaipur (d) Ministry of Home Affairs
(c) Russia (d) India Siddha
_ (c) National Dope Testing Laboratory
_ (b) New Development Bank, formerly, Codes (NDTL) is a premier analytical testing
referred to as the BRICS Development A B C D A B C D and research organisation established
Bank, is a multilateral development (a) 1 2 3 4 (b) 1 3 2 4 as an autonomous body under the
bank established by the BRICS States (c) 4 3 2 1 (d) 4 2 3 1 Ministry of Youth Affairs and Sports,
(Brazil, Russia, India, China, South Government of India. It is the only
Africa). _ (c) National Institute of Ayurveda, laboratory in the country responsible
established in 1976 at Jaipur by the
The bank is headquartered in for human sports dope testing. It was
Ministry of Health and Family Welfare,
Shanghai (China). The first regional established in 2008 with an aim to get
Government of India. It is the lone
office of the NDB is in Johannesburg permanently accredited by International
institute of Ayurveda under the AYUSH.
(South Africa). Olympic Committee and world
National Institute of Homoeopathy
As Canada is not a member of BRICS, anti-doping agency to do the testing
is an autonomous organisation under
it is not a member of the New for the banned drugs in human sports.
Ministry of AYUSH, Government of
Development Bank. India. It was established on 10th Hence, option (c) is correct.
7. The Public Financial Management December, 1975 in Kolkata.
11. In how many phases was the general
System (PFMS) is a web-based National Institute of Unani Medicine
is an autonomous organisation for
election 2019 conducted in India?
online software application (a) 6 phases (b) 7 phases
research and training in Unani
designed, developed, owned and medicine in India. It was established in (c) 8 phases (d) 9 phases
implemented by the 1984 at Bengaluru. Under Ministry of
AYUSH.
_ (b) The 2019 Indian general election
(a) Department of Financial Services was held in 7 phases from 11th April to
(b) Institute of Government Accounts National Institute of Siddha is a 19th May, 2019 to constitute the 17th
institute for study and research of Lok Sabha. The votes were counted
and Finance
Siddha medicine. It was established in and result was declared on 23rd May,
(c) Controller General of Accounts 2005 at Chennai. 2019. The voting turnout was over 67%
(d) National Institute of Financial Hence, option (c) is the correct match. the highest ever as well as the highest
Management participation by women voters.
9. Which of the following statements
_ (c) The Public Financial Management about ‘Invest India’ is/are correct ? The BJP-led National Democratic Alliance
System (PFMS), earlier known as (NDA) won 353 seats whereas
Central Plan Schemes Monitoring 1. It is a joint venture (not for profit) Congress-led United Progressive Alliance
System (CPSMS), is a web-based company. (UPA) won 91 seats. Since, no party was
online software application developed able to secure at least 10% of seats of
and implemented by the office of
2. It is the National Investment
Promotion and Facilitation Agency total strength of Lok Sabha i.e. 55, there is
Controller General of Accounts (CGA).
of India. no opposition leader in 17th Lok Sabha.
PFMS was initially started during 2009
as a Central Sector Scheme of Select the correct answer using the 12. Which one of the following
Planning Commission with the codes given below statements about the Organisation
objective of tracking funds released of Islamic Cooperation is not correct?
(a) Only 1 (b) Only 2
under all plan schemes of Government
(c) Both 1and 2 (d) Neither 1 nor 2 (a) Its permanent secretariat is
of India and real time reporting of
located at Jeddah.
CDS Solved Paper 2019 (II) 29

(b) It endeavours to safeguard and 15. ‘Triples’ is a new format of 18. Why was India’s GS Lakshmi in news?
protect interests of the, Muslim
(a) Boxing (b) Judo (a) She was the first Indian to play
world in the spirit of promoting
(c) Chess (d) Badminton cricket for an English County
international peace and harmony
Club.
among various people of the _ (d) The world body governing
badminton, BFA has launched two (b) She became the first female ICC
world.
(c) It is the largest inter-governmental new formats of the game-‘Air match referee.
Badminton’ and ‘Triples’ with new (c) She was awarded the Ramon
organisation of the world.
dimension of the court and an Magsaysay Award for the year
(d) It has consultative and cooperative innovative shuttlecock called Airshuttle.
relations with the UN. 2019.
In triples format, the match will be
played between a team of three players (d) She was the recipient of the
_ (c) The Organisation of Islamic each with presence of at least one Booker Prize in the year 2019.
Cooperation (OIC) is an international
female. Traditional competitive
organisation founded in 1969,
badminton has been an indoor game.
_ (b) The ICC appointed India’s GS
consisting of 57 member states, with Lakshmi as first ever female match
53 being Muslim majority countries. But in the new format launched Air referee. Lakshmi was a match referee
The permanent secretariat is the Badminton will be outdoor sports. in domestic women’s cricket in
executive organ of the organisation, 2008-09 and has also overseen three
located in Jeddah (Saudi Arabia). 16. Who among the following was the women’s ODI matches and three
The organisation has consultative and
Chairman of the Committee on women’s T-20 matches in capacity of a
cooperative relations with the UN and Deepening Digital Payments match referee.
other inter-governmental organisations appointed by the RBI?
to protect the vital interest of the (a) HR Khan (b) Nandan Nilekani 19. Who among the following was
Muslims. It endeavours to safeguard (c) NR Narayana Murthy elected as the President of
and protect interest of the Muslim world (d) Sanjay Jain Indonesia for the second term?
in the spirit of promoting international (a) Joko Widodo
peace and harmony among various _ (b) The Reserve Bank of India had (b) Prabowo Subianto
people of the world. constituted a high-level committee on
Deepening of Digital Payments under (c) Sandiaga Uno (d) Jusuf Kalla
The OIC is the second largest
the Chairmanship of ‘Nandan _ (a) Joko Widodo (better known as
inter-government organisation in the
Nilekani’, former Chairman, UIDAI, in Jokowi) won a second term in office as
world after the UN.
January, 2019. SEBI constituted a Indonesian President. He got 55% of
Hence, option (c) is not correct. committee under the Chairmanship of votes against his opponent Prabowo
‘HR Khan’ (Retd Deputy Governor of Subianto. In 2014, also Jokowi won by a
13. Who among the following won the RBI) to review Foreign Portfolio slightly narrower margin. Voters
Italian Open Women’s Tennis Investors (FPI) regulations 2014. enduring support for Jokowi can be
Singles Title 2019? ‘Sanjay Jain’ and ‘HR Khan’ are partly explained his sensible policies.
(a) Karolina Pliskova Member of Committee on Deepening
of Digital Payments, not chairman. 20. In India, 21st May is observed as
(b) Johanna Konta
Committee on Deepening Digital (a) NRI Day
(c) Naomi Osaka (d) Serena Williams Payments submitted its report in May
(b) National Youth Day
2019.
_ (a) Karolina Pliskova won the Italian (c) National Technology Day
Open Women’s Tennis Singles Title
2019, defeating Johanna Konta in the 17. ‘The Sasakawa Award’ of United (d) National Anti-Terrorism Day
final. Naomi Osaka lost the tennis Nations is given in recognition of
_ (d) Every year 21st May is observed as
single title. the work done in the field of Anti-terrorism Day in India to wean
(a) disaster reduction away the youth from terrorism and
14. Which among the following IN ship(s) showing as to how it is prejudicial to the
(b) peace keeping
participated in the SIMBEX-19? national interest. The date is chosen to
(c) health services
1. INS Kolkata 2. INS Shakti (d) poverty alleviation commemorate the death anniversary
3. INS Vikrant of one of the most eminent PM of India,
_ (a) United Nations Office for Disaster ‘Rajiv Gandhi’ (20th August, 1941-21st
Select the correct answer using the Risk Reduction (UNDRR) conferred May, 1991).
codes given below ‘Sasakawa Award 2019’ for Disaster
(a) 1, 2 and 3 (b) 1 and 2 Reduction to Dr. Pramod Kumar 21. Arrange the following in the
(c) 2 and 3 (d) Only 1 Mishra, Additional Principal Secretary chronological order of their
to Prime Minister of India. implementation :
_ (b) SIMBEX (Singapore India Maritime
Bilateral Exercise) is an annual bilateral The award was announced at the 6th 1. The Indian Factory Act (First)
naval exercise between India and session of global platform for Disaster
2. The Vernacular Press Act
Singapore. On the Indian side, two Risk Reduction, 2019 at Geneva. The
vessels ‘INS Kolkata’ and ‘INS Shakti’ award was given in recognition of his 3. The Morley-Minto Reforms
participated in the exercise. long-term dedication to improve the 4. The Cornwallis Code
Meanwhile, Singapore was resilience of communities most Select the correct answer using the
represented by ‘RSN Steadfast’ and exposed to disasters. The UN
‘RSN Valiant’. INS Vikrant is an Sasakawa Award is the most
codes given below
Indigenous Aircraft Carrier 1 which is prestigious international award for (a) 4, 2, 1, 3 (b) 2, 4, 1, 3
expected to enter the service in 2023. disaster risk management. (c) 3, 4, 1, 2 (d) 2, 1, 3, 4
30 CDS Solved Paper 2019 (II)

_ (a) Option (a) is the correct sequence. Council and Southern Zonal Council. 26. The total number of members in
The Cornwallis code is a body of The North Eastern States are under
another statutory body, the North
the Union Council of Ministers in
legislation enacted in 1793 by the East
India Company to improve the Eastern Council. India shall, not exceed
governance of its territories India. (a) 10% of the total number of
The Vernacular Press Act (1878) was
24. Which provision of the
members of the Parliament
enacted by the Britishers to curtail the Constitution of India provides that (b) 15% of the total number of
freedom of Indian press and prevent the President shall not be answerable members of the Parliament
the expression of criticism towards to any Court in India for the (c) 10% of the total number of
British policies. exercise of powers of his office? members of the Lok Sabha
During Lord’s Ripon time, the ‘First (a) Article-53 (b) Article-74 (d) 15% of the total number of
Factory Act’ was adopted in 1881 to (c) Article-361 (d) Article-363
improve the service conditions of the members of the Lok Sabha
factory workers in India. The act _ (c) Article-361 of the Constitution of _ (d) The Union Council of Ministers
banned the appointment of children India provides that the President or the Exercises Executive Authority in the
below the age of seven in factories. It Governor of a State, shall not be Republic of India. It consists of
also reduced the working hours of answerable to any court for the senior ministers, called Cabinet
children. exercise and performance of the Ministers, Minister of State or Deputy
The Morley-Minto Reforms (1909) powers and duties of his office or for Ministers. According to the Constitution
was an act of the Parliament of the UK any act done or purporting to be done of India, the total numbers of ministers
that brought about a limited increase by him in the exercise and performance in the Council of Ministers must not
in the Involvement of Indians in the of these powers and duties. exceed 15% of the total numbers of
member of the Lok Sabha.
governance of British India. Article 53 Executive Power of the
22. Article-371A of the Constitution of Union The executive power of the 27. Which one of the following is the
union shall be vested in the President most noticeable characteristic of
India provides special and shall be exercised by him either
privileges to which states? the mediterranean climate?
directly or through officers
subordinate to him in accordance with (a) Limited geographical extent
(a) Nagaland (b) Mizoram
this Constitution. (b) Dry summer
(c) Sikkim (d) Manipur
Article 74 of the Constitution of the (c) Dry winter
_ (a) Article 371A of the Constitution of Republic of India provides for a (d) Moderate temperature
India provides special privileges to the
Council of Ministers which shall aid the
state of Nagaland with an aim to
President in the exercise of his
_ (b) Mediterranean climate is
preserve their tribal culture. characterised by dry summer, mild and
functions. wet winters. The climate receive its
Article 371 also confers special
Article 363 of the Constitution bars name from the mediterranean basin,
privileges to the state of Assam,
the jurisdiction of all courts in any where this climate type is most
Meghalaya, Sikkim, Manipur and
dispute arising out of any agreement common. Climate zones are located
Arunachal Pradesh. Article 371 gives
which was entered into or executed along the Western sides of continents,
the power to the President of India to
before the commencement of the between 30° and 45° North and South
establish separate development of the equator.
boards for Vidarbha, Marathwada Constitution by any ruler of an Indian
regions of Maharashtra and the rest of state to which the Government of India 28. Which one of the following rivers
the State and Saurashtra, Kutch and was party.
takes a ‘U’ turn at Namcha Barwa
rest of Gujarat. Special provisions with
respect to Andhra Pradesh,
25. Which law prescribes that all and enters India?
Karnataka, Goa are dealt in Articles proceedings in the Supreme Court (a) Ganga (b) Tista
371D and 371E, 371J, 371I shall be in English language? (c) Barak (d) Brahmaputra
respectively. (a) Article-145 of the Constitution of
India
_ (d) Brahmaputra originates on the
23. How many Zonal Councils were Angsi Glacier located on the Northern
set-up vide Part-III of the States (b) Article-348 of the Constitution of side of the Himalayas as Yarlung
India Tsangpo river and flows in Southern
Re-organisation Act,1956? Tibet to break through the Himalayas in
(c) The Supreme Court Rules, 1966
(a) Eight (b) Seven great Gorges.
(d) An Act Passed by the Parliament
(c) Six (d) Five Tsangpo (Brahmaputra) enters India

_ (d) Zonal Councils are Advisory _ (b) Article-348 of the Constitution of after taking a ‘U’ turn at Namcha
India prescribes language to be used Barwa and flows in Arunachal Pradesh
Councils and are made up of the
in the Supreme Court and in the High where it is known as ‘Dihang’ or
States of India that have been grouped
into five zones to foster cooperation Court and for acts, bills, etc. ‘Siang’ rivers.
among them. These were set-up vide All proceeding in the Supreme Court
Part III of the States Reorganisation and in every High Court, the
29. What was the Dutt-Bradley Thesis?
Act, 1956. authoritative texts of all bills to be (a) The Working Committee of the
The present composition of each introduced or amendment there to be Indian National Congress
these Zonal Council is under Northern moved in either House of Parliament decided that Congress should
Zonal Council, Central Zonal Council, or either House of State Legislature, play a crucial role in realising the
Eastern Zonal Council, Western Zonal shall be in English.
independence of India
CDS Solved Paper 2019 (II) 31

(b) The Socialist Party decided to play 32. Who among the following started 36. Which one of the following states
foremost part in anti-imperialist the Indian Agriculture Service? does not have a Legislative
struggle Council?
(a) Lord Curzon (b) William Bentinck
(c) Revolutionary socialist Batukeshwar
(c) Lord Minto (d) Lord Rippon (a) Karnataka
Dutt put forth a ten-point plan to
(b) Telangana
work for the success of _ (c) The Indian Agriculture Service was
started in 1906 under the All India (c) Jammu and Kashmir
anti-imperialist front
Board of Agriculture. It was during the (d) Arunachal Pradesh
(d) It was a Communist Party viceroyalty of Lord Minto. However, the
document, according to which the training of staff could not be taken out _ (d) Legislative Council is the Upper
National Congress could play a due to lack of funds and primary aim House in those States of India that
remained as merely education. have a bicameral legislature its
great part and a foremost part in establishment is defined in Article-169
realising the anti-imperialist 33. Chandimangal was composed in of the Constitution of India.
people’s front which one of the following languages As of 2019, 7 out of 29 States have
_ (d) Dutt-Bradley Thesis was a during the 16th century CE? Legislative Council namely—Andhra
Communist Party document, Pradesh, Bihar, Karnataka,
according to which the National
(a) Sanskrit (b) Tamil Maharashtra, Jammu and Kashmir,
Congress could play a great part and a (c) Bengali (d) Oriya Telangana and Uttar Pradesh.
foremost part in realising the
anti-imperialist people’s front.
_ (c) The Chandimangal is an important 37. What is SWAYAM?
subgenre of Mangal Kavya, the most
The Anti-imperialistic people’s front in significant genre of medieval Bengali (a) Study Webs of Active-Learning for
India written by Rajni Palme Dutt and literature. Young Aspiring Minds
Ben Bradley, popularly known as the The text belonging to this subgenre (b) Study Webs of Active-Learning for
Dutt-Bradley Thesis. praise Chandi or Abhaya, primarily a Youth Aspiring Minds
Anti-imperialist people are opposed to folk goddess, but subsequently identified (c) Study Webs of Active-Learning for
colonialism, colonial empires, with puranic goddess Chandi. Young Aspiration Minds
hegemony, imperialism and the This identification was probably (d) Study Webs of Active-Learning for
territorial expansion of a counter completed a few centuries before the
beyond its established borders. Youth of Aspiration Minds
earliest composition of the
Hence, option (d) is the correct Chandimangal Kavya. _ (a) ‘SWAYAM’ stands for Study Webs
answer. of Active-Learning for Young Aspiring
34. In December, 1962, which Soviet Minds. It is a programme of Ministry of
30. The Khuntkatti tenure was leader declared that China was Human Resource Development,
prevalent in which one of the responsible for the Sino-Indian Government of India. It provides
opportunities for life long learning.
following regions of India during War of 1962? Here, learner can choose from
the British Colonial Rule? (a) Khrushchev (b) Bulganin hundred of courses, virtually every
(a) Bundelkhand (b) Karnataka (c) Suslov (d) Malenkov course that is taught at the university,
(c) Chota Nagpur college, school level and these shall be
(d) Madras Presidency
_ (a) The Sino-Indian War, also known as offered by best of the teachers in India
the Indo-China war and Sino-Indian and elsewhere.
border conflict, was a war between
_ (c) Tribal people in Chota Nagpur area, China and India that occurred in 1962. 38. Which one of the following is not
which is present day in Jharkhand,
practised Khuntkatti Tenure (system) On 2nd October, Soviet leader Nikita enumerated in the Constitution of
(joint holding tribal lineages) till Khrushchev declared that China was India as a Fundamental Duty of
mid-19th century. responsible for the Sino-Indian War of
1962. citizens of India?
31. Who was the author of the book (a) To safeguard public property
‘Plagues and Peoples’? 35. Which of the following statements (b) To protect and improve the natural
with regard to the ‘Make in India’
(a) William H McNeil initiative is/are correct? environment
(b) WI Thomas (c) To develop the scientific temper
1. It was launched in the year 2018.
(c) Rachel Carson and spirit of inquiry
2. Its objective is to foster innovation.
(d) David Cannadine (d) To promote international peace
Select the correct answer using the
and security
_ (a) Plagues and Peoples is a book on codes given below
epidemiological history by William
(a) Only 1 (b) Only 2 _ (d) The Fundamental Duties of citizens
Hardy McNeil published in New York were added to the Constitution by the
City in 1976. It was a critical and (c) Both 1and 2 (d) Neither 1 nor 2 42nd Amendment in 1976, upon the
popular success, offering a radically recommendation of the Swaran Singh
_ (d) ‘Make in India’ initiative was lunched Committee that was constituted by the
new interpretation of the extra ordinary by Prime Minister Modi in September,
impact of infectious disease on culture 2014 as part of a wider set of nation government.
as a means of enemy attack. building initiatives. It aim to transform Under Article 51, it mentions to
The book ranges from examining the India into a global design and promote international peace and
effects of small pox in Mexico, the manufacturing hub. It covers twenty five security. It is not enumerated in the
bubonic plague in China, to the sectors of the economy. Constitution of India as Fundamental
typhoid epidemic in Europe. Hence, option (d) is correct. Duty of Citizens of India.
32 CDS Solved Paper 2019 (II)

39. Who among the following in his that they can spend. It is also a result without this acceleration, the object
of consumer spending as well as would move in a straight line,
book ‘The Managerial Revolution’ private saving. Personal Disposable according to Newton’s laws of motion.
argued that a managerial class Income = Personal Income − Direct Hence, the acceleration of the car is a
dominated all industrial societies, taxes paid by households and non-zero but not a constant.
both capitalist and communist, by miscellaneous fees, fines, etc.
virtue of its technical and scientific Hence, option (a) is correct. 45. An echo is heard after 5 s of the
knowledge and its administrative production of sound which moves
42. The working of the price mechanism with a speed of 340 m/s. What is the
skills? in a free-market economy refers to
(a) James Burnham
distance of the mountain from the
which one of the following? source of sound which produced the
(b) Robert Michels
(a) The interplay of the forces of echo?
(c) Gaetano Mosca
demand and supply.
(d) Vilfredo Pareto (a) 0.085 km (b) 0.85 km
(b) Determination of the inflation rate
(c) 0.17 km (d) 1.7 km
_ (a) James Burnham in his book ‘The in the economy.
Managerial Revolution’ argued that a (c) Determination of the economy’s _ (b) Given, speed of sound = 340 m/s
managerial class dominated all
industrial societies, both capitalist and propensity to consume.
communist, by virtue of its technical (d) Determination of the economy’s
d
and scientific knowledge and its full employment output. O Reflecting
administrative skills. v=340 m/s mountain
_ (a) Free market is an economic system t=5 s
40. Which one of the following based on supply and demand with little
or no government control. Let the distance of the mountain from
conditions laid down in the the source of sound is d.
Constitution of India for the issue Its depends on interplay of the forces
of demand and supply. To heard echo after 5 s, sound will
of a writ of Quo-Warranto is not travel 2d distance. Since Echo is
It is a summary description of all
correct? voluntary exchanges that take place in repection of sound which is reflected
(a) The office must be public and it a given economic environment. from any surface.
must be created by a statute Distance
∴ Speed =
(b) The office must be a substantive 43. Indexation is a method whose use Time
one can be associated with which one of 2d
(c) There has been a contravention of ⇒ 340 =
the following? 5
the Constitution or a state in
(a) Controlling inflation ⇒ d = 170 × 5 = 850 m = 0.85 km
appointing such person to that
(b) Nominal GDP estimation
office
(c) Measurement of savings rate
46. A 100 W electric bulb is used for 10
(d) The appointment is in tune with a h/day. How many units of energy are
(d) Fixing of wage compensation
statutory provision consumed in 30 days?
(d) Writ of ‘Quo-Warranto’ in literal _ (a) Indexation is a method or technique (a) 1 unit (b) 10 units
used by organisations or government
sense means by what authority or (c) 30 units (d) 300 units
to control Inflation by connecting
warrant it is issued by the court to
prices and asset value to inflation.
enquire into legality of claim of a _ (c) Given, power of electric bulb
person to public office. Hence, it This is done by linking adjustments = 100W
prevents illegal usurpation of public made to the value of a good, service or Duration used each day = 10h
office by a person. Hence, option (d) is another metric, to a predetermined Number of days = 30
not correct. index.
Total energy consumed by the electric
41. Which one of the following equals 44. A car undergoes a uniform circular bulb = Power × Duration
Personal Disposable Income? motion. The acceleration of the car is = 100 × 10 h/day × 30 days

(a) Personal Income − Direct taxes


(a) zero = 30000 = 30 kWh
(b) a non-zero constant So, energy consume is 30 units.
paid by households and
(c) non-zero but not a constant
miscellaneous fees, fines, etc. 47. Which of the following statements
(d) None of the above
(b) Private Income − Saving of Private relating to the Fifth Schedule of the
Corporate Sectors − Corporation _ (c) Circular motion is a movement of Constitution of India is not correct?
an object along the circumference of a
Tax circle or rotation along a circular path. It (a) It relates to the special provision
(c) Private Income − Taxes can be uniform with constant angular for administration of certain areas
(d) Total expenditure of households − rate of rotation and constant speed or
in the States other than Assam,
Income Tax gifts received non-uniform with a changing rate of
rotation. Meghalaya, Tripura and Mizoram.
_ (a) Personal Income measures the Since, the object’s velocity vector is (b) Tribal Advisory Councils are to be
income that is received by individuals, constantly changing direction, the constituted to give advice under
but not necessarily earned. moving object is undergoing the Fifth Schedule.
But personal disposable income acceleration by a centripetal force in (c) The Governor is not authorised, to
represents what people actually have the direction of the centre of rotation make regulations to prohibit or
CDS Solved Paper 2019 (II) 33

restrict the transfer of land by or 2. As provided in the Code of Civil 52. Which one of the following Indian
among members of the Procedure, High Courts have cities is not located on a river bank?
Scheduled Tribes. original appellate advisory
(a) Agra
(d) The Governors of the States in jurisdiction at the state level.
(b) Bhagalpur
which there are scheduled areas Which of the statement(s) given (c) Bhopal
have to submit reports to the above is/are correct? (d) Kanpur
President regarding the
(a) Only 1 (b) Only 2 _ (c) Agra is a city in the State of Uttar
administration of such areas. Pradesh in India situated on the banks
(c) Both 1 and 2 (d) Neither 1 nor 2
_ (c) The Fifth Schedule of the of river Yamuna.
Constitution deals with the _ (d) The Advocate General of a State in Bhagalpur is one of the important city
India is appointed by the Governor of a
administration and control of of the State of Bihar. It is situated on
State.
scheduled areas and Scheduled Tribe the banks of the river Ganga.
in any State except the four States of A High Court is primarily a Court of
Assam, Meghalaya, Tripura and Appeal. It hears appeals against the Kanpur is situated in State of Uttar
Mizoram. It says, Tribal Advisory Judgements of Subordinate Courts Pradesh, it is situated on Ganga river.
Councils are to be constituted to give functioning in its territorial jurisdiction. Hence, Bhopal is not situated on a
advice under the Fifth Schedule. It has appellate jurisdiction in both civil river bank.
and criminal matters.
The Governor is empowered to make 53. Where are Jhumri Telaiya and
regulations to prohibit or restrict the Under Act 226 original appellate Mandar Hills situated?
transfer of land by or among members jurisdiction under Article 226 of
of the Scheduled Tribes. Constitution, not civil procedure code. (a) Jharkhand
Hence, option (c) is not correct. Hence, both the statements are (b) Bihar
incorrect. (c) Assam
48. Consider the following statements (d) West Bengal
with regard to the formation of new 50. Which one of the following forms of
states and alteration of boundaries Constitution contains the features _ (*) Jhumri Telaiya is a city in Koderma
of both the Unitary and Federal district of Jharkhand (India).
of existing states. Geographically, it is located in the
Constitution? Damodar River valley. The city is
1. Parliament may increase the area
(a) Unitary (b) Federal known for Mica Production and Telaiya
of any state. dam. The dam was constructed on
(c) Quasi-Federal (d) Quasi-Unitary
2. Parliament may diminish the area River Barakar by the Damodar Valley
of any state. _ (c) Quasi-Federal forms of Corporation (DVC) and opened in the
Constitution contains the features of year 1953. It is a part of Damodar
3. Parliament cannot alter the both the unitary and Federal Valley Project.
boundary of any state. Constitution. Mandar Hill is a small mountain
4. Parliament cannot alter the name The Constitution of India has both the situated in Banka district under
of any state. unitary as well as federal features. Bhagalpur division of state of Bihar.
Which of the statements given Federal features are dual polity, This hill has many references in Hindu
above is/are not correct? written Constitution, division of mythology as Mandarachal Parvat.
powers, supremacy of the As per references found from Puranas
(a) 1 and 2 (b) 2 and 3 Constitution, Rigid Constitution, and Mahabharata this hill was used for
(c) 3 and 4 (d) 4 only independent judiciary, Bicameralism. ‘Samudra Manthan’.
Unitary features are strong centre,
_ (a) According to Article 3 of Indian single Constitution, flexibility of the 54. Which one of the following is not
constitutions a state has no say over
Constitution etc. correct regarding South India?
the formation of new states beyond
communicating its views to Parliament 51. Which one of the following Indian (a) Diurnal range of temperature is less
Article 3 assigns to parliament the States has no international (b) Annual range of temperature is less
power to enact legislation for the (c) Temperature is high throughout
boundary?
formation of new states.
(a) Bihar the year
Parliament may create new states in a (d) Extreme climatic conditions are found
number of ways-(a) separating (b) Chhattisgarh
territory from any state (b) uniting two (c) Uttarakhand _ (d) South Indian regions have a
or more states (c) uniting part of states (d) Meghalaya tropical climate with the monsoons
and (d) uniting any territory to a part of playing a major part. Diurnal range of
_ (b) India is the 7th largest country in the temperature is less in South India.
any state. world and it is the only country in the
Indian subcontinent to share its land Annual range of temperature is also
49. Consider the following statements: frontiers with every member country of due to proximity to equator.
1. The Advocate General of a State the subcontinent . Temperature is high throughout the
in India is appointed by the Afghanistan, Bhutan, Bangladesh, year. Climatic conditions are not
China, Myanmar, Nepal and Pakistan extreme here as they are in Northern
President of India upon the
are bordering countries of India. part of India.
recommendations of the Governor
Chhattisgarh do not have any Hence, option (d) is not correct.
of the concerned state.
international boundary.
34 CDS Solved Paper 2019 (II)

55. Which one of the following _ (c) Nayanars were a group of 63 saints 61. Which one of the following
statements regarding sex in the 6th to 8th century who were statements about the Government
devoted to the Hindu God Shiva in
composition is not correct? Tamil Nadu. of India Act, 1919 is not correct?
(a) In some countries, sex ratio is (a) It extended the practice of
They along with the Alwars, influenced
expressed as number of males the Bhakti movement in Tamil. The communal representation
per thousand females. names of the Nayanars were first (b) It made the Central Executive
(b) In India, sex ratio is expressed as compiled by Sundarar. responsible to the Legislature
number of females per thousands
males. 59. Match List I with List II and select (c) It is also known as the
(c) At world level, sex ratio is about the correct answer using the codes Montague-Chelmsford Reforms
102 males per 100 females. given below the lists. (d) It paved the way for federalism by
(d) In Asia, there is high sex ratio. clearly separating the
List II
List I
(Related responsibilities of the Centre and
_ (d) In some countries, sex ratio is (Ethnic Territorial
Occupational the Provinces
expressed as number of males per Segment)
Pattern)
thousand females. Whereas, in India, it _ (d) Government of India Act 1919, also
is expressed as number of females per A. Maruta Makkal 1. Pastoralists known as Montague-Chelmsford
thousand males. B. Kuravan Makkal 2. Fishing people Reforms. It extended the principle of
At world level, sex ratio is about 102 communal representation to Sikhs,
C. Mullai Makkal 3. Ploughmen
males per 100 females. Europe has Indian Christians, Anglo-Indians and
highest sex ratio in the world whereas D. Neytal Makkal 4. Hill people Europeans. It divided the provincial
Asia has lowest sex ratio. subjects into 2 parts transferred and
Codes reserved. Transferred subjects were to
Hence, option(d) is incorrect. A B C D be administered by the Governor with
56. Who among the following has given (a) 3 1 4 2 the aid of ministers responsible to the
(b) 2 1 4 3 Legislative Council.
the concept of Human Development?
(c) 3 4 1 2 Reserved subjects were to be
(a) Amartya Sen administered by the Governor and his
(d) 2 4 1 3
(b) Mahbub-ul-Haq Executive Council without being
(c) Sukhamoy Chakravarty _ (c) According to Tamil literature, the responsible to the Legislative Council.
(d) GS Chaddha basic unit of ethnic identification in Hence, option (d) is incorrect.
Dravidian culture was ‘Nadu’ and these
_ (b) The concept of Human were divided into five types on the 62. The concept of ‘Four Pillar State’,
Development was developed by basis of natural sub-region.
economist Mahbub-ul-Haq.
free from district magistracy for
The ethnic territorial segments are India was suggested by
Human Development is defined as the
Maruta Ploughmen (a) Lala Lajpat Rai
process of enlarging people’s
Makkal inhabiting fertile
freedom and opportunities and (b) Ram Manohar Lohia
land.
improving their well-being. (c) Raja Ram Mohan Roy
Kuravan Hill people who leave
57. Which one of the following regions Makkal the forest to work in (d) Subhash Chandra Bose
is an important supplier of citrus Panai. _ (b) Ram Manohar Lohia believes in
fruits? Mullai Makkal Pastoralist. decentralisation of economic and
Neytal Makkal Fishing people living political powers. So he gave the
(a) Equatorial region in coastal villages. concept of Four Pillar State, which is
(b) Mediterranean region based on the principle of division of
(c) Desert region Palai Makkal People of dry plains powers.
(d) Sub-humid region According to him four pillar constitute
60. Who among the following Mughal four limbs of the State. They are the
_ (b) Citrus fruits are the highest value emperors was a follower of the village, the district, the province and
fruit crop in terms of international trade. Naqshbandiyya leader, Khwaja the centre with sovereign powers.
They are produced all over the world. Ubaydullah Ahrar?
Mediterranean region is an important 63. Which one among the following is
supplier of citrus fruits. (a) Babur not a part of the Fundamental
Oranges account for the majority of (b) Humayun Rights (Part III) of the Constitution
citrus production, but the industry also (c) Akbar of India?
sees significant quantities of grape (d) Jahangir (a) Prohibition of traffic in human
fruits, pomelos, lemons.
_ (a) Mughal emperor Babur was the beings and forced labour
58. Who were the Nayanars? follower of Khwaja Ubaydullah Ahrar, a (b) Prohibition of employment of
member of Naqshbandi Sufi order.
(a) Those who were immersed in children in factories
This order was introduced in India
devotion to Vishnu by Khwaja Baqi Billah from the (c) Participation of workers in
(b) Those who were devotees of Buddha beginning the mystics of this order management of industries
(c) Leaders who were devotees of Shiva stressed on the observance of the (d) Practice any profession or to carry
(d) Leaders who were devotees of shariat and denounced all innovations on any occupation, trade or
or biddat.
Basveshwara business
CDS Solved Paper 2019 (II) 35

_ (c) Participation of workers in precipitation. Thus, option (a), (b) and _ (d) The cultivation of jute is mainly
management of industries is not a part (d) are correct. These areas have confined to the Eastern region of the
of the Fundamental Rights (Part III) of moderate temperature but country. The jute crop is grown in
the Constitution of India. experiences seasonal changes and nearly 83 districts of West Bengal,
Participation of workers in considerable valuation. Assam, Odisha, Bihar, Uttar Pradesh,
management of industries is directive Temperate coniferous forest biome Tripura and Meghalaya.
principle of State policy. has high (and not low) variation in Jute is not cultivated in Andhra
Fundamental Rights are enshrined in annual temperature. It ranges from an Pradesh significantly.
Part-III of the Constitution from average of −40°C in winters to 10°C in Hence, option (d) is not correct.
Article-12-35. Prohibition of traffic in summers.
human beings and forced labour is Hence, option (c) is not correct. 69. Consider the following statements:
Article-23. 1. According to Mahavamsa, Ashoka
66. Match List I with List II and select
Prohibition of employment of children turned to the Buddha’s dhamma
in factories is Article-24. Practice any the correct answer using the codes
when his nephew Nigrodha
profession, or to carry on any given below the lists. preached the doctrine to him.
occupation, trade business is Article
19(1)g. List I List II 2. Divyavadana ascribes Ashoka
(Peak) (Name of Hill) being drawn to the Buddha’s
64. Which one of the following is not A. Anamudi 1. Nilgiri teaching to the influence of
geographical requirement for B. Doddabetta 2. Satpura Ashokavandana Samudra, a
cultivation of cotton? C. Dhupgarh 3. Aravalli merchant-turned monk.
(a) Temperature reaching 25°C or D. Guru Shikhar 4. Annamalai 3. Dipavamsa speaks of Samudra, the
more in summer. Codes 12 year old son of a merchant, as
(b) Moderate to light rainfall. A B C D the key figure in Ashoka’s coming
(c) Medium laom soil with good (a) 3 2 1 4 under the influence of the
drainage. (b) 3 1 2 4 Buddhist dhamma.
(d) A growing period of at least 100 (c) 4 1 2 3 Which of the statements given
frost free days. (d) 4 2 1 3 above is/are correct?
_ (d) Cultivation of cotton requires a _ (c) Anamudi is the highest peak in the (a) Only 1 (b) Only 2
growing period of atleast 200 frost free Western Ghats in India. It is located at (c) 1 and 2 (d) 1 and 3
days, and not just 100 frost free days. junction of Cardamom hills,
Other requirements for cotton Annamalai hills and Palani hills. _ (a) According to Mahavamsa, Ashoka
turned to the Budha’s dhamma when
cultivation include, temperature Doddabetta is the highest peak in the his nephew Nigrodha preached the
reaching 25°C or more in summer, Nilgiri mountains, situated in Nilgiris doctrine to him.
moderate to light rainfall (55-100 cm), distric of TamilNadu.
medium loam soil with good drainage Divyavadana ascribes Ashoka being
and high water retention capacity. Dhupgarh is highest point on the drawn to the Budha’s teaching to the
Satpura mountains, situated in influence of Upagupta (a Buddhist
Rainfall during harvesting season is Panchmaehi, Madhya Pradesh. monk).
harmful for its production. India is the
largest producer of cotton in the world. Guru Shikhar, a peak in Rajasthan is Dipavamsa refers to three visits to
the highest point of the Aravalli range. the Sri Lanka by the Buddha, the
65. Which one of the following Hence, option (c) is correct. places being Kelaniya, Deegavapi
statements regarding temperate Raja Maha Viharaya. The Dipavamsa
67. Coral reefs are not found in which lauds the Theravada as a great
coniferous forest biome is not one of the following regions? banyan tree.
correct?
(a) Lakshadweep Islands Hence, statement 2 and 3 are
(a) They are characterised by very (b) Gulf of Kutch incorrect and 1 is correct.
little undergrowth. (c) Gulf of Mannar
(b) They have a growing period of 50 70. Name the site that gives us valuable
(d) Gulf of Cambay
to 100 days in a year. information about India’s maritime
(c) There is low variation in annual _ (d) In India, coral reefs are found in links on the Coromandel coast.
Andaman and Nicobar Islands, Gulf of
temperature. Kutch, Gulf of Mannar, Lakshadweep (a) Bharukachchha
(d) There is high range in spatial Islands, Gulf of Khambat etc. It is not (b) Karur
distribution of annual precipitation. found in Gulf of Cambay. (c) Arikamedu
Hence, option (d) is not correct. (d) Anuradhapura
_ (c) Temperate coniferous forest are
found in areas with cool winters, warm _ (c) Arikamedu gives us valuable
summers and abundant rainfall. Trees
68. In which one of the following states
information about India’s maritime
like spruce, pine and cedar grows is jute not significantly cultivated? links on Coromandel coast. It is
here, but they have little undergrowth. (a) Assam situated in South India’s Puducherry.
They have a growing period of 50 to (b) West Bengal Arikamedu was an important port of
100 days in a year. Rainfall is (c) Odisha Chola Kingdom. It also helps in trade
abundant but there is high range in (d) Andhra Pradesh with Roman people.
spatial distribution of annual
36 CDS Solved Paper 2019 (II)

71. Where are the largest quantity of 75. The rate of evaporation of liquid does region of their higher concentration, i.e.
aganist the diffusion or concentration
cichlids found in India? not depend upon
gradient. Active transport always
(a) Backwaters of Kerala (a) temperature requires cellular energy to achieve this
(b) Sunderbans (b) its surface area exposed to the movement.
(c) Narmada atmosphere
79. Chlorophyll in photosynthetic
(d) Godavari (c) its mass
prokaryotic bacteria is associated with
(d) humidity
_ (d) Cichlids are fish from the family (a) plastids
cichlidae in the order cichlid forums. _ (c) The rate of evaporation of liquid (b) membranous vesicles
They are the largest verteberate does not depend upon its mass,
because it is a surface phenomenon. (c) nucleoids (d) chromosomes
families in the world.
Factors affecting evaporation are:
They are found mostly in Africa and An increase in surface area
_ (b) Chlorophyll in photosynthetic
South America. They are foundless in prokaryotic bacteria is associated with
An increase in temperature
brackish and saltwater habitats. membranous vesicles. These organisms
Humidity
lack membrane bound organelles (such
They are largest in number in the
76. Rutherford’s alpha particle scattering as chloroplast). They have infoldings of
Godavari river of Indian subcontinent.
experiment on thin gold foil was the plasma membrane present in the
72. Which Greek philosopher coined responsible for the discovery of
scattered form in cytoplasm for
the term “Geography” in the 3rd attachment of chlorophyll. These
(a) electron (b) proton carryout photosynthesis.
century B.C.E.? (c) atomic nucleus (d) neutron
(a) Euclid (b) Plato 80. What do you mean by ‘Demographic
(c) Eratosthenes (d) Clio _ (c) Rutherford’s alpha particle scattering Dividend’?
experiment leads to the discovery of
_ (c) Geography derives from the Greek atomic nucleus. He proposed that, (a) A rise in the rate of economic growth
Word ‘geographia’, which would be ‘to there is a positively charged spherical due to a higher share of working
describe or write about the earth’. centre in an atom, called nucleus. age people in a population.
Eratosthenes (3rd century BCE) was 77. Food chain is (b) A rise in the rate of literacy due to
the first person to use the word development of educational
(a) relationship between autotrophic
geography. Geography is a field of institutions in different parts of the
science devoted to the study of lands, organisms
country.
features, in habitants and (b) exchange of genetic material
(c) A rise in the standard of living of
governance. between two organisms
the people due to the growth of
(c) passage of food (and thus energy)
73. Who is the author of the 16th from one organism to another
alternative livelihood practices.
century Sanskrit text, the Vraja (d) modern entrepreneur establishment
(d) A rise in the gross employment ratio
Bhakti Vilasa which focuses on the of a country due to government
providing food outlets
Braj region in North India? policies.
_ (c) Food chain is the linear sequence
(a) Todar Mal of transfer of matter and energy in the _ (a) ‘Demographic Dividend’ means a
(b) Narayana Bhatta form of food from organism to rise in the rate of economic growth due
organism. It begins from the producer to a higher share of working age
(c) Chaitanya
organisms, i.e. plants, and ends with people in a population.
(d) Rupa Goswami
decomposer species like bacteria, The change in working age people is
_ (b) Narayana Bhatta Goswami was fungi, etc. typically brought by a decline in fertility
Brahmin born in South India. Later, he and mortality rates.
came to Vraja and took initiation from 78. Which one of the following is active
Krishna Das Brahmachari. He wrote transport? 81. Which of the following organisms is
the 16th Century Sanskrit text Vraja (a) It is the movement of a substance responsible for sleeping- sickness?
Bhakti Vilasa which focuses on the Braj against a diffusion gradient with (a) Leishmania (b) Trypanosoma
region in North India. (c) Ascaris (d) Helicobacter
the use of energy from respiration
74. Bose-Einstein condensate is a (b) It is the movement of a substance
_ (b) Trypanosoma is responsible for
against a diffusion gradient sleeping-sickness. It is a vector borne
(a) solid state of matter
without the use of energy disease, transmitted to humans by the
(b) fifth state of matter
(c) It is the movement of a substance tse-tse fly. Its symptoms include, fever,
(c) plasma
against a diffusion gradient with headache, joint pains and poor mental
(d) state of condensed matter
the use of energy from and physical coordination.
_ (b) Bose-Einstein Condensate (BEC) photosynthesis
is a fifth state of matter. The BEC is 82. Which one of the following body
(d) It is the movement of a substance
formed by working a gas of extremely parts/organs of the human body does
along a diffusion gradient with the
low density, about one- hundred not have smooth muscles?
tons and the density of normal air to
use of energy from respiration
(a) Ureters (b) Iris of eye
super low temperatures. Other states _ (a) Active transport is the movement of (c) Bronchi of lungs
of matter : Solid, liquid, gas and molecules across a membrane, from a
region of their lower concentration to a (d) Biceps
plasma.
CDS Solved Paper 2019 (II) 37

_ (d) Amongst the options that are given, enzymes which are capable of 89. Which one of the following factors is not
‘biceps’ does not have smooth breaking down all organic material
present inside the cell.
considered in determining the Mini
muscles. Smooth muscles are
involuntary muscles as their During the disturbance in cellular
mum Support Price (MSP) in India?
functioning cannot be directly metabolism or when the cell gets (a) Cost of production
controlled by our own will. damaged, lysosomes may burst and (b) Price trends in international and
The muscle present in the biceps as the enzymes digest their own cell. domestic markets
well as triceps are called skeletal or Therefore, they are also known as (c) Cost of living index
striated or voluntary muscles. We can suicidal bags of a cell. (d) Inter-crop price parity
control the movements of these _ (c) In formulating Minimum Supports
muscles. 86. Which one of the following statements Price (MSP), the Commission for
with regard to economic models is Agricultural Cost and Prices (CACP)
83. What is Inter-cropping? not correct? considers
(a) It is the time period between two Cost of production
(a) They involve simplification of
cropping seasons. Changes in input prices
complex processes. Input-output price parity
(b) It is growing two or more crops in (b) They represent the whole or a part Trends in market prices
random mixture. of a theory. Demand and supply
(c) It is growing two or more crops in (c) They can be expressed only Inter-crop price parity
definite row patterns. through equations. Effect in industrial cost structure
(d) It is growing of different crops on a Effect on cost of living
(d) They help in gaining an insight into
piece of land in a pre-planned Effect on general price level
cause and effect. International price situation
succession.
_ (c) Economic model is a theoretical Parity between prices paid and prices
_ (c) Inter-cropping is growing two or construct representing economic received by the farmers
more crops simultaneously on the processes by a set of variables Effect on issue prices and implication
same field in a definite pattern. A few and a set of logical relationship for subsidy
rows of one crop alternate with a few between them. This can be expressed Hence, option (c) is not consider in
rows of a second crop, for example through equations, models etc. determing the MSP.
Soyabean + Maize.
They involve simplification of complex
The crops are selected such their processes. They help in gaining an
90. Which one of the following is not a
nutrient requirements are different. insight into cause and effect. dimension of the human development
This ensures maximum utilisation of index?
nutrients supplied and both the 87. The value of the slope of a normal (a) A long and healthy life
cultivations give better returns. demand curve is (b) Knowledge
84. Magnification is (a) positive (b) negative (c) Access to banking and other
(c) zero (d) infinity financial provisions
(a) actual size of specimen/observed
(d) A decent standard of living
size _ (b) In a market equilibrium, whenever
price of a commodity rises, the
(b) observed size of specimen/actual _ (c) Human Development Index is an
demand of commodity declines, in the index of life expectancy, education and
size
same way when prices fall, demand per capita income indicators.
(c) actual size of specimen-observed increases. So, demand and prices
size have negative relationship with each There are three dimensions of
other. index—Long and healthy life, knowledge
(d) observed size of specimen actual
and a decent standard of living.
size Therefore, the value of slope of a
normal demand curve is negative. Hence, option (c) is not a dimension of
_ (b) The ratio of the observed size of the Human Development Index.
specimen by a spherical mirror to the 88. Which one of the following is an
size of the actual, is called the linear
example of a price floor?
91. Gini Coefficient or Gini Ratio can
magnification by the spherical mirror, be associated with which one of the
it is denoted by m. (a) Minimum Support Price (MSP) for
following measurements in an
Jowar in India.
i.e. m=
I economy?
(b) Subsidy given to farmers to buy
O (a) Rate of inflation (b) Poverty index
fertilizers.
where, I = observed size of specimen (c) Income inequality
and O = actual size of the object. (c) Price paid by people to buy goods
(d) Personal income
from ration shops.
85. Which one of the following cell (d) Maximum Retail Price (MRP) _ (c) In economics, Gini Coefficient
organelles is known as ‘suicidal bags’ represents income or wealth
printed on the covers/packets of
distribution of a nation’s residents and
of a cell? goods sold in India. most commonly used to measure
(a) Lysosomes (b) Plastids income inequality.
_ (a) A price floor is a minimum price. It is
(c) Endoplasmic reticulum a regulatory tool. Price floor is defined It was developed by the Italian
(d) Mitochondria as an intervertion to raise market prices statistician and sociologist Corrado
if the government feels that price is too Gini and published in his 1912 paper
_ (a) Lysosomes are the cell organelles low MSP for Jowar or minimum wages ‘Variability and Mutability’.
which are known as ‘suicidal bags’ of a are example of price floor.
cell. They contain powerful hydrolytic
38 CDS Solved Paper 2019 (II)

92. Consider the following statements : _ (b) In order to separate the 2. The Vice-President tenders his
components of mixture, single or a resignation to the President of India.
1. Particles of matter intermix on combination of methods are used to
their own. separation of mixture can be done by 3. The Comptroller and Auditor
2. Particles of matter have force both physical and chemical process. General of India is removed from
acting between them. So, statement 1 is incorrect. his office in the like manner as the
Which of the statements given Dissolved sodium chloride can be President of India.
above is/are correct? separated from water by evaporation. 4. A Judge of the Supreme Court can
This method is used to separate a
(a) Only 1 (b) Only 2 resign his office by writing under
volatile component from a non-volatile
(c) Both 1 and 2 (d) Neither 1 nor 2 component of a mixture.
his hand addressed to the Chief
Justice of India.
_ (c) Particles of matter intermix on their So, statement 2 is correct.
own. Particles of matter attract each Select the incorrect answer using
other. The force of attraction, 96. Which one of the following option is the codes given below
responsible for keeping them close is incorrect?
inter-molecular force of attraction. (a) 1 and 2 (b) 2 and 3
(a) Elements are defined by the (c) 1, 2 and 3 (d) 1, 3 and 4
Hence, statement 1 and 2 both are
number of protons they possess.
correct.
(b) Isobars are atoms having the
_ (d) According to Constitution of India,
President can resign from his office at
93. Rate of evaporation increases with same atomic number, but any time by addressing the resignation
(a) an increase of surface area different mass number. letter to the Vice-President.
(b) an increase in humidity (c) The mass number of an atom is The Vice-President of India can resign
(c) a decrease in wind speed equal to the number of nucleons in anytime by writting to President of
(d) a decrease of temperature its nucleus. India.
(d) Valency is the combining capacity The Comptroller and Auditor General
_ (a) The evaporation is a surface of India (CAG) can be removed by the
phenomenon. The rate of evaporation of an atom.
President on the same grounds and in
increases on increasing the surface
area of the liquid. A substance that has _ (b) Atoms of different elements with same manner as a judge of Supreme
different atomic numbers, but same Court (not President).
a larger surface area will evaporate mass number are known as isobars.
faster, as there are more surface A Judge of Supreme Court can resign
molecules that are able to escape. Hence, option (b) is in correct. from his office by writting to President
of India.
94. If an object is at rest, then the time 97. If the speed of a moving magnet Hence, statements 1, 3 and 4 are
(X-axis) versus distance (Y-axis) graph inside a coil increases, the electric
incorrect and statement 2 is correct.
(a) is vertical (b) is horizontal current in the coil
(c) has 45° positive slope (a) increases (b) decreases 100. Rajya Sabha has exclusive
(d) has 45° negative slope (c) reverses jurisdiction in
(d) remains the same (a) creation of new states
_ (b) If an object is at rest, then the (b) declaring a war
time-distance graph is as below _ (a) If the speed of a moving magnet
Y inside a coil increases, the electric (c) financial emergency
current in the coil increases. Whenever, (d) authorising Parliament to legislate
Distance

a magnetic field and an electric on a subject in the state list


conductor move relative to one
another, so the conductor crosses _ (d) Due to its federal character, the
X lines of force in the magnetic field. Rajya Sabha has been given two
O
Time Hence, the current produced by exclusive or special powers that are not
Hence, graph is horizontal. enjoyed by the Lok Sabha.
electromagnetic induction is greater.
(i) It can authorise the Parliament
95. Consider the following statements to make a law on a subject
about mixture : 98. The frequency (in Hz) of a note that enumerated in the state list.
1. A substance can be separated into is one octave higher than 500 Hz is (ii) It can authorise the Parliament
to create new all India services
other kinds of matter by any (a) 375 (b) 750 common to both the Centre and
physical process. (c) 1000 (d) 2000 States.
2. Dissolved sodium chloride can be
_ (c) An octave or perfect octave is the 101. The term ‘soil impoverishment’ relates
separated from water by the interval between one musical pitch and
physical process of evaporation. another with double its frequency. to which one of the following?
Which of the statements given Hence, option (c) is correct. (a) Soil erosion
above is/are correct? (b) Soil deposition
99. Which of the following statements as (c) Soil getting very deficient in plant
(a) Only 1 per the Constitution of India are nutrients
(b) Only 2 not correct? (d) Soil getting enriched with plant
(c) Both 1 and 2
1. The President tenders his resignation nutrients
(d) Neither 1 nor 2
to the Chief Justice of India.
CDS Solved Paper 2019 (II) 39

_ (c) The term ‘soil impoverishment’ _ (a) Oxisols are very old soil and they Codes
relates to soil getting very deficient in are highly weathered. (a) Both the statements are individually
plant nutrients. Vertisols are very rich in organic true and statement II is the correct
Soil gets impoverished due to reasons matter. explanation of statement I
like : over-grazing mono-cropping, Histosols are rich in clay content and (b) Both the statements are
leaching, erosion, land use charge etc. highly basic. individually true, but statement II is
An impoverished soil leads to increase Entisols are soils which lack horizons. not the correct explanation of
in input cost of crop cultivation in the
statement I
form of fertilizers cost, pesticide cost, 104. Which one of the following (c) Statement I is true, but statement II
etc. Soil erosion refers to removal of mountains separates Black Sea and
top fertile layers of the soil. Erosion is false
may be due to natural reasons (like :
Caspian Sea? (d) Statement I is false, but statement
river erosion, glacial erosion etc.) or (a) Urals (b) Caucasus II is true
anthropogenic factor (like : mining (c) Carpathians
activities, urbanisation, etc). Usually, (d) Balkan mountains 107. Statement I The Greek travellers
soil erosion leads to soil were most impressed by the fertility
impoverishment. _ (b) The Caucasus mountains are a of India’s soil and the energy and
mountain system at the intersection of
102. Which one of the following is the Europe and Asia stretching between ability of her cultivators.
correct sequential phase in the Black sea and Caspian sea. It is home Statement II Ancient India knew
to mount Elbrus, highest peak in
successional development of Europe. the use of manure.
vegetation community in a habitat?
It includes greater Caucasus in the _ (a) Ancient India knew the use of
(a) Migration, reaction, stabilisation North and lesser Caucasus in the manure. Because of this Greek
and nudation South. travellers were most impressed by the
(b) Migration, stabilisation, reaction fertility of India’s soil and the energy
and nudation 105. Rains caused by thunderstorms and ability of India’s cultivators.
(c) Nudation, migration, reaction and during the hot weather season From Maurya period to Gupta period,
stabilisation (mid-March to mid-June) in the arrival of Greeks in India is
(d) Reaction, migration, stabilisation Karnataka are called common.
and nudation Greek travellers visited India and
(a) Kalbaisakhi
impressed by India’s natural beauty in
_ (c) The successional development of (b) Mango showers terms of plants, animals, minerals.
vegetation community in a habit at (c) Loo
Hence, both the statements are
depends on many factors. In the (d) Cherry blossoms correct and statement II is correct
begining an area is bare, with time explanation of statements I.
there begins migration of people to that _ (b) These pre-monsoon rains are as
called ‘Mango Showers’. Mango
area. This is followed by reaction of showers are common in state of 108. Statement I Non-cooperation
people to changing environment Kerala, Karnataka, Maharashtra and began in Punjab with the student
condition and lastly people try to some part of Tamil Nadu. movement inspired by Lala Lajpat
stabilise in a given area with certain set
These showers arrive generally in late Rai in January 1921.
of climatic condition. Therefore, the April and May and are usually very
correct sequence is nudation, difficult to predict.
Statement II The Sikh dominaed
migration, reaction and stabilisation. central Punjab countryside was
106. Which one of the following is the stirred by the powerful Akali upsurge.
103. Match List I with List II and select largest fresh water lake in India?
the correct answer using the codes _ (d) Non-cooperation movement was
(a) Chilika (b) Loktak started to demand Swaraj from
given below the lists. Britishers. The main goal is not
(c) Dal (d) Wular
List I Lilst II cooperate with British government and
(Soil Type) (Major Characteristic)
_ (d) Wular lake is the largest fresh water to create barriers in their daily work.
lake in India. It is sited in Bandipura Gandhiji started non-cooperation
A. Oxisols 1. Very rich in organic district in Jammu and Kashmir. The movement on 1st August, 1920 and
matter lake basin was formed as a result of ended till Febraury, 1922. The Sikh
B. Vertisols 2. Soil lacking horizons tectonic activity and is fed by the dominated central Punjab countryside
Jhelum River. was stirred by the powerful Akali
C. Histosols 3. Very old and highly
upsurge. Hence, statement I is false, but
weathered The area of this lake is seasonally from
statemen II is true.
30 to 260 sq. km.
D. Entisols 4. Rich in clay content
and highly basic 109. Statement I The Oudh Kisan
Directions (Q. Nos. 107-110) The Sabha established in 1920 failed to
Codes following items consists of two bring under its wing any Kisan Sabhas.
A B C D
statements, Statement I and Statement Statement II The Oudh Kisan
(a) 3 1 4 2
II. Examine these two statements Sabha asked the Kisans to refuse to
(b) 3 4 1 2
carefully and select the correct answer till Bedakhli land, not to offer hari
(c) 2 1 4 3
using the codes given below. and begar.
(d) 2 4 1 3
40 CDS Solved Paper 2019 (II)

_ (d) In 1917, Jawaharlal Nehru, Madan 113. In graphite, each carbon atom is 117. Consider the following statements :
Mohan Malviya and Gauri Shankar bonded to three other carbon atoms
Mishra etc. established Uttar Pradesh ‘‘Atomic number of an element is a
Farmers Organisation, but due to (a) forming a three-dimensional structure more fundamental property than
Khilafat Movement, there were (b) in the same plane giving a its atomic mass.’’ Who among the
disputes among farmers leaders. This hexagonal array following scientists has made the
resulted in creation of Awadh farmers (c) in the same plane giving a square array
organisation in 1920 under (d) in the same plane giving a
above statement?
Ramchandra. Awadh farmers were pentagonal array (a) Dmitri Mendeleev
asked to refuse to till Bedakhli land by (b) Henry Moseley
Kisan organisations. The Oudh Kisan _ (b) In graphite, each carbon atom is
bonded to three other carbon atoms in (c) JJ Thomson
Sabha established in 1920 was not
failed to bring any Kisan Sabha under the same plane given a hexagonal (d) Ernest Rutherford
its wing. Hence, statement I is false, but array. The fourth electron of each
carbon atom is free. _ (b) Given statement was stated by
statement II is true. Henry Moseley. Mendeleev’s periodic
table was, therefore accordingly
110. Statement I The united provinces 114. Soap solution used for cleaning modified.
during non-cooperation became purpose appears cloudy. This is due
one of the strongest bases of the to the fact that soap micelles can 118. Which one of the following acids is
Congress. (a) refract light (b) scatter light also known as vitamin-C?
Statement II The literary outcrop (c) diffract light (d) polarise light (a) Methanoic acid
of non-cooperation in Bengal was (b) Ascorbic acid
_ (b) Soap solution used for cleaning
quite meagre compared to the days purpose. When soap is mixed in water, (c) Lactic acid
of the Swadeshi agitation. a colloidal solution is formed. The soap (d) Tartaric acid
solution has soap micelles which are
_ (b) Non-cooperation movement an aggregate of soap molecules. _ (b) Vitamin-C is known as ascorbic
started under the guidance of These micelles are large and they acid. It is found in all citrus fruits. It’s
Mahatma Gandhi. As Swadeshi scatter light. That is why the soap deficiency in diet comes scurvy.
agitation began due to Partition of solution appears cloudy.
Bengal, many people from Bengal
119. Which one of the following is not
participated in the movement, but the 115. People prefer to wear cotton clothes found in animal cells?
literary outcrop of non-cooperation in in summer season. This is due to the (a) Free ribosomes
Bengal was quite meagre. Hence, both fact that cotton clothes are (b) Mitochondria
the statements are correct.
(a) good absorbers of water (c) Nucleolus
111. Who were Alvars? (b) good conveyors of heat (d) Cell wall
(a) Those who immersed in devotion (c) good radiators of heat _ (d) Animal cell does not possess cell
to Vishnu (d) good absorbers of heat wall. It is present in the plant cells,
(b) Devotees of Shiva outside the plasma membrane as a
_ (a) People prefer to wear cotton rigid outer covering. It maintains the
(c) Those who worshipped abstract clothes in summer season to keep shape of the cell and protects it. Cell
form of God them cool and comfortable. In wall is also present in fungi, algae and
(d) Devotees of Shakti summer, we sweat more, cotton being bacteria.
a good absorber of water helps in
_ (a) The Alvars, were Tamil Poets– absorbing the sweat and exposes it to 120. Marsilea, fern and horse-tail are
Saints of South India who favour the atmosphere for evaporation.
support Net Bhakti to the Hindu God examples of which one of the
So, we can say that cotton clothes are following plant groups?
Vishnu or his Avatar Krishna in their
good absorbers of water.
songs of longing, ecstasy and service. (a) Pteridophyta
112. Which one of the following is mono 116. Employing chromatography, one (b) Bryophyta
atomic? cannot separate (c) Gymnosperms
(a) radioisotopes (d) Angiosperms
(a) Hydrogen (b) Sulphur
(b) colours from a dye
(c) Phosphorus (d) Helium _ (a) Marsilea, fern and horse-tail are
(c) pigments from a natural colour examples of division–Pteridophyta of
_ (d) Elements Atomicity (d) drugs from blood plant kingdom. In this group, the plant
Hydrogen Diatomic body is differentiated into roots,
Sulphur Polyatomic _ (a) Chromatography is the technique stem and leaves and has
Phosphorus Tetra atomic for the separation of those solvents, specialised tissue for the conduction
that dissolve in the same solvent. of water and other substances
Helium Mono atomic
Radioisotopes cannot be dissolve in from one part of the plant body to
Hence, helium is mono atomic among the same solvent to separate it. another.
given options.
CDS Solved Paper 2019 (I) 41

CDS
Combined Defence Service

SOLVED PAPER 2019 (I)


PAPER I Elementary Mathematics

1. What is the remainder when number is divisible by 9 and it is In second step 9 ´ b = 9 + (unit digit 3)
(17 29 + 19 29 ) is divided by 18? given that the last digit of the b=6
number is odd? 9 9 9
(a) 6 (b) 2 ´ a 6 8
(c) 1 (d) 0 (a) 5 (b) 6 7992
29 29 (c) 9 (d) 11 5994´
17 + 19 ´ ´
_ (d) =
18 _ (a) Divisible rule of 9 def 132
Sum of digit of the numbers is divisible Similarly, a=8
(18 - 1)29 (18 + 1)29
= + by 9. 999
18 18 ´868
The number = 479865 AB
\ Remainder = ( - 1)29 + (1)29 7992
Sum of the digit
= - 1+ 1 5994´
=4+7 + 9+ 8+ 6+ 5+ A+ B
=0 7992´´
= 39 + A + B
867132
2. What is the largest value of n such Pairs ( A, B) also the number is odd.
The values of a, b, c, d , e
that 10n divides the product 45 and 54 are two numbers divisible and f = 8, 6, 8, 8, 6, 7
25 ´ 3 3 ´ 4 8 ´ 5 3 ´ 67 ´ 7 6 ´ 812 by 9. Option (d) is correct.
´9 9 ´ 10 6 ´ 1512 ´ 2014 Pairs [(1, 5), (5, 1), (3, 3), (6, 9), (8, 7)]
5. Three cars A, B and C started from
Option (a) is correct.
´2211 ´ 2515 ? a point at 5 pm, 6 pm and 7 pm,
(a) 65 (b) 55 4. Consider the multiplication respectively and travelled at
(c) 50 (d) 45 999 ´ abc = def 132 in decimal uniform speeds of 60 km/h, 80
notation, where a, b, c , d , e and f km/h and x km/h, respectively in
_ (a) are digits. What are the values of
25 ´ 3 3 ´ 4 8 ´ 5 3 ´ 67 ´ 7 6 ´ 812 ´ 99 the same direction. If all the three
a, b, c , d , e and f , respectively? meet at another point at the same
´10 6 ´ 15 12 ´ 20 14 ´ 22 11 ´ 25 15 (a) 6, 6, 8, 6, 8, 7 (b) 8, 6, 8, 6, 7, 8 instant during their journey, then
10 makes by = (2 ´ 5) (c) 6, 8, 8, 7, 8, 6 (d) 8, 6, 8, 8, 6, 7 what is the value of x?
Find the number of five (5) in the
_ (d) 999 ´ abc = def 132 (a) 120 (b) 110 (c) 105 (d) 100
product 999
5 3 ´ 10 6 ´ 15 12 ´ 20 14 ´ 25 15 ´ a b c _ (a) Speed of car A = 60 km/h
Number of 5 def 132 Speed of car B = 80 km/h

3 + 6 + 12 + 14 + 30 = 65 In first step 9 ´ c = unit digit 2 Speed of car C = x km/h


c=8 Cars A, B and C started from a point
[Q (10)6 = (2 ´ 5)6 6 five in this product] 9 9 9 (M )
´ a b 8
10 65 will divide the product n = 65 Car A started at 5 pm from point M
7 9 9 2
3. How many pairs ( A, B ) are possible ´ distance coverd by car A in 1h = speed
def 132 ´ time = 60 ´ 1 = 60 km
in the number 479865AB if the
42 CDS Solved Paper 2019 (I)

Then, distance between point M and N 7. Which of the following statements 1 2p - q


(a) ± , 2p ¹ q
= 60 km p q
is not true?
Car B started at 6 pm from point ( M ) 1
(a) The difference of two prime (b) ± p - q, p ¹ q
In same direction numbers, both greater than 2, is pq
Relative speed of car A and car B divisible by 2 p
(c) ± p - q, p ¹ q
= 80 - 60 = 20 km/h (b) For two different integers m, n and q
Time taken by car B meet to car A a prime number p, if p divides the q
(d) ± 2 p - q , 2p ¹ q
=
60
= 3h
product m ´ n, then p divides p
20 either m or n
1 + px 1 - qx
é Distance ù (c) If a number is of the form _ (a) =1
êQ Time = Speed ú 6n - 1 (n being a natural number),
1 - px 1 + qx
ë û
then it is a prime number 1 + px 1 + qx
After three hour car B meet car A at =
point Z at 9 : 00 pm. (d) There is only one set of three prime 1 - px 1 - qx
The distance of M to Z numbers such that there is a gap On squaring both sides, we get
= 3 ´ 80 = 240 km of 2 between two adjacent prime (1 + px )2 1 + qx
numbers =
Car C started at 7 : 00 pm (1 - px )2 1 - qx
[from point (M)] _ (c) Every prime number is of the form Use Componendo and Dividendo
( 6n - 1) or ( 6n + 1) but every number
Car C has to travel 240 km in 2 h to rule,
which is of form ( 6n - 1) or ( 6n + 1) not
meet car A and car B.
necessarily prime. (1 + px )2 + (1 - px )2
240
Speed of car C = = 120 km/h Examples showing ( 6n - 1) (1 + px )2 - (1 - px )2
2
Let n = 6 ( 6 ´ 6 - 1) = 36 - 1 = 35 1 + qx + 1 - qx
Option (a) is correct. =
(It is not a prime number) 1 + qx - 1 + qx
éQ Speed = Distance ù
êë Let n = 4 ( 6 ´ 4 - 1) = 24 - 1 = 23 Þ
Time úû
(It is a prime number) (1 + p2 x 2 + 2 px ) + (1 + p2 x 2 - 2 px )
6. Priya’s age was cube of an integral So, option (c) is correct answer. (1 + p2 x 2 + 2 px ) - (1 + p2 x 2 - 2 px )
number (different from 1) four 1 + qx + 1 - qx
8. For x > 0, what is the minimum =
years ago and square of an integral 1 + qx - 1 + qx
x +2
number after four years. How long value of x + ? [Q( a + b )2 = a 2 + b 2 + 2 ab,
should she wait so that her age 2x
(a) 1 ( a - b )2 = a 2 + b 2 - 2 ab]
becomes square of a number in the
(b) 2 2 + 2p x 2 2
2
previous year and cube of a 1 Þ =
number in the next year? (c) 2 4 px 2qx
2
(a) 7 yr (b) 12 yr (d) Cannot be determined 1 + p2 x 2 1
Þ =
(c) 14 yr (d) 21 yr x +2 2 px qx
_ (c) x + ,x > 0 …(i)
_ (c) Let say Priya age 4 yr ago = x yr 2x Þ 1 + p2 x 2 =
2p
Þ p2 x 2 =
2p
-1
Current age = x + 4 yr =x +
x
+
2 1
=x + +
1 q q
Age after 4 yr= x + 4 + 4 = x + 8 2x 2x 2 x 2p - q
x is cube and x + 8 is an square \ px = ±
Differentiation of the Eq. (i), we get q
8 and 16 are possible solution.
1 1 2p - q
Cube ( x )3 : 8, 27, 64, 125 0 = 1+ 0 - x =±
3 x2 p q
( x + 8) : 16, 35, 72, 133
1
Square : 4, 9, 16, 25, 36, 49, 64, 81, = 1 Þ x2 = 1 Option (a) is correct.
x2
100 is square
x =±1 10. In a hostel the rent per room is
So, x = 8 and x + 8 = 16 increased by 20%. If number of
x = 1 ( x > 0)
So, present age = 12 yr rooms in the hostel is also
On putting x = 1 in Eq. (i), we get
Her age becomes square of the increased by 20% and the hostel is
(1) + 2
number in the previous year and cube = (1) + always full, then what is the
or the number in next year. 2(1)
percentage change in the total
Þ Cube = square +2 3 5
= 1+ = = 2
1
collection at the cash counter?
Cube = 27 2 2 2
Square = 25 Option (c) is correct.
(a) 30% (b) 40% (c) 44% (d) 48%
Age = 26 yr
9. 1 + px 1 - qx _ (c) In a hostel the rent per room is
26 - 12 = 14 yr If = 1, then what increased by 20%.
Then, she has to wait for 14 yr 1 - px 1 + qx The number of rooms in the hostel is
also increased by 20%.
Option (c) is correct. are the non-zero solutions of x?
CDS Solved Paper 2019 (I) 43

Then, the percentage change in the 13. It is given that log10 2 = 0301
. and P1 - 43 = 36
total collection at the cash counter
log10 3 = 0.477. How many digits P1 = 36 + 43 = 79
a´b
=a+ b+ are there in (108)10 ? On putting in Eq. (i), we get
100
\ a = b = 20% P1 + P2 + P3 = 100
(a) 19 (b) 20
20 ´ 20 (c) 21 (d) 22 79 + 43 + P3 = 100
= 20 + 20 + = 44%
100 122 + P3 = 100
_ (c) Given, log 10 2 = 0.301
Option (c) is correct. P3 = - 22
log 10 3 = 0.477
11. Radha and Hema are neighbours (It is not a prime)
x = (108)10
and study in the same school. Both Then, option (d) is correct answer.
of them use bicycles to go to the On taking log both sides, we get
15. If a, b and c are positive integers
school. Radha’s speed is 8 km/h log x = log 108 10 1 16
whereas Hema’s speed is 10 km/h. such that = , then
log x = 10 log 108 1 23
Hema takes 9 min less than Radha a+
log x = 10 [log(2 2 ´ 3 3 )] 1
to reach the school. How far is the b+
[Q108 = 2 2 ´ 3 3 ] 1
school from the locality of Radha c+
and Hema? log x = 10 [log 2 2 + log 3 3 ] 2
(a) 5 km (b) 5.5 km what is the mean of a, b and c?
[Q log( a ´ b ) = log a + log b]
(c) 6 km (d) 6.5 km (a) 1 (b) 2
log x = 10 [2 log 2 + 3 log 3]
(c) 1.33 (d) 2.33
_ (c) Radha’s speed = 8 km/h [Q log a b = b log a]
1 16 16
Hema’s speed = 10 km/h _ (b) = Þ
log x = 10 [2 ´ 0.301 + 3 ´ 0.477 ] 1 23 23
a+
Let the distance of the school from the log x = 10 ´ 2.033 = 20.33 b+
1
locality of Radha and Hema = x km 1
On taking antilog both sides, we get c+
According to the question, 2
x = e 20 . 33
9 é Distance ù 1 1
x
-
x
= Q Time = = =
8 10 60 êë Speed úû
Number of digit are there is 23/16 1 + 7
= 20 . 33 » 21 16
10x - 8x 3 é
hour ù
1
= Q1 minute = Option (c) is correct. 1 1
8 ´ 10 20 êë 60 úû = =
1 1
14. The sum of three prime numbers 1+ 1+
2 x = 12 , x = 6 km 16 / 7 2
is 100. If one of them exceeds 2 +
Option (c) is correct. 7
another by 36, then one of the
1 1
12. Which of the following pair of number is = =
1 1
numbers is the solution of the (a) 17 (b) 29 1+ 1+
1 1
2 + 2 +
equation 3 x + 2 + 3 - x = 10? (c) 43 (d) None of these
7/ 2 1
3+
(a) 0, 2 (b) 0, -2 _ (d) The sum of the three prime number 2
(c) 1, -1 (d) 1, 2 ( P1 + P2 + P3 ) = 100 …(i) Then, a = 1, b = 2, c = 3
x +2 -x Solve by option a+ b+c
_ (b) 3 +3 = 10 Mean of a, b and c =
1
2 P1 - P2 = 36 …(ii) 3
Þ 3 ×3 +
x
= 10 1+ 2 + 3
3x Option (a) = =2
3
Let 3 x = y, then P2 = 17
Option (b) is correct.
1 P1 - 17 = 36
Þ 9y + = 10
P1 = 36 + 17
y Directions (Q. Nos. 16-18) Read the
Þ 2
9 y + 1 = 10 y P1 = 53
given information carefully and
From Eq. (i), we get
Þ 9 y 2 - 10 y + 1 = 0 answer the given questions below.
P1 + P2 + P3 = 100
Þ 9 y2 - 9 y - y + 1 = 0 In a certain town of population size
53 + 17 + P3 = 100
100000 three types of newspapers (I, II
Þ 9 y( y - 1) - 1( y - 1) = 0 P3 = 30 (It is not a prime) and III) are available. The percentages of
Þ ( y - 1) ( 9 y - 1) = 0 Option (b) the people in the town who read these
1
\ y = 1, P2 = 29 papers are as follows.
9
P1 - 29 = 36 Newspaper Proportion of readers
when 3 x = 1 Þ 3 x = 3 0 Þ x = 0
P1 = 36 + 29
1 I 10%
when 3 x = Þ 3 x = 3 -2 Þ x = -2 P1 = 65 (It is not a prime)
9 II 30%
Option (c)
So, x = 0, - 2
P2 = 43 III 5%
44 CDS Solved Paper 2019 (I)

Both I and II 8% 19. What is the unit place digit in the 4 6


( 3 - 2 )+2
2
Both II and III 4% expansion of 7 73 ? =
4 6
( 3 - 2 )-2
Both I and III 2% (a) 1 (b) 3 2
(c) 7 (d) 9 4 6
All the three 1% ( 3 - 2 )+2
(I, II and III) _ (c) The unit place digit = 7 + 3
4 6
é7 1 ( 3 - 2) -2
= 7ù 3
16. What is the number of people who ê ú
read only one newspaper? ê7 2 = 9ú
=
4 3 ( 3 - 2 )+2
ê 3 ú 4 3 ( 3 - 2 )-2
(a) 20000 (b) 25000 ê7 = 3ú
ê 4 ú 4 2 ( 3 - 2 )+2
(c) 30000 (d) 35000 = 1ú +
ê7 4 2 ( 3 - 2 )-2
_ (a) Percentage of people who read ê7 5 = 7ú
ë û 4´3-4 6+2 4 6-8+2
only one newspaper. = +
= I + II + III - 2 [I and II + II and III Then, cyclicity of 7 is 4 4´ 3- 4 6 -2 4 6 - 8-2
73 14 - 4 6 4 6-6
+ I and III] + 3 [I, II and III] 73 = +
7 73 = 74 Þ remainder = = 1= 71 10 - 4 6 4 6 - 10
= 10 + 30 + 5 - 2 [8 + 4 + 2 ] + 3(1) 4
= 45 - 28 + 3 = 20% unit digit. 7 -2 6 3-2 6
= +
Population of town = 100000 Option (c) is correct. 5-2 6 5-2 6
Number of people who read only one [negative sign common in both]
20. Suppose n is a positive integer
20 10 - 4 6 2 (5 - 2 6 )
newspaper = 100000 ´
100
= 20000 such that (n 2 + 48) is a perfect = = =2
5-2 6 5-2 6
Option (a) is correct. square. What is the number of
17. What is the number of people who such n? Option (d) is correct.

read atleast two newspapers? (a) One (b) Two (c) Three (d) Four 22. x , y and z are three numbers such
(a) 12000 (b) 13000 2
_ (c) x = n + 48 that x is 30% of z and y is 40% of z.
(c) 14000 (d) 15000 If x is p% of y, then what is the
On squaring both side, we get
value of p?
_ (a) Percentage of people who read x 2 = n2 + 48
atleast two newspapers. (a) 45 (b) 55
= I and II + II and III + I and III - 2 x 2 - n2 = 48 (c) 65 (d) 75
[I, II and III] ( x - n) ( x + n) = 48 _ (d) x , y and z are three numbers,
= 8 + 4 + 2 - 2(1) = 14 - 2 = 12% n is an integer 30 3z
Number of people who read atleast x = ´ z Þ x = … (i)
x -n x+n 100 10
two
40 4z
12 2 24 y= ´ z Þ y= …(ii)
newspapers = 100000 ´ = 12000 100 10
100
Option (a) is correct. 4 12 x =
p
´ y
100
18. What is the number of people who 6 8
3z
do not read any of these three Option (c) is correct. x ´ 100 10
P= = ´ 100
newspapers? y 4z
21. 4 6
(a) 62000 (b) 64000 For x = , what is the value 10
(c) 66000 (d) 68000 2+ 3 [from Eqs. (i) and (ii)]
x +2 2 x +2 3 3 ´ 100
_ (d) The percentage of people who of + =? = = 75%
read newspaper 4
x -2 2 x -2 3
I + II + III - [I and II + II and III + I Option (d) is correct.
and III] + [I, II and III] (a) 1 (b) 2 (c) 3 (d) 2
4 6
23. A plane is going in circles around
= 10 + 30 + 5 - [8 + 4 + 2 ] + 1 _ (d) x = an airport. The plane takes 3 min
2 + 3
= 32% to complete one round. The angle
x = 4 6 ( 3 - 2) of elevation
Percentage of people who do not read
any of these three newspapers. x +2 2 x +2 3 of the plane from a point P on the
+
= 100% - 32% = 68% x -2 2 x -2 3 ground at time t is equal to that at
The number of people who do not [divided by 2 and divided by 3] time (t + 30) s.
read any of these three newspapers x x At time (t + x ) s, the plane flies
+2 +2
2 vertically above the point P. What
= 100000 ´
68
= 68000 = + 3
100
x
-2
x
-2 is x equal to?
2 3 (a) 75 s (b) 90 s (c) 105 s (d) 135 s
Option (d) is correct.
CDS Solved Paper 2019 (I) 45

100
_ (c) “A” is airport and a plane is going _ (d) p and q are two integers. = = 25
in circles around it. 4
p and q are also relative prime,
P HCF = 1 Option (a) is correct.
Condition 1 26. For any two real numbers a and b,
Both p and q may be prime numbers
75s (a - b )2 + (b - a )2 is
( p, q ) = ( 3, 5) HCF = 1
A (a) always zero (b) never zero
It is possible.
(c) positive only if a ¹ b
ts Condition 2 (d) positive if and only if a > b
B C (t + 30) s
Both p and q may be composite
numbers. _ (c) a and b are any two real numbers.
15 s D 15 s a = 3, b = 2
Let ( p, q ) = ( 9, 25), HCF = 1
The plane takes 3 min to complete one It is also possible. ( a - b )2 + ( b - a )2
round. Condition 3 = ( 3 - 2 )2 + (2 - 3)2
In s = 3 ´ 60 = 180 s
One of p and q may be prime and the = (1) + 2
( - 1)2 = 1 + 1 = 2
Plane reaches at B in t s …(i) other composite.
Plane reaches at C in (t + 30) S …(ii) positive only if a ¹ b
Let ( p, q ) = ( 5, 8) HCF = 1
The angle of elevation of the plane Option (c) is correct.
from a point P on the ground at time t s It is also possible.
is equal to that at time (t + 30) s. Then, condition (1), (2) and (3) are 27. If a : b = c : d = 1 : 6, then what is the
Plane is going to D from point B possible. a2 + c 2
value of =?
distance increases respect to P. Option (d) is correct. b2 + d 2
[because PD is longest chord of
circle or diameter of circle]
25. In a class of 100 students, the average (a)
1
(b)
1
(c)
1
(d)
1
weight is 30 kg. If the average 600 60 36 6
After point D the distance of the plane
weight of the girls is 24 kg and that
decreases with respect to point P and _ (c) a : b = c : d = 1 : 6
at point C the distance is equal to arc of the boys is 32 kg, then what is
a c 1
PC and PB. the number of girls in the class? = =
b d 6
Then, it take equal time from B to D (a) 25 (b) 26 (c) 27 (d) 28
Let a = x , c = x , b = 6x ,d = 6x
and from D to C.
_ (a) Let the number of girls = x a2 + c 2 ( x )2 + ( x )2
i.e. plane takes total 30 s
The number of boys = 100 - x Now, =
2 2
15 s from B to D, 15 s from D to C. b +d ( 6x )2 + ( 6x )2
Total weight of girls = average weight
The plane takes total time to complete ´ number of girls = 24 ´ x = 24x x2 + x2
=
a round = 180 sec 36x 2 + 36x 2
Total weight of boys = 32 ´ (100 - x )
PB + BD + DC + CP = 180 sec 2x 2 1
Total weight of students = =
CP + 15 + 15 + CP = 180 sec = 30 ´ 100 = 3000 2 36
72 x
2CP = 180 - 30 = 150
\ 24x + 32(100 - x ) = 3000 Option (c) is correct.
CP = 75 sec
24x + 3200 - 32 x = 3000
Then, total time is (t + x ) s, when 28. What is 0. 53 + 053
. equal to?
plane is at point P. 8x = 200
(a) 1068
. (b) 1068
.
Major arc from B to P. x = 25 (c) 1. 068 (d) 1.068
Total time = (t + 75 + 15 + 15) Hence, the number of girls in the class
is 25. _ (a) 0. 53 + 0.53
Total time = (t + 105) s
Alternate method In first number bar on two-digit and in
24. Consider the following statements A class of 100 students. second number bar on one-digit
in respect of two integers p and q The average weight of the class LCM of bar digits (2, 1) = 2
(both > 1) which are relatively = 30 kg. Last two-digits have bar
prime. The average weight of the girls = 24 kg. 0. 5 3 5
1. Both p and q may be prime The average weight of the boys = 32 kg. + 0. 5 3 3
numbers. By Alligation 1. 0 6 8
2. Both p and q may be composite Girls Boys 1. 068
numbers. 24 32
Alternative Method
30 53 53 - 5
3. One of p and q may be prime and 0. 53 + 0. 53 = +
6 99 90
the other composite. 2
2 :6=1:3 53 48 53 ´ 90 + 48 ´ 99
Which of the above statements Þ + =
99 90 99 ´ 90
are correct? Ratio of girls and boys = 1 : 3 4770 + 4752 9522
(a) 1 and 2 only (b) 2 and 3 only 1 = = = 1068
.
The number of girls = 100 ´ 8910 8910
(c) 1 and 3 only (d) 1, 2 and 3 (1 + 3) Option (a) is correct.
46 CDS Solved Paper 2019 (I)

29. The inequality 3 N > N 3 holds Distance of Part I = 9 ´


50 15
= km 3 × 3 520 3 × ( 3 2 )260
Remainder = =
60 2 8 8
when [Q Distance = Speed ´ 3 × ( 8 + 1)260 3(1)260 3
(a) N is any natural number Time] = = =
8 8 8
(b) N is a natural number greater than 80 32
Distance of part II = 8 ´ = km \ 3 is remainder
2 60 3
(c) N is a natural number greater than 3 Option (b) is correct.
100 75
Distance of part III = 7.5 ´ =
(d) N is a natural number except 3 60 6 34. A prime number contains the digit
_ (d) 3 > N
N 3
Total Distance =
15 32 75
+ +
X at unit’s place. How many such
Let N = 2, 32 > 2 3 2 3 6 digits of X are possible?
45 + 64 + 75 (a) 3 (b) 4 (c) 5 (d) 6
9 > 8 (It is correct) =
6
Let N = 3, 3 3 > 3 3 (It is not correct) 184 _ (b) The digit x at unit’s place in prime
= km number.
Let N = 4, 34 > 43 6
Then, such digit of x are possible
81 > 64 (It is correct) Total time = 50 + 80 + 100 = 230 min
Then, N is a natural number except 3. 230 23 x = [1, 3, 7, 9]
= = h
Hence, option (d) is correct. 60 6 Option (b) is correct.

30. Which one of the following is an The average speed of this runner 35. If an article is sold at a gain of 6%
irrational number? 184 instead of a loss of 6%, the seller
Total Distance 184 ´ 6
(a) 59049 (b)
231 = = 6 = gets ` 6 more. What is the cost
Total Time 23 6 ´ 23
593 price of the article?
6
(c) 0.45454545.... (a) ` 18 (b) ` 36
= 8 km/h
(d) 0 × 12112211122211112222.... (c) ` 42 (d) ` 50
Option (b) is correct.
_ (d) The number that cannot be _ (d) Let cost price of an article = ` x
expressed in the form of p / q are 32. If a = b = c , then which
called irrational number. b +c c +a a +b If an article sold at a gain of 6%, then
For example 2 , 3, 7 , 11 etc. selling price of an article
one of the following statements is
100 + gain% ö
Option (a) correct? = Cost price ´ æç ÷
è 100 ø
59049 = 243 (a) Each fraction is equal to 1 or - 1
231 1 106 106x
Option (b) (it is p / q form) (b) Each fraction is equal to or 1 = ´x=`
593 2 100 100
1
Option (c) 0.454545........ (c) Each fraction is equal to or - 1 If an article sold at a loss of 6%, then
45 2 selling price = Cost price
(it is p/q form) 1
100 - Loss% ö
´ æç
99 (d) Each fraction is equal to only
2 ÷
Option (d) 0.121122111222 ..... è 100 ø
a b c
[it can not expressed in form of p / q ] _ (d) = = = k (let) 94 94x
b+c c+ a a+ b =
´x=`
Then, option (d) is correct answer. 100 100
a
=k 106x 94x
31. A race has three parts. The speed b+c Then, - =6
100 100
and time required to complete the a = k( b + c ) …(i)
Þ 12 x = 600
b
individual parts for a runner is =k \ x = ` 50
displayed on the following chart c+ a
b = k(c + a ) …(ii) 36. If 3 x = 4 y = 12z , then z is equal to
Part I Part II Part III c
=k (a) x y (b) x + y
Speed (km/h) 9 8 7.5 a+ b xy
(c) (d) 4 x + 3 y
Time (min) 50 80 100 c = k( a + b ) …(iii) x+ y
On adding Eqs. (i), (ii) and (iii), we get
_ (c) 3 = 4 = 12 = k (let)
x y z
What is the average speed of this
a + b + c = 2 k( a + b + c ) 1
runner?
k = 1/ 2 3 = kx …(i)
(a) 8.17 km/h (b) 8 km/h
Option (d) is correct. 1/ y
(c) 7.80 km/h (d) 7.77 km/h 4=k …(ii)

_ (b)
33. The number 3521 is divided by 8. 12 = k 1/ z …(iii)
What is the remainder?
Part I Part II Part III Q 3 ´ 4 = 12
(a) 1 (b) 3 (c) 7 (d) 9
Speed (km/h) 9 8 7.5 521 k 1/ x ´ k 1/ y = k 1/ z
3 1 1
Time (min) 50 80 100 _ (b) + 1
8 kx y
= kz
[by Eqs. (i), (ii) and (iii)]
CDS Solved Paper 2019 (I) 47

1 1 1 (a) 4 and 0 ( y + 1) ( y - 2 ) = 0
Þ + =
x y z (b) 0 and 3 y = 2, - 1
xy (c) 3 and 0 ( y = - 1) not possible because all
\ z=
x + y (d) 0 and any integer terms in equation are positive.
2 y = 2, x = 2 + y
Option (c) is correct. _ (d) The polynomial ( x + ax + b )
leaves the same remainder when x =2 + 2 = 4
37. A field can be reaped by 12 men or
divided by ( x - 1) or ( x + 1). Option (d) is correct.
18 women in 14 days. In how many
( x - 1) = 0 42. In an examination, 52% candidates
days can 8 men and 16 women reap it?
x =1 …(i) failed in English and 42% failed in
(a) 26 days (b) 24 days
( x + 1) = 0 Mathematics. If 17% failed in both
(c) 9 days (d) 8 days
x = -1 …(ii) the subjects, then what per cent
_ (c) A field can be reaped by 12 men or On putting these value in polynomial passed in both the subjects?
18 women in 14 days.
(1)2 + a(1) + b = R [remainder] (a) 77 (b) 58 (c) 48 (d) 23
12 M = 18W
2 M = 3W a + b + 1= R …(iii) _ (d)
English Mathematics
M = 3 / 2W …(i) ( - 1)2 + a( - 1) + b = R
Total work = 18W ´ 14 days b - a + 1= R …(iv)
Let 8 men and 16 women can reap the Eq. (iii) is equal to Eq. (iv), we get
field = x days 35% 17% 25%
a + b + 1= b - a + 1
( 8M + 16W ) ´ x days
2a = 0
= 18W ´ 14 days
a=0
æ 8 ´ 3 W + 16W ö ´ x = 18W ´ 14
ç ÷ b any integer.
è 2 ø Total number of students fail
[by Eq. (i)] Hence, option (d) is correct. = 35 + 17 + 25 = 77%
18W ´ 14
D= = 9 days 40. Tushar takes 6 h to complete a piece Total number of students passed in
28W
of work, while Amar completes the both the subjects = 100 - 77 = 23%
Then, Option (c) is correct.
same work in 10 h. If both of them Option (d) is correct.
38. If ( 4a + 7b ) ( 4c - 7d ) work together, then what is the 43. A man who recently died left a
= ( 4a - 7b ) ( 4c + 7d ), then which time required to complete the work? sum of ` 390000 to be divided
one of the following is correct? (a) 3 h (b) 3 h 15 min among his wife, five sons and four
a c a c
(a) = (b) = (c) 3 h 30 min (d) 3 h 45 min daughters. He directed that each
b d d b
a d 4a c _ (d) Tushar takes 6 h to complete a son should receive 3 times as much
(c) = (d) = piece of work. While Amar complete as each daughter receives and that
b c 7b d
the same work in 10 h. each daughter should receive
_ (a) ( 4a + 7 b ) ( 4c - 7d ) Then, both complete the same work twice as much as their mother
= ( 4a - 7 b ) ( 4 c + 7d ) 1 1 16
= + = receives. What was the wife’s
6 10 60
4a + 7 b 4 c + 7d share?
Þ = 60
4a - 7b 4 c - 7d \ Required time = = 3 h 45 min (a) ` 14000 (b) ` 12000
16
Use componendo and dividendo Option (d) is correct. (c) ` 10000 (d) ` 9000
rule,
41. What is the value of _ (c) Let wife’s of man receives = ` x
( 4a + 7 b ) + ( 4a - 7 b )
Each daughter receives = ` 2 x
( 4a + 7 b ) - ( 4a - 7 b ) 2 + 2 + 2 + ............ ? and each son receives = ` 6x
( 4 c + 7d ) + ( 4 c - 7d )
= (a) 1 (b) 2 According to the question,
( 4 c + 7d ) - ( 4 c - 7d )
(c) 3 (d) 4 x + 4(2 x ) + 5( 6 x ) = 390000
8a 8c 39x = 390000
= _ (d) x = 2 + 2 + 2 + ............
14b 14 d \ x = 10000
y= 2 + 2 + 2 .... Option (c) is correct.
a c
=
b d On squaring both sides, we get 44. What is the least number of
Option (a) is correct. y2 = 2 + 2 + 2... complete years in which a sum of
39. Given that the polynomial y2 = 2 + y money put out at 40% annual
2 compound interest will be more
( x 2 + ax + b ) leaves the same y - y-2 = 0
y2 - 2 y + y - 2 = 0
than tripled?
remainder when divided by ( x - 1)
y( y - 2 ) + 1( y - 2 ) = 0 (a) 3 (b) 4
or ( x + 1). What are the values of a (c) 5 (d) 6
and b, respectively?
48 CDS Solved Paper 2019 (I)

P = 100, R = 40% 17200 ´ 10 Then, x 2 - 15x + r = 0


_ (b) Let 9% = x 3 =
(18 + 15 + 10)
T = ? [the least number of complete p + q = 15 …(ii)
172000
years in which amount will be more = = ` 4000 r = pq …(iii)
than tripled] 43
p-q =1 …(iv)
P = 100
A > 300
46. What is On solving Eqs. (ii) and (iv)
n ( x - y ) 3 + (y - z ) 3 + (z - x ) 3
300 < P æç1 +
R ö p= 8
÷
è 100 ø 3( x - y ) (y - z ) (z - x ) q =7
n
equal to? r = 8 ´ 7 = 56
300 < 100 æç1 +
40 ö
÷ 1
è 100 ø (a) 1 (b) 0 (c) (d) 3 Hence, option (b) is correct.
3 < (7 / 5)n 3
49. For the inequation x 2 - 7 x + 12 > 0,
Let n = 3, 3 < (1.4)3 ( x - y)3 + ( y - z)3 + ( z - x )3
_ (a) which one of the following is
3( x - y) ( y - z) ( z - x )
3 < 2 .744 [it is not true] correct?
Let n = 4, 3 < (1.4)4 If a+ b+c=0
(a) 3 < x < 4
a 3 + b 3 + c 3 = 3abc (b) - ¥ < x < 3 only
3 < 3.8416 [it is true]
Hence, the least number of complete x - y+ y- z+ z-x =0 (c) 4 < x < ¥ only
years in which amount will be more 3( x - y) ( y - z) ( z - x ) (d) - ¥ < x < 3 or 4 < x < ¥
Then, =1
than tripled is 4 yr. 3( x - y) ( y - z) ( z - x ) 2
_ (d) x - 4x - 3x + 12 > 0
45. A person divided a sum of Option (a) is correct answer.
x ( x - 4) - 3( x - 4) > 0
` 17200 into three parts and 47. If a x = b y = c z and b 2 = ac , then ( x - 4) ( x - 3) > 0
invested at 5%, 6% and 9% per \ x > 4 or x < 3
1 1
annum simple interest. At the end what is + equal to?
x z -¥<x <3
of two years, he got the same
1 1 2 2 or 4<x <¥
interest on each part of money. (a) (b) - (c) (d) -
y y y y Hence, option (d) is correct.
What is the money invested at 9%?
(a) ` 3200 (b) ` 4000 _ (c) Given, a = b = c and b = ac
x y z 2
50. The expression 52n - 2 3n has a
(c) ` 4800 (d) ` 5000 a x = b y = c z = k (let) factor
(a) 3 (b) 7
_ (b) A person divided a sum of ` 17200 a = k 1/ x (c) 17 (d) None of these
into three parts.
b = k 1/ y 2n 3n
_ (c) 5 - 2 has a factor,
R 1 = 5%, R 2 = 6%, R 3 = 9%, Time
= 2yr c=k 1/ z Let n = 1
Let the money invested at 5%, 6% and 2 Þ 5 2 - 2 3 = 25 - 8 = 17
9% be x 1, x 2 and x 3 respectively. Q b = ac
1/ y 2
17 is factor of 5 2n - 2 3n
We got the same interest (k ) = k 1/ x ´ k 1/ z
Option (c) is correct.
x1 ´ 5 ´ 2 x2 ´ 6 ´ 2 2 æ 1 1ö
= ç + ÷
51. If tan x = 1, 0 < x < 90°, then what
100 100 k y = kè x zø

x3 ´ 9 ´ 2 é P ´ R ´Tù is the value of 2sin x cos x?


= Q SI = 2 1 1
êë = +
100 100 úû y x z (a)
1
(b) 1 (c)
3
(d) 3
10 12 18
2 2
x1 ´ = x2 ´ = x3 ´ Option (c) is correct.
100 100 100 _ (b) Given, tan x = 1, 0 < x < 90°
48. If p and q are the roots of the
Ratio of sum x = tan- 1(1)
10 12 equation x 2 - 15x + r = 0 and
x1 ´ = x2 ´ x = tan-1(tan 45° )
100 100 p - q = 1, then what is the value of
x1 6
=
r? \ x = 45° = 2 sin x cos x
x2 5 (a) 55 (b) 56 (c) 60 (d) 64 = 2 sin 45° cos 45°
Þ x1 : x2 = 6 : 5 _ (b) p and q are the roots of the 1 1
=2 ´ ´ =1
12 18 equation. 2 2
x2 ´ = x3 ´
100 100 x 2 - 15x + r = 0 p - q = 1 …(i) Option (b) is correct.
x2 3
= é ù 52. What is the value of
x3 2 ê ax 2 + bx + c = 0 ú
ê 2 æ bö ú sin 46° cos 44° + cos 46° sin 44°?
Þ x 2 : x 3 = 3 :2 c
êx - ç- ÷x + = 0 ú
è ø (a) sin2° (b) 0
x1 : x2 : x3 ê a a ú
ê a + b = - b , ab = c ú (c) 1 (d) 2
= 6 ´ 3 : 5 ´ 3 : 2 ´ 5 = 18 : 15 : 10 êë a úû
\ The money invested at
a
_ (c) Given,
[a and b are the root of Eq. (i)] sin 46° cos 44° + cos 46° sin 44°
CDS Solved Paper 2019 (I) 49

= sin 46° cos( 90° - 46° ) BX tower at B and CY tower at C wishes to take the second street
+ cos 46° sin( 90° - 46° ) DABX which is on his right side. At some
[Q sin( 90°- q) = cos q, AB = a place, he makes a 150° turn to the
cos( 90°- q) = sin q] BX
tan 30° = right and he travels for 15 min at
BA the speed of 20 km/h. After that he
= sin 46° sin 46° + cos 46° cos 46° 1 BX
= takes a left turn of 60° and travels
= sin2 46° + cos 2 46° = 1 3 a
a for 20 min at the speed of 30 km/h
[Q sin2 q + cos 2 q = 1] BX = …(i) in order to meet the second street.
3
Option (c) is correct. What is the distance between the
DABC
Alternate Method B
two streets?
sin 46° cos 44°+ cos 46° sin 44° (a) 7.5 km (b) 10.5 km
120° (c) 12.5 km (d) 15 km
= sin( 46°+ 44° )
[Q sin A cos B + cos A sin B = sin( A + B)]
a a _ (c) There are two parallel streets.
= sin 90°
150°
=1
A M C A C
53. Suppose 0 < q < 90°, then for every [ÐB = 120° each internal 30° 5 km
q, 4 sin 2 q + 1 is greater than or angle of hexagon 120°]
120°
equal to BM ^ AC Similarly, D B
O 60° 10 km
(a) 2 (b) 4 sinq DABM 3
MC = a
(c) 4 cos q (d) 4 tanq ÐABM = 60° 2 AO = travells for 15 min at the speed
_ (a) ÐAMB = 90° AC = AM + MC of 20 km/h
4sin2 q + 1 0 < q < 90° sin 60° =
AM 3 3 Distance = speed ´ time
= a+ a
AB 2 2 15
= ´ 20 = 5 km
Let q = 30° q = 45° 3 AM = 3a 60
2 =
= 4sin 30° + 1 = 4sin2 45° + 1 2 a OB = travells for 20 min at the speed
3a of 30 km/h
1 1 AM =
= 4 ´ + 1= 2 = 4 ´ + 1= 3 20
4 2 2 = ´ 30 = 10 km
CY 60
DAYC, tan 45° =
Then, this equation greater than or AC DADO, ÐDAO = 180°-150° = 30°
equal to 2. CY OD OD
1= sin 30° = =
Option (a) is correct. 3a OA 5
3a = CY 1 OD
54. Consider a regular hexagon =
Ratio of tower B and C 2 5
ABCDEF. Two towers are situated
BX : CY OD = 2.5 km
at B and C. The angle of elevation
The distance between the two streets.
from A to the top of the tower at B a
: 3a
3 DB = DO + OB
is 30°, and the angle of elevation to
the top of the tower at C is 45°. 1: 3 = 2.5 + 10 = 12.5 km
What is the ratio of the height of Option (b) is correct. Option (c) is correct.
towers at B and C? 55. What is the value of 57. If 3 tan q = cot q where 0 £ q < p ,
(a) 1 : 3 (b) 1 : 3 tan 1° tan 2° tan 3° K tan 89° ? 2
(c) 1 : 2 (d) 1 : 2 3 (a) 0 (b) 1 then what is the value of q?
(c) 2 (d) ¥ p p
_ (b) A regular hexagon ABCDEF. (a) (b)
6 4
Two towers are situated at B and C. _ (b) Given, tan1° tan2° tan 3° K tan 89° p p
(c) (d)
A 30° = (tan1° tan 89° ) (tan2° tan 88° )K 3 2
....(tan 44° tan 45° )tan 45° p
45° x _ (a) Given, 3 tan q = cot q 0 £ q <
= 1 ´ 1K 1 ´ 1 2
F
B [Q tan q tan( 90° - q) = 1] 1 éQ cot q = 1 ù
3 tan q =
y =1 [Q tan 45° = 1] tan q êë tan q úû
E Option (b) is correct. 1
C tan2 q =
3
56. There are two parallel streets each
1
D directed North to South. tan q =
3
Let side of hexagon = a A person in the first street
travelling from South to North [tan q, value positive in first quadrant]
50 CDS Solved Paper 2019 (I)

æ 1 ö
q = tan-1ç -1
÷ = tan (tan 30° ) 61. Consider the following grouped (x ) (f )
è 3ø 6 5
frequency distribution
p 7 4
q = 30° Þ q = x f
6 8 6
Option (a) is correct. 0-10 8 9 9
58. What is the value of 10-20 12 10 8
n = 42
sin 2 25° + sin 2 65° ? 20-30 10
30-40 p N 42
(a) 0 (b) 1 (c) 2 (d) 4 Here, = = 21
40-50 9 2 2
(b) Given, sin2 25° + sin2 65° 21th observation = 8
= sin2 25° + sin2( 90° - 25° ) If the mean of the above data is 22th observation = 8
2
= sin 25° + cos 25° 2 25.2, then what is the value of p? Median
(a) 9 (b) 10 æ n ö th term +
æ n + 1ö th term
[Q sin( 90°- q) = cos q] ç ÷ ç ÷
(c) 11 (d) 12 è2 ø è2 ø
=
=1 [Q sin2 q + cos 2 q = 1] 2
_ (c) 21th term + 22 th term
Option (b) is correct. =
x xi fi xi fi 2
59. What is the value of 8 + 8 16
0-10 5 8 40 = = =8
sin6 q + cos 6 q + 3 sin2 q cos 2 q - 1 ? 2 2
10-20 15 12 180
(a) 0 (b) 1 (c) 2 (d) 4
20-30 25 10 250 63. The average of 50 consecutive
_ (a) Given, natural numbers is x. What will be
sin6 q + cos 6 q + 3 sin2 q cos 2 q - 1 30-40 35 p 35p
the new average when the next
40-50 45 9 405 four natural numbers are also
= (sin2 q)3 + (cos 2 q)3 + 3 sin2 q cos 2 q Sfi = 39 + p Sxi fi included?
= 875 + 35p (a) x + 1 (b) x + 2
(sin2 q + cos 2 q) - 1
(c) x + 4 (d) x + (x / 54)
[Q( a + b )3 = a 3 + b 3 + 3ab ( a + b )] Sx i fi
x(Mean) =
2 2 3 Sfi _ (b) The average of 50 consecutive
= (sin q + cos q) - 1 natural numbers is x.
875 + 35 p
2
[Q sin q + cos q = 1] 2 . =
252 New average
39 + p Number of next terms
= (1)3 - 1 = 0 =Old average +
. ( 39 + p) = 875 + 35 p
252 2
Option (a) is correct. 982.8 + 252. p = 875 + 35 p =x +
4
=x +2
982.8 - 875 = 35 p - 252
. p 2
60. Consider the following for real
numbers a , b, g and d 107.8 = 9.8 p 64. Consider two-digit numbers which
107.8
1. sec a = 1 / 4 2. tan b = 20 p= = 11 remain the same when the digits
9.8
3. cosec g = 1 / 2 4. cos d = 2 interchange their positions. What
Option (c) is correct. is the average of such two-digit
How many of the above
statements are not possible? 62. Consider the following frequency numbers?
(a) One (b) Two distribution (a) 33 (b) 44 (c) 55 (d) 66
(c) Three (d) Four x f _ (c) Two-digit numbers which remain
the same when the digits interchange
_ (c) a, b, g and d are real numbers 8 6
their positions.
1. sec a = 1 / 4 [Q Value of sec a lies 5 4
11, 22, 33, 44, 55, 66, 77, 88, 99
between ( - ¥, - 1] È [1, ¥) 6 5
Sum of numbers
It is not possible. 10 8 Average =
2. tanb = 20 [Q value of tanb lies Number of terms
9 9
between [- ¥, ¥] ] é11 + 22 + 33 + 44 + 55 + 66ù
4 6 ê
It is possible. + 77 + 88 + 99 úû
7 4 = ë
3. cosec g = 1 / 2 [Q value of cosec g 9
lies between ( -¥, - 1] È [1, ¥) 495
What is the median for the = = 55
It is not possible. 9
distribution?
4. cos d = 2 [Q value of cos d lies (a) 6 (b) 7 (c) 8 (d) 9 Alternative method
between [-1, 1] First term + Last term
Average =
It is not possible. _ (c) Arrange in ascending order 2
Three statements are not correct. (x ) (f ) 11 + 99
= = 55
Then, 4 6 2
Option (c) is correct. 5 4 Option (c) is correct.
CDS Solved Paper 2019 (I) 51

65. Diagrammatic representation of Percentage of educations expenditure 70. The following pairs relate to
data includes which of the 480 frequency distribution of a discrete
= ´ 100 = 8%
following? 6000 variable and its frequency polygon.
1. Bar-diagram 15 cm height of bar-diagram. Which one of the following pairs is
2. Pie-diagram Length of the education in not correctly matched?
bar-diagram
3. Pictogram (a) Base line of the X-axis
8
Select the correct answer using = 15 ´ = 12
. cm polygon
100
the code given below (b) Ordinates of the Class
(a) 1 and 2 only Percentage of miscellaneous’s vertices of the frequencies
expenditure
(b) 2 and 3 only 1660 166 polygon
(c) 1 and 3 only = ´ 100 = % (c) Abscissa of the Class marks
6000 600
(d) 1, 2 and 3 vertices of the of the
Length of the miscellaneous’s in
polygon frequency
_ (d) Diagrammatic representation of bar-diagram
distribution
data includes 166
= 15 ´ = 415
. cm (d) Area of the Total
1. Bar-diagram 600
2. Pie-diagram polygon frequency of
3. Pictogram Option (b) is correct. the
Option (d) is correct. 68. If the mean of m observations out distribution

66. The data collected from which one of n observations is n and the mean _ (d) Area of the polygon is not related
of the following methods is not a of remaining observations is m, to the total frequency of the
then what is the mean of all n distribution.
primary data?
observations? Then, option (d) is correct.
(a) by direct personal interviews 2
m 71. In a rectangle, length is three times
(b) by indirect personal interviews (a) 2m -
(c) by schedules sent through n its breadth. If the length and the
enumerators m2 breadth of the rectangle are
(b) 2m +
(d) from published thesis n increased by 30% and 10%
m2 respectively, then its perimeter
_ (d) The data collected from published (c) m -
thesis is not a primary data. n increases by
Option (d) is correct. m2 40
(d) m + (a) % (b) 20%
n 3
67. The monthly expenditure of a (c) 25% (d) 27%
person is ` 6000. The distribution _ (a) The mean of m observations = n
of expenditure on various items is The sum of observations = mn _ (c) Breadth of the rectangle = x
as follows The mean of( n - m) observations = m Length of the rectangle = 3x

Item of Amount The sum of observations = ( n - m) m Perimeter of rectangle = 2( x + 3x )


expenditure (in `) The mean of all observations = 8x …(i)
1. Food 2000 Sum of all observations [Q P = 2 ( l + b )]
=
Number of observations Increases length 30%, then
2. Clothing 660
mn + ( n - m)m 130
3. Fuel and rent 1200 = Length = ´ 3x
n 100
4. Education 480 m( n + n - m) 39x
= =
5. Miscellaneous 1660 n 10
2 nm - m2 m2 Increases breadth 10%, then
= = 2m -
If the above data is represented by n n Breadth =
110
´x =
11x
a percentage bar-diagram of Option (a) is correct. 100 10
height 15 cm, then what are the New perimeter of rectangle
lengths of the two segments of the 69. Which one of the following pairs
= 2 æç
39x 11x ö
is correctly matched? + ÷ = 10x …(ii)
bar-diagram corresponding to è 10 10 ø
education and miscellaneous, (a) Median = Graphical
Increases in perimeter
respectively? location
Eqs. (i) and (ii), we get
(a) 1.25 cm and 5 cm (b) Mean = Graphical
10x - 8x = 2 x
(b) 1.2 cm and 4.15 cm location
(c) Geometric = Ogive Percentage of change in perimeter
(c) 1.2 cm and 3.5 cm 2x
Mean = ´ 100
(d) 4.15 cm and 6 cm
(d) Mode = Ogive 8x
_ (b) The monthly expenditure of a = 25%
person is 6000. _ (a) Median is graphical location.
Option (c) is correct.
Then, Option (a) is correct.
52 CDS Solved Paper 2019 (I)

72. What is the percentage decrease in 74. If the lengths of two parallel The distance between two chords
the area of a triangle if its each chords in a circle of radius 10 cm DE = OE - OD = 8 - 6 = 2 cm
side is halved? are 12 cm and 16 cm, then what is 75. Considering two opposite vertices
(a) 75% (b) 50% the distance between these two of a square of side ‘a’ as centres,
(c) 25% (d) No change chords? two circular arcs are drawn within
(a) 1 cm or 7 cm (b) 2 cm or 14 cm
_ (a) Let we have an equilateral triangle the square joining the other two
and side of triangle = x (c) 3 cm or 21 cm (d) 4 cm or 28 cm vertices, thus forming two sectors.
3 2
Area of triangle =x _ (b) Radius of circle = 10 cm What is the common area in these
4 Length of chords = 12 cm and 16 cm two sectors?
Now, Triangle each side is halved
(a) a 2 æç p + ö÷ (b) a 2 æç p - ö÷
If chords opposite side from centre. 1 1
= x /2 è
O is centre of circle.
2ø è 2ø
New area of triangle p p
2 (c) a 2 æç - 1ö÷ (d) a 2 æç + 1ö÷
3 æx ö 3 x2 è2 ø è2 ø
= ç ÷ = 6 D 6
4 è2 ø 4 4 A B
Change in area
_ (c) ABCD is a square
O and Side of a square = a
3 2 3 x2 3 3x 2
= x - = 10 D C
4 4 4 4 4
Change in percentage of area C F
8 E 8
3 3x 2
´
= 4 4 ´ 100 = 75%
AB and CD are two chords 12 cm and
3 2
x 16 cm, respectively.
4
Option (a) is correct. In DAOD,
A B
Alternate method AO = 10 cm [radius of circle] a
Here, a = 50% AD = 6 cm Shaded region is the common area of
Required percentage decrease in ( AO )2 = ( AD)2 + (OD)2 the sectors.
area of equilateral triangle (10 )2 = ( 6 )2 + (OD)2 Area of shaded region
æ a2 ö ( 50)2 = 2 ´ Area of sector - Area of square
= çç2 a - ÷÷ % = 2 ´ 50 - 2
(10) - ( 6) = OD 2 2
è 100 ø 100 90°
= 2 ´ p a2 ´ - a2
2500 100 - 36 = OD 360°
= 100 - = 100 - 25 = 75% OD = 8 cm q
100 [Q area of sector pr 2 ]
In DCEO, (CO )2 = (CE )2 + (OE )2 360°
73. The volume of a spherical balloon
(10)2 = ( 8)2 + (OE )2 p a2 p
is increased by 700%. What is the =2 ´ - a 2 = a 2 æç - 1ö÷
100 - 64 = OE, OE = 6 cm 4 è2 ø
percentage increase in its surface
area? Distance between two chords Option (c) is correct.
(a) 300% (b) 400% ( DE ) = DO + OE = 8 + 6= 14 cm 76. The corners of a square of side ‘a’
(c) 450% (d) 500% If both chords are in same side from are cut away so as to form a
centre. regular octagon. What is the side
_ (a) Let the radius of spherical balloon
=r of the octagon?
4 3 (a) a( 2 - 1) (b) a( 3 - 1)
Volume of sphere = pr
3 a a
Surface area of balloon = 4pr 2 (c) (d)
O 2 + 2 3
According to question, 10
Volume of balloon is increased by D _ (a)
700% A 10 8 8 B
A E F D
4 800 4 C 6 E 6 F x
\New Volume = pr 3 ´ = p(2 r )3 x x
3 100 3
L G
\ New radius = 2 r O is centre of circle.
x x a
\ New surface area = 4p(2 r )2 AB = 16 cm
CD = 12 cm K H
= 4p ´ 4r 2 = 16pr 2 x x
\ Required percentage increases In DAOD, In DCOE,
B C
( AO )2 = ( AD )2 + (OD )2 (CO )2 = (OE )2 + (CE )2 J I
16pr 2 - 4pr 2 a
= ´ 100 (10 )2 = ( 8 )2 + OD 2 (10 )2 = (OE )2 + ( 6 )2
4pr 2
OD 2 = 100 - 64 OE = 100 - 36 ABCD is square.
12
= ´ 100 = 300% OD = 36 OE = 8 cm The corners of square are cut away by
4
OD = 6 cm doted line.
CDS Solved Paper 2019 (I) 53

EFGHIJKL is octagon. 78. Two circles are drawn with the = l ´ b = 12 ´ 6 = 72 sq feet
Let side of octagon = x same centre. The circumference of Length of inner rectangle
= 12 - 0.5 - 0.5 = 11 feet
AE + FD = a - x [Q AE = FD] the smaller circle is 44 cm and that
Breadth of inner rectangle
2 AE = a - x of the bigger circle is double the
= 6 - 0.5 - 0.5 = 5 feet
æa - x ö smaller one. What is the area
AE = ç ÷ ...(i) Area of inner rectangle
è 2 ø between these two circles? = 11 ´ 5 = 55 sq feet
æa - x ö (a) 154 cm2 (b) 308 cm2 Area of darkred border
Similarly, AL = ç ÷
è 2 ø (c) 462 cm 2
(d) 616 cm2 = (area of outer rectangle)
- (area of inner rectangle)
DAEL ( AL )2 + ( AE )2 = ( LE )2
_ (c) Two circle are drawn with the same = 72 - 55 = 17 sq feet
2 2 centre (O).
æa - x ö æa - x ö 2 80. The perimeter of a right-angled
ç ÷ + ç ÷ =x
è 2 ø è 2 ø
triangle is k times the shortest side.
2 If the ratio of the other side to
æa - x ö R
2 ç ÷ =x
è 2 ø O r hypotenuse is 4 : 5, then what is
the value of k?
æa - x ö
x = 2 ç ÷ (a) 2 (b) 3 (c) 4 (d) 5
è 2 ø
2x = a - x _ (c) DABC is right angle triangle at B
Circumference of smaller circle = 44 A
Þ 2x + x = a cm
x ( 2 + 1) = a and Radius of smaller circle = r
x =
a Circumference of circle = 2 pr 5x
2 +1 Þ 2 pr = 44
é 1 ù r = 7 …(i)
x = a ( 2 - 1) êQ = 2 - 1ú
2 +1 Circumference of big circle B C
ë û 4x
= 2 ´ smaller circle
Then, option (a) is correct. = 2 ´ 44 = 88 AC = 5x , BC = 4x
77. Three consecutive integers form 2 pR = 88 AB is shortest side of triangle.
the lengths of a right angled R = 14 ( AB)2 + ( BC )2 = ( AC )2
triangle. How many sets of such Area between two circles
( AB)2 + ( 4x )2 = ( 5x )2
three consecutive integers is/are A = pR 2 - pr 2
AB = 25x 2 - 16x 2 = 9x 2
possible? A = p( R 2 - r 2 )
22 22 AB = 3x
(a) Only one (b) Only two = ´ [14 2 - 7 2 ]= ´ 21 ´ 7
(c) Only three (d) Infinitely many 7 7 Perimeter of triangle is k times the
= 462 cm 2 shortest side
_ (a) Three consecutive integers form AB + BC + CA = k ´ ( AB)
the lengths of a right-angled triangle. 79. A rectangular red carpet of size 3x + 4x + 5x = k ´ ( 3x )
A 6 ft ´ 12 ft has a dark red border 6 k=
12 x
= 4Þ k = 4
inches wide. What is the area of 3x
the dark red border? 81. A 12 m long wire is cut into two
x x+2 (a) 9 sq feet (b) 15 sq feet pieces, one of which is bent into a
(c) 17 sq feet (d) 18 sq feet circle and the other into a square
_ (c) Given enclosing the circle. What is the
C 6 inches radius of the circle?
B x+1 12 6
(a) (b)
2 2
( x ) + ( x + 1) = ( x + 2 ) 2 p+ 4 p+ 4
3 6
2 6 ft (c) (d)
x + x 2 + 2 x + 1 = x 2 + 4x + 4 p+ 4 p+ 2 2
x 2 - 2x - 3 = 0 (b) A square enclosing the circle
é x = 3ù
( x - 3) ( x + 1) = 0 ê x + 1 = 4ú 12 ft
ê ú
êë x + 2 = 5úû Width of red border = 6 inches
6 R
x = 3 or x = - 1 = = 0.5 feet {Q 1 ft = 12 inches}
12
Only one set have consecutive integers.
Length of outer rectangle = 12 feet
[not possible length never be
Breadth of outer rectangle = 6 feet
negative] 2R
Area of outer rectangle
Option (a) is correct.
54 CDS Solved Paper 2019 (I)

Radius of circle = R BD = DC [isosceles triangle] We know, ( x + y)2 = x 2 + y 2 + 2 xy


and Side of square = 2 R AD is also angle bisector ( x + y)2 = 100 + 2 ´ 48 = 196
Circumference of circle = 2 pR In DADB,
x + y = 14 …(iii)
Perimeter of square = 4(2 R ) = 8R 2 y + y + 90 = 180° 2 2 2
and ( x - y) = x + y - 2 xy
Total length of wire [sum of triangle’s interior angles]
2 pR + 8R = 12 y = 30° ( x - y)2 = 100 - 2 ´ 48 = 4

2 ( pR + 4R ) = 12 cos 30° =
BD x- y=2 …(iv)
AB
R( p + 4) = 6 From Eqs. (iii) and (iv), we get
3 BD
R =
6 = x =8 Þ y=6
p+ 4 2 x
The shorter side is halved
3x
Radius of circle =
6 Þ BD = 6
p+ 4 2 y1 = = 3 cm
2
3x
Similarly, DC = The longer side is doubled
82. The angles of a triangle are in the 2
ratio 1 : 1 : 4. If the perimeter of the BC = BD + DC x 2 = 8 ´ 2 = 16 cm
triangle is k times its largest side, 3x 3x Now, new hypotenuse
then what is the value of k? BC = +
2 2 = x 12 + y12 = 16 2 + 3 2
A
= 3x
= 256 + 9 = 265 cm
Perimeter of triangle equal to k times
larger side of triangle. 84. In a circle of radius 8 cm, AB and
x + x + 3x = k 3x AC are two chords such that
3xk = 2 x + 3x AB = AC = 12 cm. What is the
3x + 2 x length of chord BC ?
k=
3x (a) 2 6 cm (b) 3 6 cm
B C
D 2 (c) 3 7 cm (d) 6 7 cm
k = 1+
2 2 3
(a) 1 + (b) 1 - _ (d) In a circle,
3 3
2 83. The hypotenuse of a right angled Radius = 8 cm
(c) 2 + (d) 2 triangle is 10 cm and its AB = AC = 12 cm
3
area is 24 cm2 . If the shorter side AB and AC are chords of circle.
_ (a) The angle of triangle are in the ratio B
1 : 1 : 4. is halved and the longer side is
Then, two angle are equal. doubled, the new hypotenuse
becomes 12 8
A
(a) 245 cm (b) 255 cm
A O
4y (c) 265 cm (d) 275 cm
12
E
_ (c) ABC is right angle triangle at B.
X X A
C

y y AO = 8 cm is also radius
B C x 10 cm 12 + 8 + 8 28
s= = = 14 cm
Then, ABC is isosceles triangle 2 2
Let AB = x Area of DABO
B y C = s( s - a ) ( s - b ) ( s - c )
A perpendicular drawn from vertex A
to BC is D. AC = 10 cm = 14 ´ (14 - 12 )(14 - 8 )(14 - 8 )
A AB = x = 14 ´ 2 ´ 6 ´ 6 = 12 7 cm2
BC = y
BE is height of DBAO
2y
( AB)2 + ( BC )2 = ( AC )2 1
2y Area of DABO = ´ BE ´ AO
x 2 + y 2 = 100 …(i) 2
x x 1
1
Area of triangle = ´ AB ´ BC 12 7 = ´ BE ´ 8
2 2
1 BE = 3 7 cm
24 = ´ x ´ y
2 Length of BC = 2 ´ BE
y y
B D C xy = 48 …(ii) = 2 ´ 3 7 = 6 7 cm
CDS Solved Paper 2019 (I) 55

85. Consider the following statements: 86. A ladder is resting against a A

1. An isosceles trapezium is always vertical wall and its bottom is 2.5


m away from the wall. If it slips R
cyclic.
0.8 m down the wall, then its O1
2. Any cyclic parallelogram is a R/2 D O
bottom will move away from the R/2 2
rectangle.
wall by 1.4 m.
Which of the above statements
is/are correct?` What is the length of the ladder? B
(a) 1 only (b) 2 only (a) 6.2 m (b) 6.5 m
(c) 6.8 m (d) 7.5 m Radius of circles = R
(c) Both 1 and 2 (d) Neither 1 nor 2
(b) A ladder is resting against a AB is common chord = 10 3 cm
_ (c) vertical wall. AB 10 3
E AD = = = 5 3 cm
A 2 2
0.8 In DAO 1D,
E (O 1 A )2 = (O 1D )2 + ( AD )2
A D 2
R 2 = æç ö÷ + ( 5 3 )2
x R
è2 ø

B C
D R2
2.5 1.4 R2 - = 25 ´ 3
B C 4
AC is length of ladder. 3R 2
We produce BA and CD and meet = 25 ´ 3, R = 25 ´ 4
them at E. After slip, 4
AB = CD (given)…(1) DE is length of ladder R = 10 cm
AD || BC (given) AC = DE …(i) Then, diameter of circle
them,
DABC using pythogoras theorem = 2 R = 2 ´ 10 = 20 cm
AE = DE …(2) 2 2 2
On adding Eq. (1) and (2), we get ( 0.8 + x ) + (2.5) = ( AC ) Option (c) is correct.
AB + AE = CD - DE 2 Alternate Method
x + 0.64 + 16 . = AC 2 …(ii)
. x + 625
BE = EC We know that, Length of common chord
\ ÐEBC = ÐECB = q In DBDE using pythogoras theorem
= 3r
then, x 2 + (2.5 + 14
. )2 = ( DE )2
10 3 = 3r
ÐBAD = 180 - ÐABC
ÐBAD = 180 - q …(3) x 2 + 1521
. = DE 2 …(iii) Diameter of circle = 2 ´ 10 = 20 cm
Similarly From Eqs. (ii) and (iii) equal 88. Consider the following statements.
ÐCDA = 180º - q …(4)
by ( AC = DE ) Eq. (i) I. The number of circles that can
then
ÐABC + ÐCDA = q + 180 - q Then, be drawn through three
= 180º x 2 + 1521
. = x 2 + 0.64 + 16
. x + 625
. non-collinear points is infinity.
and ÐDAB + ÐDCB = 180º - q + q . - 6.89
1521 II. Angle formed in minor segment
x =
= 180º 16
. of a circle is acute.
Then, an isosceles x = 52
. m …(iv) Which of the above statements
trapezium is always cyclic.
On putting x = 5.2 in Eq. (iii), we get is/are correct?
Q Statement I is true.
. )2 + 1521
( 52 . = ( DE )2 (a) I only (b) II only
(2) Any cyclic parallelogram is
(c) Both I and II (d) Neither I nor II
rectangle. DE = 27.04 + 1521
.
_ (d) I. Only one circle can be drawn
A D DE = 42.5 = 6.5 m from three non-collinear points.
Option (b) is correct. II. (i) Angle formed in minor segment
of a circle is obtuse.
87. Two equal circles intersect such
(ii) Angle formed in major segment of a
that each passes through the circle is acute.
B C centre of the other. If the length of
Then, Option (d) is correct.
In parallelogram ÐB = ÐD = x … (i) the common chord of the circles is
In cyclic parallelogram, 10 3 cm, then what is the diameter 89. Consider the following
inequalities in respect of any
ÐB + ÐD = 180° of the circle?
triangle ABC:
x + x = 180° {from Eq. (i)} (a) 10 cm (b) 15 cm
2 x = 180° Þ x = 90° (c) 20 cm (d) 30 cm 1. AC - AB < BC
Then, 2. BC - AC < AB
_ (c) Two equal circles intersect such 3. AB - BC < AC
It is a rectangle. that each passes through the centre of
Statement 2nd also correct. the other. Which of the above are correct?
56 CDS Solved Paper 2019 (I)

(a) 1 and 2 only (b) 2 and 3 only 91. What is the ratio of the volume of 8R 3 22 2 R 3
= : ´ = 14 : 11
(c) 1 and 3 only (d) 1, 2 and 3 the sphere to that of the cone? 3 3 7 3 3
Option (c) is correct.
_ (d) Property of triangle (a) 6 3 : 1 (b) 7 : 2
Difference of two side of triangle is (c) 3 3 : 1 (d) 5 3 : 1 93. Consider the following statements :
less than third side of the triangle.
_ (a) 1. The surface area of the sphere is
Then, all the statement is correct. 5 times the curved surface area
Option (d) is correct. of the cone.
a h 2. The surface area of the cube is
90. Consider the following statements :
1. The perimeter of a triangle is equal to the curved surface area
greater than the sum of its three Fig. (ii) of the cylinder.
medians. Which of the above statements
is/are correct?
2. In any triangle ABC, if D is any h
(a) 1 only (b) 2 only
point on BC, then
(c) Both 1 and 2 (d) Neither 1 nor 2
AB + BC + CA > 2AD. Fig. (iii)
Which of the above statements _ (d) (1) Surface area of the sphere is 5
Radius of sphere = R times of the curved surface area of
is/are correct? cone.
(a) 1 only (b) 2 only Then, 2 ´ (Radius of sphere)
Surface area of the sphere : Curved
(c) Both 1 and 2 (d) Neither 1 nor 2 = Diagonal of cube
surface area of cone
_ (c) (1) The perimeter of a triangle is 2´R = 3a [a = side of cube]
2R = 4pR 2 : prl [Ql = r 2 + h2 ]
greater than the sum of its three
a=
medians. 3 2 2
æ R ö æ2 R ö
A l= ç ÷ + ç ÷
Height of cylinder ( h) = Side of cube è 3ø è 3ø
[fig. (ii)]
é 5R 2 5 ù
2R êl = = Rú
h=
3 êë 3 3 ú
û
F E
1 5
Radius of cylinder = ´ Side of cube = 4pR 2 : p
R
´ R = 12 : 5
2 3 3
1 2R R
r= ´ = Surface area of sphere is not 5 times
B D C 2 3 3 of the curved surface area.
It is property of triangle. Radius of cone = Radius of cylinder (2) Surface area of the cube
2R 2R
r=
R = 6 ´ a2 = 6 ´ ´ = 8R 2
3 3 3
AD, BE and CF are three medians of
triangle. Height of cone ( h) = Height of cylinder Curved surface area of the cylinder
(2)ABC is triangle and D is a point on BC. h=
2R 22 R 2 R 44R 2
= 2 prh = 2 ´ ´ ´ =
By triangle properties, 3 7 3 3 21
DABD, AB + BD > AD …(i) Volume of sphere : Volume of cone The surface area of the cube is not
DADC, AC + DC > AD ...(ii) 4 1 R2 2R equal to curved surface area of the
= pR 3 : p ´ = 6 3 :1 cylinder.
On adding Eqs. (i) and (ii), we get 3 3 3 3
Then, option (d) is correct.
AB + AC + BD + DC > 2 AD 4
Q Volume of sphere V = pr 3 ,
AB + AC + BC > 2 AD 3
1 2 Directions(Q. Nos. 94-96) Read the
[Q BD + DC = AD ] Volume of curve = pr h
3 given information carefully and
Option (c) is correct. Option (a) is correct. answer the given questions below.
Directions (Q. Nos. 91-92) Read the 92. What is the ratio of the volume of ABCD is a quadrilateral with AB = 9 cm,
given information carefully and the cube to that of the cylinder? BC = 40 cm, CD = 28 cm, DA = 15 cm and
(a) 4 : 3 (b) 21 : 16 angle ABC is a right-angle.
answer the given questions below.
A cube is inscribed in a sphere. A right
(c) 14 : 11 (d) 45 : 32 94. What is the area of triangle ADC?
_ (c) Volume of cube : Volume of (a) 126 cm2 (b) 124 cm2
circular cylinder is with in the cube
2
touching all the vertical faces. A right
cylinder (c) 122 cm (d) 120 cm2
3 2
æ2 R ö æ R ö 2R
circular cone is inside the cylinder. =ç ÷ :p ç ÷ 95. What is the area of quadrilateral
è 3ø è 3ø 3
Their heights are same and the ABCD ?
= a 3 : pr 2 h
diameter of the cone is equal to that of (a) 300 cm2 (b) 306 cm2
2
the cylinder. (c) 312 cm (d) 316 cm2
CDS Solved Paper 2019 (I) 57

96. What is the difference between Directions (Q. Nos. 97 and 98) Read _ (a) Length of cuboid = l
perimeter of triangle ABC and the given information carefully and Breadth of cuboid = b
perimeter of triangle ADC? answer the given question below. Height of cuboid = h
(a) 4 cm (b) 5 cm Anequilateral triangle ABC is l + b + h = 22 cm …(i)
(c) 6 cm (d) 7 cm inscribed in a circle of radius 20 3 cm. Length of diagonal is = 14 cm
Solution (Q.Nos. 94-96)
97. What is the length of the side of l 2 + b 2 + h2 = 14
C 28 cm D
the triangle?
l 2 + b 2 + h2 = (14)2
(a) 30 cm (b) 40 cm
(c) 50 cm (d) 60 cm = 196 cm ...(ii)

40 cm Surface area of cuboid


_ (d) Let length of the side of the triangle
15 cm = 2 ( lb + bh + hl )
=a
a Taking square of Eq. (i),
90° Circumradius of triangle =
3 ( l + b + h)2 = l 2 + b 2 + h2
B 9 cm A
a + 2( lb + bh + hl )
20 3 =
3 (22 )2 = 196 + 2 ( lb + bh + hl )
94. (a) In DABC, ÐB = 90º
a = 60 cm 484 - 196 = 2 ( lb + bh + hl )
( AC )2 = AB2 + BC 2
Option (d) is correct.
2( lb + bh + hl ) = 288 cm2
AC = ( 9)2 + ( 40)2
98. The centroid of the triangle ABC is Option (a) is correct.
AC = 81 + 1600 at a distance d from the vertex A.
AC = 41 cm What is d equal to? 100. If S is the sum of the cubes of the
Area of DADC dimensions of the cuboid and V is
(a) 15 cm
its volume, then what is (S - 3V )
A= s( s - a ) ( s - b ) ( s - c ) (b) 20 cm
(c) 20 3 cm equal to?
a+ b+c
S = (d) 30 3 cm (a) 572 cm3
2
(b) 728 cm3
[a, b and c side of triangle] _ (c) Centroid of the triangle ABC is at a
41 + 15 + 28 84 distance “d ” from the vertex A. (c) 1144 cm3
S = = = 42 cm
2 2 All the centres of triangle at one point (d) None of the above
in equilateral triangle.
A= 42( 42 - 28) ( 42 - 41) ( 42 - 15) _ (c) S is the sum of the cube of the
Then, circumradius of circumcircle is dimensions of the cuboid.
= 42 ´ 14 ´ 1 ´ 27
equal to “d” distance = 20 3 cm
S = l 3 + b 3 + h3
A = 7 ´ 3 ´ 2 ´ 3 = 126 cm2
Option (c) is correct.
V = lbh
95. (b) Area of quadrilateral ABCD
= Area of DABC + Area of DADC
Directions (Q. Nos. 99 and 100) \ (S - 3V ) = l 3 + b 3 + h3 - 3lbh
Read the given information carefully [Q ( a 3 + b 3 + c 3 - 3abc
1
Area of DABC = ´ 40 ´ 9 = 180 cm2 and answer the given question below. = (a + b + c ) (a2 + b 2 + c 2 )
2
- ( ab + bc + ca )]
Area of quadrilateral = 180 + 126 The sum of length, breadth and height
= ( l + b + h) [ l 2 + b 2 + h 2
= 306 cm2 of a cuboid is 22 cm and the length of
its diagonal is 14 cm. - ( lb + bh + hl )]
96. (c) Difference between perimeter of = 22 [196 - 144] = 22 ´ 52
DABC and DADC. 99. What is the surface area of the
éQ 2( lb + bh + hl ) = 288ù
Perimeter of DABC = 40 + 41 + 9 cuboid? ê lb + bh + hl = 144 ú
(a) 288 cm2 ë û
= 90 cm
Perimeter of DADC = 28 + 41 + 15 (b) 216 cm2 = 1144 cm3

= 84 cm (c) 144 cm2 Option (c) is correct.


Difference of perimeter of DABC (d) Cannot be determined due to
and DADC = 90 - 84 = 6 cm insufficient data
58 CDS Solved Paper 2019 (I)

PAPER II English
Directions (Q.Nos. 1-10) Each of the _ (c) ‘getting on’ is an appropriate or violent feelings (especially of
following sentences in this section has phrase. for the give blank. The word speech or writing).
a blank space and four words or group means to have a good relationship.
12. The differences include increase in
of words are given after the sentence. 7. We live.........a tower block. Out mean temperature and heavy
Select the word or group of words you apartment is on the fifteenth floor. precipitation in several regions.
consider the most appropriate for the (a) at (b) in (a) Drought (b) Oasis
blank space and indicate your response (c) over (d) above (c) Rainfall (d) Snowing
in the answer sheet accordingly.
1. How we.........to ageing is a choice _ (b) Here, preposition ‘in’ is an _ (c) ‘Rainfall’ is the correct synonym of
appropriate word for the blank, as in is the word ‘Precipitation’. Precipitation
we must make wisely? used for an enclosed space. means rain, snow or hail that falls to or
(a) respond (b) absolve condenses on the ground.
8. You were going to apply for the
(c) discharge (d) overlook 13. The portal will help victims and
job and then you decided not to so
_ (a) ‘respond’ is correct word to be what.......? complainants to anonymously
used for the given blank as respond report cyber crime.
(a) put you off (b) put you out
means to react to something.
(c) turned you off (d) turned you away (a) Incognito (b) Directly
2. Complementary medicine ........ (c) Unfailingly (d) Is Situ
fewer risks; since it is used _ (a) ‘put you off’ is the most appropriate
phrase for the given blank. The phrase
alongwith standard remendies, _ (a) ‘Incognito’ is the correct synonym
means to cause someone to lose of the word ‘Anonymously’. Both
often to lessen side -effects and interest or enthusiasm. words mean unknown or having one’s
enhance feelings of well-being. 9. .........it was raining, he went out true identity concealed.
(a) reacts (b) releases without a raincoat. 14. He is suffering from a terminal
(c) ejects (d) carries (a) Even (b) Since disease.
_ (d) ‘carries’ is appropriate word for the (c) Unless (d) Although (a) Sublunary (b) Terrific
given blank. The word means an act of
carrying something. _ (d) According to the given sentence, (c) Chronic (d) Incurable
‘Although’ is the most appropriate
3. Stress many ..........fertility, in man word to be used for the given blank. . _ (d) ‘Terminal and Incurable’ are
synonyms. Both words mean
and women. 10. I parked my car in a no-parking predicted to lead to death (of a
(a) engage (b) reduce zone, but I.........it. disease) especially slowly.
(c) inject (d) deduce (a) came up with (b) got away with 15. Doctors are reluctant to take rural
_ (b) ‘reduce’ is the aporopriate word for (c) made off with (d) got on with postings despite big salary offers.
the blank. The word means to become (a) Disinclined
diminished. _ (b) ‘got away with’ is the most
appropriate phrase to be used for the (b) Eager
4. The football match had to be blank. The phrase means to escape (c) Fervent
........because of the weather. blame punishment or undesirable’ (d) Unrepentant
consequence for an act that is wrong
(a) called on (b) called off or mistaken. _ (a) ‘Disinclined’ is synonym of the
(c) called out (d) called over word ‘Reluctant’. Both words mean

_ (b) ‘called off’ is the appropriate


Directions (Q.Nos. 11-20) Each item unenthusiastic on unwilling.
phrase for the given blank. The in this sectioin consists of a sentence 16. The authorities have reprimanded
phrase means to cancel or abandon with an underlined word followed by to subordinate officer for violating
something. four words/group of words. Select the the protocol.
5. Nobody believed Ram at first but option that is the nearest in meaning to
(a) Extolled
he .......to be right. the underlined word and mark your
(b) Purported
response on your answer sheet
(a) came out (b) carried out (c) Admonished
accordingly.
(c) worked out (d) turned out (d) Required an apology
11. A provocative message had been
_ (d) ‘turned out’ is the appropriate doing rounds on social media to _ (c) ‘Admonished’ is the correct
phrase for the blank as it means to synonym of the word ‘Reprimanded’.
happen is a particular way. instigate the mob against migrants. Both words mean meaning is blamed
(a) Dexterous (b) Inflammatory or scolded.
6. How are you......in your new job?
(c) Valiant (d) Prudent 17. For Gandhiji, India’s religious and
Are you enjoying it?
(a) keeping on (b) going on _ (b) ‘Inflammatory’ and ‘Provocative’ linguistic diversity was an asset, not
are synonyms. Both words mean a liability.
(c) getting on (d) carrying on
arousing or intended to arouse angry
CDS Solved Paper 2019 (I) 59

(a) Obligation (b) Advantage Believer is an adherent of a particular 29. The thief had very vital information
(c) Attribute (d) Reinforcement religion.
to pass on to the police.
_ (b) ‘Advantage’ is the closest 23. According to GB Shaw, mean have (a) Crucial (b) Inessential
synonym to the word ‘Asset’. Both become inert. Therefore, life force (c) Indispensable (d) Fundamental
words mean benefit or advantage. has chosen women to perform its
_ (b) The underlined word ‘vital’ means
18. How hysterical he is! functions. absolutely necessary or essential. Its
(a) Berserk (b) Inconsistent (a) Lively (b) Quiescent correct antonym is ‘inessential’ which
(c) Duplicitous (d) Insincere. (c) Dormant (d) Apathetic means not necessary.

_ (a) ‘Berserk’ is the correct synonym of _ (a) The underlined word ‘inert’ means 30. His lectures are often wordy and
the word ‘Hysterical’. Both words motion less or still. Its correct antonym pointless.
mean crazy or out of control with anger from the given options would be ‘lively’ (a) Diffuse (b) Concise
excitement. and it means full of energy, moving or
(c) Garrulous (d) Voluble
active.
19. Mahesh is mostly prejudiced in his
24. Some of the men are highly _ (b) The underlined word ‘wordy’
political opinion. means unnecessary repetition of
(a) Objectionable (b) Predatory misanthropic. language. Its antonym is ‘concise’
(c) Jaundiced (d) Intimate (a) Anti-social (b) Philosophic which means very brief or to the point.
(c) Atrophic (d) Philanthropic
_ (c) The word ‘Jaundiced’ means the Directions (Q. Nos. 31-40) Each of
same as ‘Prejudiced’. Both words _ (d) The underlined word ‘misanthropic’ the following items in this section
mean biased or influenced. means unsociable or unfriendly’. Its
correct antonym would be
consists of a sentence, the parts of
20. Do not indulge in tautology. ‘philanthropic’ which means which have been jumbled. these parts
(a) Truth telling (b) Prolixity humanitarian, liberal, unselfish etc. have been labelled as P, Q, R and S.
(c) Foretelling given below each sentence are four
25. The teacher was a very profound man.
(d) Telepathic conversation sequences namely (a), (b), (c) and 9d).
(a) Sincere (b) Erudite You are required to rearrange the
_ (b) The word ‘Prolixity’ means the same (c) Scholarly (d) Superficial jumbled parts ofthe sentence and mark
as ‘Tautology’. Both words mean a
phrase or expression in which the same _ (d) The underlined word ‘profound’ your response accordingly.
thing is said twice in different words. means felt or experienced very
strongly or in an extreme way. Its 31. the prize money for refusing her
Directions (Q.Nos. 21-30) each item correct antonym from the given (P) (Q)
options is ‘superficial’ which means pepsico was ordered
in this sectioin consists of a sentence
concerned only what is obvious or
with an underlined word followed by apparent, not deep or serious. (R)
four words. Select the option that is to compensate the woman
opposite in meaning to the undeerlined 26. His hand-writing is readable.
(S)
word and mark your response on your (a) Well-written The correct sequence should be
answer sheet accordingly. (b) Decipherable (a) RSQP (b) SPQR
21. His religious views are rather fanatical. (c) Illegible (c) RPSQ (d) QRSP
(d) Comprehensible
(a) Bigoted (b) Rabid _ (a) RSQP is the correct sequence.
(c) Moderate (d) Militant _ (c) The word ‘readable’ means 32. trade operating from a colony
which can be read clearly. Its
_ (c) The underlined word ‘fanatical’ correct antonym is ‘illegible’ which (P)
means extremely interested in means the writing which is not clear or held a meeting
something, to a degree that some hard to read.
people find unreasonable. So, among (Q)
the given options, ‘moderate’ would 27. Mohan is his steadfast friends. demanding a probe into the illegal drug
be its correct antonym. Moderate (a) Committed (b) Unwavering (R)
means make or become less extreme (c) Unfaltering (d) Unreliable
interested in something.
the residents of the city
_ (d) The word ‘steadfast’ here means (S)
22. Religious fundamentalists often loyal or dedicated. Its correct antonym The correct sequence should be
consider the followers of other is ‘unreliable’ which means (a) QRSP (b) SPQR
religions to be heretics. untrustworthy or undependable. (c) SQRP (d) RSQP
(a) Dissenter (b) Believer 28. Radha often goes tempestuous _ (c) SQRP is the correct sequence.
(c) Renegade (d) Apostate while debating.
33. the university authorities cancelled
_ (b) The underlined word ‘heretics’ (a) Calm (b) Violent
the ongoing students’ union
means a person who belongs to a (c) Fierce (d) Vehement
particular religion but whose beliefs or election and
actions seriously disagree with the _ (a) The word ‘tempestuous’ means (P)
angry or violent. Its correct antonym is
principles of that religion. So, among following students’ unrest on campus
‘calm’ which means peaceful.
the given options, ‘believer’ is the
correct antonym of the word ‘heretics. (Q)
60 CDS Solved Paper 2019 (I)

closed till further orders common white-footed ants are the jumbled up and labelled as P, Q, R and
(R) best pollinators of S you are required to find the proper
declared the institution (R) sequence of the four sentences and
(S) bees might be the best known mark your response accordingly on the
The correct sequence should be pollinators but answer sheet.
(a) QRSP (b) QPSR (S) 41. S 1 : He no longer dreamed of
(c) SQRP (d) RSQP The correct sequence should be storms, nor of women, nor of
(a) PRSQ (b) SQRP
_ (b) QPSR is the correct sequence. great occurrences, nor of great
(c) QSRP (d) PQRS
fish, nor fights, nor contests of
34. brushed past the latter’s pet dog _ (b) SQRP is the correct sequence. strength, nor of his wife.
(P)
38. say from their forties onwards S 6 : He urinated outside the shack
stabbed to death by a man and then went up the road to
(P)
(Q) it is thus a good idea wake the boy.
after his vehicle accidentally P : He never dreamed about the
(Q)
(R) and continue to exercise early enough boy.
a cargo van driver was allegedly Q : He only dreamed of places and
(R)
(S) for senior citizens to start of the lions on the beach now.
The correct sequence should be
(a) QRSP (b) QPSR (S) R : He simply woke, looked out
(c) SQRP (d) SQPR The correct sequence should be through the open door at the
(a) PRSQ (b) QRSP moon and unrolled his trousers
_ (c) SQRP is the correct sequence. (c) QSRP (d) PQRS and put them on.
35. an earthquake and tsunami
_ (c) QSRP is the correct sequence. S : They played like young cats in
(P) the dusk and he loved them as
the disaster mitigation agency 39. scientists have determined
(P) he loved the boy.
(Q) The correct sequence should be
said that the death toll from injury in animals and humans
(a) RQPS (b) SRQP
(R) (Q)
(c) QSPR (d) RRSQ
in Indonesia has crossed 1500 that is linked to the severity of
(S) spinal cord _ (c) QSPR is the correct sequence.
The correct sequence should be (R) 42. S 1 : We do not know, after 60 years
(a) PQSR (b) RPSQ (c) SQRP (d) QRPS a gene signature of education, how to protect
_ (d) QRPS is the correct sequence. (S) ourselves against epidemics
The correct sequence should be like cholera and plague.
36. scientists say they have developed a
(a) PSRQ (b) QRPS S 6 : This is the disastrous result of
new
(c) QSPR (d) PQRS the system under which we are
(P) educated.
illnesses such as heart disease and _ (a) PSRQ is the correct sequence.
cancer 40. like a muscle and repeating the process P : If our doctors could have
started learning medicine at an
(Q) (P)
earlier age, they would not
DNA tool that uses machine and stable reading circuit
make such as poor show as
learning to accurately (Q) they do.
(R) helps the child build a strong
predict people’s height and assess (R) Q : I have seen hundreds of homes.
their risk for serious the brain works I cannot say that I have found
any evidence in them of
(S) (S)
The correct sequence should be knowledge of hygiene.
The correct sequence should be
(a) PRSQ (b) RPSQ R : I consider it a very serious blot
(a) QSRP (b) SPRQ
(c) PSRQ (d) QRPS on the state of our education
(c) QSPR (d) RQPS
_ (a) PRSQ is the correct sequence. that our doctors have not found
_ (b) SPRQ is the correct sequence.
37. a rare evergreen tree in the it possible to eradicate these
Southern-Western Ghats Directions (Q. Nos. 41-50) In this diseases.
section each item consists of six
(P) S : I have the greatest doubt
sentences of a passage. The first and
researchers have found that whether our graduates know
sixth sentences are given in the
(Q) begining as S1 and S6. The middle what one should do in case one
four sentences in each have been is bitten by a snake.
CDS Solved Paper 2019 (I) 61

The correct sequence should be The correct sequence should be R : Nadia Murad is herself a
(a) RQSP (b) PRQS (a) PQRS (b) PRQS survivor of sexual war crimes,
(c) QRPS (d) PQSR (c) QSPR (d) QSRP perpetuated by IS against the
_ (c) QRPS is the correct sequence. _ (b) PRQS is the correct sequence. Yazidis.
43. S 1 : The weak have no place here, 45. S 1 : India’s museums tend to be S : Today she campaigns tirelessly
in this life or in any other life. dreary experiences. to put those IS leaders in the
Weakness leads to slavery. S 6 : Because it’s better to attract dock in international courts.
S 6 : This is the great fact : strength crowds than dust. The correct sequence should be
is life weakness is death. (a) PQRS (b) PRQS
P : Even the Louvre that attracted
Strength is felicity, life eternal; (c) SRQP (d) QRSP
an eye-popping 8.1 million
immortal; weakness is constant visitors last year compared to _ (a) PQRS is the correct sequence.
strain and misery; weakness is India’s 10.18 million foreign 47. S 1 : Few scientists manage to break
death. tourists, has hooked up with down the walls of the so-called
P : They dare not approach us, Beyonce and Jay-Z for ivory tower of academia and
they have no power to get a promotion, where they take a touch and inspire people who
hold on us, until the mind is selfie with Mona Lisa. may not otherwise be
weakened. Q : Our museums need to get cool interested in science.
Q : Weakness leads to all kinds of too. S 6 : Not many would have 6
misery, physical and mental. R : A change of approach is clearly survived this, let alone excelled
Weakness is death. called for. in the manner he did.
R : But they cannot harm us unless P : Stephen Hawking was one of
S : Troops of restless
we become weak, until the these few.
schoolchildren are often the
body is ready and predisposed Q : Around this time he was
most frequent visitors, endlessly
to receive them. diagnosed with Amyotrophic
being told to lower their voices
S : There are hundreds of thousands and not touch ther art. Lateral Sclerosis, an incurable
of microbes surrounding us. motor neuron disease and
The correct sequence should be
The correct sequence should be (a) PQRS (b) PRSQ given two years to live.
(a) PQRS (b) PRQS (c) SRPQ (d) QSRP R : Judging by the odds he faced as
(c) QRSP (d) QSRP
_ (c) SRPQ is the correct sequence. a young graduated student of
_ (d) QSRP is the correct sequence. physics at Cambridge
44. S 1 : The Nobel Prize for Economics 46. S 1 : A decade ago UN recognised University, nothing could have
in 2018 was awarded to Paul that rape can constitute a war been a more remote possibility.
Romer and William Nordhaus crime and a constitutive act of S : When he was about 20 years
for their work in two separate genocide. old, he got the shattering news
areas : economic growth and S 6 : The fact that these two peace that he could not work with the
environmental economics laureates come from two great Fred Hoyle for his PhD,
respectively. different nations underlines as he had aspired to.
S 6 : Among recent winners of that this problem has been The correct sequence should be
Nobel Prize in Economics, it’s widespread, from Rwanda to (a) PQSR (b) PRQS
hard to think of one issue Myanmar. (c) SRPQ (d) PRSQ
which is more topical and P : This year’s Nobel Peace Prize
relevant to India. has been awarded to two _ (d) PRSQ is the correct sequence.
P : But there is a common thread exceptional individuals for 48. S 1 : The climate question presents a
in the work. their fight to end the use of leapfrog era for India’s
sexual violence as the weapon development paradigm.
Q : In economic jargon it’s termed S 6 : This presents a good template
as externality. of war.
for India, building on its
R : Productive activity often has Q : Denis Mukwege is a doctor
existing plans to introduce
spillovers, meaning that is can who has spent decades treating
electric mobility through buses
impact and unrelated party. rape survivors in the
first and cars by 2030.
Democratic Republic of Congo,
S : Romer and Nordhaus both P : It is aimed at achieving a shift
where along civil war has
studied the impact of externalities to sustainable fuels, getting
repeatedly witnessed the
and came up with profound cities to commit to eco-friendly
horror of mass rapes.
insights and economic models. mobility and delivering more
62 CDS Solved Paper 2019 (I)

walkable communities, all of _ (d) QSPR is the correct sequence. tried constantly to open new markets for
which will improve the quality 50. S 1 : In a globalised world, no Indian goods in Britain and other
of urban life. country can hope to impose countries.
Q : At the Bonn conference, a new tariffs without affecting its Thereby, it increased the export of
Transport Decerbonisation own economic interests. Indian manufacturers and thus
Alliance has been declared. S 6 : The ongoing trade war also encouraged their production. This is the
R : This has to be resolutely threatens the rules-based global reason why Indian rulers tolerated and
pursued, breaking down the trade order which has managed even encouraged the establishment of
barriers to wider adoption of the Company’s factories in India. But,
to amicably handle trade disputes
rooftop solar energy at every from the very beginning, the British
between countries for decades.
level and implementing net manufacturers were jealous of the
P : So both the US and China, popularity that India textiles enjoyed in
metering systems for all which have blamed each other Britain. All of a sudden, dress fashions
categories of consumers. for the ongoing trade war, are changed and light cotton textiles began
S : Already, the country has chalked doing no good to their own to replace the coarse woolens of the
out an ambitious policy on economic fortunes by engaging English. Before, the author of the
renewable energy, hoping to in this tit-for-tat tariff battle. famous novel, Robinson Crusoe,
generate 175 gigawatts of power Q : Apart from disadvantaging its complained that Indian cloth had “crept
from green sources by 2022. consumers, who will have to into houses, our closets and bed
The correct sequence should be pay higher prices for certain chambers curtains, cushions, chairs and
(a) SRQP (b) SPRQ goods, tariffs will also disrupt at last beds themselves were nothing but
(c) PRSQ (d) QRSP the supply chain of producers calicos or India stuffs”.
_ (a) SRQP is the correct sequence. who rely on foreign imports. The British manufacturers put pressure
R : China, which is fighting an on their government, to restrict and
49. S 1 : The dawn of the information age
economic slowdown, will be prohibit the sale of Indian goods in
opened up great opportunities
equally affected. England. By 1720, laws had been passed
for the beneficial use of data.
forbidding the wear or use of printed or
S 6 : To some, in this era of big data S : The minutes of the US Federal dyed cotton cloth. In 1760 a lady had to
analytics and automated, Reserve June policy meeting pay a fine of 200 for possessing an
algorithm based processing of show that economic uncertainty imported handkerchief ! Moreover,
zettabytes of information, the due to the trade war is already heavy duties were imposed on the import
fear that their personal data may affecting private investment in of plain cloth. Other European countries,
be unprotected may conjure up the US, with many investors except Holland, also either prohibited
visions of a dystopian world in deciding to scale back or delay the import of Indian cloth or imposed
which individual liberties are their investment plans. heavy import duties. In spite of these
compromised. The correct sequence should be laws, however, Indian silk and cotton
P : But it is the conflict between (a) SQPR (b) QPSR textiles still held their own in foreign
the massive scope for progress (c) QRPS (d) PSRQ markets, until the middle of the
provided by the digital era and _ (b) QPSR is the correct sequence. eighteenth century when the English
the fear of loss of individual textile industry began to develop on the
autonomy that is foregrounded Directions (Q. Nos. 51-70) In this basis of new and advanced technology.
in any debates about data section you have few short passages. 51. The East India Company was
protection laws. After each passage, you will find some encouraging the export of Indian
Q : It also enhanced that perils of items based on the passage. First, read
manufacturers because
unregulated and arbitrary use a passage and answer the items based
(a) it was a philanthropic trading
of personal data. on it you are required to select your
answer based on the contents of the corporation
R : It is against this backdrop that (b) it wanted Indian manufacturers to
passage and opinion of the author
the white paper made public to prosper in trade and commerce
only.
elicit views from the public on (c) it profited from the sale of Indian
the shape and substance of a PASSAGE 1 goods in foreign markets
comprehensive data protection From 1600 to 1757 the East India (d) it feared Indian Kings who would
laws assumes significance. Company’s role in India was that of a not permit them trade in India
S : Unauthorised leaks, backing trading corporation which brought goods
and other cyber crimes have or precious metals into India and _ (c) East India Company was
encouraging the export of Indian
rendered data bases vulnerable. exchanged them for Indian goods like
goods as it got huge profits by the sale
The correct sequence should be textiles and spices, which it sold abroad.
of Indian goods in foreign markets.
(a) SQRP (b) QPRS Its profits came primarily from the sale
(c) SRPQ (d) QSPR of Indian goods abroad. Naturally, it
CDS Solved Paper 2019 (I) 63

52. The people of England used Indian Emmerson Mnangagwa. A battle to 57. Mrs Mugabe is supported by
cloths because succeed the 93-years-old liberation (a) Mr Mnangagwa
(a) they loved foreign and imported hero-turned. President had already (b) Mr Mugabe
clothes been brewing within the ruling (c) Generation 40
(b) the Indian textile was light cotton Zimbabwe African National (d) Zanu-PF
(c) the Indian cloths were cheaper Union-Patriotic Front (Zanu-PF), with
the old guard backing Mr Mnangagwa, _ (c) As stated in the passage Mrs
(d) the Indian cloths could be easily Mugabe was supported by Generation
transported himself a freedom fighter and 40, a group of younger leaders.
‘Generation 40’, a grouping of younger
_ (b) The British people used Indian leaders supporting Mr Mugabe’s 58. Mr Mugabe’s political weakness
clothes because the clothes were light became apparent when
52-years-old wife, Grace. Ms Mugabe,
cotton and of breathable fabric.
known for her extravagant lifestyle and (a) he endorsed his wife
53. What did the British manufacturer interfering ways, has been vocal in (b) he turned against the army
do to compete with the Indian recent months about her political (c) he suffered from health issues
manufacturers? ambitions. ‘Mr Mugabe was seen to (d) he dismissed Mr Mnangagwa
have endorsed her when on November 6
(a) They pressurised the government
he dismissed Mr Mnangagwa. But Mr
_ (d) Mr Mugabe’s political weakness
to levy heavy duties on export of became apparent when he dismissed
Mugabe, who has ruled Zimbabwe since Mr Mnangagwa, who was the
Indian clothes
its independence in 1980, erred on two Vice-President of Zimbabwe.
(b) They pressurised the government
counts: he underestimated the deep
to levy heavy duties on import of 59. The secuirty forces of Zimbabwe
connections Mr Mnangagwa has within
Indian clothes staged a coup against the President
the establishment and overestimated
(c) They requested people to change because
his own power in a system he has
their fashion preferences (a) they wanted Mrs Mugabe as the
helped shape. In the good old days, Mr
(d) They lowered the prices of the President
Mugabe was able to rule with an iron
Britain made textile (b) they were aware of Mugabe’s
grip. But those days are gone. Age and
_ (b) The British manufacturers were health problems have weakened his failing wealth
jealous of Indian manufacturers and hold on power, while there is a (c) they disliked Mugabe’s
they pressurised the British groundswell of anger among the public extravagant lifestyle
Government to levy heavy duties on (d) they did not want a Mugabe
over economic mismanagement. So,
import of Indian clothes.
when he turned against a man long dynasty
54. Which source is cited by the author seen by the establishment as his
_ (d) The security forces of Zimbabwe
to argue that Indian textile was in successor, Mr Mugabe left little doubt staged a coup against the President
huge demand in 18th century that he was acting from a position of because they did not want a Mugabe’s
England? political, weakness. This gave the dynasty.
(a) The archival source security forces the confidence to turn 60. Why does the military not want to
(b) The scientific source against him and make it clear they call it a coup d’etat?
(c) The journalistic source didn’t want a Mugabe dynasty. The
(a) Because coup is immoral
(d) The literary source military doesn’t want to call its action a
(b) Because coup is illegal
coup detat, for obvious reasons.
_ (d) The literary source, novel (c) Because coup would lead to
‘Robinson Crusoe’ is cited by the A coup would attract international international censure and
author to argue that Indian textile was condemnation, even sanctions. But it is sanctions
in huge demand in 18th century certain that the army chief, Gen (d) Because it would make the public
England. Constantino Chiwenga, is in charge, His
revolt
55. “New and advanced technology” in plan, as it emerges, is to force Mr
Mugabe to resign and install a _ (c) The military does not want to call it
the paragraph refers to a coup d’etat as coup would lead to
transitional government, perhaps
(a) the French Revolution international censure and sanctions.
under Mr Mnangagwa, until elections
(b) the Glorious Revolution of England
(c) the Industrial Revolution
are held. PASSAGE 3
(d) the Beginning of Colonialism 56. In the paragraph, who has been Over-eating is one of the most wonderful
called liberation hero? practices among those who think that
_ (c) New and Advanced technology in (a) Constantino Chiwenga
they can afford it. In fact, authorities say
the paragraph refers to the Industrial that nearly all who can get as much as
Revolution. (b) Emmerson Mnangagwa
they desire, over-eat to their
(c) Robert Mugabe
PASSAGE 2 (d) Army Chief
disadvantage. This class of people could
save a great more food then they can
Zimbabwe’s prolonged political crisis
reached the boiling point earlier this _ (c) As stated in the passage save by missing one meal per week and
93-years-old Robert Mugabe. the at the same time they could improve
month when President Robert Mugabe President of Zimbabwe has been their health. A heavy meal at night, the
dismissed the Vice-President, called a liberation hero. so-called ‘dinner’, is the fashion with
64 CDS Solved Paper 2019 (I)

many and often is taken shortly before (b) Two times own religion and the destruction of the
retiring. It is unnecessary and could be (c) Once others, I pity him from the bottom of my
forgone, not only once a week but daily (d) Has not been specified heart and point out to him that upon the
without loss of strength. From three to
five hours are needed to digest food. _ (d) It has not been specified in the banner of every religion will soon be
passage as how many times a day written in spite of resistance: ‘Help and
While sleeping, this food not being should we have food? not fight,’ ‘Assimilation and not
required to give energy for work, is in Destruction,’ ‘Harmony and Peace and
many cases converted into excess fat, 66. According to the passage, people
not Dissension.’
giving rise to over-weight. The evening over-eat
meal should be light, taken three or four (a) because they can afford to 67. According to the author of the
hours before retiring. This prevents (b) because they are hungry passage, people should
over-eating, conserves energy and (c) because they have to work more (a) change their religions
reduces the cost of food. (d) because they have to conserve (b) follow their religions and persuade
61. Why should those who over-eat energy others to follow it
(c) follow their own religions and
refrain from doing so? _ (a) According to the passage, people respect other religions
(a) Because over-eating leads to loss over eat because they can afford and (d) disrespect other religions
of wealth can get as much as they desire
(b) Because over-eating is bad for generally over-eat the food. _ (c) According to the passage, people
should follow their own religions and
health PASSAGE 4 respect other religions also.
(c) Because over-eating conserves food
(d) Because over-eating is immoral
Much has been said of the common 68. The Parliament of Religions is
ground of religious unity. I am not going (a) a Christian organisation
and unhealthy just now to venture my own theory. But (b) a Buddhist organisation
_ (b) Over-eating should be refrained as if anyone here hopes that this unity will (c) a Hindu organisation
it is bad for health and a waste of food. come by the triumph of anyone of the
(d) a platform for discussion about
62. Over-eating is more prevalent religions and the destruction of the
every religion of the world
among others, to him I say, “Brother, yours’ is
an impossible hope.” Do I wish that the _ (d) It is a platform for discussion about
(a) the rich every religion of the world.
Christian would become Hindu? God
(b) the poor
(c) everybody
forbid. Do I wish that the Hindu or 69. What does the author think about
Buddhist would become Christian? God those who dream about the exclusive
(d) the bourgeoisie
forbid. survival of their own religions and
_ (a) Actually over-eating is more The seed is put in the ground and earth the destruction of the others?
common among those who can afford
and air and water are placed around it. (a) He hates them
the food.
Does the seed become the earth, or the (b) He desires to imprison them
63. The writer is asking the readers air, or the water? No. It becomes a plant. (c) He pities them
(a) to skip the heavy dinner and take It develops after the law of its own (d) He praises them
light evening meal instead growth, assimilates the air, the earth
(b) to stop eating anything at night _ (c) There is nothing like ‘My Religions’
and the water, converts them into plant only. The author takes a pity on those
(c) to take food only during the day substance and grows into a plant. people who dream about the exclusive
(d) to eat food before the sunset Similar is the case with religion. The survival of their own religions and the
destruction of the others. All religions
_ (a) The writer is asking the readers to Christian is not to become a Hindu or a
are same and all basics of any religion
skip the heavy dinner just before Buddhist, nor a Hindu or a Buddhist to
retiring.They should, instead take light are also the same.
become a Christian. But each must
evening meals. 70. According to the passage, what is
assimilate the sprit of the others and yet
64. What is the most appropriate time preserve his individuality and ‘impossible hope’?
for having evening meal? grow according to his own law of growth. (a) One day, all the people of the
(a) An hour after the sunset If the Parliament of Religions has shown world will follow only one religion
(b) Three or four hours before sleeping anything to the world, it is this: it has (b) One day, there will be no religion
(c) Before the sunset proved to the world that holiness, purity (c) Purity and charity are the
(d) Just before sleeping and charity are not the exclusive exclusive possessions
(d) Banner of every religion will soon
_ (b) Evening meal, ideally, should be possessions of any church in the world
taken three or four hours before sleeping. and that every system has produced men be written

65. According to the passage, how many


and women of the most exalted _ (a) According to the passage, the
character. impo- ssible hope is that one day, all
times a day should we have food? the people of the world will follow only
In the face of this evidence, if anybody
(a) Three times one religion.
dreams of the exclusive survival of his
CDS Solved Paper 2019 (I) 65

Directions (Q. Nos. 71-90) Each item 78. With a population of over one 86. Anywhere in the world(a)/ when
in this section has a sentence with three billion, (a)/ India is second most there is conflict(b)/ women and
parts labelled as (a), (b) and (c). Read popular country(b)/ in the world children suffer the mos.(c)/No error(d)
each sentence to find out whether there ofter China.(c)/ No error(d)
_ (b) For place, ‘where’ should be used
is any error in any part and indicate instead of ‘when’. So, here in part (b)
you response on the answer sheet _ (b) Here, article ‘the' should be used
before second as ‘most’ is the replace ‘when’ with ‘where’ to make the
against the corresponding letter i.e., superlative degree word. Before given sentence grammatically correct.
(a) or (b) or (c). If you find no error, superlative degree, use of ‘the’ is must.
87. The man is(a)/ the foundational director
your response should be indicated 79. There are hundred of superstitions(a)/ (b)/ of this company.(c)/ No error(d)
as (d)
which survive(b)/ in the various
71. Except for few days(a)/in a year _ (b) Here, in part (b) of the given
parts of the country. (c)/ No error. (d) sentence, article ‘a’ will be used in
during the monsoon(b)/ the river (d) There is no error in the sentence as place of ‘the’ before ‘foundational’ to
cannot flow on its own.(c)/ No it is grammatically correct sentence. make the sentence more appropriate.
error.(d)
80. It is(a)/ in the temperate countries of 88. Parents of LGBT community members(a)/
_ (b) There is no error, sentence is are coming in(b)/ with a little help
grammatically correct. Northern Europe(b)/ that the
beneficial effects of cold is most from NGOs.(c)/ No error(d)
72. Being apprised with our approach clearly manifest.(c)/ No error(d) _ (b) Here, ‘coming in’ should be
(a)/ the whole neighbourhood (b)/ replaced with ‘coming up’ to make the
came out to meet the minister.(c)/ _ (c) With plural noun ‘effects’ verb ‘are’ given syntax correct.
should be used in place of ‘is’ to make
No error(d) the given sentence grammatically 89. To love one art form is great(a)/ but
_ (a) Here, in part (a) ‘of’ should be used correct. to be able to appreciate another(b)/
in place of ‘with’ because the word and find lateral connections are
‘apprised’ is followed by preposition 81. The effects of female employment(a)/
‘of’. on gender equality(b)/ now appear priceless.(c)/No error(d)
73. The celebrated grammarian
to be trickling at the next generation. _ (d) There is no error and the sentence
(c)/No error(d) is grammatically correct.
Patanjali(a)/ was(b)/ a
contemporary to Pushyamitra _ (c) Here in part (c) preposition ‘to’ is 90. Female literacy rate has gone
used in place of ‘at’ after the word up by11%(a) in the past decade as
Sunga.(c)/No error(d) ‘trickling’ to make the given sentence opposed to(b)/ a 3% increase in male
_ (c) Here, remove ‘to’ and use ‘of’ correct. literacy.(c)/No error(d)
before pushyamitra to make the given
sentence grammatically correct.
82. Since the 15 minutes that she _ (c) Write ‘literacy rate’ in place of
drives,(a)/ she confesses that she ‘literacy’ to make the sentence
74. His appeal for funds(a)/ met(b)/ a feels like(b)/ a woman with meaningfully and grammatically correct.
poor responses. (c)/ No error (d)/ wings.(c)/ No error(d) Directions (Q. Nos. 91-100) Each of
_ (b) Here, in part(b) ‘met with’ should be _ (a) Remove ‘since’ and use ‘in’ here. the following sentences in this section
used in place of ‘met’ to make the given Use of since is incorrect according to
sentence grammatically correct. has a blank space with four words or
the given syntax.
group of words given. Select whichever
75. Buddhism teaches that(a)/ freedom 83. India won(a)/ by an innings(b)/ and word or group of words you consider
from desires(b)/ will lead to escape three runs.(c)/No error(d) most appropriate for the blank space
suffering. (c)/ No error (d) and indicate your response on the
_ (d) The sentence has no error .
_ (c) Write ‘from’ between escape and answer sheet accordingly.
suffering to make the given sentence CLOZE COMPREHENSION 1
grammatically correct. 84. Each one(a)/ of these chairs(b)/ are The question whether war is ever
76. This hardly won liberty(a)/ was not broken.(c)/ No error(d) justified, and if so under what
to be (b)/ lightly abandoned. (c)/ No _ (c) Here, in part (c) singular verb circumstances, is one which has
error (d) ‘is’ should be used in place of been forcing itself ………
‘are’ because the subject of
_ (a) Here use of adverb hardly is the sentence (Each one) is 91. (a) upon (b) on
incorrect. It should be ‘This hard’ won
liberty to make the sentence
singular, so verb also be used in (c) at (d) over
singular form. the attention of all thoughtful
grammatically correct.
85. Few creature(a)/ outwit(b)/ the fox men. On this question I find
77. My friend said(a)/ he never myself in the somewhat ……… .
remembered(b)/ having read a more in Aesop’s Fables.(c)/ No error(d)
92. (a) delightful position of
enjoyable book.(c)/ No error(d)/ _ (a) Here, in part(a) ‘Few creatures’
should be used in place of ‘Few (b) painful
_ (d) No error, the sentence is creature’ to make the given syntax (c) pleasant
grammatically correct. correct. (d) lovely
66 CDS Solved Paper 2019 (I)

holding that no single one of the 92. (b) In the given context, ‘painful’ is the variants of naturally occurring
combatants is justified in the appropriate word to fill the given enzymes that could be used to
blank.
present war, while not taking the ……… .
93. (d) With respect to the word ‘war’,
extreme Tolstoyan view that war option (d) ‘crime’ is the appropriate 104. (a) constitute
is under all circumstances a ……… choice. (b) sink
93. (a) duty (b) obligation 94. (a) ‘Feeling’ is the appropriate word to (c) manufacture
(c) responsibility (d) crime fill the given blank according to given (d) resolve
Opinions on such a subject as war context. ‘Feeling’ is an emotional state biofuels and pharmaceuticals. The
or reaction.
are the outcome of ……… other half went to George P
95. (c) Here, conjunction ‘both’ will be
94. (a) feeling (b) sentiment used to fill the given blank as ‘both Smith, also of the US and Sir
(c) reason (d) patriotism …… and’ is a conjunction pair used Gregory P Winter, from the UK,
rather than of thought : given a to join one statement or fact to who evolved antibodies to …………
man’s emotional temperament, his another. 105. (a) combat
convictions, ………… 96. (c) Here, ‘suggested’ is the (b) support
appropriate word to fill the given blank
95. (a) however (b) as well as as it means ‘put-forward for (c) observe
(c) both (d) despite consideration’. (d) invite
on war in general and on any 97. (a) With the word ‘ethical’, option (a) autoimmune diseases and even
particular war which may occur ‘questions’ is the suitable choice. metastatic cancer through a
during his lifetime, can be ……… ‘Ethical questions’ are those process called phage display.
questions that involve consideration of
96. (a) thought (b) intimated conflicting moral choices and _ Solutions (Q.Nos. 101-105)
(c) suggested (d) held dilemmas, with several alternative 101. (a) The whole passage is about
with tolerable certainty. The solutions. evolution and its scientific usage. So,
arguments used will be mere 98. (c) In the given context, option (a) option (a) ‘evolution’ is the correct
‘considered’ is the appropriate word to fill the given blank. ‘Evolution’
reinforcements to convictions is the gradual development of
choice.
otherwise reached. The something over a period of time.
99. (a) In the given context, ‘considered’
fundamental facts in this as in all is the appropriate word to fill the given 102. (d) Experiments are done is the
ethical ……… blank. ‘Considered’ means ‘having laboratory to get amazing results. So,
97. (a) questions (b) answer been thought about carefully’. option (d) ‘laboratory’ is the suitable
100. (d) With the word ‘juridical or choice in the given context.
(c) statements (d) experiences
quasi-juridical’ option (d) ‘stand point’ 103. (b) The word ‘synthesize’ means ‘to
are feelings; all that thought can produce a substance by combining
is the appropriate choice. The word
do is to clarify and systematise the ‘stand point’ means ‘a set of beliefs other substances chemically’. Hence,
expression of those feelings and it and ideas from which opinions and it is best suited for the given blank in
is such clarifying and decisions are formed’. the given context.
systematising of my own feelings 104. (c) ‘Biofuels and pharmaceuticals’ are
CLOZE COMPREHENSION 2 manufactured with the help of
that I wish to ……… naturally occurring enzymes. So,
98. (a) engage (b) praise The Nobel Prize for Chemistry
option (c) ‘manufacture’ is the suitable
(c) attempt (d) commend this year is a tribute to the power choice in the given context.
in the present article. In fact, the of ……… 105. (a) In context of autoimmune
question of rights and wrongs of a 101. (a) evolution diseases, ‘combat’ is the correct word
(b) devolution. to fill the given blank as it means ‘to
particular war is generally ……… take action to reduce or prevent
99. (a) considered (c) revolution.
something bad or undesirable’.
(b) observed (d) involution.
(c) transferred The laureates harnessed evolution Directions (Q. Nos. 106-120) given
(d) opined and used it in the ……… below are some idioms/phrases
from a juridical or quasi-juridical 102. (a) microscope followed by four alternatives meaning
(b) field (c) market to each. choose the response (a), (b), (c)
………
(d) laboratory or (d) which is the most appropriate
100. (a) possibility. expression.
(b) formula. with amazing results. Frances H
Arnold, an American who was 106. A match made in heaven
(c) force. (a) a marriage that is solemnised formally
(d) standpoint given one-half of the prize, used
‘directed evolution’ to ……… (b) a marriage that is unsuccessful
_ Solutions (Q.Nos. 91-100) 103. (a) inhibit
(c) a marriage that is likely to be
91. (a) With the word ‘itself’, preposition happy and successful
‘upon’ is the suitable choice.
(b) synthesize (d) a marriage of convenience
(c) hamper
(d) hold back _ (c) The idiom ‘A match made in heaven’
means ‘a very successful marriage as
CDS Solved Paper 2019 (I) 67

both partners are compatible with _ (b) The idiom ‘A rotten apple’ means (b) express the correct meaning of
each other’. So, option (c) express the ‘one bad person in a group of good given idiom.
correct meaning of given idiom. people’. So, option (b) express the 117. Cook the books
107. A culture vulture correct meaning of given idiom.
(a) to record false information in the
(a) someone who is very keen to 112. To vote with your feet accounts of an organisation
experience art and literature (a) to show that you do not support (b) to do something that spoils
(b) someone who wants to defend something someone’s plan
ancient culture (b) to replace something important (c) to tell a false story
(c) someone who is ashamed of one’s (c) to change something you must do (d) to be very angry
own culture (d) to express a particular opinion
(d) someone who looks at her/his _ (a) The idiom ‘Cook the books’ means
_ (a) Idiom ‘To vote with your feet’ ‘to record false information in the
culture critically means ‘to show that you do not support accounts of an organisation’. So,
something’. So, option (a) express the option (a) express the correct meaning
_ (a) Idiom ‘A culture vulture’ means ‘a correct meaning of given idiom. of given idiom.
person who is very keen to experience
art and literature’. So, option (a) express 113. Verbal diarrhea 118. Change your tune
the correct meaning of given idiom. (a) to be sick (a) to listen to good music
108. A death blow (b) to talk to much (b) to do things that you are not
(a) to be nearly dead (c) to be in a difficult situation willing to
(b) to be deeply afraid of death (d) to be a good orator (c) to change your opinion completely
(c) to beat someone to death because it will bring you an
(d) an action or event which causes _ (b) The idiom ‘Verbal diarrhea’ means advantage
something to end or fail ‘to talk to much’. So, option (b)
(d) to pretend to be very friendly
express the correct meaning of given
_ (d) The idiom ‘A death blow’ means idiom. _ (c) The idiom ‘Change your tune’
‘an action or event which causes 114. To sail close to the wind means ‘to change one’s opinion
something to end or fail’. So, option (d) completely as it will bring you
express the correct meaning of given (a) to pretend to be something that an advantage’. So, option (c)
idiom. you are not express the correct meaning of given
109. The jewel in the crown (b) to be in some unpleasant situation idiom.
(a) someone who has many skills (c) to be destroyed by a belief 119. Blue blood
(b) something that one wants (d) to do something that is dangerous (a) to swallow poison
(c) the most valuable thing in a group _ (d) The idiom ‘To sail close to the wind’ (b) to be overly interested in
of things means ‘to do something that is someone
(d) the jewel in the crown of the king dangerous or that may be illegal or (c) to suddenly become jealous
improper’. So, option (d) express the (d) to belong to a family of the highest
_ (c) Idiom ‘The jewel in the crown’ correct meaning of given idiom.
means ‘the most valuable thing in a social class
group of similar things’. So, option (c)
115. A double entendre
express the correct meaning of given (a) to look at someone or something twice _ (d) The idiom ‘Blue blood’ means ‘a
member of a wealthy upper class
idiom. (b) a situation in which you cannot family or ancestry’. So, option (d)
110. To live in a fool’s paradise succeed express the correct meaning of given
(a) to live a life that is dishonest (c) a word which has two meanings idiom.
(b) to be happy because you will not (d) something that causes both 120. Cut the crap
accept how bad a situation really is advantages and problems (a) an impolite way of telling someone
(c) to believe that things you want will to stop saying things that are not
happen _ (c) The idiom ‘A double entendre’
means ‘a word or phrase that has two true
(d) to enjoy yourself by spending a lot
meanings’. So, option (c) express the (b) to stop needing someone else to
of money correct meaning of given idiom. look after you
_ (b) Idiom ‘To live in a fool’s paradise’ 116. To cut your own throat (c) to talk about something important
means ‘to be happy because one do
(a) to stop doing something (d) to upset someone by criticising
not know or will not accept how bad a
situation really is’. So, option (b) express (b) to do something because you are them
the correct meaning of given idiom. angry
_ (a) The idiom ‘Cut the crap’ means
111. A rotten apple (c) to behave in a relaxed manner ‘a rude way of telling someone to
(a) to remove something which is rotten (d) to allow someone to do something stop saying things that are not true
(b) one bad person in a group of good or not important’. So, option (a)
_ (b) The idiom ‘To cut you own throat’ express the correct meaning of given
people means ‘to do something which may
idiom.
(c) a loving and kind person cause problem for oneself’. So, option
(d) a disorganised person with bad habits
68 CDS Solved Paper 2019 (I)

PAPER III General Studies


1. Henry T Colebrooke was a Later, Santhal Pargana was _ (b) The Settlement Act of 1856 was
Professor of Sanskrit in which one constituted by the Britishers. introduced under the guidance of Lord
of the following institutions? Dalhousie in Awadh region, this
4. The Limitation Law, which was settlement was not in favour of
(a) Fort William College passed by the British in 1859, Talukdar as it gave priority to village
(b) Serampore Mission addressed which one of the Zamindar.
(c) Kashi Vidyapith following issues? As a result of this act, Talukdars were
(d) Asiatic Society left as a interlopers with no permanent
(a) Loan bonds would not have any
stake in the land.
_ (a) Henry Thomas Colebrooke was a legal validity
Sanskrit scholar and orientalist. He (b) Loan bonds signed between 7. The Inter-State Council was set-up
was appointed to a writership in India moneylender and ryots would in 1990 on the recommendation of
during that time he learnt Sanskrit. (a) Punchhi Commission
have validity only for three years
He was a Professor of Sanskrit at (c) Land bonds could not be executed (b) Sarkaria Commission
college of Fort William College in (c) Rajamannar Commission
by moneylenders
1805. He was elected as President of (d) Mungerilal Commission
the Asiatic Society of Calcutta. He (d) Loan bonds would have validity
also translated two treatises, the for ten years _ (b) Article-263 in Part XI of the
Mitacshara of Vijnaneshwara and the Constitution provides for the
Dayabhaga of Jimutavahana in
_ (b) The Limitation Law was passed by establishment of an Inter-State
the British in 1859 with an objective
English. Council for better coordination among
that loan bond signed between money
the states and between Centre and
2. The Deccan Agriculturalists’ Relief lenders and ryots would have validity
States.
only for three years.
Act of 1879 was enacted with which In 1990, the Inter-State Council was
It took final shape when government
one of the following objectives? of India enacted Limitation Act in established on the recommendation
(a) Restore lands to the dispossessed 1963. of Sarkaria Commission (1983-87).
peasants It consists of the following members.
5. Who among the following (i) Prime Minister as the Chairman
(b) Ensure financial assistance to
peasants during social and
was known during the days of (ii) Chief Minister of all the States and
religious occasions
the Revolt of 1857, as ‘Danka Union Territories
(c) Restrict the sale of land for Shah’? (iii) Administrators of Union Territories
indebtedness to outsiders (a) Shah Mal not having Assemblies
(d) Give legal aid to insolvent (b) Maulvi Ahmadullah Shah (iv) Governor’s of States under
peasants (c) Nana Sahib President rule
(d) Tantia Tope (v) Six Central Cabinet Ministers
_ (d) Deccan Agriculturalists Relief Act
in 1879 was enacted with the _ (b) Maulvi Ahmadullah Shah was 8. Which among the following writs
objective to give legal aid to insolvent known as ‘Danka Shah’, was one of is issued to quash the order of a
peasants. the leading figures of the great Revolt Court or Tribunal?
of 1857.
In Deccan region of Maharashtra, (a) Mandamus (b) Prohibition
money lenders from Gujarat exploited He gave tough fight to the Britishers in
(c) Quo Warranto (d) Certiorari
peasants, usurped their land. the Awadh region of Uttar Pradesh,
This led to large scale riots in 1875.
even liberated Faizabad from British _ (d) Certiorari is issued to quash the
rule. order of a Court or Tribunal. There are
3. The Damin-i-Koh was created by He was symbol of Hindu-Muslim unity five major types of writs Habeas
and fought until his death with the Corpus, Mandamus, Prohibition, Quo
the British government to settle Warranto and Certiorari. In India, both
which one of the following hand of British agent in 1858.
Supreme Court and High Court are
communities? 6. The Summary Settlement of 1856 empowered with writ jurisdiction.
(a) Santhal (b) Mundas was based on which one of the Certiorari It is issued by Supreme
(c) Oraons (d) Saoras following assumptions? Court and High Court to transfer a
particular matter or quash the order of
_ (a) Damin-i-Koh was the name given (a) The Talukdars were the rightful
lower court.
to the forested hilly areas of Rajmahal owners of the land
hills (Present day Jharkhand), (b) The Talukdars were interlopers Mandamus It is a command issued
inhabited by Santhal tribe. The with no permanent stakes in the by a court to a public authority, lower
Britishers intruded in their areas and court or tribunal to perform their
land
interfered in their socio-economic life, duties.
(c) The Talukdars could evict the
thus, restricting their movement in peasants from the lands Prohibition Supreme Court and High
forest. Court may prohibit lower courts who
(d) The Talukdars would take a
All atrocities on Santhal’s resulted exceeds their jurisdiction are act in
portion of the revenue which country to the rule of natural justice.
into large scale rebellion in 1855. flowed to the State
CDS Solved Paper 2019 (I) 69

Quo Warranto This unit is issued to (d) Article-15 and Article-16 _ (c) Paul Allen was co-founder of
enquire into legality of the claim of a Microsoft along with the Bill Gates.
person or public office. _ (a) The President can proclam
emergency under Article-352 when Microsoft is an American Multinational
Habeas Corpus is used by the courts security of nation or a part of it is Company with its headquarter is in
to find out if a person has been threatened by war as external Washington. The CEO of Microsoft as
illegally detained. aggression or armed rebellion. on 2019 is Satya Nadella.
9. Which among the following Article-20 and Article-21 which
14. The mobile app ‘eVIGIL’ is helpful
statements about the power to ensure protection of life and personal
liberty cannot be suspended at any in
change the basic structure of the (a) conducting free and fair
cost.
Constitution of India is/are correct? e-tendering process in
l
Article-22 Protection against arrest
1. It falls outside the scope of the and detention in certain cases. government offices.
amending powers of the l
Article-19 Protection of certain (b) fighting against corruption in
Parliament. sights regarding freedom of speech
etc.
public services.
2. It can be exercised by the l
Article-15 Prohibition of (c) removing garbage from the
people through representatives discrimination on grounds of municipal areas.
in a Constituent Assembly. religion, race, caste, sex or place of (d) reporting violation of model code
birth. of conduct in election-bound
3. It falls within the constituent l
Article 16 Equality of opportunity in
matters of public employment. states.
powers of the Parliament.
Select the correct answer using 11. Which one of the following _ (d) The Election Commission of India
had launched eVIGIL app for citizens
the codes given below Articles of the Constitution of to report any violation of the Model
(a) 1 and 3 (b) 1 and 2 India lays down that no citizen can Code of Conduct (MCC) during
(c) Only 1 (d) 2 and 3 elections. The android based app
be denied the use of wells, tanks
_ (c) and bathing ghats maintained out aimed at empowering people across
1. The concept of basic structure was the country to share evidence of
of state funds? malpractice by political parties, their
laid down by the Supreme Court in
the Keshavanand Bharti Case
(a) Article-14 candidates and activists directly to
(1973). The basic structure doctrine (b) Article-15 ECI.
states that the Constitution of India (c) Article-16
(d) Article-17
15. ‘Prahaar’ is
has certain basic features that
cannot be altered or destroyed
(a) a battle tank
through amendments by the
_ (b) Article-15 of the Constitution (b) a surface-to-surface missile
abolishes untouchability and forbids
Parliament. (c) an aircraft carrier
its practice in any form. So, no person
Article 368 does not give absolute would be denied to use the wells, (d) a submarine
powers to the Parliament to amend tanks and bathing ghats maintained
any part of the Constitution.
_ (b) Prahaar is a surface-to-surface
out of state funds. short range tactical ballistic missile.
Thus, statement (1) is correct. In 1955, government enacted Prahaar has been indigenously
2. The amendment in basic structure protection of Civil Right Act to abolish developed by Defence Research and
can also not be brought via untouchability from society. Development Organisation (DRDO).
Constituent Assembly since it was It is a quick-reaction, all-weather,
dissolved in 24th  January,  1950. 12. Who amongst the following
all-terrain, highly accurate battle field
Hence, statement (2 )is incorrect. organised the All India Scheduled support tactical missile with advance
3. Constituent power is the power to Castes Federation? manoeuvring capability. It is
formulate a Constitution or to (a) Jyotiba Phule capable of carrying multiple types of
propose amendments to or revisions (b) Periyar warheads weighing around 200 kg
of the Constitution and to ratify such (c) BR Ambedkar and neutralising different types of
proposal. The Constitution of India targets.
(d) MK Karunanidhi
vests constituent power upon the
Parliament subject to the special _ (c) All India Scheduled Castes
16. Who among the following is/are
procedure laid down therein. Federation was founded by BR the recipient/recipients of
However, scope to amend the basic Ambedkar in 1942 to organise Rajiv Gandhi Khel Ratna Award,
structure is limited. schedule caste people against 2018?
Hence, statement (3) is incorrect. Brahmanical ideology.
(a) Virat Kohli
He also established Bahishkrit
10. When a proclamation of (b) S Mirabai Chanu and Virat Kohli
Hitkarni Sabha, Independent Labour
emergency is in operation, the Party and started weekly paper (c) Neeraj Chopra
right to move a court for the ‘Mooknayak’. He was appointed as (d) Hima Das and Neeraj Chopra
enforcement of all Fundamental the first Law Minister of India.
_ (b) The Rajiv Gandhi Khel Ratna
Rights remains suspended, except 13. Paul Allen, who died in October, Award is the highest sporting honour
(a) Article-20 and Article-21 2018, was the co-founder of in our country. It was started in
(b) Article-21 and Article-22 1991-92. In 2018 Virat Kohli and
(a) Oracle (b) IBM
Mirabai Chanu were bestowed with
(c) Article-19 and Article-20 (c) Microsoft (d) SAP
70 CDS Solved Paper 2019 (I)

this award in the field of Cricket and 21. Which one of the following is not (a) Only 1 (b) 1 and 2
Weightlifting. (c) 2 and 3 (d) Only 3
an assumption in the law of
In 2019, Deepa Malik and Bajrang demand? _ (b) An indifference curve are convex of
Punia were awarded Rajiv Gandhi combinations of goods which derive
Khel Ratna in the field of (a) There are no changes in the taste
the same level of satisfaction so that
Paralympic and Freestyle Wrestling and preferences of consumers
the consumer is indifferent to any of
respectively. (b) Income of consumers remains the combination he consumes.
constant
17. Pakyong Airport is located in Hence statement (1) is correct.
(c) Consumers are affected by
(a) Sikkim Higher indifference curve means
demonstration effect
(b) jammu and Kashmir higher level of satisfaction as higher
(d) There are no changes in the price indifference curve consists of more of
(c) Arunachal Pradesh
of substitute goods two goods or the same quantity of
(d) Mizoram
one good and more quantity of the
_ (a) Recently Prime Minister _ (c) Consumers are affected by other good. Hence statement (2) is
demonstration effect, is not an
inaugurated Pakyong airport in the correct. Two indifference curves
assumption in the law of demand.
State of Sikkim. It is Sikkim’s first cannot cut each other.
airport constructed by the Airport The law of demand is a fundamental
Thus, statement 3 is incorrect.
Authority of India. concept of economics which states
that at a higher price consumers will 24. Consider the following statements
18. The United Nations has been demand a lower quantity of goods about a joint-stock company.
observing International Day of and vice-versa. 1. It has a legal existence.
Rural Women on 22. Which one of the following 2. There is limited liability of
(a) 15th July (b) 15th August statements is not correct? shareholders.
(c) 15th September (d) 15th October 3. It has a democratic
(a) When total utility is maximum,
_ (d) The United Nations observes marginal utility is zero. management.
International Day of Rural Women on (b) When total utility is decreasing, 4. It has a collective ownership.
15th October. Which of the statements given
marginal utility is negative.
International Day of Rural Women is (c) When total utility is increasing above are correct?
observed to celebrate the crucial role
marginal utility is positive. (a) 1 and 2 (b) 1, 2 and 3
that women and girls play in ensuring
the sustainability of rural households (d) When total utility is maximum, (c) 3 and 4 (d) 1, 2, 3 and 4
marginal and average utility are
and communities, thus, improving _ (d) A joint-stock company is a
rural livelihoods and overall equal to each other. business entity in which shares of the
well-being. The First International Day company's stock can be bought and
of Rural Women was observed on _ (d) Total utility means total benefit sold by share holders.
obtained by a person from
15th October, 2008. Each shareholder owns company
consumption of goods and services,
19. Who among the following is the whereas, marginal utility means the stock in proportion, evidenced by
amount of utility, a person gains from their shares.
first Indian to win Pulitzer Prize?
the consumption of each successive All the above are features of a
(a) Arundhati Roy
unit of a commodity. joint-stock company
(b) Gobind Bihari Lal
Relation between total utility and (i) It has a legal existence .
(c) Vijay Seshadri
marginal utility (ii) There is limited liability of shareholders
(d) Jhumpa Lahiri
(i) When total utility is maximum, for satisfaction of the debt of the
_ (b) The Pulitzer Prize is awarded for marginal utility is zero company the personal property of the
the achievement in the field of (ii) When total utility increase, marginal shareholders cannot be used.
journalism, literature and musical utility is positive (iii) It has a democratic management.
composition, presented by Columbia
(iii) When total utility decrease, marginal (iv) It has a collective ownership.
University.
utility is negative
Govind Bihari Lal was first Indian to Hence, all the statements are
Hence, option (d) is not correct. correct.
win Pulitzer prize in 1937. Vijay
Seshadri and Jhumpa Lahiri won the 23. Consider the following statements 25. When some goods or productive
Pulitzer Prize in the year of 2014 and about indifference curves. factors are completely fixed in
2006 respectively. amount, regardless of price, the
1. Indifference curves are convex
20. Saurabh Chaudhary excels in to the origin. supply curve is
which one of the following sports? (a) horizontal
2. Higher indifference curve repre (b) downward sloping to the right
(a) Archery (b) Shooting sents higher level of
(c) Boxing (d) Judo satisfaction. (c) vertical
(d) upward sloping to the right
_ (b) Saurabh Chaudhary is an Indian 3. Two indifference curves cut
shooter. He won the gold medal at each other. _ (d) When some goods or productive
2018 Asian Games in 10m Air pistol. Which of the statements given factors are completely fixed in
He created history by becoming amount regardless of price, the
above is/are correct? supply curve is upward sloping to the
youngest Indian to win gold medal at
the Asian Games. right.
CDS Solved Paper 2019 (I) 71

Supply curve is a graphic 29. Which European traveller had 32. ‘Sub-prime crisis’ is a term
representation of the correlation observed, ‘‘A Hindu woman can associated with which one of the
between the cost of a good or
go anywhere alone, even in the following events?
service and the quantity supplied for
most crowded places and she need (a) Economic recession
a given period.
never fear the impertinent looks (b) Political instability
The upward sloping means that the
and jokes of idle loungers’’? (c) Structural adjustment programmes
firms will be willing to increase the
production in response to a higher (a) Francois Bernier (d) Growing social inequality
market price because the higher price (b) Jean-Baptiste Tavernier _ (a) Sub-prime crisis is Economic
may make additional production (c) Thomas Roe recession that took place in the USA
profitable. (d) Abbe JA Dubois from 2007 to 2010.
26. Who designed the Bombay Economic recession is a slow down
_ (d) Abbe Jean Antoine Dubois was a or massive contraction in economic
Secretariat in the 1870s? French Catholic missionary in India.
He was an Indologist and authored the activities. The primary cause was the
(a) Henry St Clair Wilkins collapse of the housing bubble. As a
book Hindu manners, custom and
(b) Sir Cowasjee Jehangir result of this crisis, job growth, saving
ceremonies. In his book, he wrote
Readymoney about position of women in the society. and investment reduced to
(c) Purushottamdas Thakurdas abysmal.
He had observed that a Hindu woman
(d) Nusserwanji Tata can go anywhere alone, even in the 33. Which one of the following is not
most crowded places and she need
_ (a) The Bombay Secretariat was a change brought about by the
completed in 1874 and designed by never fear the impertinent looks and Indian Independence Act of 1947?
Captain Henry St Clair Wilkins in the jokes of idle loungers. He was called
(a) The Government of India Act,
Venetian Gothic style. as Dadda Swami by the local people.
1935 was amended to provide an
With its arcaded verandahs 30. Who was the author of the book interim Constitution.
and huge gable over the ‘Plagues and Peoples’?
West facade, it was a monument to (b) India ceased to be a dependency.
the civic pride of Bombay’s British (a) William H McNeil (c) The Crown was the source of
rulers. (b) WI Thomas
authority till new Constitution was
(c) Rachel Carson
27. Who was the founder of Mahakali framed.
(d) David Cannadine
Pathshala in Calcutta? (d) The Governor-General was the
_ (a) ‘Plagues and Peoples’ is a book on Constitutional Head of Indian
(a) Her Holiness Mataji Maharani epidemiological history by William
Tapaswini Hardy McNeil published in New York Dominion.
(b) Sister Nivedita City in 1976. It was a critical and _ (b) India ceased to be a dependency is
(c) Madame Blavatsky popular success, offering a radically not a charge brought about by the
(d) Sarojini Naidu new interpretation of the extra ordinary Indian Independence Act of 1947.
impact of infectious disease on culture
On 20th February, 1947 British PM
_ (a) Mataji Maharani Tapaswini as a means of enemy attack.
declared that British rule will end by
founded Mahakali Pathshala in The book ranges from examining the
Calcutta in 1893 to foster women 30th June 1948, but on 3rd June,
effects of small pox in Mexico, the 1947 Mountabatten Plan was
education. bubonic plague in China, to the accepted by Congress which led to
28. Which European ruler had typhoid epidemic in Europe. enaction of Indian Independence Act.
observed, ‘‘Bear in mind that the 31. Which Indian social theorist had As per Act India was declared as an
commerce of India is the argued that the idea of a independent and Sovereign State
commerce of the world … he who homogenised Hinduism was from 15th August, 1947.
can exclusively command it is the constructed through the ‘cultural Hence, option (b) is correct.
dictator of Europe’’? arrogance of post-enlightenment
Europe?’ 34. Which one of the following is not
(a) Queen Victoria a correct statement regarding the
(a) Ashis Nandy
(b) Peter the Great of Russia provision of Legislative Council in
(b) Partha Chatterjee
(c) Napoleon Bonaparte the State Legislature?
(c) TK Oommen
(d) Gustav II Adolf
(d) Rajni Kothari (a) The States of Bihar and Telangana
_ (b) Peter the great was a Russian have Legislative Councils
Czar (emperor) in the late _ (a) Ashis Nandy is an Indian political
psychologist, social theorist and critic. (b) The total number of members in
17th century. He was best known
He has given theoretical criticism of the Legislative Council of a State
for his extensive reforms in an
European colonialism, development, shall not exceed one-third of the
attempt to establish Russia as a great total number of members in the
nation. modernity, and secularism among
many others. Legislative Assembly
As India was one of the main centres (c) One-twelfth of all members shall
of the world trade and industry, Peter According to him, the idea
be elected by electorates
observed, “Bear in mind that the homogenised Hindusim emnates
consisting of local bodies and
commerce of India is the commerce from the ‘cultural arrogance of
authorities
of the world.’’ post-enlightenment Europe’.
72 CDS Solved Paper 2019 (I)

(d) One-twelfth of all members shall (a) The Parliament may be law exercise some control over market
be elected by graduates residing Constitute Administrative prices.
in the state Tribunals both at the Union and An oligopoly is a market form
_ (c) The Constitution as provided State levels wherein a market or industry is
autonomy to state to have unicameral (b) Tribunals may look into disputes dominated by a small number of large
and bicameral legislature, 7 out of 28 sellers.
and complaints with respect to
states have bicameral legislature,
Legislative Assembly and Council. recruitment and conditions of 38. Consider the following statements.
service of persons appointed to 1. Inflation in India continued to
The Legislative Council is the Upper
House where members are indirectly public services be moderate during 2017-18.
elected. (c) Tribunals established by a law of 2. There was significant
The maximum strength of the council the Parliament can exclude the reduction in food inflation,
is fixed at one-third of the total jurisdiction of all courts to allow for particularly pulses and
strength of assembly and minimum is special leave to appeal vegetables during the period.
fixed at 40. (d) The law establishing the Tribunals Which of the statements given
Manner of Election may provide for procedures above is/are correct?
One-third By Local Bodies and
including rules of evidence to be (a) Only 1 (b) Only 2
Authorities followed (c) Both 1 and 2 (d) Neither 1 nor 2
One-third By Legislative Assemblies _ (c) The 42nd Constitutional _ (c) As per economic survey. Inflation
Amendment Act of 1976 added a new in india continued to be moderate
One-twelfth By Graduates
Part XIV-A entitled as ‘Tribunals’. It 3.3% during 2017-18.
One twelfth By Teachers consists of two Article-323A and 323B.
It was mainly due to the significant
One-sixth Nominated by Governor Article-323A empowers the reduction in food inflation, particularly
Parliament to establish Administrative pulses and vegetables during the
Hence, option (c) is incorrect.
Tribunal, in 1985 the Administrative period.
35. Which one of the following is not Tribunal Act was passed to establish
Hence, both the statements are true.
correct about the Panchayats as Central Administrative Tribunal.
laid down in Part IX of the All the statements are true except (c), 39. Which one of the following
as they are not empowered to hypothesis postulates that
Constitution of India? exclude the jurisdiction of all court to
(a) The Chairperson of a Panchayat allow for special leave to appeal.
individual’s consumption in any
needs to be directly elected by time period depends upon
Article-323B gives power to both
people in order to exercise the right Parliament and State Legislature to resources available to the
to vote in the Panchayat meetings establish tribunal for different individual, rate of return on his
(b) The State Legislature has the right disputes. capital and age of the individual?
to decide whether or not offices of 37. A market situation when many (a) Absolute Income Hypothesis
the Chairpersons in the firms sell similar, but not identical (b) Relative Income Hypothesis
Panchayats are reserved for SCs, products is termed as (c) Life-Cycle Hypothesis
STs or women (d) Permanent Income Hypothesis
(a) perfect competition
(c) Unless dissolved earlier, every (b) imperfect competition _ (c) The life-cycle hypothesis
Panchayat continues of a period (c) monopolistic competition postulates that individual
of five years consumption in anytime period
(d) oligopoly
(d) The State Legislature may by law depends on.
make provisions for audit of _ (c) Monopolistic competition is (i) The resources available to the
situation where many firms are individual.
accounts of the Panchayats competing against each other but
(ii) The rate of return on his capital.
selling products that are similar, this
_ (a) The Part IX was inserted in (iii) The age of the individual.
Constitution through the 73rd type of situation leads to imperfect
Amendment Act. competition in the market. It was developed by Ando and
This market situation is against the Modigliani.
Important Features
interest of consumers. 40. According to John Maynard
The Chairman of a Panchayat shall be
elected in such a manner as the State Perfect competition is the situation Keynes, employment depends upon
Legislature determines. So statement prevailing in a market in which buyers
(a) aggregate demand
(a) is not correct. and sellers are so numerous and well
informed that all elements of (b) aggregate supply
Rest all statements are true. It also
monopoly are absent and the market (c) effective demand
provided one-third reservation of
seats for women in State Finance price of a commodity is beyond the (d) rate of interest
Commission, State Election control of individual buyers and
sellers. _ (c) John Maynard Keynes was the
Commission etc. founder of modern macro economics.
Imperfect competition is the He was a British economist.
36. Which one of the following is not situation prevailing in a market in
correct about Administrative which elements of monopoly allow As per his book the general theory of
Tribunals? individual producers or consumers to employment, interest and money,
CDS Solved Paper 2019 (I) 73

employment depends on effective (d) The Federal Government had Select the correct answer using
demand. With the increase in the sole authority to decide on the codes given below
demand, manufacturing and other slavery (a) 1 and 2 (b) 1, 2 and 3
services require more person to (c) 3 and 4 (d) 1, 2 and 4
manufacture goods and provide _ (a) The Kansas-Nebraska Act was
services thus leading to job growth. passed by the US Congress in 1854. It (a) Only statement 1 and 2 are
allowed people in the territories of correct. Alladi Krishnaswami was
Hence, option(c) is true.
Kansas and Nebraska to decide for eminent lawyer from Chennai, in
41. Which one of the following canons themselves whether or not allow Constituents Assembly he favoured
of taxation was not advocated by slavery with in their borders. the role of Supreme Court to interpret
This act repealed the Missouri the Constitution when required. He
Adam Smith? also wanted that the Supreme Court
compromise of 1820 which
(a) Canon of equality had to draw the line between liberty
prohibited slavery.
(b) Canon of certainty and social control.
(c) Canon of convenience 44. Which one of the following issues
46. Which of the following are the
(d) Canon of fiscal adequacy was included in the Indo-US
core functions of the United
Nuclear Agreement of 2007?
_ (d) Adam Smith was an economist, Nations multidimensional
best known for his book ‘Wealth of (a) India has ‘advance right to
peacekeeping operations?
Nation’. reprocess’ US-origin safeguarded 1. Stabilisation
He presented four canon of taxation, spent fuel.
2. Peace consolidation
later many other canon were (b) India did not have the right to build
developed. Smith’s canon of taxation 3. To extend support to a losing
a strategic fuel reserve with the
include help of the other supplier
state in a war
(i) Canon of equality countries. Select the correct answer using the
(ii) Canon of certainty
(c) India should not test a nuclear codes given below
(iii) Canon of convenience (a) 1, 2 and 3 (b) 2 and 3
device.
(iv) Canon of economy (c) 1 and 3 (d) 1 and 2
(d) The US will impede the growth of
So, option (d) is correct answer.
India’s nuclear weapons _ (d) The United Nation peacekeeping
42. Which Arab scientist could be programme. was established in 1948, to help
given the credit of christening the countries to cross the difficult path
(c) The Indo-US nuclear agreement from conflict to peace. Their main work
mathematical discipline of also known as 123 agreement was peace building, peace making and
algorithm? watershed moment in Indo-US peace enforcement.
(a) Al-Khwarizmi (b) Ibn al-Haytham relationship. The deal would have
They do not take part in war with the
indirectly bring India under purview of
(c) Ibn Rushd (d) Ibn Sina objective winning or helping loosing
NSG and US laws that would not
state.
(a) The credit of christening the allow India to conduct nuclear test in
Hence, option (d) is correct.
mathematical discipline of algorithm the future. The issue of nuclear test
goes to Arab scientist Muhammad was later clarified as the moratorium 47. The South China Sea dispute
Ibn Musa Al-Khwarizmi. He was a on nuclear test was unilateral and
involves which of the following
Persian scholar who produced works voluntary and there was no pressure
in mathematic, astronomy and on India from outside. Thus, (c) is the countries?
geography. In 820 AD, he was correct answer.
1. China 2. Vietnam
appointed as the astronomer and 3. Malaysin 4. Indonesia
head of the library of House of 45. Which of the following statements
Select the correct answer using
Wisdom in Baghdad. about Alladi Krishnaswami Ayyar,
the codes given below
as a drafting member of the
43. Which one of the following (a) 1 and 4 (b) 1 and 2
Constitution of India, are correct? (c) 1, 2 and 3 (d) 2, 3 and 4
developments took place because
1. He favoured the role of the
of the Kansas-Nebraska Act of _ (c) The South China disputes involve
Supreme Court in taking
1854? both island and maritime claims
important decisions related to among several sovereign states within
(a) The Missouri Compromise was
the interpretation of the the region, namely, Brunei, the
repealed and people of Kansas
Constitution of India. People’s Republic of China (PRC),
and Nebraska were allowed to Republic of China (Taiwan), Malaysia,
determine whether they should 2. He felt that the Supreme Court the Phillippines, and Vietnam.
own slaves or not had to draw the line between
Hence, option (c) is correct.
(b) The Act did not permit the liberty and social control.
territories the right to vote over the 3. He believed in the dominance 48. The ‘Kyoto Protocol’ is an
question of slavery of the executive over the international treaty that commits
(c) The voice of the majority in regard judiciary. State parties to reduction in
to the issue of slavery was (a) poverty
muzzled
4. He favoured a dictatorial form (b) greenhouse gases emission
of governance. (c) nuclear armaments
(d) agricultural subsidy
74 CDS Solved Paper 2019 (I)

_ (b) The ‘Kyoto Protocol’ is an _ (b) A. CH3F — Its molecular shape is 56. The monomer/monomers used for
international treaty which extends the tetrahedral the synthesis of nylon 6 is/are
1992 United Nations Framework B. HCHO — Its molecular shape is
Convention on Climate Change (a) hexamethylenediamine and
trigonal planar
(UNFCCC) that commits State parties adipic acid
C. HCN — Its molecular shape is
to reduce greenhouse gases (b) caprolactam
linear
emission which are responsible for (c) urea and formaldehyde
D. NH 3 — Its molecular shape is
Global Warming and Climate (d) phenol and formaldehyde
trigonal pyramidal
Change. The Kyoto Protocol was
adopted in Kyoto, Japan on 52. Very small insoluble particles in a _ (b) Nylon 6 is only made from one kind
December 11, 1997 and entered into of monomer, a monomer is called
liquid may be separated from it by caprolactam.
force on February 16, 2005. There are
currently 192 parties singed the using O
protocol. (a) crystallisation
N–H
(b) fractional distillation
49. The ‘Beijing Declaration’ is
(c) centrifugation
concerned with which one of the (d) decantation Caprolactam
following issues?
(a) Rights of Children _ (d) There are some mixtures which 57. Which one among the following
contain insoluble solid particles stars is nearest to the Earth?
(b) Rights of Women suspended in a liquid. The solid
(c) Right to Development particles which are insoluble in a liquid (a) Sirius
(d) Reduction of Tariffs can be separated by using (b) Arcturus
decantation method. (c) Spica
_ (b) The ‘Beijing Declaration’ is
concerned with the ‘Right of Women’, (d) Proxima Centauri
53. Which one of the following
this declaration was outcome of fourth _ (d) The nearest star to the Earth is Sun
world conference on women in 1995. elements cannot be detected by
followed by the Proxima Centauri.
It advances the goals of equality,
‘Lassaigne’s test’?
Proxima Centauri is part of the star
development and peace for all (a) I (b) Cl (c) S (d) F
system known as ‘Alpha Centauri’.
women.
_ (d) Fluorine cannot be detected by
Lassaigne’s test because silver 58. Which of the following planets of
50. The ‘Gujral Doctrine’ relates to
fluoride is soluble in water and does our solar system has least mass?
which one of the following issues? not precipitate thus this method (a) Neptune (b) Jupiter
(a) Build trust between India and its cannot be used for detection of (c) Mars (d) Mercury
neighbours fluorine.
(b) Initiate dialogue with all insurgent _ (d) Mercury is the smallest and closest
54. In which of the following, planet to Sun. It has the least mass in
groups in India
functional group isomerism is not the solar system. The planet with the
(c) Undertake development activities highest mass is Jupiter.
possible?
in Naxal-dominated areas
(a) Alcohols (b) Aldehydes 59. Two identical solid pieces, one of
(d) Ensure food security
(c) Alkyl halides (d) Cyanides gold and other of silver, when
_ (a) The main objective of ‘Gujaral immersed completely in water
doctrine’ to build trust between India _ (c) In alkyl halides, functional group
and its neighbours, was initiated in isomerism is not possible. Rest of all exhibit equal weights. When
1996 by IK Gujaral, the then Finance options show functional group weighed in air (given that, density
Minister. isomerism.
of gold is greater than that of silver),
51. Match List-I with List-II and select 55. Which one of the following (a) the gold piece will weigh more
the correct answer using the code statements is not correct? (b) the silver piece will weigh more
(a) Fischer projection represents the (c) Both silver and gold pieces weigh
given below the Lists
molecule in an eclipsed conformation. equal
List-I List-II (b) Newman projection can be (d) weighing will depend on their
(Compound/Mo (Shape of represented in eclipsed, staggered masses
lecule) Molecule)
and skew conformations.
A. CH 3 F 1. Trigonal planar (c) Fischer projection of the molecule _ (c) Since, gold is more denser than
silver. But it is given that both pieces
B. HCHO 2. Tetrahedral is its most stable conformation. have identical and equal weight in
C. HCN 3. Trigonal pyramidal (d) In Sawhorse projections the lines water, it means both have equal
D. NH 3 4. Linear are inclined at an angle of 120° to volume and mass. (May be gold piece
each other. hollow inside.)
Code (As we know, force of gravity is
_ (c) Statement is given in option (c) is determined by mass and buoyant
A B C D correct. Correct statements is Fischer
(a) 2 4 1 3 projection always depicts, the force is determined by volume)
(b) 2 1 4 3 molecule in eclipsed form, hence it is So, both silver and gold pieces weight
(c) 3 4 1 2 most stable conformation. equal in air.
(d) 3 1 4 2
CDS Solved Paper 2019 (I) 75

60. If the wavelengths corresponding 63. Which one of the following from dead and decaying organic
matter.
to ultraviolet, visible and infrared functions is not carried out by
radiations are given as l ultraviolet , smooth endoplasmic reticulum? Rest organisms, i.e. Ulothrix, Riccia
and Cladophora have autotrophic
l visible and l infrared respectively, (a) Transport of materials mode of nutrition.
then which one of the following (b) Synthesis of lipid
gives the correct relationship (c) Synthesis of protein 67. Which one of the following has a
(d) Synthesis of steroid hormone bilateral symmetry in its body
among these wavelengths?
organisation?
(a) lultraviolet < linfrared < l visible _ (c) Endoplasmic reticulum has two (a) Asterias (b) Sea anemone
(b) lultraviolet > l visible > linfrared types–Smooth Endoplasmic
(c) Nereis (d) Echinus
(c) lultraviolet > linfrared > l visible Reticulum (SER) and Rough
Endoplasmic Reticulum (RER). SER _ (c) Nereis is a member of
(d) lultraviolet < l visible < linfrared have smooth surface due to lack of phylum–Annelida, which has bilateral
_ (d) As we know that, infrared light has ribosomes. Thus, they do not help in symmetry in its body organisation.
a lower frequency than visible light and protein synthesis unlike RER which
contain ribosomes (the site of protein In bilateral symmetry, a lengthwise
visible light has a lower frequency than vertical plane divided the animals into
ultraviolet light. synthesis).
two equal and opposite halves. Sea
i.e. nultraviolet > n visible > ninfrared SER help in synthesis of steroid anemone of phylum–Cnidaria and
hormones and lipids. They also form Asterias and Echinus of
Since, wavelength of a light is vesicles for the transportation of
inversely proportional to its phylum–Echinodermata have radial
materials. symmetry.
frequency, therefore
l ultraviolet < l visible < l infrared 64. Which one of the following cell 68. Which one of the following pairs
organelles mainly functions as of animals is warm- blooded?
61. An electron and a proton starting storehouse of digestive enzymes?
from rest get accelerated through (a) Crocodile and ostrich
(a) Desmosome (b) Ribosome (b) Hagfish and dogfish
potential difference of 100 kV. The
(c) Lysosome (d) Vacuoles (c) Tortoise and ostrich
final speeds of the electron and the
proton are v e and v p , respectively. _ (c) Lysosomes are called storehouse of (d) Peacock and camel
Which one of the following relation digestive enzymes. These are single
membrane bound cell organelles which _ (d) Peacock and camel are
is correct? warm-blooded animals. These
contain various digestive or hydrolytic
(a) ve > v p animals regulate their body
enzymes. temperature in any kind of
(b) ve < v p
Due to the presence of hydrolytic environment. On the other hand,
(c) ve = v p
enzymes these are also called as crocodile, hagfish, dogfish, tortoise
(d) Cannot be determined suicidal bags of cell. These enzymes are cold-blooded animals.
_ (a) As we know that, take part in hydrolysis of extracellular
69. Which one of the following States
1 materials.
KE = mv 2 = qV of India is not covered by flood
2 65. Which one of the following tissues forecasting stations set-up by the
For proton and electron, the relation is responsible for increase of girth in Central Water Commission?
of charge and mass are q p = - q e the stem of a plant? (a) Rajasthan
and mp = me × 1840 (a) Tracheid (b) Jammu and Kashmir
2qV 1 (b) Pericycle (c) Tripura
vµ Þv µ
m m (c) Intercalary meristem (d) Himachal Pradesh
Here, we can see that mass of proton (d) Lateral meristem
_ (d) Central Water Commission is the
is very high than mass of electron, so
ve > v p .
_ (d) The lateral meristem tissues are nodal organisation for flood
responsible for increase of girth in the forecasting in the country. CWC
stem of a plant. This tissue is present comes under Ministry of Water
62. If two vectors A and B are at an
along the side of organs, e.g. vascular Resource, River Development and
angle q ¹ 0°, then cambium and cork cambium, which Ganga Rejuvenation.
(a) | A | + | B| = | A + B| are responsible for the secondary It is the modal agency for flood
(b) | A | + | B| > | A + B| growth of plants. Due to this forecasting in India.
(c) | A | + | B| < | A + B| secondary growth the girth of stems
Except Himachal Pradesh all other
and roots increases.
(d) | A | + | B| = | A - B| mentioned states are covered by
66. Which one of the following organisms flood forecasting stations setup by
_ (b) Since, any one side of a triangle is the CWC.
less than the sum of the other two is dependent on saprophytic mode of
sides individually, so for any two nutrition? Hence, option (d) is not correct.
vectors A (a) Agaricus (b) Ulothrix
and B, 70. The city of Cartagena, which is
(c) Riccia (d) Cladophora famous for Protocol on Biosafety,
| A + B| £ | A| + |B|
_ (a) Agaricus is an organism which is is located in
If q ¹ 0, then dependant on saprophytic mode of (a) Colombia
| A|+|B| > | A + B|. nutrition. They obtain their nutrition
76 CDS Solved Paper 2019 (I)

(b) Venezuela The Sargasso sea is a large, warm statements explain the
(c) Brazil (18º C), saline region which is phenomenon?
(d) Guyana characterised by an abundance of
floating brown seaweed
1. Tank irrigation predates well
_ (a) Cartagena Protocol on Biosafety is (Sargassum). irrigation.
an international treaty that seeks to 2. Tank irrigation is in the areas
Hence, option (a) is not correct.
protect biodiversity from the potential
risks posed by Genetically Modified
with impervious surface layers.
74. Match List I with List II and select
Organisms (GMO). the correct answer using the codes 3. Well irrigation requires
It was singed on 15th May, 2000 in given below the lists. sufficient groundwater
Cartagena which is in Colombia. reserves.
List I (City) List II (Product)
71. Which one among the following is 4. Other forms of irrigation are
A. Detroit (USA) 1. Motarcar
the most populated state in India as not available.
B. Antwerp 2. Diamond Cutting
per Census 2011? (Belgium) Select the correct answer using
(a) Goa (b) Mizoram the codes given below
C. Tokyo (Japan) 3. Steel
(c) Meghalaya (d) Sikkim (a) 1, 2 and 3 (b) 2 and 3
D. Harbin (China) 4. Ship Building (c) 3 and 4 (d) 1 and 4
_ (c) The Census of 2011 was 15th
Census held in two phases. Codes _ (b) The tank irrigation is more in the
A B C D A B C D rocky plateau area of the country,
Among the above state Meghalaya is where the rainfall is not even and
(a) 3 4 2 1 (b) 3 2 4 1
most populated and Sikkim is least highly seasonal and the rocks are
(c) 1 4 2 3 (d) 1 2 4 3
populated. impervious.
The state of Uttar Pradesh (First rank), _ (d) The Eastern Madhya Pradesh,
Maharashtra (Second rank) and Bihar City Product Chhattisgarh, Odisha, Tamil Nadu
(Third rank) are among top three and some parts of Andhra Pradesh
populated states according to the Detroit (USA) Motarcar
have areas under tank irrigation.
Census 2011. Antwerp (Belgium) Diamond Cutting On the other hand, well irrigation is
72. Which among the following Tokyo (Japan) Ship Building most common in alluvial plains of the
Harbin (China) Steel country except the desert of
countries of South America does
Rajasthan.
the Tropic of Capricorn not pass
through? 75. Which one of the following is not Plains of UP, Bihar, Gujarat,
situated on Varanasi- Kanyakumari Karnataka and Tamil Nadu are the
(a) Chile (b) Bolivia states which are mostly under well
National Highway?
(c) Paraguay (d) Brazil irrigation. These areas have large
(a) Satna (b) Rewa ground water reserves.
_ (b) The Tropic of Capricorn is an (c) Katni (d) Jabalpur
imaginary line of latitude at 23.5° South Thus, only statements 2 and 3
of the equator. This latitude runs _ (a) The Varanasi-Kanyakumari correctly explains this phenomenon.
through following countries: Namibia, National highway is Part of NH-44.
Botswana, South Africa, Mozambique Major cities such as Rewa, Katni, 78. When hot water is placed into an
Madagascar , Australia, Chile, Jabalpur, Hyderabad, Bengaluru etc. empty water bottle, the bottle
Argentina, Paraguay and Brazil. It are located on this highway. keeps its shape and does not
does not pass through Bolivia. Satna does not lie on this Highway. soften. What type of plastic is the
Hence, option (b) is not correct. 76. Which one of the following water bottle made from ?
73. Which one of the following is not methods is not suitable for urban (a) Thermoplastic (b) PVC
correct about Sargasso Sea? rainwater harvesting? (c) Polyurethane (d) Thermosetting
(a) Rooftop recharge pit
(a) It is characterised with _ (d) Water bottle made from
(b) Recharge wells thermosetting plastics. Thermosetting
anti-cyclonic circulation of ocean
(c) Gully plug plastic is a polymer that irreversibly
currents.
(d) Recharge trench becomes rigid when heated. Also
(b) It records the highest salinity in known as thermoset, thermosetting
Atlantic ocean. _ (c) Except Gully plug, all the others polymer.
such as Rooftop recharge pit,
(c) It is located West of Gulf stream Recharge wells and Recharge trench
and East of Canary current. 79. Which of the following is/are state
are viable in urban rainwater
(d) It confined in gyre of calm and harvesting.
function/functions?
motionless water. 1. q + W 2. q
Gully plugs or Checkdams are mainly
built to prevent erosion and to settle 3. W 4. H - TS
_ (a) Sargasso sea is the calm centre of sediments and pollutant.
the anti-cylonic gyre in the North Select the correct answer using
Atlantic, comprising a large eddy of 77. If one plots the tank irrigation in the codes given below.
surface water, the boundaries of which (a) 1 and 4 (b) 1, 2 and 4
India and superimposes it with
are demarcated by major current
map of well irrigation, one may (c) 2, 3 and 4 (d) Only 1
systems such as the Gulf Stream,
find that the two are negatively
Canaries current, and North Atlantic
related. Which of the following _ (a) Internal energy (DE) = q + W .
Drift.
CDS Solved Paper 2019 (I) 77

It is a state function because it is (c) The potential energy of the gas 87. Which one of the following
independent of the path. molecules is zero. statements regarding haemoglobin
Gibbs energy (G) = H - TS (d) There is no interactive force is correct?
It is also a state function because it is between the molecules. (a) Haemoglobin present in RBCs
independent of the path. can carry only oxygen, but not
_ (a) Statement (a) is incorrect, because carbon dioxide
Heat (q) and work (W ) are not state an ideal gas consist of a very large
functions being path dependent. number of molecules which are in a (b) Haemoglobin of RBC can carry
state of continuous, rapid and random both oxygen and carbon dioxide
80. For a certain reaction, motion. They move in all direction with (c) Haemoglobin of RBCs can carry
DG - = - 45kJ/mol and DH - = - 90 different speeds, ranging from zero to only carbon dioxide
infinity, this is a postulate of an ideal (d) Haemoglobin is only used for blood
kJ/mol at 0°C. gas. clotting and not for carrying gases
What is the minimum temperature
at which the reaction will become
84. What is a constellation? _ (b) Haemoglobin of RBCs can carry
(a) A particular pattern of equidistant both oxygen and carbon dioxide and it
spontaneous, assuming that DH - stars from the Earth in the sky. not used for clotting of blood. Hence,
and DS - are independent of (b) A particular pattern of stars that
option (b) is the correct answer.
temperature? may not be equidistant from the 88. Which one of the following is the
(a) 273 K (b) 298 K Earth in the sky. correct sequence of passage of light
(c) 546 K (d) 596 K (c) A particular pattern of planets of in a compound microscope?
our solar system in the sky. (a) Condenser-Objective lens-Eye
_ (a) Q DG = DH - TDS and DH and DS piece-Body tube
are independent of temperature, (d) A particular pattern of stars,
(b) Objective lens-Condenser-Body
Also, given, DG - q = ( - ) ve at 273 K, planets and satellites in the sky
tube-Eye piece
Thus, reaction is spontaneous at due to their position in the space. (c) Condenser-Objective lens-Body
273 K. _ (b) A constellation is a group of stars tube-Eye piece
Hence, (a) is the correct option. usually in a recognisable shape or (d) Eye piece-Objective lens-Body
pattern. tube-Mirror
81. The PCl 5 molecule has trigonal In 1922, the International _ (c) The correct sequence of passage
bipyramidal structure. Therefore, the Astronomical Union (IAU) formally of light in a compound microscope is
hybridisation of p-orbitals should be accepted the modern list of 88 condenser-Objective lens-Body
(a) sp 2 (b) sp 3 (c) dsp 2 (d) dsp 3 constellations. tube-Eye piece.

85. The Hooke’s law is valid for Compound microscope is a common


_ (d) Phosphorus has five electrons in light microscope usually used in labs
valence orbitals, which can hybridised (a) only proportional region of the
and give five hybrid orbitals, viz. Three of schools and colleges. In this type
p-orbitals one each of s-andd-orbitals.
stress-strain curve of microscope, light reflected by
(b) entire stress-strain curve mirror goes to condenser which
Thus, hybridisation of P in PCl 5 is
(c) entire elastic region of the condense light on object. From here,
sp3d . light goes to objective lens, body tube
stress-strain curve
(d) elastic as well as plastic region of and eye piece, respectively.
82. In spherical polar coordinates
( g , q, a ), q denotes the polar angle the stress-strain curve 89. Which one of the following
around z-axis and a denotes the _ (a) Hooke’s law is valid for only statements is correct?
azimuthal angle raised from proportional region of the stress- strain (a) Urea is produced in liver
curve. (b) Urea is produced in blood
x-axis. Then, the y-component of
P is given by Q Hooke’s law ® stress µ strain (c) Urea is produced from digestion
of starch
(a) P sin q sin a (b) P sin q cos a 86. Which one of the following
(c) P cos q sin a (d) P cos q cos a (d) Urea is produced in lung and
statements regarding histone
r kidney
_ (a) Here, ther y-component of P is proteins is correct?
given by, y = ( P sin q) sin a (a) Histones are proteins that are
_ (a) The statement given in option (a)
is correct, i.e. urea is produced in
z → (γ, θ, α) present in mitochondrial liver. As a result of metabolism of
P
θ membrane nitrogenous compounds (amino
y
(b) Histones are proteins that are acids, proteins, nucleic acids),
α
present in nucleus in association ammonia is produced.
x
with DNA It combines with CO 2 through
83. For an ideal gas, which one of the (c) Histones are proteins associated ornithine cycle and forms urea.
following statements does not hold with lipids in the cytosol
(d) Histones are proteins associated
90. Which one of the following river
true? valleys of India is under the
(a) The speed of all gas molecules is with carbohydrates in the cytosol
influence of intensive gully erosion?
same. _ (b) Statement in option (b) regarding (a) Kosi (b) Chambal
(b) The kinetic energies of all gas histone protein is correct. These are (c) Damodar (d) Brahmaputra
molecules are not same. protein that are present in nucleus in
association with DNA.
78 CDS Solved Paper 2019 (I)

_ (b) Chambal river valley is under the (a) Form of suspended water droplets order as per their population size
influence of intensive gully erosion. caused by condensation. according to Census 2011.
Chambal forms ravines or badlands (b) Deposition of atmospheric
due to intensive gully erosion. (a) Delhi-Mumbai-Kolkata-Chennai
moisture.
(b) Mumbai-Delhi-Kolkata-Chennai
It occurs when water is channeled (c) Almost microscopically small (c) Mumbai-Kolkata-Delhi-Chennai
across unprotected land and washes drops of water condensed from
away the soil along drainage lines. (d) Kolkata-Chennai-Mumbai-Delhi
Chambal rises from Mhow district in
and suspended in air.
Vindhya Ranges and it is a chief (d) The moisture content of the _ (b) According to the Census of 2011
Mumbai is the most populated urban
tributary of Yamuna river. atmosphere at a particular time agglomerations followed by Delhi,
and place. Kolkata and Chennai.
91. Which one of the following may
There are total 53 million plus cities in
be the true characteristic of _ (d) Humidity refers to the moisture
cyclones? content of the atmosphere at a India as per Census of 2011. Least
(a) Temperate cyclones move from particular time and place. It is populated among them is Kota.
West to East with Westerlies measured in either relative terms
97. Match List I with List II and select
whereas tropical cyclones follow (relative humidity) or absolute terms
(absolute humidity). the correct answer using the codes
trade winds.
(b) The front side of cyclone is known It is relative humidity which indicate
given below the lists.
as the ‘eye of cyclone’. the likelihood of precipitation. List I List II
(c) Cyclones possess a centre of high Fog is a form of suspended water (Type of Lake) (Example)
pressure surrounded by closed droplets caused by condensation. A. Tectonic 1. Lonar Lake
isobars. Fog can be considered a type of B. Crater 2. Gangabal Lake
(d) Hurricanes are well-known cloud at very low altitude. It reduces
tropical cyclones which develop C. Glacial 3. Purbasthali Lake
visibility and is a hazard.
over mid-latitudes. D. Fluvial 4. Bhimtal Lake
Almost microscopically small drops
_ (a) Temperate cyclones are cyclones of water condensed from and
Codes
of mid-latitudes and hence are suspended in air is initial stage of
primairly under influence of permanent cloud formation. When these droplets A B C D A B C D
winds of mid-latitudes, i.e Westerlies. combine together, clouds are (a) 4 1 2 3 (b) 4 2 1 3
formed. (c) 3 1 2 4 (d) 3 2 1 4
Their movement is therefore
Eastwards of their origin with average 94. The Shompens are the vulnerable _ (a)
velocity of Type of Lake Example
tribal group of
32 km per hour in summers and 48 Tectonic Bhimtal Lake
km per hour in winters. (a) Jharkhand
(b) Odisha (Uttarakhand)
Normally, tropical cyclones move Crater Lonar Lake (Maharashtra)
from East to West under the influence (c) West Bengal
(d) Andaman and Nicobar Islands Glacial Gangabal Lake
of trade winds because trade winds
are permanent winds of tropical (Jammu and Kashmir)
latitudes. _ (d) The Shompens are inhabitant of Fluvial Purbasthali Lake
Andaman and Nicobar Islands. Many (W. Bengal)
The general direction is therefore more tribes live on this island such as
Westwards from their origin. They
advance with varying velocities. Weak
the Great Andamanes, Onge, Jarawa 98. The Andaman group of islands and
and Sentinelese. the Nicobar group of islands are
cyclones move at the speed of about
32 km per hour while hurricanes attain 95. Which one of the following cities separated by which one of the
the velocity of 180 km per hour or was not included in the list of following latitudes?
more.
smart cities in India? (a) 8° N latitude (b) 10° N latitude
Hence, option (a) is correct. (c) 12° N latitude (d) 13° N latitude
(a) Silvassa (b) Jorhat
92. The headquarters of the (c) Itanagar (d) Kavaratti _ (b) The Andaman and Nicobar group
International Tropical Timber are situated in Bay of Bengal. The
Organisation is located at _ (b) The smart city mission was Islands are an extension of Arakan
launched by Government of India to
(a) New Delhi (b) Yokohama develop smart cities in country. mountain ranges. Both the group are
(c) Madrid (d) Jakarta separated by 10°N latitude.
Silvassa and Kavaratti is included in the
The Saddle peak is highest peak and
_ (b) The International Tropical Timber list of smart cities. Itanagar (Arunachal
Organisation was established in 1986 Pradesh) was also included as a special India’s only active volcano on these
with a objective to conserve and case in Smart City Mission in August, Islands. Barren island is also located
manage tropical forests resources. 2017. in Andaman sea.
Its headquarter is located at Shillong became 100th city, added to 99. Daman Ganga Reservoir Project
Yokohama (Japan). the smart city mission. Jorhat with about 115 km of minor canals
(Assam) is not included among the
93. Atmospheric conditions are smart cities.
and distributaries is located in
well-governed by humidity. Which (a) NCT
one among the following may best 96. Find the correct arrangement of (b) Dadra and Nagar Haveli
define humidity? the following urban (c) Puducherry
agglomerations in descending (d) Goa
CDS Solved Paper 2019 (I) 79

_ (b) The Daman Ganga Reservoir _ (a) The Prime Minister’s National Relief guided by principles of natural
Project is located in Dadra and Nagar Fund (PMNRF) was established in justice.
Haveli. The Daman Ganga (also 1948 to assist displaced person from Hence, option (d) is not correct.
known as Daman river) flows through Pakistan, now resources of PMNRD
Maharashtra, Gujarat, Daman and Diu 105. Which one of the following
are utilised to render relief to families of
and Dadra and Nagar Haveli. those killed in natural calamities and
statements about the provisions of
man-made disaster. This fund is the Constitution of India with
100. Consider the following statements operated by the Prime Minister’s Office
relating to Coal India Limited: regard to the State of Jammu and
(PMO).
1. It is designated as a ‘Maha Kashmir is not correct. Prior to
Ratna’ company under the 103. Which one of the following arogational of Article 370.
statements with regard to India’s (a) The Directive Principles of State
Ministry of Coal.
surgical strike mission inside
2. It is the single largest coal Policy do not apply.
Pakistan occupied Kashmir is
producing company in the correct? (b) Article-35A gives some special
world. (a) It was conducted in the year 2018 rights to the permanent residents
3. The Headquarters of Coal India (b) It was led by the Indian Air Force of the State with regard to
Limited is located at Ranchi (c) It was not given any name employment, settlement and
(Jharkhand). (d) It was sanctioned by the United property.
Which of the statement(s) given Nations (c) Article-19(1)(f) has been omitted.
above is/are correct? (d) Article-368 is not applicable for
(a) Only 1(b) 1 and 2 _ (c) On 29th September, 2016 India
announced that it conducted “surgical the Amendment of Constitution of
(c) 2 and 3 (d) 1, 2 and 3 the State.
strikes” against the launch pads of
_ (b) Coal India Limited is a public militants across the line of control in
_ (c) The Government of India
sector company under the Ministry of Pakistan occupied Kashmir (PoK). The abrogated the Article 370 in August
Coal. operation was not given any name. In 2019, thus making all the provisions of
Its salient features are 2019, Balakot strike were conducted the Indian Constitution applicable to
(i) It is designated as a ‘Maha Ratna’ by Indian Air Force in response of Jammu and Kashmir.
company under the Ministry of coal. Pulwama attack.
Prior to its abrogation, Article 370
(ii) It is the single largest coal producing Hence, option (c) is correct. provided the State of Jammu and
company in the world. Thus Kashmir, power to have a separate
statements 1 and 2 are correct.
104. Which one of the following Constitution, a state flag and
(iii) Coal India is headquartered at
statements about the National autonomy over the internal
Kolkata, West Bengal. Green Tribunal is not correct? administration of the state.
Thus, statement 3 is incorrect. (a) It was set-up in the year 2010. According to question, option (c) is
(b) It is involved in effective and not correct.
101. Afro-Asian solidarity as a central expeditious disposal of cases
element of India’s foreign policy 106. In 1921, during which one of the
relating to environmental protection following tours, Gandhiji shaved
was initiated by which of the
and conservation of forests. his head and began wearing
following Prime Ministers?
(c) It may consider giving relief and loincloth in order to identify with
(a) Narendra Modi
compensation for damages to the poor?
(b) IK Gujaral
(c) JL Nehru persons and property. (a) Ahmedabad (b) Champaran
(d) Manmohan Singh (d) It is bound by the procedures laid (c) Chauri Chaura (d) South India
down under the Code of Civil
_ (c) Afro-Asian solidarity as a central Procedure, 1908. _ (d) Gandhiji was on Journey down to
element of India’s foreign policy was South India (Madurai) in 1921.
initiated by PM Jawaharlal Nehru. _ (d) The Legislate Act of Parliament He asked the people to wear Khadi,
JL Nehru believed that Asia had a defines the National Green Tribunal but they replied that they are too poor,
certain responsibility toward the Act, 2010 as follows, ‘‘An act to cannot afford Khadi clothes, after this
people of Africa. In this regard, he provide for the establishment of a incident Gandhiji decided to shave
followed Afro-Asian solidarity as a National Green Tribunal for the his head and began wearing loincloth
central element of India’s foreign effective and expeditious disposal of in order to identify with the poor.
policy. cases relating to environmental
protection and conservation of forests 107. Simla was founded as a hill station
102. The Prime Minister’s National and other natural resources including to use as strategic place for
Relief Fund is operated by which enforcement of any legal right relating billeting troops, guarding frontier
one of the following bodies? to environment and giving relief and
and launching campaign during
(a) The Prime Minister’s Office (PMO) compensation for damages to
persons and property and for matters the course of
(b) The National Disaster (a) Anglo-Maratha War
connected therewith or incidental
Management Authority thereto.” (b) Anglo-Burmese War
(c) The Ministry of Finance (c) Anglo-Gurkha War
The tribunal shall not be bound by the
(d) The National Development procedure laid down under the Code (d) Anglo-Afghan War
Council (NDC) of Civil Procedure, 1908, but shall be
80 CDS Solved Paper 2019 (I)

_ (c) Simla was founded as a hill station 110. Eight states have achieved more available of affordable prices for all
by the Britisher during Anglo-Gurkha or particularly, poor and disadvantage.
than 99% household electrification
Anglo-Nepalese war fought between It is implemented by Bureau of
1814-16.
prior to the launch of ‘Saubhagya
Pharma PSUs of India (BPPI) which is
Scheme’. registered as an independents
The war ended with the signing of the
treaty of Sugauli in 1816. The Which one of the following is not society under the Societies
strategic location of Simla decided its among them? Registration Act, 1860.
for times. (a) Kerala Hence, both the statements are
(b) Punjab correct.
108. Which politician in British India
had opposed to a Pakistan that (c) Himachal Pradesh 113. Consider the following statements
would mean ‘‘Muslim Raj here and (d) Madhya Pradesh about a scheme launched by the
Hindu Raj elsewhere’’? _ (d) Eight states which achieved more Government of India.
(a) Khan Abdul Ghaffar Khan than 99% household electrification
It was launched to provide social
prior to the launch of Saubhagya
(b) Sikandar Hayat Khan Scheme are Andhra Pradesh, Gujarat, security during old age and to
(c) Maulana Abul Kalam Azad Goa, Haryana, Himachal Pradesh, protect elderly persons aged 60
(d) Rafi Ahmed Kidwai Kerala, Punjab and Tamil Nadu. years and above against a future
_ (b ) Premier of Punjab Sir Sikandar So, Madhya Pradesh is not among fall in their interest income due to
Hyat Khan predicted to told the Punjab them. Pradhan Mantri Sahaj Bijli Har uncertain market conditions. The
Legislative Assembly. Ghar Yojna (Saubhagya) to ensure
scheme enables old age income
On 11th March 1941 "We do not ask electrification of all willing household
in the country. security for senior citizens
for freedom that there may be Muslim
through provision of assured
Raj here and Hindu Raj elsewhere. If 111. In October, 2018, India was elected
that is what Pakistan means I will have pension/return linked to the
as a member to the United Nations subscription amount based on
nothing to do with it.
Human Rights Council for a period government guarantee to
109. Match List I with List II and elect of
the correct answer using the codes Life Insurance Corporation
(a) five years (b) four years of India (LICI). Identify the
given below the lists. (c) three years (d) two years scheme.
List I List II (Book)
_ (c) India was elected for three years (a) Pradhan Mantri Swasthya
(Author) (till October 2021) as a member of Suraksha Yojana
A. Sekhar 1. Jawaharlal Nehru UNHRC. India received 188 votes, the (b) Pradhan Mantri Vaya Vandana
Bandyopadh : A Biography, highest polled by any of the 18
Yojana
yay Vol-I, 1889-1947 countries elected in the voting.
(c) Liveability Index Programme
B. Sarvepalli 2. From Plassey to This is the 5th time India is elected to
(d) Rashtriya Vayoshri Yojana
Gopal Partition : A the Geneva-based Council, the main
History of body of the UN charged with _ (b) Pradhan Mantri Vaya Vandana
Modern India promoting and monitoring human Yojana (PMVVY) provides an assured
rights. India will join China and Nepal, pension based on a guaranteed rate of
C. David 3. The Ascendancy
besides Pakistan, which were elected return of 8% per annum for 10 year. Its
Hardiman of the Congress
to the 47 Member Council in previous salient feature
in Uttar Pradesh,
years to serve three year terms. (i) Investment limit extended upto 15
1926-1934
112. Consider the following statements lakh.
d. Gyanendra 4. Gandhi in His
about the Bureau of Pharma PSUs (ii) Time period for subscription up to
Pandey Time and Ours
21st March, 2020.
of India (BPPI)
Codes (iii) On premature exit 98% purchase
1. It is the implementing agency price will be refunded.
A B C D A B C D
(a) 2 4 1 3 (b) 2 1 4 3
of Pradhan Mantri Bhartiya (iv) Senior citizen will get a pension upto
(c) 3 1 4 2 (d) 3 4 1 2 Janaushadhi Pariyojana (PMBJP). 10000 per month.
_ (b) 2. It has been registered as an 114. Who among the following won
Sekhar From Plassey to Partition: independent society under the India’s first ever gold medal in the
Bandyopadhyay A History of Modern India Societies Registration Act, International Youth Olympic
Sarvepalli Gopal Jawaharlal Nehru : A 1860. Games (2018) held in Argentina?
Biography, Vol-I, 1889-1947 Which of the statements given (a) Neeraj Chopra
David Hardiman Gandhi in His Time and above is/are correct? (b) Praveen Chitravel
Ours (a) Only 1 (b) Only 2 (c) Jeremy Lalrinnunga
Gyanendra Pandey The Ascendancy of the (c) Both 1 and 2 (d) Neither 1 nor 2 (d) Suraj Panwar
Congress in Uttar Pradesh,
_ (c) Both the statement are true. _ (c) Jeremy Lalrinnunga becomes
1926 - 1934 Pradhan Mantri Bhartiya Janaushadhi India’s first ever gold medalist after
Pariyojana was launched in 2015 with winning gold in the mens 62 kg
an objective to make quality education weightlifting competition.
CDS Solved Paper 2019 (I) 81

He hails from State of Mizoram. India Army Air Defence College is located 2. Cashless and paperless access to
ranked 17th with 13 medals which in Gopalpur (Odisha). It is a training quality health-care services.
include 3 Gold, 9 Silver and 1 Bronze. academy of Indian Army established
in 1940. 3. Government provides health
115. EK Janaki Ammal National Award insurance cover of up to
Rashtriya Indian Military College is
on Taxonomy is administered by located in Dehradun (Uttarakhand). It ` 5,00,000 per family per year.
the is a military school for boys 4. Pre-existing diseases are not
(a) Ministry of Agriculture and established in 1922.
covered.
Farmers Welfare 117. Which one of the following
(b) Ministry of New and Renewable Select the correct answer using
viruses is responsible for the recent the codes given below
Energy
death of lions in Gir National Park?
(c) Ministry of Health and Family (a) 1 and 3 (b) 1, 2 and 3
(a) Canine Distemper Virus (c) 2 and 4 (d) 2, 3 and 4
Welfare
(b) Nipah Virus
(d) Ministry of Environment, Forest
(c) Hendra Virus _ (b) Benefits of PMJAY Scheme have
and Climate Change been enumerated below
(d) Foot-and-Mouth Disease Virus
_ (d) EK Janaki Ammal National Award (i) Ayushman Bharat Pradhan Mantri
on Taxonomy is administered by the _ (a) Canine Distemper Virus is the main Jan Arogya Yojana (PMJAY) will
Ministry of Environment, Forest and cause of recent death of lions in Gir provide a cover of up to ` 5 lakhs
Climate change. National Park. It is a member of per family per year, for secondary
Paramyxoviridae family that causes and tertiary care hospitalisation.
This award is categorised in three canine distemper disease which
fields Plants, Animals and Microbial (ii) When fully implemented, PMJAY will
affects a wide variety of animal families, become the world’s largest fully
Taxonomy. i.e., dogs, cats, etc. government-financed health
The award is named after EK Janaki This disease is highly contagious via protection scheme.
Ammal legendary Botanist from inhalation. It affects gestrointestinal
Kerala. It is a visionary step towards
and respiratory tracts and sometimes advancing the agenda of Universal
116. Which one of the following pairs nervous system. High fever, eye Health Coverage (UHC).
inflammation, coughing, diarrhea, loss
of military training institute of (iii) PMJAY will provide cashless and
of appetite, hardening of nose, etc, are
India and location is not correctly paperless access to service for the
its common symptoms.
beneficiary at the point of service.
matched?
118. Till 2018, which of the following (iv) PMJAY will help reduce catastrophic
(a) Army War College : Mhow expenditure for hospitalisations,
(b) High Altitude Warfare School :
countries have legalised the possession
and use of recreational cannabis? which impoverishes people and will
Gulmarg help mitigate the financial risk arising
(c) Army Air Defence College : Pune
l. America 2. Canada out of catastrophic health episodes.
(d) Rashtriya Indian Military College : 3. Nigeria 4. Uruguay (v) Over 10.74 crore vulnerable entitled
Dehradun Select the correct answer using families (approximately 50 crore
beneficiaries) will be eligible for
_ (c) the codes given below
these benefits.
Institute Location (a) 1, 2 and 3 (b) 2 and 4
(vi) Entitled families will be able to use
Army War College Mhow (Madhya (c) 1 and 4 (d) 1, 2 and 4
the quality health services they need
Pradesh)
High Altitude Warfare Gulmarg (Jammu _ (b) Cannabis (Marijuana), is a psycho without facing financial hardships.
active drug used for medical and Hence, option (b) is the correct.
School and Kashmir) recreational purpose.
Army Air Defence Gopalpur (Odisha) 120. The 11th BRICS Summit in 2019
Canada became the second
College hosted by
country after Uruguay to Legalise
Rashtriya Indian Dehradun
possession and use of recreational (a) China (b) Russia
Military College (Uttarakhand) cannabis. (c) Brazil (d) India
Army War College is located in In US, it is legalised at federal level,
Mhow (Madhya Pradesh). It is a but few State allow use of cannabis _ (c) The 11th BRICS Summit in 2019
Tactical training and research was held in Brazil. BRICS is an
for recreational purpose.
institution established in 1971. association of five major emerging
High Altitude Warfare School is
119. Which of the following are the national economies. It include Brazil,
benefits of the Pradhan Mantri Jan Russia, India, China and South Africa.
located in Gulmarg (Jammu and The 12th edition of BRICS Summit will
Kashmir). It is a training and research Arogya Yojana (PMJAY)? be hosted by Russia at Saint
establishment of Indian Army set up 1. Free treatment available at all Petersburg in July 2020.
in 1948. public and empanelled private
hospitals in times of need.
CDS
Combined Defence Service

SOLVED PAPER 2018 (II)


PAPER I Elementary Mathematics

1. The highest four-digit number Cubing both sides, we get Hence, 3 five digit numbers of the form
which is divisible by each of the ( x − 2 )3 = (2 2/ 3 + 2 1/ 3 )3 XXYXX are divisible by 33 viz. 33033,
66066, 99089.
numbers 16, 36, 45, 48 is ⇒ x 3 − 2 3 − 3( x )(2 )( x − 2 )
(a) 9180 (b) 9360 = (2 2/ 3 )3 + (2 1/ 3 )3 + 3(2 )2/ 3 5. A five-digit number XY235 is
(2 )1/ 3(2 2/ 3 + 2 1/ 3 ) divisible by 3 where X and Y are
(c) 9630 (d) 9840
(Q( a − b ) = a − b − 3ab ( a − b )
3 3 3 digits satisfying X + Y ≤ 5. What is
_ (b) Number which is divisible by the number of possible pairs of
16, 36, 45 and 48 = LCM of (16, 36, ( a + b )3 = a 3 + b 3 + 3ab ( a + b )
values of ( X , Y )?
45, 18) ⇒ x − 8 − 6x 2 + 12 x
3
(a) 5 (b) 6 (c) 7 (d) 9
LCM of 16, 36, 45, 48 = 720 = 2 2 + 2 + 6( x − 2 )
Now, highest four digit number ⇒ x − 6x + 12 x = 6 + 8 + 6x − 12
3 2 _ (c) Given, XYZ 35 is divisible by 3 and
X+Y≤5
= 9999 ⇒ x 3 − 6x 2 − 6x = 2 Now, for a number to be divisible by 3,
i.e. 9999 = 720 × 13 + 639 the sum of digits must be divisible by 3.
4. How many five-digit numbers of
∴ Highest four digit number exactly i.e. X + Y + 2 + 3 + 5 or 10 + X + Y is
divisible by 16, 36, 45 and 48
the form XXYXX is/are divisible
divisible by 3.
by 33?
= 9999 − 639 = 9360 Now, two cases are possible, i.e. 12
(a) 1 (b) 3 and 15.
2. If x = y , y = z b and z = x c , then
a
(c) 5 (d) Infinite ∴Possible values of
the value of abc is ( X , Y ) = (1, 1), (2, 0), (1, 4), (2, 3),
_ (b) For a number to be divisible by
(a) 1 (b) 2 33 it must be divisible by 3 and 11. ( 3, 2 ),( 4, 1) and ( 5,0)
(c) –1 (d) 0 Now, a number is divisible by 11, if i.e. 7 possible values are there.
the difference of the sum of digits at
_ (a) Given, x = y
a
odd places and the sum of digits at
6. If x 2 − 6x − 27 > 0, then which one of
Or, x = ( zb )a (as y = zb ) even places is either zero or divisible the following is correct?
(a) −3 < x < 9 (b) x < 9 or x > −3
( )
a by 11.
Or, x = ( x c )b (as z = x c )
∴For XXYXX to be divisible by 11 (c) x > 9 or x < −3 (d) x < −3 only
Or, x = x abc 2 X + Y − 2 X i.e. Y must either be 0 or _ (c) Given, x − 6x − 27 > 0
2

Or, abc = 1 divisible by 11. Or x 2 − 9x + 3x − 27 > 0


3. If x = 2 + 2+ 2 , then the value
2/ 3 1/ 3 Now, Y cannot be divisible by 11 x ( x − 9) + 3( x − 9) > 0
because it is a single digit number.
of the expression x 3 − 6x 2 + 6x ( x + 3) ( x − 9) > 0
∴ Y =0
will be Now, for a number to be divisible by Now, for ( x + 3) ( x − 9) to be greater
3 the sum of digits must be divisible than 0.
(a) 2 (b) 1
by 3. Either ( x + 3) and ( x − 9) must be greater
(c) 0 (d) –2 i.e. X + X + 0 + X + X or 4X must be than 0.
divisible by 3. or both ( x + 3) and ( x − 9) must be less
_ (a) Given, x = 2 + 2 + 2
2/ 3 1/ 3

This would happen only when X is than 0.


⇒ x − 2 = 2 2/ 3 + 2 1/ 3
either 3, 6 or 9. ∴x > 9 and x < −3
2 CDS Solved Paper 2018 (II)

7. The number of divisors of the and 1< p < q < r ∴ Time taken by B to finish the job = 4x
1 −p days
number 38808 exclusive of the From option(a) =
divisors 1 and itself, is α+β q Now, according to the question,
1 1 1
(a) 74 (b) 72 Option (b) + =
x 4x 5
(c) 70 (d) 68 1 1 α + β −q p −q 4x + x
+ = = × = ⇒ =
1
α β αβ p r r 4x 2 5
_ (c) To find the number of divisors of
a number we must write it in the form −1 − p 5x 1
Option (c) = ⇒ =
of a mb nc 0d p … where a, b, c, d … are αβ r 4x 2 5
prime numbers.
25 1
αβ r  − p − r ⇒ x = = 6 days
Now, Option (d) = ×  = 4 4
38808 = 2 × 2 × 2 × 3 × 3 × 7 × 7 × 11 α+β p  q  q
∴Number of days taken by A to complete
Now, except (d) in all other options 1
= 2 × 3 × 7 × 11
3 2 2 1
the numerator is smaller than job 6 days
4
Now, number of divisors of 38808 denominator.
= k  x 2 + 2  , then k is
1 1
except 1 and 38808 ∴Option (b) is smallest. 11. If x 6 +
= ( 3 + 1) (2 + 1) (2 + 1) (1 + 1) − 2 −p x 6  x 
Now, from option (a) and (c), i.e.
= 4× 3× 3×2 −2 q equal to
−p −p
is greater as r > q
(a)  x 2 − 1 + 2  (b)  x 4 − 1 + 4 
= 72 − 2 = 70 and , 1 1
r r  x   x 
8. HCF and LCM of two polynomials −p
Now, from option (c) and (b), i.e.
(c)  x 4 + 1 + 4  (d)  x 4 − 1 − 4 
1 1
are ( x + 3) and ( x 3 − 9 x 2 − x + 105) r
−q  x   x 
respectively. If one of the two and
r
polynomials is x 2 − 4 x − 21, then −p
_ (b) We know that,
the other is is greater as q > p a 3 + b 3 = ( a + b )( a 2 − ab + b 2 )
r 3
(a) x 2 + 2 x − 21 (b) x 2 + 2 x + 15 1 1  1
∴ − is greatest. ∴ x6 + = ( x 2 )3 +  2 
(c) x 2 − 2 x − 15 (d) x 2 − x − 15 αβ x6 x 

_ (c) Given, HCF of polynomials  1  1 


10. Two workers ‘A’ and ‘B’ working =  x 2 + 2  x 4 − 1+ 4 
=x+3  x  x 
together completed a job in 5 days.
1
LCM of polynomials Had ‘A’ worked twice as ∴ k = x 4 − 1+ 4
= x 3 − 9x 2 − x + 105 efficiently as he actually did and
x
One polynomial = x 2 − 4x − 21 ‘B’ worked one-third as efficiently 12. If the sum of the squares of three
We know that, (HCF × LCM) of two as he actually did, the work would consecutive natural numbers is 110,
numbers = Product of two number have completed in 3 days. In how then the sum of their cubes is
∴ Other polynomial many days could ‘A’ alone (a) 625 (b) 654 (c) 684 (d) 725
( x + 3) ( x 3 − 9x 2 − x + 105) complete the job?
= _ (c) Let the three consecutive natural
x 2 − 4x − 21 1 1 numbers be
(a) 3 days (b) 4 days
( x + 3) ( x 3 − 9x 2 − x + 105) 2 6 ( x − 1), x , ( x + 1)
=
( x − 7 ) ( x + 3) 1 1 According to the question,
(c) 5 days (d) 6 days
x 3 − 9x 2 − x + 105 2 4 ⇒ ( x − 1)2 + x 2 + ( x + 1)2 = 110
=
x −7 Q ( a − b )2 = a 2 + b 2 − 2 ab
_ (d) Let the efficiency of A and B be
⇒ Other polynomial = x − 2 x − 15 2
‘a’ and ‘b’ respectively. ( a + b )2 = a 2 + b 2 + 2 ab
9. If α and β are two real numbers According to the question, ⇒ x + 1− 2x + x 2 + x 2 + 1+ 2x
2

q r  b = 110
such that α + β = − and αβ = , ( a + b )5 =  2 a +  ( 3)
 3 ⇒ 3x 2 = 110 − 2
p p
⇒ 5a + 5b = 6a + b 108
where 1 < p < q < r, then which ⇒ x =
2
= 36
3
one of the following is the ⇒ a = 4b
a 4 ∴ x =6
greatest? or =
b 1 ∴The numbers are ( 6 − 1), 6, ( 6 + 1)
1 1 1
(a) (b) + 1 i.e. 5, 6 and 7
α+β α β Now, efficiency ∝
Time Now, sum of their cubes
1 αβ
(c) − (d) ∴Ratio of time taken A and B = 1 : 4 = 53 + 63 + 7 3
αβ α+β
Let time taken by A to finish the job = 125 + 216 + 343 = 684
−q r be x days
_ (c) Given, α + β = , αβ =
p p
CDS Solved Paper 2018 (II) 3

13. The product of two integers p and or x = k1/ x = ( X + Y + Z ) (100 + 10 + 1)


Similarly, b = k1/ y and c = k1/ z = ( X + Y + Z ) (111)
q, where p > 60 and q > 60, is 7168
and their HCF is 16. The sum of Now, abc = 1 or k1/ x . k1/ y . k1/ z = 1 ∴ The sum of number is divisible by
1 1 1 X + Y + Z and 111.
these two integers is  + + 
or k  xy z
=1
(a) 256 (b) 184 (c) 176 (d) 164  1 1 1
18. The number of all pairs (m, n ), where
 + + 
 x y z m and n are positive integers, such that
_ (c) Given, p and q are integers or k = kº
where p > 60 and q > 60 also 1 1 1 2
or
1 1 1
+ + =0 + − = is
p.q = 7168 x y z m n mn 5
And HCF of p and q = 16 (a) 6 (b) 5 (c) 4 (d) 2
Now, the HCF is 16 that means both
16. If α and β are the roots of the
1 1 1 2
equation ax + bx + c = 0, then the
2 _ (c) We have, + − =
‘ p’ and ‘q’ are divisible by 16 m n mn 5
1 1 n+ m−1
So, p and q might be 64, 80, 96, value of + is ⇒ =
2
112, … aα + b aβ + b mn 5
Now, 64 × 112 = 7168 a b c 1 5n + 5m − 5 = 2 mn
(a) (b) (c) (d)
∴Required integers = 64, 112 bc ac ab abc 5m − 5 = 2 mn − 5m
And sum of these two integers Here, m and n such positive integer
= 64 + 112 = 176 _ (b) We have, α, β are the roots of the
equation ∴Possible value are
14. If log10 2 = 03010
. and ax 2 + bx + c = 0 (3, 10), (4, 5), (5, 4), (10, 3)
log10 3 = 0.4771, then the value of ∴α + β =
−b
, αβ =
c 19. If a = xy p −1 , b = yz q −1 , c = zx r −1 , then
. ) is equal to
log10 (072
a q −r b r − p c p −q is equal to
a a
1 1
(a) 0.9286 (b) 1.9286 Now, +
aα + b aβ + b (a) abc (b) xyz
(c) 1.8572 (d) 1.9572
aβ + b + aα + b (c) 0 (d) None of these
=
_ (b) We have, log10 2 = 0.3010 ( aα + b ) ( aβ + b )
_ (d) We have,
and log10 3 = 0.4771 a(α + β ) + 2 b a = xy p −1, b = yzq −1, c = zx r −1
=
. ) = log10 2( 072
log100( 072 . ) a (αβ ) + ab(α + β ) + b
2 2
Now, a q −r
b r − pc p −q
1
= log10( 072. )  −b  = ( xy p −1 )q − r ( yzq −1 )r − p ( zx r −1 )p −q
2 a  + 2b
 a  = x q − r y( p −1)(q − r ) y r − p z(q −1)( r − p )
=
1
= log10
72 c  −b  zp −q x ( r −1)( p −q )
a 2   + ab  +b
2
(q − r ) +( r −1)( p −q ) ( p −1)(q − r ) + r − p
2 100  a  a  =x y
1 z(q −1)( r − p ) + p −q
= [log10 72 − log10 100] =
b
=x q − r + pr −qr − p +q pq − pr −q + r + r − p
y
2
zqr − pq − r + p + p −q
ac
1
= [log10 9 × 8 − log10 10 2 ] 17. Consider the following statements = x 2q − p + r( p +q −1) y 2 r −q + p(q − r −1)
2
in respect of three 3-digit numbers z2 p − r +q ( r − p −1)
1
= [log10 9 + log10 8 − 2 log10 10] XYZ, YZX and ZXY:
2 20. The number of sides of two regular
1 1. The sum of the numbers is not polygons are in the ratio 5 : 4. The
= [log10 3 2 + log10 2 3 − 2 ]
2 divisible by ( X + Y + Z ). difference between their interior
1 2. The sum of the numbers is angles is 9º. Consider the following
= [2 log10 3 + 3 log10 2 − 2 ]
2 divisible by 111. statements
1 1. One of them is a pentagon and the
= [2 × 0.4771 + 3 × 0.3010 − 2 ] Which of the above statements is /
2 other is a rectangle.
are correct?
1
= [0.9542 + 0.9030 − 2 ] (a) 1 only 2. One of them is a decagon and the
2
(b) 2 only other is an octagon.
1
= [18572
. − 2 ]= 0.9286 − 1= 1.9286 3. The sum of their exterior angles is
2 (c) Both 1 and 2
720º.
15. If a = b = c and abc = 1 , then
x y z (d) Neither 1 nor 2
Which of the above statements is/are
1 1 1
the value of + + will be _ (b) We have, 3-digits numbers are correct?
x y z XYZ , YZX , ZXY
(a) 1 only
equal to XYZ = 100 X + 10Y + Z
(b) 2 only
(a) −1 (b) 0 (c) 1 (d) 3 YZX = 100Y + 10Z + X
(c) 1 and 3
_ (b) Given, a = b = c = k (let)
x y z ZXY = 100Z + 10 X + Y
XYZ + YZX + ZXY (d) 2 and 3
and abc = 1
= 100( X + Y + Z ) + 10( X + Y + Z ) _ (d) Let the numbers of sides of two
Now, ax = k
+X+ Y+ Z regular polygons is 5x and 4x
4 CDS Solved Paper 2018 (II)

5x − 2 H H 1
∴Interior angles are × 180º ⇒ + =2 +
5x P Q c( a + b + c )
4x − 2 1 1
and × 180º 23. The sum of all possible products =
1
+
1
+
4x
taken two at a time out of the ( a + b + c )  a b c 
respectively
numbers ± 1, ± 2, ± 3, ± 4 is =
1  ab + bc + ca 
According to the question,   =0
(a) 0 (b) –30 (c) 30 (d) 55 (a + b + c )  abc 
5x − 2 4x − 2
× 180º − × 180º = 9º
5x 4x _ (b) Let ± 1 , ± 2 , ± 3 , ± 4 are the 26. What is the principal amount which
180º  5x − 2 4x − 2  roots of the polynomials earns ` 210 as compound intrest for
⇒ − = 9º
x  5 4  ∴( x + 1) ( x − 1) ( x + 2 ) ( x − 2 ) ( x + 3) the second year at 5% per annum?
180º  20x − 8 − 20x + 10  ( x − 3) ( x + 4) ( x − 4) = 0 (a) ` 2000 (b) ` 3200
⇒  = 9º
x  ⇒ ( x − 1) ( x 2 − 4) ( x 2 − 9)
2
20 (c) ` 4000 (d) ` 4800
⇒ x =2 ( x 2 − 16) = 0
_ (c) Let principal be ` P
⇒ ( x − 5x + 4)
4 2
∴ Number of sides of polygons are  Rate 
Time

10 and 8 respectively ( x 4 − 25x 2 + 144) = 0 Q Amount = Principal  1 + 


 100 
⇒ x 8 − 30x 6 + 148x 4
∴One of them decagon and other is
−820x 2 + 576 = 0 ∴Amount after one year
octagon 1
The sum of the roots taken two at a  5  21P
We know that, sum of exterior angle or, = P1 +  =
time is coefficient of x 6 is −30  100  20
of regular polygon is 360º
21P P
∴ Sum of exteriors angles of 24. The remainder when or CI after one year = −P=
20 20
polygons are 360º + 360º = 720º 3x − 2x y − 13xy + 10y
3 2 2 3
Similarly, amount after two years
21. The minimum value of the is divided by ( x − 2y ) is equal to 2
 5  441
expression 2x 2 + 5x + 5 is (a) 0 (b) y = P1 +

 = P
100  400
(a) 5 (b) 15/8 (c) y − 5 (d) y + 3 441P 41P
(c) −15 / 8 (d) 0 and CI after two years = − P=
_ (a) Let f( x ) 400 400
_ (b) Let y = 2 x + 5x + 5
2
= 3x 3 − 2 x 2 y − 13xy 2 + 10 y 3 According to the question,
 5 5 g (x ) = x − 2 y 41P

P
= 210
y = 2 x 2 + x +  And
 2 2 400 20
When f( x ) is divided by g ( x )
41P − 20P
 5 25  25 ∴ f(2 y) = 3(2 y)3 − 2(2 y)2 y ⇒ = 210
y = 2 x 2 + x +  + 5− 400
 2 16  8 −13(2 y) y 2 + 10 y 3
⇒ 21P = 210 × 400
 5
2
15 f(2 y) = 24 y − 8 y 3 − 26 y 3 + 10 y 3
3
y = 2 x +  + or Principal = ` 4000
 4 8 f(2 y) = 0
15 Hence, remainder = 0 27. In an examination, 50% of the
∴Minimum value of y is candidates failed in English, 40% failed
8 25. If ab + bc + ca = 0, then the value of in Hindi and 15% failed in both the
22. If H is the harmonic mean P and Q, (b 2 − ca ) (c 2 − ab ) + (a 2 − bc ) subjects. The percentage of candidates
H H (c 2 − ab ) + (a 2 − bc ) (b 2 − ca ) who passed in both English and Hindi
then the value of + is is
P Q (a 2 − bc ) (b 2 − ca ) (c 2 − ab ) is
(a) 1 (b) 2 (a) –1 (b) 0 (c)1 (d) 2 (a) 20% (b) 25%
P +Q PQ (c) 60% (d) 75%
(c) (d) _ (b) We have, ab + bc + ca = 0
PQ P +Q _ (b) Given,
Now,
_ (b) We know that, harmonic mean of ( b 2 − ca ) (c 2 − ab ) + ( a 2 − bc ) 50% candidates failed in English
a and b is (c 2 − ab ) + ( a 2 − bc ) ( b 2 − ca ) 40% candidates failed in Hindi
2 ab
H. M = ( a 2 − bc ) ( b 2 − ca ) (c 2 − ab ) and 15% candidates failed in both
a+ b
1 1 1
2 PQ = + + English Hindi
∴ H= a 2 − bc b 2 − ca c 2 − ab
P+Q
1 1
2 P+Q = +
⇒ = a 2 + ab + ac b 2 + ab + bc 35% 15% 25%
H PQ
1
 1 1 +
⇒ 2 = H +  c 2 + bc + ca
 P Q
1 1
 1 1 = + ∴ Total percentage of candidates who
⇒ H +  = 2 a( a + b + c ) b( a + b + c )
 P Q failed either in one or two subjects
= ( 35 + 15 + 25)% = 75%
CDS Solved Paper 2018 (II) 5

∴ Percentage of candidates who 30. In a race of 1000 m, A beats B by Q There are 5 members in the family.
passed in both subjects 96 − 66
150 m, while in another race of ∴Present age of Sonu = = 6 yr
= (100 − 75)% = 25% 3000 m, C beats D by 400 m. Speed 5

28. A train 100 m long passes a of B is equal to that of D. (Assume ∴ Present age of father = 6 × 6 = 36 yr
platform 100 m long in 10 s. The that A, B, C and D run with and father’s age =12 yr
speed of the train is uniform speed in all the events). If Hence, = 36 + 12 = 48 yr
(a) 36 km/h (b) 45 km/h A and C participate in a race of 32. ‘A’ is thrice as good a workman as ‘B’
6000 m, then which one of the and takes 10 days less to do a piece of
(c) 54 km/h (d) 72 km/h
following is correct? work than ‘B’ takes. The number of
_ (d) Given, length of train = 100 m (a) A beats C by 250 m days taken by ‘B’ alone to finish the
Length of platform = 100 m
(b) C beats A by 250 m work is
Time taken to pass platform = 10 s
(c) A beats C by 115.38 m (a) 12 (b) 15
Now, distance covered by train
(c) 20 (d) 30
100 m +100 m = 200 m (d) C beats A by 115.38 m
Q Speed =
Distance _ (b) A is thrice as good a work man as B
_ (c) In a race of 1000 m, A beats B by
Time 150 m Let efficiency of B be b
∴Speed =
200
= 20 m/s So, if A covers 1000 m in ‘t’ s, ∴Efficiency of A = 3b
10 Now, A takes 10 days less to do a piece of
then B covers 850m , ‘t’ s
3600 work.
⇒ speed of train = 20 × km/h Similarly, if C covers 3000 m in ‘T’ s,
1000 1 1
then D covers 2600 m in ‘T’ s ∴ − = 10
(Q 1 km = 1000 m, 1 h = 3600 s) b 3b
Now, speed of B is equal to speed
= 72 km/h 3b − b
of D. ⇒ = 10
Now, time taken by B cover 2600 m 3b 2
29. A cyclist covers his first 20 km at 2b
an average speed of 40 kmp/h, T =
t
× 2600 ⇒ = 10
850 3b 2
another 10 km at an average speed 1
of 10 kmp/h and the last 30 km at or
T 52
= ⇒ b=
t 17 15
an average speed of 40 kmp/h. ∴Time taken by B to finish work = 15 days
Let T = 52 x and t = 17 x
Then, the average speed of the
entire journey is Now, A will cover 6000 m in 6t s or 33. Out of 85 children playing badminton
17 × 6x = 102 x or table tennis or both, the total
(a) 20 km/h
and C will cover 6000 m in 2t s number of girls in the group is 70% of
(b) 26.67 km/h or 2 × 52 x = 104x the total number of boys in the group.
(c) 28.24 km/h A takes less time it means A beats C The number of boys playing only
(d) 30 km/h Now, C takes 104x to cover 6000 m badminton is 50% of the number of
Distance So, C will cover distance in boys and the total number of boys
_ (b) We know that, Time =
Speed 6000 playing badminton is 60% of the total
2x = ×2
Time taken by cyclist to cover first 104 number of boys. The number of
20 1 = 115.38 m children playing only table tennis is
20km = = h
40 2 40% of the the total number of
So, A beats C by 115.38 m
Time taken by cyclist to cover 10 km children and a total of 12 children play
=
10
= 1h
31. The sum of ages of a father, a badminton and table tennis both. The
10 mother, a son Sonu and daughters number of girls playing only
And time taken by cyclist to cover Savita and Sonia is 96 yr. Sonu is badminton is
30 3
remaining 30 km = = h the youngest member of the
40 4 (a) 14
family. The year Sonu was born, (b) 16
Now, total distance travelled
the sum of the ages of all the (c) 17
= (20 + 10 + 30) km = 60 km
members of the family was 66 yr. If (d) 35
1 1
Total time taken =  + 1 +  h the father’s age now is 6 times that
2 3 _ (a) Total children = 85
of Sonu’s present age, then 12 yr.
Let number of boys be B
 2 + 4 + 3 9
=  = h Hence, the father’s age will be 70
 4  4 ∴Number of girls = B
(a) 44 yr (b) 45 yr 100
∴Required average speed 70
(c) 46 yr (d) 48 yr Now, B + B = 85
60
= ×4 100
9 _ (d) Sum of ages of father, mother,
Sonu, Savita and Sonia is 96 yr and ∴ B = 50
80
= km/h when Sonu was born the sum was And number of girls = 85 − 50 = 35
3
= 26.67 km/hr 66 yr.
6 CDS Solved Paper 2018 (II)

120
Now, according to the question, _ (c) Let total property of Mr. Sharma Or x = = 15
be `16x 8
Badminton Table Tennis
After Mr. Sharma’s death ∴Present age of mother = 3x + 10
Property of Mrs. Sharma = 8x = 3 × 15 + 10 = 55 yr and present age of
Boys= 25 Boys= 20 Property of Ravi and Raj = 4x and 4x daughter = x + 10 = 25 yr
Girls = 14 Girls = 14
After Ravi’s death 38. In a class of 60 boys, there are 45 boys
Property of Raj = 4x + 2 x = 6x who play chess and 30 boys who play
Boys= 5 After Raj’s death carrom. If every boy of the class plays
Girls = 7 Property of Mrs. Sharma atleast one of the two games, then
= 8x + 3x = 11x how many boys play carrom only?
∴ Number of girls playing only
Now, according to the question, (a) 30 (b) 20 (c) 15 (d) 10
badminton = 14
11x = 88000 or x = 8000 _ (c) Total boys = 60
34. A person bought two articles X ∴Property of Mr. Sharma Boys who play chess = 45
and Y from a departmental store.
= 16 × 8000 = ` 128
, ,000 Boys who play carrom = 30
The sum of prices before sales tax
36. X bought 4 bottles of lemon juice ∴Boys who play both chess and carrom
was ` 130. There was no sales tax
on the article X and 9% sales tax on and Y bought one bottle of orange = ( 45 + 30) − 60 = 15
the article Y . The total amount the juice. Orange juice per bottle costs Boys who play only carrom = 30 − 15 = 15
person paid, including the sales twice the cost of lemon juice per 39. Two equal amounts were borrowed at
tax was ` 136.75. What was the bottle. Z bought nothing but 5% and 4% simple interest. The total
price of the article Y before sales contributed ` 50 for his share of interest after 4 yr amounted to ` 405.
tax ? the drink which they mixed What was the total amount borrowed?
(a) ` 75 (b) ` 85 together and shared the cost
(a) ` 1075 (b) ` 1100
equally. It Z’s ` 50 is covered from (c) ` 1125 (d) ` 1150
(c) ` 122 (d) ` 125
his share, then what is the cost of
_ (a) Let the sum of article X and Y one bottle of orange juice? _ (c) Let the amount borrowed be ` P each
before sales tax be ` x and ` y for both the rates.
respectively (a) ` 75 (b) ` 50 (c) ` 46 (d) ` 30 Q Simple interest
According to the question, _ (b) Let the cost of 1 bottle of lemon Principal × Rate × Time
juice be ` x =
x + y = 130 …(i) 100
∴Cost of 1 bottle of orange juice
Now, after 9% sales tax on item Y ∴According to the question,
109 = `2x P× 5× 4 P× 4× 4
x+ y = 13675
. + = 405
100 Total share = ` 50 × 3 = ` 150 100 100
100x + 109 y = 13675 …(ii) ∴ 4x + 2 x = 150 ⇒ 20P + 16P = 40500
150
Solving Eqs. (i) and (ii), we get or x = = 25 ∴ P=
40500
= 1125
6
x = 55 36
∴Cost of 1 bottle of orange juice ∴ Amount borrowed = ` 1125
y = 75
= 25 × 2 = ` 50
∴Cost of item Y before sales tax ` 75 40. Twelve (12) men work 8 h per day and
35. According to Mr. Sharma’s will, 37. Ten (10) years before, the ages of a require 10 days to build a wall. If
half of his property goes to his mother and her daughter were in 8 men are available, how many hours
wife and the rest in equally the ratio 3 : 1. In another 10 yr per day must they work to finish the
divided between his two sons, from now, the ratio of their ages work in 8 days?
Ravi and Raj. Some years later, will be 13 : 7. What are their (a) 10 h (b) 12 h (c) 15 h (d) 18 h
Ravi dies and leaves half of his present ages?
(a) 39 yr, 21 yr (b) 55 yr, 25 yr
_ (c) 12 men can do a work in 10 days if
property to his brother Raj. When they work 8 h per day.
Raj dies has leaves half of his (c) 75 yr, 25 yr (d) 49 yr, 31 yr ∴12 men can do the work in 80 h
property to his widow and 1 man will do the same work in (12 × 80) h
_ (b) Let the age of mother 10 yr ago
remaining to his mother, who is be 3x yr = 960 h
still alive. The mother now owns ∴Age of daughter 10 yr ago = x yr 960
And 8 men will do the same work = h
` 88,000 worth of the property. According to the question, 8
The total worth of the property of 3x + 20 13 Now, to complete the work in 8 days,
=  120 
Mr. Sharma was x + 20 7 each men would have to work for  h
 8 
(a) ` 1,00,000 (b) ` 1,24,000 ⇒ 21x + 140 = 13x + 260
each day.
(c) ` 1,28,000 (d) ` 1,32,000 ⇒ 8x = 120
120
∴Required time = h = 15 h
8
CDS Solved Paper 2018 (II) 7

41. A milk vendor bought 28 L of milk  5 Similarly, work done by X and Y in 1 day
∴Lose in 72 min =  72 − 65  min
 11 1
at the rate of ` 8.50 /L. After =
6 72
adding some water he sold the = 6 min = min 6
11 11 1 1
mixture at the same price. If his ∴1 day work of Y = −
 72 60 × 24  min 6 12
gain is 12.5%, how much water did ∴Lose in 24 h =  × 
 11 72  2 −1 1
he add? 1440 = =
= min 12 12
(a) 4.5 L (b) 4 L 11
10 ∴Y will complete the work in 12 days.
(c) 3.5 L (d) 3 L = 130 min
11 45. Twelve (12) persons can paint 10
_ (c) Cost price of 1 L of milk = ` 850 identical rooms in 16 days. In how
43. A thief steals a car parked in a
Selling price of l L of mixture = ` 8.50 many days can 8 persons paint 20 such
house and goes away with a speed
Profit percentage = 12.5% of 40 km/h. The theft was rooms?
∴Cost price of 1 L of mixture discovered after half an hour and (a) 12 (b) 24 (c) 36 (d) 48
100
= × 8.50 immediately the owner sets off in _ (d) 12 person can paint 10 identical
112.5 another car with a speed of rooms in 16 days
68
=` 60km/h. When will the owner ∴ Time taken by 12 persons to paint 20
9
meet the thief? such identical rooms = 16 × 2 = 32 days
By rule of alligation, And time taken by 1 person to paint 20
(a) 55 km from the owner’s house
Water Milk identical rooms = ( 32 × 12 ) days
and one hour after the theft
`0 ` 17 ∴ Time taken by 8 persons to paint 20
(b) 60 km from the owner’s house
2 32 × 12 
such rooms = 
Mean
and 1.5 h after the theft  days = 48 days
Price  8 
` 68 (c) 60 km from the owner’s house
` 17 9 ` 68 and 1.5 h after the discovery of 46. There are n zeros appearing
18 9 the theft immediately after the decimal point in
Ratio of milk and water in the mixture (d) 55 km from the owner’s house . ) 25 . It is given that the
the value of (02
17 68 and 1.5 h after the theft
= : = 1: 8 value of log10 2 = 030103
. . The value of
18 9 _ (b) Speed of thief = 40 km/h n is
Now, quantity of milk in mixture Speed of owner = 60 km/h
(a) 25 (b) 19
= 28 L Q Distance = Speed × Time (c) 18 (d) 17
28 ∴ Distance travelled by thief in half
∴Quantity of water in mixture = ×1 _ (c) Let x = ( 02
. )25
8 1
hour = 40 ×
= 3.5 L 2 Taking log both sides, we get
= 20 km log x = log( 0 .2 )25
42. The minute hand of a clock Now, relative speed of owner and 2 
overtakes the hour hand after ⇒ log x = 25 log  
thief = ( 60 − 40) km/h = 20 km/h  10 
every 72 min of correct time. How ∴Time taken by owner to catch thief ⇒ log x = 25(log10 2 − log10 10)
much time does the clock lose or 20
= = 1h ⇒ log x = 25( 0 . 3010 − 1)
gain in a day of normal time? 20
10 Distance covered by thief after the ft ⇒ log x = 7.525 − 25
(a) Lose 130 min
 1 ⇒ log x = 0.525 − 18
11 = 40 ×  1 + 
 2 ⇒ x = Antilog( 0.525) × 10 − 18
1
(b) Lose 157 min
11 = 40 ×
3
= 60 km ∴18 zero appearing immediately after the
9 2 decimal point.
(c) Gain 121 min
11 ∴ The owner will meet the thief at 47. The ratio of the sum and difference of
60 km from the owner’s hour and 1.5
1 the ages of the father and the son is
(d) Gain 157 min h after the theft.
11 11 : 3. Consider the following
44. X and Y together can finish a job in statements
_ (a) As we know that in a correct 6 days. X can alone do the same
clock, the min hand gains 55 min 1. The ratio of their ages is 8 : 5.
spaces over the hour hand in 60 min job in 12 days. How long will Y
alone take to do the same job? 2. The ratio of their ages after the son
To be together again, the minute
hand must gain 60 min over the hour
attains twice the present age will
(a) 16 days (b) 12 days
hand. be 11 : 8.
(c) 10 days (d) 8 days
Q 55 min are gained in Which of the statements given above
 60  5 _ (b) X can do the job in 12 days is/are correct?
 × 60 = 65 min 1
 55  11 ∴Work done by X in 1day = (a) 1 only (b) 2 only
12
But they are together after 72 min (c) Both 1 and 2 (d) Neither 1 nor 2
8 CDS Solved Paper 2018 (II)

_ (a) We have, x − y = 0
2 2
_ (b) From statement 1 51. What percentage of students passed in
Ratio of ages of father and son and ( x − a )2 + y 2 = 1 all the three subjects?
= 8:5 ⇒ x 2 − y2 = 0 ⇒ x 2 = y2 (a) 6 (b) 5 (c) 4 (d) 3
Let father’s age = 8x Put the value y 2 in, we get _ (c) Given =
and son’s age = 5x ( x − a )2 + y 2 = 1 n( B) = 55%
∴According to the question, the ratio ⇒ ( x − a )2 + x 2 = 1 n( P ) = 62%
of the sum and difference of the ages
⇒ 2 x 2 − 2 ax + a 2 − 1 = 0 n(C ) = 60%
of father and son is 11 : 3.
8x + 5x 11 Equation have single positive n( P ∩ B) = 25%
∴ = solution
8x − 5x 3 n( P ∩ C ) = 30%
11 13 11 ∴ D=0 n( B ∩ C ) = 28%
But it is not equal to as ≠
3 3 3 ⇒ 4a 2 − 4( a 2 − 1) (2 ) = 0 n( P ∪ B ∪ C )' = 2%
∴Statement 1 is not true. ⇒ 4a 2 − 8a 2 + 8 = 0 ∴n( P ∪ B ∪ C ) = 100% − n( P ∪ B ∪ C )'
From statement 2 ⇒ a2 = 2 = 100% − 2% = 98%
Ratio of the age of father and son ⇒ a= 2 n( P ∪ B ∪ C ) = n( B) + n( P )
after the son attains twice the
present age will be 11 : 8
50. If α, β and γ are the zeros of the + n(C ) − n( P ∩ B)
polynomial − n( P ∩ C ) − n( B ∩ C ) − n( P ∩ B ∩ C )
Let father’s age be F and son’s age
be S f ( x ) = ax 2 + bx 2 + cx + d , then ⇒ 98 = 55 + 62 + 60 − 25 − 28 − 30
F + S 11 α 2 + β 2 + γ 2 is equal to + n( P ∩ B ∩ C )
∴ =
S+S 8 ⇒ n( P ∩ B ∩ C ) = 4%
b 2 − ac b 2 − 2ac
⇒ 8F + 8S = 22S (a) (b) Q Percentage of student passed in all
2
F 7 a a three subjects is 4%
⇒ 8F = 14S or =
S 4 b 2 + 2ac b 2 − 2ac
(c) 2
(d) 52. What percentage of students passed in
Let father’s age be 7 x and son’s age b a2 exactly one subject?
be 4x
_ (d) We have, (a) 21 (b) 23 (c) 25 (d) 27
∴According to the question,
f( x ) = ax 3 + bx 2 + cx + d
7 x + 4x 11x 11
= =
_ (b) percentage of students passed in
α, β , γ are the zero of the given exactly one subject
7 x − 4x 3x 3
polynomials
∴Statement 2 is correct = n( B) + n( P ) + n(C ) − 2{ n( P ∩ B)
b
∴ α +β+ γ =− + n( B ∩ C ) + n( P ∩ C )}
48. The solution of linear inequalities a + 3n( P ∩ B ∩ C )
x + y ≥ 5 and x − y ≤ 3 lies αβ + βγ + γα =
c
= 55 + 62 + 60 − 2(25 + 28 + 30) + 3( 4)
a
(a) Only in the first quadrant = 177 − 166 + 12 = 23%
d
(b) In the first and second quadrants αβγ = −
a 53. If the number of students in 360, then
(c) In the second and third quarants (α + β + γ )2 = α 2 + β 2 + γ 2 + 2 how many passed in atleast two
(d) In the third and fourth quadrants (αβ + βγ + γα ) subjects?
2
_ (b) We have,  b 2c (a) 270 (b) 263 (c) 265 (d) 260
⇒ −  = α + β + γ +
2 2 2

x + y ≥ 5 and x − y ≤ 3  a a _ (a) Percentage of student passed in


The graph of linear equalities are ⇒α2 + β2 + γ 2 exactly two subject
below b 2
2c = n( P ∪ B) + n( P ∩ C ) + n( B ∩ C )
= 2
− − 3n( P ∩ B ∩ C )
a a
(0, 5) x–y=3 b − 2 ac
2 = 25 + 28 + 30 − 12 = 71%
=
a2 % age of passed atleast two subjects
Directions (Q. Nos. 51-54) = 71% + 4% = 75%
(3, 0) 5, 0 ∴ Number of students passed
0, –3 Consider the following for the next 04
x+y=5 75
(four) items that follow : = × 360 = 270
100
In an examination of class XII, 55%
Clearly, from graph solution lies in 54. What is the ratio of number of
students passed in Biology, 62%
first and second quadrant
passed in Physics, 60% passed in students who passed in both Physics
49. It is given that equations Chemistry, 25% passed in Physics and Chemistry to number of students
x 2 − y 2 = 0 and ( x − a ) 2 + y 2 = 1 and Biology, 30% passed in Physics who passed in both Biology and
have single positive solution. For and Chemistry, 28% passed in Physics but not Chemistry?
this, the value of ‘a’ is Biology and Chemistry. Only 2% (a) 7 : 10 (b) 10 : 7
(a) 2 (b) 2 (c) − 2 (d) 1 failed in all the subjects. (c) 9 : 7 (d) 7 : 9
CDS Solved Paper 2018 (II) 9

_ (b) Percentage of student passed in 59. Let x 1 and x 2 (where x 2 > x 1 ) be 1 π


both Physics and Chemistry = 30%
61. If cos θ = , where 0 < θ < , then
the means of two sets comprising 5 2
Percentage of student passed in
n 1 and n 2 (where n 2 < n 1 ) 2 tan θ
both Biology and Physics but not is equal to
Chemistry observations respectively. If x is 1 − tan 2 θ
= (25 − 4)% = 21% the mean when they are pooled, (a) 4 / 3 (b) −4 / 3 (c) 1 / 3 (d) −2 / 3
30% then which one of the following is
∴Ratio = = 10 : 7
correct? _ (b) We have,
21% 1
(a) x1 < x < x 2 cosθ =
55. Data on ratings of hotels in a city 5
(b) x > x 2
is measured on ⇒ tanθ = sec 2 θ − 1 = 5−1=2
(a) Nominal scale (b) Ordinal scale (c) x < x1 2 tanθ 2(2 ) 4 −4
∴ = = =
(c) Interval scale (d) Ratio scale (d) (x1 − x ) + (x 2 − x ) = 0 1 − tan2 θ 1 − (2 )2 1− 4 3
x 1n1 + x 2n2
_ (b) Data on rating of hotels in a city is _ (a) x = 62. If 0 < θ < 90º, 0 < φ < 90º and
measured on ordinal scale. n1 + n2
cos θ < cos φ, then which one of the
56. The average marks of section A ⇒ n1x + n2x = n1x 1 + n2x 2 following is correct ?
are 65 and that of section B are 70. ⇒ n1( x − x 1 ) = n2( x 2 − x ) (a) θ < φ
If the average marks of both the x2 − x
sections combined are 67, then the ⇒
n
= 1 (b) θ > φ
x − x1 n2
ratio of number of students of (c) θ + φ = 90º
x2 − x
section A to that of section B is ⇒ > 1 ( n1 > n2 ) (d) No conclusion can be drawn
x − x1
(a) 3 : 2 (b) 1 : 3 (c) 3 : 1 (d) 2 : 3
_ (b) 0 < θ < 90°, 0 < φ < 90°
x 2 − x > x − x1
_ (a) Let the number of students in
Q
and cos θ < cos φ
section A = x ∴ x 2 > x > x1
θ > φ [cos x is decreasing function in
and the number of students in
60. Consider the following  π
Section B = y  0, 2 ]
∴Total number of student = x + y
statements:
∴65x + 70 y = 67 ( x + y) Statements I Median can be 63. On the top of a hemispherical dome of
⇒ 2x = 3y
computed even when the end radius r, there stands a flag of height h.
intervals of a frequency From a point on the ground, the
⇒ x : y :: 3 : 2
distribution are open elevation of the top of the flag is 30º.
57. The median of 19 observations is Statements II Median is a After moving a distance d towards the
30. Two more observations are positional average. dome, when the flag is just visible, the
made and the values of these are 8 elevation is 45º. The ratio of h to r is
Which one of the following is
and 32. What is the median of the equal to
correct in respect of the above
21 observations? 3+1
statements? (a) 2 − 1 (b)
(a) 32 (b) 30 (c) 20 2 2
(d) Cannot be determined due to (a) Both Statement I and Statement
II are true and Statement II is 3+1 ( 3 + 1) ( 2 − 1)
insufficient data (c) d (d) d
the correct explanation of 2 2 2 2
_ (b) The median of the 21 observation Statement I
is no charge because 8 is less than 30 _ (a) According to given information, we
and 32 is greater than 30 (b) Both Statement I and Statement have the following figure.
II are true and Statement II is not In ∆BDE , we have tan 45° =
DE
58. As the number of observations the correct explanation of BD
and classes increases, the shape of Statement I ⇒ BD = DE = h + r
a frequency polygon
(c) Statement I is true but Statement ⇒ BC + r = h + r
(a) Tends to become jagged E
II is false
(b) Tends to become increasingly h
smooth (d) Statement I is false but
Statement II is true F r
(c) Stays the same
(d) Varies only if data become more _ (a) We know that, median always r
reliable lies in the center of the distribution
30º 45º
therefore it is called a
B C D
_ (b) We know that, when the number positional average and so we
of observations and classes can compute it even when the end
increases then the shape of a intervals of a frequency distribution ⇒ BC = h
frequency polygon tends to become are open. Now, as ∠EBD = 45°, therefore
increasingly smooth.
∠BED = 45°
10 CDS Solved Paper 2018 (II)

Also, In ∆BFD ⇒ cos θ =


1
= cos 60° 67. Consider the following statements
2
1. cos θ + sec θ can never be equal to
FD
tan 45° =
FB ⇒ θ = 60°
1.5
⇒ FB = FD = r ∴Statement 1 is correct
Similarly, EF = r 2. 3 tanθ + cot θ = 5 cosec θ
2. sec 2 θ + cosec 2 θ can never be less
than 4.
Now, consider ∆BDE, then we have put θ = 60°
Which of the statements given above
( BE )2 = ( BD )2 + ( DE )2 Q 3 tan 60° + cot 60° = 5 cosec 60°
is / are correct?
⇒ (2 r )2 = ( h + r )2 + ( h + r )2 3 3+
1
=5×
2
3 3 (a) 1 only (b) 2 only
⇒ 4r 2 = 2( h + r )2
⇒ 2 r = (h + r )
2 2 9 + 1 10 (c) Both 1 and 2 (d) Neither 1 nor 2
⇒ =
⇒ h + r = 2r 3 3
_ (c) cos θ + sec θ
10 10
⇒ h = ( 2 − 1)r ⇒ = ⇒ cos θ +
1
h 3 3 cos θ
⇒ = 2 −1
∴Statement 2 is also correct 1
r ⇒ cos θ + ≥2
66. Consider the following cos θ
3
64. Let sin ( A + B ) = and statements ∴Statement 1 is correct
2
sec 2 θ + cosec 2θ
3 p +q2 2
cos B = , where A, B are acute 1. cos θ = 1 −
2
, where p, =
1
+
1
2 2pq cos 2 θ sin2 θ
angles. What is tan (2A − B ) equal q are non-zero real numbers, is 1
=
to ? possible only when p = q . sin2 θ ⋅ cos 2 θ
1 4 pq 4 4
(a) 1 / 2 (b) 3 (c) (d) 1
2. tan 2 θ = − 1 , where = =
3 (2 sinθ cos θ)2 sin2 2θ
(p + q )2
_ (c) We have, p , q are non-zero real ∴
4
≥4
3 sin2 2θ
sin( A + B) = numbers, is possible only
2 Statement 2 is also correct.
when p = q .
⇒ A + B = 60° … (i) 68. If sin 2 x + sin x = 1, then what is the
Which of the statements given
and cos B =
3 above is/are correct ? value of cos 12 x + 3 cos 10 x +
2
(a) 1 only 3 cos 8 x + cos 6 x?
⇒ B = 30° …
(b) 2 only (a) –1 (b) 0 (c) 1 (d) 8
(ii)
(c) Both 1 and 2 _ (c) We have,
Solving Eqs. (i) and (ii), we get
(d) Neither 1 nor 2 sin2 x + sin x = 1
A = 30°
p2 + q 2 ⇒ sin x = 1 − sin2 x = cos 2 x
∴tan(2 A − B) = tan( 60 − 30° ) _ (c) cos θ = 1 −
2

1 2 pq ⇒ sin2 x = cos 4 x
= tan 30° =
3 0 ≤ cos θ ≤ 1 2
⇒ 1 − cos 2 x = cos 4 x
p ×q2 2
⇒ cos x + cos 2 x = 1
4
65. Consider the following statements θ≤ 1− ≤1
2 pq cubing both sides,
cos θ cos θ
1. If + = 4, where 0 ≤ 2 pq − ( p2 + q 2 ) ≤ 2 pq (cos 4 x + cos 2 x )3 = (1)3
1 − sin θ 1 + sin θ
p2 + q 2 − 2 pq ≤ 0 ⇒ cos12 x + 3 cos10 x
0 < θ < 90º, then θ = 60º.
⇒ ( p − q )2 ≤ 0 + 3 cos 8 x + cos 6 x = 1
2. If 3 tan θ + cot θ = 5 cos ec θ, It is possible only when p = q
where 0 < θ < 90º, then θ = 60º. 69. If 3 sin θ + 5 cos θ = 4, then what is the
∴Statement 1 is correct
Which of the statements given value of (3 cos θ − 5 sin θ) 2 ?
4 pq
above is/are correct? tan2 θ = −1
( p + q )2 (a) 9 (b) 12
(a) 1 only (b) 2 only (c) 16 (d) 18
⇒ tan θ ≥ 0
2

(c) Both 1 and 2 (d) Neither 1 nor 2 4 pq _ (d) We have,


⇒ − 1≥ 0
( p + q )2 3 sinθ + 5 cos θ = 4
_ (c) We have,
cos θ cos θ ⇒ 4 pq − ( p + q ) ≥ 0 2 squaring both sides,
1. + =4
1 − sinθ 1 + sinθ 9 sin2 θ + 25 cos 2 θ + 30
⇒ ( p + q ) − 4 pq ≤ 0
2

cos θ (1 + sinθ + 1 − sinθ) sinθ cos θ = 16


⇒ =4 ⇒ p2 + q 2 + 2 pq − 4 pq ≤ 0
1 − sin θ 2 ⇒ 9 − 9 cos 2 θ + 25 − 25 sin2 θ
⇒ (p − q) ≤ 0 2

2 cos θ + 30 sinθ cos θ


⇒ =4 It is possible only p = q
cos θ
2 = 16
∴Statement 2 is correct
CDS Solved Paper 2018 (II) 11

⇒ 9 cos 2 θ + 25 sin2 θ − 30 Now, area of ∆ABC=


1
× base (a) 10 (b) 12 (c) 13 (d) 14
sinθ cos θ = 9 + 25 − 16 2
× height _ (b) Length, breadth and height of walls
⇒ ( 3 cos θ − 5 sinθ)2 = 18 1
are 6m, 4m and 2.5 m respectively
= × (u 2 − v 2 ) × (2uv ) ∴Area of 4 walls
70. If cot θ (1 + sin θ) = 4m and 2
= uv(u 2 − v 2 ) = 2 h ( l + b ) = 2 × 2.5( 6 + 4) = 50 m 2
cot θ (1 − sin θ) = 4n, then which
⇒ uv(u + v ) (u − v ) = 2016 Area of 4 walls of 5 rooms
one of the following is correct?
⇒ 9 × 7 × 16 × 2 = 2016 = 5 × 50 = 250 m 2
(a) (m 2 + n 2 )2 = mn Area of windows = (2.5)2 = 625
. m2
∴ u = 9 and v = 7
(b) (m 2 − n 2 )2 = mn Area of two windows
∴Perimeter of triangle
(c) (m 2 − n 2 )2 = m 2n 2 = 2 × 625
.
= (u + v ) + (u − v ) + 2uv
2 2 2 2
(d) (m 2 + n 2 )2 = m 2n 2 = 13.50 m 2
= 2u 2 + 2uv = 2u(u + v )
Total area required for painting
_ (b) We have,
= 2 × 9 × ( 9 + 7 ) = 288 unit = 250 − 13.50
cot θ(1 + sinθ) = 4m … (i)
cot θ(1 − sinθ) = 4n … (ii) 72. A circle is inscribed in an = 236.50
equilateral triangle of side of Now, one cans paints = 20 m 2
16mn = cot 2 θ(1 − sin2 θ)
length. The area of any square ∴Number of cans required for painting
16mn = cot θ cos θ
2 2
…(iii)
inscribed in the circle is =
236.50
= 11825
. = 12 cans
Now, add eqs. (i) and (ii), we get
l2 3l 2 l2 l2 20
(a) (b) (c) (d)
cot θ = 2( m + n) 2 4 4 6 74. Let S be the parallelogram obtained by
Subtracting eqs. (ii) from (i), we get
_ (d) Side of equilateral ∆ABC = l joining the mid-points of the
cot θ sinθ = 2( m − n) parallelogram T . Consider the
Radius of circle inscribed in ∆ABC
⇒ cos θ = 2( m − n) Area of ∆ABC following statements :
r=
Putting the value of cot θ and cosθ in Semi - perimeter of ∆ABC 1. The ratio of area of T to that S is
eq. (iii), we get
3 2 2:1
16mn = (2 2( m − n)2 )(2 2( m + n)2 ) l
⇒r = 4 2. The perimeter of S is half of the
= 16( m2 − n2 )2 l
3 sum of diagonals of T .
⇒ mn2( m2 − n2 )2 2
A
Which of the above statements is/are
71. If base and hypotenuse of a right correct?
triangle are (u − v ) and 2 2
(a) 1 only (b) 2 only
(u 2 + v 2 ) respectively and the (c) Both 1 and 2 (d) Neither 1 nor 2
area of the triangle is 2016 square S R
2r _ (a) Given,
units, then the perimeter of the Area of parallelogram ABCD = T
triangle may be P Q Area of parallelogram PQRS = S
(a) 224 units (b) 288 units 1
B C Area of ∆PQS = × area of||gm ABQS
(c) 448 units (d) 576 units l 2
⇒r =
1
_ (b) According to the question, 2 3 Area of ∆RSQ = × area of ||gm SQCD
2
In right angle ∆ABC Area of square PQRS
D R C
C PR 2 4r 2
= PQ 2 = = = 2r2
2 2
2
 l  2 l2 l2
u2+v2 =2   = = S Q
2 3 4×3 6

73. Walls (excluding roofs and floors)


A B
of 5 identical rooms having A P B
u2−v2 length, breadth and height 6 m, ∴Area of parallelogram PQRS =
1
Here, 4 m and 2.5 m respectively are to 2
AC 2 = AB2 + BC 2 be painted. Out of five rooms, two (area of parallelogram ABCD )
⇒ BC = AC − AB
rooms have one square window 1
2 2 2
S = T ⇒ S : T = 1: 2
each having a side of 2.5 m. Paints 2
⇒ BC = AC 2 − AB2 are available only in cans of 1 L; ⇒ T :S = 2 :1
and 1 L of paint can be used for ∴Statement 1 is correct
= (u 2 + v 2 )2 − (u 2 − v 2 )2
painting 20 m 2 . The number of 2. Perimeter of parallelogram PQRS
= 4 u 2v 2 = 2uv cans required for painting is = PQ + RQ + RS + PS
12 CDS Solved Paper 2018 (II)

1 462 × 7
PQ = AC = RS ⇒ r= = 4.2 cm 81. If the diagonal of a cube is of length l,
2 22 × 35
then the total surface area of the cube is
1
RQ = PS = BD (a) 3l 2 (b) 3l 2 (c) 2l 2 (d) 2l 2
2 78. A semi-circular plate is rolled up
∴PQ + RQ + RS + PS to form a conical surface. The _ (d) Diagonal of cube = 3 × sides of
= ( AC + BD ) angle between the generator and cube
Perimeter of S is equal to sum of
the axis of the cone is ∴ l = 3 × side of cube
diagonals T (a) 60º (b) 45º l
Side of cube =
∴Statement 2 is incorrect (c) 30º (d) 15º 3

75. The sides of a triangle are 5 cm, _ (c) Clearly, the radius of semi-circle Surface area of cube = 6(side)2
= slant height of cone = r  l  6l 2
2
6 cm and 7 cm. The area of the =6×  = = 2 l2
triangle is approximately And the circumference of  3 3
semi-circle = circumference of the
(a) 14.9 cm 2 (b) 14.7 cm 2 base of cone 82. An equilateral triangle, a square and a
2
(c) 14.5 cm (d) 14.3 cm 2 circle have equal perimeter. If T , S and
O C denote the area of the triangle, area
_ (b) The sides of triangles are 5cm , r r
6 cm and 7cm respectively θ
of the square and area of the circle
Area of ∆ =
s( s − a ) ( s − b ) ( s − c ) r A r B respectively, then which one of the
a+ b+c following is correct?
Where s = ⇒ πr = 2 πR ⇒ R =
r
2 2 (a)T < S < C (b) S < T < C
5+ 6+ 7
∴ s= =9 Now, In ∆AOB, we have
2 (c) C < S < T (d)T < C < S
r
∆ = 9( 9 − 5) ( 9 − 6) ( 9 − 7 ) 2 1 _ (a) An equilateral triangle, a square and a
sinθ = = ⇒ θ = 30º circle of perimeter are equal
∆ = 9× 4× 3×2 r 2
∆ =6 6 Let side of triangle = l cm
79. A solid right cylinder is of height Side of square = x cm
= 6 × 2.44 = 14.69 = 147
. π cm. If its lateral surface area is
Radius of circle = r cm
76. There is a path of width 5 m half its total surface area, then the
∴3l = 4x = 2 π r = k
around a circular plot of land radius of its base is
k
whose area is 144π m 2 . The total (a) π / 2 cm (b)π cm ⇒ l = k / 3, x = k / 4, r =

area of the circular plot including (c) 1/ π cm (d) 2 / π cm
the path surrounding it is Area of equilateral ∆
_ (b) Height of cylinder = π cm
(a) 349π m 2
(b) 289π m 2
3 2 3  k2  k2
Lateral surface area =
1
total surface = l =  =
4 4  9  12 3
(c) 209π m 2 (d) 149π m 2 2
area
k2
_ (b) Area of circular 1 Area of square = x 2 =
∴ 2 πrh = (2 πr 2 + 2 πrh) 16
Plot of land = 144 π m 2 2
Let r = radius of circular plot 1 k2 k2
⇒ 2 πrh = 2 πr( r + h) Area of circle = πr 2 = π =
∴ 144 π = πr 2
2 4π2 4π
⇒ r 2 = 144 ⇒ 2h = r + h k2 k2 k2
> >
⇒ r = 12 ⇒ r=h 4 π 16 12 3
Area of circular plot including the ∴ r = π cm C>S>T
path ⇒ T<S<C
80. A rectangular block of length
= π( r + 5)2 = π(12 + 5)2
20 cm, breadth 15 cm and height 83. The areas of two similar triangles are
= π(17 )2 = 289 π m 2
10 cm is cut up into exact number (7 − 4 3 ) cm 2 and (7 + 4 3 ) cm 2
77. The lateral surface area of a cone is of equal cubes. The least possible respectively. The ratio of their
462 cm 2 . Its slant height is 35 cm. number of cubes will be corresponding sides is
The radius of the base of the cone (a) 12 (b) 16
(a) 7 − 4 3 (b) 7 − 3 3
is (c) 20 (d) 24
(c) 5 − 3 (d) 5 + 3
(a) 8.4 cm (b) 6.5 cm _ (d) Volume of rectangular block
(c) 4.2 cm (d) 3.2 cm = 20 × 15 × 10 = 3000 cm 3 _ (a) Ratio of area of two similar triangle =
Ratio of square of their corresponding
Let n is the number of cube formed
_ (c) Lateral surface area of cone = πrl sides
∴462 = πr( 35) (Q l = 35) Side of cube is 5 cm possible
20 × 15 × 10 7−4 3
462 ∴n = = 24 ∴ = ratio of their
⇒ r= 5× 5× 5 7 + 4 3
π × 35
corresponding sides = 7 − 4 3
CDS Solved Paper 2018 (II) 13

84. The chord of a circle is 3 times its 86. Two cones have their heights in ⇒ 2( x 2 + y 2 ) − 2 x ( x 1 + x 2 )
radius. The angle subtended by the ratio 1 : 3. If the radii of their − 2 y( y1 + y2 ) + x 12 + x 22 + y12 + y22
this chord at the minor arc is k bases are in the ratio 3 : 1, then the = constant
times the angle subtended at the ratio of their volumes will be Which represents the locus circle.
major arc. What is the value of k ? (a) 1 : 1 (b) 2 : 1 (c) 3 : 1 (d) 9 : 1 89. If ABC is a right-angled triangle with
(a) 5 (b) 2 _ (c) Let the height of cones are h and AC as its hypotenuse, then which one
(c) 1/2 (d) 1/5 3h and their radii of bases are 3r and r of the following is correct ?
_ (c) Given, AB = 3r 1
∴Volumes of cones are π( 3r )2( h) (a) AC 3 < AB 3 + BC 3
3
OA 2 + OB2 − AB2 1 (b) AC 3 > AB 3 + BC 3
Now, cosθ1 = and π( r 2 ) ( 3h)
2OA ⋅ OB 3 (c) AC 3 ≤ AB 3 + BC 3
r 2 + r 2 − 3r 2 ∴Ratio of volumes will be
cosθ1 = (d) AC 3 ≥ AB 3 + BC 3
2r2 1
π 9 r 2h
1 3 _ (b) Here, ABC is a right angle triangle
cosθ1 = − = 3 = = 3:1
2 1 / 3 πr 2 3h 1 with AC as its hypotenuse
C
A B 87. If two lines AB and CD intersect at
r r
O such that ∠AOC = 5 ∠AOD,
θ1 then the four angles at O are
O (a) 40º, 40º, 140º, 140º
θ2 (b) 30º, 30º, 150º, 150º
(c) 30º, 45 º, 75 º, 210º A B
(d) 60º, 60º, 120º, 120º
θ1 = 120° ⇒ θ1 = kθ 2 Let AC = 5
120
_ (b) We have, AB = 3
⇒ k= A C BC = 4
360 − 120
∴ AC 3 = ( 5)3 = 125
120 1 AB3 = ( 3)3 = 27
= =
240 2
O BC 3 = ( 4)3 = 64
85. In a triangle ABC, the sides AB, Now, AC 3 > AB3 + BC 3
D B
AC are produced and the bisectors
∠AOC = 5∠AOD 90. The area of the region bounded
of exterior angles of ∠ABC and
∴ ∠AOD + ∠AOC = 180° externally by a square of side 2a cm
∠ACB intersect at D. If
⇒ 5∠AOD + ∠AOD = 180° and internally by the circle touching
∠BAC = 50º, then∠BDC is equal
⇒ 6∠AOD = 180°
the four sides of the square is
to
⇒ ∠AOD = 30° (a) (4 − π)a 2 (b) (π − 2)a 2
(a) 115º
(b) 65 ∴Other four angles are (8 − π)a 2
(π − 2)a 2
(c) (d)
(c) 55º 30°, 30°, 150°, 150° 2 2
(d) 40º
88. If a point P moves such that the _ (a) Side of square ABCD is 2a cm
_ (b) ∠BDC = 180° − (u + v ) sum of the squares of its distances ∴Radius of circle = a cm
A
from two fixed points A and B is a Area of shaded region
50º constant, then the locus of the = area of square – area of circle
point P is = (2 a )2 − πa 2 = 4a 2 − πa 2 = ( 4 − π )a 2
x y (a) A straight line 91. In the figure given below, ABC is a
B C
u v (b) A circle right-angled triangle where ∠A = 90º,
(c) Perpendicular bisector of AB AB = p cm and AC = q cm. On the
(d) An arbitrary curve three sides as diameters semi-circles
D are drawn as shown in the figure. The
_ (b) Given, PA + PB = constant
2 2
 50 + y 50 + x 
= 180° −  +  area of the shaded portion, in square
 2 2  P (x, y)
cm, is
 x + y
= 180° −  50 +  A
 2 
 50 
= 180° −  + 90°  A (x1, y1) B (x2, y2)
 2 
= 180° − (25 + 90) ∴( x − x 1 ) + ( y − y1 )2 + ( x − x 2 )2
2

= 180° − 115° = 65° + ( y − y2 )2 = constant B C


14 CDS Solved Paper 2018 (II)

π(p 2 + q 2 ) ∴∠BCD = 180° − ∠BAD


(a) pq (b)
2 = 180° − 100°
pq = 80°
(c) π(p + q )
2 2
(d)
2 A D
B C 95. In the figure given below, AB is the
_ (d) diameter of the circle whose centre is
A at O. Given that ∠ECD = ∠EDC = 32º,
then ∠CEF and ∠COF respectively
q
p (a) 9π (b) 27 π (c) 36π (d) 81π are
_ (b) Area of shaded regions = F
B p2+q2 C C
Area of semi-circle on AD + area of
Area of shaded region = semicircle on AB – area of
semi-circle on BD
area of semi-circle on AB + area of B
A
semi-circle on AC + area of ∆ABC − O E
area of semi-circle on BC
π  p2  π q2 1 D
=   +   + pq 6
2 4 2  4 2 A B 6 C 6 D
(a) 32º, 64º (b) 64 º, 64 º
π  p2 + q 2 
−   (c) 32º, 32º (d) 64º, 32º
2  4 
1 _ (b) Given, ∠ECD = ∠EDC = 32 º
= pq π π π ∠CEF = ∠ECD + ∠EDC
2 = ( 9)2 + ( 3)2 − ( 6)2
2 2 2 (exterior angle of ∆ is sum of interior
92. In the figure given below, the π opposite angle)
= ( 92 + 32 − 62 )
radius of the circle is 6 cm and 2 ∴∠CEF = 32 º + 32 º = 64º
AT = 4 cm. The length of tangent π ∠COF = 2 ∠CDF = 2( 32 º ) = 64º
= ( 81 + 9 − 36)
PT is 2
π 96. In the figure given below,
P = ( 54) = 27 π
2 ∆ABR ~ ∆PQR. If PQ = 3 cm, AB = 6
cm, BR = 82
. cm and PR = 52
. cm, then
94. In the figure given below, what is QR and AR are respectively
T
O A ∠BCD equal to? P
D
B Q
(a) 6 cm R

(b) 8 cm
(c) 9 cm C
A 30º A
(d) 10 cm 70º (a) 8.2 cm, 10.4 cm
_ (b) Here, OP = 6 B (b) 4.1 cm, 6 cm
OT = OA + AT = 10 (a) 70º (b) 75º (c) 80º (d) 90º (c) 2.6 cm, 5.2 cm
∴ PT = OT − OP _ (c) ∠CAD = ∠CBD
2 2 2 (d) 4.1 cm, 10.4 cm
P
D _ (d) Given,
6 ∆ABR ~ ∆PQR
T P
O 6 A 4
B Q
C R
A 30º
⇒ PT = (10)2 − ( 6)2 70º

⇒ 100 − 36 = 64 = 8 B A
∠CAD = ∠CBD
93. In the figure given below, ABCD is (angle on same segment) ∴
AB
=
AR
=
BR
the diameter of a circle of radius PQ PR QR
∴∠CAD = 70° 6 82
.
9 cm. The lengths AB, BC and CD =
AR
=
∴∠BAD = ∠BAC + ∠CAD 3 52
. QR
are equal. Semi-circles are drawn
on AB and BD as diameters as = 30° + 70° = 100° Solving and, we get
shown in the figure. What is the ∠BAD + ∠BCD = 180° QR = 41
. cm and AR = 10.4 cm
area of the shaded region? ( ABCD is cycle quadilateral )
CDS Solved Paper 2018 (II) 15

97. In the figure given below ABC is a r ∴ AXY is also equilateral triangle
triangle with AB perpendicular to C Similarly, XBP and CYQ is an equilateral
r r triangle
BC. Further BD is perpendicular to
AC. If AD = 9 cm and DC = 4 cm, 27 27 ∴ XY = AX , XP = BX ,
then what is the length of BD? 27 QY = CY = CQ
A P O Q B
B XY + XP + CY = 40
In ∆OPC, PC 2 = OP 2 + OC 2 AX + BX + CY = 40
(27 + r )2 = 27 2 + ( 54 − r )2 AB + CQ = 40
⇒ (27 )2 + 54r + r 2 ⇒ CQ = 40 − 30 = 10 = BP
= 27 2 + ( 54)2 − 108r + r 2 PQ = BC − ( BP + CQ )
C 4 D 9 A ⇒ r = 18 = 30 − 20 = 10
(a) 13/36 cm (b) 36/13 cm ∴Area of shaded region = 100. In the figure given below, ABCD is a
(c) 13/2 cm (d) 6 cm Area of bigger semi-circle – 2(area of square of side 4 cm. Quadrants of a
smaller semi-circle) – area of
circle of diameter 2 cm are removed
_ (d) In ∆ABD, smallest circle
BD 2 = AB2 − AD 2 π
form the four corners and a circle of
…(i) π 
= ( 54)2 − 2  (27 )2  − π(18)2 diameter 2 cm is also removed. What
In ∆BCD, 2 2 
is the area of the shaded region?
BD 2 = BC 2 − CD 2 …(ii) π
= [54 2 − 2(27 )2 − 2(18)2 ]
Adding Eqs. (i) and (ii), we get 2 D C
π
2 BD = ( AB + BC ) − ( AD + CD )
2 2 2 2 2 = [2916 − 1458 − 648]
2
= ( AC 2 ) − ( AD 2 + CD 2 ) π
= × 810 = 405 π cm2
= (13)2 − ( 4 2 + 9 2 ) 2
2 BD = 169 − 97 = 72
2
99. In the figure given below, ABC is
72 an equilateral triangle with each
BD = =6
2 side of length 30 cm. XY is parallel
to BC, XP is parallel to AC andYQ A B
98. In the figure given below, the is parallel to AB. If XY + XP + YQ is 7 7
diameter of bigger semi-circle is (a) 5 cm 2 (b) 7 cm 2
40 cm, then the value of PQ is 9 9
108 cm. What is the area of the 5 5
shaded region? A (c) 9 cm 2 (d) 9 cm 2
7 6
_ (c) Area of shaded region =
= Area of square – 4 (area of quadrant)–
area of circle
π 
X Y = ( 4)2 − 4 (1)2  − π(1)2
4 
54 cm 54 cm = 42 − 2 π
(a) 201π cm 2 22
B P Q C = 16 − 2 ×
7
(b) 186.3 π cm 2 (a) 5 cm (b) 12 cm
44
= 16 −
(c) 405 π cm 2 (c) 15 cm (d) 10 cm 7
. π cm 2
(d) 7695 _ (d) Given, 112 − 44
=
_ (c) Given, AO = OB = 54 cm AB = BC = AC = 30 7
68 5
∴ OP = OQ = 27 cm XY is parallel to BC = = 9 cm2
7 7
PAPER II English
Directions (Q. Nos. 1-10) Each of the 7. ……… sincere, he would have got the 13. He felt desolated after he lost his
following sentence in this section has a prize. business.
blank space and four words or group of (a) Had he been (a) deserted (b) joyful
words given after the sentences. Select (b) Has he been (c) strong (d) annoyed
the word or group of words you consider (c) Would he have been 3 (a) ‘deserted’ is closest in meaning to word
most appropriate for the blank space. (d) He is desolated. The word means to leave someone in
and indicate your response on the 3 (a) From the given option ‘had he been’ is a situation when they have no one to support.
answer sheet accordingly. most suitable. The sentence is ‘conditional’ type, The word desolated also means to feel
here, use of past perfect is required. miserable or gloomy.
1. My teacher was ………………… us for
being late. 8. Ten years ……… for me to live in a 14. Don’t condone such acts which lead to
(a) annoyed at (b) annoyed with foreign country. unrest in the country.
(c) annoyed about (d) annoys (a) are a long time (a) regard (b) punish
(b) is a long time (c) aware of (d) overlook
3 (b) ‘annoyed with’ is the most appropriate
option as when we are angry with people we (c) has a long time 3 (d) ‘overlook’ is closest in meaning to the
use ‘annoyed with’. (d) of time word ‘condone’. The word condone means to
accept or allow behaviour that is wrong.
2. Sandhya ……… me from the top of the 3 (b) Ten years is singular noun, so we will use
house. ‘is a long time’ to fill in the blank. 15. A good work place shall not encourage
(a) shouted to (b) shouted at 9. If I ……… you, I ……… love to accept ineptitude even in a hidden manner.
(c) shouted on (d) shouted the offer. (a) incompetence (b) courage
3 (b) With verb ‘shouted’, preposition to be (a) was …… will (c) gossip (d) radical thinking
used is ‘at’. So option ‘b’ ‘shouted at’ is correct (b) was …… would 3 (a) ‘incompetence’ is closest in meaning to
here as we shout at some or the other person. (c) were …… would the word ineptitude. The word means lack of skill
When we ‘shout to’ some person, we want to be or ability and incompetence also means the
(d) were …… will
heard. Here, option (b) should be used. same.
3 (c) Use of were and would is required in this
3. Ravi has the habit of …………………… a conditional type of sentence. 16. Learning of foreign language should
headache. not impede one’s mother tongue
(a) complaining (b) complain 10. My sister asked me ……… willing to go learning.
(c) complaining to (d) complaining of abroad for my studies. (a) facilitate (b) acts for
(a) if I were (c) hinder (d) accept
3 (d) ‘complaining of’ is the most suitable (b) if I could be
option. When we speak about illnesses, we use 3 (c) ‘hinder’ is the most suitable meaning of
(c) whether I should be
complaining of. the word ‘impede’. Both words mean to delay or
(d) whether I will prevent someone by obstructing them.
4. I always want to go alone for a ride, 3 (a) ‘if I were’ seems to be the only correct
but my mother ……… going with my option. 17. Extradition of the leader of the group
brother. was debated for hours in the meeting.
(a) insists (b) insists on Directions (Q.Nos. 11-20) Each item in (a) acceptance (b) sentence
(c) insists in (d) insisted this section consists of a sentence with (c) extension (d) deportation
an underlined word/words followed by 3 (d) ‘deportation’, is closest in meaning to the
3 (a) ‘insists’ is appropriate option. ‘Insist’
means to say firmly or demand forcefully. No
four words. Select the option that is word ‘extradition’. The word means the action of
need to use any preposition. nearer to meaning to the underlined deporting a person accused of a crime.
word/words. 18. It was felt that the decision to remove
5. The new student found it difficult to
……… with his classmates. 11. Rahul is always thrifty. the group from the exercise would be
(a) get along (b) get among (a) reckless (b) economical detrimental to the organisation.
(c) get well (d) get up (c) naive (d) extravagant (a) beneficial (b) harsh
3 (b) economical is synonym of the underlined (c) disadvantageous (d) demanding
3 (a) ‘get along’ is the phrasal verb which
should be used here in the blank. ‘Get along’ word thrifty which means using money and other 3 (c) ‘disadvantageous’ is closest in meaning to
means to have a friendly relationship with others. resources carefully. Other options are antonyms the word ‘detrimental’. The word means
and do not give similar meaning. An economic adverse, undesirable or harmful.
6. The visiting Diplomat ……… the Prime person is one who is careful with spending
Minister. money. 19. His derisive behaviour has led to the
situation we face now.
(a) called in 12. His salubrious words calmed the (a) mockery (b) conducive
(b) called at students. (c) encouraging (d) contemptuous
(c) called on (a) provoking (b) pleasant
(d) called upon 3 (d) The word ‘derisive’ means expressing
(c) ridiculous (d) thanking contempt or ridicule. So, option (d)
3 (c) ‘called on’ is the appropriate phrasal verb 3 (b) pleasant is nearer to the meaning of ‘contemptuous’ is the most suitable option to
to fill in the blank. ‘Called on’ means to come to underlined word ‘salubrious’. Other options are select.
see or visit someone. not relevant here.
CDS Solved Paper 2018 (II) 17

20. Any classroom should provide an 3 (c) As stated in the passage curcumin has 27. The woman returned the letter to the
positive effect on people without dementia and
engaging environment for learners. also on people suffering from Alzheimer’s
postman because
(a) carefree (b) appealing disease. (a) she could not pay the postage
(c) thinking (d) dreaming (b) the letter was not addressed to her
3 (b) appealing is closest in meaning to the
23. Which word in the passage means (c) she already knew the contents of the
word ‘engaging’ which means delightful or ‘earlier’? letter
pleasing. (a) Performance (b) Absorbed (d) she hated the person who wrote the
(c) Properties (d) Previously letter
Directions (21-25) In this section, you 3 (d) Previously means earlier or in the time
have few short passages. After each 3 (c) The woman returned the letter to the
gone by. postman because she already knew the
passage, you will find some items based contents of the letter.
on the passage. First, read a passage and 24. Eating turmeric
answer the items based on it. You are (a) will reduce the chance of getting 28. Mr. Hill paid the postage because
required to select your answer based on Alzheimer’s disease (a) the letter was from her brother
the contents of the passage and opinion (b) will increase curcumin (b) the woman was his relative
(c) will enhance dementia (c) the letter was addressed to him
of the author only.
(d) will reduce chance of getting cancer (d) he wanted to be kind to her
PASSAGE 1 3 (a) As inferred from the passage,
consumption of turmeric can reduce the chance
3 (d) Mr. Hill paid the postage because he
wanted to be kind to the woman as letter was
Daily consumption of a certain form of of getting Alzheimer’s disease as it contains as from her brother.
curcumin improved memory and mood in antioxidant called curcumin.
people with mild, age-related memory loss. The 29. The envelope contained
25. ……… of a disease in a region depends (a) a currency note (b) two written
research examined the effects of an easily
on the food habits too. sheets
absorbed curcumin supplement on memory
(a) Dominance (b) Prevalence (c) no sheet at all (d) a blank sheet
performance in people without dementia, as
(c) Affection (d) Death 3 (d) As mentioned in the passage, the
well as curcumin’s potential impact on the
microscopic plaques and tangled in the brains 3 (b) Prevalence is the best option for this envelope contained a blank sheet only.
of people with Alzheimer’s disease. sentence. The word means widespread
presence or a condition which is quite common.
30. The woman and her brother had
Found in turmeric, curcumin has previously agreed that
been shown to have anti-inflammatory and (a) the letter with no postage meant good
antioxidant properties in laboratory studies. It
PASSAGE 2 news
has also been suggested as a possible reason that Mr. Rowland Hill, when a young man was (b) the blank sheet meant being well
senior citizens in India, where curcumin is a walking through the Lake district, when he one (c) the blank sheet meant bad news
dietary staple, have a lower prevalence of day saw the postman deliver a letter to a woman (d) the letter with no postage meant
Alzheimer's disease and better cognitive at a cottage door. The woman turned it over unimportant news
performance. and examined it and then returned it, saying 3 (b) There was an agreement between the
she could not pay the postage, which was a woman and her brother that the blank sheet
21. Which of the following statements are shilling. Hearing that the letter was from her meant all was well with him.
true? brother, Mr. Hill paid the postage, in spite of
1. Senior citizens in India have the manifest unwillingness of the woman. As PASSAGE 3
high level of Alzheimer’s disease soon as the postman was out of sight, she In good many cases unnecessary timidity makes
because of consumption of turmeric. showed Mr. Hill how his money had been the trouble worse than it needs to be. Public
2. Senior citizens in India do not, have wasted, as far as she was concerned. The sheet opinion is always more tyrannical towards those
high prevalence of Alzheimer’s was blank. There was an agreement between her who obviously fear it than towards those who
because of consumption of brother and herself that as long as all went well feel indifferent to it. A dog will bark more
turmeric. with him, he should send a blank sheet in this loudly and bite more easily when people are
3. Consumption of turmeric enhances way once a quarter and she thus had tidings of afraid of it than when they treat him with
cognitive performance. him without expense of postage. contempt and the human herd has something
4. Curcumin is an antioxidant. 26. The story uses irony as a technique of this same characteristic. If you show that you
Select the correct answer using the because are afraid of them, you give promise of good
codes given below (a) the woman returned her own hunting, whereas if you show indifference, they
(a) 2, 3 and 4 (b) 1, 3 and 4 brother’s letter without opening it begin to doubt their own power and therefore,
(c) 1 and 4 (d) 1 and 3 (b) the woman broke the agreement of tend to let you alone.
3 (a) Statement 2, 3 and 4 are true as per the receiving blank letters to convey well 31. If we are afraid of public opinion, the
passage. being of her brother attitude of the people towards us is
22. Curcumin has positive effect on people (c) Mr. Hill accepted the letter addressed (a) sympathetic
to the woman (b) indifferent
(a) without dementia
(d) in the modern times a brother has (c) admiration
(b) with Alzheimer’s disease
no time to write a letter to his own (d) ruthless
(c) without dementia and with
Alzheimer’s disease sister 3 (d) According to the passage if we are afraid
(d) with dementia and with Alzheimer’s 3 (a) Woman returned her own brother’s letter of public opinion, the altitude of the people
disease without reading it. towards us is ruthless attitude or tyrannical.
18 CDS Solved Paper 2018 (II)

32. The statement, ‘‘A dog will bark more 35. The author calls our age curious your response on the answer sheet against the
loudly and bite more easily when because corresponding letter i.e., (a), (b), (c). If you find
people are afraid of him, than when (a) it is an age of science and scientists no error, your response should be indicated
they treat him with contempt ......’’ are curious by nature as (d).
implies that (b) it is witnessing the emergence of a 39. The letter has been written I insist on
(a) barking dogs seldom bite world civilisation
(a) (b)
(b) we should not be afraid of dogs (c) it is witnessing incidents that threaten
(c) if we are afraid of others, they will to shake the very foundations of it being sent at once. No error
leave us alone civilisation (c) (d)
(d) if we are afraid of people, they will try (d) it is an age of contradictions consisting
3 (b) Delete ‘on’ after insist. The word does not
of constructive and destructive
to scare us more require any preposition. It means to say firmly or
activities
3 (d) This statement implies that if we are demand forcefully.
3 (d) The author says that this modern age is
afraid of people, they will try to scare us more. 40. “I’m tired of my boys”, said the mother,
curious because it is an age of contradictions
33. The author compares men with dogs consisting of constructive and destructive (a)
in respect of activities.
“Both of them keep quarrelling all the time
(a) attacking others without any reason 36. It is deplorable to witness mob attacks (b)
(b) attacking others when they are weak on embassies following a clash of
(c) barking and biting policies of two official policy makers right now also they are quarrelling
(d) faithfulness to the master because with one another.” No error
3 (b) People as well as dogs try to attack others (a) students should not take part in (c) (d)
when they are weak or afraid of them. politics, but should concentrate on 3 (c) Use ‘each other’ in place of ‘one another’
their studies as reference is for two as suggested by the use
34. ‘You give promise of good hunting’ (b) they may result in the loss of lives of of word ‘both’.
means young and promising students
(a) you are vulnerable (c) they are overlooked by the policy 41. Sherly wants to know
(b) you are challenging planners themselves (a)
(c) you are indomitable (d) they are indicative of the complete whether you are going
(d) you are confused failure of the government in
controlling the rebellious students
(b)
3 (a) The statement ‘you give promise of good
hunting’ means that you are vulnerable which 3 (c) As remarked or mentioned in the
to Delhi today night.
means exposed to the possibility of being passage, the attacks by students are overlooked (c)
attacked or harmed either physically or by the policy planners or policy makers No error
emotionally. themselves.
(d)
37. One aspect of the mob indulging in
PASSAGE 4 violence and arson is that they 3 (c) Today night is not correct. We must write
We live in a curious age. We are offered ‘tonight’. That means approaching evening or
(a) destroy very costly things like vehicles
glimpses of a world civilisation slowly emerging, night.
(b) destroy very valuable artifacts and
e.g., the U.N. special agencies dedicated to books 42. The visitor’s to the zoo are requested,
health and education. But along with these are (c) get a mad delight in destruction for (a)
sights and sounds that suggest that the whole the sake of destruction only
civilisation is rapidly being destroyed. (d) are motivated by certain political
in the interest of all concerned,
Two official policies clash and instantly ideology to resort to destruction (b)
embassies are attacked by howling mobs of 3 (c) A worrying aspect of mob attack is that not to carry sticks, stones or food
students, at once defying law, custom and they get a mad delight in destruction for the inside and not to tease animals.
usage. And that this may not be merely so many sake of destruction only. (c)
hot-headed lads escaping all control and may 38. In the passage, the word ‘demolition’ No error
itself be part of the policy of the political has the meaning as the word (d)
parties, that is, mob antics as additional (a) defying
propaganda to deceive world opinion, makes 3 (a) Use of apostrophe is incorrect here. Just
(b) antics
the situation even worse. write the ‘visitors’.
(c) destruction
Parties have always been dishonest, but now it (d) urge 43. The legendary hero
seems as if power-mania is ready to destroy those 3 (c) destruction or knocking down. (a)
civilities that make international relations possible.
Directions (Q.Nos. 39-48) Each item in laid down his precious life
There is something even worse. What inspires this section has a sentence with (b)
these students to burn cars and books is not three underlined parts labelled (a), (b)
their political enthusiasm but a frenzied delight for our country . No error
and (c).
in destruction, an urge towards violent (c) (d)
demolition. Read each sentence to find out whether there is
3 (c) Use of pronoun ‘our’ is incorrect here. Use
any error in any underlined part and indicate
‘his’ in its place.
CDS Solved Paper 2018 (II) 19

44. Our gardener, which is very lazy, Directions (Q. Nos. 49-63) Each of the 50. (a) had been (b) has been (c) was (d) is
(a) following sentences in this section has a 3 (b) has been, use of present perfect is required
blank space with four words or group of here.
says that
words given. Select whichever word or
(b) group of words you consider most 51. (a) instance (b) incident (c) accident
there will be no apples this year. appropriate for the blank space. (d) events
(c) 3 (a) Instance
This cultural form …...…… 49. and
No error indicate your response on the answer 52. (a) that (b) this (c) these (d) which
(d) sheet accordingly. Japan has a name 3 (b) this
3 (a) Use of ‘which’ pronoun is incorrect. Use
which means ‘whimsical or
impromptu pictures’. It …...…… 50. in 53. (a) For (b) Beginning (c) During (d) Initially
‘who’ for gardener. Who is generally used to
refer to people where as ‘which’ is used to refer existence since the 12th century when 3 (d) Initially
to animals or things. the first ….... 51. for this art form was
54. (a) enrage (b) enlarged (c) engraved
seen. Since the language itself is read
45. When I asked the guest from right to left, the books with
(d) enraged
(a) ………… 52. art form follow the same 3 (d) Enraged means to make people angry.
what she would like to drink pattern. ……… 53. when english 55. (a) the (b) a (c) some (d) same
(b) translations were made, they flipped
3 (d) Same. The word ‘the’ should come before
the pictures and published it. This same.
she replied that she preferred coffee
…………… 54. the purists as it showed
much more than tea. No error left-handed samurai, who did not exist 56. (a) difficulty (b) difficult (c) difference
(c) (d) in the original book. Hence, nowadays different
3 (c) Use of ‘much more than’ is incorrect here. even English translations follow 3 (a) difficulty
We should write she preferred coffee to tea. The ………… 55. right to left format.
word prefer means more liking to a particular
57. (a) educated (b) to be educated (c) to
The name of this art form is Manga. be educating (d) to educate
thing.
So the ……… 56. is that in the present 3 (b) to be educated (use of passive voice is
46. No sooner did I reached there social structure, discipline has required here.
(a) become an important factor
because we want large numbers of 58. (a) as quick as possible (b) as quickly as
the children left the place possible (c) as possible as (d) quickly
children ………… 57. together and
(b) ………… 58. Educated to be what? 3 (b) as quickly as possible
with their parents. No error To be bank clerks or super 59. (a) of some kind (b) of same kind (c) of
(c) (d) salesmen, capitalists or some (d) of same
3 (a) Part ‘A’ has an error. With helping verb, commissars. When you are a 3 (a) of some kind
‘did’ we use first form of verb. So the use of superman ……… 59. as/or a super
reached is not correct, write ‘did I reach’ there. governor or a subtle parliamentary 60. (a) beast (b) bear (c) beastly (d) bare
47. I did not want to listen to him, debater, what have you done? You are 3 (d) Use of ‘bare’ which means here basic and
probably very clever, full of facts. simple.
(a)
Anybody can pick up facts; but we are 61. (a) smiling for (b) smiling to (c)
but he was adamant and human beings, not factual machines, smiling with (d) smiling at
(b) not ……… 60. routine automations. But 3 (d) smiling at
discussed about the matter. No error again, sirs, you are not interested. You
are listening to me and ……… 61. each 62. (a) until there are (b) still there is
(c) (d) (c) till there was (d) till there is
other, your are not going to do a thing
3 (c) Use of preposition ‘about’ after discussed
about radically changing the 3 (d) till there is
is not correct, remove it. The word ‘discussed’
itself means to talk about something. education system; so it will drag on 63. (a) had learnt (b) learnt (c) have learnt
……… 62. a monstrous revolution, (d) had been learnt
48. Please note which will merely be another
(a) 3 (c) have learnt
substitution-there will be much more
that the interview for the post control because the totalitarian Directions (Q. Nos. 64-68) Given below
(b) government knows how to shape the are some idioms/phrases followed by four
minds and hearts of the people, they alternative meanings to each. Choose
shall be held on 15th June, 2019 the response (a), (b), (c) or (d) which is
between 10.00 a.m. to 2.00 p.m. ……… 63. the trick.
the most appropriate expression.
(c) 49.(a) originating (b) originates (c)
originated (d) organising
64. He makes decision on the fly.
No error (a) He decides quickly without any
(d) 3 (a) originating. As whole passage is in seriousness
present tense, use of ‘originating’ is (b) He decides with all seriousness
3 (c) Use of ‘shall’ is incorrect here. As interview correct which means ‘having a specified
is singular in form, it should be replaced by ‘will (c) He decides non-chalantly
beginning’. (d) He is unwilling to decide
be held’.
20 CDS Solved Paper 2018 (II)

3 (a) The phrase ‘on the fly’ means to do have drawn intense interest and 74. and they largely relied on agriculture,
something quickly without planning or thinking
about it in advance. e.g. The new rules have (R) fishing and hunting
been created ‘on the fly’. safety questions about cell phones (P)
65. Follow suit (S) the people had a subsistence economy
(a) Following someone's suit (a) P Q R S (b) R S P Q (Q)
(b) Suiting to someone (c) S P Q R (d) S R Q P
from excavation sites indicate that
(c) Doing the same as someone else has 3 (d) SRQP is the correct order of sentences.
just done
(R)
70. by means of education rich materials found
(d) Doing the same kind of mistake
(P) (S)
3 (c) ‘Follow suit’ means ‘doing the same as
someone else has just done. e.g. When one civilisation to bring about (a) R Q P S (b) Q S P R
airline reduces ticket prices, the others usually (Q) (c) S P Q R (d) S R Q P
follow suit.
it is difficult in modern 3 (d) SRQP
66. Close shave (R)
(a) Shaving very closely
75. and that is ‘To learn to say I am sorry’
an integrated individual
(b) Miraculous escape (P)
(S)
(c) Saving someone from danger something important enough that
(a) R Q S P (b) R S P Q
(d) Easy escape (Q)
(c) S P Q R (d) P R Q S
3 (b) The idiom ‘close shave’ means miraculous
3 (a) RQSP but surely there must be
escape. A close shave means a situation where
one avoids something dangerous. e.g. The car 71. is that it is not professional enough (R)
almost hit the child who ran out in front of it. It everyone should learn it
was a ‘close shave’. (P)
have not done their home work (S)
67. At the crossroads (a) R Q S P (b) R Q P S
(Q)
(a) At important point of a decision (c) S P Q R (d) P R Q S
(b) At an important point of journey a valid criticism of the profession of
politics in India 3 (a) RQSP
(c) At the important road of a journey
(d) At an important stage or decision (R) 76. or an independent judiciary
3 (a) The idiom ‘At the cross roads’ means at as the majority of its practitioners (P)
important point of a decision. When a choice (S) a free press is
must be made, it is called at the cross roads. (a) R S P Q (b) R P S Q
e.g. He is at the cross roads in his career-either (Q)
(c) S P Q R (d) P Q R S
he stays in his current job and waits for as essential a limb of democracy as a
3 (b) RPSQ
promotion or accept new post abroad. Parliament
68. A pearl of wisdom 72. that suit partisan political objectives (R)
(a) A wise man (P) freely elected by the people
(b) An important piece of order when great historical figures are (S)
(c) An important piece of pearl appropriated
(a) R Q S P
(d) An important piece of advice (Q) (b) Q R P S
3 (d) ‘A pearl of wisdom’ means an important or we are living at a time (c) S P Q R
valuable piece of advice. e.g. The old woman (d) Q R S P
(R)
shared her ‘pearls of wisdom’ with the
and reduced into stereotypes 3 (d) QRSP
youngsters.
(S) 77. the opinion that a human life
Directions (Q. Nos. 69-88) Each of the (a) R Q P S (b) R Q S P
following items in this section consists of (P)
(c) S Q R P (d) P R Q S
a sentence the parts of which have been and that he would quite like to live
3 (b) RQSP
jumbled. These parts have been labelled that long
P, Q, R and S are given below each 73. it is in this context that (Q)
sentence in four sequences namely (a), (P) could span 125 years
(b), (c) and (d). You are required to and prosperity must be viewed (R)
rearrange the jumbled parts of the
(Q) there was a time when Gandhi
sentence and mark your response
accordingly. the role of agriculture expressed
(R) (S)
69. have become integral to most people’s
as a provider of jobs (a) S P R Q
lives
(b) R Q P S
(P) (S)
(c) S P Q R
(a) P Q R S (b) R S P Q
debate for years as the devices (d) Q R S P
(c) P R S Q (d) R S Q P
(Q) 3 (a) SPRQ
3 (c) PRSQ
CDS Solved Paper 2018 (II) 21

78. I must say what I feel that are causing social ferment 87. National Building Organisation
(P) (S) besides conducting surveys on
(a) Q R P S (b) Q S R P housing
I am a votary of truth and
(c) S P R Q (d) P R S Q (P)
(Q)
3 (a) QRPS and disseminates the statistical
to what I may have said before
83. The best part of literary flourishes information
(R)
(P) (Q)
and think at a given moment without
and locates the story with the larger collects, tabulates
regards
framework of our world (R)
(S)
(a) R Q S P (b) Q R P S (Q) on housing and building construction
(c) P S R Q (d) Q P S R long-formed journalism is that activities
3 (d) QPSR (R) (S)
(a) Q R P S (b) R P Q S
79. The man in the competition it brings back the importance of
(c) S P R Q (d) R Q S P
writing skills
(P) 3 (d) RQSP
(S)
has been elected
(a) Q R P S (b) R S P Q 88. The Himalayan range sacred to the
(Q) (c) S P R Q (d) P R S Q Gaddi people
as the chairperson of the sports 3 (d) PRSQ (P)
committee
84. Children that grow into beautiful is home
(R)
trees (Q)
in red who stood first
(P) to a chain of high altitude lakes
(S)
of a warm home and supportive (R)
(a) S P Q R (b) S R P Q
surroundings that towers over the Kangra valley
(c) P S R Q (d) Q R S P
(Q) (S)
3 (a) SPQR
are like the tender samplings (a) Q R P S
80. One of the difficulties the whole of (b) S P Q R
(R)
mankind (c) S Q R P
with the sunshine and rain (d) R Q S P
(P)
(S) 3 (c) SQRP
or affect the masses
(a) Q R P S (b) R P Q S
(Q) (c) R P S Q (d) P R S Q Directions (Q. Nos. 89-98) In this
the day after tomorrow 3 (c) RPSQ section each item consists of six
(R) sentences of a passage. The first and the
85. We with real life experiences sixth sentences are given as S1 and S6.
is that we want to transform
(P) The middle four sentences in each have
(S)
tend to learn with interest been jumbled up and labelled P, Q, R and
(a) S P Q R (b) P R S Q S. You are required to find the proper
(c) S P R Q (d) Q R S P (Q)
sequences of the four sentences.
3 (a) SPQR when we see beauty in our work
(R) 89. S1 : The giant wall of the
81. The speaker of their in action Dhauladhar range in Himachal
and connect learning Pradesh is one of the most
(P)
(S) stunning sights in the Himalayas.
has identified charging the opponents
(a) Q R S P (b) R P Q S P : As the life line of the region it
(Q) (R) (c) S P R Q (d) P R S Q acts as a watershed ridge
many issues besides 3 (a) QRSP between Chamba’s Ravi river
(S) system and Kangra’s Beas river
86. Elementary education ensuring the system.
(a) P S Q R (b) Q S R P
growth of a nation
(c) S P R Q (d) Q S P R Q : Although of modest altitude
(P) compared to other Himalayan
3 (b) QSRP
is inevitable in developing the children ranges-the highest Dhauladhar
82. The government and job markets peak is less than 5,000 m.
(Q) (R)
(P) R : Thus, the Dhauladhar could be
to further education, thereby
must offer convincing solutions stated as the life line of the region.
(S)
(Q) S : Despite of that, the range sweeps
(a) Q R P S (b) R P Q S
up an astounding 12,000 ft. from
to the crises in the rural economy (c) S P R Q (d) Q R S P
the valley floor, creating a barrier
(R) 3 (d) QRSP wall in that is striking to look at.
22 CDS Solved Paper 2018 (II)

S6 : Looming over the hill stations of 92. S1 : Tolstoy Farm was founded in destroying ourselves and most of
Dharmsala and McLeodganj, the 1910 by which time Gandhi had the life on Earth.
Dhauladhar is a popular trekking already conceptualised ideas that S : This process has accelerated
destination. he would develop in India. greatly since the industrial
(a) Q R P S (b) S P Q R P : He was rich and used his money revolution.
(c) Q S R P (d) R Q S P to buy the land and help set up S6 : The concept of ‘sustainable’ is so
3 (d) RQSP the farm. far from reality that it is almost
90. S1 : Truth is far more important than Q : A Jewish architect, Kallenbach was laughable.
the teacher. by his side through this period. (a) P Q R S (b) Q P S R
P : Without self-knowledge, the R : Tolstoy Farm became the subject (c) P Q S R (d) S R Q P
airplane becomes the most of research for different kinds of 3 (b) QPSR
destructive instrument in life; but cooperative communities across
the world.
95. S1 : Measurement is an important
with self-knowledge, it is a concept in performance
means of human help. S : He first put in the social, moral, management.
Q : Wisdom begins with religious components of his
P : It also indicates where things are
self-knowledge; and without doctrine.
not going so well, so that
self-knowledge, mere S6 : Both he and Gandhi often
corrective action can be taken.
information leads to destruction. referred to the time that they
Q : It identifies where things are going
R : In other words, you have to be spent in Tolstoy Farm as among
well to provide the foundations
the perfect teacher to create a the happiest in their lives.
for building further success.
new society; and to bring the (a) Q R S P (b) S Q P R
R : It is the basis for providing and
perfect teacher into being, you (c) S Q R P (d) R Q P S
generating feedback.
have to understand yourself. 3 (b) SQPR S : Measuring performance is
S : Therefore you, who are the
seeker of truth, have to be both 93. S1 : Decentralised planning is a relatively easy for those who are
process of planning that begins responsible for achieving quantified
the pupil and the teacher.
from the grassroots level taking targets for example sales.
S6 : So, a teacher must obviously be
into confidence all the beneficiaries. S6 : It is more difficult in the case of
one who is not within the
P : Under decentralised planning, the knowledge workers e.g. scientists
clutches of society, who does not
operation is from bottom to top. and teachers.
play power politics or seeks
(a) R Q P S (b) Q P S R
position or authority. Q : It can be said that it is more
(c) P S Q R (d) S P Q R
(a) Q R S P (b) S R Q P connected with the capitalistic
(c) Q S R P (d) R Q S P economies. 3 (b) QPSR
3 (b) SRQP R : It empowers the individuals and 96. S1 : Equity theory is concerned with
small groups to carry out their the perception people have
91. S1 : Though most of us talk of discipline, plans for their achievement of a about how they are being
what do we mean by that word?
common goal. treated compared with others.
P : The teacher would understand
each child and help him in the S : The decentralised planning is P : To be dealt with equitably is to
way required. implemented through market be treated fairly in comparison
Q : But if you have five or six in a mechanism. with another group of people or
class and an intelligent S6 : But it cannot be described as a relevant other person.
understanding teacher with a undemocratic for most national Q : Equity involves feelings and
warm heart, I am sure there states adopt such a planning perceptions and is always a
would be no need discipline. now. comparative process.
R : When you have a hundred boys (a) Q R S P (b) S R Q P R : Equity theory states, in effect,
in a class, you will have to have (c) S Q R P (d) S R P Q that people will be better
discipline; otherwise there will 3 (c) SQRP motivated if they are treated
be complete chaos. equitably and demotivated if they
S : Discipline in schools becomes 94. S1 : It is doubtful if mankind, are treated inequitably.
necessary when there is one teacher throughout his long history, has S : It is not synonymous with
to a hundred boys and girls. ever lived at all ‘sustainably’. equality, which means treating
S6 : And most of us are interested in P : But in general mankind has everyone the same, since this
mass movements, large schools regarded the environment as an would be inequitable if they
with a great many boys and girls; endless ‘resource’ to be exploited deserve to be treated differently.
we are not interested in creative and plundered. S6 : This explains only one aspect of
intelligence, therefore we put up Q : May be a few isolated tribal the process of motivation and job
huge schools with enormous groups found the necessary satisfaction, although it may be
attendances. balance with nature lived without significant in terms of morale.
(a) Q R S P (b) S R Q P the desire for endless ‘more’. (a) P Q R S (b) P Q S R
(c) Q S R P (d) R Q P S R : Now we have reached a point (c) R S Q P (d) Q P R S
3 (d) RQPS where we are on the verge of 3 (c) RSQP
CDS Solved Paper 2018 (II) 23

97. S1 : We cannot understand the power Directions (Q. Nos. 99-109) Each item 3 (a) The word ‘inadvertently’ means something
in this section consists of a sentence with done without intention or accidentally. Its
of rumours and prophecies in antonym is ‘purposely’ which means something
history by checking whether an underlined word/words followed by done deliberately or intentionally.
they are factually correct or not. four words.
P : The rumours in 1857 began to Select the option that is opposite in 106. The whole audience showed a
make sense when seen in the meaning to the underlined word/ words. disdainful attitude during the match.
(a) Sneering
context of the policies the 99. His ideas are obscure. (b) Respectful
British pursued from the late (a) New (b) Clear (c) Mocking
1820s. (c) Infamous (d) Obscene (d) Cheerful
Q : Rumours circulate only when 3 (b) clear is antonym of word ‘obscure’, which 3 (b) The word ‘disdainful’ means showing lack
they resonate with the deeper means something uncertain or doubtful. ‘Clear’ of respect or derisive. Its antonym in ‘respectful’
fears and suspicions of people. means which is easy to understand or which means showing respect or reverence.
R : Under the leadership of transparent.
Governor General Lord William 107. Efficacy of the project needs an
100. Ravi is jovial and he makes the examination.
Bentinck, the British adopted environment sanguine.
policies aimed at reforming (a) Inefficiency (b) Efficiency
(a) ‘Pessimistic’ (b) Optimistic (c) Value (d) Effectiveness
Indian society by introducing (c) Humorous (d) Rebellious
Western education, 3 (a) The word ‘efficacy’ means the ability to
3 (a) ‘Pessimistic’, the word ‘Sanguine’ means produce a desired or intended result. Its
Western ideas and Western optimistic or positive especially in a bad or antonym is ‘inefficiency’ suggesting a state of
Institutions. difficult situation. So, its antonym is ‘Pessimistic’ not achieving maximum productivity, or failure to
S : We need to see what they reflect which means gloomy or believer in that worst will make best use of time or resources.
about the minds of people who happen.
believed them-their fears and 108. Her rebuttal that she was not involved
101. There prevailed a woebegone feeling in the case was considered by the
apprehensions, their faiths and in the room.
convictions. court.
(a) Sad (b) Cheerful (a) Refusal (b) Denial
S6 : With the cooperation of sections (c) Sleepy (d) Thoughtful (c) Acceptance (d) Kindness
of Indian society they set up 3 (b) The word ‘woebegone’ means someone
English-medium schools, colleges 3 (c) The word ‘rebuttal’ means denial or
sad or miserable in appearance. Its antonym will negation. Its antonym from the given option is
and universities which taught be ‘cheerful’ which means noticeably happy and ‘Acceptance’ which means a general
Western sciences and liberal arts. optimistic. agreement, or the act the agreeing to an offer,
(a) S Q P R (b) Q S P R plan or invitation.
102. It appears that the whole group is
(c) P R S Q (d) R S P Q
mutinous. 109. The baby could not move as the place
3 (b) QSPR (a) Arrogant (b) Lucky was soggy.
98. S1 : The Constitution of India thus (c) Obedient (d) Sincere (a) Sodden (b) Dry
emerged through a process of 3 (c) The word ‘mutinuous’ means a person or (c) Hot (d) Wet
intense debate and discussion. soldier refusing to obey the authority. Its antonym 3 (b) dry. The word ‘soggy’ means wet and soft.
is ‘obedient’. The word means a person who is Its antonym is ‘dry’ which means something free
P : This was an unprecedented act of willing to comply with an order or request.
faith, for in other democracies of moisture, not wet.
the vote had been granted 103. They consider themselves as foes from Directions (Q. Nos. 110-115) Each item
slowly, and in stages. birth.
in this section has a direct statement
Q : However, on one central feature (a) Protagonists (b) Opponents
followed by its reported form in indirect
of the Constitution there was (c) Friends (d) Soul mates
speech. Select the correct statement in
substantial agreement. 3 (c) ‘Foe’ means enemy or opponent. Its indirect speech and mark it in the
R : Many of the provisions were antonym is friends which means a person with
whom one has bond of affection.
answer sheet accordingly.
arrived at through a process of
give and take, by forgiving a 104. This painting has a distinctive element 110. The captain said to his soldiers, “Move
middle ground between two which can be noticed well. forward and face the target now.’’
opposed positions. (a) Salient (b) Common (a) The captain ordered his soldiers to
(c) Great (d) Unique move forward and face the target
S : This was on the granting of the
(b) The captain informed his soldiers that
vote to every adult Indian. 3 (b) The word ‘distinctive’ means unique or they should move forward and face
S6 : In countries such as the United remarkable or uncommon. So, from the options
the target now
States and the United Kingdom, given ‘common’ is appropriate antonym which
means something which is found or done very (c) The captain asked his soldiers to
only men with education often or which is prevalent. move forward and face the target then
were allowed into the charmed (d) The captain told his soldiers that they
circle. 105. The entry was carried out move forward and face the target
(a) P R S Q inadvertently. immediately
(b) R Q S P (a) Purposely
3 (a) It is correct sentence (Imperative type) in
(c) S P R Q (b) Purposively Indirect speech. Here, principal verb is ‘ordered’
(d) Q S R P (c) Accidently which is correct as a captain orders to his
3 (b) RQSP (d) Not noticing soldiers.
24 CDS Solved Paper 2018 (II)

111. Vivek said to his friend, “Could you (b) The actor told his co-star, Sarita if she (b) One particular question was
please turn off the switch ?” would go with him for a cup of tea in answered by all the examinees in the
evening that day long answer writing section
(a) Vivek told his friend to turn off the
(c) The actor requests his co-star, Sarita (c) All the examinees answered one
switch
if she would go with him for a cup of particular question in the long answer
(b) Vivek asked his friend to please turn
tea in that evening that day writing section
off the switch
(d) The actor asked his co-star Sarita if (d) One particular question has been
(c) Vivek requested his friend to turn off
she would go with him for a cup of answered by all the examinees in the
the switch
tea in the evening that day long answer writing section
(d) Vivek told his friend that he should
turn off the switch 3 (d) It is correct sentence in Indirect speech. 3 (d) It is the correct sentence in Passive voice.
Sentence is in present perfect tense.
3 (c) As the sentence in direct speech is a type 115. The preacher said to the crowd, “The
of request, option (c) is correct statement in Sun rises everyday for all of us 118. The writer who passed away
indirect speech. without any expectations in return.” recently has authored a dozen
112. The manager said to his colleagues, (a) The preacher told the crowd that novels and a number of poetry
“We have received a serious threat to the Sun rose everyday for all of collections.
our business now and we need to act them without any expectations in (a) A dozen novels and a number of
to face it.” return poetry collections have been
(a) The manager told his colleagues that (b) The preacher told the crowd that the authored by the writer who passed
they had received a serious threat to Sun rises everyday for all of us away recently
our business then and they needed to without any expectations in return (b) A dozen novels and a number of
act to face it (c) The preacher told the crowd that the poetry collections has been authored
(b) The manager told his colleagues that Sun has risen everyday for all of by the writer who passed away
they received a serious threat to their them without any expectations in recently
business then and they needed to act return (c) A dozen novels and a number of
to face it (d) The preacher told the crowd that poetry collections were authored
(c) The manager said his colleagues that the Sun rises everyday for all of by the writer who passed away
they had received a serious threat to them without any expectations in recently
our business then and they needed to return (d) A dozen novels and a number of
act to face it. 3 (d) It is correct sentence in Indirect speech. poetry collections had been authored
(d) The manager told his colleagues that by the writer who passed away
they had received a serious threat to Directions (Q. Nos. 116-120) Each item recently
their business at that time and they in this section has a sentence in active 3 (a) It is the correct sentence in Passive voice.
needed to act to face it. voice followed by four sentences one of Sentence is in present perfect tense.
which is the correct passive voice
3 (d) As the sentence is in past tense ‘have
statement of the same. Select the correct
119. Shut the door.
received’ will change to ‘had received’ and (a) Shut the door
‘need’ will be changed to needed. one and mark it in the answer sheet
(b) Let the door be shut
accordingly.
113. Romila said to Rahim, “Where were (c) The door be shut
your ideas when we faced the troubles 116. The Members of the Parliament elect (d) The door is shut
last week ?” their group leader either by consensus 3 (b) It is the correct sentence in Passive voice.
(a) Romila asked Rahim where his ideas or by voice vote. An Imperative sentence in passive voice has the
had been when they had faced the (a) The group leader is elected by the following structure. Let + object + be + past
trouble the week before Members of the Parliament either by participle.
(b) Romila asked Rahim where his ideas consensus or by voice vote 120. India won freedom with the blood and
had been when they faced the trouble (b) The group leader was elected by the sweat of hundreds and thousands of
the last week Members of the Parliament either by Indians.
(c) Romila requested Rahim where his consensus or by voice vote
(a) India had won freedom with the blood
ideas had been when they faced the (c) The group leader has been elected by
and sweat of hundreds and
trouble the week before the Members of the Parliament either
thousands of Indians
(d) Romila told Rahim where his ideas by consensus or by voice vote
(b) Freedom had been won by India with
were when they faced the trouble the (d) The Members of the Parliament are
the blood and sweat of hundreds and
week before elected by their group leader either by
thousands of Indians
3 (a) As sentence is interrogative type, the consensus or by voice vote
(c) Freedom was won by India with the
correct statement in Indirect speech with proper 3 (a) It is the correct sentence in Passive voice. blood and sweat of hundreds and
tense used in option (a). Sentence is in simple present tense. thousands of Indians
114. The actor said to his co-star, Sarita, 117. All the examinees have answered one (d) Freedom was won by hundreds and
“Will you go with me for a cup of tea particular question in the long answer thousands of Indians with their blood
in the evening today?” writing section. and sweat
(a) The actor said to his co-star if she (a) One particular question is answered 3 (c) It is the correct sentence in Passive voice.
would go for a cup of tea with him in by all the examinees in the long The sentence in active speech is in simple past
evening today answer writing section tense.
PAPER III General Studies
1. Two reactants in a flask at room (c) Calcium carbonate and hydrochloric 3 (a) C 7H 7NO 2
acid
temperature are producing bubbles of Number of carbon atoms = 7
(d) Zinc and nitric acid
a gas that turn limewater milky. The Number of hydrogen atoms = 7
reactants could be 3 (a) Number of nitrogen atoms = 1
(a) zinc and hydrochloric acid (i) Reaction between magnesium and Number of oxygen atoms = 2
(b) magnesium carbonate and hydrochloric acid An oxygen molecule O 2 contains 2 oxygen atoms.
hydrochloric acid Total number of molecules = 7 + 7 + 1 + 2 = 17.
(c) methane and oxygen Mg( s )+2HCl( aq ) → MgCl 2 ( aq )+H 2 ( g ) ↑
So, 17 atoms are present in a molecule of the
(d) copper and dilute hydrochloric acid When magnesium reacts with hydrochloric compound.
acid, they produce hydrogen (H 2 ) gas.
3 (b) When magnesium carbonate (MgCO 3 ) 6. Which of the following is the general
reacts with hydrochloric acid (HCl), the product (ii) Reaction between copper and dilute nitric formula for saturated hydrocarbons?
will be magnesium chloride (MgCl 2 ), water (H 2O) acid (a) C nH 2 n + 2 (b) C nH 2 n − 2
and carbon dioxide (CO 2 ). 3Cu + 8HNO 3 (dilute) → (c) C nH 2 n + 1 (d)C nH 2 n − 1
MgCO 3 ( s ) + 2HCl( aq ) → MgCl 2 ( aq ) 3Cu(NO 3 )2 + 2NO + 4H 2O 3 (a) Alkanes are saturated hydrocarbons. This
+ H 2O( l ) + CO 2 ( g ) ↑
When copper reacts with dil. nitric acid, means that they contain only carbon and
Above two reactants when take in a flask at room hydrogen atoms bonded by single bonds only.
they cannot produce hydrogen (H 2 ) gas.
temperature produce bubbles of CO 2 (carbon
dioxide) gas. (iii) Reaction between calcium carbonate and The general formula for an alkane is C nH 2 n + 2 .
After that, CO 2 reacts with lime water to form hydrochloric acid In this formula, n = number of carbon.
calcium carbonate (CaCO 3 ), which is white and
CaCO 3 + HCl → CaCl 2 7. A particle moves with uniform
does not dissolve in water. Thus, causing the lime
water turn milky. + CO 2 + H 2O acceleration along a straight line from
[Lime water → Lime water is the common name Calcium carbonate reacts with hydrochloric
rest. The percentage increase in
for a diluted solution of calcium hydroxide displacement during sixth second
acid which cannot produce hydrogen (H 2 )
(Ca(OH)2 ). It is clear and colourless.] compared to that in fifth second is
gas.
Ca(OH)2 ( aq ) + CO 2 ( g ) → CaCO 3 ( s )+ H 2O( l ) about
(iv) Reaction between zinc and nitric acid (a) 11% (b) 22% (c) 33% (d) 44%
2. How many moles of CO can be
obtained by reacting 2.0 mole of CH 4 Zn + 4HNO 3 → Zn(NO 3 )2 + 2H 2O 3 (b) The particle is moving with uniform
with 2.0 mole of O 2 according to the + 2NO 2 acceleration along a straight line from rest.
equation given below? Displacement during 6th second
When zinc reacts with nitric acid, they also → 1
1 S 6th = u + a(2 n − 1)
CH 4 (g ) + O 2 → CO + 2H 2 cannot produce hydrogen (H 2 ) gas. 2
2 Note Nitric acid is an oxidizing agent which Q n=6
→ 1
(a) 2.0 (b) 0.5 produces oxides of nitrogen on reacting S 6th = u + a (12 − 1)
(c) 2.5 (d) 4.0 with metal. It do not produces hydrogen 2

gas. Given, u =0
3 (a) Step I To find the limiting reagent, → 11 11→
So, S 6th = a = a
1 4. Which of the following characteristics 2 2
CH 4 ( g ) + O 2 → CO + 2H 2 →
2 1→ 9a
is common to hydrogen, nitrogen, In 5th second = a (10 − 1) =
1 oxygen and carbon dioxide? 2 2
1 : : 1 : 2
2 (a) They are all diatomic. S − S 5th
= 6th
As per balanced equation, (b) They are all gases at room S 5th
1 →
For mole of O 2 we need = 1 mole of CH 4 temperature.
11 a 9 a

2 (c) They are all coloured. −


= 2 2 =2
∴ For 2 mole of O 2 (given) we need = 4 mole (d) They all have same reactivity. →
9a 9
of CH 4 3 (b) H 2 , N 2 , O 2 and CO 2 are all gases at room
But we have given only 2 moles of CH 4 , thus CH 4 temperature. 2
2
is out limiting reagent. H 2 ,N 2 and O 2 are diatomic but CO 2 is triatomic. ∴ % = × 100
Thus, They all are colourless gases and they all have 9
Step II Quantity of CO is obtained different reactivity. − 22%
= 22.22 ~
Q 1 mole of CH 4 give 1 mole of CO 5. The compound C7 H 7 NO 2 has 8. If two miscible liquids of same volume
∴ 2 moles of CH 4 give = 2 moles of CO but different densities P1 and P2 are
(a) 17 atoms in a molecule of the
Hence, 2 moles of CO will be produced. compound mixed, then the density of the mixture
3. Reaction between which of the (b) equal molecules of C and H by mass is given by
following two reactants will produce (c) twice the mass of oxygen atoms P1 + P2 2 P1 P2
(a) (b)
hydrogen gas? compared to nitrogen atoms 2 P1 + P2
(a) Magnesium and hydrochloric acid (d) twice the mass of nitrogen atoms 2 P1 P2 PP
compared to hydrogen atoms (c) (d) 1 2
(b) Copper and dilute nitric acid P1 − P2 P1 + P2
26 CDS Solved Paper 2018 (II)

3 (b) Density of liquid A = P1 11. Which one of the following does not 16. If the xylem of a plant is mechanically
Density of liquid B = P2 convert electrical energy into light blocked, which of the following
We assume the mass of liquid A or B = m energy? functions of the plant will be affected?
m
Volume of liquid A = (a) A candle (a) Transport of water only
P1 (b) A light-emitting diode (b) Transport of water and solutes
Mass (c) A laser (d) A television set (c) Transport of solutes only
[Formula → Volume = ]
Density 3 (a) Burning of a candle is a chemical as well (d) Transport of gases
m as physical change in which chemical energy is 3 (b) Xylem is responsible for transportation of
Volume of liquid B =
P2 converted into heat and light energy. A water and solutes (soluble mineral ions). Thus its
light-emitting diode, a laser, a television set blocking will affect transportation of water and
m m
Total volume of the mixture = + convert electrical energy into light energy.
P1 P2 solutes.
Total mass = m + m = 2 m
12. Which of the following is/are the main 17. Which one of the following agents
Mass absorbing organ/organs of plants? does not contribute to propagation of
Hence, density of mixture = (a) Root only (b) Leaf only
Volume plants through seed dispersal?
(c) Root and leaf only
2m 2m (a) Wind (b) Fungus
= = (d) Root, leaf and bark
m m 1 1 (c) Animal (d) Water
 +  m +  3 (a) Roots are the main absorbing organ in
 P1 P2   P1 P2  3 (b) Except fungus, all of rest factors are helpful
plants. This underground part of plant absorbs
2 2 in seed dispersal. The dislocation of seeds from
= (cancelling m) = water and soluble mineral salts from soil.
their parental plants by different mediums is
1 1  P2 + P1 
 +    13. Which of the following is not a known as seed dispersal. The seeds of
 P1 P2   P1 P2  primary function of a green leaf? calotropis, rafflesia and lotus are dispersed by
2 P1 P2 P + P2 wind, elephant and water respectively.
Density of mixture = = 1 (a) Manufacture of food
P1 + P2 2 (b) Interchange of gases 18. The visible portion of the
(c) Evaporation of water electromagnetic spectrum is
9. The position vector of a particle is (d) Conduction of food and water (a) infrared (b) radiowave

r = 2t 2 x$ + 3ty$ + 4z$ 3 (d) Conduction of food and water is not the (c) microwave (d) light
→ primary function of green leaf. Manufacture of 3 (d) The visible portion of the electromagnetic
Then the instantaneous velocity v and food by photosynthesis, gaseous interchange spectrum is light. Visible rays are the most
→ by stomatas and evaporation of water
acceleration a respectively lie familiar form of electromagnetic waves. It is the
(Transpiration) are primary functions of leaves. part of the spectrum that is detected by human
(a) on xy -plane and along z-direction Conduction of water and food is done by eyes. It runs from about 4 × 1014 Hz to about
(b) on yz-plane and along x-direction vascular tissues, i.e. conduction of water occurs 7 × 1014 Hz or a wavelength of about 700-400
(c) on yz-plane and along y-direction via xylem and conduction of food occurs via nm. Visible light emitted or reflected from objects
(d) on xy -plane and along x-direction phloem. around us provides us information about the
→ world. Infrared, radiowaves, microwaves, etc are
3 (d) r = 2t 2 $x + 3 t $y + 4 $z 14. Which one of the following denotes a not visible portion of electromagnetic spectrum.
r dvr ‘true’ fruit?
Instaneous velocity V = 4 tx$ + 3 $y 19. Which one of the following is the
dt (a) When only the thalamus of the flower
correct ascending sequence of states in
Hence, istantaneous velocity lie on XY plane. grows and develops into a fruit
r terms of their population density as
dv (b) When only the receptacle of the
Instaneous acceleration a = = 4 $x per Census 2011?
dt flower develops into a fruit
(c) When fruit originates only from the (a) Arunachal Pradesh–Sikkim–Mizoram–
Hence, acceleration lie along x − axis. Himachal Pradesh
calyx of a flower
10. Two persons are holding a rope of (d) When only the ovary of the flower (b) Arunachal Pradesh–Mizoram–Sikkim–
negligible mass horizontally. A 20 kg grows into a fruit Himachal Pradesh
(c) Mizoram–Arunachal Pradesh–
mass is attached to the rope at the 3 (d) True fruit develops from ovary of flower Himachal Pradesh–Sikkim
midpoint; as a result the rope after fertilization, while false or pseudo fruit
(d) Arunachal Pradesh–Himachal
deviates from the horizontal develops from other parts of flower, i.e. the
Pradesh–Sikkim–Mizoram
direction. The tension required to edible part of apple is fleshy receptacle.
completely straighten the rope is 3 (b) The correct ascending sequence of states
15. In which one of the following in terms of their population density as per
(g = 10 m/s 2 ) physiological processes, excess water Census 2011 is
(a) 200 N escapes in the form of droplets from a
(b) 20 N
l Arunachal Pradesh–17
plant? l Mizoram–52
(c) 10 N
(a) Transpiration (b) Guttation l Sikkim–86
(d) infinitely large
(c) Secretion (d) Excretion l Himachal Pradesh–550
3 (a) Given, m = 20 kg
3 (b) Guttation is special physiological process
Tension required to completely straighten in which excess water escapes in the form of 20. The rate of population growth during
the rope. liquid droplets from the tips and margins of 2001-2011 decade declined over the
From Newton’s second law of motion, leaves. It occurs during night or early morning, previous decade (1991-2001) in all of
F = ma when there is high atmospheric humidity and the following states, except
F = 20 × 10 (Q acc n due to g = 10 m/s) rate of transpiration is lower than the rate of (a) Tamil Nadu (b) Kerala
= 20 kg ×10 m/s 2 = 200 N water absorption. (c) Goa (d) Andhra Pradesh
CDS Solved Paper 2018 (II) 27

3 (a) The rate of population growth during 2001 Ba(OH)2 = Ba 2 + + 2OH − l Each layer is composed of planar
- 2011 decade and previous decade One mole of Ba(OH)2 furnishes 2 moles of hexagonal rings of carbon atoms.
(1991-2001) in the given states is stated below OH − (2 equivalents). l Electrons are mobile in graphite,
State State Growth Ba(OH)2 = molar mass therefore graphite conducts electricity.
growth (1991- 1137
( .3) + 2(16) + 2(1) = 1713
. gm/mole l It is a good electric conductor than
(2001-11) 2001) 1713. diamond.
Ba(OH)2 = equivalent weight =
1. Tamil Nadu 15.60 11.19 2
27. In which one of the following
2. Kerala 4.9 9.42 = 857
. g/eq
reactions, the maximum quantity
3. Goa 8.2 14.89 25. Which one of the following nitrogen of H2 gas is produced by the
4. Andhra Pradesh 11.0 14.59 oxides has the highest oxidation state decomposition of 1 g of compound by
on nitrogen? H 2 O/O 2 ?
Thus, decadal growth rate of Tamil Nadu was (a) NO (b) NO 2 (a) CH 4 + H 2O → CO + 3H 2
increased from the decade 1991-2001 to (c) N 2O (d) N 2O 5
2001-2011. (b) CO 4 + H 2O → CO 2 + H 2
3 (d) Oxidation state of oxygen = − 2 1
21. Which one of the following (c) CH 4 + O 2 → CO + 2H 2
Oxidation state of nitrogen = x 2
statements with regard to growth of
(a) NO = x + ( −2 ) = 0; x − 2 = 0; x = + 2 (d) C 12H 24 + 6O 2 → 12CO + 12H 2
coral reefs is not correct?
(a) Coral can grow abundantly in fresh Where, oxidation state of nitrogen is + 2. 3 (a) n =
w
where, n = no. of moles
water. (b) NO 2 ; x + 2 ( − 2 ) = 0; x − 4 = 0; x = + 4 m
(b) It requires warm water between w = given mass
23°C-25°C. In above compound, oxidation state of
nitrogen is + 4. m = atomic mass
(c) It requires shallow saltwater, not
deeper than 50 metres. 2 (i) CH 4 + H 2O → CO + 3H 2
(c) N 2O; 2 x − 2 = 0; 2 x − 2, x = =1
(d) It requires plenty of sunlight to aid 2 (12 + 4 = 16) gm + 1 gm → 6 gm
photosynthesis. Where, oxidation state of nitrogen is + 1. Q 16 gm will produce = 6 gm of H 2
3 (a) The formation of coral reefs only occurs in
(d) N 2O 5 ; 2 x + 5 ( − 2 ) = 0; 2 x − 10 = 0 ∴1 gm will produce
saltwater of oceans, where temperature is
between 23°-25°C and for high productivity. 10 6
x= =5 = = 0.375 gm of H 2
They do not exist below 50 m due to 2 16
requirement of plenty of sunlight.
In this compound, oxidation state of (ii) CO + H 2O → CO 2 + H 2
22. As per Census 2011, the nitrogen is + 5, which is the highest
concentration of Scheduled Caste oxidation state of nitrogen. (12 + 16 = 28) gm + 1 gm → 2 gm
population (going by percentage of 28 gm will produce = 2 gm of H 2
Scheduled Caste population to total
26. Which one of the following is not
true for the form of carbon known as 2
population of the State) is the highest 1 gm will produce = = 0.0714 gm of H 2
diamond? 28
in the state of
(a) Uttar Pradesh (a) It is harder than graphite. 1
(b) It contains the same percentage of (iii) CH 4 + O 2 → CO + 2H 2
(b) Himachal Pradesh 2
(c) Punjab carbon as graphite.
(d) West Bengal (c) It is a better electric conductor then (12 + 4 = 16) gm + 1 gm → 4 gm of H 2
graphite.
3 (c) Punjab has the largest share of Schedule ∴16 gm will produce = 4 gm of H 2
(d) It has different carbon to carbon
Caste people in its population at 31.9%. ∴1 gm will produce
distance in all directions.
Himachal Pradesh and West Bengal follow
Punjab with 25.2% and 23.5%. Uttar Pradesh 3 (c) (i) Diamond → It has a crystalline lattice. 4
= = 025
. gm of H 2
has 20.7% Schedule Caste people in its l In diamond, each carbon atom 16
population. undergoes sp3 hybridisation and linked (iv) C 12H 24 + 6O 2 → 12CO + 12H 2
23. Which one of the following states has to four other carbon atoms.
more than two major ports? l The structure extends in space and (144 + 24 = 168) gm +1 gm → 24 gm of H 2
(a) Maharashtra (b) West Bengal produces a rigid three-dimensional 168 gm will be produced = 24 gm of H 2
(c) Odisha (d) Tamil Nadu network of carbon atom.
24
3 (d) Tamil Nadu has more than two major l In this structure, directional covalent ∴1 gm will be produced =
ports i.e., Chennai Port, Tuticorin Port and bonds are present throughout the 168
Ennore Port. Maharashtra has two major ports lattice. = 0142
. gm of H 2
i.e., Jawaharlal Nehru Port Trust and Mumbai
Port. Both West Bengal and Odisha have one
l There are no free electrons, in the
structure of Diamond. Therefore, it is a
28. When a convex lens produces a real
major port.
image of an object, the minimum
very poor electric conductor.
24. The equivalent weight of Ba(OH)2 is distance between the object and image
(ii) Graphite is equal to
(given, atomic weight of Ba is 137.3)
l Graphite has layered structure. (a) the focal length of the convex lens
(a) 85.7 (b) 137.3
l Its layers are held by Van der Waals (b) twice the focal length of the convex
(c) 154.3 (d) 171.3
forces. lens
3 (a) The equivalent weight is that weight which
will furnish 1 mole of H + or 1 mole of OH − ions.
28 CDS Solved Paper 2018 (II)

(c) four times the focal length of the 3 (b) Boiling water can quickly entinguish fire 3 (c) After the childbirth, the first yellow and thick
convex lens more quickly because the process of fire milk produced by a lactating mother is known as
(d) one half of the focal length of the extinguishing involves absorption of heat. colostrum. It contains IgA antibodies in
Absorption of heat in converting hot water to abundant, which are essential for the
convex lens
steam is more than the heat absorbed in development of immune response against many
3 (c) Let s be the distance heating cold water to the boiling temperature. diseases in new born baby.
1 1 1 Hence, boiling water can extinguish fire more
+ = 36. Which one of the following statements
u s−u f quickly than ice, cold water and hot water.
explains higher mutation rate and faster
⇒ f( s − u ) + fu − u ( s − u ) = 0 32. In which of the following, heat loss is evolution found in RNA virus?
⇒ fs − fu + fu − su + u 2 = 0 primarily not due to convection? (a) RNA is relatively unstable compared to
⇒ fs − su + u 2 = 0 (a) Boiling water DNA.
u (b) Land and sea breeze
⇒ u 2 = (u − f )s or s = (b) Virus can multiply only within the living
u −f (c) Circulation of air around blast furnace cell of a host.
ds (u − f ) (2u ) − u 2 (d) Heating of glass surface of a bulb (c) Metabolic processes are absent in
On differentiating, = due to current in filament virus.
du (u − f )2
3 (d) Convection is the process of heat transfer (d) Virus can remain latent for a long
(u 2 − 2uf ) u(u − 2 f )
= = =0 of the bulk movement of molecules within fluids period.
(u − f )2 (u − f )2 such as gases and liquids. In this process, the
3 (a) RNA is relatively unstable compared to
or u (u − 2 f ) = 0 ⇒ u = 0, or u = 2 f heat transfers is by the actual motion of matter.
DNA, which explains higher mutation rate
It happens in liquid and gases. Boiling water,
Thus, image and object is 2(2 f ) = 4f land and sea breeze, circulation of air around
and faster evolution found in RNA virus. The
main reasons of unstability of RNA are
Hence, the minimum distance between the blast furnace, etc are the example of
presence of ribose, sugar and single strand
object and image is equal to four times the convection. Whereas heating of glass surface of
a bulb due to current in filament occurs due to structure.
focal length of the convex lens.
conduction of heat. 37. Which one of the following is the
29. The direction of magnetic field at any 33. Which one of the following features correct ascending sequence of states
location on the earth’s surface is with regard to percentage of urban
is an indication for modification of
commonly specified in terms of population (2011)?
stem of a plant?
(a) field declination (a) Tamil Nadu–Mizoram–Goa–
(a) Presence of ‘eye’ on potato
(b) field inclination Maharashtra
(b) ‘Scale’ found in onion
(c) both field declination and field (b) Goa–Mizoram–Maharashtra–Kerala
(c) ‘Tendril’ found in pea
inclination (c) Maharashtra–Kerala–Mizoram–Goa
(d) ‘Hair’ present in carrot
(d) horizontal component of the field (d) Mizoram–Goa–Maharashtra–Kerala
3 (a) Potato is a modified stem, known as
3 (d) It is termed as horizontal component of 3 (c) Percentage of urban population in Goa
tuber. The eye of potato is a very small axillary
the field. At a place, it is defined as the is 62.17%, Mizoram is 51.51% , Maharashtra is
bud, which contains small internodes and
component of earth’s magnetic field along the
nodes. It can give rise to a new plant via asexual 45.23% and Kerala is 47.72%.
horizontal in the magnetic meridian.
reproduction. The tendrils of pea and scales of
Its value is different at different places. Field onion are modification of leaves, while hairs on 38. Which one of the following places
declination is the acute angle between magnetic carrot are part of root. does not fall on leeward slope?
meridian and geographical meridian at a place. (a) Pune (b) Bengaluru
Inclination is the angle which resultant earth’s 34. Which of the following roles is/are (c) Leh (d) Mangaluru
magnetic field at a place makes with the played by epididymis, vas deferens,
horizontal surface. seminal vesicles and prostate in male 3 (d) Leeward slope is the part of a mountain that
does not face the wind i.e. the slope opposite the
30. A circuit has a fuse having a rating of reproductive system of human?
windward slope. Mangaluru does not fall on
5 A. What is the maximum number of (a) Spermatogenesis and maturation of leeward slope. Mangaluru is situated on the West
100 W-220 V bulbs that can be safely sperms coast of India and is bounded by the Arabian Sea
connected in parallel in the circuit? (b) Maturation and motility of sperms to its West and the Western Ghats to its East.
(c) Spermatogenesis and motility of Mangaluru experiences moderate to gusty winds
(a) 20 (b) 15 (c) 11 (d) 10
sperms during day time and gentle winds at night.
3 (c) Given, (d) Motility of sperms only
P = 100 W (power of 1 bulb) 39. South Arcot and Ramanathapuram
3 (b) Epididymis, vas deferens, seminal vesicles receive over 50 percent of their annual
V = 220 Volt I = 5A and prostate in male reproductive system of rainfall from which one of the
Let, x be maximum number of bulbs that can be human are involved in maturation and motility of following?
safely connected in parallel circuit. sperms. Spermatogenesis occurs in testes.
(a) South-West monsoon
Potential difference ( V ) = 220 volt 35. Which one of the following is the (b) North-East monsoon
P = VI special type of milk produced by a (c) Bay of Bengal branch of summer
100 × x = 220 × 5 lactating mother, essential for the monsoon
220 × 5 (d) Western disturbances
x= = 11 development of immune response of
100 newborn baby in human? 3 (b) Tamil Nadu receives rainfall in the winter
31. Which one of the following can (a) Breast milk produced after a month season due to North-East trade winds. The
of childbirth normal annual rainfall of the state is about
extinguish fire more quickly? 945 mm (37.2 in) of which 48% is through the
(a) Cold water (b) Boiling water (b) Transitional milk
North-East monsoon and 32% through the
(c) Hot water (d) Ice (c) Colostrum South-West monsoon.
(d) Mineralised milk
CDS Solved Paper 2018 (II) 29

40. The Eight Degree Channel separates (a) Intensification of cropping over 48. Match List I with List II and select the
which of the following? already cultivated land correct answer using the code given
(a) India from Sri Lanka (b) Increasing cultivable area by bringing below the Lists
(b) Lakshadweep from Maldives cultivable and fallow land under
plough List I List II
(c) Andaman from Nicobar Islands
(d) Indira Point from Indonesia (c) Using High Yielding Varieties (HYV) (Major Dam) (State)
seeds A. Cheruthoni Dam 1. Madhya Pradesh
3 (b) Lakshadweep islands are separated from (d) Switching over from cash crops to
Maldives by Eight Degree Channel. Minicoy is B. Indira Sagar Dam 2. Tamil Nadu
food crops
separated from rest of the Lakshadweep by C. Krishnaraja 3. Karnataka
Nine Degree Channel. 3 (c) Use of High Yielding Varieties (HYV) Sagar Dam
seeds were not a part of strategies followed by
41. Match List I with List II and select the Government of India to increase food grain D. Mettur Dam 4. Kerala
correct answer using the code given production in India immediately after
below the Lists independence. HYV seeds are not used in India Codes
till beginning of Green Revolution. A B C D A B C D
List I List II
(Classification of (Example) 44. Which one of the following is a (a) 2 1 3 4 (b) 2 3 1 4
Town) West-flowing river? (c) 4 3 1 2 (d) 4 1 3 2
A. Industrial Town 1. Vishakhapatnam (a) Mahanadi (b) Godavari
3 (d) The Mettur Dam is the largest in Tamil
(c) Krishna (d) Narmada Nadu located across the river Cauvery.
B. Transport Town 2. Bhilai
3 (d) Narmada is the largest West-flowing river The Cheruthoni Dam, located in Idukki district,
C. Mining Town 3. Singrauli
of the Peninsular India. Narmada flows Kerala, India is a 138m tall concrete gravity dam.
D. Garrison 4. Ambala Westwards through a rift valley between the The Indira Sagar Dam is a multipurpose project of
Cantonment Vindhyan Range on the North and the Satpura Madhya Pradesh on the Narmada river.
Town Range on the South. Krishnaraja Sagar, also popularly known as KRS,
45. Khasi language is included in is a lake and the dam that creates it. They are close
Codes
to settlement of Krishnaraja Sagar in the Indian
A B C D A B C D (a) Munda branch of Austro-Asiatic state of Karnataka.
(a) 2 1 3 4 (b) 2 3 1 4 sub-family
(c) 4 3 1 2 (d) 4 1 3 2 (b) Mon-Khmer branch of Austro-Asiatic 49. Which one among the following Union
sub-family Territories of India shares the shortest
3 (a) Industrial Town-Bhilai, Garrison (c) North Assam branch of Sino-Tibetan length of National Highways?
Cantonment Town-Ambala, Transport Town-
family (a) Chandigarh (b) Delhi
Vishakhapatnam, Mining Town-Singrauli
(d) Assam-Myanmari branch of (c) Daman and Diu
42. Which of the following statements Sino-Tibetan family (d) Dadra and Nagar Haveli
with regard to the land use situation 3 (b) Khasi is an Austro-Asiatic language 3 (a) Length of National Highway in Chandigarh
in India is/are correct? spoken primarily in Meghalaya state in India by is 15km, in Delhi is 80km, Daman and Diu is 22
1. There has been a tremendous the Khasi people. It is also spoken by a sizable km Dadar and Nagar Haveli is 31km.
decline in area under forest in population in Assam and Bangladesh. Khasi is
part of the Austro-Asiatic language family and is 50. Which one among the following passes
recent years. related to Cambodian and Mon languages links Lhasa with Ladakh?
2. The rate of increase in land use of Southeast Asia and the Munda branch of (a) Lanak La
in recent years is the highest in that family which is spoken in East-central
(b) Burzil
case of area under India.
(c) Babusar
non-agricultural use. 46. The headquarters of Metro Railway (d) Khyber
3. Land use such as barren and Zone is located in 3 (a) Lanak La is a well-established frontier
wasteland, area under pastures (a) New Delhi (b) Mumbai point between Ladakh and Tibet. It is on the
and tree crops have experienced (c) Kolkata (d) Chennai Southeastern boundary of the Aksai Chin region
decline in recent year. 3 (c) Apart from the headquarters of the that is controlled by China.
Select the correct answer using the Eastern and the South Eastern Railways, 51. According to the latest Reserve Bank
code given below. Kolkata also has the headquarters of the of India study on State finances, capital
(a) 1 only (b) 1 and 2 only Kolkata Metro Railways which is now a zone of
the Indian Railways. spending is maximum on
(c) 2 and 3 only (d) 1, 2 and 3
(a) rural development
3 (c) India’s tree and forest cover has 47. Which one among the following is (b) water supply and sanitation
registered an increase of 1% or 8,021 sq. not a tributary of river Luni? (c) urban development
km in two years since 2015, according to the (a) Khari (b) Sukri (d) education
latest assessment by the government. Land (c) Jawai (d) Banas
devoted to non-agricultural use has 3 (c) Capital outlay by all states is expected to
increased three times since independence. It 3 (d) Major tributaries of Luni river are the Sukri, touch a staggering ` 5.37 trillion in 2018-19 , up
is set to increase further and faster. Mithri, Bandi, Khari, Jawai, Guhiya and Sagi from ` 4.7 trillion in 2017-18 (RE), revealed by the
from the left and the Jojari river from the right. latest Reserve Bank of India (RBI) study on state
43. Which one of the following was not a The Luni river begins near Ajmer in the Pushkar finances. The spending amounts to 2.9 per cent
part of the strategies followed by the valley of the Western Aravalli Range at an of Gross Domestic product (GDP). Under
Government of India to increase food elevation of about 550 m. AMRUT scheme, states have to spend large
grain production in India immediately The Banas is a river of Rajasthan state in Western amounts on urban development. Thus, capital
India. It is a tributary of the Chambal river. spending is maximum on urban development.
after independence?
30 CDS Solved Paper 2018 (II)

52. According to the World Bank’s Doing 3 (a) The natural rate of unemployment is the 60. In the first century AD, which among
Business Report, 2018, India’s ranking name that was given to a key concept in the the following was not a major item of
study of economic activity by Milton Friedman.
has improved in 2018 as compared to Indian exports to Rome?
The natural rate of unemployment is the
2017 in which of the following areas? difference between those who would accept a (a) Pepper (b) Spikenard
1. Paying taxes job at the current wage rate and those who are (c) Tortoise shell (d) Nutmeg
2. Resolving insolvency able and willing to take a job. 3 (d) In the first century AD, periplus of Erythrean
sea, provides vivid accounts of Indo-Roman
3. Starting a business 55. The Harappan site at Kot Diji is close trade relations . It was trade between the Indian
4. Getting electricity to which one of the following major subcontinent and the Roman Empire in Europe
Select the correct answer using the sites of that civilization? and the Mediterranean. In India, the ports of
code given below. (a) Harappa (b) Mohenjo-daro Barbaricum (modern Karachi), Barygaza, Muziris
(a) 1 only (b) 1 and 2 only (c) Lothal (d) Kalibangan and Arikamedu on the Southern tip of India acted
as the main centres of that trade. Items exported
(c) 1, 2 and 3 (d) 2, 3 and 4 3 (b) Kot Diji is located in the vicinity of several
from these places are spikenard, costus, cotton
3 (b) India improved its ranking from other important historic sites. It sits to the East
cloth of all kinds, silk cloth, mallow cloth, yarn,
100th position in 2017 to 77th position in 2018 of Mohenjo-daro, a group of mounds that
long pepper etc. Tortoiseshell receives
(a jump of 23 positions) among 190 countries. contain the remains of largest city of the Indus
more mention in the Periplus than any other
civilization.
Also India has improved its rank by 53 positions object of trade.
in the last two years and 65 positions in the last 56. The story Gandatindu Jataka was 61. Which of the following statements
four years [2014 – 2018]. written in which language? relating to the Government of India
World Bank’s Ease of Doing Business Index (a) Sanskrit (b) Telugu
ranks 190 countries based on 10 parameters, Act, 1858 is/are correct?
(c) Tamil (d) Pali
including starting a business, construction 1. The British Crown assumed
permits, getting electricity, getting credit, paying 3 (d) The Jatakas were written in Pali around
the middle of the first millennium CE. One
sovereignty over India from the
taxes, trade across borders, enforcing contracts, East India Company.
and resolving insolvency. story known as the Gandatindu Jataka
describes the plight of the subjects of a 2. The British Parliament enacted the
53. The Fourteenth Finance Commission wicked king. These included elderly women first statute for the governance of
assigned different weights to the and men, cultivators, herders, village boys
and even animals. When the king went in India under the direct rule of the
following parameters for distribution disguise to find out what his subjects British.
of tax proceeds to the states thought about him, each one of them cursed 3. This Act was dominated by the
1. Income distance him for their miseries. To escape from this principle of absolute imperial
situation, people abandoned their village and
2. Population went to live in the forest. control without any population
3. Demographic changes participation in the administration
4. Area
57. According to the Tamil Sangam texts, of the country.
who among the following were the
Arrange the aforesaid parameters in Select the correct answer using the
large landowners?
descending order in terms of their code given below.
(a) Gahapatis (b) Uzhavars
weights. (a) 1 and 2 only (b) 2 only
(c) Adimais (d) Vellalars
(a) 1-2-3-4 (b) 1-2-4-3 (c) 1, 2 and 3 (d) 1 and 3 only
3 (d) Vellalars were, originally an elite caste of
(c) 1-3-2-4 (d) 4-3-2-1
Tamil agricultural landlords in Tamil Nadu , 3 (c) The Government of India Act, 1858 was an
3 (b) The 14th Finance Commission of India is Kerala states in India and in neighbouring Sri Act of the British Parliament that transferred the
constituted in the Chairmanship of the former Lanka. government and territories of the East India
RBI Governor Mr. Y.V. Reddy for the period of Company to the British Crown. The Company’s
April 1, 2015 to March 31, 2020. The 14th 58. According to the Manusmriti, women rule over British territories in India came to an end
Finance Commission has recommended a can acquire wealth through which of and it was passed directly to the British
record 10% increase in the states’ share in the government. Queen Victoria, who was the
the following means? monarch of Britain, also became the sovereign of
Union taxes to 42% as compared to the (a) Purchase (b) Investment
13th Finance Commission. The total devolution British territories in India as a result of this Act.
(c) Token of affection (d) Inheritance This act was dominated by the principle of
to states during the five year (2015-20) period
will be ` 39.48 lakh crore. Criteria and weights 3 (d) Manusmriti provides a woman with absolute imperial control without any popular
for the horizontal distribution of the tax is as property rights to six types of property in verses participation in administration of the country.
follows: 9.192-9.200. One of them is through Under this act, all the powers of Crown were
inheritance. exercised by Secretary of State of India and was
Criteria Weight (%) assisted by Council of India (Council of fifteen
59. The dialogue on Varna between King members).
1. Income Distance 50
Avantiputta and Kachchana, a
2. Population (1971) 17.5 disciple of Buddha, appears in which 62. Which of the following statements
3. Area 15 relating to the Indian Councils Act,
one of the following Buddhist texts?
1861 is/are correct?
4. Demographic Change (2011) 10 (a) Majjhima Nikaya (b) Samyutta Nikaya
(c) Anguttara Nikaya (d) Ambattha Sutta 1. The Act introduced a grain of
5. Forest Cover 7.5
popular element by including
3 (a) The story, based on a Buddhist text in Pali
non-official members in the
54. The natural rate of unemployment known as the Majjhima Nikaya, is part of
adialogue between a king named Avantiputta Governor-General’s Executive
hypothesis was advocated by
and a disciple of the Buddha named Kachchana. Council.
(a) Milton Friedman (b) A. W. Phillips
While it may not be literally true, it reveals 2. The members were nominated
(c) J. M. Keynes (d) R. G. Lipsey Buddhist attitudes towards varna. and their functions were confined
CDS Solved Paper 2018 (II) 31

exclusively to consideration of was unanimously adopted on 22nd of January, from the Light Combat Aircraft (LCA)
legislative proposals placed 1947. It laid down the fundamentals and programme, which began in the 1980s to replace
philosophy of the constitutional structure. The India’s ageing MIG-21 fighters. In 2003, the LCA
before it by the Governor- Constituent Assembly declared it’s firm resolve was officially named ‘Tejas’.
General. to proclaim India an Independent Sovereign
3. The Governor-General did Republic and draw up a Constitution for her 67. As per the extant policy, Foreign Direct
not have effective legislative governance. Ltd Investment is permitted in the defence
power. sector under the automatic route upto
64. The 2+2 Bilateral Dialogue was held which one of the following limits?
Select the correct answer using the in September 2018 between (a) 26 per cent (b) 74 per cent
code given below. (a) External Affairs and Defence Ministers (c) 51 per cent (d) 49 per cent
(a) 1 and 2 only (b) 2 and 3 only of India with their US counterparts
3 (d) On 10th November, 2017 the government
(c) 1, 2 and 3 (d) 1 only (b) Finance and Defence Ministers of relaxed Foreign Direct Investment (FDI) norms in
3 (a) The Indian Councils Act, 1861 was India with their Russian counterparts the defence sector by allowing FDI upto 49 per
passed by British Parliament on 1st August, (c) Home and Defence Ministers of India cent under automatic route and beyond that
1861 to make substantial changes in the and their counterparts in Pakistan through the FIPB’s approval. The government has
composition of the Governor-General council. (d) External Affairs and Defence also done away with the earlier requirement of
Ministers of India with their mandatory permission from the Cabinet
l The executive council of Governor-
counterparts in Pakistan. Committee on Security (CCS) beyond 49 per cent.
General was added a fifth finance
member. For legislative purposes, the 3 (a) The first edition of 2+2 dialogue between 68. The policy on strategic partnership in
Governor-General’s Council was India-US held in New Delhi. In the inaugural defence was approved by the Ministry
enlarged. Now, there were to be between meeting, Minister of External Affairs (MEA) of Defence in May 2017. Which of the
Sushma Swaraj and Minister of Defence Smt following is not among the four
6 and 12 additional members (nominated
Nirmala Sitharaman met with their US
by the Governor-General). counterparts Secretary of State Mr. Michael R.
segments identified by the Ministry for
l There were appointed for a period of Pompeo and Secretary of Defence Mr. James acquisition through the strategic
2 years. Out of these, atleast half of the N. Mattis. They welcomed the launch of the 2+2 partnership route?
additional members were to be Dialogue as a reflection of the shared (a) Artillery guns
non-official (British or Indian). commitment by Prime Minister Shri Narendra (b) Fighter aircraft and helicopters
Modi and President Mr. Donald Trump to (c) Submarines
l Their functions were confined to
provide a positive, forward-looking vision for the (d) Armoured fighting vehicles and main
legislative measures. India-US strategic partnership and to promote
l Any bill related to public revenue or debt, battle tanks
synergy in their diplomatic and security efforts.
military, religion or foreign affairs could They resolved to continue meetings in this 3 (a) The policy on Strategic Partnerships in
not be passed without the Governor- format on an annual basis. Defence sector was approved by Defence
General’s assent. Acquisition Council (DAC) in May, 2017.
65. Who is the Chairman of the Defence The policy is intended to institutionalise a
l The Viceroy had the power to overrule the Planning Committee set up in April transparent, objective and functional mechanism
council, if necessary. 2018? to encourage broader participation of the private
l The Governor-General also had the power (a) The Prime Minister sector, in addition to DPSUs / OFB, in the
to promulgate ordinances without the (b) The National Security Advisor manufacture of defence platforms and
council’s concurrence during (c) The Defence Minister equipment such as aircraft, submarines,
emergencies. helicopters and armoured vehicles. It will serve
(d) The Chief of the Army Staff
to enhance competition, increase efficiencies,
63. Which of the following statements 3 (b) On April 18, 2018, the government has facilitate faster and more significant absorption of
relating to the historic objectives revamped the existing defence planning system technology, create a tiered industrial ecosystem,
resolution, which was adopted by the by establishing a Defence Planning ensure development of a wider skill base and
constituent assembly, is/are correct? Committee(DPC) under the Chairmanship of the trigger innovation, leading to reduction in
National Security Adviser (NSA). The committee dependence on imports and greater self-reliance
1. The Objectives Resolution will be a permanent body and it will prepare in meeting national security objectives. The
inspired the shaping of the national security strategy besides undertaking following four segments have been identified for
Constitution through all its strategic defence review and formulating acquisition under Strategic Partnership (SP)
subsequent stages. international defence engagement strategy. It route:
will consist of Chairman Chiefs of the Staff l Fighter Aircraft l Helicopters
2. It was not just a resolution, but a
Committee (COSC), service chiefs, Defence
declaration, a firm resolve and a Secretary, Foreign Secretary and Secretary l Submarines
pledge. (expenditure) in Finance Ministry. l Armoured Fighting Vehicles (AFVs)/ Main
3. It provided the underlying Battle Tanks (MBTs).
66. ‘Tejas’ is the name of which one of
philosophy of our Constitution.
the following? 69. The acronym ‘CAATSA’ refers to a
Select the correct answer using the (a) Main battle tank piece of legislation enacted by which
code given below. (b) Nuclear submarine one of the following countries?
(a) 1 and 2 only (b) 1 only (c) Light combat aircraft (a) United Kingdom
(c) 1, 2 and 3 (d) 2 and 3 only (d) Aircraft carrier (b) United States of America
3 (c) Objectives Resolution’ was the resolution (c) Russia
3 (c) Tejas is an Indian single-seat, single-jet
moved by Jawaharlal Nehru on the 13th of engine, multirale light fighter designed by (d) India
December, 1946 in the 1st Session of India’s the Aeronautical Development Agency (ADA) 3 (b) The Countering America’s Adversaries
Constituent Assembly, which was charged with and Hindustan Aeronautics Ltd (HAL) for Through Sanctions Act, CAATSA is a United
the objective to frame India’s Constitution. It the Indian Air Force and Indian Navy. It came States federal law that imposed sanctions on
32 CDS Solved Paper 2018 (II)

Iran, North Korea and Russia. CAATSA is a level of 3 per cent for the third 76. Who among the following European
specifically enacted legislation. Its “ultimate successive year.
goal”, in the words of a senior State Department
travellers never returned to Europe
official, “is to prevent revenue from flowing to Statement II Special Category States and settled down in India?
the Russian Government.” had run up a higher level of fiscal (a) Duarte Barbosa (b) Manucci
deficit in 2017-2018 compared to (c) Tavernier (d) Bernier
70. Who among the following is the 2016-17.
Convener of the ‘Task Force’ set up in 3 (b) Niccolao Manucci (19 April, 1638–1717)
3 (b) State governments in aggregate were to was an Italian writer and traveller. He wrote a
November 2017 by the Government of memoir about the Indian subcontinent during
revert to well below 3% fiscal deficit threshold in
India to review the Income-tax Act the Mughal era. His records have been a source
Financial Year 2017-18. This year saw a change
and draft a new direct tax law? from the previous few years with all deficit of history about Shah Jahan, Aurangzeb, Dara
(a) Girish Ahuja (b) Mukesh Patel indicators worsening for special category states Shikoh, Shah Alam, Raja Jai Singh and Kirat
(c) Arbind Modi (d) Mansi Kedia than that of non-special category states and Singh. He spent almost his entire life in India.
most special category states recording GFDs Italian doctor Manucci, never returned to Europe,
3 (c) The finance ministry sets up a six-member and settled down in India.
task force to draft a new direct tax law that will above the 3% mark.
better serve the country’s economic needs by 73. Statement I There has been a sharp 77. The class of Amar Nayakas in
widening the tax base, improving compliance Vijayanagara is a reference to which of
decline in savings rate in Indian
and ease of doing business. Arbind Modi, the following?
member, Central Board of Direct Taxes, was economy between 2007-2008 to
2015-2016. (a) Village Chieftains
named convener of the six-member panel that
has been tasked to draft a new law. Modi was (b) Senior Civil Servants
Statement II There has been a fall in (c) Tributary Chiefs
also a key contributor to the direct taxes code
proposed by the previous United Progressive
household and public savings. (d) Military Commanders
Alliance Government. 3 (c) Gross domestic savings have been falling 3 (d) The Amara Nayaka system was a major
71. With regard to the cabinet decision in as a percentage of GDP. However, there has political innovation of the Vijayanagara Empire.
been rise in household and public savings Nayakas were military chiefs usually maintained
July 2018, the percentage increase in
recorded. law and order in their areas of control. They
Minimum Support Price (MSP) is maintained forests and kept armed supporters.
maximum in which one of the 74. Statement I Private investments in They use to control and expand fertile land and
following crops? research have severely lagged public agricultural settlements.
(a) Jowar (Hybrid) (b) Bajra investments in India.
(c) Maize (d) Soya bean
78. The important source for Akbar’s
Statement II Universities play a reign, Tarikh-i-Akbari was written by
3 (a) Giving a major boost for the farmers’ relatively small role in the research which one of the following Persian
income, the Cabinet Committee on activities of the country. language scholars?
Economic Affairs chaired by Prime Minister
Shri Narendra Modi has approved the 3 (c) According to the Economic Survey 2017- (a) Arif Qandahari
increase in the Minimum Support Prices 18, India under-spends on R&D even relative to (b) Bayazid Bayat
(MSPs) for all kharif crops for 2018-19 season. its level of development. In India, the (c) Abdul Qadir Badauni
Among these, the maximum increase in the government is not just the primary source of (d) Nizamuddin Ahmad
MSP is MSP of jowar which is from 1700 to R&D funding but also its the primary user of
3 (a) Tarikh-i-Akbari, also known as
` 2430. these funds. Tarikh-i-Qandahari, or Muzaffar Nama, the first
India’s spending on R&D (about 0.6% of GDP) is chronicle of Emperor Akbar’s reign is unaimously
Directions (Q. Nos.72-75) The following well below that in major nations such as the US important source of information on the 16th
four (4) items consists of two statements, (2.8%), China (2.1%), Israel (4.3%) and Korea century history of India; particularly the formative
Statement I and Statement II. Examine (4.2%). Private investments in research have years of his career. It was written by Arif
these two statements carefully and severely lagged public investments in India. The Qandahari.
select the correct answer using the code Survey highlighted that universities play a Though the major pre-occupation of modern
given below. relatively small role in the research activities of the scholars has been with Abul Fazl’s Akbar Nama
country. and Badauni’s Muntakhab-ut-Tawarikh, it
Code
supplements in many ways of the former’s works
(a) Both the statements are 75. Statement I Agriculture in India still while Farishta in his Gulshan-i-Ibrahimi and
individually true and Statement II accounts for a substantial share in Nihawandi in his Maasir-i-Rahimi explicitly
is the correct explanation of total employment. acknowledge their indebtedness to it.
Statement I. Statement II There has been no 79. The aristocrat Muqarrab Khan was a
(b) Both the statements are decline in volatility of agricultural great favourite of which Mughal
individually true and Statement II growth in India. Emperor?
is not the correct explanation of 3 (c) The Indian economy has moved (a) Akbar (b) Jahangir
Statement I. decisively to a higher path of growth in (c) Farrukhsiyar (d) Shah Alam
(c) Statement I is true but Statement Agriculture’s share of total employment has 3 (b) Muqarrab Khan was favourite of Mughal
II is false. declined. It is still a dominant source of emperor Jahangir. Muqarrab Khan of
employment, and presently it accounts for 52 Golconda (titled Khan-Zaman Fath Jang) was the
(d) Statement I is false but Statement per cent of the country’s total labour force. The most experienced commander in Golconda,
II is true. declining trend in agricultural growth has during the reign of Abul Hasan Qutb Shah.
emerged as a major concern for researchers Muqarrab Khan is known to have been an ally of
72. Statement I The overall fiscal deficit Afzal khan and defended Golconda’s Southern
of the States in India during 2017-18 and policymakers but volatility in agriculture has
realms against Maratha raids. Aurangzeb sent
stayed above the FRBM threshold declined significantly in India. the Golconda noble Muqarrab Khan to hunt
CDS Solved Paper 2018 (II) 33

down and kill Shambhaji. Later in 1687, 83. Which one of the following Articles 3 (c) The 10th edition of DEFEXPO has been
Aurangzeb ordered Mu’azzam (Shah Alam) to held in Chennai from 11 to 14 April, 2018. The
march against the sultanate of Golconda.
of the Constitution of India deals with location of the event is Tiruvidanthal,
Meanwhile he came close to Muqarrab Khan. the special provision with respect to Kancheepuram district on the East Coast Road
the State of Assam? near Chennai, Tamil Nadu. DEFEXPO 2018, for
80. Who was the first Nawab Wazir of (a) Article 371A (b) Article 371B the first time, project India’s Defence
Awadh in the 18th Century? (c) Article 371C (d) Article 371D manufacturing capabilities to the world. This is
(a) Nawab Safdarjung reflected in the tagline for the Expo, ‘India: The
3 (b) The Article 371B of Indian Constitution Emerging Defence Manufacturing Hub’.
(b) Nawab Saadat Ali Khan has special provision with respect to the State of
(c) Nawab Shuja-ud-Daula Assam. It is mostly related to Sixth schedule 87. The two defence industrial corridors
(d) Nawab Saadat Khan and functioning of Legislative Assembly and announced by the Finance Minister in
3 (b) Saadat Khan Burhanul Mulk or Saadat Ali Sixth schedule. his 2018 Budget speech are coming up
Khan was appointed Nawab in 1722 and 84. Provisions of which one of the in which of the following states?
established his court in Faizabad near Lucknow.
following Articles of the Constitution (a) Odisha and West Bengal
He took advantage of a weakening Mughal
Empire in Delhi to lay the foundation of the of India apply to the State of Jammu (b) Punjab and Haryana
Awadh dynasty. and Kashmir? (c) Gujarat and Maharashtra
(a) Article 238 (b) Article 370 (d) Uttar Pradesh and Tamil Nadu
81. According to the French traveller
(c) Article 371 (d) Article 371G 3 (d) The government will develop two defence
Tavernier, the majority of houses in
Varanasi during the 17th century 3 (b) Article 370 of the Indian Constitution is an industrial production corridors and bring out an
article that grants special autonomous status to industry-friendly military production policy to
were made of promote the defence industry in Uttar Pradesh
the State of Jammu and Kashmir. The article is
(a) brick and mud and Tamil Nadu. The first of the two corridors will
drafted in Part XXI of the Constitution, which
(b) stone and thatch relates to Temporary, Transitional and Special be constructed between Chennai and Bengaluru,
(c) wood and stone Provisions. linking Kattupalli port, Chennai, Tiruchi,
(d) brick and stone Coimbatore and Hosur. The second industrial
85. Which one of the following Schedules corridor will link Agra, Aligarh, Lucknow, Kanpur,
3 (a) In 1665, the French traveller Jean Baptiste
to the Constitution of India provides Jhansi and Chitrakoot.
Tavernier described the architectural beauty of
the Vindu Madhava temple on the side of the for setting up of Autonomous District It will be constructed keeping in mind the
Ganges. According to him, the majority of Councils? development requirements of Bundelkhand
houses in Varanasi during the 17th century were (a) Third Schedule (b) Fourth Schedule region.
made of brick and mud. (c) Fifth Schedule (d) Sixth Schedule The government had earlier announced that it
would develop two such corridors.
82. Which of the following statements 3 (d) The Constitution of India makes special
relating to the duties of the Governor provisions for the administration of the tribal 88. What is India’s first Indigenous
is/are correct? dominated areas in the four states viz. Assam, Aircraft Carrier (IAC) called?
Meghalya, Tripura and Mizoram. Though these (a) Vikrant (b) Virat
1. The duties of the Governor as a areas fall within the executive authorities of the
Constitutional Head of the State (c) Vaibhav (d) Varaha
state, provision has been made for the creation
do not become the subject matter of the district councils and regional councils for 3 (a) INS Vikrant (IAC-I) is the first indigenous
of questions or debate in the the excercise of the certain legislative and aircraft carrier built in India and the first Vikrant
judicial powers. class aircraft carrier built by Cochin shipyard
Parliament.
Each district is an autonomous district and (CSL) in Kochi, Kerala for the Indian Navy. The
2. Where the Governor takes a motto of the ship is Jayema Sam Yudhi Sprdhah,
Governor can modify/divide the boundaries of
decision independently of his the said Tribal areas by notification. Currently, which is taken from Rigveda 1.8.3 and can be
Council of Ministers or where he there are ten such councils in the region. translated as “I defeat those who fight against
acts as the Chief Executive of the The Constitution of India makes special me”. India’s first domestically built aircraft carrier
provisions for the administration of the tribal was built in India under the so-called Indigenous
State under President’s rule, his Aircraft Carrier (IAC) program.
actions are subject to scrutiny by dominated areas in four states viz. Assam,
Meghalaya, Tripura and Mizoram. As per Article 89. Which one of the following
the Parliament.
244 and 6th Schedule, these areas are called manufacturers is engaged in upgradation
Select the correct answer using the “Tribal Areas”.The Sixth Schedule envisages
establishment of Autonomous District Councils
of the Swedish 155-mm Bofors
code given below.
(ADCs). These councils have been given Howitzer under the project ‘Dhanush’?
(a) 1 only
Legislative, Administrative and Judicial powers (a) Bharat Electronics Limited
(b) 2 only
under the Sixth schedule. No law of the Centre (b) Ordnance Factory Board
(c) Both 1 and 2
or the state in respect of the legislative powers (c) Bharat Dynamics Limited
(d) Neither 1 nor 2 conferred on the autonomous district councils (d) Mishra Dhatu Nigam
3 (b) Since Governor is appointed by the could be extended to those areas without their
President or Union Government, his duties as prior approval. The district councils are also 3 (b) Dhanush is a 155mm x 45mm
constitutional Head of the State can become the empowered to constitute village councils and calibre artillery gun and is also called the
subject matter of questions or debate in the also village courts. Desi Bofors. Dhanush will be the first artillery gun
Parliament even in that condition when the to be acquired by the army since the purchase of
Governor takes a decision independently of his 86. In which one of the following States Bofors guns from Sweden in 1987.
Council of Ministers or where he acts as the was ‘DEFEXPO 2018’ held in April It is a modified version of the Bofors upgraded by
Chief Executive of the State under Presidents 2018? Ordnance Factory Board (OFB). It is a
rule, his actions are subject to scrutiny by the conglomerate of 39 ordnance factories with
(a) Goa (b) Karnataka
Parliament. another two new projects being set up at
(c) Tamil Nadu (d) Andhra Pradesh Nalanda in Bihar and Korwa in Uttar Pradesh.
34 CDS Solved Paper 2018 (II)

90. Which one of the following is the 3 (c) Robert Fortune (1812-80) was a Scottish 98. Which one of the following regarding
botanist, plant hunter and traveller. He is best
official mascot of Tokyo 2020 known for stealing tea plants from China in the
the tenure of the elected members of
Olympic Games? employment of the British East India Company. the Autonomous District Council is
(a) Soohorang correct?
(b) Vinicius de Moraes 95. Subhas Chandra Bose started the (a) Five years from the date of election
(c) The Hare, the Polar Bear and the ‘Azad Hind Radio’ in which of the (b) Five years from the date appointed for
Leopard following countries? the first meeting of the council after the
(d) Miraitowa (a) Japan (b) Austria election
(c) Germany (d) Malaysia (c) Six years from the date of
3 (d) The mascot for the 2020 Tokyo Olympics
is named Miraitowa. Miraitowa is a combination 3 (c) Azad Hind Radio was a propaganda radio administration of oath
of the Japanese words for future and eternity. service that was started under the leadership (d) Six years from the date of election
of Netaji Subhas Chandra Bose in Germany in 3 (b) The Sixth schedule to the Constitution
91. ‘Mission Satyanishtha’, a programme 1942 to encourage Indians to fight for freedom. empowers the Governor to determine the
on ethics in public governance, was administrative areas of the autonomous councils.
launched recently by the 96. Which political party formally
Each district council or regional council provided
accepted the Cabinet Mission Plan on
(a) Indian Railways under the Sixth schedule is a corporate body by
(b) Central Bureau of Investigation
6th June, 1946, which had rejected the name of District Council or Regional Council of
(c) Supreme Court
demand for a sovereign Pakistan? (name of the district or name of the region)
(d) Enforcement Directorate (a) The Hindu Mahasabha having perpetual succession and a common seal
(b) The Congress with the right to sue and be sued. The councils
3 (a) ‘Mission Satyanishtha’ launched on consists of 30 members, 26 are elected from the
(c) The Muslim League
27th July, 2018 aims at sensitizing all railway single member constituencies on the basis of
employees about the need to adhere to good (d) The Unionist Party
adult franchise and not more than 4 persons are
ethics and to maintain high standards of 3 (c) Cabinet Mission of 1946 came to India nominated by the Governor on the advice of the
integrity at work. Mission Satyanishtha launched with an aim to discuss the transfer of power Chief Executive Member for a term of five years.
by Railway Ministry aims to adhere to good from the British Government to the Indian The tenure of the elected members of the
ethics and to maintain high standards of leadership, with the aim of preserving India’s Autonomous District Council is five years from
integrity at work. This mission is a first of its kind unity and granting it independence. The Cabinet the date appointed for the first meeting of the
event held by any government organisation. Mission Plan of 1946 proposed that there shall Council after the election. They are known as
be a Union of India which was to be empowered MDC (Member of the District Council).
92. The College of Fort William was to deal with the defense, foreign affairs and
established by which one of the communication. The Muslim League first 99. Who among the following shall cause
following Governor-Generals? approved the plan. But when Congress the accounts of the Autonomous
(a) Warren Hastings declared that it could change the scheme District and Regional Council Funds to
(b) Lord Cornwallis through its majority in the Constituent Assembly, be audited?
(c) Richard Wellesley they rejected the plan.
(a) The Comptroller and Auditor General
(d) William Bentinck 97. The elected President of the All India of India
3 (c) Fort William College (also called the Kisan Sabha, which met in (b) The Chartered Accountant empanelled
College of Fort William) was an academy and Vijayawada (1944), was by the Government of India
learning centre of Oriental studies established (a) Sahajananda Saraswati (c) The State Government Auditors
by Lord Wellesley in 1800, by the then Governor (b) Vinoba Bhave (d) Any Chartered Accountant
General of British India. Fort William College (c) Achyut Rao Patwardhan 3 (a) The district and the regional councils are
aimed at training British officials in Indian (d) Narendra Dev Responsible for framing rules for the
languages and, in the process, fostered the
3 (a) The eighth annual session of the All India management of finances with the approval of the
development of languages such as Bengali and
Kisan Sabha was held, at Governor. They are also given mutually exclusive
Urdu.
Bezwada(Vijayawada), in Andhra, on March 14 powers to collect land revenues, levy and collect
93. The Economic historian, who has used and 15, 1944, under the presidentship of Swami taxes on lands.
the data collected by Buchanan- Sahajanand Saraswati. The session was unique The District Councils enjoy autonomy and the Acts
Hamilton to support the thesis of in many respects in the history of the Sabha. of the Parliament and the State legislatures on the
deindustrialization in the 19th The total gathering at the open session subject under them do not normally apply to the
numbered about a lakh including over 10,000 autonomous districts.
century India, is women—as big as at Gaya in 1939. The Sixth schedule of the Constitution of India also
(a) Tirthankar Roy
The presidential procession on 14th morning envisages audit of accounts of district and regional
(b) Amiya Kumar Bagchi was a disciplined mass of nearly 10,000 people councils of autonomous regions by the Comptroller
(c) Sabyasachi Bhattacharya formed into a one-mile-long mobile column and Auditor General of India (CAG).
(d) Irfan Habib which marched for three hours through
Bezwada town and its outskirts while about as 100. Who has the power of annulment or
3 (a) Tirthankar Roy, a professor of London
school of Economics has given his thesis on many thousands were witnessing the arousing suspension of acts and resolutions of
de-industrial through data of Buchanan- demonstration in amazement. The main political the Autonomous District and Regional
Hamilton data. resolution was on the release of national leaders Councils?
to end the deadlock. It was of paramount (a) The Governor
94. Tea growing in India in the 19th importance to our national life to-day because
its main drive goes towards the achievement of (b) The President
century was made possible by
Hindu-Muslim (c) The Chief Minister of the State
(a) Joseph Banks
(b) James Cook and Congress-League unity leading to the (d) The Prime Minister
release of the leaders, ending the deadlock, and
(c) Robert Fortune 3 (a) Governors of four states viz. Assam,
winning a National Government of National
(d) Robert Owen Meghalaya, Tripura and Mizoram are empowered
defence.
CDS Solved Paper 2018 (II) 35
to declare some tribal dominated (c) To discuss issues in dealing with 106. Which one of the following is not
districts/areas of these states as autonomous violent disturbances in an undertaking
districts and autonomous regions by order.
among the duties of the Chief
under the control of the Union Ministers?
No separate legislation is needed for this.
Government.
The Governor also has power to include any (a) To communicate to the Governor of
(d) To discuss issues for putting down the
other area, exclude any area, increase, the State all decisions of the Council of
demands of the industrial labour
decrease, diminish these areas, unite two Ministers relating to the administration
districts/regions, and alter the names and 3 (a) To discuss state matters in general of the affairs of the State and
boundaries of these autonomous district condition does not lies under procedure and proposals for legislation
councils. Governor has the power of conduct of business of Parliament. Under article
(b) To furnish information relating to the
annulment of suspension of Acts and 246 of Constitution, Parliament has right to make
laws on Union list and Concurrent list of Schedule administration of the State and
Resolutions of the Autonomous District and
regional council. seven. Parliament can discuss the issue of Police proposals for legislation as the
action against the Schedule Castes and Tribes Governor may call for
101.The audit reports of the Comptroller under the Scheduled Castes and Tribes (c) To communicate to the President all
and Auditor General of India (Prevention of Atrocities) Act, 1989. Issue of decisions of the Council of Ministers
relating to the accounts of the Industrial labours come under Concurrent list, so relating to the administration of the
Union shall be submitted to it can be discussed in Parliament. Parliament can State in the monthly report
also discuss issues in dealing with violent
(a) the President (d) To submit for the consideration of the
disturbances in an undertaking under the control
(b) the Speaker of the Lok Sabha of the Union Government. Council of Ministers any matter on
(c) the Prime Minister which a decision has been taken by a
(d) the Vice President
104. Which of the following is not under the Minister but has not been considered
powers and functions of the Election by the Council, if the governor so
3 (a) Under Article 151 of Constitution, the Commission of India? requires
reports of the Comptroller and
Auditor-General of India relating to the (a) Superintendence, direction and control 3 (c) To communicate to the President all
accounts of the Union shall be submitted to of the preparation of electoral rolls decisions of the Council of Ministers relating to
the President, who shall cause them to be laid (b) Conduct of elections to the Parliament the administration of the State in the monthly
before each House of Parliament. and to the Legislature of each State report is the duty of Prime Minister under Article 78.
(c) Conduct of election of the office of the The Chief Minister is appointed by the Governor.
102. Which of the following is not President and the Vice President Article. 164 of the Constitution provides that there
related to the powers of the (d) Appointment of the Regional shall be a Council of Ministers with the Chief
Governor? Commissioners to assist the Election Minister at its hand to aid and advise the Governor.
(a) Diplomatic and Military powers Commission in the performance of the Under Article 167 of the Constitution, it shall be the
functions conferred on the Commission duty of the Chief Minister of each state
(b) Power of appoint Advocate
l to communicate to the Governor of the
General 3 (d) According to Article 324 of Indian
Constitution, the Election Commission of India has State all decisions of the Council of
(c) Summoning, proroguing and
superintendence, direction and control of the Ministers relating to the administration of
dissolving State Legislature
entire process for conduct of elections to the affairs of the State and proposals for
(d) Power to grant pardons, reprieves, Parliament and Legislature (state legislative legislation.
respites or remission of assembly and state legislative council) of every l to furnish such information relating to
punishments State and to the offices of President and
the administration of the affairs of the
3 (a) Governor is the Head of the State, as Vice-President of India. Appointment of the
Regional Commission in the performance of the State and proposals for legislation as
President being the head of the country. He
functions conferred on the Commission is not the Governor may call for.
enjoys executive, legislative as well as judicial
functions. Following are the powers and done by the Election Commission of India. l if the Governor so requires, to submit for
duties performed by Governor in states are the consideration of the Council of
105. Which one of the following criteria is not Ministers any matter on which a decision
l They have power to appoint Advocate required to be qualified for appointment as
has been taken by a Minister but which
General. Judge of the Supreme Court? has not been considered by the Council.
l They can summoning, proroguing and (a) At least five years as a Judge of a High
dissolving State Legislature. Court 107. Which one of the following is not
l They have power to grant pardons, (b) At least ten years as an Advocate of a considered a part of the Legislature of
reprieves, respites or remission of High Court States?
punishments. But in case of Military (c) In the opinion of the President, a (a) The Governor
power, Governor don’t have such power distinguished Jurist (b) The Legislative Assembly
as like President. (d) At least twenty years as a Sub-Judicial (c) The Legislative Council
Magistrate (d) The Chief Minister
103. Which one of the following
regarding the procedure and 3 (d) Article 124 (3) of the Constitution prescribes 3 (d) Under Article 168 of the Constitution, for
that for appointment as a Judge of the Supreme every State there shall be a Legislature which
conduct of business in the
Court a person must be shall consist of the Governor, and where there
Parliament is not correct? are two Houses of the Legislature of a State, one
l a citizen of India,
(a) To discuss State matters shall be known as the Legislative Council and the
l has been a judge of any High Court for at
other as the Legislative Assembly, and where
(b) To discuss issues of the use of least 5 years, or there is only one House, it shall be known as the
Police force in suppressing the l has been an advocate in a High Court for Legislative Assembly.
Scheduled Caste and Scheduled 10 years or is in the opinion of the Hence, Chief Minister is not the part of
Tribe communities President a distinguished jurist. Legislature.
36 CDS Solved Paper 2018 (II)

108. Which one of the following regarding 110. Which one of the following 3 (b) The Central Board of Direct Taxes has
launched an app ‘Aaykar Setu’. The app helps to
the ordinance-making power of the statements regarding the Human file Income Tax Return (ITR) online, locate the
Governor is not correct? Rights Council is not correct? nearest Tax Return Prepares (TRP), provides
(a) It is not a discretionary power. (a) It is an inter-governmental body calculators and other tools, helps manage
(b) The Governor may withdraw the within the United Nations system PAN and TDS.
ordinance anytime. made up of all members of the UN. 114. SWAYAM is
(c) The ordinance power can be (b) It is responsible for the promotion
(a) a network that aims to tap the talent
exercised when the Legislature is not and protection of all human rights
pool of scientists and entrepreneurs
in session. around the globe.
towards global excellence
(d) The aid and advice of Ministers is not (c) It replaced the former United Nations
(b) a Massive Open Online Courses
required for declaring the ordinance. Commission on Human Rights.
(MOOCs) initiative on a national platform
3 (d) The ordinance-making power of the (d) It is made up of 47 UN Member
(c) an empowerment scheme for advancing
Governor is exercised under Article 214. The States which are elected by the UN
the participation of girls in education.
Governor can issue ordinance only when two General Assembly.
(d) a scheme that supports differently
conditions are fulfilled ;
3 (a) The Human Rights Council is an abled children to pursue technical
l the governor can only issue inter-governmental body within the United education.
ordinances when the legislative Nations system made up of 47 states
3 (b) Under SWAYAM or Study Webs of Active
assembly of a state both houses in responsible for the promotion and protection of
–Learning for young aspiring minds programme
session or where there are two houses all human rights around the globe. It replaced
of Ministry of Human Resource Development,
in a state both houses are not in the former United Nations Commission on
Government of India, professors and faculties of
session. Human Rights. Hence, all UN members are not
centrally funded institutions like IITs, IIMs, central
of its part.
l the governor must be satisfied that universities will offer online courses to citizens of
circumstance exist which render it 111. Ace athlete Neerja Chopra is an India.
necessary for him to take immediate accomplished player in 115. Under the PRASAD Tourism Scheme,
action. (a) Hammer throw (b) Javelin throw which one of the following has not
The Court cannot question the validity or the (c) Shot put throw (d) Discus throw been identified as a religious site for
ordinance on the ground that there was no development?
3 (b) Neeraj Chopra is an Indian track and field
necessity or sufficient ground for issuing the
athlete, who competes in the Javelin Throw. He (a) Ajmer (Rajasthan)
ordinance by the Governor. The existence of
represented India at the 2018 Asian Games (b) Haridwar (Uttrakhand)
such necessity is not a justiciable discretionary.
where he won a gold medal, setting the (c) Somnath (Gujarat)
The exercise of ordinance-making power is not
national record of 88.06 m, and at the 2018 (d) Velankanni (Tamil Nadu)
discretionary. The Governor exercises this
Commonwealth Games where he also clinched
power on the advice of the Council of Minister. 3 (b) PRASAD (Pilgrimage Rejuvenation and
the gold medal.
He may withdraw it at any time. Spiritual Augmentation Drive) scheme has been
109. Which one of the following 112. According to the updated World launched to identify and develop pilgrimage
Bank data for 2017, India is the sixth tourist destinations on the principles of high
statements regarding the Universal tourist visits, competitiveness and sustainability
biggest economy of the world (in
Declaration of Human Rights is not to enrich the religious tourism experience. Twelve
terms of GDP). Which one of the
correct? cities namely Amaravati (Andhra Pradesh), Gaya
(a) The UN General Assembly adopted
following is not ahead of India? (Bihar), Dwaraka (Gujarat), Amritsar (Punjab),
the Human Rights Charter on 10th (a) Japan Ajmer (Rajasthan), Kanchipuram (Tamil Nadu),
December, 1948. (b) UK Vellankani (Tamil Nadu), Puri (Odisha), Varanasi
(b) Some of the provisions of the (c) France (Uttar Pradesh), Mathura (Uttar Pradesh),
(d) Germany Kedarnath (Uttarakhand) and Kamakhya (Assam)
Fundamental Rights enshrined in the
have been identified for development under
Constitution of India are similar to the 3 (c) In 2017, India became the sixth largest Pilgrimage Rejuvenation and Spirituality
provisions of the Universal economy with a gross domestic product of Augmentation Drive (PRASAD) by the Ministry of
Declaration of Human Rights. $2.60 trillion, relegating France to the seventh Tourism.
(c) The Rights to Property is not a part of position. As per the data, GDP of France stood
the Universal Declaration of Human
at $2.58 trillion. 116. Name the Indian cricketer who is not
Rights. 113. Which one of the following is correct inducted to the ICC Cricket Hall of
(d) India is a signatory to the Universal Fame (till July 2018).
about ‘Aaykar Setu’?
Declaration of Human Rights. (a) Rahul Dravid
(a) It is a mechanism for achieving (b) Sunil Gavaskar
3 (c) The Universal Declaration of Human excellence in public sector delivery (c) Sachin Tendulkar
Rights (UDHR) was adopted by the United related to GST. (d) Anil Kumble
Nations General Assembly at its third session (b) With the use of a mobile app, it
on 10th December, 1948. India is a signatory to facilitates online payment of taxes. 3 (c) The ICC Cricket ‘Hall of Fame’ recognises
the achievements of the legends of the cricket.
the Universal Declaration of Human Rights. (c) It is a communication strategy
Since 1932 till date just five Indians have been
Our Indian Constitution was greatly influenced designed to collect information and included in this list. Rahul Dravid, the former wall
by the Universal Declaration of Human Rights build a database of tax defaulters. of Indian cricket is the latest Indian player who is
specially the part three i.e. Fundamental rights. (d) It enables electronic filing and included in the list of ICC Hall of Fame in 2018.
Right to Property (Housing) is also a human processing of import and export Other are Bishan Singh Bedi, Kapil Dev, Sunil
right under this declaration. declarations. Gavaskar in 2009 and Anil Kumble (2015).
CDS Solved Paper 2018 (II) 37

117. The Central Water Commission has (a) Archaelogical Survey of India 3 (b) Eka Movement or Unity Movement is a
(b) India Tourism Development Peasant Movement which surfaced in Hardoi,
recently entered into a collaborative Bahraich and Sitapur during the end of 1921 by
agreement with which one of the Corporation
Madari Pasi, an offshoot of
following entities for flood (c) Geological Survey of India
Non-cooperation Movement. The initial thrust
(d) Geographical Indications (GI) of India
forecasting? was given by the leaders of Congress and
(a) Skymet 3 (d) Department of Industrial Policy and Khilafat movement.
(b) Google Promotion (DIPP) under Ministry of
Commerce and Industry has unveiled tricolour 120. Which organization was started at the
(c) MetService
logo for Geographical Indication (GI) certified Haridwar Kumbh Mela in 1915?
(d) AccuWeather
products. (a) Sanatan Dharma Sabha
3 (b) The Central Water Commission (CWC) The logo has tagline “Invaluable Treasures of (b) Dev Samaj
has entered into an agreement with Google to Incredible India” printed below it. (c) Brahmin Sabha
improve flood forecast systems and (d) Hindu Mahasabha
disseminate flood-related information by using 119. Who took over the ‘Eka Movement’
technology developed by the tech giant. The started by the Congress in Awadh 3 (d) Hindu Mahasabha was founded in 1915 by
initiative is likely to help crisis management during 1921-1922? Madan Mohan Malviya. It worked with Arya
agencies to deal extreme hydrological events in Samaj and other Hindu organisations. It was
(a) Bhagwan Ahir directly linked with Rashtriya Swam Sevak Sangh
a better manner.
(b) Madari Pasi founded in 1925 at Nagpur by K.B.Hegewar. The
118. The tagline ‘Invaluable Treasures of (c) Baba Ramchandra first All India Hindu Mahasabha Conference was
Incredible India’ is associated with (d) Shah Naeem Ata organised at Haridwar in 1915 during Kumbh
the logo for Mela.
01
MATHEMATICS Quadratic Equations and Inequalities 3

NUMBER SYSTEM
Generally (10-12) questions have been asked from this chapter. Questions, from this section usually
test your basic knowledge of numbers and are mostly based on various properties of multiplication
and division. A good number of statement based questions have been asked from this chapter.

NUMBER SYSTEM
Numbers are collection of certain symbols or figures called digits. The common number system in use
is decimal number system. In this system, we use ten symbols each representing a digit. These are 0, 1,
2, 3, 4, 5, 6, 7, 8, and 9. A combination of these figures representing a number is called a numeral.

Types of Numbers
1. Natural numbers Numbers which are used for counting i.e. 1, 2, 3, 4, ... are called natural numbers.
The set of natural numbers is denoted by ‘ N ’. Smallest natural number is 1 but we cannot find the
largest natural number as successor of every natural number is again a natural number.
2. Whole numbers Natural numbers including zero are known as whole numbers. The set of whole
numbers is denoted by W.
• Every natural number is a whole number.
• Zero (0) is the only whole number which is not a natural number.
3. Even numbers The numbers which are divisible by 2 are called as even numbers. e.g 2, 4, 6, 8, 10,
… . In general these are represented by 2 m, where m ∈ N.
4. Odd numbers The number which are not divisible by 2 are called as odd numbers. e.g. 1, 3, 5, 7,
9, … . In general, these are represented by ( 2 m − 1) , where m ∈ N.
5. Prime numbers Those numbers which are divisible by 1 and the number itself are known as prime
numbers. e.g. 2, 3, 5, 7,..., etc. are prime numbers.
• If a number is not divisible by any of the prime numbers upto square root of that number, then it is a prime
number.
• 2 is the only even number which is prime.
• The prime numbers upto 100 are : 2, 3, 5, 7, 11, 13, 17, 19, 23, 29, 31, 37, 41, 43, 47, 53, 59, 61, 67, 71,
73, 79, 83, 89 and 97 i.e. there are 25 prime numbers upto 100.
4 CDS Pathfinder

6. Coprime numbers Two natural numbers x and y are The set of rational numbers is represented by ‘Q’.
said to be coprime, if they do not have any common 3 −3
divisor other than 1. e.g. , , 7, − 6 are rational numbers.
5 7
e.g. (9, 2), (5, 6), (11, 15) are the pairs of coprime • The decimal expansion of every rational number is
numbers. either terminating or non-terminating repeating.
• If x and y are any two coprimes, a number p is 1 1 8
divisible by x as well as by y, then the number is also e.g. = 0 . 2, = 0 . 333. . ., = 0 . 181818 . . . etc.
5 3 44
divisible by xy.
• Coprime are also called as relatively prime numbers. • The recurring decimal have been given a short notation
• Twin primes Twin primes are pair of primes as 0. 3333 = 0. 3, 0 . 181818 = 0. 18
which differ by 2. e.g. (3, 5), (7, 9), (11, 13) etc.
Note • Zero is a rational number, since we can write 0 = 0 / 1.
7. Composite numbers A composite number is any
• Every natural number, whole number and integer is a
number greater than one that is not a prime number. rational number.
e.g. 4, 6, 8, 9, … all are composite numbers.
• ‘1’ is neither prime nor composite.
IRRATIONAL NUMBERS
The number which cannot be expressed in the form p/q,
INTEGERS where p and q both are integers and q ≠ 0 are known as
The collection of positive numbers, negative numbers irrational numbers. The irrational numbers when
and zero are called integers. The set of integers is expressed in decimal form are in non-terminating and
denoted by Z or I. non-repeating form. e.g. 2, 5 , 7 , 0.101005001, etc.
Thus, Z or I = {. . . , − 4, − 3, − 2, − 1, 0, 1, 2, 3, 4, . . . } is the 22
• Here, it is notable that exact value of π is not or
set of integers. Every natural number and whole number 7
is a part of integer. So, N ⊂ W ⊂ I. 22
3.14, as is a rational number while π is irrational.
7
Types of Integers
Integers are of three types IMPORTANT FACTS
(i) Positive Integers It is a set of all positive numbers. 1. If a + b = x + y , where a and x are rational and
It is denoted by I + , I + = {1, 2, 3, 4, . . . } b and y are irrational, then a = x and b = y .
2. The sum or difference of a rational and an irrational
(ii) Negative Integers It is a set of all negative numbers.
number is irrational number.
It is denoted by I − , I − = {. . . , − 3, − 2, − 1}
3. The product of rational and irrational number is also
(iii) Non-negative Integers An integer that is either 0 or an irrational number.
positive is called non-negative integer. {0, 1, 2, 3, . . . } 4. If we add, subtract, multiply or divide two irrational
numbers, we may get an irrational number or rational
Note ‘0’ is neither positive nor negative. number.

EXAMPLE 1. The smallest 3 digit prime number is EXAMPLE 2. The rational number lying between 2
a. 101 b. 103 c. 109 d. 113 and 3 is
Sol. a. The smallest 3 digit number is 100, which is divisible 49 56
a. b.
by 2. 28 35
∴ 100 is not a prime numbers. 63 85
c. d.
101 < 11 and 101is not divisible by 2, 3, 5 and 7. 45 68
∴ 101is a prime number. Sol. b. We have, 2 = 1. 414 ... and 3 = 1. 732...
Hence, 101is the smallest 3 digit prime number. 49 7 63 7
= = 1. 75, = = 1. 4
28 4 45 5
RATIONAL NUMBERS 56 8
= = 1. 6,
85 5
= = 1. 25
35 5 68 4
The numbers which are expressed in the form of p / q
Clearly, 1. 6 lies between 2 and 3.
where p and q are integers and are coprimes, q ≠ 0 are
56
called rational numbers. Hence, lies between 2 and 3.
35
MATHEMATICS Number System 5

2. | x − 2 | = x − 2, if x > 2, | x − 2 | = 0 , if x = 2
REAL NUMBERS | x − 2 | = 2 − x, if x < 2
The collection of all rational and all irrational numbers
together forms the set of real numbers and is denoted by
Some Properties of Absolute Values
‘R’. Thus, all natural numbers, whole numbers, integers,
rational and irrational numbers are real numbers. 1. | x | ≥ 0 for all real x.
2. | x | = a means x = a or x = − a
Properties of Real Numbers 3. | x | > a means x > a or x < − a
Properties of real numbers are as follows
4. If n = x 2 ; n = x 2 = | x | = x, if x > 0 = − x , if x < 0
General Properties of R
1. If x and y are two real numbers, then either EXAMPLE 3. Find the value of x which satisfy the
x > y, y > x or x = y. inequalities | x | ≥ x and 2x − 1 > 3.
2. If x and y are two real numbers, then a. All positive number
1 1 b. All positive number greater than 2
(i ) x > y ⇒ < (ii ) x > y ⇒ − x < − y
x y c. All negative number less than −2
(iii ) x > y ⇒ x + a > y + a d. All negative number
(iv ) x > y ⇒ xa > ya, when a > 0 Sol. b. | x | ≥ x is true for all real values of x. Now, consider
3. If xy = 0 ⇒ x=0 or y=0 2x − 1 > 3 or 2x > 4 or x > 2.
So, the solution set is all positive number greater than 2.
Properties of operations on R
Let ‘∗’ be any operation defined on R.
1. Closure Property If a ∈ R and b ∈ R, then a * b ∈ R.
FACTORS IN SET
2. Associative Property If a, b, c ∈ R, then OF INTEGERS
a *( b * c ) = ( a * b)* c. Let a, b ∈ I, we say that a is a factor of b, if there exists
3. Commutative Property If a, b ∈ R, then a * b = b * a. an integer p such that b = ap; in short we write a / b read
4. Identity Let ‘I’ be the identity, then it ‘a’ divides ‘b’.
I * a = a * I = a ∀ a ∈ R. If a is factor of b, then b is called a multiple of a.
5. Inverse Let a ′ be the inverse of a, then
a ′ * a = a * a ′ = I ∀ a ∈ R. Properties of factor and multiples
• The above properties hold for addition and For real numbers a and b, if b = ac then ‘a’ is a factor
multiplication on R. ‘b’ and we write a / b.
• The additive identity of R is O and additive inverse of
(i) a / b, b / c ⇒ a / c (This law is known as transitivity)
a ∈ R is −a.
• The multiplicative identity of R is 1 and multiplicative (ii) a / a, ∀a ∈ R (This law is known as reflexivity)
inverse of a ∈ R is 1/ a. (iii) a / b and a / c ⇒ a / b + c and a / b − c
• R is closed for subtraction, if a, b ∈ R, then a − b ∈ R. (iv) If p is prime number and p divides ab where a,b are
• If a, b, c ∈ R, then a × ( b + c ) = ( a × b) + ( a × c ), is the
integers, then p divides ‘a’ or p divides ‘b’. Thus
distributive property on R.
p / ab ⇒ p / a or p / b.
Absolute Value of a Real Number
The absolute value of a real number x is denoted by | x |. EXAMPLE 4. How many factors of 2 5 × 3 6 are perfect
Thus, | 3 | = 3 and | − 4 | = 4. squares?
 x, when x > 0 a. 9 b. 12
 c. 18 d. 4
If x is any real number, then | x | =  0, when x = 0
 − x, when x < 0 Sol. b. Any factor of this number should be of the form
 2a × 3 b .
e.g. For the factor to be a perfect square a, b have to be even
1. If x = 5, | 5 | = 5, if x = 0, | 0 | = 0 a can take values 0, 2, 4 and b can take values 0, 2, 4, 6.
If x = − 5, | − 5 | = − ( −5) = 5 ∴Total number of perfect squares = 3 × 4 = 12
6 CDS Pathfinder

To find the Unit’s Place Digit of a given EXAMPLE 7. When a positive integer n is divided by
5, the remainder is 2. What is the remainder when the
Expression number 3n is divided by 5?
When Number is in the form of Product a. 1 b. 2 c. 3 d. 4
To find the unit digit in the product of two or more Sol. a. Let n = 5q + 2 and 3n = 3( 5q + 2)
number we take unit digit of every number and then
⇒ 3n = 15q + 6 = 15q + ( 5 + 1) = 5( 3q + 1) + 1
multiply them. Then, the unit digit of the resultant when 3n is divided by 5, then remainder is 1.
product is the unit digit of the product of original
numbers. Divisibility Test
When Number is in the form of Index Divisor Condition with Example
n
Let the exponential number of the form be a and n ∈ I. 2 If the unit place of a number is ‘0’ or divisible by 2 i.e. unit
place is even. e.g. 17980, 314782, 6148, 316 etc.
1. In case, if a is any of (0, 1, 5, 6), then the unit’s place 3 If the sum of the digits of the given number is divisible by
digit is 0, 1, 5 and 6, respectively. 3. e.g. 24375, here 2 + 4 + 3 + 7 + 5 = 21 ÷ 3 = 7
2. In case, if a is any of (4 and 9) Hence, 24375 is divisible by 3.
(i) and if power is odd, then the unit’s place digit is 4 4 If last two digits of number is divisible by 4.
and 9, respectively. e.g. 589372, 72 is divisible by 4.
(ii) and if power is even, then the unit’s place digit is 6 Hence, 589372 is also divisible by 4.
and 1, respectively. 5 If digit at unit place is 5 or 0.
3. In case, if a is any of (2, 3, 7, 8), then see the e.g. 895, 700 etc.
following steps 6 If given number is divisibly by both 2 and 3.
Step I First, divide the exponent of a by 4. e.g. 759312. Here the last digit is divisible by 2. And
(7 + 5 + 9 + 3 + 1 + 2 ) = 27, is divisible by 3.
Step II If any remainder comes on division. Put it as the
Hence, 759312 is divisible by 6.
power of a and get the result.
7 If twice the number at units place is subtracted from rest of
Step III If any remainder does not come on division. Put the digits and the remainder is divisible by 7.
4 as the power of a and get the result. e.g. (a) 875 = 87 − 2( 5) = 87 − 10 = 77 ÷ 7 = 11
EXAMPLE 5. Find the unit digit of 207 × 781 × 39 × 94. Hence, 875 is divisible by 7.
(b) 5103 = 510 − 2( 3) = 510 − 6 = 504 ÷ 7 = 72
a. 4 b. 2 c. 1 d. 5
Hence, 5103 is divisible by 7.
Sol. b. Taking unit digit of every number and then Note Trick is applicable for number greater than 99.
multiplying them = 7 × 1× 9 × 4 = 7 × 36 8 If last three digits are divisible by 8.
Again taking unit digit and then multiplying = 7 × 6 = 42 e.g. (a) 96432 → 432 ÷ 8 = 54
∴Required unit digit = 2 (b) 16000 → 000 ÷ 8 = 0
9 If sum of all the digits are divisible by 9.
EXAMPLE 6. What is the last digit in 7 402 + 3 402 ? e.g. 317349 ⇒ ( 3 + 1 + 7 + 3 + 4 + 9) = 27 ÷ 9 = 3
a. 0 b. 4 c. 8 d. None of these Hence, 317349 is divisible by 9.

Sol. c. On division of 402 by 4, we get 2 as remainder. 10 If last digit of a number is ‘0’. e.g. 130, 36980, etc.
11 If the difference between sum of digits at even places and
∴ Last digit of 7402 = Last digit of 72 = Last digit of 49 = 9
sum of digits at odd places is divisible by 11.
Last digit of 3402 = Last digit of 32 e.g. 10615.
= Last digit of 9 = 9 Sum of digits at odd place = 1 + 6 + 5 = 12
Sum of digits at even place = 0 + 1 = 1
∴ Last digit of 7402 + 3402 = Last digit of (9 + 9) = 8
Difference = 12 − 1 = 11 ÷ 11 = 1
So, 10615 is divisible by 11.
DIVISION ON NUMBERS
EXAMPLE 8. What is the remainder when 41000 is
(DIVISION ALGORITHM) divided by 7? e 2014 I
Let ‘a’ and ‘b’ be two integers such that b ≠ 0. On a. 1 b. 2 c. 4 d. 5
dividing ‘a’ by ‘b’, ‘q’ will be the quotient and ‘r’ will be
Sol. c. On division of 4 , 4 , 4 , 4 , 4 , 4 , and 47 by 7,
1 2 3 4 5 6
the remainder, then the relationship between a, b, q and r
is a = bq + r , where 0 ≤ r < b. Or in general, we have we get remainders 4, 2, 1, 4, 2, 1and 4 respectively.
Now, 44 gives us same remainder as 41, so the cyclicity is
Dividend = Divisor × Quotient + Remainder of 3.
MATHEMATICS Number System 7

So, any power of 3 or a multiple of 3 will give a 2. Multiplication of a Given


remainder of 1.
Number by a Power of 5
So, 4999 will give a remainder of 1.
Method Put as many zeros to the right of the
41000
∴ Remainder of =4 multiplicand as is the power of 5 in the multiplier.
7
Divide the number so formed by 2 raised to the same
Theorem of Divisibility power as is the power of 5.
1. If N is a composite number of the form e.g. Multiply 6798 by 125
N = a p ⋅ b q ⋅ c r . . . , where a, b and c are primes, then 6798000
Here, 6798 × 125 = 6798 × 5 3 =
the number of divisors of N, represented by mis 23
given by m = ( p + 1) ( q + 1) (r + 1) . . .
6798000
2. The sum of the divisors of N, represented by S is = = 849750
8
given by
( a p + 1 − 1) ( b q + 1 − 1) ( c r + 1 − 1) Important Identities
S= ⋅ ⋅
( a − 1) ( b − 1) ( c − 1) The identities given below are very useful for quick
multiplication.
SOME IMPORTANT RESULTS ON DIVISION
Here, if a, b and c are real numbers, then
1. If p divides q and r, then p also divides their sum and
difference also.
1. (i) ( a + b)2 = ( a + b)(a + b) = a 2 + b2 + 2ab
2. For any natural number n , ( n 3 − n ) is divisible by 6. (ii) ( a + b)3 = a 3 + b3 + 3ab ( a + b)
3. The product of three consecutive natural numbers is 2. (i) ( a − b)2 = ( a − b)( a − b) = a 2 + b2 − 2ab
always divisible by 6.
4. ( x m − a m ) is divisible by ( x + a ) for even values of m.
(ii) ( a − b)3 = a 3 − b3 − 3ab ( a − b)

5. ( x m + a m ) is divisible by ( x + a ) for odd values of m. 3. (i) ( a + b) = ( a − b)2 + 4ab


6. ( x m − a m ) is divisible by ( x − a ) for all values of m.
(ii) ( a − b) = ( a + b)2 − 4ab

EXAMPLE 9. 19 5 + 21 5 is divisible by 4. (i) ( a + b)2 + ( a − b)2 = 2 ( a 2 + b2 )


a. Only 10 b. Only 20 (ii) ( a + b)2 − ( a − b)2 = 4ab
c. Both 10 and 20 d. Neither 10 nor 20
2 2
 a + b  a − b
Sol. c. We can check divisibility of 195 + 215 by 10 by 5. ab =   − 
adding the unit digits of 95 and 15 which is equal to  2   2 
9 + 1 = 10. So, it must be divisible by 10. 6. a 2 − b 2 = ( a − b)( a + b)
Now, for divisibility by 20 we add 19 and 21 which is
equal to 40. EXAMPLE 10. The value of
So, it is clear that it is also divisible by 20. 1.073 × 1.073 − 0.927 × 0.927 (3 4 ) 4 × 9 6
+ is
So, 195 + 215 is divisible by both 10 and 20. 1.073 − 0.927 (27)7 × 3 9
1 1
Shortcut Methods for Multiplication a. 2
3
b. 2
5
1. Multiplication of a Given c. 2
1
d. 3
Number by 9, 99 etc. 9
Method Place as many zeros at the right of the .
1073 × 1073
. − 0.927 × 0.927 ( 34 ) 4 + (9)6
Sol. c. +
multiplicand as is the number of nines in the multiplier .
1073 − 0.927 ( 27)7 × ( 3) 9
and subtract from the number, so formed from the
. ) 2 − (0.927) 2 ( 34 ) 4 × ( 32)6
(1073
multiplicand, to get the result. e.g. = + 37
1073
. − 0.927 ( 3 ) × ( 3) 9
(i) Multiply 8886 by 9999
(1073
. + 0.927) (1073
. − 0.927) 328
So, 8886 × 9999 = 88860000 − 8886 = 88851114 = + 30
1073
. − 0.927 3
(ii) Multiply 56985 by 999 1 1 1
= 2+ 2 = 2+ = 2
So, 56985 × 999 = 56985000 − 56985 = 56928015 3 9 9
8 CDS Pathfinder

The operations have to be carried out in the order, in


SIMPLIFICATION •
which they appear in the word VBODMAS.
The word simplification refers to a procedure of • The order of brackets to be simplied is (), {}, [ ].
converting a complex arithmetical expression into a
simple expression. EXAMPLE 11. Simply the following expression 52 − 4
For simplifying an expression, follow the rule of (17 − 12) + 4 × 7.
‘VBODMAS’. The letter of word means a. 38 b. 85 c. 60 d. 72
V — Under line portion i.e. bar Sol. c. 52 − 4 of (17 − 12) + 4 × 7
B — Brackets = 52 − 4 of 5 + 4 × 7 (simplifying parenthesis)
O — Orders or powers = 52 − 4 × 5 + 4 × 7 (simplifying of)
D — Division = 52 − 20 + 4 × 7 (simplifying multiplication)
M — Multiplication = 52 − 20 + 28 (simplifying multiplication)
A — Addition = 32 + 28 (simplifying subtraction)
S — Subtraction = 60 (simplifying addition)

PRACTICE EXERCISE
1. Let ‘a’ and ‘b’ be natural number, not necessarily 8. In a division operation, the divisor is 5 times the
distinct. For all values of ‘a’ and ‘b’ the natural quotient and twice the remainder. If the
number would be remainder is 15, then what is the dividend?
(a) (a + b ) (b) a / b (c) a − b (d) log (ab ) (a) 175 (b) 185 (c) 195 (d) 250
2. x and y are two natural numbers such that x is 9. If 42 ∗ 8 is a multiple of 9, then the digit
less than y,q is the quotient and r is the represented by ∗ is
remainder when y is divided by x. Therefore, (a) 0 (b) 1 (c) 2 (d) 4
(a) r = 0 (b) r < 0 (c) r > x (d) 0 ≤ r < x 16 16 ∗ 9 9 9
10. The value at ∗ in × − × + × = 1 is
3. If p is a prime number and p divides ab i.e. p/ ab, 7 7 7 7 7 7
where ‘a’ and ‘b’ are integers, then (a) 33 (b) −33 (c) −11 (d) 32
(a) p/ a or p/ b (b) p/ a and p/ b 11. The value of 10 ÷ 4 + 6 × 4 is
(c) p/ a − b (d) None of these
(a) 4 (b) 1/4 (c) 5 (d) None of these
4. Which one of the following is a prime number? a b c a+ b+ c
(a) 161 (b) 171 12. If = = , then the value of is
4 5 6 b
(c) 173 (d) 221
(a) 3 (b) 2 (c) 6 (d) 4
5. If n is a natural number, then n is
(a) always a whole number
13. The value of [a − b + ( b − a )] − [2a − 2b + ( b − 2a )] is
(b) always a natural number (a) b − 2 a (b) a − 2 b (c) b + 2 a (d) b
(c) sometimes a natural number and sometimes an k
14. If 3.325 × 10 = 0.0003325, the value of ‘k’ is
irrational number
(d) always an irrational number (a) 4 (b) −4 (c) −3 (d) −2

6. The product of a rational number and an 15. If [0.04 × 0.4 × x ] = 0.4 × 0.04 × y, then the
irrational number is value of x / y is
(a) natural number (b) an irrational number (a) 0.0016 (b) 0.16 (c) 0.016 (d) 0.160
(c) a composite number (d) a rational number 170
16. The unit digit in the product (127) is
7. What is the value of x for which x, x + 1, x + 3 are (a) 3 (b) 9 (c) 7 (d) 3
all prime numbers?
(a) 0 (b) 1 17. The unit digit in the product ( 771 × 659 × 365 ) is
(c) 2 (d) 101 (a) 6 (b) 2 (c) 4 (d) 1
MATHEMATICS Number System 9

18. When n is divided by 4, the remainder is 3. 30. If k is any even positive integer, then ( k2 + 2k) is
What is the remainder when 2n is divided by (a) divisible by 24
4? (b) divisible by 8 but may not be divisible by 24
(a) 0 (b) 2 (c) 6 (d) 3 (c) divisible by 4 but may not be divisible by 8
1 1 1 1 1 1 (d) divisible by 2 but may not be divisible by 4
− − × −
19. If 4 6 48 ÷ 4 6 48 = x, the value of 31. The number 2784936 is divisible by which one of
1 1 1  1 1 1
− −  × −  the following numbers?
4  6 48 4  6 48 (a) 86 (b) 87 (c) 88 (d) 89
x is
20 21 21 20 32. Which among the following is the largest four-digit
(a) (b) − (c) (d) − number that is divisible by 88?
21 20 20 21
(a) 9988 (b) 9966 (c) 9944 (d) 8888
20. If x is negative real number, then 1
1 1 33. The pair of rational numbers that lies between
(a)| x| = x (b)| x| = − x (c)| x| = (d)| x| = −
x x 4
3
x 3 x− y and is
21. If = , then the value of is 4
y 5 x+ y 262 752 24 74
1 1 1 (a) , (b) ,
(a) − (b) (c) (d) −6 1000 1000 100 100
4 4 6 9 31 252 748
(c) , (d) ,
7− x 40 40 1000 1000
22. If x is real, then < 2, if and only if
3 34. By adding x to 1254934, the resulting number
(a) 1 < x < 13 (b) −1 < x < 13 (c) x < 13 (d) x > 13 becomes divisible by 11, while adding y to 1254934
23. A number is divisible by 25 only, if makes the resulting number divisible by 3. Which
(a) the last digit of the number is zero one of the following is the set of values for x and y?
(b) the last digit of the number is 5 (a) x = 1, y = 1 (b) x = 1, y = − 1
(c) the last two digit of the number is divisible by 15 (c) x = − 1, y = 1 (d) x = − 1, y = − 1
(d) the last two digit of the number is divisible by 25 35. What is the last digit in the expansion of 34798 ?
2.48 × 2.48 − 1.52 × 1.52 (a) 1 (b) 3 (c) 7 (d) 9
24. The value of is
0.96 36. What least value must be given to ⊗, so that the
(a) 4 (b) 0.96 (c) 16 (d) 15.04 number 84705 ⊗ 2 is divisible by 9?
25. Which one of the following is correct regarding (a) 0 (b) 1 (c) 2 (d) 3
the number 222222? 37. What is the total number of three digit numbers
(a) It is divisible by 3 but not divisible by 7 with unit digit 7 and divisible by 11?
(b) It is divisible by 3 and 7 but not divisible by 11 (a) 6 (b) 7 (c) 8 (d) 9
(c) It is divisible by 2 and 7 but not divisible by 11
7 3
(d) It is divisible by 3, 7 and 11 38. Find the value of ‘a’ and ‘b’ 3 × b = 8.
a 15
26. What is the sum of positive integers less than (a) 2, 11 (b) 11, 2 (c) 1, 1 (d) 2, 1
100 which leave a remainder 1 when divided
by 3 and leave a remainder 2 when divided by 39. If a , b and c are real numbers such that
4? a < b and c < 0, then which of the statements is
(a) 416 (b) 620 (c) 1250 (d) 1314 true?
27. What is the last digit in the expansion of (a) (a/c ) < (b/c ) (b) ac < bc
754 (c) (c/a) > (c/b ) (d) ac > bc
( 2457) ?
(a) 3 (b) 7 (c) 8 (d) 9 40. If we divide a positive integer by another positive
integer, what is the resulting number?
28. A three-digit number is divisible by 11 and
(a) It is always a natural number
has its digit in the unit’s place equal to 1. The
(b) It is always an integer
number is 297 more than the number obtained
by reversing the digits. What is the number? (c) It is a rational number
(a) 121 (b) 231 (c) 561 (d) 451 (d) It is an irrational number

29. The remainder on dividing given integers a 41. What can be said about the expansion of 212n − 64 n ,
and b by 7 are, respectively 5 and 4. What is where n is a positive integer?
the remainder when ab is divided by 7? (a) Last digit is 4 (b) Last digit is 8
(a) 3 (b) 4 (c) 5 (d) 6 (c) Last digit is 2 (d) Last two digits are zero
10 CDS Pathfinder

42. If p is an integer, then every square integer is of 54. What will be the remainder when 19100 is
the form divided by 20?
(a) 2 p or (4 p − 1) (b) 4p or (4 p − 1) (a) 19 (b) 20 (c) 3 (d) 1
(c) 3p or (3 p + 1) (d) 4p or (4 p + 1) 12
55. 7 − 4 12
is exactly divisible by which of the
43. If r and s are any real numbers such that 0 ≤ s ≤ 1 following number?
and r + s = 1, then what is the maximum value of (a) 34 (b) 33 (c) 36 (d) 35
the product? p q p +2 q +1
3 1 1
56. If 2 + 3 = 17 and 2 −3 = 5, then find the
(a) 1 (b) (c) (d) value of p and q.
4 2 4
(a) −2, 3 (b) 2, − 3 (c) 3, 2 (d) 2, 3
44. A number when divided by 2, 3 or 5 gives
49 1
remainder 1. The number is 57. If = 3+ , where x , y and z are natural
15 1
(a) 31 (b) 47 (c) 49 (d) 53 x+
1
45. The largest integer that divides product of any y+
four consecutive integers is z
numbers, then what is z equal to?
(a) 4 (b) 6 (c) 12 (d) 24
(a) 1 (b) 2 (c) 3
46. Which one of the following three digit numbers (d) Cannot be determined due to insufficient data
divides 9238 and 7091 with the same remainder
in each case? 58. The least number which is a perfect square and
(a) 113 (b) 209 (c) 317 (d) 191
has 540 as a factor is
(a) 8100 (b) 6400 (c) 4900 (d) 3600
47. For a positive integer n, define d( n ) = The 2 3
59. If A is real and 1 + A + A + A = 40, then A is
number of positive divisors of n. What is the equal to
value of d( d( d(12)))?
(a) −3 (b) −1 (c) 1 (d) 3
(a) 1 (b) 2 (c) 4 (d) None of these
60. How many factors of 1080 are perfect squares?
48. If three sides of a right angled triangle are (a) 4 (b) 6 (c) 8 (d) 5
integers in their lowest form, then one of its
sides is always divisible by 61. Consider the following statements:
(a) 6 (b) 5 (c) 7 (d) None of these I. In a given whole number, if the sum of the odd
numbered digit is equal to the sum of even
49. If −1 ≤ x ≤ 3 and 1 ≤ y ≤ 3, then the maximum numbered digits, then the number is divisible
value of ( 3 y − 4x ) is by 11.
(a) 18 (b) 13 (c) 5 (d) −6 II. In a given whole number, if the difference of
50. If ‘ p’ is an integer greater than 3, then on dividing sum of odd numbered digits and even numbered
digits is divisible by 11, then the number is
p11 + 1 by p − 1, we would get the remainder as
divisible by 11.
(a) 2 (b) 0 (c) −2 (d) −1
Which of the statement(s) given above is/are
51. If A is the set of squares of natural numbers and correct?
x and y are any two element of A, then the (a) Only I (b) Both I and II
correct statement is (c) Only II (d) None of these
(a) x + y belongs to A (b) x − y belongs to A
x 62. Consider the following statements:
(c) belongs to A (d) x y belongs to A
y A number a1a2a3a4 a5 is divisible by 9, if
I. a1 + a2 + a3 + a4 + a5 is divisible by 9.
52. A ten-digit number is divisible by 4 as well as by
II. a1 − a2 + a3 − a4 + a5 is divisible by 9.
5. What could be the possible digit at the ten’s
place in the given number? Which of the statement(s) given above is/are
(a) 0, 1, 2, 4, or 6 (b) 1, 2, 4, 6 or 8 correct?
(c) 2, 3, 4, 6 or 8 (d) 0, 2, 4, 6 or 8 (a) Only I (b) Only II
(c) Both I and II (d) Neither I nor II
53. If x < 0 < y, then which one of the following
relations is correct? 63. Consider the following statements:
(a)
1
<
1
<
1
(b)
1
>
1
<
1 If p is a prime such that p + 2 is also a prime,
2
x xy y2 x 2 xy y2 then
1 1
(c) <
1 1
(d) > I. p( p + 2) + 1 is a perfect square.
x y x y II. 12 is a divisor of p + ( p + 2), if p > 3
MATHEMATICS Number System 11

Which of the statement(s) given above is/are PREVIOUS YEARS QUESTIONS


correct?
(a) Only I (b) Only II 69. What number should be added to 231228 to
(c) Both I and II (d) Neither I nor II make it exactly divisible by 33? e 2012 I
64. Consider the following statements: (a) 1 (b) 2 (c) 3 (d) 4
I. If x and y are composite integers, so also is x + y. 70. If a positive integer leaves remainder 28 when
II. If x and y are composite integers and x > y, then divided by 143, then what is the remainder
x − y is also a composite integer. obtained on dividing the same number by 13?
III. If x and y are composite integers, so also in xy. e 2012 I
(a) 0 (b) 2 (c) 9 (d) 10
Which of the statement(s) given above is/are
correct? 71. Which one of the following is neither prime
(a) I, II and III (b) I and II number nor composite number? e 2012 II
(c) Only III (d) None of these (a) 1 (b) 2 (c) 3 (d) None of these

65. Consider the following statements: 72. Which one of the following has least number of
I. The product of any three consecutive integers is divisors? e 2012 II
divisible by 6. (a) 88 (b) 91 (c) 96 (d) 99
II. Any integer can be expressed in one of the 73. How many numbers between −11 and 11 are
three forms 3k, 3k + 1, 3k + 2, where k is an multiples of 2 or 3? e 2012 II
integer. (a) 11 (b) 14 (c) 15 (d) None of these
Which of the statement(s) given above is/are 74. Consider the following statements
correct? I. If n is a prime number greater than 5, then
(a) Only I (b) Only II n4 − 1 is divisible by 2400.
(c) Both I and II (d) Neither I nor II II. Every square number is of the form 5n or (5n − 1)
66. Consider the following statements: or (5n + 1), where n is a whole number.
I. A natural number is divisible by 2, if its last Which of the statement(s) given above is/are
digit is divisible by 2. correct? e 2012 II
II. A natural number is divisible by 2, if its last (a) Only I (b) Only II
digit is either zero or 2. (c) Both I and II (d) Neither I nor II
III. A natural number is divisible by 2, if its last
digit is even. 75. If N , ( N + 2) and ( N + 4) are prime numbers,
then the number of possible solutions for N are
Which of the statements given above are correct?
(a) 1 (b) 2 e 2013 I
(a) I and II (b) I and III (c) II and III (d) All of these (c) 3 (d) None of these
67. Consider the following statements for natural 76. The two-digit number, which when divided by
numbers a, b and c : sum of the digits and product of the digits,
I. If ‘a’ is divisible by ‘b’ and ‘b’ is divisible by ‘c’, respectively leaves the same remainder the
then a must be divisible by ‘c’. number is and the difference of quotients is one.
II. If ‘a’ is a factor of both ‘b’ and ‘c’, then ‘a’ must (a) 14 (b) 23 (c) 32 (d) 41 e 2013 I
be a factor of ‘b + c’.
77. If x is positive even integer and y is negative
III. If ‘a’ is a factor of both ‘b’ and ‘c’, then ‘a’ must
be a factor of ‘b − c’. odd integer, then x y is e 2013 I
(a) odd integer (b) even integer
Which of the statements given above are correct? (c) rational number (d) None of these
(a) I, II and III (b) I and II
(c) II and III (d) None of these 78. Consider the following statements
I. There is a finite number of rational numbers
68. Consider the following statements: between any two rational numbers.
I. Set of positive powers of 2 is closed under II. There is an infinite number of rational numbers
multiplication. between any two rational numbers.
II. The set {1, 0, − 1} is closed under multiplication. III. There is a finite number of irrational number
III. The number 35 has exactly four divisors. between any two rational numbers.
IV. The set {1, 0, − 1} is closed under addition. Which of the statement(s) given above is/are
Which of the statements given above are correct? correct? e 2013 I
(a) I, II and III are true (b) Only III (a) Only I (b) Only II
(c) Only IV (d) All of these (c) Only III (d) Both II and III
12 CDS Pathfinder

79. If k is a positive integer, then every square 88. What is the remainder when (1235 × 4523 × 2451)
integer is of the form e 2013 II is divided by 12? e 2014 II
(a) 4k (b) 4k or 4k + 3 (a) 1 (b) 3 (c) 5 (d) 7
(c) 4k + 1 or 4k + 3 (d) 4k or 4k + 1
89. What is the number of divisors of 360? e 2014 II
80. If b is the largest square divisor of c and a2 (a) 12 (b) 18
divides c, then which one of the following is (c) 24 (d) None of these
correct (where, a, b and c are integers)? e 2013 II
(a) b divides a (b) a does not divide b 90. The multiplication of a three-digit number XY 5
(c) a divides b (d) a and b are coprime with digit Z yields X 215. What is X + Y + Z
equal to? e 2014 II
81. Every prime number of the form 3k + 1 can be (a) 13 (b) 15 (c) 17 (d) 18
represented in the form 6m + 1 (where, k and m
are integers), when e 2013 II
91. How many pairs of X and Y are possible in the
number 763X 4Y 2, if the number is divisible
(a) k is odd
by 9? e 2014 II
(b) k is even (a) 8 (b) 9 (c) 10 (d) 11
(c) k can be both odd and even
(d) No such form is possible 92. If an = 3 − 4n , then what is a1 + a2 + a3 +…+ an
equal to? e 2014 II
82. Consider the following statements (a) − n (4n − 3) (b) − n (2 n − 1) (c) −n2 (d) − n (2 n + 1)
I. 7710312401 is divisible by 11.
II. 173 is a prime number. 93. Consider all those two-digit positive integers less
than 50, which when divided by 4 yield unity as
Which of the statement(s) given above is/are
remainder. What is their sum? e 2014 II
correct? e 2013 II
(a) 310 (b) 314 (c) 218 (d) 323
(a) Only I (b) Only II
(c) Both I and II (d) Neither I nor II 94. p, q and r are prime numbers such that
p < q < r < 13. In how many cases would
83. Consider the following statements ( p + q + r ) also be a prime number? e 2014 II
I. To obtain prime numbers less than 121, we are (a) 1 (b) 2 (c) 3 (d) None of these
to reject all the multiples of 2, 3, 5 and 7.
II. Every composite number less than 121 is 95. The digit in the units place of the product
divisible by a prime number less than 11. 81 × 82 × 83 × 84 ×…× 99 is e 2015 I
(a) 0 (b) 4 (c) 6 (d) 8
Which of the statement(s) given above is/are
correct? e 2013 II 96. What is the remainder obtained when
(a) Only I (b) Only II
1421 × 1423 × 1425 is divided by 12? e 2015 I
(a) 1 (b) 2 (c) 3 (d) 4
(c) Both I and II (d) Neither I nor II
96
97. What is the remainder when 4 is divided by 6?
84. Consider the following statements
I. No integer of the form 4k + 3, where k an integer, e 2015 I
can be expressed as the sum of two squares. (a) 4 (b) 3
(c) 2 (d) 1
II. Square of an odd integer can expressed in the
form 8k + 1, where k is an integer. 98. What is the maximum value of m, if the number
Which of the statement(s) given above is/are N = 35 × 45 × 55 × 60 × 124 × 75 is divisible by
correct? e 2014 I 5m ? e 2015 I
(a) Only I (b) Only II (a) 4 (b) 5
(c) 6 (d) 7
(c) Both I and II (d) Neither I nor II
85. What is 262 + 972 equal to? e 2014 I
99. If a − b = 4 and a2 + b2 = 40, where a and b are
(a) 27 2 + 932 (b) 342 + 932 (c) 82 2 + 412 (d) 792 + 62 2 positive integers, then a3 + b6 is equal to e 2015 I
(a) 264 (b) 280
86. If n is a whole number greater than 1, then (c) 300 (d) 324
n2( n2 − 1) is always divisible by e 2014 I 100. If n is a natural number and n = p1x1 p2x2 p3x3 ,
(a) 12 (b) 24 (c) 48 (d) 60
where p1, p2 , p3 are distinct prime factors, then
87. What is the remainder when (1723 + 2323 + 2923 ) the number of prime factors for n is e 2015 I
is divided by 23? e 2014 II (a) x1 + x2 + x3 (b) x1x2x3
(a) 0 (b) 1 (c) 2 (d) 3 (c) (x1 + 1)(x2 + 1)(x3 + 1) (d) None of these
MATHEMATICS Number System 13

37 1 111. What is the maximum value of m, if the number


101. If = 2+ ,where x , y and z are natural
13 1 N = 90 × 42 × 324 × 55 is divisible by 3m ? e 2016 I
x+
1
y+ (a) 8 (b) 7 (c) 6 (d) 5
z 10 10
numbers, then what is z equal to? e 2015 I 112. 7 − 5 is divisible by e 2016 I
(a) 1 (b) 2 (c) 3 (a) 5 (b) 7 (c) 10 (d) 11
(d) Cannot be determined due to insufficient data 113. If a3 = 117 + b3 and a = 3 + b, then the value of
102. A student was asked to multiply a number by 25. a + b is (given that a > 0 and b > 0) e 2016 I
He instead multiplied the number by 52 and got (a) 7 (b) 9 (c) 11 (d) 13
the answer 324 more than the correct answer.
The number to be multiplied was e 2015 I 114. Let m be a non-zero integer and n be a positive
(a) 12 (b) 15 (c) 25 (d) 32 integer. Let R be the remainder obtained on
dividing the polynomial x n + m n by ( x − m ).
103. The number of pairs ( x , y ), where x , y are integers
Then, e 2016 I
satisfying the equation 21x + 48 y = 5, is e 2015 II (a) R is a non-zero even integer
(a) Zero (b) One (c) Two (d) Infinity (b) R is odd, if m is odd
104. The largest natural number which divides every (c) R = s 2 for some integer s, if n is even
3
natural number of the form ( n − n ) ( n − 2), (d) R = t 3 for some integer t, if 3 divides n
where n is a natural number greater than 2, is
61 1
(a) 6 (b) 12 e 2015 II 115. If = 3+ where x , y and z are natural
(c) 24 (d) 48 19 1
x+
105. The digit in the units place of the resulting 1
y+
number of the expression ( 234)100 + ( 234)101, is x
(a) 6 (b) 4 e 2015 II numbers, then what z is equal to? e 2016 I
(c) 2 (d) 0 (a) 1 (b) 2 (c) 3 (d) 4
106. A number when divided by 7 leaves a remainder 116. Let x and y be positive integers such that x is
3 and the resulting quotient, when divided by 11 prime and y is composite. Which of the following
leaves a remainder 6. If the same number when statements are correct?
divided by 11 leaves a remainder m and the I. ( y − x) can be an even integer.
resulting quotient when divided by 7 leaves a
II. xy can be an even integer.
remainder n. What are the values of m and n,
respectively? e 2015 II III. 0.5 (x + y) can be an even integer.
(a) 1 and 4 (b) 4 and 1 (c) 3 and 6 (d) 6 and 3 Select the correct answer using the code given
107. The seven digit number 876 p37q is divisible by below e 2016 I
(a) I and II (b) II and III
225. The values of p and q can be respectively (c) I and III (d) All of these
(a) 9, 0 (b) 0, 0 e 2015 II
(c) 0, 5 (d) 5,9 117. Consider the following statements:
108. Let x and y be positive integers such that x > y. I. Every natural number is a real number.
The expressions 3x + 2 y and 2x + 3 y, when II. Every real number is a rational number.
divided by 5 leave remainders 2 and 3, III. Every integer is real number.
respectively. What is the remainder when ( x − y ), IV. Every rational number is a real number.
is divided by 5? e 2015 II Which of the above statements are correct?
(a) 4 (b) 2 (c) 1 (d) 0 (a) I, II and III (b) I, III and IV e 2016 I
(c) II and III (d) III and IV
109. The sum of first 47 terms of the series
1 1 1 1 1 1 1 1 1 118. Consider the following statements in respect of
+ − − − + + + − − ... , is two different non-zero integers p and q.
4 5 6 4 5 6 4 5 6 e 2015 II
1 1 9 I. For ( p + q) to be less than ( p − q), q must be
(a) 0 (b) − (c) (d) negative.
6 6 20
II. For ( p + q) to be greater than ( p − q), both p and
110. The value of the expression q must be positive.
( 243 + 647)2 + ( 243 − 647)2 Which of the above statements is/are correct?
is equal to
243 × 243 + 647 × 647 e 2016 I (a) Only I (b) Only II e 2016 I
(a) 0 (b) 1 (c) 2 (d) 3 (c) Both I and II (d) Neither I nor II
14 CDS Pathfinder

119. If a and b are negative real numbers and c is a 120. If m and n are distinct natural numbers, then
positive real number, then which of the following which of the following is/are integer/integers?
is/are correct? m n m n
I. + II. mn  +  (m2 + n2 )−1
I. a − b < a − c n m  n m
a b
II. If a < b, then < mn
c c III.
1 1 m2 + n2
III. <
b c
Select the correct answer using the code given
Select the correct answer using the code given
below e 2016 (I)
below e 2016 I
(a) I and II (b) Only II
(a) Only I (b) Only II
(c) Only III (d) II and III (c) II and III (d) Only III

ANSWERS
1 2 3 4 5 6 7 8 9 10
11 12 13 14 15 16 17 18 19 20
21 22 23 24 25 26 27 28 29 30
31 32 33 34 35 36 37 38 39 40
41 42 43 44 45 46 47 48 49 50
51 52 53 54 55 56 57 58 59 60
61 62 63 64 65 66 67 68 69 70
71 72 73 74 75 76 77 78 79 80
81 82 83 84 85 86 87 88 89 90
91 92 93 94 95 96 97 98 99 100
101 102 103 104 105 106 107 108 109 110
111 112 113 114 115 116 117 118 119 120

HINTS AND SOLUTIONS 


1. (a) ( a + b ) always represent a natural 6. (b) We know that, the product of a 256 9x 81
⇒ − + =1
number ∀ a , b ∈ N . rational number and an irrational 49 49 49
2. (d) As, y = qx + r, so 0 ≤ r < x. number is an irrational number. ⇒ 256 + 81 − 9x = 49
3. (a) As p is prime, so p/a or p/b. 7. (c) If x = 2, then, x, x + 1 and x + 3 are ⇒ 9x = 288 ⇒ x = 32
all prime numbers.
4. (c) Consider the given options ⇒ 9x = 288 ⇒ x = 32
8. (c) Q Dividend = D × Q + R
(a) 161 is divisible by 7. Hence, 161 is 11. (a) By BODMAS, 10 ÷ 4 + 6 × 4
not a prime number. Given, D = 5Q and D = 2R
= 10 ÷ ( 4 + 6) × 4
(b) Since, 1 + 7 + 1 = 9. Therefore, 171 When R = 15, D = 2 × 15 = 30
is divisible by 3. Hence, 171 is not a D 30 = 10 ÷ 10 × 4 = 1 × 4 = 4
prime number. ∴ Q= = =6 a b c
5 5 12. (a) = = = k (say)
(c) 173 < ( 14)2 4 5 6
∴ Dividend = 30 × 6 + 15 = 195
173 is not divisible by any of the ⇒ a = 4k , b = 5k and c = 6k
numbers 2, 3, 5, 7, 11, 13. Hence, 9. (d) As, 4 + 2 + * + 8 = 14 + * ; 42 * 8 is
divisible by 9, if 14 + * divisible by 9. a + b + c 4k + 5k + 6k 15k
173 is a prime number. So, = = =3
So, 14 + ∗ = 18 (nearest multiple of 9) b 5k 5k
(d) 221 is divisible by 13. Hence, 221 is
not a prime number. ⇒ ∗ = 18 − 14 = 4 13. (d) [ a − b + ( b − a )]
5. (c) Consider, 2, 4 ∈ N 16 16 * 9 9 9 − [ 2a − 2b + ( b − 2a )]
10. (d) Here, × − × + × =1
So, 4 = 2, a natural number 7 7 7 7 7 7 ⇒ [ a − b + b − a ] − [ 2a − 2b + b − 2a ]
and 2 = irrational number. Put * = x ⇒ 0 − [ 2a − 2a − b ] = 0 + b = b
MATHEMATICS Number System 15

0.0003325 7 − x
= 22. (a) As, <2 30. (b) If k is any even positive integer, then
k
14. (b) 10 (given)
3. 325  3  k 2 + 2k is divisible by 8 but may not be
3.325 × 10−4 7−x 7 − x
= = 10−4 ⇒ < 2 or −  <2 divisible by 24.
3.325 3  3 
Let k = 2m,m ∈ N , then
⇒ 10k = 10−4 [Q | x | < a ⇒ x < a or − x < a]
k 2 + k ⋅ 2 = 4m 2 + 4m = 4m (m + 1)
∴ k = −4 x−7
⇒ 7 − x < 6 or <2 which is divisible by 8.
3
15. (c) 0.04 × 0.4 × x = 0.4 × 0.04 × y
⇒ − x < − 1 or x − 7 < 6 31. (c) Given number is 2784936.
(given) Sum of digits at odd places= 25 and sum
x 0.4 × 0.04 ⇒ x > 1 or x < 13
∴ = of digits at even places = 14
y 0.04 × 0.4 ⇒ 1 < x < 13
∴ Difference = 25 − 14 = 11
23. (d) A number is divisible by 25 when its
x 0.4 × 0.4 × 0.04 × 0.04 So, number is divisisble by 11.
= = 0.016 last 2 digits are either zero or divisible by
y 0.04 × 0.4 25. Also last three digits of 2784936 i.e. 936
(squaring both sides) ( 2.48)2 − ( 1. 52)2 is divisible by 8.
24. (a) Hence, 2784936 is divisible by both 8
16. (b) Since, unit digit in 74 is 1. 0.96
( 2.48 − 1.52)( 2.48 + 152. ) and 11 i.e. 88.
∴ 7168 = ( 74 )42 give unit digit 1. =
0.96 32. (c) A number divisible by 88, if it is
∴ 7170 = 7168 × 72 gives the unit digit divisible by 8 and 11.
[Q a 2 − b 2 = ( a − b )( a + b )]
as 1 × 7 × 7 = 9
0.96 × 4 In the given options 9944 and 8888 are
17. (c) Unit place in 7 4 = 1, so unit place in = =4
0.96 divisible by 88. Hence, maximum
768 is 1. number is 9944.
25. (d) Given, number is 222222.
∴ Unit place in 7 68 × 7 3 = 3. Similarly, 1 3
Here, sum of digits 33. (d) Q = 0.25 and = 0.75
unit place in 6 59 is 6 and unit place in 4 4
= 2 + 2 + 2 + 2 + 2 + 2 = 12,
34 is 1, so unit place in 365 = 364 × 3 = 3. Only option (d) lies between 0.25 and
which is divisible by 3. So, given number 252
∴ Unit place in 7 × 6 × 3 71 59 65
is the is divisible by 3. 0.75. Since = 0. 252
unit place of 3 × 6 × 3 = 4 1000
Now, sum of odd terms of digits − Sum
of even terms of digits = 6 − 6 = 0, 748
18. (b) As, n is divided by 4 the remainder is and = 0.748
3, so it is divisible by 11. 1000
n = 4q + 3, where q is quotient. Also, in a number if a digit is repeated 34. (b) Difference of sums of even and odd
six times, then the number is divisible by places digit of 1254934
⇒ 2n = 8q + 6
⇒ 2n = (8k + 4) + 2 = 4 ( 2k + 1) + 2
7, 11 and 13. = ( 1 + 5 + 9 + 4) − ( 2 + 4 + 3)
So, if 2n is divided by 4 the quotient
Hence, the given number is divisible by = 19 − 9 = 10
3, 7 and 11.
is 2k + 1 and remainder is 2. This number will be divisible by 11,
1 1 1 1 1 1 26. (a) Required numbers are of the form of after adding x, if x = 1.
− − × − 12q − 2
19. (c) Given, 4 6 48 ÷ 4 6 48 Also, the sum of digits of 1254934
1 1 1 1 1 1 i.e. 10, 22, 34, 46, 58, 70, 82, 94
− −  × −  = 1 + 2 + 5 + 4 + 9 + 3 + 4 = 28
4  6 48  4  6 48  ∴ Total sum = 10 + 22 + 34 + 46 + 58
1254934 will be divisible by 3, after
+ 70 + 82 + 94 = 416 adding y , if y = − 1
=x
12 − 8 − 1 1 1 27. (d) The last digit in the expansion of 35. (d) Last digit in the expansion of 34798
− 754 754
⇒ x = 48 ÷ 24 48 ( 2457) is equal to last digit of ( 7) . = Last digit in the expansion of
12 − (8 − 1) 1  8 − 1 ( 7)754 = ( 74 )188 × 72 = 1 × 72
×  (34 )1199 ⋅ 32
48 4  48  = ( 7)2 = 49
3 2−1 = Last digit in the expansion of 32 = 9
Hence, last digit is 9.
⇒ x = 48 ÷ 48 36. (b) The given number is divisible by 9, if
5 7 28. (d) On taking option (d),
sum of the digits is divisible by 9.
48 4 × 48 The reverse digit of 451 is 154.
Here, sum of digits
3 4 21 Now, 154 + 297 = 451 is equal to the
⇒ x = ÷ ⇒x = original number. = 8 + 4 + 7 + 0 + 5 + 2 + ⊗ = 26 + ⊗
5 7 20
29. (d) Let a = 7 p + 5 and b = 7q + 4 If ⊗ = 1 , then 26 + 1 = 27 is divisible by
20. (b) Clearly, absolute value is defined by 9.
| x | = − x. where, p and q are natural numbers.
x− y x/ y − 1 3/5− 1 ∴ ab = ( 7 p + 5) ( 7q + 4) 37. (c) The total number of three-digit
21. (a) = = numbers with unit digit 7 and divisible
x + y x/y + 1 3/5 + 1 ab = 49 pq + ( 4 p + 5 q ) 7 + 20
by 11 are 187, 297, 407, 517, 627, 737,
= 7 ( 7 pq + 4 p + 5 q ) + 7 × 2 + 6 847, 957.
(3 − 5 ) / 5 −2 1
= = =− when ab is divided by 7, we get the
(3 + 5 ) / 5 8 4 remainder 6. ∴ Total numbers = 8
16 CDS Pathfinder

38. (b) Apply hit and trial method from the 48. (b) Given, by pythagoras theorem, 49 1
57. (c) =3+
given option. As, here when 15 1
(3 ) + ( 4 ) = ( 5 )
2 2 2
x+
a = 11, b = 2, then y+
1
Let the sides of a right triangle be 3, 4,
7 3 7 3 z
3 ×b =3 ×2 5.
a 15 11 15 1 49 4
Hence, one of its sides is always divisible ⇒ = −3 =
40 33 1 15 15
= × =8 by 5. x+
11 15 yz + 1
49. (b)Q1 ≤ y ≤ 3 ⇒ 3 ≤ 3 y ≤ 9 …(i) z
39. (d) Since, a < b ⇒ a − b < 0. Also, c < 0
and −1 ≤ x ≤ 3 ⇒ − 4 ≤ 4x ≤ 12 1 4
∴ ( a − b ) c > 0 ⇒ ac − bc > 0 ⇒ ac > bc ⇒ z =
⇒ −12 ≤ −4x ≤ 4 …(ii) x+ 15
40. (c) When we divide a positive integer by yz + 1
From Eqs. (i) and (ii), we get
another positive integer, the resultant
−9 ≤ 3 y − 4x ≤ 13 yz + 1 4
will be a rational number i.e. in the form ⇒ =
xyz + x + z 15
of p/q, where p and q are positive integers Maximum value is 13.
and q ≠ 0. 50. (a) When p11 + 1 is divided by ( p − 1), ⇒ 15( yz + 1) = 4 ( xyz + x + z )
41. (d) 212 n − 64 n = ( 212 )n − ( 64 ) n then the remainder is ⇒ 15 yz + 15 = 4xyz + 4x + 4z
= ( 4096)n − ( 1296)n ( 1)11 + 1 = 1 + 1 = 2 ⇒ 15 − 4x = ( 4xy + 4 − 15 y )z
∴ 212 n − 64 n = ( 4096 − 1296) k 51. (d) Let x = a 2 and y = b 2 for some 15 − 4x
z=
As x n − y m is always divisible by ( x − m ) a , b ∈ N , then xy = a 2 b 2 = ( ab )2 , where ( 4xy + 4 − 15 y )
ab ∈ N .
= 2800 ( k ) As x, y and z are natural numbers.
So, xy ∈ A .
Hence, last two digits are always be zero.
x a2
2 Again, let x = 3 and y = 1
But = 2 =   ∉ A as ∉ N .
a a
42. (c) We know that  b 15 − 4(3)
y b b Then, z =
4 = 3 p + 1, for p = 1, 9 = 3 p, for p = 3 4(3)( 1) + 4 − 15( 1)
52. (d) Since, a ten-digit number is divisible
16 = 3 p + 1, for p = 5, 15 − 12 3
by 4 as well as by 5, then this number = =
25 = 3 p + 1, for p = 8 must be divisible by 20. 12 + 4 − 15 1
36 = 3 p, for p = 12 We known that any number is divisible ⇒ z = 3, which is a natural number.
Hence, every square integer is of form by 20, if last two digits is divisible by 20. 58. (a) Required number is
either 3 p or 3 p + 1. It means unit place will be zero and ten’s
x 2 = 540 × q = 3 × 3 × 3 × 2 × 2 × 5 × q
place may be 0, 2, 4, 6 or 8.
43. (d) Given, r + s = 1 In order to make x 2 a perfect square, the
1 53. (c) As, x < 0 < y [given]
For maximum product, r = s = least number we must have to put is
2 ⇒ x < 0 and y > 0 q = 3 × 5 = 15
1 1 1
∴ rs = × = ∴
1
< 0 and
1 1 1
>0 ∴ < ∴ x 2 = 540 × 15 = 8100
2 2 4 x y x y
59. (d) We have, 1 + A + A 2 + A 3 = 40
44. (a) Required number n
54. (d) On division of ( 19) by 20, we get ⇒ ( 1 + A ) + A 2 ( 1 + A ) = 40
= [LCM of 2, 3 and 5] +1 remainder either 19 or 1.
= 30 + 1 = 31 ⇒ ( 1 + A )( 1 + A 2 ) = 40
Since, last digit of ( 19)100 is 1.
Only A = 3 satisfies the above equation,
45. (d) The largest integer that divides ( 19)100
product of any four consecutive integers ∴ Remainder of is 1. so, A = 3.
20
is 24. 60. (a) We have, 1080 = 23 × 33 × 5
e.g. 1, 2, 3, 4 are four consecutive 55. (b) We know that ( xn − y n ) is divisible
integers. by ( x − y ) for all ‘n’ and is divisible by For any perfect square, all the powers of
( x + y ) for even ‘n’. the primes have to be even numbers.
Multiplication = 1 × 2 × 3 × 4 = 24
∴( 712 − 412 ) is divisible by ( 7 + 4) = 11 So, if the factor is of the form
which divided by 24.
and ( 7 − 4) = 3 2a × 3b × 5c
46. (a) When we divide the number 9238
Thus, ( 7 12
− 4 ) is divisible by 33.
12
The values a can be 0 and 2, b can be 0
and 7091 by 113, we get the same
remainder 85. 56. (c) Here, 2 p + 3q = 17 ...(i) and 2 and c can take the value 0.
47. (d) d ( d ( d ( 12))) = d ( d ( 6)) 2p + 2
− 3q +1
=5 Totally, there are 4 possibilities.
[Q positive integer divisor of or 4 ⋅ 2 − 3 ⋅ 3q = 5
p
...(ii) 61. (b) Clearly, both statements satisfies
12 = 1, 2, 3, 4, 6, 12] On multiplying Eq. (i), by 3 and adding divisibility rule of 11.
d ( d ( 6)) = d ( 4) it with Eq. (ii), we get 62. (a) As, we know that a number
[Q positive integer divisor of 7 ⋅ 2 p = 56, 2 p = 8 = 23 , ⇒ p = 3 a a a a a is divisible by 9, if sum of
6 = 1, 2, 3, 6] 1 2 3 4 5
Put p = 3 in Eq. (i), we get the digits, i.e. a + a + a + a + a
and d( 4) = 3 1 2 3 4 5
[Q positive integer divisor of 23 + 3q = 17, 3q = 17 − 8 = 9 = 32 is divisible by 9. Hence, only statement I
4 = 1, 2, 4] ∴ q=2 is true.
MATHEMATICS Number System 17

63. (c) Taking p = 11 72. (b)Here, 88 = 2 × 2 × 2 × 11 = ( 2)3 × ( 11)1 So, a 2 will divide bx or a will divide b.
p + 2 = 13 [a prime number] 91 = ( 7) × ( 13)
1 1 [since, it cannot divide x as it is not a
whole square]
I. 11 × 13 + 1 = 144 [a square number] 96 = 2 × 2 × 2 × 2 × 2 × 3
II. 11 + 13 = 24 [12 is a divisor of 24] 81. (b) Every prime number of the form
= ( 2)5 × (3)1
3k + 1 can be represented in the form
Hence, both statements I and II are and 99 = 3 × 3 × 11 = (3)2 × ( 11)1 6m + 1 only, when k is even.
correct. So, 91 has least number of divisors. 82. (c) I. In 7710312401, difference between
64. (c) I. If x = 15 and y = 14, then 73. (c) Following are the numbers between sum of even place digits and the sum of
x + y = 15 + 14 = 29, which is a − 11and 11 which are multiples of 2 or 3. odd place digits 0. So, it is divisible by
prime number. So, if x and y are 11.
composite, then x + y is not always − 10, − 9, − 8, − 6, − 4, − 3, − 2,
II. Since, 173 < ( 14)2 and it is not
composite. 0, 2, 3, 4, 6, 8, 9, 10 divisible by 2, 3, 5, 7, 11 and 13. So,
II. If x = 15 and y = 14, then So, the numbers of multiples 2 or 3, it is a prime number.
x − y = 15 − 14 = 1 which is neither between –11 and 11 are 15. Hence, both statements I and II are
prime nor composite, hence again correct.
x − y is not always composite. 74. (b) I. Given, n is a prime number greater
than 5. 83. (c) Both the statements given are
III. Third condition is satisfied for all correct. As 121 is the square of 11. So, to
Now, n4 − 1 = ( n2 − 1)( n2 + 1)
measure. obtain prime numbers less than 121, we
Hence, only III is correct. = ( n − 1)( n + 1)( n2 + 1) reject all the multiples of prime numbers
65. (c) I. The product of any three Put n = 11, less than 11 i.e. 2, 3, 5 and 7. Similarly,
consecutive integers is divisible by 6. n4 − 1 = ( 11 − 1)( 11 + 1)( 121 + 1) every composite number less than 121 is
[prime number greater than 5] divisible by a prime number less than 11
II. Here, 3k = {. . . − 6, − 3, 0, 3, 6, . . .}
= 10 × 12 × 122 i.e. 2, 3, 5 or 7.
3k + 1 = {. . . − 5, − 2, 1, 4, 7, . . .}
= 14640 which is not divisible by 2400. 84. (a) I. f ( k ) = 4k + 3
and 3k + 2 = {. . . − 4, − 1, 2, 5, 8, . . .}
So, statement I is not true. For k = 1, f ( 1) = 4 × 1 + 3 = 7
∴ {3k , 3k + 1, 3k + 2} For k = 2, f ( 2) = 4 × 2 + 3 = 11
II. Every square number can be of the
= {. . . − 6, − 5, − 4, − 3, − 2, − 1 form 5n or (5n ± 1) or (5n ± 4). For k = 3, f (3) = 4 × 3 + 3 = 15
0, 1, 2, 3, 4, 5, 6 . . . } So, statement II is true. Values of f ( k ) for k = 1, 2 ,... cannot
Hence, it is true. be expressed as sum of two squares,
75. (a) When N is a natural number, then since 12 + 22 = 5, 12 + 32 = 10,
66. (d) All statements are true. there is only one possible case that N, (N 22 + 32 = 13.
+ 2), (N + 4) are prime numbers.
67. (a) (i) If b / a and c / b, then a = bx and II. f ( k ) = 8k + 1
b = cy for x, y ∈ N When N = 3, then N, (N + 2), (N + 4) For k = 1, f ( 1) = (8 × 1) + 1 = 9
⇒ a = bx = cy ( x) = cxy ⇒ c / a = 3, 5, 7 all are primes numbers. For k = 2, f ( 2) = (8 × 2) + 1 = 17
(ii) If a / b and a / c, then b = ax and 76. (c) From options, For k = 3, f (3) = (8 × 3) + 1 = 25
c = ay for xy ∈ N , (c)
32
=
32
=2 (remainder) For k = 4, f ( 4) = (8 × 4) + 1 = 33
⇒ ( b + c ) = ax + ay = a ( x + y ) (3 + 2 ) 5 For k = 5, f (5) = (8 × 5) + 1 = 41
So, a / ( b + c ) 32 32 f ( k ) = 8k + 1 is square of an odd
and = =2 (remainder)
(iii) From (ii), we have ( b − c ) = a( x − y ) (3 × 2 ) 6 integer only for some values of k.
⇒ a / ( b − c) ∴ Remainder are same. So, only statement I is correct.
68. (d) All are true. and difference of quotients = 6 − 5 = 1 85. (d) To check which option is equal to
26 2 + 972 , we take the sum of unit
69. (c) Given, 33 ) 231228 ( 7006 77. (c) If x is a positive even integer and y is digit’s square of both number of the
231 negative odd integer, then x y is a
question as well as the answer options.
rational number.
228 Whichever answer option shows the
198 78. (b) We know that, between any two same result will be the answer.
30 rational numbers, there are an infinite Here, in 262 + 972 , 62 = 36
number of rational and irrational numbers.
Now, 33 − 30 = 3 and 72 = 49
Hence, only statement II is correct.
So, on adding 3 to 231228, it will be So, 36 + 49 = 85
completely divisible by 33. 79. (d) If k is a positive integer, then every
square integer is of the form 4k or 4k + 1, For option (a), 72 + 32 = 49 + 9 = 58
70. (b) Given, N = 143k + 28 as every square number is either a For option (b), 42 + 32 = 16 + 9 = 25
⇒ N = 143k + 26 + 2 multiple of 4 or exceeds multiple of 4 by For option (c), 22 + 12 = 4 + 1 = 5
⇒ N = 13 ( 11k + 2) + 2 unity. For option (d), 9 + 22 = 81 + 4 = 85
2

∴When the number is divided by 13 the 80. (c) Since, b is largest square divisor of c. Only option (d) satisfies the condition.
remainder is 2. So, c = bx
86. (a) If n is greater than 1, then n2 ( n2 − 1)
71. (a) 1 is neither prime number nor [where, x is not a whole square number] is always divisible by 12.
composite number. Also, a 2 divides c. Illustration 1 Put n = 2, then
18 CDS Pathfinder

n2 ( n2 − 1) = ( 2)2 ( 22 − 1) = 4 × 3 = 12 10 103. (a) 21x + 48 y = 5 ⇒ 3( 7x + 16 y ) = 5


= [ 2 × 13 + ( 10 − 1)4]
Illustration 2 Put n = 3, then 2 If x , y are integer, then LHS of the
n2 ( n2 − 1) = (3)2 (32 − 1) = 9 × 8 = 72 10 10 × 62
= [ 26 + 36] = = 310 above equation is multiple of 3, but the
2 2 RHS of above equation is not multiple
1723 + 2923 2323
87. (a) We have, + of 3.
23 23 94. (b) The prime numbers less than 13 are
2, 3, 5, 7, 11. ∴ There is no any integral values of x
( 1723 + 2923 ) is divisible by
Also, p < q < r < 13 and y exist.
( 17 + 29) i.e. 46
and p + q + r is a prime number. 104. (c) Let x = ( n3 − n)( n − 2), where n > 2
So, it is divisible by 23.
Also, 2323 is always divisible by 23. Hence, only two possible pairs exist i.e. Take n = 3, we get
∴ Remainder = 0 (3, 5, 11) and (5, 7, 11). x = (33 − 3)(3 − 2) = ( 27 − 3)( 1) = 24
88. (b) Let E = ( 1235 × 4523 × 2451) 95. (a) Product of unit digits [which is divisible by 6, 12 and 24]
= ( 12 × 102 + 11)( 12 × 376 + 11) = 1× 2 × 3 × 4 × 5 × 6 × 7 × 8 × 9 Take n = 4, we get x = ( 43 − 4)( 4 − 2)
× ( 12 × 204 + 3 ) × 1 × 2 × 3 × ........ × 8 × 9 = ( 64 − 4) × 2 = 120
When we divide E by 12, then =0 [which is again divisible by 6, 12 and 24]
Remainder = Remainder when ∴ Required digit in the unit place is 0. Now, take n = 5, we get
11 × 11 × 3 or 363 is divided by 12 = 3 x = (53 − 5)(5 − 2) = ( 125 − 5) × 3
96. (c) See question 88.
89. (c) Q 360 = 23 × 32 × 51 = 120 × 3 = 360
∴ Number of divisors 97. (d) See example 8.
= (3 + 1)( 2 + 1)( 1 + 1) [which is again divisible by 6, 12 and 24]
98. (c) N = 35 × 45 × 55 × 60 × 124 × 75
= 4 × 3 × 2 = 24 Hence, 24 is the largest natural number.
= 5 × 7 × 5 × 9 × 5 × 11 × 5 × 12 × 124
90. (a) Given, three-digit number = XY 5 105. (d) We have, ( 234)100 + ( 234)101
× 52 × 3
XY 5 ⇒ ( 234)100 ( 1 + 234) = 235( 234)100
and =Z = 56 × 7 × 9 × 11 × 12 × 124 × 3
X 215
Hence, the maximum value of m is 6. We know that square of any number
Here, Z can take values 1, 3, 5, 7 and 9. having 4 at unit place is a number in
But only 9 satisfies it, 99. (b) Given a − b = 4 …(i) which 6 at unit place.
135
On squaring both sides, we get
Q ×9 Any exponent of a number 6 at unit
1215 ( a − b )2 = ( 4)2 ⇒ a 2 + b 2 − 2ab = 16 place is always 6 at unit place.
then X = 1, Y = 3 and Z = 9 ⇒ 40 − 2ab = 16 [Qa 2 + b 2 = 40] Q ( 235)(…6) = … 30
Now, X + Y + Z = 1 + 3 + 9 = 13 ⇒ 2ab = 24 ⇒ ab = 12 Resulting number have 0 at unit place.
91. (d) Given number is 763X 4Y 2. 106. (a) Let the number be y.
Q a+ b = a + b + 2ab
2 2
Since, given number is divisible by 9. ∴ y = 7q + 3 and q = 11p + 6
= 40 + 2 × 12 = 8
∴ 7 + 6 + 3 + X + 4 + Y + 2 = 9k ∴ y = 7( 11p + 6) + 3
⇒ 22 + X + Y = 9k ∴ a+ b =8 … (ii)
⇒ y = 77 p + 45
It is clear that LHS is divisible by 9, if On solving Eq. (i) and Eq. (ii), we get
When divided by 11 remainder is 1 and
X + Y = 5,14 a = 6 and b = 2
quotient is 7 p + 4 and when the
When sum of X and Y is 5, then Now, a 3 + b 6 = 63 + 26 = 23 × 33 + 26 quotient is divided by 7 remainder is 4.
possible pairs are (1, 4), (4, 1), (2, 3),
= 2 3 (3 3 + 2 3 ) 107. (c) Seven digits number 876 p37q is
(3, 2), (0, 5), (5, 0). When sum of X and
Y is 14, then possible pairs are = 8 ( 27 + 8) = 8 × 35 = 280 divisible by 225, if this number is
(5, 9),(9, 5),(6, 8),(8, 6) and (7, 7). divisible by 9 and 5.
Hence, the value of a 3 + b 6 is 280.
Hence, the possible pairs are 11. If this number is divisible by 9.
100. (c) If factor of given number is of the Then, sum of its digits is divisible by 9.
92. (b) Given, an = 3 − 4n
form of p α , p α ... p α n , then number Now, sum of digits
∴ Σan = Σ(3 − 4n) 1 1 2 2 n

[ n × ( n + 1)] of prime factors are =8+ 7+ 6+ p+3+ 7+ q


= 3n − 4
2 (α + 1) (α + 1) … (α n + 1) = 31 + p + q
1 2
= 3 n − 2 n2 − 2 n Hence, the prime factor of n are ∴ p + q = 5 or p + q = 14
= n − 2n2 = − n( 2n − 1) ( x + 1)( x + 1) and ( x + 1) ∴ Given number is divisible by 225
1 2 3
when q = 5
93. (a) Let the two-digit numbers less than, 101. (b) See question 57.
50 which when divided by 4 yield unity if q = 5, p = 0 or 9.
102. (a) Let x be the required number.
as remainder be 13, 17, … , 49. 108. (a) We have, when 3x + 2 y is divided by
∴ 52x − 25x = 324 ⇒ 27x = 324 5, remainder is 2.
Here, first term, a = 13, common 324
difference, d = 4 and n = 10 ⇒ x= = 12 ∴ 3x + 2 y = 5q + 2 …(i)
n 27
∴ Required sum = [ 2a + ( n − 1)d ] and when 2x + 3 y is divided by 5, the
2 Hence, the required number is 12. remainder is 3.
MATHEMATICS Number System 19

∴ 2x + 3 y = 5m + 3 …(ii) Also, a 3 − b 3 = 117 I. Q p + q< p − q


Subtract Eq. (ii) from Eq.(i), we get ⇒ ( a − b )( a + b 2 + ab ) = 117
2
⇒ q + q< 0
x − y = 5( q − m ) − 1 117 ⇒ 2q < 0
⇒ a 2 + b 2 + ab = = 39 …(ii)
x − y = 5( q − m ) − 5 + 4 3 ⇒ q< 0
x − y = 5( q − m − 1) + 4 On subtracting Eq. (i) from Eq. (ii) ∴ q must be negative.
∴ x − y is divided by 5 remainder is 4. 3ab = 30 ⇒ ab = 10 Hence, statement I is correct.
109. (b) The sum of first 47 terms of the Now, a + b = ( a − b )2 + 4ab II. Q p + q> p − q
series
= 9 + 40 = 49 = 7 ⇒ q + q> 0
1 1 1 1 1 1 1 ∴ 2q > 0
+ − − − + + 114. (a) If R is the remainder obtained by
4 5 6 4 5 6 4
dividing the polynomial xn + m n by ∴ q must be positive irrespective of p.
1 1
+ − … 47 term ( x − m ), then ( xn + m n − R) is divisible Hence, statement II is incorrect.
5 6 by ( x − m ).
119. (d) I. a − b < a − c
It is clear that sum of first 6 term is zero. Let f ( x) = xn + m n − R ⇒b − c> 0 ...(i)
Similarly, sum of first 42 terms is zero. Q f ( x) is divisible by ( x − m ).
As, b is negative real number and c is
Sum of last 5 terms ∴ f (m ) = 0 positive real number, then Eq. (i) is
1 1 1 1 1 −1 ⇒ (m ) n + (m ) n − R = 0 not true.
= + − − − =
4 5 6 4 5 6 R = 2(m )n II. If a < b, when a and b are negative
( 243 + 647)2 + ( 243 − 647)2 As, m is a non-zero integer and n is a real numbers and c is a positive
110. (c) a b
243 × 243 + 647 × 647 positive integer, then R is a non-zero real number, then < is always
even integer. c c
2[( 243)2 + ( 647)2 ]
= =2 true for b > a.
( 243)2 + ( 647) 2 115. (c) See question 57. 1 1
III. < is always true, as c is a positive
[∴( a + b ) + ( a − b ) = 2 ( a + b )]
2 2 2 2
116. (d) x is prime and y is composite b c
111. (b) We have, N = 90 × 42 × 324 × 55 number. real number and b is a negative real
Since, x is prime number. number.
= 32 × 10 × 3 × 14 × 34 × 4 × 55
∴ x may be {2, 3, 5, 7, 11, …} and y may 120. (d) I. If m and n are distinct natural
= 37 × 10 × 14 × 4 × 55 m n
7
be {4, 6, 8, 9, 10, 12, 14, 15, …} numbers, then + is integer if and
Hence, N is divisible by 3 . Here, y may be even or odd number and n m
So, the maximum value of m is 7 when x is odd number except 2. only if m = n. Hence statement I is
N is divisible by 3m . incorrect.
∴ ( y − x) can be even number.
II. mn  +  (m 2 + n2 )−1
m n
112. (d) 710 − 510 = ( 75 )2 − (55 )2 Also, xy can be even number.  n m
= ( 75 + 55 ) ( 75 − 55 ) If y is odd integer and x ≥ 3.  m 2 + n2   1 
Then, 0.5( x + y ) is an even integer. = mn    =1
= ( 16807 + 3125) ( 75 − 55 )  mn   m 2 + n2 
= 19932 × ( 75 − 55 ) Hence, statements I, II and III are
correct. Hence, statement II is correct for all
Since, 19932 is divisible by 11. values of m and n.
117. (b) A real number is collection of all mn
Hence, 710 − 510 is divisible by 11. III. Now, 2 is a fraction.
rational and irrational numbers. m + n2
113. (a) We have, a − b = 3
So, statement II is false. So, statement III is incorrect.
Squaring both sides, we get
118. (a) Given, p and q are non-zero integers.
a 2 + b 2 − 2ab = 9 …(i)
02
20 CDS Pathfinder

SEQUENCE
AND SERIES
Usually (1-2) questions have been asked from this chapter. Questions are mostly based on
relation between arithmatic geometric and harmonic mean.

SEQUENCE
A set of numbers arranged in a definite order according to some definite rule is called a sequence.
e.g. 2, 4, 6, 8, ... is a sequence.

SERIES
If a1 , a 2 , a 3 ,... a n is a sequence, then the expression a1 + a 2 + a 3 + ... + a n is called the series.
The series is said to be finite or infinite depending upon the last term is given or not. e.g.
(i) a1 , a 2 , a 3 , . . . , a n is a finite sequence and is denoted as { a k } n .
k=1

(ii) a1 , a 2 , a 3 , a 4 , . . is an infinite sequence and is denoted by { a n } ∞


n = 1 or simply { a n }.

Here, a1 is called the first term and in case of finite sequence a n is called the last term.
e.g. 2, 4, 6, 8, … , 100 is a finite sequence.
1, 2, 3, 4, … is an infinite sequence.

PROGRESSION
Sequence following certain patterns are called progressions.
e.g. 2, 3, 4, 5, … is a progression, here each term is increasing by 1.

Arithmetical Progression (AP)


An arithmetic progression is a sequence in which the difference between any term and its preceding
term is constant throughout. The constant ‘d’ is called the common difference. The first term of an AP
is represented by ‘a’.
MATHEMATICS Sequence and Series 21

If an AP has first term = a and common difference = d, a(r n − 1)


then the general form of an AP is
• Sum of the n terms of a GP is S n = ,
r −1
a, a + d , a + 2d , a + 3d , . . . , a + ( n − 1) d a(1 − r n )
if r > 1 and S n = , if r < 1.
• nth term of an AP is Tn = a + ( n − 1) d . 1−r
Sum of the first n terms of an AP is a
• • Sum of infinite terms of a GP is S n = .
n n 1−r
S n = [ 2a + ( n − 1) d ] or S n = [ a + l ]
2 2
where, l = last term Geometric Mean (GM)
If three terms are in GP, then the middle term is called
EXAMPLE 1. Find the sum of 11 terms of the geometric mean of the other two. If a, b and c are in
−7, − 2, 3, 8 K . GP, then b is the GM of a and c.
a. 200 b. 198 c. 326 d. 137
Let a and b be two numbers and G be the GM between
Sol. d. Here, a = −7, l = −7 + (10 × 5) = 43 them. Then, a, G, b are in GP, G = ab, a > 0, b > 0.
 a + l  −7 + 43
∴ S 11 = n  = 11×  
 2  2  EXAMPLE 3. If the 4th, 10th and 16th terms of a GP
36 are x, y and z respectively, then x, y , z are in
= 11× = 198
2 a. AP b. GP
c. AGP d. HP
Arithmetic Mean (AM) Sol. b. Let the first term and common ratio be a and r,
When three terms are in AP, then the middle term is respectively.
called arithmetic mean of the other two. Given, T4 = x ⇒ ar 4 − 1 = x ⇒ ar 3 = x …(i)
10 − 1 9
If a, b and c are in AP, then b is AM of a and c. T10 = y ⇒ ar = y ⇒ ar = y …(ii)
16 − 1 15
Let a and b be two numbers and M be the AM between T16 = z ⇒ ar = z ⇒ ar =z …(iii)
a+b
them. Then, a, M, b are in AP, M = . On multiplying Eq. (i), by Eq. (iii), we get
2
ar 3 × ar 15 = xz ⇒ a2r 18 = xz
EXAMPLE 2. Find the arithmetic mean (AM) ⇒ ( ar 9 ) 2 = xz ⇒ y 2 = xz [from Eq. (i)]
between 3 and 9. Hence, x, y and z are in GP.
a. 4 b. 6
c. 8 d. None of these Harmonic Progression (HP)
3 + 9 12 A sequence is said to be harmonic progression (HP). If
Sol. b. The arithmetic mean is = = 6.
2 2 the reciprocals of its terms are in arithmetic progression
So, 3, 6, 9 are in AP. (AP).
1 1 1
Geometric Progression e.g. The sequence, 1, , , … is an HP because the
3 5 7
A geometric progression is a progression of numbers,
sequence 1, 3, 5, 7 … is an AP.
whose first term is non-zero and each of the term is
1 1 1 1
obtained by multiplying its preceding term by a If a1 , a 2 , a 3 , . . ., a n are in HP, then , , ,...,
constant quantity. This constant quantity is called the a1 a 2 a 3 an
common ratio of the GP. are in AP.
Thus, if t 1 ,t 2 and t 3 are in GP, then common ratio 1
• nth term of an HP is T n =
Second term t 2 1  1 1
r= = + ( n − 1) − 
First term t1 a1  a 2 a1 

If ‘a’ is the first term and ‘r’ is the common ratio, then
GP can be written as a, ar , ar 2 , ar 3 , …, ar n−1 = ( a ≠ 0 )
Harmonic Mean (HM)
If three terms are in HP, then the middle term is called
• n th term of a GP is Tn = ar n−1 = l. the harmonic mean of the other two. If a, b and c are
[where, l = last term] in HM, then b is the HM of a and c.
22 CDS Pathfinder

2
Let a and b be two numbers and H be the HM between  a + c 4 p2r 2 
⇒  
them.

 2 
  ( p + r) 2  = apcr [from Eqs. (i) and (ii)]
 
Then, a, b and c are in HP ( a + c) 2 pr
2ab ⇒ = ac
∴ H= ( p + r) 2
a+b
( a + c) 2 ( p + r) 2
⇒ =
Relation between Arithmetic, Geometric ac pr
and Harmonic Mean p2 r 2 a2 c2 p r a c
⇒ + + 2= + +2 ⇒ + = +
Let A, G and H be the arithmetic, geometric and pr pr ac ac r p c a
harmonic means between a and b, then
Sum to n Terms of Special Series
(i) A ≥ G ≥ H (ii ) G 2 = AH The sum of first n terms of some special series is given
below
EXAMPLE 4. If a, b, c are in AP, p, q, r are in HP and
p r 1. The sum of first n natural numbers
ap, bq, cr are in GP, then + is equal to
n ( n + 1)
r p = ∑ n = 1 + 2 + 3+…+ n =
a c a c 2
a. − b. +
c a c a 2. The sum of square of the first n natural numbers
b q b q
d. + d. −
q b q b n ( n + 1)( 2n + 1)
= ∑ n 2 = 12 + 22 +…+ n 2 =
a+ c 6
Sol. b. Since, a, b, c are in AP ⇒ b= ...(i)
2 3. The sum of cubes of the first n natural numbers
2pr 2
p, q, r are in HP ⇒ q= …(ii)  n ( n + 1)
p+ r = ∑ n 3 = 13 + 23 +…+ n 3 = 
 2 
and ap, bq, cr are in GP, b2q2 = apcr

PRACTICE EXERCISE
3 + 5 + 7 +L+ n 5. An AP consists of n (odd terms) and its middle
1. If = 7, then the value of n
5 + 8 + 11 + L + 10 terms term is m. Then, the sum of the AP is
is 1
(a) 2mn (b) mn (c) mn (d) mn2
(a) 35 (b) 36 (c) 37 (d) 40 2

2. If sum of n terms of an AP is 3n2 + 5n and 2 3 4


6. The sum of 1 + + + + L ∞ upto n terms is
Tm = 164, then m is equal to 5 52 53
(a) 26 (b) 27 (c) 28 (d) None of these 25 15
(a) (b)
16 16
3. In a GP, if the ( m + n )th term be p and ( m − n )th 5 3
(c) (d)
term be q, then its mth term is 16 2
(a) pq (b) p/q (c) q/p (d) p+ q
7. The sum of n terms of an AP is an ( n − 1). The
4. The sum of the first ‘n’ terms of the series sum of the squares of these terms is equal to
1 3 7 15
+ + + + L is a2
2 4 8 16 (a) a2n2 (n − 1)2 (b) n (n − 1) (2 n − 1)
6
(a) 2 n − n − 1 (b) 1 − 2 − n 2 a2 2a 2
−n n
(c) n (n − 1) (2 n − 1) (d) n (n + 1) (2 n + 1)
(c) n + 2 −1 (d) 2 − 1 3 3
MATHEMATICS Sequence and Series 23

8. If S be the sum to infinity of a GP, whose first 14. If the sum of first ‘n’ natural numbers is
term is a, then the sum of first n terms is n( n + 1)
n
. Then, what will be the sum of first ‘n’
2
S  1 − 
a
(a)
 S terms of the series of alternate positive and
 negative numbers when ‘n’ is even?
a 
n
(b) S 1 −  1 −  
 S  12 − 22 + 32 − 42 + 52 − K

 a 
n n (n + 1) n2 (n + 1) − n (n + 1)
(c) a 1 −  1 −   I.
2
II.
2
III.
2
 S 

(d) None of the above
Which of the above statement(s) is/are correct?
(a) Only I (b) Only III (c) Only II (d) None of these
9. If the non-zero numbers a , b, c are in AP and
tan− 1 a , tan− 1 b, tan− 1 c are also in AP, then
PREVIOUS YEARS QUESTIONS
(a) a = b = c (b) b 2 = 2 ac
(c) a2 = bc (d) c 2 = ab 15. If A, G and H are the arithmetic, geometric and
1 1 1 1 harmonic means between a and b respectively,
10. + = + , then a , b, c are in then which one of the following relations is
b− a b− c a c
(a) AP (b) GP
correct?
(c) HP (d) None of these (a) G is the geometric mean between A and H e 2015 I
(b) A is the arithmetic mean between G and H
11. The value of x + y + z is 15, if a , x , y , z , b are in (c) H is the harmonic mean between A and G
1 1 1 5
AP while the value of + + is , if a , x , y , z , b (d) None of the above
x y z 3
16. Consider the following statements in respect of
are in HP. Then, a and b are
n( n + 1)
(a) 1, 9 (b) 3, 7 the expression S n = , where ‘n’ is an
(c) 7, 3 (d) None of these 2
integer.
12. If the mth and nth term of a HP are n and m I. There are exactly two values of n for which
respectively, then the mnth term is S n = 861.
1
(a) 0 (b) 1 (c) 2 (d) II. S n = S − ( n +1) and hence for any integer m we have
2 two values of n for which S n = m .
13. If a , 2a + 2, 3a + 3 are in GP, then what is the Which of these statement(s) is/are correct? e 2016 (I)
fourth term of the GP? (a) Only I (b) Only II
(a) − 13.5 (b) 13.5 (c) − 27 (d) 27 (c) Both I and II (d) Neither I nor II

ANSWERS
1 2 3 4 5 6 7 8 9 10
11 12 13 14 15 16

HINTS AND SOLUTIONS 


n 1 3 7 15
[ 2 × 3 + ( n − 1) 2] 2. (b) Q Tm = Sm − S 4. (c) + + + +... + n terms
m −1
1. (a) Q 2 =7 ⇒ 164 = 3 ( 2m − 1) + 5 ⋅ 1 2 4 8 16
10
=  1 −  +  1 −  +  1 − 
[ 2 × 5 + ( 10 − 1) × 3] 1 1 1
2 ⇒ 6m = 162  2  4  8
n( n + 2) ∴ m = 27
⇒ =7 + ... +  1 − n 
1
5 × 37 3. (a) Tm + n = arm + n − 1 = p,  2 
⇒ n2 + 2n − 1295 = 0 Tm − n = arm − n − 1 = q
= n −  + + + ... + n 
1 1 1 1
2
⇒ n + 37n − 35n − 1295 = 0 On multiplying, we get a 2 r 2 m − 2 = pq 2 4 8 2 
⇒ ( n + 37) ( n − 35) = 0 1  1 − ( 1 / 2)n 
∴ Tm = arm − 1 = pq = n−   = n+ 2 − 1
−n
∴ n = 35 2  1− 1/ 2 
24 CDS Pathfinder

5. (c) Middle term = T ( c − b + a) ( b − c − a) ∴mnth term of HP = 1


n +1 ⇒ =
c( b − a ) a( b − c )
2 13. (c) a , 2a + 2, 3a + 3 are in GP.
n+ 1
∴ a +  − 1 d = m [given] ⇒
1
=−
1
⇒ ( 2 a + 2 ) 2 = (3 a + 3 )a
 2  c( b − a ) a( b − c )
⇒ 4a 2 + 4 + 8a = 3a 2 + 3a
2a + ( n − 1) d = 2m …(i) ⇒ ba − ca = − cb + ac
n ⇒ a 2 + 5a + 4 = 0
Now, Sn = [ 2a + ( n − 1) d ] = nm ⇒ ab + bc = 2ac
2 ⇒ a = −1, − 4
2ac
6. (a) The given sequence is arithmetic ∴ b= Now, a = −1 does not satisfy the given
1 a+ c series.
geometric series, where r = and d = 1 Hence, a , b , c are in HP.
5 ∴ −4, − 6, − 9 are in GP.
a dr 3
S∞ = +
∴ t = −4   = −13.5
11. (a) Q a, x, y, z, b are in AP. 3
1 − r ( 1 − r )2 a + b  2
∴ x + y + z = 3 
4
1 
1×  2 
1 5 14. (b) When ‘n’ is even.
= + a + b
15 = 3
1 2
1−  1 − 1 ⇒  Let n = 2m, then
5  5
  2 
= 12 − 22 + 32 − 42 + 52 − K
= +
5 5
=
25 ⇒ a + b = 10 …(i)
= ( 12 − 22 ) + (32 − 42 ) + (52 − 62 )
4 16 16 Also, a , x , y , z , b are in HP.
+ K + ( 2m − 1)2 − ( 2m )2
7. (c) Let Sn = an( n − 1), then ⇒
1 1 1 1 1
, , , , are in AP. = ( 1 + 2)( 1 − 1) + (3 + 4)( − 1)
S = a ( n − 1)( n − 2) a x y z b
n −1 + (5 + 6)( − 1) + K
∴ Tn = Sn − S = 2a ( n − 1) a + b
+ + = 3
1 1 1
n −1 ⇒  + ( 2m − 1 + 2m )( − 1)
Tn 2 = 4a 2 ( n − 1)2 x y z  2ab 
( n − 1)( n)( 2n − 1) = − ( 1 + 2 + 3 + 4 + ... + 2m )
∴ Sum = ΣTn2 = 4a 2 ⇒ − n ( n + 1)
6 5 3 × 10
= [Q a + b = 10] = −2m( 2m + 1) =
2a 2 n( n − 1)( 2n − 1) 3 2ab 2 2
=
3 ⇒ ab = 9 15. (a) Given, A, G and H are the arithmetic,
8. (b) Let r be the common ratio of GP, On solving Eqs. (i) and (ii), we get geometric and harmonic means between
then a and b, respectively.
a = 1, b = 9 or b = 1, a = 9
S=
a
, r = 1−
a a+ b 2ab
∴A = , G = ab and H =
1− r S 12. (b) Let a be the first term and d the 2 a+ b
a( 1 − r n ) a common difference of corresponding ( i) ( ii ) ( iii )
∴ Sn = = (1 − rn ) AP.
1− r 1− r On multiplying Eq. (i) and (iii), we get
1
 a n
= S 1 −  1 − 
So, m th and nth term of AP are and a+ b 2ab
  S  
n ∴ AH = × = ab = ( ab )2
1 2 a+ b
.
9. (a) Since, 2b = a + c …(i) m [from Eq. (ii)]
and 2 tan − 1 b = tan − 1 a + tan − 1 c ∴
1
= a + (m − 1) d …(i) AH = G 2
2b a+ c n
⇒ = Hence, the option (a) is correct.
1 − b2 1 − ac and
1
= a + ( n − 1) d …(ii) n ( n + 1)
⇒ b 2 = ac [from Eq. (i)] m 16. (a) I. Sn = = 861
⇒ 4b 2 = 4ac On solving Eqs. (i) and (ii), we get, 2
2
⇒ ( a + c )2 − 4ac = 0 [from Eq. (i)] 1 1 ⇒ n + n − 861 × 2 = 0
d = and a =
⇒ ( a − c )2 = 0 mn mn ⇒ ( n + 42) ( n − 41) = 0
⇒ a=c=b ∴mnth term of AP ⇒ n = − 42, 41
1 1 1 1 1 1 Hence, statement I is correct.
10. (c) Given, + = + = + (mn − 1) ×
b−a b−c a c mn mn II. Given, Sn = S
− ( n + 1)
1 1 1 1 1 + mn − 1 If Sn = m, then we have two values of
⇒ − = − = =1
b−a c a b−c mn n if and only if m is positive integer.
Hence, statement II is incorrect.
03
25

HCF AND LCM OF


NUMBERS
Usually (2-3) questions have been asked from this chapter. Generally questions based on LCM and HCF
are related to traffic lights, racitracks, largest size of tile etc. This concept is also useful in the chapters
of time and distance, time and work, pipes and cristerns etc.

FACTORS
If number ‘a’ divides the number b without leaving a remainder, then ‘a’ is said to be the factor of ‘b’.
e.g. (i) 4 is a factor of 16 as 16 = 4 × 4 (ii) 9 is a factor of 729 as 729 = 9 × 9 × 9

Prime Factors
The factors that cannot be again factorized i.e. the factors which are prime numbers are called prime factors.
e.g. (i) 9 is a factor of 729, but 3 is a prime factor of 729.
(ii) 8 and 9 are factors of 72, but 2 and 3 are prime factors of 72.

Least Common Multiple (LCM)


1. Common Multiple A common multiple of two or more numbers is a number which is exactly divisible
(without leaving remainder) by each of them.
e.g. 30 is a common multiple of 2, 3, 5, 6, 10 and 15 because 30 is exactly divisible by each number.
Similarly, 72 is a common multiple of 2, 3, 4, 6, 9, 12 and 18.
2. Least Common Multiple The least common multiple of two or more given numbers is the least number
which is exactly divisible by all the given numbers.
e.g. 84, 162, 252 are the common multiples of 2, 3, 4, 7. But, 84 is the LCM of 2, 3, 4, 7.

Methods of Finding LCM


1. Prime Factorization Method Write down the given numbers as the product of prime factors. Then, the
LCM is the product of the highest powers of all the prime factors.
26 CDS Pathfinder

EXAMPLE 1. The LCM of 30, 250, 490 is Sol. b. LCM (5, 6, 7, 8) = 840.
a. 46750 b. 36750 Here, R = 3 ⇒ Number is of the form 840 k + 3.
c. 26750 d. None of these Least value of k for which (840 k + 3) when divided by 9
leaves no remainder is 2.
Sol. b. Here, 30 = 2 × 3 × 5
250 = 5 × 5 × 2 × 5 = 2 × 53 ∴Required number = 840 × 2 + 3 = 1683
and 490 = 7 × 7 × 2 × 5 = 2 × 5 × 72
∴ LCM of 30, 250 and 490 = 2 × 53 × 72 × 3 = 36750 Highest Common Factor (HCF)
2. Division Method Write the given number in a row and 1. Common Factor A common factor of two or more
divide them with the common prime divisor. On numbers is a number which divides each of them exactly.
division. Write the quotient in each case below the e.g. 2 is common factor of 2, 10, 20.
number. If any number is not divisible by the
2. Highest Common Factor The Highest Common
respective divisor, then write it as such in the next row. Factor (HCF) of two or more numbers is the largest
Keep on dividing the quotients until you get 1. number that divides all the given numbers exactly. It is
Multiply all the divisors to get the required LCM. also known as Greatest Common Divisor (GCD).
HCF is always a factor of LCM.
EXAMPLE 2. What is the LCM of 120, 144, 160 and
e.g. HCF of the numbers 18 and 24 is 6.
180.
a. 1450 b. 1620 c. 1440 d. 1380 Methods of Finding HCF
Sol. c. 1. HCF by Prime Factorization Write the given number
2 120, 144, 160, 180
as product of prime factors and then find the product
2 60, 72, 80, 90
of least powers of common prime factors. This
2 30, 36, 40, 45
product is the required HCF of given numbers.
3 15, 18, 20, 45
3 5, 3, 10, 15 EXAMPLE 5. The HCF of 65, 75 and 105 is
5 5, 1, 10, 5
a. 4 b. 5 c. 6 d. 8
2 1, 1, 2, 1
1, 1, 1 1 Sol. b. Here, 65 = 13 × 5 , 75 = 5 × 5 × 3
and 105 = 7 × 3 × 5
∴LCM of 120, 144, 160 and 180 = 2 × 2 × 2 × 3 × 3 × 5 × 2 ∴HCF of 65, 75 and 105 = 5
= 1440
2. HCF by Division Method Suppose we have to find
IMPORTANT POINTS the HCF of two given numbers, divide the large
number by the smaller one. Now, you will get the
1. The least number which when divided by x, y and z
leaving the remainders a , b and c , respectively is remainder. Divide the divisor by the remainder. Repeat
given by [LCM of ( x, y , z ) − p ], where this process until no remainder is left, the last divisor
p = ( x − a ) = ( y − b ) = ( z − c ). used in this process is the desired greatest common
2. The least number which when divided by x, y and z divisor i.e. HCF. In order to find the HCF of three
leaving the same remainder R in each case is given numbers, then, HCF of [(HCF of any two) and (third
by [LCM of ( x, y , z ) + R ]. number)] gives the HCF of given three numbers.

EXAMPLE 3. What is the least number which when EXAMPLE 6. The HCF of 204, 1190 and 1445 is
divided by 42, 72 and 84 leaves the remainders 25, 55 a. 85 b. 15 c. 17 d. 75
and 67, respectively?
Sol. c. Here, 1190)1445 (1
a. 521 b. 512 c. 504 d. 487
1190
Sol. d. Here, difference = ( 42 − 25) = (72 − 55) = (84 − 67) = 17
255)1190 (4
Now, LCM (42, 72, 84) = 504
1020
∴ Required number = 504 − 17 = 487
170) 255 (1
EXAMPLE 4. Find the least number which when 170
divided by 5, 6, 7 and 8 leaves a remainder 3 but 85)170 (2
when divided by 9, leaves no remainder. 170
a. 1620 b. 1683 ×
c. 1635 d. 1672 So, HCF of 1190 and 1445 is 85.
MATHEMATICS HCF and L CM of Numbers 27

Now, 85) 204 (2 Sol. a. HCF of numerators i.e. 14, 21 and 7 is 7 and
170 LCM of denominators i.e. 3, 9 and 15 is 45. So, HCF of
7
34) 85 (2 given fractions = .
45
68
17) 34 (2
34
Relation between LCM and
× HCF of Two Numbers
∴ HCF of 85 and 204 is 17.
Product of two numbers = (Their HCF) × (Their LCM)
Hence, HCF of 204,1190 and 1145 is 17.
EXAMPLE 10. The LCM of two numbers is 90 times
IMPORTANT POINTS their HCF. The sum of LCM and HCF is 1456. If one of
the numbers is 160, then what is the other number?
1. For integers x, y and z , if HCF (x, y) = 1 and e 2014 II
HCF (x, z) = 1, then HCF of ( x, y , z ) is always 1.
a. 120 b. 136 c. 144 d. 184
2. The greatest number that will divide x, y and z leaving
Sol. c. Let the HCF of two numbers be x.
remainders a , b and c , respectively is given by HCF of
LCM of two numbers be 90x.
( x − a ), ( y − b ), ( z − c ).
According to the question,
3. The greatest number that will divide x, y and z leaving
the same remainder in each case is given by [HCF of LCM + HCF = 1456
| x − y | , | y − z | , | z − x |] ⇒ 90x + x = 1456
⇒ 91 x = 1456
EXAMPLE 7. Find the greatest number which will ⇒ x = 16
divide 400, 435 and 541 leaving 9,10 and 14 as ∴ HCF of two numbers = 16
remainders respectively. and LCM of two numbers = 90 × 16 = 1440
a. 19 b. 17 c. 13 d. 9 e 2014 I We know that,
Sol. b Required number = HCF of (400-9, 435-10, 541-14) LCM × HCF = Product of two numbers
= HCF of (391, 425, 527) = 17
⇒ 1440 × 16 = 160 × Second number
EXAMPLE 8. For any integers ‘a’ and ‘b’ with HCF 1440 × 16
∴Second number = = 144
(a, b) = 1, what is HCF (a + b, a − b) equal to? 160
a. It is always 1 b. It is always 2 e 2014 I
c. Either 1 or 2 d. None of these EXAMPLE 11. What is the greatest number that
Sol. c. Put arbitrary values of a and b.
divides 13850 and 17030 and leaves a remainder 17?
Illustration 1 Let a = 9 and b = 8. a. 477 b. 159 c. 107 d. 87 e 2012 II
∴ HCF (8 + 9, 9 − 8) ⇒ HCF (17, 1) = 1 Sol. b. Required number = HCF of (13850 − 17), (17030 − 17)
Illustration 2 Let a = 23 and b = 17. = HCF of (13833, 17013) = 159

∴ HCF (17 + 23, 23 − 17) ⇒ HCF ( 40, 6) = 2 EXAMPLE 12. There are three drums with 1653 litre
Hence, HCF (a + b, a − b) can either be 1 or 2. 2261 litre and 2527 litre of petrol. The greatest
possible size of the measuring vessel with which we
How to Calculate LCM and can measure the petrol of any drum while every time
HCF of Fractions the vessel must be completely filled is
a. 31 b. 27 c. 19 d. 41
The LCM and HCF of fractions can be obtained from Sol. c. The maximum capacity of the vessel = HCF of 1653,
the following formula 2261 and 2527 = 19
HCF of numerators
1. HCF of fractions =
LCM of denominators EXAMPLE 13. John, Kate and smith at same time,
same point and in same direction to run around a
LCM of numerators
2. LCM of fractions = circular in 150 seconds. Find after what time will they
HCF of denominators meet again?
14 21 7 a. 30 min b. 25 min c. 20 min d. 15 min
EXAMPLE 9. The HCF of , , is Sol. b. LCM of 250, 300 and 150 = 1500 sec
3 9 15
7 2 7 = 25 min.
a. b. 3 c. d. None of these Hence, john, kate and smith meet after 25 min.
45 5 30
28 CDS Pathfinder

PRACTICE EXERCISE
1. If x = 23 × 32 × 54 and y = 22 × 32 × 5 × 7, then 13. What is the HCF of a 2b4 + 2a 2b2 and ( ab)7 − 4a 2b9?
HCF of x and y is (a) ab (b) a2 b 2 (c) a2 b 3 (d) a3 b 2
(a) 180 (b) 360 (c) 540 (d) 35
14. If a number is exactly divisible by 11 and 13,
2. LCM of 2 × 3 × 5 and 2 × 5 × 7 is
3 4
which of the following types the number must be?
(a) Divisible by (11+13) (b) Divisible by (13 −11)
(a) 2 12
× 3× 5 ×7
2
(b) 2 × 5 × 7 × 9
4
(c) Divisible by (11×13) (d) Divisible by (13 ÷11)
(c) 2 4 × 3 × 5 × 7 (d) 2 3 × 3 × 5 × 7
15. What is the sum of the digits of the least number
4 3 7
3. LCM of , and is which when divided by 52, leaves 33 as
5 10 15 remainder, when divided by 78 leaves 59 and
2 8 4 when divided by 117, leaves 98 as remainder ?
(a) 8 (b) (c) 20 (d) 16
3 15 5 (a) 17 (b) 18 (c) 19 (d) 21
3 9 15
4. What is the HCF of , and ? 16. For any integer n, what is the HCF of integers
2 7 14 m = 2n + 1 and k = 9n + 4 ?
3 3 3
(a) (b) (c) (d) 3 (a) 3 (b) 1 (c) 2 (d) 4
7 14 2
17. For any three natural numbers a , b and c, if HCF
5. The least number divisible by 12, 15, 20 and is a
 a b
perfect square is ( a , b) = c, then HCF  ,  is
 c c
(a) 900 (b) 400 (c) 36 (d) 256
(a) a / c (b) b / c (c) c (d) Always 1
6. The least number which when divided by 5, 6, 7
and 8 leaves a remainder 3 is 18. Raj, Rachit and Asha begin to tag around a circular
(a) 423 (b) 843 (c) 1683 (d) 2523 stadium. They complete their revolutions in 42 s,
56 s and 63 s, respectively. After how many seconds
7. The HCF of two numbers is 1/5th of their LCM. will they be together at the starting point?
If the product of the two numbers is 720, then (a) 366 (b) 252 (c) 504 (d) 605
the HCF of the numbers is
(a) 13 (b) 12 (c) 14 (d) 18
19. Find the side of the largest possible square slabs
which can be paved on the floor of a room
8. The LCM of two numbers is 39780 and their 2m 50cm long and 1m 50cm broad. Also, find the
ratio is 13 : 15. Then, the numbers are number of such slabs to pave the floor.
(a) 273, 315 (b) 2652, 3060 (a) 25,20 (b) 30,15 (c) 50,15 (d) 55,10
(c) 516, 685 (d) None of these
20. Four bells begin to toll together and toll,
9. If the highest common factor of two positive respectively at intervals of 5, 6, 8 and 12 s. How
integers is 24, then their least common multiple many times will they toll together in an hour
cannot be excluding the one at the start?
(a) 72 (b) 216 (c) 372 (d) 600 (a) 10 (b) 19 (c) 13 (d) 9

10. If the HCF of three numbers 144, x and 192 is 21. 21 mango trees, 42 apple trees and 56 orange
12, then the number x cannot be trees have to be planted in rows such that
(a) 180 (b) 84 (c) 60 (d) 48 each row contains the same number of trees of
one variety only. What is the minimum number
11. Consider those numbers between 300 and 400 of rows in which the above trees may be planted?
such that when each number is divided by 6, 9 (a) 3 (b) 15 (c) 17 (d) 20
and 12, it leaves 4 as remainder in each case.
What is the sum of the numbers? 22. A person has four iron bars whose lengths are
(a) 692 (b) 764 (c) 1080 (d) 1092 24 m, 36 m, 48 m and 72 m, respectively. This
person wants to cut pieces of same length
12. What is the smallest positive integer which when from each of four bars. What is the least number
divided by 4, 5, 8 and 9 leaves remainder 3, 4, 7 of total pieces, if he is to cut without any
and 8, respectively? wastage?
(a) 119 (b) 319 (c) 359 (d) 719 (a) 10 b) 15 (c) 20 (d) 25
MATHEMATICS HCF and L CM of Numbers 29

23. For two natural numbers m and n, let gmn 33. For any integer n, HCF of (22n + 7, 33n + 10) is
denote the greatest common factor of m and n. equal to e 2014 I
Consider the following in respect of three natural (a) n (b) 1 (c) 11 (d) None of these
numbers k, m and n.
I. gm ( nk) = g(mn) k II. gmn gnk = gmk 34. In a fire range, 4 shooters are firing at their
respective targets. The first, the second, the
Which of the above statement(s) is/are correct?
third and the fourth shooter hit the target once
(a) Only I (b) Only II
in every 5 s, 6 s, 7 s and 8 s, respectively. If all
(c) Both I and II (d) Neither I nor II
of them hit their target at 9 : 00 am, when will
24. Consider the following in respect of integers a they hit their target together again? e 2014 I
and b (a) 9 : 04 am (b) 9 : 08 am
I. HCF (a , b) = HCF (a + b, b) (c) 9 : 14 am (d) None of these
II. HCF (a , b) = HCF (a , b − a ) for b > a 35. If a and b be positive integers, then HCF of
Which of the above statement(s) is/are correct?  a b 
 ,  equal to? e 2014 I
(a) Only I (b) Only II  HCF ( a,b) HCF ( a,b)
(c) Both I and II (d) None of these a
(a) a (b) b (c) 1 (d)
HCF (a, b )
PREVIOUS YEARS’ QUESTIONS 36. The HCF of two natural numbers m and n is 24
and their product is 552. How many sets of
25. The HCF and LCM of two natural numbers are values of m and n are possible? e 2014 II
12 and 72, respectively. What is the difference (a) 1 (b) 2 (c) 4
between the two numbers, if one of the numbers (d) No set of m and n is possible satisfying the given
is 24? e 2012 I conditions
(a) 12 (b) 18 (c) 21 (d) 24
37. The LCM of two integers is 1237. What is their
26. The sum of two numbers is 232 and their HCF is HCF? e 2014 II
29. What is the number of such pairs of numbers (a) 37 (b) 19
satisfying the above condition? e 2012 I (c) 1 (d) Cannot be determined
(a) One (b) Two (c) Four (d) None of these
38. There are 48 cricket balls, 72 hockey balls and
27. The product of HCF and LCM of 18 and 15 is 84 tennis balls and they have to be arranged in
e 2012 II several rows in such a way that every row
(a) 120 (b) 150 (c) 175 (d) 270 contains the same number of balls of one type.
28. Three planets revolve round the Sun once in 200, What is the minimum number of rows required
for this to happen? e 2014 II
250 and 300 days, respectively in their own
(a) 12 (b) 16 (c) 17 (d) 19
orbits. When do they all come relatively to the
same position at a certain point of time in their 39. Consider all positive two digit numbers each of
orbits? e 2012 II which when divided by 7 leaves a remainder 3.
(a) After 3000 days (b) After 2000 days What is their sum? e 2015 II
(c) After 1500 days (d) After 1200 days (a) 661 (b) 666 (c) 676 (d) 777

29. The LCM of two numbers is 2376 while their 40. What is the sum of digits of the least multiple of
HCF is 33. If one of the numbers is 297, then the 13, which when divided by 6, 8 and 12 leaves 5, 7
other number is e 2013 I
and 11, respectively, as the remainders? e 2015 II
(a) 5 (b) 6 (c) 7 (d) 8
(a) 216 (b) 264 (c) 642 (d) 792
30. The HCF of two numbers is 98 and their LCM is 41. The LCM of two numbers is 12 times their HCF.
The sum of HCF and LCM is 403. If one of the
2352. The sum of the numbers may be e 2013 II
numbers is 93, then the other number is e 2015 II
(a) 1372 (b) 1398 (c) 1426 (d) 1484 (a) 124 (b) 128 (c) 134 (d) 138
31. If for integers a, b and c, HCF ( a , b) = 1 and HCF 42. Consider the following in respect of natural
( a , c) = 1 , then which one of the following is numbers a , b and c e 2016 I
correct? e 2013 II I. LCM (ab, ac) = a LCM (b, c)
(a) HCF (a, bc ) = 1 (b) HCF (a, bc ) = a II. HCF (ab, ac) = a HCF (b, c)
(c) HCF (a, bc ) = b (d) None of these III. HCF (a , b) < LCM (a , b)
32. What is the number of integral solutions of the IV. HCF (a , b) divides LCM (a , b).
equations HCF (a, b) = 5 and a + b = 65 ? e 2014 I Which of the above statement(s) is/are correct?
(a) Less than 65 (b) Infinitely many (a) III and II (b) III and IV
(c) Exactly one (d) None of these (c) I, II and IV (d) All of these
30 CDS Pathfinder

ANSWERS
1 2 3 4 5 6 7 8 9 10
11 12 13 14 15 16 17 18 19 20
21 22 23 24 25 26 27 28 29 30
31 32 33 34 35 36 37 38 39 40
41 42

HINTS AND SOLUTIONS 


1. (a) Given, x = 23 × 32 × 54 9. (c) In the given options, only 372 is not 16. (b) Since, m = 2n + 1 is an odd integer,
and y = 22 × 32 × 5 × 7 divisible by 24. Therefore, LCM of so its factors may be 1 or 3 and
∴ HCF = 22 × 32 × 5 = 4 × 9 × 5 = 180 numbers cannot be 372. k = 9 n + 4 its factors may be 1, 2 and 4.
2. (c) Here, say a = 23 × 3 × 5 and 10. (d) Here, we know that Hence, HCF of (m , k ) is 1.
144 = 12 × 2 × 2 × 3 17. (d) It is always 1
b = 24 × 5 × 7, then and 192 = 12 × 2 × 2 × 2 × 2 Illustrations Let a = 21 and b = 35
LCM = 24 × 3 × 5 × 7 By taking option (d), 48 = 12 × 2 × 2
Then, HCF (21, 35) = 7
3. (d) Here, LCM Hence, the value of x will not be 48
∴ HCF  ,  = HCF (3, 5) = 1
21 35
LCM of 4, 3, 7 84 4 otherwise the HCF of given numbers  7 7
= = = 16 becomes 48.
HCF of 5, 10, 15 5 5
11. (a) LCM of 6, 9 and 12 = 36 18. (c) Required time = LCM of 42,56 and
4. (b) HCF  , ,  =
3 9 15 HCF (3, 9, 15) 63 s LCM of 42, 56 and 63 is
So, number is the form of 36 p + 4.
 2 7 14  LCM (2, 7, 14) 2 42, 56, 63
Since, the required numbers are between
3 3 21, 28, 63
= 300 and 400.
14 ∴ p = 9 and 10 7 7, 28, 21
5. (a) 2 1, 4, 3
12, 15, 20 ∴ Required sum = 328 + 364 = 692
2 6, 15, 10 ∴ Required time
12. (c) Here,
3 3, 15, 5 = 2 × 3 × 7 × 4 × 3 = 504 s.
4−3=5− 4=8− 7= 9−8= 1
5 1, 5, 5 19. (c) Side of largest possible square slab is
Now, 4 = 2 × 2, 5 = 5 the HCF of 250 cm and 150 cm
1, 1, 1
8 = 2 × 2 × 2, 9 = 3 × 3 250 = 5 × 5 × 5 × 2
LCM of 12, 15 and 20 = 2 × 2 × 3 × 5
∴ LCM = 5 × 2 × 2 × 2 × 3 × 3 = 360 150 = 5 × 5 × 3 × 2
∴ Required perfect square
Required number = 360 − 1 = 359 ∴ HCF is 50. Then, number of slabs
= 2 × 2 × 3 × 3 × 5 × 5 = 900
13. (b) a 2 b 4 + 2a 2 b 2 = a 2 b 2 ( b 2 + 2) …(i) Area of floor 250 × 150
6. (b) To find out the least number, firstly = = = 15
we find out the LCM of given numbers. and ( ab ) − 4a b = a b − 4a b
7 2 9 7 7 2 9
Area of slab 50 × 50
∴ LCM (5, 6, 7, 8) = a 2 b 2 ( a 5 b 5 − 4b 7 ) …(ii) 20. (a) Here, LCM of 5, 6, 8 and 12 is 360, so
= LCM (5, 2 × 3, 7, 23 ) = 840 From Eqs. (i) and (ii), we get the bells will toll after 360 s.
∴ Required number = 840 + 3 = 843 HCF = a 2 b 2 So, in an hour they will toll together
60 × 60
7. (b) Let LCM = 5x, then HCF = x 14. (c) LCM of 11 and 13 will be ( 11 × 13). = = 10 times
Now, product of numbers = 720 Hence, if a number is exactly divisible 360
So, 5x × x = 720 by 11 and 13, then the same number 21. (c) The HCF of (21, 42, 56) is 7.
⇒ 5x2 = 720 ⇒ x = 12 must be exactly divisible by their LCM ∴ The minimum number of rows
i.e. ( 11 × 13). 21 42 56
8. (b) The numbers are 13x and 15x. = + +
So, x is the HCF. Now, 15. (a) Here, 52 − 33 = 78 − 59 7 7 7
= 117 − 98 = 19 = 3 + 6 + 8 = 17
HCF × LCM = Product of numbers 52 = 13 × 2 × 2
Now, 22. (b) Here, 24 = 12 × 2, 36 = 12 × 3
x × 39780 = 13x × 15x ⇒ 78 = 13 × 2 × 3
48 = 12 × 4, 72 = 12 × 6
⇒ x × 39780 = 13 × 15 × x2 ⇒ 117 = 13 × 3 × 3
39780 ∴ LCM = 13 × 2 × 2 × 3 × 3 = 468 ∴ HCF ( 24, 36, 48, 72) = 12
⇒ x= = 204. 24 36 48 72
13 × 15 ∴ Required number = 468 − 19 = 449 Total piece = + + +
12 12 12 12
∴ Numbers are 13 × 204 = 2652 and Hence, the sum of the digits is 17.
15 × 204 = 3060 = 2 + 3 + 4 + 6 = 15
MATHEMATICS HCF and L CM of Numbers 31

23. (d) I. Let three natural numbers are 30. (a) It is given that, the HCF of two 38. (c) Given, number of cricket balls
m = 16, n = 15, k = 20 numbers is 98. This means that both the = 48 = 24 × 3
∴ gm = g =4 numbers are multiples of 98. Therefore,
( nk ) 16 ( 300 ) the sum of these two numbers must also Number of hockey balls = 72 = 23 × 32
and g = g = 20 be a multiple of 98. Among all the four and number of tennis balls
(m n ) k 240 ( 20 )
∴ g ≠ g options given, only option (a) satisfies = 84 = 22 × 3 × 7
m ( nk ) ( mn ) k
this condition.
II. g mn g = g g = 1×5 =5 ∴ HCF of 48, 72 and 84 = 22 × 3 = 12
nk 16 , 15 15 , 20
and g = g =4 31. (a) For integers a, b and c, if HCF (a, b) = Now, minimum number of rows
mk 16 , 20 1 and HCF (a, c) = 1 then, HCF (a, b c)
∴ g mn g ≠ g 48 72 84
nk mk =1 = + +
So, neither I nor II is correct. 12 12 12
32. (a) HCF ( a , b ) = 5
24. (c) I. Let a = 4, and b = 10 = 4 + 6 + 7 = 17
Let a = 5x and b = 5 y
∴ a + b = 14 39. (c) The required numbers are 10, 17,
∴ 5x + 5 y = 65 ⇒ x + y = 13
24,…, 94.
HCF ( 4, 10) = 2 ∴ Number of pairs of ( x, y ) = ( 1, 12),
Total number of numbers is 13.
and HCF (14, 10) = 2 (2, 11), (3, 10), (4, 9), (5, 8), (6, 7)
Sum of these numbers
∴ HCF ( a , b ) = HCF ( a + b , b ) Hence, number of solutions is less than
13 13
II. Let a = 6 and b = 15 65. = [ 10 + 94] = × 104
2 2
∴ b − a = 15 − 6 = 9 33. (b) HCF of (22n + 7, 33n + 10) is
always 1. = 13 × 52 = 676
HCF (6, 15) = 3
Illustration For n = 1, HCF (29, 43) 40. (d) Here, 6 − 5 = 1, 8 − 7 = 1
HCF (6, 9) = 3
=1 12 − 11 = 1
∴ HCF ( a , b ) = HCF ( a , b − a )
LCM × HCF For n = 2, HCF (51, 76) = 1 LCM of 6,8 and 12 = 24
25. (a) Second number =
First number For n = 3, HCF (73, 109)= 1 Required number
72 × 12 34. (c) Time after which they will hit the = 24 k − 1, k is any natural number
= = 36 target together again
24 For k = 6, the number = 144 − 1 = 143
∴ Difference between two numbers = LCM of 5, 6, 7 and 8 which is multiple of 13
= 36 − 24 = 12 = 5 × 3 × 7 × 2 × 2 × 2 = 840 s So, sum of digits = 1 + 4 + 3 = 8
26. (b) Let two numbers be 29x and 29 y. Duration after which they will hit target 41. (a) Let other number be b and HCF be x.
together
29 x + 29 y = 232 ⇒ x + y = 8 ⇒ LCM = 12x
840
∴ ( x, y ) = ( 1, 7), (3, 5) = = 14 min.
60 We have, x + 12x = 403 ⇒ 13x = 403
Hence, one such pair is 87 and 145. So, they will hit the target together after ∴ x = 31
and the other pair is 203 and 29. 14 min. Product of two numbers
27. (d) Here, 18 = 2 × 3 × 3 and 15 = 3 × 5 Hence, they will hit together again at = LCM × HCF
9 : 14 am. ∴ 93 × b = x × 12x
HCF of 18 and 15 = 3
 a b  ⇒ 93 × b = 12 × 31 × 31
LCM of 18 and 15 = 2 × 3 × 3 × 5 = 90 35. (c) HCF  ,  is
 HCF a, b HCF a, b 
∴ b = 124
∴ Product of HCF and LCM of both always equal to 1.
numbers = 3 × 90 = 270 42. (d) a , b and c are natural numbers.
Illustration Let the two positive
28. (a) Given that, three planets revolves the I. LCM of ( ab , ac ) = abc
integers be a = 24 and b = 36.
Sun once in 200, 250 and 300 days, a × LCM of ( b , c ) = abc
 
∴HCF  
respectively in their own orbits. 24 36 Hence, statement I is correct.
,
∴ Required time = LCM of 200, 250  HCF ( 24, 36) HCF ( 24, 36)  II. HCF ( ab , ac ) = a HCF ( b , c )
 
and 300 = 3000 days HCF of ( ab , ac ) = Common factor of
HCF  ,  ⇒ HCF (2, 3) = 1
24 36
( ab , ac )
Hence, after 3000 days they all come  12 12 
relatively to the same position at a and a × HCF ( b , c ) = a × common
certain point of time in their orbits. 36. (d) LCM of two natural numbers factor of ( b , c )
Product of m and n 552 Hence, statement II is correct.
29. (b) Given, LCM of two numbers = 2376 = = = 23
HCF of two numbers = 33 HCF of m and n 24 III. We know that HCF is always less
Here, no set of m and n is possible than LCM.
One of the number = 297
satisfying the given conditions as LCM Hence, statement III is correct.
(HCF of two numbers) × (LCM of is always a multiple of HCF. IV. HCF ( a , b ) divides LCM ( a , b )
two numbers) because a common factor
37. (c) Given, LCM of two integers is 1237,
= (First number) × (Second number) between a , b always divides ( a × b ).
which is a prime number.
33 × 2376 Hence, statement IV is correct.
∴ Second number = = 264 So, their HCF is 1.
297
04
32 CDS Pathfinder

DECIMAL FRACTIONS
Regularly (1-2) questions have been asked from this chapter. It is one of the most common chapters which
we are studying from the very starting age.

FRACTION
If any unit is divided into some parts, then ...... each of these parts is called fraction of that unit. A
fraction is represented as p/q, where q ≠ 0 and here q is called as denominator and p is called as
numerator.
e.g. 1/ 2, 3/ 4, 6 /7 etc. are fractions.

Simple fraction
6 2
The fraction which has denominator other than power of 10, is called simple fraction. e.g. , etc.
5 9
Note Simple fraction is also known as vulgar fraction.

DECIMAL FRACTION
Fraction that has powers of 10 in the denominator are called decimal fraction. e.g.
1 1
(i) is the tenth part of 1 is written as 0.1. (ii) is the hundredth part of 1 is written as 0.01.
10 100

Types of Decimals
1. Recurring Decimals A decimal number in which a digit or set of digits repeats regularly is called
non-terminating repeating decimals or recurring decimals. To represent these fractions, a line is
drawn on repeating digits.
1
e.g. = 0 .1428571428571 . . . = 0. 142857
7
2
= 0 .66666 . . . = 0 .6
3
4
= 1.33333 . . . = 1.3
3
MATHEMATICS Decimal Fractions 33

2. Pure recurring decimal A decimal fraction in Sol. c. Here,


which all the digits after the decimal points are 51.300
repeated, is called a pure recurring decimal. 7.078
e.g. 0.786786786 . . . = 0. 786 1.380
+ 0.900
0.77777 . . . = 0.7
60.658
EXAMPLE 1. Which one of the following is a
non-terminating repeating decimal? Note Adding/Annexing zeroes to the extreme right of a decimal
fraction does not change its value.
13 3 3 137
a. b. c. d.
8 16 11 25
3
Multiplication
Sol. c. Q = 0 ⋅ 272727... While multiplying two or more decimal fractions,
11
consider them without decimal point and multiply them
= 0 ⋅ 27 as usual. In the product so obtained the decimal point is
13 13 × 125 1625 marked off as many places from the right as in the sum
= = = 1.625
8 8 × 125 1000 of the decimal places in the given numbers.
3 3 × 625 1875
= = = 0.1875 EXAMPLE 4. 5 × 0 . 25 × 6.301 × 0.00394 = ?
16 16 × 625 10000
a. 0.0310324220 b. 0.0310324210
137 137 × 4 548
= = = 5.48 c. 0.0310324250 d. None of these
25 25 × 4 100
From above it is clear that all of these are terminating Sol. c. For the product of 5 × 0.25 × 6.301 × 0.00394
3 Here, 5 × 25 × 6301 × 394 = 310324250
decimals. Hence, is a non-terminating repeating
11 Total decimal places ( 2 + 3 + 5) = 10
decimal. ∴ 5 × 0. 25 × 6. 301 × 0. 00394 = 0.031032425
3. Mixed recurring decimal A decimal fraction in
which some digits after the decimal point are not Division
repeated while some are repeated, is called a mixed While dividing the given decimal fraction by a natural
recurring decimal. number, divide it without the decimal point to get
e.g. 0.179999 . . . = 0.179 quotient. Here, in the quotient so obtained, place the
0.053939 . . . = 0.0539 decimal point after as many places from the right as are
there in the dividend.
Note The decimal expansion of a rational number is either
terminating or non-terminating recurring. In other words, a For dividing a decimal fraction by a decimal fraction,
number whose decimal expansion is terminating or multiply the dividend and the divisor by a suitable power
non-terminating recurring is a rational number. of 10 to make the divisor a whole number and then
proceed as in the previous rule.
Operations on Decimal Fractions,
Addition and Subtraction EXAMPLE 5. 0.01834 ÷ 13 = ?
a. 141.077 b. 1.41077
While making addition or subtraction of decimal c. 0.00141077 d. None of these
fractions the numbers are placed in such a way that the
decimal point lie in one column. Then, the numbers Sol. c. Here, for 0.01834 ÷ 13
can be added or subtracted as usual. 1834
⇒ = 141077
.
EXAMPLE 2. 7.093 − 3.57 = ? 13
a. 3.523 b. 3.513 So, 0 . 01834 ÷ 13 = 0 . 00141077
c. 3.143 d. 3.532
EXAMPLE 6. 0 .0066 ÷ 0 .22 = ?
Sol. a. Here, 7.093
– 3.570 a. 0.03 b. 0.3 c. 3 d. None of these
3.523 Sol. a. 0 . 0066 ÷ 0 . 22
EXAMPLE 3. 51.3 + 7 .078 + 1.38 + 0 .9 = ? 0.0066 × 10000 66 3
So, = = = 0.03
a. 61.668 b. 59.238 0.22 × 10000 22 × 100 100
c. 60.658 d. None of these
34 CDS Pathfinder

To Convert a Pure Recurring Decimal 2 5 16 3


EXAMPLE 10. Arrange , , , in ascending
into a Simple Fraction 3 6 25 7
In order to convert a pure recurring decimal fraction
order.
2 5 16 3 16 5 3 2
into simple (vulgar) fraction, we write the repeated a. , , and b. , , and
figures only once in the numerator without decimal 3 6 25 7 25 6 7 3
point and write as many nines in the denominator as the 3 16 2 5
c. , , and d. None of these
number of repeating figure. 7 25 3 6
5 45
e.g. 0. 5 = or 0. 45 = Sol. c. Firstly, we convert each of the fraction in decimal
9 99 form.
2 5 16 3
EXAMPLE 7. 1.27 in the form p/q is equal to = 0. 666, = 0.833, = 0.64, = 0. 428571
3 6 25 7
127 73 14 11
a. b. c. d. Here, 0. 428571 < 0.64 < 0. 666 < 0.8333
100 100 11 14 3 16 2 5 3 16 2 5
99 + 27 126 14 Hence, < < < and so , , and are in
Sol. c. 1. 27 = 1 + 0. 27 = 1 + 27 / 99 = = = 7 25 3 6 7 25 3 6
99 99 11 ascending order.
7 5 6
To Convert Mixed Recurring Decimal EXAMPLE 11. Arrange , , in descending order
8 6 7
into a Simple Fraction
a. 6/7, 5/6, 7/8 b. 7/8, 5/6, 6/7
The numerator is obtained by taking the difference
c. 7/8, 6/7, 5/6 d. 6/7, 7/8, 5/6
between the number formed by all the digits after decimal
point. (Here, repeated digits are taken only once) and the Sol. c. LCM of 8, 6, 7 = 168
number formed by non-repeating digits. The
7 7 × 21 147 5 5 × 28 140
denominator is obtained by placing, as many nines as in ∴ = = ⇒ = =
repeating digits followed by as many zeroes as the 8 8 × 21 168 6 6 × 28 168
number of non-repeating digits. 6 6 × 24 144
and = =
7 7 × 24 168
EXAMPLE 8. The vulgar fraction of 0.1236 is
140 144 147
120 102 93 Q < <
a. b. c. d. None of these 168 168 168
823 825 825
1236 − 12 1224 102 7 6 5
Sol. b. 0.1236 = = = So, descending order of fractions is , , .
9900 9900 825 8 7 6

EXAMPLE 9. What is the value of 1. 34 + 4.12 ? HCF and LCM of


a.
133
b.
371
c. 5
219
d. 5
461 Decimal Fractions
90 90 990 990
For finding the HCF or LCM of decimal fractions, first
134 − 1 133 412 − 41 371 make the given fractions to have same number of
Sol. d. Q 1. 34 = = and 4.12 = =
99 99 90 90 decimal places by annexing zeroes, if needed. Now, find
133 371 1330 + 4081 5411 461 the HCF or LCM of the numbers without considering
∴134
. + 4.12 = + = = =5
99 90 990 990 990 decimal. Finally, in the result mark as many decimal
places as are there in each of the given numbers.
Comparison of Fractions By EXAMPLE 12. The HCF and LCM of 1.600, 8 and 2.4 is
Converting in Decimal Form a. 0.800 and 24 b. 0.6000 and 38
To compare the fraction using decimal, convert each one c. 0.900 and 42 d. None of these
of the given fractions in decimal form and now arrange
them in ascending or descending order as per the Sol. a. We can write the given numbers as 1.600, 8.000,
requirement. 2.400.
So, without decimal, the numbers are 1600, 8000 and
By Equating Denominators 2400.
Now, HCF of 1600, 8000 and 2400 is 800.
For comparison of fractions, take LCM of the
denominator of all fractions. So, that their denominators ∴ HCF of 1.600, 8.000 and 2.400 is 0.800.
are same. Now, the fractions having largest numerator is And LCM of 1600, 8000, 2400 is 24000.
the largest fraction. ∴ LCM of 1.600, 8.000 and 2.400 is 24.000.
MATHEMATICS Decimal Fractions 35

PRACTICE EXERCISE
1. If ( 15 . 9273 − x ) = 11. 0049, then the value of x is p
15. If 2 . 5252525 . . . =(in the lowest form), then
(a) 4.9224 (b) 0.4922 (c) 0.4294 (d) 6.932 q
q
what is the value of ?
2. If ( 15. 39 + 0 . 236 + 5 . 290 + 0 . 0002 ) = x , then the p
value of x is (a) 0.4 (b) 0.42525 (c) 0.0396 (d) 0.396
(a) 0.20916 (b) 2.0916 (c) 209.16 (d) 20.9162
16. What decimal of an hour is a second?
3. If 175 × 1. 24 = 2.17, then the value of 1.75 × 124 is (a) 0.25 (b) 0.0256 (c) 0.00027 (d) 0.0125
(a) 217 (b) 0.0217
(c) 2.17 (d) None of these 3 5
17. If 5 = 2.24, then the value of is
4. If 111.744 ÷ 28.8 = 3.88, then the value of 2 5 − 0.48
(a) 1.68 (b) 16.8 (c) 168 (d) 0.168
1117.44 ÷ 288 is
(a) 3.88 (b) 0.388 (c) 388.0 (d) 38.8  ( 0. 1) − ( 0. 01)
2 2

18. The value of  + 1 is equal to
( 0. 5) 4
− ( 0.4)
4
 0. 0001 
5. The value of is equal to
( 0. 5) + ( 0.4)2
2 (a) 1001 (b) 11 (c) 101 (d) 100
(a) 0.9 (b) 0.09 (c) 9 (d) 0.009 3
19. The value of is equal to
0.004 × 0.0008 0. 3 − 3.03
6. The value of equals to 3+
0.02 3 × 0. 91
(a) 0.000016 (b) 0.00016 (a) 0.75 (b) 1.5 (c) 15 (d) 0.15
(c) 0.0016 (d) None of these
20. What should be subtracted from the
7. Which of the following sets of the fractions is in multiplication of 0.527 and 2.013 to get 1?
ascending order? (a) 0.939085 (b) 0.060851 (c) 1.91984 (d) 2.16085
6 7 5 11 5 6 7 11
(a) , , , (b) , , ,
8 9 6 13 6 8 9 13 21. The value of 0. 3467 + 0. 1333 is equal to
11 5 7 6 11 7 6 5 (a) 0.48 (b) 0.4801
(c) , , , (d) , , ,
13 6 9 8 13 9 8 6 (c) 0. 48 (d) 0.48
8. When 0.232323…is converted into a fraction, 22. The greatest fraction out of
2 5 11
, ,
7
and is
then the result is 5 6 12 8
1 2 23 23 7 5 11 2
(a) (b) (c) (d) (a) (b) (c) (d)
5 9 99 100 8 6 12 5
9. If 3. 245 × 10k = 0.0003245, then the value of k is 3
23. Which of the following fractions is greater than
(a) 4 (b) –4 (c) 3 (d) 5 4
5
10. The value of ( 0.6 + 0.8 + 0.7 ) is and less than .
6
1 1 1 1 1 2 4 9
(a) 2 (b) 2 (c) 2 (d) 1 (a) (b) (c) (d)
8 9 3 9 2 3 5 10
11. The value of ( 6. 88 − 2. 58) is 24. Consider the following statements:
(a) 4. 30 (b) 4.29 (c) 3.22 (d) 4. 38 1
I. cannot be written as a terminating decimal.
12. The expression (11.98 × 11.98 + 11.98 × x + 0.02 22
× 0.02) will be a perfect square for x equal to 2
II. can be written as a terminating decimal.
(a) 0.02 (b) 0.2 (c) 0.04 (d) 0.4 15
1
x+ y III. can be written as a terminating decimal.
13. If 2. 5x = 0. 5 y, then the value of is 16
x− y
Which of the statement(s) given above is/are
(a) −1.3 (b) −1.5 (c) 1.5 (d) 1.3
correct?
14. 7.2 exceeds its one-tenth by (a) Only I (b) Only II
(a) 8.48 (b) 5.48 (c) 6.48 (d) 5.28 (c) I and III (d) II and III
36 CDS Pathfinder

p
25. Consider the following decimal numbers: 28. Representation of 0. 2341 in the form , where p
q
I. 1.16666666… II. 1.181181118…
and q are integers, q ≠ 0, is e 2013 I
III. 2.010010001… IV. 1.454545…
781 1171 2341 2339
(a) (b) (c) (d)
Which of the above numbers represent(s) rational 3330 4995 9990 9990
number(s)?
29. Let p be a prime number other than 2 or 5. One
(a) Only IV (b) II and III
(c) I and IV (d) None of these
would like to express the vulgar fraction 1/ p in
the form of a recurring decimal. Then, the
decimal will be e 2015 I
PREVIOUS YEARS’ QUESTIONS (a) a pure recurring decimal and its period will be
necessarily ( p − 1)
26. What is the value of 0.007 + 17. 83 + 310. 0202 ? (b) a mixed recurring decimal and its period will be
necessarily ( p − 1)
(a) 327.86638 (b) 327.86638 e 2012 I
(c) a pure recurring decimal and its period will be some
(c) 327.86683 (d) 327.8668 factor of ( p − 1)
(d) a mixed recurring decimal and its period will be
27. What is the value of 0.242424…? e 2012 II
some factor of ( p − 1)
23 8 7 47
(a) (b) (c) (d)  
99 33 33 198 30. The value of ( 0. 63 + 0. 37) is e 2015 II
100 100 1000
(a) 1 (b) (c) (d)
91 99 999

ANSWERS
1 2 3 4 5 6 7 8 9 10
11 12 13 14 15 16 17 18 19 20
21 22 23 24 25 26 27 28 29 30

HINTS AND SOLUTIONS


1. (a) x = 15.9273 − 11.0049 = 4.9224 =
. )2 − ( 0.4)2 ][( 05
[( 05 . )2 + ( 0.4)2 ] 10k = 10−4
2. (d) 15.3900 . )2 + ( 0.4)2 ]
[( 05 So, k = − 4
0.2360 = ( 05
. )2 − ( 0.4)2 = ( 05
. − 0.4)( 05
. + 0.4) 6 8 7
10. (c) 0.6 + 0.8 + 0.7 = + +
5.2900 [Rule 1] 9 9 9
= 0.1 × 0.9 = 0.09 21 7 1
+ 0.0002 [Rule 2] = = =2
9 3 3
20.9162 0.004 × 0.0008 0.0000032
6. (b) = 11. (a) Here, 6.88 − 2.58
x = 20.9162 0.002 0.02
=  6 +
88   58 
= 0.00016  − 2 + 
3. (a) 1.75 × 1.24 = 2.17  99   99 
6 7 5
⇒ 1.75 × 124 = 217 7. (a) Here, = 0.75, = 0.7 ,
= ( 6 − 2) + 
88 58 
8 9 6 − 
111.744 11  99 99 
4. (a) Since, = 3.88 = 083
. , = 0846
.
28.8 88 − 58 30
13 =4 + =4 + = 4.30
111744 So, 0.75 < 0.7 < 083
. < 0846
. 99 99
⇒ = 3.88 [Rule 6]
28800 12. (c) Given expression

Now,
1117.44 11744
= = 3.88 [Rule 4] 8. (c) Given, 0.232323 …= 0. 23 = ( 1198
. )2 + ( 0.02)2 + 1198
. ×x
288 28800 23
(which is a recurring decimal) = For the given expression to be a perfect
99 square, we must have
. )2 ] 2 − [( 0. 4)2 ] 2
[( 05 9. (b) Here, 10k =
0.0003245
5. (b) 3.245 . × x = 2 × 1198
1198 . × 0.02
. )2 + ( 0. 4)2
( 05
3.245 × 10−4 ⇒ x = 0.04
[( a 2 − b 2 ) = ( a + b ) ( a − b )] = [by using ( a + b )2 = a 2 + b 2 + 2ab]
3.245
MATHEMATICS Decimal Fractions 37

13. (b) 2.5x = 0.5 y 3 3 25. (c) Since, 1.16666… and 1.454545… are
19. (b) =
x .
05 3 + 0.3 − 3.03 3 − 273 recurring numbers and we know that,
⇒ = = 0. 20
y 2⋅5 3 × 0.91 3 × 91 recurring numbers represent rational
numbers.
Now, the expression is [dividing
3 3
numerator and denominator by y] = = = 1.5 Hence, statements I and IV are rational
x + y x / y + 1 0.20 + 1 120
. 3−1 2 numbers.
= = =
x− y x / y − 1 0.20 − 1 −080 . 26. (b) 0.007 = 0.007777777
20. (b) Let x should be subtracted
= −15
. ( 0.527 × 2.013) − x > 1 ⇒ 17.83 = 1783838383
.
Shortcut Method 1.060851 − x > 1 310.0202 = 310. 020222222
x 05 . x + y 05. + 25
.
= ⇒ = x = 0.060851
y 2.5 x− y . − 25
05 . = 3278663838
. on adding
21. (b) 0.34 67 + 0.1333 = 32786638
.
[using componendo and dividendo]
x+ y 3467 − 34 1333 − 13
⇒ =
3
= − 1.5 = + 27. (b) Given, 0.242424... = 0. 24
x − y −2 9900 9900
3433 + 1320 4753 [which is a recurring decimal number]
7.2 = = 24 8
14. (c) 7. 2 − 9900 9900 = =
10 99 33
4801 − 48
⇒ 7. 2 − 0. 72 = 6.48 = = 0. 4801
2341 − 2 2339
p 9900 28. (d) 0. 2341 = =
15. (d) Given, = 2.52 …(i) 9990 9990
2 5 11
q 22. (c) = 0.4, = 083
. , = 0.916
5 6 12 29. (c) Value p may be 3, 7, 11, 13.
Now, 100 multiply both sides, we get
7 1
100 p and = 0875. = 0.3 , Period = 1
= 252.52 …(ii) 8 3
q
On subtracting Eq. (i) from Eq. (ii), we
Clearly, the greatest fraction is 0.916 i.e. Here, p − 1 = 2 and 1 is a factor of 2.
11 1
get . = 0. 142857, Period = 6
99 p q 99 12 7
= 250 ⇒ = = 0.396
q p 250 3 5
23. (c) = 0.75, = 0833
. Here, p − 1 = 6 and 6 is a factor of 6.
1 4 6
16. (c) Required decimal fraction = 1
= 0. 09, Period = 2
60 × 60 1
= 05
2 4
. , = 0.66, = 08
. and
9
= 0.9 11
1 2 3 5 10
= = 0.00027 Here, p − 1 = 10 and 2 is a factor of 10.
3600 Clearly, 0.8 lies between 0.75 and  
3 5 3 × 2. 24 0.8333. 30. (c) We have, 0. 63 + 0.37
17. (a) =
2 5 − 0.48 2 × 2. 24 − 0.48 4 3 5
∴ lies between and . Let x = 0.63 6363 63 …
5 4 6
[Q 5 = 2.24] 100x = 63.636363 …, 99x = 63
1 2
6.72 6.72 24. (c) In and , 22 and 15 are not in 63
= = = 168
. 22 15 and x =
4. 48 − 0. 48 4 1 99
the form of 2m × 5n but in . 
16 Similarly, y = 0.37 = 0373737
.
 ( 0.1) − ( 0.01)
2 2

18. (d) Here, + 1
  16 in the form of 24 × 50 . So,
1 37
 0 .0001  can be Q y=
16
0.01 − 0.0001  99
+ 1
0.0099
= + 1=  written as a terminating decimal.
0.0001  0.0001  ∴ x+ y =
63 37 100
+ =
Hence, statements I and III are correct. 99 99 99
= ( 99 + 1) = 100
05
38 CDS Pathfinder

SQUARE ROOTS
AND CUBE ROOTS
Generally (1-3) questions have been asked from this chapter. This section will be useful in solving
simplification questions and will save lots of time while doing fuzzy calculations.

SQUARE
The square of a number is obtained by multiplying the number by itself.
e.g. Square of 5 = 5 × 5 = 25, square of 6 = 6 × 6 = 36
Perfect square A natural number n is called a perfect square or a square number, if there exists a
natural number m such that n = m2 .
e.g. The numbers 1, 4, 9, 16, 25, ... are perfect square.

Properties of Perfect Square


The properties of perfect squares are discussed below.
(i) A number having 2, 3, 7 or 8 at unit’s place is never a perfect square.
e.g. 172, 2783
(ii) The squares of odd numbers are odd and the squares of even numbers are even.
(iii) The difference of squares of two consecutive natural numbers is equal to their sum.
e.g. 7 2 − 6 2 = 7 + 6 = 13
(iv) The product of four consecutive natural numbers plus one is a always square.
(v) A number ending in an odd number of zeroes is never a perfect square.
Square Root
The square root of a number x is that number which when multiplied by itself gives x as the product.
It is denoted by .
In general, y 2 = x ⇒ y = x
Here, y is the square root of x, if and only if x is the square of y.
e.g. 4 = 2× 2 = 2
Thus, 2 is the square root of 4.
MATHEMATICS Square Roots and Cube Roots 39

x 4 Step III Put the quotient above the period and write the
EXAMPLE 1. If = , then the value of x is product of divisor and quotient just below the
64 8
first period.
a. 16 b. 12 c. 8 d. 4
x 4
Step IV Subtract the product of divisor and quotient from
Sol. a. We have, = , the first period and bring down the next period to
64 8 the right of the remainder. This becomes the new
On squaring both sides, we get dividend.
2
 x  4
2
x 16 Step V Double the quotient and enter it with a blank on
  =   ⇒ = ⇒ x = 16
 64  8 64 64 its right.
Step VI Guess a largest possible digit to fill the blank
Methods to Find Square Root which will also become the next digit in the
The square root of a given number can be determined quotient, such that when the new divisor is
by using any of the following two methods multiplied to the new quotient the product is less
1. Prime Factorization Method than or equal to the dividend.
2. General Method/Division Method Step VII Repeat the above steps till all the periods have
been taken up.
Prime Factorization Now, the quotient so obtained is the required
In this method, we express the given number as the square root of the given number.
product of prime factors. Now, for finding square root,
we take the product of these prime factors choosing one EXAMPLE 4. The square root of 1522756 is
out of every pair of same prime factors. a. 1182 b. 1222
c. 1234 d. 1334
EXAMPLE 2. The square root of 213444 is Sol. c
a. 332 b. 368 c. 432 d. 462
1 1 52 27 56 1234
Sol. d. The prime factorisation of 213444 is
1
213444 = 2 × 2 × 3 × 3 × 7 × 7 × 11 × 11
22 52
∴ 213444 = 2 × 2 × 3 × 3 × 7 × 7 × 11× 11
×2 44
Now, taking one number from each pair and multiplying
them, we get 213444 = 2 × 3 × 7 × 11 = 462 243 0827
×3 729
EXAMPLE 3. What is the square root of 2464 9856
0. 324 × 0.64 × 129.6
? ×4 9856
0 . 729 × 1. 024 × 36 ×
a. 4 b. 3 Hence, the square root of 1522756 is 1234.
c. 2 d. 1
EXAMPLE 5. The least number of four digits which
0.324 × 0.64 × 129.6 324 × 64 × 1296 is a perfect square is e 2012 II
Sol. d. =
0.729 × 1024
. × 36 729 × 1024 × 36 a. 1204 b. 1024
( 22 × 34) × ( 26) × ( 24 × 34) ( 2 × 32) × 23 × ( 22 × 32) c. 1402 d. 1420
= =
( 36) × ( 210) × ( 22 × 32) 33 × 25 × ( 2 × 3) Sol. b. Here, the greatest three-digit number = 999
18 × 8 × 36
= =1 31
27 × 32 × 6
3 9 99
Division Method ×3 9
To find the square root of a given number using this 61 99
method, the following steps are to be followed. 61
Step I In the given number, place bars over every pair of 38
digits starting with the unit’s digit. Each pair and Here, the greatest number of three digits which is
the remaining one digit (if any) on the extreme left perfect square = 999 − 38 = 961 = ( 31) 2
is called a period.
So, the smallest four-digit number which is perfect
Step II Think of the largest number whose square is less square = ( 31+ 1) 2 = ( 32) 2 = 1024
than or equal to the first period.
40 CDS Pathfinder

CUBE POWERS OR EXPONENTS


If a number is multiplied three times with itself, then the An expression that represents repeated multiplication of
result of this multiplication is called the cube of that number. the same factor is called a power. It is written as ‘x n ’
e.g. 8 = 2 × 2 × 2 = 23 and is read as ‘x’ raised to the power n.
Here, x is called the ‘base’.
Properties of Cube and n is called the ‘exponent’.
1. Cubes of all even natural numbers are even and all e.g. 65 = 6 × 6 × 6 × 6 × 6 = 7776.
odd natural numbers are odd.
2. Cubes of the numbers ending in digit 0, 1, 4, 5, 6 and Laws of Exponents
9 are the numbers ending in the same digit. x
Let be any rational number and m, n be any integers.
3. Cubes of negative integers are negative. y
4. Cubes of numbers ending in digits 3 and 7 ends in Then,
m n m+ n
digit 7 and 3, respectively.  x  x  x
(i)   ×   =  
 y  y  y
CUBE ROOT m n m− n
 x  m
n mn
 x  x  x  x
The cube root of a given number is the number whose (ii)   ÷   =  (iii)    =  
cube is the given number.  y  y  y  y    y
In general, we can say that a 3 = b, then cube root of ‘b’ −n n 0
 x  y  x
is ‘a’. Cube root of a number ‘a’ is denoted as 3 a. (iv)   =  (v)   =1
 y  x  y
e.g. 3 64 = 4 × 4 × 4 = 4. Thus, 4 is the cube root of 64.
EXAMPLE 8. If 2m + 21 + m = 24, then the value of m is
x 5
EXAMPLE 6. If 3 = , then the value of x is a. 1 b. 2 c. 3 d. 5
27 3
1+ m
a. 125 b.25 c. 27 d. 9 Sol. c. Given, 2 + 2 m
= 24 ⇒ 2 + 2 × 2m = 24
m 1

x 5 ⇒ 2m(1 + 2) = 24 ⇒ 2m × 3 = 24 ⇒ 2m = 8 = 23
Sol. a. We have 3 =
27 3 Now, on comparing both sides, we get
On cubing both sides, we get ∴ m=3
3
 x  5
3
x 125 Hence, the value of m is 3.
 3  =   ⇒ =
 27  3 27 27
∴ x = 125 SURDS OR RADICALS
Let x be a rational number and n be a positive integer,
Method to Find Cube Root such that n x is irrational, then n x is called a radical or
Method to calculate the cube root of a number is as follow surd of order n and here x is called the radicand.
• A surd of order 2 is called a quadratic surd i.e. 2, 3.
Prime Factorization Method • A surd of order 3 is called a cubic surd. i.e. 3 2, 3 3.
In this method, we express the given number as the • A surd of order 4 is called a biquadratic surd i.e. 4 5 , 4 7 .
product of prime factors.
Now, for finding cube root, we take the product of Laws of Radicals
these prime factors choosing one out of every three, The laws of exponents which are applicable to the surds
from the factors. are
EXAMPLE 7. Find the cube root of 373248. (i) ( n x ) n = x
a. 42 b. 52 c. 62 d. 72 (ii) n xy = n x ⋅ n y
Sol. d. Here, 373248 = 8 × 8 × 8 × 9 × 9 × 9 x n
x
(iii) n =
373248 = 2 × 2 × 2 × 2 × 2 × 2 y n y
× 2× 2× 2× 3× 3× 3× 3× 3× 3
(iv) ( n x ) m = ( n x m )
⇒ 3
373248 = 2 × 2 × 2 × 3 × 3 = 72
Hence, the cube root of 373248 is 72. (v) m n
x = mn x = n m x
MATHEMATICS Square Roots and Cube Roots 41

Types of Surds Multiplication and Division of Surds


p q
1. Pure surds A surd which has only 1 as a rational Suppose given surds are a, b and r c , then first find
factor is called a pure surd. the LCM of p, q and r to make the radial powers of the
e.g. 2, 3 2, 4 3 all are pure surds. surds same and then their multiplication and division
can be done using following rules.
2. Mixed surds A surd which is not a pure surd or has n
factor other than unity is called a mixed surd. x x
(i) n x × n y = n xy (ii) =n
4 n y y
e.g. 2 3, 5 3 12, 3 35 are mixed surds.
3
3. Like and Unlike surds When the radicands of two EXAMPLE 11. Simplify 6 12 ÷ 3 ⋅ 3 2.
surds are same, then those are known as like surds. a. 3 3 b. 3
e.g. 6 3 and 22 ⋅ 3. c. 2 3
d. 3
1
When radicands of two surds are different, then they 3
3
are called unlike surds. e.g. 4 7 and 6 5. Sol. d. Here, the order of 3, 2 is 2 and 3, respectively.
So, LCM 2, 3 and 6 = 6
Comparison of Two Surds
3 = 31/ 2 = ( 36/ 2)1/ 6 = ( 33)1/ 6 = ( 27)1/ 6 = 6 27
1. If two surds are of the same order, then the one
whose radicand is larger, is the larger of the two.
3
2 = ( 2)1/ 3 = ( 26/ 3)1/ 6 = ( 22)1/ 6 = 6 4
e.g. 17 > 13, 3 21 > 3 16 6
12 6
12 6
12 12 1
So, = = =6 =6
2. If any two surds have different orders are to be 3⋅ 3 2 6
27 ⋅ 6 4 6 27 × 4 27 × 4 9
compared, then we will first reduce them to the same  1 2
1/ 6 2/ 6 1/ 3
=    =  = 
1 1 1
but smallest order and then compare them. =3
   3  3
 3  3
Note Let n x be a surd of order n.
m
Then, n x = x m/ n i.e. x 1/ n = ( x m/ n )1/ m is a surd of order m. Rationalisation of Surds
Process of converting surd into rational number is called
EXAMPLE 9. Convert 2 into a surd of order 4.
as rationalisation of surds.
a. 4 2 b. 4 8 c. 4 4 d. None of these
When the product of two surds is a rational number, then
Sol. c. 2=2 1/ 2
= (2 4 / 2 1/ 4
) = (2 )
2 1/ 4
= 4
4
each surd is called a rationalising factor of each other.
EXAMPLE 10. Consider the following in respect of e.g. 3
25 × 3 5 = 3 25 × 5 = 3 125 = 3 5 3 = 5
the numbers 2, 3 3, and 6 6 . e 2014 (I)
So, 3 25 and 3 5 are rationalising factor of each other.
6
I. 6 is the greatest number.
II. 2 is the smallest number. Rationalising factors of a given surd
Which of these statements is/are correct? 1
• Rationalising factor of = a
a. Only I b. Only II c. Both I and II d. Neither I nor II a
Sol. d. Taking LCM of 2, 3 and 6 = 12 1
• Rationalising factor of = am b
Now, 2 = 2 1/ 2
=2 6 /12
= 12
2 =
6 12
64 a± b
1 2
6 = (6) 6 = (6)12 = 12 62 = 12 36
6 5+ 3 5− 3
EXAMPLE 12. + is equal to
3
3 = 31/ 3 = 34/12 = 12 81 5− 3 5+ 3
So, neither I nor II is correct. a. 16 b. 8 c. 4 d. 15
Operations on Surds Sol. b.
5+ 3
+
5− 3
5− 3 5+ 3
Addition and Subtraction of Surds ( 5 + 3)( 5 + 3) ( 5 − 3) ( 5 − 3)
= + ×
Addition or subtraction takes place only between like ( 5 − 3)( 5 + 3) ( 5 + 3) ( 5 − 3)
surds, using laws of numbers. e.g. ( 5 + 3) 2 ( 5 − 3) 2 ( 5 + 3) 2 + ( 5 − 3) 2
(i) 2 3 + 4 3 + 5 = 6 3 + 5 = + =
5 − 3
2 2
5 − 3
2 2 ( 5) 2 − ( 3) 2
(ii) 6 5 − 3 2 + 7 5 + 4 2 = (6 5 + 7 5 ) + ( 4 2 − 3 2 ) 2[( 5) + ( 3) ]
2 2
2( 5 + 3)
= = =8
= 13 5 + 2 5− 3 2
42 CDS Pathfinder

PRACTICE EXERCISE
x 4 14. Find the value of
1. If = , then the value of x is
49 7
343 + 307 + 273 + 241 + 225
(a) 9 (b) 25 (c) 16 (d) 8
(a) 18 (b) 19 (c) 19 (d) 18 + 3
2. Which of the following cannot be a digit in the
1 1 1 1
unit place of a perfect square? 15. What is − + −
(a) 1 (b) 5 (c) 7 (d) 0 9− 8 8− 7 7− 6 6− 5
1
3. The number 0.0001 is + equal to?
5− 4
(a) a rational number less than 0.01
1
(b) a rational number (a) 0 (b) 1 (c) 5 (d)
3
(c) an irrational number
(d) Neither a rational number nor an irrational number 16. What is one of the square roots of 9 − 2 14?
4. If m and n are natural numbers, then m
n is (a) 7 − 3 (b) 6 − 3 (c) 7 − 5 (d) 7 − 2
(a) always irrational 17. If p = r = m and r = p = n , then which one of
x y w z
(b) irrational unless n is the mth power of an integer
the following is correct?
(c) irrational unless m is the nth power of an integer
(a) x w = yz (b) x z = yw
(d) irrational unless m and n are coprime
(c) x + y = w + z (d) x − y = w − z
5. What is that fraction which when multiplied by
18. If a x = b y = cz and abc = 1, then what is
itself gives 227.798649?
(a) 15.093 (b) 15.099 (c) 14.093 (d) 9.0019 xy + yz + zx equal to
(a) xyz (b) x + y + z (c) 0 (d) 1
6. If x = 3018 + 36 + 169 , then the value of x is 5+1 5−1
19. If a = and b = , then the value of
(a) 55 (b) 44 (c) 63 (d) 42 5−1 5+1
7. If 3 x = x / 5, then x is equal to  a 2 + ab + b2 
 2  is
(a) 5 5 (b) 55
6
(c) 56
5
(d) 5  a − ab + b2 
(a) 3/4 (b) 4/3 (c) 3/5 (d) 5/3
8. What is the value of
29.16 + 0.2916 + 0.002916 + 0. 00002916 ? 20. If 3 5 + 125 = 17.88, then what will be the value
(a) 5.9949 (b) 5.9894 (c) 5.9984 (d) 5.9994 of 80 + 6 5
− 3/ 2 (a) 13.41 (b) 20.46 (c) 21.66 (d) 22.35
9. ( 5 )−5/ 2 × ( 5)
1 1 1 21. If N = 20.15 and N b = 16, then b is equal to
(a) (b) (c) (d) None of these
625 25 125 (a) 80 / 3 (b) 5 / 3 (c) 4 (d) 3 / 5
10. If a = 3, b = 9 and c = 10, then the value of 22. If 6 = 2, 6 = 5 and 6 = 15, then Q equals to
A B Q

13 + a + 112 + b + c − 1 is (a) B + 3 (b) 5A + B (c) B2 − 2 B (d) B − A + 1


4 6 12
(a) 15 (b) 18 (c) 16 (d) 10 23. Arrange 3 , 10 , 25 in descending order.
11. If 0. 9 × 0.09 × x = 0. 9 × 0.09 × z, then the value (a) 6 10 > 4 3 > 12 25 (b) 12 25 > 4 3 > 6 10
x (c) 6 10 > 12 25 > 4 3 (d) 4 3 > 12 25 > 6 10
of is
z
24. If ( 3.7)x = ( 0.037) y = 10000, then what is the value
(a) 0.081 (b) 0.810 (c) 0.81 (d) 8.09
1 1
of − ?
12. 2 2 2 2 2 is equal to x y
1 1
(a) 1 (b) 2 (c) (d)
(a) 0 (b) 2 (c) 1 (d) 2 31/ 32 2 4
7+ 5
13. If 35 = 5. 9160, then the value of is 25. The greatest six digit number which is a perfect
7− 5 square is
(a) 9.1060 (b) 10.9160 (c) 11.9160 (d) 12 (a) 998004 (b) 998006 (c) 998049 (d) 998001
MATHEMATICS Square Roots and Cube Roots 43

26. What is the smallest number by which 26244 37. If 16 × 8n + 2 = 2m , then m is equal to e 2013 I
must be divided to get a perfect cube? (a) n + 8 (b) 2 n + 10
(a) 4 (b) 6 (c) 36 (d) 16 (c) 3 n + 2 (d) 3 n + 10
27. What is the smallest number that must be added 2
to 1780 to make it a perfect square?
38. When a ball bounces, it rises to of the height
3
(a) 39 (b) 49 (c) 59 (d) 69 from which it fell. If the ball is dropped from a
28. A gardener plants 17956 trees in such a way height of 36 m, how high will it rise at the third
that there are as many rows as there are trees in bounce? e 2013 I
a row. The number of trees in a row are 1 2 1 2
(a) 10 m (b) 10 m (c) 10 m (d) 12 m
(a) 136 (b) 164 (c) 134 (d) 166 3 3 3 3
29. A group of student decided to collect as many 39. The product of four consecutive natural numbers
paise from each member of the group as is the plus one is e 2014 I
number of members. If the total collection (a) a non-square
amounts to ` 32.49 the number of members in the (b) always sum of two square numbers
group, is (c) a square
(a) 37 (b) 47 (c) 57 (d) 27 (d) None of the above
30. A general arranges his soldiers in rows to form a 40. The difference of two consecutive cubes e 2014 I
perfect square. He find that in doing, so (a) is odd or even (b) is never divisible by 2
60 soldiers are left out. If the total number of (c) is always even (d) None of these
soldiers be 8160. Then, the number of soldiers in
each row is ( 0.75)3
41. The square root of + [0.75 + ( 0.75)2 + 1] is
(a) 90 (b) 91 (c) 92 (d) 80 1 − 0.75
e 2015 I
31. A ball is dropped from a height 64 m above the
(a) 1 (b) 2 (c) 3 (d) 4
ground and every time it hits the ground it rises
to a height equal to half of the previous. What is 5 + 10
the height attained after it hits the ground for 42. What is equal to?
5 5 − 2 20 − 32 + 50
the 16th time?
e 2015 I
(a) 2 −12 m (b) 2 −11 m (c) 2 −10 m (d) 2 −9 m
(a) 5 (b) 5 2 (c) 5 5 (d) 5
a + 3b + a − 3b 91 1
32. If x = , then 3bx 2 − 2ax + 3b is 43. If x = , then the value of 3 − is
a + 3b − a − 3b 216 (1 −x )1/ 3 e 2015 II
equal to (given that, b ≠ 0). 9 5 4 4
(a) (b) (c) (d)
(a) 1 (b) 0 (c) ab (d) 2ab 5 9 9 5
44. Which one of the following is correct? e 2015 II
PREVIOUS YEARS’ QUESTIONS (a) 2 < 4 6 < 3 4 (b) 2 > 4 6 > 3
4
(c) 4 6 < 2 < 3 4 (d) 4 6 > 2 > 3
4
33. If a x = b, b y = c and xyz = 1, then what is the 1 1
value of cz ? e 2012 I 45. If x = 3 + 2, then the value of x3 + x + + , is
x x3
(a) a (b) b (c) ab (d) a/b
e 2015 II
34. If 196 x = x , then x is equal to which one of
4 6 3
(a) 10 3 (b) 20 3 (c) 10 2 (d) 20 2
the following? e 2012 I
46. If x = 21/ 3 + 2− 1/ 3 , then the value of 2x3 − 6x − 5 is
x6 x2
(a) (b) 14x4 (c) (d) 14x2 equal to e 2016 I
14 14 (a) 0 (b) 1 (c) 2 (d) 3
35. If 10 + 3 x = 4, then what is the value of x? 47. If 4x 2 y = 128 and 33 x 32 y − 9xy = 0, then the value
e 2012 I of x + y can be equal to e 2016 I
(a) 150 (b) 216 (c) 316 (d) 450 (a) 7 (b) 5 (c) 3 (d) 1
36. The expression [( 2 ) 2 ] 2
gives e 2013 I a + 2b + a − 2b
(a) a natural number 48. If x = , then bx 2 − ax + b is
a + 2b − a − 2b
(b) an integer and not a natural number
(c) a rational number but not an integer equal to (given that b ≠ 0) e 2016 I

(d) a real number but not a rational number (a) 0 (b) 1 (c) ab (d) 2ab
44 CDS Pathfinder

ANSWERS
1 2 3 4 5 6 7 8 9 10
11 12 13 14 15 16 17 18 19 20
21 22 23 24 25 26 27 28 29 30
31 32 33 34 35 36 37 38 39 40
41 42 43 44 45 46 47 48

HINTS AND SOLUTIONS 


x 4 x 4 4
1. (c) Given, = or = = ( 5 )− 4 =  
1 = 343 + 307 + 17
49 7 7 7  5
= 343 + 18 = 361 = 19
Now, on squaring both sides, we get (1 / 2 ) × 4 2
=   =   =
1 1 1 1 1 1
15. (c) − +
( x )2 = ( 4)2 ⇒ x = 16 5 5 25 9− 8 8− 7 7− 6
Hence, the value of x is 16. 1 1
10. (b) Given, a = 3, b = 9 and c = 10 − +
6− 5 5− 4
2. (c) The digits 2, 3, 7 and 8 cannot be at ∴ 13 + a + 112 + b + c−1
the unit place of a perfect square =( 9 + 8) − ( 8 + 7)
= 13 + 3 + 112 + 9 + 10 − 1
number. +( 7+ 6) − ( 6 + 5)
= 16 + 121 + 9 = 4 + 11 + 3 = 18
3. (b) 0.0001 = 0.01 which is a rational +( 5+ 4)
number. x 0.9 × 0.09
11. (a) Here, = [on rationalisation]
z 0.9 × 0.09
4. (b) If m and n are natural numbers, then = 9+ 4 =3+ 2=5
m n is irrational unless n is mth power of 0.9 × 0.09 × 0.9 × 0.09
=
an integer. 0.9 × 0.09 16. (d) 9 − 2 14 = 7+ 2−2× 7 × 2

5. (a) Let fraction be x, then = 0.9 × 0.09 = ( 7− 2) =2


7− 2
2
⇒ 
x = 227.798649
2 x
 = ( 0.9 × 0.09 )
2
 z 17. (a) Given, p x = r y = m and
⇒ x= 227.798649 = 15.093
x rw = pz = n
⇒ = 0.081
6. (a) Given, x = 3018 + 36 + 169 z Now, px = r y
On multiply power with w on both
= 3018 + 36 + 13
12. (d) Here, 2 2 2 2 2 sides, we get
= 3018 + 49 = 3018 + 7 ( p x )w = ( r y )w ⇒ p xw = r yw
= 2 2 2 2 × 21 / 2 = 2 2 2 × 23 / 4 ⇒ px w = ( rw ) y ...(i)
= 3025 = 55
On putting r w
= p in Eq. (i), we get
z
Hence, the value of x is 55. = 2 2 × 27 / 8 = 2 × 215 / 16 = 231 / 32
p xw = ( p z ) y ⇒ p x w = p zy
7. (c) 3 x = x /5 7+ 5
13. (c) Here, On comparing both sides, we get
On cubing both sides, we get 7− 5
∴ xw = zy
x = x 3 / 53 7+ 5 7+ 5 ( 7+ 5 )2
On squaring both sides, we get = × = 18. (c) Given, a x = b y = c z = k [Let]
7− 5 7+ 5 7−5
x6 ⇒ a=k 1/x
, b=k 1/ y
x = x 6 / 56 ⇒ 56 = = x5
x 7 + 5 + 2 35 12 + 2 35 and c = k1 / z
= = 1
+
1
+
1
⇒ x 5 = 56 ⇒ x = 5 56 2 2 ∴ abc = k x y z

8. (d) 29.16 + 0.2916 + 0.002916 [Q ( a + b )2 = a 2 + b 2 + 2ab ] 1 1 1


+ +
+ 0.00002916 = 6 + 35 = 6 + 5.9160 = 119160
. ⇒ 1 = kx y z = k0
= 5.4 + 054
. + 0.054 + 0.0054 [Q abc = 1, given]
14. (b) 343 + 307 + 273 + 241 + 225
= 5.9994 On comparing both sides, we get
1 1 1
9. (b) ( 5 )− 5 / 2 × ( 5 )− 3 / 2 = 343 + 307 + 273 + 241 + 15 + + =0
x y z
= ( 5 )− 5 / 2 − 3 / 2 = ( 5 )− 8 / 2
= 343 + 307 + 273 + 16
⇒ xy + yz + zx = 0
MATHEMATICS Square Roots and Cube Roots 45

5+1 5+1 ⇒
4 4
− =3 − 1⇒
1 1 1
− = a + 3b + a − 3b
19. (b) a = × 32. (b) Given, x =
5−1 5+1 x y x y 2 a + 3b − a − 3b
1 1 1
( 5 + 1)2 3 + 5 Hence, the value of − is ⋅ By rationalising, we get
= = x y 2
5−1 2 ( a + 3b )2 + ( a − 3b )2
x=
5−1 5−1 25. (d) Here, the greatest six-digit number ( a + 3b )2 − ( a − 3b )2
b= × = 999999
5+1 5−1 2 ( a + 3b )( a − 3b )
999 +
( 5 − 1) 3− 5
2 ( a + 3b )2 − ( a − 3b )2
= = 9 99 99 99
5−1 2 81 a + 3b + a − 3b + 2 a 2 − 9b 2
2 2 x=
3 + 5  3 − 5  189 1899 a + 3b − a + 3b
∴ a2 + b2 =   + 
 2   2  ×9 1701 2a + 2 a 2 − 9b 2
x=
2( 9 + 5) 1989 19899 a + 3b − a + 3b
= =7
4 ×9 17901
(a + a 2 − 9b 2 )
5+1 5−1 1998 =
Also, ab = × =1 (3 b )
5−1 5+1 Here, the greatest number of six digit
a 2 + ab + b 2 ( a 2 + b 2 ) + ab
which is perfect square 3bx2 − 2ax + 3b = 0
∴ 2 = = 999999 − 1998 = 998001
a − ab + b 2 ( a 2 + b 2 ) − ab 33. (a) Given, a x = b , b y = c and x yz = 1
7+ 1 4 26. (c) We have, Now, a x = b
= =
7−1 3 26244 = 2 × 2 × 3 × 3 × 3 × 3 × 3 × 3 × 3 × 3 On multiplying both sides by y in
So, to get a perfect cube, power, we get
20. (d) 3 5 + 125 = 1788
.
26244 must be divided by 2 × 2 × 3 × 3 ( ax ) y = ( b) y ⇒ a xy = b y
⇒ 3 5 + 5 5 = 1788
. i.e., 36.
Here, b = c y
⇒ ax y = c
⇒ 8 5 = 1788
.
27. (d) We know that, ( 42)2 = 1764 and Again, multiply with z in power, we get
1788
.
⇒ 5= = 2.235 ( 43)2 = 1849
8 ( a x y )z = c z ⇒ a x y z = cz
Since, 1764 < 1780 < 1849
∴ 80 + 6 5 = 4 5 + 6 5 Hence, the smallest number that must be Given, xyz = 1 ⇒ a = c z
= 10 5 = 10 × 2.235 = 22.35 added to 1780 is ( 1849 − 1780), i.e. 69. ∴ cz = a
21. (a) N = 2 0.15
28. (c) Given, total number of tress = 17956 Hence, the value of c z is a.
⇒ N = ( 2) 3 / 20
⇒ ( N ) = ( 2)
b 3 b / 20
∴ Number of trees in each row x6
34. (d) 196x4 = x6 , 196 = = x2
But, N b = 16 = 17956 = 134 x4
∴ 16 = ( 2)3 b / 20 ⇒ 24 = ( 2)3 b / 20 29. (c) Total rupees collected = ` 32.49 ⇒ x= 196 = 14
3b 80 = 32.49 × 100 paise = 3249 paise ⇒ x3 = 14x2
⇒ 4= ⇒ b=
20 3 ∴ Number of member in the group 35. (b) Given, 10 + 3 x = 4
22. (d) Since 15 = 3 × 5 = 3 × 6B = 3249 = 57 On squaring both sides, we get
6 61
As, 3 = = A = 61 − A 30. (a) Here, number of soldier arranged 10 + 3 x = 16
2 6
= 8160 − 60 = 8100 ⇒ 3 x = 16 − 10 = 6
∴ 15 = 3 × 6B = 61 − A × 6B
As number of soldier in each row is Now, on cubing both sides, we get
⇒ 6Q = 61 − A × 6B ⇒ 6Q = 61 − A + B equal to number of row. x = ( 6)3 = 216
⇒ Q = 1− A + B So, number of soldier in each row Hence, the value of x is 216.
23. (a) LCM of 4, 6, 12 = 12 = 8100 = 90 36. (d) Expression = [( 2 ) 2
] 2
4
3 = 12 33 = 12 27 1
31. (c) After Ist hit, height of the ball 2
6
10 = 12 102 = 12 100 1 = ( 2) (2 ) 2
= ( 2 )(2 ) 2

= ( 64)
25 = 12 25
12 1  1 − 1
2 1
×2 2 
 2


= ( 2) 2 = ( 2)(2 )
Clearly, 12
100 > 12
27 > 12
25 After IInd hit, height of the ball
Thus, 6 10 > 4 3 > 12 25 2 which denotes a real number but not a
=   ( 64)
1
rational number.
24. (c) Given, (3.7)x = ( 0.037) y = 10000  2
37. (d) Given, 16 × 8n + 2
= 2m
⇒ (3.7)x = 104 and ( 0.037) y = 104 After 16th hit, height of the ball
⇒ ( 2) × 2 3 (n +
4 2)
= 2m
37 = 104 / x + 1 and 37 = 104 / y + 3
16

=  
1
 2
( 64) ⇒ 2(4 + 3 n + 6)
= 2m ⇒ 2(3 n + 10 ) = 2m
⇒ 104 / x + 1 = 104 / y + 3
On comparing, we get
1
On comparing both sides, we get = ( 26 ) = 2−10 m 3n + 10 = m
4 4 216
+ 1= + 3 ⇒ m = 3n + 10
x y
46 CDS Pathfinder

38. (b) After first bounce, height of ball 5 + 10 1


+ 3 x +  = 24 3
x3 +
1
=
1
5 5−4 5−4 2+5 2 x3  x
=   × 36
2
3 (5 + 10 ) ( 5 − 2 ) Q x3 + 3 + x + = 24 3 − 2  x + 
1 1 1
= × 
( 5 + 2) ( 5 − 2) x x x
and after third bounce, height of ball
3 = 24 3 − 2 × 2 3
=   × 36
2 [by rationalisation]
3 5 5 − 5 2 + 50 − 20 = 24 3 − 4 3 = 20 3
=
8 8 × 4 32 2 5 − 2
2 2 46. (a) Given, x = 2 1/3
+ 2 −1 / 3
= × 36 = = m = 10 m
27 3 3 3 ∴ 2x − 6x − 5 = 2 ( 21 / 3 + 2− 1 / 3 )3
3
[Q ( a − b )( a + b ) = a 2 − b 2 ]
Hence, the required height at third − 6 ( 21 / 3 + 2−1 / 3 ) − 5
2 5 5 −5 2 + 5 2 − 2 5 3 5
bounce is 10 m. = = = 5 = 2[ 2 + 2 −1
+ 3 ( 21 / 3 + 2−1 / 3 )]
3 5−2 3
− 6 ( 21 / 3 + 2−1 / 3 ) − 5
39. (c) Product of four consecutive numbers 91
43. (a) We have, x = 1
plus one is always a square. 216 = 4 + 2 × + 6 ( 21 / 3 + 2−1 / 3 )
3
2
Illustration 1 Let four consecutive
⇒ 1− x = 1−
91
=
125  5 
=  − 6 ( 21 / 3 + 2−1 / 3 ) − 5
numbers be 3, 4, 5 and 6. 216 216  6  = 4 + 1− 5 = 0
∴ (3 × 4 × 5 × 6) + 1 = 361 = ( 19)2 1
5 1 6
⇒ (1 − x) 3 = ⇒ = 47. (b) We have, 4x 2 y = 128
Illustration 2 Let four consecutive 6 1
5
numbers be 9, 10, 11 and 12. ( 1 − x) 3 ⇒ 2 2 x 2 y = ( 2)7
∴ ( 9 × 10 × 11 × 12) + 1= 11881= ( 109)2 ⇒ 2 2 x + y = ( 2)7
1 6 9
⇒ 3− =3− = ⇒ 2x + y = 7
40. (b) The difference of two consecutive 1
5 5
cubes is never divisible by 2. ( 1 − x) 3 ⇒ y = 7 − 2x …(i)
Illustration 1 Let the two consecutive 44. (b) We have, 2, 6 1/4
,4 1/3 Now, 3 3x
⋅3 2y
=9 xy

numbers be 4 and 5. ⇒ 33 x+2y = 32 xy


The LCM of 2, 4, 3 are 12.
∴ (5)3 − ( 4)3 = 125 − 64 = 61 1 1 1 ⇒ 3x + 2 y = 2xy …(ii)
Illustration 2 Let the two consecutive ( 26 ) 12 ; ( 63 ) 12 ; ( 44 ) 12 ⇒ 3x + 2 ( 7 − 2x) = 2x ( 7 − 2x)
numbers be 9 and 10. 1 1 1
[from Eq. (i)]
∴ ( 10)3 − ( 9)3 = 1000 − 729 = 271 ( 64) 12 ; ( 216) 12 ; ( 256) 12
⇒ 3x + 14 − 4x = 14x − 4x2
( 0.75)3 1 1
⇒ 4x − 15x + 14 = 0
2
41. (b) + [ 0.75 + ( 0.75)2 + 1] 2<6 4
<4 3
or 2> 4
6> 3
4
1 − 0.75 ⇒ 4x2 − 8x − 7x + 14 = 0
( 0.75)3 45. (b) We have, x = 3 + 2 ⇒ 4x ( x − 2) − 7 ( x − 2) = 0
= + [ 175
. + ( 0.75)2 ]
0.25 1 1 3− 2 ⇒ ( 4x − 7) ( x − 2) = 0
Q = ×
= 3 × ( 0.75)2 + 175
. + ( 0.75)2 3+ 2 3− 2 7
x ⇒ x = or x = 2
3− 2 4
= 4 × ( 0.75)2 + 175
. = 4=2 = = 3− 2 7
3−2 ∴ y = or y = 3
Hence, the required square root is 2. 1 2
∴ x+ =2 3 21
5 + 10 x ∴ x + y= or x + y = 5
42. (d) 4
5 5 − 2 20 − 32 + 50 On cubing both sides, we get
48. (a) Refer to question 32.
06
MATHEMATICS Area and Perimeter of Plane Figures 47

TIME AND DISTANCE


Regularly (4-7) questions have been asked from this chapter. Generally the type of questions vary from
simple to quite complex. But if you know the basics, then this section becomes easy to score area.

Distance
The length of the path travelled by any object or a person between two places is known as distance.
The unit of distance is m or km.

Time
The duration in hours, minutes or seconds spent to cover a certain distance is called time.

Speed
The distance travelled by any object or a person per unit time is known as speed of that object or a
person. The unit of speed is m/s or km/h.
distance
So, speed =
time

EXAMPLE 1. There are 20 poles with a constant distance between each pole. A car takes 24 s to
reach the 12th pole. How much time will it take to reach the last pole?
a. 25.25 s b. 17.45 s c. 35.75 s d. 41.45 s
Sol. d. Suppose distance between each pole is 1m.
∴ Total distance = 19 m
It takes 24 s to cover 11 m.
24
∴ To cover 19 m, it will take × 19 = 4145
. .
11

Conversion of Unit
5
(i) To convert speed of an object from km/h to m/s, multiply it by .
18
5
e.g. 36 km/h = 36 × m/s = 10 m/s
18
48 CDS Pathfinder

(ii) To convert speed of an object from m/s to km/h,


18 Important Rules and Formulae
multiply it by .
5 Rule 1 If A travels with speed x km/h for t 1 h and with
18 x t + yt 2
e.g. 5 m/s = 5 × km/h = 18 km/h speed y km/h for t 2 h, then average speed = 1
5 t1 + t 2
Average Speed EXAMPLE 4. A man walks at the rate of 5 km/h for
The average speed of an object is defined as total 6 h and at 4 km/h for 12 h. Find out the average
distance travelled divided by total time taken. speed (in km/h) of the man.
Total distance 1 1 1
i.e. Average speed = a. 2 b. 5 c. 7 d. None of these
Total time 3 3 3
EXAMPLE 2. A man completes 30 km of a journey Sol. d. Here, x = 5 km/h, y = 4 km/h, t1 = 6 h and t 2 = 12 h
at 6 km/h and the remaining 40 km of the journey in xt1 + yt 2 5 × 6 + 4 × 12 30 + 48
∴ Average speed = = =
5 h. Then, his average speed for the whole journey is t1 + t 2 6 + 12 18
a. 5 km/h b. 7 km/h 78 1
= = 4 km/h
c. 7.5 km/h d. None of these 18 3
Total distance travelled 70
Sol. b. Average speed = = = 7 km/h Rule 2 If a man/vehicle covers two equal distances with
Total time taken 10
the speed of x km/h and y km/h respectively, then the
Relative Speed average speed of the man/vehicle for complete journey
The speed of an object with respect to other is called 2xy
will be .
relative speed. Suppose two bodies are moving with x+ y
speeds of x km/h and y km/h respectively, then their
relative speed will be EXAMPLE 5. A man covers half of his journey at
6 km/h and the remaining half at 3 km/h. Then, his
(i) ( x + y) km/h, if both the bodies are moving in average speed is
opposite directions.
a. 1 km/h b. 2 km/h c. 3 km/h d. 4 km/h
(ii) ( x − y) km/h, if both the bodies are moving in same
Sol. d. Here, x = 6 km/h and y = 3 km/h
direction.
2xy 2 × 6 × 3 36
EXAMPLE 3. Two persons 27 km apart setting out at ∴ Average speed = = = = 4 km/h
x+ y 6+ 3 9
the same time are together in 9 h, if they walk in the
same direction but in 3 h, if they walk in opposite Rule 3 If a man changes his speed to  x  of his usual
directions. Then, their rates of walking (speeds) are  y
a. 2 km/h and 4 km/h b. 3 km/h and 5 km/h
speed and gets late by t min, then the usual time taken by
c. 4 km/h and 8 km/h d. None of these t ×x
Sol. d. Let the first person be walking faster with speed him = .
( y − x)
x km/h and second walking with speed y km/h.
Case I Both walking in same directions.  x
Note If a person changes his speed to   of his usual and
∴ Distance travelled by first person in 9 h = 9x km  y
t ×x
and distance travelled by second person in 9 h = 9 y km reaches early by t min, then usual time taken by him = .
( x − y)
As both are 27 km apart
∴ 9x − 9y = 27 ⇒ x − y = 3 ...(i) EXAMPLE 6. If a man travels with a speed of 2/5
Case II Both walking in opposite directions. times of his original speed and he reached his office
∴ Distance travelled by first person in 3 h = 3x 15 min late to fixed time, then the time taken with his
and distance travelled by second person in 3 h = 3y original speed, is
So, by condition, 3x + 3y = 27 ⇒ x + y = 9 ...(ii) a. 10 min b. 15 min c. 20 min d. 25 min
On adding Eqs. (i) and (ii), we get Sol. a. Here, x = 2 , y = 5 and t = 15 min
⇒ 2x = 12 ⇒ x = 6 km/h x × t 2 × 15
∴ Required time = =
Put the value of x in Eq. (ii), we get y − x 5− 2
6 + y = 9 ⇒ y = 3 km/h 2 × 15
= = 2 × 5 = 10 min
So, their speeds are 6 km/h and 3 km/h. 3
MATHEMATICS Time and Distance 49

(ii) Time taken by a train x m long in passing


Rule 4 When a man travels from A to B with a speed
an object of length y m = Time taken by the
x km/h and reaches t 1 h later after the fixed time and when train to cover ( x + y) m.
he travels with a speed y km/h from A to B, he reaches his
destination t 2 h before the fixed time, then distance EXAMPLE 9. A 100 m long train is moving at a speed
xy (t 1 + t 2 ) of 60 km/h. In what time will it cross a signal pole?
between A and B = km.
y−x a. 6 s b. 12 s c. 15 s d. None of these
5 50
Sol. a. Here, speed of train = 60 km/h = 60 × = m/s
EXAMPLE 7. Walking at 3 km/h, Rajeev reaches his 18 3
school 5 min late, if he walks at 4 km/h, he will be and distance covered by train in passing a pole
5 min early. The distance of Rajeev’s school from his = Length of the train
house is Q Length of train = 100 m
1 1 Distance travelled by train
a. 1 km b. 2 km c. 2 km d. 5 km ∴Time taken to pass the pole = = 6s
2 2 Speed of train
5 5
Sol. b. Here, x = 3 km/h, y = 4 km/h, t1 = h and t 2 = h
60 60 Rule 7 Time taken by a train of length l1 metre moving
3 × 4 + 
5 5
xy (t1 + t 2)  60 60 at a speed of s 1 m/s to cross another train of length l 2
∴ Required distance = = moving at a speed of s 2 m/s in same/opposite direction is
y−x 4−3
l1 + l 2
10
= 3× 4 × = 2 km s1 ± s 2
60
Rule 5 A person travels from A to B with a speed Use ‘+’ for opposite direction.
Use ‘–’ for same direction.
x km/h and reaches t 1 h late. Later he increases his speed
by y km/h to cover the same distance and he still gets late EXAMPLE 10. Two trains of lengths 110 m and
by t 2 h, then the distance between A and B 130 m travel on parallel track. If they move in the
x same direction, the first one which is faster takes one
= (t 1 − t 2 )( x + y) . minute to pass the other one completely. If they move
y
in opposite directions then they pass each other in 3s,
EXAMPLE 8. A boy walking at a speed of 20 km/h then the speed of the trains is
reaches his school 30 min late. Next time he a. 41 m/s and 39 m/s b. 32 m/s and 43 m/s
increased his speed by 5 km/h but still he is late by c. 42 m/s and 38 m/s d. None of these
10 min. What is the distance of the school from his Sol. c. Let v1 be the velocity of faster train and v 2 be the
home? velocity of slower train.
100 50 100 2 Case I If they move in the same direction, then
a. km b. km c. km d. km
3 3 7 5 110 + 130
= 60 s ⇒ v1 − v 2 = 4 …(i)
1 v1 − v 2
Sol. a. Given, x = 20 km/h, y = 5 km/h, t1 = 30 min = h
2 Case II If they move in opposite directions,
1 110 + 130
and t 2 = 10 min = h = 3 ⇒ v1 + v 2 = 80 …(ii)
6 v1 + v 2
x
∴ Required distance = (t1 − t 2) ( x + y) On adding Eqs. (i) and (ii), we get 2v1 = 84 ⇒ v1 = 42 m/s
y
Now, putting the value of v1 in Eq. (ii), we get
=  −  ( 20 + 5) ×
1 1 20 100
= km 10
 2 6 5 3 42 + v 2 = 80, v 2 = 38 m/s = 3 × 4 × = 2 km
60

Problem Based on Trains Rule 8 If two trains start at the same time from points
Problems based on trains are same as the problems A and B towards each other and after crossing each other,
related to speed, Time and Distance. The only difference they take t 1 and t 2 time in reaching points B and A
is that the length of the moving object (train) is taken t
respectively, then ratio of their speed = 2
into consideration in these types of problems. t1
Rule 6 (i) Time taken by a train x m long in passing a S1 t
or = 2
single post or pole or standing man = Time S2 t1
taken by the train to cover x m.
50 CDS Pathfinder

EXAMPLE 11. Two trains, one from Howrah to Patna


Rule 10 Let the speed of boat in downstream = u km/h
and the other from Patna to Howrah, start
simultaneously. After they meet, the trains reach their and speed of boat in upstream = v km/h, then
destinations after 9 h and 16 h, respectively. What is 1
the ratio of their speeds? (i) Speed of boat in still water = (u + v ) km/h
2
a. 4:3 b. 3:1 1
(ii) Speed of stream (current) = (u − v ) km/h
c. 4:5 d. 3:2 2
Sol. a. Let us name the trains as A and B. Then, A’s speed :
B’s speed = t 2 : t1 = 16 : 9 = 4 : 3. EXAMPLE 13. The speed of boat upstream and
speed of boat downstream are 7 km/h and 13 km/h,
Problems Based on Boat and Stream respectively. Then, the speed of stream and speed of
Boats and streams is an application of speed, time and
boat in still water is
distance. Some of the important terms are explained a. 10 km/h and 3 km/h b. 15 km/h and 9 km/h
below c. 20 km/h and 6 km/h d. 40 km/h and 12 km/h
• Still water If the speed of water is zero, then water is Sol. a. Let the speed of boat in downstream and upstream
considered to be still water. be u km/h and v km/h, respectively.
• Stream water If the water of a river is moving at a Given, v = 7 km/h and u = 13 km/h
certain speed, then it is called stream water. 1 1
Speed of boat in still water = (7 + 13) = ( 20) = 10 km/h
2 2
• Downstream motion In water, the direction along the 1 1
stream is called downstream. Speed of the stream = (13 − 7) = (6) = 3 km/h
2 2
• Upstream motion In water, the direction against the
stream is called upstream.
Some rules and problems related to boat and stream are
RACES
given below A contest of speed in running, driving, riding, sailing or
rowing over a specified distance is called a race.
Rule 9 If the speed of a boat in still water be x Suppose A and B are two contestants in a race. If A
km/h and speed of stream be y km/h, then beats B by x m and length of the track is dm.
(i) Speed of boat downstream = ( x + y) km/h Then, Distance travelled by B when A finishes the race
(ii) Speed of boat upstream = ( x − y) km/h Speed of A d
= d − x and =
Speed of B d − x
EXAMPLE 12. A sailor goes 8 km downstream in
40 min and returns back in 1 h. Then, the speed of Note (i) A gives B a start of x m means when A starts at starting
the sailor in still water and the speed of the current is point, B starts x m ahead from starting point.
a. 5 km/h and 3 km/h b. 10 km/h and 2 km/h (ii) A gives B a start of t s means A starts t s after B starts
c. 7 km/h and 10 km/h d. None of these from the same point.

Sol. b. Let the speed of the sailor in still water = x km/h EXAMPLE 14. In a 200 m race, A can beat B by 50m
and speed of the current (stream) = y km/h and B can beat C by 8m. In the same race, A can beat
Then, speed of the sailor downstream = ( x + y) km/h C by what distance?
and speed of the sailor upstream = ( x − y) km/h a. 60 m b. 72 m c. 56 m d. 66 m
40 2 Sol. c. A can beat B by 50 m.
∴ Time to travel 8 km downstream = 40 min = h= h
60 3 speed of A 200 200
∴ = =
8 2 24  distance  speed of B 200 − 50 150
⇒ = ⇒x+ y = Q = time
x+ y 3 2  speed  Also, B can beat C by 8m,
⇒ x + y = 12 …(i) speed of B 200 200
∴ = =
and time to return = 1h ⇒
8
=1 speed of C 200 − 8 192
x−y speed of A speed of B speed of A
Now, = ×
⇒ x− y =8 …(ii) speed of C speed of C speed of B
On adding Eqs. (i) and (ii), we get x = 10 and y = 2 200 200 200
= × =
Hence, speed of sailor in still water = 10 km/h 150 192 144
Speed of current (stream) = 2 km/h So, A beats C by (200 − 144) = 56 m
MATHEMATICS Time and Distance 51

PRACTICE EXERCISE
from B to A at a speed of 60 km/h. The time of
1. A car completes a journey in 6 h with a speed of
their meeting is
50 km/h. At what speed must it travel to
(a) 8 : 30 am (b) 8 : 00 am (c) 9 : 00 am (d) 9 : 15 am
complete the journey in 5 h?
(a) 60 km/h (b) 55 km/h (c) 45 km/h (d) 61 km/h 11. A train T1 leaves a place P at 5:00 am and
2. Kiran covers a certain distance at 80 km/h and reaches another place Q at 9 : 00 am another
returns back to the same point at 20 km/h. Then, train T2 leaves the place Q at 7 : 00 am and
the average speed during the whole journey is reaches the place P at 10 : 30 am. The time at
(a) 35 km/h (b) 32 km/h (c) 30 km/h (d) 28 km/h which the two trains cross each other is
(a) 8 : 26 am (b) 7 : 56 am (c) 8 : 15 am (d) 8 : 00 am
3. Normally Sarita takes 3 h to travel between two
stations with a constant speed. One day her 12. A certain distance is covered at a certain speed.
speed was reduced by 12 km/h and she took If half of the distance is covered in double time,
45 min more to complete the journey. Then, the then the ratio of the two speeds is
distance between the two stations is (a) 4 : 1 (b) 1 : 4 (c) 2 : 1 (d) 1 : 2
(a) 60 km (b) 120 km (c) 180 km (d) 95 km 13. Two trains start running at the same time from
4. Rani goes to school at 10 km/h and reaches the two stations which are 210 km apart and going
school 6 min late. Next day, she covers this in opposite directions cross each other at a
distance at 12 km/h and reaches the school distance of 100 km from one of the station. The
9 min earlier than the scheduled time. What is ratio of their speed is
the distance of her school from her house? (a) 11 : 9 (b) 10 : 11 (c) 11 : 10 (d) 9 : 11
(a) 16 km (b) 12 km (c) 10 km (d) 15 km 14. A police car is ordered to chase a speeding car
5. A man travels first 50 km at 25 km/h next 40 km that is 5 km ahead. The thief car is travelling at
at 20 km/h and then 90 km at 15 km/h. His an average speed of 80 km/h and the police car
average speed (in m/s) for the whole journey is pursues it at an average speed of 100 km/h. How
(a) 18 (b) 5 (c) 10 (d) 36 long does it take for the police car to overtake
the other car?
6. A boy is running at a speed of p km/h to cover a
(a) 17 min (b) 19 min (c) 13 min (d) 15 min
distance of 1 km. But due to the slippery ground,
his speed is reduced by q km/h ( p > q ). If he 15. Points A and B are 70 km apart on a highway. A
takes r h to cover the distance, then car starts from A and another car starts from B
1 pq 1 1
(a) = (b) = p + q (c) r = p − q (d) = p−q at the same time. If they travel in the same
r p+ q r r direction they meet in 7 h, but they travel
towards each other they meet in 1 h. What are
7. A train passes telegraph post in 40 s moving at a
the speeds of the cars?
rate of 36 km/h. Then, the length of the train is
(a) 30 km/h, 40 km/h (b) 36 km/h, 40 km/h
(a) 400 m (b) 500 m (c) 450 m (d) 395 m
(c) 19 km/h, 20 km/h (d) 40 km/h, 50 km/h
8. A person can run around a circular path of
radius 21 m in 44 s. In what time will the same 16. Assume that the distance that a car runs on 1 L
person run a distance of 3 km? of petrol varies inversely as the square of the
speed at which it is driven. It gives a run of
(a) 18 min 40 s (b) 16 min 30 s
25 km/L at a speed of 30 km/h. At what speed
(c) 18 min 30 s (d) 16 min 40 s
should it be driven to get a run of 36 km/L?
9. A car is ahead of a scooter by 30 km. Car goes at (a) 12.5 km/h (b) 25 km/h (c) 30 km/h (d) 40 km/h
the rate of 50 km/h and the scooter goes at the
rate of 60 km/h. The scooter overtakes the car 17. A man standing on a railway platform observes
after that a train going in one direction takes 4 s to
1 pass him. Another train of same length going in
(a) 3 h (b) 3.5 h (c) 4 h (d) 3 h
4 the opposite direction takes 5 s to pass him. The
time taken (in seconds) by the two trains to cross
10. Two towns A and B are 250 km apart. A bus each other will be
starts from A to B at 6 : 00 am at a speed of 32 33 40 49
40 km/h. At the same time another bus starts (a) (b) (c) (d)
9 7 9 9
52 CDS Pathfinder

18. A bullock cart has to cover a distance of 80 km 28. A person x started 5 min earlier at 60 km/h from
in 10 h. If it covers half of the journey in 3/5th a place P, then another person y followed him at
time, what should be its speed to cover the 48 km/h, started his journey at 3:05 pm. Which
remaining distance in the time left? of the following is/are correct.
(a) 5 km/h (b) 10 km/h (c) 15 km/h (d) 18 km/h I. At 3 : 15 pm, x and y are 7 km apart.
3 II. At 3 : 25 pm, y will overtake x
19. By walking at of his usual speed, a man reaches
4 (a) Only I (b) Only II
his office 25 min later than usual. His usual time is (c) Both I and II (d) Neither I nor II
(a) 60 min (b) 70 min (c) 75 min (d) 80 min
29. A boy walking at a speed of 20 km/h reaches his
20. A train covers a distance in 50 min, if it runs at school 30 min late. Next time he increases his
a speed of 48 km/h on an average. The speed at speed by 4 km/h but still he is late by 10 min.
which the train must run to reduce the time of
Which of the following statement is/are correct?
journey to 40 min, will be
I. The distance of the school from his home is
(a) 10 km/h (b) 20 km/h (c) 40 km/h (d) 60 km/h
50 km/h.
21. Two trains travel in the same direction at II. If he increases his speed by 10 km/h but still
50 km/h and 32 km/h, respectively. A man in the late by 10 min, then distance of the school will
slower train observes that 15 s elapse before the be 20 km.
faster train completely passes him. What is the (a) Both I and II are correct (b) Only I is correct
length of the faster train? (c) Only II is correct (d) Neither I nor II are correct.
625
(a) 75 m (b) 125 m (c) 150 m (d) m
3
Directions (Q. Nos.30-31) Ramesh and Prateek start
22. A father and his son start at a point A with running at same time in opposite directions from
speeds of 12 km/h and 18 km/h, respectively and
two points and after passing each other they
reach another point B. If his son starts 60 min
complete their journeys in ‘x’ and ‘y’ h, respectively.
after his father at A and reaches B, 60 min
before his father, what is the distance between A Then, Speed of Ramesh : Speed of Prateek = y : x
and B ?
(a) 90 km (b) 72 km (c) 36 km (d) None of these 30. Find the ratio of the speeds of Ramesh and
Prateek, if Ramesh and Prateek completed their
23. Two trains of lengths 100 m and 150 m are journeys in 16 h and 25 h after passing each other.
travelling in opposite directions at speeds of
(a) 5 : 4 (b) 5 : 3 (c) 4 : 5 (d) 3 : 5
75 km /h and 50 km/h, respectively. What is the
time taken by them to cross each other? 31. What if two persons completed their journey
(a) 7.4 s (b) 7.2 s (c) 7 s (d) 6.8 s with speeds in the ratio 7 : 9, then how much
24. A man can walk uphill at the rate of 2.5 km/h time they should have taken to complete their
and downhill at the rate of 3.25 km/h. If the journey after they meet each other.
total time required to walk a certain distance up (a) 81 : 49 (b) 7: 9 (c) 49 : 81 (d) 14 : 18
the hill and return to the starting position is 4 h
36 min, what is the distance he walked up the hill? PREVIOUS YEARS’ QUESTIONS
(a) 3.5 km (b) 4.5 km (c) 5.5 km (d) 6.5 km
25. A boat goes 30 km upstream and 44 km 32. A train 280 m long is moving at a speed of
downstream in 10 h. In 13 h it can go 40 km 60 km/h. What is the time taken by the train to
upstream and 55 km downstream. The speed of cross a platform 220 m long? e 2012 I
the boat in still water is (a) 45 s (b) 40 s (c) 35 s (d) 30 s
(a) 9 km/h (b) 8 km/h (c) 4 km/h (d) 3 km/h 33. A car travels along the four sides of a square at
26. A motorboat takes 2 h to travel a distance of speeds v , 2 v , 3 v and 4 v , respectively. If u is the
9 km down the current and it takes 6 h to travel average speed of the car in its travel around the
the same distance against the current. What is square, then which one of the following is
the speed of the boat in still water (in km/h)? correct? e 2012 I
(a) 3 (b) 2 (c) 1.5 (d) 10 (a) u = 2 .25 v (b) u = 3 v
(c) v < u < 2 v (d) 3 v < u < 4 v
27. Ram travels from P to Q at 10 km/h and return at
15 km/h. Shyam travels from P to Q and return 34. A car is travelling at a constant rate of 45 km/h.
at 12.5 km/h. If he takes 12 min less than Ram, The distance travelled by car from 10 : 40 am to
then what is the distance between P and Q? 1 : 00 pm is e 2012 II
(a) 60 km (b) 45 km (c) 36 km (d) 30 km (a) 165 km (b) 150 km (c) 120 km (d) 105 km
MATHEMATICS Time and Distance 53

35. A person travels a certain distance at 3 km/h 45. A man rides one-third of the distance from A to
and reaches 15 min late. If he travels at 4 km/h, B at the rate of x km/h and the remainder at the
he reaches 15 min earlier. The distance he has to rate of 2y km/h. If he had travelled at a uniform
travel is e 2013 I rate of 6z km/h, then he could have ridden from
(a) 4.5 km (b) 6 km (c) 7.2 km (d) 12 km A to B and back again in the same time. Which
36. If a body covers a distance at the rate of x km/h one of the following is correct? e 2014 II

and another equal distance at the rate of y km/h, (a) z = x + y (b) 3z = x + y


1 1 1 1 1 1
then the average speed (in km/h) is e 2013 I (c) = + (d) = +
x+ y 2x y x+ y z x y 2z x y
(a) (b) x y (c) (d)
2 x+ y xy 46. In a flight of 600 km, an aircraft was slowed
37. A sailor sails a distance of 48 km along the flow down due to bad weather. Its average speed for
of a river in 8 h. If it takes 12 h to return the the trip was reduced by 200 km/h and the time
same distance, then the speed of the flow of the of flight increased by 30 min. The duration of the
river is e 2013 I flight is e 2015 I
(a) 0.5 km/h (b) 1 km/h (c) 1.5 km/h (d) 2 km/h (a) 1 h (b) 2 h (c) 3 h (d) 4 h
38. A train running at the speed of 72 km/h goes 47. With a uniform speed, a car covers a distance in
past a pole in 15 s. What is the length of the 8 h. Had the speed been increased by 4 km/h,
train? e 2013 II the same distance could have been covered in 7 h
(a) 150 m (b) 200 m (c) 300 m (d) 350 m and 30 min. What is the distance covered?
e 2015 I
39. Two cars A and B start simultaneously from a (a) 420 km (b) 480 km (c) 520 km (d) 640 km
certain place at the speed of 30 km/h and
45 km/h, respectively. The car B reaches the 48. A car travels the first one-third of a certain
destination 2 h earlier than A. What is the distance with a speed of 10 km/h, the next
distance between the starting point and one-third distance with a speed of 20 km/h and
destination? e 2013 II the last one-third distance with a speed of
(a) 90 km (b) 180 km (c) 270 km (d) 360 km 60 km/h. The average speed of the car for the
whole journey is e 2015 I
40. A man cycles with a speed of 10 km/h and
reaches his office at 1 : 00 pm. However, when (a) 18 km/h (b) 24 km/h (c) 30 km/h (d) 36 km/h
he cycles with a speed of 15 km/h, he reaches his 49. A man rows 32 km downstream and 14 km
office at 11 : 00 am. At what speed should he upstream, and he takes 6 h to cover each distance.
cycle, so that he reaches his office at 12 noon? What is the speed of the current? e 2015 I
e 2013 II (a) 0.5 km/h (b) 1 km/h (c) 1.5 km/h (d) 2 km/h
(a) 12.5 km/h (b) 12 km/h (c) 13 km/h (d) 13.5 km/h 2
50. A runs 1 times as fast as B. If A gives B a start
41. A train takes 9 s to cross a pole. If the speed of the 3
train is 48 km/h, the length of the train is e 2014 I of 80 m, how far must the winning post from the
(a) 150 m (b) 120 m (c) 90 m (d) 80 m starting point be so that A and B might reach it
at the same time? e 2015 I
42. A train takes 10 s to cross a pole and 20 s to (a) 200 m (b) 300 m (c) 270 m (d) 160 m
cross a platform of length 200 m. What is the
length of the train? e 2014 II 51. A thief is noticed by a policeman from a distance
(a) 50 m (b) 100 m (c) 150 m (d) 200 m of 200 m. The thief starts running and the
policeman chases him. The thief and the
43. A train travels at a speed of 40 km/h and policeman run at the speed of 10 km/h and
another train at a speed of 20 m/s. What is the 11 km/h, respectively. What is the distance
ratio of speed of the first train to that of the between them after 6 min? e 2015 I
second train? e 2014 II (a) 100 m (b) 120 m (c) 150 m (d) 160 m
(a) 2 : 1 (b) 5 : 9 (c) 5 : 3 (d) 9 : 5
52. Two persons A and B start simultaneously from
44. The distance between two points (A and B ) is two places c km apart and walk in the same
110 km. X starts running from point A at a direction. If A travels at the rate of p km/h and B
speed of 60 km/h and Y starts running from travels at the rate of q km/h, then A has
point B at a speed of 40 km/h at the same time. travelled before he overtakes B a distance of
They meet at a point C, somewhere on the line e 2015 I
AB. What is the ratio of AC to BC ? e 2014 II
(a)
qc
km (b)
pc
km (c)
qc
km (d)
pc
km
(a) 3 : 2 (b) 2 : 3 (c) 3 : 4 (d) 4 : 3 p+ q p−q p−q p+ q
54 CDS Pathfinder

53. By increasing the speed of his car by 15 km/h, a 60. A bike consumes 20 mL of petrol per kilometre,
person covers 300 km distance by taking an hour if it is driven at a speed in the range of
less than before. The original speed of the car 25-50 km/h and consumes 40 mL of petrol per
was e 2015 II kilometre at any other speed. How much
(a) 45 km/h (b) 50 km/h (c) 60 km/h (d) 75 km/h petrol is consumed by the bike in travelling a
distance of 50 km, if the bike is driven at a
54. Two trains, one is of 121 m in length at the speed of 40 km/h for the first 10 km, at a speed
speed of 40 km/h and the other is of 99 m in of 60 km/h for the next 30 km and at a speed of
length at the speed of 32 km/h are running in 30 km/h for the last 10 km? e 2015 II
opposite directions. In how much time will they (a) 1 L (b) 1.2 L (c) 1.4 L (d) 1.6 L
be completely clear from each other from the
moment they meet ? e 2015 II 61. A passenger train takes 1 h less for a journey of
(a) 10 s (b) 11 s (c) 16 s (d) 21 s 120 km, if its speed is increased by 10 km/h
from its usual speed. What is its usual speed?
55. The speeds of three buses are in the ratio e 2016 I
2 : 3 : 4. The time taken by these buses to travel (a) 50 km/h (b) 40 km/h (c) 35 km/h (d) 30 km/h
the same distance will be in the ratio. e 2015 II
(a) 2 : 3 : 4 (b) 4 : 3 : 2 62. A man walking at 5 km/h noticed that a 225 m
(c) 4 : 3 : 6 (d) 6 : 4 : 3 long train coming in the opposite direction crossed
him in 9 s. The speed of the train is e 2016 I
56. Two trains are moving in the same direction (a) 75 km/h (b) 80 km/h (c) 85 km/h (d) 90 km/h
at 1.5 km/min and 60 km/h, respectively. A man
in the faster train observes that it takes 27 s to
63. A cyclist moves non-stop from A to B, a distance
of 14 km, at a certain average speed. If his
cross the slower train. The length of the slower
average speed reduces by 1 km/h, then he takes
train is e 2015 II
20 min more to cover the same distance. The
(a) 225 m (b) 230 m (c) 240 m (d) 250 m
original average speed of the cyclist is e 2016 I
57. In a race of 100 m, A beats B by 4 m and A beats (a) 5 km/h (b) 6 km/h
C by 2 m. By how many metres (approximately) (c) 7 km/h (d) None of these
would C beat B in another 100 m race assuming 64. In a race of 1000 m, A beats B by 100 m or 10 s.
C and B run with their respective speeds as in If they start a race of 1000 m simultaneously
the earlier race? e 2015 II
from the same point and if B gets injured after
(a) 2 (b) 2.04 (c) 2.08 (d) 3.2 running 50 m less than half the race length and
58. Three athletes run a 4 km race. Their speeds are in due to which his speed gets halved, then by how
the ratio 16 : 15 : 11. When the winner wins the much time will A beat B ? e 2016 I
race, then the distance between the athlete in the (a) 65 s (b) 60 s (c) 50 s (d) 45 s
second position to the athlete in the third 65. In a race A, B and C take part. A beats B by
position is e 2015 II 30 m, B beats C by 20 m and A beats C by 48 m.
(a) 1000 m (b) 800 m (c) 750 m (d) 600 m
Which of the following is/are correct?
59. A motorboat, whose speed is 15 km/h in still I. The length of the race is 300 m.
water goes 30 km downstream and comes back in II. The speeds of A, B and C are in the ratio
a total of 4h and 30 min. The speed of the 50 : 45 : 42.
stream is e 2015 II
Select the correct answer using codes given below.
(a) 4 km/h (b) 5 km/h
(a) Only I (b) Only II e 2016 I
(c) 6 km/h (d) 10 km/h
(c) Both I and II (d) Neither I nor II

ANSWERS
1 a 2 b 3 c 4 d 5 b 6 d 7 a 8 d 9 a 10 a
11 b 12 a 13 c 14 d 15 a 16 b 17 c 18 b 19 c 20 d
21 a 22 b 23 b 24 d 25 b 26 a 27 d 28 a 29 c 30 a
31 a 32 d 33 c 34 d 35 b 36 c 37 b 38 c 39 b 40 b
41 b 42 d 43 b 44 a 45 c 46 a 47 b 48 a 49 c 50 a
51 a 52 b 53 c 54 b 55 d 56 a 57 b 58 a 59 b 60 d
MATHEMATICS Time and Distance 55

HINTS AND SOLUTIONS 


1. (a) Let speed be v km/h. 7. (a) Length of train = Distance covered 12. (a) Let x km distance be covered in y h.
We know that, in 40 s at the rate of 36 km/h. Then, speed of object in first case
Distance = Speed × Time [Rule 6 (i)] x
= km/h
6 × 50 = 5 × v ∴ Length of train y
6 × 50 5 As, half of this distance is covered in
⇒ v= = 60 km/h = 40 × 36 × = 400 m
18 double time.
5
2xy 8. (d) Distance travelled in 44 s = 2 πr Then, speed of object in second case
2. (b) Average speed = [Rule 2] 22 x x 1 x
x + y =2× × 21 = 132 m = ÷ 2y = × = km/h
7 2 2 2y 4y
2 × 80 × 20
= = 32 km/h ∴ Speed =
132
= 3 m/s ∴ Ratio of first and second speeds
80 + 20 44 x x 1
= : = 1: = 4 : 1
Q speed = distance  y 4y 4
3. (c) Let Sarita’s speed be u and distance is
constant.  time 
15 3000 13. (c) Let the speed of two trains be x km/h
Then, u × 3 = (u − 12) Time taken to travel 3 km = and y km/h, respectively. Then, the time
4 3
Q 3 h + 45 min = 15 h  1000 taken by first train to cover
= 1000 s = min = 16 min 40 s
 4  60 110 km = Time taken by second train to
45 × 4 cover 100 km .
⇒ u= = 60 km/h 9. (a) Distance between car and scooter 110 100 x 110
3 = 30 km Thus, = ⇒ =
x y y 100
∴ Distance = Speed × Time Relative speed
∴ x : y = 11 : 10
= 60 × 3 = 180 km y = 60 − 50 = 10 km/h
So, the time taken by scooter to overtake 14. (d) Distance travelled by thief car in one
6
4. (d) Here, x = 10, y = 12, t = h and 30 hour = 80 km
1
60 the car = =3h
10 Distance travelled in one hour by police
9
t = h car = 100 km
2 10. (a) Let the buses meet x h after 6:00 am.
60
So, police travels extra 20 km in 1 h.
xy( t + t ) Then, the distance covered by the two
∴ Required distance =
1 2 So, to overtake thief, police car has to
y−x buses is 250 km. travel 5 km extra.
∴ 40x + 60x = 250 ∴ Time =
5 1
= h=
60
min = 15 min
[Rule 4] 250 20 4 4
⇒ x= = 2.5 h = 2h and 30 min
10 × 12 × 15 100
= 15. (a) Let the speed of car A = x km/h
60 × 2 So, they will meet at 8 : 30 am.
Let the speed of car B = y km/h
11. (b) Let the distance between P and Q be
= 15 km Total distance covered by b other the
x km and let the two trains meet y h
cars in 1h = 70 km
5. (b) Total distance covered after 7 am.
Then,T covers x km in 4h and T covers x ( 1) + y( 1) = 70
= 50 + 40 + 90 = 180 km 1 2
x + y = 70 …(i)
1
Total time taken x km in 3 h.
2 [in opposite direction]
= 
50 40 90  x
+ +  = 10 h ∴ Speed of train T = km/h Distance covered by both the cars in
 25 20 15  1
4 7 h = 70 km
∴ Average speed for the whole journey 2x
and speed of train T = km/h But in same direction
2
7 7x − 7 y = 70 ⇒ x − y = 10 ...(ii)
Total distance travelled
=
Total time taken According to the question, added Eq. (i) and (ii), we get
x ( y + 2) 2xy 2x = 80, x = 40 km/h, y = 30 km/h
=
180
= 18 km/h + =x
10 4 7
16. (b) Let the speed of car be v and distance
18 × 5 ( y + 2) 2 y
∴ 18 km/h = m/s = 5 m/s ⇒ + =1 covered by car in one litre be A.
18 4 7 1 K
7( y + 2) + 8 y ∴ A∝ 2 ⇒ A= 2
6. (d) Actual speed of boy = ( p − q ) km/h ⇒ =1 v v
1 28 Then, A = 25 km/h and v = 30 km/h
Time taken to cover 1 km = ⇒ 7 y + 14 + 8 y = 28 K
p−q 14 14 So, 25 = ⇒ K = 900 × 25
∴ y= h= × 60 min = 56 min (30)2
1 1
∴ =r ⇒ = p−q 15 15 22500
p−q r ⇒ K = 22500 ⇒ A =
So, trains will meet at 7 : 56 am. v2
56 CDS Pathfinder

When A = 36 km/L 22. (b) Let distance between A and On solving the above equations, we get
22500 22500 150 B be x km. x = 8, y = 3
v2 = ⇒v = =
36 36 6 By given condition, Thus, the speed of boat in still water
x x = 8 km/h
Hence, the speed of a car is 25 km/h. − = 2 ⇒ 6x = 2 × 18 × 12
12 18
17. (c) Let the length of each train be I m. 26. (a) Let the speed of motorboat be
2 × 18 × 12
⇒ Speed of first train =   m/s ∴ x= = 72 km
I x km/h and the speed of water be y km/h.
 4 6
Now, speed of boat downstream
and speed of second train =   m/s
I Hence, the required distance is 72 km.
5 = ( x + y ) km/h [Rule 9]
23. (b) Relative speed when trains are in
As, both trains are moving in opposite and speed of boat upstream
opposite direction
direction. = ( x − y ) km/h
=V + V
1 2
Time taken to cross each other By given condition,
= 75 + 50 = 125 km/h
I+I 9
= [Rule 7] 125 × 5 = 2 ⇒ 2x + 2 y = 9 …(i)
I I = m/s x+ y
+ 18
4 5 and total distance covered 9
  and = 6 ⇒ 6x − 6 y = 9
 2I   20 × 2  40 = ( 100 + 150) = 250 m x− y
=  s=  = s ⇒ 2x − 2 y = 3
9
  I  9  9 ∴ Time taken to cross each other …(ii)
 
 20  Total covered distance On solving Eqs. (i) and (ii), we get
=
18. (b) Given, total distance to cover in 10 h Relative speed x = 3 km/h and y = 1.5 km/h

= 80 km 250 × 18 ∴ Speed of boat = 3 km/h


= = 7.2 s
3 125 × 5 27. (d) Let the distance between P and
If it covers 40 km in th of time i.e.
5 24. (d) Let he walked up the hill at a Q = d km. Total time taken by Ram
40 km in 6 h. d d 25d
distance of x km. = + =
∴ Remaining time = 10 − 6 = 4 h 10 15 150
By given condition, 2d 4d
Remaining distance = 40 km x x 36 Total time taken by Shyam = =
+ = 4 12.5 25
25. 3.25 60 According to question,
∴ Required speed = ( 40 ÷ 4) km/h
⇒ x 
1 1  276 25d 4d 12
=
40
= 10 km/h +  = − =
 25
. 3.25  60 150 25 60
4
25 − 24  1
19. (c) Here, x = 3 m/s, y = 4 m/s ⇒ x  +
2 4
 =
276 ⇒ d  =
 5 13  60  150  5
and t = 25 min
276 65 d 1
x ×t ∴ x = × = 6.5 km ⇒ = ⇒ d = 30 km
∴Required time = [Rule 3] 60 46 150 5
y−x
3 × 25 25. (b) Let speed of boat in still water 28. (a) Distance travelled by x in 15 min
= = 75 min 15
4 −3 = x km/h = 60 × = 15 km
60
20. (d) Distance travelled in 1h = 48 km and speed of boat in current water
Distance travelled by y in 10 min
∴ Distance travelled in 50 min = y km/h 10
= 48 × = 8 km
48 ∴ Downstream speed = ( x + y ) km/h 60
= × 50 = 40 km
60 Upstream speed = ( x − y ) km/h Difference = ( 15 − 8) km = 7 km
40
Time to be reduced = h [Rule 9] Hence, at 3:15 pm they are 7 km apart.
60
∴ Required speed According to the question, So, statement I is true. As speed of x is
40 × 60 30 44 greater than y. So, y will never overtake x.
=
40
= = 60 km/h + = 10 …(i)
40 / 60 40 x− y x+ y Thus, statement II is false.
40 55 29. (c) I. Here, x = 20 km/h, y = 4 km/h,
21. (a) Let the length of faster train be x km. and + = 13 …(ii)
x− y x+ y t = 30 min, t = 10 min
Q Trains travel in the same direction. 1 1 1 2
Let = u and =v …(iii)
∴ Relative speed = (50 − 32) = 18 km/h x− y x+ y
According to formula,
15 ∴ Required distance
Elapsed time = 15 s = h Then, 30u + 44v = 10 x
3600 = (t − t ) (x + y) [Rule 5]
distance 40u + 55v = 13 1 2
y
Now, time =
(30 − 10)
( 20 + 4)  
speed On solving the above equations, we get 20
=
15 x 1
u = ,v=
1 60  4
⇒ = ⇒ x × 3600 = 18 × 15 5 11 20 20
3600 18 = × 24 × = 5 × 8 = 40 km
18 × 15 From Eq. (iii), we get 60 4
∴x= = 0.075km = 75 m So, I is incorrect.
3600 x − y = 5 and x + y = 11
MATHEMATICS Time and Distance 57

II. Here, x = 20 km/h, y = 10 km/h 140 40. (b) Let t h be the time taken by the man
= 140 min = h
t = 30 min, t = 10 min 60 to reach his office at speed of 15 km/h.
1 2
According to formula, So, required distance travelled by car Then, time taken to reach office at the
= speed × time speed of 10 km/h = ( t + 2) h
∴ Required distance 140
= 45 × = 105 km s ×t = s ×t
 30 − 10  60 Now,
 ( 20 + 10)  20 
1 1 2 2
=
 60   10  35. (b) Given, x = 3 km/h, y = 4 km/h, ⇒ 10 × ( t + 2) = 15 × t
 
[Rule 5] t = 15 min =
15
h ⇒ 10 t + 20 = 15 t ⇒ 5 t = 20
1
20 20 60 ∴ t =4h
= × 30 × = 20 km 15
60 10 and t = 15 min = h Now, distance covered to reach office
2
So, II is correct.
60 =s × t
xy ( t + t ) 1 1
∴ Required distance = 1 2
= 10 × ( 4 + 2) = 10 × 6 = 60 km
30. (a) x = 16, y = 25 y−x
According to formula, Now, speed required to reach office at 12
[Rule 4]
Ramesh’ s Speed y 25 5 noon in 5 h.
∴ = = = 
3×4  +
15 15 
 ∴ Speed =
60
= 12 km/h
Prateek’ s Speed x 16 4  60 60 
= 5
or 5:4 4 −3
30 1 41. (b) Speed of train = 48 km/h
31. (a) Here, we know the relation = 12 × = 12 × = 6 km
=  48 ×
5
Speed of Ramesh y 7 y 60 2  m/s
= ⇒ =  18 
Speed of Prateek x 9 x 36. (c) If a body covers a distance at the rate
of x km/h and another equal distance at Let the length of train be x m
49 y
On squaring both sides, = = 81 : 49 the rate of y km/h, then =
2xy 5
81 x x = 48 × ×9
x+ y 18
32. (d) To cover a distance by train [Rule 2] x = 120 m
= 280 + 220 = 500 m
37. (b) Let speed of the flow of water Length of the train is 120 m.
Speed of train = 60 km/h be v km/h and rate of sailing of sailer be
5 42. (d) Let the speed of a train be x m/s and
= 60 × m/s u km/h.
18 length be y m.
50 Speed of sailor downstream
= m/s Condition I When t = 10 s
3 = (u + v ) km/h [Rule 9] distance
time =
∴ Time taken by train Speed of sailor upstream = (u − v ) km/h speed
total distance 48 y
= Condition I u + v = ⇒ 10 = ∴ y = 10x ...(i)
speed 8 x
500 ⇒ u+ v=6 ...(i)
= = 30 s Condition II When t = 20 s
50 / 3 48
Condition II u − v = and total distance = y + 200 m
12
Total distance
33. (c) Let side of a square be x. ⇒ u−v=4 ...(ii) ∴ Time =
x Speed
On subtracting Eqs. (ii) from (i), we get
y + 200
D 3v C
v = 1 km/h 20 = ⇒ 20x = y + 200
x
x 4v 2v x 38. (c) Speed of train = 72 km/h y
20 × = y + 200
5
= 72 × 10
v 18
A B ⇒ 2 y − y = 200 ⇒ y = 200
x
= 20 m/s Hence, the length of train is 200 m.
total distance
Q Average speed ( u ) = Length of train = distance covered
total time 43. (b) Given, speed of a train = 40 km/h
(x + x + x + x) x = 20 × 15
= x x x x = 40 ×
5
m/s
+ + + x = 300 m 18
v 2v 3v 4v
4x Length of the train is 300 m. Speed of another train = 20 m/s
=

x +
1 1
+
1
+
1 39. (b) Let the distance between two points ∴ Required ratio

 v 2v 3v 4v  be x km Speed of first train
=
4×v 48v Time taken by A − Time taken by B = 2 Speed of second train
= = = 1.92 v x x
1 1 1 25 ⇒ − =2
1+ + + 40 ×
5
2 3 4 30 45 40 × 5 2 ×5
= 18 = =
3 − 2
which lies in the interval v < u < 2v. ⇒ x =2 20 20 × 18 18
 90 
34. (d) Given, speed of a car = 45 km/h 10 5
= = or 5 : 9
Time taken by the car ⇒ x = 90 × 2 = 180 km 18 9
58 CDS Pathfinder

44. (a) Given, distance between two points 1 1 1 = ( x − y ) km / h


− =
(A and B) = 110 km x − 200 x 600 × 2 According to the question,
Their relative speed x − ( x − 200) 32 Q speed = distance 
1 =6
= 60 + 40 = 100 km/h ⇒ = x+ y  time 
x ( x − 200) 1200
Time after which they meet 14
and =6
=
Total distance ⇒ x2 − 200x − 240000 = 0 x− y
Relative speed ⇒ ( x − 600)( x + 400) = 0 ⇒ 6x + 6 y = 32 … (i)
110 ⇒ x = 600 km / h and 6x − 6 y = 14 … (ii)
=
100 or x ≠ −400 km/h On subtracting Eq. (ii) from Eq. (i), we
C 600 get
∴ Required time = = 1 h.
A
110 km
B
600 12 y = 18
= 1. 10 h 47. (b) Let the distance between A and B be y=
18
= 1.5 km/h
Distance covered by A in 1.1 h x km and speed be V km/h. 12
= AC = 60 × 1.1 = 66 km 50. (a) Let the speed of B be x m/s.
x km
Remaining distance 2 5x
A B ∴ Speed of A = 1 x = m/s
= BC = 110 − 66 = 44 km 3 3
∴ Required ratio = AC : BC Case I Given, distance = x km, speed
Ratio of speed of rates of A and B
= 66 : 44 = 3 : 2 = V km/h and time = 8 h. 5x
Distance x = : x = 5 :3
45. (c) Let the total distance be d. ∴ Speed = ⇒ V = … (i) 3
1 Time 8
Time taken to cover rd distance, Q 2 m are gained in a race of 5 m.
3 Case II If speed = (V + 4) km/h 5
1 1 15 ∴ 1 m are gained in a race of m.
d and time = 7 h = h 2
d  distance 
t =3 = Q time = 
2 2
1
x 3x  speed  x 2x So, 80 m are gained in a race of
then, V +4= ⇒ V +4=
5
1
Remaining distance = d − d = d
2 15 / 2 15 × 80 m = 200 m
3 3 x 2x 2
2 ⇒ +4= [from Eq .(i)]
Time taken to cover rd distance, 8 15 51. (a) Given, speed of thief = 10 km/h
2 3 2x x x
d ⇒ − =4 ⇒ =4 10 × 1000 500
2 d d = m/min = m/min
t = 3 = = 15 8 120 60 3
2
2y 6y 3y ⇒ x = 480 km and speed of policeman = 11 km/h
Time taken to cover distance from A to 48. (a) x/3 km x/3 km x/3 km
11 × 1000 550
B and B to A, t1 t2 t3 = m/min = m/min
A C D B 60 3
2d d x km
∴ t= =
6z 3z Let total distance of AB be x km. Now, distance travelled by thief in 6 min
According to the question, x/3 x 500
For distance AC, t = = h = × 6 = 1000 m
d d d 1 3
t + t =t ⇒ + = 10 30
1 2 x/3 x
3x 3 y 3z and distance travelled by policeman in
For distance CD, t = = h.
2
20 60 550

1
+
1
=
1

1 1 1
+ = 6 min = × 6 = 1100 m
3 x 3 y 3z x y z x/3 x 3
For distance BD, t = = h.
3
60 180 ∴ Difference = ( 1100 − 1000) = 100 m
46. (a) Let the original speed of an aircraft
be x km/h and its reduced speed ∴ Total time taken Hence, the distance between them after
= ( x − 200) km/h. x x x 6 min is 100 m.
=t +t +t = + +
1 2 3
Condition I 30 60 180 52. (b) Let A and B will meet after t h at
10x x
Time taken by aircraft to cover 600 km = = point E.
600 180 18
= h. Total distance A B E
x ∴ Average speed =
Total time taken Distance travelled by A = pt h
Condition II Time taken by aircraft to x
cover = x = 18 km / h and distance travelled by B = qt h
600
600 km = h 18 According to the question,
( x − 200)
49. (c) Let speed of a boat in still water be pt = qt + c ⇒ pt − qt = c
According to the question, c
x km/h and speed of the stream be y km/h. ⇒ t( p − q ) = c ⇒ t = …(i)
600 1 
Q 30 min = h  p−q
600 1
− = Then, speed of the boat downstream
x − 200 x 2  2  Q Distance travelled by A = pt
= ( x + y ) km / h [Rule 9] pc
On dividing both sides by 600, we get = [from Eq. (i)]
and speed of the boat upstream p−q
MATHEMATICS Time and Distance 59

53. (c) Let the original speed of car be 57. (b) When A covers 100 m, then B covers ⇒ x = 85 km/h
x km/h. 96 m and C covers 98 m. Hence, the speed of the train is 85 km/h.
300
Time taken to cover 300 km = h i.e. when C covers 98 m, then B covers
x 63. (c) Refer to question 46.
96 m.
If the speed of car is increased by 64. (a) Since, either A beats B by 100 m or
∴ when C covers 100 m, then B covers
15 km/h. 10 s. It means that B runs 100 m in 10 s.
100 × 96
300 300 100
Then, = −1 m, i.e. 97.96 ∴ Speed of B = = 10 m / s
x + 15 x 98 10
300 300
⇒ 1= − So,C beats B by approximately 2.04 m. B A
x x + 15
58. (a) Ratio of speeds are 16 : 15 : 11. 900 m 100 m
  Let speed of winner athletic be 16x km/h. 1000 m
x + 15 − x 
= 300  Similarly, speed of athletic in second
 x( x + 15)  Q B gets injured at a distance of 450 m
  and third position be 15x and 11x, and his speed gets halved.
4500 respectively.
⇒ 1= 2 So, time taken by B to cover 1000 m
x + 15x Total distance travelled by winner is 4 km. 450 550
⇒ x2 + 15x − 4500 = 0 4 = + = 155
∴ Time = h 10 5
⇒ ( x + 75)( x − 60) = 0 16x Q Ratio of speed of A and B is equal to
∴ x = 60 km/h Distance travelled by second athletic in ratio of distance covered by A and B.
4 4 15
54. (b) Total length of trains h= × 15x = km A : B = 1000 : 900 = 10 : 9
16x 16x 4
= 121 + 99 = 220 m 10 100
Similarly, distance travelled by third Now, speed of A = × 10 = m/s
9 9
Relative speed of trains = ( 40 + 32) athletic in
km/h 4 4 11 Time taken by A to cover
h= × 11x = km 1000
= 72 km/h 16x 16x 4 1000 m = × 9 = 90 s
5 ∴ Difference between their distances 100
= 72 × m/s = 20 m/s
= 
18 15 11 Hence, A beat B by length of time
−  km
distance 220  4 4 = ( 155 − 90) = 65 s
∴ Time = = = 11 s
speed 20 = 1 km = 1000 m 65. (c) I. Let the length of race be x m.
55. (d) Let the distance be x. 59. (b) Refer to example 12. Then, distance covered by A = x m.
Now, ratio of time taken to travel the 60. (d) Petrol consumed by the bike Distance covered by B when A
distance by each bus is = ( 10 × 20) + (30 × 40) reaches the destination = x − 30
x x x 12 12 12
: : ⇒ : : + ( 10 × 20) mL Distance covered by C when A
2 3 4 2 3 4 reaches the destination = x − 48
= ( 200 + 1200 + 200) mL x − 30
= 6: 4:3 Ratio of speed of B to A =
= 1600 mL = 1. 6 L x
56. (a) Speed of faster train is 1.5 km/min.
61. (d) Refer to question 53. x − 48
i.e. 90 km/h or 25 m/s Ratio of speed of C to A =
62. (c) Let the speed of the train = x km/h x
and speed of slower train = 60 km/h
Then, relative speed of train and distance covered by C when B
50
= m/s reaches the destination = x − 20
3 = ( x + 5) km/h x
Since, these trains are moving in same Ratio of speed B to C =
and length of the train x − 20
direction.
= 225 m = 0. 225 km [given] x − 30 x − 48  B C B 
= = ×
So, relative speed of train
Time taken by train to cross the man x x − 20  A A C 
=  25 −  m/s =
50 25
m/s 0.225 ⇒ x2 − 50x + 600 = x2 − 48x
 3 3 = h
x+ 5 ∴ x = 300 m
Time taken by crossing slower from
According to the question, II. The speeds of A,B and C are in the
= 27 s ratio
0.225 9
∴ Distance = speed × time =
x + 5 3600 300 : 270 : 252 = 50 : 45 : 42
25
= × 27 = 225 m ⇒ x + 5 = 90
3
07
60 CDS Pathfinder

TIME AND WORK


Usually (2-4) questions have been asked from this chapter. Generally questions are asked on Man-
hours formulae, total wages paid for a work, number of days to complete a certain work and pipes
and cisterns.

In this chapter, we will study the relationship among the quantity of work given, wages given,
stipulated time, number of persons etc., and after it, we will be able to calculate the work in stipulated
time by arranging some persons according to the work but before that lets discuss some basic rules.

IMPORTANT RULES AND FORMULAE


Rule 1 1
If a person can do a piece of work in ‘n’ days, then he will do of the work in one day and if a
n
1
person can do th of work in one day, then he will complete the work in n days.
n
e.g. If Raj can do a piece of work in 20 days, then he will do 1/20th of the work in one day.
Note In the problems related to time and work it is always considered that a man/woman works at uniform rate.

Rule 2 A and B can do a piece of work in x days and y days, respectively. Then, time taken by ( A + B) to
xy
complete the work is equal to reciprocal of ( A + B)’s one day’s work, i.e. .
x+ y

Rule 3 If A and B can complete a work in x days and A alone can finish that work in y days, then
xy
number of days required to complete the work by B = days
y−x
Rule 4 If A and B can do a piece of work in x days. B and C can do same work in y days, C and A can
2xyz
do same work in z days. Then, they will complete the same work in days by working
xy + yz + zx
together.
61

EXAMPLE 1. Raj can do a piece of work in 20 days EXAMPLE 4. 15 men complete a work in 16 days. If
and Rohan can do it in 12 days. How long will they 24 men are employed, then the time required to
take if both work together? complete that work will be e 2014 I
1 1 1 1 a. 7 days b. 8 days c. 10 days d. 12 days
a. 5 days b. 7 days c. 3 days d. 9 days
2 2 2 2 Sol. c. Let the work done be 1.
1 Here, M1 = 15, D1 = 16, W1 = W2 = 1, M 2 = 24 and D2 = ?
Sol. b. Raj’s one day’s work =
20 Now, according to the formula,
1
Rohan’s one day’s work = M1D1W2 = M 2D2W1 ⇒ 15 × 16 × 1 = 24 × D2 × 1
12 15 × 16 240
1 1 3+ 5 8 ⇒ D2 = ⇒ = = 10 days
∴ (Raj + Rohan)’s one day’s work = + = = 24 24
20 12 60 60 Therefore, 10 days are required to complete the work.
∴ Number of day’s taken by Raj and Rohan together to
Rule 6 If ‘m’ men or ‘n’ women can do a piece of
60 1
complete the work = days or 7 days work in ‘a’ days, then x men and y women can do the
8 2
1
Here, x = 20 days and y = 12 days same work in days.
x y
∴Required time =
xy
[Rule 2] +
x+ y m× a n × a
20 × 12 20 × 12 15 1
= = = = 7 days EXAMPLE 5. If 3 men or 4 women can reap a field
20 + 12 32 2 2
in 43 days. How long will 7 men and 5 women take to
EXAMPLE 2. A and B together can do a piece of reap it?
work in 12 days and A alone can do it 18 days. In how a. 3 days b. 7 days c. 12 days d. 15 days
many days can B alone do it? Sol. c. Here, m = 3, n = 4, a = 43, x = 7 and y = 5
1 1
a. 14 days b. 24 days c. 36 days d. 28 days ∴ Required days = = [Rule 6]
x y 7 5
Sol. c. Here, x = 12 and y = 18 + +
m × a n × a 3 × 43 4 × 43
xy 12 × 18
∴Time taken by B = = = 36 days [Rule 3] 1 1
y − x 18 − 12 = = = 12days
1  7 5 1 43
 +  ×
EXAMPLE 3. A and B can do a piece of work in 43  3 4 43 12
3 days. B and C in 9 days and A and C in 12 days. Find Rule 7 If A can do a work in x days and B can do y% fast
the time in which A, B and C can finish the work, 100 x
working together. than A, then B will complete the work in days.
(100 + y)
1
a. 3 days b. 3 days
4 EXAMPLE 6. x can do a work in 16 days. In how
15
c. 3 days d. None of these
many days will the work be completed by y, if the
19 efficiency of y is 60% more than that of x? e 2013 II
Sol. c. Here, x = 3, y = 9 and z = 12 a. 10 days b. 12 days c. 25 days d. 30 days
According to rule, Sol. a. Here, x = 16 days and y = 60% faster
2xyz 100 x 100 × 16 1600
Required time taken by, A, B and C = [Rule 4] ∴ Required days = = = = 10 days
xy + yz + zx 100 + y 100 + 60 160
2 × 3 × 9 × 12 6 × 12 72 15 [Rule 7]
= = = =3 days
3 × 9 + 9 × 12 + 3 × 12 19 19 19 Rule 8 If A, B and C can do a piece of work in x, y and
Rule 5 If two groups, M 1 persons of the first group can z days, respectively and they received ` k as wages by
do ‘W1 ’, work in ‘D1 ’ days working T1 h in a day earning working together, then
a sum of ` R1 and M 2 persons of the second group can do yz
share of A = ` ×k
W2 work in D2 days working T2 h in a day earning a sum xy + yz + zx
of ` R2 . If each person of both group has the same xz
share of B = ` ×k
efficiency of work, then xy + yz + zx
M 1 D1T1 M 2 D2T2 xy
= and share of C = ` ×k
W1 R1 W2 R2 xy + yz + zx
62 CDS Pathfinder

Note If A and B can do a piece of work in x days and y days,


respectively and they received ` k as wages by working
yk xk
PIPES AND CISTERNS
together, then share of A = ` and share of B = ` . Problems on pipes and cisterns are based on the basic
x+ y x+ y
concept of time and work. Pipes are connected to a tank or
Wages are directly proportional to the work done and cistern and are used t4o fill or empty the tank or cistern.
indirectly proportional to the time taken by the individual.
EXAMPLE 7. X completes a job in 2 days and Y Important Rules and Formulae
completes it in 3 days and Z takes 4 days to complete Rule 9 If there are two pipes A and B takes ‘a’ and ‘b’ h
it. If they work together and get ` 3900 for the job, respectively to fill a tanker, then the two pipes together
then how much amount does Y get?  1 1
a. ` 1800 b. ` 1200 c. ` 900 d. ` 800 fill  +  part of the tank in 1h and time taken to fill
 a b
Sol. b. Here, a = 2 days, b = 3 days and c = 4 days and  ab 
k = ` 3900 the tank will be   h. If a pipe fills a tank in ‘a’ h, then
 a + b
ac
Now, amount of y = ×k [Rule 8] 1
ab + bc + ca in 1 h only of the tank is filled.
2× 4 a
= × 3900
2× 3+ 3× 4 + 4 × 2
8 8 × 3900 EXAMPLE 10. Pipe A can fill a tank in 45 h and pipe
= × 3900 = = ` 1200
6 + 12 + 8 26 B can fill it in 36 h. If both the pipes are opened in the
empty tank. In how many hours will it be full?
SOME OTHER FORMULAS a. 10 h b. 15 h c. 20 h d. 28 h
1. If A and B can do a piece of work in x and y days, Sol. c. Here, a = 45 h and b = 36 h.
respectively. A and B started working together but A left ab 45 × 36 1620
the work t days before completing the work, then time ∴ Required time = = = = 20 h
a + b 45 + 36 81
(x + t)y
taken to complete the work will be days.
(x + y) Rule10 (i) If a pipe can fill a tank in ‘a’ h and another
2. A and B do a piece of work in ‘a’ and ‘b’ days, can fill the tank in ‘b’ h but a third pipe
respectively. Both begin together but after some days, empties the filled tank in ‘ c’ h, then in one
A leaves the work and the remaining work is  1 1 1
completed by B in x days. Then, the time after which A hour  + −  part of the tank will be
(b − x ) a  a b c
left is given by T =
a +b filled (when all the three pipes are open) and
3. If A and B can do a piece of work in x and y days, time taken to fill the tank will be
respectively. They start working together and after t abc
h.
days B leaves the work, then time taken to finish the bc + ac − ab
x
whole work will be × ( y − t ) days.
y (ii) If a pipe can fill a tank in a1 h and another
pipe can empty the filled tank in a 2 h, then
EXAMPLE 8. Akshu can do a piece of work in 10 1 1
days and Harshal can do same work in 12 days. They in one hour  −  part of the tank will be
 a1 a 2 
started working together but Akshu left the work 2
days before completion of work, then time taken to filled. (when both pipes are open) and time
complete the work?  a a 
taken to fill the tank will be  1 2  h.
6
a. 6
3 3
days b. 5 days c. 4 days
2
d. 7 days  a 2 − a1 
11 10 2 5
6 12 × (10 + 2) 12 × 12 6 EXAMPLE 11. Two pipes ‘A’ and ‘B’ can fill a tank in
Sol. a. 6 days = = =6 days
11 10 + 12 22 11 36 min and 45 min, respectively. A waste pipe ‘C’ can
empty the tank in 30 min. In how much time the tank
EXAMPLE 9. A can do a piece of work in 10 days is full if all three pipes are opened?
and B can do same work in 15 days. They started
a. 60 min b. 90 min c. 115 min d. None of these
working together but after 2 days, A left work. The
remaining work was completed by B alone, then time Sol. a. Here, a = 36, b = 45 and c = 30.
taken to complete the work? So, time taken to fill the tank will be
abc
3 2 3 5 = [Rule 8]
a. 8 days b. 8 days c. 5 days d. 2 days ac + bc − ab
11 3 4 3
2 10 26 2 36 × 45 × 30
Sol. b. 8 days = (15 − 2) = = 8 days = = 60 min
3 15 3 3 36 × 30 + 45 × 30 − 36 × 45
63

PRACTICE EXERCISE
3 9. A and B can do a piece of work in 40 days and 50
1. X can do of a work in 12 days. In how many
4 days, respectively. Both begin together but after
days X can finish the 1/2 work? a certain time, A leaves off. In this case B
(a) 8 days (b) 16 days (c) 12 days (d) 24 days finishes the remaining work in 20 days. After
how may days did A leave?
2. A can do a piece of work in 10 days and B can do 1
the same work in 12 days. How long will they (a) 14 days (b) 13 days (c) 13 days (d) 15 days
3
take to finish the work, if both work together?
(a) 5
5
days (b) 6
5
days
10. P and Q can do a job in 2 days, Q and R can do it
11 11 in 4 days and P and R in 12/5 days. What is the
1
(c) 5 days (d) None of these
number of days required for P alone to do the
5 job?
(a) 5/2 (b) 3 (c) 14/5 (d) 6
3. A, B and C working together take 30 min to
address a pile of envelopes. A and B together 1
11. of a work is completed in half a day by
would take 40 min, A and C together would take 48
45 min. How long would each take working 1
5 persons. Then, of the work can be completed
alone? 40
(a) A : 72 min, B : 90 min, C : 120 min by 6 persons in how many days?
(b) A : 42 min, B : 90 min, C : 120 min 1
(a) 1 (b) 2 (c) 3 (d)
(c) A : 72 min, B : 90 min, C : 100 min 2
(d) A : 72 min, B : 80 min, C : 120 min
12. A garrison of ‘n’ men had enough food to last for
30 days. After 10 days, 50 more men joined
4. A and B together can do a piece of work in 12 them. If the food now lasted for 16 days, what is
days and A alone can do it in 36 days. In how the value of n?
many days can B alone do it? (a) 200 (b) 240 (c) 280 (d) 320
(a) 18 days (b) 12 days
13. Two pipes A and B can fill a tank in 12 and
(c) 15 days (d) 20 days
16 min, respectively. If both the pipes are opened
5. Ram can do a piece of work in 6 days and Shyam simultaneously, after how much time should B be
can finish the same work in 12 days. How much closed so that the tank is full in 9 min?
work will be finished, if both work together for (a) 3 min (b) 5 min (c) 4 min (d) 2 min
2 days?
14. 2 men undertake to do a job for ` 1400. One can
(a) One-fourth of the work (b) One-third of the work
do it alone in 7 days and the other in 8 days.
(c) Half of the work (d) Whole of the work With the assistance of a boy they finish the work
6. A and B can do given work in 8 days; B and C can in 3 days. How should the money be divided?
do the same work in 12 days and A, B, C complete (a) ` 600, ` 525, ` 275 (b) ` 550, ` 500, ` 350
it in 6 days. In how many days can A and C finish (c) ` 650, ` 470, ` 280 (d) None of these
it?
15. A, B and C can do a piece of work individually
(a) 12 (b) 8 (c) 14 (d) 16
in 8, 10 and 15 days, respectively. A and B start
7. A can finish a work in 8 days and B can do it in working but A quits after working for 2 days.
12 days. After A had worked for 3 days, B also After this, C joins B till the completion of
joins A to finish the remaining work. In how work. In how many days will the work be
many days will the remaining work be finished? completed?
(a) 2 days (b) 3 days (a) 53/9 days (b) 34/7 days
(c) 4 days (d) 5 days (c) 85/13 days (d) 53/10 days

8. Two taps can fill a tub in 5 min and 7 min, 16. A can do a piece of work in ‘x ’ days and B can
respectively. A pipe can empty it in 3 min. If all do the same work 3x days. To finish the
the three are kept open simultaneously, when work together they take 12 days. What is the
will the tub be full? value of ‘x’?
(a) 60 min (b) 85 min (c) 90 min (d) 105 min (a) 8 (b) 10 (c) 12 (d) 16
64 CDS Pathfinder

17. If 6 men and 8 boys can do a piece of work in PREVIOUS YEARS’ QUESTIONS
10 days while 26 men and 48 boys can do the
same in 2 days, what is the time taken by 15
25. 45 people take 18 days to dig a pond. If the pond
would have to be dig in 15 days, then the
men and 20 boys in doing the same type of work?
number of people to be employed will be e 2012 I
(a) 4 days (b) 5 days (c) 6 days (d) 7 days
(a) 50 (b) 54 (c) 60 (d) 72
18. Four taps can individually fill a cistern of water 26. A and B can do a piece of work in 10 h. B and C
in 1h, 2h, 3h and 6h, respectively. If all the four can do it in 15 h, while A and C take 12 h to
taps are opened simultaneously, the cistern can complete the work. B independently can
be filled in how many minutes? complete the work in e 2012 I
(a) 20 (b) 30 (c) 35 (d) 40 (a) 12 h (b) 16 h (c) 20 h (d) 24 h
19. 76 ladies complete a job in 33 days. Due to some 27. X can do a piece of work in 25 days. Y is 25%
reason some ladies did not join the work and more efficient than X. The number of days taken
therefore, it was completed in 44 days. The by Y is e 2012 II
number of ladies who did not report for the work is (a) 15 days (b) 20 days (c) 21 days (d) 30 days
(a) 17 (b) 18 (c) 19 (d) 20
28. A mason can build a tank in 12 h. After working
20. 9 men finish one-third work in 10 days. The for 6 h, he took the help of a boy and finished
number of additional men required for finishing the work in another 5 h. The time that the boy
the remaining work in 2 more days will be will take alone to complete the work is e 2013 I
(a) 78 (b) 81 (c) 55 (d) 30 (a) 30 h (b) 45 h (c) 60 h (d) 64 h

21. Ravi and Sneha working separately can finish a 29. X can complete a job in 12 days. If X and Y work
job in 8 and 12 h, respectively. If they work for 2
together, they can complete the job in 6 days. Y
an hour alternately, Ravi beginning at 9:00 am. 3
When will the job be finished? alone can complete the job in e 2013 I
(a) 10 days (b) 12 days (c) 15 days (d) 18 days
(a) 7 : 30 pm (b) 7 : 00 pm (c) 6 : 30 pm (d) 6 : 00 pm
22. Consider the following statements: 30. Pipe A can fill a tank in 10 min and pipe B can
empty it in 15 min. If both the pipes are opened
I. If 18 men can earn ` 1440 in 5 days, then in an empty tank, the time taken to make it
10 men can earn ` 1280 in 6 days. full is e 2013 I
II. If 16 men can earn ` 1120 in 7 days, then (a) 20 min (b) 25 min
21 men can earn ` 800 in 4 days. (c) 30 min (d) None of these
Which of the statement(s) given above is/are 31. 4 goats and 6 sheeps can graze a field in 50
correct? days. 2 goats and 9 sheeps can graze the field in
(a) Only I (b) Only II e 2013 II
(c) Both I and II (d) Neither I nor II (a) 100 days (b) 75 days (c) 50 days (d) 25 days

Directions (Q. Nos. 23-24) If a1 men and b1 boys can


32. A can finish a work in 15 days, B in 20 days and
C in 25 days. All these three worked together
complete a work in x days, while a2 men and b 2
and earned ` 4700. The share of C is e 2013 II
boys can complete the same work in y days, then
(a) ` 1200 (b) ` 1500 (c) ` 1800 (d) ` 2000
One day work of 1 man (yb 2 − xb1)
= 33. 20 workers working for 5 h per day complete a
One day work of 1 boy ( xa1 − ya2)
work in 10 days. If 25 workers are employed to
23. If 14 men and 12 boys can finish a work in 4 work 10 h per day, what is the time required to
days, while 8 men and 16 boys can finish the complete the work? e 2013 II
same work is 5 days. Compare the 1 day work of (a) 4 days (b) 5 days (c) 6 days (d) 8 days
1 man and 1 boy. 34. 18 men can earn ` 360 in 5 days. How much
1 1 money will 15 men earn in 9 days? e 2013 II
(a) 2 (b) 1 (c) (d) 3
2 2 (a) ` 600 (b) ` 540 (c) ` 480 (d) ` 360
24. 28 men and ‘m’ boys can finish a work in 4 days, 35. 2 men and 1 woman can complete a piece of
while 20 men and 6 boys can finish the same work in 14 days, while 4 women and 2 men can
work in 5 days. Now, if the ratio of 1 day work of do the same work in 8 days. If a man gets ` 90
1 man and 1 boy is 1 : 2, what will be the value per day, what should be the wages per day of a
of ‘m’? woman? e 2013 II
(a) 7 (b) 6 (c) 8 (d) 12 (a) ` 48 (b) ` 60 (c) ` 72 (d) ` 135
65

36. A can do a piece of work in 4 days and B can 40. A and B are two taps which can fill a tank
complete the same work in 12 days. What is the individually in 10 min and 20 min, respectively.
number of days required to do the same work However, there is a leakage at the bottom, which
together? e 2013 II can empty a filled tank in 40 min. If the tank is
(a) 2 days (b) 3 days (c) 4 days (d) 5 days empty initially, then how much time will both
the taps take to fill the tank with leakage?
37. A, B and C can do a piece of work individually in e 2015 II
8, 12 and 15 days, respectively. A and B start
(a) 2 min (b) 4 min (c) 5 min (d) 8 min
working but A quits after working for 2 days.
After this, C joins B till the completion of work. 41. If 4 men working 4 h per day for 4 days complete
In how many days will the work be completed? 4 units of work, then how many units of work
e 2014 II will be completed by 2 men working for 2 h per
8 6 7 3 day in 2 days? e 2015 II
(a) 5 days (b) 4 days (c) 6 days (d) 3 days 1 1
9 7 13 4 (a) 2 (b) 1 (c) (d)
2 8
38. A is thrice as efficient as B and hence completes
a work in 40 days less than the number of days 42. If m persons can paint a house in d days, then
taken by B. What will be the number of days how many days will it take for ( m + 2) persons to
taken by both of them when working together? paint the same house? e 2015 II
m+ 2 md
e 2014 II (a) md + 2 (b) md − 2 (c) (d)
(a) 22.5 days (b) 15 days (c) 20 days (d) 18 days md m+ 2

39. The efficiency of P is twice that of Q, whereas 43. Two pipes A and B can fill a tank in 60 min and
the efficiency of P and Q together is three times 75 min, respectively. There is also an outlet C. If
that of R. If P , Q and R work together on a job, A, B and C are opened together, then the tank is
in what ratio should they share their earnings? full in 50 min. How much time will be taken by
e 2015 I C to empty the full tank? e 2016 I
(a) 2 : 1 : 1 (b) 4 : 2 : 1 (c) 4 : 3 : 2 (d) 4 : 2 : 3 (a) 100 min (b) 110 min (c) 120 min (d) 125 min

ANSWERS
1 a 2 a 3 a 4 a 5 c 6 b 7 b 8 d 9 b 10 b
11 d 12 a 13 c 14 a 15 d 16 d 17 a 18 b 19 c 20 b
21 c 22 d 23 a 24 b 25 b 26 d 27 b 28 c 29 c 30 c
31 d 32 a 33 a 34 b 35 b 36 b 37 a 38 b 39 a 40 d
41 c 42 d 43 a

HINTS AND SOLUTIONS 


3 5 1 1 4 −3 1
1. (a) Since, X can do of work in =5 days = − = =
4 11 30 40 120 120
12 days. [Rule 1] 3. (a) B’s one min’s work A’ one min’s work = ( A + B )’s one min
12 × 4 work − B’s one min work
So, X can do 1 work in days. = ( A + B + C )’s one min’s work
3 1 1 9−4
= − =
1 12 × 4 × 1 − ( A + C )’s one min’s work 40 90 360
∴X can do work in = 8 days
2 3×2 1 1 6−4 2 1 5 1
= − = = = = =
2. (a) Here, x = 10 and y = 12 30 45 180 180 90 360 72
∴ Number of days taken by A and B C ’s one min’s work = ( A + B + C )’s one
Hence, A, B and C alone can finish the
xy 12 × 10 work in 72 min, 90 min and 120 min,
= = [Rule 2] min work − ( A + B )’s one min work respectively.
x + y 12 + 10
66 CDS Pathfinder

4. (a) Here, x = 12 and y = 36 1 1 1 14. (a) Let the boy completes the work in
∴ 1 min work of all taps = + −
xy 12 × 36 5 7 3 x days.
∴ Time taken by B = =
y − x 36 − 12 21 + 15 − 35 1 According to the condition,
= =
105 105 1 1 1 1 1 1 1 1
[Rule 3] + + = ⇒ = − −
Hence, tap will be filled in 105 min, if 7 8 x 3 x 3 8 7
12 × 36
= = 18 days they work together. 1 56 − 21 − 24 11
24 ⇒ = =
9. (b) Here, a = 40 days, b = 50 days, x 168 168
1
5. (c) ∴ One day work of Ram = x = 20 and T = ? 168
6 ∴ x= days
( b − x )a 11
1 ∴ Required time =
∴ One day work of Shyam = a+ b So, money is to be shared in the ratio
12 (50 − 20) × 40 30 × 40
= = 1 1 11
Hence, one day work of, Ram and : : or 24 : 21 : 11
Shyam ( 40 + 50) 90 7 8 168
2+ 1 3 40 1
1 1 1 = = 13 days Thus,
= + = = = 3 3 A ’s amount =
24
× 1400 = ` 600
6 12 12 12 4 12 56
1 10. (b) Here, x = 2, y = 4 and z =
∴ Two day’s work = 5 B ’s amount =
21
× 1400 = ` 525
2 56
∴ Time taken by ( P + Q + R)
Thus, if they work together for 2 day’s, 2xyz 11
then half of the work will be complete. = [Rule 4] Boy’s amount = × 1400 = ` 275
xy + yz + zx 56
6. (b) Here, x = 8, y = 12, z = 12 1
15. (d) A’s work in one day = , B’s work in
12
Now, time taken by ( A + B + C ) 2×2×4× 8
= 5 1
2xyz one day =
= [Rule 4] 2×4+ 4×
12 12
+ ×2 10
xy + yz + zx 5 5 1
192 C’ work in one day =
2 × 8 × 12 × z = 15
⇒ 6= 40 + 48 + 24
8 × 12 + 12 × z + z × 8 ( A + B )’s work in one day
192 12 1 1 5+ 4 9
= = days = + = =
⇒ 96 + 20z = 32z 112 7 8 10 40 40
⇒ 96 = 12z 2×9 9
Now, P one days work ( A + B )’s work in two days = =
⇒ z=8 7 1 7−3
40 20
= − = 9 11
So, A and C can do the work in 8 days. 12 4 12 Remaining work = 1 − =
1 20 20
7. (b) One day’s work of A = 4 1
= = ( B + C )’s work in one day
8 12 3
1 1 1 3+ 2 5 1
One day’s work of B = Hence, P alone can do in 3 days. = + = = =
12 10 15 30 30 6
MD MD
3 11. (d) Using the formula 1 1 = 2 2 Since, ( B + C ) complete the work in 6
3 day’s work of A = W W
8 1 2 days.
3 11
Remaining work of A = 1 − 12. (a) By given condition, ∴ work will be completed in
8 20
n × 30 = n × 10 + ( n + 50 ) × 16 11 11 × 3 33
5 6× = =
= ⇒ 20 n = 16 n + 800
days
8 20 10 10
800 ∴ Total number of days
One day’s work of A and B together ∴ n= = 200
4 33 20 + 33 53
1 1 3+ 2 5 = 2+ = = days
= + = = 13. (c) Here, x = 12, y = 16 and t = 9 10 10 10
8 12 24 24
Two pipes A and B can fill a tank in x h 1
Number of days to finish the work 16. (d) 1 day work of A =
and y h, respectively. If both the pipes ax x
5 5
= ÷ = 3 days opened simultaneously, then the time 1
8 24 1 day work of B =
after which B should be closed, so that 3x
1 the tank is filled in
8. (d) 1 min work of first tap = ∴ 1 day work of both A and B
5 th = [ y( 1 − t / x )]n 1 1 4
1 = + =
1 min work of second tap = Required time after which B should be x 3x 3x
closed = y  1 −  = 16 1 − 
7 t 9
given, one day work of both A and B
1  x  12  1
and 1 min work of third = − =
3 3 12
= 16 × = 4 min
12
67

4 1 1 3600
⇒ = ⇒ 3x = 48 ⇒ x = 16 21. (c) Work done by Ravi in one hour = = =8h
3x 12 8 450
Hence, the value of x is 16. 1 1 1
Work done by Sneha in one hour = So, 1 h work of B = −
12 8 12
17. (a) Given, 6 M + 8 B = 10 days ...(i)
Total work done by them in 2 h 3−2 1
and 26 M + 48 B = 2 days ... (ii) = =
1 1 5 24 24
15 M + 20 B = ? = + =
8 12 24 Hence, B alone can do in 24 h.
Here, M D = M D
1 1 2 2 Work done is 4 pairs of hours 27. (b) Here, x = 25 days and y = 25
⇒ ( 6 M + 8 B ) × 10 = ( 26 M + 48 B ) × 2 5 5 100x 100 × 25
= ×4= ∴ Required days = =
⇒ 60 M + 80 B = 52 M + 96 B 24 6 100 + y 100 + 25
⇒ 8M = 16B Remaining work after 8 hours 2500
= = 20 days
∴ M = 2B 5 1 125
= 1− =
Then, 15 M + 20 B = 15 × 2 B + 20 B 6 6 1
28. (c) Mason work for 1 h =
= 30 B + 20 B =50 B Now, its Ravi’s turn 12
On putting the value of M in Eq. (i), we 1 1 1 6 1
So, work left after 9 hours = − = Mason Work for 6 h = =
get 6 8 24 12 2
⇒ 6 × 2B + 8 B = 10 days 1 Work left = 1 −
1
=
1
⇒ 12 B + 8 B = 10 days work will be done by sneha in
24 2 2
∴ 20 boys finish the work in 10 days 1 Now, let the boy can finish the work
One boy finish the work in 12 × h = 30 min
24 in x h.
10 × 20 days = 200 days 1 1 x + 12
Total time taken = 9 h and 30 min Then, their 1 h work = + =
200 12 x 12x
and 50 boys finish the work in days Required time = 6 : 30 pm
50 x + 12 1 5x + 60 1
∴ ×5= ⇒ =
= 4 days 22. (d) I. 18 men can earn in 5 days = ` 1440 12x 2 12x 2
18. (b) Part filled by first tap in 1h = 1 1440
1 man can earn in 1 day = ` ⇒ 10x + 120 = 12x ⇒ 120 = 2x
1 18 × 5
Part filled by second tap in 1h = ∴ x = 60 h
2 ∴ 10 men can earn in 6 days
1
1 1440 29. (c) X ’s one day work = and ( X + Y )’s
Part filled by third tap in 1h = = × 6 × 10 12
3 18 × 5 3
one day work =
Part filled by fourth tap in 1h =
1 = ` 960 ≠ ` 1280 20
6 II. 16 men can earn in 7 days = ` 1120 ∴ Y ’s one day work
Total tank filled by all taps in 1h 1120 3 1 4 1
1 man can earn in 1 day = = − = =
1 1 1
= 1+ + + 16 × 7 20 12 60 15
2 3 6 ∴ 21 men can earn in 4 days ∴ Number of day’s taken by Y to
∴ Required time =
1 1120 complete the work = 15 days
1 1 1 = × 21 × 4 = ` 840 ≠ ` 800
1+ + + 16 × 7 30. (c) Part filled by pipe A in 1 min =
1
2 3 6 So, neither statement I nor II is correct. 10
6 6 6
= = h= × 60 min 23. (a) Here, a = 14, b = 12, x = 4, and part emptied by pipe B in 1 min
6 + 3 + 2 + 1 12 12 1 1 1
a = 8, b = 16 and y = 5 =
2 2 15
= 30 min
One day work of 1 man ( yb2 − xb1 ) ∴ Total tank filled in minutes
=
19. (c) Given, M = 76, D = 33 One day work of 1 boy ( xa − ya ) 1 1 3−2 1
1 1 1 2 = − = =
Let number of ladies who did not 10 15 30 30
report for the work = x 5 × 16 − 4 × 12
= =2 Hence, the tank will be filled in 30 min.
By given condition, 4 × 14 − 5 × 8
24. (b) Refer to Question No 23. 31. (d) Here, m = 4, n = 6, a = 50 and x = 2,
M = 76 − x and D = 44 y=9
2 2
We know that, 25. (b) Using the formula, M D = M D 1
1 1 2 2
∴ Required days = x
MD =MD [Rule 5] 26. (d) Here, x = 10, y = 15, z = 12. +
y
1 1 2 2
∴ 76 × 33 = ( 76 − x) × 44 ∴ Time taken by ( A + B + C ) m× a n× a
76 × 3 2xyz 1
⇒ 76 − x = = 19 × 3 ⇒ x = 19 = [Rule 4] = [Rule 6]
2 9
4 xy + yz + zx +
4 × 50 6 × 50
Hence, the number of ladies is 19. 2 × 10 × 15 × 12 1 1 100
= = = = = 25 days
20. (b) Refer to Question No 11 1 3 4
10 × 15 + 15 × 12 + 12 × 10 + 4
[Using Rule 5] 100 100 100
68 CDS Pathfinder

32. (a) Here, x = 15 days, y = 20 days and 1 x x−2 Part of tank emptied by leakage in 1 min
⇒ + + =1
z = 25 days and K = `4700 4 12 15 1
=
  15 + 5x + 4( x − 2) 40
Share of C = ` 
kxy
 ⇒ =1
 x y + yz + zx  60 Part of tank filled by both taps and
⇒ 15 + 5x + 4x − 8 = 60 ⇒ 9x + 7 = 60
leakage is 
4700 × 15 × 20 1 1 1 1
=` 53 8 + −  =
⇒ 9x = 53 ⇒ x = =5  10 20 40  8
15 × 20 + 20 × 25 + 25 × 15
9 9
4700 × 15 × 20 Hence, total time taken to fill the tank is
=` = ` 1200 Hence, the work will completed in
1175 8 8 min.
5 days.
33. (a) Refer to Question No 11 9 41. (c) Here, M = 4,T = 4,W = 4,D = 4
1 1 1 1
[Using Rule 5] 38. (b) Let efficiency of A be x days. and M = 2, T = 2, D = 2, W = ?
2 2 2 2
34. (b) Refer to Question No 22 Then, efficiency of B = 3x days. Using the formula,
1 M ×T ×D M ×T ×D
35. (b)Q 14 days work of 2 men and 1 women ∴ B’s one day work = 1 1 1 = 2 2 2 [Rule 5]
3x W W
= 8 days work of 4 women and 2 men 1 2
1
⇒ 1 day work of 28 men and 14 women and A’ s one day work = 4×4×4 2×2×2
x ⇒ =
= 1day work of 32 women and 16 men Again, let time taken by B = t days 4 W
2
⇒ 28 men + 14 women Then, time taken by A = ( t − 40) days 8 1
= 32 women + 16 men ⇒ W = =
t t − 40 2
16 2
⇒ 28 m − 16 m = 32 w − 14 w Now, = ⇒ t = 3( t − 40)
3x x 42. (d) m persons paint a house in d days.
m 18 3
⇒ 12 m = 18 w ⇒ = = ⇒ 3t − t = 120, t =
120
= 60 days ∴ 1 person paints a house in
w 12 2 2 (m × d ) days.
So, efficiency of 1 man and 1 woman is So, B complete the work in 60 days and
3 : 2. and m + 2 persons paint a house in
A complete the work in 20 days.  
So, their wages must be in the same ratio  md  days.
90 3 ( A + B )'s 1 day work
∴ = m + 2
=
1
+
1
=
4
=
1  
x 2
60 20 60 15
[where, x = wages per day of a woman] 43. (a) Since, two pipes A and B fill a tank in
90 × 2 Hence, they will complete the work in 60 min and 75 min, respectively.
∴ x= = ` 60 15 days when they work together.
3 ∴ Part of tank filled by pipe A in 1 min
39. (a) Let efficiency of P be x days. 1
1 =
36. (b) A’s one day work = Then, efficiency of Q = 2x days
4 60
1
and B’s one day work =
1 ∴ Q’s one day work = and part of tank filled by pipe B in 1 min
12 2x 1
1 =
One day work of A and B together and P ’s one day work = 75
1 1 3+ 1 4 1 x
= + = = = Now, part of tank filled by A and B
4 12 12 12 3 Now, ( P + Q )’s one day work together in 1 min
1 1 3
Days required by A and B together to do = + = =
1
+
1
=
9
=
3
the work x 2x 2x 60 75 300 100
1 ∴( P + Q ) will complete the whole work
= Let part of tank emptied by pipe C in 1
One day work of A and B together 2x
in days. 1
3 min = .
= 3 days C
1 According to the question,
37. (a) Work done by A in one day = So, net part of tank filled by pipes A, B
8 R will complete this work = 2x days and C together in 1 min
1 1
Work done by B in one day = ∴ R’ s one day work = =
3
− =
1 1
[Given]
12 2x 100 C 50
1
and work done by C in one day = 1 1 1
Required ratio = : :
1 1
= 1: : 1 3 1 1
15 x 2x 2x 2 2 ⇒ = − =
C 100 50 100
Let the work will be completed in = 2 : 1: 1
x days. ∴ C = 100
1
2 x x−2 40. (d) Part of tank filled by A in 1 min = Hence, the time taken by pipe C to
Then, + + =1 10
8 12 15 empty the tank is 100 min.
1
Part of tank filled by B in 1 min =
20
08
69

PERCENTAGE
Generally (2-3) questions have been asked from this chapter. Generally questions which are asked from this
chapter are tricky but you can easily solve them by using the short-tricks formulae in very less time.

PER CENT
Per cent is a fraction whose denominator is 100 and numerator of the fraction is called the rate per
cent. Per cent is denoted by the symbol ‘%’.

Important Rules and Formulae


a
Rule 1 For expressing a% as a fraction, we can write a % = , i.e. divide ‘a’ by 100 and reduce it to
100
25 1
the lowest form, e.g. 25% = =
100 4
a a
=  × 100 %
a
Rule 2 For expressing a fraction as a per cent, we can write
b b b 
2 2 200
e.g. = × 100% = %
3 3 3

Rule 3 To find how much per cent one quantity is of another quantity, we can write
The quantity to be expressed in per cent
Required percentage = nd
× 100%
2 quantity (in respect of which the per cent has to be obtained)

EXAMPLE 1. 20g is what per cent of 1000g?


a. 2% b. 5% c. 7% d. 9%
20
Sol. a. Required percentage = × 100% = 2%
1000
Rule 4 Percentage increase or decrease in a quantity when it increases or decreases by some value.
Increase or decrease in the value
Percentage = × 100 %
Initial value
70 CDS Pathfinder

EXAMPLE 2. A person’s salary has increased from


Rule 8 If the value of an object is first changed
` 7200 to ` 8100. What is the percentage increase in
his salary? e 2013 II
(increased or decreased) by x% and then changed (increased
2 1 or decreased) by y% , then
a. 25% b. 18% c. 16 % d. 12 % ( ± x)( ± y)

3 2 Net effect =  ± x ± y + %
8100 − 7200  100 
Sol. d. Percentage increase in salary = × 100%
7200 Here, + sign is used in case of increment and − sign is used
900 1
= × 100% = 12.5% = 12 % in case of decrement.
7200 2

Rule 5 If A is x% more than ‘B’, then EXAMPLE 5 The price of an article is first increased by
 x  20% and later on the price were decreased by 25% due
B would be  × 100 % less than ‘A’. to reduction in sales. Find the net percentage change in
(100 + x)  final price of article.
Rule 6 If A is x% less than ‘B’, then B would be a. 20% b. 10% c. 30% d. 15%
Sol. b. Here, a = 20 % , b = 25 %
 x 
(100 − x) × 100 % more than A.
10
So, required percentage = × 100 = 10%
  100
 ( ± a) ( ± b) 
Required change = ( ± a) + ( ± b) + %
EXAMPLE 3. Raj get 10% less marks than Rohit in  100 
an examination. What percentage of marks does
Rohit gets more than Raj?  20 × ( −25)   positive sign for increase 
= 20 − 25 + %
1 1 1  100  negative sign for decrease
a. 11 % b. 11 % c. 12 % d. None of these
6 9 9 = [ −5 − 5]%
Sol. b. Given, x = 10% = −10 %
10 ∴Net percentage change is a decrease of 10 % because
∴ Rohit more percentage than Raj = × 100
100 − 10 final result is negative.
10 × 100 1
= = 11 % Rule 9 If the population of a town is P and it increases
90 9 (or decreases) at the rate of R% per annum, then
Rule 7 If the price of a commodity is increased
n
(i) Population after n yr = P 1 ±
(or decreased) by x%, then the decrease (or increase) in R

consumption, so as not to increase or (decrease) the  100 
 x  P
expenditure is  × 100 %. (ii) Population, n yr ago =
 100 ± x  n
1 ± R 
 
 100 
EXAMPLE 4. If the price of the cooking gas rises
by 15%, by what per cent should a family reduce its
consumption so as not to exceed the budget on EXAMPLE 6 The population of a town is 352800. If it
cooking gas? increases at the rate of 5% per annum, then what will
1 1 1 be its population 2 yr hence, also find the population
a. 12 % b. 13 % c. 14 % d. None of these 2 yr ago.
23 23 23
a. 315000 b. 316500 c. 200045 d. 320000
Sol. b. Let initial price of cooking gas be ` 100.
Sol d. Given that, p = 352800, R = 5% and n = 2
Price after increase = ` 115
According to the formula,
On ` 115 he should reduce ` 15 on ` 100, he should
R n 5 2
reduce Population after 2 yr = P 1+ 
 = 352800 × 1+ 
15 1  100  100
= × 100 = 13 %
 100 + 5 = 352800 ×  21 × 21 = 388962
115 23 2
= 352800 ×    
Shortcut Method  100   20 20
Here, x = 15% Population 2 yr ago
 x 
∴Reduction in consumption =  × 100 % P 352800 20 20
 100 + x  = n = = 352800 × × = 320000
1 R  1 5 2 21 21
15 1  +   + 
= × 100% = 13 %  100  100
115 23
71

Rule 10 If the present population of a city is P and there an increment of 45%, then find the population of city
is a increment or decrement of R1 %, R2 % and R3 % in at the end of year 2007.
first, second and third year respectively, then a. 1083875 b. 1083000 c. 1089000 d. 1135000

Population of city after 3 yr Sol a. Given that, P = 1000000 , R1 = 15 %, R2 = 35%


(decrease) and R3 = 45%
 R  R  R  Population of city at the end of year 2007.
= P 1 ± 1  1 ± 2  1 ± 3 
 100   100   100   R   R   R 
= P 1+ 1  1− 2  1+ 3 
 100  100  100
Note Use ‘+’ sign for increases and ‘–’ sign for decreases.
= 1000000 1+
5   35   45 
 1−  1+ 
 100  100  100
EXAMPLE 7 Population of a city in 2004 was
1000000. If in 2005 there is an increment of 15%, in 115 65 145
= 1000000 × × ×
2006 there is a decrement of 35% and in 2007 there is 100 100 100
= 1083875

PRACTICE EXERCISE
1. If 90% of A = 30% of B and B = x % of A, the value 10. To pass an examination, a candidate needs 40%
of x is marks. All questions carry equal marks. A
(a) 700 (b) 600 (c) 300 (d) 1100 candidate just passed by getting 10 answers
correct by attempting 15 of the total questions.
2. When 40% of a number is added to 42, the result How many questions are there in the
is the number itself. The number is examination?
(a) 70 (b) 90 (c) 82 (d) 72 (a) 25 (b) 30
3. If x% of y is 13x, then the value of y is (c) 40 (d) 45
(a) 880 (b) 1300 (c) 1200 (d) 700 11. If salary of X is 20% more than salary of Y, then by
how much percentage is salary of Y less than X ?
4. What is the number when 20% of number is 30%
(a) 25 (b) 20
of 40?
50 65
(a) 90 (b) 80 (c) 60 (d) 50 (c) (d)
3 4
5. What is the number when increased by 20% 2
becomes 300? 12. Water contains 14 % of hydrogen and the rest is
7
(a) 250 (b) 200 (c) 180 (d) 280
oxygen. In 350 g of water, oxygen will be
6. If 50% of ( x − y ) = 40% of ( x + y ), then what (a) 300 g (b) 250 g
percent of x is y? (c) 200 g (d) None of these
1 1 1 1
(a) 10 % (b) 11 % (c) 13 % (d) 21 % 13. The income of ‘A’ is 20% higher than that of ‘B’.
9 9 9 9
The income of ‘B’ is 25% less than of C. What per
7. Two candidates fought an election one get 65% of cent less is A ’s income from ‘C’s income?
the votes and won by 300 votes. The total (a) 7% (b) 8%
number of votes polled in the election is (c) 10% (d) 12.5%
(a) 700 (b) 950 (c) 1000 (d) 900 14. 38 L of milk was poured into a tub and the tub
1 was found to be 5% empty. To completely fill the
8. A man spends ` 3500 and saves 12 % of his
2 tub, what amount of additional milk must be
income. His monthly income (in `) is poured?
(a) 4000 (b) 3800 (c) 4200 (d) 4500 (a) 1 L (b) 2 L (c) 3 L (d) 4 L
9. In an examination 52% of the candidates failed 15. 10% of the inhabitants of a certain city left that
in English, 42% in Mathematics and 17% in city. Later on 10% of the remaining inhabitants
both. The number of those who passed in both of that city again left the city. What is the
the subjects is remaining percentage of population of that city?
(a) 23% (b) 40% (c) 53% (d) 33% (a) 80% (b) 80.4% (c) 80.4% (d) 81%
72 CDS Pathfinder

16. If after 24% of wastage the net output of a 27. Two numbers are less than a third number by
coal-mine is 68400 quintals. Then, the total 30% and 40%, respectively. How much per cent
output of the coal-mine in quintals is is the second number less than the first?
2
(a) 70000 (b) 90000 (c) 80000 (d) 89000 (a) 35% (b) 36% (c) 14% (d) 14 %
7
17. A rise of 25% in the price of grapes compels a
person to buy 1.5 kg of grapes less for ` 240. 28. A sample of 5 L of glycerine is formed to be
Then,the original price of grapes per kg is adulterated to the extent of 20%. Find how
glycerine should be added to bring down
(a) ` 40 (b) ` 32 (c) ` 30 (d) ` 28
percentage of impurity to 5%?
18. The price of an item is increased by 20% and (a) 10 L (b) 25 L (c) 15 L (d) 20 L
then decreased by 20% the final price as 29. The daily wages of a worker increase by 20% but
compared to original price is the number of hours worked by him also dropped
(a) 4% less (b) 4% more (c) 20% less (d) 20% more by 20%. If originally he was getting ` 200 per
19. Sohan saves 14% of his salary while George week, his wages per week now is
saves 22%. If both gets the same salary and (a) ` 160 (b) ` 192 (c) ` 210 (d) ` 198
George saves ` 1540. Then, the salary of each of 30. 140 L of a liquid contains 90% of acid and the
them is rest water. How much water must be added to
(a) ` 9500 (b) ` 17000 (c) ` 7000 (d) ` 7500 make the water 12.5% of the resulting mixture?
20. A’s salary is half that of B. If A got a 50% rise in (a) 4 L (b) 10 L (c) 12 L (d) 3 L
his salary and B got a 25% rise in his salary, 31. A person spends 30% of monthly salary on rent,
then the percentage increase in combined 25% on food, 20% on children’s education and
salaries of both is 12% on electricity and the balance of ` 1040 on
1
(a) 13% (b) 33 % (c) 33% (d) 45% the remaining items. What is the monthly salary
3 of the person?
21. A man donated 4% of his salary to a charity and (a) ` 8000 (b) ` 9000 (c) ` 9600 (d) ` 10600
deposited 10% of the rest in the bank. If now he 32. In an office, 40% of the staff is female and rest is
has ` 10800, then his income was male. 60% of the male and 40% of female voted for
(a) ` 13500 (b) ` 14500 Ramesh. The percentage of votes Ramesh got was
(c) ` 40000 (d) ` 12500 (a) 24% (b) 42% (c) 50% (d) 52%
22. Rohit saves 30% of his salary. When his 33. In a class-X of 30 students, 24 passed in first
expenses increased by 30%, he is able to save class; in another class-Y of 35 students, 28
` 1215 per month. His monthly salary is passed in first class. In which class was the
(a) ` 13500 (b) ` 14500 percentage of students passed first class more?
(c) ` 30000 (d) ` 12500 (a) Class-X (b) Class-Y
(c) Both X and Y (d) None of these
23. A man spends 75% of his income. If his income is
increased by 20% and he increased his expenditure
by 10%. His savings percentage is increased by Directions (Q. Nos. 34-36) The population of the
town is 126800. It increases by 15% in the 1st year
(a) 25% (b) 50% (c) 75% (d) 10%
due to increase crime in the city.
24. The price of wheat has increased by 60%. In
order to restore to the original price, the new 34. What is the population of the town at the end of
price must be reduced by 2nd year if the population decreases by 20% in
(a) 37.5% (b) 33% (c) 34% (d) 40% the second year?
(a) 174984 (b) 135996 (c) 116656 (d) 145820
25. If the numerator of a fraction increased by 20%
and its denominator be diminished by 10%. The 35. What is the population of the town, if the population
16 decrease by 15% at the end of 2nd year?
value of the fraction is , then the fraction is
27 (a) 123749 (b) 123479
(a) 4/9 (b) 3/2 (c) 3/8 (d) 9/4 (c) 123947 (d) None of these
26. 1 L of water is evaporated from 6 L of a solution 36. What is the population of the town, if the
having 4% of sugar. The percentage of sugar in population increases by 20% at the end of 2nd
the remaining solution is year?
4 4 (a) 194784 (b) 174984 (c) 179484 (d) 178494
(a) 4% (b) 5% (c) 4 % (d) %
5 5
MATHEMATICS Percentage 73

PREVIOUS YEARS’ QUESTIONS 44. A person could save 10% of his income. But 2 yr
later, when his income increased by 20%, he
37. A man losses 20% of his money. After spending could save the same amount only as before. By
25% of the remainder, he has ` 480 left. What is how much percentage has his expenditure
the amount of money he originally had? e 2012 I increased? e 2015 I
(a) ` 600 (b) ` 720 (c) ` 800 (d) 840 2 1
(a) 22 % (b) 23 %
38. The price of an article is ` 25. After two 9 3
2 2
successive cuts by the same percentage, the price (c) 24 % (d) 25 %
9 9
becomes ` 20.25. If each time the cut was x%,
then e 2012 II 45. 20% of a number when added to 20 becomes the
(a) x = 9 (b) x = 10 (c) x = 11 (d) x = 115
. number itself, then the number is e 2015 II
(a) 20 (b) 25 (c) 50 (d) 80
39. What is 5% of 50% of 500? e 2012 II
(a) 12.5 (b) 25 (c) 1.25 (d) 6.25 46. A’s salary was increased by 40% and then
40. X , Y and Z had taken a dinner together. The cost decreased by 20%. On the whole A’s salary is
of the meal of Z was 20% more than that of Y increased by e 2015 II
(a) 60% (b) 40% (c) 20% (d) 12%
and the cost of the meal of X was 5/6 as much as
the cost of the meal of Z. If Y paid ` 100, then 47. In an election 10% of the voters on the voter list
what was the total amount that all the three of did not cast their vote and 60 voters cast their
them had paid? e 2013 II ballot papers blank. There were only two
(a) ` 285 (b) ` 300 candidates. The winner was supported by 47% of
(c) ` 355 (d) None of these total voters in the voter list and he got 308 votes
41. A water pipe is cut into two pieces. The longer more than his rival. The number of voters on the
piece is 70% of the length of the pipe. By how voter list is e 2015 II
much percentage is the longer piece longer than (a) 3600 (b) 6200 (c) 6028 (d) 6400
the shorter piece? e 2013 II 48. The salary of a person is increased by 10% of his
400
(a) 140% (b) % original salary. But he received the same amount
3
even after increment. What is the percentage of
(c) 40% (d) None of these
his salary he did not receive? e 2016 I
42. If m% of m + n % of n = 2% of ( m × n ), then what (a) 11% (b) 10% (c)
100
% (d)
90
%
percentage of m is n? e 2014 II 11 11
(a) 50% (b) 75%
(c) 100% (d) Cannot be determined
49. The expenditure of a household for a certain
month is ` 20000, out of which ` 8000 is spent
43. The price of a commodity increased by 5% from on education, ` 5900 on food, ` 2800 on
2010 to 2011, 8% from 2011 to 2012 and 77% from shopping and the rest on personal care. What
2012 to 2013. What is the average price increase percentage of expenditure is spent on personal
(approximate) from 2010 to 2013? e 2014 II care? e 2016 I
(a) 26% (b) 32% (c) 24% (d) 30% (a) 12% (b) 16.5% (c) 18% (d) 21.8%

ANSWERS
1 c 2 a 3 b 4 c 5 a 6 b 7 c 8 a 9 a 10 a
11 c 12 a 13 c 14 b 15 d 16 b 17 b 18 a 19 c 20 b
21 d 22 a 23 b 24 a 25 a 26 c 27 d 28 c 29 b 30 a
31 a 32 d 33 c 34 c 35 c 36 b 37 c 38 b 39 a 40 d
41 b 42 c 43 d 44 a 45 b 46 d 47 b 48 c 49 b
74 CDS Pathfinder

HINTS AND SOLUTIONS 


1. (c) Given, 90% of A = 30% of B 8. (a) Let monthly income be ` x. Now, difference between C’ s salary and
90 A 30B 1 A ’ s salary
= [Rule 1] ⇒ 87 % of x = ` 3500
100 100 2 9x x
= x− =
A 3 175 10 10
⇒ = ⇒ B = 3A ⇒ x = ` 3500
B 9 2 × 100 ∴ The required percentage
xA
Now, B = x % of A, 3 A = [Rule 1] x
100 3500 × 2 × 100 = × 100% = 10%
∴ x= = ` 4000 10 × x
∴ x = 300 175
Hence, the value of x is 300. 9. (a) Number of candidates who passed in 14. (b) Amount of milk in tub = 38 L
2. (a) Let the number be x, both the subject = 100 − ( x + y − z )% Q Tub found to be 5% empty
then, 42 + 40% of x = x = 100 − (52 + 42 − 17)% ∴ Total quantity of milk in per cent
40
= 23% = 95%
⇒ x = 42 + x [Rule 1]
100 Now, completely fill the tub, total
3 10. (a) Let the total number of questions in
⇒ x = 42 examination be x. amount of additional milk
5 38 × 5
42 × 5 By given condition, 40% of x = 10 = = 2L
∴ x= = 70 x × 40 1000 95
3 ⇒ = 10 ⇒ x = = 25
100 40 15. (d) Remaining percentage
Hence, the required number is 70.
=  1 −
10   10 
3. (b) Given, x% of y = 13x
11. (c) Let the salary of y be ` x .  1− 
 100   100 
x 120x 6x
⇒ y = 13x [Rule 1] ∴ Salary of x = ` =` 90 90
100 100 5 = × = 81%
∴ y = 13 × 100 = 1300 100 100
Difference of their salaries
16. (b) Let total output be x quintal.
= `  − x = `
4. (c) Let the number be x. 6x x
By given condition, 20% of x  5  5 ∴ Useful output = 100 − 24 = 76%
76
= 30% of 40 ∴ Required percentage ⇒ x = 68400
20x 30 × 40 x/5 50 100
⇒ = ⇒ 20x = 30 × 40 = × 100% = % 68400 × 100
100 100 6x / 5 3 ⇒ x=
∴ x = 60 76
[Rule 3]
Hence, the required number is 60. ∴ x = 90000 quintal
Shortcut Method
5. (a) Let the number be x. Here, x = 20% 17. (b) Let original price of grapes be ` x.
20x 25
∴ x+ = 300 ∴ Required percentage ∴ Increased price = x + x
100
=  × 100 % [Rule 5]
x 100
300 × 100
⇒ x= = 250  100 + x  125
=` x …(i)
120
=  × 100 %
20 100
6. (b) Given, 50% of ( x − y ) = 40% of ( x + y )  100 + 20  Increased price of 1.5 kg of grapes
50 40 2000 50
× (x − y) = × (x + y) = %= % 25
100 100 = × 240 = ` 60
120 3 100
⇒ 5x − 5 y = 4x + 4 y [Rule 1] 12. (a) Percentage of oxygen in water 60
⇒ x = 9y … (i) 2 100 5 ∴ Increased price of 1 kg =
Let r % of x = y ⇒
r
×x= y = 100 − 14 = 100 − = 85 % 1.5
7 7 7
100 = ` 40 …(ii)
r
⇒ × 9y = y [from Eq. (i)] Percentage of oxygen in 350 g of water
100 So, from Eqs. (i) and (ii), we have
5 600 350
100 1 = 85 % of 350 = × = 300 g 125
∴ r= = 11 % 7 7 100 x = ` 40
9 9 100
13. (c) Let the income of C be ` x.
7. (c) Let total number of votes polled be x. 40 × 100
25x ∴ x= = ` 32
∴Income of B = x − x × 25% = x − 125
∴ 65% of x − 35% of x = 300 100
65 35 18. (a) The price of item first increased by
x− x = 300 = x−
x 3x
=
100 100 20% and then decreased by 20%.
4 4
30 300 × 100 20 × ( −20) 
⇒ x = 300 ⇒ x =
and, the income of A =
3x 3x
+ × 20% ∴ Net effect =  20 − 20 + 
100 30  100 
4 4
∴ x = 1000 votes 3x 3x 1 9x −400
= + × = = = −4% [Rule 8]
Hence, the total number of votes is 1000. 4 4 5 10 100
MATHEMATICS Percentage 75

19. (c) Let salary of each of them be ` x. 24. (a) New price reduced by 30. (a) Water in the mixture = 10% of 140 L
=   × 100%
George saves 22% of x and his saving x 10
[Rule 7] = × 140 = 14 L
amount is ` 1540.  100 + x  100
22
⇒ x = 1540
= 
60  Let x L of water added in the mixture,
100  × 100% = 37.5% 14 + x 
1540 × 100
 100 + 60  then   × 100 = 125
.
⇒ x= = ` 7000 x  140 + x 
22 25. (a) Let fraction be .
y ⇒ 1400 + 100x = 1750 + 125
. x
20. (b) Let A ’s salary = x. Then,
120% ofx 4x ⇒ . x = 350
875
B’s salary = 2x New fraction = =
90% of y 3 y 350
New salary of A = 150% of x =
3
x ⇒ x = = 4L
2 875.
According to question,
5 31. (a) Let the monthly salary of the person
Total salary of B = 125% of 2x = x [by given condition]
2 4x 16 x 16 3 4 be ` x.
= ⇒ = × = By given condition,
Total combined salary 3 y 27 y 27 4 9
( 100 − 30 − 25 − 20 − 12) × x
=  x + x = 4x
3 5
= 1040
2 2  26. (c) Amount of sugar in 6 L of solution 100
4 1040 × 100
∴ Required increment in salary = × 6 = 0. 24 L ⇒
13x
= ` 1040 ⇒ x =
4x − 3x x 100 1 100 100 13
= = × 100 = =33 %
3x 3x 3 3 After evaporation, sugar in 5 L = 0. 24 L ⇒ x = ` 8000
[Rule 4] ∴ Percentage of sugar Hence, the monthly salary of the person
 0. 24  4 is ` 8000.
21. (d) Let total income be ` x. = × 100 = 4 %
 5 
Income deposited = 10% of  x −
4    5 32. (d) Let total number of staff be 100.
x
 100  Female staff = 40
10  4  96 27. (d) Let the third number be z.
= x − x = x ( 100 − 30) 7z Male staff = ( 100 − 40) = 60
100  100  1000 ∴ First number x = ×z =
Votes casted by females
100 10
Remaining income = ` 10800 40
4 96x Second number = × 40 = 16
∴ x+ + 10800 = x ( 100 − 40) 6z 100
100 1000 y= ×z = 60
136x 100 10 Votes casted by males = × 60 = 36
⇒ 10800 = x − 100
1000 Difference between first and second
864x 10800 × 1000 number Votes casted by both males and females
⇒ 10800 = ⇒x = = 16 + 36 = 52
7z 6z z
1000 864 = (x − y) = − =
∴ x = ` 12500 10 10 10 ∴ Percentage votes obtained = 52%.
22. (a) Let the salary of Rohit be ` 100, then Hence, the required percent [Rule 4] 33. (c) For class-X, let the student passed in
saving = ` 30 z first class = a%.
100 2
Expenses = ` 70 = 10 × 100% = % = 14 % Then, by condition given in question,
7z 7 7
New expenses = ( 100 + 30)% of ` 70 a × 30
10 a% of 30 = 24 ⇒ = 24
= ` 91 100
28. (c) Glycerine in the given sample = 80%
New saving = ` ( 100 − 91) = ` 9 ∴ a = 80%
of 5 L
He saves ` 9, his salary = ` 100 80 Now, for class-Y let the student passed
If he saves ` 1215. = ×5= 4L in first class = b%.
100
Then, his salary = `  × 1215
100 According to the question,
 9  Let x L of glycerine be added, then
4+ x b% of 35 = 28
= ` 13500 × 100 = 95 b
(5 + x ) ⇒ × 35 = 28
23. (b) Let income of man be ` 100. 100
⇒ 80 + 20x = 95 + 19x
Then, his expenditure = ` 75 ∴ b = 80%
∴ x = 15 L
and savings = ` 25 Hence, both classes have equal
New income = ` ( 100 + 20) = ` 120 29. (b) Increased wages of the worker percentage of students getting first class.
= 200 + 20% of 200 = ` 240
New expenditure = ` ( 75 + 7.5) 34. (c) Given, R = 15% and R = 20% .
Also, let he worked for x h. 1 2
= ` 82.50 ∴ Required population
New saving = ` ( 120 − 825. ) = ` 37.50 ∴ Reduced working hours
 R   R 
=x−
20
x = 080 = P 1+ 1  1− 2  [Rule 10]
Saving difference = 37.50 − 25.0 = 12.5 . x
 100   100 
100
∴ Percentage increase saving
= 126800  1 +
240 15   20 
12.5 Required wages = × 08
. x = ` 192  1− 
= × 100 = 50% x  100   100 
25
76 CDS Pathfinder

= 126800  1 +
3  1 = ( 100 + 20) = ` 120 Winner was supported by 47% of total
 1− 
 20   5 5 voter. i.e. 47% of x.
and, the cost of the meal of X = as
Hence, rival got vote = (90% of x − 60)
= 126800     = 116656
23 4 6
 20   5  much as the cost of the meal of − 47% of x.
5
35. (c) Given, R = 15%, R = 15% Z = × 120 = ` 100 = 43% of x − 60
1 2 6
It is given that difference between their
 R  R 
∴ Population = P  1 + 1   1 − 2  ∴ Total amount that all the three of vote is 308.
 100  100  them has paid Then, 47% of x − 43% of x + 60 = 308
= 126800  1 +
15   15  = 100 + 100 + 120 = ` 320 ⇒ 4% of x = 308 − 60
1− 
 100   100  4
41. (b) (Pipe) ⇒ x = 248
115 85 100
= 126800 × × = 123947
248 × 100
100 100
70% 30% ∴ x= = 6200
4
36. (b) Given, R = 15% and R = 20%
1 2 ∴ Increase in percentage of longer piece 48. (c) Let the original salary be x.
∴ Required population compared to shorter piece
Then, increased salary
= P  1 +
15   20  70 − 30
 1+  = × 100% [Rule 4]
= 
   100  110  11x
100 30 x= `
 100  10
= 126800     = 174984
23 6 40 400
 20   5  = × 100% = % Q He received the same salary even after
30 3
increment.
37. (c) Refer to question 21. 42. (c) Refer to question 6.
Amount of salary he did not receive
38. (b) Refer to question 34. 43. (d) Average price increase 11x x
= − x =`
5 + 8 + 77 
= 
39. (a) 5% of 50% of 500 90 10 10
% = % = 30%
=
5
×
50
× 500 = 12.5 [Rule 1]  3  3 ∴ Amount of salary in percentage
100 100  x 
44. (a) Refer to question 23.
40. (d) Given, the cost of meal of Y = ` 100  
45. (b) Refer to question 2. = 10 × 100 %
Now, according to the question,  11x 
 
46. (d) Refer to question 18.  10 
Cost of the meal of Z = 20% more than
that of Y x 100
47. (b) Let the number of voters on the voter = × 100% = %
=  100 + × 100
20 list be x. 11x 11
 100  Total cast vote = 90% of x − 60 49. (b) Refer to question 31.
09
SIMPLE INTEREST
Regularly (1-2) questions have been asked from this chapter. Generally direct formula based
questions are asked from this chapter and hence making it easy to score area.

INTEREST
When money is borrowed by a person, then customarily the money lender used to charge some extra
money in lieu of the money lent by him. This extra money earned by the money lender is called
interest.
Some terms related to interest are given below
Principal (P) The money which is borrowed from a money lender, is called Principal.
Amount (A) The sum of the principal and the interest is called Amount i.e. Amount (A) = Principal ( P ) +
Interest ( I ) .
Rate (R) It is the rate at which the interest is charged on Principal.
Time (T) The time period, for which the money is lent or deposited or borrowed, is called Time.

SIMPLE INTEREST
If the interest is calculated on the original Principal for any length of time, then it is called simple
interest.
Principal × Rate × Time P × R×T
Simple Interest (SI) = or SI =
100 100
100 × A
Amount ( A ) = Principal ( P ) + Interest ( I ) and P =
100 + RT
Where, A → Amount
R → Rate of simple Interest
T → Time

EXAMPLE 1. The sum required to earn a monthly interest of `400 at 10% per annum at simple
interest is
a. ` 2000 b. ` 12000 c. ` 24000 d. ` 48000
78 CDS Pathfinder

1
Sol. d. Given, SI = ` 400, R = 10% and T = 1 month = yr Rule 2 If a certain Principal Amounts to ` A1 in t 1 yr
12
and to ` A 2 in t 2 yr, then the sum ( P ) is given by
P ×R ×T 100 × SI 100 × 400 × 12
SI = or P = = = ` 48000  A t − A1 t 2 
100 R ×T 10 × 1 ` 2 1  and the rate per cent ( R) per annum is
 t1 − t 2 
EXAMPLE 2. In what time will the simple interest on 100 ( A 2 − A1 )
` 400 at 10% per annum be the same as the simple given by  %.
interest on ` 1000 for 4 yr at 4% per annum?  A1 t 2 − A 2 t 1 
a. 2 yr b. 3 yr c. 4 yr d. 6 yr
EXAMPLE 5. A certain sum amounts to ` 1586 in
Sol. c. Here, P = `1000, T = 4 yr, and R = 4% 2 yr and ` 1729 in 3 yr. Find the rate and the sum.
P × R × T 1000 × 4 × 4 a. 8%, ` 1200
∴ Simple interest = = = ` 160
100 100 b. 9%, ` 1300
Now, simple interest = ` 160, P = ` 400, R = 10%, c. 10%, ` 1000
100 × SI 100 × 160 d. 11%, ` 1300
then, T = = = 4 yr
P ×R 400 × 10 Sol. d. Here, A1 = ` 1586, t1 = 2 yr, A2 = 1729 and t 2 = 3 yr
A2t1 − A1t 2 1729 × 2 − 1586 × 3
EXAMPLE 3. A sum at simple interest of 4% per ∴ Required principle = =
annum amounts to ` 3120 in 5 yr. Find the sum. t1 − t 2 2− 3
a. ` 2500 b. ` 1300 c. ` 4000 d. ` 2600 = ` 1300
( A − A1) × 100 
Sol. d. Here, T = 5 yr, R = 4%, A = ` 3120 and required rate R =  2 %
100 × A 100 × 3120 100 × 3120  A1t 2 − A2t1 
We know that, P = = = (1729 − 1586) × 100 143 × 100
100 + RT 100 + 4 × 5 120 = = = 11%
= ` 2600 1586 × 3 − 1729 × 2 1300
Rule 3 At the same rate of simple interest, if a sum of
Important Rules and Formulae money becomes n1 times of itself in t 1 yr and n2 times in
Rule 1 If a sum of money becomes n times in ‘T ’ yr at ( n − 1)
t 2 years, then t 2 = 2 t 1 yr.
simple interest, then rate of interest will be, ( n1 − 1)
 100( n − 1)
R=  % EXAMPLE 6. A sum of money becomes 3 times in 5 yr
 T 
at simple interest. In how many years, will the same sum
become 6 times at the same rate of simple interest?
EXAMPLE 4. At what rate per cent per annum will a
sum of money double in 8 yr? a. 10 yr
1 b. 12 yr
a. 12% b. 12 % c. 12.5 yr
2
c. 13% d. 15% d. 10.5 yr
Sol. b. Here, n = 2 and T = 8 yr Sol. c. Here, n1 = 3, t1 = 5yr and n2 = 6, t 2 = ?
100(n − 1) 100 × ( 2 − 1) 100 1 (n2 − 1) (6 − 1) 25
∴ Rate = = = = 12 % ∴ Required time (t 2) = t1 ×5= = 12.5 yr
T 8 8 2 (n1 − 1) ( 3 − 1) 2

PRACTICE EXERCISE
1. Find the amount on a sum of ` 400 for 3 yr at 3. If a certain sum is doubled in 8 yr on simple
simple interest at 5% per annum. interest, in how many years will it be four times?
(a) ` 460 (b) ` 415 (c) ` 435 (d) ` 412 (a) 24 yr (b) 16 yr (c) 32 yr (d) 12 yr
2. Find what sum of money will amount to ` 900 in 4. A sum of money at simple interest amount to
4 yr at 5% per annum on simple interest? ` 1260 in 2 yr and `1350 in 5 yr, then the rate
(a) ` 750 (b) ` 650 per cent per annum is
(c) ` 500 (d) ` 550 (a) 30% (b) 10% (c) 2.5% (d) 5%
MATHEMATICS Simple Interest 79

the same rate of interest. Which of the following


5. The difference of 13% per annum and 12% of a
statement is true in this regard?
sum in 1 yr is ` 110. Then, the sum is
(a) A will get twice the amount of interest that B would get
(a) ` 12000 (b) ` 13000 (c) ` 11000 (d) ` 16000
(b) B will get twice the amount of interest that A would get.
6. The simple interest on a sum of money at 10% per (c) A and B will get the same amount as interest.
annum for 6 yr is half the sum. Then, the sum is (d) B will get four times the amount of interest that A
would get
(a) ` 5000 (b) not possible (c) ` 4000 (d) ` 6000
18. Out of a sum of ` 625 a part was lent at 5% and
7. The sum which amounts to ` 840 in 5 yr at the the other at 10% simple interest. If the interest
rate of 8% per annum simple interest is on the first part after 2 yr is equal to the
100 × 8 × 5 + 100 100 + 840 interest on the second part after 4 yr, then the
(a) ` (b) `
840 100 + 5 × 8 second sum (in `) is
100 × 840 840 × 5 × 8 (a) ` 125 (b) ` 200 (c) ` 250 (d) ` 300
(c) ` (d) `
100 + 5 × 8 100
19. A man invests an amount of ` 15860 in the
8. A certain sum at simple interest amounts to names of his three sons A, B and C in such a way
` 1040 in 3 yr and to ` 1360 in 7 yr. Then, the sum is that they get the same interest after 2, 3 and 4
(a) ` 750 (b) ` 800 (c) ` 900 (d) ` 1000 yr, respectively. If the rate of simple interest is
9. A sum becomes 6 fold at 5% per annum. At what 5%, then the ratio of the amounts invested
rate, the sum becomes 12 fold? among A, B and C will be
1 1 1
(a) 10% (b) 12% (c) 9% (d) 11% (a) 10 : 15 : 20 (b) : :
10 15 20
10. A man borrowed ` 40000 at 8% simple interest (c) 110 : 115 : 120 (d)
1
:
1
:
1
per year. At the end of second year he paid back 110 115 120
certain amount and at the end of fifth year he
paid back ` 35960 and cleared the debt. What is
20. A sum was put at simple interest at a certain
rate for 2 yr. Had it been put at 3% higher rate,
the amount did he pay back after the second year?
it would have fetched ` 72 more. The sum is
(a) ` 16200 (b) ` 17400
(a) ` 1200 (b) ` 1600 (c) ` 1900 (d) ` 1400
(c) ` 18600 (d) None of these
11. A sum of ` 1550 was lent partly at 5% and partly 21. Harsha makes a fixed deposit of ` 20000 in Bank
at 8% simple interest. The total interest received of India for a period of 3 yr. If the rate of
after 3 yr was ` 300. The ratio of money lent at interest be 13% SI per annum charged half
5% to 8% is yearly, what amount will he get after 42 months?
(a) 11 : 12 (b) 16 : 15 (c) 12 : 21 (d) 11 : 13 (a) ` 27800 (b) ` 28100 (c) ` 29100 (d) ` 30000

12. Rahim buys a house and pays ` 8000 cash 22. A sum of money lent on simple interest triples
and ` 9600 at 5 yr credit at 4% per annum simple itself in 15 yr and 6 months. In how many year
interest. Then, the cash price of the house still it be doubled?
(a) ` 10000 (b) ` 9600 (c) ` 17000 (d) ` 16000 (a) 6 yr and 3 months (b) 7 yr and 9 months
(c) 8 yr and 3 months (d) 9 yr and 6 months
13. At what rate per cent per annum simple interest,
will a sum of money triple itself in 25 yr? 23. Mr Pawan invests an amount of ` 24200 at the
(a) 8% (b) 9% (c) 10% (d) 12% rate of 4% per annum for 6 yr to obtain a simple
interest, later he invests the principal amount as
14. A man invested ` 1000 on a simple interest at a well as the amount obtained as simple interest
certain rate and ` 1500 at 2% higher rate. The
for another 4 yr at the same rate of interest.
total interest in 3 yr is ` 390. What is the rate of
What amount of simple interest will be obtained
interest for ` 1000?
at the end of the last 4 yr?
(a) 4% (b) 5% (c) 6% (d) 8%
(a) ` 4800 (b) ` 4850.32 (c) ` 4801.28 (d) ` 4700
15. In what time the simple interest on a sum of 24. A person invested some amount at the rate of
money be 3 / 8 of the principal with rate of 12% simple interest and the remaining at 10%.
interest 3(1 / 8)%? He received yearly an interest of ` 130. Had he
(a) 9 yr (b) 6 yr (c) 12 yr (d) 15 yr
interchanged the amounts invested, he would
16. If the rate of simple interest is 12% per annum have received an interest of ` 134. How much
the amount that would fetch interest of ` 6000 money did he invest at different rates?
per annum is (a) ` 500 at the rate of 10%, ` 800 at the rate of 12%
(a) ` 7200 (b) ` 72000 (c) ` 50000 (d) ` 48543.69 (b) ` 700 at the rate of 10%, ` 600 at the rate of 12%
17. A lends a sum of money for 10 yr at 5% simple (c) ` 800 at the rate of 10%, ` 400 at the rate of 12%
interest, B lends double that amount for 5 yr at (d) ` 700 at the rate of 10%, ` 500 at the rate of 12%
80 CDS Pathfinder

Directions (Q. Nos. 25-27) If SI for a certain sum P1 PREVIOUS YEARS’ QUESTIONS
for time T1 and rate of interest R1 is I1 and SI for
another sum P2 for time T2 and rate of interest R2 is 28. The principal on which a simple interest of ` 55
P R T − PR
1 1T1 will be obtained after 9 months at the rate of
I2, then difference of SI = I2 − I1 = 2 2 2 .
100 3( 2 / 3)% per annum is e 2013 I
(a) ` 1000 (b) ` 1500
25. Simple interest for the sum of ` 1500 is ` 50 in
(c) ` 2000 (d) ` 2500
4 yr and ` 80 in 8 yr, the rate of SI is
(a) 0.6% (b) 5% (c) 0.05% (d) 0.5% 29. In how much time would the simple interest on a
principal amount be 0.125 times the principal
26. Simple interest for the sum of ` 1230 for 2 yr is amount at 10% per annum? e 2015 I
` 10 more than the simple interest for ` 1130 for
1 3 1 3
the same duration. Find the rate of interest. (a) 1 yr (b) 1 yr (c) 2 yr (d) 2 yr
4 4 4 4
(a) 5% (b) 6% (c) 8% (d) 2%
30. If a sum of money at a certain rate of simple
27. If the annual payment oa ` A will discharge a interest per year doubles in 5 yr and at a
debt of ` 1092 due in 2 yr at 12% simple interest, different rate of simple interest per year becomes
then three times in 12 yr, then the difference in
I. A will be ` 515 (approx.). the two rates of simple interest per year is
II. A will be ` 530 (approx.), if interest rate is 6%. e 2016 I
(a) Only I (b) Both I and II 1 1
(a) 2% (b) 3% (c) 3 % (d) 4 %
(c) Only II (d) Neither I nor II 3 3

ANSWERS
1 2 a 3 a 4 c 5 c 6 b 7 c 8 b 9 d 10 b
11 b 12 d 13 a 14 a 15 c 16 c 17 c 18 a 19 b 20 a
21 c 22 b 23 c 24 d 25 d 26 a 27 b 28 c 29 a 30 c

HINTS AND SOLUTIONS


1. (a) Given, P = ` 400, R = 5% Q Sum 4. (c) Simple interest in 3 yr
and T = 3 yr 100 × A 100 × 900 = ` ( 1350 − 1260) = ` 90
= = = ` 750
P × R×T 100 + RT 100 + 5 × 4
Simple interest = ∴ Simple interest for 2 yr
100 3. (a) Let the sum be ` x, so amount 2
400 × 3 × 5 = × 90 = ` 60
SI = = ` 60 = ` 2x 3
100 ∴ SI = ` x
Q Principal = ` ( 1260 − 60) = ` 1200
∴Amount = P + SI = 400 + 60 = ` 460 Let R be rate of interest.
100 × SI 100 × x ∴ Rate,
2. (a) Let the sum of money be ` x. ∴ R= = = 125
. % 100 × SI 100 × 60 60 5
P ×T x ×8 R= = = =
Q Amount = Sum + SI Now, the needed amount = ` 4x P ×T 1200 × 2 24 2
x ×5 × 4
∴ Amount = x + ∴ SI = ` ( 4x − x) = ` 3x R = 25
. %
100
100 × SI 100 × 3x Shortcut Method
Q SI = P × R × T  ∴T= = = 24 yr
 P×R x × 125
. Here, A = ` 1260, t = 2 yr,
100  1 1
But amount = ` 900 Shortcut Method A = ` 1350 and t = 5yr
2 2
Here, n = 2, n = 4 and t = 8 yr
20x 1 2 1 ∴ Required rate
∴ 900 = x + ( n − 1)
100 ∴ Required time = 2 ×t  (A − A ) 
6x 900 × 5 ( n − 1) 1 = 2 1
 × 100% [Rule 2]
⇒ 900 = ⇒x= = ` 750 1  A1 t 2 − A2 t1 
5 6 [Rule 3]
Shortcut Method ( 4 − 1) 3 ( 1350 − 1260)
= × 8 = × 8 = 24 yr = × 100% = 25
. %
Here, A = ` 900, R = 5% and T = 4yr ( 2 − 1) 1 1260 × 5 − 1350 × 2
MATHEMATICS Simple Interest 81

5. (c) Let the sum be ` x. Shortcut Method 13. (a) Let the principal be ` P
x × 13 × 1 x × 12 × 1 As, R = 5%, n = 6, n = 12. As, amount = 3P and T = 25 yr
Then, − = 110 1 1 2
100 100 According to the formula, ∴ SI = 3P − P = 2P
x
⇒ = 110 n −1 100 × SI
100 12 − 1 Q Rate =
R = 2 ×R = × 5 [Rule 3] principal × T
⇒ x = 110 × 100 = ` 11000 2
n −1 1
6−1
1 100 × 2P
Shortcut Method 11 = = 8%
= × 5 = 11% P × 25
Here, P = ` 110 , r = 13%, r = 12% 5
1 2
and t = t = 1yr 14. (a) Let a man invest ` 1000 at a R%.
1 2 10. (b) Total borrowed money = ` 40000
∴ Short trick Now, rate is increased by 2%.
and rate of interest = 8%
110 × 100 ∴ New rate = ( R + 2)%
= 40000 × 8 × 2
13 − 12 The interest for 2 yr = By given condition,
100
= ` 11000 1000 × R × 3 1500 × ( R + 2) × 3
= ` 6400 + = 390
6. (b) Let the sum (principal) be ` x. 100 100
x Let he paid ` x at the end of second
∴ Simple interest = ` year. ⇒ 30R + 45R + 90 = 390
2 ⇒ 75R = 300 ⇒ R = 4%
and T = 6 yr, R = 10% per annum ∴ Interest will be calculated on
1 25
P × R×T ` ( 40000 − x + 6400) 15. (c) Here, rate of interest = 3 % = %
∴ SI = 8 8
100 ( 46400 − x) × 3 × 8
Interest for 3 yr = Let principal be ` x.
x x × 10 × 6 1 6 100 3
⇒ = ⇒ = and simple interest = ` x
2 100 2 10 6
= ` ( 46400 − x) 8
Which is not true, so it is not a possible 25 25
x× × T
case. 6 3 8
∴ ( 46400 − x) + 46400 − x = 35960 ∴ x=
7. (c) Let the sum be ` 100. 25 8 100
300
21576 × 25 ⇒ = T ⇒ T = 12 yr
Then, amount = (Sum + SI) ⇒ x= = ` 17400 25
100 × 8 × 5  31
=  100 +  = ( 100 + 8 × 5) 16. (c) Given, rate of interest = 12% per
 100  11. (b) Let sum lent at rate 5% be ` x. annum
So, when the amount is ( 100 + 8 × 5), Then, sum lent at rate 8% Simple interest = ` 6000 per annum
then sum = 100
= ` ( 1550 − x) Let principal is ` P.
When the amount is ` 840, then sum x ×5 ×3 P × 1 × 12
100 × 840  ∴ Simple interest at rate 5% = ∴ 6000 =
= `   100 100
 100 + 8 × 5  6000 × 100
Simple interest at rate 8% Q P= = ` 50000
8. (b) Simple interest for 4 yr ( 1550 − x) × 8 × 3 12
=
= ` ( 1360 − 1040) = ` 320 100 Hence, the required Principal amount is
Simple interest for 3 yr x × 15 ( 1550 − x) × 24 ` 50000.
∴ + = 300
= `  × 3 = ` 240
320 17. (c) For A Let the amount be ` x.
100 100
 4  15x 37200 24x Rate of interest = 5% and Time = 10 yr
⇒ + − = 300 x × 5 × 10 x
∴ Sum or principle 100 100 100 ∴ Simple interest = =
= ` ( 1040 − 240) = ` 800 100 2
⇒ x = ` 800
Shortcut Method Amount lent at rate 8% For B The amount be ` 2x.
Here, A = ` 1040 , t = 3 yr, = ` ( 1550 − 800) = ` 750 Rate of interest = 5% and Time = 5 yr
1 1
2x × 5 × 5 x
A = ` 1360 and t = 7yr ∴ Required ratio =
800 16
= = 16 : 15 SI = =
2 2
100 2
∴ Required principle (sum) 750 15
So, A and B both will get the same
A t −At  12. (d) Let the amount remaining to pay be amount as interest.
= ` 2 1 1 2 [Rule 2]
 t −t  ` x.
 1 2  18. (a) Let the first part and the second part
∴ Price of house = ` ( x + 8000) be ` x and ` ( 625 − x), respectively.
1360 × 3 − 1040 × 7 x × 4 ×5
= = `800 ⇒ 9600 − =x Q The simple interest on both part in
3−7 100 same.
9. (d) SI at 5% = 6 P − P = 5P x
⇒ 9600 − = x x × 5 × 2 ( 625 − x) × 10 × 4
P ×5 ×T 5 So, =
∴ 5P = ⇒ T = 100 yr 6x 9600 × 5 100 100
100 ⇒ 9600 = ⇒ =x ⇒ 10x = ( 625 − x)40
Now, for new rate ( R), 5 6
⇒ x = ( 625 − x)4
P × R × 100 ⇒ x = ` 8000
11P = ∴ x = ` 500
100 ∴ Cash price of the house ∴ Second part
∴ R = 11% = ` (8000 + 8000) = ` 16000 = ` ( 625 − 500) = ` 125
82 CDS Pathfinder

19. (b) Let the amount of A = ` a, Let amount doubled be t yr. 27. (b) Case I Annual payment
1
time = 2 yr and rate = 5% Now, SI = 2P − P = ` P =
100P
∴ Simple Interest of SI × 100 RT (T − 1)
∴ t = 100T +
a × 2 × 5 10a 1
P×R 2
A= =
100 100 1092 × 100
P × 100 × 31 =
⇒ t = 24( 2 − 1)
Let the amount of B = ` b, rate = 5% 1
P × 400 100 × 2 +
and time = 3 yr. 31 2
∴ Simple interest of = = 7 yr and 9 months 1092 × 100
b × 3 × 5 15b
4 = = ` 515.09 ≈ ` 515
B= = P × R×T
212
100 100 23. (c) Case I SI = Case II Annual payment
Let the amount of C = ` c, time = 4 yr 100
24200 × 4 × 6 1092 × 100
and rate = 5% R= = ` 5808 =
12( 2 − 1)
c × 4 × 5 20c 100 100 × 2 +
∴Simple interest of C = = 2
100 100 ∴ Amount = Principal + SI
a × 10 b × 15 c × 20 1092 × 100
But = = = = ` 530.09 ≈ ` 530
SI = 24200 + 5808 = 30008 206
100 100 100
⇒ 10 a = 15 b = 20 c = k 30008 × 4 × 4 28. (c) Let P be the principal amount.
Case II SI = = ` 4801.28
k k k 100 9
So, a= , b= , c= SI = ` 55, time, t = 9 months = yr
10 15 20 24. (d) Let the person invest amount x and y 12
1 1 1 2 11
∴ a : b :c = : : into two different rates of interest. and rate, r = 3 % = %
10 15 20 3 3
x × 12 × 1 y × 10 × 1
∴ + = 130 P × R×T
20. (a) Let the sum be ` x and the original 100 100 Q SI =
rate r%, then 100
x×r×2 Q SI = PRT  SI × 100 55 × 100
Simple interest =  100  ⇒ P= = × 3 × 12
100 R×T 11 × 9
Now, rate is increased by 3%. ⇒ 12x + 10 y = 13000 …(i)
∴ New rate = ( r + 3)% y × 12 × 1 x × 10 × 1 = 5 × 100 × 4 = ` 2000
x × ( r + 3) × 2
and + = 134
Q Simple interest = 100 100 ∴ Principal ( P ) = ` 2000
100 ⇒ 12 y + 10x = 13400 …(ii) 29. (a) Let the principal be ` x and the time
x × ( r + 3) × 2 x × r × 2
∴ − = 72 be t yr.
100 100 On solving Eqs. (i) and (ii), we get
Rate = 10%
( xr + 3x)2 2xr
⇒ − = 72 x = ` 500 and y = ` 700 P × R×T
100 100 Q Simple interest =
25. (d) Here, I = 50, I = 80, T = 4 yr, 100
2xr + 6x − 2xr
⇒ = 72 1 2 1
According to the question,
100 T = 8 yr, and P = ` 1500
∴ x = ` 1200 2 0.125 × Principal = Simple interest
According to formula,
P × 10 × T
21. (c) As rate of interest is charged half P × R × (T − T ) ∴ 0.125 P =
yearly, I −I = 2 1 100
2 1
13 100 125P 10 × P × T
So, rate = % half yearly 1500 × R × (8 − 4) ⇒ =
2 ⇒ 80 − 50 = 1000 100
100
time  × 2 half yearly = 7 half yearly
42 125
⇒ =T
 12  ⇒ 30 = 15 × R × 4 100
20000 × 13 × 7 ⇒ 2R = 1 5 1
SI = = ` 9100 ⇒ T = = 1 yr
100 × 2 ⇒
1
R = = 0.5% 4 4
∴ Amount ( A ) = 20000 + 9100 2 30. (c) Let principal = ` P, then amount of
= ` 29100 26. (a) Here, I − I = ` 10, P = ` 1230, money = ` 2P
1 2 1
22. (b) Let initial amount be ` P, then P = ` 1130 and T = 2 yr ∴ SI = 2P − P = `P
A = ` 3P and 2
P × r ×5
31 According to formula, Now, P= ⇒ r = 20%
T = 15 yr and 6 months = yr 100
2 T × R × (P − P )
I −I = 2 1 Amount of money after 12 yr = `3P
Q SI = A − P = 3P − P = ` 2P 2 1
100
∴ SI = 3P − P = 2P
⇒ P×
31
×
R
= 2P R × 2( 1130 − 1230)
−10 = P × R × 12 50
2 100 100 Now, 2P = ⇒ R= %
2 × 2 × 100 −200R 100 3
⇒ R= % −10 =
31 100 ∴ Difference between two interest rates
=  20 −  % =
400 50 10 1
= % ∴ R = 5% %=3 %
31  3 3 3
10
83

COMPOUND INTEREST
Usually (1-2) questions have been asked from this chapter. Questions from this chapter are based on
the direct application of compound interest formula.

COMPOUND INTEREST
When the interest is calculated on the amount of previous year, then it is known as compound interest.
It will make a deposit or loan grow at a faster rate than simple interest.

Important Rules and Formulae


Consider P = Principal (amount borrowed) , A = Amount (Principal + Interest)
R = Rate of interest per annum and n = Number of years
Rule 1 When interest is compounded annually, then
 R
n  R
n 
(i) Amount = P 1 +  (ii) CI = A – P = P 1 +  − 1
 100   100  

EXAMPLE 1. The amount and the compound interest on ` 24000 compounded annually for 2 yr at
the rate of 10% per annum is
a. ` 39040 and ` 4040 b. ` 29040 and ` 5040 c. ` 19040 and ` 3040 d. None of these
Sol. b. Here, P = ` 24000, R = 10% per annum and n = 2 yr
R n 10  2 110 2
Q A = P 1 +  = 24000 1 +  = 24000 
110 110
 = 24000 × × = ` 29040
 100  100  100 100 100
Q Compound interest = Amount − Principal = ` 29040 − ` 24000 = ` 5040
So, amount = ` 29040 and compound interest = ` 5040
Rule 2 When interest is compounded half-yearly (every 6 months), then
 r 
2n  R 
2n 
(i) Amount = P 1 +  (ii) CI = 1 +  − 1
 100 × 2  100 × 2 
84 CDS Pathfinder

EXAMPLE 2. The compound interest on ` 24000


1 Rule 4 When interest is compounded annually but time
compounded semi-annually for 1 yr at the rate of a
2 is given in fraction (say, t yr), then
b
10% per annum is t
 R  1 + ( a/b)R
a. ` 3783 b. ` 2783 c. ` 2763 d. ` 3763 Amount = P 1 +  × 
 100   100 
Sol. a. Given, P = ` 24000
Rate of interest = 10% per annum EXAMPLE 4. The amount and compound interest on
and
1
time = 1 yr = yr
3 ` 5000 compounded annually for 2 yr 6 months at the
2 2 rate of 10% per annum is
Since, the interest is compounded half-yearly. a. ` 6252.5 and ` 1342.5 b. ` 6000 and ` 1300
10
Then, rate (R) = % half-yearly c. ` 6250 and ` 1340 d. ` 6352.5 and ` 1352.5
2
3 Sol. d. Here, time = 2 yr and 6 months
and time (n) = 2 × = 3 half-yearly a 1
2 ∴ t = 2 yr and = yr
 b 2
 5 3  R  n  1+ a R 
∴ A = 24000 1 +  Q A = P 1+ 100 
 100 R t 
  
Amount = P 1+  × b 
 100  100 
Q
 1 3  21 3  
= 24000 1 +  = 24000   [Rule 1]  
 20  20
21 21 21  1
⋅ 10
= 24000 × × × = ` 27783  10  2  2
20 20 20 ∴ A = 5000 1 +  1 + 
 100  100 
Now, compound interest = A − P  
= ` 27783 − ` 24000 = ` 3783
110 2  105
Rule 3 When interest is compounded quarterly = 5000    
 100  100
(every 3 months), then 11 11 21
= 5000 × × × = ` 6352.50
4n 10 10 20
 R 
(i) Amount = P 1 +  ∴ Compound interest = A − P
 100 × 4 CI = 6352.50 − 5000 = ` 1352.50
 R 
4n 
(ii) CI = P 1 +  − 1 Rule 5 When rate of interest for n1 , n2 and n3 yr are
 100 × 4 
R1 , R2 and R3 respectively, then
n1 n2 n3
EXAMPLE 3. The amount and the compound  R   R2   R3 
Amount = P 1 + 1  1 +  1 + 
interest on ` 100000 compounded quarterly for 9  100   100   100 
months at the rate of 4% per annum is
a. ` 106060 and ` 6060.10 b. ` 103030 and ` 3030.10 EXAMPLE 5. The compound interest on ` 5000 for
c. ` 103030.10 and ` 3030.10 d. ` 106060.10 and ` 6060.10 4 yr if the rate of interest is 10% per annum for the
Sol. c. Given, P = ` 100000, rate of interest = 4% per annum first two years and 15% for the next two years is
and time = 9 months a. ` 3000 b. ` 3001 c. ` 3002 d. None of these
Since, the interest is compounded quarterly. Sol. b. Here, R1 = 10%, n = n 1 + n2 = 4 yr
4 n 1 = 2 yr, R2 = 15% , n2 = 2 yr
Then, rate (R) = = 1% quarterly
4
and principal = ` 5000
9
and time (n) = 9 months = × 4 = 3 quarter yearly R n1 R n2
12 Q Amount = P 1 + 1  1+ 2 
n
 100  100
Q Amount, A = P 1+
R 
 10  2  15  2
 100 ∴ A = 5000 1 +  1 + 
 100  100
1 3 101 3
∴ A = 100000 1 +  = 100000   110 2  115 2
 100  100
= 5000 ×    
 100  100
101 101 101 11 11 23 23
= 100000 × × × = ` 103030.10 = 5000 × × × × = ` 8001.125
100 100 100 10 10 20 20
∴Compound interest = A − P = ` 103030.10 − ` 100000 ∴ Compound interest
= ` 3030.10 = A − P = ` 8001125
. − ` 5000 ~
− ` 3001
MATHEMATICS Compound Interest 85

Rule 6 The difference D between simple and compound EXAMPLE 7. A sum of money on compound interest
amount to ` 9680 in 2 yr and to ` 10648 in 3 yr. What
interest accrued on ` P at the rate of interest of R% is is the rate of interest per annum?
given by a. 5% b. 10% c. 15% d. 20%
PR2
(i) For 2 yr, D = Sol. b. Given, A1 = ` 10648 and A2 = ` 9680
(100 )2
( A2 − A1)
PR2 ( 300 + r ) ∴ Rate of compound interest = × 100%
(ii) For 3 yr, D = A1
(100 )3 10648 − 9680
= × 100% = 10%
EXAMPLE 6. The difference between the compound 9680
interest and the simple interest on a certain sum at
12% per annum for 2 yr is ` 90. Then, sum is DEPRECIATION
a. ` 6250 b. ` 6350 c. ` 6520 d. ` 6950
The value of certain things like the machine, vehicle etc.,
Sol. a. Let the sum be ` 100. decreases over a period of time. The decrement in the
Case I Then, simple interest on ` 100 for 2 yr value things is called depreciation. The depreciation per
unit time is called the rate of depreciation.
 100 × 12 × 2
=   = ` 24 Rule 8 If the present value of a article is P and it
 100 
Case II Amount when ` 100 is borrowed for 2 yr depreciate at the rate of R% per annum. Then,
12  2  1+ R  ,Rule 1 (i) 
n n
= 100 × 1 +   A = P 

(i) Value of article after n yr = P × 1 −
R 

 100  100
   100 
= 100 ×
28 28
× 
=` 
3136
 P
25 25  25  (ii) Value of article n yr ago = n
 R 
∴ Compound interest for 2yr =  − 100 = ` 
3136 636 −
  1 
 25   25   100 
Difference between CI and SI =  − 24 = `
636 36
 25  25 EXAMPLE 8. Given that carbon − 14 (C 14 ) decays at a
36 constant rate in such a way that it reduces to 50% in
If the difference between CI and SI is ` , then the 5568 yr. Then, the age of an old wooden piece in
25
sum = ` 100 which the carbon is only 12.5% of the original is
If the difference between CI and SI is ` 1, a. 16000 yr b. 16244 yr c. 16702 yr d. None of these
 100 × 25
then the sum = `   Sol. d. Let the rate of decay be R% and the age of the
 36  wooden piece be n yr.
If the difference between CI and SI is ` 90, then the sum
R  5568
Then, 
50  
⇒ 1 −
R 
 = 1 −   = (0.5)1/ 5568 …(i)
= 100 × × 90 = ` 6250
25
 100  100  100
 36 
12.5  R n R n
Here, the sum is ` 6250. ⇒ = 1 −  ⇒ (0.125) = 1 − 
100  100  100
Shortcut Method
⇒ (0.125)1/ n = 1 −
R 
 ⇒ (0. 5) 3/ n = 1 −
R 
Let the required sum be ` P.  …(ii)
 100  100
Here, difference = ` 90 and R = 12% 1
∴ Difference between CI and SI From Eqs. (i) and (ii), (0.5) 3/ n = (0.5) 5568
PR 2 (P)(12) 2 P × 144
90 = 2 = ⇒ 90 = On comparing both sides, we get
(100) (100) 2 100 × 100 3 1
= ⇒ n = 3 × 5568 = 16704 yr
∴ P = ` 6250 n 5568
Required sum = ` 6250
Rule 7 If a certain sum at compound interest becomes INSTALMENTS
A1 , in n yr and A 2 in ( n + 1) yr, then When a borrower pays the sum in parts, then we say
that he/she is paying in instalments.
(A 2 − A1 )
(i) Rate of compound interest = × 100% x x x
n A1 ∴ P= + +…+
A   R  2 n
(ii) Sum = A1  1  1 +  1 + R  
1 +
R

 A2   100   100   100 
86 CDS Pathfinder

x = value of each instalment  100 (100) 2 (100) 3 


⇒ P =x + 2 + 
(100 + R) (100 + R) (100 + R) 3 
n
 R
Total amount paid in instalments, A = P 1 + 
 100   100   100  2
 100 
3

= 486680   +  +  
n = number of instalments  100 + 15  100 + 15  100 + 15 
EXAMPLE 9. Sapna borrowed some money on  20  20 2  20 3 
= 486680 ×  +  +  
compound interest and returned it in 3 yr in equal  23  23
 23 
annual instalments. If rate of interest is 15 % per
20  20 400
annum and annual instalment is ` 486680, then the = 486680 × 1 + + 
sum borrowedes. 23  23 529

a. 111300 b. 122400 = 1111200


c. 1111200 d. 154320 ∴ Principal borrowed = ` 1111200

Sol. c. Given that, rate of interest, R = 15% pa


SOME IMPORTANT RESULTS
Annual instalment, x = ` 486680
Total number of instalments, n = 3 1. Simple interest and compound interest are equal for first
year period on the same sum and at the same rate.
 
 x x x  2. If A and B are the amounts of a certain sum for two
∴ P= + 2 + 3
consecutive years, then simple interest for 1 yr = B − A
 1 + R  1 + R  1 + R   3. If certain sum at compound interest becomes x times in
 100  100  100 
 n 1 yr and y times in n 2 yr, then x 1 /n 1 = y 1 /n 2
MATHEMATICS Compound Interest 87

PRACTICE EXERCISE
1. Kiran purchased a scooter for ` 24000. The value 11. A saving bank gives interest which compounds
of scooter is depreciating at the rate of 5% per annually. Raju deposited ` 100 and received
annum. Then, its value after 3 yr is ` 121 at the end of second year. Rate of
(a) ` 20577 (b) ` 20977 (c) ` 20677 (d) ` 20877 compound interest per annum is
(a) 10% (b) 15% (c) 11.5% (d) 20.5%
2. If P be the principal amount and the rate of
interest be r% per annum and the compound 12. The amount of a certain sum at compound
interest is calculated k times in a year, then interest for 4 yr at 10% in ` 4410. The sum is
what is the amount at the end of n yr? (a) 3012.08 (b) 3015 (c) 3020.16 (d) 3016.9
nk nk
(a) P  1 +
r 
(b) P  1 +
kr 
  13. The compound interest on ` 5000 for 3 yr at 8%
 100k   100  for first year, 10% for second year and 12% for
n/ k n/ k
(c) P  1 +
kr 
(d) P  1 +
kr  third year will be
 
 100   100k  (a) ` 1560.40 (b) ` 1500 (c) ` 1565.60 (d) ` 1652.80

3. The amount of a certain sum at compound 14. An amount of ` x at compound interest at 20%
interest for 2 yr at 5% is ` 4410. The sum is per annum for 3 yr becomes y. What is y : x?
(a) ` 4000 (b) ` 4200 (c) ` 3900 (d) ` 3800 (a) 3 : 1 (b) 36 : 25
(c) 216 : 125 (d) 125 : 216
4. A person borrowed ` 7500 at 16% compound
interest. How much does he have to pay at the 15. The compound interest on ` 2000 for 1 yr at the
end of 2 yr to clear the loan? rate of 8% per annum, when the interest is
(a) ` 9900 (b) ` 10092 (c) ` 11000 (d) ` 11052 compounded semi-annually
(a) ` 163.20 (b) ` 2163.20 (c) ` 3153.20 (d) ` 1163
5. If the rate of interest is 10% per annum and
is compounded half-yearly, then the principal of 16. ` 16000 invested at 10% per annum compounded
` 400 in 3/2 yr will amount to semi-annually amounts to ` 18522. Then, the
(a) ` 463.00 (b) ` 463.05 period of investment is
(c) ` 463.15 (d) ` 463.20 1 5
(a) 1 yr (b) 3 yr (c) 2 yr (d) yr
2 2
6. At compound interest, if a certain sum of money 25
doubles in n yr, then the amount will be four 17. A sum compounded annually becomes times
fold in 16
(a) 2 n2 yr (b) n2 yr (c) 2n yr (d) 4n yr of itself in 2 yr. Then, the rate of interest per
annum is
7. The simple interest on a certain sum of money (a) 25% (b) 20% (c) 15% (d) 7
1
%
for 3 yr at 8% per annum is half the compound 2
interest on ` 4000 for 2 yr at 10% per annum.
What is the sum placed on simple interest? 18. A sum of ` 3200 invested at 10% per annum
(a) ` 1550 (b) ` 1650 (c) ` 1750 (d) ` 2000
compounded quarterly amounts to ` 3362, then
the time period is
8. What is the least number of complete years in (a) 1
1
yr (b)
1
yr (c) 2 yr (d) 1 yr
which a sum of money at 20% compound interest 2 2
will be more than doubled?
19. A sum amount to ` 9680 in 2 yr and to ` 10648
(a) 7 (b) 6 (c) 5 (d) 4
in 3 yr compounded annually. Then, the sum and
9. The difference between the simple interest and rate of interest, respectively are
the compound interest (compounded annually) on (a) ` 8000, 10% (b) ` 8500, 10%
` 1250 for 2 yr at 8% per annum will be (c) ` 8500, 9% (d) ` 8000, 9%
(a) ` 18 (b) ` 2 (c) ` 8 (d) ` 4
20. If the value of a machine depreciates by 10% of
10. The compound interest on a sum for 2 yr is ` 832 its value at the beginning of the year and its
and the simple interest on the same sum at the present value is estimated as ` 10935, then what
same rate for the same period is ` 800. What is was its value three years back?
the rate of interest? (a) ` 15000 (b) ` 7000
(a) 6% (b) 8% (c) 10% (d) 12% (c) ` 8050 (d) None of these
88 CDS Pathfinder

21. A sum of money doubles itself at compound 26. From the given statements, identify which of the
interest in 15 yr. It will becomes 8 times in following or both are correct and then select the
(a) 30 yr (b) 40 yr appropriate option.
(c) 60 yr (d) 45 yr I. The Present worth of ` 169 due in 2 yr at 4% Pa
compound interest is 156.25.
22. A man borrows ` 4000 at 8% per annum on
compound interest. At the end of every year he II. If the simple interest on a certain sum for 2 yr is
` 120 and compound interest is ` 129, then the
pays ` 1500 as part payment of loan and
rate of interest must be 15%.
interest. How much does he still owe to the bank
after three such annual payments? Which statement(s) is/are correct?
(a) ` 1799 (b) ` 2000 (a) Only II (b) Only I
(c) ` 169.25 (d) None of these (c) Neither I nor II (d) Both I and II

23. A sum of ` 10000 deposited at compound interest


becomes double after 5 yr. After 20 yr, the
PREVIOUS YEARS’ QUESTIONS
amount will be 27. What is the compound interest on ` 1600 at 25%
(a) ` 160000 (b) ` 40000 per annum of 2 yr compound annually? e 2012 II
(c) ` 50000 (d) ` 60000 (a) ` 700 (b) ` 750 (c) ` 800 (d) ` 900
24. Vinod and Karan each invested ` 15000 for 3 yr 28. The difference between compound interest and
at the same rate of interest but Vinod's simple interest for 2 yr at the rate of 10% over
investment is compounded annually while principal amount of ` X is ` 10. What is the
Karan’s investment is charged on simple value of X? e 2014 II
interest. (a) ` 100 (b) ` 1000 (c) ` 500 (d) ` 5000
I. Vinod receive more interest than karan. 29. The difference between compound interest and
II. Data is insufficient to calculate the interest. simple interest at the same rate of interest R per
Which one is correct? cent per annum on an amount of ` 15000 for
2 yr is ` 96. What is the value of R? e 2015 I
(a) Only I (b) Only II
(a) 8% (b) 10%
(c) Neither I nor II (d) Both I and II
(c) 12% (d) Cannot be determined
25. An amount is invested in a bank at compound
rate of interest. The total amount, including 30. There is 60% increase in an amount in 6 yr at
interest, after first and third year is ` 1200 and simple interest. What will be the compound
` 1587, respectively. interest on ` 12000 after 3 yr at the same rate of
interest? e 2015 I
I. The principal amount will be ` 1403. (a) ` 2160 (b) ` 3120 (c) ` 3972 (d) ` 6240
II. The rate of interest is 16%
31. A sum of ` 10000, is deposited for 1 yr at the
Which statement(s) is/are correct? rate of interest 10% compounded half yearly.
(a) Both I and II (b) Only I What will be the interest at the end of one year?
(c) Only II (d) Neither I nor II e 2015 II
(a) ` 1000 (b) ` 1025 (c) ` 1050 (d) ` 1100

ANSWERS
1 a 2 a 3 a 4 b 5 b 6 c 7 c 8 d 9 c 10 b
11 a 12 a 13 d 14 c 15 a 16 a 17 a 18 b 19 a 20 a
21 d 22 c 23 a 24 b 25 d 26 d 27 d 28 b 29 a 30 c
31 b
MATHEMATICS Compound Interest 89

HINTS AND SOLUTIONS 


n
1. (a) Given, P = ` 24000, R = 5% per ∴ x  1 +
R  9. (c) Given, P = ` 1250, R = 8% per
 = 2x
 100  annum and n = 2 yr
annum and n = 3 yr P × R×T
n 1 + R  = 2
n
Q Simple interest =

∴ A = P  1 −
R    …(i) 100
 [Rule 8 (i)]  100 
 100  1250 × 8 × 2
∴ Simple interest =
3 Let it becomes four fold in N yr.
= 24000  1 −
5  100
 N
x  1 +
 100  R  = ` 200
Then,  = 4x
3
 100 
Compound interest
= 24000 × 
95  Q
 1 + R 
N
 100  ⇒ =4  n

= P   1 +
  R 
 100   − 1 [Rule 1. (ii)]
= ` 20577 N
 100  
22 =  1 +
R 
2. (a) Given, principal amount = ` P ⇒ 
2
Q CI = 1250  1 +
8 
r  100   − 1250
Rate of interest, R = % per annum and  100 
k 2n N
⇒  1 +
R 
=  1 +
R  2
= 1250 × 
Time, T = nk   108 
 100   100   − 1250
T  100 
A = P  1 +
R 
Q  [Rule 1. (i)] [from Eq. (i)]
 100  = 1458 − 1250 = ` 208
∴ N = 2n yr
 
nk
∴ Difference in SI and CI
A = P 1 +
r  7. (c) Let the principal amount be ` P.
∴ = 208 − 200 = ` 8
 100 k  1
  By given condition, SI = CI Shortcut Method
2
3. (a) Let the principal be = ` x. P ×8 ×3 1 Here, p = ` 1250 and R = 8%
⇒ =
Given, n = 2 yr, R = 5% , A = ` 4410 100 2 PR2
∴ Required difference =
n
  2 ( 100)2
A = P  1 +
R   10 
Q

 [Rule 1. (i)]  4000  1 +  − 4000
100   100   [Rule 6. (i)]
2 1250 × 64
4410 = x  1 +
5  = = `8
∴  [Rule 1 (ii)]
 100  100 × 100
24P 1 
− 4000
121
2 ⇒ = 4000 × 10. (b) Given, CI = ` 832, SI = ` 800,
⇒ 4410 = x  
21 100 2  100 
 20  n = 2 yr
1
= [ 4840 − 4000] 
n

CI = P  1 +
4410 × 400 R 
⇒ x= = ` 4000 2  − 1
441 24P  100  
⇒ = 420
4. (b) Given, P = ` 7500, R = 16% and 100 [Rule 1. (ii)]
n = 2 yr 420 × 100  2

832 = P  1 +
∴ P= = ` 1750 R 
n ∴  − 1 …(i)
Q A = P  1 +
R  24  100 
 [Rule 1. (i)]  
 100  8. (d) Let the sum of money be ` P. P × R×T
2 Also, SI =
= 7500  1 +
16  ∴ Amount = ` 2P 100

 100  n
P × R× 2
A = P  1 +
R 
2 ⇒  [Rule 1. (i)] ⇒ 800 =
 100 
= 7500 
116  100

 100  n 40000
2P = P  1 +
20  ⇒ P=
29 29 ⇒ 
= 7500 × ×  100  R
25 25 n From Eq. (i),
2P  6 
= 12 × 841 = ` 10092 ⇒ = 
P 5 40000  R2 2R 
832 =  + 
5. (b) Refer to example 2. n R  10000 100 
2 =  
6

6. (c) Let the principal be ` x. 5 40000  R
⇒ 832 =  + 2
∴ Rate = ` R, Amount = ` 2x 100  100 
On putting n = 4, we get
and Time = n yr 4
 6  = 1296 = 2 (approx) ⇒ 832 = 4R + 800
 
n
A = P  1 +
R 
 [Rule 1. (i)] 5 625 32
∴ R= = 8%
Q
 100 
∴ Least number of years = 4 4
90 CDS Pathfinder

n
11. (a) Given, principal (P) = ` 100 and time ( n) = 1 × 2 = 2 half-yearly  
3362 = 3200  1 +
10 
Amount (A) received after 2 yr = ` 121  n
 ⇒
CI = P   1 +
R   4 × 100 
Q  − 1  
Let rate of interest = R% per annum   100   n 2 n
n 3362  410   41   41 
Q A = P  1 +
R  [Rule 1. (ii)] ⇒ =  ⇒  =  
 [Rule 1. (i)]
3200  400   40   40 
 100  2
CI = 2000  1 +
4 
 − 2000 ⇒ n = 2 ⇒ 4t = 2 yr [as n = 4t ]
2
 100 
121 = 100  1 +
R 
∴  2 1
 100  ∴ t = yr
= 2000 ×   − 2000
26
2  25  2
121  100 + R 
⇒ =  = 2163. 20 − 2000 = ` 163.20 19. (a) Given, A = ` 9680, A = ` 10648
100  100  1 2
2 2 16. (a) Given, principal (P) = ` 16000 and n = 2 yr
100 + R 
⇒   = 
11
 Amount received at the end of period ∴ Rate of compound interest
 10   100  A −A
(A) = ` 18522 2 1
11 100 + R = × 100% [Rule 7 (i)]
⇒ = Let time = t yr A
10 100 1
11 × 100 ∴ n = 2t 10648 − 9680
⇒ 100 + R = Rate = 10% per annum i.e 5% half-yearly = × 100% = 10%
10 n
9680
A = P  1 +
R  A 
n
⇒ 100 + R = 110 Q  [Rule 1. (i)]
 100  ∴ Sum = A  1  [Rule 7 (ii)]
∴ R = 110 − 100 = 10 1 
n  A2 
18522 = 16000  1 +
5 
∴ Rate of interest = 10% ∴  2
 100  = 9680 
9680 
12. (a) Refer to example 3. 
n  10648 
⇒ 
18522   21 
 =  2 2
13. (d) Given, P = ` 5000,  16000   20 
= 9680 
110   10 
 = 9680  
R = 8%, R = 10%, R = 12% n 3 n  121   11 
⇒ 
9261  21   21   21 
1 2 3
 =   ⇒  =   9680 × 100
and n = n = n = 1 yr  8000   20   20   20  = = ` 8000
1 2 3
121
Q Amount ⇒ n=3
n n n
n = 2t ⇒ 2t = 3 20. (a) Let the value of machine 3 yr ago be
 R  1 R  2 R  3 Q
= P1 + 1  1+ 2  1+ 3  3 1 ` x.
 100   100   100  ∴ t = yr = 1 yr and given, P = ` 10935, R = 10% and
2 2
[Rule 5] n = 3 yr
17. (a) Let sum/ principal be ` x and the rate n
x = P  1 −
R 
 12    [Rule 8 (i)]
= 5000 ×  1 +
8  10   be R% per annum. Q

 1+ 1+  100 
 100   100   100   ∴ Amount, A =
25
x and n = 2 yr 3
∴ x  1 −
10 
 27 11 28  16  = 10935
=  5000 × × ×  = ` 6652.80  100 
 25 10 25 
n
A = P  1 +
R 
Q  [Rule 1. (i)] 3
x 
 100  90 
∴Compound interest = 665280
. − 5000 ⇒  = 10935
2  100 
x = x  1 +
= ` 1652.80 25 R 
∴  10935 × 10 × 10 × 10
16  100  ∴ x= = ` 15000
14. (c) Given, P = ` x , R = 20% per annum, 9×9×9
n = 3 yr 25  R 
2
⇒ = 1 +  21. (d) Let the sum be ` x and rate be R %
and A=` y 16  100  per annum.
n
A = P  1 +
R  2
 5 = 1+ R 
2
∴ Amount, A = ` 2x
Q  [Rule 1. (i)] ⇒    
 100   4  100  and time, n = 15 yr
3 n
y = x  1 +
20 
A = P  1 +
5  R  R 1 R 
∴  ⇒ = 1 +  ⇒ = Q  [Rule 1. (i)]
 100  4  100  100 4  100 
3 15
y = x   ⇒ x  1 +
R 
6 y 216 ∴ R = 25%
⇒ = Then,  = 2x
5 x 125  100 
18. (b) Here, P = ` 3200,
⇒ y : x = 216 : 125 1 + R 
15
A = ` 3362 ⇒   =2 …(i)
15. (a) Given, principal ( P ) = ` 2000  100 
[since, amount is payable quarterly]
Time = 1 yr 10 Suppose the sum becomes eight times in
∴ R = 10% per annum = % quarterly ‘n’ yr, then
Rate of interest = 8% per annum 4 n
x  1 +
and let time = t R 
Since, the interest is compounded  = 8x [by given condition]
 100 
semi-annually. ⇒ n = 4t
n n
A = P  1 +
R 
⇒  1 +
8 R 
Then, rate ( R) = % = 4 half-yearly  =8= 2
3
Q  [Rule 1. (i)] …(ii)
2  100   100 
MATHEMATICS Compound Interest 91

3
 R  
15 Amount after 3rd yr = ` 1587 27. (d) P = ` 1600, r = 25% and n = 2 yr
[but 23 =   1 +   from Eq. (i)] n
∴ A = P 1+
R 
  100  
3
x  1 +
R  [Rule 1. (i)]
 = ` 1587 …(ii)  100 
n 45  100 
⇒  1 + R  = 23 =  1 + R 
    2
= 1600  1 +
 100   100  On dividing Eq. (ii) by Eq. (i), we get 25 
[on comparing] 2
 100 
 1 + R  = 1587 = 529
∴ n = 45 yr   5 5
 100  1200 400 = 1600 × × = ` 2500
Shortcut Method 4 4
R 23 R 3
1+ = ⇒ =
Since, a sum of money at compound 100 20 100 20 ∴ Compound interest
interest doubles itself in 15 yr. = ` 2500 − `1600 = ` 900
∴ R = 15%
Hence, it will becomes eight times
Put R = 15% in Eq. (i), 28. (b) Refer to example 6.
(23 times) in = 15 × 3 = 45 yr 1200 × 100
x  1 +
15  29. (a) Given, principal = ` 15000
 = ` 1200 ⇒ x =
22. (c) Amount due at the end of 1 yr  100  115
= 4000  1 +
8  ∴ x = ` 1043.478 Rate, R = ?
 = ` 4320
 100  Difference, D = ` 96
26. (d) I. Given, R = 4%, n = 2 yr and
∴ Amount due after 1 yr PR2
A = ` 169, P = ? For 2 yr Difference, D =
= ( 4320 − 1500) = ` 2820 n ( 100)2
A = P  1 +
R 
 [Rule 1. (i)]
Amount due at the end of second year  100  [Rule 6 (i)]
= 2820  1 +
8  15000 × R2
 = ` 3045.60
2
169 = P  1 +
4  ⇒ 96 =
 100   100 × 100
 100 
∴ Amount due after second year 2 ⇒ R2 = 64
⇒ 169 = P  
26
= (3045.60 − 1500) = ` 1545.60  25  ∴ R = 8%
Amount due at the end of third year
169 × 25 × 25 30. (c) ∴ Let the principal be `x.
= 1545.60  1 +
8  P= = ` 156.25
 = ` 1669.25 26 × 26 Then, SI = `
60x
 100 
100
Amount due after the payment of third II. Given, SI = ` 120 , n = 2 yr
Principal × Rate × Time
instalment Q SI =
and CI = ` 129. 100
= ( 1669. 25 − 1500) = ` 169.25 P × R×T 60x x × Rate × 6
SI = ⇒ = [QTime = 6 yr]
23. (a) Let the sum be ` x, then 100 100 100
5 5 P × R× 2 60 = Rate × 6
x  1 +
R   R  120 = ⇒ PR = ` 6000
 = 2x ⇒  1 +  =2
 100   100  100
∴ Rate = 10%
6000
…(i) ∴ P= …(i) Again principal = ` 12000, Time = 3 yr
The amount after 20 yr, R
Time
Amount = Principal  1 +
4  n
 Rate 
CI = P   1 +
 R 
R  
20 5
 − 1 [Rule 1. (ii)] 
x  1 +
R 
 = x   1 +      100 
 100   100 
  100   3
= 12000  1 +
 2
 10 
129 = P   1 +
= 2 x = 16x [from Eq. (i)]
4 R  
 − 1 .  100 
= 16 × 10000 = ` 160000   100   3
= 12000  
11
[put x = ` 10000] 1290000 = P [( 100 + R)2 − 1002 ].
 10 
24. (b) Data is insufficient as the rate of = P [ R2 + R × 200].
11 × 11 × 11
interest is not given to calculate further. 6000 2 = 12000 ×
= [ R + R × 200] 1000
25. (d) Let amount be ` x and rate of R
interest is R % annually. [from Eq. (i)] = 12 × 121 × 11 = ` 15972
1290000 = 6000R + 1200000 ∴ CI = Amount − Principal
According to the questions,
90000
Amount after 1st yr = ` 1200 R= = 15% = 15972 − 12000 = ` 3972
6000
x  1 +
R  31. (b) Refer to example 2.
 = 1200 …(i) Hence, both statement are correct.
 100 
11
92 C D S Pathfinder

PROFIT AND LOSS


Generally (2-4) questions have been asked from this chapter. Generally questions are based on basic
formulae and tricks explained in this chapter which are helpful to solve the problems more easily
and quickly.

Cost Price (CP)


The price, at which an article is bought is called its cost price. All the overhead expenses in the
transection like freight, damage etc., are added to the cost price.

Selling Price (SP)


The price at which an article sold is called its selling price.
Profit (SP>CP) When an article is sold at a price more than its cost price, then profit is earned.
• Profit = Selling price − Cost price
• Loss = Cost price − Selling price
Loss (CP>SP) When an article is sold at a price lower than its cost price, then loss is incurred.
Profit (SP − CP)
• Profit % = × 100% = × 100%
CP CP
Loss (CP − SP)
• Loss % = × 100 % = × 100%
CP CP
Note Profit and loss percentage is always calculated as the percentage of CP unless otherwise specified.

List Price/Marked Price


The price of an article which is displayed on the price tag is known as marked price. It is the normal
price of the thing/products without a discount. Sometimes the shopkeeper increases or decreases the
cost price, then this price is taken as the list price of the article.
MATHEMATICS Profit and Loss 93

Important Rules and Formulae Rule 4 When two different articles are sold at the
Rule 1 If there is a profit of r%, then same selling price, getting gain/loss of x% on the first and
gain/loss of y% on the second, then the overall % gain or
(100 + r ) × CP 100 × SP % loss in the transection is given by
(i) SP = (ii) CP =
100 (100 + r )
 100( x + y) + 2xy 
%. The above expression represent
(100 + x) + (100 + x)
EXAMPLE 1. If cost price of a fan is ` 720. If there is  
2 overall gain or loss accordingly as its sign is positive or
a profit of 16 %. Then, selling price is
3 negative.
a. ` 840 b. ` 940
c. ` 1050 d. None of these When two different articles are sold at the same selling
2
price getting a gain of x% on the first and loss of x% on
Sol. a. Given, r = 16 % and cost price = ` 720 the second, then the overall % loss in the transection is
3 2
given by   %.
x
720 × 100 + 16 
2
CP × (100 + r)  3  10 
∴Selling price (SP) = =
100 100
720 × 100 +
50 Note That in such questions there is always a loss.

 3  = 720 × 350 = ` 840
= EXAMPLE 4. A trader sells two cycles at ` 1188 each
100 300
and gains 10% on the first and loses 12% on the
Rule 2 If there is a loss of r%, then second. What is the profit or loss percent on the
whole?
(100 − r ) 100 × SP
(i) SP = × CP (ii) CP = a. 1% loss b. 1% gain c. No loss no gain d. 3.2% loss
100 (100 − r )
Sol. d. When there is a profit of x% and loss of y%, then
EXAMPLE 2. When Selling price is ` 75 and there is the resultant
Profit/Loss percentage =  x − y −
a loss of 12%. Then, cost price is xy 
%
 100
a. ` 22 b. ` 44
c. ` 55 d. ` 85.22 Here, x = 10% and y = 12
10 × 12
Sol. d. Given, r = 12% and selling price = ` 75 ∴Profit/Loss percentage = 10 − 12 −  % = −3.2%
 100 
100 × SP 100 × 75 100 × 75
∴ Cost price = = = = 85.22
(100 − r) (100 − 12) 88 EXAMPLE 5. Harish sold two scooters, each for
` 24000. If he makes 20% profit on the first and 15%
Rule 3 (i) When there are two successive profits or losses
loss on the second. What is his gain or loss percent in
of x% and y%, then the resultant profit or the transection?
loss percent is given by 20 20 32 32
( ± x)( ± y) a. loss % b. gain % c. loss % d. gain %

± x ± y +  %. 41 41 73 73
 100  Sol. a. Here, x = 20 and y = −15
Note l Take positive for profit and negative for loss.  100( x + y) + 2xy 
∴ Overall gain/loss % =  %
(100 + x) + (100 + y) 
l If the sign of result is positive, then there is a total gain.
l If the sign of result is negative, then there is a total loss.
100( 20 − 15) + 2 × 20 × −15
EXAMPLE 3. By selling a watch for ` 132 a trademan = %
 (100 + 20) + (100 − 15) 
got two successive profits of 10% and 20%, then the
resultant profit percentage is −100 −20
= %= %
205 41
a. 22% b. 30%
c. 32% d. 34% Which represents loss.
Sol. c. Given, the successive profits are 10%, 20%. EXAMPLE 6. Vikram sold two horses for ` 990 each
 20 × 10 gaining 10% on the one and lossing 10% on the
So, resultant profit percentage = 10 + 20 + %
 100  other. Find his gain or loss percent.
= 32% a. 1% gain b. 1% loss c. 4 % gain d. 4 % loss
94 CDS Pathfinder

Sol. b. Here, x = 10 EXAMPLE 9. If the cost price of 18 chairs be equal


x 2 to selling price of 16 chairs. The gain percent is
∴ Overall loss % =   %
 10 a. 12% b. 12.5%
2 c. 14% d. 15.5%
=   % = 1%
10
 10 Sol. b. Given, n = 18, m = 16 [Q n > m]

Rule 5 A person buys two items for ` S. One is sold at  18 − 16 Q n − m × 100
∴ Profit percentage =   × 100
 16   m 
a loss of r% and the other at a gain of R%, if each item
was sold at the same price, then 2 100
= × 100 = = 12.5%
S × (100 + R) 16 8
Cost price of item sold at loss =
(100 − r ) + (100 + R)

Cost price of item sold at gain =


S × (100 − r ) DISCOUNT
(100 − r ) + (100 + R) The reduction allowed on the marked price of an article
is called as discount. Discount is always reckoned on the
EXAMPLE 7. Sudhir bought two boxes for ` 1300. marked price.
He sold one box at a profit of 20% and the other at a
loss of 12%. If the selling price of both boxes is the Selling price = Marked price − Discount
same, then the cost price of each box is Rule 8 If discount allowed is r%, then
a. ` 700 and ` 500 b. ` 750 and ` 550 (100 − r )
c. ` 800 and ` 600 d. None of these Selling price = × Marked price
100
Sol. b. Given, cost of two boxes S = ` 1300, profit on first
box R = 20% EXAMPLE 10. If the marked price of a fan is ` 700
and loss on second box r = 12% and a discount of 10% is given on it, then what is the
Cost price of box sold at loss selling price of the fan?
1300 × (100 + 20) 1300 × 120 a. ` 500 b. ` 575
= = = ` 750 c. ` 610 d. ` 630
(100 − 12) + (100 + 20) 88 + 120
Cost price of box sold at gain Sol. d. Given, r = 10%
1300 × (100 − 12) 1300 × 88 (100 − 10)
= = = ` 550 Selling price = × 700
(100 − 12) + (100 + 20) 88 + 120 100
Rule 6 If a trader professer to sell his goods at cost 90 × 700
= = ` 630
price, but uses false weights, then gain % 100
true weight − false weight Rule 9 If a shopkeeper marks his items at x% above the
= × 100
false weight cost price and allows customers a discount of y% then
 xy 
EXAMPLE 8. A dishonest dealer sell his goods at there is  x − y −  % profit or loss according to
cost price, but he uses a weight of 960 gm for the kg  100 
weight, then the percentage of gain is positive or negative sign, respectively.
1 1 1 1
a. 2 % b. 4 % c. 6 % d. 3 %
3 6 9 2 EXAMPLE 11. A dealer marked his goods 20% above
true weight − false weight the cost price and allows a discount of 10%. Then, his
Sol. b. Gain % = × 100
false weight gain percent is
1000 − 960 40 1 a. 2% b. 4%
= × 100 = × 100 = 4 % c. 6% d. 8%
960 960 6
Rule 7 If cost price of ‘n’ articles is equal to the selling Sol. d. Given, x = 20% and y = 10%
Profit/Loss percentage =  x − y −
price of ‘m’ articles, then xy 
%
 100
 n − m
Profit percentage =   × 100% ( n > m) 20 × 10
 m  =  20 − 10 −  % = 8%
 100 
 m − n
and loss percentage =   × 100% ( m > n)
 m  Hence, positive sign shows profit i.e. profit of 8%.
MATHEMATICS Profit and Loss 95

Successive Discounts Rule 11 If marked price of an item is ` x and the


(Discount Series) successive discount rates are r1 %, r 2 %, r 3 % and so on,
then selling price of the item
Rule 10 Single discount equivalent to three successive
 r  r  r 
discounts r1 %, r 2 % and r 3 % SP = x × 1 − 1  1 − 2  1 − 3 
 100   100   100 
  r  r  r 
= 1 − 1 − 1  1 − 2  1 − 3  × 100%
  100   100   100  
  EXAMPLE 13. The marked price of a watch is ` 1600.
Note This formula also can be apply for more than three
The shopkeeper gives successive discount of 10%, r%
successive discounts. to the customer. If the customer pays ` 1224 for the
watch, then the value of r is
EXAMPLE 12. A single discount which is equivalent a. 20% b. 35% c. 37% d. 15%
to two successive discounts of 20% and 5% is
Sol. d. Given, cost price x = ` 1600, r1 = 10%
a. 20% b. 22% c. 24% d. 25%
and r2 = r %
Sol. c. Given, r1 = 20%, r2 = 5%
∴ Selling price = Cost price 1− 1  1− 2 
r r
∴Single discount equivalent to two succesive discount  100  100
20% and 5%
1224 = 16001−
10   r 
= 1− 1−
20   5   1− 
 1−  × 100%  100  100
  100  100 
r 1224 r 1224
= 1− ×  × 100% = 1−  × 100%
4 19 19 ⇒ 1− = ⇒ = 1−
 5 20  25 100 160 × 9 100 1440
6 ∴ r = 15%
= × 100% = 24%
25

PRACTICE EXERCISE
1. By selling an article for ` 247.50, Sonu get a 7. Sneha gains 10% on selling a pen. If she sells it
profit of 12.5%. The cost of the article is at double the price, the profit per cent is
(a) ` 220 (b) ` 205 (c) ` 210 (d) ` 200 (a) 120% (b) 60% (c) 100% (d) 200%

2. If cost price of a fan is ` 720 and its SP is ` 840. 8. On selling an article for ` 240, a trader loses
4%. In order to gain 10%, he must sell the article
Find the gain percent. for
2 1 7
(a) 16% (b) 16 % (c) 16 % (d) 16 % (a) ` 275 (b) ` 280 (c) ` 285 (d) ` 300
3 3 3
9. By selling 8 dozen pencils, a shopkeeper gains
3. By selling an article for ` 110, a man losses 12%. the selling price of 1 dozen pencils. What is the
For how much should he sell it to gain 8%? gain?
(a) ` 120 (b) ` 125 (c) ` 135 (d) ` 140 1 1 2 1
(a) 12 % (b) 13 % (c) 14 % (d) 87 %
2 7 7 2
4. A man buys 4 tables and 5 chairs for ` 1000. If
he sells the tables at 10% profit and chairs 20% 10. A man purchased a watch for ` 400 and sold it at
profit, he earns a profit of ` 120. Then, what is a gain of 20% of the selling price. The selling
the cost of one table? price of the watch is
(a) ` 300 (b) ` 320 (c) ` 440 (d) ` 500
(a) ` 200 (b) ` 220 (c) ` 240 (d) ` 260
11. A person A sells a table costing ` 2000 to a
5. If selling price of 8 articles is equal to the cost person B and earns a profit of 6%. The person B
price of 10 articles, then per cent gain or loss is sells it to another person C at a loss of 5%. At
(a) 20% (b) 25% (c) 30% (d) 35% what price did B sell the table?
6. A man sells fans at the same price on one he (a) ` 2054 (b) ` 2050 (c) ` 2024 (d) ` 2014
gain 20% and losses 20% on the other. His gain 12. What price did the seller mark at the printed
or loss is price of a watch purchased at ` 380, so that after
(a) 4% loss (b) 4% gain giving 5% discount, there is 25% profit?
(c) Neither gain nor loss (d) 1% loss (a) ` 400 (b) ` 450 (c) ` 500 (d) ` 600
96 CDS Pathfinder

13. A discount series of 10%, 20% and 40% is equal 23. A fruit-seller buys lemons at 2 for a rupee and
to a single discount of sells them at 5 for three rupees. What is his gain
(a) 50% (b) 60% (c) 56.8% (d) 70.28% per cent?
1 1 (a) 10% (b) 15% (c) 20% (d) 25%
14. Successive discounts of 12 % and 7 % are
2 2 24. A trader marks 10% higher than the cost price.
given on the marked price of a cupboard. If the He gives a discount of 10% on the marked price.
customer pays ` 2590, then what is the marked In this kind of sales how much per cent does the
price? trader gain or loss?
(a) ` 3108 (b) ` 3148 (c) ` 3200 (d) ` 3600 (a) 5% gain (b) 2% gain (c) 1% loss (d) 3% loss
15. The difference between a discount of 40% on 25. One saree was purchased for ` 564 after getting
` 1000 and two successive discounts of 35% and a discount of 6% and another saree was
5% on the same amount is purchased for ` 396 after getting a discount of
(a) ` 15.50 (b) ` 16.50 (c) ` 17.50 (d) ` 18.00 1%. Taking both the items as a single
transaction, what is the percentage of discount?
16. A dealer buys an article listed at ` 100 and gets
(a) 3.5% (b) 4% (c) 7% (d) 7.5%
two successive discounts of 10% and 20%. He
spends 10% of the cost price on transport etc. At 26. A dishonest dealer professes to sell his good at
what price should he sell the article to earn a cost price but uses a false weight and thus gains
profit of 15%? 25%. For a kilogram he uses a weight of
(a) ` 90 (b) ` 91 (c) ` 91.08 (d) ` 91.10 (a) 700 g (b) 750 g (c) 800 g (d) 850 g
17. An item costing ` 200 is being sold at 10% loss. 27. A person bought two old scooters for ` 9000. By
If the price is further reduced by 5%, then the selling one at a profit of 25% and the other at a
selling price will be loss of 20%, to neither gain nor loses. The cost of
(a) ` 170 (b) ` 171 (c) ` 175 (d) ` 179 each scooter is
18. A man bought a number of oranges at 3 for a (a) ` 3500, ` 500 (b) ` 4500, ` 4500
rupee and an equal number at 2 for a rupee. At (c) ` 4000, ` 5000 (d) ` 5300, ` 3700
what price per dozen should he sell them to 28. The manufacturer of a certain item can sell all
make a profit of 20%? he can produce at the selling price of ` 60 each.
(a) ` 4 (b) ` 5 (c) ` 6 (d) ` 7 It costs him ` 40 in materials and labour to
19. A milk vendor bought 28 L of milk at the cost of produce each item and he has overhead expenses
` 8.50 per L. After adding some water, he sold of ` 3000 per week in order to operate the plant.
the mixture at the same price. If he gains 12.5%, The number of items he should produce and sell in
then how much water did he add? order to make a profit of atleast ` 1000 per week is
(a) 5.5 L (b) 4.5 L (a) 400 (b) 300 (c) 250 (d) 200
(c) 3.5 L (d) 2.5 L 1
29. A person sold a table at a profit of 6 %. If he
20. A man sold two watches, each for ` 495. If he 2
gained 10% on one watch and suffered a loss of had sold it for ` 1250 more, he would have
10% on the other, then what is the loss or gain gained 19%
percentage in the transaction? I. The CP of the table is ` 10000.
(a) 1% gain (b) 1% loss II. CP will ` 19000, if he had sold it for ` 2400 more
(c) 100/99% loss (d) No gain no loss and gained same profit
21. Jyoti bought a computer system for ` 40000. She Which one is correct?
sold it to Brajesh at a loss of 4%. If Brajesh sells (a) Only II (b) Only I
it for ` 40320 to Yash, then the profit per cent (c) Neither I nor II (d) Both I and II
earned by Brajesh is
30. A bookseller sells a book at a gain of 10%. If he
(a) 3% (b) 5%
had bought it at 4% less and sold it for ` 6 more,
(c) 7% (d) 10% 3
he would have gained 18 %
22. List price of a video cassette is ` 100. A dealer 4
sells three video cassettes for ` 274.50 after I. The SP of the book is ` 165.
allowing discount at certain rate. The rate of II. The CP of the book is ` 150.
discount allowed is Which one is correct?
(a) 7% (b) 7.5% (a) Neither I nor II (b) Both I and II
(c) 8% (d) 8.5% (c) Only I (d) Only II
MATHEMATICS Profit and Loss 97

e 2013 II
PREVIOUS YEARS’ QUESTIONS
(a) 10% loss (b) 10% profit
31. A person bought 8 quintal of rice for certain (c) 20% profit (d) 20% loss
rupees. After a week, he sold 3 quintal of rice at
10% profit, 3 quintal of rice with neither profit 39. On a 20% discount sale, an article costs ` 596.
nor loss and 2 quintal at 5% loss. In this What was the original price of the article?
transaction, what is the profit? e 2012 I e 2014 I
(a) 10% (b) 20% (c) 25% (d) None of these (a) ` 720 (b) ` 735 (c) ` 745 (d) ` 775

32. The cost of two articles are in the ratio 3 : 5. If 40. A man buys 200 oranges for ` 1000. How many
there is 30% loss on the first article and 20% oranges for ` 100 can be sold, so that his profit
gain on the second article, then what is overall percentage is 25%? e 2014 II
percentage of loss or gain? e 2012 I (a) 10 (b) 14 (c) 16 (d) 20
(a) 2.25% gain (b) 5.25% loss 41. When an article is sold at 20% discount, the
(c) 2% loss (d) None of these selling price is ` 24. What will be the selling
33. A cloth store is offering Buy 3, get 1 free. What price when the discount is 30%. e 2014 II
is the net percentage discount being offered by (a) ` 25 (b) ` 23 (c) ` 21 (d) ` 20
the store? e 2012 II 42. A shopkeeper sells his articles at their cost price
1 but uses a faulty balance which reads 1000 g for
(a) 20% (b) 25% (c) 30% (d) 33 %
3 800 g. What is his actual profit percentage?
34. A person sold an article for ` 136 and got 15% (a) 25% (b) 20% e 2014 II
loss. Had he sold it for ` x, he would have got a (c) 40% (d) 30%
profit of 15%. Which one of the following is 43. A person selling an article for ` 96 finds that his
correct? e 2012 II
loss per cent is one-fourth of the amount of
(a) 190 < x < 200 (b) 180 < x < 190
rupees that he had paid for the article. What can
(c) 170 < x < 180 (d) 160 < x < 170
be the cost price? e 2014 II
35. A man buys a television set which lists for (a) Only ` 160 (b) Only ` 240
` 5000 at 10% discount. He gets an additional (c) Either ` 160 or ` 240 (d) Neither ` 160 nor ` 240
2% discount (after the first discount) for paying
cash. What does he actually pay for the set? 44. A milkman claims to sell milk at its cost price
e 2012 II but he is making a profit of 20% since, he has
mixed some amount of water in the milk. What
(a) ` 4410 (b) ` 4400 (c) ` 4000 (d) ` 4500
is the percentage of milk in the mixture? e 2015 I
36. A merchant earns a profit of 20% by selling a 250 200
basket containing 80 apples which costs ` 240 (a) 80% (b) % (c) 75% (d) %
3 3
but he gives one-fourth of it to his friend at cost
price and sells the remaining apples. In order to 45. The value of a single discount on some amount
earn the same profit, at what price must he sell which is equivalent to a series of discounts
each apple? e 2012 II of 10%, 20% and 40% on the same amount, is
(a) ` 3.00 (b) ` 3.60 (c) ` 3.80 (d) ` 4.80 equal to e 2015 II
37. A person sold an article for ` 3600 and got a (a) 43.2% (b) 50% (c) 56.8% (d) 70%
profit of 20%. Had he sold the article for ` 3150, 46. A cloth merchant buys cloth from a weaver and
how much profit would he have got? e 2013 II cheats him by using a scale which is 10 cm longer
(a) 4% (b) 5% (c) 6% (d) 10% than a normal metre scale. He claims to sell
38. Two lots of onions with equal quantity, one cloth at the cost price to his customers, but while
costing ` 10 per kg and the other costing ` 15 per selling uses a scale which is 10 cm shorter than
kg, are mixed together and whole lot is sold at a normal metre scale. What is his gain? e 2016 I
2 1
` 15 per kg. What is the profit or loss? (a) 20% (b) 21% (c) 22 % (d) 23 %
9 3

ANSWERS
1 a 2 b 3 c 4 a 5 b 6 a 7 a 8 a 9 c 10 d
11 d 12 c 13 c 14 c 15 c 16 c 17 b 18 c 19 c 20 b
21 b 22 d 23 c 24 c 25 b 26 c 27 c 28 d 29 b 30 b
31 d 32 d 33 b 34 b 35 a 36 c 37 b 38 c 39 c 40 c
41 c 42 a 43 c 44 b 45 c 46 a
98 CDS Pathfinder

HINTS AND SOLUTIONS 


1. (a) Given, selling price of article Selling price of an article for a profit of ⇒ Selling price
25 10%
= ` 247.50 and gain =
= 1000 ×  1 −
% 35   5 
2 240 × 100 110 240 × 110 1− 
= × =  100   100 
∴ Cost price 96 100 96
  = ` 275 [Rule 11]
  65 95
100 = × × 1000 = ` 617.50
= ` × 247.50 [Rule 1] 9. (c) Let selling price of 1 dozen pencil be
100 100
  100 + 25   ` x.
  2  ∴ Selling price of 8 dozen pencils = ` 8x ∴Difference = ( 617. 50 − 600) = ` 17.50
100 × 2 × 24750. and profit = `x 16. (c) List price of article be ` 100.
=` = ` 220
225 Q Cost price of 8 dozen pencils Cost price for dealer
2. (b) Total gain = SP – CP = 8x − x = ` 7x ( 100 − 20) ( 100 − 10)
= (840 − 720) = ` 120
x 2 = × × 100
∴ Gain, percent = × 100% = 14 % 100 100
120 2 7x 7
∴ Gain percent = × 100 = 16 % 80 × 90 × 100
720 3 10. (d) Given, CP = ` 400, let selling price be = = ` 72
100 × 100
3. (c) Given, SP = ` 110 and loss = 12% ` x.
Money spent on transport
Then, 400 + 20% of x = x
∴ Costprice = `  × 110 = ` 125
100 10
 88  x 4x = × 72 = ` 7.20
⇒ 400 + = x ⇒ = 400 100
5 5
Now, CP = ` 125, gain required = 8% 400 × 5 ∴Total cost price = 72 + 7. 20 = ` 79.20
⇒ x= ⇒ x = ` 500
( 100 + 8)
∴ SP = `  × 125 = ` 135 4 ∴ Selling price
 100  ∴ Selling price = ` 500 ( 100 + 15)
= × 79. 20
4. (a) Let cost of one table and one chair 11. (d) The cost price of table for person B 100
be ` x and ` y, respectively. 2000 115 × 79.20
= 2000 + 6 × = = ` 91.08
∴ Cost of 4 tables and 5 chairs 100 100
= ` 4x + 5 y = 2000 + 120 = ` 2120 17. (b) Given, cost of article = ` 200
∴ 4x + 5 y = 1000 ...(i) Selling price for person B ∴ Selling price of article = 95% of
∴ Now, profit on 4 tables 2120 × 5 ( 90% of 200)
4x = 2120 −
= 4x × 10% = ` 100
=
95
×
90
× 200 = ` 171
10 = 2120 − 106 = ` 2014 100 100
Profit on 5 chairs = 5 y × 20% = ` y
4x 12. (c) Let marked price be ` x. 18. (c) Given, CP of 3 oranges of Ist variety
∴ + y = 120 ...(ii)
10 Selling price after 5% discount =`1
5 19 1
Now, on solving Eqs. (i) and (ii), we get =x− x = x CP of 1 orange of Ist variety = `
x = ` 200 100 20 3
5. (b) Refer to example 9. Profit = SP − CP =
19
x − 380 CP of 2 oranges of IInd variety = ` 1
20 1
6. (a) Refer to example 4. CP of 1 orange of IInd variety = `
19
7. (a) Let the selling price be ` 100. x − 380 2
SP × 100 Profit % = 20 × 100 Total CP of 2 oranges of different
∴ Cost price = [Rule 1] 380 1 1 5
100 + 10 19 variety = + = `
100 × 100 1000 x − 380 3 2 6
= =` 25 = 20 × 100
110 11 380 Profit on 2 oranges of different variety
Now, if SP is ` 200. 20 5 20 5 1
x = 475 × = ` 500 = 20% of = × =`
∴ Gain = `  200 −
1000  1200 6 100 6 6
 =` 19
 11  11 ∴ SP of 2 oranges of different variety
13. (c) Refer to example 12.
1200 / 11
Gain percent = × 100 = 120% 14. (c) Refer to example 13.
5
= + =`1
1
1000 / 11 6 6
8. (a) Given, selling price of an article 15. (c) Case I Discount = 40 % Hence, SP of 12 oranges is ` 6.
⇒ Selling price = 60% of 1000 19. (c) Given, cost price of 1L = ` 8.50
= ` 240
60
Loss = 4% = × 1000 = ` 600 ∴Total CP of milk = 28 × 850
. = ` 238
100
∴Cost price of an article for the loss of 4% ∴ Profit = 125. % of 238
Case II Two successive discounts are
240 × 100 125
.
=` [Rule 2] of 35% and 5%. = × 238 = ` 29.75
96 100
MATHEMATICS Profit and Loss 99

Let he added x L of water. Total amount after discount 19x 213x


⇒ − = 1250
∴ Profit = x × 85
. ⇒ 29. 75 = x × 85
. = 564 + 396 = ` 960 100 200
∴ Discount per cent ⇒ x = 1250 × 8 = 10000
∴ x = 3 .5 L
213x
20. (b) Refer to example 6. 1000 − 960 40 II. If SP = + 2400 and gain
= × 100 = % = 4% 200
1000 10
21. (b) Cost price of computer for Brajesh = 19%, then
4 26. (c) Here, true weight = 1000 213x
= 40000 − ( 40000) = ` 38400 + 2400 − x
100 and gain = 25% 19 = 200 × 100
Selling price of computer for Brajesh 1000 − false weight x
= ` 40320 ⇒ 25 = × 100 19x 13x
false weight ⇒ − = 2400
∴ Profit amount = (40320 − 38400) 100 200
= ` 1920 false weight ⇒ x = 2400 × 8 = 19200
⇒ = 1000 − false weight
∴ Profit percentage earned by Brajesh 4 Hence, I is correct but II is incorrect.
1920 4
= × 100 = 5% ⇒ false weight = 1000 × = 800 30. (b) Let CP = x
38400 5 110x 11x
22. (d) Given, list price of a video cassette then SP = =
Hence, he uses the weight 800 g. 100 10
= ` 100 27. (c) Let cost price of one scooter be ` x. 96x 24x
Now, CP = 96% of x = =`
Let the rate of discount be r%. ∴ Cost price of other scooter 100 25
Selling price of 3 video cassette According to the questions
= ` ( 9000 − x)
= ` 274.50 SP = ` 
11x
+ 6
∴ Selling price of Ist scooter
∴ Selling price of 1 video cassette  10 
25x 125x
27450
. =x+ =  11x  3 24x
=` = ` 91.50 100 100 ∴  + 6 = 118 % of
3  10  4 25
r Also, selling price of 2nd scooter
∴ 100 − × 100 = ` 91.50 11x + 60 475 24x 57x
⇒ = × =
= ( 9000 − x)  1 −
100 20 
 10 400 25 50
⇒ 100 − 91.50 = r ⇒ 8.50 = r  100 
⇒ 570x = 550x + 3000
= ( 9000 − x) 
80 
∴ Rate of discount = 85
. %  3000
 100  ⇒ x= = 150
1
23. (c) CP of 1 lemon = Total selling price of both scooters
20
2 ∴ CP = ` 150
125x 80
SP of 1 lemon =
3 = + ( 9000 − x) 11
100 100 ∴ SP = × 150 = ` 165
5 10
125x 80
∴ Gain % ∴ + ( 9000 − x) = 9000 Hence, I and II both are correct.
3 1 100 100
− [given] 31. (d) Let CP of 8 quintal be ` x.
6 −5
= 5 2 × 100 = × 2 × 100 = 20% 45x ∴ CP of 1 quintal = `
x
1 10 ⇒ + 7200 = 9000
100 8
2
∴ x = 4000 ∴ SP of 3 quintal rice at 10% profit
24. (c) Let the cost price be ` x. ∴ Cost price of 1st scooter = ` 4000 3x 3x 1 3x 3x 33x
x × 110 11x Hence, cost price of 2nd scooter = + × = + =
Marked price = =`f 8 8 10 8 80 80
100 10 = ( 9000 − x) = ` 5000 SP of 3 quintal rice without profit or
11x 90 99x 3x
∴ SP = × = 28. (d) Let the number of items be x. loss = `
10 100 100 8
Then, selling price of items = 60 x
∴ Required gain/loss per cent SP of 2 quintal rice at 5% loss
99x Cost of material of items = 40 x
−x 2x 2x 5 x x
Overhead expenses = ` 3000 = − × = −
= 100 × 100 = − 1% [Rule 8] 8 8 100 4 4 × 20
x ∴ 60 x − ( 40 x + 3000) = 1000
19x 19x
⇒ 20 x = 4000 = =
[negative sign i.e. loss] 4 × 20 80
4000 33x 3x 19x
25. (b) Let marked price of two sarees be ` x ∴ x= = 200 ∴ Total SP = + +
and ` y, respectively. 20 80 8 80
6x 94x 29. (b) I. Let CP = x 33x + 30x + 19x 82x
∴ x− = 564 ⇒ = 564 = =
100 100 213 80 80
Then, SP = x.
y SP − CP
⇒ x = ` 600 and y − = 396 200 ∴ Profit percentage = × 100%
100 213x CP
If SP = + 1250, then gain = 19% 82x

99 y
= 396 ⇒ y = ` 400 200 −x
(82 − 80)x
100 213x = 80 × 100% = × 100%
+ 1250 − x x 80x
∴ Total MP amount = 600 + 400 ⇒ 19 = 200 × 100 2
= × 100 = 2.5%
= ` 1000 x 80
100 CDS Pathfinder

32. (d) Let the CP of two articles be 3x and 1 20 x


But he sold of his apples i.e. 20 apples ⇒ x = 24 + x ⇒ x = 24 +
5 x, respectively. 4 100 5
3x × 70 21x for ` 60. x 4x
∴ SP of first article = = ⇒ x− = 24 ⇒ = 24
100 10 ∴ SP of remaining 60 apples 5 5
= ( 288 − 60) = ` 228 24 × 5
[Rule 2] ⇒ x= = 30
and SP of second article 228 4
∴ SP of 1 apple = = ` 3.80
5x × 120 60 ∴ Cost price of an article = ` 30
= = 6x
100 37. (b) Let the cost price of the article be ` x. Now, selling price after 30% discount
∴ Total SP Given, profit percentage = 20% 30
= 30 − 30% of 30 = 30 − × 30
21x 60x + 21x 81x 20x 120x 100
= 6x + = = Now, x + = 3600 ⇒ = 3600
10 10 10 100 100 = 30 − 9 = ` 21
∴ Total CP = 3 x + 5 x = 8x ∴ x = ` 3000 42. (a) Q Actual profit percentage
81x 81x − 80x x Fair weight − Unfair weight
∴ Profit = − 8x = = Now, profit percentage when the article = × 100 %
10 10 10 is sold for ` 3150 Unfair weight
∴Overall percentage of gain percentage 3150 − 3000 150 1000 − 800
× 100 = × 100 = 5% = × 100% = 25%
x 3000 3000
× 100 800
10 x × 100
= = = 1. 25% 38. (c) Let each lot of onion contains x kg. 43. (c) Let the cost price of an article be ` x.x
8x 10 × 8x Then, total cost price of these two lots
and selling price of an article = ` 96
33. (b) We know that, together = 10 x + 15x = 25x
[given]
Net percentage discount Selling price of whole lot
x − 96 1
Discount 1 = 15 × ( x + x) = 15 × 2x = 30 x × 100 = x
= × 100% = × 100% = 25% x 4
Cost price 4 30x − 25x
∴ Profit percentage = × 100 ⇒ x2 − 400 x + 38400 = 0
34. (b) We know that, 25x
5x ⇒ x2 − 160 x − 240 x + 38400 = 0
Cost price = 
100  × SP = × 100 = 20%
 25x ⇒ ( x − 160) ( x − 240) = 0
 100 − Loss% 
Hence, the profit is 20%. ∴ x = 160 or 240
[Rule 2]
39. (c) Let the original price be ` x. Hence, the cost price of the article is
= 
100 
 × 136 either
 100 − 15  Since, at discount of 20% article costs
` 596. ` 160 or ` 240.
136 × 100
= = ` 160 80 596 × 100
85 Then, 596 = ×x ⇒x= 44. (b) Let CP of 1 L of milk be ` x.x
100 80 ∴
Q Given, profit percentage = 15% SP of 1 L of milk
= ` 745 = xx × 120% = ` 1. 2x
∴ Selling price (x)
Hence, the original price of an article is Now, as in ` 12. x,x the quantity of milk
160 × ( 100 + 15) 160 × 115 sold = 1 L
= = = ` 184 ` 745.
100 100 ∴ In ` x,x quantity of milk sold
40. (c) Given, cost price of 200 oranges
So, option (b) is correct because 1 5
= ` 1000 = ×x= L
180 < x < 190. 12
. x 6
1000
35. (a) Given, list price of television = ` 5000 ∴ Cost price of 1 orange = = `5
200 According to the question,
and discount = 10% 100 + 25 
Selling price of 1 orange = 5 
CP of milk and SP of mixture are same,
∴ After discount television cost  100  therefore in mixture, quantity of milk
5
= 5000 − 5000 × 10% =5×
125
= ` 6.25
must be L.
5000 × 10 6
= 5000 − = ` 4500 100
100 Hence, the required percentage
Now, in ` 6.25 number of oranges can be
5 250
Additional discount = 2% sold = 1 = × 100% = %
6 3
∴ Actually price of television In ` 100, number of oranges can be sold
4500 × 2 1 45. (c) Refer to example 12.
= 4500 − = ` 4410 = × 100 = 16
100 6.25 46. (a) Let the amount of cloth = 100 cm
36. (c) Q CP of 80 apples = ` 240 Hence, 16 oranges can be sold in ` 100 Then, amount of cloth purchased
240 for profit 25%. = 110 cm
∴ CP of 1 apple = =`3
80 41. (c) Let the cost price of article be ` x. and amount of cloth sold = 90 cm
∴ CP of 20 apples = `3 × 20 = ` 60 Discount = 20% and selling price = 24 110 − 90
∴ Gain % × 100%
To earn a profit of 20%, SP = 120% of [given] 100
20
240 = ` 288 ∴ x = 24 + 20% of x = × 100% = 20%
100
MATHEMATICS Simple Interest 101

12
RATIO AND
PROPORTION
Regularly (1-2) questions have been asked from this chapter. Generally both the topics i.e. ratio and
proportion have same importance but more emphasis is given on proportion. The concepts of this
chapter are extremely used in other chapters like-age, average. etc.

RATIO
A ratio is the comparison of two or more quantities of the same type (or kind) by division. i.e. if a and
b are two quantities of same kind (or same unit), then the fraction a / b is called the ratio a to b and
we write it as a : b. Here, a is called antecedent and b is called the consequent.
e.g. If a fruit box contains 8 oranges and 7 lemons then the ratio of oranges to lemons is 8 to 7 or 8:7.

Note If both terms a and b of a ratio are multiplied or divided by the same quantity, then ratio remains unchanged.
a na a b a a/n
i.e. a : b is same as na : nb ⇒ = and a : b is same as : ⇒ = .
b nb n n b b/n

EXAMPLE 1. If the ratio of 90 cm to 1.5 m is same as 3 : x, then x is


a. 4 b. 5 c. 2 d. 1
Sol. b. 1.5 m = 150 cm (units must be same)
90 90 / 30 3
Ratio of 90 cm to 1.5 m = = = .
150 150 / 30 5
As this ratio is same as 3 : x
3 3
∴ =
x 5
⇒ 3x = 5 × 3
⇒ x=5
102 CDS Pathfinder

1 1
Types of Ratios 6. Reciprocal Ratio If a : b is a ratio, then : is its
a b
1. Compound Ratio When two or more ratios are reciprocal ratio, i.e. b : a .
multiplied together, they are said to be compound 1 1
ratio. e.g. Reciprocal ratio of 3 : 7 is : i.e. 7 : 3.
3 7
a c e g
If , , and are all ratios, then their compound EXAMPLE 4. Compound ratio of the duplicate ratio
b d f h
of 5 : 6, the reciprocal ratio of 25:42 and the
ratio is aceg : bdfh.
2 4 1 2× 4×1 8 subduplicate ratio of 36:49 is
e.g. Compound ratio of , and = = . a. 1 : 2 b. 2 : 3 c. 1 : 1 d. 4 : 9
3 7 3 3 × 7 × 3 63
5 5 25
2. Duplicate Ratio When a ratio is compounded with Sol. c. The duplicate ratio of 5 : 6 is × =
6 6 36
itself, the resulting ratio is called the duplicate ratio.
1 1 42
So, a 2 : b 2 is duplicate ratio of a : b. The reciprocal ratio of 25 : 42 is : i.e.
25 42 25
EXAMPLE 2. The duplicate ratio of the ratio 2 2 : 3 5 36 36 6
and subduplicate ratio of is =
is 49 49 7
a. 4 : 9 b. 8 : 45 c. 2 : 3 d. 6 : 45 25 42 6 1
∴ Compound ratio is × × = = 1: 1
Sol. b. The duplicate ratio of 2 2 : 3 5 is 36 25 7 1
2 2 2 2 4 4 8
× = = = 8 : 45 IMPORTANT POINTS
3 5 3 5 9 25 45
3. Triplicate Ratio If a ratio is compounded three l Usually, the ratio is expressed in its lowest terms.
times with itself, then resulting ratio is called l Ratio exists only between quantities of the same kind.
triplicate ratio. l Ratio is a fraction, so it has no units.
So, a 3 : b 3 is the triplicate ratio of a : b. l Ratio is taken only between positive quantities.

EXAMPLE 3. If 4x + 3 : 9x + 10 is the triplicate ratio


of 3 : 4. Then, the value of x is PROPORTION
a. 2 b. 6 c. 8 d. 12 The equality of two ratios is called proportion.
Sol. b. Since, 4x + 3 : 9x + 10 is the triplicate ratio of 3 : 4. ‘: :’ is the sign of proportion.
4x + 3 3 3 3 4x + 3 27 So, if a : b = c : d , we write it as
Therefore, = × × ⇒ =
9x + 10 4 4 4 9x + 10 64 a : b : : c : d and say that a, b, c and d are in proportion.
⇒ 64( 4x + 3) = 27(9x + 10) The quantities a, b, c and d are called the first, second,
⇒ 256x + 192 = 243x + 270
third and fourth terms of proportion respectively. First
and fourth terms are called extreme terms and second
⇒ 256x − 243x = 270 − 192 ⇒ 13x = 78 and third terms are called means or middle terms.
78
∴ x= =6 If quantities a, b, c and d are in proportion, then
13
a c
Hence, the value of x is 6. = ⇒ ad = bc
b d
4. Subduplicate Ratio If two numbers are in ratio, i.e. product of extreme terms = product of middle terms.
then the ratio of their square roots is called sub This is also called cross-product rule.
duplicate ratio.
If a : b is ratio, then the subduplicate ratio is a : b. Types of Proportion
e.g. Subduplicate ratio of 9 : 16 is 9 : 16 = 3 : 4. 1. Continued Proportion The non-zero quantities of
same kind, a, b, c, d, e, f, … are said to be continued
5. Subtriplicate Ratio If two numbers are in ratio, a b c e
proportion, if = = = = …
then the ratio of their cube roots is called b c d f
subtriplicate ratio.
EXAMPLE 5. If a, b, c, d and e are in continued
If a : b is ratio, then the subtriplicate ratio is 3 a : 3 b.
proportion, then a/e is equal to
e.g. Subtriplicate ratio of 64 : 27 is 3 64 : 3 27 = 4 : 3 a. a3 /b3 b. a 4 /b 4 c. b3 /a3 d. b 4 /a 4
MATHEMATICS Ratio and Proportion 103

Sol. b. Since, , , , and are in continued proportion. EXAMPLE 8. The fourth proportional to
2
Q = = = ⇒ = = = ⇒ = p 2 − pq + q 2 , p 3 + q 3 , p − q is
2 4 3 2 6 4 a. p + q b. p − q c. p2 + q2 d. p2 − q2
1
= = 2 ⋅ = 2 , = = 4 ⋅ 2 = 3 Sol. d. Let the fourth proportional be x.
4
Then, 2
− + 2
: 3
+ 3
= − :x
∴ = =
( 4
/ 3
) 4 2
− + 2
− ( − )( 3
+ 3
)
⇒ = ⇒x=
2. Mean Proportional If a, b and c are in continued
3
+ 3
x 2
− + 2

proportion, then b is called the mean proportional of ( − )( 3


+ 3
)
a and c. ∴ x= =( − )( + ) =( 2
− 2
)
2
− + 2
a b
i.e. = ⇒ b 2 = ac ⇒ b = ac
b c
Properties of Proportion
EXAMPLE 6. What is the mean proportional between If four non zero quantities a, b, c and d are in proportion,
(15 + 200 )and (27 − 648)? e 2012 I then properties of proportion are as follows.
a. 4 b. 14 7 c. 3 5 d. 5 3 1. Invertendo If a : b : : c : d, then b : a : : d : c
Sol. c. Let mean proportional between (15 + 200) and a c b d
i.e. = ⇒ =
( 27 − 648) is x b d a c
Then, x = (15 + 200)( 27 − 648) 2. Alternendo If a : b :: c : d, then a : c :: b : d
a c a b
= (15 + 10 2)( 27 − 18 2) i.e. = ⇒ =
b d c d
= 5 ( 3 + 2 2).9( 3 − 2 2) = 45[( 3) 2 − ( 2 2) 2 ] 3. Componendo (Adding the Denominator)
= 45 (9 − 8) = 45 = 3 5 If a : b :: c : d, then ( a + b): b : : ( c + d ): d
3. Third Proportional If a : b : : b : c, then c is called a c a + b c +d
i.e. = ⇒ =
the 3rd proportional to a and b. Now, c will be b d b d
calculated as below: 4. Dividendo (Subtracting the Denominator)
b2 If a : b :: c : d, then a : ( a − b): : c : ( c − d )
a : b :: b : c ⇒ a : b = b : c ⇒ c =
a a c a c
i.e. = ⇒ =
EXAMPLE 7. The two numbers such that their mean b d a − b c −d
proportional is 24 and the third proportional is 1536, are 5. Componendo and Dividendo
a. 3 and 98 b. 6 and 96 c. 4 and 92 d. None of these If a : b :: c : d, then ( a + b): ( a − b): : ( c + d ): ( c − d )
Sol. b. Let and be the required numbers, then mean a c a + b c +d
proportional between and is 24. i.e. = ⇒ =
b d a − b c −d
∴ : 24 = 24 : ⇒ = 242, = 576 …(i)
EXAMPLE 9. If (a − b) : (a + b) = 1 : 5, then what is
and third proportional to and is 1536.
⇒ : = : 1536 (a 2 − b 2 ) : (a 2 + b 2 ) equal to?
2
⇒ 2
= 1536 ⇒ = …(ii) 13 4 5
1536 a. b. c. d. None of these
5 13 13
On putting the value of in Eq. (i), we get
1 − + 5 + + − 5+ 1
2
Sol. c. Given, ⇒ = ⇒ = =
⇒ ⋅ = 576 ⇒ 3
= 576 × 1536 5 + − 1 + − + 5−1
1536
(using componendo and dividendo rule)
⇒ = (96 × 6) × (96 × 16) = 963 ⇒
3
= 96 2
2 6 3 3 9
576 576 ⇒ = = ⇒ = ⇒ 2 =
∴ = = =6 [from Eq. (i)] 2 4 2 2 4
96
Again, using componendo and dividendo rule
Hence, the required numbers are 6 and 96. 2
+ 2
9 + 4 13
4. Fourth Proportional If four numbers or quantities ⇒ = =
2
− 2
9−4 5
a, b, c and d are in proportion, then d is known as
fourth proportional.
2
− 2 5
⇒ = ∴( 2
− 2
) :( 2
+ 2
) = 5 : 13
2
+ 2 13
i.e. a:b::c:d
104 CDS Pathfinder

MIXTURE OR ALLIGATION VARIATIONS


If two or more quantities are mixed together in a certain When a change in a certain quantity leads to a certain
ratio, then the product is called mixture. change in another quantity then it is called variations.

Mean Price Direct Variation


Cost price of a unit quantity of the mixture is called the If x varies directly with y, then as x increses (decrease) y
mean price. also increases (decreases)
i.e. x ∝ y
Rule of Mixture or Alligation
It is the rule to find the ratio in which two or more ⇒ x = ky. where, k is the constant of variation.
ingredients are mixed or to be mixed.
If two quantities are mixed in the ratio n1 : n2 , then
Inverse Variation
n1 P2 − Pm If x varies inversely with y, then an increase is the value
= of x causes a decreases in the value of y.
n2 Pm − P1
1
where P1 , P2 and Pm are cheaper price, dearer price and i.e. x ∝
mean price respectively. y
k′
It can also be expressed as ⇒ x=
Cheaper price Dearer price y
P1 P2 where, k ′ is the constant of variation.
Mean price
EXAMPLE 11. If a quantity y varies as the sum of
Pm three quantities of which the first varies as x, the
second varies as − x + x 2 , the third varies as x 3 − x 2 ,
P2 – Pm Pm– P1
then what is y equal to?
Cheaper quantity n1 P2 − Pm
∴ = = a. kx3 , where k is a constant
Dearer quantity n2 Pm − P1 b. kx + lx2 + mx3 , where k , l and m are constants
c. kx2 , where k is a constant
Note P1 < Pm < P2
d. kx, where k is a constant
EXAMPLE 10. In what proportion, must wheat at Sol. b. Since, first term ∝ x
` 6.20 per kg be mixed with wheat at ` 7.20 per kg, so
that the mixture be worth ` 6.50 per kg? ⇒ First term= 1x ,
a. 6 : 4 b. 7 : 4 c. 7 : 3 d. 5 : 2 ⇒ Second term ∝ ( − x + x2)

Sol. c. Given, cost price of cheaper quantity = ` 6.20 per kg ⇒ Second term = 2( − x + x2 )

Cost price of dearer quantity ` 7.20 per kg and third term ∝ ( x3 − x2) ⇒ Third term = 3 ( x3 − x 2 )
Mean price = ` 6.50 per kg Also, ∝ [ 1x + 2 (− x + x ) + 2
3( x
3
− x )] (given)
2

According to the rule of alligation, ⇒ = 4[( 1 − 2) x +( 2 − 3) x


2
+ 3
3x ]

CP of cheaper CP of dearer = 4 ( 1 − 2) x+( 2 − 3) 4x


2
+ 3 4x
3

(620 paise) (720 paise)


= x+ x + 2
x 3
Mean price
(650 paise) where, = 4 ( 1 − 2 ),

(650 – 620) =( 2 − 3) 4,
(720 – 650) = 30 paise
= 70 paise and = 3 4

∴ Required ratio = 70 : 30 = 7 : 3
MATHEMATICS Ratio and Proportion 105

PRACTICE EXERCISE
1. The compounded ratio of 2a : 6b, 7a : 49b and 15. Divide 1870 into three parts in such a way that
3a : 12b is half of the first part, one-third of the second part
(a) a : 84 b 3
(b) a : 84 b 3
(c) a : 24 b (d) 2 a : 3 b and one-sixth of the third part are equal. Then,
the third part is
2. The subduplicate ratio of 16x 4 : 625 y 6 is (a) 340 (b) 510 (c) 1020 (d) 1320
2 3 2 2 2 3
(a) 4x : 25 y (b) 4x : 25 y (c) 4x : 25 y (d) 8x : 125 y 16. Out of the ratios 7 : 20, 13 : 25, 17 : 30 and
11 : 15, the smallest one is
3. If x + 5 : 3x + 4 is the duplicate ratio of 5 : 8 , then (a) 11 : 15 (b) 7 : 20 (c) 11 : 16 (d) 17 : 30
the value of x is
17. What is the ratio of the numbers of 0.5 of a
(a) 16 (b) 18 (c) 20 (d) 22 number is equal to 0.07 of another ?
4. If x : 6 : : 5 : 3 , then the value of x is (a) 50 : 7 (b) 5 : 7 (c) 1 : 14 (d) 7 : 50

(a) 8 (b) 10 (c) 12 (d) 13 18. What is the ratio whose terms differ by 40 and
2
5. The third proportional to 9 and 12 is the measure of which is ?
7
(a) 12 (b) 14 (c) 16 (d) 18 (a) 14 : 56 (b) 15 : 56 (c) 16 : 56 (d) 16 : 72
3 5 3 2
19. If P : Q = : and Q : R = : , then what is
6. The mean proportional between ( 2 + 3 ) and 5 7 4 5
( 8 − 48 ) is P : Q : R equal to?
(a) 2 (b) 3 (c) 4 (d) 5 3 5 2 9 15 2 3 3 2 3 5 3
(a) : : (b) : : (c) : : (d) : :
5 7 5 20 28 7 5 4 5 5 7 4
7. The fourth proportional to 7, 11, 14 is
(a) 16 (b) 18 (c) 20 (d) 22 20. Two numbers are in the ratio 3 : 5. If 9 is
subtracted from each number, then they are in
8. In a ratio which is equal to 7 : 8, if the the ratio of 12 : 23. What is the second number?
antecedent is 35, then the consequent is (a) 44 (b) 55 (c) 66 (d) 77
(a) 30 (b) 32 (c) 36 (d) 40
21. The mean proportional between two numbers is
9. If x : y = 1 : 3, y : z = 5 : k, z : t = 2 : 5 and t : x = 3 : 4, 28 and their third proportional to them is 224.
then what is the value of k? The two numbers are
(a) 1/2 (b) 1/3 (c) 2 (d) 3 (a) 7 and 112 (b) 14 and 56 (c) 28 and 28 (d) 21 and 36
10. If the ratio of x to y is 25 times the ratio of y to 22. ` 770 have been divided among A, B, C in such a
x, then what is the ratio of x to y? 2
way that A receives th of what B and C together
(a) 1 : 5 (b) 5 : 1 (c) 25 : 1 (d) 1 : 25 9
receive. What is A’s share?
11. If p% of ` x is equal to t times q% of ` y, then
(a) ` 154 (b) ` 140 (c) ` 250 (d) ` 254
what is the ratio of x to y ?
(a) pt : q (b) p : qt (c) qt : p (d) q : pt 23. x varies directly as y and inversely as square of
z. When y = 4 and z = 14, then x = 10. If y = 16
12. If a : b = 2 : 3 and x : y = 3 : 4 , then the value of and z = 7, what is x?
4ay − 3bx
is (a) 180 (b) 160 (c) 154 (d) 140
5ax − 2by
m + 3n + m − 3n
(a)
5
(b)
5
(c)
4
(d)
5 24. If x = , then
3 6 5 4 m + 3n − m − 3n
x3 + 3x 341 (a) 3 nx2 − 2 mx + 3 n = 0 (b) 2 nx2 − 2 mx + 3 n = 0
13. If = , then the value of x is
3x 2 + 1 91 (c) 3 nx − 2 mx − 3 n = 0
2
(d) 3 nx2 + 3mx + 3 n = 0

(a) 16 (b) 14 (c) 13 (d) 11 1 1 2


25. If p + r = 2q and + = , then
14. What is the number which has to be added to each q s r
term of the ratio 49 : 68, so that it becomes 3 : 4? (a) p : s = r : q (b) p2 : r 2 = q : s
(a) 3 (b) 5 (c) 8 (d) 9 (c) p : q = r : s (d) p2 = 2 r + s
106 CDS Pathfinder

26. If q is the mean proportional between p and r, 36. A mixture contains milk and water in the ratio
p2 − q 2 + r 2 5 : 1. On adding 5 L of water, the ratio of milk
then is equal to and water becomes 5 : 2. What is the quantity of
p− 2 − q − 2 + r − 2 milk in the original mixture?
2 3
(a) p q (b) q 3 (c) q 4 (d) p2 r 2q 4 (a) 5 L (b) 25 L
27. The monthly incomes of A and B are in the ratio (c) 27.5 L (d) 32.5 L
4 : 3. Each of them saves ` 600. If the ratio of the 37. If x varies as the mth power of y , y varies as the
expenditure is 3 : 2, then what is the monthly
nth power of z and x varies as the p th power of
income of A ?
z, then which one of the following is correct?
(a) ` 2400 (b) ` 1800 (c) ` 2000 (d) ` 3600
(a) p = m + n (b) p = m − n
28. Three numbers are in the ratio 3 : 2 : 5 and the (c) p = mn (d) None of these
sum of their squares is 1862. What are the three
numbers?
38. The wages of labourers in a factory has increased
in the ratio 22 : 25 and their number decreased in
(a) 18, 12, 30 (b) 24, 16, 40
the ratio 3 : 2. What was the original wage bill of
(c) 15, 10, 25 (d) 21, 14, 35
the factory, if the present bill is ` 5000?
29. The speeds of three cars are in the ratio 4 : 3 : 2. (a) ` 4000 (b) ` 6000
What is the ratio between the times taken by the (c) ` 8000 (d) None of these
cars to cover the same distance?
(a) 2 : 3 : 4 (b) 3 : 4 : 6
39. The ratio of A to B is x : 8 and the ratio of B to C
(c) 1 : 2 : 3 (d) 4 : 3 : 2 is 12 : z. If the ratio of A to C is 2 : 1, then what
is the ratio of x : z ?
30. The ratio between the ages of A and B is 2 : 5. (a) 2 : 3 (b) 3 : 2 (c) 4 : 3 (d) 3 : 4
After 8 yr, their ages will be in the ratio 1 : 2.
What is the difference between their present 40. A bag contains ` 112 in the form of ` 1, 50 paise
ages? and 10 paise coins in the ratio 3 : 8 : 10. What is
(a) 20 yr (b) 22 yr (c) 24 yr (d) 25 yr the number of 50 paise coins?
(a) 112 (b) 108 (c) 96 (d) 84
31. Let y is equal to the sum of two quantities of which
one varies directly as x and the other inversely as x. 41. In a class, the number of boys is more than
If y = 6 when x = 4 and y = 10/ 3, when x = 3, then the number of girls by 12% of the total students.
what is the relation between x and y? What is the ratio of number of boys to that of
(a) y = x + (4/ x) (b) y = − 2 x + (4/ x) girls?
(c) y = 2 x + (8/ x) (d) y = 2 x − (8/ x) (a) 11 : 14 (b) 14 : 11
(c) 28 : 25 (d) 25 : 28
32. If ` 8400 is divided among A, B and C in the
1 1 1 42. A sum of ` 53 is divided among A, B, C in such a
ratio : : , what is the share of A?
5 6 10 way that A gets ` 7 more than what B gets and B
(a) ` 1800 (b) ` 3000 (c) ` 3600 (d) ` 4000 gets ` 8 more than what C gets. The ratio of
their shares is
33. A certain amount of money has to be divided (a) 25 : 18 : 10 (b) 6 : 7 : 8
between two persons P and Q in the ratio 3 : 5. (c) 12 : 14 : 9 (d) 15 : 8 : 30
But it was divided in the ratio of 2 : 3 and there
by Q loses ` 10. What was the amount? 43. A person P started a business with a capital of
` 2525 and another person Q joined P after some
(a) ` 250 (b) ` 300 (c) ` 350 (d) ` 400
months with a capital of ` 1200. Out of the total
34. 20 L of a mixture contains milk and water in the annual profit of ` 1644, P ′s share was ` 1212.
ratio 4 : 3. If 6 L of this mixture are replaced by When did Q join as partner?
6 L of milk, the ratio of milk to water in the new (a) After 2 months (b) After 3 months
mixture will become (c) After 4 months (d) After 5 months
(a) 7 : 3 (b) 8 : 3 (c) 9 : 7 (d) 4 : 6
44. Seats for Mathematics, Physics and Biology in a
35. Two vessels are full with milk and water in the school are in the ratio 5 : 7 : 8. There is a proposal
ratios 1 : 3 and 3 : 5, respectively. If both are to increase these seats by 40%, 50% and 75%,
mixed in the ratio 3 : 2, what is the ratio of milk respectively What will be the ratio of increased
and water in the new mixture? seats?
(a) 4 : 15 (b) 3 : 7 (a) 2 : 3 : 4 (b) 6 : 7: 8
(c) 6 : 7 (d) None of these (c) 6 : 8: 9 (d) 5 : 7 : 12
MATHEMATICS Ratio and Proportion 107

45. The ratio between the number of passengers PREVIOUS YEARS’ QUESTIONS
travelling by I and II class between the two
railway stations is 1:50, whereas the ratio of I 51. If x : y = 7 : 5, then what is the value of
and II class fares between the same station is ( 5x − 2 y ) : ( 3x + 2 y )? e 2012 I
3:1, If on a particular day, ` 1325 were collected (a) 5/4 (b) 6/5 (c) 25/31 (d) 31/42
from the passengers travelling between these
stations, then what was the amount collected 52. Two numbers are in the ratio 2 : 3. If 9 is added
from the II class passengers? to each number, they will be in the ratio 3 : 4.
What is the product of the two numbers? e 2012 I
(a) ` 750 (b) ` 1000
(a) 360 (b) 480 (c) 486 (d) 512
(c) ` 850 (d) ` 1250
46. If x varies inversely as the square of y in such a 53. A milkman bought 15 kg of milk and mixed 3 kg
way that, if x = 1, then y = 6. of water in it. If the price per kg of the mixture
becomes ` 22, what is cost price of the milk
I. If y = 3, then x = 4 II. If y = 6, then x = 1 per kg? e 2012 II
Which of the following options is correct? (a) ` 28.00 (b) ` 26.40 (c) ` 24.00 (d) ` 22.60
(a) Only II (b) Both I and II
54. Sex ratio is defined as the number of females per
(c) Only I (d) Neither I nor II
1000 males. In a place, the total inhabitants are
47. If a : b = c : d = e : f = 1 : 2, then 1935000, out of which 935000 are females. What
3a + 5c + 7e 1 a 2 + c2 + e2 1 is the sex ratio for the place? e 2012 II
I. = II. = (a) 935 (b) 1000
3b + 5d + 7 f 3 b2 + d 2 + f 2 2
(c) 1935 (d) 9350
Which of the following option is correct?
(a) Only I (b) Only II 55. In a certain school, the ratio of boys to girls is
(c) Both I and II (d) Neither I nor II 7 : 5. If there are 2400 students in the school,
then how many girls are there? e 2013 I
48. A cat takes 5 leaps for every 4 leaps of a dog but
(a) 500 (b) 700 (c) 800 (d) 1000
3 leaps of the dog are equal to 4 leaps of the cat.
Now, the 56. If A : B = 2 : 3, B : C = 5 : 7 and C : D = 3 : 10, then
I. Ratio of the speeds of the cat to that of the dog what is A : D equal to? e 2014 I
is 15 : 16. (a) 1 : 7 (b) 2 : 7 (c) 1 : 5 (d) 5 : 1
II. Ratio of the distance of the cat to that of the dog 57. ( x + y ) : ( x − y ) = 3 : 5 and xy = positive imply that
is 15:16, covered in 30 min.
(a) x and y are both positive e 2014 II
Which of the following option is correct? (b) x and y are both negative
(a) Only I (b) Only II (c) one of them is positive and one of them is negative
(c) Both I and II (d) Neither I nor II (d) no real solution for x and y exists

49. Determine the ratio of the number of people 58. The ratio of ages of A and B is 2 : 5 and the ratio
having characteristic X to the number of people of ages of B and C is 3 : 4. What is the ratio of
having characteristic Y in a population of 100 ages of A, B and C? e 2014 II
subjects from the following table: (a) 6 : 15 : 20 (b) 8 : 5 : 3
Having X and Y 10 (c) 6 : 5 : 4 (d) 2 : 15 : 4
Having X but not Y 30
Having Y but not X 20 59. The height of a tree varies as the square root of
Having neither X nor Y 40 its age (between 5 to 17 yr). When the age of the
(a) 4:3 (b) 3:2 (c) 1:2 (d) 2:3 tree is 9 yr, its height is 4 ft. What will be the
50. Fresh grapes contain 90 percent water by weight height of the tree at the age of 16 yr? e 2014 II
while dried grapes contain 20 percent water by (a) 5 ft 4 inch (b) 5 ft 5 inch
weight. What is the weight of dry grapes contain (c) 4 ft 4 inch (d) 4 ft 5 inch
20 percent water by weight. What is the weight 60. 16 L of a mixture contains milk and water in the
of dry grapes available from 20 kg of fresh ratio 5 : 3. If 4 L of milk is added to this mixture,
grapes? the ratio of milk to water in the new mixture
(a) 2 kg (b) 2.4 kg would be e 2015 I
(c) 2.5 kg (d) None of these (a) 2 : 1 (b) 7 : 3 (c) 4 : 3 (d) 8 : 3
108 CDS Pathfinder

61. If a : b = 3 : 5 and b : c = 7 : 8 , then 2a : 3b : 7c is 64. The annual incomes of two persons are in
equal to e 2015 II the ratio 9 : 7 and their expenses are in the ratio
(a) 42 : 105 : 320 (b) 15 : 21 : 40 4 : 3. If each of them saves ` 2000 per year, what
(c) 6 : 15 : 40 (d) 30 : 21 : 350 is the difference in their annual incomes?
e 2016 I
62. The speeds of three buses are in the ratio 2 : 3 : (a) ` 4000 (b) ` 4500 (c) ` 5000 (d) ` 5500
4. The time taken by these buses to travel the
a b c
same distance will be in the ratio. e 2015 II 65. If = = , then which of the following is/are
(a) 2 : 3 : 4 (b) 4 : 3 : 2 b c d
(c) 4 : 3 : 6 (d) 6 : 4 : 3 correct?
b3 + c3 + d3 d a 2 + b2 + c2 a
63. In a mixture of milk and water of volume 30 L, I. = II. =
a3 + b3 + c3 a b2 + c2 + d 2 d
the ratio of milk and water is 7 : 3. The quantity
of water to be added to the mixture to make the Select the correct answer using the code given
ratio of milk and water 1 : 2 is e 2015 II below. e 2016 I
(a) 30 (b) 32 (a) Only I (b) Only II
(c) 33 (d) 35 (c) Both I and II (d) Neither I nor II

ANSWERS
1 b 2 a 3 c 4 b 5 c 6 a 7 d 8 d 9 a 10 b
11 c 12 b 13 d 14 c 15 c 16 b 17 d 18 c 19 b 20 b
21 b 22 b 23 b 24 a 25 c 26 c 27 a 28 d 29 b 30 c
31 d 32 c 33 d 34 a 35 b 36 b 37 c 38 d 39 c 40 a
41 b 42 a 43 b 44 a 45 d 46 b 47 d 48 c 49 a 50 c
51 c 52 c 53 b 54 a 55 d 56 a 57 d 58 a 59 a 60 b
61 c 62 d 63 c 64 a 65 a

HINTS AND SOLUTIONS 


1. (b) The required compounded ratio is 5. (c) Let the third proportional to 9 and 8. (d) Let the consequent be x.
2a 7a 3a a3 12 be x. ∴ 35: x = 7:8
× × = = a 3 : 84b 3
6 b 49 b 12 b 84 b 3 ∴ 9 : 12 : : 12 : x 35 7
Then, = [by given condition]
9 12 x 8
⇒ = ⇒ 9x = 144
2. (a) The subduplicate ratio of 12 x 35 × 8
⇒ 7x = 35 × 8 ∴ x = = 40
16x4 : 625 y 6 is 144 7
∴ x= = 16
9 9. (a) Given, x : y = 1 : 3, y : z = 5 : k,
16x4 4x 2
= = 4x2 : 25 y 3
6 25 y 3 6. (a) Mean proportional between z : t = 2 : 5 and t : x = 3 : 4
625 y
( 2 + 3 ) and (8 − 48 ) x y z t
5 2 × × × =1
3. (c) Duplicate ratio of 5 : 8 = = (2 + 3 ) (8 − 48 ) y z t x
82
52 x+5 x+5 1 5 2 3 1
⇒ = ⇒
25
= = (2 + 3 ) 4( 2 − 3 ) ⇒ × × × = 1⇒ k =
8 2 3x + 4 64 3x + 4 3 k 5 4 2
10. (b) Given, = 25  
x y
⇒ 25(3x + 4) = 64 ( x + 5) = 2 (2 + 3) (2 − 3)
y  x
⇒ 75x + 100 = 64x + 320
= 2 ( 4 − 3) = 2 × 1 = 2 x2 25 x 5
⇒ 75x − 64x = 320 − 100 ⇒ = ⇒ = or , 5 : 1
[Q ( a − b )( a + b ) = a − b ]
2 2
y2 1 y 1
⇒ 11x = 220
220 7. (d) Let fourth proportional be x, then 11. (c) According to the question,
∴ x= = 20
11 7 : 11 : : 14 : x p % of x = t ( q% of y )
7 14 xp yq
4. (b) Given, x : 6 : : 5 : 3 ⇒ = ⇒ 7x = 11 × 14 ⇒ = ×t
x 5 30 11 x 100 100
∴ = ⇒ 3x = 30 ⇒ x = = 10 ⇒ x = 2 × 11
6 3 3 ∴ x : y = qt : p
∴ x = 22
MATHEMATICS Ratio and Proportion 109

4ay − 3bx 17 17 10 170 ⇒ b 3 = 28 × 28 × 224


12. (b) ⇒ = × =
5ax − 2by [dividing numerator and 30 30 10 300 ⇒ b = 56
denominator by by] 11 11 20 220
and = × = Again from Eq. (i) say,
4ay 3bx 4  a  − 3  x  15 15 20 300 28 × 28
−     105 7 a= = 14
by by  b  y As, is the smallest, so or, 7 : 20 is 56
= = 300 20
5ax 2by a  x
− 5     − 2 the smallest ratio. 22. (b) As, A : ( B + C ) = 2 : 9 [given]
by by  b  y Sum of ratios = 2 + 9 = 11
17. (d) Let the numbers be A and B.
4  − 3  8 − 9
2 3 2
∴ 0.5 A = 0.07B [by given condition] So, A ’s part = 770 × = ` 140
3  4 3 4 11
= = ⇒
A 0.07
= =
7
i.e. 7 : 50
5    − 2
2 3 5 1
−2 B 0.5 50 23. (b) Given, x ∝ y and x ∝ 2
 3   4 2 z
32 − 27 5 18. (c) Let the ratio be x and ( x + 40). y
Now, x ∝ 2 ⇒ x = 2
ky
12 12 5 2 5 x 2 z z
= = = × = Then, by given condition, =
5−4 1 12 1 6 x + 40 7 Q x = 10 , y = 4 and z = 14
2 2 k⋅4 1960
x3 + 3x 341 ⇒ 7x = 2x + 80 ⇒ x = 16 ∴ 10 = ⇒k = = 490
13. (d) Given, 2 = 196 4
3x + 1 91 So, the required ratio is 16 : 56. Now, z = 7 and y = 16, then
By componendo and dividendo rule, 3 5
19. (b) Given, P : Q = : …(i) 490 × 16
x3 + 3x + 3x2 + 1 341 + 91 432 5 7 x= = 160
= = 3 2 7×7
x3 + 3x − 3x2 − 1 341 − 91 250 and Q:R= : …(ii)
4 5 m + 3n + m − 3n
x
( x + 1)3 216 3 3 5 3 24. (a) =
⇒ = From Eq. (i), P : Q = × : × 1 m + 3n − m − 3n
( x − 1)3 125 5 4 7 4
[Q ( a + b )3 = a 3 + b 3 + 3ab( a + b )] 9 15 By componendo and dividendo rule,
= : …(iii)
On cube roots both sides, we get 20 28 x+ 1 m + 3n + m − 3n
x+ 1 6 3 5 2 5 =
⇒ = ⇒ 5( x + 1) = 6( x − 1) From Eq. (ii), Q : R = × : × x−1 m + 3n + m − 3n
x−1 5 4 7 5 7
15 2 + m + 3n − m − 3n
∴ x = 11 = : …(iv)
28 7
14. (c) Let the number be x − m + 3n + m − 3n
From Eqs. (iii) and (iv),
49 + x 3
∴ = 9 15 2 x+ 1 m + 3n
68 + x 4 P :Q: R = : : ⇒ =
20 28 7 x−1 m − 3n
⇒ 196 + 4x = 204 + 3x
∴ x=8 20. (b) Let two numbers be 3x and 5x, On squaring both sides, we get
respectively. ( x + 1)2 m + 3n
15. (c) Let the three parts be A , B and C . =
By given condition, According to the question, ( x − 1)2 m − 3n
1 1
A= B= C =x
1
[say] (3x − 9) : ( 5x − 9 ) :: 12 : 23 Again applying componendo and
2 3 6 3x − 9 12 dividendo rule,
1 ⇒ =
∴ A = x ⇒ A = 2x 5x − 9 23 ( x + 1)2 + ( x − 1)2 m + 3n + m − 3n
2 =
1 ⇒ 69x − 207 = 60x − 108 ( x + 1)2 − ( x − 1)2 m + 3n − m + 3n
B = x ⇒ B = 3x 69x − 60x = 207 − 108 ⇒ 9x = 99 2( x + 1)
2
2m
3 ⇒ =
1
C = x ⇒ C = 6x ∴ x = 11 2 ( 2x) 2(3n)
6 ∴ Second number = 5x = 5 × 11 = 55 x2 + 1 m
As, A + B + C = 1870 ⇒ = ⇒ 3n( x2 + 1) = 2mx
21. (b) Let the numbers are a and b and its 2x 3n
[by given condition]
mean proportional is x ∴ 3nx2 − 2mx + 3n = 0
∴ 2x + 3x + 6x = 1870
∴ a : x :: x : b p+ r
⇒ 11x = 1870 25. (c) Given, p + r = 2q ⇒ 2 =
a : 28 : : 28 : b (Q x = 28) q
1870
∴ x= = 170 ⇒ 282 = ab 1 1 2
11 …(i) and + =
q s r
∴ Third part i.e. C = 6 × 170 = 1020 Again, a : b : : b : 224
1 1 p+ r s + q p+ r
b2 ⇒ + = ⇒ =
16. (b) LCM of 20, 25, 30, 15 is 300. ⇒ b 2 = a × 224 ⇒ a = …(ii) q s qr sq qr
7 7 15 105 224
So, = × = ⇒ r ( s + q ) = s( p + r )
20 20 15 300 From Eq. (i) say, p r
⇒ rq = sp ⇒ =
13 13 12 156 b2
⇒ = × = 282 = ×b q s
25 25 12 300 224 ∴ p: q = r: s
110 CDS Pathfinder

26. (c) As, given q 2 = pr 10 37. (c) Given, x ∝ y m ...(i)


when y = and x = 3 ,
p2 − q2 + r2 p2 − q2 + r2 3 y ∝ zn ...(ii)
= 10 m
then = 3l + ⇒ 9 l + m = 10…(iii) x ∝z p
p −2 − q −2 + r −2 1

1
+
1 3 3 ...(iii)
p2 pr r 2 On solving Eqs. (ii) and (iii), On putting the values of x and y from
p 2 − q 2 + r 2 ( p 2 r 2 )( p 2 − q 2 + r 2 ) l = 2 and m = −8 Eqs. (ii) and (iii) in Eq. (i), we get
= = z p ∝ (z n )m ⇒ z p ∝ z mn
r 2 − pr + p 2 ( r 2 − pr + p 2 ) From Eq. (i),
8
y = 2x −
2 2
p r x ∴ p = mn
1 1 1 38. (d) Let the initial and final salary be
( p 2 r 2 )( p 2 − pr + r 2 ) 32. (c) Given, A : B : C = : :
= 5 6 10 ` 22x and ` 25x, respectively.
( r 2 − pr + p 2 )
= 6:5:3 and let initial and final number of
= p r = ( pr )2 = ( q 2 )2 = q 4
2 2
6
∴ Share of A = × 8400 employees be 3y and 2y, respectively.
27. (a) Let monthly incomes of A ’s and B’s 6+5+3
∴ Present bill = Final salary × Final
be 4x and 3x respectively 6
= × 8400 = ` 3600 number of employees
Each saving = ` 600 14 5000 = 25x × 2 y
Q Income − saving = Expenditure 33. (d) Let the amount be ` x. 5000
⇒ = xy ⇒ xy = 100
4x − 600 3 5x 5 50
∴ = In first condition,Q’s part = = x
3x − 600 2 5+3 8 ∴ Original bill = Initial salary × Initial
⇒ 2 ( 4x − 600) = 3 (3x − 600) number of employees
In second condition, Q’s part
= 22x × 3 y
8x − 1200 = 9x − 1800 3x 3
= = x = 66 xy = 66 × 100 = ` 6600
⇒ 1800 − 1200 = 9x − 8x ⇒ x = 600 2+3 5
39. (c) Given,
∴ A ’s income = 4x = 4 × 600 = ` 2400 5 3
By given condition, x − x = 10 A : B = x : 8 and B : C = 12 : z
28. (d) Let three numbers be 3x, 2x and 5x, 8 5 A A B x 12 3x
respectively. x ⇒ A :C = = × = × =
⇒ = 10 ⇒ x = ` 400 C B C 8 z 2z
According to the question, 40
But A :C = 2: 1
(3x)2 + ( 2x)2 + ( 5x )2 = 1862 34. (a) Remaining mixture = 20 − 6 = 14 L 3x 2
⇒ = ⇒ x:z = 4:3
4 2z 1
⇒ 9x2 + 4x2 + 25x2 = 1862 Quantity of milk = 14 × =8L
1862 7 40. (a) Let the number of coins of ` 1, 50
⇒ x2 = = 49 ∴ x =7 ∴ Water contents = 14 − 8 = 6 L paise and 10 paise be 3x, 8x and 10x,
38
Q Milk in new mixture respectively
Hence, the numbers are 3 × 7, 2 × 7 and
= 8 + 6 = 14 L and water = 6 L According to the question,
5 × 7 i.e. 21, 14 and 35.
3x 8x 10x
1 ∴ Ratio of milk and water in new + + = 112
29. (b) Speed ∝ 1 2 10
Time 14 7
mixture = = ⇒ 3x + 4x + x = 112
1 1 1
∴ Required ratio = : : = 3 : 4 : 6 6 3 112
4 3 2 ⇒ x= = 14
Hence, the required ratio 7 : 3. 8
30. (c) Let the ages of A and B be 2x yr and 35. (b) Let x be the part of milk in new ∴ Number of 50 paise coins
5x yr, respectively. mixture. = 14 × 8 = 112
After 8 yr age of A = ( 2x + 8) yr By law of mixture, 41. (b) Let the number of boys and girls be x
and age of B = ( 5x + 8) yr 3 and y , respectively.
2x + 8 1 −x
According to the question, = 8 3 3 3 ∴ Total number of students = ( x + y )
5x + 8 2 ∴ = ⇒ − 2x = 3x −
1 2 4 4 According to the question,
x−
⇒ 4x + 16 = 5x + 8 4 Number of boys = Number of girls
⇒ x=8 ⇒ 5x = =
6 3
∴ x=
3 + Total Number of students × 12%
∴ Difference between their present ages 4 2 10 x = y + ( x + y ) × 12%
= 5x − 2x = 3x = 3 × 8 = 24 yr Required ratio =
3 3
= or, 3 : 7 ( x + y ) × 12
x− y =
31. (d) By given condition, 10 − 3 7 100
m 36. (b) Let quantities of milk and water be ⇒ 25x − 25 y = 3x + 3 y
y = lx + …(i)
x 5x and x L, respectively. ⇒ 22x = 28 y
5x 5
where, l and m are proportionality According to the question, = ∴ x : y = 14 : 11
constants. x+5 2
42. (a) Let C gets ` x .
when y = 6 and x = 4, ⇒ 10x = 5x + 25
m Now, by given conditions,
then 6 = 4l + ∴ x=5
4 Hence, the quantity of milk in the Amount of B = ` ( x + 8)
⇒ 16 l + m = 24 …(ii) original mixture = 5 × 5 = 25 L Amount of A = ` ( x + 15)
MATHEMATICS Ratio and Proportion 111

∴ x + ( x + 8) + ( x + 15) = 53 II. On putting y = 6 in Eq. (ii), we get 53. (b) Let cost price of milk be ` x per kg.
⇒ x = 10 36 Then, according to the rule of mixture,
x = =1
∴ A : B : C = ( 10 + 15) : ( 10 + 8) : 10 36 22 : ( x − 22) = 15 : 3
= 25 : 18 : 10 x =1 22 15
⇒ =
Both statements I and II are correct. x − 22 3
43. (b) Let Q join for x months. a c e 1
= = = 22
∴ Ratio of capital 47. (d) Given that, ⇒ =5
= 2525 × 12 : 1200 × x
b d f 2 x − 22
b d f ∴ x = ` 26.40
= 2525 : 100x = 101 : 4x ⇒ a= , c= , e=
2 2 2
101 54. (a) Total number of inhabitants = 1935000
∴ P’s profit = × 1644  3b + 5d + 7 f 
101 + 4x 3a + 5c + 7e   Total number of females = 935000
I. = 2 2 2 
101 × 1644 3b + 5d + 7 f  3b + 5d + 7 f  ∴ Total number of males
⇒ 1212 =  
101 + 4x = 1935000 − 935000 = 1000000
1

1
=
1 (3b + 5d + 7 f ) 1 ∴ sex ratio =
935000
× 1000 = 935
137 101 + 4x = 2 = 1000000
3b + 5d + 7 f 2
⇒ 4x = 36 ⇒ x=9 55. (d) Given, ratio of boys to girls = 7 : 5
b2 d 2 f2
Hence, Q joined for 9 months i.e. he + + and total number of students = 2400
a 2 + c2 + e 2 4 = 1
joined after 3 months. II. = 42 4
Sum of ratio = 7 + 5 = 12
b2 + d 2 + f 2 b +d + f2
2
4
44. (a) Originally, let the numbers of seats 5
∴ Number of girls = × 2400 = 1000
for Mathematics, Physics and Biology ∴ Neither I nor II correct. 12
be 5x, 7x and 8x, respectively. 48. (c) I. 4 leaps of cat = 3 leaps of dog 56. (a) Given, A : B = 2 : 3, B : C = 5 : 7
Number of increased seats are (140% of 3
⇒ 1 leap of cat = leap of dog and C : D = 3 : 10
5x) (150% of 7x) and (175% of 8x). 4 A A B C
Required ratio can be obtained as ∴ = × ×
Cat takes 5 leap for every 4 leaps of dog. D B C D
⇒  × 5x  :  × 7x 
140 150 ∴ Required Ratio 2 5 3 1
 100   100  = × × =
= (5 × cat’s leap) : (4 × dog’s leap) 3 7 10 7
:  × 8x 
175
=  5 × dog' s leap : (4 × dog’s leap) ∴ A : D = 1: 7
3
 100   4 
 21  57. (d) Given that, ( x + y ) : ( x − y ) = 3 : 5
⇒  7x : x : 14x  × 2 = 15 : 16 x+ y 3
 2  ∴ =
⇒ ( 14x : 21x : 28x ) ÷ 7x Thus,
Speed of cat
=
15 x− y 5
Speed of dog 16 On applying componendo and dividendo
∴ 2:3: 4
Distance (cat) s (cat) × t rule, we get
II. =
45. (d) Let the number of passengers
Distance (dog) s (dog) × t (x + y) + (x − y) 3 + 5
travelling by class I and class II be x and =
(x + y) − (x − y) 3 − 5
50x respectively. s (cat) × 30  s (cat) 15 
= = =  2x 8 x
Then, amount collected from class I and s (dog) × 30  s (dog) 16  ⇒ = ⇒ = −4
II will be ` 3 × x and ` 50x respectively. 2 y −2 y
Thus, both statements I and II are ∴ x = − 4y
Given, 3x + 50x = 1325 correct.
But it is given , xy = positive
53x = 1325 ⇒ x = 25 49. (a) Number of people having ∴ − 4 y × y = positive
∴ Amount collected from class II characteristic X
⇒ − 4 y 2 = positive, which is not possible.
= 50 × 25 = ` 1250 = 10 + 30 = 40
Hence, no real solution for x and y exists.
1 k Number of people having characteristic Y
46. (b) As x ∝ 2 ⇒ x = 2 …(i) 58. (a) Given, ratio of ages of A and B i.e.
y y = 10 + 20 = 30 A : B = 2:5
If x = 1 Required ratio = 40:30 = 4:3. Ratio of ages of B and C i.e.
and y=6 50. (c) Fresh grapes contain 10% pulp. B :C = 3 : 4
2:5
k ∴20 kg fresh grapes contain 2 kg pulp. ∴ Ratio of ages of A, B and C =
1= 2 ⇒ k = 36
6 3:4
Dry grapes contain 80% pulp.
On putting the value of k in Eq. (i), we
2 kg pulp would contain = 2 × 3 : 3 × 5 : 5 × 4 = 6 : 15 : 20
get
36 2 20 59. (a) Let height of tree be h ft and age be
x = 2 …(ii) = = 25
. kg dry grapes.
08
. 8 a yr.
y
36 51. (c) Refer to question 12 Then, according to the question,
I. On putting y = 3 in Eq. (ii), x =
9 h∝ a ⇒ h=k a ..(i)
52. (c) Refer to question 20
x =4 where, k is a constant.
∴ Their product = 18 × 27 = 486
112 CDS Pathfinder

If age = 9 yr, then height = 4 ft 40 a ⇒ 9 + y = 42


From Eq. (i), we get 4 = k 9 Now, 2a : 3b : 7c = 2a : 5a :
3 ∴ y = 33 L
4
⇒ 4 = k ×3 ∴ k = = 2:5:
40
= 6 : 15 : 40
3 64. (a) Refer to question 27.
4 3
If age = 16 yr, then h = k 16 = × 4 a b c
62. (d) Let the distance be x. 65. (a) = = = k (say)
3 b c d
Q k = 4  Now, ratio of time taken to travel the
 ⇒ a = bk , b = ck, c = dk
3  distance by each bus is
x x x 12 12 12 ∴ a = dk 3 , b = dk 2 , c = dk
16 1 ⇒
∴ h= ft = 5 = 5 ft 4 inch : : : : b 3 + c3 + d 3
3 3 2 3 4 2 3 4 I. 3
a + b 3 + c3
60. (b) Refer to question 34. ⇒ 6:4:3
b 3 + c3 + d 3
a 3 63. (c) Since, milk = 7x and water = 3x =
61. (c) We have, = ( bk ) + ( ck )3 + ( dk )3
3
b 5 Then, 7x + 3x = 30 ⇒ 10x = 30
1 ( b 3 + c3 + d 3 ) 1 d
b=
5a
and
b 7
= ∴ x =3 = = =
k 3 ( b 3 + c3 + d 3 ) k 3
Q
3 c 8 a
So, milk = 21 L and water = 9 L
8 8 5a 40 a Hence, statement I is correct.
⇒ c= b= × = Let y L of water to be added.
a 2 + b 2 + c2 b
7 7 3 21 21 1 II. Similarly 2 = k2 =
Then, = b + c2 + d 2 d
5a 40 a 9+ y 2
Q a:b:c= a: :
3 21 Hence, statement II is incorrect.
13
MATHEMATICS > Logarithm 113

LOGARITHM
Usually (2-3) questions have been asked from this chapter. Generally questions are based on
fundamental rules of logarithm. It is especially used to solve complicated mathematical calculations.
Applications of square, cube, surds and indices are extremely used here.

DEFINITION
If ‘a’ is a positive real number, other than 1 and ‘b’ is a rational number such that a b = N, then we say
that logarithm of N to base ‘a’ is b or ‘b’ is the logarithm of N to base ‘a’, written as log a N = b
So, a b = N ⇔ log a N = b , e.g. 7 0 = 1 ⇔ log 7 1 = 0, 811/4 = 3 ⇔ log 81 3 = 1 / 4

EXAMPLE 1. Which one of the following has the value of x = 4?


3
a. log2 x = 3 b. log5 x = 3 c. log81 x = d. log 2
x=4
2
Sol. d. a. log 2 x = 3 ⇔ x = 23 = 8 b. log 5 x = 3 ⇔ x = 53 = 125 [by definition]
c. log 81 x = 3 / 2 ⇔ x = (81) 3/ 2
= (3 )4 3/ 2
, ⇔ x=3 6
⇔ x = 729
d. log 2
x = 4 ⇔ x = ( 2) 4, x = ( 21/ 2) 4 = 22 = 4

EXAMPLE 2. If log5 y = x and log 2 z = x, then the value of 20 x in terms of y and z is


a. zy b. z2 y c. xy 2 d. z/y
Sol. b. Given, log 5 y = x and log 2 z = x ⇒ y = 5 and z = 2
x x
…(i)
Now, 20 = ( 2 × 5) = 2
x 2 x 2x
× 5 = (2 ) ⋅ 5 = z ⋅ y
x x 2 x 2
[from Eq. (i)]
∴ 20 = z ⋅ y
x 2

Fundamental Rules of Logarithms


Rule 1 If m and n are positive rational number, then log a ( mn) = log a m + log a n
If m1 , m2 , m3 , . . . , mn are positive rational numbers, then
log a ( m1 m2 m3 . . . mn ) = log a m1 + log a m2 + . . . + log a mn
114 CDS Pathfinder

EXAMPLE 3. The value of log10 (15) is Sol. b. log 3 4 ⋅ log 4 5 ⋅ log 5 6 ⋅ log 6 7 ⋅ log 7 8 ⋅ log 8 9
a. log10 2 − log10 5 b. log10 5
log 4 log 5 log 6 log 7 log 8 log 9
c. log10 3 + log10 5 d. log10 2 + log10 5 = × × × × ×
log 3 log 4 log 5 log 6 log 7 log 8
Sol. c. log10 15 = log 10 ( 3 × 5) = log10 3 + log10 5
log 9 log 32 2 log 3
= = = =2
Rule 2 If m and n are positive rational numbers, then log 3 log 3 log 3

 m
log a   = log a m − log a n
 n SOME USEFUL RESULTS
logarithm of 1 with any base is always zero i.e.
EXAMPLE 4. The value of x in
l

loga 1 = 0
log x − log (x − 1) = log 3 is
l logarithm of base to itself is always 1 i.e. loga a = 1
a. 2/3 b. 3/2 c. 1/2 d. 1/4
l x > y ⇒ loga x > loga y for a > 1
Sol. b. log x − log ( x − 1) = log 3
l x > y ⇒ loga x < loga y for 0 < a < 1
x x 1
⇒ log = log 3 ⇒ =3 for x > 0, 1 ≠ a > 0, loga n ( x ) = loga x
x−1 x−1
l

n
⇒ x = 3 ( x − 1) ⇒ x = 3x − 3 ⇒ −2x = − 3 l for 1 ≠ a ,b > 0, x > 0, a logb x = x logb a
If ‘a’ is a positive real number and n is a positive
∴ x = 3/ 2 l

rational number, then


Rule 3 If m and n are positive rational numbers, then, x
(i) log y n x = loga n .
log a ( mn ) = n log a m a
y
(ii) a log a n = n
EXAMPLE 5. The value of x in
(iii) loga n = log n 2 = log n 3 = loga m n m
log x 4 + log x 16 + log x 64 = 12 is a2 a3

a. 1 b. 2 c. 3 d. 4
Sol. b. log x 4 + log x 16 + log x 64 = 12 Common Logarithm and Natural
⇒ log x 22 + log x 24 + log x 26 = 12 Logarithm
⇒ 2 log x 2 + 4 log x 2 + 6 log x 2 = 12 Logarithm can have any positive base other than 0. But
⇒ 12 log x 2 = 12 ⇒ log x 2 = 1
there are two most important base which are generally used.

∴ x=2
1. Common Logarithm Logarithm to the base ‘10’ is
called common logarithm. It is also called Brigg’s
log 256 logarithm.
EXAMPLE 6. The value of x in = log x is
log 16 e.g. log 100 = log 10 100 = 2
a. 1 b. 10 c. 100 d. 0 log 1000 = log 10 1000 = 3
log 256 log 162 2 log 16 2. Natural Logarithm Logarithm to the base e is
Sol. c. = = =2
log 16 log 16 log 16 called natural logarithm. It is also called Napier
⇒ log x = 2 ⇒ log10 x = 2 [Q log x = log10 x] logarithm. log e x is usually denoted by ln x. e is
∴ x = 10 = 100
2 irrational number between 2 and 3.
Rule 4 If m is a positive rational number and a, b are ln x = y ⇒ e y = x
positive real numbers i.e. a ≠ 1 and b ≠ 1, then

log a m =
log b m ANTILOGARITHM
log b a The positive number ‘a’ is called the antilogarithm of a
EXAMPLE 7. The value of number b, if log a = b. If ‘a’ is antilogarithm of b, it is
log3 4 ⋅ log4 5 ⋅ log5 6 ⋅ log6 7 ⋅ log 7 8 ⋅ log8 9 is written as a = antilog b.
So, a = antilog b ⇔ log a = b
a. 1 b. 2 c. −1 d. None of these
MATHEMATICS > Logarithm 115

Characteristic and Mantissa first significant figures is at nth place, where n is the
characteristic.
of a Logarithm
The logarithm of positive real number ‘n’ consists of EXAMPLE 8. The characteristic of the logarithm of
two parts the number 0.00000014 is
1. The integral part is known as the characteristic. It is a. 1 b. 7
always an integer (positive, negative or zero). c. −7 d. None of these
1
2. The decimal part is called as the mantissa. The Sol. c. 0.00000014 = 1.4 × = 1.4 × 10−7
mantissa is never negative and is always less than 10000000
one. ∴ The characteristic of log 0.00000014 = − 7

To Find the Characteristic EXAMPLE 9. The characteristic of the logarithm of


the number 8.188 is
Case I The characteristic of the log of a number
greater than 1 is positive and numerically one less than a. 2 b. 1
the number of digits to the left of the decimal part. c. −1 d. 0

e.g. Sol. d. 8.188 = 8.188 × 1 = 8.188 × 100


Value Characteristic
∴ The characteristic of log 8.188 = 0
log 3.257 1− 1 = 0
log 32.57 2 −1= 1 EXAMPLE 10. The characteristic of the logarithm of
log 3257
. 3−1= 2 the number 566.37 is
a. 2 b. −2
Case II The characteristic of log of a number less than c. 4 d. None of these
1 is negative and numerically one more than the number Sol. a. 566.37 = 56637
. × 102
of zeroes immediately after the decimal point. It is
represented by bar over the digit. ∴ The characteristic of log 566. 37 = 2
The bar over the characteristic indicates that it is negative. EXAMPLE 11. The characteristic of the logarithm of
e.g. the number 313 is
Value Characteristic
a. 3 b. 2
log 0.3257 − ( 0 + 1) = −1 i.e. 1 c. 1 d. 0
log 0.03257 − (1 + 1) = −2 i.e. 2
Sol. b. 313 = 313
. × 102
log 0.003257 − (2 + 1) = −3 i.e. 3
∴ The characteristic of log 313 = 2

Rules for Inserting Decimal Point


Two rules are used for inserting a decimal point. SOME IMPORTANT POINTS
Rule 5 When the characteristic of the logarithm is 1. The base of a logarithm is never taken as zero and
negative.
positive, we insert the decimal point after the ( n + 1) th 2. Log of negative integers are not defined and also
digit, where n is the characteristic. loge 0 is not defined.
3. Logarithmic function is positive as well as negative
Rule 6 When the characteristic of the logarithm is
but exponential function is always positive.
negative, we insert the decimal point such that the
116 CDS Pathfinder

PRACTICE EXERCISE
1. What is the value of log100 0. 1 ? 1 1 1
16. The value of + + is
(a) 1/2 (b) –1/2 (c) –2 (d) 2 logxy ( xyz ) log yz ( xyz ) logzx ( xyz )
(a) xyz (b) 2 (c) 0 (d) 1
2. The value of 3 log 3 + 2 log 2 is
(a) log 108 (b) log 106 (c) log 109 (d) None of these 17. The value of
1 1 1
1 + + is
3. If loga 2 = , then the value of a is 1 + logx ( yz ) 1 + log y ( xz ) 1 + logz ( xy )
6
1
(a) ( 2 )6 (b) (6)1/ 2 (c) 3 (d) −6 (a) 1 (b) (c) x = yz (d) 0
xy2
4. If log3 x = − 2, then the value of x is 18. If log4 ( x 2 + x ) − log4 ( x + 1) = 2, then the value of x
1 1 1 1
(a) (b) − (c) (d) − is
9 9 8 8 (a) 4 (b) 8 (c) 16 (d) 1
5. Find the logarithm of 1728 to the base 2 3. 19. If log4 x + log2 x = 6, then the value of x is
(a) 3.124 (b) 3.1732 (c) 6 (d) 5 (a) 16 (b) 4 (c) 2 (d) 1
6. What is the value of 20. Given log10 2 = 0.3010, the value of log10 5 is
(log1/ 2 2)(log1/ 3 3)(log1/ 4 4) . . .(log1/ 1000 1000)? (a) 0.6990 (b) 0.6919 (c) 0.6119 (d) 0.7525
(a) 1 (b) –1 (c) 1 or –1 (d) 0 x y
21. If log + log = log ( x + y ), then
7. What is the value of y x
1 (a) x + y = 1 (b) x − y = 0 (c) x − y = 1 (d) x = y
log 10 25 − 2log 10 3 + log 10 18?
−5
2 22. The characteristic in log 6.7482 × 10 is
(a) 2 (b) 3 (c) 1 (d) 0
(a) 6 (b) − 4 (c) 5 (d) − 5
8. What is the value of [log 10 (5 log 10 100)] 2 ?
23. If 10x = 1.73 and log10 1730 = 3. 2380, then x
(a) 4 (b) 3 (c) 2 (d) 1
equals to
9. The value of log y x ⋅ logz y ⋅ logx z is (a) 2.380 (b) 0.2380 (c) 2.2380 (d) 1.380
2x + 3 x −1
(a) log xyz (b) xyz (c) 1 (d) 0 24. If 2 =6 , then x equals
10. The value of log3 ( 27 × 9 × 9 ) is 4 3 4 log 2 + log 3 3 log 2 + 2 log 3
(a) (b)
1 1 1 log 3 − log 2 log 3 − 2 log 2
(a) 4 (b) 4 (c) 8 (d) 4 log 48
3 3 6 (c) (d) None of these
log 7
11. The value of log2 [log2 log2 log2 ( 65536)] is
25. The value of 10log10 m + 2 log10 n + 3 log10 p is
(a) 8 (b) 16 (c) 4 (d) 1
(a) m2 np3 (b) mn2 p3 (c) m3 np2 (d) None of these
12. What is the value of [log 13(10)]/[log 169(10)] ?
26. Given that log10 2 = 0.3010, log10 3 = 0.4771 and
1
(a) (b) 2 (c) 1 (d) log10 13 108
2 log10 7 = 0.8491, then log10 is equal to
7
13. What is the value of (a) 2.6123 (b) 1.6088 (c) 1.6320 (d) 2.4558
1 
 log 10 125 − 2log 10 4 + log 10 32 + log 10 1 ? 27. If a , b and c are three consecutive integers, then
3 
1 2 log ( ac + 1) is equal to
(a) 0 (b) (c) 1 (d)
5 5 (a) log (2b ) (b) (log b )2 (c) 2 log b (d) None of these
14. If log r 6 = m and log r 3 = n , then what is 28. If logr p = 2, logr q = 3, then the value of log p q is
log r (r / 2) equal to? 1 2 3
(a) m − n + 1 (b) m + n − 1 (c) 1 − m − n (d) 1 − m + n (a) (b) (c) (d) 6
3 3 2
3 4 5 x log x
15. What log 10   + log 10   + log 10
is   +L 29. If log x 2 y 2 = a and log = b, then is equal to
2 3 4 y log y
upto 8 terms equal to? a − 3b a + 3b a + 2b a − 2b
(a) (b) (c) (d)
(a) 0 (b) 1 (c) log10 5 (d) None of these a + 2b a − 2b a − 2b a + 3b
MATHEMATICS > Logarithm 117

30. If log10 5 = 0.70, then log5 10 is equal to 37. Consider the following statements
(a) 1.35 (b) 1.40 (c) 1.43143 (d) 1.56 I. (log10 0.1)2 + log10 10 ⋅ log10 100 = 3
II. log10log10 10 = 1 III. log10 10 + log10 10 = 1
31. The value of
 1  1  1 Which of the statements given above are correct?
log3 1 +  + log3 1 +  + log3 1 + 
 3  4  5 (a) I and III (b) II and III
 1 (c) I and II (d) All are correct
+ . . . + log3 1 +  is
(a) −1 + 2 log 3 5
 24 x
) ... ∞
(b) 2 38. If y = ( a x )( a , then which one of the following
(c) 3 (d) 4 is correct?
32. If log ( x + y ) = log x + log y and x = 1.1568, then y (a) log y = xy log a (b) log y = x + y log a
is equal to (c) log y = y + x log a (d) log y = ( y + x) log a
(a) 7.3776 (b) 7 .3776 (c) 5.3776 (d) 5.3116
33. If log8 x + log4 x + log2 x = 11. Then, the value of x PREVIOUS YEARS’ QUESTIONS
is 39. What is the logarithm of 0.0001 with respect to
(a) 128 (b) 16 (c) 32 (d) 64 base 10? e 2012 II
(a) 4 (b) 3 (c) − 4 (d) − 3
34. What is the value of log y x5 logx y 2 logz z3 ?
(a) 10 (b) 30 (c) 20 (d) 60 40. If log10 a = p and log10 b = q , then what is the
value of log10( a pbq )? e 2012 II
35. If (log3 x )(log x 2x )(log2x y ) = logx x 2, then what is
p2
the value of y? (a) p + q
2 2
(b) p − q
2 2
(c) p q 2 2
(d) 2
9 q
(a) (b) 9 (c) 18 (d) 27
2 41. What are the possible solutions for x of the
n
equation x x
= x x , where x and n are positive
36. If log10 2, log10( 2x − 1), log10 ( 2x + 3) are three
integers? e 2015 I
consecutive terms of AP, then which one of the (a) 0, n (b) 1, n (c) n, n2 (d) 1, n2
following is correct?
I. x = 1 II. x = log 2 5
1
42. The value of log10 3125 − 4 log10 2 + log10 32 is
(a) Both I and II (b) Only II 5 e 2016 I
(c) Only I (d) None of these (a) 0 (b) 1 (c) 2 (d) 3

ANSWERS
1 b 2 a 3 a 4 a 5 c 6 b 7 c 8 d 9 c 10 d
11 d 12 b 13 c 14 d 15 c 16 b 17 a 18 c 19 a 20 a
21 a 22 d 23 b 24 a 25 b 26 b 27 c 28 c 29 c 30 c
31 a 32 a 33 d 34 b 35 b 36 b 37 a 38 a 39 c 40 a
41 a 42 b

HINTS AND SOLUTIONS 


0.1 = log  1 2=
1
⇒ a1 / 6 = On comparing both sides, we get x = 6
1. (b) log   3. (a) Q log a 2
100 10 2  10  6
6. (b) (log 2)(log 3)(log 4)
∴ a = ( 2 )6 1/2 1/3 1/4
log   = log ( 10)−1
1 1 1
= . . . (log 1000)
10 
2 10  2 10
4. (a) log x = − 2 ⇒ x = 3 −2 =
1
=
1 1 /1000
 log 2   log 3   log 4 
1 1 3
32 9 =     
= − log 10 = − [Rule 3]  log 1 / 2   log 1 / 3   log 1 / 4 
10
2 2 5. (c) Let log 1728 = x
2 3  
2. (a) 3 log 3 + 2 log 2 = log 33 + log 22  log a 
... 
⇒ ( 2 3 )x = 1728 log 1000 
 log 1/ 1000  Q log b a = log b 
= log 27 + log 4 [Rule 3] Q 1728 = 26 ( 3 )6 = ( 2 3 )6    
= log ( 27 × 4) = log 108 [Rule 1] ( 2 3 )x = ( 2 3 )6 [Rule 4]
118 CDS Pathfinder

 log 2   log 3   log 4  = log 5 − 4 log 2 + 5 log 2 19. (a) Given, log x + log x = 6
=      10 10 10 4 2
 − log 2   − log 3   − log 4  = log 5 + log 2 = log 5 × 2 log x log x
10 10 10 ⇒ + =6
 log 1000  = log 10 = 1 [Rule 1] log 4 log 2
...   10
 − log 1000  14. (d) Given, log r 6 = m and log r 3 = n ⇒
log x
+
log x
=6
Q log r 6 = log r ( 2 × 3) = log r 2 + log r 3 2 log 2 log 2
= ( −1) × ( −1) × ( −1) × . . . × ( −1)
[Q number of terms is odd] ∴ log r 3 + log r 2 = m ⇒ 3 log x = 12 log 2
= −1 ⇒ n + log r 2 = m ⇒ log x = 4 log 2 ⇒ log x = log 24
1 ⇒ log r 2 = m − n ⇒ log x = log 16
7. (c) log 25 − 2 log 3 + log 18
10 10 10 On comparing both sides, we get
2  r
∴ log r   = log r r − log r 2 [Rule 2]
= log 251 / 2 − log 32 + log 18  2 ∴ x = 16
10 10 10
[Rule 3] = 1−m + n 20. (a) log 5 = log
10
10 10
15. (c) log   + log   + log  

= log 5 − log 9 + log 18 3 4 5 2
10 10 10 10   10   10   = log 10 − log 2 = 1 − 0.3010
2 3 4
5 × 18 90 10 10
= log = log + L + 8th term
10
9 10
9 = 0.6990
 n + 2 x y
= log 10 = 1 Q Tn = log   21. (a) Given, log + log = log ( x + y )
10 10  
 n + 1
2 y x
8. (d) [log (5 log 100)] x y
10 10 ⇒ log ⋅ = log ( x + y )
= [log (5 log 102 )]2  10  y x
10 10 ⇒ T = log  
= [log (10 log 10)]2 8 10  9 ⇒ log 1 = log ( x + y )
10 10
= [log ∴ log   + log   + log   ∴ x+ y =1
2 3 4 5
10] [Q log 10 = 1]
10 10
10   10   10  
= 12 = 1 2 3 4 22. (d) The characteristic in
9. (c) log y x ⋅ log z y ⋅ log x z 
+ L + log  
10 log 6.7482 × 10−5 is −5.
10 
log x log y log z 9 23. (b) Given, 10x = 1. 73, x = log 1. 73
= × × =1 10
= log  × × × L ×  [Rule 1]
log y log z log x 3 4 5 10 = log 1730 − log 1000
10 
9
10 10
 log b  2 3 4 = log 1730 − log 103
Q log a b = log a  10 10
= log   = log 5
  10
= 3.2380 − 3 = 0.2380
10 
2 10
10. (d) Let log ( 27 × 4
9× 3
9) = x 24. (a) 22 x + 3
= 6x − 1
3
16. (b) The given expression is
⇒ 3 = 27 ×
x 4
9× 3
9 Taking log on both sides, we get
= log xyz ( xy ) + log xyz ( yz ) + log xyz (zx)
⇒ 3x = 33 × 32 / 4 × 32 / 3 ( 2x + 3) log 2 = ( x − 1) log 6
= log xyz ( xy ⋅ yz ⋅ zx) [Rule 1]
3x = 325 / 6 ⇒ 2x log 2 + 3 log 2
= log xyz ( xyz )2 = 2 log xyz xyz = 2
On comparing both sides, we get = ( x − 1) (log 2 + log 3)
25 1 ⇒ 2x log 2 + 3 log 2 = x (log 2 + log 3)
⇒ x= =4 17. (a) The given expression is
6 6 1 − log 2 − log 3
=
11. (d) log log log log 216 log x ( yz ) + log x x ⇒ x (log 2 − log 3) = − 4 log 2 − log 3
2 2 2 2
= log log log ( 16) [Q log 2 = 1] 1 4 log 2 + log 3
2 2 2 2
+ ∴ x=
= log log log ( 24 ) = log log ( 4) log y ( xz ) + log y y log 3 − log 2
2 2 2 2 2

= log log ( 22 ) = log ( 2) = 1 1


2 2 2 + 25. (b) 10 log 10 m + 2 log
10
n + 3 log
10
p
log z ( xy ) + log z z
log (10) log (10) m + log n 2 + log p3
12. (b) 13 = 13 = 10 log
10 10 10
1 1 1
log
169
(10) log (10) = + + ⇒ 10 log 10 mn 2 p 3
= mn2 p 3 [Q
13 2 log x xyz log y xyz log z xyz
log 10  = p]
log a p
Q log c = log a c 
1 a
= 13
= log xyz x + log xyz y + log xyz z
1 
 ab b  108
log 10
= log xyz xyz = 1 [Rule 1] 26. (b) log = log 108 − log 7
2 13 10
7 10 10

1
= =2 18. (c) log ( x2 + x) − log ( x + 1) = 2 = log 22 × 33 − log 71 / 2
1/ 2 4 4 10 10
 x2 + x  x2 + x
1 ⇒ log   = 2 ⇒ 42 = = 2 log 2 + 3 log 3−
1
13. (c) We have, log 125 − 2 log 4 4  
log 7
 x+ 1 x+ 1 10 10 10
3 10 10 2
+ log 32 + log 1 = 2 × (0. 3010) + 3(0.4771)
10 10 ⇒ 16x + 16 = x2 + x
= log (125) 1/3
− 2 log (2) 2 1
10 10 ⇒ x − 15x − 16 = 0
2 − (0.8491)
2
+ log 5
(2) + 0 [Q log 1 = 0] ∴ x = 16 or x = − 1 [not possible]
10 10 = 0.6020 + 1.4313 − 0.4245 = 1.6088
MATHEMATICS > Logarithm 119

x ...∞
27. (c) Let first integer be a. 11log x 38. (a) Given, y = ( a x )(a )
⇒ = 11 ⇔ log x = 6 log 2
Then, b = a + 1 and c = a + 2 6 log 2
⇒ x = 26 , ∴ x = 64 ∴ y = ( ax ) y
∴ ac + 1 = a ( a + 2) + 1
34. (b) ∴ log y x5 log x y 2 log z z 3 On taking log both sides, we get
= a 2 + 2a + 1 = ( a + 1 )2
= 5 log y x ⋅ 2 log x y ⋅ 3 log z z log y = y log a x
ac + 1 = b 2
[Qlog a b n = n log a b] ∴ log y = xy log a
So, log ( ac + 1) = log b 2 = 2 log b
28. (c) Given, log r p = 2 and log r q = 3 = 5 log y x ⋅ 2 log x y ⋅ 3 × 1 39. (c) Let, log 0.0001 = x
10
log r q 3 5 log x log y  log b  1
= × 2⋅ × 3 Q log a b = Then, x = log
By relation, log q =
p
= log y log x  log a  10
( 10)4
log r p 2
= 5 × 2 × 3 = 30
= log 1 − log ( 10)4
29. (c) Given, log x2 y 2 = a 35. (b) (log x) (log x 2x) (log y ) = log x x2
10 10
3 2x =0−4=−4
[Q log (mn) = log m + log n] log x log 2x log y log x2
⇒ × × = 40. (a) Given, log a = p and log b = q
∴ log x + log y = a
2 2
log 3 log x log 2x log x 10 10
⇒ 2 log x + 2 log y = a …(i) log ( a b ) = log a + log b q
p q p
 log a 
Q log b a = log b 
10 10 10
Also, log x − log y = b …(ii)
  ⇒ log ( a p b q ) = p log a + q log b
On solving Eqs. (i) and (ii), we get 10 10 10
log y 2 log x ∴ log ( a p b q ) = p × p + q × q
a + 2b a − 2b ⇒ = [Q log a b = b log a ] 10
log x = and log y = log 3 log x
4 4 = p2 + q2
log x a + 2b ⇒ log y = 2 log 3 ⇒ log y = log 32
∴ = [Q log m = log n ⇒ m = n]
log y a − 2b 41. (a) Given, x x
= n
xx , where x and n
log 10 ⇒ log y = log 9 are positive integers.
10 1
30. (c) log 10 = = = 143143
. ∴ y=9 On taking log both sides, we get
5 log 5 0.70
10 36. (b) Given that, log 2, log ( 2x − 1) and
[Q log 10 = 1] 10 10 log x x
= log[ n xx ]
10 log ( 2x + 3) are in AP.
10
⇒ x log x = log( xx )1 / n = log xx/n
31. (a) log  1 +  + log  1 + 
1 1 then,
3
3 3 
4 2 log ( 2x − 1) = log 2 + log ( 2x + 3) ⇒
x
x log x = log x
10 10 10
+ log  1 +  + . . . + log  1 +
1 1 ⇒ log ( 2x − 1)2 = log ( 2 x + 1 + 6)
n
3 

5 3 
24 
10 10 x
⇒ ( 2x − 1)2 = ( 2x + 1 + 6) ⇒ x log x − log x = 0
4 5 6 n
= log + log + log + ⇒ 2 + 1 − 2.2x = 2 ⋅ 2x + 6
2x

log x x −  = 0

3 3 3 x
3 4 5 ⇒ 22 x − 4.2x − 5 = 0 ⇒
25  n 
. . . + log ⇒ ( 2 ) − 4 ( 2x ) − 5 = 0
x 2
3
24 log x ≠ 0
⇒ y2 − 4y − 5 = 0
Q
x
= log 4 − log 3 + log 5 − log 4 ∴ y = 5, − 1 ∴ x− =0
3 3 3 3
n
+ log 6 − log 5 Hence, 2x = −1 or 2x = 5 x
3 3
⇒ x=
+ . . . + log 25 − log 24 ∴ x = log ( −1) is not possible n
3 3 2
or x = log (5) ∴
x
=n
= − log 3 + log 25 = − 1 + 2 log 5 2
3 3 3 Then, x = log ( 5 ) is answer. x
32. (a) Given, log ( x + y ) = log x + log y 2
On squaring both sides, we get
Hence, II is correct.
⇒ log ( x + y ) = log xy x2
37. (a) I. [log (0.1)]2 + log 10 ⋅ log 100 =n
10 10 10
⇒ x + y = xy x
= [ − log 10]2 + log 10 ⋅ log 102
⇒y=
x
=
11568
.
=
11568
. 10 10 10
⇒ x 2 − nx = 0
x − 1 11568
. − 1 0.1568 = ( −1)2 + (1) ⋅ 2 log 10
10 ⇒ x( x − n) = 0
= + 1 + (1) ⋅ 2 = 1 + 2 = 3 ∴ x = 0, x = n
= 7.37755 = 7.3776
Hence, statement I is correct.
33. (d) Given, log x + log x + log x = 11 Hence, the possible solution of x is 0, n.
8 4 2 II. log log 10 = log (1) = 0 ≠ 1
log x log x log x 10 10 10
1
⇒ + + = 11 Hence, statement II is incorrect. 42. (b) log 3125 − 4 log 2 + log 32
10 10 10
log 8 log 4 log 2 III. log 10 + log 10 5
10 10
log x log x log x
1
= log 10 + log 10
1 = log [(5)5 ]1 / 5 − log ( 2)4 + log 32
⇒ 3
+ 2
+ = 11 2 10
2 10
10 10 10
log 2 log 2 log 2
1 1 = log 5 − log 16 + log 32
log x log x log x = + =1 10 10 10

⇒ + + = 11 2 2
5 × 32 
3 log 2 2 log 2 log 2 Hence, statement III is correct. = log   = log10 = 1
10 
16 
14
120 CDS Pathfinder

ALGEBRAIC
OPERATIONS
Generally (3-4) questions have been asked from this chapter. Questions which are asked from
this chapter are mostly based on direct identities and factor theorem.

Algebraic Expressions
A combination of constants and variables connected by the four fundamental operations +, −, × and ÷ is
called an algebraic expression.

POLYNOMIALS
A polynomial is an algebraic expression consisting of variables and coefficients, having non-negative
integral powers. e.g. x 3 + 5x 2 − 1, 3x 3 + 4x 2 y + 17 etc.
(i) 3x 2 + 9x − 1 + 7 / x is not a polynomial as it contains a term, namely 7 / x, having negative integral
power of variable x.
(ii) 5x 2 − 7 x 7/2 + x − 1 is not a polynomial as the term −7 x 7/2 contains rational power of variable x.

Degree of a Polynomial
The exponent of the highest degree term in a polynomial, is known as degree of polynomial.
e.g. (i) 4x 3 − 9x 2 + 7 x + 9 is a polynomial with variable x of degree 3.
(ii) 4x 3 y − 3x 2 + 2xy + 1 is a polynomial of degree 4.

Various Types of Polynomials


Polynomials are classified on the basis of degree of polynomial and number of terms which are given
below.
Based on Degree of Polynomial
(i) Constant polynomial A polynomial of degree zero, is called constant polynomial. e.g. f ( x) = 8.
Note The polynomial f( x) = 0 is called the zero polynomial. The degree of zero polynomial is not defined.
MATHEMATICS Algebraic Operations 121

(ii) Linear polynomial A polynomial of degree 1, is • ( a 3 − b 3 ) = ( a − b) ( a 2 + b 2 + ab)


called linear polynomial. e.g. 9x + 7, x − 9, x + 2 etc. • ( a + b + c ) 2 = a 2 + b 2 + c 2 + 2 ( ab + bc + ac )
(iii) Quadratic polynomial A polynomial of degree 2, is • a 3 + b 3 + c 3 − 3abc = ( a + b + c ) ( a 2 + b 2 + c 2
called quadratic polynomial. − ab − ac − bc)
The general form of quadratic polynomial in x is • If a + b + c = 0, then a 3 + b 3 + c 3 = 3abc
ax 2 + bx + c, where a, b and c are real and a ≠ 0. • a 4 + a 2 b 2 + b 4 = ( a 2 + ab + b 2 ) ( a 2 − ab + b 2 )
e.g. x − 7 x + 12, 5 y − 7, z − 4 etc., are all quadratic
2 2 2

polynomials. Methods of Factorisation


(iv) Cubic polynomial A polynomial of degree 3, is These methods are given below
called cubic polynomial.
Factorisation by Taking Out the
The general form of a cubic polynomial in x is
Common Factor
ax 3 + bx 2 + cx + d , where a, b, c and d are real and a ≠ 0.
If each term of an expression has a common factor, take
e.g. 3x 3 + 2x 2 + 7 x − 1 , 8 y 3 + 5 y + 2, x 3 − 8 etc., are out the common factors.
all cubic polynomials.
EXAMPLE 1. Factorise value of 4(3a −2b) 2 − 5(3a −2b) is
(v) Biquadratic polynomial A polynomial of degree 4,
is called a biquadratic polynomial. a. ( 3a − 2b)(12a − 8b − 5) b. ( 2a − 3b)(12a − 8b)
The general form of a biquadratic polynomial in x is c. ( 2a − 3b)( 8 a − 12b − 5) d. None of these
ax 4 + bx 3 + cx 2 + dx + e, where a, b, c, d and e are Sol. a. 4 ( 3a − 2b) − 5( 3a − 2b) = ( 3a − 2b)[ 4( 3a − 2b) − 5]
2

real numbers and a ≠ 0. = ( 3a − 2b)[12a − 8b − 5]


e.g. 6x 4 + 3x 3 + 2x 2 + x + 2, x 4 − 8 etc.,
Factorisation by Grouping
are all biquadratic polynomials.
Sometimes in a given polynomial, it is not possible to
Based on Number of Terms take out a common factor directly. However, on
rearranging the terms of the polynomial and grouping
(i) Monomial A polynomial containing only one term them such that all the terms have a common factor, the
is called a monomial. e.g. 6x 2 polynomial can be easily factorised.
(ii) Binomial A polynomial containing two terms is
called a binomial. e.g. 5x 3 + 7 xy EXAMPLE 2. Factorise x 2 + y − xy − x
(iii) Trinomial A polynomial containing three terms is a. ( x − y )( x + 1) b. ( x − y )( x 2 − 1)
called a trinomial. e.g. 7 x 2 y + 5x + 6 c. ( x + y )( x2 + 1) d. ( x − y )( x − 1)

Sol. d. Method I x2 + y − xy − x = x2 − x + y − xy
FACTORISATION = x ( x − 1) + y(1 − x) = x ( x − 1) − y( x − 1) = ( x − 1)( x − y)
To express a polynomial as the product of other Method II x2 + y − xy − x = x2 − xy + y − x
polynomials of degree less than that of the given = x ( x − y) − 1( x − y) = ( x − y)( x − 1)
polynomial is called as factorisation.
4 4
e.g. x 2 − 49 = x 2 − 7 2 = ( x − 7 )( x + 7 ) EXAMPLE 3. Factorise x 2 + 2 + 4 − 2x −
x x
Important Identities  2  2   2  2 
a.  x −   x + + 2 b.  x −   x + − 2
 x  x   x  x 
• ( a 2 − b 2 ) = ( a + b)( a − b)  2  2 
c.  x +  x + − 2 d. None of these
• ( a + b) 2 = a 2 + b 2 + 2ab and ( a − b) 2 = a 2 + b 2 − 2ab  x  x 
( a + b) 2 − ( a − b) 2 = 4ab
Sol. c.  x2 + 2 + 4 − 2x −  =  x2 + 2 + 4 − 2  x + 
• 4 4 4 2
• ( a + b) 2 + ( a − b) 2 = 2 ( a 2 + b 2 )  x x  x   x
( a + b) 3 = a 3 + b 3 + 3ab ( a + b) 2
=  x +  − 2  x +  =  x +   x + − 2
• 2 2 2 2
• ( a − b) 3 = a 3 − b 3 − 3ab ( a − b)  x  x  x  x 
• ( a 3 + b 3 ) = ( a + b) ( a 2 + b 2 − ab)
122 CDS Pathfinder

Factorisation of Perfect Sol. c. 4 3x2 + 5x − 2 3 = 4 3x2 + 8x − 3x − 2 3


Square Polynomials [Q 4 3 × ( − 2 3) = − 24 = 8 × ( − 3)]
= 4x ( 3x + 2) − 3( 3x + 2)
If the polynomial is given as the perfect square
= ( 3x + 2)( 4x − 3)
quadratic polynomial, then use the following Identities
to factorise it. Factorisation of Sum and
• a 2 + 2ab + b 2 = ( a + b) 2 • a 2 − 2ab + b 2 = ( a − b) 2 Difference of Cubes
In case, the polynomial is given in the form of a 3 + b 3 or
EXAMPLE 4. Factorise 16 x 2 − 8x + 1
a 3 − b 3 . To factorise it, following identities can be
a. ( 2x − 1)2 b. ( x − 4)2 c. ( x − 2)2 d. ( 4 x − 1)2 applied.
Sol. d. 16x2 − 8x + 1 = ( 4x) 2 − 2( 4x) + 12 = ( 4x − 1) 2 • a 3 + b 3 = ( a + b)( a 2 − ab + b 2 )
= ( 4x − 1)( 4x − 1) • a 3 − b 3 = ( a − b)( a 2 + ab + b 2 )

EXAMPLE 5. Factorise x 2 − 2 5x + 5 EXAMPLE 9. Factorise (2x + 3y ) 3 − (2x − 3y ) 3


a. ( x 5 − 1)2 b. ( x − 5)2 c. ( x + 5)( x − 5) d. ( x − 5)2 a. 18( 4 x2 + 3y 2 ) b. 18 y ( 3x2 + 4 y 2 )
c. xy ( 4 x + 3y )
2 2
d. 18 y ( 4 x2 + 3y 2 )
Sol. d. x2 − 2 5x + 5 = x2 − 2 5x + ( 5) 2 = ( x − 5) 2
= ( x − 5)( x − 5) Sol. d. ( 2x + 3y) 3 − ( 2x − 3y) 3
Put 2x + 3y = a and 2x − 3y = b
Factorising the Difference of Two Squares
⇒ a3 − b3 = ( a − b)( a2 + ab + b2) = ( 2x + 3y − 2x + 3y)
In case, the polynomial is given in the form of a 2 − b 2 , [( 2x + 3y) 2 + ( 2x + 3y)( 2x − 3y) + ( 2x − 3y) 2 ]
to evaluate it, following identity is applied. = 6y[ 4x2 + 9y 2 + 12xy + 4x2 − 9y 2 + 4x2
• a 2 − b 2 = ( a − b)( a + b) + 9y 2 − 12xy ]
= 6y[12x + 9y ] = 18y[ 4x + 3y 2 ]
2 2 2

EXAMPLE 6. Factorise 2a 5 − 32a


Hence, ( 2x + 3y) 3 − ( 2x − 3y) 3 = 18y[ 4x2 + 3y 2 ]
a. ( a + 4)( a − 2)( a + 2)
2
b. 2a ( a + 4)( a − 2)( a + 2)
2

c. 2a ( a2 + 4) d. None of these Factorisation of Polynomial of form


a3 + b3 + c 3 − 3abc
Sol. b. 2a5 − 32a = 2a( a4 − 16)
Here, it is easy to use.
= 2a[( a2) 2 − ( 4) 2 ] = 2a( a2 + 4)( a2 − 4)
• a 3 + b 3 + c 3 − 3abc
= 2a( a2 + 4)( a2 − 22) = 2a( a2 + 4)( a − 2)( a + 2) = ( a + b + c )( a 2 + b 2 + c 2 − ab − bc − ac )
• If a + b + c = 0, then a 3 + b 3 + c 3 = 3abc
Factorisation of Quadratic Polynomials
Quadratic polynomials of the type ax 2 + bx + c, where EXAMPLE 10. Factorise x 3 + 27 y 3 + 8x 3 − 18xyz
a ≠ 0, a and b are coefficients of x 2 and x respectively a. ( x + 3y + 2z)( x2 + 9 y 2 + 4 z2 + 3xy + 6 yz + 2xz)
and c is constant, can be factorised by splitting the
b. ( x + 3y + 2z)( x2 + 9 y 2 + 4 z2 − 3xy − 6 yz − 2xz)
middle term. We find two numbers p and q such that
c. ( x − 3y − 2z)( x2 + 9 y 2 − 4 z2 − 3xy + 6 yz + 2xz)
p + q = b and pq = ac, then
d. None of the above
ax 2 + bx + c = ax 2 + ( p + q)x + c = ax 2 + px + qx + c
Sol. b. x3 + 27y 3 + 8z 3 − 18xyz
EXAMPLE 7. Factorise x 2 + 9x + 14 = x3 + ( 3y) 3 + ( 2z) 3 − 3( x)( 3y)( 2z)
= ( x + 3y + 2z)[ x2 + 9y 2 + 4z 2 − x ( 3y)
a. ( x + 2)( x + 7) b. ( x − 2)( x − 7) − 3y( 2z) − x ( 2z)]
c. ( x + 2)( x − 7) d. ( x − 2)( x + 7) = ( x + 3y + 2z)([ x2 + 9y 2 + 4z 2 − 3xy − 6yz − 2xz ])

Sol. a. x + 9x + 14 = x + (7 + 2) x + 14 = x2 + 7x + 2x + 14
2 2 EXAMPLE 11. Factorise
= x ( x + 7) + 2( x + 7) = ( x + 7)( x + 2) (2x − 3y ) 3 + (3y − 5z) 3 + (5z − 2x) 3
a. 3( 2x − 3y )( 3y − 5z)( 5z − 2x)
EXAMPLE 8. Factorise 4 3x 2 + 5x − 2 3 b. ( 2x − 3y )( 3y − 5z)( 5z − 2x)
a. ( 2x + 3)( 4 x − 3) b. ( 2x − 3)( 4 x + 3) c. 3( 2x − 3y )( 3y + 5z)( 5z − 2x)
c. ( 3x + 2)( 4 x − 3) d. None of these d. None of the above
MATHEMATICS Algebraic Operations 123

Sol. a. Here, 2x − 3y = a, 3y − 5z = b and 5z − 2x = c EXAMPLE 14. If two factors of a 4 − 2a 3 − 9a 2 + 2a + 8


Q a + b + c = 0 ⇒ a 3 + b 3 + c 3 = 3abc are (a +1) and (a −1), then what are the other two
∴ ( 2x − 3y) 3 + ( 3y − 5z) 3 + ( 5z − 2x) 3 factors?
= 3( 2x − 3y)( 3y − 5z)( 5z − 2x) a. ( a − 2) and ( a + 4) b. ( a + 2) and ( a + 4)
c. ( a + 2) and ( a − 4) d. ( a − 2) and ( a − 4)
Remainder Theorem
Sol. c. Let f ( a) = a4 − 2a3 − 9a2 + 2a + 8
Let p( x) be a polynomial of degree n greater than or and ( a + 1) and ( a − 1) are two factors of f ( a).
equal to 1 (i.e. n ≥ 1) and a be any real number. If p( x) is Now, by division method,
divided by the linear polynomial ( x − a), then the a2 –1) a4 – 2a3 – 9a2 + 2a + 8 (a2 – 2a – 8
remainder is p( a). Remainder can be evaluated by a4 – a2
substituting x − a = 0, i.e. x = a in p( x) .
– 2a3 – 8a2 + 2a + 8
EXAMPLE 12. The remainder when – 2a3 2a
12x 3 − 13x 2 − 5x + 9 is divided by (3x + 2) is
– 8a2 +8
a. 1 b. 2 c. 3 d. 0 – 8a2 +8
Sol. c. Let p( x) = 12x3 − 13x2 − 5x + 9 and q( x) = 3x + 2 ×
−2
When p ( x) is divided by ( 3x + 2), then remainder is p   . So, the required factor of f ( a) is ( a2 − 2a − 8).
 3 Further factorise it by splitting the middle term, i.e.
−2 −2 3 −2 2 −2
Now, p   = 12   − 13   − 5   + 9 a2 − 2a − 8 = a2 − 4a + 2a − 8
 3  3  3  3
= a( a − 4) + 2( a − 4) = ( a + 2)( a − 4)
−8
= 12 ×   − 13 × +
4 10 Hence, ( a + 2) and ( a − 4) are other two factors of f ( a).
+9=3
 27 9 3
Hence, the required remainder is 3. Factorisation of Polynomials
Factor Theorem Using Factor Theorem
Let p( x) be a polynomial, if degree n ≥ 1 and a be any To factorise the polynomial, follow the following steps
real number. Then, (i) Let p ( x) be a given polynomial.
(i) If p( a) = 0, then ( x − a) is a factor of p( x) . (ii) find all the possible factors of constant term in p ( x).
(ii) If ( x − a) is a factor of p( x), then p( a) = 0. (iii) Take any one of the factors, say (α ) and find p (α ). If
p (α ) = 0 the ( x − α ) is a factor of p ( x).
EXAMPLE 13. For what values of k will
4x 5 + 9x 4 − 7 x 3 − 5x 2 − 4kx + 3k 2 (iv) Divide the polynomial p ( x) by ( x − α ) to find its all
contain x −1 as a factor? other factors.
1 1 1
a. 3, − b. 3, −1 c. 0, d. 1, EXAMPLE 15. Factorise 2x 3 − 5x 2 − 19x + 42 .
2 3 3
Sol. d. Given, P( x) = 4x5 + 9x4 − 7x3 − 5x2 − 4kx + 3k 2 a. ( x − 2) ( 2x − 7) ( x + 3) b. ( x + 2) ( 2x + 7) ( x − 3)
Since, ( x − 1) is a factor of P( x). ∴ P(1) = 0 c. ( x − 1) ( 3x − 7) ( x − 4) d. ( 2x + 3) ( 4 x − 7) ( x − 1)
⇒ 4 × (1) 5 + 9 × (1) 4 − 7 × (1) 3 − 5 × (1) 2 −4 × k × (1) + 3 × k 2 = 0 Sol. a. Let p ( x) = 2x3 − 5x2 − 19x + 42
⇒ 3k 2 − 4k + 1 = 0 ⇒ 3k 2 − 3k − k + 1 = 0 Hence, constant term is 42 and its all factors are = ± 1,
1 ± 2, ± 3, ± 6, ± 7, ± 14, ± 21 and ± 42.
⇒ ( 3k − 1)( k − 1) = 0 ⇒ k = , 1
3
At x = 1, p (1) = 2(1) 3 − 5(1) 2 − 19 + 42
Division Synthetic Algorithm = 2 − 5 − 19 + 42 = 20 ≠ 0
So, ( x − 1) is not a factor of p ( x).
If p( x) and g( x) any two polynomials with g( x) ≠ 0, then
At x = 2, p ( 2) = 2( 2) 3 − 5( 2) 2 − 19( 2) + 42 = 58 − 58 = 0
we can find polynomials q( x) and r ( x) such that
So, ( x − 2) is a factor of p ( x).
p( x) = g( x) × q( x) + r ( x) Now, on dividing p ( x) by ( x − 2), we get ( 2x 2 − x − 2) as
i.e. Dividend = (Divisor × Quotient) + Remainder quotient i.e. other factor of p( x).
where, r ( x) = 0 or degree of r ( x) < degree of g( x) , then So, p ( x) = ( x − 2)[ 2x2 − x − 21]
we say that p( x) divided by g( x), gives q ( x) as quotient = ( x − 2) ( 2x2 − 7x + 6x − 21)
and r ( x) as remainder.
= ( x − 2)[ x ( 2x − 7) + 3 ( 2x − 7)]
If the remainder r ( x) is zero, we say that divisor g ( x) is ∴ p ( x) = ( x − 2)( 2x − 7)( x + 3)
a factor of p ( x).
124 CDS Pathfinder

PRACTICE EXERCISE
x = 5, y = 3 and z = 2, then 10. What are the factors of  x 2 − 2x − 9 ?
1. If the value of 1
x + y 2 + z 2 − 2xy + 2 yz − 2zx is
2 3 
1 1
(a) 125 (b) 0 (c) −25 (d) 10 (a) (x − 9)(x + 3) (b) (x − 9)(x − 3)
3 3
2. The factors of x − 2 3x + 3 are 2
1 1
(c) (x + 9)(x + 3) (d) (x + 9)(x − 3)
(a) (x + 3 )2 (b) (x − 3 )2 3 3
(c) (x + 3 )(x − 3) (d) (x + 2 )(x + 3) 11. What are the factors of ( 6 3x 2 − 47x + 5 3 )?
3. The factors of ( a b − 16c ) are 4 4 4
(a) (3 3x + 5 3 )(2 x − 5 3 )
(a) 4(a2 b 2 + c 2 )(ab − 2c )(ab + 2c ) (b) (3 3x − 5 3 )(x 3x + 1)
(b) (a2 b 2 − 4 c 2 )(ab + 2c )2 (c) (2 x + 5 3 )(3 3x + 1)
(c) (a b + 4 c )(ab + 2c )(ab − 2 c )
2 2 2 (d) (2 x − 5 3 )(3 3x − 1)
(d) (a2 b 2 − 4 c 2 )2 (ab + 2c )(ab + 4 c ) 12. The factors of ( 8a3 + 125b3 − 64c3 + 120abc) are
4. The factors of ( x 8 − y 8 ) are (a) (2 a + 5b − 4 c ) (2 a + 5b + 4c )
(b) (2 a − 5b − 4 c ) (2 a + 5b + 4c )
(a) (x + y )(x + y )(x + y)(x − y)
4 4 2 2
(c) (2 a + 5b − 4c ) (4a2 + 25b 2 + 16c 2 − 10ab + 20bc + 8ac )
(b) (x2 + y2 )2 (x + y)(x − y)
(d) (2 a + 5b + 4c ) (4a2 + 25b 2 + 16c 2 − 10ab + 20bc + 8ac )
(c) (x4 + y4 )(x2 + y2 )2
(d) (x2 + y2 )(x − y)2 13. If x1/ 3 + y1/ 3 + z1/ 3 = 0, then
5. The factors of  a 2 + a +  are (a) x + y + z = 0 (b) ( x + y + z)3 = 27 xyz
1
 4 (c) x + y + z = 3 xyz (d) x3 + y3 + z3 = 0
2
(a)  a +  (b)  a +  (a + 2 )
1 1 1 1
 2  2 14. If x + = 5, then the value of x3 + 3 is
x x
2
(c)  a +   a −  (d)  a − 
1 1 1 (a) 8 5 (b) 2 5 (c) 5 5 (d) 7 5
 2  2  2
15. If  x +  = 6, then  x 2 + 2  is equal to
1 1
6. The factors of 8 − 4x − 2x3 + x 4 are  x   x 
(a) (2 − x)(4 − x3 ) (b) (2 + x)(4 − x3 ) (a) 32 (b) 38 (c) 34 (d) 44
(c) (2 + x)(3 − x ) 3
(d) (2 − x)(x3 − 4) 1 1 9
16. If x − = , then what is 9x 2 + 2 equal to?
7. The factors of ( a − b − 4ac + 4c ) are 2 2 2 x 3 x
(a) (a + 2c + b )(a − 2c − b ) (b) (a − 2c + b )(a − 2c − b ) (a) 18 (b) 19 (c) 20 (d) 21
(c) (a − 2 b + c )(a + b + 2c ) (d) (a − 2 b )(a + 2 b + 2c )
17. If a + b + c = 6 and a + b + c = 26, then what is
2 2 2

8. What are the factors of x 2+ 4 y 2 + 4 y − 4xy ab + bc + ca equal to?


−2x − 8? (a) 0 (b) 2 (c) 4 (d) 5
(a) (x − 2 y − 4) and (x − 2 y + 2 )
−4
(b) (x − y + 2 ) and (x − 4 y + 4) 18. If ( x + x ) = 322, then what is one of the value
4

(c) (x − y + 2 ) and (x − 4 y − 4) of ( x − x − 1 )?
(d) (x + 2 y − 4) and (x + 2 y + 2 ) (a) 18 (b) 16 (c) 8 (d) 4
2 1
9. Factorise a − 3 − 2a + 3
19. If a + b + c = 0, then what is the value of
a a
a 2 b2 c2
(a)  a +   a2 + 2  − 2  a −  + +
1 1 1 ?
 a  a   a bc ca ab
1  2 
(b)  a −   a + 2 + 1 + 2  a − 
  (a) − 3
1 1 (b) 0 (c) 1 (d) 3
 a  a   a
 1  1 20. If a + b + c = 10 and ab + bc + ca = 31, then the
(c)  a +   a − 
 a  a value of a 2 + b2 + c2 is
(d) None of the above (a) 48 (b) 38 (c) 28 (d) 18
MATHEMATICS Algebraic Operations 125

1 1 32. If the remainder of the polynomial


21. The factors of x 2 + + 1 − 2x − are
4x 2
x a0 + a1x + a2x 2 + L + an x n when divided by ( x − 1)
(a)  x −
1 
+ 2  (b)  x +
1 
− 2 
1 1
 x −  x + is 1, then which one of the following is correct?
 2x  2x   2x  2x  (a) a0 + a2 + L = a1 + a3 + L
(c)  x −
1 
− 2  (d)  x +
1 
− 2 
1 1 a0 + a2 + L = 1 + a1 + a3 + L
 x −  x − (b)
 2x  2x   2x  2x 
(c) 1 + a0 + a2 + L = − (a1 + a3 + L )
22. The factors of ( a 2 − b2 ) ( c2 − d 2 ) − 4abcd are (d) 1 − a0 − a2 − L = a1 + a3 + L
(a) (ac + bd + bc + ad ) (ac − bd − bc − ad ) 33. If u , v and w are real numbers such that
(b) (ac − bd − bc + ad ) (ac + bd + dc + ab ) u3 − 8v3 − 27w3 = 18 uvw, then which one of the
(c) (ac − bd + bc + ad ) (ac − bd − bc − ad )
following is correct?
(d) (ac − bd − bc − ad ) (ac + bd + dc + ab )
(a) u − v + w = 0 (b) u = − v = − w
1 17 (c) u − 2 v = 3w (d) u + 2 v = − 3w
23. If ‘a’ is an integer such that a + = , then the
a 4
 1 34. The value of ( a1/ 8 + a − 1/ 8 ) ( a1/ 8 − a − 1/ 8 ) ( a1/ 4 + a − 1/ 4 )
value of  a −  is
 a ( a1/ 2 + a − 1/ 2 ) is
(a) 4 (b)
13
(c)
17
(d)
15 (a) (a + a− 1 ) (b) (a − a− 1 ) (c) (a2 + a− 2 ) (d) (a2 − a− 2 )
4 4 4
24. x 4 + 4 y 4 is divisible by which one of the following? 35. The polynomial f ( x ) = x 4 − 2x3 + 3x 2 − ax + b,
(a) (x2 + 2 xy + 2 y2 ) (b) (x2 + 2 y2 )
when divided by ( x − 1) and ( x + 1) leaves the
remainders 5 and 19, respectively. The values of
(c) (x − 2 y )
2 2
(d) None of these a and b are
25. If x ( x + y + z ) = 9, y ( x + y + z ) = 16 and (a) a = 4, b = 3 (b) a = 3, b = 4
z ( x + y + z ) = 144, then x equal to (c) a = 5, b = 8 (d) a = 9, b = 7
9 9 9 16
(a) (b) (c) (d) 36. If the expression ax 2 + bx + c is equal to 4 when
5 7 13 13
x = 0, leaves a remainder 4 when divided by x + 1
26. If x = ( b − c) ( a − d ), y = ( c − a ) ( b − d ) and z = ( a − b) and a remainder 6 when divided by x + 2., then
( c − d ), then what is x3 + y3 + z3 equal to? the values of a , b and c are respectively:
(a) xyz (b) 2xyz (c) 3xyz (d) − 3xyz (a) 1, 1, 4 (b) 2, 2, 4 (c) 3, 3, 4 (d) 4, 4, 4

27. If the expression ( px + x − 2x − q ) is divisible by


3 2
37. If ( x − xy + x y − y
3/ 2 1/ 2 1/ 2 3/ 2
) is divided by
( x − 1) and ( x + 1), what are the values of p and q x1/ 2 − y1/ 2 the quotient is
respectively? (a) x − y (b) x + y (c) x1/ 2 + y1/ 2 (d) x2 − y2
(a) 2, − 1 (b) − 2, 1 (c) − 2, − 1 (d) 2, 1
38. If, 9x 2 + 3 px + 6q when divided by 3x + 1 leaves a
28. If the expression px3 + 3x 2 − 3 and 2x3 − 5x + p 3
when divided by x − 4 leave the same remainder,
remainder −
and qx 2 + 4 px + 7 is exactly
4
then what is the value of p? divisible by x + 1, then the values of p and q
(a) − 1 (b) 1 (c) − 2 (d) 2 respectively are
29. Let f ( x ) = a0x + a1x
n n −1
+ a2 x n −2
+...+ an −1x + an , (a) 0, 7 / 4 (b) −7 / 4, 0
7
where a0 , a1 , a2... an are constants. If f ( x ) is (c) , 0 (d) None of these
4
divided by ax − b, the remainder is
−b  −a 39. If a + b + c = 1, then which of the following
(a) f   (b) f  (c) f   (d) f  
b a
 statements are true?
 a  a   b  b
I. (a + b)(b + c)(c + a ) = bc + ac + ab − abc
30. Which one of the following statements is correct?
(a) Remainder theorem is a special case of factor II. a 2 + b2 − c2 + 2ab = a + b − c
theorem (a) Only I (b) Only II
(b) Factor theorem is a special case of remainder (c) Both I and II (d) Neither I nor II
theorem
1
(c) Factor theorem and remainder theorem are two 40. If x + = 4, then the value of expression
independent results x
(d) None of the above 1
I. x3 + = 52 II. x = 2 + 3
31. What is x ( y − z ) ( y + z ) + y ( z − x ) ( z + x ) + z( x − y ) x3
( x + y ) equal to? Which of the following is correct?
(a) (x + y) ( y + z) ( z + x) (b) (x − y) (x − z) ( z − y) (a) Only I (b) Only II
(c) (x + y) ( z − y) (x − z) (d) ( y − x) ( z − y) (x − z) (c) Neither I nor II (d) Both I and II
126 CDS Pathfinder

PREVIOUS YEARS’ QUESTIONS 52. x( y 2 − z 2 ) + y( z 2 − x 2 ) + z( x 2 − y 2 ) is divisible by


(a) Only ( y − z) e 2013 II
41. What is the value of k that ( 2x − 1) may be a (b) Only ( z − x)
factor of 4x 4 − ( k − 1)x3 + kx 2 − 6x + 1? e 2012 I (c) Both (y − z) and ( z − x)
(a) 8 (b) 9 (c) 12 (d) 13 (d) Neither ( y − z) nor ( z − x)
42. x 4 + xy3 + x3 y + xz3 + y 4 + yz3 is divisible by 53. Consider the following statements
(a) Only (x − y) e 2012 I I. x + 3 is the factor of x3 + 2x2 + 3x + 8.
(b) Only (x 3 + y 3 + z 3) II. x − 2 is the factor of x3 + 2x2 + 3x + 8.
(c) Both (x + y) and (x 3 + y 3 + z 3 ) Which of the statement(s) given above is/are
(d) None of the above correct? e 2013 II
43. If x3 + 5x 2 + 10k leaves remainder −2x when (a) Only I (b) Only II
divided by x 2 + 2, then what is the value of k? (c) Both I and II (d) Neither I nor II

e 2012 I 54. x3 + 6x 2 + 11x + 6 is divisible by e 2014 I


(a) –2 (b) –1 (c) 1 (d) 2 (a) Only (x + 1) (b) Only (x + 2 )
1 (c) Only (x + 3) (d) All of these
44. If x + = a , then what is the value of
x 55. What should be added to the x ( x + a ) ( x + 2a )
1 1
x3 + x 2 + 3 + 2 ? ( x + 3a ), so that the sum be a perfect square?
x x e 2012 I
(a) 9 a2 (b) 4a2 e 2014 I
(a) a3 + a2 (b) a3 + a2 − 5a
(c) a4 (d) None of these
(c) a3 + a2 − 3a − 2 (d) a3 + a2 − 4a − 2
56. If  x 2 +
1  17  1
45. If p ( x ) is a common multiple of degree 6 of the 2
= , then what is  x3 − 3  equal to?
 x  4  x 
polynomials f ( x ) = x3 + x 2 − x − 1 and e 2014 I
75 63 95
g ( x ) = x3 − x 2 + x − 1, then which one of the (a) (b) (c) (d) None of these
16 8 8
following is correct? e 2012 I
(a) p(x) = (x − 1)2 (x + 1)2 (x2 + 1) 57. What is the remainder when x5 − 5x 2 + 125 is
(b) p(x) = (x − 1) (x + 1) (x2 + 1)2 divided by x + 5? e 2014 II
(c) p(x) = (x − 1)3 (x + 1) (x2 + 1) (a) 0 (b) 125 (c) − 3125 (d) 3125
(d) p(x) = (x − 1)2 (x4 + 1) 58. If ax + by − 2 = 0 and axby = 1, where a ≠ 0, b ≠ 0,
46. What is the value of k which will make the then what is ( a 2x + b2 y ) equal to? e 2014 II
expression 4k + 12k + k a perfect square?e 2012 II
2
(a) a + b (b) 2ab (c) a + b
3 3
(d) a 4 + b 4
(a) 5 (b) 7 (c) 8 (d) 9
47. The factors of 5 px − 10qy + 2rpx − 4 qry is/are 59. If ( x + k) is the common factor of x 2 + ax + b and
(a) Only (5 + 2 r ) e 2013 I x 2 + cx + d, then what is k equal to? e 2014 II
(b) Only ( px − 2qy) (a) (d − b ) / (c − a ) (b) (d − b ) / (a − c )
(c) Both (5 + 2 r ) and ( px − 2qy) (c) (d + b ) / (c + a ) (d) (d − b ) / (c + a )
(d) Neither (5 + 2 r ) nor ( px − 2qy)
60. Consider the following statements:
48. ( a + 1)4 − a 4 is divisible by e 2013 I I. (a − b − c) is one of the factors of
(a) − 2 a 2 + 2 a − 1 (b) 2 a 3 − 2 a − 1 3abc + b3 + c3 − a3 .
(c) 2 a 3 − 2 a + 1 (d) 2 a 2 + 2 a + 1 II. (b + c − 1) is one of the factors of 3bc + b3 + c3 − 1.
49. One of the factors of the polynomial Which of the statement(s) given above is/are
x 4 − 7x3 + 5x 2 − 6x + 81 is e 2013 I correct? e 2014 II
(a) x + 2 (b) x − 2 (c) x + 3 (d) x − 3 (a) Only I (b) Only II
(c) Both I and II (d) Neither I nor II
50. If the expression x + 3x + 4x + k has a factor
3 2

x + 5, then what is the value of k? e 2013 II


61. For what value of k is ( x − 5) a factor of
(a) −70 (b) 70 (c) 48 (d) −48 x3 − 3x 2 + kx − 10 ? e 2015 I
(a) −8 (b) 4 (c) 2 (d) 1
51. The quantity which must be added to
(1 − x )(1 + x 2 ) to obtain x3 is e 2013 II
62. If x + y + z = 0, then x3 + y3 + z3 + 3xyz is equal to
(a) 2 x3 + 3x2 + x + 1 (b) 2 x3 + x2 + x − 1 e 2015 I
(c) 2 x3 − x2 + x − 1 (d) − x3 + x − 1 (a) 0 (b) 6xyz (c) 12xyz (d) xyz
MATHEMATICS Algebraic Operations 127

63. The expression x3 q 2 − x3 pt + 4x 2 pt −4x 2q 2 66. If the polynomial x 6 + px5 + qx 4 − x 2 − x − 3 is


+3xq 2 − 3xpt is divisible by divisible by ( x 4 − 1), then the value of
e 2015 I
(a) Only (x − 1) (b) Only (x − 3) p2 + q 2 is e 2016 I
(c) Both (x − 1) and (x − 3) (d) Neither (x − 1) nor (x − 3) (a) 1 (b) 9 (c) 10 (d) 13

64. Which one of the following is correct? e 2015 II 67. If the linear factors of ax 2 − ( a 2 + 1) x + a are p
(a) (x + 2 ) is a factor of x4 − 6x3 + 12 x2 − 24x + 32 and q, then p + q is equal to e 2016 I

(b) (x + 2 ) is a factor of x + 6x − 12 x + 24x − 32


4 3 2 (a) (x − 1) (a + 1) (b) (x + 1) (a + 1)
(c) (x − 1) (a − 1) (d) (x + 1) (a − 1)
(c) (x − 2 ) is a factor of x4 − 6x3 + 12 x2 − 24x + 32
(d) (x − 2 ) is a factor of x4 + 6x3 − 12 x2 + 24x − 32 x 10 y
68. If = − and x − y = 8, then the value of
65. For what value of k ( x + 1) is a factor of y 3 x
x3 + kx 2 − x + 2 ? e 2016 I
xy is equal to e 2016 I

(a) 4 (b) 3 (c) 1 (d) −2 (a) 36 (b) 24 (c) 16 (d) 9

ANSWERS
1 b 2 b 3 c 4 a 5 a 6 a 7 b 8 a 9 d 10 a
11 d 12 c 13 b 14 b 15 c 16 b 17 d 18 d 19 d 20 b
21 b 22 c 23 d 24 a 25 c 26 c 27 d 28 b 29 a 30 b
31 b 32 d 33 c 34 b 35 c 36 a 37 b 38 c 39 c 40 d
41 d 42 c 43 c 44 c 45 a 46 d 47 c 48 d 49 d 50 b
51 c 52 c 53 d 54 d 55 c 56 b 57 c 58 a 59 a 60 c
61 a 62 b 63 c 64 c 65 d 66 c 67 a 68 d

HINTS AND SOLUTIONS 


1. (b) x2 + y 2 + z 2 − 2xy + 2 yz − 2zx 5. (a)  a 2 + a +  = a 2 + a + a + ∴ z 2 − 2z − 8 = z 2 − 4z + 2z − 8
1 1 1 1
 4 2 2 4 = z(z − 4) + 2(z − 4)
= ( x − y − z ) 2 = (5 − 3 − 2 ) 2 = 0
= (z + 2)(z − 4)
= a  a +  +  a + 
1 1 1
[Q x = 5, y = 3 and z = 2]
 2 2  2 Now, put z = x − 2 y
2. (b) x − 2 3 x + 3
2
2 ∴ ( x − 2 y + 2)
=  a +   a +  =  a + 
1 1 1
=x − 3x− 3x+3
2
 and ( x − 2 y − 4) are required factors.
2  2  2
= x( x − 3 ) − 3 ( x − 1 2
3) 9. (d) a 3 − 3 − 2a +
6. (a) 8 − 4x − 2x3 + x4 a a
= ( x − 3 ) ( x − 3 ) = ( x − 3 )2 3
= 4 ( 2 − x) − x 3 ( 2 − x)
= a −   − 2  a − 

3 1  1
3. (c) ( a b − 16c ) = [( a b ) − ( 4c ) ]
4 4 4 2 2 2 2 2
= ( 2 − x) ( 4 − x 3 )  a  a
=  a −   a 2 + 2 + 1 − 2  a − 
1 1 1
= [ a 2 b 2 + 4c 2 ] [ a 2 b 2 − 4c 2 ] 7. (b) ( a 2 − b 2 − 4ac + 4c 2 )
 a  a   a
[Q a 2 − b 2 = ( a − b ) ( a + b )] = ( a 2 − 4ac + 4c 2 ) − b 2
=  a −   a 2 + 2 − 1
1 1
= [ a 2 b 2 + 4c 2 ] [( ab )2 − ( 2c )2 ] [( a )2 − 2( 2c )( a ) + ( 2c )2 ] − b 2  a  a 
= ( a − 2c )2 − b 2 ( x2 − 6x − 27)
= [ a 2 b 2 + 4c 2 ] [( ab + 2c ) ( ab − 2c )] 10. (a)  x2 − 2x − 9 =
1
= ( a − 2c − b ) ( a − 2c + b ) 3  3
4. (a) x8 − y 8 = [( x4 )2 − ( y 4 )2 ] 1
8. (a) x2 + 4 y 2 + 4 y − 4xy − 2x − 8 = [ x2 − 9x + 3x − 27]
= [ x4 + y 4 ][ x4 − y 4 ] 3
= x2 − 4xy + 4 y 2 − 2x + 4 y − 8 1
= [ x4 + y 4 ][( x2 )2 − ( y 2 )2 ] = [ x( x − 9) + 3 ( x − 9)]
= ( x − 2 y )2 − 2 ( x − 2 y ) − 8 3
= ( x4 + y 4 )( x2 + y 2 )( x2 − y 2 ) 1
Let x − 2 y = z = ( x + 3) ( x − 9)
= ( x4 + y 4 ) ( x2 + y 2 ) ( x + y ) ( x − y ) 3
128 CDS Pathfinder

11. (d) 6 3 x2 − 47x + 5 3 1 25. (c) Given, x ( x + y + z ) = 9,


18. (d) Given, x4 + = 322
= 6 3 x − 45x − 2x + 5 3
2 x4 y ( x + y + z ) = 16
2
⇒  x2 + 2  − 2 = 322
1 and z ( x + y + z ) = 144
= 3 3 x ( 2x − 5 3 ) − 1 ( 2x − 5 3 )
 x 
= ( 2x − 5 3 ) (3 3 x − 1) 2 On adding all three equations, we get
⇒  x2 + 1  = 324 = 182
  x( x + y + z ) + y( x + y + z ) + z( x + y + z )
12. (c) 8a + 125b − 64c + 120abc
3 3 3
 x2 
1 = 9 + 16 + 144
= ( 2a )3 + (5b )3 − ( 4c )3 + 120abc ⇒ x + 2 = 18
2
x ⇒ ( x + y + z ) ( x + y + z ) = 9 + 16 + 144
= ( 2a )3 + (5b )3 + ( − 4c )3 − 3( 2a ) 2
⇒ ( x + y + z )2 = 169
⇒  x −  + 2 = 18 ⇒ x − = 4
1 1
(5 b ) ( − 4 c )  ⇒ x + y + z = 13
x x
= ( 2a + 5b − 4c ) ( 4a 2 + 25b 2 + 16 c 2 Q x ( x + y + z) = 9
19. (d) Given, a + b + c = 0 9
− 10ab + 20bc + 8ac) ⇒ x ( 13) = 9 ⇒ x =
⇒ a 3 + b 3 + c 3 = 3 abc 13
[Q a 3 + b 3 + c 3 − 3abc = ( a + b + c )
On dividing both sides by abc, we get 26. (c) Given, x = ( b − c ) ( a − d ),
( a 2 + b 2 + c 2 − ab − bc − ac )]
a3 b3 c3 y = ( c − a) ( b − d )
⇒ + + =3
13. (b) Since, x 1 / 3 + y 1 / 3 + z1 / 3 = 0 abc abc abc and z = ( a − b) ( c − d )
∴( x 1 / 3 )3 + ( y 1 / 3 )3 + (z1 / 3 )3 a 2
b 2
c 2
Q x + y + z = ( b − c) ( a − d )
⇒ + + =3
−3 x 1 / 3 y 1 / 3z1 / 3 = 0 bc ac ab + ( c − a) ( b − d )
⇒ x + y + z − 3( xyz )1 / 3 = 0 + ( a − b) ( c − d )
20. (b) Q ( a + b + c )2 = a 2 + b 2 + c 2
=0
⇒ x + y + z = 3( xyz ) 1/3
+ 2ab + 2bc + 2ac
Hence, x3 + y 3 + z 3 = 3xyz
⇒ ( x + y + z )3 = 27xyz ( 10)2 = ( a 2 + b 2 + c 2 ) + 2 (31)
3 27. (d) Here, px3 + x2 − 2x − q is divisible
a 2 + b 2 + c 2 = 100 − 62
14. (b) Q  x +  = x3 + 3 + 3  x + 
1 1 1 by ( x − 1) and ( x + 1).
 x x  x ⇒ a 2 + b 2 + c 2 = 38
1 ∴ p ( 1)3 + ( 1)2 − 2 ( 1) − q = 0
( 5 )3 = x3 + 3 + 3 ( 5 ) 1 1 ⇒ p−q=1 …(i)
x 21. (b) x + 2
+ 1 − 2x −
4x 2 x and p ( − 1)3 + ( − 1)2 − 2 ( − 1) − q = 0
Q x + 1 = 5 
⇒ p + q =3
=  x2 + + 1 − 2  x +
  1 1 …(ii)
x 
 4x 2   2x  On solving Eqs. (i) and (ii), we get
1
⇒ x3 + 3 = 5 5 − 3 5 = 2 5 2 p = 2 and q = 1
=  x +
1
− 2  x +
x 1
 
 2x   2x  28. (b) Let f ( x) = px3 + 3x2 − 3
15. (c) Given,  x +  = 6
1
and g ( x) = 2x3 − 5x + p.
=  x +
 x 1  x + 1 − 2
  
 2x   2x  On dividing f ( x ) by ( x − 4), remainder is
On squaring both sides, we get
2
 x + 1  = 62 22. (c) ( a 2 − b 2 )( c 2 − d 2 ) − 4abcd f ( 4) = p ( 4)3 + 3 ( 4)2 − 3
  = 64 p + 48 − 3 = 64 p + 45
 x = a 2 c 2 − a 2 d 2 − b 2 c 2 + b 2 d 2 − 4abcd
1
⇒ x + 2 + 2 = 36
2 = ( a 2 c 2 + b 2 d 2 − 2abcd ) Similarly g( 4) = 2( 4)3 − 5 ( 4) + p
x − ( b c + a d + 2abcd )
2 2 2 2
= 128 − 20 + p = 108 + p
1
⇒ x2 + 2 = 34 = ( ac − bd )2 − ( bc + ad )2 But f ( 4) = g ( 4) [given]
x
= ( ac − bd + bc + ad )( ac − bd − bc − ad ) ∴ 64 p + 45 = 108 + p
1 1
16. (b) Given, x − = 2 ⇒ 63 p = 63 ⇒ p = 1
23. (d) Q  a −  =  a + 1 − 4
x 3 1
  29. (a) ax − b = 0 ⇒ x = b / a
3  a  a
⇒ 3x − = 1
x 2
 17  − 4 By remainder theorem, if f ( x ) is divisible
=   by ax − b, the remainder is f ( b / a ).
On squaring both sides, we get  4
2
 17  30. (b) Factor theorem is a special case of
 3x − 3  = ( 1)2 Q given  a +  =
1
   remainder theorem.
 x  a 4 
9 289 − 64 31. (b) x ( y − z )( y + z ) + y (z − x) (z + x)
9x 2 + 2 − 2 × 9 = 1 = =
225 15
= + z (x − y) (x + y)
x 16 16 4
9 = x ( y 2 − z 2 ) + y (z 2 − x 2 )
⇒ 9x2 + 2 = 19 24. (a) x4 + 4 y 4 + z ( x2 − y 2 )
x
= x 4 + 4 y 4 + 4x 2 y 2 − 4x 2 y 2 = x ( y 2 − z 2 ) + yz 2 − yx2
17. (d) Q ( a + b + c )2 = a 2 + b 2 + c 2 + 2 = ( x 2 + 2 y 2 ) 2 − ( 2 xy ) 2 + zx2 − zy 2
( ab + bc + ca ) = ( y − z ) ( xy + xz − x2 − yz )
= ( x2 + 2 y 2 − 2xy )
∴ ( 6)2 = 26 + 2 ( ab + bc + ca ) = ( y − z ) [ y ( x − z ) + x (z − x)]
( x 2 + 2 y 2 + 2 xy )
⇒ 2 ( ab + bc + ca ) = 10 = ( y − z ) (z − x ) ( x − y )
From above it is clear that x4 + 4 y 4 is
divisible by x2 + 2 y 2 + 2xy = ( x − y ) ( x − z ) (z − y )
⇒ ab + bc + ca = 5
MATHEMATICS Algebraic Operations 129

4 3
32. (d) Let f ( x) = a + a x + a x2 37. (b) x 3 / 2 − xy 1 / 2 + x 1 / 2 y − y 3 / 2
⇒ 4 ×   − ( k − 1) ×  
1 1
0 1 2
+ L + an x n
= x( x 1/2
−y 1/2
) + y( x 1/2
−y 1/2
)  2  2
2
Since, ( x − 1) is a factor of f ( x). = (x −y )( x + y ) + k ×   − 6 × + 1 = 0
1/2 1/2 1 1
Put x = 1 in f ( x), then  2 2
⇒ Quotient
f ( 1) = a + a + a + L + an ( x 1 / 2 − y 1 / 2 )( x + y ) 1 ( k − 1) k
0 1 2
= =x+ y ⇒ − + −2=0
⇒ 1 = a + a + a + L + an ( x 1 / 2 − y1 / 2 ) 4 8 4
0 1 2
∴ 1− a − a −L= a + a + L k ( k − 1) 1
0 2 1 3 38. (c) Let f ( x ) = 9x 2 + 3 px + 6q ⇒ − =2−
33. (c) Given, (u )3 + ( − 2v )3 + ( − 3w )3 4 8 4
Given, f ( −1 / 3) = −3 / 4
= 3 × ( − 2) ( − 3) uvw 2 2k − ( k − 1 ) 8−1 7
⇒ 9 −  + 3 p  −  + 6q = −3 / 4 ⇒ = =
1 1
∴ u + ( − 2v ) + ( − 3w ) = 0
 3  3 8 4 4
[Q a 3 + b 3 + c 3 = 3abc , then
⇒ 1 − p + 6 q = −3 / 4 ⇒ 8k − 4k + 4 = 56
a + b + c = 0]
⇒ u − 2v − 3w = 0 ⇒ 24q − 4 p + 7 = 0 …(i) ⇒ 4k = 52 ⇒ k = 13
∴ u − 2v = 3w Let g( x ) = qx + 4 px + 7
2 Hence, the value of k is 13.
34. (b) We know that, ( a + b ) ( a − b ) Since, ( x + 1) is a factor of g( x ) 42. (c) Given, x4 + xy 3 + x3 y + xz 3
= a2 − b2 ∴ g( −1) = 0 ⇒ q − 4 p + 7 = 0…(ii) + y 4 + yz 3
− 1/8
So, [( a1/8
+ a ) ( a1 / 8 − a −1 / 8 ) On solving Eq. (i) and Eq. (ii), we get = ( x + xy + xz ) + ( x y + y 4 + yz 3 )
4 3 3 3

( a1 / 4 + a − 1 / 4 ) ( a1 / 2 + a − 1 / 2 ) ] q = 0 and p = 7 / 4 = x( x3 + y 3 + z 3 ) + y( x3 + y 3 + z 3 )
= [( a1 / 8 )2 − ( a −1 / 8 )2 ( a1 / 4 + a −1 / 4 )] 39. (c) I. ( a + b )( b + c )( c + a ) = ( x + y ) ( x3 + y 3 + z 3 )
= ( a1 / 4 − a −1 / 4 )( a1 / 4 + a −1 / 4 ) = ( 1 − c )( 1 − a )( 1 − b ) Hence, given polynomial is divisible by
( a1 / 2 + a −1 / 2 ) [Q a + b + c = 1] both
= [ a1 / 2 − a − 1 / 2 ] [ a1 / 2 + a − 1 / 2 ] = ( 1 − c ) [ 1 − b − a + ab ] ( x + y ) and ( x3 + y 3 + z 3 ).
−1
= (a − a ) = 1 − b − a + ab − c + bc + ac − abc
43. (c) x2 + 2 ) x3 + 5x2 + 10k ( x + 5
35. (c) f ( x) = x4 − 2x3 + 3x2 − ax + b = 1 − ( a + b + c ) + ab + bc + ac − abc
x3 + 2x
= ab + bc + ac − abc − −
Put x = 1 as f ( 1) = 5, f ( − 1) = 19
II. a 2 + b 2 − c 2 + 2ab 5x2 – 2x + 10k
f ( 1) = 14 − 2 ( 1)3 + 3 ( 1)2 − a + b
= a 2 + b 2 + 2ab − c 2 5x2 + 10
⇒1− 2 + 3 − a + b =5 − −
= ( a + b )2 − c 2
⇒ − a + b =3 …(i) - 2x - 10 + 10k = Remainder
= ( a + b + c )( a + b − c ) = a + b − c
and f ( −1) = ( −1)4 − 2( −1)3 + 3 But given that, remainder = − 2x
[Q a + b + c = 1]
( − 1)2 − a ( − 1) + b ∴ − 2x − 10 + 10 k = − 2x
So, both statements are correct.
⇒ 1 + 2 + 3 + a + b = 19 1 ⇒ −10 + 10 k = 0
40. (d) We have, x + = 4
⇒ a + b = 13 …(ii) x ⇒ 10 k = 10 ⇒ k = 1
3
I.  x +  = x3 + 3 + 3 ( x)
1 1 1
On solving Eqs. (i) and (ii), we get 44. (c) Given that, x + = a ...(i)
a = 5, b = 8  x x x
×    x + 
1 1 1 1
36. (a) Let f ( x ) = ax 2 + bx + c Then, x3 + x2 + 3 + 2
 x  x x x
Given f ( 0) = 4 1
( 4)3 = x 3 + 3 + 3 ( 4) =  x3 + 3  +  x2 + 2 
1 1
⇒0 + 0 + c = 4 ⇒a − b + 4 = 4 ⇒c = 4 
x x   x 
On dividing f ( x ) by x + 1, remainder is 4 1 1
64 − 12 = x3 + 3 ⇒ x3 + 3 = 52 3 2
=  x +  − 3  x +  +  x +  − 2
1 1 1
∴ f ( −1) = 4 x x
1  x  x  x
⇒ a−b+ c= 4 ⇒a−b+ 4= 4 II. x + = 4 ⇒ x2 − 4x + 1 = 0
⇒ a−b = 0 …(i) x = a 3 − 3a + a 2 − 2 = a 3 + a 2 − 3a − 2
4 ± 16 − 4 × 1 × 1
On dividing f ( x ) by x + 2, remainder is 6 x= 45. (a) Given, f ( x) = x3 + x2 − x − 1
2×1
∴ f ( −2 ) = 6 and g( x) = x3 − x2 + x − 1
4± 2 3
⇒ 4a − 2b + c = 6 = = 2± 3
2 p( x ) = f ( x) ⋅ g ( x)
⇒ 4a − 2b = 2 …(ii) = ( x3 + x2 − x − 1) ( x3 − x2 + x − 1)
Hence, both statements are correct.
⇒ a−b+ 4= 4 = [ x 2 ( x + 1) − 1( x + 1)]
On solving Eq. (i) and eq. (ii), we get 41. (d) Let f ( x ) = 4x4 − ( k − 1)x3 + kx2
−6x + 1 …(i) [ x 2 ( x − 1) + 1( x − 1)]
a = 1, b = 1
Since, ( 2x − 1) is a factor of f ( x ) = [( x + 1)( x − 1)][( x − 1)( x 2 + 1)]
2
Hence,
= ( x + 1)( x + 1)( x − 1)( x − 1)( x 2 + 1)
f   = 0
1
a = 1, b = 1, c = 4. ∴
 2 = ( x + 1)2 ( x − 1)2 ( x 2 + 1)
130 CDS Pathfinder

= x y 2 − x 3 + x 3 − x y2 = 0 − 3 × . x x −  =
46. (d) Put the value of all options in 1 1 1 27
⇒ x3 −
4k + 12k + k
2
So, (z − x) is also a factor, so it is also x3 x  x 8
divisible by (z − x).
+ 3 ×  
when, k = 9, 1 27 3
⇒ x3 − 3 =
53. (d) Put x = − 3 in p ( x ), we get x 8  2
then, 4( 9)2 + 12( 9) + 9 = 441= ( 21)2
47. (c) Given expression Let p ( x ) = x 3 + 2x 2 + 3x + 8 [From Eq. (i)]
1 27 9
= 5 px − 10 qy + 2rpx − 4 qry p( −3) = ( − 3) + 2 ( − 3) + 3 ( − 3) + 8
3 2
⇒ x − 3 =3
+
x 8 2
= (5 px + 2rpx) − ( 10 qy + 4 qry ) = −27 + 18 − 9 + 8
 1  63
∴ x − 3  =
= px (5 + 2r ) − 2 qy (5 + 2r ) = − 10 ≠ 0
3

= (5 + 2r ) ( px − 2 qy )  x  8
So, ( x + 3) is not the factor of
So, factors of given expression are both x3 + 2x2 + 3 x + 8. 57. (c) Let f ( x ) = x5 − 5x2 + 125
(5 + 2r ) and ( px − 2qy ). put x = −5,
Similarly, put x = 2 in p ( x ), we get Now,
48. (d) Given, ( a + 1)4 − a 4 p( 2) = ( 2)3 + 2 ( 2)2 + 3 ( 2) + 8 ∴ Required remainder = f ( − 5)
= {( a + 1)2 − a 2 } {( a + 1)2 + a 2 } = 30 ≠ 0 So, x − 2 is also not the factor
= {( a + 1) + a } {( a + 1) − a } = ( −5)5 − 5 ( − 52 ) + 125
of x3 + 2x2 + 3x + 8.
{ a 2 + 1 + 2a + a 2 } = − 3125 − 125 + 125 = − 3125
= ( 2a + 1) ( 1) ( 2a 2 + 2a + 1) 54. (d) Let f ( x) = x3 + 6x2 + 11x + 6
58. (a) Given, ax + by − 2 = 0
= ( 2a + 1) ( 2a 2 + 2a + 1) Put x = − 1 in f ( x), we get
So, ( a + 1)4 − a 4 is divisible by ⇒ a x + by = 2
( 2a 2 + 2a + 1) f ( −1) = ( −1)3 + 6( −1)2 + 11( −1) + 6
Squaring on both sides, we get
= −1 + 6 − 11 + 6
49. (d) Let f ( x) = x4 − 7x3 + 5x2 − 6x + 81 ( ax + by )2 = ( 2)2
= −12 + 12 = 0
f (3) = (3)4 − 7(3)3 + 5(3)2 − 6(3) + 81 ⇒ a x + b y 2 + 2axby = 4
2 2 2
Hence, ( x + 1) is a factor of f ( x).
= 81 − 189 + 45 − 18 + 81 = 0 ⇒ a 2 x2 + b 2 y 2 + 2 = 4 [Q axby = 1 ]
∴ f ( x ) = ( x + 1)( x2 + 5x + 6 )
∴( x − 3) is a factor of f ( x )
= ( x + 1)( x2 + 3x + 2x + 6 ) ⇒ a 2 x2 + b 2 y 2 = 2
50. (b) Let f ( x) = x + 3x + 4x + k
3 2
= ( x + 1)( x + 2 )( x + 3 ) ⇒ a 2 x2 = 1 and b 2 y 2 = 1
Q ( x + 5) is a factor of f ( x) Hence, ( x + 1), ( x + 2 ) and ( x + 3 ) are ⇒ ax = 1 and by = 1
∴ f ( −5 ) = 0 the factors of f ( x ). 1 1
⇒ x = and y =
⇒ ( −5)3 + 3( −5)2 + 4( −5) + k = 0 55. (c) x( x + a ) ( x + 2a ) ( x + 3a ) a b
⇒ −125 + 75 − 20 + k = 0 = ( x + a )( x + 2a )x ( x + 3a )
1 1
∴ a2 x + b2 y = a2 ⋅ + b2 ⋅ = a + b
⇒ −70 + k = 0 ⇒ k = 70 a b
= ( x + 3ax + 2a )( x + 3ax )
2 2 2
Hence, the value of k is 70.
= ( x 2 + 3ax + a 2 + a 2 ) 59. (a) Given, x + k is the common factor of
51. (c) Let p( x) be added to obtain x . 3
( x 2 + 3ax + a 2 − a 2 ) x2 + ax + b and x2 + cx + d. Then, put
Then, ( 1 − x)( 1 + x ) + p( x) = x
2 3 x = − k in given polynomial.
= ( x 2 + 3ax + a 2 )2 − a 4
⇒ 1 + x2 − x − x3 + p( x) = x3 ∴ k 2 − ka + b = 0
4
So, a must be added to and k 2 − kc + d = 0
⇒ p( x) = x3 − 1 − x2 + x + x3 x ( x + a )( x + 2a )( x + 3a ) to make it a
∴ p( x) = 2x3 − x2 + x − 1 perfect square. Then, k 2 − ka + b = k 2 − kc + d
d −b
So, 2x3 − x2 + x − 1 is added to  x2 + 1  = 17 ⇒ k ( c − a) = d − b ⇒ k =
56. (b) Given,   c−a
( 1 − x)( 1 + x ) to obtain x .
2 3  x2  4
60. (c) I. We have, 3abc + b 3 + c 3 − a 3
52. (c) Let f(x) = x( y 2 − z 2 ) + y(z 2 − x 2 ) 1 17
⇒ x + 2 −2+ 2=
2
x 4 = − ( a 3 − b 3 − c 3 − 3abc )
+ z( x 2 − y 2 ) 2
 x − 1  + 2 = 17 = − [ a 3 + ( − b )3 + ( − c )3 − 3( a )( − b )( − c )]
If it is divisible by ( y − z ), then y − z is ⇒  
 x 4 = − ( a − b − c )( a 2 + b 2 + c 2 + ab
a factor of f ( x ). 2 − bc + ac)
∴ y −z = 0 ⇒ y =z ⇒  x − 1  = 17 − 2
  So, ( a − b − c ) is a factor of
 x 4
On putting y = z, we get 2 3abc + b 3 + c 3 − a 3 .
x(z 2 − z 2 ) + z(z 2 − x 2 ) + z( x 2 − z 2 ) ⇒  x − 1 = 9
  Hence, statement I is correct.
 x 4
= z 3 − zx 2 + zx 2 − z 3 = 0 II. Now, 3bc + b 3 + c 3 − 1
⇒ x −  =
1 3
Hence, y − z is a factor, so it is divisible   …(i)
= b 3 + c 3 − ( 1)3 − 3bc( −1)
 x 2
by ( y − z ).
= ( b + c − 1)[ b 2 + c 2 + 12 − bc + c + b ]
On putting z = x, we get On cubing both sides, we get
3 3 So, ( b + c − 1) is a factor of
x ( y2 − x 2 ) + y(x 2 − x 2 )  x − 1 =  3 
    3bc + b 3 + c 3 − 1 .
 x  2
+ x (x 2 − y 2 ) Hence, statement II is also correct.
MATHEMATICS Algebraic Operations 131

61. (a) Let p( x) = x3 − 3x2 + kx − 10 64. (c) In option (c), we have = x( ax − 1) − a ( ax − 1)


Then, ( x − 5) is a factor of p( x). ( 2)4 − 6 ( 2)3 + 12( 2)2 − 24 × 2 + 32 = ( x − a )( a x − 1)
∴ p(5) = 0 = 16 − 6(8) + 12( 4) − 48 + 32 If p and q are linear factors of f ( x), then
= 16 − 48 + 48 − 48 + 32 = 0 p = ( x − a ) and q = ( ax − 1)
⇒ 53 − 3(5)2 + k × 5 − 10 = 0
Hence, ( x − 2) is a factor of ∴ p + q = ( x − a ) + ( ax − 1)
⇒ 125 − 75 + 5k − 10 = 0
x 4 − 6x 3 + 12x 2 − 24x + 32 = ( x − 1)( a + 1)
⇒ 40 + 5k = 0 ⇒ k = −8.
65. (d) Let f ( x) = x + kx − x + 2
3 2
68. (d) We have, x − y = 8 …(i)
62. (b) Q x3 + y 3 + z 3 − 3xyz = ( x + y + z ) x 10 y
If ( x + 1) is a factor of f ( x). and = −
( x2 + y 2 + z 2 − xy − yz − zx) Then, f ( −1) = 0 y 3 x
Given, x+ y +z = 0 ⇒ ( −1)3 + k( −1)2 − ( −1) + 2 = 0 x y 10
⇒ + =
∴ x + y + z 3 − 3xyz = 0
3 3
⇒ −1 + k + 1 + 2 = 0 ⇒ k = − 2 y x 3
⇒ x 3 + y 3 + z 3 + 3xyz = 6xyz 66. (c) Let f ( x ) = x6 + px5 + qx4 ( x )2 + ( y )2 10
⇒ =
− x − x−3
2 xy 3
63. (c) x q − x pt + 4x pt − 4x q
3 2 3 2 2 2

Q f ( x ) is divisible by ( x4 − 1). 10
+ 3xq 2 − 3xpt ⇒ x+ y = xy
Then, f ( 1) = f ( − 1) = 0 3
= x3 q 2 − x3 pt − 4x2 q 2
On squaring both sides, we have
+ 4x pt + 3xq − 3xpt
2 2 Now, f ( 1) = 1 + p + q − 1 − 1 − 3 = 0
100
⇒ p+ q=4 …(i) ( x + y )2 = xy
= x ( q − pt ) − 4x2 ( q 2 − pt )
3 2
9
and f ( − 1) = 1 − p + q − 1 + 1 − 3 100
+3x( q − pt ) 2
⇒ ( x − y ) 2 + 4 xy = xy
= ( x3 − 4x2 + 3x)( q 2 − pt ) ⇒ q− p=2 …(ii) 9
On solving Eqs. (i) and (ii), we get q = 3 [Q ( x + y )2 = ( x − y )2 + 4xy ]
= x( x − 4x + 3)( q − pt )
2 2
and p = 1 ⇒ (8)2 =  − 4 xy [from Eq. (i)]
100
= x ( x2 − 3x − x + 3)( q 2 − pt )  9 
∴ p 2 + q 2 = 32 + 12 = 10
= x [ x ( x − 3) − 1 ( x − 3)]( q 2 − pt ) 64
67. (a) Let f ( x) = ax2 − ( a 2 + 1) x + a ⇒ 64 = xy ⇒ xy = 9
= x ( x − 3)( x − 1)( q 2 − pt ) 9
= a x2 − x − a 2 x + a
15
132 CDS Pathfinder

HCF AND LCM OF


POLYNOMIALS
Regularly (1-2) questions have been asked from this chapter. Generally questions are based on your
prior knowledge of GCD and LCM of numbers and expressions.

DIVISOR
A polynomial d ( x) is said to be a divisor of polynomial p( x) if d ( x) is a factor of p( x) i.e., p( x) can be
written as p( x) = d ( x) ⋅ q( x), where q( x) is a quotient polynomial e.g. ( x − 3) is a divisor of ( x − 3) 2 ( x + 1) .

HCF (GCD) of Polynomials


A polynomial h ( x) is called the HCF or GCD of two or more given polynomials, if h( x) is a
polynomial of highest degree dividing each one of the given polynomial without leaving any remainder.

Note The coefficient of highest degree term is always taken as positive.

EXAMPLE 1. The HCF of polynomials (x + 2) 2 (x − 3) 2 and (x + 1) (x + 2) 2 (x − 3) is


a. ( x + 2) ( x − 3) b. ( x + 2)2 ( x − 3) c. ( x + 1) ( x + 2) ( x − 3) d. ( x + 2)2 ( x − 3)2
Sol. b. Let p( x) = ( x + 2) 2 ( x − 3) 2 and q( x) = ( x + 1) ( x + 2) 2( x − 3)
The common factor between two polynomials is ( x + 2) 2 ( x − 3) , ∴ HCF = ( x + 2) 2 ( x − 3)

HCF by Factorisation Method


Following are the steps for calculating HCF through factorisation method
Step I Resolve the given polynomials in the complete factored form.
Step II Find the HCF of the numerical factors (if any) of given polynomial.
Step III Find the factors of highest degree which is common to all given polynomials.
Step IV The product of all such common factors and HCF of the numerical factors is the HCF of given
polynomials.
MATHEMATICS HCF and LCM of Polynomials 133

EXAMPLE 2. The HCF of p(x) = 24(6 x 4 − x 3 − 2x 2 ) and −39 is taken out as common factor from remainder.
q(x) = 20(2x 6 + 3x 5 + x 4 ) is x3 − 11x − 6) x4 − 39x − 22 (x

a. 4( 2x + 1) b. x2 ( 2x + 1) c. 4 x2 d. None of these x4 − 11x2 − 6x


– + +
Sol. d. Here, P( x) = 24 (6x4 − x3 − 2x2) = 23 ⋅ 3⋅ x2 ⋅ (6x2 − x − 2) 11x2 − 33x − 22
= 2 ⋅ 3⋅ x ⋅ (6x − 4x + 3x − 2) = 2 ⋅ 3⋅ x ( 2x + 1) ⋅ ( 3x − 2)
3 2 2 3 2
Again 11 is taken out as common factor from remainder
and q( x) = 20 ( 2x6 + 3x5 + x4) = 22 ⋅ 5⋅ x4( 2x2 + 3x + 1)
x2 − 3x − 2) x3 − 11x − 6 (x + 3
= 22 ⋅ 5⋅ x4 ⋅ ( 2x2 + 2x + x + 1) = 22. 5. x4( 2x + 1)( x + 1)
x3 − 3x2 − 2x
HCF of numerical factor = 22 = 4 , Highest degree common – + +
factor = x2 ( 2x + 1) 3x − 9x − 6
2

∴ Required HCF = 4x2 ( 2x + 1) 3x2 − 9x − 6


− + +
HCF by Division Method ×
To find the HCF of polynomials which cannot be ∴ HCF of 11x4 − 39x3 − 8
factorised easily, we use successive division method.
and x4 − 39x − 22 is x2 − 3x − 2 .
Step I Arrange the given polynomials in descending order Also, HCF of 2x2 and 2x is 2x.
of powers of its variables.
∴ Required HCF = 2x ( x2 − 3x − 2)
Step II If any common factor is present in the terms of
each polynomial, it should be taken out. LCM of Polynomials
Step III Divide the polynomial of highest degree by the A polynomial h( x) is called the LCM of two or more
polynomial of lowest degree. polynomials, if it is a polynomial of smallest degree
or If both the polynomials are of the same degree then which is divided by each one of the given polynomials
any one of them can be taken as divisor or dividend. without leaving any remainder.
Step IV After the first division take the remainder as the
new divisor and first divisor as new dividend. EXAMPLE 4. The LCM of 12x 2 y 3 z 2 and
Step V Continue this process of dividing the last divisor 18x 4 y 2 z 3 is
by the last remainder until the remainder becomes a. x4 y 3 z3
zero.
b. 9 x2 y 2 z3
Step VI The product of common factors obtained from step
c. 36 x3 y 3 z 4
II and last divisor is the HCF of given polynomials.
d. 36 x4 y 3 z3
Note If the first term of a remainder is negative at any stage, the
Sol. d. Here, 12x2y 3z 2 = 22 × 31 × x2 × y 3 × z 2
l

sign of all of its term must be changed.


l If at any stage, the remainder contains common factor it 18x4 y 2z 3 = 21 × 32 × x4 × y 2 × z 3
should be taken out.
∴ Required LCM = 22 × 32 × x4 × y 3 × z 3
EXAMPLE 3. The HCF of 22x 6 − 78x 5 − 16 x 2 and = 36x4 y 3z 3
2x 5 − 78x 2 − 44x is
a. ( x 2 − 3x − 2) b. 2x( x 2 − 3x − 2) LCM by Factorisation Method
c. 22x ( x − 3x − 2)
2
d. None of these Following are the steps for calculating LCM through
factorisation method
Sol. b. Let p ( x) = 22x6 − 78x5 − 16x2 = 2x2(11x4 − 39x3 − 8)
Step I Resolve the given polynomials in the
q ( x) = 2x5 − 78x2 − 44x = 2x ( x4 − 39x − 22) complete factored form
Let us divide (11x4 − 39x3 − 8) by ( x4 − 39x − 22) Step II Find the LCM of the numerical factors
x4 − 39x − 22) 11x4 − 39x3 − 8 (11 (if any) of given polynomials
11x4 − 242 − 429x Step III The required LCM is the product of LCM
– + + of numerical factors and each factor raised to
− 39x3 + 429x + 234 the highest power.
134 CDS Pathfinder

EXAMPLE 5. The LCM of x 3 − 2x 2 − x + 2 and EXAMPLE 6. The HCF of two polynomials is x + 3


x 3 − x 2 − 4x + 4 is and their LCM is x 3 − 7 x + 6. If one of the polynomials
is x 2 + 2x − 3. Then, the other is
a. ( x 2 − 1)( x 2 − 4) b. ( x 2 + 2)( x 2 − 4)
c. ( x 2 + 1)( x 2 + 2) d. None of these a. x2 − x − 6 b. x2 − x + 6
c. x2 + x + 6 d. None of these
Sol. a. Here, x3 − 2x2 − x + 2 = x2( x − 2) − 1( x − 2)
Sol. d. Here, HCF = x + 3 and LCM = x3 − 7x + 6
⇒ ( x − 2)( x2 − 1) = ( x − 2)( x − 1)( x + 1)
= ( x − 1)( x + 1)( x − 2) Also, one polynomial = x2 + 2x − 3
and x3 − x2 − 4x + 4 = x2( x − 1) − 4( x − 1) As, LCM × HCF = Product of two polynomials
⇒ ( x − 1)( x2 − 4) = ( x − 1)( x − 2)( x + 2) LCM × HCF
Other polynomial =
∴ Required LCM = ( x − 1)( x + 1)( x − 2)( x + 2) One polynomial
= ( x2 − 1)( x2 − 22 ) ( x + 3) × ( x3 − 7x + 6)
=
[Q ( a − b )( a + b) = a − b ]
2 2
x2 + 2x − 3
= ( x − 1)( x 2 − 4 )
2
( x + 3)( x3 − 7x + 6)
=
( x + 3)( x − 1)
For any two polynomials p( x) and q( x)
p ( x) × q ( x) = (Their HCF) × (Their LCM) ( x + 3)( x − 1)( x2 + x − 6)
i.e. Product of two polynomials
= = x2 + x − 6
( x + 3)( x − 1)
= Product of their HCF and LCM
So, other polynomial is x2 + x − 6.

PRACTICE EXERCISE
1. The LCM of ( x − 1)( x − 2) and x 2( x − 2)( x + 3) is 6. What is the HCF of ( x4 − x2 − 6) and ( x4 − 4x2 + 3)?
(a) (x − 1) (b) (x − 1)(x − 2 )(x + 3) (a) x2 − 3 (b) x + 2 (c) x + 3 (d) x2 + 3
(c) x2 (x − 1)(x − 2 )(x + 3) (d) None of these
7. The HCF of the polynomial A and B where
2. The LCM of 2 ( a 2 − b2 ), 3 ( a3 − b3 ), 4 ( a 4 − b4 ) is A = ( x + 3)2( x − 2)( x + 1)2
(a) 6(a − b )(a + b )(a2 + b 2 ) (b) 12(a4 − b 4 )(a2 + ab + b 2 ) and B = ( x + 1)2( x + 3)( x + 4) is given by
(c) a3 − b 3 (d) 12(a4 − b 4 ) (a) (x + 1) 2 (x + 3) (b) (x + 1)(x + 3)2
3. The HCF of two expressions a and b is 1. Their (c) (x + 1)(x + 3) (d) (x + 3)2 (x + 1)2
LCM is
1 8. The HCF of 22x( x + 1)2 and 36x 2( 2x 2 + 3x + 1) is
(a) (a + b ) (b) a − b (c) ab (d)
ab (a) 2 x(x + 1) (b) x(x + 1) (c) 2(x + 1) (d) 2 (x + 1)2
4. The LCM of the polynomials
( x + 3)2( x − 2)( x + 1)2; ( x + 1)3 ( x + 3)( x + 4) is 9. The LCM of a 2 − b2 − c2 − 2bc,
b2 − c2 − a 2 − 2ac and c2 − a 2 − b2 − 2ab is
(a) (x − 2 )(x + 1)3 (x + 3)2 (x + 4)
(a) (a + b + c )
(b) (x − 2 )(x + 1)3 (x + 3)(x + 4) (b) (a − b − c ) (a + b + c )
(c) (x − 2 )(x + 3)(x + 4) (c) (a + b + c ) (c − a − b )
(d) (x − 2 )2 (x + 1)(x + 3)2 (x + 4) (d) (a + b + c ) (a − b − c ) (b − c − a) (c − a − b )

5. HCF of 4 y 4x − 9 y 2x3 and 4 y 2x 2 + 6 yx3 is 10. LCM of [( x + 3) ( x − 2)]2 and [( x − 2) ( x − 6)] is


(a) y2 x (2 y + 3x) (b) yx (3x + 2 y) (a) (x + 3) (x − 2 )3 (x − 6) (b) (x + 3) (x − 2 )2 (x − 6)
(c) yx (x + 3)
2
(d) None of these (c) (x + 3) (x − 2 ) (x − 6) (d) (x + 3) (x − 6)
MATHEMATICS HCF and LCM of Polynomials 135

11. If ( z − 1) is the HCF of ( z 2 − 1) and pz 2 − q( z + 1), 21. If f( x ) and g( x ) are two polynomials with integral
then coefficients which vanish at x = 1 / 2, then what is
(a) 2 p = q (b) p = 2q (c) 3 p = 2q (d) 3 p = 2 p the factor of HCF of f( x) and g( x) ?
12. The HCF of two expressions is 3x 2 + 4x − 4 and (a) x − 1 (b) x − 2 (c) 2 x − 1 (d) 2 x + 1

their LCM is 3x + 4x − 7x − 4x + 4.
4 3 2
The 22. The HCF of two polynomials p( x ) and q( x ) is
expressions are 2x ( x + 2) and LCM is 24x ( x + 2)2 ( x − 2). If
(a) (x − 1)(3x + 4x − 4) and (3x + 4x − 4)
2 2
p( x) = 8x3 + 32x2 + 32x, then what is q ( x ) equal to?
(b) (x + 1)(3x2 + 4x − 4) and (x + 2 )(3x2 + 4x − 4) (a) 4x3 − 16x (b) 6x3 − 24x (c) 12 x3 + 24x (d) 12 x3 − 24x
(c) (x + 2 )(3x + 4x + 4) and (x − 1)
2
23. If ( x − 2) is the HCF of ( ax 2 + bx + c) and
(d) (x + 1)(3x2 + 4x − 4) and (x − 1)(3x2 + 4x − 4)
( bx 2 + ax + c), then value of c is
13. The LCM and HCF of two polynomials p ( x ) and (a) 2 (a + b ) (b) (a + b ) (c) −3 (a + b ) (d) − (a + b )
q( x ) are 36x ( x + a )( x − a ) and x ( x − a ),
2 3 3 2
24. We have three polynomials A = 8 p + p2 + 12,
respectively. If p ( x ) = ( 4x 2 )( x 2 − a 2 ), then q( x ) B = p2 + 2 p − 24 and C = p2 + 15 p + 54
(a) 4x (x − a )
3 3 3
(b) 12 x (x − a )
3 2 2
I. Their LCM is ( p + 6) ( p − 4) (p + 2) ( p + 9)
(c) 9x3 (x3 − a3 ) (d) 36x3 (x3 − a3 ) II. Their HCF is ( p + 6)( p − 2)
14. The LCM of two polynomials p ( x ) and q ( x ) is Then, which of the following codes is/are correct
x3 − 7x + 6. If p ( x ) = x 2 + 2x − 3 and (a) Only I (b) Only II
q( x ) = x + x − 6, then the HCF is
2 (c) Neither I nor II (d) Both I and II

(a) (x + 3) (b) (x − 3) 25. Which of the following statements are true?


(c) (x + 3)(x − 2 ) (d) (x − 1) I. HCF of x2 − 6x + 9 and x3 − 27 is (x − 3).
15. If ( x + k) is the HCF of ( x 2 + ax + b) and II. LCM of 10x2yz, 15xyz , 20xy2z 2 is 120x2y2z 2.
( x 2 + px + q ), then the value of k is III. HCF of (6x2 − 7x − 3) and (2x2 + 11x − 21) is
(2x − 3).
b + q b −q  a + b  a − b
(a)   (b)   (c)   (d)   Select the correct answer using the codes given below
 a + p  a − p  p+ q  p−q (a) I and III (b) I, II and III
(c) II and III (d) None of these
16. What is the value of k for which the HCF of
2x 2 + kx − 12 and x 2 + x − 2k − 2 is ( x + 4)? 26. Consider the following statements :
(a) 5 (b) 7 (c) 10 (d) −4 I. The HCF of x+ y and x10 − y10 is x + y.
17. If the HCF of ( x + x − 12) and ( 2x − kx − 9) is
2 2 II. The HCF of x+ y and x10 + y10 is x + y.
III. The HCF of x− y and x10 + y10 is x − y.
( x − k), then what is the value of k?
(a) − 3 (b) 3 (c) − 4 (d) 4 IV. The HCF of x− y and x10 − y10 is x − y.
Which of the statement(s) given above is/are
18. If GCD of the polynomials ( x − 2x + px + 6) and 3 2
correct?
( x 2 − 5x + q ) is ( x − 3). Then, the value of 5q + 6 p is (a) I and II (b) II and III (c) I and IV (d) II and IV
(a) −1 (b) 1
(c) 0 (d) None of these Directions (Q. Nos. 27-29) A student wrote five
polynomials such as A = pq − np, B = pq − mq,
19. The sum and the difference of two expressions is
C = q 2 − 3nq + 2n2, D = pq − 2 pn − mq + 2mn,
5x − x − 4 and x + 9x − 10 respectively, then
2 2

their LCM would be equal to E = pq − np − mq + mn. Now, he divide the


(a) (x − 1) (b) (2 x + 3)(3x + 7 )
polynomials into groups and calculate the HCF and
LCM.
(c) (2 x − 3)(3x + 7 ) (d) (x − 1)(2 x − 3)(3x + 7 )
27. Calculate the HCF of A, C and E
20. Find the values of a and b so that the polynomials (a) (q − 2 n) (b) ( p − n) (c) (q − n) (d) (q − n)(q − 2 n)
p ( x ) and q( x ) have ( x + 1)( x + 3) as their HCF
28. Calculate the LCM of D and E
p ( x ) = ( x 2 + 3x + 2)( x 2 + 2x + a ) (a) ( p − m)(q − 2 n) (b) ( p − m)(q − n)(q − 2 n)
and q( x ) = ( x 2 + 7x + 12)( x 2 + 7x + b) (c) (q − n)(q − 2 n)(m − p) (d) (q − n)( p − m)
(a) −3, 6 (b) 3, − 6 29. The HCF of all five polynomials together is
(c) 6, − 3 (d) None of these (a) (q − n) (b) (q − 2 n) (c) 1 (d) ( p − m)
136 CDS Pathfinder

PREVIOUS YEARS’ QUESTIONS 37. What is the HCF of 8 ( x5 − x3 + x ) and 28 ( x 6 + 1) ?


30. What is the LCM of a3 b − ab3 , a3 b2 + a 2b3 and (a) 4(x4 − x2 + 1) (b) 2(x4 − x2 + 1) e 2014 I
ab ( a + b) ? e 2012 I (c) (x4 − x2 + 1) (d) None of these
(a) a2 b 2 (a2 − b 2 ) (b) ab (a2 − b 2 )
38. What is the highest common factor of
(c) a2 b 2 + ab 3 (d) a3 b 3 (a2 − b 2 ) 2x3 + x 2 − x − 2 and 3x3 − 2x 2 + x − 2 ? e 2014 II
31. What is the HCF of 36 ( 3x4 + 5x3 − 2x2 ), (a) x − 1 (b) x + 1 (c) 2 x + 1 (d) 2 x − 1
9 ( 6x + 4x − 2x) and 54 ( 27x − x) ?
3 2 4
e 2012 I 39. The HCF and LCM of two polynomials are ( x + y )
(a) 9x (x + 1) (b) 9x (3x − 1) (c) 18x (3x − 1) (d) 18x (x + 1) and ( 3x5 + 5x 4 y + 2x3 y 2 − 3x 2 y3 − 5xy 4 − 2 y5 ),
32. What is the HCF of the polynomials respectively. If one of the polynomials is ( x 2 − y 2 ),
x3 + 8, x2 + 5x + 6 and x3 + 2x2 + 4x + 8 ? e 2013 II then the other polynomial is e 2015 I
(a) x + 2 (b) x + 3 (c) (x + 2 )2 (d) None of these (a) 3x4 − 8x3 y + 10x2 y2 + 7 x y3 − 2 y4
33. The LCM of ( x3 − x2 − 2x) and ( x3 + x2 ) is e 2013 II (b) 3x4 − 8x3 y − 10x2 y2 + 7 x y3 + 2 y4
(a) x 3 − x 2 − 2 x (b) x 2 + x (c) x 4 − x 3 − 2 x 2 (d) x − 2 (c) 3x4 + 8x3 y + 10x2 y2 + 7 x y3 + 2 y4
(d) 3x4 + 8x3 y − 10x2 y2 + 7 x y3 + 2 y4
34. The HCF of ( x4 − y 4 ) and ( x6 − y 6 ) is e 2013 II
(a) x 2 − y 2 (b) x − y (c) x 3 − y 3 (d) x4 − y4 40. If ( x + 1) is the HCF of Ax 2 + Bx + C and
35. What is the LCM of x 2 + 2x − 8, x3 − 4x 2 + 4x and Bx 2 + Ax + C where A ≠ B, then the value of C is
x 2 + 4x ? e 2013 II e 2015 II
(a) x(x + 4)(x − 2 )2 (b) x(x + 4)(x − 2 ) (a) A (b) B (c) A − B (d) 0
(c) x(x + 4)(x + 2 )2 (d) x(x + 4)2 (x − 2 ) 41. The sum and difference of two expressions are
36. What is the HCF of a b + 2a b and ( ab) − 4a b ?
2 4 2 2 7 2 9 5x 2 − x − 4 and x 2 + 9x − 10 respectively. The HCF
of the two expressions will be e 2016 I
e 2013 II
(a) ab (b) a2 b 3 (c) a2 b 2 (d) a3 b 2 (a) x + 1 (b) (x − 1) (c) (3x + 7 ) (d) (2 x − 3)

ANSWERS
1 c 2 b 3 c 4 a 5 b 6 a 7 a 8 a 9 d 10 b
11 b 12 d 13 c 14 a 15 b 16 a 17 b 18 c 19 d 20 a
21 c 22 b 23 c 24 a 25 a 26 c 27 c 28 b 29 c 30 a
31 c 32 a 33 c 34 a 35 a 36 c 37 a 38 a 39 c 40 d
41 b

HINTS AND SOLUTIONS 


1. (c) LCM = product of the largest power Product of expressions 6. (a) Let p( x ) = x 4 − x 2 − 6
3. (c) LCM =
of each factor HCF
= x 4 − 3x 2 + 2x 2 − 6
= x2 ( x − 1)( x − 2)( x + 3) a×b
= = ab = x 2 ( x 2 − 3) + 2( x 2 − 3)
2.(b) Here, 2 ( a 2 − b 2 ) = 2 ( a + b ) ( a − b ) 1
= ( x 2 + 2)( x 2 − 3)
3 ( a 3 − b 3 ) = 3 ( a − b ) ( a 2 + ab + b 2 ) 4. (a) Given, ( x + 3)2 ( x − 2)( x + 1)2
q( x ) = x 4 − 4x 2 + 3
and 4 ( a 4 − b 4 ) = 4 ( a + b ) ( a − b ) and ( x + 1) 3 ( x + 3)( x + 4)
= x 4 − 3x 2 − x 2 + 3
( a2 + b2 ) LCM
= ( x − 2)( x + 1) 3 ( x + 3)2 ( x + 4) = x 2 ( x 2 − 3) − 1( x 2 − 3)
LCM of numerical coefficients = 12
and LCM of algebraic expressions = ( x 2 − 3)( x 2 − 1)
5. (b) 4 y 4 x − 9 y 2 x3 = y 2 x( 4 y 2 − 9x2 )
= ( a − b )( a + b )( a + b )2 2 HCF of p( x ), q( x ) = x 2 − 3
= y 2 x( 2 y − 3x)( 2 y + 3x)
( a 2 + ab + b 2 ) 7. (a) A = ( x + 3)2 ( x − 2)( x + 1)2 and
4 y 2 x2 + 6 yx3 = 2 yx2 ( 2 y + 3x)
= ( a 4 − b 4 )( a 2 + ab + b 2 ) B = ( x + 1)2 ( x + 3)( x + 4)
∴ Required HCF = xy( 2 y + 3x)
∴ LCM of polynomials ∴ HCF of polynomials
= 12( a − b )( a + ab + b )
4 4 2 2 = ( x + 3)( x + 1)2
MATHEMATICS HCF and LCM of Polynomials 137

8. (a) Here, HCF of 22 and 36 is 2. 14. (a) Given, Subtracting Eqs. (i) and (ii), we have
Now, x ( x + 1)2 = x ( x + 1) ( x + 1) p( x) = x2 + 2x − 3 = ( x + 3)( x − 1) 2q( x) = 4x2 − 10x + 6
x ( 2x + 3x + 1) = x ( 2x + 1) ( x + 1)
2 2 2 q( x) = x + x − 6 = ( x + 3)( x − 2)
2
⇒ q( x) = 2x2 − 5x + 3
Common factors of x ( x + 1) 2 and LCM = 2x 2 − 3x − 2x + 3
and x 2 ( 2x 2 + 3x + 1) are x ( x + 1). = x − 7x + 6 = ( x − 1)( x + x − 6)
3 2
= x( 2x − 3) − 1( 2x − 3)
Hence, required HCF = 2x ( x + 1) = ( x − 1)( x + 3)( x − 2) = ( x − 1)( 2x − 3)
p ( x) × q ( x)
∴ HCF = ∴ LCM of p( x) and q( x)
9. (d) a − b − c − 2bc = a − ( b + c + 2bc )
2 2 2 2 2 2
LCM = ( x − 1)( 2x − 3)(3x + 7)
= a 2 − ( b + c )2 = ( a + b + c ) ( a − b − c ) ( x + 3)( x − 1) × ( x + 3)( x − 2)
b 2 − c 2 − a 2 − 2ac = b 2 − ( c 2 + a 2 + 2ac ) = 20. (a) p ( x) = ( x + 2)( x + 1)( x2 + 2x + a )
( x − 1)( x + 3)( x − 2)
= b 2 − ( c + a )2 q ( x) = ( x + 3)( x + 4)( x2 + 7x + b )
= ( x + 3) As HCF is ( x + 1)( x + 3), then both
= ( b − c − a) ( b + c + a)
15. (b) Since, ( x + k ) is the HCF, it will ( x + 1) and ( x + 3) must be factors of
= ( a + b + c) ( b − c − a ) divide both the polynomials without p( x) and q( x).
and c 2 − a 2 − b 2 − 2ab leaving any remainder, thus x = − k will For p(x), ( x + 1) is already a factor, so
= c 2 − ( a 2 + b 2 + 2ab ) = c 2 − ( a + b )2 make both of them zero.
( x + 3) must be a factor of x2 + 2x + a.
= ( c − a − b) ( c + a + b) ∴ k − pk + q = k − ak + b
2 2
So, ( −3)2 + 2( −3) + a = 0
= ( a + b + c) ( c − a − b ) or − ak + b = − pk + q
⇒ 9−6+ a=0
∴ Required LCM ⇒ ak − pk = b − q
= ( a + b + c) ( a − b − c) b−q ∴ a = −3
∴ k=
( b − c − a) ( c − a − b) a− p For q (x), ( x + 3) is already factor.
10. (b) Given, [( x + 3) ( x − 2)2 ] ∴( x + 1) must be a factor of
16. (a) Since, ( x + 4) is HCF, so it will
and [( x − 2) ( x − 6)] divide both the expressions i.e. x = − 4 x2 + 7x + b.
∴ LCM = ( x + 3) ( x − 2)2 ( x − 6)
will make each one zero. ∴( −1)2 + 7( −1) + b = 0
∴ 2( −4) + k( −4) − 12 = 0
2
∴ b=6
11. (b) Since, (z − 1) is the HCF, so it will
divide each one of the given polynomials. ⇒ 32 − 12 = 4k So, a = − 3 and b = 6 is solution.
So, z = 1 will make each one zero. ∴ 20 = 4k ⇒ k =5 1
21. (c) Given, f ( x) and g( x) vanish at x =
∴ p ( 1)2 − q ( 1 + 1) = 0 ⇒ p = 2q 17. (b) Since, HCF of x + x − 12 and 2 2
12. (d) Let p( x ) and q( x ) be two polynomials 2x2 − kx − 9 is ( x − k ), then ( x − k ) will So, ( 2x − 1) is a factor of f ( x) and g( x)
and p( x ) ÷ HCF = a be the factor of 2x2 − kx − 9. both.
and q( x ) ÷ HCF = b ∴ 2k 2 − k 2 − 9 = 0 Hence, HCF of f ( x) and
⇒ k2 − 9 = 0 g( x) = 2x − 1 .
∴ p( x ) × q( x ) = LCM × HCF
⇒ k = ±3
⇒ a × HCF × b × HCF = LCM × HCF 22. (b) Refer to question 13.
and factor of x2 + x − 12 are
⇒ a × b × HCF = LCM 23. (c) As ( x − 2) is the HCF of
( x + 4)( x − 3).
LCM ( ax 2 + bx + c ) and ( bx 2 + ax + c )
ab = Hence, value of k is 3.
HCF So, it will divide both the expressions,
3x 4 + 4x 3 − 7x 2 − 4x + 4 18. (c) Here, ( x − 3) is GCD, so is a factor of
∴ a ( 2)2 + b( 2) + c = 0
= both of them.
3x 2 + 4x − 4 ∴ Putting x = 3, in both makes each ⇒ 4a + 2b + c = 0 …(i)
= ( x + 1)( x − 1) polynomial zero. and b ( 2)2 + a ( 2) + c = 0
Let a = ( x + 1) and b = ( x − 1), 3 3 − 2(3)2 + p(3) + 6 = 0 ⇒ p = −5 ⇒ 4b + 2a + c = 0 …(ii)
then the required expression are 32 − 5(3) + q = 0 ⇒ q = 6 adding Eqs. (i) and (ii), we get
( x + 1) (3x2 + 4x − 4) ∴ 5 q + 6 p = 5 ( 6) + ( −5)( 6) ⇒ 6a + 6b + 2c = 0
and ( x − 1) (3x2 + 4x − 4). = 30 − 30 = 0
⇒ 2 c = −6 a − 6 b
13. (c) Here, LCM = 36x3 ( x + a )( x3 − a 3 ) 19. (d) Let the expressions be p( x) and q( x), ⇒ c = −3 ( a + b )
and HCF = x2 ( x − a ) then
p( x) = 4x2 ( x2 − a 2 ) p ( x) + q ( x) = 5x2 − x − 4 …(i) 24. (a) A = p 2 + 8 p + 12 = ( p + 2)( p + 6)
But p( x) × q ( x) = HCF × LCM p ( x) − q( x) = x2 + 9x − 10 …(ii) B = p 2 + 2 p − 24 = ( p − 4)( p + 6)
HCF × LCM On adding Eqs. (i) and (ii), we have C = p 2 + 15 p + 54 = ( p + 9)( p + 6)
q ( x) =
p ( x) 2 p( x) = 6x2 + 8x − 14 I. LCM of A, B
x ( x − a ) 36x ( x + a )( x − a )
2 3 3 3
⇒ p( x) = 3x2 + 4x − 7 and C = ( p + 2)( p + 6)( p − 4)( p + 9)
=
4x 2 ( x 2 − a 2 ) = 3x 2 + 7x − 3x − 7 Thus, I is correct
= 9x 3 ( x 3 − a 3 ) = x(3x + 7) − 1(3x + 7) II. HCF of A, B and C = ( p + 6)
∴ p( x) = (3x + 7)( x − 1) Hence, II is incorrect.
138 CDS Pathfinder

25. (a) I. x2 − 6x + 9 = ( x − 3)( x − 3) 31. (c) Let P (x) = 36(3x4 + 5x3 − 2x2 ) Now, LCM of ( x2 + 2x − 8),
and x − 27 = x − (3)
3 3 3
= 36x (3x + 5x − 2)
2 2
( x3 − 4x2 + 4x) and ( x2 + 4x)
= ( x − 3)( x2 + 3x + 9)
= 36x 2 ( x + 2) (3x − 1) = x ( x − 2)2 ( x + 4)
∴ HCF = x − 3.
Hence, it is true. Q (x) = 9( 6x + 4x − 2x)3 2
36. (c) a b + 2a 2 b 2 = a 2 b 2 ( b 2 + 2) ...(i)
2 4

II. LCM of 10x2 yz , 15xyz = 18x(3x2 + 2x − 1) and ( ab )7 − 4a 2 b 9 = a 7 b 7 − 4a 2 b 9


2 2
and 20xy z is 60x y z . 2 2 2 = 18 x(3x − 1)( x + 1) = a 2 b 2 ( a 5 b 5 − 4b 7 ) ...(ii)
Hence, it is false. R (x) = 54( 27x − x) = 54x( 27x − 1)
4 3
From Eqs. (i) and (ii), HCF = a b 2 2
III. 6x2 − 7x − 3 = ( 2x − 3)(3x + 1) = 54x (3x − 1)( 9x 2 + 1 + 3x )
37. (a) Let p( x) = 8( x5 − x3 + x)
and 2x2 + 11x − 21 HCF of [36, 18, 54] = 18
= ( x + 7)( 2x − 3) = 4 × 2 × x( x4 − x2 + 1)
∴ HCF of [P (x ), Q (x ), R (x )]
Hence, HCF = ( 2x − 3), it is also and q( x) = 28( x6 + 1)
true. = 18x (3x − 1)
= 7 × 4[( x2 )3 + ( 1) 3 ]
Hence, the statement I and III are 32. (a) Let f ( x) = x + 8 = x + 2
3 3 3
correct. = 4 × 7 × ( x2 + 1)( x4 − x2 + 1)
= ( x + 2) ( x − 2x + 4) 2
∴ HCF of p( x) and q( x)
26. (c) We know that,( x + y ) and ( x − y ) are
= ( x + 2) ( x − 2) ( x − 2) = 4( x4 − x2 + 1)
the factors of ( x 10 − y 10 ).
g( x) = x + 5x + 6
2
Hence, statements I and IV are true. 38. (a) Let f ( x) = 2x3 + x2 − x − 2
= x 2 + 3x + 2x + 6
27. (c) We have, A = pq − np = p ( q − n) …(i) = ( x − 1)( 2x2 + 3x + 2)
= ( x + 3) ( x + 2)
C = q − 3nq + 2n
2 2
and g( x) = 3x3 − 2x2 + x − 2
= q − 2nq − nq + 2n
2 2 and h( x) = x3 + 2x2 + 4x + 8
= ( x − 1)(3x2 + x + 2)
= q ( q − 2n) − n( q − 2n) = ( x + 2) ( x 2 + 4)
Hence, the highest factor of f ( x) and
= ( q − n)( q − 2n) …(ii) ∴ HCF of { f ( x), g ( x), h ( x)} = ( x + 2) g( x) is ( x − 1).
E = pq − np − mq + mn 33. (c) Let f (x) = x3 − x2 − 2x 39. (c) Given, HCF = ( x + y ) and
= p( q − n) − m( q − n) = x ( x − x − 2)
2
LCM = 3x5 + 5x4 y + 2x3 y 2
= ( p − m )( q − n) …(iii) = x ( x + 1) ( x − 2) − 3 x 2 y 3 − 5 xy 4 − 2 y 5
∴ HCF of A, C and E = ( q − n)
and g (x ) = x + x 3 2
= x (3x + 5xy + 2 y 2 ) − y 3
3 2
28. (b) We have, E = ( p − m )( q − n) ... (i)
= x2 ( x + 1) = x ⋅ x ( x + 1) (3x 2 + 5xy + 2 y 2 )
D = pq − 2np − mq + 2mn
∴ LCM of [ f ( x), g( x)] = (3x + 5xy + 2 y )( x3 − y 3 )
2 2
= p( q − 2n) − m ( q − 2n)
= x2 ( x + 1) ( x − 2) We know that,
= ( p − m )( q − 2n) …(ii)
∴ LCM of D and E = x (x
2 2
− x − 2) Product of two polynomials
= ( p − m )( q − n)( q − 2n) = HCF × LCM
= x 4 − x 3 − 2x 2
29. (c) The factors of given polynomials are ∴ Required polynomial
34. (a) Let f ( x) = ( x 4 − y 4 )
as follows ( x + y )( x3 − y 3 )(3x2 + 5xy + 2 y 2 )
= ( x2 − y 2 ) ( x2 + y 2 ) =
A = pq − np = p ( q − n) …(i) ( x − y )( x + y )
= ( x − y ) ( x + y ) ( x2 + y 2 )
B = pq − mq = q ( p − m ) …(ii) ( x − y ) ( x 2 + y 2 + xy )
and g( x) = ( x6 − y 6 )
C = ( q − n)( q − 2n) (3x 2 + 5xy + 2 y 2 )
…(iii) = ( x3 + y 3 ) ( x3 − y 3 ) =
D = ( p − m )( q − 2n) …(iv) = ( x + y ) ( x2 − x y + y 2 ) ( x − y ) (x − y)

E = ( p − m )( q − n) …(v) ( x2 + x y + y 2 ) = ( x + y + xy )(3x2 + 5xy + 2 y 2 )


2 2

There is no such factor which is = ( x − y ) ( x + y ) ( x2 − x y + y 2 ) = 3x4 + 8x3 y + 10x2 y 2 + 7xy 3 + 2 y 4


common to all given five polynomials. 40. (d) ( x + 1) is the HCF of
( x2 + x y + y 2 )
Thus, HCF (A, B, C , D, E) = 1
Ax 2 + Bx + C and Bx 2 + Ax + C
∴ HCF of
30. (a) Here, a 3 b − ab 3 = ab ( a 2 − b 2 ) ∴ A( −1)2 + B( − 1) + C = 0
[ f ( x), g( x)] = ( x − y ) ( x + y )
= ab ( a − b ) ( a + b ) ⇒ A−B+C =0
= x2 − y 2 ⇒ C =B− A
a3b2 + a2b3 = a2b2 ( a + b)
35. (a) x 2 + 2 x − 8 = ( x − 2) ( x + 4) and B( − 1)2 + A ( −1) + C = 0
∴ LCM [( a 3 b − ab 3 ), ( a 3 b 2 + a 2 b 3 ), ⇒ B− A+C =0
ab ( a + b )] x 3 − 4x 2 + 4x = x [ x 2 − 4x + 4] ⇒ C = A−B
= a2b2 ( a + b) ( a − b) = x ( x − 2)2 ∴ C =0
= a b (a − b )
2 2 2 2
x + 4x = x ( x + 4)
2
41. (b) Refer to question 19.
16
MATHEMATICS Rational Expressions 139

RATIONAL EXPRESSIONS
Usually (1-2) questions have been asked from this chapter. Generally questions are asked from this
chapter are based on simplification of rational expressions.

EXAMPLE 1. The lowest term of an expression


RATIONAL EXPRESSIONS a3 −b3
p ( x) is
An expression in the form of , where p ( x) and q ( x) a 2 + ab + b 2
q ( x) a
a. a + b b. a − b c. ab d.
are polynomials and q ( x) ≠ 0 is called a rational b
expression. Sol. b. Rational expression
p ( x) a3 − b 3 ( a − b) ( a2 + ab + b2)
• In the rational expression , p ( x) is called the = 2 = =a−b
q ( x) a + ab + b
2
( a2 + ab + b2)

numerator and q ( x) is called the denominator of the [Q( a3 − b3) = ( a − b)( a2 + ab + b2)]
rational expression. EXAMPLE 2. The lowest term of an expression
• Every polynomial is a rational expression. Since, p ( x) 3x 2 − 11x − 4
p ( x) is
can always be written as and a constant function 1 6x 2 − 7x − 3
1
x+ 4 x+ 4 x−4 x−4
is a polynomial of degree 0. a. b. c. d.
2x + 3 2x − 3 2x + 3 2x − 3
• Every rational expression need not be a polynomial.
3x2 − 11x − 4 3x2 − 12x + x − 4
Sol. d. =
Working Rule to Reduce the Given 6x2 − 7x − 3 6x2 − 9x + 2x − 3
3x ( x − 4) + 1( x − 4) ( 3x + 1) ( x − 4) x−4
Rational Expression in its Lowest Term = = =
3x ( 2x − 3) + 1( 2x − 3) ( 3x + 1) ( 2x − 3) 2x − 3
1. Firstly, factorize both the polynomials p ( x) and q ( x).
EXAMPLE 3. The lowest term of an expression
2. Find the HCF of p ( x) and q ( x). If HCF of p ( x) and 12x 3 y 5 z 4
p ( x) is
q ( x) is one, then rational expression is in its 18x 2 y 6 z 5
q ( x)
2x xy 2x 3x
lowest terms. a. b. c. d.
yz z 3yz 2yz
3. If HCF is not equal to 1. Then, divide both p ( x) and
12x3y 5z 4 6 × 2 × x2 × x × y 5 × z 4 2x
q ( x) by their HCF and the rational expression is Sol. c. 2 6 5 = =
18x y z 6 × 3 × x2 × y × y 5 × z × z 4 3yz
obtained in the lowest terms.
140 CDS Pathfinder

p ( x)
Operations on Rational 3. Multiplication of Rational Expressions If
q ( x)
and
Expressions g ( x)
are two rational expressions, then their product
1. Addition or Subtraction of Rational Expressions h ( x)
with like denominators p ( x) g ( x) p ( x) ⋅ g ( x)
is given by × =
P ( x) h ( x) q ( x) h ( x) q ( x) ⋅ h ( x)
It and are two rational expressions then
q ( x) q ( x) p ( x) q ( x)
• Multiplicative inverse of is .
p ( x) h ( x) p ( x) ± h ( x) q ( x) p ( x)
± =
q ( x) q ( x) q ( x) • 1 is the multiplicative identity.
2. Addition or subtraction of Rational Expressions
with unlike denominators x2 −1 x +2
EXAMPLE 6. The product of and is
To add or subtract rational expressions with unlike x +1
2
x +1
denominators, follow the steps given below ( x − 1) ( x − 2)
a. b.
Step I Write each denominator in the factor form ( x2 + 1) ( x2 + 1)
Step II Find the LCM of the denominators ( x − 1)( x + 2) x+ 2
c. d.
Step III Rewrite each rational expression with LCM as ( x2 + 1) ( x2 − 1)
the denominator
x2 − 1 ( x + 2) ( x − 1) ( x + 1) ( x + 2)
Step IV Add or subtract the numerators. Sol. c. Here, product = × =
x +1
2
( x + 1) ( x2 + 1) ( x + 1)
x +1 x −1 [Q a2 − b2 = ( a − b)( a + b)]
EXAMPLE 4. The sum of and is ( x − 1) ( x + 2)
x −1 x +1 =
( x2 + 1)
x +1
2
1 2x + 2
2
x +2
2
a. b. c. d. p ( x) g ( x)
x2 − 1 x2 + 1 x2 − 1 x2 − 1 4. Division of Rational Expressions If and
q ( x) h ( x)
x + 1 x − 1 ( x + 1) ( x + 1) ( x − 1) ( x − 1)
Sol. c. + = × + × are two rational expressions, then their division is
x − 1 x + 1 ( x − 1) ( x + 1) ( x + 1) ( x − 1)
p ( x) g ( x) p ( x) h ( x)
( x + 1) 2 + ( x − 1) 2
given by ÷ = ×
= q ( x) h ( x) q ( x) g ( x)
( x2 − 1)
p ( x) g ( x)
= Product of and the reciprocal of
x2 + 1+ 2x + x2 + 1− 2x 2x + 2
2
= = 2 q ( x) h ( x)
x −1
2
x −1
h ( x)
i.e. .
g ( x)
[Q ( a + b) 2 = a2 + b2 + 2ab and ( a − b) 2 = a2 + b2 − 2ab]

EXAMPLE 5. What should be subtracted from EXAMPLE 7. The lowest term of an expression
7x 4 x 2 + 8x + 12 x 2 + 4x − 12
to get ? ÷ is
x + x − 12
2
x+4 x 2 − 7 x + 12 x −4
a.
1
b.
3
c.
1
d.
3 ( x − 2) x−2
a. b.
x−3 x−3 x+ 3 x+ 3 ( x − 3)( x + 2) x+ 2
( x + 2) x+ 3
Sol. b. Let p( x) is to be subtracted, then c. d.
7x 4 ( x − 3)( x − 2) x−2
− p( x) =
x + x − 12
2
x+ 4 x2 + 8x + 12 x2 + 4x − 12
7x 4 7x 4 Sol. c. Here, ÷
⇒ p( x) = 2 − = − x2 − 7x + 12 x−4
x + x − 12 x + 4 ( x + 4) ( x − 3) ( x + 4)
x2 + 6x + 2x + 12 x−4
7x 4( x − 3) 7x − 4 ( x − 3) = ×
= − = x2 − 4x − 3x + 12 ( x2 + 6x − 2x − 12)
( x + 4)( x − 3) ( x + 4)( x − 3) ( x + 4) ( x − 3)
( x + 6) ( x + 2) x−4 x+ 2
3x + 12 3 ( x + 4) 3 = × =
= = = ( x − 4) ( x − 3) ( x + 6) ( x − 2) ( x − 3) ( x − 2)
( x + 4) ( x − 3) ( x + 4) ( x − 3) x − 3
MATHEMATICS Rational Expressions 141

PRACTICE EXERCISE
1. Which of the following are rational expressions?  x − 1  x + 1
8. If A =   and B =  x − 1 , then ( A + B ) is
2

I.
x3 − 3x2 + 2
II.
z3 − 3z 2  x + 1  
x2 + 1 2z + 3 4x4 + 8x2 − 4 4x4 + 8x2 + 4
(a) (b)
x −x+2
2
x + 3x − 1
3 2
x − 2x + 1
4 2
x4 − 2 x2 + 1
III. IV.
x+3 x2 + x − 1 4x4 + 8x2 + 4
(c) (d) None of these
Select the correct answer using the codes given below x4 + 2 x + 1
(a) I, II and III (b) II, III and IV
9. The value of
(c) All of these (d) None of these
1 a2 b2
a + 2 ( a + 1) + − is
2. The simplified form of − . (1 − a ) (1 − b) (1 − a ) ( b − a ) ( b − 1) ( a − b)
a + 3 ( a + 2)
1 a+2 (a) 1 + a (b) 1 − a2
(a) (b)
a2 + 5a + 6 a2 + 5a + 6 (c) 1 − b 2 (d) None of these
(a + 2 ) 2
(c) 0 (d)  x y  y z   z x
a 2 + 5a + 6  y − x   z − y  x − z 
10. The value of    is
 x + 1 2 1 11 11 1
3.  2 −  expressed as a rational expression is  2 − 2  2 − 2  2 − 2
 x − 1 x x y  y z  z x 
x2 − 2 − (x − 2 ) (a) x2 y2 z2 (b) − x2 y2 z2
(a) (b)
x (x2 − 1) x (x − 1) (c) 1 (d) None of these
2 (x + 1) 1
(c) (d) 11. What is the simplified form of
(x3 − 1) x (x + 1)
 x 2 − 3x + 2  x 2 − 9   x3 + 2x 2 + 4x 
( x − 1) ( x − 2) ( x 2 − 9x + 14)   ÷   ×   ?
4. The expression in  x3 − 8   x + 7x + 12  x 2 + 3x − 4 
2

( x − 7) ( x 2 − 3x + 2) x x −2
(a) (b)
lowest terms is x−3 x−3
1 1 x x+ 3
(a) (b) (x − 7 ) (c) (x − 2 ) (d) (c) (d)
x−7 x−2 x+ 3 x+ 4
a−c b− c 12. If pq + qr + rp = 0, then what is the value of
5. + is equal to
( a − b) ( x − a ) ( b − a ) ( x − b) p2 q2 r2
b−a x−c + + ?
(a) (b) p2 − qr q 2 − rp r 2 − pq
( x − b ) ( x − a) ( x − a) ( x − b )
(a) 0 (b) 1
b−a
(c) (d) None of these (c) −1 (d) 3
(x − b) (x − c )
13. If x + y + z = 0, then what is the value of
x−3
6. The sum of the rational expression and its 1 1 1
x2 + 1 + + ?
reciprocal is x2 + y2 − z 2 y2 + z 2 − x2 z 2 + x2 − y2
x3 − 3x2 + x − 3 x4 + 3x2 − 6x + 10
(a) (b) 1
x + 3x − 6x + 10
4 2
x3 − 3x2 + x − 3 (a) (b) 1
x2 + y2 + z2
x − 3x + 6x + 10
4 2
(x − 1) 3
(c) (d) (c) −1 (d) 0
x3 − 3x2 + x − 3 (x + 3)2
1+ x 1 2a + 1
14. If a = , then what is + 2 equal to?
7. The value of 2− x a+1 a −1
a b c (1 + x)(2 + x) (1 − x)(2 − x)
+ + (a) (b)
( a − b) ( a − c) ( b − c) ( b − a ) ( c − a ) ( c − b) 2x − 1 x −2
(a) 2a (b) 0 (c) 2b (d) b − c (1 + x)(2 − x ) (1 − x)(2 − x)
(c) (d)
2x − 1 2x + 1
142 CDS Pathfinder

15. If
1
+
2
+
1009
= 1, then what is the PREVIOUS YEARS’ QUESTIONS
x + 1 y + 2 z + 1009
x y z ( x 2 + y 2 )( x − y ) − ( x − y )3
value of + + ? 18. What is equal to?
x + 1 y + 2 z + 1009 x 2 y − xy 2 e 2013 II
(a) 0 (b) 2 (c) 3 (d) 4 (a) 1 (b) 2 (c) 4 (d) − 2
1 1 2b 4b3 8b7
x 2 − 3x + 2 x 2 − 5x + 4 19. What is − − 2 − −
16. What is ÷ equal to? a − b a + b a + b2 a 4 + b4 a 8 − b8
x 2 − 5x + 6 x 2 − 7x + 12
x +3 x+ 1 equal to? e 2014 II
(a) (b) 1 (c) (d) 2
x−3 x+ 3 (a) a + b (b) a − b (c) 1 (d) 0

17. If x + y + z = 0, then what is


xyz 20. If a − by − cz = 0, ax − b + cz = 0 and
2 2

( x + y )( y + z )( z + x ) ax + by − c2 = 0, then the value of


equal to (where, x ≠ − y , y ≠ − z , z ≠ − x ) ? x y z
+ + will be
(a) −1 (b) 1 a+ x b+ y c+ z e 2016 I
(c) x y + yz + zx (d) None of these (a) a + b + c (b) 3 (c) 1 (d) 0

ANSWERS
1 a 2 a 3 b 4 c 5 b 6 b 7 b 8 b 9 d 10 b
11 a 12 b 13 d 14 c 15 b 16 b 17 a 18 b 19 d 20 c

HINTS AND SOLUTIONS


x 3 + 3x 2 − 1 a−c b−c −a b
1. (a) Here, is not a rational 5. (b) Here, + = −
x2 + x − 1 ( a − b )( x − a ) ( b − a )( x − b ) ( a − b )( c − a ) ( b − c )( a − b )
( a − c )( x − b ) − ( b − c )( x − a ) c
expression, since the denominator is not
= −
a polynomial. ( a − b )( x − a )( x − b ) ( c − a )( b − c )
a + 2 ( a + 1) ( − a )( b − c ) − b( c − a ) − c( a − b )
− ax − ab − xc + bc − ( bx − ab − cx + ac ) =
2. (a) We know = ( a − b )( b − c )( c − a )
a + 3 ( a + 2) ( a − b )( x − a )( x − b )
( a + 2)2 − ( a + 1)( a + 3) − ab + ac − bc + ab − ac + bc
= ax + bc − bx − ac =
( a + 3)( a + 2) = ( a − b )( b − c )( c − a )
( a − b )( x − a )( x − b ) 0
a 2 + 4 + 4a − ( a 2 + 4a + 3) x( a − b ) − c ( − b + a ) = =0
= = ( a − b )( b − c )( c − a )
a 2 + 5a + 6 ( a − b )( x − a )( x − b )
1 8. (b) Here,
= 2 ( x − c )( a − b ) ( x − c)
a + 5a + 6 = = x − 1   x + 1 
2
( a − b )( x − a )( x − b ) ( x − a )( x − b ) ( A + B )2 =   + 
x + 1 2  + 1  x − 1  
 x
3. (b) Here,  2
1 2
−  = − x −3 x2 + 1
 x − 1 x x − 1 x 6. (b) Here, reciprocal of is
2
 (x − 1)2 + (x + 1)2   2 (x2 + 1) 
2

x + 1 ( x − 3)
2
=  = 2 
[Q x2 − 1 = ( x + 1)( x − 1)]  (x + 1) (x − 1)   x − 1 
x − 2x + 2 −( x − 2) So,
= = x −3 x2 + 1 ( x − 3)2 + ( x2 + 1)2 [Q ( a + b )2 + ( a − b )2 = 2( a 2 + b 2 )]
x ( x − 1) x ( x − 1) + =
x +1
2
x −3 ( x2 + 1)( x − 3) 4( x 4 + 2x 2 + 1) 4x 4 + 8x 2 + 4
= = 4
4. (c) We know that, x + 9 − 6x + x 4 + 1 + 2x 2
2
x 4 − 2x 2 + 1 x − 2x 2 + 1
=
( x − 1)( x − 2)( x2 − 9x + 14) x 3 − 3x 2 + x − 3
9. (d) Given,
( x − 7)( x2 − 3x + 2) x + 3x2 − 6x + 10
4
= 1 a2
x 3 − 3x 2 + x − 3 +
( x − 1)( x − 2)( x2 − 7x − 2x + 14) ( 1 − a )( 1 − b ) ( 1 − a )( b − a )
=
( x − 7)( x2 − 2x − x + 2) 7. (b) We know that, b2

( x − 1)( x − 2)( x − 7)( x − 2) a
+
b ( b − 1)( a − b )
= =x−2
( x − 7)( x − 2)( x − 1) ( a − b )( a − c ) ( b − c )( b − a ) ( b − a) + a2( 1 − b) − b 2( 1 − a)
c =
+ ( 1 − a )( 1 − b )( b − a )
( c − a )( c − b )
MATHEMATICS Rational Expressions 143

b − a + a 2 − a 2 b − b 2 + ab 2 1
+
2
+
1009
=1
= 15. (b) Given,
( 1 − a )( 1 − b )( b − a ) x + 1 y + 2 z + 1009
( b − a ) + ( a 2 − b 2 ) + ab( b − a ) 1 2 1009
= ⇒ − 1+ − 1+ − 1 = 1− 3
( 1 − a )( 1 − b )( b − a ) x+ 1 y+2 z + 1009
( b − a ) − ( a + b )( b − a ) + ab( b − a ) 1 − a − b + ab x y z
= = =1 ⇒ − − − = −2
( 1 − a )( 1 − b )( b − a ) 1 − b − a + ab x + 1 y + 2 z + 1009
x y z
∴ + + =2
 x y   y z  z x x + 1 y + 2 z + 1009
 −   −   − 
 y x z y x z
10. (b) Given, x 2 − 3x + 2 x 2 − 5x + 4
 1 1 1 1 1 1 16. (b) Given, ÷
 2 − 2  2 − 2  2 − 2 x2 − 5x + 6 x2 − 7x + 12
 x y  y z   z x  x − 3x + 2 ( x2 − 7x + 12)
2
= 2 ×
 x − y   y − z  z − x 
2 2 2 2 2 2
x − 5x + 6 ( x 2 − 5x + 4)
   
=  2
xy   zy   xz  ( x − 1)( x − 2) ( x − 4)( x − 3)
= × =1
 y − x  z − y   x − z 
2 2 2 2 2
( x − 3)( x − 2) ( x − 4)( x − 1)
 2 2  2 2  2 2 
 x y  z y  xz  17. (a) Given, x + y + z = 0
( x2 − y 2 )( y 2 − z 2 )(z 2 − x2 ) ⇒ x + y = −z , y + z = − x and z + x = − y
x2 y 2 z2 x 4 y 4z 4 ∴
xyz
=
xyz
=
xyz
= −1
=− =− = − x 2 y 2z 2 ( x + y )( y + z )(z + x) ( −z )( − x)( − y ) − xyz
( x − y )( y − z )(z − x )
2 2 2 2 2 2
x 2 y 2z 2
x 4 y 4z 4 ( x 2 + y 2 )( x − y ) − ( x − y )3
18. (b) Given,
 x 2 − 3x + 2   x 2 − 9   x 3 + 2x 2 + 4x  x 2 y − xy 2
11. (a) Given,   ÷ 2  × 2 
 x − 8   x + 7x + 12   x + 3x − 4 
3
x 3 + xy 2 − x 2 y − y 3 − ( x 3 − y 3 − 3x 2 y + 3xy 2 )
=
 x − 3x + 2 x + 7x + 12   x + 2x + 4x 
2 2 3 2 x 2 y − xy 2
= ×  × 
 x −8
3
x 2 − 9   x 2 + 3x − 4  x 3 + xy 2 − x 2 y − y 3 − x 3 + y 3 + 3x 2 y − 3xy 2
=
( x − 1)( x − 2) ( x + 4)( x + 3) x( x2 + 2x + 4) x 2 y − xy 2
= × ×
( x − 2)( x + 4 + 2x) ( x − 3)( x + 3) ( x − 1)( x + 4)
2 2x 2 y − 2xy 2 2( x 2 y − xy 2 )
= = =2
=
x
[Q a 3 − b 3 = ( a − b )( a 2 + b 2 + ab )] x 2 y − xy 2 x 2 y − xy 2
x −3 1 1 2b 4b 3 8b 7
19. (d) − − 2 − 4 − 8
12. (b) Given, pq + qr + r p = 0 a−b a+ b a + b 2
a +b 4
a − b8
p2 q2 r2 ( a + b) − ( a − b) 2b 4b 3
8b 7
∴ + 2 + 2 = − 2 − 4 − 8
p − qr q − r p r − pq
2 ( a − b )( a + b ) a +b 2
a +b 4
a − b8
3 7
p2 q2 r2 2b 2b 4b 8b
= + 2 + 2 = 2 − − −
p + rp + pq q + pq + qr r + qr + rp
2 a − b2 a2 + b2 a4 + b4 a8 − b8
2b( a 2 + b 2 ) − 2b( a 2 − b 2 ) 4b 3 8b 7
p2 q2 r2 p+ q+ r = − 4 − 8
= + + = =1 ( a − b )( a + b )
2 2 2 2
a +b 4
a − b8
p ( p + r + q) q ( p + q + r) r ( p + q + r) p + q + r 3 3 7
4b 4b 8b
= 4 − −
13. (d) Given, x + y + z = 0 ⇒ x + y = − z. a − b4 a4 + b4 a8 − b8
On squaring both sides, we get x2 + y 2 + 2xy = z 2 4b 3 ( a 4 + b 4 ) − 4b 3 ( a 4 − b 4 ) 8b 7
= − 8 =0
x 2 + y 2 − z 2 = − 2 xy ( a − b )( a + b )
4 4 4 4
a − b8

Similarly, y 2 + z 2 − x 2 = −2 yz and z 2 + x 2 − y 2 = −2zx 20. (c) Given, a 2 − by − cz = 0 …(i)



1
+
1
+
1 ax − b 2 + cz = 0 …(ii)
x2 + y 2 − z 2 y 2 + z 2 − x2 z 2 + x2 − y 2 and ax + by − c 2 = 0 …(iii)
1 1 1 1z + x + y  On adding Eqs. (i), (ii) and (iii), we get
= + + =  =0
−2xy −2 yz −2zx 2  xyz  b 2 − a 2 + c2 x b2 − a 2 + c2
x= ⇒ = 2
2a a+ x a + b 2 + c2
1+ x
3   y a 2 − b 2 + c2 z a 2 + b 2 − c2
1+ x 1 2a + 1 3a  2 − x Similarly, = 2 and =
14. (c) Given, a = So, + 2 = 2 = b+ y a +b +c 2 2
c+z a 2 + b 2 + c2
2−x a + 1 a − 1 a − 1  1+ x 2
  −1
 2 − x ( b 2 − a 2 + c2 ) + ( a 2 − b 2 + c2 )
3( 1 + x)( 2 − x) 3( 1 + x)( 2 − x) ( 1 + x)( 2 − x) + ( a 2 + b 2 − c2 )
= = = x y z
∴ + + = =1
1 + x + 2x − ( 4 + x − 4x)
2 2
6x − 3 ( 2x − 1) a+ x y + b c+z a 2 + b 2 + c2
17
144 CDS Pathfinder

LINEAR EQUATIONS
Generally (3-5) questions have been definitely asked from this chapter in CDS examination. Beside this,
the linear equations are also applicable in solving various word problems related to number system,
mensuration, time and work, etc.

A linear equation is an equation for a straight line.


So, the equation which has degree 1, i.e., which has linear power of the variables, is called a linear
equation.

LINEAR EQUATIONS IN ONE VARIABLE


A linear equation in one variable is an equation which can be written in the form of ax + b = 0,
or ax = c, where a, b, c are real numbers with a ≠ 0.
2 y
e.g. 5x + 8 = 9 − x , y + 7 = and t + 3t = 9 − t are linear equations in one variable.
3 2

Solving a linear Equation in One Variable


1. Simplify both sides of the equation.
2. Use the addition and subtraction properites to get all variables of terms on the LHS and all constant
terms on the RHS.
3. Simplify and divide both sides of the equation by the coefficient of the variable.

EXAMPLE 1. Solve 2(x − 3) − (5 − 3x) = 3 (x + 1) − 4 (2 + x).


a. 1 b. −1 c. 0 d. 3
Sol. a. 2 ( x − 3) − ( 5 − 3x) = 3 ( x + 1) − 4 ( 2 + x) ⇒ 2x − 6 − 5 + 3x = 3x + 3 − 8 − 4x
⇒ 5x − 11 = − x − 5 ⇒ 6x = 6 ⇒ x = 1, Hence, the value of x is 1.

LINEAR EQUATION IN TWO VARIABLES


An equation which can be put in the form ax + by + c = 0, where a, b and c are real numbers and a, b
both are not zero, is called a linear equation in two variables.
e.g. 2x + 3y = 5, 2x + 3y = 0 and 2a + 3b = 0 are linear equations in two variables.
MATHEMATICS Linear Equations 145

Sol. a. Given equations are, kx + 2y − 5 = 0 and


Pair of Linear Equations
3x + y − 1 = 0
Two linear equations in the two same variables are called On comparing the given equations, with standard form
pair of linear equations in two variables. The general form of equations
of pair of linear equations in two variables x and y is a1x + b1y + c1 = 0 and a2x + b2y + c2 = 0
a1 x + b1 y + c1 = 0 and a 2 x + b2 y + c 2 = 0 We have, a1 = k, b1 = 2, c1 = − 5
where, a1 , a 2 , b1 , b2 and c1 , c 2 are all real numbers and and a2 = 3, b2 = 1, c2 = − 1
a12 + b12 ≠ 0, a 22 + b22 ≠ 0.
(i) Here, the equations have a unique solution, if
a1 b1

Consistency Pair of a2 b2
k 2
linear Equations i.e. if ≠
3 1
⇒ k ≠6
A pair of linear equations which has no solution is called (ii) The equations have no solution, if
an inconsistent pair of linear equations. A pair of linear a1 b1 c1
= ≠
equations which has atleast one solution is called a a2 b2 c2
consistent pair of linear equations. Let us consider two k 2
linear equations as a1 x + b1 y + c1 = 0 and a 2 x + b2 y + c 2 = 0. ∴ = ⇒ k =6
3 1
Pair of linear equations in two variables will represent
two straight lines, both to be considered together. The EXAMPLE 3. The value of k for which the system
table given below repersents the three possibilities. of equations kx − y = 2, 6 x − 2y = 3 has infinitely many
Ratios Consistency Graphical Algebraic solutions is
Representation interpretation a. 3 b. 4
a1 b Consistent Exactly one c. 12 d. No such value of k exists.
≠ 1
a2 b 2 solution (unique).
Sol. d. Given equations are,
kx − y − 2 = 0 and 6x − 2y − 3 = 0
Intersecting lines. On comparing the given equations, with standard form
a1 b c Dependent of equations
= 1 = 1 Infinitely many
a2 b 2 c 2
(consistent) solutions. a1 x + b1 y + c1 = 0 and a2 x + b2 y + c 2 = 0
We have, a1 = k , b1 = − 1, c1 = − 2
and a2 = 6, b2 = − 2, c2 = − 3
Coincident lines.
Here, the equations have infinite number of solutions, if
a1 b c Inconsistent No solution.
= 1 ≠ 1 a1 b1 c1
a2 b 2 c 2 = =
a2 b2 c2
a1 k b1 − 1 1 c −2 2
But, = , = = and 1 = =
Parallel lines. a2 6 b2 − 2 2 c2 −3 3
b1 c1
So, ≠ .
b2 c2
HOMOGENEOUS PAIR OF Thus, the system has no such value of k, for which the
LINEAR EQUATIONS given system has infinitely many solutions.

The pair of linear equations a1 x + b1 y = 0, a 2 x + b2 y = 0 has Algebraic Methods of Solutions


a b
1. A unique solution, if 1 ≠ 1 and the solution is Different algebraic methods for solving pair of linear
x = 0, y = 0. a 2 b2 equations are discussed below in detail.
a b
2. An infinite number of solutions, if 1 = 1 .
a 2 b2 1. Substitution Method
This pair of linear equations is always consistent. In this method, we substitute the value of one variable
in terms of other variable to solve the pair of linear
EXAMPLE 2. The values of k for which the system equations, so this method is called substitution method.
kx + 2y = 5 and 3x + y = 1 has
Steps used in this method are given below.
(i) unique solution (ii) no solution are
a. k ≠ 6 and k = 6 b. k ≠ 3 and k = 3 Step I Find the value of one variable x (or y) in terms of
c. k ≠ 2 and k = 2 d. None of these other variable i.e. y (or x) from an equation.
146 CDS Pathfinder

Step II Substitute this value of x (or y) in other equation, 3. Cross-multiplication Method


and reduce it to a linear equation in one variable
Let us consider a general system of two simultaneous
i.e. in terms of y (or x) which can be solved easily.
linear equations
Step III Substitute the value of y (or x) obtained in Step II a1 x + b1 y + c1 = 0 …(i)
in the equation which is used to obtain the value a 2 x + b2 y + c 2 = 0 …(ii)
of the other variable in Step I.
a1 b1
where, ≠ then the solution of the system is given by,
EXAMPLE 4. The solutions of the system of a2 b2
equations x + y = 14 and x − y = 4, is x=
b1c2 − b2 c1 c a − c2 a1
and y = 1 2
a. x = 5 and y = 9 b. x = 9 and y = 5 a1b2 − a2 b1 a1b2 − a2 b1
x y 1
c. x = 5 and y = 3 d. x = 3 and y = 5 ∴ = =
b1c2 − b2 c1 c1a2 − c2 a1 a1b2 − a2 b1
Sol. b. Given equations are, x + y = 14 …(i) This method is called cross multiplication method.
and x−y =4 …(ii)
from eq. (ii), y = x−4 …(iii) Remembering Technique
On substituting the value of y from Eq. (iii) in Eq. (i), we get (Cross-multiplication)
x+ x − 4 = 14 ⇒ 2x = 18 ⇒ x = 9 The following diagram helps in remembering the above
On substituting x = 9 in Eq. (iii), we get solution.
y =9 − 4 = 5 ⇒ y = 5 x y 1
Hence, x = 9 and y = 5 b1 c1 a1 b1

2. Elimination Method b2 c2 a2 b2

In this method, one variable is eliminated from both The arrows between the two numbers indicate that they
equations to have an equation in one variable, so it are to be multiplied. The downward arrows indicate first
is called elimination method. product while upward arrows indicate the second product.
The second product is to be subtracted from the first.
Steps used in this method are given below.
Step I Make the coefficient of one variable ( x or y ) EXAMPLE 6. The system of equations
numerically equal by multiplying both equations ax − by = a 2 − b 2 and x + y = a + b has the solution.
by some suitable non zero constant. a. x = a and y = b b. x = − a and y = − b
Step II Now, add or subtract both equations, so that one c. x = b and y = a d. x = − b and y = − a
variable is eliminated and the remaining equation
Sol. a. Given, pairs of linear equations are
is in one variable only.
a x − by − ( a2 − b2) = 0 ...(i)
Step III Solve the equation in one variable to get the value and x + y − ( a + b) = 0 ...(ii)
of this variable (x or y). On comparing Eqs. (i) and (ii) with a1x + b1y + c1 = 0
Step IV Substitute this value (x or y) in either of the given and a2x + b2y + c2 = 0, we get
equations to get the value of other variable. a1 = a, b1 = − b, c1 = − ( a2 − b2)
a2 = 1, b2 = 1, c2 = − ( a + b)
EXAMPLE 5. The solution of the given system of
By cross multiplication, we have
equations 2x + 5y = 11 and 3x + 4y = 13 is x y 1
a. (4, 2) b. (3, 1) c. (5, 2) d. (1, 1) –b –(a2–b2) a –b
Sol. b. Given equations are, 2x + 5y =11 ...(i)
1 –(a+b) 1 1
and 3x + 4y = 13 ...(ii) x y 1
⇒ = =
On multiplying Eq. (i) by 3 and Eq. (ii) by 2, we get b( a + b) + ( a2 − b2) − ( a2 − b2 + a( a + b)) a + b
6x + 15y = 33 …(iii) x y 1 ab + a2 b2 + ab
⇒ = = ⇒ x = and y =
6x + 8y = 26 …(iv) ab + a2 b2 + ab a + b a+ b a+ b
On subtracting Eq. (iv) from Eq. (iii), we get a( a + b) b ( b + a)
⇒ x= = a and y = =b
(6 x + 15y) − (6x + 8y) = 33 − 26 ⇒ 7y = 7 ⇒ y = 1 a+ b a+ b
So, x = a and y = b is the required solution.
Put y = 1 in Eq. (i), 2x + 5 × 1 = 11
⇒ 2x = 11 − 5 ⇒ 2x = 6 ⇒ x = 3
Hence, solution of system of equations is (3, 1).
MATHEMATICS Linear Equations 147

⇒ 4A + 6 = 18 ⇒ 4A = 12 ⇒ A = 3
EQUATIONS REDUCIBLE TO A 1 1 1 1
Now = B = 2 ⇒ x = and = A = 3⇒ y =
PAIR OF LINEAR EQUATIONS x 2 y 3
1 1
We have several situations when there are two So, x = and y = is the required solution.
2 3
equations that are not linear, but can be reduced to a pair
of linear equations. By making suitable substitutions. 3 2 9 4
EXAMPLE 9. If + = 2 and − = 1,
2 3 5 4 x+y x−y x+y x−y
EXAMPLE 7. If + = 13 and − = − 2 , then x
x y x y then what is value of ?
x y
what is the value of ? 3 2 1
y a. b. 5 c. d.
2 3 5
2 3 1 1
a. b. c. d.
3 2 3 2 Sol. b. Given equations are,
1 1 3 2
Sol. b. Let = p and = q + =2 …(i)
x y x+ y x−y
So, we have, 2p + 3q = 13 9 4
…(i) and − =1 …(ii)
and 5p − 4q = − 2 …(ii) x+ y x−y
1 1
On multiplying Eq. (i) by 4 and Eq. (ii) by 3 and then Let = a and =b
adding, we get x+ y x−y
8 p + 15p = 52 − 6 ⇒ 23p = 46 ⇒ p = 2 ∴ 3a + 2b = 2 …(iii)
On putting p = 2 in Eq. (i), we get 9a − 4b = 1 …(iv)
9 On multiplying Eq. (iii) by 2 and adding with Eq. (iv), we
2 × 2 + 3q = 13 ⇒ 3q = 13 − 4 = 9 ⇒ q = = 3
3 get
1 1 1 1 ⇒ 6 a + 9 a = 5 ⇒ 15a = 5
Now, x= ⇒ x = and y = ⇒ y= .
p 2 q 3 1
∴ a=
x 3 3
Hence, the value of is . 1 1
y 2 ∴ = ⇒ x+ y =3 …(v)
x+ y 3
EXAMPLE 8. Find the solution of the system of On putting the value of a in Eq. (iii), we get
equations 4x + 3y = 18xy and 2x − 5y = − 4xy. 1
3 × + 2b = 2 ⇒ b =
1
1 1 1 1 3 2
a. x = 2, y = 3 b. x = 3, y = 2 c. x = , y =
d. x = , y = 1 1
3 2 2 3 ⇒ = ⇒ x−y =2 …(vi)
Sol. d. Given equations are, 4x + 3y = 18xy ...(i) x−y 2
5
and 2x − 5y = −4xy ...(ii) On adding Eqs. (v) and (vi), we get, 2x = 5 ⇒ x =
2
On dividing both the equations by xy, we get 5 1
From Eq. (v), x + y = 3 ⇒ y = 3 − =
4x 3y 18xy 2x 5y − 4xy 2 2
+ = and − = 5
xy xy xy x y xy xy
x 2
4 3 2 5 ∴ = =5
or + = 18 and − =−4 y 1
y x y x
2
1 1
On putting = A and = B, we get
y x
4A + 3B = 18 …(iii)
LINEAR EQUATION IN
2A − 5B = − 4 …(iv) THREE VARIABLES
On multiplying Eq. (iv) by 2 and subtracting from Eq. (iii), An equation in the form of ax + by + cz = r , where a, b, c
we get and r are real numbers and a, b and c are not all zeros is
4A + 3B = 18
called a linear equation in three variables
4A − 10B = − 8
− + + e.g. 3x + 4y − 7 z = 2 , −2x + y − z = 6 and x + y + z = 2 are all
13B = 26 linear equation in three variables.
⇒ B=2 Use substitution and elimination method to solve the
On putting, the value of B in Eq. (iii), we get system of three equations in three variables.
148 CDS Pathfinder

EXAMPLE 10. Solve the system x + y + z = 5,


2x − y + z = 9 and x − 2y + 3z = 16.
Problem Based on Numbers
Let the digit in unit’s place be x and that in ten’s place
a. ( 2, − 1, 4 ) b. ( 3, 5, 6 ) c. (1, − 2, 5 ) d. ( 6 , 1, − 3)
be y. Then, the two-digits number is given by 10 y + x.
Sol. a. Given equations are, x + y + z = 5 …(i) On interchanging the positions of the digits, the digit in
2x − y + z = 9 …(ii) unit place becomes y and that in ten’s place becomes x,
x − 2y + 3z = 16 …(iii) and the number becomes 10 x + y. Formulate the
First we eliminate y by adding Eqs. (i) and (ii), we get equations and then solve them.
x+ y + z = 5
2x − y + z = 9 EXAMPLE 12. A number consists of two digits,
3x + 2z = 14 …(iv) whose sum is 8. If 18 is added to the number, the
digits are reversed. The number is equal to
Next we eliminate y by multiplying Eq. (i) by 2 and then
adding it to Eq. (iii), we get a. 26 b. 35 c. 53 d. 62
2x + 2y + 2z = 10 Sol. b. Let the unit's place digit and ten's place digit be x
x − 2y + 3z = 16 and y, respectively.
3x + 5z = 26 …(v) ∴Original number = 10y + x
Now subtract Eq. (iv) from Eq. (v), we get Number obtained by reversing the order of digit = 10x+ y
3z = 12 ⇒ z = 4 Condition I Sum of digits = 8
Substitute z = 4 in Eq. (iv), 3x + 2 × 4 = 14 ⇒ 3x = 6 ⇒ x = 2. ⇒ x+ y =8 …(i)
Finally put x = 2 and z = 4 in Eq. (i), we get Condition II 10x + y = 10y + x + 18
2 + y + 4 = 5 ⇒ y = −1 ⇒ 10x − 10y + y − x = 18 ⇒ 9x − 9y = 18
⇒ x−y = 2 …(ii)
∴ (2, −1, 4) is the required solution.
On adding Eqs. (i) and (ii), we get
x + y + x − y = 8 + 2 ⇒ 2x = 10
APPLICATION OF LINEAR ∴ x=5
EQUATIONS On putting x = 5 in Eq. (i), we get
5+ y = 8 ⇒ y = 3
The application of linear equations are discussed below.
Hence, Required number = 10y + x = 10 × 3 + 5 = 35
Problem Based on Ages
If the problem involves finding out the ages of two Problem Based on Fractions
persons, then take the present age of one person as x Let the numerator of the fraction be x and denominator
and that of the other as y. be y, then the fraction is x / y.
Then, ‘a’ years ago, age of Ist person was ( x − a) years Formulate the linear equations on the basis of
and that of 2nd person was ( y − a) years. conditions given and solve them.
After b years, age of Ist person will be ( x + b) years and
EXAMPLE 13. If we add 1 to the numerator and
that of 2nd person will be ( y + b) years. subtract 1 from the denominator, a fraction becomes
Formulate the equations and then solve them. 1
1. It also becomes , if we only add 1 to the
2
EXAMPLE 11. Age of X is six times that of Y. After 4 yr, denominator. The fraction is
X is 4 times elder to Y. What is the present age of Y? a. 4/5 b. 3/5 c. 2/5 d. 1/5
a. 4 yr b. 5 yr c. 6 yr d. 7 yr
Sol. b. Let the numerator and the denominator of the
Sol. c. Let the present age of X and Y be x and y yr, fraction be x and y, respectively.
respectively. x
∴ Fraction =
Condition I Age of X = Age of Y × 6, x = 6y ...(i) y
After 4 yr, Age of X = ( x + 4) yr , Age of Y = ( y + 4) yr Condition I When 1 is added to the numerator and 1 is
Condition II ( x + 4) = ( y + 4) × 4, x + 4 = 4y + 16 ...(ii) x+1
subtract from the denominator, then new fraction =
y −1
Put the value of x = 6y in Eq. (ii), we get
⇒ 6y + 4 = 4y + 16 ⇒ 2y = 12 According to the question,
x+1
∴ y =6 = 1 ⇒ x + 1= y − 1 ⇒ x − y = − 2 …(i)
So, the present age of Y is 6 yr. y −1
MATHEMATICS Linear Equations 149

Condition II When 1 is added to denominator, then EXAMPLE 15. A railway ticket for a child costs half
x
new fraction = the full fare but the reservation charge is the same on
y+1 half tickets as much as on full ticket. One reserved first
x 1
According to the question, = ⇒ 2x = y + 1 class ticket for a journey between two stations is ` 362,
y+1 2
⇒ 2x − y = 1 …(ii) one full and one half reserved first class tickets cost
On subtracting Eq. (ii) from Eq. (i), we get ` 554. What is the reservation charge?
−x = − 3 ⇒ x = 3 a. ` 18 b. ` 22 c. ` 38 d. ` 46
Put the value of x in Eq. (i), we get Sol. b. Let full fare and reservation charge be ` x and
3 − y = − 2⇒ y = 5 ` y, respectively. Then, full ticket = x + y
x 3
Hence, the fractions is = x
y 5 half ticket = + y
2
According to question, x + y = 362 ...(i)
Problem Based on Distance, and ( x+ y) +  + y = 554 = x + 2y = 554 ...(ii)
x 3
Speed and Time 2  2
Speed is the distance covered by an object per unit time. On multiplying Eq. (i) by 2, we get
Distance 2x + 2y = 724 ...(iii)
i.e. Speed =
Time On subtracting Eq. (ii) from Eq. (iii), we get
If the speed of boat in still water be x km/h and speed x
= 170 ⇒ x = 340
of stream be y km/h. Then, the speed of boat 2
downstream = ( x + y) km/h On putting value of x in Eq. (i), we get
and speed of boat upstream = ( x − y) km/h 340 + y = 362 ⇒ y = 22
Hence, the reservation charge is ` 22.
EXAMPLE 14. A motorboat takes 2h to travel a
distance of 9 km down the current and it takes 6h to Problems Based on Mensuration
travel the same distance against the current. What is Make use of given formulae to formulate the equation
the speed of the boat in still water ?
1. Area of rectangle = length × breadth
a. 3 km/h b. 2 km/h c. 1.5 km/h d. 1 km/h
Perimeter of rectangle = 2 (length+Breadth)
Sol. a. Let speed of the motorboat in still water be x km/h
and speed of the stream be y km/h. 2. The sum of angles of a triangle is 180°
Then, speed of the motorboat downstream = ( x + y) km/h 3. The sum of opposite angles of cyclic quadrilateral is
and speed of the boat upstream = ( x − y) km/h 180°
Condition I When motorboat goes 9 km downstream, then 4. In parallelogram, opposite angles are equal.
9  Distance 
2= Q Time =  EXAMPLE 16. The length of a rectangle is 8 cm
x+ y  Speed 
more than its breadth. If the perimeter of the
⇒ 2x + 2y = 9 …(i) rectangle of 68 cm, then its length and breadth are
Condition II When motorboat goes 9 km upstream, then a. 21 cm, 13 cm b. 14 cm, 23 cm
9 c. 19 cm, 20 cm d. 9 cm, 15 cm
6= ⇒ 6x − 6y = 9 …(ii)
x−y Sol. a. Let the length and breadth of a rectangle be x and y,
On multiplying Eq. (i) by 3, we get respectively.
6x + 6y = 27 …(iii) Condition I x= y + 8 ⇒ x− y =8 …(i)
Now, adding Eqs. (ii) and (iii), we get
Condition II Perimeter of rectangle = 68 cm
12x = 36 ⇒ x = 3 km/h
⇒ 2[ x + y ] = 68, x + y = 34 …(ii)
Problem Based on Fixed On adding Eq. (i) and (ii), we get
and running Charges 2x = 42 ⇒ x = 21
Let the fixed charge of any commodity be ` x and rate On putting x = 21in Eq. (ii), we get
of running charges be ` y, then total cost = x + total 21 + y = 34 ⇒ y = 34 − 21 = 13
running charges. Hence, length of rectangle = 21cm
Formulate the equations and then solve them. and breadth of rectangle = 13 cm
150 CDS Pathfinder

EXAMPLE 17. The area of a rectangle is decreased ⇒ xy − 2x + 2y − 4 = xy − 28


by 28 m 2 , if the length is increased by 2 m and the ⇒ 2x − 2y = 24 ⇒ x − y = 12 …(i)
breadth is decreased by 2 m. The area of a rectangle Condition II If, length = ( x − 1) m and breadth = ( y + 2) m
is increased by 33 m 2 , if the length is decreased by
1m and the breadth is increased by 2 m, then the area ∴ Now the area of rectangle = ( x − 1)( y + 2) m 2
of rectangle is According to the question,
a. 253 m 2
b. 235 m 2
c. 532 m 2
d. 352 m 2
( x − 1)( y + 2) = xy + 33
Sol. a. Let the length and breadth of a rectangle be x and y, ⇒ xy + 2x − y − 2 = xy + 33
respectively. ⇒ 2x − y = 35 …(ii)
∴ Original area of rectangle = xy m 2 On subtracting Eq. (i) from (ii), we get, x = 23
Condition I If length = ( x + 2) m and breadth = ( y − 2) m On putting x = 23 in Eq. (i), we get,
∴ Area of rectangle = ( x + 2)( y − 2) m 2 23 − y = 12
According to the question, ∴ y = 23 − 12 = 11
( x + 2)( y − 2) = xy − 28 So, original area of rectangle = xy = 23 × 11= 253 m 2

PRACTICE EXERCISE
1. Find x, if 25x − 19 − [3 − { 4x − 5}] = 3x − ( 6x − 5). 7. The system of equations 6x + 5 y = 11 and
1 15
(a) x = 1 (b) x = − 1 (c) x = (d) x = 2 9x + y = 21 has
2 2
2. If a number is subtracted from three-fourth of (a) a unique solution (b) many solution
itself, the value so obtained is − 130. Then, what (c) no solution (d) None of these
is the number?
8. The sum of two numbers is 2490 and if 6.5% of
(a) 540 (b) 560 (c) 420 (d) 520
one number is equal to 8.5% of the other, then
3. Sum of two numbers is 21 and their difference is numbers are
11, then the greatest number is (a) 1414, 1076 (b) 1411, 1079
(a) 5 (b) 16 (c) 9 (d) 10 (c) 1412, 1078 (d) None of these
4. Which of the following equations have x = 2 and 9. Given two linear equations a1x + b1 y = c1 and
y = 1 as a solution? a1 b1
a2x + b2 y = c2 , if ≠ , then the graph is
I. 2x + 5 y = 9 II. 5x + 3 y = 14 a2 b2
III. 2x + 3 y = 7 IV. 2x − 3 y = 1 (a) parallel (b) intersection at one point
Select the correct answer using the codes given (c) coincident (d) None of these
below
10. For what value of k, the following equations will
(a) I and IV (b) II and III (c) Only I (d) I, III and IV
be inconsistent? 4x + 6 y = 11 and 2x + ky = 7
x 12
5. The solution of the pair of equation + y = 0.8 (a) k = − 3 (b) k = (c) k = 12 (d) k = 3
2 5
7
and = 10 is 11. For what value of k, the system of equations has
y
x+ infinitely many solutions 2x − ky = 4 and
2
2 3 2 3x + 2 y = 6?
(a) x = , y = (b) x = , y = 5 4 −4 2 3
5 5 3 (a) (b) (c) (d)
2 5 3 2 3 3 3 2
(c) x = , y = (d) x = , y =
5 3 5 5 12. For what value of α, the system of equation
6. A system of two simultaneous linear equations in αx + 3 y = α − 3, 12x + αy = α will have a unique
two variables has a unique solution if their graphs solution?
(a) are coincident (b) are parallel
(a) α = ± 6 (b) α = 6 (c) α ≠ ± 6 (d) α = − 6
(c) intersect in one point (d) None of these
MATHEMATICS Linear Equations 151

13. Consider the following statements: 44 30 55 40


23. If + = 10, + = 13, then find
I. The solution of equation 2x + 3 y = 425 and x+ y x − y x+ y x − y
3x + 2 y = 350 is a positive integral pair. the value of x and y.
II. for k = −2, the equations x − ky = 2 and 3x + 6 y = −5 (a) x = 2, y = 8 (b) x = 8, y = 3
have infinitely many solutions. (c) x = 8, y = 2 (d) x = 3, y = 8
III. (2, 5) and (−1, 3) are solutions of 2x + 5 y = 13 24. Consider the following sets of equations
Select the correct answer using the codes given I. 2x − y = 0 and 6x − 3 y = 0
below: II. 3x − 4 y = 0 and 12x − 20 y = 0 Then,
(a) Only I (b) Both I and II (a) Both sets I and II possess unique solution
(c) Both II and III (d) All of these (b) Set I possesses unique solution and set II has
infinitely many solutions
14. A horse and two cows together cost ` 680. If a
horse costs ` 80 more than a cow, then the cost (c) Set I possesses infinitely many solutions and set II
possess unique solution
of horse is
(d) None of the sets I and II possesses a unique solution
(a) ` 170 (b) ` 280 (c) ` 200 (d) ` 220
4
15. Sunita has 10 paise and 50 paise coins in her 25. A fraction becomes , if 1 is added to both
5
purse. If the total number of coins is 17 and numerator and denominator. If however 5 is
their total value is ` 4.50, then number of 10 subtracted from both numerator and denominator
paise coins is 1
(a) 9 (b) 7 (c) 10 (d) 5 the fraction becomes . Then, the fractions is
2
7 9 3 4
16. The solution of the equation (a) (b) (c) (d)
3x − y + 1 2x + y + 2 3x + 2 y + 1 9 7 5 3
= = is given by
3 5 6 26. For what value of k, the following system of
which one of the following? equations 3x + 4 y = 6 and 6x + 8 y = k represents
(a) x = 2, y = 1 (b) x = 1, y = 1 coincident lines?
(c) x = − 1, y = − 1 (d) x = 1, y = 2 (a) 12 (b) 11 (c) 13 (d) 10

17. The line 3x − 5 y = −10 cuts Y-axis at 27. The area of a rectangle remains the same if the
(a) (0, 2) (b) (0, 1) (c) (0, 3) (0, 4) length is increased by 7 m and the breadth is
decreased by 3 m. The area remains unaffected if
18. The equations px + q = 0 and rx + s = 0 are the length is decreased by 7 m and the
consistent, if breadth is increased by 5 m, then area of
(a) ps = qr (b) ps + qr = 0 (c) pq − rs = 0 (d) pq + rs = 0 rectangle is
(a) 280 m2 (b) 320 m2 (c) 420 m2 (d) 400 m2
19. A streamer goes downstream and covers the
distance between two ports in 4 h while it covers 28. The value of x in the solution of the equation
the same distance upstream in 5 h. If the speed 2x + y
= 2x − y
= 8 is
of the stream is 2 km/h, then the speed of the 3 1 1
(a) 0 (b) (c) (d)
streamer in still water is 2 4 8
(a) 20 km/h (b) 19 km/h (c) 18 km/h (d) 19.5 km/h
29. A and B each have a certain number of mangoes.
20. The solution of the system of linear equations A says to B : “If you give 30 of your mangoes, I
0.4x + 0.3 y = 1.7 and 0.7x − 0.2 y = 0.8 is will have twice as many as left with you” B
(a) x = 3, y = 2 (b) x = 2, y = −3 replies “If you give me 10, I will have thrice as
(c) x = 2, y = 3 (d) None of these many as left with you”. How many mangoes did
A has?
21. If 2x − 3 y = 0 and 7x + 2 y = 0, then find the (a) 41 (b) 62 (c) 34 (d) 32
value of x + y.
(a) 1 (b) 2 (c) 3 (d) 0 30. There are two examination rooms A and B. If 10
candidates are sent from room A to room B,
22. A man starts his job with a certain monthly the number of students in each room is the
salary and earns a fixed increment every year. If same. If 20 candidates are sent from B to A, the
his salary was ` 1500 after 4 yr of services and number of students in A is double the number
` 1800 after 10 yr of service. What was his starting of students in B. Then, number of students in
salary? room B is
(a) ` 1300 (b) ` 1200 (c) ` 50 (d) ` 1100 (a) 40 (b) 100 (c) 80 (d) 60
152 CDS Pathfinder

31. A train started from a station with a certain 41. What is the value of k for which the system of
number of passengers. At the first halt, 1/3rd of equations x + 2 y − 3 = 0 and 5x + ky + 7 = 0 has no
its passengers got down and 120 passengers got solution?
in. At the second halt, half of the passengers 3 14 1
(a) − (b) − (c) (d) 10
got down and 100 persons got in. Then, the 14 3 10
train left for its destination with 240 passengers.
How many passengers were there in the train 42. What is the solution of the equation x − y = 0.9
when it started ? and 11( x + y )−1 = 2 ?
(a) 540 (b) 480 (c) 360 (d) 240 (a) x = 3.2 and y = 2.3 (b) x = 1 and y = 0.1
(c) x = 2 and y = 11
. (d) x = 1.2 and y = 0. 3
32. The system of equations x + 2 y = 3 and 3x + 6 y = 9
has 43. Pooja started her job with certain monthly
(a) unique solutions (b) no solution salary and gets a fixed increment every year.
(c) infinitely many solutions (d) finite number of solutions If her salary was ` 4200 after 3 yr and ` 6800
after 8 yr of service, then what are her
33. The sum of digits of a two-digit number is 8 and initial salary and the annual increment,
the difference between the number and that respectively ?
formed by reversing the digits is 18. What is the (a) ` 2640, ` 320 (b) ` 2460, ` 320
difference between the digits of the number? (c) ` 2460, ` 520 (d) ` 2640, ` 520
(a) 1 (b) 2 (c) 3 (d) 4
44. A person bought 5 tickets from the station P to a
34. If ( x , y ) = ( 4, 1) is the solution of the pair of linear station Q and 10 tickets from the station P to a
equations mx + y = 2x + ny = 5, then what is station R. He paid ` 350. If the sum of fare of a
m + n equal to? ticket from P to Q and a ticket from P to R is `
(a) −2 (b) −1 (c) 1 (d) 1 42, then what is the fare from P to Q ?
a+2 b+1 (a) ` 12 (b) ` 14
35. If 2 + 3 = 17 and 2
a b
−3 = 5, then
(c) ` 16 (d) ` 18
(a) a = 2, b = 3 (b) a = − 2, b = 3
(c) a = 2, b = − 3 (d) a = 3, b = 2 45. The Community Relief fund receives a large
donation of $ 2800. The foundation agrees to
36. The sum of two numbers is 80. If the larger spend the money on $ 20 school bags, $ 25
number exceeds four times the smaller by 5, sweaters, $ 5 books. They want to buy 200 items
what is the smaller number ? and send them to schools in earthquake-hit
(a) 5 (b) 15 (c) 20 (d) 25
areas. They must order as many books as school
2 3 9 4 9 21 bages and sweaters combined.
37. If + = and + = , where x, y ≠ 0
x y xy x y xy How many of each item should they order?
and y ≠ 0, then what is the value of x + y ? (a) (40, 60, 100) (b) (20, 30, 80)
(a) 2 (b) 3 (c) 4 (d) 8 (c) (50, 100, 60) (d) 40, 80, 25)

38. Under what condition do the equation kx − y = 2 Directions (Q. Nos. 46-47) Some money is divided
and 6x − 2 y = 3 have a unique solution? among Rajesh, Sonal and Chetan in such a way that
(a) k = 3 (b) k ≠ 3 (c) k = 0 (d) k ≠ 0 2 times share of Rajesh, 3 times share of Sonal and
5 times share of Chetan are all equal.
39. Let there be three simultaneous linear equations
in two unknowns, which are non-parallel and Now, answer the following questions based on above
information.
non-collinear. What can be the number of
solutions (if they do exist)?
46. If the sum of 6 times the share with Rajesh and
(a) One or infinite (b) Only one
6 times the share with Sonal is ` 150, then find
(c) Exactly two (d) Exactly three
the share of Chetan?
40. A number consists of two digits, whose sum is (a) ` 5 (b) ` 6 (c) ` 10 (d) ` 12
10. If 18 is subtracted from the number, digits of
the number are reversed. What is the product? 47. If the sum of shares of Rajesh, Chetan and Sonal
is ` 155, then find the share of Rajesh.
(a) 15 (b) 18
(c) 24 (d) 32 (a) ` 75 (b) ` 35 (c) ` 50 (d) ` 25
MATHEMATICS Linear Equations 153

PREVIOUS YEARS’ QUESTIONS 57. A bus starts with some passengers. At the first
stop, one-fifth of the passengers gets down
48. The graphs of ax + by = c, dx + ey = f will be and 40 passengers get in. At the second stop,
I. Parallel, if the system has no solution. half of the passengers gets down and 30 get in.
II. Coincident, if the system has finite number of The number of passengers now is 70. The
solutions. number of passengers with which the bus started
III. Intersecting, if the system has only one solutions was e 2013 II
(a) 40 (b) 50 (c) 60 (d) 70
Which of the statement(s) given above is/are
correct? e 2012 I 58. If x + y − 7 = 0 and 3x + y − 13 = 0, then what is
(a) Both I and II (b) Both II and III 4x 2 + y 2 + 4xy equal to? e 2013 II
(c) Both I and III (d) All of these
(a) 75 (b) 85 (c) 91 (d) 100
49. The sum of two numbers is 20 and their product is x y 2 3
75. What is the sum of their reciprocals? e 2012 I 59. If + = 4 and + = 1, then what is x + y
2 3 x y
1 1 4 7
(a) (b) (c) (d) equal to? e 2013 II
15 5 15 15
(a) 11 (b) 10 (c) 9 (d) 8
50. If 3x + y = 81 and 81x − y = 3, then what is the value
60. If x + y = 5, y + z = 10 and z + x = 15, then which
of x? e 2012 I
17 17 17 15 one of the following is correct? e 2014 I
(a) (b) (c) (d)
16 8 4 4 (a) z > x > y (b) z > y > x (c) x > y > z (d) x > z > y
a b x a b 61. The present age of Ravi’s father is 4 times Ravi’s
51. If − = and + = x − y, then what is the
b a y b a present age. 5 yr back, Ravi’s father was seven
value of x ? e 2013 I
times as old as Ravi was at that time. What is
a+ b a+ b a−b the present age of Ravi’s father? e 2014 I
(a) (b) (c) (d) None of these
a b a (a) 84 yr (b) 70 yr (c) 40 yr (d) 35 yr

52. The system of equations 3x + y − 4 = 0 and 62. The sum of two positive number x and y is 2.5
6x + 2 y − 8 = 0 has e 2013 I times their difference. If the product of numbers
(a) a unique solution x = 1, y = 1 is 84, then what is the sum of those two
(b) a unique solution x = 0 , y = 4 numbers? e 2014 I
(c) no solution (a) 26 (b) 24 (c) 22 (d) 20
(d) infinite solutions 63. Two chairs and one table cost ` 700 and one
53. The sum of two numbers is 7 and the sum of chair and Two tables cost ` 800. If cost m tables
their squares is 25. The product of the two and m chairs is ` 30000, then what is m equal
numbers is e 2013 I to? e 2014 I
(a) 6 (b) 10 (c) 12 (d) 15 (a) 60 (b) 55 (c) 50 (d) 45

Directions (Q. Nos. 54-55) A number consists of two 64. A certain number of two digits is three times the
sum of its digits. If 45 is added to the number,
digits whose sum is 10. If the digits of the number
then the digits will be reversed. What is the sum
are reversed, then the number is decreased by 36.
of the squares of the two digits of the number?
54. Which of the following is/are correct? (a) 41 (b) 45 e 2014 II
I. The number is divisible by a composite number. (c) 53 (d) 64
II. The number is a multiple of a prime number. 65. A student was asked to multiply a number by 25.
Select the correct answer using the codes given He instead multiplied the number by 52 and got
below e 2013 I the answer 324 more than the correct answer.
(a) Only I (b) Only II The number to be multiplied was e 2015 I
(c) Both I and II (d) Neither I nor II (a) 12 (b) 15 (c) 25 (d) 32

55. What is the products of the two digits? 4


e 2013 I 66. A tin of oil was full. When 6 bottles of oil were
(a) 21 (b) 24 (c) 36 (d) 42 5
taken out from this tin and 4 bottles of oil were
56. Ten chairs and six tables together cost ` 6200, poured into it, it was 3 / 4 full. Oil of how many
three chairs and two tables together cost ` 1900. bottles can the tin contain? (All bottles are of
The cost of 4 chairs and 5 tables is e 2013 I equal volume) e 2015 II
(a) ` 3000 (b) ` 3300 (c) ` 3500 (d) ` 3800 (a) 35 (b) 40 (c) 45 (d) 50
154 CDS Pathfinder

67. A number consists of two digits, whose sum is 7. 70. There are three brothers. The sums of ages of
If the digits are reversed, the number is increased two of them at a time are 4 yr, 6 yr and 8 yr.
by 27. The product of digits of the number is The age difference between the eldest and the
e 2015 II youngest is e 2016 I
(a) 6 (b) 8 (c) 10 (d) 12 (a) 3 yr (b) 4 yr (c) 5 yr (d) 6 yr
p q q p x 71. The annual incomes of two persons are in the
68. If + = m and + = n, then what is equal
x y x y y ratio 9 : 7 and their expenses are in the ratio
to? e 2016 I 4 : 3. If each of them saves ` 2000 per year, then
np + mq np + mq what is the difference in their annual incomes?
(a) (b)
mp + nq mp − nq e 2016 I
np − mq np − mq (a) ` 4000 (b) ` 4500 (c) ` 5000 (d) ` 5500
(c) (d)
mp − nq mp + nq
72. Let a two digits number be k times the sum of its
69. The value of k, for which the system of equations digits. If the number formed by interchanging
3x − ky − 20 = 0 and 6x − 10 y + 40 = 0 has no the digits is m times the sum of the digits, then
solution, is e 2016 I the value of m is e 2016 I
(a) 10 (b) 6 (c) 5 (d) 3 (a) 9 − k (b) 10 − k (c) 11 − k (d) k − 1

ANSWERS
1 a 2 d 3 b 4 d 5 a 6 c 7 c 8 b 9 b 10 d
11 b 12 c 13 a 14 b 15 c 16 b 17 a 18 a 19 c 20 c
21 d 22 a 23 b 24 c 25 a 26 a 27 c 28 b 29 c 30 c
31 d 32 c 33 b 34 a 35 d 36 b 37 c 38 b 39 b 40 c
41 d 42 a 43 d 44 b 45 a 46 b 47 a 48 c 49 c 50 b
51 d 52 d 53 c 54 b 55 a 56 a 57 b 58 d 59 b 60 a
61 c 62 d 63 a 64 c 65 a 66 b 67 c 68 c 69 c 70 b
71 a 72 c

HINTS AND SOLUTIONS 


1. (a) We have, 25 x − 19 − [3 − { 4x − 5}] 3. (b) Let numbers be x and y, respectively. ⇒ 13 = 14, it is false.
= 3x − ( 6x − 5) Then by given condition, III. 2x + 3 y = 7 ⇒ 2 ( 2) + 3 ( 1) = 7
⇒ 25x − 19 − [3 − 4x + 5] x + y = 21 …(i) 7 = 7 , it is true.
= 3x − 6x + 5 x − y = 11 …(ii) IV. 2x − 3 y = 1 ⇒ 2 ( 2) − 3 ( 1) = 1
⇒ 25x − 19 + 4x − 8 = − 3x + 5 On adding Eqs. (i) and (ii), we get
1 = 1 , it is true.
⇒ 29x + 3x = 5 + 27 2x = 32
32 So, x = 2 and y = 1
32 x= = 16
⇒ 32x = 32 ⇒ x = =1⇒ x=1 2 is a solution of I, III and IV.
32
On putting y = 16 in Eq. (i), we get
Hence, the value of x is 1. 5. (a) Given equations are,
x + 16 = 21
x + 2 y = 16
. ...(i)
2. (d) Let the number be x. ∴ x=5
y 7
According to the question, Hence, the greatest number is 16. and x+ = ...(ii)
3x 3x − 4x 2 10
− x = − 130 ⇒ = − 130 4. (d) Put x = 2 and y = 1in each equation
4 4 On multiplying Eq. (i) by 10 and
I. 2x + 5 y = 9 ⇒ 2 ( 2) + 5 ( 1) = 9
−x Eq. (ii) by 10, we get
⇒ = − 130 ⇒ x = 520 9 = 9 , it is true.
4 or 10x + 20 y = 16 …(iii)
II. 5x + 3 y = 14
Hence, the number is 520. ⇒ 5( 2) + 3( 1) = 14 and 10x + 5 y = 7 …(iv)
MATHEMATICS Linear Equations 155

On subtracting, Eq. (iv) from Eq. (iii), a 2 b1 −k c 4 On subtracting Eq. (i) from Eq. (iii), we
we get Now, 1 = , = and 1 = get
a 3 b 2 c 6
( 10x + 20 y ) − ( 10x + 5 y ) = 16 − 7
2 2 2 ( x + 5 y ) − ( x + y ) = 45 − 17
Since, the system has infinitely many 28
⇒ 15 y = 9 ⇒ y =
9
⇒y=
3 ⇒ 4 y = 28 ⇒ y = =7
solutions 4
15 5
1 = 1 = 1, 2 = − k = 2 ∴ Number of 10 paise coins
a b c
On putting the value of y in Eq (i), we
get a
2
b
2
c 3
2
2 3 = x = 17 − y = 17 − 7 = 10
8 6 8 −4
x + 2y = , x + = ⇒ − 3k = 4 ⇒ k = 16. (b) Given,
5 5 5 3
8 6 2 3x − y + 1 2x + y + 2 3x + 2 y + 1
⇒ x= − = 12. (c) Given equations are, = =
5 5 5 3 5 6
αx + 3 y = α − 3 and 12x + αy = α ( I) ( II) ( III)
6. (c) A pair of linear equations in two
Here, a = α, b = 3, c = α − 3 Taking Ist and IInd terms,
variables has a unique solution if their 1 1 1
graphs intersect in one point. a = 12, b = α, c = α 5(3x − y + 1) = 3( 2x + y + 2)
2 2 1
a 6 2 b 5 2 Since, system has unique solution, ⇒ 9x − 8 y = 1 …(i)
7. (c) Here, 1 = = , 1 = = a b
1 ≠ 1 ⇒ α 3 Taking IInd and IIIrd terms
a 9 3 b
2
15 / 2 3
2 So, ≠
c a b c a b 12 α 6 ( 2x + y + 2) = 5(3x + 2 y + 1)
11 2 2
and 1 = ⇒ 1 = 1 ≠ 1 ⇒ α ≠ 36
2
⇒ α≠± 6 ⇒ 3x + 4 y = 7 …(ii)
c 21 a b c
2 2 2 2
13. (a) I. 2x + 3 y = 425, 3x + 2 y = 350 On solving Eqs. (i) and (ii), we get y = 1
So, the system has no solution. and x = 1
Solving both the equations, we get
8. (b) Let the numbers be x and 2490 − x. x = 40 and y = 115.
65
. 13x 17. (a) Given, 3x − 5 y = −10
6.5% of x = ×x= II. When k = −2
100 200 At Y-axis, x = 0
1 = 1, 1 = 2 = 1, 1 = −2
.
85 a b c 10
8.5% of ( 2490 − x) = ( 2490 − x) ⇒ 0 − 5 y = −10 ⇒ y = =2
100 a 3 b 6 3 c 5 5
2 2 2
17 a b c So, the point on Y-axis is (0, 2).
= ( 2490 − x) Q 1 = 1 ≠ 1
200 a b c 18. (a) Given, px + q = 0 and rx + s = 0
13x 17( 2490 − x) 2 2 2
−q −s
By given condition, = Hence, the given equations have no ⇒ x= and x =
200 200 solution. p r
⇒ 13x = 17( 2490 − x) −q −s
III. (2, 5) is not a solution of So, = ⇒ ps = qr
⇒ 13x + 17x = 42330 2x + 5 y = 13 as p r
42330
⇒ x= = 1411 2( 2) + 5( 5) = 4 + 25 = 29 ≠ 13. 19. (c) Let the speed of the streamer in still
30
Hence, only I is correct. water = x km/h
∴ Second number = ( 2490 − x)
then speed of streamer downstream
= 2490 − 1411 = 1079 14. (b) Let cost of one horse and one cow be
`x and `y, respectively. = ( x + 2) km/h
9. (b) As this is the case of unique solution speed of streamer upstream
a b Condition I x + 2 y = 680 ...(i)
1 ≠ 1 , so the graph of the equations
= ( x − 2) km/h
a
2
b Condition II x = y + 80
2
Distance travelled by streamer
will intersect at one point. ⇒ x − y = 80 ...(ii)
downstream in 4h
On subtracting Eq. (ii) from Eq. (i), we
10. (d) Given equations are, = 4 ( x + 2) km
get
4x + 6 y = 11 and 2x + ky = 7 ( x + 2 y) − ( x − y ) = 680 − 80 [Q distance = time × speed]
Here, a = 4, b = 6, c = 11 ⇒ 3 y = 600 and distance travelled by streamer
1 1 1
and a = 2, b = k , c = 7 ∴ y = 200 upstream in 5 h = 5( x − 2)
2 2 2
a 4 b 6 c 11 On putting y = 200 in Eq. (i), we get So, 4 ( x + 2) = 5 ( x − 2)
Here, 1= , 1 = and 1 =
a 2 b k c 7 x + 2 × 200 = 680 ⇒ 4x + 8 = 5x − 10
2 2 2
Since, system is inconsistent, so ⇒ x + 400 = 680 ⇒ x = 680 − 400 Hence, x = 18 km/h is the speed of
a b c ∴ x = 280 streamer in still water.
1 = 1 ≠ 1 ⇒ 4 = 6
a b c 2 k Hence, the cost of one horse is ` 280. 20. (c) Given, 0.4x + 03
. y = 17
. and
2 2 2
0.7x − 0.2 y = 0.8
⇒ 4k = 12 ⇒ k = 3 15. (c) Let number of 10 paise coins be x and
Hence, the value of k is 3. number of 50 paise coins be y. On multiplying above equation by 10,
we get
11. (b) Given equations are, According to the questions,
x + y = 17 4x + 3 y = 17 ...(i)
2x − ky = 4 and 3x + 2 y = 6 Then, …(i)
and 7x − 2 y = 8 ...(ii)
Here, a = 2, b = − k, c = 4 and 10x + 50 y = 450 …(ii)
1 1 1
and a = 3, b = 2, c = 6 On multiplying Eq. (i) by 2 and Eq. (ii)
From Eq. (ii), x + 5 y = 45 …(iii)
2 2 2 by 3, we get
156 CDS Pathfinder

8x + 6 y = 34 ...(iii) 24. (c) Set I 2x − y = 0 and 6x − 3 y = 0 27. (c) Let the length of rectangle be x m
and 21x − 6 y = 24 ...(iv) Here, a = 2, b = − 1 and breadth be y m.
1 1
On adding Eqs. (iii) and (iv), we get and a = 6 , b = −3 ∴ Original area of rectangle = xy
2 2
(8x + 6 y ) + ( 21x − 6 y ) = 34 + 24 a
1 = 2 = 1 ⇒
b
1 = 1
Case I Length = ( x + 7) m and breadth

⇒ 29x = 58 a 6 3 b 3 = ( y − 3) m
∴ x=2 2 2
∴ New area of rectangle
a b
On putting x = 2 in Eq. (i), we get ∴ 1 = 1
a b = ( x + 7) ( y − 3)
8 + 3 y = 17 ⇒ y = 3 2 2
Given, xy = xy − 3x + 7 y − 21
Hence, x = 2 and y = 3 is the required So, system of equations have infinitely
⇒ 3x − 7 y = − 21 …(i)
solution. many solutions.
21. (d) Given, 2x − 3 y = 0 Case II Length = ( x − 7)
Set II 3x − 4 y = 0 and 12x − 20 y = 0
7x + 2y = 0 and Breadth = ( y + 5)
and Here, a = 3, b = − 4
Here, a = 2, b = − 3
1 1
∴ New area of rectangle
1 1 and a = 12, b = − 20
2 2 = ( x − 7) ( y + 5)
a = 7 and b = 2 a b −4
2 2
∴ 1 = 3 = 1 ⇒ 1 = =
1 Given, xy = xy + 5x − 7 y − 35
As the equations are homogeneous a 12 4 b −20 5
equations and also 2 2 ⇒ 5x − 7 y = 35 …(ii)
a b 1 1 On subtracting Eq. (ii) from Eq. (i), we
1 ≠ 1 . So, equation has one solution, Here ≠
4 5 get
a b 3x − 7 y = − 21
2 2 So, system of equatons has unique
∴ x = y = 0 ⇒x+ y = 0 solution. 5x − 7 y = 35
Hence, the value of x + y is zero. x
25. (a) Let the fraction be . – + −
22. (a) Let the starting salary be ` x and y − 2x = − 56
x+ 1 4
annual increment be ` y.
Condition I = ∴ x = 28 m
By given condition, x + 4 y = 1500 ...(i) y+ 1 5 On putting x = 28 in Eq. (ii), we get
and x + 10 y = 1800 ...(ii) 5 × 28 − 7 y = 35 ⇒ − 7 y = 35 − 140
⇒ 5( x + 1) = 4 ( y + 1)
On subtructing Eq. (i) from Eq. (ii), we
⇒ 5x − 4 y = − 1 …(i) ∴ y = 15
get ∴ Area of rectangle
x −5 1
6 y = 300 ⇒ y = 50 Condition II =
y −5 2 = xy = 15 × 28 = 420 m 2
On putting y = 50 in Eq. (i), we get
⇒ 2( x − 5) = ( y − 5) 28. (b) Given, 2x + y
= 8 , 2x − y
= 8
x + 200 = 1500 ⇒ 2x − y = 5 x+ y x− y
…(ii) or 2 = 2 2, 2 =2 2
∴ x = ` 1300 On multiplying Eq. (ii) by 4 and or 2x + y
= 23 / 2 , 2x − y
= 23 / 2
1 1
23. (b) Put = A and =B subtracting from Eq. (i), we get On comparing both sides, we get
x+ y x− y
5x − 4 y = − 1 x+ y =
3
...(i)
⇒ 44 A + 30 B = 10 ...(i) 8x − 4 y = 20 2
and 55A + 40B = 13 ...(ii) − + − 3
and x − y = ...(ii)
On multiplying Eq. (i) by 4 and Eq. (ii) −3 x = − 21 2
by 3 and subtracting, we get ∴ x=7 On adding Eqs. (i) and (ii), we get 2x = 3
176 A + 120B = 40 Put the value of x in Eq. (i), 3
⇒ x=
⇒ 35 − 4 y = − 1 ⇒ y = 9 2
165 A + 120B = 39 3
− − − 7 Hence, the value of x is .
Hence, the fraction is .
11A =1 9 2
⇒ A = 1 / 11 ⇒ x + y = 11 ...(iii) 26. (a) Given equations are, 29. (c) Let A has x mangoes and B has y
On putting the value of A in Eq. (i), we 3x + 4 y = 6 ...(i) mangoes.
get and 6x + 8 y = k ...(ii) Case I x + 30 = 2 ( y − 30)
4 + 30 B = 10 ⇒ 30B = 6 Here, a = 3, b = 4, c = b ⇒ x + 30 = 2 y − 60
1 1 1
1 a = 6, b = 8, c = k
∴ B = , x− y =5 ...(iv) 2 2 2 x − 2 y = − 90 …(i)
5 Since the system of equations represents Case II ( y + 10) = 3 ( x − 10)
On adding Eqs. (iii) and (iv), we get concident line
y = 3x − 30 − 10
2x = 16 a b c 3 4 6
∴ 1 = 1 = 1, i.e. = = 3x − y = 40 …(ii)
∴ x=8 a
2
b
2
c
2
6 8 k
On putting x = 8 in Eq. (iii), we get 48 On solving Eqs. (i) and (ii), we get
⇒ 4k = 6 × 8 ⇒ k = = 12
8 + y = 11 ⇒ y = 3 4 x = 34, y = 62
Hence, x = 8 and y = 3. Hence, the value of k is 12. So, A has 34 mangoes.
MATHEMATICS Linear Equations 157

30. (c) Let number of students in room A be 35. (d) Given, 2a + 3b = 17 41. (d) Since, given system of equations
x and in room B be y. and 2 a + 2
− 3b +1
=5 x + 2 y − 3 = 0 and 5x + ky + 7 = 0 has
So, by given condition no solution.
⇒ 2a × 22 − 3b × 31 = 5
x − 10 = y + 10 ⇒ x − y = 20 …(i) a b c
⇒ 4 ⋅ 2a − 3 ⋅ 3b = 5 Then, 1 = 1 ≠ 1

and x + 20 = 2 ( y − 20) Let 2a = x and 3b = y


a
2
b c
2 2
x − 2 y = − 60 …(ii) x + y = 17 1 2 −3
Then, …(i) ∴ = ≠ ⇒ k = 10
On subtracting Eq. (ii) from Eq. (i), we 5 k 7
4x − 3 y = 5 …(ii)
get
On multiplying Eq. (i) by 3 and adding 42. (a) Given, x − y = 0.9 …(i)
( x − y ) − ( x − 2 y ) = 20 + 60 ⇒ y = 80 to Eq. (ii), we get −1
and 11( x + y ) =2 …(ii)
Hence, number of students in room 3x + 3 y = 51
B is 80. ⇒ 2x + 2 y = 11
4x − 3 y = 5
31. (d) Let the number of passengers in the On multiplying Eq. (i) by 2 and adding
7x = 56 Eqs. (i) and (ii), we get
starting be x.
∴ x =8
Number of passengers after first halt On putting the value of x in Eq. (i), we 4x = 128
. ⇒ x = 3.2
=  x −  + 120 = x + 120
x 2 get From Eq. (i) y = 3.2 − 0.9 = 2.3
 3  3 8 + y = 17 ⇒ y = 9
43. (d) Let Pooja’s initial salary be ` x and
and number of passengers after second Now, 2a = x ⇒ 2a = 8 = ( 2)3
halt fixed increment every year be ` y.
∴ a = 3 and 3b = y = 9 ⇒ 3b = 32
=  x + 120 + 100
1 2 By given condition, x + 3 y = 4200 …(i)
∴ b=2
2  3 
Hence, a = 3 and b = 2 and x + 8 y = 6800…(ii)
But number of passengers after second
36. (b) Let smaller number be x. On subtracting Eq. (ii) from Eq. (i), we
halt = 240
and larger number = 80 − x get
1 2
∴ x + 120 + 100 = 240 By given condition, 80 − x = 4x + 5
( x + 3 y ) − ( x + 8 y ) = 4200 − 6800
2  3 
⇒ −5 y = −2600 ⇒ y = `520
2 2 ⇒ 5x = 75 ⇒ x = 15
⇒ x + 120 = 280 ⇒ x = 160 On putting y = 520 in Eq. (i), we get
3 3 Hence, the smaller number is 15.
x = ` 2640
⇒ x = 240 37. (c) Given equations are,
2 3 9 44. (b) Let the fare of ticket from station P
32. (c) Given system of equations are + = ⇒ 2 y + 3x = 9 …(i) to station Q is ` x and that from station
x y xy
x + 2 y = 3 and 3x + 6 y = 9 P to station R is ` y.
4 9 21
a b c
1 = 1 = 1 = 1.
and + = ⇒ 4 y + 9x = 21 …(ii) By given condition, x + y = 42
Here, x y xy
a b c 3 and 5x + 10 y = 350
2 2 2 On solving Eqs. (i) and (ii), we get
Hence, given system of the equation has On solving Eqs. (i) and (ii), we get
x = 1 and y = 3
infinitely many solutions. x = 14 and y = 28
∴ x + y = 1+ 3 = 4
33. (b) Let x be the ten’s digit and y be the Hence, fare from station P to station Q
38. (b) Since, the equation kx − y = 2 and
unit's digit of two-digit number. is ` 14.
6x − 2 y = 3 have a unique solution
By given condition, x + y = 8 k 1 45. (a) Let x represent school bags, y
∴ ≠ ⇒ k ≠3
and ( 10x + y ) − ( 10 y + x) = 18 …(i) 6 2 represent sweaters, z represent books.
System of equations:
⇒ 9x − 9 y = 18 ⇒ x − y = 2 …(ii) 39. (b) Three lines a x + b y + c = 0
1 1 1 x + y + z = 200 …(i)
On adding Eqs. (i) and (ii), we get a x+b y+c =0
2 2 2 z=x+ y …(ii)
2x = 10 ⇒ x = 5 and a x+b y+c =0
3 3 3 20x + 25 y + 5z = 2800 …(iii)
On putting x = 5 in Eq. (i), we get which are non-parallel and non-collinear
putting z = x + y in Eq. (i)
⇒ y = 8 − 5 = 3 ⇒ x = 5 and y = 3 they have only one solution if they meet
in a common point in this case these ( x + y ) + x + y = 200 , 2x + 2 y = 200
∴ Required difference of digits,
lines are called concurrent lines. x + y = 100 , x = 100 − y
x− y =5−3= 2
40. (c) Let the unit’s digit be x and ten’s putting x = 100 − y in Eq. (iii)
34. (a) Given, ( x, y ) = ( 4, 1) and digit’s be y. Then the two-digit number 20 ( 100 − y ) + 25 y + 5
mx + y = 2x + ny = 5 = 10 y + x [( 100 − y ) + y ] = 2800
On putting x = 4 and y = 1, we get By given condition, x + y = 10 …(i) 2000 − 20 y + 25 y + 500 = 2800
∴ m( 4) + 1 = 2 × 4 + n = 5 and 10 y + x − 18 = 10x + y
5 y = 300 ⇒ y = 60
I II III ⇒ 9x − 9 y = − 18 ⇒ x − y = − 2
∴ 4m + 1 = 5 and 8 + n = 5 x = 100 − 60 ⇒ x = 40
On solving Eqs. (i) and (ii), we get
⇒ n = − 3 and m = 1 z = 40 + 60 ⇒ z = 100
x = 4 and y = 6
∴ m + n = 1−3 = −2 ∴ Required product = xy = 4 × 6 = 24 40 school bags, 60 sweaters, 100 books
158 CDS Pathfinder

46. (b) Let the share of Rajesh be R, Sonal 52. (d) Given equations are, ⇒ 9x + 6 y = 5700 …(ii)
be S and Chetan be C, respectively. 3x + y = 4 …(i) On subtracting Eq. (ii) from Eq. (i), we
According to question, 2R = 3S = 5C 6x + 2 y = 8 …(ii) get
also, 6 R + 6S = 150 Here, a = 3 , b = 1 and c = 4 ( 10x + 6 y ) − ( 9x + 6 y ) = 6200 − 5700
1 1 1
3 a = 6 , b = 2 and c = 8
⇒ × 6 S + 6 S = 150 and
2 2 2 ⇒ x = 500
2 a b c
1 = 1 = 1 = 1 ∴ x = `500
⇒ 9S + 6S = 150 ⇒ S = 10 Q
a b c 2
3 3 2 2 2 From Eq. (i), 5000 + 6 y = 6200
∴ C = S = × 10 = ` 6.
5 5 So, the system of equations has ⇒ 6 y = 1200 ⇒ y = ` 200
infinitely many solutions.
47. (a) R + C + S = 155 ∴ Cost of 4 chairs and 5 tables
2 2 53. (c) Let the two numbers be x and y,
R + R + R = 155 = 4x + 5 y = 4 × 500 + 5 × 200
5 3 respectively.
Given, sum of two numbers = 7 = 2000 + 1000 = `3000
15R + 6R + 10R
= 155 ⇒ R = ` 75 ⇒ x+ y =7 …(i) 57. (b) Let bus starts with x number of
15
48. (c) The graph of ax + by = c, and sum of their squares = 25 passengers.
dx + ey = f will be coincident, if the ⇒ x2 + y 2 = 25 …(ii) After 1st stoppage, number of passengers
system has infinite number of solutions. Now, we have x 5x − x + 200
= x − + 40 =
So, statement II is false. ( x + y ) 2 = ( x 2 + y 2 ) + 2 xy 5 5
4x + 200
Thus, statements I and III are correct. ⇒ ( x + y ) 2 − ( x 2 + y 2 ) = 2 xy =
5
49. (c) Let the two numbers be x and y, 1
⇒ xy = [( x + y )2 − ( x2 + y 2 )] After 2nd stoppage, number of passengers
respectively. According to the question, 2 4x + 200 4x + 200
x + y = 20 …(i) 1 − + 30
= [( 7)2 − ( 25)] 5 5×2
and xy = 75 …(ii) 2 According to question,
1 1 y+x 20 4 4x + 200 4x + 200
⇒ + = = = [from Eqs. (i) and (ii)] ⇒ − + 30 = 70
x y xy 75 15 5 10
1 1
= ( 49 − 25) = × 24 = 12 4x + 200  1
[from Eqs. (i) and (ii)] 2 2  1 −  + 30 = 70
50. (b) Given equations are, Hence, the product of the two numbers 5  2
4x + 200 1
3x+ y = 81 ⇒ 3x + y = 34 is 12. ⇒ × = 40
5 2
⇒ x+ y =4 …(i) Solutions (Q. Nos. 54-55) Let the digit in
⇒ 4x + 200 = 400 ⇒ 4x = 200
and 81x − y = 3 ⇒ (34 )x − y = 31 unit’s place by y and that in ten’s place
200
1 be x . ∴ x= = 50
⇒ x− y = …(ii) Then, the two-digit number is given by 4
4
10x + y. Hence, the number of passengers is 50.
On adding Eqs. (i) and (ii), we get
17 17
Number obtain by reversing the order of 58. (d) We have,x + y − 7 = 0
⇒ 2x = ⇒ x= digit = 10 y + x ⇒ x+ y =7 …(i)
4 8
Now, by given condition, and 3x + y − 13 = 0
51. (d) Given equations are, x + y = 10 …(i) ⇒ 3x + y = 13 …(ii)
a b x
− = …(i) and ( x + 10 y ) + 36 = ( y + 10x) On subtracting Eq. (i) from Eq. (ii), we
b a y get
a b ⇒ − 9 y + 9x = 36
and + =x− y …(ii) 3x + y = 13
b a ⇒ x− y =4 ...(ii)
x+ y =7
From Eqs. (i) and (ii), On adding Eqs. (i) and (ii), we get − − −
 a − bx − x 2x = 14, x = 7 2x = 6
 
a b x   ∴ x=3
+ =x− = b a On putting the value of x in Eq. (i), we
b a  − 
a b a

b get On putting the value of x in Eq. (i), we
 
 b a b a 7 + y = 10 ⇒ y = 3 get
3+ y =7 ⇒ y =4
⇒  +   −  = x − − 1
a b a b a b 54. (b) The required number is 73.
 b a  b a b a  So, the number is a multiple of a prime Now, 4x2 + y 2 + 4xy = ( 2x + y )2
 a2 b2   a 2 − b 2 − ab  number. = ( 2 × 3 + 4)2
⇒  2 − 2 = x 
b a   ab  55. (a) Required product of two digits = ( 6 + 4)2
ab  a4 − b4  = 3 × 7 = 21 = 102 = 100
⇒ x= 2 × 
( a − b 2 − ab )  a 2 b 2  x y 3x + 2 y
56. (a) Let the cost of one chair and one 59. (b) Given, + = 4 ⇒ =4
( a4 − b4 ) 1 table be ` x and ` y, respectively. 2 3 6
⇒ x= 2 ⋅ ⇒ 3x + 2 y = 24 …(i)
a − b 2 − ab ab By given condition,
2 3 2 y + 3x
( a − b )( a + b )( a 2 + b 2 ) 10x + 6 y = 6200 …(i) and + = 1⇒ =1
= x y xy
ab( a 2 − b 2 − ab ) and 3x + 2 y = 1900
⇒ 2 y + 3x = xy …(ii)
MATHEMATICS Linear Equations 159

p q q p
On comparing Eqs. (i) and (ii), we get 63. (a) Let the cost of one chair be ` x and 68. (c) Given, + = m and + = h
xy = 24 that of one table be ` y. x y x y
1 1
There are 8 possibilities for x and y, Then, 2x + y = 700 …(i) Let = u and = v
respectively. x y
and x + 2 y = 800 …(ii)
1 × 24 = 24 6 × 4 = 24 Then, pu + qv = m …(i)
On multiplying Eq. (ii) by 2 and
2 × 12 = 24, 8 × 3 = 24 and qu + pv = n …(ii)
subtracting from Eq. (i), we get
3 × 8 = 24, 12 × 2 = 24 2x + y = 700
On solving Eqs. (i) and (ii), we get
4 × 6 = 24, 24 × 1 = 24 mp − nq mq − np
2x + 4 y = 1600 u= 2 and v = 2
On putting all the values of x and y in − − − p − q2 q − p2
given equations we get that only x = 4 − 3 y = − 900 ⇒ y = 300 x 1 / u v −(mq − np )
and y = 6 satisfy the equations. ∴ = = =
On putting y = 300 in Eq. (ii), we get y 1/ v u mp − nq
∴ x + y = 4 + 6 = 10 np − mq
x + 2 × 300 = 800 =
60. (a) Given equations are, x = 800 − 600 = 200 mp − nq
x + y =5 …(i) Since, m chairs and m tables are to be 69. (c) Given equations are,
y + z = 10 …(ii) purchased for ` 30000.
z + x = 15 …(iii) 3x − ky − 20 = 0 …(i)
∴Cost of m table and m chairs = 30000
from Eq. (i), we have y = 5 − x 6x − 10 y + 40 = 0 …(ii)
put the value of x in Eq. (ii) , ∴ 200m + 300m = 30000
since, the given system has no solution
z + 5 − x = 10 30000
⇒ m= ⇒ m = 60 a b c
∴ 1 = 1 ≠ 1 ⇒ 3 = k ⇒k =5
⇒ z−x =5 …(iv) 500
a b c 6 10
Add Eq. (iv) and Eq. (iii) 64. (c) Let the two-digit number be 10x + y, 2 2 2
(z + x ) + (z − x ) = 15 + 5 Condition I 70. (b) Let the ages of three brothers be a, b
⇒ 2z = 20 ⇒ z = 10 According to the question, and c.
from Eq. (iii), we have 10 + x = 15 10x + y = 3( x + y )
⇒ x =5 Then, a + b = 4, b + c = 6
⇒ 10x + y − 3x − 3 y = 0
and hence y = 5 − x = 5 − 5 = 0 and c+ a =8
⇒ 7x − 2 y = 0 …(i)
∴ correct sequence is z > x > y Condition II On solving these three equations, we get
⇒ 10x + y + 45 − 10 y − x = 0 a = 3, b = 1 and c = 5
61. (c) Let present age of Ravi be x yr.
⇒ 9x − 9 y + 45 = 0 ∴ Age difference between eldest and
∴ Present age of Ravi’s father = 4x yr
⇒ x − y = − 5 …(ii) youngest = 5 − 1 = 4 yr
Now, 5 yr ago, age of Ravi’s father
On solving Eqs. (i) and (ii), we get 71. (a) Let annual incomes of two persons
= ( 4x − 5) yr
x = 2 and y = 7 be 9x and 7x and expenses be 4 y and 3 y,
and age of Ravi = ( x − 5) yr respectively.
∴ Sum of the squares of digits
According to the question, Then, according to the question,
= ( 2)2 + ( 7)2 = 4 + 49 = 53
4x − 5 = 7( x − 5) 9x − 4 y = 2000 …(i)
⇒ 4x − 5 = 7x − 35 ⇒ 3x = 30 65. (a) Let x be the required number.
and 7x − 3 y = 2000 …(ii)
∴ x = 10 ∴ 52x − 25x = 324
324 From Eqs. (i) and (ii), we get
∴ Present age of Ravi = x = 10 yr ⇒ 27x = 324 ⇒ x = = 12
27 9x − 4 y = 7x − 3 y ⇒ y = 2x
Present age of Ravi’s father On putting the value of y in Eq. (i), we
Hence, the required number is 12.
= 4x = 4 × 10 = 40 yr get
66. (b) Let a tin of oil contain x bottles of 9x − 8x = 2000 ⇒ x = ` 2000
62. (d) Let the two positive numbers be x oil.
and y, respectively. According to the ∴ Difference between their annual
According to the question, incomes = 9x − 7x = 2 x = ` 4000
question,
4 6 4 3 4 3 6 4
( x + y ) = 25
. (x − y) − + = ⇒ − = − 72. (c) Let the unit’s place number be y and
5 x x 4 5 4 x x
⇒ x + y = 25
. x − 25
. y 1 2
ten’s place number be x.
x 7 7 ⇒ = ⇒ x = 40 Then, number = 10x + y
⇒ 35
. y = 15
. x ⇒ = or x = y 20 x
y 3 3 Now, after interchanging the digits,
∴ A tin contains 40 bottles of oil.
Now, product of numbers xy = 84 New number = 10 y + x and sum of
67. (c) Let the number be 10x + y .
7 84 × 3 digits = x + y
⇒ y × y = 84 ⇒ y 2 = We have, x + y = 7 …(i)
3 7 According to the question,
and 10 y + x = 10x + y + 27
⇒ y 2 = 12 × 3 10x + y = k( x + y ) …(i)
⇒ 9 y − 9x = 27 ⇒ y − x = 3 …(ii)
7 and 10 y + x = m( x + y ) …(ii)
∴ y =6⇒ x = × 6 = 14 Solving Eqs. (i) and (ii), we get y = 5
3 and x = 2 On adding Eqs. (i) and (ii), we get
∴ Sum of numbers = x + y ∴ Number is 25. 11 ( x + y ) = ( k + m ) ( x + y )
= 14 + 6 = 20 Product of number = 2 × 5 = 10 ⇒ k + m = 11 ⇒ m = 11 − k
18
160 CDS Pathfinder

QUADRATIC EQUATIONS
AND INEQUALITIES
Regularly (9-10) questions have been asked from this chapter. Questions from this section
generally focus on finding roots of quadratic equation and their factorisation.

QUADRATIC EQUATION
A quadratic equation is an equation whose degree is 2, meaning that the highest exponent of variable is 2.
The general form of a quadratic equation is ax 2 + bx + c = 0, where a, b and c are real numbers and a ≠ 0.
e.g. 2x 2 + 3x + 5 = 0, x 2 + x + 1 = 0 and 5x 2 + 6x + 8 = 0 are all quadratic equations.

Roots of a Quadratic Equation


A real number α is said to be a root of the quadratic equation ax 2 + bx + c = 0 ( a ≠ 0 ) , if it satisfies the
equation i.e. aα 2 + bα + c = 0. We can also say that x = α is a solution of the quadratic equation.
e.g. 2 is the root of the quadratic equation, x 2 − 6x + 8 = 0 since ( 2) 2 − 6( 2) + 8 = 0.
Note A quadratic equation has exactly two roots.
If α be the root of the quadratic equation ax 2 + bx + c = 0, then ( x − α ) is the factor of ax 2 + bx + c.

Solutions of Quadratic Equations


Solutions of quadratic equations can be determined by using any of the following two methods
1. By Factorisation
In this method, the middle term of the quadratic equation is splitted and the given quadratic equation
is converted into a product of two linear factors. Then, the roots of the equation are obtained by
equating each factor equal to zero.
Let the quadratic equation be ax 2 + bx + c = 0 and its linear factors are ( px + q) and (rx + s ) , then
ax 2 + bx + c = ( px + q)(rx + s )
−q −s
Now, ax 2 + bx + c = 0 ⇒ ( px + q)(rx + s ) = 0 ⇒ px + q = 0 or rx + s = 0 ⇒ x = or x=
p r
−q −s
Thus, and are the roots of the equation, ax 2 + bx + c = 0.
p r
MATHEMATICS Quadratic Equations and Inequalities 161

EXAMPLE 1. Solve the equation x 2 − 7 x + 12 = 0, and Nature of Roots of Quadratic Equation


find the value of x. Let D = b 2 − 4ac be the discriminant of the quadratic
a. −3, − 4 b. −3, 4 c. 3, − 4 d. 3, 4
equation, ax 2 + bx + c = 0, where a ≠ 0. Then, the
Sol. d. Given, x2 − 7x + 12 = 0 following cases arise
x2 − 3x − 4x + 12 = 0 [splitting the middle term]
(i) If D > 0, then the two roots are real and distinct.
⇒ x ( x − 3) − 4 ( x − 3) = 0
(ii) If D = 0, then the two roots are real and equal and are
⇒ ( x − 3) ( x − 4) = 0 −b
⇒ ( x − 3) = 0 or ( x − 4) = 0 ⇒ x = 3 or x = 4 given by α = β = .
2a
So, x = 3, 4 are roots of the given equation. (iii) If D < 0, then there are no real roots, i.e. given
EXAMPLE 2. Solve the equation equation has imaginary roots.
3a 2 x 2 + 8abx + 4b 2 = 0, a ≠ 0, and find the value of x.
2b 2b − 2b 2b − 2b − 2b 2b −2b IMPORTANT POINTS
a. , b. , c. , d. ,
3a a 3a a 3a a 3a a 1. If a , b , c ∈Q and D is a perfect square, then equation
Sol. c. The equation is 3a2x2 + 8abx + 4b2 = 0 has rational roots.
2. If one roots of quadration equation is p + q , then
⇒ 3a2x2 + 2abx + 6abx + 4b2 = 0
other root will be p − q .
⇒ ax ( 3ax + 2b) + 2b ( 3ax + 2b) = 0
∴ ( 3ax + 2b) ( ax + 2b) = 0 EXAMPLE 4. If the equation x 2 + 2(1 + k )x + k 2 = 0
⇒ 3ax + 2b = 0 or ax + 2b = 0
has equal roots, then what is the value of k?
− 2b − 2b
∴ x= or a.
1
b. −
1
3a a 2 2
2. By Using the Quadratic Formula c. 1 d. − 1
Let the quadratic equation be ax 2 + bx + c = 0, where Sol. b. Given equation, x2 + 2(1 + k ) x + k 2 = 0.
a ≠ 0. Then, solution of this equation can be find using If it has equal roots, then D = 0
the formula.
⇒ {2 (1 + k )}2 − 4k 2 = 0
− b ± b 2 − 4ac
x= ⇒ 4(1 + k 2 + 2k ) − 4k 2 = 0
2a
• If α and β be considered as roots of the quadratic
⇒ 4 + 4k 2 + 8k − 4k 2 = 0
4
equation, then ⇒ 4 + 8k = 0 ⇒ k = −
8
− b + b 2 − 4ac − b − b 2 − 4ac 1
α= and β = ∴ k=−
2a 2a 2
• The quantity b 2 − 4ac is called the discriminant of the 1
Hence, the value of k is − .
quadratic equation ax 2 + bx + c = 0 and is denoted by D. 2
So, D = b 2 − 4ac.
Roots Under Particular Conditions
EXAMPLE 3. Solve the equation x − 9x + 18 = 0 and 2 Consider the quadratic equation ax 2 + bx + c = 0.
find the value of x. (i) Both roots are positive, if a and b are opposite in
a. 3, 6 b. −3, − 6 c. −3, 6 d. 3,− 6 −b c
sign and a and c are same in sign, i.e. > 0 and > 0.
a a
Sol. a. We have, x2 − 9x + 18 = 0
(ii) Both roots are negative, if a, b and c are of same sign.
Here, a = 1, b = − 9 and c = 18
b c
−b± b2 − 4ac − ( − 9) ± ( − 9) 2 − 4 (1) (18) i.e. > 0 and > 0.
∴ x= = a a
2a 2 (1) (iii) Roots are of opposite signs, if a and c are of opposite
9 ± 81 − 72 9± 9 9± 3 −a
= = = sign, i.e. > 0.
2 22 c
9+ 3 9−3
∴ x= or x = ⇒ x = 6 or 3 (iv) Roots are equal but opposite in signs, if b = 0.
2 2
Hence, the values of x are 6 and 3. (v) Roots are reciprocal to each other, if a = c.
162 CDS Pathfinder

EXAMPLE 5. What is the least integral value of k for ( a2 − b 2 ) 2 a2 − b 2


⇒ α −β = = ...(ii)
which the equation x 2 − 2(k −1) x + (2k + 1) = 0 has both ( a2 b 2 ) 2 a2 b 2
the roots positive? On solving Eqs. (i) and (ii), we get
1 1 1
a. 1 b. − c. 4 d. 0 α = 2 and β = 2
2 b a
Sol. a. Both the roots of the equation ax2 + bx + c = 0 are
x2 x 1
b c
positive, if − > 0 and > 0 EXAMPLE 7. If one root of the equation + + =0
a a a b c
is reciprocal to the other, then which one of the
Given equation is x2 − 2 ( k − 1) x + ( 2k + 1) = 0.
following is correct?
b 2( k − 1) a. a = b b. b = c c. ac = 1 d. a = c
Now, − = ⇒ 2( k − 1) > 0, if k > 1
a 1 2
c 2k + 1 x x 1 ...(i)
and = ⇒ ( 2k + 1) > 0, if k > −
1 Sol. d. Given, + + =0
a 1 2 a b c
∴ k >1 Now, let one root be α, then the other root is 1/ α.
We know that,
Hence, the least value k as per given in options is 1. Constant term 1 /c
Product of roots = =
Coefficient of x2 1 / a
Relation between Roots and ⇒
1 a
α. = ⇒ c=a
Coefficients α c
which is the required relation.
1. Quadratic Equation
Let α, β be the roots of the equation ax 2 + bx + c = 0 Symmetric Functions of the Roots
( a ≠ 0 ) , then Let α and β be the roots of a quadratic equation. An
b − Coeff icient of x expression in α and β which remains same when α and β
Sum of the roots = α + β = − = are interchanged is known as a symmetric function in α
a Coeff icient of x 2
and β. To evaluate the value of symmetric function of the
c Constant term roots, express the given function in terms of (α + β ) and
Product of the roots = αβ = = αβ.
a Coeff icient of x 2
You can use the following results.
2. Cubic Equation • (α 2 + β 2 ) = [(α + β ) 2 − 2αβ ]
If α, β, γ are the roots of the cubic equation • (α 3 + β 3 ) = [(α + β) 3 − 3αβ (α + β )]
ax 3 + bx 2 + cx + d = 0, a ≠ 0, then
• (α − β ) 2 = [(α + β ) 2 − 4αβ ]
Sum of roots = α + β + γ = − b / a
• (α 3 − β 3 ) = [(α − β ) 3 + 3αβ (α − β )]
Product of roots two at a time = αβ + βγ + γα = c / a
d EXAMPLE 8. If α and β are the roots of the equation
Product of three roots = αβγ = −
a x 2 − 6 x + 6 = 0, what is α 3 + β 3 + α 2 + β 2 + α + β equal
EXAMPLE 6. What are the roots of the quadratic to?
equation a 2b 2 x 2 − (a 2 + b 2 ) x + 1 = 0? a. 150 b. 138 c. 128 d. 124
− Coefficient of x
1 1 1 1 1 1 1 1 Sol. b. Here, α + β = =6
a. 2
, b. − ,− c. ,− d. − , Coefficient of x2
a b2 a 2
b2 a 2
b2 a 2
b2
Constant term
Sol. a. Let roots of the equation a b x − ( a + b ) x + 1 = 0
2 2 2 2 2 and αβ= =6
Coefficient of x2
be α and β.
∴ (α + β) 2 = 62 ⇒ α 2 + β 2 + 2αβ = 36
a2 + b2
∴ Sum of roots = α + β = ...(i) ⇒ α 2 + β 2 = 36 − 2 (6 ) = 24
a2b2
1 ∴ (α 3 + β 3) + (α 2 + β 2) + (α + β)
and product of roots = αβ = 2 2
a b = (α + β) (α 2 + β 2 − αβ) + (α 2 + β 2) + (α + β)
2 2
 a2 + b  4 = 6 ( 24 − 6) + ( 24) + (6)
Now, α − β = (α + β) 2 − 4 αβ =  2 2  − 2 2
 a b  a b = 6 (18) + 30 = 108 + 30 = 138
MATHEMATICS Quadratic Equations and Inequalities 163

3
Formation of a Quadratic Equation EXAMPLE 11. Solve 2x − = 5. Then, the value of x is
x
If α, β are the roots of a quadratic equation, then the
a. 1/3, −3 b. 1/2, −3 c. −1/2, 3 d. 1/2, 4
equation is ( x − α )( x − β ) = 0 ⇒ x 2 − (α + β ) x + αβ = 0
3
Sol. c. Here, 2x − =5 ...(i)
i.e. x 2 − (sum of the roots) x + product of the roots = 0 x
On multiplying Eq. (i) by x ⇒ 2x2 − 3 = 5x
EXAMPLE 9. The quadratic equation whose roots are ⇒ 2x − 5x − 3 = 0 ⇒ 2x − 6x + x − 3 = 0
2 2
3 and −1, is
⇒ ( 2x + 1) ( x − 3) = 0 ⇒ x = − 1/ 2 or x = 3
a. x 2 − 4 x + 3 = 0 b. x 2 − 2x − 3 = 0 −1
c. x 2 + 2x − 3 = 0 d. x 2 + 4 x + 3 = 0 Hence, the solution is x = , 3.
2
Sol. b. Given that, the roots of the quadratic equation are
3 and – 1.
Rule 3 x + b + x = c or a − x 2 = bx + c
Let α = 3 and β = −1
• Square both sides to get a quadratic equation without
∴ Sum of roots = α + β = 3 − 1 = 2
the radical.
and product of roots = αβ = ( 3)( −1) = −3
• Solve the equation by factorisation or by quadratic
So, required quadratic equation is formula.
x 2− (α + β) x + αβ = 0
⇒ x 2− ( 2 ) x + ( −3 ) = 0
EXAMPLE 12. Solve 2x + 9 + x = 13, then the value
of x is
⇒ x 2− 2x − 3 = 0 a. 4, 10 b. 8, 20 c. 3, 15 d. −4 , − 10
Equations Reducible to Quadratic Equations Sol. b. Given, 2x + 9 + x = 13 ⇒ 2x + 9 = 13 − x
The equations which at the outset are not quadratic On squaring both sides, we get ( 2x + 9) 2 = (13 − x ) 2
equations, but can be reduced to quadratic equations by
suitable substitutions or simplifications are called ⇒ 2x + 9 = 169 + x2 − 26x
equations reducible to quadratic equations. ⇒ x 2 − 28x + 160 = 0 ⇒ x 2 − 20x − 8x + 160 = 0
⇒ ( x − 8) ( x − 20) = 0 ⇒ x = 8 or x = 20
Rule 1 ax 2n + bx n + c = 0, where n ≥ 2 Hence, the solution is x = 8, 20.
Use substitution x n = y
Rule 4 ax + b ± cx + d = e
⇒ x 2n = y 2 and equation reduces to ay 2 + by + c = 0.
or ax + b ± cx + d ± ex + f = 0
EXAMPLE 10. Solve the quadratic equation To solve such type of equations, follow the steps given
x 4 − 26 x 2 + 25 = 0, and find the value of x. below.
a. ± 1, ± 25 b. ± 1, ± 5 • Square both sides once, so that only one term
c. 1, 5 d. 1, 25 containing radical is obtained.
• Keep the term containing radical on one side and all
Sol. b. Put x = z ⇒ x = z
2 4 2
other terms on the other side.
x4 − 26x2 + 25 = 0 can be written as • Square again to get, the quadratic equation and solve it.

z 2 − 26z + 25 = 0 ⇒ z 2 − 25z − z + 25 = 0 EXAMPLE 13. Solve the following equation


⇒ z ( z − 25) − 1( z − 25) = 0 4 − x + x + 9 = 5. Then, the value of x is
⇒ ( z − 1) ( z − 25) = 0 ⇒ z = 1or z = 25 a. 1, 5 b. −1, 3 c. 0 , − 5 d. 1, 4
when z = 1⇒ x 2 = 1 ⇒ x = ± 1
Sol. c. Given, 4 − x + x + 9 = 5
when z = 25 ⇒ x2 = 25 ⇒ x = ± 5
On squaring both sides, we get ( 4 − x + x + 9) 2 = 25
Hence, the solutions are ± 1 and ± 5.
⇒ ( 4 − x) + ( x + 9) + 2 ( 4 − x x + 9) = 25
Q
Rule 2 Px + =R ⇒ 2 ( 4 − x) ( x + 9) = 12 ⇒ ( 4 − x) ( x + 9) = 6
x Again, squaring both sides, we get
Reduce the given equation to a quadratic equation by ⇒ ( 4 − x ) ( x + 9 ) = 36 ⇒ − x 2 + 36 − 5x = 36
multiplying both sides by ‘x’. So, Px 2 + Q = Rx ⇒ x2 + 5x = 0 ⇒ x ( x + 5) = 0 ⇒ x = 0 or x = − 5
or Px 2 − Rx + Q = 0 ∴ x = 0, − 5 is the required solution.
164 CDS Pathfinder

EXAMPLE 14. Solve 11y − 6 + y − 1 − 4y + 5 = 0. Now,


Then, the value of y is x2 + 5x = − 8 x2 + 5x = − 2
a. {5, 6/5} b. {1, 2} ⇒ x2 + 5x + 8 = 0 ⇒ x2 + 5x + 2 = 0
c. {3, 5/2} d. None of these
− 5 ± 25 − 32 − 5 ± 25 − 8
Sol. a. Given, 11y − 6 + y − 1 − 4y + 5 = 0 ⇒ x= ⇒ x=
2 2
11y − 6 + y − 1 = 4y + 5 − 5± −7 − 5 ± 17
∴ x= ∴ x=
On squaring both sides, we get 2 2
Hence, the number of real roots of the given equation is 2.
[ 11y − 6 + y − 1]2 = [ 4y + 5]2
 1  1
⇒(11y − 6) + ( y − 1) + 2 (11y − 6) ( y − 1) = ( 4y + 5) Rule 6 (i) a  x 2 +  + b  x +  + c = 0
 x 2  x
⇒ 12y − 7 + 2 11y 2 − 17y + 6 = 4y + 5  1  1
(ii) a  x 2 +  + b  x −  + c = 0
⇒ 2 11y 2 − 17y + 6 = 12 − 8y  x 2  x
⇒ 11y 2 − 17y + 6 = 6 − 4y To solve an equation of the form (i) .
 1
Again, squaring both sides, we get put  x +  = y and to solve an equation of
 x
11y 2 − 17y + 6 = (6 − 4y) 2
 1
⇒ 11y 2 − 17y + 6 = 36 + 16y 2 − 48y
the form (ii), put  x −  = y
 x
⇒ 5y 2 − 31y + 30 = 0
 1 
EXAMPLE 16. Solve 4  x 2 + 2  − 4  x +
1
Here, a = 5, b = − 31and c = 30  − 7 = 0;
 x   x
31 ± ( − 31) 2 − 4 ( 5) ( 30)  − b ± b2 − 4ac  x ≠ 0 and find the real roots of x.
⇒ y= Q y = 
2× 5  2a  a. {−2, −1/2} b. {0, 2}
31 ± 361 31 ± 19 c. {2, 1/2} d. None of these
⇒ y= ⇒y =
Sol. c. 4 x 2 +
1
 − 4  x +  − 7 = 0
10 10 1
…(i)
31 + 19 31 − 19  x 2  x
⇒ y= and y = 1
10 10 Put x+ =y
x
y = 5,  is the required solution.
6
∴ On squaring both sides, we get
 5 2
 x + 1 = y 2 ⇒ x2 + 1 + 2 = y 2 ⇒ x 2 + 1 = y 2 − 2
Rule 5 ( x + a) ( x + b) ( x + c ) ( x + d ) + k = 0, where  x x2 x2
a + b = c + d and k may or may not be zero. ⇒ 4 ( y 2 − 2) − 4y − 7 = 0 [from Eq. (i)]
To solve such type of equations, put x 2 + ( a + b)x = T . ⇒ 4y − 8 − 4y − 7 = 0 ⇒ 4y − 4y − 15 = 0
2 2

EXAMPLE 15. Solve the equation ⇒ 4y − 10y + 6y − 15 = 0 ⇒ 2y( 2y − 5) + 3( 2y − 5) = 0


2

(x + 1) (x + 2) (x + 3) (x + 4) − 8 = 0, and find the number ⇒ ( 2y − 5) ( 2y + 3) = 0 ⇒ y =


5
or −
3
of real roots of this equation. 2 2
5 3
a. 1 b. 2 c. 4 d. No real roots Now, For y= For y = −
2 2
Sol. b. ( x + 1) ( x + 2) ( x + 3) ( x + 4 ) − 8 = 0 ⇒ x+ =
1 5
⇒ x+ =−
1 3
x 2 x 2
Here, 1 + 4 = 2 + 3,
x2 + 1 5 x2 + 1 3
so, [( x + 1) ( x + 4 )][( x + 2) ( x + 3)] − 8 = 0 ⇒ = ⇒ =−
x 2 x 2
[ x2 + 5x + 4][ x2 + 5x + 6] − 8 = 0
⇒ 2x 2 + 2 = 5x ⇒ 2x 2 + 2 = − 3x
Put x2 + 5x = t, so the equation becomes
⇒ 2x 2 − 5x + 2 = 0 ⇒ 2x2 + 3x + 2 = 0
( t + 4) ( t + 6) − 8 = 0
⇒ ( x − 2) ( 2x − 1) = 0 Here, D = 9 − 16 = −7 < 0
⇒ t 2 + 10 t + 24 − 8 = 0 ⇒ t 2 + 10 t + 16 = 0 1
∴ x = 2 or x = So, there are no real roots.
⇒ t 2 + 8 t + 2t + 16 = 0 2
⇒ t (t + 8) + 2 (t + 8) = 0 ⇒ (t + 8) (t + 2) = 0 [QD = b2 − 4ac ]
⇒ t = − 8 or − 2 1
Hence, the values of x are 2 and .
2
MATHEMATICS Quadratic Equations and Inequalities 165

Rule 7 ax 4 + bx 3 + cx 2 + bx + a = 0 (iii) Sometimes it may happen that, out of the roots of


the quadratic equation only one satisfies the given
• To solve such type of equations, we divide the equation condition in the problem. So, the root which does
by x 2 , to obtain a( x 2 + 1 / x 2 ) + b ( x + 1 / x) + c = 0, and, not satisfy the condition of the problem will be
1 rejected.
then put x + = y
x
EXAMPLE 18. A positive number, when increased by
EXAMPLE 17. If x 4 − 3x 3 − 2x 2 + 3x + 1 = 0, x ≠ 0, 10 equals 200 times its reciprocal. What is number?
then the roots of the equation is a. 100 b. 10 c. 20 d. 200
 −3 ± 13   3 ± 13 
a. ± 2,  b. ± 1,  Sol. b. Let the positive number be x.
 2   2 
Then, according to the question,
 3 ± 11  200
c. ± 3,  d. None of these x + 10 = ⇒ x 2+ 10x = 200
 2  x
⇒ x2 + 10x − 200 = 0 ⇒ x2 + 20x − 10x − 200 = 0
Sol. b. x4 − 3x3 − 2x2 + 3x + 1 = 0
⇒ x ( x + 20) − 10( x + 20) = 0 ⇒ ( x − 10) ( x + 20) = 0
On dividing throughout by x 2 and rearranging the terms, ∴ x = 10, − 20
we get But x ≠ −20, since x is a positive number.
 x2 + 1  − 3  x − 1 − 2 = 0 …(i) So, the required number is 10.
 x2   x
1
Put x− =y EXAMPLE 19. Out of group of swans, 7/2 times the
x square root of the number are swimming in the pool.
On squaring both sides, we get
While the two remaining are playing outside the pool.
1 1
x2 + 2 − 2 = y 2 ⇒ x2 + 2 = y 2 + 2 What is the total number of swans?
x x
⇒ ( y 2 + 2) − 3y − 2 = 0 [from Eq. (i)]
a. 4 b. 8 c. 12 d. 16
⇒ y 2 − 3y = 0 ⇒ y ( y − 3) = 0 Sol. d. Let total number of swans be x.
7
Number of swans swimming in the pool = x
∴ y = 0 or 3 Remaining swans = 2 2
For y = 0 For y = 3 7 7
1 1 By given condition, x+ 2= x ⇒ x = x−2
⇒ x− =0 ⇒ x− =3 2 2
x x 49
x −1
2
x −1
2 On squaring both sides, we get x = x2 + 4 − 4 x
⇒ =0 ⇒ =3 4
x x ⇒ 4x2 − 65x + 16 = 0, 4x2 − 64x − x + 16 = 0
⇒ x2 = 1 ⇒ x2 − 3x − 1 = 0 ⇒ 4x ( x − 16) − 1( x − 16) = 0 ⇒ ( x − 16)( 4x − 1) = 0
3± 9 + 4 Q x ≠ 1 
∴ x=±1 ⇒ x= ∴ x = 16
2  4 
3 ± 13 Thus, total number of swans = 16
∴ x=
2

So, x = ± 1,
3 ± 13 
 is the required solution. INEQUALITIES
 2 
Two real numbers or two algebraic expressions related
Note An equation of degree n has n roots.
by the symbols >, <, ≤ or ≥ form inequality. Here, the
symbols < (less than), > (greater than), ≤ (less than or
Word Problems Involving equal) and ≥ (greater than or equal) are known as
inequality signs.
Quadratic Equation e.g. 5 < 7, x ≤ 2, x + y ≥ 11 are all inequalities.
In this section, we will discuss some problems based on
practical applications of quadratic equation. Linear Inequalities
In such type of problems, A linear inequality is an inequality which involves a
(i) We formulate a quadratic equation in variable x with linear function i.e. each variable occurs in first degree
the help of the given conditions. only and there is no term involving the product of the
(ii) We then solve the quadratic equation to find the variables.
answer to the given problem. e.g. ax + b ≤ 0, ax + by + c > 0, ax ≤ 4.
166 CDS Pathfinder

Linear inequality in one variable Clearly, here (0, 0) does not Y


satisfies the inequality, so we (0, 2)
A linear inequality which has only one variable is called shade the plane which does not
0)
linear inequality in one variable. contain (0, 0). (3,
 − b Also, since x ≥ 0 and y ≥ 0, so the X´ X
e.g. ax + b < 0, where a ≠   , 4x + 7 ≥ 0 solution set must be in Ist
 a
quadrant only.
Linear inequality in two variables Hence, the shaded region Y´
represents the solution set of the given system of
A linear inequality which has two variables, is called linear inequalities.
inequality in two variables.
e.g. 3x + 11y ≤ 0, 4t + 3y > 0 Note (i) If the inequality is strict ( > or <), graph a dashed line. If the
inequality is not strict ( ≥ or ≤), graph a solid line.
General Rules to Solve (ii) In order to solve a system of two or more linear
inequalities with the same variables, graph each inequality
Linear Inequalities on the same set of axes. The intersection of the region of
all the inequalities is the required solution set.
• Equal numbers may be added or subtracted from both
sides of an inequality without changing the sign of in EXAMPLE 21. Find the solution set of −2x + 3y > 6
equality. and 4x − 6 y > 12 .
e.g. If a > b, then for any number c,
a. x < −1, y > −3 b. x < 1, y > 3
a + c > b + c or a − c > b − c.
c. x ≥ 3, y ≥ 2 d. No solution
• If both sides of an inequality are multiplied or divided by
the same positive number, then the sign of inequality Sol. d. Consider, −2x + 3y = 6
remains the same. e.g. Y
0 −3
a b x
If a > b and c > 0, then > and ac > bc. y 2 0 (0, 2)
c c
• If both sides of an inequality are multiplied or divided Again, 4x − 6y = 12 X´ X
(– 3, 0) O
by the same negative number, then the sign of x 0 3 (3, 0)
inequality is reversed. y −2 0 (0, – 2)
a b
e.g. If a > b and c < 0, then < and ac < bc. Now, graph each equation. Since,
c c Y´
there is no intersection of these two
shaded regions, therefore there is no
Graphical Solution of Linear solution.
Inequalities in Two Variables
Let the inequality be ax + by + c ≥ 0. Quadratic Inequalities
An equation of the form
Step I Consider it as ax + by + c = 0 by changing the
ax 2 + bx + c ≥ 0 or ax 2 + bx + c ≤ 0
inequality sign to equality sign, Draw the graph
of this equation. or ax 2 + bx + c > 0 or ax 2 + bx + c < 0
Step II Choose any point not on the line, [usually (0, 0 )]. where, a ≠ 0 is called a quadratic inequality in one
variable x. It is same as quadratic equations except for
Find whether this point satisfy the inequality or
the inequality sign.
not. If it does, then shade the half-plane
containing that point. Otherwise shade the other Solution of Quadratic Inequalities
half-plane.
Solving the inequality means finding the values of x
EXAMPLE 20. The solution set of inequality that make the inequality true. Solution sets of
2x + 3y ≥ 6, x ≥ 0 and y ≥ 0 is quadratic inequalities are expressed in the form of
a. x = 1, y = 1 b. x = − 1, y = − 1 intervals. Here are the steps:
c. x = 3, y = 1 d. None of these • Replace the inequality symbol with an equal sign.
• Solve the quadratic equation by factorisation or by
Sol. d. Draw the graph of 2x + 3y = 6.
quadratic formula and find the real roots of the
The values of ( x, y) satisfying 2x + 3y = 6 are equation.
x 0 3 • Plot the two real roots on the numbers line. They
y 2 0 divide the line into three sections or three intervals.
MATHEMATICS Quadratic Equations and Inequalities 167

• Pick a number from each interval and test it in the Take x = − 4, Take x = −2 , Take x = 0
original inequality. ⇒ ( −4) 2 + 4( −4) + 3 > 0,
• If the inequality holds true for the choosen point. then ( −2) 2 + 4( −2) + 3 < 0, 02 + 4(0) + 3 > 0
that interval is the solution of the given quadratic + – +
inequality.
–3 –1
Note (i) If the quadratic equation does not have real roots, then the So, the intervals which satisfy the given equation are
quadratic expression is always positive (or always negative) (− ∞ , − 3] and [−1, ∞)
depending on the sign of a meaning that the solution set will
either be empty or the entire real number line. i.e. x ≤ −3 and x ≥ −1
e.g. 15x 2 − 18x + 7 > 0 has no real roots as EXAMPLE 23. The real values of x which satisfy
D = b 2 − 4ac = 64 − 420 < 0 . Since a > 0, so the
expression is always positive and hence the solution set x 2 − 4x + 3 ≥ 0 and x 2 − 3x − 4 ≤ 0 is
will be the entire real number line. a. ( −1, 1) ∪ ( 3, 4) b. [ −1, 1] ∩ [ 3, 4]
(ii) When the inequality has an additional ‘=’ sign ( ≥ or ≤), use c. [ −1, 1] ∪ [ 3, 4] d. ( −1, 1) ∪ [ 3, 4]
closed intervals like [a, b] to indicate that the two end points
are also included in the solution set. If the inequality is strict Sol. c. Given, x − 4x + 3 ≥ 0
2
(>or<), use open intervals like (a, b) as end points are not
included. ⇒ x2 − 4x + 3 = 0 ⇒ ( x − 1)( x − 3) = 0 ⇒ x = 1, 3
+ – +
EXAMPLE 22. The solution of the inequality
1 3
x 2 + 4x + 3 ≥ 0 is ⇒ x ∈ ( −∞ , 1] or x ∈[ 3, ∞). ⇒ x ∈ ( − ∞ 1] ∪ [ 3, ∞).
a. x < − 3 and x > − 1 b. x ≤ − 3 and x ≥ − 1 Again, x2 − 3x − 4 ≤ 0 ⇒ x2 − 3x − 4 = 0
c. x < − 3 and x ≥ − 1 d. x ≤ − 3 or x > − 1 ⇒ ( x + 1)( x − 4) = 0 ⇒ x = −1, 4
+ – +
Sol. b. We have, x2 + 4x + 3 ≥ 0
Change the inequality to equality sign and solve –1 4
x2 + 4x + 3 = 0 ⇒ x2 + 3x + x + 3 = 0 ⇒ x ∈[ −1, 4]
∴ Solution set is
⇒ x ( x + 3) + ( x + 3) = 0 ⇒ ( x + 1)( x + 3) = 0 ⇒ x = −1, − 3.
x = ( − ∞ , 1] ∪ [ 3, ∞) ∩ [ −1, 4] = [ −1, 1] ∪ [ 3, 4]
Place the value of x on the number line to create
intervals.

PRACTICE EXERCISE
1. The quadratic equation has maximum 4+ 7
5. The quadratic equation whose roots are
(a) one root (b) two roots 2
(c) four roots (d) three roots 4− 7
and is
2
2. The values of x in the equation (a) 4x2 + 16x + 9 = 0 (b) 4x2 − 16x − 9 = 0
a b x − ( a + b ) x + 1 = 0, a ≠ 0, b ≠ 0 is
2 2 2 2 2
(c) 4x − 16x + 9 = 0
2
(d) 4x2 + 16x − 9 = 0
(a) 1/a2 (b) 1/b 2 6. If α and β are the roots of the equation
2 2
(c) 1/a , 1/b (d) None of these x 2 − 8x + p = 0 and α 2 + β 2 = 40, then p is equal to
(a) 12 (b) 10 (c) 9 (d) 11
3. The value of ‘a’ for which the equation
ax 2 − 2 5 x + 4 = 0 has equal roots is
7. If α and β are the roots of the equation
x 2 − 5x + 6 = 0, then the value of α 2 − β 2
(a) 5/4 (b) 4/5
(a) 5 (b) − 5 (c) ± 5 (d) ± 4
(c) − 5/ 4 (d) − 5/ 3
8. If α , β are the roots of the equation
4. If one root of 3x 2 = 8x + ( 2k + 1) is seven times the ax 2 + bx + c = 0, then an equation whose roots are
other, then the value of k is 1 / α and 1 / β is
(a) 5/3 (b) − 5/ 3
(a) bx2 + ax + c = 0 (b) ax2 − bx + c = 0
(c) 2/3 (d) − 3/2
(c) cx2 + ax + b = 0 (d) cx2 + bx + a = 0
168 CDS Pathfinder

9. If α , β are the roots of a quadratic equation such 19. Sum of roots is − 1 and sum of their reciprocals is
that α + β = 24 and α − β = 8, then the equation is 1/6, then equation is
(a) x2 − 24x − 128 = 0 (b) x2 + 24x + 128 = 0 (a) x2 − 6x + 1 = 0 (b) x2 − x + 6 = 0
(c) x2 + 24x − 128 = 0 (d) None of these (c) 6x2 + x + 1 = 0 (d) x2 + x − 6 = 0

10. Which one of the following is the equation whose 20. If sum of the roots of the equation
roots are respectively three times the roots of the ax 2 + bx + c = 0 is equal to the sum of their squares,
equation ax 2 + bx + c = 0? then which one of the following is correct?
(a) ax2 + bx + c = 0 (b) ax2 + 3bx + 9c = 0 (a) a2 + b 2 = c 2 (b) a2 + b 2 = a + b
(c) ax − 3bx + 9c = 0
2
(d) ax2 + bx + 3c = 0 (c) 2 ac = ab + b 2 (d) 2c + b = 0
11. How many real values of x satisfy the equation 21. If the roots of x 2 − lx + m = 0 differ by 1, then
x 2/ 3 + x1/ 3 − 2 = 0?
(a) l 2 = 4m − 1 (b) l 2 = 4m + 2
(a) only one value (b) two values
(c) three values (d) No value (c) l = 4m + 12
(d) l 2 = 4m + 1

12. If α , β are the roots of the quadratic equation 22. If α, β are


the roots of the equation
2x − 4x + 1 = 0.
2 1
Then, the value of x − (1 + a ) x + (1 + a 2+ a 4 ) = 0, then α 2 + β 2 is
2 2

1 1 2
+ is equal to equal to
α + 2β β + 2α
12 17 11 13 (a) a4 + a2 (b) a2
(a) (b) (c) (d)
17 12 17 17 (c) (a + a )
2 4 2
(d) None of these
x +2 −x 1 1 1
13. Solve the equation 2 +2 = 5 and find the 23. If the roots of + = are equal in
roots of the equation. x+ p x+q r
(a) {0, 2} (b) {−2, 0} (c) {0, 1} (d) {−1, 0} magnitude and opposite in sign, then product of
14. If α and β are roots of the equation x 2 + p = 0 roots is
1 2 p2 + q 2
where p is a prime, then which equation has the (a) − (p + q2) (b)
2 2
roots 1/α and 1/β? p+q 1
1 1 1 1 (c) (d) ( p + q )2
(a) 2
− = 0 (b) px2 + 1 = 0 (c) px2 − 1 = 0 (d) 2 + = 0 2 2
x p x p
24. If α , β are the roots of 3x 2 + 2x + 1 = 0, then the
15. The roots of the equation x + px + q = 0 are 1 2
1−α 1−β
equation whose roots are and is
and 2. The roots of the equation qx 2 − px + 1 = 0 1+α 1+β
must be (a) x2 + 2 x + 3 = 0 (b) x2 − 2 x + 3 = 0
−1 1
(a) and 1 (b) and 1 (c) x2 + 2 x − 3 = 0 (d) x2 − 2 x − 3 = 0
2 2
−1 25. If one root of px 2 + qx + r = 0 is double of the
(c) and − 1 (d) None of these
2
other root, then which one of the following is
16. Which one of the following is the quadratic correct?
equation whose roots are reciprocal to the roots (a) 2 q 2 = 9 pr (b) 2 q 2 = 9 p (c) 4q 2 = 9 r (d) 9q 2 = 2 pr
of the quadratic equation 2x 2 − 3x − 4 = 0 ?
(a) 3x2 − 2 x − 4 = 0 (b) 4x2 + 3x − 2 = 0 26. If the roots of the equation x 2 + x + 1 = 0 are in
(c) 3x2 − 4x − 2 = 0 (d) 4x2 − 2 x − 3 = 0
the ratio m : n, then
m n
(a) + + 1= 0 (b) m+ n + 1= 0
17. The value of x satisfying the equation n m
x + 4 = x − 2 is (c)
m n
+ + 1= 0 (d) m + n + 1 = 0
n m
(a) 0, 5 (b) 0, 4
(c) 5 (d) None of these 27. What is one of the value of x in the equation
18. If the roots of the quadratic equation x 1 − x 13
+ = ?
px 2 + qx + r = 0 are reciprocal to each other, then 1− x x 6
(a) q = r (b) p = r (a)
5
(b)
7
(c)
9
(d)
11
(c) q divides r (d) p divides q 13 13 13 3
MATHEMATICS Quadratic Equations and Inequalities 169

28. What are the roots of the equation 40. The solution of the equation
−1 −1 −1 −1
(a + b + x) =a +b +x ? x 2 − 16 − ( x − 4) = x 2 − 5x + 4 is
(a) a, b (b) − a, b (c) a, − b (d) − a, − b
(a) 4, 5, −  (b) {4, 5} (d) 5, − 
13 13
(c) {4}
29. The number of roots of the quadratic equation  3  3
8 sec2 φ − 6 sec φ + 1 = 0 is 41. If sinθ and cosθ are the roots of the equation
(a) n (b) 2 (c) 0 (d) No solution ax 2 − bx + c = 0, then which one of the following
2
− 7x + 7
30. The number of real roots of 32x = 9 is is correct?
(a) 3 (b) 1 (c) 2 (d) 4 (a) a2 + b 2 + 2 ac = 0 (b) a2 − b 2 + 2 ac = 0
(c) a2 + c 2 + 2 ab = 0 (d) a2 − b 2 − 2 ac = 0
31. If x = 2 + 2, then
(a) x2 + 4x + 2 = 0 (b) x2 − 2 x − 2 = 0 42. The equation (1 + n 2 ) x 2 + 2ncx + ( c2 − a 2 ) = 0 will
(c) x2 − 4x + 2 = 0 (d) x2 − 4x − 2 = 0 have equal roots, if
(a) c 2 = 1 + a2 (b) c 2 = 1 − a2
x 2 − bx (c) c 2 = 1 + n2 + a2 (d) c 2 = (1 + n2 ) a2
32. For what value of k will the equation
ax − c
k−1 43. What is the condition that the equation
= have roots reciprocal to each other?
k+1 ax 2 + bx + c = 0, where a ≠ 0 has both the roots
1+ c c+1 b+c
(a) (b) (c) a − c (d) positive?
1− c c −1 c −1
(a) a, b and c are of same sign
33. If ‘α’ and ‘β’ be the roots of ax − bx + b = 0, the 2 (b) a and b are of same sign
(c) b and c have the same sign opposite to that of a
α β
value of + is (d) a and c have the same sign opposite to that of b
β α
44. The equation whose roots are twice the roots of
(a) a / b (b) b/ a (c) a/ b (d) − a / b
the equation x 2 − 2x + 4 = 0 is
34. If the equations 2x − 7x + 3 = 0 and
2
(a) x2 − 2 x + 4 = 0 (b) x2 − 2 x + 16 = 0
4x 2 + ax − 3 = 0 have a common root, then what (c) x2 − 4x + 8 = 0 (d) x2 − 4x + 16 = 0
is the value of a?
(a) − 11 or 4 (b) − 11 or − 4 (c) 11 or − 4 (d) 11 or 4 45. If α and β are the roots of the equation
35. If the roots of x + bx + c = 0 be α and β and those
2 x 2 + px + q = 0, then − α −1 , − β −1 are the roots of
of x 2 + px + q = 0 be k α and k β, then which one of the following equations?
(a) cb 2 = qp2 (b) qc 2 = b 2 p2 (a) qx2 − px + 1 = 0 (b) q 2
+ px + 1 = 0
(c) qb 2 = cp2 (d) None of these (c) x + px − q = 0
2
(d) x − px + q = 0
2

36. If − 4 is a root of the equation x 2 + px − 4 = 0 and 46. If one root of the equation ax 2 + x − 3 = 0 is − 1,
the equation x + px + q = 0 has equal roots, then
2
then what is the other root?
the values of p and q are, respectively 1 1 3
(a) (b) (c) (d) 1
9 9 9 4 2 4
(a) − 3 and (b) 3 and (c) and 3 (d) 4 and 3
4 4 4 47. If the equation ( a 2 + b2 ) x 2 − 2 ( ac + bd ) x
37. If α , β are the roots of 2x 2 − 6x + 3 = 0, then the + ( c2 + d 2 ) = 0 has equal roots, then which one of
α β   1 1 the following is correct?
value of  +  + 3  +  + 2 αβ is
 β α  α β (a) ab = cd (b) ad = bc
(c) a 2 + c 2 = b 2 + d 2
(d) ac = bd
(a) 12 (b) 23 (c) 13 (d) − 13
38. If the roots of the equation x + 2ax + b = 0, are 2 48. What is the solution of the equation
real and distinct and they differ by atmost 2m, x x+3 3
then b lies in the interval − =− ?
x+3 x 2
(a) (a2 − m2 , a2 ) (b) [a2 − m2 , a2 )
(a) 1 (b) 2 (c) 4 (d) None of these
(c) (a2 , a2 + m2 ) (d) None of these
49. What are the roots of the equation
39. The solution of the equation
( x + 2) ( x − 5) ( x − 6) ( x + 1) = 144 is 4x − 3 ⋅ 2x + 2 + 32 = 0?
(a) {7, − 3} (b) {7, − 3, 2} (c) {7, − 3, 2, 1} (d) {7, 2} (a) 1, 2 (b) 3, 4 (c) 2, 3 (d) 1, 3
170 CDS Pathfinder

50. If α and β are the roots of the equation 61. An equation equivalent to the quadratic
x − x − 1 = 0, then what is the value of (α + β )?
2 4 4 equation x 2 − 6x + 5 = 0 is
(a) 7 (b) 0 (a) x2 − 5x + 6 = 0 (b) 5x2 − 6x + 1 = 0
(c) 2 (d) None of these (c) | x − 3| = 2 (d) 6x2 − 5x + 1 = 0
10
51. When the roots of the quadratic equation 62. If the sum of a number and its reciprocal is ,
ax + bx + c = 0 are negative and reciprocals of each
2
then the numbers are 3
1 1 1 1
other, then which one of the following is correct? (a) 3, (b) 3, − (c) −3, (d) −3, −
(a) b = 0 (b) c = 0 (c) a = c (d) a = − c 3 3 3 3

52. If sum as well as product of roots of a quadratic 63. Divide 16 into two parts such that the twice of
the square of the greater part exceeds, the
equation is 9, then what is the equation?
square of the smaller part by 164. Then, the
(a) x2 + 9x − 18 = 0 (b) x2 − 18x + 9 = 0
greater part is
(c) x + 9x + 9 = 0
2
(d) x2 − 9x + 9 = 0 (a) 58 (b) 10 (c) 6 (d) 15

53. What are the roots of the equation 64. The number of straight lines that can connect
log10( x 2 − 6x + 45) = 2 ? x ( x − 1)
‘x’ points is given by the equation y = .
2
(a) 9, − 5 (b) − 9, 5 (c) 11, − 5 (d) − 11, 5
How many points does a figure have if only 15
1 1 lines can be drawn connecting them?
54. The sum of the roots of the equation +
x+a x+ b (a) 15 (b) 10 (c) 6 (d) 5
1
= is zero. What is the product of the roots of the 65. The two successive natural numbers whose
c squares have sum 221 are
equation? (a) 10 and 11 (b) 11 and 12
(a + b) (a + b) (c) − 10 and − 11
(a) − (b) (d) None of these
2 2
( a2 + b 2 ) ( a2 + b 2 ) 66. The two consecutive positive odd integers, the
(c) − (d) sum of whose squares is 130.
2 2
(a) − 7 and − 9 (b) 7 and 9
55. For what value of k, will the roots of the equation (c) 7 and 5 (d) 3 and − 5
kx 2 − 5x + 6 = 0 be in the ratio of 2 : 3?
67. The solution set of inequation 2x + 1 ≥ 7 is
(a) 0 (b) 1 (c) − 1 (d) 2
(a) x ≥ 8 (b) x ≥ 3
56. What is the ratio of sum of squares of roots to the (c) x ≥ 6 (d) None of these
product of the roots of the equation
x−1
7x 2 + 12x + 18 = 0 ? 68. The values of x satisfying inequation ≥ 4 is
3
(a) 6 : 1 (b) 1 : 6 (c) − 6 : 1 (d) − 6 : 7
(a) x ≤ 13 (b) x ≥ 12 (c) x ≥ 13 (d) x = 13
x ( x − 1) − ( m + 1) x
57. If the roots of the equation = 69. The values of x satisfying 3x + 2 ≤ 5x − ( 4 − x ) is
( x − 1) ( m − 1) m
(a) x ≤ 2 (b) x ≥ 2
are equal, then what is the value of m ? (c) x = 2 (d) None of these
(a) 1 (b) 1 / 2 (c) 0 (d) − 1/2
70. The solution set of x for the inequations
58. If 3x + 27( 3− x ) = 12, then what is the value of x? 2x + 3 ≥ 8 and 3x + 1 ≤ 12 is
5 11 5 11
(a) 1 (b) 2 (c) 1, 2 (d) 0, 1 (a) < x≤ (b) < x<
2 3 2 3
59. What is the magnitude of difference of the roots of 5
(c ) ≤ x ≤
11 5
(d) ≥ x ≥
11
x 2 − ax + b = 0 ? 2 3 2 3
a2 − 4b b2 − 4 a 1 3  1
(a) (b) 71. The values of x satisfying  x + 4 ≥ ( x − 6)
2 5  3
(c) 2 a2 − 4b (d) b 2 − 4 ab are
(a) x ≥ 120 (b) x ≤ 120 (c) x ≤ 12 (d) x ≥ 12
60. What can be said about the roots of the equation
x 2 − x − 2 = 0? 72. 4x − 1 ≤ 0, then the solution set is
2

−1 1 −1 1
(a) both of them are integers (a) ≤ x≤ (b) < x<
2 2 2 2
(b) both of them are natural numbers
−1 1
(c) the latter of the two is negative (c) ≥ x≥ (d) None of these
2 2
(d) None of the above
MATHEMATICS Quadratic Equations and Inequalities 171

73. The shaded region, including the boundary in the Select the answer using the codes given below.
given graph, is exactly represented by (a) Only I (b) Only II
(c) Both I and II (d) Neither I nor II
Y
A (0, 6) 81. Consider the following statements
I. Every quadratic equation has atleast one real
root.
II. A quadratic equation with integral coefficients
has integral roots.

O
X III. If the coefficient of x2 and the constant term of
B a quadratic equation have opposite signs, then
(4, 0)
the quadratic equation has real roots.

(a) 3x + 2 y ≤ 12, x < 0, y ≥ 0 (b) 3x + 2 y ≤ 12, x ≥ 0, y ≥ 0 Which of the above statements is/are correct?
(c) 3x + 2 y < 12, x ≥ 0, y ≥ 0 (d) 3x + 2 y > 12, x ≥ 0, y ≥ 0 (a) Only I and II (b) Only II and III
(c) Only III (d) All of these
74. When is the expression x 2 + 3x − 10 positive only?
(a) x ≤ −5 (b) x ≥ 2 (c) − 5 < x < 2 (d) x < −5 or x > 2
82. Which of the following are quadratic equations?
1 3
75. The solution set for the quadratic inequation I. x 2 + 2
=2 II. x + = x2
x x
x 2 − 5x + 6 ≥ 0 is
III. 2x2 − x + 2 = x2 + 4x − 4 IV. x3 + 6x2 + 2x − 1 = 0
(a) [ − ∞, 2 ] ∪ [ 3, ∞ ] (b) [ − ∞, 2 ] ∪ [ 3, ∞ ]
(c) [ − ∞, 2 ] ∪ [ 3, ∞ ] (d) [2, 3] Select the correct answer using the codes given
x−2 x−3 below.
76. All real values of x for which < are (a) I, III are quadratic (b) II, III and IV are quadratic
3x + 1 3x − 2
(c) Only III is quadratic (d) None of these
−1 2 − 1 2 −1 2
(a)  ,  (b)  , (c)  ,  (d) R
 3 3   3 3   3 3  83. Consider the following statements
I. x = 1 is a root of 3x2 − 2x − 1 = 0.
77. If x + 2 y ≤ 3, x > 0 and y > 0, then one of the
solution is II. x = − 2 2 is a root of x2 + 2x − 4 = 0.
(a) x = − 1, y = 2 (b) x = 2, y = 1 III. x2 + x + 1 = 0; x = 1, x = − 1.
(c) x = 1, y = 1 (d) x = 0, y = 0 1 2
IV. 9x2 − 3x − 2 = 0; x = − , x = .
78. The shaded region in the given figure is the 3 3
solution set of the inequalities Which of the equation(s) given above is/are
Y correct?
(a) I and II (b) I, II and IV
x

(c) III and IV (d) Only IV


+
y=

Directions (Q. Nos. 84-86) A ball is thrown


2

x+
3y upwards from a rooftop, 80 m above the ground.
= It will reach a maximum vertical height and
3
X then fall back to the ground. The height of the ball
O
from the ground at time t is h which is given by,
(a) x+ y ≤ 2, x + 3 y ≥ 3, x ≥ 0, y ≥ 0 h = −16t 2 + 64 t + 80 Now, answer the following
(b) x+ y ≥ 2, x + 3 y ≥ 3, x ≥ 0, y ≥ 0
questions based on the above information.
(c) x+ y ≥ 2, x + 3 y ≤ 3, x ≥ 0, y ≥ 0
(d) x+ y ≤ 2, x + 3 y ≤ 3, x ≥ 0, y ≥ 0 84. What is the height reached by the ball after
1 sec?
79. If a + b = 2m , b + c = 6m, a + c = 2, where m is a
2
(a) 150 m (b) 128 m
real number and a ≤ b ≤ c, then which one of the (c) 64 m (d) None of these
following is correct?
(a) 0 ≤ m ≤ 1/2 (b) − 1 ≤ m ≤ 0 85. What is the maximum height reached by the
(c) 1/ 3 ≤ m ≤ 1 (d) 1 < m ≤ 2 ball?
(a) 110 m (b) 132 m
80. If α and β are the roots of the equation (c) 144 m (d) cannot be determined
( x 2 − 3x + 2 = 0), then which of the following
86. How long will it take before hitting the ground?
equation has the roots (α + 1) and (β + 1)? (a) 5 sec (b) 6 sec
I. x2 + 5x + 6 = 0 II. x2 − 5x − 6 = 0 (c) 3 sec (d) cannot say
172 CDS Pathfinder

PREVIOUS YEARS’ QUESTIONS 97. If x 2 = 6 + 6 + 6 + 6 + L ∞ , then what is one


87. If the roots of the equation x 2 − 2ax + a 2 + a − 3 = 0 of the values of x equal to? e 2013 II
(a) 6 (b) 5 (c) 4 (d) 3
are real and less than 3, then which one of the
following is correct? e 2012 I 98. Consider the following statements in respect of
(a) a < 2 (b) 2 < a < 3 (c) 3 < a < 4 (d) a > 4 the quadratic equation ax 2 + bx + b = 0, where
88. If one of the roots of quadratic equation a ≠ 0.
7x 2 − 50x + k = 0 is 7, then what is the value of k? I. The product of the roots is equal to the sum of
e 2012 I
the roots.
(a) 7 (b) 1 (c) 50/7 (d) 7/50 II. The roots of the equation are always unequal
and real.
89. Two students A and B solve an equation of the Which of the above statements is/are correct?
form x 2 + px + q = 0. A starts with a wrong value e 2014 I
of p and obtains the roots as 2 and 6. B starts (a) Only I (b) Only II
with a wrong value of q and gets the roots as (c) Both I and II (d) Neither I nor II
2 and – 9. What are the correct roots of the
equation? e 2012 II
99. If the roots of the equation Ax 2 + Bx + C = 0 are
(a) 3 and − 4 (b) − 3 and − 4 (c) − 3 and 4 (d) 3 and 4 −1 and 1, then which one of the following is
correct? e 2014 I
90. If one of the roots of the equation x − bx + c = 0 is 2
(a) A and C are both zero
the square of the other, then which of the (b) A and B are both positive
following option is correct? e 2013 I (c) A and C are both negative
(a) b 3 = 3 bc + c 2 + c (b) c 3 = 3 bc + b 2 + b (d) A and C are of opposite sign
(c) 3 bc = c + b + b
3 2
(d) 3 bc = c 3 + b 3 + b 2 100. If the roots of the equation ( a 2 − bc) x 2
+ 2 ( b2 − ac) x + ( c2 − ab) = 0 are equal, where
91. The difference of the roots of the equation
2x 2 − 11x + 5 = 0 is b ≠ 0, then which one of the following is correct?
e 2013 I
e 2014 I
(a) 4.5 (b) 4 (c) 3.5 (d) 3
(a) a + b + c = abc (b) a2 + b 2 + c 2 = 0
92. The sum of the squares of two numbers is 97 and (c) a3 + b 3 + c 3 = 0 (d) a3 + b 3 + c 3 = 3abc
the squares of their difference is 25. The product
of the two numbers is e 2013 I 101. If m and n are the roots of the equation
(a) 45 (b) 36 (c) 54 (d) 63 ax 2 + bx + c = 0, then the equation whose roots
1 1 are ( m 2 + 1) / m and ( n 2 + 1) / n is
93. If x + = 2, then what is value of x − ? e 2014 I
x x e 2013 I (a) acx2 + (ab + bc )x + b 2 + (a − c )2 =0
(a) 0 (b) 1 (c) 2 (d) −2 (b) acx2 + (ab − bc ) x + b 2 + (a − c )2 =0
(c) acx2 + (ab − bc ) x + b 2 − (a − c )2 =0
94. There are some benches in a classroom having the
number of rows 4 more than the number of (d) acx2 + (ab + bc ) x + b 2 − (a − c )2 =0
columns. If each bench is seated with 5 students, 102. In solving a problem, one student makes a
there are two seats vacant in a class of 158 mistake in the coefficient of the first degree
students. The number of rows is e 2013 I term and obtains −9 and −1 for the roots. Another
(a) 4 (b) 8 (c) 6 (d) 10 student makes a mistake in the constant term
95. Which one of the following is factor of of the equation and obtains 8 and 2 for the
1  1 roots. The correct equation was e 2014 I
x2 + + 8 x +  + 14 ? (a) x2 + 10x + 9 = 0 (b) x2 − 10x + 16 = 0
x2  x e 2013 II
1 1 (c) x − 10x + 9 = 0
2
(d) None of these
(a) x + +1 (b) x + + 3
x x
103. If m and n ( m > n ) are the roots of the equation
1 1
(c) x + + 6 (d) x + +7 7( x + 2a )2 + 3a 2 = 5a( 7x + 23a ), where a > 0, then
x x
what is 3m − n equal to? e 2014 II
96. If α and β are the roots of the equation (a) 12a (b) 14a (c) 15a (d) 18a
α2 + β2 104. If one of the roots of the equation px + qx + r = 0 2
x − x − 1 = 0, then what is
2
equal
(α 2 − β 2 ) (α − β ) is three times the other, then which one of the
to? e 2013 II following relations is correct? e 2014 II
2 3 4 (a) 3q 2 = 16 pr (b) q 2 = 24 pr
(a) (b) (c) (d) None of these
5 5 5 (c) p = q + r (d) p + q + r = 1
MATHEMATICS Quadratic Equations and Inequalities 173

105. If m and n are the roots of the equation 114. 5x – 2y =10


x 2 + ax + b = 0 and m 2, n 2 are the roots of the 4
equation x 2 − cx + d = 0, then which of the 3
following is/are correct? 2x + 6 y =21
2
I. 2b − a 2 = c II. b2 = d
1
Select the correct answer using the codes given
below. e 2014 II
(a) Only I (b) Only II 0 1 2 3 4
(c) Both I and II (d) Neither I nor II The linear inequations, for which the shaded
106. The sign of the quadratic polynomial ax 2 + bx + c area in the figure given above is the solution set,
is always positive, if e 2015 I are e 2016 I
(a) a is positive and b 2 − 4ac ≤ 0 (a) 2 x + 6 y ≤ 21, 5x − 2 y ≤ 10
(b) a is positive and b 2 − 4ac ≥ 0 (b) 2 x + 6 y ≤ 21, 5x − 2 y ≥ 10
(c) a can be any real number and b 2 − 4ac ≤ 0 (c) 2 x + 6 y ≥ 21, 5x − 2 y ≤ 10
(d) a can be any real number and b 2 − 4ac ≥ 0 (d) 2 x + 6 y ≥ 21, 5x − 2 y ≥ 10
100
107. The number of values of x satisfying x + > 50,
115. Let p and q be non-zero integers. Consider the
x
where x is a natural number less than or equal polynomial A ( x ) = x 2 + px + q. It is given that
to 100, is e 2015 II ( x − m ) and ( x − km ) are simple factors of A ( x ),
(a) 51 (b) 53 (c) 55 (d) 57 where m is a non-zero integer and k is a positive
108. If the roots of the quadratic equation integer, k ≥ 2. Which one of the following is
x 2 − 4x − log10 N = 0 are all real, then the correct? e 2016 I
(a) (k + 1)2 p2 = kq (b) (k + 1)2 q = kp2
minimum value of N is e 2015 II
1 1 1 (c) k 2q = (k + 1)p2 (d) k 2 p2 = ( k + 1)2 q
(a) (b) (c) (d) 10000
100 1000 10000
116. Which of the points P ( 5, − 1), Q ( 3, − 2) and
109. What is 4 + 4 − 4 + ... equal to? e 2015 II R (1, 1) lie in the solution of the system of
13 − 1 13 + 1 inequations x + y ≤ 4 and x − y ≥ 2? e 2016 I
(a) 3 (b) (c) (d) 0
2 2 (a) Q and R (b) P and R
110. If k = x − y + 2z, where −2 ≤ x ≤ 1 and −1 ≤ y ≤ 2 (c) P and Q (d) P, Q and R
and 3 ≤ z ≤ 6, then which one of the following is a + 2b + a − 2b
correct? e 2015 II 117. If x = , then bx 2 − ax + b is
(a) 0 ≤ k ≤ 9 (b) 5 ≤ k ≤ 11 (c) 2 ≤ k ≤ 14 (d) 2 ≤ k ≤ 11 a + 2b − a − 2b
equal to (given that, b ≠ 0) e 2016 I
111. If the sum of the roots of ax 2 + bx + c = 0 is equal
(a) 0 (b) 1 (c) ab (d) 2ab
to the sum of the squares of their reciprocals,
then which one of the following relations is 118. If 3x 2 − 7x − 30 − 2x 2 − 7x − 5 = x − 5 has α
correct? e 2016 I and β as its roots, then the value of αβ is e 2016 I
(a) ab 2 + bc 2 = 2 a2c (b) ac 2 + bc 2 = 2 b 2 a (a) − 15 (b) − 5 (c) 0 (d) 5
(c) ab 2 + bc 2 = a2c (d) a2 + b 2 + c 2 = 1
119. If the roots of the equation lx 2 + mx + m = 0 are
112. If the equations x 2 − px + q = 0 and x 2 + qx − p = 0
p q m
have a common root, then which one of the in the ratio p : q, then + + is equal to
following is correct? e 2016 I q p l
e 2016 I
(a) p − q = 0 (b) p + q − 2 = 0 (a) 0 (b) 1 (c) 2 (d) 3
(c) p + q − 1 = 0 (d) p − q − 1 = 0
1 1
120. The solution of the inequation 1 + − ≥ 0 is
113. Under what condition on p and q, one of the x x2
roots of the equation x 2 + px + q = 0 is the square (given that, x ≠ 0) e 2016 I
of the other? e 2016 I (a) x > 0 (b) x < 0
(a) 1 + q + q 2 = 3 pq (b) 1 + p + p2 = 3 pq −1 − 5 −1 + 5 −1 − 5 −1 + 5
(c) ≤ x≤ (d) x ≤ or x ≥
(c) p3 + q + q 2 = 3 pq (d) q 3 + p + p2 = 3 pq 5 2 2 2
174 CDS Pathfinder

ANSWERS
1 b 2 c 3 a 4 b 5 c 6 a 7 c 8 d 9 d 10 b
11 b 12 a 13 b 14 b 15 c 16 b 17 c 18 b 19 d 20 c
21 d 22 b 23 a 24 b 25 a 26 a 27 c 28 d 29 d 30 c
31 c 32 b 33 b 34 a 35 c 36 b 37 c 38 b 39 b 40 b
41 b 42 d 43 d 44 d 45 a 46 c 47 b 48 a 49 c 50 a
51 c 52 d 53 c 54 c 55 b 56 d 57 d 58 c 59 a 60 a
61 c 62 a 63 b 64 c 65 a 66 b 67 b 68 c 69 b 70 c
71 b 72 a 73 b 74 d 75 a 76 b 77 c 78 d 79 c 80 d
81 c 82 c 83 b 84 b 85 c 86 a 87 a 88 a 89 b 90 a
91 a 92 b 93 a 94 b 95 c 96 b 97 d 98 d 99 d 100 d
101 a 102 c 103 c 104 a 105 b 106 a 107 c 108 c 109 c 110 c
111 a 112 d 113 c 114 a 115 b 116 c 117 a 118 a 119 a 120 d

HINTS AND SOLUTIONS 


1. (b) An equation of n degree has n roots. 5. (c) Here, sum of roots, 8. (d) Here, α + β = − b / a and αβ = c / a
So, quadratic equation has two roots. 4+ 7 4− 7 Now, roots of required equation are
S= + =4
2. (c) a 2 b 2 x2 − a 2 x − b 2 x + 1 = 0 2 2 1 1
, .
Product of roots, α β
⇒ a 2 x ( b 2 x − 1) − 1 ( b 2 x − 1) = 0
1 1 α + β −b / a −b
⇒ ( b x − 1) ( a x − 1) = 0
2 2 4+ 7   4 − 7  16 − 7 9 ⇒S = + = = =
P=    = = α β αβ c/ a c
So, either a 2 x − 1 = 0 ⇒ a 2 x = 1  2   2  4 4
1 1 1 a
or b 2 x − 1 = 0 or b 2 x = 1 The required equation is P= ⋅ = =
α β c/ a c
⇒ x = 1/ a 2 or x = 1/ b 2 x2 − Sx + P = 0
9 ∴ Required quadratic equation is
3. (a) Given, ax2 − 2 5x + 4 = 0 has x 2 − 4x + = 0
equal roots.
4 x2 − S x + P = 0
⇒ 4x2 − 16 x + 9 = 0 −
∴ Discriminant x2 −   x + = 0
b a

6. (a) Given, x2 − 8x + p = 0  c  c
= ( − 2 5 ) 2 − 4 ( a) 4 = 0
[Q D = B 2 − 4 AC ] Sum of roots α + β = 8 and product of ⇒ cx2 + bx + a = 0
⇒ 20 − 16 a = 0 ⇒ a = 5 / 4 roots αβ = p
9. (d) Given, α + β = 24 and α − β = 8
α 2 + β 2 = (α + β )2 − 2 αβ …(i)
4. (b) Given, 3x2 = 8x + ( 2k + 1) On solving, we get α = 16 and β = 8
⇒ 40 = (8)2 − 2 ( p ) [Q α 2 + β 2 = 40] ∴ Sum of roots = α + β = 24
or 3x2 − 8x − ( 2k + 1) = 0
⇒ 40 − 64 = − 2 p and product of roots = 16 × 8 = 128
Let first and second root be α and 7α ,
respectively. ⇒ − 24 = − 2 p ⇒ p = 12 So, required equation is
8
∴ Sum of roots = α + 7α = 7. (c) Given, x2 − 5x + 6 = 0 x2 − 24x + 128 = 0.
8 1 3
⇒ 8α = ⇒ α = Sum of roots = −
Coefficient of x
3 3 10. (b) Let α and β be the roots of the
Coefficient of x 2
1 7 equation ax2 + bx + c = 0.
So, roots are and . ⇒ α+β =5 −b c
3 3 Constant term Then, α + β = and αβ =
1 7 7 Product of roots = a a
Also, product of roots = × = Coefficient of x 2
3 3 9 −3 b 9c
⇒ αβ = 6 Now, 3α + 3β = and 3α ⋅3 β =
7 − ( 2k + 1) a a
⇒ = Now, (α − β )2 = (α + β )2 − 4αβ
9 3 ∴ Required equation = x2 −
= (5 ) 2 − 4 × 6 = 1
7 (Sum of roots) x
⇒ = − ( 2k + 1) ⇒ α − β = ± 1 and α + β = 5
3 + Product of roots
⇒ α 2 − β 2 = (α + β ) (α − β ) −3 b 
⇒ x2 − 
9c
⇒ 7 = − 6 k − 3 ⇒ 10 = − 6 k x+ =0
= 5 ( ± 1)  a  a
−5
⇒ k= ⇒ α −β = ± 5
2 2
⇒ ax2 + 3bx + 9c = 0
3
MATHEMATICS Quadratic Equations and Inequalities 175

11. (b) Given equation is 15. (c) The equation is x2 + px + q = 0. By given condition,
x 2/3
+ x
1/3
−2=0 Sum of roots = − p = 1 + 2 α + β = α2 + β2
⇒ (x 1/3 2
) + x 1/3
−2=0 ⇒ p = −3 ⇒ α+β = (α + β )2 − 2αβ
Let x1 / 3 = x Product of roots = q = 1 × 2 = 2 2
=  −  − 2  
b b c
⇒ −
⇒ x2 + x − 2 = 0, ∴ Equation qx2 − px + 1 = 0 becomes a  a  a
It is a quadratic equation in x. 2x 2 − ( − 3) x + 1 = 0 −b b 2 2c
⇒ = − ⇒ − ba = b 2 − 2 ca
∴ Discriminant of x + x − 2 = 0 is
2
⇒ 2x 2 + 3x + 1 = 0 a a2 a
B 2 − 4 AC = 1 2 − 4 ( 1) ( − 2) = 9 > 0 ⇒ ( 2x + 1) ( x + 1) = 0 ⇒ 2 ac = b 2 + ab
Hence, two real values of x satisfy the

1
x = − or x = − 1 21. (d) Here, roots are α and α + 1 .
given equation. 2 ∴ α + (α + 1) = l [sum of roots]
4 1 l −1
12. (a) Here, α + β = = 2, αβ = 16. (b) Given, 2x2 − 3x − 4 = 0 ⇒ 2α = l − 1 ⇒ α =
2 2 For getting a reciprocal roots, we replace 2
1 1 Also, α (α + 1) = m or α 2 + α = m
Now, + 1
x by , we get
α + 2β β + 2α 2
x
⇒  l − 1 +  l − 1 = m
2    
β + 2α + α + 2β 2   − 3   − 4 = 0
1 1  2   2 
=  x  x
(α + 2 β ) (β + 2 α ) ⇒ ( l − 1) 2 + 2 ( l − 1) = 4m
2 3
⇒ − −4= 0 ⇒ l 2 − 1 = 4m ⇒ l 2 = 4 m + 1
3α + 3β x2 x
=
αβ + 2α 2 + 2β 2 + 4αβ 22. (b) Here, α + β = ( 1 + a 2 )
⇒ − 4x 2 − 3x + 2 = 0
1
⇒ 4x 2 + 3x − 2 = 0 and αβ = ( a 4 + a 2 + 1)
3(α + β ) 3( 2) 12
= = = 2
2 (α + β )2 + αβ 2 ( 2)2 + 1 17 17. (c) As, x + 4 = x − 2 Q α 2 + β 2 = (α + β )2 − 2 αβ
2 On squaring both sides, we get = ( 1 + a 2 ) 2 − ( a 4 + a 2 + 1)
13. (b) Given equation is 2x + 2 + 2− x = 5 ( x + 4) = ( x − 2) 2
= 1 + a 4 + 2a 2 − a 4 − a 2 − 1
1
⇒ 2x 22 + 2−x = 5 ⇒ 4 ⋅ 2x + x = 5 ⇒ x + 4 = x + 4 − 4x
2
∴ α2 + β2 = a2
2 ⇒ x 2 − 5x = 0 ⇒ x = 0 , x = 5
1 1 1 1
Put 2x = y, 4 y + = 5 23. (a) Here, + =
y But for x = 0, 0+ 4 = 0 − 2 x+ p x+ q r
4≠−2
⇒ 4y2 + 1 = 5y ⇒ 4y2 − 5y + 1 = 0 ⇒ r ( x + p + x + q) = ( x + p) ( x + q)
So, x = 5 is the only solution.
⇒ ( y − 1) ( 4 y − 1) = 0 ⇒ x2 + ( p + q − 2r ) x + pq
1
⇒ y − 1 = 0 or 4y − 1 = 0 18. (b) Let roots of equation be α and . − ( p + q) r = 0
α
⇒ y = 1 or 4y = 1 Let roots be α and ( − α ), then
∴ Product of roots
1 α + ( − α) = 0
⇒ y = 1, 1 Constant term r p+ q
4 =α × = = ⇒ − ( p + q − 2r ) = 0 ⇒ r =
By condition, α Coefficient of x 2
p 2
r So, product of roots = pq − ( p + q ) r
2x = y 2x = y ⇒ 1= ⇒ r= p
p ( p + q)
⇒ 2x = 1 ⇒ 2x =
1 = pq − ( p + q ) ⋅
4 19. (d) Let roots be α and β, then 2
1 α+β=−1 ( p + q ) 2 −1 2
⇒ 2 =2
x 0
⇒ 2 x = 2 = 2 −2 = pq − = ( p + q2 )
1 1 1 β+α 1 2 2
2 + = ⇒ =
∴ x=0 ∴ x = −2 α β 6 αβ 6 2 1
24. (b) Here, α + β = − and αβ =
−1 1 3 3
Hence, the roots of the equation are 0 = ⇒ αβ = − 6
and − 2. αβ 6 1− α 1− β
⇒ S= +
14. (b) Since, α and β are roots of the ∴ Required equation is, 1+ α 1+ β
x2 − (α + β ) x + αβ = 0 ( 1 − α )( 1 + β ) + ( 1 + α )( 1 − β )
equation x + p = 0.
2
=
( 1 + α )( 1 + β )
∴ α + β = 0 and αβ = p ⇒ x2 − ( −1)x + ( −6) = 0 1
1 1 α+β ∴ x2 + x − 6 = 0 2−2×
So, + = =0 2 − 2αβ 3
= =
α β αβ 20. (c) Let α and β be the roots of the 1 + (α + β ) + αβ 1 + ( −2 / 3) + 1 / 3
1 1 1 1 equation 4 3
and ⋅ = = = × =2
α β αβ p ax2 + bx + c = 0 3 2
b
∴ Required equation x2 − 0 ⋅ x +
1
=0 ∴ Sum of roots (α + β ) = − 1 − α 1 − β 1 − (α + β ) + αβ
p a P= × =
c
and product of roots (αβ ) = 1 + α 1 + β 1 + (α + β ) + αβ
⇒ px 2 + 1 = 0 a
176 CDS Pathfinder

1 − ( −2 / 3 ) + 1 / 3 2 29. (d) 8 sec 2 φ − 6 sec φ + 1 = 0 Now, qb 2 = k 2 αβ (α + β )2


= = =3
1 + ( −2 / 3 ) + 1 / 3 2 / 3 ⇒ 8 sec φ − 4 sec φ − 2 sec φ + 1 = 0
2
= k 2 (α + β )2 αβ
∴ Required equation is
⇒ ( 4 sec φ − 1) ( 2 sec φ − 1) = 0 = [ k (α + β )]2 αβ
x2 − Sx + P = 0 or x2 − 2x + 3 = 0
⇒ sec φ = 1/ 4 or sec φ = 1/ 2 ∴ qb = p 2 c
2

25. (a) Given, px + qx + r = 0


2
But sec φ ≥ 1 or sec φ ≤ − 1 36. (b) Since, − 4 is a root of
Let the roots be α and β. Hence, the equation has no solution. x 2 + px − 4 = 0
By given condition, β = 2α 2
r 30. (c) Here, 32 x − 7x + 7
= 9 = 32 ( − 4) + p ( − 4) − 4 = 0
2
Product of roots (αβ ) = = 2 α 2
p On comparing, we get ⇒ 16 − 4 p − 4 = 0 ⇒ p = 3
r 2x 2 − 7x + 7 = 2 As roots of x2 + px + q = 0 are equal.
⇒ α =
2
…(i)
2p 2x 2 − 7x + 5 = 0 ∴ D = B 2 − 4 AC = 0
q
Sum of roots (α + β ) = − = 3 α …(ii) Here, D = b 2 − 4 a c = 49 − 4 ( 2) (5) = 9 ⇒ p 2 − 4 q = 0 or 32 − 4 q = 0 ⇒ q =
9
p 4
q2 Since, D > 0, so it has two real roots.
On squaring Eq. (ii), we get 9α 2 = 2 So, p = 3, q =
9
p 31. (c) Here, x= 2 + 2 ⇒x− 2= 2 4
 r  q2 On squaring both sides, we get
⇒ 9  = [from Eq. (i)] 37. (c) Here, α + β = 3 and αβ = 3 / 2
 2 p  p2 ( x − 2)2 = 2 ⇒ x 2 − 4 x + 4 = 2 α β   1 1
  ⇒  +  + 3  +  + 2 αβ
⇒ x2 − 4 x + 2 = 0 β α α β
⇒ 9rp = 2q 2
α m x2 − bx k − 1 α 2 + β 2 3(α + β )
26. (a) Let roots be α , β, then = 32. (b) = = + + 2 αβ
β n ax − c k+1 αβ αβ
[given] ⇒ ( x2 − bx) ( k + 1 ) = ( k − 1 ) ( ax − c ) (α + β )2 − 2αβ 3(α + β )
α + β = − 1, αβ = 1 = + + 2αβ
⇒ x2 k + x 2 − bxk − bx = kax αβ αβ
m n α β − kc − ax + c 32 − 2 ⋅ 3 / 2 3⋅3
+ + 1= + + 1 = + + 2  
3
n m β α ⇒ ( k + 1 ) x − x ( bk + b + ka − a )
2
3/2 3/2  2
α+β −1 + kc − c = 0 6 9
= + 1= + 1= − 1+ 1= 0 = + + 3 = 4 + 6 + 3 = 13
αβ 1 Since, roots are reciprocal to each other. 3/2 3/2
x 1− x 1 So, product = 1 38. (b) Let α, β be the roots of
27. (c) Let = y ⇒ =
1− x x kc − c x 2 + 2ax + b = 0
y i.e. = 1 ⇒ kc − c = k + 1 …(i)
1 13 ( k + 1) ⇒ α + β = −2a and αβ = b.
∴ y+ = ⇒ ( y + 1) 6 = 13 y
2
c+ 1
y 6 ⇒ k ( c − 1) = c + 1 ⇒ k = By hypothesis, |α − β| ≤ 2m
⇒ 6 y 2 − 13 y + 6 = 0 c−1
⇒ (α − β)2 ≤ 4m 2
⇒ 6 y2 − 9y − 4y + 6 = 0 33. (b) Here, α + β = b / a and αβ = b / a
⇒ (α + β)2 − 4αβ ≤ 4m 2
⇒ 3 y ( 2 y − 3 ) − 2 ( 2 y − 3) = 0 α β α+β b/a b
So, + = = = ⇒ 4a 2 − 4b ≤ 4m 2
⇒ (3 y − 2 ) ( 2 y − 3 ) = 0 β α αβ b/a a
⇒ a 2− b ≤ m 2 …(ii)
2 3
∴ y = and and discriminant of Eq. (i) is greater
3 2 34. (a) Given, 2x 2 − 7 x + 3 = 0 than 0.
When, y =
2

x
=
4 ∴ 2x − 6 x − x + 3 = 0
2
⇒ 4a 2 − 4b > 0 ⇒ b < a 2 …(iii)
3 1− x 9 ⇒ 2x ( x − 3) − 1 ( x − 3) = 0 From Eqs. (ii) and (iii),
4 b ∈[ a 2 − m 2 , a 2 )
⇒ 9x = 4 − 4x ⇒ x = ⇒ ( 2x − 1 ) ( x − 3) = 0
13 1 39. (b) Refer to example 15. [Rule 5]
3 x 9 When, x=
When, y = ⇒ = 2 40. (b) Refer to example 14. [Rule 4]
2 1− x 4 2
4   + a   − 3 = 0
1 1 41. (b) Since, sin θ and cos θ are the roots of
9
⇒ 4x = 9 − 9 x ⇒ x =  2  2 the equation ax2 − bx + c = 0.
13 a a
⇒1 + − 3 = 0 ⇒ =2⇒ a=4 b
1 1 1 1 ∴ sin θ + cos θ = ...(i)
28. (d) Given, = + + 2 2 a
a+ b+ x a b x c
When, x = 3, 4 (3)2 + a (3) − 3 = 0 and sin θ cos θ =
1 1 1 1
∴ − = + ⇒ 36 + 3 a − 3 = 0 ⇒ a = − 11
a
a+ b+ x x a b On squaring both sides of Eq. (i), we get
− ( a + b) ( a + b) ∴ a = − 11 or 4
⇒ = b2
⇒ sin 2 θ + cos 2 θ + 2 sin θ cos θ = 2
( a + b + x )x ab 35. (c) Here, for equation x2 + bx + c = 0 a
⇒ x2 + ( a + b ) x + ab = 0 α + β = − b and αβ = c 2 2
− 2
⇒ 1 + 2  = 2 ⇒ 2   =
c b c b a
⇒ ( x + a) ( x + b) = 0 and for x2 + px + q = 0  a a  a a 2
⇒ x = − a, − b
kα + kβ = − p and k 2 αβ = q ⇒ 2ac = b 2 − a 2 ⇒ a 2 − b 2 + 2ac = 0
MATHEMATICS Quadratic Equations and Inequalities 177

42. (d) The equation will have equal roots, x x+3 3 1


+
1
=
1
48. (a) Given, − =− 54. (c) Given,
if B − 4 AC = 0
2 x+3 x 2 x+ a x+ b c
x+3 1 ( x + b) + ( x + a) 1
∴ ( 2nc )2 − 4 ( 1 + n2 ) ( c 2 − a 2 ) = 0 Let, y =
x
, then = ⇒ =
x+3 x y ( x + a )( x + b ) c
⇒ 4n c − 4 ( c + n c − a
2 2 2 2 2 2
1 3 ⇒ 2cx + ( a + b ) c = x2 + ( a + b )x + ab
− n2 a 2 ) = 0 y− =− ⇒ 2y − 2 = − 3y
2
y 2 ⇒ x2 + ( a + b − 2c ) x + ab − ac − bc = 0
⇒ − 4 c 2 + 4 a 2 + 4 n2 a 2 = 0 Let the roots of above equation be α
⇒ 2y2 + 3y − 2 = 0
⇒ c 2 = a 2 ( 1 + n2 ) ⇒ 2y2 + 4y − y − 2 = 0 and β.
43. (d) a and c have the same sign opposite ⇒ 2 y ( y + 2) − 1 ( y + 2) = 0 Given, α+ β=0
to that of b.
⇒ ( 2 y − 1) ( y + 2) = 0 ⇒ − ( a + b − 2c ) = 0 ⇒ a + b = 2c …(i)
44. (d) Let the roots of equation If, ( 2 y − 1) = 0 Now, αβ = ab − ac − bc
x − 2x + 4 = 0 be α and β.
2
1 x x 1 = ab − ( a + b ) c
⇒ y= ⇒ y= ⇒ = ( a + b)
Then, α + β = 2 and αβ = 4 2 x+3 x+3 2 = ab − ( a + b )
2
On taking, α → 2α and β → 2β On squaring both sides, we get
[from Eq. (i)]
2α + 2β = 4 and 2α ⋅ 2β = 4 × 4 = 16 x
=
1
2ab − ( a 2 + b 2 + 2ab ) ( a2 + b2 )
So, the new equation becomes x+3 4 = =−
2 2
x2 − 4x + 16 = 0 ⇒ 4x = x + 3 ⇒ x = 1
55. (b) Let the roots of the equation
45. (a) Since, α and β are the roots of the or y + 2=0 ⇒ y=−2
kx2 − 5x + 6 = 0 be α and β.
equation x2 + px + q = 0 Since, y cannot be negative. 5 6
∴ α + β = and αβ =
∴ α + β = − p and αβ = q Hence, x = 1 is the required solution. k k
 1 1 49. (c) Given, 4x − 3 ⋅ 2x + 2
+ 32 = 0 α 2 2
Now, −1
−α −β = −  +  −1 Given, = ⇒ α= β
α β ⇒ 2 2x
− 3 ⋅ 2 ⋅ 22 + 32 = 0
x β 3 3
α + β p ⇒ 22x − 12 ⋅ 2x + 32 = 0 ∴
2 5 2
β + β = and β 2 =
6
=−  =
 αβ  q ⇒ 2 2x
− 8 ⋅ 2x − 4 ⋅ 2x + 32 = 0
3 k 3 k
5 5 9
 − 1   − 1 = 1 = 1 ⇒ ( 2 x − 8) ( 2 x − 4) = 0 ⇒ β = and β 2 =
 
 α   β  αβ q
and 3 k k
Either, 2 =8 ⇒ x=3
x
3 9 9 9
or 2x = 4 ⇒ x = 2 β = and β = ⇒2
=
Hence, required equation is k k k2 k
x 2 − ( − α −1 − β −1 ) x + ( − α −1 ) 50. (a) Since, α and β are the roots of the ⇒ k = 1 and k ≠ 0
−1 equation x2 − x − 1 = 0.
(− β ) = 0 [Q leading coefficient is never zero.]
p 1 ∴ α + β = 1 and αβ = − 1
⇒ x2 − x+ =0 56. (d) Let α and β be the roots of the
q q Now, α 4 + β 4 = (α 2 + β 2 ) 2 − 2 (αβ )2
equation 7x2 + 12x + 18 = 0.
⇒ q x 2 − px + 1 = 0 = [(α + β ) − 2αβ ] − 2 (αβ )
2 2 2
−12 18
= ( 1 + 2) 2 − 2 = 9 − 2 = 7 ∴ α+β= and αβ =
46. (c) Since, one root of the equation 7 7
ax2 + x − 3 = 0 is − 1 . 51. (c) Let the roots of equation
⇒ α + β + 2αβ =
2 2 144
1
∴ a ( − 1) + ( − 1) − 3 = 0 ⇒ a = 4
2 ax2 + bx + c = 0 be − α and − . 49
c α 144 36 108
∴ 4x 2 + x − 3 = 0 ∴ Product of roots = ⇒ α2 + β2 = − =−
a 49 7 49
Let other root of this equation be α.
( − α )  −  =
1 c c
c ⇒ 1= ⇒c = a −
108
Now, product of roots =  α a a α2 + β2 6
a ∴ = 49 = − = − 6 : 7
3 3 52. (d) Let the roots of the quadratic αβ 18 7
∴ − 1⋅ α = − ⇒ α= equation be α and β. 7
4 4
Then, α + β = 9 and αβ = 9 x ( x − 1) − (m + 1) x
47. (b) Since, the roots of equation Hence, equation is 57. (d) Given, =
( x − 1)(m − 1) m
( a + b ) x − 2( ac + bd ) x
2 2 2
x − (α + β ) x + (αβ ) = 0
2
⇒ m ( x2 − x − m − 1)
+ ( c + d ) = 0 are equal.
2 2 ⇒ x 2 − 9x + 9 = 0
= x (mx − x − m + 1)
∴ B = 4 AC
2 53. (c) Given, log ( x2 − 6x + 45) = 2
10 ⇒ mx2 − mx − m (m + 1)
⇒ 4 ( ac + bd )2 = 4 ( a 2 + b 2 ) ( c 2 + d 2 ) ⇒ ( x2 − 6x + 45) = 102 = 100
= mx2 − x2 − mx + x
⇒ a 2 c 2 + b 2 d 2 + 2abcd = a 2 c 2 ⇒ x2 − 6x − 55 = 0
⇒ x − x − m (m + 1) = 0
2
⇒ x − 11x + 5x − 55 = 0
2
+ a 2 d 2 + b 2 c2 + b 2 d 2 Let one root be α, then other root is
⇒ x ( x − 11) + 5( x − 11) = 0 also α.
⇒ ( ad − bc )2 = 0
⇒ ( x + 5) ( x − 11) = 0 ∴ α + α = 1, ⇒ α =
1
∴ ad = bc ∴ x = 11, − 5 2
178 CDS Pathfinder

Also, α 2 = −m (m + 1) x( x − 1) 1  3 x + 4 ≥ 1 ( x − 6)
64. (c) Here, y = 15, then 15 = 71. (b)  
2
 1  = −m (m + 1) 2 2 5  3
⇒  
 2 ⇒ x2 − x − 30 = 0 ⇒
3 1
x + 2≥ x − 2
⇒ 4m 2 + 4m + 1 = 0 ⇒ x2 − 6x + 5x − 30 = 0
10 3
⇒ ( 2m + 1) 2 = 0 ⇒ 9x + 60 ≥ 10 x − 60
1 ⇒ x ( x − 6) + 5 ( x − 6) = 0
∴ m=− ⇒ − x ≥ − 120
2 ⇒ ( x + 5) ( x − 6) = 0
[multiplying both sides by − 1]
58. (c) Given, 3x + 27 (3− x ) = 12 Either x = 6 or x = − 5
⇒ x ≤ 120
Let 3x = y But x ≠ − 5, so x = 6
27 Thus, all real numbers x which are less
∴ y+ = 12 Thus, figure has 6 points. than or equal to 120 satisfies the
y
65. (a) Let the natural numbers be x and inequality.
⇒ y 2 − 12 y + 27 = 0
⇒ y − 9 y − 3 y + 27 = 0
2 x + 1 , respectively. 72. (a) We have, 4x2 − 1 ≤ 0
⇒ ( y − 3) ( y − 9) = 0 ⇒ y = 3, 9 Then, x + ( x + 1) = 221
2 2
⇒ ( 2x)2 − 1 ≤ 0
⇒ 3 = 3 or 3 = 9 = 3
x x 2 ⇒ 2x2 + 2x + 1 = 221 ⇒ ( 2x − 1) ( 2x + 1) ≤ 0
∴ x = 1 or x = 2 ⇒ 2x2 + 2x − 220 = 0 So, either ( 2x − 1) ≥ 0
59. (a) Let the roots of the given equation be ⇒ x2 + x − 110 = 0 and ( 2x + 1) ≤ 0 …(i)
α and β. ⇒ ( x + 11) ( x − 10) = 0 or ( 2x − 1) ≤ 0
∴ α + β = a and αβ = b ⇒ x = − 11 and x = 10 and ( 2x + 1) ≥ 0 …(ii)
Now, |α − β | = (α + β )2 − 4αβ x ≠ −11 [Qnumbers are natural] From Eq. (i), 2x ≥ 1 and 2x ≤ − 1
So, for x = 10, next consecutive natural 1 −1
⇒ x ≥ and x ≤
= a 2 − 4b number 2 2
= x + 1 = 10 + 1 = 11 which is not possible.
60. (a) x2 − x − 2 = 0 66. (b) Let the consecutive positive odd From Eq. (ii), ( 2x − 1) ≤ 0
( x − 2) ( x + 1) = 0 ⇒ x = 2,−1 integers be 2x + 1 and 2x + 3,
and ( 2x + 1) ≥ 0
So, both the roots are integers. so ( 2x + 1)2 + ( 2x + 3)2 = 130
2x ≤ 1 and 2x ≥ − 1
61. (c) x2 − 6x + 5 = 0 ⇒ ( x − 5) ( x − 1) = 0 ⇒ ( 4x2 + 4x + 1) 1 −1
x ≤ and x ≥
⇒ x = 5 or 1 + ( 4x2 + 12x + 9) = 130 2 2
⇒ x + 2x − 15 = 0
2 −1 1
Also, |x − 3 | = 2 ⇔ ( x − 3 ) = 2 ∴ ≤ x ≤ is the required solution set.
2 2
or − ( x − 3) = 2 ⇒ x = 5 or x = 1 ⇒ ( x + 5) ( x − 3) = 0
⇒ x = 3, x = − 5 , but x ≠ − 5 73. (b) Here, 3x + 2 y = 12 passes through
∴ x2 − 6 x + 5 = 0
the points ( 0, 6) and ( 4, 0) . So, its
and |x − 3 |= 2 are equivalent. [Q integers are positive.] graph is line AB. Also, 3x + 2 y ≤ 12 as
1 ∴ Two consecutive integers are the shaded region is below AB.
62. (a) Let the number are x and .
x 2x + 1 = 7 and 2x + 3 = 9 x = 0 and y = 0 are the Y-axis and
X-axis, respectively. x ≥ 0 and y ≥ 0
1 10 x2 + 1 10 i.e. 7 and 9.
Then, x + = , = implies the region on the right hand
x 3 x 3 67. (b) Here, 2x + 1 ≥ 7 ⇒ 2x ≥ 7 − 1 side of Y-axis and region above X-axis,
⇒ 3x2 − 10x + 3 = 0 respectively.
⇒ 2x ≥ 6 ⇒ x ≥ 3
⇒ 3x 2 − 9x − x + 3 = 0 x −1 The three portion x ≥ 0, y ≥ 0 and
⇒ 3x ( x − 3) − 1 ( x − 3) = 0 68. (c) Here, ≥4 3x + 2 y ≤ 12 intersects to give the
3
⇒ (3x − 1) ( x − 3) = 0 shaded portion OAB.
⇒ x − 1 ≥ 12 ⇒ x ≥ 13

1
x= ,x=3 74. (d) Given, x 2 + 3x − 10 > 0
69. (b) Here, 3x + 2 ≤ 5x − ( 4 − x)
3 ⇒ x 2 + 5x − 2x − 10 > 0
⇒ 3x + 2 ≤ 5x − 4 + x
63. (b) Let the smaller part = x and greater ⇒ ( x + 5) ( x − 2) > 0
part = 16 − x ⇒ 3x + 2 ≤ 6x − 4 Either, ( x + 5) > 0 and ( x − 2) > 0
By given condition, ⇒ 3x − 6x ≤ − 4 − 2 ⇒ − 3x ≤ − 6 or ( x + 5) < 0 and ( x − 2) < 0
2 ( 16 − x)2 − x2 = 164 ⇒ − x≤ − 2 ⇒ x≥ 2 ⇒ x > 2 or x < −5
⇒ 2 ( 256 + x2 − 32 x) − x2 = 164 70. (c) Here, 2x + 3 ≥ 8 ⇒ 2x ≥ 8 − 3 75. (a) x2 − 5x + 6 ≥ 0
⇒ x2 − 64 x + 348 = 0 5 ⇒ ( x − 2)( x − 3) ≥ 0
⇒ 2x ≥ 5 ⇒ x ≥
⇒ ( x − 58) ( x − 6) = 0 2
Again, 3x + 1 ≤ 12 + – +
⇒ x = 58, x = 6
11 2 3
Here x ≠ 58 ⇒ 3x ≤ 11 ⇒ x ≤
3 ⇒ x ≤ 2 or x ≥ 3
∴ x=6 By combining values, we get ⇒ x ∈ [ −∞ ,2] or [3, ∞ ]
and, hence larger part 5 11
≤ x≤ ⇒ x ∈ [ −∞ ,2] ∪ [3, ∞ ]
= 16 − x = 16 − 6 = 10 2 3
MATHEMATICS Quadratic Equations and Inequalities 179

x−2 x −3 ⇒ m 2 − 3m + 1 ≤ m 2 + 3m − 1 III. x2 + x + 1 = 0 ; x = 1, x = − 1
76. (b) <
3x + 1 3x − 2 and m + 3m − 1 ≤ − m + 3m + 1
2 2 Put x = 1, we get 12 + 1 + 1 = 3 ≠ 0
x−2 x −3 ∴ LHS ≠ RHS
⇔ − <0 ⇒ 6m ≥ 2 and 2m 2 ≤ 2
3x + 1 3x − 2 1 ∴ x = 1 is not a solution.
( x − 2)(3x − 2) − ( x − 3)(3x + 1) ⇒ m ≥ and −1 ≤ m ≤ 1
⇔ <0 3 Put x = −1, we get
(3x + 1)(3x − 2) 1
∴ ≤m≤ 1 ( −1)2 + ( −1) + 1 = 1 − 1 + 1
7 3
⇔ <0 = 1≠ 0
(3x + 1)(3x − 2) 80. (d) Given equation, x 2 − 3x + 2 = 0 ∴ LHS ≠ RHS
⇔ (3 x + 1 ) (3 x − 2 ) < 0 ⇒ x 2 − 2x − x + 2 = 0 ∴ x = − 1 is not a solution.
⇔ 3  x +  3  x −  < 0
 ⇒ ( x − 2)( x − 1) = 0 ⇒ x = 2, 1
1 2 IV. 9x 2 − 3x − 2 = 0
 3  3 Let α = 1 and β = 2 Put x = −1 / 3
⇔  x + 1  x − 2  < 0 ∴ α + 1 = 2 and (β + 1) = 3. ⇒ 9 ( −1 / 3 ) 2 − 3 ( −1 / 3 ) − 2 = 0
   
 3  3 Now, sum of roots = 2 + 3 = 5 ∴ LHS = RHS
– and product of roots = 2 × 3 = 6 Now, put x = 2 / 3
+ +
∴ Required equation ⇒ 9 ( 2 / 3)2 − 3 ( 2 / 3) − 2 = 0
–∞ –1/3 2/3 +∞
−1 2 = x 2 − (sum of roots) ∴ LHS = RHS
⇔ < x< + product of roots = 0
3 3 84. (b) Given, h = −16 t 2 + 64 t + 80
− 1 2 ⇒ x 2 − 5x + 6 = 0 put t = 1
So, x ∈  ,  Hence, the equation is neither I nor II. ⇒ h = −16 ( 1) 2 + 64 ( 1) + 80 = 128 m
 3 3 
81. (c) I. Every quadratic equation has two 85. (c) By rearranging, we get
77. (c) Here, as x > 0 and y > 0 so both are roots, which may or may not be real. h = −16 ( t 2 − 4t − 5)
positive and satisfies x + 2 y ≤ 3. II. x2 − 4x + 2 = 0 has integral ⇒ h = −16 [( t − 2)2 − 9]
∴ When, x = 3 and y = 1, we get coefficients but does not have ⇒ h = −16 ( t − 2)2 + 144
x + 2 y = 5. integral roots. when the height is maximum, t = 2
Clearly, this does not satisfy x + 2 y ≤ 3 III. Since, discriminant = b 2 − 4ac ∴ maximum height = 144 m
When x = 1 and y = 1, If a and c have opposite sign, then 86. (a) When the ball hits the ground h = 0
then, x + 2 y ≤ 3. b 2 − 4ac ≥ 0 ⇒ −16 t 2 + 64t + 80 = 0
∴ The quadratic equation has real ⇒ t 2 − 4t − 5 = 0
So, x = 1, y = 1 is one of the solutions.
roots ⇒ t2 −5 t + t −5 = 0
78. (d) As, ( 0, 0 ) satisfies x + 3 y ≤ 3.
82. (c) I. x2 +
1
=2 ⇒ ( t − 5)( t + 1) = 0 ⇒ t = 5 , − 1
And ( 0, 0 ) satisfies x + y ≤ 2.
x2 Since, the time cannot be negative.
Clearly, shaded region is the position or x4 − 2x2 + 1 = 0 So, t = 5 sec
common to the line is not a quadratic equation. 87. (a) Let f ( x ) = x 2 − 2ax + a 2 + a − 3
x + 3 y = 3 and below it and that on the 3
line x + y = 2 and below it. II. x + = x2 f (3) > 0 ⇒ 9 − 6a + a 2 + a − 3 > 0 …(i)
x
So, shaded region is the solution set Also, since roots are real
x2 + 3
x + y ≤ 2, x + 3 y ≤ 3, x ≥ 0, y ≥ 0. ⇒ = x2 ∴ B 2 − 4 AC ≥ 0
x
79. (c) Given, a + b = 2m 2 …(i) or x2 + 3 = x3 ⇒ 4 a − 4 ( a 2 + a − 3) ≥ 0
2
…(ii)
b + c = 6m …(ii) ⇒ x3 − x2 − 3 = 0 From Eqs. (i) and (ii), we have
and a+ c=2 …(iii) is not a quadratic equation. a 2 − 5 a + 6 > 0 and a − 3 ≤ 0
III. 2x2 − x + 2 = x2 + 4x − 4 ⇒ a 2 − 3a − 2a + 6 > 0
On adding Eqs. (i), (ii) and (iii), we get
or 2x2 − x + 2 − x2 − 4x + 4 = 0 ⇒ ( a − 3)( a − 2) > 0
2 ( a + b + c ) = 2m 2
+ 6m + 2
or x2 − 5x + 6 = 0 ⇒ a ≤ 3 and a < 2 or a > 3
⇒ a + b + c=m 2
+ 3m + 1 …(iv) is a quadratic equation.
On subtracting Eq. (ii) from Eq. (iv), we Combining the above inequalities, we
IV. x3 + 6x2 + 2x − 1 = 0
get have a < 2
is not a quadratic equation.
a = m 2 − 3m + 1 Hence, only III statement is correct. 88. (a) Given, 7x2 − 50x + k = 0
On subtracting Eq. (iii) from Eq. (iv), Here, a = 7, b = − 50 and c = k
83. (b) I. 3x 2 − 2x − 1 = 0
we get
Put x = 1 ⇒ 3 ( 1)2 − 2 ( 1) − 1 = 0 Since, α and β are the roots of the given
b = m 2 + 3m − 1 ∴ LHS = RHS
equation
−b 50
On subtracting Eq. (i) from Eq. (iv), we ∴α + β = ⇒α+β=
II. x 2 + 2x − 4 = 0 a 7
get 1
Put x = −2 2 ⇒ β= [Q α = 7, given]
c = − m 2 + 3m + 1 ⇒ ( − 2 2 ) 2 + 2( − 2 2 ) − 4 = 0 7
As, a ≤ b and b ≤ c c 1 k
∴ LHS = RHS and αβ = ⇒ 7× = ⇒ k=7
a 7 7
180 CDS Pathfinder

89. (b) Let α and β be the roots of the ⇒ ( x2 + y 2 ) − 2x y = 25 −b c


∴ α+β = = 1 and αβ = = −1
quadratic equation ⇒ 97 − 2x y = 25 [from Eq. (i)] a a
x2 + px + q = 0. ⇒ 2x y = 72 α2 + β2 (α + β )2 − 2αβ
=
Given that, A starts with a wrong value ∴ x y = 36 (α 2 − β 2 )(α − β ) (α − β )2 (α + β )
of p and obtains the roots as 2 and 6.
1 (α + β )2 − 2αβ
But this time q is correct. 93. (a) Given that, x + = 2 …(i) =
x [(α + β ) 2 − 4αβ ](α + β )
i.e. Product of roots
= q = α ⋅ β = 6 × 2 = 12 ...(i) On squaring both sides, we get
2 ( 1) 2 − 2 × −1 3
and B starts with a wrong value of q and  x + 1  = 4 ⇒ x2 + 1 + 2 = 4 = =
  [( 1) 2 − 4 × −1] × 1 5
gets the roots as 2 and − 9. But this time  x x2
p is correct. 1 97. (d) Given,
⇒ x2 + 2 = 2 …(ii)
i.e. Sum of roots x x2 = 6 + 6+ 6+ 6+ L ∞
= p = α + β = − 9 + 2 = − 7 ...(ii) 2
Now,  x −
1  2 1 So, x 2 = 6 + x2 ⇒ x2 = 6 + x
We know that,  = x + 2  − 2
 x   x 
(α − β )2 = (α + β )2 − 4αβ ⇒ x2 − x− 6 = 0
= 2 − 2 = 0 [from Eq. (ii)]
= ( −7) − 4 × 12 = 49 − 48 = 1
2
1 ⇒ x 2 + 2x − 3x − 6 = 0
[from Eqs. (i) and (ii)] ∴ x− =0
⇒ α −β = 1 ...(iii)
x ⇒ x( x + 2) − 3( x + 2) = 0
From Eqs. (ii) and (iii), α = − 3 and 94. (b) Let the number of columns be x. ⇒ ( x − 3)( x + 2) = 0
β=−4 Then, number of rows = x + 4 ∴ x = 3, −2
which are correct roots. According to the question, 98. (d) Given, a x + b x + b = 0
2

90. (a) Let one root be α, then other roots is x( x + 4) × 5 − 2 = 158 b b


⇒ x2 + x + = 0
α2 ⇒ 5x( x + 4) = 160 ⇒ x( x + 4) = 32 a a
−b
Given equation is x − bx + c = 0
2 ⇒ x2 + 4x − 32 = 0 ∴ Sum of roots, α + β =
( −b) a
∴ Sum of roots = α + α 2 = − ⇒ x2 + 8x − 4x − 32 = 0
b
1 and products of roots, αβ =
⇒ x ( x + 8) − 4 ( x + 8) = 0 a
⇒ α(α + 1) = b ...(i)
c ⇒ ( x + 8)( x − 4) = 0 Hence, product of roots is not equal to
and product of roots = α ⋅ α 2 =
1 So, x = 4 as x = −8 is not possible. the sum of roots,
⇒ α 3 = c ⇒ α = c1 / 3 ...(ii) ∴ Number of rows So, statement I is not correct.
From Eqs. (i) and (ii), Now, for roots to be real and unequal,
= x+ 4 = 4+ 4 =8
D>0
1/3
(c1/3
+ 1) = b ...(iii)
95. (c) Given, x2 + 2 + 8 x +  + 14
c 1 1 ⇒ b 2 − 4 ac > 0
On cubing both sides, we get x  x ⇒ b − 4 a ( b) > 0
2
[Q c = b]
c ( c1 / 3 + 1)3 = b 3 1
Let x + = y ...(i) ⇒ b 2 − 4ab > 0 ⇒ b 2 > 4ab
x
⇒ c {c + 1 + 3 c 1/3
(c 1/3
+ 1)} = b 3
∴ b > 4a
On squaring both sides, we get
⇒ c ( c + 1 + 3b ) = b 3 [from Eq. (iii)] 2 So, if b > 4a, then roots are unequal and
∴  x + 1 = x 2 + 1 + 2 = y 2
⇒ b 3 = 3 bc + c 2 + c   real, so statement II is not always true, it
 x x2 depends on values of a and b.
91. (a) Let α and β be the roots of the given 1
⇒ x 2 + 2 = y2 − 2 ...(ii) 99. (d) Given, Ax2 + Bx + C = 0 ...(i)
quadratic equation 2x2 − 11x + 5 = 0. x
Since, the given roots are − 1 and 1 .
( −11) 11
∴ α+β=− = ...(i) Now, on putting values from Eqs. (i) and
∴ Sum of roots = − 1 + 1 = 0
2 2 (ii) in the given equation, we get
5 and product of roots = 1 × ( − 1) = − 1
and α ⋅β = ...(ii) y 2 − 2 + 8 y + 14 = y 2 + 8 y + 12 ⇒ quadratic equation is x2 −
2 (sum of roots) x + product of roots = 0
= y 2 + 2 y + 6 y + 12
Now, (α − β )2 = (α + β )2 − 4αβ ⇒ x2 − 0 ⋅ x − 1 = 0 ⇒ x2 − 1 = 0
2 = y ( y + 2) + 6 ( y + 2)
=   − 4  
11 5 On comparing with above equation
 2  2 = ( y + 6) ( y + 2)
from Eq. (i), we get
=  x + + 6  x + + 2
2 1 1
=
121 40 81
− = =  9 A = 1 and C = −1
     
4 4 4  2 x x
 1  So, A and C are of opposite sign.
9 So, one factor is  x + + 6 .
∴Difference of roots = (α − β ) = = 4 .5  x  100. (d) Given, ( a 2 − bc )x2 + 2( b 2 − ac )x
2
96. (b) Given, x2 − x − 1 = 0 + ( c 2 − ab ) = 0
92. (b) Let the two numbers be x and y. Since, the given roots are equal.
Now, by given condition, Here, a = 1, b = −1 and c = −1 ∴ D=0
x2 + y 2 = 97 …(i) Q α and β are the roots of the equation i.e. [ 2( b 2 − ac )] 2 − 4( a 2 − bc )
and ( x − y )2 = 25 …(ii) x2 − x − 1 = 0 ( c 2 − ab ) = 0
MATHEMATICS Quadratic Equations and Inequalities 181

⇒ 4( b 4 + a 2 c 2 − 2ab 2 c ) When mistake is done in constant term, II. From Eq. (ii), m 2 n2 = b 2
−4( a c − bc − a b + ab c ) = 0
2 2 3 3 2 then the roots of equation are 8 and 2. ⇒ b2 = d
⇒ 4 b + 4a c − 8ab c − 4a c
4 2 2 2 2 2 ∴ Equation is ( x − 2)( x − 8) Hence, statement II is correct.
+ 4bc 3 + 4a 3 b − 4ab 2 c = 0 = x2 − 10x + 16 ...(ii) 106. (a) If a is positive and b 2 − 4ac ≤ 0,
⇒ 4b − 12ab c + 4bc + 4a b = 0
4 2 3 3 From Eqs. (i) and (ii), we get then the sign of quadratic polynomial
x2 − 10x + 9 = 0 is the correct equation. ax2 + bx + c is always positive.
⇒ b 3 + c 3 + a 3 − 3abc = 0
100
∴ a 3 + b 3 + c 3 = 3abc 103. (c) Given equation, 107. (c) We have, x + > 50
x
101. (a) Given m and n are the roots of the 7( x + 2a )2 + 3a 2 = 5a ( 7x + 23a )
and 1 ≤ x ≤ 100 ⇒ x 2 − 50x + 100 > 0
given equation ax2 + bx + c = 0 ⇒ 7( x2 + 4a 2 + 4ax ) + 3a 2
50 ± 2500 − 400
∴ Sum of roots = m + n = − b /a …(i) = 35ax + 115a 2 ⇒ x =
c 2
and product of roots = mn = …(ii) ⇒ 7x − 7ax − 84a 2 = 0
2
a 50 ± 10 21
⇒ x2 − ax − 12a 2 = 0 =
m +1 n +1
2 2 2
Now, + ⇒ ( x + 3a )( x − 4a ) = 0
m n x < 25 − 5 21 or x > 25 + 5 21
m n + n + mn2 + m
2 ⇒ x = − 3a and x = 4a ⇒ x < 2. 087 or x > 47. 91
=
mn Since, m and n are the roots of the given ⇒ x = 1 and 2 or x = 48, 49,… ,100
mn (m + n) + (m + n) equation.
= ∴ Number of total values of
mn Let m = 4a and n = − 3a (Q m > n)
(m + n)(mn + 1) x = 2 + 53 = 55
= ∴ 3 m − n = 3 ( 4a ) − ( − 3a )
mn 108. (c) We have, x 2 − 4x − log N = 0
= 12 a + 3 a = 15 a 10
−  + 1
bc We know that, roots are real, if D ≥ 0
  − b( a + c ) 104. (a) Given equation is px2 + qx + r = 0.
= a ac = ⇒ 16 − 4( − log10 N ) ≥ 0
ac Let one root of the equation be α.
a Then, other root = 3α ⇒ 16 + 4 log N ≥ 0
10
[using Eqs. (i) and (ii)] q ⇒ log N ≥ − 4
∴ Sum of roots = α + 3α = − 10
m 2 + 1 n2 + 1 p 1
and × −q −q ⇒ N ≥ 10 −4
⇒ N ≥
m n 10000
⇒ 4α = ⇒ α= ...(i)
(m 2 + 1)( n2 + 1) p 4p 1
= r Minimum value of N =
mn and product of roots = (α ) ⋅ (3α ) = 10000
m 2 n2 + n2 + m 2 + 1 p
=
 −q 109. (c) Let x = 4+ 4 − 4 + ...
2
mn r r
(mn)2 + (m + n)2 − 2mn + 1 ⇒ 3α =
2
⇒ 3  =
= p  4p  p
mn ∴ x = 4+ 4−x
2 2 [from Eq. (i)]
 c  +  − b  − 2  c  + 1 3q 2
r ⇒ x2 = 4 + 4−x
 a  a   a ⇒ = ⇒ 3q 2 p = 16 p 2 r
= 16 p 2 p ⇒ ( x 2 − 4)2 = 4 − x
c
a ⇒ 3 q 2 = 16 pr ⇒ ( x 2 − 4)2 + x = 4 …(i)
c 2 + b 2 − 2ac + a 2 13 + 1
= 105. (b) I. Given, m and n are the roots of the For option (c), x =
ac equation 2
b 2 + ( a − c )2 From Eq. (i),
= x2 + ax + b = 0. 2
ac
∴ Sum of roots, m + n = − a ...(i)   13 + 1 2  13 + 1
We know that, quadratic equation can LHS =    − 4 +
be written as and product of roots, mn = b ...(ii)   2   2
x2 − (Sum of roots) x + Product of roots Also, given m 2 and n2 are the roots 2
 13 + 1 + 2 13 − 16  13 + 1
=0 of the equation =  +
− b( a + c )   4  2
⇒ x2 −  x − cx + d = 0.
2
 x
 ac  = 4 = RHS
∴ m 2 + n2 = c ...(iii)
 b 2 + ( a − c )2  110. (c) We have, −2 ≤ x ≤ 1
+   =0 and m 2 n2 = d ...(iv) …(i)
 ac  −1 ≤ y ≤ 2
On squaring both sides of Eq. (i), we
⇒ acx2 + b( a + c )x + b 2 + ( a − c )2 = 0 −2 ≤ − y ≤ 1 …(ii)
get Q
⇒ acx2 + ( ab + bc )x + b 2 + ( a − c )2 = 0 3≤z≤ 6
m 2 + n2 + 2mn = a 2 and
102. (c) When mistake is done in first degree [from Eqs. (i) and (ii)] 6 ≤ 2z ≤ 12 …(iii)
term, then the roots of the equation are ⇒ c + 2b = a 2 ⇒ c = a 2 − 2b On adding Eqs. (i), (ii) and (iii), we get
− 9 and − 1.
⇒ 2b − a 2 = − c −2 − 2 + 6 ≤ x − y + 2z ≤ 1 + 1 + 12
∴ Equation is ( x + 1) ( x + 9)
= x2 + 10x + 9 ...(i) Hence, statement I is not correct. 2 ≤ k ≤ 14
182 CDS Pathfinder

111. (a) Let α and β be the roots of equation 115. (b) We have, A( x ) = x 2 + px + q ⇒ 3x 2 − 7x − 30 − x 2 + 10x
ax 2 + bx + c = 0. Q ( x − m ) and ( x − km ) are factors of + 7x − 2x 2 − 20
−b A( x ), then m and km are roots of
Then, α+β= = ( 2x − 10) 2x 2 − 7x − 5
a A (x ) = 0
c ⇒ ( 10x − 50)
and αβ = ∴ m + km = − p ⇒ m ( k + 1) = − p
a −p
⇒ m= …(i) = ( 2x − 10) 2x 2 − 7x − 5
 1
2 2
( k + 1)
α + β =   +  
1
Now,
α β and m ⋅ km = q ⇒ m 2 k = q ⇒ 10( x − 5) = 2( x − 5) 2x 2 − 7x − 5
p2
β2 + α2 ⇒ ⋅k = q [from Eq. (i)]
⇒ α+β= ( k + 1) 2 ⇒ 2x 2 − 7x − 5 = 5 …(i)
(αβ ) 2
⇒ ( k + 1) q = kp
2 2 Again, on squaring both sides of Eq. (i),
(α + β )2 − 2αβ we get
116. (c) Given points are P (5, − 1), Q(3, − 2)
=
(αβ ) 2 and R ( 1, 1). 2x 2 − 7x − 5 = 25

− b b 2 / a 2 − 2c / a If points P,Q and R lie in the solution of ⇒ 2x 2 − 7x − 30 = 0 …(ii)


⇒ = inequations x + y ≤ 4 and x − y ≥ 2,
a c2 / a 2 If α and β are roots of Eq. (ii), then
then points P, Q and R satisfy these
inequations. −30
⇒ ab 2 − 2a 2 c = − bc 2 αβ = = − 15
For point P (5, − 1), 5 − 1 ≤ 4 [true] 2
⇒ ab + bc = 2a c
2 2 2
5 + 1 ≥ 2 [true] 119. (a) Since, the roots of equation
112. (d) Here, a = 1, b = − p , c = q For point Q (3, − 2), 3 − 2 ≤ 4 [true] lx 2 + mx + m = 0 are in the ratiop : q.
and a ′ = 1, b ′ = q , c ′ = − p 3 + 2 ≥ 2 [true] Then, we can take roots as pk and qk.
Applying common root condition, and for point R ( 1, 1), 1 + 1 ≤ 4 [true] ∴ Sum of roots = pk + qk = −
m
( a ′ c − ac ′ )2 = ( bc ′ − b ′ c )( ab ′ − a ′ b ) 1 − 1 ≥ 2 [false] l
m
⇒ ( q + p )2 = ( p 2 − q 2 ) ( q + p ) So, the point R ( 1, 1) does not satisfy the ⇒ ( p + q) k = − …(i)
l
inequations.
⇒ ( p + q) = ( p − q) ( p + q) and product of roots
Hence, points P and Q lie in the solution m
⇒ p−q=1 = ( pq ) k 2 = …(ii)
of inequations. l
⇒ p − q − 1= 0 a + 2b + a − 2b
117. (a) Given, x = On dividing Eq. (i) by Eq. (ii), we get
113. (c) Let α and α be the roots of equation
2
a + 2b − a − 2b ( p + q )k
By rationalising, we get =−1
x 2 + px + q = 0. ( pq ) k 2
Then, α ⋅ α2 = q ( a + 2b )2 + ( a − 2b )2
 p + q m/l
∴ α = ( q) 1/ 3 …(i) + 2 ( a + 2b )( a − 2b ) ⇒   =−k=
 pq  ( p + q)
x =
and α + α2 = − p ( a + 2b )2 − ( a − 2b )2 [using Eq. (i)]
∴ ( q) 1/3
+ ( q) 2 /3
=−p …(ii) ( p + q )2 m p+ q m
a + 2b + a − 2b + 2 a 2 − 4b 2 ⇒ = ⇒ =±
⇒x =
On cubing both sides, we get a + 2b − a + 2b pq l pq l
2a + 2 a 2 − 4b 2 p+ q m
q + q 2 + 3 q ( q1 / 3 + q 2 / 3 ) = − p 3 ⇒ =−
⇒x = pq l
⇒ q + q + 3 q ( − p) = − p
2 3 4b
a+ a 2 − 4b 2 p q m
[from Eq. (ii)] ⇒x = ⇒ + + =0
2b q p l
⇒ q + q 2 + p 3 = 3 pq
⇒ bx 2 − ax + b = 0 120. (d) We have, 1 +
1 1
− 2 ≥0
⇒ p 3 + q 2 + q = 3 pq
x x
118. (a) Given, 3x 2 − 7x − 30
114. (a) Consider the line, x2 + x − 1
= ( x − 5) + 2x 2 − 7x − 5 ⇒ ≥0
2x + 6 y = 21 …(i) x2
The point (0, 0) does not satisfy Eq. (i) On squaring both sides, we get As, x 2 ≥ 0, then x 2 + x − 1 ≥ 0
but the point (0,0) satisfy the equation 3x − 7x − 30 = ( x − 5)
2 2 If x 2 + x − 1 = 0,
2x + 6 y ≤ 21 . + ( 2x 2 − 7x − 5) −1 ± 5
then x =
Now, consider the line, + 2( x − 5) 2x 2 − 7x − 5 2
5x − 2 y = 10 … (ii) ⇒ 3x 2 − 7x − 30 = x 2 + 25 − 10x   −1 + 5     −1 − 5  
∴ x −    x −   ≥ 0
The point (0, 0) does not satisfy   2    2 
+ 2x 2 − 7x − 5
Eq. (ii) but the point (0,0) satisfy the + ( 2x − 10) 2x − 7x − 52 −1 − 5 −1 + 5
⇒ x≤ or x ≥
equation 5x − 2 y ≤ 10. 2 2
19
MATHEMATICS Set Theory 183

SET THEORY
Usually (1-2) questions have been asked from this chapter. Generally questions are asked
from these topics more results on operations on set.

SET
A well defined collection of objects, is called a set. The objects in a set are called its members or
elements. Sets are usually denoted by the capital letters A, B, C, X, Y and Z etc. And the elements of a set
are denoted by small letters a, b, c etc. If x is an element of set A, we can write x ∈ A , which means that ‘x
belongs to A’ or that x is an element of A.
If x does not belong to A, we can write, x ∉ A . e.g.
• The collection of vowels in English alphabet is a set A containing five elements namely a, e, i, o and u,
where a ∈ A but b ∉ A.
• The collection of first four prime numbers is a set A containing the elements 2, 3, 5 and 7, where 3 ∈ A
but 1 ∉A.

Representation of Sets
Sets are generally represented by following two ways:
1. Roster or Tabular form or listing method In this method, all the elements of a set are listed, within
curly braces { } being separated by commas. e.g.
(i) If A is a set of first eight prime numbers, then A = { 2, 3, 5, 7, 11, 13, 17, 19}
(ii) If B is a set of squares of first five natural numbers, then B = {1, 4, 9, 16, 25}
(iii) If A is a set of vowels of English alphabets, then A = { a, e, i, o, u}
Note The order in which the elements are written in a set makes no difference and also the repeated elements are taken
only once each.

2. Set Builder form or Rule method In this method, instead of listing all the elements of a set, we write
the set by some special property or properties satisfied by all its elements and write it as.
(i) The set B of all even natural numbers can be written as
B = { x : x is a natural number and x = 2n for n ∈ N} or B = { x : x ∈ N, x = 2n, n ∈ N} and neads it as
‘B’ is the set consisting of all elements x such that x has the property of even natural number. The
symbol ‘:’ or ‘1’ stands or such that
(ii) The set A = { 3, 5, 7, 9, 11}, then it is represented as
A = { x : x = 2n + 1 where n ∈ N, n < 6}
184 CDS Pathfinder

(iii) The set A = {0, 1, 4, 9, 16, . . . } can be written as 7. Subsets Let A and B be two sets. If every element
A = { x : x = n 2 , n ∈ Z}, where Z is the set of of A is an element of B, then A is called a subset of B.
integers. If A is subset of B, then we can write A ⊆ B which is
read as ‘A is a subset of B’ or ‘A is contained in B’.
Types of Sets (i) Every set is a subset of itself i.e. A ⊆ A, B ⊆ B.
1. Empty set A set which does not contain any (ii) Empty set is a subset of every set i.e. φ ⊆ A.
element is called an empty set or null set or void set.
e.g. Let A = { 2, 4, 6} and B = {6, 4, 2, 8} .
It is denoted by φ or { }.
Then, A ⊆ B but B ⊆/ A i.e. A is a subset of B but B is
e.g. A = set of all odd numbers divisible by 2
not a subset of A.
and B = { x : x ∈ N and 5 < x < 6}
The sets which have atleast one element are called Note The total number of subsets of a finite set containing n
non void or non-empty set. elements is 2 n.

2. Singleton set A set consisting of a single element is 8. Super set If A is a subset of B, then we say that B is
called a singleton set. e.g.
superset of A and we write B ⊇ A .
(i) The set {5} is a singleton set. e.g. If A = {1, 2, 3, 4} and B = {1, 2, 3, 4, 5, 6} ,
(ii) { x | x ∈ W and x + 6 = 6} = {0}
then, B ⊇ A .
which is a singleton set.
9. Comparability of sets Two sets A and B are said to
3. Finite set A set which consists of a definite number
be comparable, if either A ⊂ B or B ⊂ A or A = B,
of elements, is called a finite set. Empty set is also a
finite set. e.g. otherwise A and B are said to be incomparable.
(i) The set {1, 2, 3, 4} is a finite set, because it contains e.g. Let A = {1, 2, 3}, B = {1, 2, 4, 6} and C = {1, 2, 4}
a definite number of elements i.e. only 4 elements. Since, A ⊆/ B and B ⊆/ A.
(ii) B = Set of vowels in English Alphabets So, A and B are incomparable but C ⊂ B and so
= {a, e, i, o, u}; B and C are comparable sets.
(iii) A = Set of even prime natural numbers. 10. Proper subset If A ⊆ B and A ≠ B, then A is called
⇒ A = { 2} a proper subset of B and we write A ⊂ B.
Note The number of distinct elements contained in a set A is e.g. If A = {1, 2, 3, 4, . . . }
called the cardinal number of A and is denoted as n( A ). and B = {. . . , −3, −2, −1, 0, 1, 2, 3, . . . . },
If A = { a, e, i , o, u}, then n ( A ) = 5.
then A ⊂ B.
4. Infinite set A set which consists of infinite number
Note The total number of proper subset of a finite set containing n
of elements is called an infinite set. It is represented elements is (2 n − 1).
by writing a few elements of the set followed by
e.g. 11. Universal set If there are some sets under
(i) Set of square of natural numbers is an infinite set, consideration, then there happens to be a set which is
because natural numbers are infinite and it can be a superset of each one of the given sets. Such a set is
represented as {1, 4, 9, 16, 25, ...}. known as the universal set and it is denoted by U.
(ii) Set of all points in a plane. e.g.
5. Equal sets Two sets A and B are said to be equal, if • If A = {1, 2, 3}, B = { 2, 3, 4, 5, 7 } and
they have exactly the same elements and we write C = { 2, 4, 6, 8}, then U = {1, 2, 3, 4, 5, 6, 7, 8} is a
A = B. Otherwise, two sets are said to be unequal universal set for A, B and C.
and we write A ≠ B.
• For the set of all integers, the universal set can be
e.g. Let A = {1, 2, 3, 4} and B = { 4, 3, 1, 2}, then A = B
the set of rational numbers or the set of real
because each element of A is in B and vice-versa.
numbers.
6. Equivalent sets Two sets A and B are equivalent, if 12. Power set The collection of all subsets of a set A, is
their cardinal numbers are same i.e. n ( A ) = n ( B) and called power set of A and it is denoted by P ( A ) . In
we write A ↔ B or A ~ B
P ( A ), every element is a set.
e.g. Let A = {1, 2, 3} and B = { 4, 5, 6},
e.g. Let A = {1, 2, 3}
then n ( A ) = n ( B)
A ↔ B or A ~ B Then, P ( A ) = { φ , {1}, { 2}, { 3}, {1, 2}, { 2, 3}, { 3,1}, {1, 2, 3}}
MATHEMATICS Set Theory 185

Thus, A ∪ B = { x : x ∈ A or x ∈ B}
Properties of Power Sets U
In the given Venn diagram A ∪ B is
1. Each element of a power set is a set.
denoted by the shaded region.
2. If set A has n elements, then P ( A ) has 2n elements. A B
e.g. If A = {1, 2, 3, 4}
3. If A is an empty set φ or { }, then P ( A ) has just one and B = {1 , 2, 3, 5 , 7}, then A ∪ B = {1, 2, 3, 4, 5, 7}
element i.e. P( A ) = { φ}.
4. If A ⊆ B, then P ( A ) ⊆ P ( B) . Note • x ∈ A ∪ B ⇔ x ∈ A or x ∈ B
• x ∉ A ∪ B ⇔ x ∉ A and x ∉ B • A ⊆ A ∪ B, B ⊆ A ∪ B.
EXAMPLE 1. The set S = {x ∈ N: x + 3 = 3} is a
a. null set b. singleton set 2. Intersection of sets Let A and B be any two sets.
c. infinite set d. None of these The intersection of A and B is the set of all those
elements which belongs to both A and B. It is
Sol. a. Given, S = {x ∈ N : x + 3 = 3} denoted by A ∩ B and read as A intersection B.
∴ S = {} Thus, A ∩ B = { x : x ∈ A and x ∈ B} U
So, S is a null set. A ∩ B is represented by the shaded
region in the given Venn diagram A B
EXAMPLE 2. If A = {x : x is an odd integer} and
B = {x : x 2 − 8x + 15 = 0 }. Then, which one of the e.g. If A = {1, 2, 3, 4} and
B = {1, 3, 7, 9}, then A ∩ B = {1, 3}.
following is correct. e 2013 I
a. A = B b. A ⊆ B c. B ⊆ A d. A ⊆ B C Note • x ∈ A ∩ B ⇔ x ∈ A and x ∈ B
• x ∉ A ∩ B ⇔ x ∉ A or x ∉ B • A ∩ B ⊆ A and A ∩ B ⊆ B
Sol. c. Given that, A = {x : x is an odd integer}
and B = {x : x2 − 8x + 15 = 0} = {x : x2 − 5x − 3x + 15 = 0} 3. Disjoint sets Two sets A and B U
= {x : x ( x − 5) − 3( x − 5) = 0} are said to be disjoint, if they have
no common element i.e. A B
= {x : ( x − 5)( x − 3) = 0} = {3, 5}
Since, B has two odd elements.
A ∩B=φ
∴ B⊆A e.g. If A = {1, 2, 4}, B = { 3, 5, 6},
then A ∩ B = φ
EXAMPLE 3. If A = {a, b }, then power set of A is So, A and B are disjoint sets.
a. {φ, a , b} b. {φ, a , b , A}
4. Difference of sets Let A and B be two sets. The
c. {φ, a , b , ab} d. {φ, { a }, {b }, A} difference of two sets is the set of all those elements
of A which do not belong to B. It is denoted by
Sol. d. A = {a, b}
A − B and read as A minus B.
The subsets of A are φ, {a}, {b} {a, b}.
∴ A − B = { x : x ∈ A and x ∉B}
∴ P ( A) = {φ , {a}, {b}, {a, b} = {φ , {a}, {b}, A}
U

VENN DIAGRAM A–B


‘Venn diagram’ is the diagrammetic A B
representation of various types of sets U
and operations on sets. The shaded part in the given Venn diagram
A represents A − B.
The universal set is usually
Similarly, the difference B − A is U
represented by a rectangular region
the set of all those elements of B
and other subsets of the universal set
that do not belong to A. B–A
are represented by circles inscribed in it. Each inscribed
circle represents a set (subset of universal set). ∴ B − A = { x : x ∈ B and x ∉ A } A B
The shaded part in the given Venn diagram
Operations on Sets represents B − A .
1. Union of Sets Let A and B be any two sets. The e.g.
union of A and B is the set of all the elements of A and • If A = { 2 , 4, 6, 8, 10} and B = { 2, 4, 6, 12, 14}, then
all the elements of B, the common elements being A − B = {8, 10} and B − A = {12, 14}.
taken only once. It is denoted by A ∪ B and read as ‘A • If A = {1, 2, 3} and B = {1, 2, 3, 4}, then A − B = φ.
union B.’
186 CDS Pathfinder

Note The sets A − B, A ∩ B and B − A are mutually disjoint sets i.e.


Symmetric Difference of Two Sets the intersection of any of these two sets is an empty set.
Let A and B be two sets. The U
symmetric difference of sets A and Laws of Algebra of Sets
B is the set ( A − B) ∪ ( B − A ) and is 1. Idempotent laws For any set A, we have
denoted by A ∆ B i.e. it is a set
A–B B–A (i) A ∪ A = A (ii) A ∩ A = A
consisting of all those members of A
which are not in B or all those 2. Identity Laws For any set A,
which are in B but not in A. (i) A ∪ φ = A (ii) A ∩ U = A
i.e. φ and U are identity elements for union and
Thus, A ∆ B = { x : x ∈ A but x ∉ B} ∪ { x : x ∈ B but x ∉ A }
intersection, respectively.
A ∆ B = ( A − B) ∪ ( B − A )
The shaded part in given Venn diagram represents 3. Commutative laws For any two sets A and B, we
A ∆ B. have
e.g. If A = {1, 2, 3, 4, 5, 6, 7, 8} and B = {1, 3, 5, 6, 7, 8, 9}, (i) A ∪ B = B ∪ A (ii) A ∩ B = B ∩ A
∴ A − B = { 2, 4} and B − A = {9}, then A ∆ B = { 2, 4, 9}. 4. Associative laws If A , B and C are any three sets,
then
Complement of a Set (i) ( A ∪ B) ∪ C = A ∪ ( B ∪ C )
Let U be the universal set and A be A’ U (ii) A ∩ ( B ∩ C ) = ( A ∩ B) ∩ C
any set such that A ⊂ U . Then, the 5. Distributive laws If A , B and C are any three sets,
complement of A with respect to U is A
then
the set of all those elements of U
(i) A ∪ ( B ∩ C ) = ( A ∪ B) ∩ ( A ∪ C )
which are not in A. It is denoted by
A ′ or A C or U − A (ii) A ∩ ( B ∪ C ) = ( A ∩ B) ∪ ( A ∩ C )
Thus, A ′ = { x : x ∈U and x ∉ A } 6. De-Morgan’s laws If A and B are any two sets,
then
The shaded portion in the above Venn diagram shows
the complement of set A. If A is a subset of the (i) ( A ∪ B) ′ = A ′ ∩ B ′ (ii) ( A ∩ B) ′ = A ′ ∪ B ′
universal set U, then its complement A ′ is also a subset
of U. e.g. More Results on
• Let U = {1, 2, 3, 4, 5, 6, 7 , 8, …}and A = {1, 3, 5, 7 , . . . } . Operations on Sets
Then, the complement of A is 1. If A , B and C are any three sets, then
A ′ = U − A = { 2, 4, 6, 8, . . . } (i) A − ( B ∩ C ) = ( A − B) ∪ ( A − C )
• If U = { a, b, c, d , e, f } and A = { a, e}, (ii) A − ( B ∪ C ) = ( A − B) ∩ ( A − C )
then A ′ = { b, c, d , f }. (iii) A ∩ ( B − C ) = ( A ∩ B) − ( A ∩ C )
EXAMPLE 4. If A = {0, 1 2, 3, 4}, B = {1, 2, 3}, (iv) A ∩ ( B ∆ C ) = ( A ∩ B) ∆ ( A ∩ C )
C = {3, 5, 7, 9}, then ( A − B) ∩ (B − C) is (v) ( A − B) ∪ ( B − A ) = ( A ∪ B) − ( A ∩ B)
a. {1, 2} b. { 0 , 4 } c. { 0 , 1, 2, 4 } d. φ 2. If A , B and C are finite sets and U be the finite
Sol. d. Given, A = { 0, 1, 2, 3, 4}, B = { 1, 2, 3} and C = {3, 5, 7, 9} universal set, then
(i) n ( A ∪ B) = n ( A ) + n ( B) − n ( A ∩ B)
∴ A − B = { 0, 4} and B − C = { 1, 2}
(ii) n ( A ∪ B) = n ( A ) + n ( B) ⇔ A , B are disjoint sets.
∴ ( A − B) ∩ ( B − C ) = φ
(iii) n ( A − B) = n ( A ) − n ( A ∩ B)
Properties of Operations on Sets i.e. n ( A − B) + n ( A ∩ B) = n ( A )
(iv) Number of elements which belong to exactly one
1. (A ′ )′ = A 2. A ∪ A ′ = U
of A or B
3. A ∩A′ = φ 4. φ ′ = U = n ( A ∆ B) = n ( A ) + n ( B) − 2n ( A ∩ B)
5. U′=φ 6. A − B = A ∩ B ′ (v) If A, B and C are finite sets, then
7. B−A = B∩ A′ 8. A − B ⊆ A n ( A ∪ B ∪ C ) = n ( A ) + n ( B) + n (C ) − n ( A ∩ B)
9. B−A ⊆ B 10. A − B ≠ B − A − n (B ∩ C) − n (A ∩ C) + n (A ∩ B ∩ C)
11. A − φ = A and A − A = φ 12. A ⊆ B ⇒ A − B = φ (vi) n ( A ′ ∪ B ′ ) = n (( A ∩ B) ′ ) = n (U ) − n ( A ∩ B)
13. A −B = B−A ⇔A = B (vii) n ( A ′ ∩ B ′ ) = n (( A ∪ B) ′ ) = n (U ) − n ( A ∪ B)
MATHEMATICS Set Theory 187

EXAMPLE 5. If a set A contains 60 elements and ∴ n (E ∪ H ) = n (E) + n (H ) − n (E ∩ H )


another set B contains 70 elements and there are 50 40 = n ( E ) + 22 − 12 ⇒ n ( E ) = 30
elements in common, then how many elements does Number of people who speak English only
A ∪ B contain? = n ( E ) − n ( E ∩ H ) = 30 − 12 = 18
a. 130 b. 100 c. 80 d. 70
Sol. c. Here, n ( A) = 60, n ( B) = 70 and n ( A ∩ B) = 50
Ordered Pairs
An ordered pair consists of two objects or elements in a
Now, n( A ∪ B) = n( A) + n( B) − n( A ∩ B)
given fixed order.
= 60 + 70 − 50 = 130 − 50 = 80
If A and B are two sets and a ∈ A , b ∈ B, then the
EXAMPLE 6. A group of 40 people who speaks either ordered pair of elements a and b is denoted by (a, b).
english or Hindi, out of these 12 speak both English The natural numbers and their squares can be
and Hindi and 22 speak Hindi. How many people represented by ordered pair in the following way.
speak only english not Hindi?
(1, 1), (2, 4), (3, 9), (4, 16), ....
a. 30 b. 10 c. 18 d. 28
Two ordered pairs (a, b) and (c, d) will be equal, if and
Sol. c. Let E be the set of students who speak English and H be the set only if a = c and b = d .
of students who speak Hindi.
Given, n ( E ∪ H ) = 40, n ( H ) = 22 and n ( E ∩ H ) = 12 Note { a, b} = { b, a} but ( a, b ) ≠ ( b, a ) .

PRACTICE EXERCISE
1. If A = { 5, 6, 7} and B = { 7, 8, 9}, then A ∪ B is 8. The number of non-empty proper subsets of set
equal to A = { 2, 5, 7, 10} is
(a) {5, 6, 7, 8, 9} (b) {7, 8, 9} (c) {5, 6, 7} (d) φ (a) 16 (b) 15 (c) 14 (d) 8

2. Given that A = { 2, 6, 8, 9}, B = { 7, 8, 9, 12}, then 9. Which of the following statements is false for the
B − A is equal to sets A, B and C, where
(a) {7, 8, 9, 12} (b) {7, 12} A = { x|x is letter of the word ‘BOWL’}
(c) {2, 6, 7, 8, 9, 12} (d) {2, 6, 8, 9, 12} B = { x|x is a letter of the word ‘ELBOW’}
3. If U is the universal set of all natural numbers C = { x|x is a letter of the word ‘BELLOW’}
and A = { 1, 2, 3, 4, 5}, then compute A ∩ U . (a) A ⊂ B (b) B ⊃ C
(a) {1, 2, 3, 4} (b) φ (c) {1, 2, 3, 4, 5} (d) U (c) B ≠ C (d) B is a proper subset of C

4. The set { 2, 4, 16, 256,... } can be represented as 10. If A and B are two sets, then A ∩ ( A ∪ B) equals
which one of the following? (a) A (b) B
n
(a) {x ∈ N| x = 2 2 , n ∈ N} (c) φ (d) None of these
n
(b) {x ∈ N| x = 2 2 , n = 0, 1, 2,...}
11. The smallest set B such that
(c) {x ∈ N| x = 2 4 n , n = 0, 1, 2,...}
B ∪ { 1, 2} = { 1, 2, 3, 5, 9} is
(d) {x ∈ N| x = 2 2 n , n = 0, 1, 2,...}
(a) {3, 5, 9} (b) {3, 5, 8}
5. If P and Q are any two sets and P ⊂ Q , then (c) {1, 2, 3} (d) None of these
(a) P ∩ Q = φ (b) P ′ ∩ Q = P (c) P ∩ Q = P (d) P ∩ Q = Q 12. If U = { 0, 1, 2, 3, 4, 5, 6, 7, 8, 9}, P = { 0, 1, 2, 3},
6. Which one of the following is a true statement? Q = { 2, 3, 4, 5}, and R = { 4, 5, 6}, then Q ′ ∩ ( P ∪ R )
(a) ( A − B) ∩ (B − A) = φ (b) ( A − B) ∩ (B − A) = A is equal to
(c) ( A − B) ∩ (B − A) = U (d) ( A − B) ∩ (B − A) = B (a) {0, 1, 6} (b) {0, 1, 2, 3, 4, 5}
7. If P and Q are any two sets, then P ∪ Q = P ∩ Q, if (c) {6, 7, 8, 9} (d) {2, 3}
(a) P is the empty set (b) Q is the empty set 13. If P is a non-empty set, then ( P ′ )′ is equal to
(c) Both P and Q are empty sets
(d) P and Q are non-empty sets (a) φ (b) U (c) U − P (d) P
188 CDS Pathfinder

A U U
14. The shaded region in the adjoining diagram is A
B B
U

(c) (d)
(a) A ∪ A ′ (b) U (c) A ′ (d) A ∩ A ′ 20. Which one of the following is a correct statement?
15. The shaded region in the adjoining diagram (a) φ ∈ φ (b) φ ∉P (φ) (c) φ = P (φ) (d) φ ∈ P (φ)

21. Let A =  3, π , 2 , , − 5, 3 + 7 . The subset of A


represents 2
U
 7 
containing all the elements from it which are
irrational numbers is
A B
(a) 3, , − 5 (b) 3, π, , − 5, 3 + 7 
2 2
(a) ( A ∩ B)′ (b) A ∪ B (c) A − B (d) A ∩ B  7   7 
(c) { π, 2 , 3 + 7} (d) {3, − 5}
16. The shaded portion in the following Venn
diagram represents 22. If U = { x : x ∈ N }, A = { x : x is an odd number },
then A′ is equal to
B (a) {x : x is an even number} (b) {x : x is an odd number}
(c) {x : x is a natural number} (d) {x : x is an integer}

A 23. If two sets are disjoint, then their intersection is


C (a) null set (b) singleton
(c) a infinite set (d) None of these
(a) A ∪ (B ∩ C ) (b) A ∪ (B ∪ C )
(c) ( A ∩ B) ∩ C (d) ( A ∩ B) ∪ C 24. If A and B are two non-empty sets, then
A − ( A − B) equals
17. The shaded region in the adjoining diagram is (a) B (b) A − B (c) A ∩ B (d) A ′ ∩ B′
25. Let two sets A and B have 2n and 4n elements
B respectively, where n is a natural number. What
can be the minimum number of elements in A ∪ B ?
A (a) 2n (b) 3n (c) 4n (d) 6n
C 26. If P = { x : x − 3x + 2 = 0}, Q = { x : x + 4x − 12 = 0},
2 2

then P − Q is
(a) A ∩ (B − C ) (b) A − (B ∪ C ) (a) {1, 2} (b) {2} (c) {1} (d) {4, 3}
(c) A ∩ (B ∪ C ) (d) A ∪ (B ∩ C ) 27. If A = {( 2 2n
− 3n − 1)| n ∈ N } and
18. In the Venn diagram below, shaded portion B = { 9 ( n − 1)| n ∈ N }, then which one of the
represents following is correct?
A B C U (a) A ⊂ B (b) A ⊂ A (c) A = B
(d) Neither A is a subset of B nor B is a subset of A
28. If A, B and C are any three sets, then
(a) A − (B ∪ C ) = ( A − B) ∪ ( A − C )
(b) A − (B ∪ C ) = ( A − B) − ( A − C )
(a) A ∪ B ∪ C (b) A ∩ B ∩ C (c) A − (B ∪ C ) = ( A − B) ∩ ( A − C )
(c) ( A ∩ B) ∪ (B ∩ C ) (d) A ∪ B ∩ C (d) A − (B ∪ C ) = ( A ∪ B) − ( A ∪ C )
19. The Venn diagram for A ∪ B when B ⊂ A is 29. If P and Q are two sets such that
U A B U n ( P ) = m , n (Q ) = n and n ( P ∩ Q ) = p, then
B
A n ( P ∪ Q ) is equal to
(a) m + n (b) m + n + p (c) m + n − p (d) m − n − p
30. A and B are two sets such that n ( A) = 17,
n( B) = 23, n ( A ∪ B) = 38. Then, n ( A ∩ B) is
(a) (b) (a) 40 (b) 78 (c) 2 (d) None of these
MATHEMATICS Set Theory 189

31. Let A = { x : x 2 − 6x + 8 = 0} and 42. In an examination, 52% candidates failed in


B = { x : 2x + 3x − 2 = 0}. Then, which one of the
2 English and 42% failed in Mathematics. If 17%
following is correct? candidates failed in both English and
(a) A ⊆ B (b) B ⊆ A
Mathematics, what percentage of candidates
passed in both the subjects?
(c) Neither A ⊆ B nor B ⊆ A (d) A = B
(a) 18% (b) 21% (c) 23% (d) 25%
32. If A = { 1, 2, 3, 4}, B = { 2, 3, 4, 5}, C = { 1, 3, 4, 5, 6, 7}, 43. State which of the following statements about
then A ∩ ( B ∪ C ) is equal to sets is/are true?
(a) {1, 2, 3, 4, 5, 6, 7} (b) {1, 2, 3, 4} I. Every subset of a finite set is finite.
(c) {1, 2, 3, 4, 5} (d) φ II. φ is a subset of {0}.
33. Let the set A and B be given by A = { 1, 2, 3, 4} Select the correct answer using the codes given
and B = { 2, 4, 6, 8, 10} and the universal set below
U = { 1, 2, 3, 4, 5, 6, 7, 8, 9, 10}, then ( A ∪ B)′ is (a) Only I (b) Only II
(a) {2, 4} (b) U (c) Both I and II (d) Neither I nor II
(c) {1, 3, 5, 6, 7, 8, 9, 10} (d) {5, 7, 9} 44. Which of the following is/are examples of empty
34. What is {[( A ∪ B)′ ∩ A]} − ( A − B) equal to? set?
I. A = { x : x + 3 = 3, x ∈ I }
(a) φ (b) A (c) B (d) B′
II. B = { x : x is a positive even integer and prime}
35. Which one of the following is a correct statement?
III. C = { x : x2 = 16, x is odd integer}
I. { a } ∈ {{ a }, { b}, { c} II. { a } ⊆ {{ a }, b, c}
Select the correct answer using the codes given
III. { a , b} ⊆ {{ a }, b, c} IV. a ⊆ {{ a }, b, c} below
(a) Only I (b) Only II (c) III and IV (d) Only IV (a) Only I (b) I and III (c) Only III (d) I, II and III
36. Which one of the following is an infinite set? 45. Consider the following statements :
(a) {x : x is a whole number less than or equal to 1000} I. A′ ∪ B = ( A ∩ B)′ II. (φ′ )′ = ∪
(b) {x : x is a natural number less than 1000}
III. A ∩ (B ∪ C ) = ( A ∩ B) ∪ ( A ∩ C )
(c) {x : x is a positive integer less than or equal to 1000}
(d) {x : x is an integer and less than 1000} Which of the statement(s) given above is/are
correct?
37. Which one of the following is not correct in (a) Only II (b) I and III (c) Only III (d) II and III
respect of the sets A and B ?
(a) If A ⊆ B, then B ∪ A = B 46. Let P = Set of all integral multiples of 3
(b) If A ⊆ B, then A ∩ ( A − B) = φ Q = Set of all integral multiples of 4
(c) If A ⊆ B, then B ∩ A = A R = Set of all integral multiples of 6
(d) If A ∩ B = φ, then either A = φ or B = φ Consider the following relations
↔ ↔ ↔
38. If R ∈ PQ , S ∉ PQ and S does not lie on PQ I. P ∪ Q = R II. P ⊂ R III. R ⊂ (P ∪ Q )
extended, then
↔ ↔ ↔ ↔ Which of the relation(s) given above is/are correct?
(a) PQ ∩ RS = φ (b) PQ ∩ RS = {R} (a) Only I (b) Only II (c) Only III (d) II and III
↔ ↔ ↔ ↔
(c) PQ ∩ RS = {S} (d) PQ ∩ RS = {P}
47. Consider the following statements:
39. In a group of 1000 people, there are 750 people I. Set of all points of a given line is a infinite set.
who can speak Hindi and 400 who can speak II. The set of all birds in a zoo is a infinite set.
English. How many can speak Hindi only? III. Good books in a school library is a set.
(a) 600 (b) 150 (c) 300 (d) 500
Which of the statement(s) given above is/are
40. In a committee 50 people speak French, 20 speak correct?
Spanish and 10 speak both Spanish and French. (a) Only I (b) Only II (c) Only III (d) All of these
How many speak atleast one of these two languages? 48. Given that the set A = { 0, 1, 2, 3}, which of the
(a) 60 (b) 50 (c) 30 (d) 70
following statements about A are true?
41. Every student in a class of 42 students, studies I. A is a finite set.
atleast one of the subjects, Mathematics, English II. A is a subset of the set of integers.
and Commerce, 14 students study Mathematics, III. {1, 2} is a proper subset of A.
20 Commerce and 24 English. 3 students study IV. A is the null set.
Mathematics and Commerce, 2 English and Select the correct answer using the codes given
Commerce and there is no student who studies below
all the three subjects. The number of students
(a) I, II and III (b) I and IV
who study Mathematics but not Commerce is
(c) I and III (d) All of these
(a) 4 (b) 3 (c) 12 (d) 11
190 CDS Pathfinder

49. State which of the sets given below are infinite set? 56. Consider the following in respect of the sets A
I. Set of all concentric circles. and B. e 2013 I
II. { x : x is a multiple of 2, x is an integer.} I. ( A ∩ B) ⊆ A II. ( A ∩ B) ⊆ B III. A ⊆ ( A ∪ B)
III. The set of lines which are parallel to X-axis. Which of the statement(s) given above is/are
IV. The set of positive integers greater than 100. correct?
(a) I and II (b) II and III (c) I and III (d) All of these
Select the correct answer using the codes given
below 57. In a school there are 30 teachers who teach
(a) I and II (b) II and III Mathematics or Physics. Of these teachers, 20
(c) Only I (d) All of these teach Mathematics and 15 teach Physics, 5 teach
both Mathematics and Physics. The number of
50. Which of the following sets are equivalent? teachers teaching only Mathematics is e 2013 I
I. A = {1, 2, 3, 4, 5}, B = { 7, 8, 9, 10, 11} (a) 5 (b) 10 (c) 15 (d) 20
II. A = { x, y, z } B = { p, q}
58. In a class of 110 students, x students take both
III. P = {2, 4, 6, 8}, R = { a , b, c, d } Mathematics and Statistics, x + 20 students take
IV. A = {36, 39, 42, 45} , B = {42, 39, 45, 36} Mathematics and x + 30 students take Statistics.
Select the correct answer using the codes given There are no students who take neither Mathematics
below nor Statistics. What is x equal to? e 2013 II
(a) I, II and IV (b) II, III and IV (a) 15 (b) 20 (c) 25 (d) 30
(c) I, III and IV (d) None of these 59. If A is a non-empty subset of a set E, then what
is E ∪ ( A ∩ φ ) − ( A − φ ) equal to? e 2014 I
Directions (Q. Nos. 51-52) Answer the questions (a) A (b) Complement of A
based on the following information. (c) φ (d) E
In a survey of 250 students, it was found that 150 play 60. If A and B are any two non-empty subsets of a
cricket, 100 play basketball and 120 play football, set E, then what is A ∪ ( A ∩ B) equal to? e 2014 I
further, 30 of them play both basketball and football, 50
(a) A ∩ B (b) A ∪ B (c) A (d) B
play both cricket and basketball and 60 play both cricket
and football. 61. Out of 105 students taking an examination
English and Mathematics, 80 students pass in
51. The maximum number of students who play all English, 75 students pass in Mathematics, 10
the three sports students fail in both the subjects. How many
(a) 20 (b) 10 (c) 15 (d) None of these students pass in only one subject? e 2014 I

52. If 5 students play none of the three sports then (a) 26 (b) 30 (c) 35 (d) 45
numbers of students who play at least two sports 62. Let A denotes the set of quadrilaterals having
(a) 100 (b) 110 (c) 120 (d) 130 two diagonals equal and bisecting each other.
Let B denotes the set of quadrilaterals having
diagonals bisecting each other at 90°. Then,
PREVIOUS YEARS’ QUESTIONS A ∩ B denotes e 2015 II

53. If A = {x : x is an even natural number}, B = {x : x (a) the set of parallelograms (b) the set of rhombuses
is a natural number and multiple of 5} and C = { (c) the set of squares (d) the set of rectangles
x : x is a natural number and multiple of 10}, 63. Let S be a set of first fourteen natural numbers.
then what is the value of A ∩ ( B ∪ C )? e 2012 I The possible number of pairs ( a , b), where a,
(a) {10, 20, 30,…} (b) {5, 10, 15, 20,…} b ∈ S and a ≠ b such that ab leaves remainder 1
(c) {2, 4, 6,…} (d) {20, 40, 60,…} when divided by 15, is e 2016 I
(a) 3 (b) 5 (c) 6 (d) None of these
54. Which one of the following is a null set? e 2012 II
(a) A = {x is a real number : x > 1 and x < 1} 64. In a gathering of 100 people, 70 of them can speak
(b) B = {x : x + 3 = 3} Hindi, 60 can speak English and 30 can speak
(c) C = {φ} French. Further, 30 of them can speak both Hindi
(d) D = {x is a real number : x ≥ 1 and x ≤ 1} and English, 20 can speak both Hindi and French.
If x is the number of people who can speak both
55. Let x ∈ {2, 3, 4} and y ∈ {4, 6, 9, 10}. If A be the
English and French, then which one of the following
set of all order pairs (x, y) such that x is a factor
is correct? (Assume that everyone can speak
of y. Then, how many elements does the set A
atleast one of the three languages) e 2016 I
contain? e 2012 II
(a) 12 (b) 10 (c) 7 (d) 6 (a) 10 < x ≤ 30 (b) 0 ≤ x < 8 (c) x = 9 (d) x = 8
MATHEMATICS Set Theory 191

ANSWERS
1 a 2 b 3 c 4 b 5 c 6 a 7 c 8 c 9 a 10 a
11 a 12 a 13 d 14 c 15 c 16 a 17 b 18 c 19 a 20 d
21 c 22 a 23 a 24 c 25 c 26 c 27 a 28 c 29 c 30 c
31 c 32 b 33 d 34 a 35 a 36 d 37 d 38 b 39 a 40 a
41 d 42 c 43 c 44 c 45 c 46 c 47 a 48 a 49 d 50 c
51 a 52 b 53 a 54 a 55 d 56 d 57 c 58 b 59 b 60 c
61 c 62 c 63 d 64 a

HINTS AND SOLUTIONS 


1. (a) A ∪ B = {5, 6, 7} ∪ { 7, 8, 9} 9. (a) Given, A = { B , O , W , L } 21. (c) As, π, 2 , 3 + 7 are irrational
= {5, 6, 7, 8, 9} B = {B, O, W , L, E } numbers.
So, required set is { π , 2 , 3 + 7 }.
2. (b) B − A = { 7, 8, 9, 12} − { 2, 6, 8, 9} C = {B, O, W , L, E }
22. (a) Here, U = { 1, 2, 3, 4, 5, ...}
= { 7, 12} ∴ A ⊂ B and B = C
and A = { 1, 3, 5, 7, 9, 11,… }
3. (c) Given, A = { 1, 2, 3, 4, 5} and A ⊆ U . 10. (a) Clearly, A ∩ ( A ∪ B ) = A
So, A ′ = U − A = { 2, 4, 6, 8, 10,… }
⇒A ∩ U = { 1, 2, 3, 4, 5} ∩ {U } 11. (a) B ∪ { 1, 2} = { 1, 2, 3, 5, 9} ∴ A′ = { x : x is an even number}
= { 1, 2, 3, 4, 5} ⇒ B = {3, 5, 9} is the required smallest 23. (a) Disjoint sets have no common
set element.
4. (b) Let A = { 2, 4, 16, 256,...}
0 12. (a) Here, ( P ∪ R) = { 0, 1, 2, 3, 4, 5, 6} e.g. A = { 1, 2, 3} , B = { 4, 5, 6}
for n = 0, 22 = 21 = 2
1 Q ′ = U − Q = { 0, 1, 6, 7, 8, 9} ⇒ A∩B=φ
for n = 1, 22 = 22 = 4
2 So, Q ′ ∩ ( P ∪ R) = { 0, 1, 6} 24. (c) A − ( A − B ) = A − ( A ∩ B ′)
for n = 2, 22 = 24 = 16 = A ∩ ( A ∩ B ′) ′ = A ∩ ( A ′ ∪ B )
13. (d) Let U = { 1, 2, 3, 4, 5, 6} and
Thus, = ( A ∩ A ′) ∪ ( A ∩ B )
n P = { 1, 3, 4}
A = { x ∈ N | x = 22 , n = 0, 1, 2,...}
Then, P ′ = U − P = { 2, 5, 6} = φ ∪ ( A ∩ B) = A ∩ B
5. (c) Q P ⊂ Q and ( P ′)′ = U − P ′ = { 1, 3, 4} = P 25. (c) Q n ( A ∩ B ) = 2n,
∴ n ( A ∪ B) = n ( A) + n ( B)
Q U 14. (c) The shaded region is U − A = A ′.
− n ( A ∩ B)
P 15. (c) The shaded region represents the
elements in A and not in B, so it = 2n + 4n − 2n = 4n
represents A − B. Hence, minimum number of elements
16. (a) We have, in A ∪ B is 4n.
(B ∩ C) A ∪ (B ∩ C) 26. (c) Here, P = { x: x2 − 3x + 2 = 0}
So, P ∩ Q = P ⇒ P = { x : ( x − 1) ( x − 2) = 0 }
6. (a) Let x ∈ A − B ⇒ x ∈ A and x ∉B B B
⇒ P = { x : x = 1, x = 2}
⇒ x ∉B − A A A ∴ P = { 1, 2} and
Also, x ∈ B − A ⇒ x ∈ B Q = { x : x2 + 4x − 12 = 0}
and x ∉ A ⇒ x ∉ A − B C C ⇒ Q = { x : ( x + 6) ( x − 2) = 0}
So, ( A − B ) ∩ ( B − A ) = φ ⇒ Q = { − 6, 2}
17. (b) The shaded region represents the
7. (c) Only if P and Q both are empty set or elements in A and not in B or C . So, P − Q = { 1, 2} − { − 6, 2} = { 1}
P and Q have same elements, then
P ∪ Q = P ∩ Q. ∴ A − (B ∪ C ) 27. (a) Given, A = {( 22 n − 3n − 1)| n ∈ N }
18. (c) Clearly, ( A ∩ B ) ∪ ( B ∩ C ) = { 0, 9, 54, 243, … }
8. (c) Number of elements in A = 4
19. (a) Clearly, (a) is correct. and B = { 9 ( n − 1) | n ∈ N }
∴ Total number of non-empty
20. (d) Since, P( φ) = { φ} = { 0, 9, 18, 27,… }
Proper subsets of A = 2 − 2 n

Hence, φ ∈ P( φ) From above, it is clear that A ⊂ B.


= 24 − 2 = 14
192 CDS Pathfinder

↔ ↔
28. (c) ∴ PQ ∩ RS = { R} n ( E ∪ M ) = n( E ) + n( M ) − n ( E ∩ M )
∪ ∪ = 52 + 42 − 17 = 77%
A B A B where R is point of intersection of the
straight lines PQ and RS. ∴ Total candidates passed = ( 100 − 77)
39. (a) Here, H = People, who can speak Hindi = 23%
(A–B)
A–(B ∪ C) C ∩(A–C) C E = People, who can speak English 43. (c) Clearly, both I and II are true.
Given, n ( H ∪ E ) = 1000, n ( H ) = 750, 44. (c) I. A = { 0} II. B = { 2 }
From the above two figures, shaded III. C = { } ; + 4 is not an odd integer
n ( E ) = 400
portion of
A − ( B ∪ C ) = ( A − B) ∩ ( A − C ) n (H ∪ E ) = n (H ) + n (E ) Here, only III is empty set.

29. (c) n ( P ∪ Q ) = n ( P ) + n (Q ) − n (H ∩ E ) 45. (c) By distributive law in sets


− n ( P ∩ Q) 1000 = 750 + 400 − n ( H ∩ E ) A ∩ ( B ∪ C ) = ( A ∩ B) ∪ ( A ∩ C )
∴ n ( P ∪ Q) = m + n − p ⇒ n ( H ∩ E ) = 1150 − 1000 = 150 So, only III is correct.
30. (c) n ( A ∪ B ) = n ( A ) + n ( B ) Number of people who can speak Hindi 46. (c) Here, P = {… − 6, − 3, 0, 3, 6,… },
− n ( A ∩ B) only Q = {… − 8, − 4, 0, 4, 8,… } and
∴ n ( A ∩ B) = n ( A) + n ( B) = n (H ) − n (H ∩ E ) R = {… − 12, − 6, 0, 6, 12,… }
− n ( A ∪ B ) = 17 + 23 − 38 = 2 = 750 − 150 = 600 I. P ∪ Q = {… − 8, − 6, − 4, − 3, 0,
31. (c) Given, A = { x : x2 − 6x + 8 = 0} 40. (a) Let S be the set of people who speak 3 , 4 , 6 , 8 ,… } ≠ R
⇒ A = { x :( x – 4)( x − 2)} spanish and F be the set of people who II. P ⊄ R as 3 ∈ P but 3 ∉R.
∴ A = { 4, 2} speak French III. R ⊂ ( P ∪ Q ) is true.
And B = { x : 2x 2 + 3x − 2 = 0} ∴ n ( S ) = 20, n ( F ) Hence, only statement III is correct.
⇒ B = { x:( 2x − 1)( x + 2)} = 50, n ( S ∩ F ) = 10 47. (a) I. There are infinite points lie on a line
B =  , − 2
1 ⇒ n (S ∪ F ) = n (S) + n (F ) segment so, it is an infinite set.

2  − n (S ∩ F ) II. Number of birds in zoo are
Hence, neither A ⊆ B nor B ⊆ A. = 20 + 50 − 10 = 60 countable so, it is a finite set.
32. (b) ( B ∪ C ) = { 1, 2, 3, 4, 5, 6, 7} 41. (d) Here, M = Students who study III. It is not a well-defined set.
∴ A ∩ ( B ∪ C ) = { 1, 2, 3, 4} Mathematics Hence, only I is correct.
∩ { 1, 2, 3, 4, 5, 6, 7} E = Students who study English 48. (a) I. A is a finite set.
= { 1, 2, 3, 4} C = Students who study Commerce II. As all elements of A are integers, so
∴ n ( M ∪ E ∪ C ) = 42 A is subset of integers.
33. (d) ( A ∪ B ) = { 1, 2, 3, 4, 6, 8, 10}
Also, n( M ) = 14, n( E ) = 24, n(C ) = 20, III. { 1, 2} is a proper subset of A.
( A ∪ B )′ = U − ( A ∪ B )
IV. A ≠ φ.
= { 1, 2, 3, 4, 5, 6, 7, 8, 9, 10} n ( M ∩ C ) = 3, n ( E ∩ C ) = 2
So, I, II and III are true.
− { 1, 2, 3, 4, 6, 8, 10} and n ( M ∩ E ∩ C ) = 0
49. (d) All sets are infinite set.
= {5, 7, 9} 14
) C(20)
M( 3 50. (c) Equivalent sets have same cardinal
34. (a) {( A ∪ B )′ ∩ A } − ( A − B )
0 numbers. Here, cardinal numbers of I,
= {(U − ( A ∪ B )) ∩ A } − ( A − B ) 2
III, IV sets are same.
= {(U ∩ A ) − {( A ∪ B ) ∩ A }}
E(24)
− ( A − B)
Solutions (Q. No. 51-52) Let C,
= { A − A} − ( A − B) = φ − ( A − B) = φ From the Venn diagram, it is clear that
B and F denotes the number of
number of students who study
35. (a) {a} is an element of {{ a },{ b },{ c }}, Mathematics but not Commerce students who play cricket,
subsets of {{ a }, b , c } are φ, {{ a }, { b }, { c }, basketball and football
{{ a }, b }, { b , c }, {{ a }, c }, {{ a }, b , c } = 14 − 3 = 11
respectively.
Hence, only statement I is correct. 42. (c) Let number of failed candidates in
Mathematics and English be M and E, Given, n (C ) = 150, n (B) = 100,
36. (d) {x : x is an integer and less than respectively. n (F) = 120, n (B ∩ F) = 30,
1000} = [..., 998,999] n (C ∩ B) = 50 and n (C ∩ F) = 60
Given, candidates failed in English,
i.e. x ∈ ( − ∞ , 1000) is an infinite set. n( E ) = 52%, Let x be the number of students
37. (d) If A ∩ B = φ, then it is not necessary Candidates failed in Mathematics, who play all the three sports
that either. n( M ) = 42% B (100)
A = φ or B = φ. )C
Candidates failed in both English and 50
↔ ↔ (1
38. (b) Since R ∈ PQ i.e R belongs to PQ. Mathematics, x
↔ ↔
n ( E ∩ M ) = 17%
Again S , PQ and S does not lie on PQ
extended Q Total candidates, failed F (120)
MATHEMATICS Set Theory 193

51. (a) n (C ∪ B ∪ F ) = n (C ) + n ( B ) ∴A ∪ ( A ∩ B ) = A ∪ (Shaded portion)


U
+ n ( F ) − n (C ∩ F ) = A
− n (C ∩ B ) − n ( B ∩ F ) + n (C ∩ B ∩ F ) A B If A and B are two non-empty disjoint
⇒ n (C ∪ B ∪ F ) = 150 + 100
sets.
+ 120 − 60 − 50 − 30 + x
then, A ∩B = φ
⇒ n (C ∪ B ∪ F ) = 230 + x
Since, maximum value of n (C ∪ B ∪ F ) (A ∩ B) ∴ A ∪ ( A ∩ B) = A ∪ φ = A
is 250 I. ( A ∩ B ) ⊆ A [true] 61. (c) Let E and M represent the students
Hence, maximum value of II. ( A ∩ B ) ⊆ B [true] pass in English and Mathematics.
x = 250 − 230 = 20 III. A ⊆ ( A ∪ B ) [true] Given, n (U ) = 105, n( E ) = 80,
52. (b) Here, n(C ∪ B ∪ F ) = 250 − 5 = 245 So, all three statements are correct. n( M ) = 75, and n( E ∩ M ) = 10
Shaded region = ( A ∪ B )
∴ 245 = 230 + x ⇒ x = 15 Now, n( E ∪ M ) = n(U ) − n( E ∩ M )
57. (c) Let M and P be the teachers who = 105 − 10 = 95
B (100)
0 )C teach Mathematics and Physics,
(15 35 respectively. Also, n( E ∪ M ) = n( E ) + n( M )
15
45 15 Given, − n( E ∩ M )
n ( M ∪ P ) = 30, n ( M ) = 20, n ( P ) = 15 ⇒ 95 = 80 + 75 − n( E ∩ M )
F (120) and n ( M ∩ P ) = 5 ⇒ n( E ∩ M ) = 60
Number of students who play atleast ∴ Number of teachers teaching only The number of students who pass in
two sports = 15 + 35 + 45 + 15 = 110 Mathematics is only one subject
n ( only M ) = n ( M ) − n ( M ∩ P ) = n( E ) + n( M ) − 2 n ( E ∩ M )
53. (a) Given, A = { 2, 4, 6, 8, 10, 12, ...},
= 20 − 5 = 15 = 80 + 75 − 2 × 60
B = {5, 10, 15, 20, 25, 30, ...}
and C = { 10, 20, 30, 40, 50, 60, ...} 58. (b) Venn diagram of given conditions is = 155 − 120 = 35
as shown in figure given below
Now, 62. (c) A = diagonal equal and bisecting each
Students take Students take other.
( B ∪ C ) = {5, 10, 15, 20, 25, 30, ...} only Mathematics only Statistics
∴ A ∩ ( B ∪ C ) = { 2, 4, 6, 8, 10, 12, ...} A is square or rectangle. and B diagonal
bisecting each other at 90°.
∩ {5, 10, 15, 20, 25, 30, ...} So, A ∩ B = the set of squares.
x + 20 x x + 30
= { 10, 20, 30, ...} 63. (d) The possible set of pairs (a, b) such
54. (a) From option (a), A = {x is a real that ab leaves remainder 1 when divided
number : x > 1 and x < 1 }. Students take by 15 are (2, 8), (8, 2) (7, 13)
both Mathematics and Statistics
Since, there is no such element which is and (13, 7).
greater than 1 and less than 1. Total students = Students those take ∴ Number of possible set of pairs = 4
So, A is a null set. only Mathematics + Student those take
64. (a) Let A , B and C be the number of
From option (b), B = {x : x + 3 = 3} = {0} only Statistics + Student those take
people who can speak Hindi, English
= Singleton set both Mathematics and Statistics
and French.
From option (c),C = {φ} = Singleton set Q 110 = x + 20 + x + x + 30
Then, n ( A ) = 70, n ( B ) = 60,
From option (d), D = {x is a real number ⇒ 110 = 3x + 50 ⇒ 3x = 60
: x ≥ 1 and x ≤ 1} n(C ) = 30, n ( A ∩ B ) = 30,
∴ x = 20
= {1} = Singleton set n ( A ∩ C ) = 20, and n ( B ∩ C ) = x
59. (b) E ∪ {( A ∩ φ) − ( A − φ)} n ( A ∪ B ∪ C ) = n ( A ) + n ( B ) + n (C )
55. (d) Given that, x ∈ {2, 3, 4}
= E ∪ φ − A = E − A = A′ −n ( A ∩ B ) − n ( B ∩ C )
and y ∈ {4, 6, 9, 10}
[Q ( A ∩ φ) = φ and ( A − φ) = A] −n ( A ∩ C ) + n ( A ∩ B ∩ C )
and also A =( x, y ), such that x is a factor
of y. 60. (c) Since, A and B are non-empty subsets ⇒ 100 = 70 + 60 + 30 − 30 − 20
of E.
∴ A = {(2, 4), (2, 6), (2, 10), (3, 6), (3, 9), −x + n ( A ∩ B ∩ C )
(4, 4)} A B E ⇒ x = 10 + n ( A ∩ B ∩ C )
Hence, A contain 6 elements. Also, n ( A ∩ B ∩ C ) can’t be more than
56. (d) From figure, n ( A ∩C )
⇒ 10 ≤ x ≤ 30
20
194 CDS Pathfinder

MEASUREMENTS OF
ANGLES AND
TRIGONOMETRIC RATIOS
Generally (13-14) questions have been asked from this chapter. Generally questions asked from this
chapter are based on the trigonometric identities and formulae. So, detailed study of this chapter
will help you to score in exam as good number of questions have been asked from this chapter.

Trigonometry is the branch of Mathematics which deals with the measurements of sides and angles of
triangles and the problems based on these angles.

ANGLE
An angle is considered as the figure obtained by rotating a given ray about its
endpoint. The original ray is called the initial side and the ray into which the
initial side rotates is called terminal side. In figure, OA is initial side and OB is the
terminal side of ∠AOB. The point O is called its vertex.

QUADRANTS
Let XOX ′ and YOY ′ be two lines at right angles in the plane of paper. These
two perpendicular lines divide the plane of the paper into four equal parts, these
four parts are known as four quadrants and lines XOX ′ and YOY ′ are known as
X-axis and Y-axis, respectively.
The parts XOY , YOX ′, X ′ OY ′ and Y ′ OX are known as 1st, 2nd, 3rd and 4th
quadrant, respectively.

Relation between Angles, Radius and Arc Length


If l ’ is the length of a circle of radius r. And this arc subtends an angle θ radians
l
at the centre of the circle, then l = r θ ⇒ θ =
r
195

Relation between Degree and Radian Sol. a. I. We know that,


arc
Radius = (given, arc length is constant)
A circle subtends an angle, at the centre whose radian angle
measure is 2π and its degree measure is 360 °. It follows 1
∴ Radius ∝
that angle
2π radian = 360 ° So, angular measure in radian decreases, if the
π radian = 180 ° radius of the arc increases.
180 π
∴ 1 radian = degree = 57 ° 16′ 22′ ′ (approx) II. 1800°× = 10π
π 180°
π Hence, only statement I is correct.
1 degree = radian = 0 . 01746 rad (approx)
180
EXAMPLE 4. The angle between the hour hand and
Note 1 radian is written as 1c . the minute hand of a clock at half-past three is
a. 54° b. 63° c. 75° d. 85°
EXAMPLE 1. The radian measure corresponding to
− 22° 30′ is Sol. c. Angle traced by hour hand in 1 hr = 30°
7 °
Angle traced by hour hand in 3 hr =  30 ×  = 105°
1
πc πc πc πc 
a. b. − c. d. − 2 2
8 8 4 4
Angle traced by minute hand in 1 min = 6°
Sol. b. As, 180° = π Angle traced by minute hand in 30 min = 6 × 30° = 180°
45 ° π c πc Angle between two hands = 180°−105° = 75°
∴−22°30 ′ = −  22 +
1
 = −   = − 
45
×  =−
 2  2  2 180 8
TRIGONOMETRIC RATIOS
IMPORTANT POINTS
The ratio of the sides of a right
l The angle between two consecutive digits in a clock is angled triangle with respect to its
π
30 ° =  radian  . angle are called trigonometric ratios.
6 
The side opposite to the right angle is
l The hour hand rotates through an angle of 30° in one called the hypotenuse. Relative to the
1 °
hour i.e.   in one minute. angle θ, the side opposite to angle θ is
 2
caused the prependicular side and the remaining side is
l The minute hand rotates through an angle of 6° in one called base.
minute.
(i) For angle θ, perpendicular = BC, base = AB
EXAMPLE 2. If an arc 24π cm long of a circle and hypotenuse = AC
subtends an angle of 72° at its centre, then the (ii) For angle α, perpendicular = AB, base = BC
radius of the circle is
and hypotenuse = AC
a. 30 cm b. 40 cm c. 50 cm d. 60 cm
Trigonometric ratios of θ in right angled ∆ABC are
Sol. d. Given, length of arc ( l ) = ( 24π) cm defined below
π  c  2π  c Perpendicular BC P
Angle (θ) = 72° = 72 ×  =  sin θ = = =
 180  5 Hypotenuse AC H
Let r be the radius of circle Base AB B
l 24π cos θ = = =
Since, θ = ⇒ r= = = 60 cm
l
2π Hypotenuse AC H
r θ
5 Perpendicular
BC P
tan θ = ==
EXAMPLE 3. Consider the following statements: Base AB B
I. The angular measure in radian of a circular arc of HypotenuseAC H
sec θ = = =
fixed length subtending at its centre decreases, if Base AB B
the radius of the arc increases. Base AB B
II. 1800° is equal to 5π radian. cot θ = = =
Perpendicular BC P
Which of the statements given above is/are correct? e 2012 I
Hypotenuse AC H
a. Only I b. Only II cosec θ = = =
c. Both I and II d. Neither I nor II Perpendicular BC P
196 CDS Pathfinder

Relation Between T-ratios TRIGONOMETRIC IDENTITIES


1 1 1
(i) sin θ = (ii) cos θ = (iii) tan θ = A statement of equality involving trigonometric ratios of
cosec θ sec θ cot θ
an angle is called a trigonometric identity. It is valid for
sin θ cos θ all values of the angles. The three important identities
(iv) tan θ = (v) cot θ =
cos θ sin θ are
8 1. sin 2 θ + cos 2 θ = 1 2. 1 + tan 2 θ = sec 2 θ
EXAMPLE 5. If sin θ = , then the other
17 3. 1 + cot 2 θ = cosec 2 θ
trigonometric ratios cos θ, tan θ, cosec θ, sec θ, cot θ are
15 8 17 17 15 15 8 17 −17 8 Note sin2 A + cos 2 B cannot be equal to 1 because the angles are
a. , , , , b. − , , , , different.
17 15 8 15 8 17 15 8 15 15
17 15 8 15 8 tan A − sin A
c. , , , , d. None of these
15 8 17 17 15 EXAMPLE 7. What is the value of ?
sin 3 A e 2013 I
8 perpendicular
Sol. a. As, sin θ = = a.
sec A
b.
sec A
17 hypotenuse 1− cos A 1 + cos2 A
In right angled ∆PQR, sec A
c. d. None of these
∴ PQ2 + QR 2 = PR 2 ⇒ PQ2 = PR 2 − QR 2 1+ cos A
sin A
⇒ PQ = PR 2 − QR 2 = 172 – 82 − sin A
tan A − sin A cos A (1− cos A) (1+ cos A)
Sol. c. = = ×
= 289 − 64 = 225 = 15 sin3 A sin3 A cos A sin2 A (1+ cos A)
1− cos2 A sin2 A
PQ 15 RQ 8 = =
∴ cos θ = = , tan θ = = cos A sin A (1+ cos A) cos A sin2 A (1+ cos A)
2
PR 17 PQ 15
1 1 sec A
PR 17 PR 17 = ⋅ =
cosec θ = = , sec θ = = cos A 1+ cos A 1+ cos A
RQ 8 PQ 15
PQ 15 Values of Trigonometric Ratios for Some Specific Angles
and cot θ = =
RQ 8 Angles 0° 30° 45° 60° 90°
1 1 3
1 sin θ 0 1
EXAMPLE 6. If cot A = , then the value of 2 2 2
( 2 − 1) 1 1
3
sin A cos A is cos θ 1 0
2 2 2
1 1 1 2
a. b. c. d. 1
4 2 2 4 tan θ 0 1 3 ∞
3
1 base 1
Sol. d. Q cot A = = cot θ ∞ 3 1 0
2 − 1 perpendicular 3
2 −1  1  sec θ
2

⇒ tan A = Q tan A =  1 2 2
1  cot A 3
2
In right angled ∆ABC, cosec θ ∞ 2 2 1
3
∴ AC 2 = AB 2 + BC 2
⇒ AC = AB 2 + BC 2 EXAMPLE 8. What is the value of sin 3 60 °cot 30 °
= 12 + ( 2 − 1) 2 = 1 + 2 + 1 − 2 2 = 4 − 2 2 −2sec 2 45°+3cos 60 ° tan 2 45°− tan 2 60 ° ?
35 –35 −11 11
BC 2 −1 AB 1 a. b. c. d.
∴ sin A = = and cos A = = 8 8 8 8
AC 4 −2 2 AC 4 −2 2
Sol. b. sin360° cot 30°−2sec 2 45°+3cos 60° tan2 45°− tan2 60°
2 −1 1 2 −1
∴ sin A cos A = ⋅ =  3
3
1
4 −2 2 4−2 2 4−2 2 =   × 3 − 2 × ( 2) 2 + 3 × × (1) 2 − ( 3) 2
 2 2
2 −1 1 2 2 9 3 9 − 32 + 12 − 24 21− 56 −35
= = × = = −4+ −3= = =
2 2 ( 2 − 1) 2 2 2 4 8 2 8 8 8
197

π
Sign of trigonometric ratio in different Sol. a. Given, 0 < θ <
4
, then 1− 2sin θ cos θ
quadrants
= sin2 θ + cos2 θ − 2sin θ cos θ
Y
II Quadrant [Q sin2 θ + cos2 θ = 1]
(90°<θ<180°) sinθ I Quadrant (0<θ<90°) = (cos θ − sin θ) 2
and cosec θ are positive. All trigonometric
All other trigonometric ratios are positive. Q 0 < θ < π ,cos θ > sin θ , so we take (cosθ − sinθ) 2 
ratios are negative.  4 
X´ X = cos θ − sin θ
IV Quadrant
III Quadrant
(270°<θ < 360°) cos θ Domain and Range of Trigonometrical Functions
(180°<θ<270°) tan θ
and sec θ are positive.
and cot θ are positive. Trigonometric Domain Range
All other trigonometric
All other trigonometric Ratio
ratios are negative.
ratios are negative. sin θ R [− 1, 1 ]

cos θ R [− 1, 1 ]
π
π tan θ R ~ (2 n + 1) , n ∈ I  ( − ∞, ∞ ) = R
EXAMPLE 9. If 0 < θ < , then what is 1 − 2sinθ cos θ  2 
4 cot θ R ~ { n π, n ∈ I} ( − ∞, ∞ ) = R
equal to? e 2014 II
π
sec θ R ~ (2 n + 1) , n ∈ I  ( − ∞, − 1 ] ∪ [1, ∞ )
a. cosθ − sinθ b. sinθ − cosθ  2 
c. ± (cosθ − sinθ) d. cosθ sinθ cosec θ R ~ { n π, n ∈ I} ( − ∞, − 1 ] ∪ [1, ∞ )

The behaviour of the trigonometric function in different quadrants are defined in following table.
Trigonometric Ist Quadrant IInd Quadrant III Quadrant IVth Quadrant
Ratio (θ increase from 0 to π /2) (θ increase from π /2 to π) (θ increase from π to 3 π /2) 3π
(θ increase from to 2 π)
2
sin θ Increases from 0 to 1 Decreases from 1 to 0 Decreases from 0 to –1 Increases from –1 to 0
cos θ Decreases from 1 to 0 Decreases from 0 to –1 Increases from –1 to 0 Increases from 0 to 1
tan θ Increases from 0 to ∞ Increases from – ∞ to 0 Increases from 0 to ∞ Increases from – ∞ to 0
cot θ Decreases from ∞ to 0 Decreases from 0 to – ∞ Decreases from ∞ to 0 Decreases from 0 to – ∞
sec θ Increases from 1 to ∞ Increases from – ∞ to –1 Decreases from –1 to – ∞ Decreases from ∞ to 1
cosec θ Decreases from ∞ to 1 Increases from 1 to ∞ Increases from – ∞ to –1 Decreases from –1 to – ∞

EXAMPLE 10. Which one of the following statements TRIGONOMETRIC RATIOS


is true in respect of the expression sin 31°+ sin 32° ?
a. Its value is 0 b. Its value is 1
OF ALLIED ANGLES
c. Its value is less than 1 d. Its value is greater than 1 Two angles are said to be allied when their sum or
1 difference is either zero or a multiple of 90 °.
Sol. d. We know that, sin 30° =
2 The angles like − θ, 90 ° ± θ, 180 ° ± θ, 360 ° ± θ etc., are
Value of sin increases 0° to 90° angles allied to the angle θ, if θ is measured in degrees.
∴ sin 31° > sin 30° and sin 32° > sin 30° Angle sin θ cos θ tan θ cot θ sec θ cosec θ
1 1
⇒ sin 31° > and sin 30° > −θ − sinθ cosθ − tanθ − cot θ sec θ − cosec θ
2 2
Adding both 90°− θ cosθ sinθ cot θ tanθ cosec θ sec θ
1 1 90°+ θ − sinθ − cot θ − tanθ − cosec θ sec θ
∴ sin 31°+ sin 31° ⇒ + cosθ
2 2
180°− θ sinθ − cosθ − tanθ − cot θ − sec θ cosec θ
⇒ sin 31°+ sin 32° > 1
180°+ θ − sinθ − cosθ tanθ cot θ − sec θ − cosec θ
IMPORTANT POINTS 270°− θ − cosθ − sinθ cot θ tanθ − cosec θ − sec θ

l | sin θ | ≤ 1 l| cos θ | ≤ 1 l sec θ ≥ 1 270°+ θ − cosθ sinθ − cot θ − tanθ cosec θ − sec θ
l sec θ ≤ −1 lcosec θ ≥ 1 l cosec θ ≤ −1 360°− θ − sinθ cosθ − tanθ − cot θ sec θ − cosec θ
l tan θ and cot θ can take any value. 360°+ θ sinθ cosθ tanθ cot θ sec θ cosec θ
198 CDS Pathfinder

EXAMPLE 11. What is the value of ( A − B) ( A + B)


• sin A − sin B = 2 sin ⋅ cos
sec(90 °−θ)sinθ sec 45°? 2 2
3 ( A + B) ( A − B)
a. 1 b. c. 2 d. 3 • cos A + cos B = 2 cos ⋅ cos
2 2 2
Sol. c. Given, sec (90°− θ) sin θ sec 45° = cosec θ × sin θ × 2 ( A + B) ( A − B)
• cos A − cos B = − 2 sin ⋅ sin
1 2 2
= × sin θ × 2 = 2
sin θ ( A + B) (B − A )
= 2 sin ⋅ sin
2 2
sin 135° − cos 120 °
EXAMPLE 12. The value of is sin ( A + B)
sin 135° + cos 120 ° • tan A + tan B =
cos A cos B
a. 2 + 3 2 b. 2 − 3 2 c. 3 − 2 2 d. 3 + 2 2 sin ( A − B)
• tan A − tan B =
Sol. d. sin 135° = sin (180° − 45° ) = sin 45° =
1 cos A cos B
2 sin ( A + B)
1 • cot A + cot B =
cos 120° = cos (180° − 60° ) = − cos 60° = − sin A sin B
2
sin ( B − A )
1  1
− − 
• cot A − cot B =
sin 135° − cos 120°  2 2+1 2+1 sin A sin B
∴ = 2 = ×
sin 135° + cos 120° 1
+  − 
1 2 −1 2 +1
2  2
π
EXAMPLE 13. If A − B = , then the value of
( 2 + 1) 2 3
= = 2 + 1+ 2 2 = 3 + 2 2 cos A cos B + sin A sin B is
2−1
a. 1/2 b. 1
c. 3/2 d. None of these
Sum, Difference and Product Formulae
Sol. a. cos A cos B + sin A sin B = cos ( A − B)
• sin ( A + B) = sin A cos B + cos A sin B
π Q A − B = π 
= cos   =
1
• sin ( A − B) = sin A cos B − cos A sin B
 3 2  3
• cos ( A + B) = cos A cos B − sin A sin B
• cos ( A − B) = cos A cos B + sin A sin B 3 cos 23° –sin23°
EXAMPLE 14. The value of is
tan A + tan B 2
• tan ( A + B) =
1 − tan A tan B a. cos 7° b. sin 53° c. cos 53° d. sin 7°
tan A − tan B 3 cos 23° − sin 23°
• tan ( A − B) = Sol. c. =
3 1
cos 23°– sin 23°
1 + tan A tan B 2 2 2
cot A cot B − 1 = cos 30° cos 23°– sin 30° sin 23°
• cot ( A + B) =
cot A + cot B = cos ( 30° + 23° ) = cos 53°
cot A cot B + 1 [Q cos ( A + B) = cos A cos B – sin A sin B ]
• cot ( A − B) =
cot B − cot A tan 47 ° + tan 43°
EXAMPLE 15. The value of is
• sin ( A + B) sin ( A − B) = sin A − sin B
2 2
1 − tan 47 ° tan 43°
= cos 2 B − cos 2 A a. 1 b. ∞
• cos ( A + B) cos ( A − B) c. 0 d. −1
= cos 2 A − sin 2 B = cos 2 B − sin 2 A tan A + tan B tan 47° + tan 43°
Sol. b. Q = tan ( A + B) ⇒
• 2sin A cos B = sin ( A + B) + sin ( A − B) 1 − tan A tan B 1 − tan 47° tan 43°
• 2cos A sin B = sin ( A + B) − sin ( A − B) = tan ( 47° + 43° ) = tan 90° = ∞
2cos A cos B = cos ( A + B) + cos ( A − B)
EXAMPLE 16. The value of sin 20 °sin 40 °sin 60 °sin 80 °

• 2sin A sin B = cos ( A − B) − cos ( A + B) is
( A + B) ( A − B) 1 3 1 3
• sin A + sin B = 2 sin ⋅ cos a. b. c. d.
2 2 2 4 8 16
199

Sol. d. sin 20° sin 40° sin 80° sin 60° EXAMPLE 18. The value of 3 sin 15° − 4 sin 3 15° is
= sin 60°[sin 80° sin 40° ]sin 20° 1 1
a. 1 b. c. d. 0
3 2 2
= [sin 80° sin 40° ]sin 20°
2
Sol. c. As, 3 sin θ − 4 sin3 θ = sin 3 θ
3 1
= × [ 2sin 80° sin 40° ]sin 20° 1
2 2 So, 3 sin 15° − 4 sin3 15° = sin 3(15° ) = sin 45° =
2
3
= [cos(80°−40° ) − cos(80° + 40° )]sin 20°
4 Trigonometric Ratios of Submultiple Angles
3 θ θ
= [cos 40°− cos 120° ]sin 20° • sin θ = 2 sin cos
4 2 2
=
3
cos 40°−  −   sin 20°
1 2 θ θ θ θ
 2  • cos θ = cos − sin 2 = 2 cos 2 − 1 = 1 − 2 sin 2
4  2 2 2 2
Q cos 120° = − 1 1 − cos θ θ 1 + cos θ θ
 = tan = cot
2
• •
sin θ 2 sin θ 2
3 1 1 − cos θ 2 θ + θ θ
= cos 40°+  sin 20° • = tan 1 cos
= cot 2
4  2 1 + cos θ 2 •
1 − cos θ 2
3 θ θ
= [sin( 40°+20° ) − sin( 40°−20° ) + sin 20° ]
8 2 tan 1 − tan 2
• sin θ = 2 • cos θ = 2
=
3
[sin 60°− sin 20°+ sin 20° ] θ 2 θ
8 1 + tan 2
1 + tan
2 2
3 3 3 3
= × sin 60° = × = Maximum and Minimum Values
8 8 2 16
cos 4x + cos 3x + cos 2x of Trigonometrical Functions
EXAMPLE 17. is equal to We know that, − 1 ≤ sin θ ≤ 1, − 1 ≤ cos θ ≤ 1
sin 4x + sin 3x + sin 2x
Maximum and minimum value of a trigonometrical
a. cot 3x b. tan 3x c. cot x d. cot 2x
cos 4x + cos 3x + cos 2x (cos 4x + cos 2x) + cos 3x
function of the form a sin θ ± b cos θ are a 2 + b2
Sol. a. =
sin 4x + sin 3x + sin 2x (sin 4x + sin 2x) + sin 3x and − a 2 + b 2 , respectively.
 4x + 2x  4x − 2x
2cos  cos  + cos 3x
 2   2  EXAMPLE 19. The maximum value and minimum
= value of 8 sin θ cos θ + 4 cos 2θ is
 4x + 2x  4x − 2x
2sin  cos  + sin 3x
 2   2  a. 4 and −4 b. 16 and −16
2cos 3x ⋅ cos x + cos 3x cos 3x ( 2cos x + 1) c. 4 2 and −4 2 d. None of these
= = = cot 3x
2sin 3x ⋅ cos x + sin 3x sin 3x ( 2cos x + 1) Sol. c. Here, 8 sin θ cos θ + 4 cos 2θ = 4 sin 2θ + 4 cos 2θ
Trigonometric Ratios of Multiple Angles So, maximum value = 42 + 42 = 4 2
2 tan θ
• sin 2θ = 2 sin θ cos θ = and minimum value = − 42 + 42 = − 4 2
1 + tan 2 θ
• cos 2θ = cos 2 θ − sin 2 θ = 2 cos 2 θ − 1 EXAMPLE 20. Solve cos θ + sin θ = 2, then the value
1 − tan θ2 of θ is
= 1 − 2 sin 2 θ = a. π/3 b. π/4 c. π/6 d. π/2
1 + tan 2 θ
cos θ sin θ
1 + cos 2θ 1 − cos 2θ Sol. b. cos θ + sin θ = 2 , + =1
• = cos 2 θ and = sin 2 θ 2 2
2 2 (Q dividing throughout by 2)
2 tan θ
• tan 2θ = • sin 3 θ = 3sin θ − 4sin 3 θ ⇒ cos θ cos 45° + sin θ sin 45° = 1
1 − tan θ
2
cos (θ − 45° ) = 1 = cos 0°
cos 3θ = 4 cos 3 θ − 3 cos θ • tan 3θ = 3 tan θ − tan θ
3
• ∴ θ − 45° = 0 ⇒ θ = 45° = π /4
1 − 3 tan 2 θ
200 CDS Pathfinder

EXAMPLE 21. The value of φ for maximum value of 3. Cosine rule In a ∆ABC, if a, b, c be the sides
sin 3φ + cos 3φ is opposite to angles A, B and C respectively, then
a. 2 b. 90° c. 1 d. 15° b2 + c 2 − a 2
cos A = , cos B = c 2 + a 2 − b 2 and
Sol. d. sin 3φ + cos 3φ = 2  
1 1 2bc
cos 3φ + sin 3φ
 2 2 
a 2 + b2 − c 2
= 2 (sin 45° cos 3φ + cos 45° sin 3φ) cos C =
2ab
= 2 sin ( 45° + 3φ)
∴ The maximum value occurs when EXAMPLE 22. A triangle ABC is inscribed in a circle.
sin ( 45° + 3φ) = 1 = sin 90° If sum of the squares of sides of the triangle is equal
∴ 45° + 3φ = 90° ⇒ φ = 15° to twice the square of the diameter, then
sin 2 A + sin 2 B + sin 2 C is equal to
Properties of Triangle a. 2 b. 3 c. 4 d. None of these
1. Circumcircle of a triangle The A Sol. a. Let the radius of inscribed circle be R.
circle passing through the vertices a b c
c ∴ R= = =
of a ∆ABC is called circumcircle. 2sin A 2sin B 2sin C
Its radius R is called the B b
R a b c
⇒ sin A = , sin B = and sin C =
circumradius and its centre is a 2R 2R 2R
known as circumcentre. C a2 + b 2 + c 2
sin2 A + sin2 B + sin2 C =
a b c 4 R2
Here, R = = =
2sin A 2sin B 2sin C a2 b2 c2
⇒ sin2 A = 2 , sin B =
2
2 and sin C =
2
2. Sine rule In a ∆ABC, if a, b, c be A 4R 4R 4R 2
the three sides opposite to the According to the question,
angles A, B, C respectively, then
b c a2 + b2 + c2 = 2 × 2R 2 = 8R 2
a b c
= = 8R 2
sin A sin B sin C ∴ sin2 A + sin2 B + sin2 C = =2
4R 2
C a B

PRACTICE EXERCISE
1. A unit radian is approximately equal to 5. The angle between the minute hand and the
(a) 57° 17′ 43′′ (b) 57° 16′ 22′′ hour hand of a clock when the time is 7:35 pm is
(c) 57° 17′ 47′′ (d) 57° 17′ 49′′ (a) 16° 45′ (b) 17 ° 30′ (c) 18°15′ (d) 19° 30′
2. Find the radian measure corresponding to the 6. When do the hands of a clock coincide between
degree − 47° 30′. 5 pm and 6 pm?
− 19 π 17 π (a) 5:30 pm (b) 5:27:16 pm
(a) rad (b) rad
72 72 (c) 5:32:16 pm (d) 5:28:56 pm
13 π −15 π 1
(c) rad (d) rad
72 72 7. If sin A = , then cos A ⋅ cosec A + tan A ⋅ sec A is
3
3. The length of a pendulum is 60 cm. The angle equal to
through which it swings when its tip describes 16 2 + 3 4 2 + 3
(a) (b)
an arc of length 16.5 cm is 8 8
(a) 15° 30′ (b) 15° 45′ (c) 16°15′ (d) 16° 45′ 3+2 3 −1
(c) (d)
4. A railway train is moving on a circular curve of 4 8
radius 1500 m at a speed of 90 km/h. Through 8. The value of cos 15° − sin 15° is equal to
what angle has it turned in 11 seconds? 1 1 1 3
(a) 12° (b) 16° 30′ (c) 10° 30′ (d) 11° 40′ (a) (b) (c) (d)
3 2 2 2
MATHEMATICS Measurements of Angles and Trigonometric Ratios 201

1 1
1 − sin θ 22. If cosθ = 0.96, then + is equal to
9. If 3 tanθ = 4, then is equal to sin θ tan θ
1 + sin θ
(a) 0.98 (b) 3 (c) 4 (d) 7
1 2 1
(a) (b) (c) (d) None of these
2 3 3 23. If 0 < x < 45° and 45° < y < 90°, then which one of
the following is correct?
10. If tan A = 1 and tan B = 3, then
(a) sin x = sin y (b) sin x < sin y
cos A ⋅ cos B − sin A ⋅ sin B is equal to (c) sin x > sin y (d) sin x ≤ sin y
1+ 3 1− 3 2 2
(a) (b) (c) (d) 1 24. For what value of θ is (sin θ + cosec θ) = 2.5, where
2 2 2 2 3
0 < θ ≤ 90° ?
11. The value of cosec2 θ − 2 + sin2 θ is always (a) 30° (b) 45° (c) 60° (d) 90°
cos x sin x
(a) less than zero (b) non-negative 25. If = n and = m, then ( m 2 − n 2 ) sin2 y
(c) zero (d) 1 cos y sin y
is equal to
12. What is the value of
(a) 1 − n2 (b) 1 + n2 (c) m2 (d) n2
sin 15° + sin 20° + sin 25°+L + sin 75°?
2 2 2 2

13 11 26. If p = tan2 x + cot2 x, then which one of the


(a) 0 (b) (c) 6 (d) following is correct?
2 2
(a) p ≤ 2 (b) p ≥ 2 (c) p < 2 (d) p > 2
13. What is the value of
5 sin 75° sin 77° + 2 cos 13° cos 15° 7 sin 81° 27. The difference of the two angles in degree
− ? measure is 1 and their sum in circular measure
cos 15° sin 77° cos 9° is also 1. What are the angles in circular measure?
(a) − 1 (b) 0 (c) 1 (d) 2
π   1 + π 
(a)  − (b)  −
1 1 90   1 + 90 
, ,
14. What is log (tan 1° ) + log (tan 2° ) + log (tan 3° )  2 360   2 360  2 π 2 π
+ L + log (tan 89° ) equal to? π  1 π 
(c)  −
1
,  +  (d) None of these
(a) 0 (b) 1 (c) 2 (d) − 1  2 180   2 180 
1
15. If tan θ + = 2, then the value of 28. What is the value of [(1 − sin2 θ) sec2 θ + tan2 θ]
tan θ
(cos2 θ + 1) when 0 < θ < 90° ?
1
tan2 θ + is equal to (a) 2 (b) > 2 (c) ≥ 2 (d) < 2
tan2 θ
π
(a) 6 (b) 4 (c) 2 (d) 3 29. If 7 cos2 θ + 3 sin2 θ = 4 and 0 < θ < , then what is
13 2 sin θ − 3 cos θ the value of tan θ? 2
16. If sec θ = , then is equal to 7
5 4 sin θ − 9 cos θ (a) 7 (b) (c) 3 (d) 3
3
(a) 1 / 3 (b) − 3 (c) 3 (d) 3
17. sin ( 45° + A) − cos ( 45° − A) is equal to 30. If cos 1° = p and cos 89° = q, then which one of
(a) 1 / 2 (b) 1 (c) 0 (d) 2 the following is correct?
(a) p is close to 0 and q is close to 1
1 + sin θ (b) p< q
18. What is equal to?
1 − sin θ (c) p=q
(a) sec θ − tan θ (b) sec θ + tan θ (d) p is close to 1 and q is close to 0
(c) cosec θ + cot θ (d) cosec θ − cot θ π
31. If 0 ≤ θ < and p = sec2 θ, then which one of the
19. sin ( n + 1) A sin ( n − 1) A + cos ( n + 1) A cos ( n − 1) A 2
is equal to following is correct?
(a) sin 2 A (b) cos 2 A (c) tan 2 A (d) cot 2 A (a) p < 1 (b) p = 1 (c) p > 1 (d) p ≥ 1
cos 11° + sin 11° 32. What is the value of cos 1° cos 2° cos 3°… cos 90° ?
20. The value of is
cos 11° − sin 11° (a)
1
(b) 0 (c) 1 (d) 2
(a) tan 56° (b) tan 34° (c) cot 56° (d) tan 11° 2
1 − cos B 2 tan A
a 2 − b2 33. If tan A = , then what is equal
21. If sinθ = , then tanθ is equal to sin B 1 − tan2 A
a 2 + b2
to?
a2 + b 2 2 ab a2 − b 2 ab tan B
(a) (b) (c) (d) (a) (b) 2 tanB (c) tan B (d) 4 tan B
2 ab a2 − b 2 2 ab a2 + b 2 2
202 CDS Pathfinder

34. What is cot 15° cot 20° cot 70° cot 75° equal to? 5 1
45. If tan A = and tan B = , then A + B is equal to
(a) − 1 (b) 0 (c) 1 (d) 2 6 11
π π 3π
sin θ cos θ (a) (b) (c) (d) π
35. If + = 2 with 0 < θ < 90°, then what is θ 2 4 2
cos θ sin θ
1
equal to? 46. The value of (cos A − sin A) is
(a) 30° (b) 45° (c) 60° (d) 75° 2
π π
(a) cos  + A (b) cos  + A
36. In a right ∆ ABC, right angled at B, the ratio of 3  2 
2 tan A  π   π 
AB to AC is 1 : 2, then is equal to (c) cos  + A (d) sin  + A
1 − tan2 A 4  4 
(a) 2 (b) 1 (c) 3 (d) undefined π π π π
47. sin cos − cos sin is equal to
37. In figure, AD = DB, ∠B = 90°, then cosθ is equal to 4 12 4 12
1 3 1
A (a) (b) 3 (c) (d)
2 2 2
x sin φ y sin θ x
48. If tan θ = and tan φ = , then
b 1 − x cos φ 1 − y cos θ y
Da
is equal to
sin φ sin φ sin θ sin θ
(a) (b) (c) (d)
θ B sin θ cos θ sin φ 1 − cos φ
C

2 b 2 − a2
49. If sin (θ + φ) = 2 sin (θ − φ), then
a
(a) (b) (a) cot φ = 3 tan θ (b) tan θ = 3 tan φ
4b 2 − 3 a2 4b 2 − 3 a2
(c) sin θ = 3 sin φ (d) sin φ = sin 2 θ
4b 2 − 3 a2
(c) (d) None of these 50. If sin (θ + α ) = cos (θ + α ), then tan θ is equal to
2 1 − tan α tan α + 1 1 − cot α sin2(θ + α )
(a) (b) (c) (d)
38. If 0° ≤ x ≤ 90° and sin x + 3 cos x = 1, then what 1 + tan α tan α − 1 1 + cot α cos (θ + α )
is the value of x? θ−φ θ+φ
(a) 30° (b) 45° (c) 60° (d) 90° 51. cos2 − sin2 is equal to
2 2
39. If tan A = 2 − 1, then the value of cosec A ⋅ sec A (a) cos φ sin θ (b) cos 2θ sin φ
is equal to (c) cos θ cos φ (d) sin θ sin φ

(a)
1
(b)
2
(c) 2 2 (d)
3 52. If x = r sin θ cos φ, y = r sin θ sin φ, z = r cos θ, then
2 2 2 x 2 + y 2 + z 2 is equal to
40. tan θ = 3 and θ lies in third quadrant, then the (a) r 2 cos 2 φ (b) r 2 sin2 θ + r 2 cos 2 φ
value of sinθ is (c) r 2 (d)
1
1 1 −3 3 r2
(a) (b) − (c) (d)
10 10 10 10
7 3π
53. If cotθ = and π < θ < , then the value of
41. The value of (sin 20° cos 70° + cos 20° sin 70° ) is 24 2
(a) 1 (b) 0 (c) − 1 (d)
1 cos θ − sin θ is
2 19 18
(a) (b)
25 25
42. If φ and θ are supplementary angles, then
17 18
(a) sin θ = sin φ (b) cos θ = cos φ (c) (d)
25 25
(c) tan θ = tan φ (d) sec φ = cosec θ
π 1
43. If x + y = 90°, then what is the value of 54. If α + β = and sinα = , then β is
2 2
cos x cosec y − cos x sin y ? 1°
(a) 30° (b) 60° (c) 45° (d) 22
2
(a) cos x (b) sin x (c) cos x (d) sin x
55. If sin x − cos x = 0, then what is the value of
44. The value of 2 + 2 + 2 cos 4 θ is sin4 x + cos4 x ?
θ 3 1 1
(a) 2 cos 2 θ (b) 2 cos 2 θ (c) 2 cos θ (d) 2 cos (a) 1 (b) (c) (d)
2 4 2 4
MATHEMATICS Measurements of Angles and Trigonometric Ratios 203

56. If 3 cos x = 5 sin x , then the value of 67. cos 3 θ + sin 3 θ is maximum, when θ is
5 sin x − 2 sec x + 2 cos x
3 (a) 15° (b) 30° (c) 45° (d) 60°
is
5 sin x + 2 sec x − 2 cos x
3
68. If sin x + sin y + sin z = (sin x + sin y + sin z )2,
2 2 2

361 271 541 127 then which of the following expressions must
(a) (b) (c) (d)
2397 979 979 979 necessarily vanish?
(a) tan x + tan y + tan z (b) cos x + cos y + cos z
1 + cos x 1 1 1 1 1 1
57. is equal to (c) + + (d) + +
1 − cos x sin x sin y sin z cos x cos y cos z
(a) sec x + tan x (b) cosec x + cot x
(c) sec x − tan x (d) cosec x − cot x
69. If sin x + sin2 x = 1, then the value of
cos x + cos4 x is equal to
2

58. cos4 x − sin4 x is equal to (a) −1 (b) 2 (c) − 2 (d) 1


(a) 2 cos 2 x − 1 (b) 2 sin2 x − 1
70. cos 35° + cos 85° + cos 155° is
(c) sin2 θ − cos 2 θ (d) None of these 1 1
(a) 0 (b) (c) (d) cos 275°
sin θ 3 2
59. is equal to
1 + cos θ 71. sin 12° sin 24° sin 48° sin 84° is
1 − cos θ sin θ cos θ − 1 1 + sin θ 1 3 3
(a) (b) (c) (d) (a) (b) (c) (d) 0
sin θ cos θ sin θ cos θ 16 64 15
 5 cos θ − 4 3 + 5 sin θ  p
60. If 0 ≤ θ ≤ 90° , then  −  is 72. If tan θ = , then the value of p cos 2 θ + q sin 2 θ is
 3 − 5 sin θ 4 + 5 cos θ  q
(a) p (b) q
equal to
1 1 q (3q 2 − p2 ) p (3q 2 − p2 )
(a) 0 (b) 1 (c) (d) (c) (d)
4 2 p2 + q 2 p2 + q 2
4 5 2π 4π
61. If cos (α + β ) = and sin (α − β ) = , α , β lies 73. If x = y cos = z cos , then xy + yz + zx is
5 13 3 3
π equal to
between 0 and , then the value of tan 2α is
4 (a) 1 (b) − 1 (c) 0 (d) 2
56 56 43 34
(a)
33
(b)
23
(c)
33
(d)
33
74. If sin θ + cos θ = m and sec θ + cosec θ = n, then
tan ( 45° + x ) n ( m 2 − 1) is equal to
62. is equal to 1 m
tan ( 45° − x ) (a) (b) m (c) 2m (d)
2 2
2m 2
 1 + tan x   1 − tan x 
(a)   (b)   75. If cosec θ − sin θ = a3 , sec θ − cos θ = b3 , then
 1 − tan x   1 + tan x 
a 2b2 ( a 2 + b2 ) is equal to
x
2 sin2 (a) − 1 (b) 1 (c) 2 (d) − 2
(c) 2 (d) None of these
x 76. The maximum value and minimum value of
1 + cos 2
2 (1 + cos 2x ) are
−1 1
63. What is the simplest value of (a) − 1 and 1 (b) 1 and 2 (c) and (d) 0 and 2
(1 − sin A cos A) (sin A − cos A)
2 2 2 2
?
cos A (sec A − cosec A) (sin3 A + cos3 A) 77. If tan θ + tan φ = a and cot θ + cot φ = b, then
(a) sin A (b) cos A (c) sec A (d) cosec A cot (θ + φ ) is equal to
1 1 1 1
64. If A + B = 45°, then (1 + tan A) (1 + tan B) is equal (a) + (b) −
a b a b
to (c) a − b (d) a + b
(a) 1 (b) − 1 (c) 2 (d) − 2
78. tan 75° − tan 30° − tan 75° tan 30° is equal to
 cos 2B − cos 2 A
65.   is equal to (a) − 1 (b) 1 (c) 0 (d) 2
 sin 2B + sin 2 A
79. The value of sin (15° + A) − sin (15° − A) is equal
2 2
(a) tan ( A − B) (b) tan ( A + B) (c) cot ( A − B) (d) cot ( A + B)
to
66. sec2 x + tan2 x = 7, the value of x is (a)
1 1
cos 2 A (b) sin 2 A (c)
1
tan 2 A (d) cot 2 A
(a) 15° (b) 30° (c) 45° (d) 60° 2 2 2
204 CDS Pathfinder

sin 38° − cos 68° 91. What is the value of x in the equation
80. The value of is equal to
cos 68° + sin 38° cosec 2 30° sec2 45°
x = tan2 60° − tan2 30° ?
(a) 3 tan 8° (b) 3 cot 8° 8 cos2 45° sin2 60°
(c) 3 sin 38° (d) 3 sin 8° 1 3
(a) x = 1 (b) x = 2 (c) x = (d) x =
81. 2 cos x − cos 3x − cos 5x is equal to 2 2
(a) 8 cos 3 x sin2 x (b) 12 cos 2 x sin3 x 92. Under which one of the following conditions is
3
(c) 16 cos x sin x 2
(d) 32 cos x sin x 3 the trigonometrical identity
sin x/(1 + cos x ) = (1 − cos x )/ sin x true?
82. cos 4x is equal to (a) x is not a multiple of 360°
(a) 1 + 2 sin 2 x 2
(b) 2 cos 2 x 2 (b) x is not an odd multiple of 180°
(c) x is not a multiple of 180°
(c) 1 − 8 sin x cos x2 2
(d) 1 + 8 sin2 x cos 2 x
(d) None of the above
83. 2 sin A cos3 A − 2 sin3 A cos A is equal to 93. If cos x = k cos ( x − 2 y ), then tan ( x − y ) tan y is
1 1 1 1 equal to
(a) sin 4 A (b) cos 4 A (c) tan 4 A (d) cot 4 A
2 2 2 2 1+ k 1− k 2k k −1
(a) (b) (c) (d)
84. If a cos θ − b sin θ = c, then what is the value of 1− k 1+ k k+1 2k + 1
a sin θ + b cos θ ? sin ( x + y ) − 2 sin x + sin ( x − y )
94. The value of is
(a) ± a + b +c
2 2 2
(b) ± a − b + c
2 2 2 cos ( x + y ) − 2 cos x + cos ( x − y )
(a) cot x (b) tan x (c) sin x (d) cosec x
(c) ± a + b −c
2 2 2
(d) ± a2 − b 2 − c 2
95. If sin x + sin y = a and cos x + cos y = b, then
85. If cos x + cos2 x = 1, then the value of x+ y
tan is
sin x + 2 sin x + sin x is
8 6 4
2
(a) 0 (b) − 1 (c) 2 (d) 1 4 b a 4
(a) (b) (c) (d)
a + b
2 2
a b a − b2
2
86. If m = cosec x − sin x and n = sec x − cos x, then
tan x is equal to 96. The largest hand of a clock is 42 cm long, then
2/ 3 1/ 3 the distance covered by the extremity in 20 min
(a)   (b)  
n n
 m  m is
2 (a) 88 cm (b) 80 cm (c) 82 cm (d) 84 cm
(c)   (d)  
n n
 m  m 97. If 0° < x < 90° and 2 sin x + 15 cos x = 7, then 2

1 what is the value of tan x ?


87. If 2 cos θ = x + , then 2 cos 3 θ is equal to 2 3 3 4
x (a) (b) (c) (d)
3 2 4 3
1 1 x2
(a) x3 + (b) x2 + (c) (d) None of these
x 3
x 2
1+ x2 98. If ABC is a triangle and A + B + C = 180°, then
tan A + tan B + tan C is
1 A+ B B+ C
88. If cosθ ≥ in the first quadrant, then which one (a) tan A tan B tan C (b) tan + tan
2 2 2
of the following is correct? tan A + tan B
(c) (d) cot A cot B cot C
π π π π tan C
(a) θ ≤ (b) θ ≥ (c) θ ≤ (d) θ ≥
3 3 6 6
3 12
99. If x = a sec θ cos φ, y = b sec θ sin φ and z = c tanθ,
89. If sin A = and cos B = , then the value of x2 y2 z2
5 13 then the value of + − is
tan A − tan B a2 b2 c2
is equal to
1 + tan A tan B (a) 9 (b) 0 (c) 1 (d) 4
23 16 1 13 100. Which one of the following statements is correct?
(a) (b) (c) (d)
16 63 63 63
(a) The squares of the tangents of the angles 30°, 45°,
90. If B + C = 60°, then which of the following is 60° are in G.P.
correct statement? (b) The squares of the sines of the angles 30°, 45°, 60°
are in G.P.
(a) sin (120° − B) = sin (120° − C )
(c) The squares of the secants of the angles 30°, 45°,
(b) sin (120° + B) = sin (180° + C ) 60° are in A.P.
(c) cos (120° − B) = sin (120° + C ) (d) The squares of the tangents of the angles 30°, 45°,
(d) tan (B) = tan (120° + C ) 60° are in A.P.
MATHEMATICS Measurements of Angles and Trigonometric Ratios 205

101. If A, B, C and D are the successive angles of a π π


110. If A = and B = , then which of the following
cyclic quadrilateral, then what is 6 3
cos A + cos B + cos C + cos D are equal to? is/are correct?
(a) 4 (b) 2 (c) 1 (d) 0 I. sin A + sin B = cos A + cos B
II. tan A + tan B = cot A + cot B
102. Which one of the following is correct?
(a) There is only one θ with 0° < θ < 90° such that Select the correct answer using the codes given
sin θ = a, where a is a real number below
(b) There is more than one θ with 0° < θ < 90° such that (a) Only I
sin θ = a, where a is a real number (b) Only II
(c) There is no θ with 0° < θ < 90° such that sin θ = a, (c) Both I and II
where a is a real number
(d) Neither I nor II
(d) There are exactly two θ ’s with 0° < θ 90° such that
sin θ = a, where a is a real number 111. For any quadrilateral ABCD which of the
103. If sin ( B + C − A) = cos (C + A − B) = tan ( A + B − C ) following statements are true?
= 1, then the angles A, B, C which are positive I. sin ( A + B) + sin (C + D ) = 0
acute angles are, respectively II. cos ( A + B) + cos (C + D ) = 0
1° 1°
(a) 45° , 80° , 105° (b) 22 , 67 , 45° Select the correct answer using the codes given
2 2 below
(c) 20°, 70°, 90° (d) 30°, 60°, 90° (a) Only I (b) Only II
104. If A, B, C and D be the angles of a cyclic (c) Both I and II (d) None of these
quadrilateral taken in order, then cos (180° + A) 112. Consider the following statements
+ cos (180° + B) + cos (180° + C ) + cos (180° + D ) is I. 1° in radian measure is less than 0.03 radians.
equal to
II. 1 radian in degree measure is greater than 45°.
(a) 2 (cos A + cos B) (b) 2 (cos A + cos D)
(c) 0 (d) 4 cos A Which of the above statement is/are correct?
(a) Only II (b) Only I
105. If ABC is a right angled triangle at C and having
(c) Neither I nor II (d) Both I of II
u units, v units and w units as the lengths of its
sides opposite to be vertices A, B and C 113. Consider the following statements
respectively, then what is tan A + tan B equal to? 1
I. sin θ ⋅ sin(60°+ θ ) ⋅ sin(60°− θ ) = sin 3 θ
u2 w2 4
(a) (b) 1 (c) u + v (d)
vw uv 1
II. cos θ ⋅ sin(30°+ θ ) ⋅ sin(30°− θ ) = cos 3 θ
4
106. The Earth takes 24 h to rotate about its own axis. 1
Through what angle will it turn in 4 h and 12 min? III. sin θ ⋅ cos(30°+ θ ) ⋅ cos(30°− θ ) = sin 3 θ
4
(a) 63° (b) 64° (c) 65° (d) 70°
Which of the above statement is/are correct?
107. Assume the Earth to be a sphere of radius R.
(a) Only I
What is the radius of the circle of latitude 40°S?
(a) R cos 40° (b) R sin 80° (b) Only II
(c) R sin 40° (d) R tan 40° (c) Only III
(d) All of the above
108. If sin 3 θ = cos (θ − 2° ), where 3θ and (θ − 2° ) are
acute angles, then what is the value of θ? 114. If ABC is a right angled triangle, then which of
(a) 22° (b) 23° the following statements are correct?
(c) 24° (d) 25° I. sin ( A + B) = sin C
109. Consider the following statements:  A + B C
II. sin   = cos
cos 2 θ − sin 2 θ  2  2
I. = cos 2 θ (1 + tan θ ) (1 − tan θ ) (A + B − C )
cos 2 θ + sin 2 θ III. tan = cot C
1 + sin θ 2
II. = (tan θ + sec θ )2 (A − B − C )
1 − sin θ IV. tan = − cot A
2
Which of the statement(s) given above is/are
Select the correct answer using the codes given
correct?
below
(a) Only I (b) Only II
(a) I and II (b) I, II and III
(c) Both I and II (d) Neither I nor II
(c) I, II and IV (d) All of these
206 CDS Pathfinder

Directions. Q. Nos. (115-117) Let sin( A + B) = 1 and Which of the above statement(s) is/are correct?
π
sin( A − B) = , where A , B ∈ 0, .
1
(a) Only I (b) Only II e 2012 I
2  2 
 (c) Both I and II (d) Neither I nor II

115. What is the value of A? 126. Consider the following statements


π π π π I. sin 2 1°+ cos 2 1° = 1
(a) (b) (c) (d)
6 3 4 8 II. sec2 33°− cot2 57° = cosec2 37°− tan 2 53°
116. What is the value of tan ( A + 2B) tan (2 A + B)? Which of the above statement(s) given above
(a) −1 (b) 0 (c) 1 (d) 2 is/are correct? e 2012 I
(a) Only I (b) Only II
117. What is the value of sin A − sin B ?2 2
(c) Both I and II (d) Neither I nor II
(a) 0 (b) 1 / 2 (c) 1 (d) 2
127. Consider the following statements:
Directions. Q. Nos. (118-120) ABC is an obtuse I. There is only one value of x in the first quadrant
triangle and tanA, tanB are the roots of the equation that satisfies sin x + cos x = 2.
3x 2 − 2 3x + 1 = 0. II. There is only one value of x in the first quadrant
that satisfies sin x − cos x = 0.
118. The measure of angle C is Which of the above statement(s) given above
2π 5π
(a) π / 3 (b) π / 2 (c) (d) is/are correct? e 2012 I
3 6 (a) Only I (b) Only II
(c) Both I and II (d) Neither I nor II
119. Find the value of sin C + cos C.
(a)
2
(b)
3+1
(c)
3 −1
(d) 3 +
1 128. If sin θ cos θ = 3 / 4, then sin4 θ + cos4 θ is equal
3+1 2 2 2 to e 2012 II
(a) 7/8 (b) 5/8 (c) 3/8 (d) 1/8
1 + tan2 C
120. If 2sin B = , then the value of B is
cosec2 C 129. If the angle θ is in the first quadrant and
tan θ = 3, then what is the value of (sin θ + cos θ )?
π π 2π π
(a) (b) (c) (d) e 2012 II
2 3 3 6 1 2 3 4
(a) (b) (c) (d)
10 10 10 10
PREVIOUS YEARS’ QUESTIONS 130. If 0° < θ < 90°, then all the trigonometric ratios
121. The expression sin2 x + cos2 x − 1 = 0 is satisfied can be obtained when e 2012 II
by how many values of x? e 2012 I (a) only sin θ is given
(a) Only one value of x (b) Two values of x (b) only cos θ is given
(c) Infinite values of x (d) No value of x (c) only tan θ is given
(d) any one of the six ratios is given
122. If 3 sin x + 5 cos x = 5, then what is the value of
( 3 cos x − 5 sin x ) ? e 2012 I
131. What is the value of sin A cos A tan A + cos A
(a) 0 (b) 2 (c) 3 (d) 5 sin A cot A ? e 2012 II
123. If p = a sin x + b cos x and q = a cos x − b sin x, then (a) sin2 A + cos 2 A (b) sin 2 A + tan 2 A
what is the value of p2 + q 2? e 2012 I (c) sin2 A + cot 2 A (d) cosec 2 A + cot 2 A
(a) a + b (b) ab (c) a2 + b 2 (d) a2 − b 2 sin θ 1 + cos θ
132. What is the value of + ?
124. If α and β are complementary angles, then what 1 + cos θ sin θ e 2012 II
1 (a) 2 cosec θ (b) 2 sec θ

 sin α cos α  2 (c) sec θ (d) cosec θ
is cosec α ⋅ cosec β  +  equal to?
 sin β cos β  e 2012 I 133. What is the value of sec2 D − tan2 D ? e 2013 I
(a) 0 (b) 1
(c) 2 (d) None of these (a) 1/2 (b) 2/3
(c) 1 (d) None of these
125. Consider the following statements
cot 30° + 1 134. If cos A + cos2 A = 1, then what is the value of
I. = 2(cos 30° + 1)
cot 30° − 1 2 (sin2 A + sin4 A)? e 2013 I
II. 2 sin 45° cos 45°− tan 45° cot 45° = 0 (a) 4 (b) 2 (c) 1 (d) 1/2
MATHEMATICS Measurements of Angles and Trigonometric Ratios 207

135. (1 − tan A)2 + (1 + tan A)2 + (1 − cot A)2 143. What is cosec ( 75° + θ) − sec (15° − θ) equal to?
+ (1 + cot A) is equal to
2
e 2013 I
e 2013 II
(a) 0 (b) 1 (c) 2 sin θ (d) 2cosθ
(a) sin2 A cos 2 A (b) sec 2 A cosec 2 A
2
(c) 2 sec A cosec A 2
(d) None of these 144. If 5 sin θ + 12 cos θ = 13, then what is
5 cos θ − 12 sin θ equal to? e 2013 II
1 + 2 sin θ cos θ (a) − 2 (b) − 1 (c) 0 (d) 1
136. If a 2 = , then what is the value of
1 − 2 sin θ cos θ 145. If sin θ − cos θ = 0, then what is sin4 θ + cos4 θ
a+1
? equal to? e 2013 II
a−1 e 2013 I 3 1 1
(a) 1 (b) (c) (d)
(a) sec θ (b) 1 (c) 0 (d) tan θ 4 2 4

x2 − y2 cos2( 45° + θ ) + cos2( 45° − θ )


137. If sin θ = , then which one of the following 146. What is equal to?
x2 + y 2 tan( 60° + θ ) tan( 30° − θ ) e 2013 II

is correct? e 2013 I (a) −1 (b) 0 (c) 1 (d) 2


2x y 2x y 147. If tan θ + sec θ = m, then what is sec θ equal to?
(a) cos θ = (b) cos θ =
x 2 − y2 x 2 + y2 e 2013 II
x− y x y (x − y) m2 − 1 m2 + 1 m+ 1 m2 + 1
(c) cos θ = (d) cos θ = (a) (b) (c) (d)
x 2 + y2 x 2 + y2 2m 2m m m
138. Consider the following statements for 0 ≤ θ ≤ 90°: 3
148. If α , β and γ are acute angles such that sinα = ,
I. The value of sin θ + cos θ is always greater 2
3
than 1. cosβ = and tan γ = 1, then what is α + β + γ
II. The value of tan θ + cot θ is always greater 2
than 1. equal to? e 2013 II
(a) 105° (b) 120°
Which of the statement(s) given above is/are
(c) 135° (d) 150°
correct?
(a) Only I (b) Only II e 2013 I (1 + sec θ − tan θ ) cos θ
149. What is equal to?
(c) Both I and II (d) Neither I nor II (1 + sec θ + tan θ ) (1 − sin θ ) e 2013 II
(a) 1 (b) 2 (c) tan θ (d) cot θ
Directions (Q. Nos. 139-141) Read the following
information carefully to answer the questions that 150. If ∆ABC is right angled at C, then what is
follow. cos( A + B) + sin( A + B) equal to? e 2013 II
The angles A, B, C and D of a quadrilateral ABCD are in 1
(a) 0 (b) (c) 1 (d) 2
the ratio 1 : 2 : 4 : 5. 2
151. Consider the following statements:
139. What is the value of cos ( A + B) ? e 2013 I
I. tan θ increases faster than sin θ as θ increases.
(a) 0 (b) 1/2
II. The value of sin θ + cos θ is always greater
(c) 1 (d) None of these
than 1.
140. What is the value of cosec (C − D + B)? e 2013 I Which of the statement(s) given above is/are
(a) 1 (b) 2 (c) 3 (d) 4 correct? e 2013 II

141. Consider the following statements: (a) Only I (b) Only II


(c) Both I and II (d) Neither I nor II
I. ABCD is a cyclic quadrilateral.
II. sin ( B − A ) = cos ( D − C ) 152. The value of cos 25° − sin 25° is e 2014 I
(a) positive but less than 1
Which of the statement(s) given above is/are
(b) positive but greater than 1
correct? e 2013 I
(c) negative
(a) Only I (b) Only II
(d) 0
(c) Both I and II (d) Neither I nor II
153. In a right angled ∆ABC, right angle at B, if
142. If sin θ + cos θ = 3, then what is tan θ + cot θ 4
cos A = , then what is sin C is equal to?
equal to? e 2013 II 5 e 2014 I
(a) 1 (b) 2 (c) 2 (d) 3 3 4 3 2
(a) (b) (c) (d)
5 5 4 5
208 CDS Pathfinder

154. If α and β are complementary angles, then what sec x


166. What is equal to?
is cos α cosec β − cos α sin β equal to? cot x + tan x e 2014 II
e 2014 I
(a) sinx (b) cos x (c) tanx (d) cot x
(a) sec β (b) cos α (c) sin α (d) − tan β
167. What is (1 + cot x − cosec x )(1 + tan x + sec x ) equal
155. If sec θ + tan θ = 2, then what is the value of
to? e 2014 II
sec θ? e 2014 I (a) 1 (b) 2 (c) sinx (d) cos x
3 5 5
(a) (b) 2 (c) (d)
2 2 4 168. What is (cosec x − sin x )(sec x − cos x )(tan x + cot x )
156. What is cos ec ( 75° + θ) − sec (15° − θ) equal to? e 2014 II
(a) sin x + cos x (b) sin x − cos x
− tan( 55° + θ ) + cot ( 35° − θ) equal to? e 2014 I
(c) 2 (d) 1
3
(a) − 1 (b) 0 (c) 1 (d) sin x − cos x + 1
2 169. What is equal to?
sin x + cos x − 1 e 2014 II
157. What is sin 25° sin 35° sec 65° sec 55° equal to? sin x − 1 sin x + 1 sin x − 1 sin x + 1
(a) (b) (c) (d)
e 2014 I cos x cos x cos x + 1 cos x + 1
1
(a) −1 (b) 0 (c) (d) 1
2 170. What is (sin2 x − cos2 x ) (1 − sin2 x cos2 x ) equal
2 cos θ − sin θ to? e 2014 II
158. If 2 cot θ = 3, then what is equal
2 cos θ + sin θ (a) sin4 x − cos 4 x (b) sin6 x − cos 6 x
to? e 2014 I (c) cos 8 x − sin8 x (d) sin8 x − cos 8 x
2 1 1 3
(a) (b) (c) (d)
3 3 2 4 171. What is (sin x cos y + cos x sin y )
1 (sin x cos y − cos x sin y ) equal to? e 2014 II
159. If sin θ cos θ = , then what is sin6 θ + cos6 θ equal (a) cos 2 x − cos 2 y (b) cos 2 x − sin2 y
2
to? e 2014 I (c) sin2 x − cos 2 y (d) sin2 x − sin2 y
1
(a) 1 (b) 2 (c) 3 (d)
4
172. If A + B + C = 180°, then cot A cot B + cot B cot C
+ cot C cot A is equal to e 2014 II
160. If cos x + sec x = 2 , then what cosn x + secn x equal (a) − 1 (b) 2 (c) π (d) 1
to, where n is a positive integer? e 2014 I
173. If sin x + cosec x = 2, then what is sin9 x + cosec9x
(a) 2 (b) 2 n − 2 (c) 2 n − 1 (d) 2 n
equal to? e 2014 II
161. If sin θ + 2 cos θ = 1, where 0 < θ < π / 2, then what (a) 2 (b) 18 (c) 512 (d) 1024
is 2 sin θ − cos θ equal to? e 2014 I
(a) − 1 (b) 1 / 2 (c) 2 (d) 1
174. If sin x + cos x = p and sin3 x + cos3 x = q, then
what is p3 − 3 p equal to e 2014 II
π
162. If tan 8 θ = cot 2 θ, where 0 < 8 θ < , then what is (a) 0 (b) −2q (c) 2q (d) 4q
2
the value of tan 5θ? e 2014 I 175. Consider the following statements e 2014 II
1
(a) (b) 1 (c) 3 (d) 0 I. sin 1° > sin 1 II. cos 1° < cos 1
3
Which of the statement(s) given above is/are
163. If sin ( A + B ) = 1, where 0 < B < 45°, then what is correct?
cos( A − B) equal to? e 2014 I (a) Only I (b) Only II
(a) sin 2B (b) sin B (c) cos 2B (d) cos B (c) Both I and II (d) Neither I nor II
164. At what point of time after 3 O’clock, hour hand 176. The value of cosec2 67°+ sec2 57°– cot2 33°– tan2 23°
and the minute hand of a clock occur at right is e 2015 I
angles for the first time? e 2014 I (a) 2 2 (b) 2 (c) 2 (d) 0
1
(a) 9 O’clock (b) 4 h 37 min
6 177. If tan( A + B) = 3 and tan A = 1, then tan( A – B)
8 8 is equal to e 2015 I
(c) 3 h 30 min (d) 3 h 32 min
11 11 1
(a) 0 (b) 1 (c) (d) 2
3
165. If tan θ + cot θ = 2, then what is sin θ + cos θ equal
to? e 2014 II 178. If tan A + cot A = 4, then tan4 A + cot4 A is equal
1 1 to e 2015 I
(a) (b) (c) 2 (d) 1
2 3 (a) 110 (b) 191 (c) 80 (d) 194
MATHEMATICS Measurements of Angles and Trigonometric Ratios 209

179. ABC is a triangle right angled at B and 188. If tan( x + 40)° tan( x + 20)° tan( 3x )°
AB : BC = 3 : 4. What is sin A + sin B + sin C equal tan( 70 − x )° tan( 50 − x )° = 1, then the value of x is
to? e 2015 I equal to e 2015 II
11 12
(a) 2 (b) (c) (d) 3 (a) 30 (b) 20 (c) 15 (d) 10
5 5
189. The value of
180. If sin x + cos x = C, then sin6 x + cos6 x is equal to  π  π  π
e 2015 I 32 cot2   − 8 sec2   + 8 cos3   is equal to
 4  3  6 e 2015 II
1 + 6 C 2 – 3C 4 1 + 6 C 2 – 3C 4
(a) (b) (a) 3 (b) 2 3 (c) 3 (d) 3 3
16 4
1 + 6 C 2 + 3C 4 1 + 6 C 2 + 3C 4 190. If x = a cosθ and y = b cotθ, then
(c) (d)
16 4 ( ax −1 − by −1 ) ( ax −1 + by −1 ) is equal to e 2015 II
2
3 – tan A (a) 0 (b) 1 (c) tan2 θ (d) sin2 θ
181. If = k, where k is a real number, then
1 – 3 tan2 A 191. If θ is an acute angle and
cosec A (3 sinA – 4 sin3 A) is equal to e 2015 I sin θ cos θ = 2 cos3 θ − 1.5 cos θ, then what is sinθ
2k 2k 1 equal to? e 2015 II
(a) (b) , where ≤ k ≤ 3
k–1 k–1 3 5 −1 1− 5 5+ 1 5+1
(a) (b) (c) (d) −
2k 1 2k 4 4 4 4
(c) , where k < or k > 3 (d)
k–1 3 k+1 192. A clock is started at noon by 10 minutes past 5,
1 – sin x 1 – sin x cos x through what angle, the hour hand moves?
182. If p = , q= , r= , then e 2015 II
1 + sin x cos x 1 + sin x
(a) 160° (b) 145° (c) 150° (d) 155°
which of the following is/are correct? e 2015 I
I. p = q = r II. p2 = qr 193. Consider the following statements e 2015 II
I. sin 66° is less than cos 66°.
Select the correct answer using the codes given
below II. sin 26° is less than cos 26°.
(a) Only I (b) Only II Which of the above statement(s) is/are correct?
(c) Both I and II (d) Neither I nor II (a) Only I (b) Only II
(c) Both I and II (d) Neither I nor II
183. Consider the following identity
cos A sin A 194. Consider the following statements.
I. + = sin A + cos A 2
1 – tan A 1 – cot A 1 + tan 2 θ  1 − tan θ 
I. =  is true for all
II. (1 – sin A – cos A )2 = 2(1 – sin A )(1 + cos A ) 1 + cot2 θ  1 − cot θ 
Which of the above identity/identities is/are π π
0 < θ < ,θ ≠
correct? 2 4
1
(a) Only I (b) Only II e 2015 I II. cot θ = is true for θ = 45° only.
(c) Both I and II (d) Neither I nor II tan θ
Which of the above statements is/are correct?
184. If a and b are positive, then the relation
2a + 3b e 2015 II
sinθ = is (a) Only I (b) Only II
3b e 2015 II
(c) Both I and II (d) Neither I nor II
(a) not possible (b) possible, if a = b
(c) possible, if a > b (d) possible, if a < b π
195. In a ∆ABC, if A − B = , then C + 2B is equal to
2
185. The minimum value of cos2 x + cos2 y − cos2 z is e 2016 I
2π 3π π
e 2015 II (a) (b) (c) π (d)
3 4 2
(a) − 1 (b) 0
2 2
(c) 2 (d) 2  sin 35°   cos 55° 
196.   −  + 2 sin 30° is equal to
186. If tan θ + sec θ = 2, then tanθ is equal to e 2015 II  cos 55°   sin 35°  e 2016 I
3 5 3 5 (a) −1 (b) 0 (c) 1 (d) 2
(a) (b) (c) (d)
4 4 2 2
4 π
cos θ  π 197. If tan θ + cot θ = , where 0< θ< , then
187. is equal to  where, θ ≠  3 2
1 − sin θ  2 e 2015 II sin θ + cos θ is equal to e 2016 I
tan θ − 1 1 + sin θ tan θ + 1 1 + cos θ 3 −1 3+1
(a) (b) (c) (d) (a) 1 (b) (c) (d) 2
tan θ + 1 cos θ tan θ − 1 sin θ 2 2
210 CDS Pathfinder

198. If p = cot θ + tan θ and q = sec θ − cos θ, then 1 + cos θ


II. = cosec θ + cot θ
( p q )2/ 3 − ( q 2 p)2/ 3 is equal to
2 1 − cos θ
e 2016 I
(a) 0 (b) 1 (c) 2 (d) 3 Which of the above is/are identity/ identities?
x y x y e 2016 I
199. If − tanθ = 1 and tan θ + = 1, then the (a) Only I (b) Only II
a b a b (c) Both I and II (d) Neither I nor II
x2 y2
value of 2 + 2 is
cos2 θ − 3cos θ + 2 π
a b e 2016 I 203. If = 1, where 0 < θ <
, then
(a) 2sec θ 2
(b) sec θ
2
(c) cos θ
2
(d) 2 cos θ 2
sin θ 2
2
which of the following statement(s) is/are correct?
200. Which of the following is correct in respect of the
equation 3 − tan2 θ = α(1 − 3tan2 θ )? I. There are two values of θ satisfying the above
(given that α is a real number.) e 2016 I equation.
II. θ = 60° is satisfied by the above equation.
(a) α∈  , 3 (b) α∈  −∞,  ∪ [3, ∞ )
1 1
 3   3  Select the correct answer using the codes given
(c) α∈  −∞,  ∪ [3, ∞ )
1 below e 2016 I
(d) None of these
 3 (a) Only I (b) Only II
201. A person goes to a market between 4 pm and (c) Both I and II (d) Neither I nor II
5 pm. When he comes back, he finds that the 204. Consider the following statements
hour hand and the minute hand of the clock I. There exists a positive real number m such that
have interchanged their positions. For how much cos x = 2m + 1.
time (approximately) was he out of his house?
II. mn ≥ m + n, for all m, n belonging to set of
e 2016 I
natural numbers.
(a) 55.38 min (b) 55.48 min (c) 55.57 min (d) 55.67 min
Which of the above statement(s) is/are correct?
202. Consider the following (a) Only I (b) Only II e 2016 I
1 − cos θ (c) Both I and II (d) Neither I nor II
I. = cosec θ − cot θ
1 + cos θ

ANSWERS
1 b 2 a 3 b 4 c 5 b 6 b 7 a 8 b 9 c 10 b
11 b 12 b 13 b 14 a 15 c 16 c 17 c 18 b 19 b 20 a
21 c 22 d 23 b 24 a 25 a 26 b 27 a 28 b 29 d 30 d
31 d 32 b 33 c 34 c 35 b 36 d 37 a 38 d 39 c 40 c
41 a 42 a 43 b 44 c 45 b 46 c 47 d 48 c 49 b 50 a
51 c 52 c 53 c 54 b 55 c 56 b 57 b 58 a 59 a 60 a
61 a 62 a 63 a 64 c 65 a 66 d 67 a 68 c 69 d 70 a
71 a 72 d 73 c 74 c 75 b 76 d 77 b 78 b 79 b 80 a
81 c 82 c 83 a 84 c 85 d 86 b 87 a 88 a 89 b 90 a
91 a 92 c 93 b 94 b 95 c 96 a 97 d 98 a 99 c 100 a
101 d 102 a 103 b 104 c 105 d 106 a 107 a 108 b 109 c 110 c
111 a 112 d 113 d 114 d 115 b 116 c 117 b 118 c 119 c 120 b
121 c 122 c 123 c 124 b 125 c 126 c 127 b 128 b 129 d 130 d
131 a 132 a 133 c 134 b 135 c 136 d 137 b 138 b 139 a 140 b
141 d 142 a 143 a 144 c 145 c 146 c 147 b 148 c 149 a 150 c
151 a 152 a 153 b 154 c 155 d 156 b 157 d 158 c 159 d 160 a
161 c 162 b 163 a 164 d 165 c 166 a 167 b 168 d 169 b 170 b
171 d 172 d 173 a 174 b 175 d 176 b 177 c 178 d 179 c 180 b
181 c 182 c 183 a 184 a 185 a 186 a 187 b 188 c 189 d 190 b
191 a 192 d 193 b 194 a 195 d 196 c 197 c 198 b 199 d 200 c
201 a 202 c 203 b 204 d
MATHEMATICS Measurements of Angles and Trigonometric Ratios 211

HINTS AND SOLUTIONS


1. (b) We know that, π radian = 180° 6. (b) Let the hands of a clock coincides 4
9. (c) Given, 3 tan θ = 4 ⇒ tanθ =
180° 180° after ‘t’ min. 3
⇒ 1 radian = = ×7
π 22 Angle traced by hour hand in 1 − sin θ 1 − sin θ 1 − sin θ
= ×
630° 3° 3 × 60 1 1 + sin θ 1 + sin θ 1 − sin θ
= = 57 = 57° + min t min = t
11 11 11 2 1 – sin θ
4 =
= 57° + 16′ + min Angle traced by minute hand in 1 – sin 2 θ
11 t min = 6t 1 − sin θ 1 sin θ
4 = = −
= 57° + 16′ + × 60 s Angle between hour hand and minute cos θ cos θ cos θ
11 hand at 5 pm = 5 × 30 = 150°
= 57° + 16′ + 21.8′′ = sec θ − tan θ
1 150 × 2 300
= 57° 16′ 218
. ′′ = 57° 16′ 22′′ ∴ 6t − t = 150° ⇒ t = = = 1 + tan 2 θ − tan θ
2 11 11
30  °
2. (a) − 47° 30′ = −  47 +
2
1 +  
 3 4 4
 = 27 + = 27 min 16 sec = −
60  11 3 3
1 ° Q 1′ =  1  ° 
= −  47 +     Therefore, required time = 5:27:16 pm 16 4 5 4 1
 2   60   = 1+ − = − =
1 9 3 3 3 3
94 + 1 ° −95° 7. (a) Given, sin A = ,
= −   = 3
 2  2 C 10. (b) Given, tan A = 1 = tan 45°
−95 π −19 π ⇒ A = 45° and tan B = 3 = tan 60°
= × = rad
2 180 72 ∴ B = 60°
3. (b) Here, r = 60 cm, l = 16.5 cm 3 Now, cos A cos B − sin A sin B
c 1
= cos 45° cos 60°− sin 45° sin 60°
∴ θ = = 
l 16. 5 

r  60  1 1 1 3 1− 3
° ° = × − × =
= 
180   165 × 7
165. . 180 2 2 2 2 2 2
×  =  × A B
 60 π   60 22  2√2
° 11. (b) cosec 2 θ − 2 + sin 2θ
=   = 15° 45′
63 In right angled ∆ABC ,
 4 = (sin θ − cosec θ)2
AB = AC
2 2
− BC 2
=3 − 1 =8
2 2
Hence, it is always non-negative.
4. (c) Speed of train = 90 km/h ⇒ AB = 2 2
12. (b) sin 2 15° + sin 2 20° + sin 2 25°
=  90 ×  m/sec = 25 m/sec
5 AB 2 2
 18  ∴ cos A = =
AC 3 + ... + sin 2 75°
Distance moved in 11 sec
and sec A =
AC
=
3 = (sin 15°+ sin 75° )
2 2

= ( 25 × 11) m = 275m AB 2 2
+ (sin 2 20°+ sin 2 70° ) + ...+ sin 2 45°
∴ l = 275 m, r = 1500 m BC 1
tan A = = = [sin 2 15°+ sin 2 ( 90°−15° )]
l  275 
c AB 2 2
Q θ= ⇒ θ=   + [sin 2 20°+ sin 2 ( 90°−20° )]+ ...+ sin 2 45°
r  1500  and cosec A =
AC
=3
° ° BC = (sin 2 15°+ cos 2 15° )
= 
275 180   275 180
×  = × × 7 ∴ cos A ⋅ cosec A + tan A ⋅ sec A + (sin 2 20°+ cos 2 20° )+ ...+ 1 / 2
 1500 π   1500 22 
°
 21 =
2 2 3
⋅ +
1

3 = 6 + 1 / 2 = 13 / 2
=   = 10°30′
 2 3 1 2 2 2 2 ( Q sin 2 θ + cos 2θ = 1)
3 16 2 + 3
5. (b) Angle traced by hour hand in =2 2+ = 5 sin 75° sin 77°+ 2 cos 13° cos 15°
1 hr = 30° 8 8 13. (b)
cos 15° sin 77°
Angle traced by hour hand in 8. (b) cos 15° − sin 15° = cos 15° 7 sin 81°

°
− sin ( 90° − 75° ) cos 9°
hrs =  30 ×  = 227°30′
91 91
 5 cos 15° sin 77°+ 2 sin 77° cos 15°
12 12  = cos 15° − cos 75° =
Angle traced by minute hand in 15°+75° 75°−15° cos 15° sin 77°
= 2 sin ⋅ sin 7 cos 9°
1min = 6° 2 2 −
Angle traced by minute hand in Q cos C − cos D = 2 sin C + D sin D − C  cos 9°
35 min = ( 6 × 35)° = 210°   7 cos 15°⋅ sin 77° 7 cos 9°
 2 2  = −
∴ Required angle = 227°30′ − 210° 1 1 1 cos 15°⋅ sin 77° cos 9°
= 2 sin 45°⋅ sin 30° = 2 ⋅ = =7−7=0
= 17°30′ 2 2 2
212 CDS Pathfinder

14. (a) log (tan 1° ) + log (tan 2° ) +…+ log (tan 89° ) 23. (b) As we know, sin x is increasing from 0 to 90°.
= log (tan 1° tan 2°… tan 45° … tan 88° tan 89° ) ∴ sin y > sin x
= log [(tan 1° cot 1° ) (tan 2° cot 2° ) … tan 45° ] 24. (a) Given, (sin θ + cosec θ ) = 2.5
[Q tan 89° = tan( 90° − 1° ) = cot 1°]  
⇒  sin θ + 1  = 5 ⇒ 2 sin 2 θ − 5 sin θ + 2 = 0
= log( 1°⋅1° ... 1° ) = 0 
 sin θ  2
1
15. (c) Q Given, tan θ + =2
tan θ ⇒ 2 sin 2 θ − 4 sin θ − sin θ + 2 = 0
2
On squaring both side, we get  tan θ +
1  ⇒ 2 sin θ (sin θ − 2) − 1 (sin θ − 2) = 0
 =2
2
 tan θ  1
⇒ ( 2 sin θ − 1 ) (sin θ − 2) = 0 ⇒ sin θ = (Q sin θ ≠ 2)
1 1
⇒ tan 2 θ + + 2 = 4 ⇒ tan 2 θ + =2 2
tan 2 θ tan 2 θ ∴ θ = 30°
13 cos x sin x
16. (c) Given, sec θ = 25. (a) Given, = n and =m ...(i)
5 cos y sin y
∴ tan θ = sec 2 θ − 1 =
169
−1 =
144 12
=  sin 2 x cos 2 x 
Now, ( m 2 − n2 ) sin 2 y =  2 − 2
 sin y
25 25 5  sin y cos 2 y 
2 sin θ
2 sin θ − 3 cos θ −3 ( 1 − cos 2 x) cos 2 y − cos 2 x ( 1 − cos 2 y )
=
∴ = cos θ cos 2 y
4 sin θ − 9 cos θ sin θ
4 −9
cos θ cos 2 y − cos 2 x
= = 1 − n2 [from Eq. (i)]
(dividing numerator and denominator by cos θ) cos 2 y
12
2 tan θ − 3 2× −3 26. (b) Given, p = tan 2 x + cot 2 x = tan 2 x +
1
5 9/5 9
= = = = =3 tan 2 x
4 tan θ − 9 4 × 12 − 9 3 / 5 3 1/2
, ≥ 2  tan 2 x ⋅
1 1 
5 Q AM ≥ GM ⇒ tan 2 x + 
tan 2 x  tan 2 x 
17. (c) sin ( 45° + A ) − cos ( 45° − A )
= sin( 45°+ A ) − cos[( 90°− ( 45°+ A )] = sin( 45°+ A ) − sin( 45°+ A ) 1
⇒ tan 2 x + ≥ 2 ⇒ p≥ 2
=0 tan 2 x
1 + sin θ ( 1 + sin θ)( 1 + sin θ) 27. (a) Let angles in circular measure are A and B, then degree
18. (b) =
measures will be A 
1 − sin θ ( 1 − sin θ)( 1 + sin θ) 180   180  .
 and B  
 π   π 
( 1 + sin θ)2 ( 1 + sin θ)2 1 + sin θ 1 sin θ
= = = = + By given condition, A+ B=1 …(i)
1 − sin θ
2
cos 2 θ cos θ cos θ cos θ
 180   180 
= sec θ + tan θ A  − B  =1 …(ii)
 π   π 
19. (b) sin ( n + 1) A sin ( n − 1) A + cos ( n + 1) A cos ( n − 1) A 180
= cos [( n + 1) A − ( n − 1) A ] On multiplying Eq. (i) by and adding to Eq. (ii), we get
π
[Q cos A cos B + sin A sin B = cos( A – B )] π
2 A 
180  180 1 11  180
+ 1 = +
1
= cos [ nA + A − nA + A ] = cos 2 A  = + 1⇒ A = ×
 π  π 2 180  π  2 360
cos 11°+ sin 11°
π  1 π
B = 1 − A = 1 −  +
20. (a) 1
cos 11°− sin 11° ∴  = −
 2 360  2 360
Divide numerator and denominator by cos 11°
1 + tan 11° tan 45°+ tan 11° 28. (b) [( 1 − sin 2 θ) sec 2 θ + tan 2 θ] (cos 2 θ + 1)
= = = tan ( 45° + 11° ) = tan 56°
1 − tan 11° 1 − tan 45° tan 11° = [cos 2 θ ⋅ sec 2 θ + tan 2 θ] (cos 2 θ + 1) (Q sin 2 θ + cos 2 θ = 1)
a2 − b2 = ( 1 + tan θ) (cos θ + 1) = sec θ (cos θ + 1)
2 2 2 2

sin θ sin θ a2 + b2 (Q sec 2 θ − tan 2 θ = 1)


21. (c) tan θ = = =
cos θ 1 − sin 2 θ ( a 2 − b 2 )2 = sec 2 θ ⋅ cos 2 θ + sec 2θ = 1 + sec 2 θ > 2
1− 2
( a + b 2 )2 (Q sec 2 θ > 1 for 0 < θ < 90° )
a2 − b2 29. (d) Given, 7 cos 2 θ + 3 sin 2 θ = 4 ⇒ 7 cos 2 θ + 3 ( 1 − cos 2 θ) = 4
a2 + b2 a2 − b2 a2 − b2 ⇒ 3 − 4 cos 2 θ = 4 ⇒ cos 2 θ = 1/ 4
= = =
(a + b ) − (a − b )
2 2 2 2 2 2 2 2
4a b 2ab
Now, tan 2 θ = sec 2θ − 1 =
1
− 1 ⇒ tan 2 θ = 4 − 1 = 3
a2 + b2 cos 2θ
∴ tanθ = 3 (Q 0 < θ < π / 2)
22. (d) Given, cosθ = 0.96, sin θ = 1 − cos 2 θ = 1 − (0.96)2
30. (d) We know that, the value of cos θ is decreasing in the interval
sin θ 0.28 7 0 ≤ θ ≤ 90°
= 1 − 0.9216 = 0.28, tan θ = = =
cos θ 0.96 24 ∴ cos 1° > cos 89° ⇒ p > q
1 1 100 24 196 Also, cos 1° is close to 1 and cos 89° is close to 0.
+ = + = =7
sin θ tan θ 28 7 28 Hence, option (d) is correct.
MATHEMATICS Measurements of Angles and Trigonometric Ratios 213

31. (d) We know in the interval θ ∈( 0, π / 2),sec 2 θ is increasing from 1 38. (d) Given, sin x + 3 cos x = 1
to ∞. 1 3 1
∴ p≥ 1 On dividing both sides by 2, we get sin x + cos x =
2 2 2
32. (b) Q cos 90° = 0 ⇒ sin x sin 30° + cos x cos 30° = 1 / 2
∴ cos 1° cos 2° cos 3°… cos 90° = 0 (Q cos 90° = 0) ⇒ cos ( x − 30° ) = cos 60° ⇒ x − 30° = 60°
1 − cos B ∴ x = 90°
33. (c) Given, tan A =
sin B 1 + tan 2 A
( 1 − cos B ) 39. (c) cosec A ⋅ sec A = ⋅ 1 + tan 2 A
2⋅ tan A
2 tan A sin B
∴ = 1 + tan 2 A 1 + ( 2 − 1)2
1 − tan 2 A  1 − cos B 
2
= = (Q tan A = 2 – 1)
1−   tan A 2−1
 sin B 
2( 1 − cos B ) sin B 2 ( 1 − cos B ) sin B 1 + 2 + 1 − 2 2 4 − 2 2 2 2( 2 − 1)
= = = = = =2 2
sin 2 B − 1 − cos 2 B + 2 cos B −2 cos 2 B + 2 cos B 2−1 ( 2 − 1) ( 2 − 1)
2 sin B( 1 − cos B ) 3
= =
sin B
= tan B 40. (c) Given, tan θ = ⇒ sinθ = 3 cos θ ⇒ sinθ = 3 1 − sin 2θ
2 cos B ( 1 − cos B ) cos B 1
Squaring both sides, we get sin 2 θ = 9( 1 − sin 2θ)
34. (c) cot 15° cot 20° cot 70° cot 75° 9 3
⇒ 10 sin 2 θ = 9 ⇒ sinθ = =−
= tan ( 90° − 15° ) tan ( 90° − 20° ) cot 70° cot 75° 10 10
= tan 75° tan 70°
1

1
=1 (Q θ lies in IIIrd quadrant, so sin θ will be negative)
tan 70° tan 75° 41. (a) sin 20° cos 70° + cos 20° sin 70°
sin θ cos θ = sin( 20°+70° ) = sin 90° = 1
35. (b) Given, + =2
cos θ sin θ [Q sin A cos B + cos A sin B = sin( A + B )]
∴ sin 2 θ + cos 2 θ = 2 sin θ cos θ 42. (a) φ + θ = 180° ⇒ θ = 180° − φ ⇒ sin θ = sin ( 180° − φ)
⇒ sin 2θ = 1 = sin 90° ⇒ 2θ = 90° ⇒ θ = 45° sin θ = sin φ
36. (d) Given, in right angled ∆ABC , ∠B = 90° and AB : AC = 1 : 2 43. (b) cos x cosec y − cos x sin y
Let AB = x, AC = 2x = cos x ⋅ cosec ( 90°− x) − cos x ⋅ sin( 90°− x)
C ( Q x + y = 90°, given)
= cos x ⋅ sec x − cos x
2

√2 x
= 1 − cos 2 x = sin 2 x = sin x

44. (c) 2+ 2 + 2 cos 4 θ = 2+ 2( 1 + cos 4 θ)


A x B
⇒ AB 2 + BC 2 = AC 2 (using pythagoras theorem) = 2+ 2( 2 cos 2θ) =
2
2+ 4 cos 2 2θ = 2 + 2 cos 2θ
⇒ x + BC
2 2
= ( 2 x) 2
= 2( 1 + cos 2θ) = 2( 2 cos θ) =
2
4 cos θ = 2 cos θ
2

⇒ = 2x − x ⇒ BC = x
BC 2 2 2
5 1
45. (b) Given, tan A = and tan B =
BC x 6 11
∴ tan A = = =1
AB x 5 1 55 + 6
+
tan A + tan B 6 11 61

2 tan A
=
2( 1) 2
= =∞ (undefined) Q tan ( A + B ) = = = 66 = =1
1 − tan A tan B 1 − 5 ⋅ 1 66 − 5 61
1 − tan 2 A 1 − 12 0
a 6 11 66
37. (a) Given, AD = DB = and ∠B = 90° π π
2 ⇒ tan ( A + B ) = 1 ⇒ tan( A + B ) = tan ⇒ A + B =
4 4
In right angled ∆ ABC , BC 2 + AB 2 = AC 2 1 cos A sin A
46. (c) (cos A − sin A ) = −
⇒ BC 2 + a 2 = b 2 ⇒ BC = b2 − a2 2 2 2
π π π
In right angled ∆ BCD, CD = BC 2 + BD 2 = cos cos A − sin sin A = cos  + A 
2

2
 a2  4 4 4 
CD 2 = ( b 2 − a 2 )2 +   =  b 2 − a 2 +
a

 2  4 [Q cos A cos B – sin A sin B = cos( A + B )]
4b − 3a 4b − 3a π π π π π π
= sin  −
2 2 2 2
⇒ CD 2 = ⇒ CD = 47. (d) sin ⋅ cos − cos ⋅ sin 
4 2 4 12 4 12  4 12 
BC b2 − a2 2 b2 − a2 [Q sin A cos B – cos A sin B = sin ( A − B )]
∴ cosθ = = = π
= sin   =
1
CD 4b − 3a2 2
4b 2 − 3a 2
 6 2
2
214 CDS Pathfinder

x sin φ sin x
48. (c) Given, tan θ = ⇒ x sin φ = tan θ − x cos φ tan θ 56. (b) Given, 3 cos x = 5 sin x ⇒ =
3 3
⇒ tan x = ,
1 − x cos φ cos x 5 5
tan θ sin θ sin θ 5 sin x − 2 sec 3 x + 2 cos x 5 tan x − 2 sec 4 x + 2
⇒ x= = = =
sin φ + cos φ tan θ cos θ sin φ + cos φ sin θ sin(θ + φ) 5 sin x + 2 sec 3 x − 2 cos x 5 tan x + 2 sec 4 x − 2
sin φ x sin θ (dividing numerator and denominator by cos x)
Similarly, y = , Hence, =
sin ( θ + φ) y sin φ 5 tan x − 2( 1 + tan 2 x)2 + 2
=
5 tan x + 2( 1 + tan 2 x)2 − 2
49. (b) Given, sin ( θ + φ) = 2 sin ( θ − φ) 2
− 2  1 +
3 9 1156
⇒ sin θ cos φ + cos θ sin φ = 2 (sin θ cos φ − cos θ sin φ) 5 ×  + 2 3+ 2−2×
5  25  625
3 sin φ sin θ = =
⇒ 3 cos θ sin φ = sin θ cos φ ⇒ = 3  9
2
3 − 2 + 2 ×
1156
cos φ cos θ 5 × + 2 1 +  −2
5  25  625
⇒ 3 tan φ = tan θ or tan θ = 3 tan φ 3125 − 2312 813 271
sin(θ + α ) = = =
50. (a) Given, sin ( θ + α ) = cos ( θ + α ) ⇒ =1 625 + 2312 2937 979
cos(θ + α )
tan θ + tan α 1 + cos x ( 1 + cos x) 1 + cos x
⇒ tan ( θ + α ) = 1 ⇒ =1 57. (b) = ×
1 − tan θ tan α 1 – cos x ( 1 − cos x) 1 + cos x
⇒ tan θ + tan α = 1 − tan θ tan α 1 + cos x 1 cos x
= = + = cosec x + cot x
1 − tan α
⇒ tan θ ( 1 + tan α ) = 1 − tan α ⇒ tan θ = sin x sin x sin x
1 + tan α
58. (a) cos 4 x − sin 4 x = (cos 2 x − sin 2 x) (sin 2 x + cos 2 x)
θ− φ θ+ φ
51. (c) cos 2 − sin 2
2 2 = [(cos 2 x) − ( 1 − cos 2 x)] = 2 cos 2 x − 1 (Q sin 2 x + cos 2 x = 1)
 θ − φ θ + φ  θ − φ − θ + φ
= cos  +  ⋅ cos   sin θ sin θ 1 − cos θ sin θ( 1 − cos θ) 1 − cos θ
 2 2   2 2  59. (a) = × = =
1 + cos θ 1 + cos θ 1 − cos θ sin 2 θ sin θ
[Q cos 2 A − sin 2 B = cos ( A + B ) cos ( A − B )]
5 cos θ − 4 3 + 5 sin θ
= cos θ ⋅ cos ( − φ) = cos θ ⋅ cos φ 60. (a) −
3 − 5 sin θ 4 + 5 cos θ
52. (c) x2 + y 2 + z 2 = r 2 sin 2 θ cos 2 φ + r 2 sin 2 θ sin 2 φ + r 2 cos 2 θ (5 cos θ − 4)( 4 + 5 cos θ) − (3 + 5 sin θ)(3 − 5 sin θ)
=
= r 2 sin 2 θ (cos 2 φ + sin 2 φ) + r 2 cos 2 θ (3 − 5 sin θ)( 4 + 5 cos θ)
= r sin 2 θ + r 2 cos 2 θ = r 2 (sin 2 θ + cos 2 θ) = r 2
2 ( 25 cos 2 θ − 16) − ( 9 − 25 sin 2 θ)
=
7 49 625 (3 − 5 sin θ)( 4 + 5 cos θ)
53. (c) Given, cot θ = , cosec 2 θ = 1 + cot 2 θ = 1 + = 25(sin 2 θ + cos 2 θ) − 25 25 − 25
24 576 576 = = =0
25 (3 − 5 sin θ)( 4 + 5 cos θ) (3 − 5 sin θ)( 4 + 5 cos θ)
∴ cosecθ = ±
24 16 9 3
61. (a) sin (α + β ) = 1 − cos 2 ( α + β ) = 1− = =
24 576 49 25 25 5
⇒ sin θ = ± , cos 2 θ = 1 − sin 2 θ = 1 − =
25 625 625 25 144 12
cos ( α − β ) = 1 − sin 2 ( α − β ) = = 1− =
7 3π 169 169 13
∴ cosθ = ± , As π < θ <
25 2 sin(α + β )  3 5  3
∴ tan ( α + β ) = = ×  =
∴ sin θ and cosθ both are negative. cos(α + β )  5 4  4
− 24 −7 sin(α − β ) 5 13 5
∴ sin θ = , cos θ = tan (α − β ) = = × =
25 25 cos(α − β ) 13 12 12
− 7 24 17
∴ cos θ − sin θ = + = ∴ tan ( 2α ) = tan[(α + β ) + (α − β )]
25 25 25 3 5
+
1 tan(α + β ) + tan(α − β ) 4 12 56
54. (b) Given, sin α = = sin 30° ⇒ α = 30° = = =
2 1 − tan(α + β ) ⋅ tan(α − β ) 1 − ×3 5 33
π 4 12
Also, β = − α = 90° − 30° = 60°
2 tan 45°+ tan x 1 + tan x
62. (a) tan ( 45° + x) = =
55. (c) Given, sin x − cos x = 0 ⇒ sin x = cos x ⇒ tan x = 1 1 − tan 45° tan x 1 − tan x
π π tan 45°− tan x 1 − tan x
⇒ tan x = tan ⇒ x = tan ( 45° − x) = =
4 4 1 + tan 45° tan x 1 + tan x
4 4 1 + tan x
π π  1   1 
∴ sin 4 x + cos 4 x = sin 4 + cos 4 =  +   tan( 45°+ x) 1 − tan x  1 + tan x 
2
4 4  2  2 ∴ = =
tan( 45°− x) 1 − tan x  1 − tan x 
1 1 1
= + = 1 + tan x
4 4 2
MATHEMATICS Measurements of Angles and Trigonometric Ratios 215

( 1 − sin A cos A )(sin 2 A − cos 2 A ) 71. (a) Refer to example 16.


63. (a)
cos A (sec A − cosec A )(sin 3 A + cos 3 A ) p
72. (d) Given, tan θ = ,
( 1 − sin A cos A )(sin 2 A − cos 2 A ) q
=  1 − tan 2 θ  2 tan θ 
 cos A  1 − 1  (sin A + cos A )
  p cos 2θ + q sin 2θ = p  2 
+ q  
  cos A sin A    1 + tan θ   1 + tan 2 θ 
 
(sin A + cos A − sin A cos A )
2 2
  p2   2p 
  1−     2 2   2 
= p
q2  + q q  = p  q − p  +  2 pq 
[( 1 − sin A cos A )(sin A ⋅ cos A )      q2 + p2   q2 + p2 
p2 p2
× (sin A + cos A ) (sin A − cos A )] 1+  1+ 2     
 q2   q 
= = sin A
[cos A (sin A − cos A )(sin A + cos A ) p( q 2 − p 2 + 2q 2 ) p(3q 2 − p 2 )
= =
( 1 − sin A cos A )] p2 + q2 p2 + q2

64. (c) ( 1 + tan A ) ( 1 + tan B ) 2π 4π − y −z


73. (c) Given, x = y cos = z cos ⇒ x= = =k (let)
= ( 1 + tan A ) [ 1 + tan ( 45° − A )] 3 3 2 2
tan 45°− tan A  x = k , y = − 2k , z = − 2k
= ( 1 + tan A )  1 + (Q A + B = 45°)
 1 + tan 45° tan A  ⇒ xy + yz + zx = k ( − 2k ) + ( − 2k ) ( − 2k ) + ( − 2k ) k
1 − tan A 
= ( 1 + tan A )  1 + = − 2k 2 + 4k 2 − 2k 2 = 0
 1 + tan A 
74. (c) Given, sin θ + cos θ = m ...(i)
1 + tan A + 1 − tan A 
= ( 1 + tan A )  1 1
 1 + tan A  and sec θ + cosec θ = n ⇒ + =n
cos θ sin θ
= ( 1 + tan A )  =2
2
sin θ + cos θ
 1 + tan A  ⇒ =n
sin θ cos θ
( A + B) ( B − A) m m
65. (a) Q cos A − cos B = 2 sin ⋅ sin ⇒ = n ⇒ sin θ cos θ = [from Eq. (i)]
2 2 sin θ cos θ n
( A + B) ( A − B)
and sin A + sin B = 2 sin ⋅ cos
2 2 On squaring Eq. (i), we get sin 2 θ + cos 2 θ + 2 sin θ cos θ = m 2
cos 2B − cos 2 A 2 sin( A + B ) sin( A − B ) m 2m
∴ = = tan ( A − B ) 1 + 2 = m2 ⇒ = m2 − 1 [Q sin 2 θ + cos 2 θ = 1]
sin 2B + sin 2 A 2 sin( A + B ) cos( A − B ) n n
66. (d) sec 2 x + tan 2 x = 7 (Q sec 2 x = 1 + tan 2 x) ⇒ 2m = ( m 2 − 1) n
1 + tan x + tan x = 7 ⇒ 1 + 2 tan x = 7
2 2 2
75. (b) Given, cosecθ − sin θ = a 3 ⇒
1
– sin θ = a 3
⇒ 2 tan 2 x = 6 ⇒ tan x = 3 = tan 60° sin θ
∴ x = 60° 1 − sin 2 θ cos 2 θ
⇒ = a3 ⇒ = a 3 ⇒ cos 2 θ = a 3 sin θ …(i)
67. (a) cos 3θ + sin 3θ = 2  sin 3θ sin θ sin θ
1 1
cos 3θ +
 2 2 
1 1 − cos 2 θ
= 2 (sin 45° cos 3θ + cos 45° sin 3θ) = 2 [sin ( 45° + 3θ)] and sec θ − cos θ = b 3 ⇒ – cos θ = b 3 ⇒ = b3
cos θ cos θ
cos3θ + sin 3θ is maximum when sin( 45°+3θ) is maximum
The maximum value of sin ( 45° + 3θ) = 1 = sin 90° sin 2 θ
⇒ = b 3 ⇒ sin 2 θ = b 3 cos θ …(ii)
⇒ 45° + 3θ = 90° ⇒ θ = 15° cos θ
sin 2 θ
68. (c) sin 2 x + sin 2 y + sin 2 z = (sin x + sin y + sin z )2 On putting cos θ = in Eq. (i), we get
b3
⇒ sin x sin y + sin y sin z + sin z sin x = 0
sin 4 θ
[Q ( a + b + c )2 = a 2 + b 2 + c 2 + 2 ( ab + bc + ca )] = a 3 sin θ ⇒ sin 3 θ = a 3 b 6
b6
On dividing both sides by sin x sin y sin z
∴ sin θ = ab 2 …(iii)
1 1 1
⇒ + + =0
sin x sin y sin z Similarly, cos θ = a 2 b …(iv)
On squaring and adding Eqs. (iii) and (iv), we get
69. (d) Given, sin x + sin 2 x = 1 …(i)
1 = a2b4 + a4 b2 ⇒ 1 = a2b2 ( a2 + b2 )
⇒ sin x = 1 − sin 2 x = cos 2 x (Q sin 2 x + cos 2 x = 1)
∴cos 2 x + cos 4 x = (sin x) + sin 2 x = 1 [from Eq. (i)] 76. (d) As, we know that cos θ ≥ − 1 and cos θ ≤ 1
∴ cos 2x ≥ − 1 ⇒ 1 + cos 2x ≥ 0
70. (a) (cos 35° + cos 85° ) + cos 155°
cos 2x ≤ 1 ⇒ 1 + cos 2x ≤ 2
= 2 cos 60° cos 25° + cos ( 180° − 25° )
1 So, minimum value of 1 + cos 2x is 0 and maximum value of
= 2 × cos 25° − cos 25° = cos 25° − cos 25° = 0 1 + cos 2x is 2.
2
216 CDS Pathfinder

1 1 − tan θ tan φ 1 − cos 2 x


77. (b) cot ( θ + φ) = =
tan ( θ + φ) tan θ + tan φ n sec x − cos x
86. (b) = = cos x
m cosec x − sin x 1 − sin 2 x
1 tan θ tan φ
= − sin x
tan θ + tan φ tan θ + tan φ 3
sin 2 x sin x
= 
sin x 
= ⋅ 
=
1

1 cos x cos x  cos x 
2

tan θ + tan φ   1/3


 1 + 1  ⇒ (tan x)3 =  
n
⇒ tan x =  
n
 tan θ tan θ  m m
 
1
1 1 1 1 87. (a) 2 cos θ = x + …(i)
= − = − x
tan θ + tan φ (cot θ + cot φ) a b 3
On cubing both sides, we get 8 cos 3 θ =  x + 
1
[Q tan θ + tan φ = a, or cot θ + cot φ = b]  x
8 cos 3 θ = x3 + 3 + 3x ⋅  x + 
1 1 1
78. (b) Q tan 75° = tan ( 45° + 30° )
x x x
tan 45°+ tan 30° 1 + tan 30°
⇒ tan 75° = ⇒ tan 75° = 1
1 – tan 45° tan 30° 1 − tan 30° 8 cos 3 θ = x3 + 3 + 3 ( 2 cos θ) [from Eq. (i)]
x
⇒ tan 75°− tan 75°⋅ tan 30° = 1 + tan 30° 1
⇒ 8 cos 3 θ − 6 cos θ = x3 + 3
⇒ tan 75°− tan 30°− tan 75°⋅ tan 30° = 1 x
1 1
79. (b) sin 2 ( 15° + A ) − sin 2 ( 15° − A ) = sin ( 15° + A + 15° − A ) ⇒ 2 [ 4 cos 3 θ − 3 cos θ] = x3 + 3 ⇒ 2 [cos 3θ] = x3 + 3
x x
⋅ sin ( 15° + A − 15° + A ) 88. (a) We know that, the value of cosθ decreases in the interval
[Q sin 2 A − sin 2 B = sin ( A + B ) ⋅ sin ( A − B )] 0≤ θ≤ π / 2
1 1 π π
= sin 30° sin ( 2 A ) = sin 2 A Q cos θ ≥ ⇒ cos θ ≥ cos ⇒ θ≤
2 2 3 3
sin 38° − cos 68° cos( 90° − 38° ) − cos 68° 3 12
80. (a) = 89. (b) Given, sin A = and cos B =
cos 68° + sin 38° cos 68° + sin( 90° − 52° ) 5 13
sin A sin A 3/5 3/5 3
52° + 68°   68° − 52°  ∴ tan A = = = = =
2 sin   sin   cos A 1 − sin 2 A 1 − (3 / 5 ) 2 4 /5 4
cos 52° − cos 68°  2   2 
= = 144
cos 68° + cos 52° 2 cos  52° + 68°  cos  68° − 52°  1−
    sin B 1 − cos 2 B
169 = 5 / 13 = 5
 2   2  and tan B = = =
cos B cos B12 12 / 13 12
sin 60°⋅ sin 8°
= = tan 60°⋅ tan 8° = 3 tan 8° 13
cos 60°⋅ cos 8° 3 5 9 −5

81. (c) 2 cos x − (cos 3x + cos 5x) tan A − tan B
= 4 12 = 12
1 + tan A tan B 1 + 3 ⋅ 5 48 + 15
= 2 cos x − 2 cos 4x ⋅ cos x = 2 cos x( 1 − cos 4x)
4 12 4 × 12
= 2 cos x ⋅ 2 sin 2 2x = 4 cos x( 2 sin x cos x)2 = 16 cos 3 x sin 2 x 1
1 48 16
82. (c) cos 4x = 1 − 2 sin 2 2x = 1 − 2 ( 2 sin x cos x)2 = 3 = × =
63 3 63 63
= 1 − 2( 4 sin 2 x cos 2 x) = 1 − 8 sin 2 x cos 2 x
48
83. (a) 2 sin A cos 3 A − 2 sin 3 A cos A
90. (a) Given, B + C = 60° ⇒ B = 60° − C
= 2 sin A cos A (cos 2 A − sin 2 A )
sin ( 120° − B ) = sin [ 120° − ( 60° − C )] = sin ( 60° + C )
2 sin 2 A cos 2 A 1
= sin 2 A (cos 2 A ) = = sin 4 A = sin [ 180° − ( 60° + C )] = sin ( 120° − C )
2 2
x ⋅ cosec 2 30° sec 2 45°
84. (c) Given, a cos θ − b sin θ = c 91. (a) Given, = tan 2 60° − tan 2 30°
8 cos 2 45° sin 2 60°
On squaring both sides, we get 2
x × ( 2)2 × ( 2 )2
= ( 3 )2 −  
1
a 2 cos 2 θ + b 2 sin 2 θ − 2ab cos θ sin θ = c 2 ⇒
2
 3
2  3
8 × 
⇒ a 2 ( 1 − sin 2 θ) + b 2 ( 1 − cos 2 θ) − 2ab sin θ cos θ = c 2 1 
 × 
 2  2 
⇒ a 2 + b 2 − c 2 = a 2 sin 2 θ + b 2 cos 2 θ + 2ab sin θ cos θ
x×4×2 1 8x 8
⇒ ( a sin θ + b cos θ)2 = a 2 + b 2 − c 2 ⇒ =3− ⇒ = ⇒x= 1
1 3 3 3 3
8× ×
⇒ a sin θ + b cos θ = ± a 2 + b 2 − c2 2 4
sin x 1 − cos x
85. (d) Given, cos x + cos 2 x = 1 ⇒ cos x = 1 − cos 2 x = sin 2 x …(i) 92. (c) =
1 + cos x sin x
sin 8 x + 2 sin 6 x + sin 4 x = (cos x)4 + 2 cos 3 x + cos 2 x
The above identity is possible for all values of x except multiples of
= (cos 2 x + cos x)2 [from Eq. (i)] 180°. Since, for x = 180°, sin x = 0 and 1 + cos x = 0.
= ( 1)2 = 1
MATHEMATICS Measurements of Angles and Trigonometric Ratios 217

1 cos( x − 2 y ) 99. (c) Given, x = a sec θ cos φ, y = b sec θ sin φ and z = c tanθ,
93. (b) Given, cos x = k cos ( x − 2 y ) ⇒ =
k cos x x 2 y 2 z2 a 2 sec 2 θ cos 2 φ b 2 sec 2 θ sin 2 φ c 2 tan 2 θ
2
+ 2 − 2 = + −
Apply componendo and dividendo theorem a b c a2 b2 c2
x – 2y + x x – x + 2y = sec θ [cos φ + sin φ] − tan θ = sec θ − tan 2 θ = 1
2 2 2 2 2
2 sin sin
1 − k cos( x − 2 y ) − cos x 2 2 100. (a) The squares of the tangents of the angles 30°, 45° and 60° are in
= =
1 + k cos( x − 2 y ) + cos x x – 2y + x x – 2y – x G.P.
2 cos cos 2
⇒ tan 2 30°, tan 2 45°, tan 2 60° are in G.P. ⇒   , ( 1)2 , ( 3 )2
2 2 1
2 sin( x − y ) sin y  3
= = tan( x − y ) tan y
2 cos( x − y ) cos y 1
are in G.P. ⇒ , 1, 3 are in G.P.
sin( x + y ) − 2 sin x + sin( x − y ) 3
94. (b)
cos( x + y ) − 2 cos x + cos( x − y ) 1
which is true as 12 = × 3 ⇒ 1 = 1
3
sin( x + y ) + sin( x – y ) – 2 sin x
= 101. (d) We know that, in a cyclic quadrilateral sum of opposite angle is
cos( x + y ) + cos( x – y ) – 2 cos x
180°.
x+ y + x− y x+ y −x+ y
2 sin ⋅ cos − 2 sin x ∴ A + C = 180° …(i)
= 2 2
(x + y + x − y) (x + y − x + y) and B + D = 180° …(ii)
2 cos ⋅ cos − 2 cos x
2 2 ∴ cos A + cos B + cos C + cos D
2 sin x cos y − 2 sin x 2 sin x (cos y − 1) sin x = cos A + cos B + cos ( 180° − A ) + cos ( 180° − B )
= = = = tan x
2 cos x cos y − 2 cos x 2 cos x (cos y − 1) cos x = cos A + cos B − cos A − cos B = 0
x+ y x− y [∴ cos ( 180°− θ) = − cos θ]
95. (c) Given, sin x + sin y = a ⇒ 2 sin cos =a …(i)
2 2 102. (a) We know that, for 0° < θ < 90°, there exist only one θ such that
x+ y (x − y) sin θ = a.
and cos x + cos y = b ⇒ 2 cos ⋅ cos = b …(ii)
2 2 103. (b)Q sin ( B + C − A ) = 1 = sin 90° ⇒ B + C − A = 90° …(i)
On dividing Eq. (i) from Eq. (ii), we get Q cos (C + A − B ) = 1 = cos 0°
x+ y x− y ∴ C + A − B = 0° …(ii)
2 sin cos
2 2 a x+ y a
∴ = ⇒ tan = and tan ( A + B − C ) = 1 = tan 45°
x+ y (x − y) b 2 b
2 cos cos ∴ A + B − C = 45° …(iii)
2 2
On adding Eqs. (i), (ii) and (iii), we get
96. (a) Here, r = length of largest hand of clock
A + B + C = 135° …(iv)
l = distance covered by largest hand of clock
On subtracting Eqs. (i), (ii), (iii) from Eq. (iv), we get
Angle traced in 60 min = 360°
2 A = 45°, 2B = 135°, 2C = 90°
360° 2π 1° 1°
Angle traced in 20 min = × 20 = 120° ⇒ θ = A = 22 , B = 67 , C = 45°
60 3 2 2
l 2 π 42 × 2 × 22
As, θ = ⇒ l = r θ, ⇒ l = 42 × = = 88 cm
r 3 3×7 104. (c) cos ( 180° + A ) + cos ( 180° + B )
+ cos ( 180° + C ) + cos ( 180° + D )
97. (d) Given, 2 sin x + 15 cos 2 x = 7 ⇒ 2 sin x + 15 − 15 sin 2 x = 7
= − cos A − cos B − cos C − cos D
⇒ 15 sin 2 x − 2 sin x − 8 = 0
= − cos A − cos B − cos ( 180° − A ) − cos ( 180° − B )
⇒ 15 sin 2 x − 12 sin x + 10 sin x − 8 = 0
(Q A + C = B + D = 180°)
⇒ 3 sin x ( 5 sin x − 4) + 2 ( 5 sin x − 4) = 0
= − cos A − cos B + cos A + cos B = 0
⇒ ( 3 sin x + 2) ( 5 sin x − 4) = 0
105. (d) In ∆ABC ,
⇒ sin x =
4 Q sin x ≠ − 2 , 0° < x < 90° 
  BC u v
5  3  tan A = = and tan B =
2 2 AC v u
1 −   3
4 3
∴ cos x = 1 − sin 2 x = =   =
5 5 5
A
sin x 4 / 5 4
∴ tan x = = =
cos x 3 / 5 3 v w
98. (a) In ∆ABC , A + B + C = 180°
∴ A + B = 180° − C ⇒ tan ( A + B ) = tan ( 180° − C ) C u B
tan A + tan B
⇒ = − tan C Also, u + v = w
2 2 2
(by pythagoras theorem) …(i)
1 − tan A tan B
⇒ tan A + tan B = − tan C + tan A tan B tan C u v u 2 + v2 w2
∴ tan A + tan B = + = = [from Eq. (i)]
⇒ tan A + tan B + tan C = tan A tan B tan C v u uv uv
218 CDS Pathfinder

π cos π A + B  180°−C 
106. (a)Q In 24 h, Earth rotate about its own RHS cos A + cos B = cos + II. sin   = sin  
axis = 360° 6 3  2   2 
3 1 3+1
= + = = sin  90° −  = cos
In 1 h Earth rotate about its own axis C C
2 2 2  2 2
360° ⇒ sin A + sin B = cos A + cos B
= = 15° + −
III. tan  
A B C
24 π π 
II. LHS tan A + tan B = tan + tan  2 
In 4 h Earth rotate about its own axis 6 3
( 180°−C ) − C
= 15° × 4 = 60° 1 4 = tan
= + 3= 2
Since, in 60 min Earth rotate about its 3 3 ( 180°−2C )
own axis = 15° π π = tan
RHS cot A + cot B = cot + cot 2
In 12 min Earth rotate about its own 6 3
1 4 = tan ( 90° − C ) = cot C
15°×12 = 3+ =
axis = = 3° (A − B −C ) A − (B + C )
60° 3 3 IV. tan = tan
⇒ tan A + tan B = cot A + cot B 2 2
∴ In 4 h 12 min Earth rotate about its A − ( 180°− A )
own axis Hence, both statements are true. = tan
2
= 60° + 3° = 63° 111. (a) Q A + B + C + D = 360°
= tan [ − ( 90° − A )]
107. (a) In right angled ∆OAB, ∴ A + B = 360° − (C + D )
= − tan ( 90° − A ) = − cot A
∴ sin ( A + B ) = sin [360° − (C + D )]
Hence, all options are correct.
= − sin (C + D )
A r B 115. (b) Given, sin( A + B ) = 1
40° ∴ sin ( A + B ) + sin (C + D ) = 0
R and sin( A − B ) = 1 / 2
Also, cos( A + B ) = cos[360° − (C + D )] π
O ∴ A+B = …(i)
cos ( A + B ) = cos (C + D ) 2
π
Hence, only statement I is correct. ∴ A−B = …(ii)
6
π 3.14
112. (d) I. 1° = radian = = 0.017 On adding Eqs. (i) and (ii), we get
AB r 180 180 2π π π
cos 40° = ⇒ cos 40° = 2A = ⇒ A = and B =
OB R = 0.02 radians (approx) 3 3 6
⇒ r = R cos 40° which is less than 0.03 radians. 116. (c) Now, tan ( A + 2B ) ⋅ tan( 2 A + B )
180 π π 2π π
II. 1 radian = = tan  +  ⋅ tan 
So, the radius of the circle of latitude degree
π + 
40°S is R cos 40°. 3 3  3 6
180
108. (b) Given, sin 3θ = cos ( θ − 2° ) = = 57.32 degree 2π 5π
3.14 = tan   ⋅ tan  
⇒ sin 3θ = sin [ 90° − ( θ − 2° )]  3   6 
which is greater than 45°.
π π π π
⇒ 3θ = 90° − θ + 2° Hence, both statements are true.

= tan  +  ⋅ tan  + 
 2 6  2 3
92°
⇒ 4θ = 92° ⇒ θ = = 23° π π −1
4
113. (d) I. sin θ ⋅ sin( 60°+ θ) ⋅ sin( 60°− θ) =  − cot   − cot  = ( − 3 )  = 1
= sinθ[sin 2 60°− sin 2θ]  6   3   3
109. (c) I. RHS = cos 2 θ( 1 + tan θ)( 1 − tan θ)
= sin θ  − sin 2 θ
3 117. (b) Now, sin 2 A − sin 2 B
= cos 2 θ ( 1 − tan 2 θ)  4  = sin 2 ( π / 3) − sin 2 ( π / 6)
 cos 2 θ − sin 2 θ  1 1 2
= cos 2 θ   = [3 sin θ − 4sin θ] = sin 3θ
3
 3 2
=   −   = − = =
1 3 1 2 1
 cos 2 θ  4 4
 2
 2  4 4 4 2
cos 2 θ − sin 2 θ II. cosθ ⋅ sin (30°+θ) ⋅ sin(30°− θ)
= = LHS
cos 2 θ + sin 2 θ = cosθ[sin 2 30°− sin 2θ] 118. (c) Since, tanA, tanB are the roots of
equation 3x 2 − 2 3x + 1 = 0
= cos θ  − ( 1 − cos 2 θ)
1 + sin θ ( 1 + sin θ)2 1
II. LHS = =  4  ∴ tan A + tan B =
2 3
1 − sin θ 1 − sin 2 θ 3
1 + sin θ 
2 1 1
= = [ 4 cos θ − 3cosθ] = cos 3θ
3
 4 4 and tan A ⋅ tan B = 1 / 3
 cos θ 
tan A + tan B
= (sec θ + tan θ )2 III. sin θ ⋅ cos(30°+ θ) ⋅ cos(30°− θ) ∴ tan( A + B ) =
1 − tan A ⋅ tan B
Hence, both statements are correct. = sin θ [cos 2 30°− sin 2 θ]
2 3
π π
= sin θ  − sin 2 θ = sin 3θ
3 1 π
110. (c) Given, A = and B = = 3 = 3 = tan
6 3  4  4 1− 1/ 3
π π 3
I. LHS sin A + sin B = sin + sin Hence, all three statements are correct.
6 3 ∴ A + B = π /3
1 3 1+ 3 114. (d) In right angled ∆ABC , Now, A + B + C = π
= + = π 2π
2 2 2 A + B + C = 180° ⇒ C = π − ( A + B) = π − =
I. sin( A + B ) = sin ( 180° − C ) = sin C 3 3
MATHEMATICS Measurements of Angles and Trigonometric Ratios 219

−1 / 2
119. (c) Now, sin C + cos C 1  sin 90°  128. (b) Given that, sin θ ⋅ cos θ =
3
…(i)
=  
= sin( π − ( A + B )) + cos( π − ( A + B )) (sin α sin β ) 1/2
°−α β 4
 cos( 90 ) sin 
[as A + B + C = π, C = π − ( A + B )] [from Eq. (i)] ∴ sin 4 θ + cos 4 θ
= sin( A + B ) − cos( A + B )
=
1
× (sin α sin β )1 / 2 = 1 = (sin 2 θ + cos 2 θ)2 − 2 sin 2 θ .cos 2 θ
= sin( π / 3) − cos( π / 3)
(sin α sin β ) 1/2
= ( 1)2 − 2 (sin θ ⋅ cos θ )2
3 1 3−1
= − = cot 30°+1 2
2 2 2 125. (c) I. = 2 (cos 30° + 1)  3 3 3 5
cot 30°−1 = 1− 2   = 1− 2. = 1− =
1 + tan 2 C  4  16 8 8
120. (b) 2 sin B = 3+1  3 
cosec 2C ⇒ =2 + 1
3−1  2  129. (d) Given that, θ lies is in first quadrant
1 + tan 2 ( 2 π / 3) and tanθ=3
= 3+1 3+1  3 + 2
cosec 2 ( 2 π / 3) ⇒ × =2  ∴ tan 2 θ = 9
3−1 3+1  2 
1 + tan ( π − π / 3)
2
On adding both sides by 1, we get
= 3 + 1+ 2 3
cosec 2 ( π − π / 3) ⇒ = 3+ 2 ⇒ 1 + tan 2 θ = 10
3−1
1 + tan π / 3 2
⇒ sec 2 θ= 10 ⇒ secθ= 10
2 sin B = 4+ 2 3
cosec 2 π / 3 ⇒ = 3+ 2
2 (since, θ lies in first quadrant)
1+ 3 4
= = ×3 = 3 2 (2 + 3) 1
4/3 4 ⇒ = 3+ 2 ⇒ cosθ = ...(i)
2 10
3 π π
sin B = = sin , B = ⇒ 3+ 2= 3+ 2 Q sin 2 θ = 1 − cos 2 θ = 1 −
1
=
9
2 3 3 10 10
So, it is true. 3
121. (c) Given that, sin x + cos x − 1 = 0
2 2
⇒ sinθ = ...(ii)
⇒ sin 2 x + cos 2 x = 1 II. 2 sin 45° cos 45° − tan 45° cot 45° = 0 10
3 1 4
⇒ 2 × 
which is an identity of trigonometric 1 1  Now, sin θ + cos θ = + =
×  – 1× 1= 0
ratio and always true for every real value  2 2 10 10 10
of x. 1
So, the equation has infinite solutions. or 2 × – 1× 1= 0 130. (d) If 0°< θ < 90°, then all the
2 trigonometric ratios can be obtained
122. (c) Given that, 3 sin x + 5 cos x = 5 when any one of the six ratios is given.
⇒ 1 − 1 = 0, It is true.
On squaring both sides, we get
Hence, both statements I and II are 131. (a) sin A .cos A . tan A + cos A . sin A . cot A
9 sin 2 x + 25 cos 2 x + 30 sin x cos x = 25 true. sin A cos A
= sin A . cos A . + cos A . sin A .
⇒ 9( 1− cos 2 x) + 25( 1− sin 2 x) 126. (c) I. We know that, sin 2 θ + cos 2 θ = 1 cos A sin A
+ 30 sin x cos x = 25 ∴ sin 2 1° + cos 2 1° = 1 = sin 2 A + cos 2 A
⇒ 9 + 25 − { 9 cos 2 x + 25 sin 2 x
II. We have sin θ 1 + cos θ
− 30 sin x cos x} = 25 132. (a) +
sec 2 33°− cot 2 57° 1 + cos θ sin θ
⇒ 9 = (3 cos x − 5 sin x)2
⇒ 3 cos x − 5 sin x = 3 = cosec 2 37°− tan 2 53° sin 2 θ + ( 1 + cos θ)2
=
LHS = sec 33°− cot 2 57°
2
sin θ ( 1 + cos θ)
123. (c) Given, p = a sin x + b cos x and
q = a cos x − b sin x = sec 2 ( 90°−57° ) − cot 2 57°
= cosec 2 57°− cot 2 57° = 1 sin 2 θ + 1 + cos 2 θ + 2 cos θ
⇒ p 2 = a 2 sin 2 x + b 2 cos 2 x =
RHS = cosec 2 37°− tan 2 53° sin θ ( 1 + cos θ)
+ 2ab sin x cos x
and q 2 = a 2 cos 2 x + b 2 sin 2 x = cosec 2 37°− tan 2 ( 90°−53° ) sin 2 θ + cos 2 θ + 1 + 2 cos θ
=
− 2 ab sin x cos x = cosec 2 37°− cot 2 37° = 1 sin θ ( 1 + cos θ)
∴ p 2 + q 2 = a 2 (sin 2 x + cos 2 x) + b 2 LHS = RHS 1 + 1 + 2 cos θ
(cos 2 x + sin 2 x) =
Hence, both statements are correct. sin θ ( 1 + cos θ)
=a + b
2 2
127. (b) I. Given that, sin x + cos x = 2 2 ( 1 + cos θ)
124. (b) Given, α + β = 90° ...(i) = = 2 cosecθ
⇒ (sin x + cos x) = 4
2 sin θ ( 1 + cos θ)
− 1/2
 sin α cos α 
∴ ( cosec α ⋅ cosec β )  +  ⇒ (sin 2 x + cos 2 x) + 2 sin x cos x = 4
 sin β cos β  133. (c) sec 2 D – tan 2 D = 1
⇒ 1 + sin 2x = 4 ⇒ sin 2x = 3
1 Since, it is a trigonometric identity.
= As maximum value of sinθ is 1.
(sin α sin β )1 / 2 134. (b) Given, cos A + cos 2 A = 1
− 1/2 Therefore, no value of x satisfy above
 sin α cos β + cos α sin β  ⇒ cos A = 1 − cos 2 A = sin 2 A ...(i)
  equation. Thus, statement I is false.
 sin β cos β 
II. sin x − cos x = 0 (Q sin 2 θ + cos 2 θ = 1)
− 1/2
1  sin(α + β )  ⇒ tan x = 1 = tan π / 4 Now, 2(sin A + sin 4 A )
2
=   It x lies in first quadrant, then
(sin α sin β )  sin β cos β 
1/2
tan x = tan π / 4 ⇒ x = π / 4 = 2 (sin 2 A + cos 2 A ) [from Eq. (i)]
Thus, statement II is true. = 2 ⋅ ( 1) = 2
220 CDS Pathfinder

135. (c) ( 1 − tan A )2 + ( 1 + tan A )2 + ( 1 − cot A )2 + ( 1 + cot A )2 140. (b) Now, cosec (C − D + B ) = cosec( 120° − 150° + 60° )
= 2 [ 1 + tan 2 A ] + 2[ 1 + cot 2 A ] = cosec (180° − 150° ) = cosec 30° = 2
[Q ( a − b ) + ( a + b ) = 2( a + b )]
2 2 2 2
141. (d) If ABCD is a cyclic quadrilateral, then sum of opposite angles
= 2 sec A + 2 cosec A = 2 
1 1  should be 180° but here
2 2
+ 
 cos 2 A sin 2 A  ∠A + ∠C = 30° + 120° = 150° ≠ 180°
and ∠B + ∠D = 60° + 150° = 210° ≠ 180°
 sin 2 A + cos 2 A  2 ⋅ ( 1)
=2  = = 2 sec 2 A ⋅ cosec 2 A So, statement I is incorrect.
 sin 2
A ⋅ cos 2
A  sin 2
A ⋅ cos 2 A
Now, sin ( B − A ) = cos (D − C )
1 + 2 sin θ ⋅ cos θ
136. (d) Given, a2 = ⇒ sin ( 60° − 30° ) = cos ( 150° − 120° )
1 − 2 sin θ ⋅ cos θ
1 3
(sin 2 θ + cos 2 θ) + 2 sin θ ⋅ cos θ ⇒ sin 30° = cos 30° ⇒ ≠
⇒ a2 = 2 2
(sin 2 θ + cos 2 θ) − 2 sin θ ⋅ cos θ So, statement II is also incorrect.
(sin θ + cos θ)2 a sin θ + cos θ 142. (a) Given, sin θ + cos θ = 3
⇒ a2 = ⇒ =
(sin θ − cos θ)2 1 sin θ − cos θ On squaring both sides, we get (sin θ + cos θ)2 = ( 3 )2
a + 1 (sin θ + cos θ) + (sin θ − cos θ) ⇒ sin 2 θ + cos 2 θ + 2 sin θ cos θ = 3 ⇒ 1 + 2 sin θ cos θ = 3
⇒ =
a − 1 (sin θ + cos θ) − (sin θ − cos θ) 3−1
⇒ sin θ cos θ = =1 ...(i)
(applying componendo dividendo formula) 2
sin θ cos θ sin θ + cos θ
2 2
1
a + 1 2 sin θ Now, tan θ + cot θ = + = =
⇒ = = tan θ cos θ sin θ sin θ cos θ sin θ cos θ
a − 1 2 cos θ
1
x2 − y 2  x2 − y 2 
2
From Eq. (i), tan θ + cot θ = =1
137. (b) Given, sin θ = ⇒ cos 2 θ = 1 − sin 2 θ = 1 −  2  1
x + y
2 2
 x + y2 
143. (a) cos ec ( 75° + θ ) − sec ( 15° − θ ) = cos ec ( 75° + θ )
( x2 + y 2 )2 − ( x2 − y 2 )2 4x 2 y 2  2xy 
2
− sec [ 90° − ( 75° + θ )]
= = 2 = 
( x2 + y 2 )2 ( x + y 2 )2  x2 + y 2  = cos ec ( 75° + θ ) − cos ec ( 75° + θ ) = 0

∴ cos θ = 2x y / x 2 + y 2 144. (c) Given, 5 sin θ + 12 cos θ = 13


On squaring both sides, we get
138. (b) Let f (θ) = sin θ + cos θ
25 sin 2 θ + 144 cos 2 θ + 120 sin θ cos θ = 169
Maximum and minimum value of a cos θ + b sin θ is
⇒ 25( 1 − cos 2 θ) + 144( 1 − sin 2 θ) + 120 sin θ cos θ = 169
− a 2 + b 2 ≤ a cosθ + b sin θ ≤ a2 + b2
⇒ 25 − 25 cos 2 θ + 144 − 144 sin 2 θ +120 sin θ cos θ = 169
∴ − 1 + 1 ≤ cos θ + sin θ ≤ 1+ 1 ⇒ 25 cos 2 θ + 144 sin 2 θ − 120 sin θ cos θ = 169 − 169
⇒ − 2 ≤ cos θ + sin θ ≤ 2 ⇒ ( 5 cos θ − 12 sin θ)2 = 0 ⇒ 5 cos θ − 12 sin θ = 0
for 0 ≤ θ ≤ 90°, 1 ≤ cosθ + sin θ ≤ 2 145. (c) Given, sin θ − cos θ = 0
Hence, statement I is false. ∴ sin θ = cos θ
1 Since, sin θ and cosθ are equal for θ = 45°.
And let g (θ) = tan θ + cot θ = tan θ + (Q AM ≥ GM)
tan θ ∴ sin 4 θ + cos 4 θ = (sin 45° )4 + (cos 45° )4
1 4 4
1 1 1+ 1 2 1
= 
tan θ + 1 1   1 
tan θ ≥  tan θ ⋅ 1  2  +   = + = = =
⇒  2  2 4 4 4 4 2
2  tan θ 
cos 2 ( 45°+ θ) + cos 2 ( 45°− θ)
⇒ (tan θ + cot θ) ≥ 2 146. (c)
tan( 60°+ θ) tan(30°− θ)
So, (tan θ + cot θ) is always greater than 1.
cos 2 ( 45°+ θ) + cos 2 [ 90°− ( 45°+ θ)]
Hence, statement II is true. =
tan( 60°+ θ) ⋅ tan[ 90°− ( 60°+ θ]
Solutions (Q. Nos. 139-141) Let the angle of A, B, C and D of a cos 2 ( 45°+ θ) + sin 2 ( 45°+ θ) 1
quadrilateral be x, 2x, 4x and 5x. = = =1
tan( 60°+ θ) ⋅ cot( 60°+ θ) 1
Q x + 2x + 4x + 5x = 360°
(Q sum of all interior in a quadrilateral is 360°) 147. (b) Given, tanθ + sec θ = m ⇒ tanθ = m − sec θ
⇒ 12x = 360° , x = 30° On squaring both sides, we get
∴ ∠A = x = 30° , ∠B = 2x = 60° ⇒ tan 2 θ = m 2 + sec 2 θ − 2m sec θ
∠C = 4x = 120° , ∠D = 5x = 150° ⇒ tan 2 θ = m 2 + 1 + tan 2 θ − 2m sec θ
m2 + 1
139. (a) Now, cos ( A + B ) = cos (30° + 60° ) = cos 90° = 0 ⇒ 0 = m 2 + 1 − 2m sec θ ⇒ sec θ =
2m
MATHEMATICS Measurements of Angles and Trigonometric Ratios 221

148. (c) Q sinα =


3
⇒ sin α = sin 60° 156. (b) cosec( 75° + θ ) − sec ( 15° − θ ) − tan( 55° + θ ) + cot( 35° − θ )
2 = cosec ( 75° + θ ) − sec [ 90° − ( 75° + θ )]
 3
⇒ α = 60° Q sin 60° =  − tan(55° + θ ) + cot[ 90° − ( 55° + θ )]
 2 
= cosec ( 75° + θ ) − cosec ( 75° + θ )
3  3
Now, cosβ = ⇒ β = 30° Q cos30° =  − tan( 55° + θ ) + tan( 55° + θ ) = 0
2  2 
157. (d) sin 25° sin 35° sec 65° sec 55°
and tan γ = 1 ⇒ γ = 45° (Q tan 45° = 1) = sin 25°⋅ sin 35°⋅
1

1
∴ α + β + γ = 60° + 30° + 45° = 135° cos 65° cos 55°
1 1
 1 + 1 − sin θ  cos θ = sin 25°⋅ sin 35° ⋅ ⋅
  cos( 90°−25° ) cos( 90°−35° )
( 1 + sec θ − tan θ) cos θ  cos θ cos θ 
149. (a) = 1 1
( 1 + sec θ + tan θ) ( 1 − sin θ)  1 + 1 + sin θ  ( 1 − sin θ) = sin 25° ⋅ sin 35° ⋅ ⋅ =1
  sin 25° sin 35°
 cos θ cos θ 
 cos θ + 1 − sin θ  cos θ 3
  158. (c) Given, 2 cot θ = 3 ⇒ cot θ =
 cos θ  2
= 3
(cos θ + 1 + sin θ)( 1 − sin θ) 2× −1
2 cos θ − sin θ 2 cot θ − 1 2 3−1 2 1
cos θ ∴ = = = = =
cos θ + 1 − sin θ 2 cos θ + sin θ 2 cot θ + 1 2 × 3 + 1 3 + 1 4 2
= 2
cos θ + 1 + sin θ − sin θ cos θ − sin θ − sin 2 θ
cos θ 159. (d) sin 6 θ + cos 6 θ = (sin 2 θ )3 + (cos 2 θ )3
cos θ + 1 − sin θ = (sin 2 θ + cos 2 θ )(sin 4 θ + cos 4 θ − sin 2 θ cos 2 θ )
= (Q 1 − sin 2 θ = cos 2 θ)
cos θ + 1 − sin 2 θ − sin θ cos θ [Q a 3 + b 3 = ( a + b )( a 2 + b 2 − ab )]
cos θ = (sin θ + cos θ ) − 2 sin 2 θ cos 2 θ − sin 2 θ cos 2 θ
2 2 2
cos θ + 1 − sin θ cos θ + 1 − sin θ
= = =1 1 3 1
= ( 1 − 3 sin 2 θ cos 2 θ ) = 1 − 3 × = 1 − =
cos θ + cos 2 θ − sin θ cos θ cos θ (cos θ + 1 − sin θ) 4 4 4
cos θ cos θ
160. (a) Given, cos x + sec x = 2 ...(i)
150. (c) In right angled ∆ABC , if ∠C is 90°, then 1
⇒ cos x + = 2 ⇒ cos x + 1 = 2 cos x
2
∠ A + ∠B = 180° − 90° = 90° cos x
Now, cos( A + B ) + sin( A + B ) = cos 90° + sin 90° = 0 + 1 = 1 ⇒ cos 2 x − 2 cos x + 1 = 0 ⇒ (cos x − 1)2 = 0 ⇒ cos x = 1
151. (a) Only statement I is correct as tanθ increases faster than sin θ as 1
∴ sec x = =1
θ increases while statement II is wrong as the value of sin θ + cos θ cos x
is not always greater than 1. It may also be equal to 1. ∴ cos n x + sec n x = ( 1)n + ( 1)n = 1 + 1 = 2
152. (a) Since, value of cosθ decreases, from 0° to 90° and at 45° it is 161. (c) Given, sin θ + 2 cos θ = 1
equal to the value of sin θ. On squaring both sides, we get (sin θ + 2 cos θ )2 = 1
Similarly, value of sin θ increases from 0 to 90° and at 45° it is ⇒ sin 2 θ + 4 cos 2 θ + 4 sin θ cos θ = 1
equal to the value of cosθ.
⇒ ( 1 − cos θ) + 4( 1 − sin 2 θ) + 4 sin θ cos θ = 1
2
For 0° < θ < 45°, cos θ > sin θ
⇒ − (cos 2 θ + 4 sin 2 θ) + 4 sin θ cos θ = 1 − 5
So, value of cos 25° − sin 25° is always positive but less than 1.
⇒ cos 2 θ + 4 sin 2 θ − 4 sin θ cos θ = 4
153. (b) In right angled ∆ABC, ∠B = 90°
⇒ ( 2 sin θ − cos θ)2 = 4 ⇒ 2 sin θ − cos θ = 2
∠C = 180°− ( ∠B + ∠A ) = 180°−90°−∠A = 90°−∠A
∴ sin C = sin( 90°− A ) = cos A = 4 / 5 162. (b) Given, tan 8θ = cot 2θ
154. (c) Since, α and β are complementary angle. ⇒ tan 8θ = tan( 90° − 2θ) ⇒ 8θ = 90° − 2θ ⇒ θ = 9°
∴ tan 5θ ⇒ tan 45° = 1
∴ α = 90° − β
cos α
Now, cos α cosec β − cos α sin β = − cos α sin β 163. (a) Q sin ( A + B ) = 1 = sin 90° ⇒ ( A + B ) = 90°
sin β ⇒ A = 90° − B
cos α Now, cos ( A − B ) = cos A cos B + sin A sin B
= − cos α cos( 90°−β )
cos( 90° − β ) = cos ( 90° − B ) cos B + sin ( 90° − B ) sin B
cos α = sin B cos B + cos B sin B = 2 sin B cos B = sin 2 B
= − cos α ⋅ cos α = 1 − cos 2 α = sin 2 α = sin α
cos α
12
164. (d) Clock will make right angle at ( 5n + 15) × min past n.
155. (d) Given, sec θ + tan θ = 2 ...(i) 11
By trigonometric identity, sec 2 θ − tan 2 θ = 1 Here, n = 3
⇒ (sec θ + tan θ)(sec θ − tan θ) = 1⇒ sec θ − tan θ = 1 / 2 ...(ii) ∴ ( 5 × 3 + 15) ×
12
min past 3 = 30 ×
12
min past 3
On adding Eqs. (i) and (ii), we get 11 11
8 8

1
2 sec θ = + 2 ⇒ sec θ =
5 = 32 min past 3 i.e. 3 h 32 min
2 4 11 11
222 CDS Pathfinder

165. (c) Given, tan θ + cot θ = 2 = (sin 2 x − cos 2 x ) [(sin 4 x + cos 4 x + 2 sin 2 x cos 2 x )
sin θ cos θ sin 2 θ + cos 2 θ − sin 2 x cos 2 x]
⇒ + =2 ⇒ =2
cos θ sin θ sin θ cos θ = (sin x − cos x ) (sin x + cos x + sin x cos 2 x )
2 2 4 4 2

1 1 = (sin 2 x )3 − (cos 2 x )3 [Q ( a − b )( a 2 + b 2 + ab ) = a 3 − b 3 ]
⇒ =2 ⇒ sin θ cos θ = ...(i)
sin θ cos θ 2
= sin 6 x − cos 6 x
Now, (sin θ + cos θ)2 = sin 2 θ + cos 2 θ + 2 sin θ cos θ
1
= 1+ 2 × = 1+ 1 [from Eq. (i)] 171. (d) (sin x ⋅ cos y + cos x ⋅ sin y ) (sin x ⋅ cos y − cos x ⋅ sin y )
2 = sin ( x + y ) ⋅ sin ( x − y ) = sin 2 x − sin 2 y
⇒ (sin θ + cos θ )2 = 2 ⇒ sin θ + cos θ = 2
[Q sin 2 A − sin 2 B = sin( A + B ) sin( A − B )]
sec x 1 / cos x
166. (a) = 172. (d) In ∆ABC , A + B + C = 180° ⇒ A + B = 180° –C
cot x + tan x cos x + sin x
sin x cos x tan( A + B ) = tan( 180° –C ) = − tan C
1 / cos x 1 sin x cos x tan A + tan B
= = × = sin x ⇒ = − tan C
cos 2 x + sin 2 x cos x 1 1 − tan A tan B
sin x cos x tan A + tan B + tan C = tan A tan B tan C
167. (b) ( 1 + cot x − cosec x )( 1 + tan x + sec x ) On dividing both sides by tan A tan B tan C
1 1 1
+ + =1
= ( 1 + cot x − cosec x ) 1 + + sec x
1
  tan B tan C tan A tan C tan A tan B
cot x
( 1 + cot x − cosec x)( 1 + cot x + cosec x) ⇒ cot B cot C + cot A cot C + cot A cot B = 1
=
cot x 1
173. (a) sin x + cosecx = 2 ⇒ sin x + = 2 ⇒ sin 2 x + 1 = 2 sin x
( 1 + cot x)2 − ( cosec x)2 sin x
=
cot x ⇒ sin 2 x − 2 sin x + 1 = 0 ⇒ (sin x − 1)2 = 0 ⇒ sin x = 1
12 + cot 2 x + 2 cot x − cosec 2 x cosec x =
1
=1
=
cot x sin x
1 + 2 cot x − ( cosec 2 x − cot 2 x) 1 + 2 cot x − 1 ∴ sin 9 x + cosec 9 x = ( 1)9 + ( 1)9 = 1 + 1 = 2
= = =2
cot x cot x 174. (b) Given, sin x + cos x = p ...(i)
168. (d) ( cosec x − sin x )(sec x − cos x )(tan x + cot x ) and sin 3 x + cos 3 x = q ...(ii)

=  − sin x  − cos x 


1 1 sin x cos x  On cubing Eq. (i) on both sides, we get
+ 
 sin x   cos x   cos x sin x  sin 3 x + cos 3 x + 3 sin x cos x (sin x + cos x ) = p 3
( 1 − sin x)( 1 − cos x)(sin x + cos x) cos x sin x × 1
2 2 2 2 2 2
⇒ q + 3 sin x cos x( p ) = p 3 [from Eqs. (i) and (ii)] …(iii)
= = =1
sin x cos x ⋅ sin x cos x sin 2 x cos 2 x On squaring Eq. (i) on both sides, we get
sin x − cos x + 1 (sin x − cos x) + 1 (sin x + cos x) + 1 sin 2 x + cos 2 x + 2 sin x cos x = p 2
169. (b) = ×
sin x + cos x − 1 (sin x + cos x) − 1 (sin x + cos x) + 1 p2 − 1
⇒ sin x cos x = (Q sin 2 x + cos 2 x = 1)
(sin x − cos x + 1)(sin x + cos x + 1) 2
= 3( p 2 − 1) p
(sin x + cos x)2 − 1 On putting this value in Eq. (iii), we get q + = p3
2
[Q ( a − b )( a + b ) = a 2 − b 2 ]
⇒ 2q + 3 p 3 − 3 p = 2 p 3 ⇒ p 3 − 3 p = − 2q
sin 2 x + sin x cos x + sin x − cos x sin x − cos 2 x
175. (d) Since, 1 radian = 57° 16′22′′
− cos x + sin x + cos x + 1
= So, sin 1°< sin 1 and cos 1°> cos 1
sin 2 x + cos 2 x + 2 sin x cos x − 1 Hence, neither statement I nor II is correct.
sin x + 2 sin x − cos 2 x + 1
2
= 176. (b) cosec 2 67°+ sec 2 57° –cot 2 33° –tan 2 23°
1 + 2 sin x cos x − 1
sin 2 x + 2 sin x − ( 1 − sin 2 x) + 1 = cosec 2 (90° –23° ) + sec 2 (90° –33° ) – cot 2 33° –tan 2 23°
= [Q cos 2 x = 1 − sin 2 x] = sec 2 23°+ cosec 2 33° – cot 2 33° – tan 2 23°
2 sin x cos x
sin 2 x + 2 sin x − 1 + sin 2 x + 1 = 1 + tan 2 23°+1 + cot 2 33° – cot 2 33° – tan 2 23° = 2
=
2 sin x cos x (Q 1 + tan 2 q = sec 2 q and 1 + cot 2 q = cosec 2 q )
2 sin 2 x + 2 sin x 2 sin x (sin x + 1) sin x + 1 177. (c) Given, tan ( A + B ) = 3 = tan 60°
= = = ∴ A + B = 60° ...(i)
2 sin x cos x 2 sin x cos x cos x
and tan A = 1 ⇒ tan A = tan 45°
170. (b) (sin 2 x − cos 2 x ) ( 1 − sin 2 x cos 2 x ) ∴ A = 45°
From Eq. (i), A + B = 60° ⇒ 45°+ B = 60° ⇒ B = 15°
= (sin 2 x − cos 2 x ) [(sin 2 x + cos 2 x )2 − sin 2 x cos 2 x]
Now, tan( A – B ) = tan( 45° – 15° ) = tan 30° = 1 / 3
(Q sin 2 x + cos 2 x = 1)
Hence, the value of tan( A – B ) is 1 / 3.
MATHEMATICS Measurements of Angles and Trigonometric Ratios 223

178. (d) Given, tan A + cot A = 4 ⇒ 3 – tan 2 A = k – 3k tan 2 A II. [( 1 – sin A ) – cos A ]2
On squaring both sides, we get ⇒ 3k tan A – tan A = k – 3
2 2 = ( 1 – sin A )2 + cos 2 A
(tan A + cot A )2 = ( 4)2 –2 cos A( 1 – sin A )
⇒ tan 2 A(3k – 1) = k – 3
= 1 + sin 2 A – 2 sin A + cos 2 A
⇒ tan 2 A + cot 2 A + 2 tan A cot A = 16 k –3
⇒ tan 2 A = –2 cos A( 1 – sin A )
⇒ tan 2 A + cot 2 A + 2 = 16 3k – 1
= 1 + sin 2 A + cos 2 A – 2 sin A
⇒ tan 2 A + cot 2 A = 14 k −3 1
where > 0 ⇒ k < or k > 3 –2 cos A( 1 – sin A )
Again, squaring both sides, we get 3k − 1 3 = 2 – 2 sin A – 2 cos A( 1 – sin A )
(tan 2 A + cot 2 A )2 = ( 14)2 Now, cosec A (3 sin A – 4 sin 3 A ) = 2 ( 1 – sin A ) – 2 cos A( 1 – sin A )
⇒ tan A + cot A + 2 tan A
4 4 2
= cosec A × sin A (3 – 4 sin 2 A ) = 2 ( 1 – sin A )( 1 – cos A ) ≠ RHS
cot A = 196
2 1
= × sin A (3 – 4 sin 2 A ) Hence, only statement I is correct.
⇒ tan 4 A + cot 4 A + 2 = 196 sin A 2a + 3b 2a
Q cosecA = 1  184. (a) We have, sinθ = = 1+
⇒ tan 4 A + cot 4 A = 194   3b 3b
 sinA  Since, sinθ is always smaller or equal to
179. (c) In right angled ∆ABC , AB : BC = 3: 4
 tan 2 A  1 but 1 +
2a
> 1.
AB 3 = 3 − 4 sin 2 A = 3 − 4  
or = C
 1 + tan A 
2 3b
BC 4 k −3 Hence, it is not possible.
Now, in ∆ABC 3−
3 − tan 2 A 3k − 1
4 = = 185. (a) Since, 0 ≤ cos 2 x ≤ 1
AC 2 = AB 2 + BC 2 1 + tan 2 A 1 + k − 3
3k − 1 Q − 1 ≤ cos 2 x + cos 2 y − cos 2 z ≤ 2
= 32 + 42 ∴ Minimum value of the given
A 3 B 9k − 3 − k + 3
= 9 + 16 = 25 = expression is − 1 .
BC 4 3k − 1 + k − 3
⇒ AC = 5 ⇒ sin A = = 8k 2k 186. (a) We have, tan θ + sec θ = 2
AC 5 = = ,
4k − 4 k − 1 ⇒ sec θ = 2 − tan θ
AB 3
sin B = 90° = sin 90° = 1, sinC = = 1
AC 5 where k < or k > 3 On squaring both sides, we get
3
4
Now, sin A + sin B + sin C = + 1 +
3 sec 2 θ = 4 + tan 2 θ − 4 tan θ
1 – sin x
5 5 182. (c) Given, p = ⇒ 1 + tan 2 θ = 4 + tan 2 θ − 4 tan θ
4 + 5 + 3 12 1 + sin x 3
= = ⇒ 4 tan θ = 3 ⇒ tan θ =
5 5 ( 1 – sin x)( 1 – sin x) 4
p=
180. (b) Given, sin x + cos x = C ( 1 + sin x)( 1 – sin x) cos θ 1 + sin θ
187. (b) We have, ×
On squaring both sides, we get 1 – sin x 1 – sin x 1 − sin θ 1 + sin θ
= = cos θ ( 1 + sin θ) cos θ ( 1 + sin θ)
(sin x + cos x)2 = C 2 1 – sin 2 x cos x = =
1 − sin 2 θ cos 2 θ
⇒ sin 2 x + cos 2 x + 2 sin x cos x = C 2 r=
cos x
×
( 1 – sin x) cos( 1 – sin x)
= 1 + sin θ
⇒ 1 + 2 sin x cos x = C 2 1 + sin x ( 1 – sin x) 1 – sin 2 x =
cos θ
C2 −1 cos x ( 1 – sin x) 1 – sin x
⇒ sin x cos x = …(i) r= = 188. (c) We have,
2 cos 2 x cos x
⇒ p=q=r tan( x + 40)° tan( x + 20)° tan(3x )°
Now, sin 6 x + cos 6 x = (sin 2 x)3
1 − sin x cos x tan( 70 − x )° tan(50 − x )° = 1
+(cos 2 x)3 Also, q × r = ×
cos x 1 + sin x [ Q cot( 90° − θ) = tan θ]
= (sin x + cos x)[sin x + cos 4 x
2 2 4
1 − sin x ⇒ tan( x + 40)° tan( x + 20)° tan(3x )°
– sin 2 x cos 2 x] = = p2
1 + sin x cot( 90° − 70° + x )°
[Q a + b = ( a + b )( a 2 + b 2 – ab )]
3 3
cot( 90° − 50° + x )° = 1
= 1 [(sin 2 x + cos 2 x)2 – 3 sin 2 x cos 2 x] Hence, both statements are correct.
cos A sin A ⇒ tan( x + 40)° tan( x + 20)° tan(3x )°
= ( 1 – 3 sin 2 x cos 2 x) 183. (a) I. LHS = +
1 – tan A 1 – cot A cot( 20 + x )° cot( 40 + x )° = 1
∴ sin 6 x + cos 6 x = 1 – 3 sin 2 x cos 2 x
cos A sin A ⇒ tan(3x )° = 1 ⇒ tan(3x )° = tan 45°
 C 2 – 1
2
= +
= 1 – 3  [from Eq. (i)] 1–
sin A
1–
cos A ⇒ 3x = 45 ⇒ x = 15
 2  π π
− 8 sec 2  
cos A sin A
189. (d) We have, 32 cot 2
 C 4 + 1 – 2C 2  cos 2 A sin 2 A 4 3
= 1 – 3  = + π
 4  cos A – sin A sin A – cos A + 8 cos 3  
4 – 3C 4 – 3 + 6C 2 1 + 6C 2 – 3C 4  6
= = cos 2 A – sin 2 A
=  3
3
4 4 (cos A – sin A ) = 32 ⋅ ( 1) − 8 ⋅ ( 2)2 + 8 ⋅  
3 – tan 2 A (cos A – sin A )(cos A + sin A )  2 
181. (c) Given, =k =
1 – 3 tan 2 A (cos A – sin A ) = 32 − 8 ⋅ ( 4) + 8 ⋅
3 3
⇒ 3 – tan 2 A = k( 1 – 3 tan 2 A ) = (cos A + sin A ) = RHS
8
= 32 − 32 + 3 3 = 3 3
224 CDS Pathfinder

2 2
190. (b) We have, x = a cosθ and y = b cot θ sin 35°   cos 55°  + 2 sin 30° 200. (c) Q 3 − tan 2 θ = α ( 1 − 3 tan 2 θ)
196. (c)   − 
 a b a b a b 2 2  cos 55°   sin 35°  ⇒ (3α − 1) tan 2 θ = α − 3
∴  −  +  = 2 − 2
x yx y x sin( 90°−55° ) 
2 α −3
= ⇒ tan 2 θ =
y
 cos 55°  3α − 1
= sec θ − tan θ = 1
2 2
α −3
cos( 90°−35° ) 
2 As, tan 2 θ ≥ 0, then ≥ 0 ⇒α≥3
−
3 1
191. (a) We have, sin θ cos θ=2 cos 3 θ − cos θ + 2× 3α − 1
2  sin 35°  2
⇒ α∈  −∞ ,  ∪ [3, ∞ )
1 1
or, α <
⇒ 2 sin θ = 4 cos 2 θ − 3 cos 55° 
2
 sin 35°  + 1
2
 3
=   − 
3
⇒ 2 sin θ = 4 − 4 sin θ − 3 2
 cos 55°   sin 35° 
201. (a) To exchange the position, both hands
⇒ 4 sin 2 θ + 2 sin θ − 1 = 0, = 12 − 12 + 1 = 1 has to cover 360° together.
−2 ± 4 + 16 −2 ± 2 5 4 Angle traced by hour hand in 1 min
⇒ sinθ = = 197. (c) We have, tan θ + cot θ =
8 8 3 = ( 1 / 2)°
−1 ± 5 sin θ cos θ 4 Angle traced by minute hand in 1 min = 6°
⇒ sinθ = ⇒ + =
4 cos θ sin θ 3 Let the required time be t min. Then,
Since, θ is acute angle 1 360 × 2
sin 2 θ + cos 2 θ 4 6t + t = 360° ⇒ t =
⇒ = 2 13
∴ sinθ > 0 cos θ ⋅ sin θ 3
720
5−1 1 4 = min = 55.38 min
⇒ sinθ = ⇒ = 13
4 sin 2θ 3
1 − cos θ 1 − cos θ 1 − cos θ
192. (d) Hours moved by hour hand = 5 hrs 2 202. (c) I. = ×
10 min 1 + cos θ 1 + cos θ 1 − cos θ
3
⇒ sin 2θ =
10 31 ( 1 − cos θ)2 ( 1 − cos θ)2
=5+ = hrs 2 = =
60 6 π π 1 − cos 2 θ sin 2 θ
⇒ 2θ = ⇒ θ =
Angle traced by hour hand in 1 hr = 30° 3 6 1 − cos θ
= = cosec θ − cot θ
∴ Angle traced by hour hand is π π sin θ
∴ sin θ + cos θ = sin + cos
 31 hrs = 30 × 31 = 155° 6 6 Hence, statement I is true.
 
 6 6 1 3 3+1 1 + cos θ 1 + cos θ 1 + cos θ
= + = II. = ×
193. (b) I. If 45°< θ < 90°, then sin θ > cos θ 2 2 2 1 − cos θ 1 − cos θ 1 + cos θ
∴ sin 66°> cos 66° 198. (b) We have, ( 1 + cos θ)2 1 + cos θ
= =
II. When 0°< θ < 45°, cos θ > sin θ p = cot θ + tan θ = cosecθ ⋅ sec θ sin 2 θ sin θ
∴ cos 26°> sin 26° q = sec θ − cos θ = sin 2 θ ⋅ sec θ = cosec θ + cot θ
Hence, statement II is true.
or sin 26°< cos 26°. ∴ ( p 2 q )2 / 3 − ( q 2 p )2 / 3
Hence only II is correct. = ( cosec 2θ ⋅ sec 2 θ ⋅ sin 2 θ ⋅ sec θ)2 / 3 cos 2 θ − 3 cos θ + 2
203. (b) Q =1
1 + tan 2 θ sec 2 θ sin 2 θ
194. (a) I. LHS = = − (sin 4 θ ⋅ sec 2 θ ⋅ cosecθ ⋅ sec θ)2 / 3
1 + cot θ cosec 2θ
2 ⇒ cos 2 θ − 3 cos θ + 1 + 1 − sin 2 θ = 0
= (sec 3 θ)2 / 3 − (sin 3 θ ⋅ sec 3 θ)2 / 3
= tan 2 θ ⇒ cos 2 θ − 3 cos θ + 1 + cos 2 θ = 0
= sec 2 θ − sin 2 θ ⋅ sec 2 θ
1 − tan θ 
2
 tan θ[cot θ − 1] 
2 ⇒ 2 cos 2 θ − 3 cos θ + 1 = 0
RHS =   = = sec 2 θ( 1 − sin 2 θ) = 1
 1 − cot θ   1 − cotθ  ⇒ ( 2 cos θ − 1) (cos θ − 1) = 0
x y 1
199. (d) Given, − tan θ = 1 ...(i) ⇒ cos θ = or, cos θ = 1
= ( − tan θ)2 = tan 2 θ a b 2
∴ LHS = RHS x y π
and tan θ + = 1 ...(ii) ⇒ θ = [Q 0 < θ < π / 2]
1 a b 3
II. cot θ = ⇒ tan θ ⋅ cot θ = 1
tan θ On solving Eqs. (i) and (ii), we get Hence, only II is correct.
which true for all value of θ. x 1 + tan θ y 1 − tan θ 204. (d) I. As, cos x lies between −1and 1, then
= and =
So, statement II is false. a sec θ
2
b sec 2 θ cos x = 2m + 1 does not exist for positive
Hence, only statement I is correct. value of m.
x 2 y 2 ( 1 + tan θ)2 + ( 1 − tan θ)2 II. The given relation m n ≥ m + n is true
195. (d) We have, A + B + C = π ∴ + =
π a2 b2 sec 4 θ for m , n ∈ N and m > 1 and n > 1.
⇒ + B + B + C = π [A = B + π/ 2] 2 sec 2 θ
2 = Thus, statement II is not true.
π π sec 4 θ
⇒ 2B + C = π − = Hence, neither I nor II is correct
2 2 = 2 cos 2 θ
21
MATHEMATICS > Height and Distance 225

HEIGHT AND DISTANCE


Regularly (1-2) questions have been asked from this chapter. Generally this is the simplest application
of trigonometry related to our real world and a little practice can get you command over this section.

It is an important application of trigonometry which helps


us to find the height of any object and distance of that IMPORTANT POINTS
object from any point which are not directly measurable. If l If the observer moves towards the perpendicular line
the angle of elevation/depression from a point is known. (tower/building), then angle of elevation increases
and if the observer moves away from the perpendicular
Line of sight line (tower/building), then the angle of elevation
decreases.
A line of sight is the line drawn from the eye of an observer
l If the angle of elevation of Sun above a tower decreases,
to the point, where the object is viewed by the observer. then the length of shadow of a tower increases.
D
l If height of tower is doubled and the distance between
ig ht the observer and foot of the tower is also doubled, then
o fs
e Tower the angle of elevation remains same.
Lin
Horizontal line EXAMPLE 1. The angle of elevation of the top of a
A E
Person tower of height 100 3 m from a point at a distance
B C
of 100 m from the foot of the tower on a horizontal
Horizontal Line plane is
The line of sight which is parallel to ground level is a. 30° b. 60° c. 75° d. 105°
known as horizontal line. Sol. b. Let φ be the angle of elevation.
Given, AC = Height of tower
ANGLE OF ELEVATION = 100 3 m C
Angle of elevation is defined as an angle formed by the
and AB = 100 m
line of sight with the horizontal
(Object) P In right angled ∆ABC,
line when an object is viewed in 100√3 m
the upward direction. ht AC 100 3
ig s tan φ = = = 3
of AB 100 φ
Let P be the position of the object e B A
above the horizontal line OA and Lin Angle of ⇒ tan φ = tan 60° 100 m
θ elevation
O be the eye of the observer, then Eye ⇒ φ = 60°
O Horizontal line A
∠AOP is called angle of elevation. Hence, angle of elevation is 60°.
226 CDS Pathfinder

EXAMPLE 2. The angle of elevation of a tower at a


point is 45°. After going 40 m towards the foot of the ANGLE OF DEPRESSION
tower, the angle of elevation of the tower becomes Angle of depression is defined as an angle formed by
60°. Then, the height of the tower is the line of sight with the horizontal line when an object
a. 20( 3 + 1) m b.
20 3
m
is viewed in the downward direction.
3+1 Horizontal line
A
O
c.
40 3
m d.
40 3
m θ Angle of
depression
3+1 3 −1

Lin
e
P

of
Sol. d. Let PQ ( = h) be the height of the

s ig
tower. Let S and R be the points

ht
P
where the angles subtended are
h Let P be the position of the object below the horizontal
45° and 60°, respectively.
line OA and O be the eye of the observer, then ∠AOP
In right angled ∆PQR ,
60°
is called angle of depression.
PQ 45°
tan 60° = Q
RQ S R
40 m x
IMPORTANT POINTS
h h
⇒ 3= ⇒ x= …(i)
x 3 l The angle of depression of a point P as seen from a
point O is numerically equal to the angle of elevation of O
PQ PQ h
In right angled ∆PQS, tan 45° = = = as seen from P .
SQ SR + RQ 40 + x
l Angle of elevation and angle of depression are always
h acute angle.
⇒ h = 40 + x ⇒ h = 40 + [from Eq. (i)]
3 l Unless mentioned, the height of the observer is not
 1  3 − 1 considered.
⇒ h 1 −  = 40 ⇒ h   = 40
 3  3 
EXAMPLE 4. The angle of depression of two ships
40 3 from the top of a light house are 45° and 30° towards
∴ h= m
3 −1 East. If the ships are 200 m apart, the height of the
light house is
EXAMPLE 3. The angles of elevation of the top of a 200 200 100 100
a. m b. m c. m d. m
tower from two points at distances ‘a’ and ‘b’ (a > b) 3 −1 3+1 3 −1 3+1
from the base and in the same straight line with it are
complementary. Then, the height of the tower is Sol. a. Let AB( = h) be the X A
height of the light house 30°
1 a 45°
a. b. ab c. d. ab and D, C be the ships.
ab b
h
A Given, DC = 200 m
Sol. b. Let AD = h be the height of the
∠ADC = ∠XAD = 30°
tower and the angle of elevation 30° 45°
of the top of tower at B and C be and ∠ACB = ∠XAC = 45° D C y B
h 200 m
(90° − θ) and θ, respectively. Now, in ∆ACB,
θ AB h
In right angled ∆ADC, 90
°– θ tan 45° = ⇒1= ⇒ h=y …(i)
AD h  B CB y
Q tan θ = 
P C b D
tan θ = = a
CD b  B  and in ∆ADB, tan 30º =
AB

1
=
h
h = b tan θ …(i) BD 3 200 + y
AD h ⇒ 200 + y = 3 h ⇒ 200 + h = 3 h [from Eq. (i)]
In right angled ∆ADB, tan (90° − θ) = ⇒ cot θ =
BD a ⇒ 200 = 3 h − h ⇒ 200 = h ( 3 − 1)
[Q tan (90° − θ) = cot θ]  200 
∴ h=  m
h a  3 − 1
⇒ = cot θ ⇒ = tan θ …(ii)
a h
From Eqs. (i) and (ii), we get
EXAMPLE 5. From the top of a building 60 m high
the angles of depression of the top and bottom of a
h = b   ⇒ h2 = ab
a
 h
tower are observed to be 30° and 60°. Then, the
height of the tower is
∴ h = ab a. 10 m b. 20 m c. 30 m d. 40 m
MATHEMATICS > Height and Distance 227

Sol. d. Let x be the height of the tower. AB 60


= tan 60° ⇒ = 3
Here, AB = 60 m and BD = y m BD y
In right angled ∆ABD, 60
⇒ y= …(i)
A 3
60°
30° In right angled ∆AEC,
AE AE 1
= tan 30° ⇒ = [QCE = DB]
C 30° EC BD 3
y E AE 1 AE 1
⇒ = ⇒ = [from Eq. (i)]
60 m
y 3 60 3
3
x 1 60
⇒ AE = × = 20 m [Q 3 × 3 = 3]
3 3
60° ∴ x = AB − AE = 60 − 20 = 40 m
D y B Hence, the height of tower is 40 m

PRACTICE EXERCISE
1. The length of shadow of a tree is 16 m when the 7. An aeroplane flying horizontally 1 km above the
angle of elevation of the Sun is 60°. What is the ground is observed at an elevation of 60°. After
height of the tree? 10 s, its elevation is observed to be 30°, the
(a) 8 m (b) 16 m (c) 16 3 m (d) 3m uniform speed of the aeroplane in km/h is
240
2. From a light house the angles of depression of (a) (b) 240 3
2
two ships on opposite sides of the light house are
observed to 30° and 45°. If the height of light (c) 240 (d) 260 3
house is h, then what is the distance between 8. A person of height 2m wants to get a fruit which
the ships? is on a pole of height (10 / 3) m. If he stands at a
(d)  1 +
1  distance of ( 4 / 3 ) m from the foot of the pole,
(a) ( 3 + 1) h (b) ( 3 − 1) h (c) 3h h
 3 then the angle at which he should throw the
3. The angle of elevation of the top of a tower from stone, so that it hits the fruit is
the bottom of a building is twice that from its (a) 60° (b) 45° (c) 90° (d) 30°
top. What is the height of the building, if the
height of the tower is 75 m and the angle of 9. A vertical tower stands on a horizontal plane
elevation of the top of the tower from the bottom and is surmounted by a vertical flagstaff of
of the building is 60°? height h. At a point on the plane, the angle of
elevation of the bottom of the flagstaff is α and
(a) 25 m (b) 37.5 m (c) 50 m (d) 60 m
that of the top of the flagstaff is β. Then, the
4. A radio transmitter antenna of height 100 m height of the tower is
stands at the top of a tall building. At a point on h tanβ h tan α
(a) (b)
the ground, the angle of elevation of bottom of tan α − tan β tan α + tan β
the antenna is 45° and that of top of antenna is h tan β h tan α
60°. What is the height of the building? (c) (d)
tan α + tan β tan β − tan α
(a) 100 m (b) 50 m (c) 50 ( 3 + 1) m (d) 25 3 m
5. The shadow of a tower is 15 m when the Sun’s 10. From a window (h m high above the ground) of a
house in a street, the angle of elevation and
altitude is 30°. What is the length of the shadow
depression of the top and the foot of another
when the Sun’s altitude is 60°?
house on the opposite side of the street are θ and
(a) 3 m (b) 4 m (c) 5 m (d) 6 m φ, respectively. Then, the height of the opposite
6. The angle of elevation of a moon when the length house is
of the shadow of a pole is equal to its height is (a) h tan θ cot φ (b) h [tan θ cot φ + 1]
(a) 60° (b) 45° (c) 90° (d) 30° (c) h [cot θ tan φ + 1] (d) h cot θ tan φ
228 CDS Pathfinder

11. The length of the shadow of a person s cm tall 18. The angle of elevation of the top of an
when the angle of elevation of the Sun is α is incomplete vertical pillar at a horizontal
p cm. It is q cm when the angle of elevation of the distance of 100 m from its base is 45°. If the
Sun is β. Which one of the following is correct angle of elevation of the top of complete pillar at
when β = 3 α? the same point is to be 60°, then the height of
 tan α − tan 3α   tan 3α − tan α  the incomplete pillar is to be increased by
(a) p − q = s   (b) p − q = s  
 tan 3α tan α   3 tan 3α tan α  (a) 50 2 m (b) 100 m
(c) 100 ( 3 − 1) m (d) 100 ( 3 + 1) m
 tan 3α − tan α   tan 2 α 
(c) p − q = s   (d) p − q = s  
 tan 3α tan α   tan 3α tan α  19. A man on the top of a vertical observation tower
observes a car moving at a uniform speed
12. Two posts are ‘K ’ m apart and the height of one coming directly towards it. If it takes 12 min for
is double that of the other. If from the middle the angle of depression to change from 30° to 45°,
point of the line joining their feet, an observer how soon after this will the car reach the
finds the angular elevations of their tops to be observation tower?
complementary, then the height (in metre) of the
(a) 14 min 35 s (b) 15 min 49 s
shorter post is
K K (c) 16 min 23 s (d) 18 min 5 s
(a) 3K (b) (c) 2 K (d)
2 2 2 20. You are stationed at a radar base and you
13. A flagstaff 5 m high stands on a building of 25 m observe an unidentified plane at an altitude
height. At an observer at a height of 30 m, the h = 1000 m flying towards your radar base at an
flagstaff and the building subtend equal angles. angle of elevation = 30°. After exactly one
The distance of the observer from the top of the minute, your radar sweep reveals that the plane
flagstaff is is now at an angle of elevation = 60° maintaining
3 5 5 3 3 the same altitude. What is the speed (in m/s) of
(a) m (b) 3 m (c) m (d) 5 m the plane?
2 2 2 2
(a) 15.58 m/s (b) 19.25 m/s
14. A man is watching from the top of a tower a boat
(c) 18 m/s (d) 11.25 m/s
speeding away from the tower. The boat makes
an angle of depression of 45° with the man’s eye 21. The angle of elevation of the top of the tower
when at a distance of 60 m from the bottom of from a point on the ground is sin −1 (3/5). If the
tower. After 5 s, the angle of depression becomes point of observation is 20 m away from the foot
30°. What is the approximate speed of the boat of the tower, what is the height of the tower?
assuming that it is running in still water? (a) 9 m (b) 18 m (c) 15 m (d) 12 m
(a) 31.5 km/h (b) 36.5 km/h (c) 38.5 km/h (d) 40.5 km/h 22. A balloon of radius r makes an angle α at the
15. Suppose the angle of elevation of the top of a tree eye of an observer and the angle of elevation of
at a point E due East of the tree is 60° and that its centre is β. The height of its centre from the
at a point F due West of the tree is 30°. If the ground level is given by
distance between the points E and F is 160 ft, (a) r sin β cosec α /2 (b) r cosec α /2 sin α
then what is the height of the tree? (c) r cosecα sin β (d) None of these
40
(a) 40 3 ft (b) 60 ft (c) ft (d) 23 ft
3 Directions (Q. Nos. 23-25) The height of a tower is h
16. A telegraph post gets broken at a point against a and the angle of elevation of the top of the tower is
storm and its top touches the ground at a α . When observer on moving a distance h / 2
distance 20 m from the base of the post making towards the tower, the angle of elevation becomes β.
an angle 30° with the ground. What is the height 23. What is the value of cot α − cot β?
of the post?
(a) 1 / 2 (b) 2 / 3 (c) 1 (d) 2
40
(a) m (b) 20 3 m (c) 40 3 m (d) 30 m
3 24. If α = 30°, then find the value of cot β.
2
17. If the angle of elevation of a cloud from a point (a) 3 − 1 / 2 (b)
2 3 −2
h m above a lake is β and the angle of depression
1 1
of its reflection in the lake is α, then the height of (c) (d)
1+ 3 1− 3
the cloud is
h cosec (α − β ) 25. If tan β = 4, then distance between observer and
(a) (b) h cosec (α − β ) sin (α − β )
cosec (α − β ) tower when angle of elevation is β
h cosec (α + β ) 3h h
(c) h sin (α + β ) cosec (α − β ) (d) (a) h (b) h / 4 (c) (d)
sin (α − β ) 4 3
MATHEMATICS > Height and Distance 229

PREVIOUS YEARS’ QUESTIONS 34. What is the angle of elevation of the Sun, when
x
26. What is the angle of elevation of the Sun when the the shadow of a pole of height x m is m?
shadow of a pole is 3 times the length of the 3
e 2013 II
pole? e 2012 I (a) 30° (b) 45° (c) 60° (d) 75°
(a) 30° (b) 45°
(c) 60° (d) None of these 35. A spherical balloon of radius r subtends angle
60° at the eye of an observer. If the angle of
27. From the top of a cliff 200 m high, the angles of elevation of its centre is 60° and h is the height
depression of the top and bottom of a tower are of the centre of the balloon, then which one of
observed to be 30° and 45°, respectively. What is the following is correct? e 2013 II
the height of the tower? e 2012 I
(a) h = r (b) h = 2 r
(a) 400 m (b) 400 3 m
(c) h = 3 r (d) h = 2 r
(c) 400 / 3 m (d) None of these
36. The angle of elevation of the top of a tower 30 m
28. The angles of elevation of the top of a tower from high from the foot of another tower in the
two points which are at distances of 10 m and same plane is 60° and the angle of elevation
5 m from the base of the tower and in the same of the top of the second tower from the foot of
straight line with it are complementary. The the first tower is 30°. The distance between
height of the tower is e 2012 II
the two towers is m times the height of the
(a) 5 m (b) 15 m (c) 50 m (d) 75 m shorter tower. What is m equal to? e 2014 I
29. A ladder 20 m long is placed against a wall, so 1 1
(a) 2 (b) 3 (c) (d)
that the foot of the ladder is 10 m from the wall. 2 3
The angle of inclination of the ladder to the 37. The shadow of a tower standing on a level
horizontal will be e 2012 II plane is found to be 50 m longer when the
(a) 30° (b) 45° Sun’s elevation is 30°, then when it is 60°.
(c) 60° (d) 75° What is the height of the tower? e 2014 I
25
(a) 25 m (b) 25 3 m (c) m (d) 30 m
Directions (Q. Nos. 30-33) Read the following 3
information carefully to answer the questions that 38. From a certain point on a straight road, a person
follow. observe a tower in the West direction at a
As seen from the top and bottom of a building of height distance of 200 m. He walks some distance along
h m, the angles of elevation of the top of a tower of the road and finds that the same tower is 300 m
(3 + 3)h South of him. What is the shortest distance of
height m are α and β, respectively. the tower from the road? e 2014 II
2 e 2013 I
300 500 600 900
30. If β = 30°, then what is the value of tan α? (a) m (b) m (c) m (d) m
13 13 13 13
(a) 1/2 (b) 1/3
(c) 1/4 (d) None of these 39. If from the top of a post a string twice the length
of the post is stretched tight to a point on the
31. If α = 30°, then what is the value of tan β? ground, then what angle will the string make
(a) 1 (b) 1/2 with the post? e 2014 II
(c) 1/3 (d) None of these π π 5π π
(a) (b) (c) (d)
6 4 12 3
32. If α = 30° and h = 30 m, then what is the
distance between the base of the building and 40. The angle of elevation of a cloud from a point
the base of the tower? 200 m above a lake is 30° and the angle of
depression of its reflection in the lake is 60°. The
(a) (15 + 15 3 ) m (b) (30 + 15 3 ) m height of the cloud is e 2015 I
(c) (45 + 15 3 ) m (d) None of these (a) 200 m (b) 300 m (c) 400 m (d) 600 m
33. If β = 30° and if θ is the angle of depression of the 41. From the top of a tower, the angles of depression
foot of the tower as seen from the top of the of two objects P and Q (situated on the ground
building, then what is the value of tan θ? on the same side of the tower) separated at a
(3 − 3) (3 + 3 ) distance of 100( 3 − 3 ) m are 45° and 60°,
(a) (b)
3 3 3 3 respectively. The height of the tower is e 2015 I
(2 − 3) (a) 200 m (b) 250 m
(c) (d) None of these
3 3 (c) 300 m (d) None of these
230 CDS Pathfinder

42. The angles of elevation of the top of a tower from 44. An aeroplane flying at a height of 3000 m passes
two points P and Q at distance m 2 and n 2 vertically above another aeroplane at an instant,
respectively, from the base and in the same when the angles of elevation of the two planes
straight line with it are complementary. The from some point on the ground are 60° and 45°,
height of the tower is e 2015 I respectively. Then, the vertical distance between
(a) (mn)1/ 2 (b) mn1/ 2 the two planes is e 2015 II
1/ 2 (a) 1000 ( 3 − 1) m (b) 1000 3 m
(c) m n (d) mn
(c) 1000 (3 − 3 ) m (d) 3000 3 m
43. A pole is standing erect on the ground which is 45. Two observers are stationed due North of a
horizontal, The tip of the pole is tied tight with a
tower (of height x m) at a distance y m from each
rope of length 12 m to a point on the ground. If
other. The angles of elevation of the tower
the rope is making 30° with the horizontal, then observed by them are 30° and 45°, respectively.
the height of the pole is e 2015 II
Then, x / y is equal to e 2016 I
(a) 2 3 m (b) 3 2 m 2 −1 3 −1 3+1
(c) 3 m (d) 3 m (a) (b) (c) (d) 1
2 2 2

ANSWERS
1 c 2 a 3 c 4 c 5 c 6 b 7 b 8 d 9 d 10 b
11 c 12 b 13 d 14 a 15 a 16 b 17 c 18 c 19 c 20 b
21 c 22 a 23 a 24 a 25 b 26 a 27 d 28 c 29 c 30 b
31 a 32 c 33 a 34 c 35 c 36 b 37 b 38 c 39 d 40 c
41 c 42 d 43 d 44 c 45 c

HINTS AND SOLUTIONS 


1. (c) Let the height of the tree be h m. ∴ y=h …(i) and in right angled ∆BCE ,
∴ BC = h m and AB = 16 m and in right angled ∆PTA , CE 75
tan 60° = ⇒ 3=
h x
C tan30° = ⇒ x = 3h …(ii) BC
x ⇒ x 3 = 75
∴ Required distance, AB = x + y ⇒( 75 3 − h 3 ) 3 = 75 [from Eq. (i)]
hm x + y = 3h + h = h ( 3 + 1) m ⇒ 75 × 3 − 3h = 75
75 × 2
3. (c) Let height of the building be h m ⇒ 3h = 75 × 3 − 75 ⇒ h =
60° and distance between building and 3
A B ∴ h = 50 m
16 m tower be x m.
In right angled ∆ABC , ∴ AB = h m and BC = x m Hence, the height of the building is
BC h h E 50 m.
tan 60° = = ⇒ 3=
(75 – h) m

AB 16 16 4. (c) Let BC be a building of height h m


⇒ h = 16 3 m and CD be a height of antenna. and let
Hence, the height of the tree is 16 3 m. distance between A and B = x m
30°
A x D 75 m D
2. (a) In right angled
h h
∆PBT, tan 45° = ⇒ 1=
y y h 100 m
P
30° 45° 60° C
B x C
In right angled ∆ADE,
h 60° hm
ED 1 75 − h
tan 30° = ⇒ =
30° 45° AD 3 x 45°
A xm B
A x T y B ⇒ x = 75 3 − h 3 …(i)
MATHEMATICS > Height and Distance 231

In right angled ∆ABC , 7. (b) Let A be the observer. When the 9. (d) Let BC be the tower and CD be the
BC h aeroplane was at point B the angle of flagstaff.
tan 45° = = ⇒x = h ...(i)
AB x elevation was 60° and after 10 s when ∴ ∠BAC = α and ∠BAD = β
and in right angled ∆ABD, it was at point C the angle of elevation
was 30°. In right angled ∆ABC ,
DB DC + CB 100 + h
tan 60° = = = Here, in the figure, D
BA AB x BE = CD = 1 km = 1000 m
100 + h
⇒ 3= [from Eq. (i)] B C h
h
⇒ 3h = 100 + h 1 km 1 km C
100
⇒ ( 3 − 1) h = 100 ⇒ h = 60° β
3−1 30°
A D A α
E B
100 ( 3 + 1)
⇒ h= × In right angled ∆BAE, BC
( 3 − 1) ( 3 + 1) BE 1000 tan α = …(i)
tan 60° = = AB
= 50 ( 3 + 1) m AE AE
and in right angled ∆ABD,
Hence, the height of the building is ⇒ AE = 1000 cot 60°
BD BC + h
=
1000 = = tan β …(ii)
50 ( 3 + 1) m. m …(i) AB AB
3
5. (c) Let the height of the tower be h m On dividing Eq. (ii) by Eq. (i), we get
and length of the shadow ( BC ) be x m. In right angled ∆ACD,
DC 1000 BC + h tan β
In right angled ∆ACD, tan 30° = = =
CD AD AD BC tan α
tan 30° =
AC ⇒ AD = 1000 cot 30° = 1000 3 ⇒ ( BC + h ) tan α = BC tan β
1 h 15 ∴ ED = AD − AE ⇒ BC (tan β − tan α ) = h tan α
⇒ = ⇒h= m …(i) 1000 2000
3 15 3 = 1000 3 − = m h tan α
D 3 3 ∴ BC =
tan β − tan α
So, distance travelled by plane in 10 s,
then
2000 10. (b) Let O be the window and AB be
h BC = DE = m
3 the house on the opposite side of the
Distance street.
∴ Speed of aeroplane =
30° 60° Time A
A x 2000
B C
2000 × 60 × 60
15 m = 3 m/s = km/h h´
and in right angled ∆BCD, 10 10 3 × 1000 O θ
φ P
tan 60° =
CD
⇒ 3= ⇒
h h
=x = 240 3 km/h
BC x 3 Hence, the uniform speed of the h h
15 aeroplane is 240 3 km/h.
∴ x= =5m O´ B
3
8. (d) Let TP be the man and SQ be the
[from Eq. (i)] Let AP = h ′
pole. S
Hence, the length of shadow is 5 m. and BP = OO ′ = h
When sun’s altitude is 60°.
T θ In right angled ∆AOP,
R 10/3 m
6. (b) Let the height of pole be h m, then AP h′
2m tan θ = = …(i)
PQ = RQ = h OP OP
In right angled ∆PRQ , Q
P 4/√3 m and in right angled ∆BOP,
moon
R PB h
Let ∠STR = θ tan φ = = …(ii)
OP OP
Now, SR = SQ − RQ
h h ′ tan θ
pole 10 4 ∴ = [by Eq. (i) ÷ Eq. (ii)]
= − 2 = m [QRQ = TP] h tan φ
θ
3 3
P Q In right angled ∆STR , ⇒ h ′ = h tan θ cot φ
h
RQ h tan θ =
SR SR
= [Q TR = PQ] ∴ Height of the house
tan θ = = =1 TR PQ
PQ h AB = AP + PB = h + h ′
⇒ tan θ = 1 = tan 45° ∴ θ = 45° 4 3 1
= × = = tan 30° = h tan θ cot φ + h
Hence, the angle of elevation of moon 3 4 3
= h [tan θ cot φ + 1]
is 45°. ∴ θ = 30°
232 CDS Pathfinder

11. (c) In right angled ∆BAC , On multiplying Eq. (i) by Eq. (ii), we get In right angled ∆ACB, tan 45° =
AB
BC s s 4h 2h AC
tan α = = ⇒p = …(i) ⇒ × =1
tanα h
AC p K K ⇒ 1= ⇒ h = 60 m ...(i)
K2 60
B ⇒ 8h 2 = K 2 ⇒ h 2 =
8 Now, in right angled ∆ADB,
K 60
∴ h= m tan 30° =
AB
=
AB
=
2 2 AD AC + CD 60 + x
Hence, the height of shorter post is
[from Eq. (i)]
s K
m. 1 60
2 2 ⇒ = ⇒ 60 + x = 60 3
3 60 + x
13. (d) Let O be an observer at a height ⇒ x = 60( 3 − 1) = 60( 173
. − 1)
30 m. Let x be the distance of observer
= 60 × 0.73 = 438
. m
α β from the top of the flagstaff.
A D q C Now, given time = 5 s
i.e. OC = x
p Distance
x C We know that, Speed =
O
α Time
In right angled ∆BDC , α 5m 438
. 18 788.4
BC s s B ∴ Speed of boat = × =
tan β = = ⇒q= …(ii) 5 5 25
DC q tan 3α
30 m 25 m = 31.5 km/h
[Q β = 3α, given] 15. (a) Let AC = h ft = Height of a tree
On subtracting Eq. (ii) from Eq. (i), we A and x ft = Distance between A and F
get then AE = ( 160 − x) ft
s s Here, AB be the tower of the flag staff
p−q= − = 25 m and BC be the flag staff = 5 m In right angled ∆AFC ,
tan α tan 3α
 tan 3α − tan α  In right angled ∆OBC , tan 30° =
AC h
= ⇒
1
=
h
= s 
tan α =
BC 5
=
AF x 3 x
 tan α tan 3α 
  OC x ⇒ x = 3h …(i)
⇒ x = 5 cot α ...(i) C N
12. (b) Let PQ and RS be the two posts such
In right angled ∆OCA,
that PQ = 2RS [given] h W E
AC 30
Q tan 2 α = = [Q AC = AB + BC ]
OC x F 30° 60° E
S 30 x A (160 – x) S
∴ tan 2 α = [from Eq. (i)] 160 ft
2h 5 cot α
and in right angled ∆AEC ,
h ⇒ tan 2α = 6 tan α
θ 90°– θ tan 60° =
AC
=
h
2 tan α
R M P ⇒ = 6 tan α AE 160 − x
K 1 − tan 2 α
h
  ⇒ 3= [Qtan 60° = 3]
Given, M is the mid-point of RP. 2 tan α 160 − x
Q tan 2α = 
K  1 − tan α 
2 ⇒ 3( 160 − x) = h
∴ RM = PM = [Q RP = K given]  
2 2 ⇒ 3( 160 − 3h ) = h [from Eq. (i)]
⇒ 3 − 3 tan 2 α = 1 ⇒ tan 2 α =
Let ∠RMS = θ 3 ⇒ 160 3 − 3h = h ⇒ 4h = 160 3
∴ ∠QMP = 90° − θ 2 ∴ h = 40 3 ft
⇒ tan α = or cot α = 3 / 2
Let RS = h, then PQ = 2h 3 Hence, the height of the tree is 40 3 ft.
3
Now, in right angled ∆PMQ, or x=5 m [from Eq. (i)] 16. (b) Let the height of the post be h m, and
PQ
2 AB = x m.
tan ( 90° − θ) = 14. (a) Let height of towerAB be h m and Given, BC = 20 m
MP D
2h 4h distance between C and D be x m.
⇒ cot θ = =
(h – x) m

k/2 k B
4h 30°
45°
or cot θ = …(i)
K
A hm
In right angled ∆SRM, hm (h – x) m
SR h
tanθ = =
RM K / 2 xm
2h 30°
⇒ = tan θ …(ii) 45° 30°
K A C x D C 20 m B
60 m
MATHEMATICS > Height and Distance 233

In right angled ∆ABC , 18. (c) Let the height of the incomplete 20. (b) Let the radar base is at point A. The
AB 1 x pillar be x m and the increasing height plane is at point D in the first sweep and
tan 30° = ⇒ =
BC 3 20 be PC = h m. at point E in the second sweep. The
20 Given, AB = 100 m distance it covers in the one minute
⇒ x= m …(i) P interval is DE.
3
E D
BC 20
and cos 30° = = h
AC h−x
3 20 40 C
⇒ = ⇒ h−x= 1000 m
2 h−x 3 °
60
From Eq. (i), putting the value of x , x 30°
A B C
40 20 60 3
h= + = × = 20 3 m 60° From the figure,
3 3 3 3 45°
A B
100 m In right angled ∆ADC , we get
Hence, the height of the post is 20 3 m.
In right angled ∆ABC , DC 1000 1000
tan30° = = ⇒ AC =
17. (c) Let P be the cloud and P ′ its image in AC AC tan 30°
BC x
the lake. Let T be the point ‘h’ m above tan 45° = = ⇒ x = 100 m...(i) Similarly, in ∆EAB, we get
the surface of the lake and let x be the AB 100
EB 1000 1000
height of the cloud. and in right angled ∆APB, tan 60° = = ⇒ AB =
AB AB tan 60°
P PB PC + BC x+ h
tan 60° = = = Total distance covered by plane in 1 min,
β AB AB 100
T Mx then
α ⇒ x + h = 100 3
h DE = AC − AB
⇒ h = 100 3 − x = 100 3 − 100 1000 1000
S N DE = −
[from Eq. (i)] tan 30° tan 60°
∴ h = 100 ( 3 − 1) m 1000
x = 1000 3 − = 1154.70 m
Hence, the height of the incomplete 3
pillar is 100( 3 − 1) m.
The speed of the plane is given by
P´ 19. (c) Let AB be the tower and C , D be the s = distance covered/time taken
∴ ST = h two positions of the car.
= DE / 60 = 19.25 m/s.
Then, ∠ACB = 45° , ∠ADB = 30°.
Also, NP = NP ′ = x [say] 21. (c) R
Let AB = h , CD = x and AC = y .
Then, PM = x − h
B
P ′M = x + h

tower
In right angled ∆PTM, x h
PM
TM
= tan β h ()
3
q = sin–1 5
∴ x − h = TM tan β …(i) P 20 m Q
In right angled ∆P ′TM, 30° 45° Let P be the point of observation and
P ′M D x C y A QR be the tower.
= tan α
Given that, θ = sin −1   , PQ = 20 m.
TM 3
In right angled ∆ABC , we get 5
x + h = TM tan α …(ii) AB h
= tan 45° = 1 ⇒ =1 Let the height of the tower, QR = h and
From Eqs. (i) and (ii), we get AC y
x − h tan β PR = x
= ⇒ y=h …(i)
x + h tan α From the right angled ∆PQR,
In right angled ∆ABD, we get
 3 
⇒ sin sin −1   = h / x
QR
Using componendo and dividendo rule, AB
= tan30° =
1

h
=
1 sin θ =
we get PR   5  
AD 3 x+ y 3
x − h + x + h tan β + tan α 3 h 5h
= ⇒ x + y = 3h …(ii) ⇒ = ⇒x= …(i)
x − h − x − h tan β − tan α 5 x 3
∴ x = 3h − h = h ( 3 − 1)
2x tan β + tan α From pythagoras theorem, we get,
⇒ = [by Eqs. (i) and (ii)]
−2h tan β − tan α PQ 2 + QR2 = PR2
Now, h ( 3 − 1) is covered in 12 min. ⇒ 202 + h 2 = x2
x tan β + tan α
⇒ = So, h will be covered in 2
h tan α − tan β
202 + h 2 =   [using Eq. (i)]
5h
  ⇒
x sin β cos α + cos β sin α 12 12 3
⇒ =  × h = min. 2
h sin α cos β − cos α sin β  h ( 3 − 1)  ( 3 − 1) 25h 2 16 h
  ⇒ 202 + h 2 = ⇒ = 202
sin(α + β ) 9 9
= =  1200 
sin(α − β )  min ≈ 16 min 23 s 3 × 20
 73  ∴ h= = 15 m
∴ x = h sin (α + β ) cosec (α − β ) 4
234 CDS Pathfinder

x + h/2 28. (c) Refer to example 3.


22. (a) Let O be the position of the man’s eye In ∆ABD, cot α =
and C be the centre of the balloon. h
29. (c) Let θ be the inclination of the ladder
B 3 x + h/2
∴ = to the horizontal.
4 h
h
C ⇒ 3h = 4x + 2h ⇒ x =
4 B
α/2 h
Hence, required distance is
α/2 A 4

m
O D

20
β 26. (a) Let θ be the angle of elevation of Sun
α and height of the pole be h m.
Now, ∠BOC = ∠COA = θ
2 A A 10 m O
Given, CA = BC = r
In right angled ∆COD, sin β =
CD
h Now, in right angled ∆ AOB ,
OC AO 10 1
θ cos θ = = =
∴ CD = OC sin β …(i) AB 20 2
C B
In right angled ∆ COA, √3 h ⇒ cos θ = cos 60°
α CA
sin = In right angled ∆ABC , ∴ θ = 60°
2 OC
α AB h Hence, the angle of inclination of the
∴ OC =
r tan θ = ⇒ tan θ =
α = r cosec 2 BC 3h ladder is 60°.
sin
1
2 ⇒ tan θ = = tan 30° Solutions (Q. Nos. 30-33)
α 3
From Eq. (i), CD = r cosec sin β 30. (b) Given, β = 30°
2 ∴ θ = 30° AE
Hence, the height of centre of the In right angled ∆ADE, tan β =
α Hence, the angle of elevation of Sun is 30°. DE
balloon is r sin β cosec . AE 1 AE
2 27. (d) Let AE be the height of the cliff and tan30° = ⇒ =
h m (BD) be the height of the tower. DE 3 DE
23. (a) Let AB be the tower of height h m,
BC be x m and DC be h / 2 m. A 3 + 3 
30° ⇒ DE = 3 AE = 3   h
200 − h

A 45°  2 
3
30° ⇒ BC = DE = ( 1 + 3 ) h ... (i)
B x C 2
200 m

h A
h
α β α
B C
D C x B 45° θ 3+√3 h m
h/2 x E
D 2
In right angled ∆ABD,
In right angled ∆ABC , hm
h x+ h/ 2 200 − h
tanα = ⇒ cot α = . tan30° =
AC
=
x+ h/ 2 h β θ
BC x D E
…(i) 1 200 − h
Now, in right angled ∆ABC , ⇒ = [Q BC = DE ]
3 x
h
tanβ = ⇒ cot β =
x
…(ii) ⇒ x = ( 200 − h ) 3 …(i) Now, in right angled ∆ABC ,
x h AC
and in right angled ∆ADE, ⇒ tanα =
Subtracting eq. (ii) from eq. (i), we get BC
x + h/2 x AE 200
cot α − cot β = − tan 45° = = ⇒ BC tan α = ( AE − CE )
h h DE x
200 = ( AE − BD ) [Q BD = CE ]
x + h/ 2−x h/ 2 1 ⇒ 1= ⇒ x = 200 m
= = = x 3 + 3 
h h 2 ⇒ BC tanα =   h−h
From Eq. (i), 200 = ( 200 − h ) 3  2 
24. (a) We have, cot α − cot β = 1/2
⇒ 200 = 200 3 − h 3 3 + 3 − 2
⇒ cot 30°− cot β = 1/ 2 =h 
⇒ h 3 = 200( 3 − 1)  2 
⇒ cot β = cot 30°−1 / 2
 3 − 1  1+ 3 
h = 200   m
= 3 − 1/ 2 3
∴ ⇒ (1 + 3 ) h tan α =   h
 3  2  2 
25. (b) tanβ = 4 ⇒ cot β = 1/ 4  
we have, cot α − cot β = 1 / 2 Hence, the height of tower is [from Eq. (i)]
3 200( 3 − 1) 1
⇒ cot α = cot β + 1/2 = 1/ 4 + 1/2 = m. ∴ tan α =
4 3 3
MATHEMATICS > Height and Distance 235

31. (a) Given, α = 30 ° ( 3 − 1) 3 37. (b) Let h m be the height of the tower
⇒ tan θ = ⋅
In right angled ∆ABC , 3 3 and BC be x m.
AC 1 AC (3 − 3 ) In right angled ∆BCD,
tan α = tan 30° = , = ∴ tan θ =
BC 3 BC DC h
3 3 tan 60° = =
⇒ BC = 3 AC = 3 ( AE − CE ) BC x
34. (c) Let θ be the angle of elevation, h
⇒ 3= ⇒ h=x 3 ...(i)
= 3 ( AE − BD ) [Q BD = CE ] A x
3 + 3  D
= 3 − 1 h
 2 
3
= (1 + 3) h ...(ii) x
h
2
Now, in right angled ∆ADE,
θ
AE 30° 60°
tan β = B C C
DE x/ 3 A B
AE In right angled ∆ABC , 50 m x
⇒ tan β = [Q DE = BC ]
AC x 3x Now, in right angled ∆ACD,
BC tanθ = = x = = 3
3 + 3  3( 1 + 3 ) BC x DC
  h h 3 tan30 ° =
 2  2 AC
⇒ = Here, tanθ = 3 = tan 60°
3 3 (1 + 3) ∴ θ = 60° [Q tan 60° = 3 ] =
DC
=
h
(1 + 3) h
2 h AB + BC 50 + x
2 35. (c) Given, radius of circle (OC ) = r
∴ tan β = 1 1 x 3
⇒ = [from Eq. (i)]
32. (c) Given, α = 30° and h = 30 m O 3 50 + x
In right angled ∆ABC , r ⇒ 50 + x = 3x ⇒ x = 25 m
1 AC
tan α = tan 30° = = C ∴ h = 25 3 m
3 BC h
BC ° Hence, the height of tower is 25 3 m.
⇒ = ( AE − CE ) = ( AE − BD ) 60
3 38. (c) Let person be at point C and observes
60°
[Q BD = CE ] A B a tower in West direction at B.
3 + 3  In right angled ∆ABO, ∴ BC = 200 m
⇒ BC = 3  − 1 h
 2  OB OB N
sin 60° = ⇒ AO = ...(i)
(1+ 3) AO sin 60°
⇒ BC = 3 ⋅ 30
2 Now, in right angled ∆AOC , A
= ( 3 + 3) ⋅ 15 60° OC OC
sin = ⇒ AO = ...(ii) 300 m
∴ DE = BC = ( 45 + 15 3 ) m 2 AO sin30° D
[Q DE = BC ] From Eqs. (i) and (ii), θ
C
33. (a) Given, β = 30° OB
=
OC

h
=
r W E
B 200 m
In right angled ∆ ADE , tan β =
AE sin 60 ° sin 30 ° 3 1
DE 2 S
2
3 + 3  ∴ h = 3r He walks some distance and reach at A.
 h Now, he observe tower in South
 2 
⇒ tan 30° = 36. (b) Let h m be the height of shorter direction at B.
DE tower and the distance between the two ∴ AB = 300 m
 1+ 3  towers is mh m.
3 h Given, ∠ABD = 30° and ∠BAC = 60°
Let BD be the shortest distance of tower
1  2  from the road, which is a perpendicular
⇒ = C
3 DE distance.
3
⇒ DE = ( 1 + 3 ) h …(i) If ∠ABD = θ , then
D ∠CBD = 90° − θ
2
In right angled ∆BDE, 30 m [Q angle between S and W = 90° ]
BD h h
tan θ = = In right angled ∆ADB,
DE DE BD BD
h cosθ = ⇒ cosθ = ...(i)
⇒ tan θ = [from Eq. (i)] A
60° 30°
B AB 300
3 mh m
(1 + 3) h
2 In right angled ∆CDB,
In right angled ∆ABD,
2 ( 3 − 1) cos ( 90° − θ ) =
BD
⇒ tan θ = tan 30° =
h

1
=
1
3 ( 3 + 1) ( 3 − 1) mh 3 m
BC
BD
2( 3 − 1) ⇒ m= 3 ⇒ sinθ = ...(ii)
⇒ tan θ = 200
3⋅ 2 Hence, the value of m is 3.
236 CDS Pathfinder

We know that, cos 2 θ + sin 2 θ = 1 In right angled ∆QMB, In ∆ABC , sin30° =


AB
=
h
2 2
 BD  +  BD  = 1 MQ H + 200 AC 12
⇒     tan 60° = = 1
 300   200  BM BM ⇒
h
=
H + 200 12 2
[from Eqs. (i) and (ii)] ⇒ 3= [from Eq. (i)]
40000 + 90000  3( H − 200) 12 2 3
⇒ BD 2  =1 ∴ h= = = 3m
 90000 × 40000  ⇒ H + 200 = 3( H − 200)
2 2

BD 
130000  ⇒ H + 200 = 3H − 600 44. (c) Let A and B be the position of two
⇒ 2
=1
 3600000000  planes and D be a point.
⇒ 2H = 800
360000 ∴ H = 400 m A
⇒ BD 2 =
13 Hence, the height of the cloud is 400 m.
360000 600
∴ BD = =

3000 m
m 41. (c) Let BC = h m be height of tower. Let B
13 13 P and Q be the points, where the
39. (d) Let AB be the height of the post, AC angle subtended are 45° and 60°, h
be the string and the angle made by respectively. 60°
D 45° C
string with the post be θ. 45° C x
A h
60° In ∆BCD, tan 45° =
x
θ ⇒ h=x
2h h
3000
h In ∆ACD, tan 60° = 3 =
x
60° 3000 3
45° Q x= × = 1000 3 m
P Q x B 3 3
B C
100 (3–√3) m ∴ AB = 3000 − h
AB h 1 In right angled ∆BCQ,
Now, cosθ = = ⇒ = cos 60° = 3000 − 1000 3
AC 2h 2
π tan 60° =
BC
⇒ 3=
h = 1000(3 − 3 ) m
∴ θ= BQ x
3 AC
h 45. (c) In right angled ∆ADC , tan 45° =
40. (c) Let P be the cloud at a height Hm ⇒ x= ...(i) DC
above the level of the water in the lake 3 AC
⇒ DC = =x …(i)
and Q be its image in the water. In right angled ∆PBC , tan 45°
P BC BC A
tan 45° = =
(H – 200)

PB PQ + QB
h
⇒ 1= x
B 30°
H 100(3 − 3 ) + x
M
200 m
60°
200 m
⇒ 100 (3 − 3 ) + x = h 30° 45°
h
⇒ 100(3 − 3 ) + = h [from Eq. (i)] B y D C
A O 3
h In right angled ∆BCA, we get
⇒ h− = 100(3 − 3 )
AC x
3 tan30° = ⇒
h( 3 − 1) BC DC + BD
H ⇒ = 100 3( 3 − 1) 1 x
3 ⇒ = ⇒ y + x = 3x
3 y+x
⇒ h = 100 3 × 3
∴ h = 300 m ⇒ y = x ( 3 − 1)
Q x 1
42. (d) Refer to example 3. ⇒ =
∴ OQ = OP = H y ( 3 − 1)
43. (d) AB is a pole and AC is rope.
Given, ∠PBM = 30° and ∠MBQ = 60° x 1 × ( 3 + 1)
A ⇒ =
In right angled ∆PBM, y ( 3 − 1)( 3 + 1)
PM H − 200
tan30° = = x ( 3 + 1)
pe

⇒ =
m
√1

BM BM
Ro

y ( 3 )2 − ( 1)2
2

h
1 H − 200
⇒ = Pole
x ( 3 + 1)
3 BM ∴ =
30° y 2
⇒ BM = 3( H − 200) ...(i) C B
22
237

LINES AND ANGLES


Usually (2-3) questions have been asked from this chapter. And this is the easiest topic of geometry
and so candidate can easily score marks in examinations.

In this chapter, we will study about the properties of Ray


the angles which formed, when two lines intersect each A ray extends indefinitely in one direction. This is
other and when a line intersect two or more parallel →
lines at distinct points. exhibited by an arrow i.e. PQ .
• P is called the initial point of the ray. The ray has no
Basic Terms and Definitions definite length.
Point Collinear Points
The figure of which length, breadth and height cannot
Three or more points are said to be collinear if a single
be measured is called a point. It is infinitesind.
straight line passes through them. Here, A, B and C are
Line Segment collinear.

The straight path between two points P and Q is called


a line segment.
Non-collinear Points
This can be represented as PQ.
Three or more points not lying on a single straight line
• P and Q are called the end points of the line segment.
are called non-collinear points. Here, A, B and C are
• The line segment has a definite length.
non-collinear points.
• Distance between P and Q is called the length of the
line segment PQ.
Line
A line segment extended endlessly on both sides, is

called a line. It is denoted by PQ Intersecting Lines

or QP. Two lines having a common
point are called intersecting
• A line is a set of infinite number of points.
lines. This common point
• A line have no end points and no definite length.
is the point of intersection
i.e. O.
238 CDS Pathfinder

Concurrent Lines 5. Reflex angle An angle whose measure is more than


180°, but less than 360°, is called a reflex angle.
Three or more lines intersecting A S
at the same point are said to be O O A
concurrent. This common point P Q
is the point of concurrence i.e. O. B
R B
In the given figure, 180 ° < ∠ AOB < 360 °
Non Intersecting lines/Parallel Lines
6. Complete angle An angle whose
If two lines lie in the same plane and do not intersect A
measure is 360°, is called a complete O
when produced on either side, then such
lines are said to be parallel to each other.
l angle. In the given figure, ∠AOA is a complete angle.
m
If l and m are two parallel lines, we write l || m and read
it as l is parallel to m .
Pairs of Angle
1. Complementary Angles
ANGLES Two angles are said to be
complementary, if the sum of
An angle is formed by two rays with a common initial
their measure is 90°. Thus, ∠θ1
point. Let ‘O’ is the initial point, then O is called the
and ∠θ 2 are complementary, if
vertex.
∠ θ1 + ∠ θ 2 = 90 ° θ1
θ2
A • Complementary angles are O
complement of each other.
R

P • Complement of x is (90 ° − x ).
m
ar
angle
vertex EXAMPLE 1. The measure of an angle which is 28°
O arm B more than its complement is
Where, P is a point in the interior of ∠AOB and R is a a. 23° b. 59° c. 77° d. None of these
point in the exterior of ∠AOB. Sol. b. Let the measure of the required angle be x°. Then,
measure of its complement = 90° − x
Types of Angle ∴ x − (90° − x) = 28° ⇔ 2x = 118°
1. Acute angle An angle whose measure is more than
⇔ x = 59°
0°, but less than 90°, is called an acute angle.
Hence, the measure of the required angle is 59°.
A
EXAMPLE 2. The measure of the complement of an
angle of 48° 36 ′ 24′ ′ is
O B a. 41°23′ 36′′ b. 42°23′ 36′ c. 41°24′ 36′′ d. 42°24′ 36′′
In the given figure, 0 ° < ∠AOB < 90 ° Sol. a. As, 90° = 89° 59′ 60′′
2. Right angle An angle whose measure is
A ∴ Complement of an angle of ( 48° 36′ 24′′)
90°, is called a right angle. = Angle of [90° − 48° 36′ 24′′ ]
In the given figure, ∠AOB = 90 °. = Angle of [89° 59′ 60′′ − 48° 36′ 24′′]
O B
= Angle of ( 41° 23′ 36′′)
3. Obtuse angle An angle whose measure is more than
90° but less than 180°, is called an obtuse angle. 2. Supplementary Angles
A
Two angles are said to be supplementary, if the sum of
their measures is 180°. Thus, ∠ θ1 and ∠θ 2 are
O B supplementary if θ1 + θ 2 = 180 °
In the given figure, 90 ° < ∠ AOB < 180 °.
θ2
4. Straight angle An angle whose θ1
• • •
measure is 180°, is called a straight A O B
angle. • Supplementary angles are supplement of each other.
In the figure, ∠ AOB = 180 ° • Supplement of x is (180 ° − x).
MATHEMATICS Lines and Angles 239

EXAMPLE 3. The measure of an angle, which is 32° If two lines PQ and RS intersects at a point O, then the
less than its supplement is pair of ∠POR and ∠QOS or pair of ∠POS and ∠ROQ
a. 31° b. 64° c. 74° d. 148° is said to be a pair of vertically opposite angles.
Sol. c. Let the measure of the required angle be x. Then, P
S
measure of its supplement = (180° − x)
∴ (180° − x) − x = 32° ⇔ 2x = 148° ⇔ x = 74° O
Q
EXAMPLE 4. Two supplementary angles are in the R
ratio 3 : 2. Then, the measurement of the smaller angle is • Vertically opposite angles are always equal
a. 36° b. 72° c. 108° d. 112° i.e. ∠POS = ∠ROQ and ∠POR = ∠SOQ .
Sol. b. Let the supplementary angles be 3x and 2x, • Sum of the angles around a point is 360 °.
respectively. Then, according to the definition of
EXAMPLE 5. In the given figure if DOC is a straight
supplementary angle.
ray OB is bisector of ∠AOC, where ∠AOC = 110 ° and
3x + 2x = 180° ∠COE = 120 °, then the sum of ∠x, ∠y and ∠z is
⇒ 5x = 180° ⇒ x = 36°
A
∴ Angles will be 3x = 3 × 36 = 108° B

and 2x = 2 × 36 = 72°. 110°


y z
Thus, the smaller angle is 72°. O
D x C
120°
Bisector of an Angle
A ray which divides the angle into
P E
two equal parts, is called the bisector
of an angle. S a. 160° b. 115° c. 170° d. 180°
If a ray OS is the bisector of ∠POQ, Sol. d. Since, DOC is a straight line
then ∠POS = ∠QOS. O Q ∠AOC + ∠DOA = 180° [Linear pair]
1 110° + ∠DOA = 180° [Q ∠AOC = 110° ]
• ∠POS = ∠QOS = ∠POQ
2 ∠DOA = 70°
∠DOA = ∠y = 70°
Adjacent Angles Also, ray OB is angle bisector of ∠AOC
Two angles are said to be adjacent, if ∠AOC 110°
∠z = = = 55°
(i) they have a common vertex. 2 2
R
(ii) they have a common arm Q Line BE and DC intersect each other at O.
Q
(iii) their non-common arms are on ∠x = ∠z = 55° [Q vertically opposite angle]
either side of the common arm. O P
∴ ∠x + ∠y + ∠z = 55°+70°+55° = 180°
Here, ∠POQ and ∠ROQ are adjacent angles.

Linear Pairs of Angles Some Other Angles


Let l and m are parallel lines and n is the transversal
If the non-common arms of two adjacent angles form a
which cuts these parallel lines. The different angles
line, then these angles are called linear pair of angles.
formed are as follows.
∠ROP and ∠ROQ form a linear pair of angles.
n
R

2 1 l
3
P O Q 4
∴ ∠POQ = ∠ROP + ∠ROQ = 180 °
6 5 m
Vertically Opposite Angles 7 8
Two angles are called a pair of vertically opposite
angles, if their arms form two pairs of opposite rays.
240 CDS Pathfinder

1. Corresponding angles Corresponding angle pairs EXAMPLE 6. In the given figure, PQ || RS . The value of
are x is
∠1 and ∠5, ∠ 2 and ∠6, ∠ 4 and ∠8, ∠ 3 and ∠7 and T
all the corresponding pair are equal x
100
i.e., ∠1 = ∠5, ∠2 = ∠6, ∠4 = ∠8 and ∠3 = ∠7. R S
50
2. Vertically opposite angles Vertically opposite P Q

angles pairs are a. 50° b. 80° c. 75° d. 65°


∠1 and ∠ 3, ∠ 4 and ∠ 2, ∠8 and ∠6, ∠5 and ∠7 and Sol. a. Draw AB || PQ
all vertically opposite angles are equal ∴ ∠ATP = ∠TPQ = 50° [alternate angles]
i.e., ∠1 = ∠3, ∠4 = ∠2, ∠8 = ∠6 and ∠5 = ∠7. and ∠BTR + ∠TRS = 180°
[Angles on the same side of transversal]
3. Alternate angles Alternate angles pairs are T
∠ 3 and ∠5, ∠ 4 and ∠6 and they are equal A B
i.e., ∠3 = ∠5 and ∠4 = ∠6. x
100°
S
• The sum of interior angles on the same side of R
50°
transversal is equal to 180 °. P Q
i.e., ∠3 + ∠6 = 180 ° and ∠4 + ∠5 = 180 °
⇒ ∠BTR = 80°
• The sum of exterior angles on the same side of and ∠ATP + x + ∠BTR = 180° [ Q ATB is a straight line]
transversal is equal to 180°.
i.e., ∠2 + ∠7 = 180 ° and ∠1 + ∠8 = 180 ° ∴ x = 180° − ( 50° + 80° ) = 50°

PRACTICE EXERCISE
1. Three lines intersect each other in pairs. What is 7. If ∠1 = ( 5x − 20)° and ∠7 = ( 2x + 10)°, then ∠7 is
the total number of angles so formed? l
(a) 3 (b) 6 (c) 9 (d) 12
1 2
2. An angle is 14° more than its complement. Then, m 3 4
its measure is
(a) 166° (b) 86° (c) 76° (d) 52°
5 6
3. The measure of an angle is twice the measure of n 8 7
its supplementary angle. So, its measure is
(a) 120° (b) 60° (c) 100° (d) 90°
(a) 38° (b) 10°
4. What is the least number of straight lines for a (c) 30° (d) 70°
bounded plane figure?
8. The measure of complementary angle of 12° 25′
(a) 1 (b) 2 (c) 3 (d) 4
40′′ is
3 (a) 77° 34′ 20′′ (b) 77° 36′ 20′′ (c) 77° 24′ 20′′ (d) 77° 34′
5. The supplement of of a right angle is
5
(a) 122° (b) 126° (c) 130° (d) 132°
9. ∠POR and ∠QOR form a linear pair and if
a − b = 80°, then the values of a and b are,
6. In the given figure, if PQ||SR, then the relation respectively
between ∠a and ∠b is R
a
P A Q


B P O Q
S R
b
(a) 95°, 85° (b) 108°, 72°
(a) ∠a ≠ ∠b (b) ∠a < ∠b (c) ∠a = ∠b (d) ∠a > ∠b (c) 130°, 50° (d) 105°, 75°
MATHEMATICS Lines and Angles 241

10. In the given figure, the value of y is 17. The value of x in the given figure is
C F A
P 45°
5y O 2y
A B
5y x B

E D 30°
(a) 25° (b) 35° (c) 15° (d) 40° Q C
11. AB and CD are two parallel lines. PQ cuts AB and (a) 75° (b) 185°
CD at E and F, respectively. EL is the bisector of (c) 285° (d) 245°
∠FEB. If ∠LEB = 35°; then ∠CFQ will be 18. In the given figure, AB||CD, ∠DPL =
1
∠NPO,
(a) 110° (b) 85° (c) 70° (d) 95° and OP⊥ XY the value of x° is 2
12. AB and CD are two A B X
parallel lines. The 65° E F
points B and C are M 60°
35° A B
joined such that
∠ABC = 65°. A line D O L
CE is drawn making C
N x
angle of 35° with the line CB, EF is drawn parallel C D
P
to AB, as show in figure, then ∠CEF is equal to Y
(a) 160° (b) 155° (c) 150° (d) 145° (a) 30° (b) 40° (c) 15° (d) 25°
13. In the given figure, if EC|| AB, ∠ECD = 70°, 19. In the given figure, AB||CD and they cut PQ
∠BDO = 20°, then ∠OBD is equal to and QR at E, F and G , H respectively. Then find
the value of x + y
E A A C
G
x x Q
y
70° O E
140°
C 20° D H
P 70°
B
(a) 70° (b) 60° (c) 50° (d) 20°
14. Two parallel lines AB and CD are intersected by F
a transversal line EF at M and N , respectively. R
B D
The lines MP and NP are the bisectors of the
interior angles BMN and DNM on the same side (a) 120° (b) 130°
(c) 150° (d) 132°
of the transversal. Then, ∠MPN is equal to
(a) 90° (b) 45° (c) 135° (d) 60° 20. Two parallel lines are cut by a transversal, then
15. In the given figure if l|| m, then the value of x is which of the following are true?
A I. Pair of alternate interior angles are congruent.
l
100° II. Pair of corresponding angles are congruent.
III. Pair of interior angles on the same side of the
O x+5 transversal are supplementary.
30° m
B Select the correct answer using the codes given
(a) 105° (b) 100° (c) 110° (d) 115° below
16. In the given figure, if ∠COE = 90°, then the (a) I, II and III are true (b) I and III are true
value of x is (c) I and II are true (d) II and III are true
A
D
21. Consider the following statements related to
z three lines L1 , L2 and L3 in the same plane.
2x O y
I. If L 2 and L3 are both parallel to L1 , then they
x
90° are parallel to each other.
C B II. If L 2 and L3 are both perpendicular to L1 then
E they are parallel to each other.
III. If there is acute angle between L1 and L3 , then
(a) 120° (b) 60° (c) 45° (d) 30° L 2 is parallel to L3 .
242 CDS Pathfinder

Which of the statement(s) given above is/are 27. In the given figure, which of the
correct? following statements must be true? b c
(a) I and II (b) II and III I. a + b = d + c II. a + c + e = 180° a d
(c) All of these (d) None of these III. b + f = c + e IV. a + b + c = d + e + f f e
22. In the given figure, AB||CD, then which one of Select the correct answer using the codes
the following is true? given below
A E B (a) Only I (b) I, II and III
p° (c) II, III and IV (d) All of these
q° F 28. In the given figure, AB is parallel to CD. If
∠BAF = 98° and ∠AFC = 144°, then ∠ECD is
r° equal to?
C G D A B
(a) p + q − r = 180° (b) p + q + r = 180° 98°
(c) p − q + r = 180° (d) p + q − 2 r = 180° F 144°
23. LM is a straight line and O is a point on LM.
Line ON is drawn not coinciding with OL or OM. C D
If ∠MON is one-third of ∠LON , then what is
∠MON equal to? E
(a) 45° (b) 60° (a) 62° (b) 64°
(c) 75° (d) 80° (c) 82° (d) 84°

24. Consider the following statements : 29. Consider the following statements :
If two parallel lines are intersected by a
Two lines intersected by a transversal are
transversal, then
parallel, if
I. each pair of corresponding angles are equal.
I. The pairs of corresponding angles are equal.
II. each pair of alternate angle are unequal.
II. The interior angles on the same side of the
transversal are supplementary. Which of the statement(s) given above is/are
correct?
Which of the statement(s) given above is/are (a) Only I (b) Only II
correct? (c) Both I and II (d) Neither I nor II
(a) Only I (b) Only II
(c) Both I and II (d) Neither I nor II 30. In the given figure, PQ is parallel to RS. Then
∠NMS is equal to
25. In the given figure AC|| BD and AE|| BF . What L
is the value of ∠x ? P
55° 47°
Q
C D
67° N

130°
A B
x R M S
60°
(a) 20° (b) 23° (c) 27° (d) 47°
G
E F 31. The line segments AB and CD intersect at O. OF
is the internal bisector of obtuse ∠BOC and OE
(a) 130° (b) 110°
(c) 70° (d) 50° is the internal bisector of acute ∠AOC. If
∠BOC = 130°, then what is the measure of
26. The necessary conditions for the line l and m to ∠FOE ?
be parallel when these lines are intersected by a (a) 90° (b) 110° (c) 115° (d) 120°
transversal line n is that
I. Interior angles on the same side are equal.
32. In the given figure below, AB is parallel to LM.
What is the angle a equal to?
II. Corresponding angles are equal.
A b B
III. Vertically opposite angles are equal.
a
IV. Alternate interior angles are equal.
L c M
Select the correct answer using the codes given
below (a) π + b + c (b) 2 π − b + c
(a) II and III (b) I, II and III (c) II and IV (d) I, II and IV (c) 2 π − b − c (d) 2 π + b − c
MATHEMATICS Lines and Angles 243

33. In the given figure, PQ is parallel to RS. What is Which of the statement(s) given above is/are
the angle between the lines PQ and LM? correct? e 2012 I
P
Q (a) Only III (b) Only I (c) I and III (d) II and III
55°
40. In the given figure, LOM is a straight line.
What is the value of x° ? e 2012 II
L
M Q P
155°
30°
R 25° S
50°
(a) 175° (b) 177° (c) 179° (d) 180° (x + 20)° (x – 10)°
L M
O
34. The length of a line segment AB is 2 units. It is
(a) 45° (b) 60° (c) 70° (d) 80°
divided into two parts at the point C such that
AC 2 = AB × CB. What is the length of CB ? 41. If the arms of one angle are respectively parallel
(a) 3 + 5 units (b) 3 − 5 unit to the arms of another angle, then the two
(c) 2 − 5 unit (d) 3 units angles are e 2013 I
(a) Neither equal nor supplementary
Directions (Q.Nos 35-36) Read the information and (b) not equal but supplementary
answer the questions.' (c) equal but not supplementary
In the given figure, the lines CB and AC of a (d) Either equal or supplementary
triangle ABC are extended to D and F, respectively 42. The complement angle of 80° is e 2015 I
and CF || GE. 18 5π
(a) radian (b) radian
D 95° E F π 9
π 9
(c) radian (d) radian
G 18 5π

C y° 43. Let OA,OB,OC and OD be rays in the


x° anti-clockwise direction? Such that
∠AOB = ∠COD = 100°, ∠BOC = 82°
35° z° and ∠AOD = 78°.
A B
Consider the following statements
35. The sum of the value of x, y and z. I. AOC and BOD are lines.
(a) 250° (b) 180° (c) 230° (d) 110° II. ∠BOC and ∠AOD are supplementary. e 2015 I
36. The value of the angle which equals one-fifth of Which of the statement(s) given above is/are
the supplement of z ? correct?
(a) 50° (b) 26° (c) 10° (d) 130° (a) Only I (b) Only II
(c) Both I and II (d) Neither I nor II

PREVIOUS YEARS’ QUESTIONS 44. If a transversal intersects four parallel straight


lines, then the number of distinct values of the
37. AB is a straight line. C is a point whose distance angles formed will be e 2016 I
from AB is 3 cm. What is the number of points (a) 2 (b) 4 (c) 8 (d) 16
which are at a distance of 1 cm from AB and
5 cm from C ? e 2012 I
45. p
(a) 1 (b) 2 (c) 3 (d) 4 80° z 40 x 3
°
38. The ratio of two complementary angle is 1 : 5.
What is the difference between the two angles?
e 2012 I 100° x y
q
(a) 60° (b) 90°
In the figure given above, p and q are parallel
(c) 120° (d) Cannot be determined
lines. What are the values of the angles x, y
39. Consider the following statements : and z ? e 2016 I
If two straight lines intersect, then (a) x = 80°, y = 40°, z = 100°
I. vertically opposite angles are equal. (b) x = 80° , y = 50°, z = 105°
II. vertically opposite angles are supplementary. (c) x = 70°, y = 40° , z = 110°
III. adjacent angles are complementary. (d) x = 60°, y = 20°, z = 120°
244 CDS Pathfinder

ANSWERS
1 d 2 d 3 a 4 c 5 b 6 c 7 c 8 a 9 c 10 c
11 a 12 c 13 c 14 a 15 a 16 d 17 c 18 c 19 c 20 a
21 a 22 a 23 a 24 c 25 b 26 c 27 c 28 a 29 a 30 a
31 a 32 c 33 d 34 b 35 c 36 b 37 d 38 a 39 b 40 b
41 b 42 c 43 d 44 a 45 d

HINTS AND SOLUTIONS 


1. (d) We know that, when two lines 7. (c) Here, ∠ 1 = ∠5
intersect each other, it makes 4 angles. [corresponding angles] P
Since, the total number of pairs = 3 ∠7 = ∠5 (vertical opposite) A
E
B
35°
⇒ ∠1 = ∠7 (angle)
L
So, 5x − 20° = 2x + 10° F
3x = 30° ⇒ x = 10° C D
Hence, Q
∠ 7 = 2x + 10° = 2 ( 10° ) + 10° = 30° ⇒ ∠AEB = ∠AEF + ∠BEF = 180°
[straight line]
∴ Total number of angles = 3 × 4 = 12 8. (a) Complementary angle of 12° 25′40′′ ⇒ ∠AEF = 180° − 70° = 110°
= 90° −12° 25′ 40′′ ⇒ ∠CFQ = ∠AEF = 110°
2. (d) Let angle be x and its complement be
90° − x. = 89° 59′ 60′′ − 12° 25′ 40′′ [corresponding angles]
According to the question, = (89 − 12)° + (59′ − 25′) + ( 60 − 40)′′ 12. (c) Since, AB || CD
= 77° + 34′ + 20′′ = 77° 34′ 20′′
x = ( 90° − x) + 14 A B

⇒ 2x = 104° 9. (c) Since, ∠POR and ∠QOR is a linear 65° E F


104° pair.
⇒ x= = 52° ∴ ∠POR + ∠ROQ = 180°
35°
2
R D
3. (a) Let angle be x and its supplementary C
= 180° − x a
∴ ∠ABC = ∠BCD = 65°
Then, x = 2 ( 180° − x) b [alternate angles]
⇒ 3x = 360° P O Q ∠ECD = 65° − ∠BCE
⇒ x = 120° = 65° − 35° = 30°
⇒ a + b = 180° ... (i) ∠CEF + ∠ECD = 180°
4. (c) The least number of straight lines for and a – b = 80° ... (ii) [Angles on the same side of
a bounded plane figure is 3. On solving Eqs. (i) and (ii), we get transversal]
3 2a = 260° ⇒ a = 130° ⇒ ∠CEF = 180° − 30° = 150°
5. (b) Given, angle = of right angle
5 and b = 180° – 130° = 50° 13. (c) Since, EC || AB
3 a = 130° , b = 50°
= × 90° = 3 × 18° = 54° So,
5 E A
10. (c) Since, AOB is a straight line.
Supplement of 54° = ( 180°−54° )
∴ ∠AOB = 180°
= An angle of measure 126° 70°
∠AOC + ∠COF + ∠FOB = 180° O
6. (c) We have, ∠QAB = ∠a 20°
C D
[straight line]
[vertically opposite angles]
5 y + 5 y + 2 y = 180° B
a [Q ∠COF = ∠DOE , vertically ∠AOD = ∠ECO
P Q
A opposite angles] [Q corresponding angles]
B ⇒ 12 y = 180° ⇒ ∠AOD = 70°
S R 180° So, ∠BOD = 180°−70°
b ⇒ y= = 15°
12°
= 110° [linear point]
and ∠QAB = ∠b Hence, the value of y is 15°.
Now, in ∆BOD
[corresponding angles] 11. (a) Given, ∠LEB = 35°
⇒ ∠a = ∠b ∠OBD + ∠BOD + ∠ODB = 180°
∠FEB = 2 × ∠LEB = 2 × 35° = 70° ⇒ ∠OBD = 180° − ( 110 + 20)° = 50°
MATHEMATICS Lines and Angles 245

1 and Q XY ||QC , ∠QCB = ∠CBY = 30° 24. (c) Both the statements I and II are
14. (a) Given, ∠PMN = ∠BMN
2 [alternate angles] correct.
1
and ∠PNM = ∠DNM ⇒ ∠ABC = ∠ABY + ∠CBY 25. (b) Since, AC || BD
2
⇒ ∠ABC = 45° + 30° = 75° C D
As, ∠BMN + ∠DNM = 180°
Q x = 360° − ∠ABC = 360° − 75°
[angles on the same side of transversal] 130°
∴ x = 285° B
E A
M 18. (c) Q AB || CD, ∠ONP = ∠XMB = 60° x
A B 60°
[corresponding angles]
G
P ∠OPN = 90° − ∠ONP E F
= 90° − 60° ⇒ ∠OPN = 30°
1 1 ∴ ∠DBA = 180° − 130° = 50°
C
N
D But, ∠DPL = ∠NPO = 30° [interior angle]
2 2
F ⇒ ∠DPL = 15° ⇒ x = 15° [Q BAC = 130° ]
∴ In ∆MPN , ∠PMN + ∠PNM = 90° DBG is straight line.
19. (c) Since, AB || CD and PQ is transversal
⇒ ∠MPN = 180° − ( ∠PMN + ∠PNM ) ∴ ∠DBA + ∠ABF + ∠FBG = 180°
∠PEF = ∠EGH [corresponding angles] [linear pairs]
[angle sum property]
⇒ ∠EGH = 70° [Q ∠PEF = 70°] ⇒ 50° + ∠ ABF + 60° = 180°
∴ ∠MPN = 180° − 90° = 90°
Now, ∠EGH + ∠HGQ = 180° ⇒ ∠ABF = 70°
15. (a) Draw a line n passing through O and [linear pair] Since, AE || BF
parallel to l and m ⇒ ∠HGQ = 180°−70° = 110° ∴ x = 180° − ∠ABF
Since, l || n, ∠1 + 100° = 180° Also, ∠DHQ + ∠GHQ = 180°
= 180° − 70° = 110°
[Sum of the interior angles on the [linear pair]
26. (c) When two lines are parallel
same side of the transversal] ∠GHQ = 180°−140° = 40°
intersected by a transversal then
∠1 = 80° ∴ x + y = 110° + 40° = 150° corresponding as well as alternate
Since, n || m, ∠2 = 30° [alternate angles] 20. (a) All the three statements are true. interior angles are equal.
A 21. (a) Only statements I and II are true.
l Hence, the statement II and IV are
100° 22. (a) Draw FH || AB || CD correct.

O
1 (x + 5)
n (sum of interior angles) 27. (c) We have, a = d , b = e and c = f
2
30° m ∴ ∠1 + p = 180° ...(i) [vertically opposite angles]
B A E B and a + b + c = d + e + f = 180°
Now, ∠AOB = ∠1 + ∠2 p II. a + b + c = 180° ⇒ a + c + e = 180°
= (80° + 30° ) = 110° 1 III. b + f = c + e
But, ∠AOB = ( x + 5)° = 110° F
IV. a + b + c = d + e + f
H 2
x = ( 110 − 5)° = 105° 180–r Hence, statements II, III and IV are true.
r
16. (d) Here, ∠BOD = ∠AOC 28. (a) Draw a line MN parallel to AB and
C G D
∴ 2x = y [vertically opposite angles] CD.
∠2 + 180° − r = 180° ...(ii) A B
Now, COD is a straight line.
(sum of interior angle)
∠COD = 180° 98°
On adding Eqs. (i) and (ii), we get
⇒ ∠COE + ∠EOB + ∠BOD = 180° N
⇒ 90°+ x + 2x = 180° ∠1 + ∠ 2 + p + 180° − r = 360° M
F 144°
⇒ 3x = 90° ⇒ x = 30° ⇒p + q − r = 180° [Q∠1 + ∠2 = q °]
1
Hence, the value of x is 30°. 23. (a) Given that, ∠ MON = ∠ LON C
D
17. (c) From figure, PA || QC 3
x
Let ∠ LON = x, then, ∠ MON = E
Draw a line XY parallel to PA and QC. 3
⇒ ∠AFN = 180° − 98° = 82°
P A N (sum of interior angles)
45° x ∠CFN = 144° − 82° = 62°
x/3
and ∠ECD = ∠CFN = 62°
x B L O M
X Y [corresponding angles]
We know that,
30° ∠ LON + ∠ MON = 180° [linear pair] 29. (a) If two parallel lines are intersected by
a transversal, then each pair of
Q C x 180°×3
⇒ x + = 180° ⇒ x = = 135° corresponding angles and of alternate
Q PA||XY , 3 4
x 135° angles are equal.
⇒ ∠ABY = ∠PAB = 45°
Thus, ∠ MON = = = 45° Therefore, statement I is correct.
[alternate angles] 3 3
246 CDS Pathfinder

30. (a) Since, PQ || RS ∴ ∠EOP = ∠OPB = b [alternate angle] 37. (d) ∴ Required number of points
∴ ∠PLM = ∠LMS = 55° and ∠EON = ∠ONM = c = 4 (P , P , P , P )
1 2 3 4
[alternate angle] [alternate angle] C
Draw a line EF which is parallel to PQ. ⇒ ∠PON = b + c
Then, ∠QLN = ∠LNE = 47° Q ∠PON + a = 2 π
Q ∠ENL + ∠MNE = 67° ∴ a = 2 π − ∠PON = 2 π − b − c 5 cm 5 cm
L 33. (d) 3 cm

5c
P

m
P Q
55° 47° Q

5c

m
55°
21 cm 21 cm
67° N P1 P2
E F 1 cm
L
M A B
30° 155° 1 cm
R 25° S P3 P4
R M S
PQ || RS …(i) 3 cm 3 cm
⇒ 47° + ∠MNE = 67°
Since, ∠PQR = ∠QRS = 30°+25° = 55° α 1
⇒ ∠MNE = 67° − 47° 38. (a) Given that, =
[alternate angle] β 5
⇒ ∠MNE = 20° Let α = k and β = 5k
PQ || RS …(i)
Similarly, EF || RS, then
and ∠SRL + ∠RLM = 180° Sum of two complementary angles
∠ENM = ∠NMS = 20°
⇒ RS || LM …(ii) = 90°
[alternate angle]
From Eqs. (i) and (ii), we get ⇒ α + β = 90°,
31. (a) Given, ∠BOC = 130° PQ || LM
A D
α = 90° − β
So, the angle between the lines PQ and ⇒ k = 90° − 5 k
LM is 180°.
O ⇒ k = 15°
E 34. (b) Given, AC 2 = AB × CB
∴ α = 15° and β = 75°
⇒ x 2 = 2 × ( 2 − x)
∴ Difference between angles
C B A B
F x C (2 – x) = 75° − 15° = 60°
2
Q AOB is a straight line. 39. (b) Here, AB and CD are two lines.
⇒ x 2 = 4 − 2x
Q ∠BOC + ∠AOC = 180° [linear pair] A D
⇒ x + 2x − 4 = 0
2
⇒ 130° + ∠AOC = 180°
−2 ± 4 + 16
⇒ ∠FOC + ∠FOC = 130° ⇒ x=
2×1
⇒ ∠AOC = 50°
⇒ x = − 1± 5 C B
Now, ∠BOC = 130°
Now, BC = 2 − ( − 1 ± 5) = 3 − 5 If two straight lines intersect, then
⇒∠BOF + ∠FOC = 130° vertically opposite angles are equal.
(neglect 3 + 5 Q3 + 5 > 2)
[Q OF is bisector of ∠BOC ]
35. (c) We have, ∠DGE = ∠GCF = 95° 40. (b) From the given figure,
⇒ ∠FOC = 65°
[corresponding angles] ∠ LOQ + ∠QOP + ∠POM = 180°
Now, ∠AOC = 50°
Also, ∠GCF + ∠FCB = 180° [straight line]
⇒ ∠AOE + ∠EOC = 50°
[linear pair] ∴ ( x° + 20° ) + 50° + ( x° − 10° ) = 180°
⇒ ∠EOC + ∠EOC = 50°
⇒ 95°+ y = 180° ⇒ 2x° + 60° = 180°
⇒ ∠EOC = 25°
⇒ y = 180°−95° = 85° ⇒ 2x° = 120°
[Q OE is bisector of ∠AOC ]
Again, ∠ACB + y = 180° ∴ x ° = 60°
∴ ∠EOF = ∠EOC + ∠FOC
⇒ x = 180°−85° = 95° 41. (b) Case I When both pairs of arms are
= 25° + 65° = 90°
Now, In ∆ABC ∠A + ∠B + ∠C = 180° parallel same sense.
32. (c) Draw a line parallel to AB i.e.
EF || AB. ⇒ 35°+ z + 95° = 180° A
P ⇒ z + 130° = 180° D
A B
b ⇒ z = 180°−130° = 50°
∴ x + y + z = 95° + 85° + 50° = 230° C
B G
E F
36. (b) Supplement of z = 130°
O a F
E
1 130°
c ∴ th of 130° = = 26° Here, ∠ ABC = ∠DEF
L N M 5 5
MATHEMATICS Lines and Angles 247

π π
Case II When both pairs of arms are = 10 × = radian. 44. (a) If, line P intersect four parallel lines
parallel in opposite sense. 180° 18 l, m, n and o, then 16 angles will be
43. (d) formed.
A B P
F E
100° 1 2
82° l
O 78° A
B C m
C
n
D D
o
Given, ∠AOB = ∠COD = 100°,
Here, ∠ ABC = ∠DEF ∠BOC = 82° As these lines are parallel, hence distinct
Case III When one pair of arms is
and ∠AOD = 78° angle will be ∠1 and ∠2.
parallel in same direction and other pairs
are parallel in opposite direction. If, AOC is a straight line, then 45. (d) In the given figure, lines p and q are
∠AOB + ∠BOC = 180° parallel.
A A
⇒ 100°+82° = 180° p
D 80° z x/3
⇒ 182° ≠ 180°

40
°
B If, BOD is a straight line, then
C ∠BOA + ∠AOD = 180°
F E 100° x y
⇒ 100° + 78° = 180° q
C B
Here, ∠ ABC + ∠DEF = 180° ⇒ 178° ≠ 180° x
∴ x = 40° + [alternate angles]
So, the two angles are not equal but If, ∠BOC and ∠AOD 3
supplementary. 2x
are supplementary angles, then ⇒ = 40° ⇒ x = 60°
42. (c) Let the angle be θ. 3
∠BOC + ∠AOD = 180° y = x /3 [alternate angle]
∴ θ = 80°
⇒ 82°+78° = 180° 60° x
Complement angle y= = 20°, z + 40° + = 180°
⇒ 160° ≠ 180° 3 3
= 90° – θ = 90° – 80° = 10°
⇒ z + 20° = 180° − 40°
[Q sum of two complement angles is Hence, statements I and II are incorrect.
⇒ z = 120°
90°]
23
248 CDS Pathfinder

TRIANGLES
Generally (8-10) questions have been asked from this chapter. Generally questions are asked from
the topic of pythagoras theorem, similarity of triangles and mid-point theorem.

TRIANGLE
A plane (closed) figure bounded by three line segments is called a triangle. It is A
denoted by ∆.
∆ABC has
• three vertices, namely A , B and C. • three sides, namely AB, BC and CA .
• three angles, namely ∠A , ∠B and ∠C.
• Sum of three angles of a triangle is 180 °. i.e., ∠A + ∠B + ∠C = 180 °
B C
D
1 1
• Area of a triangle = × base × height = × BC × AD
2 2

Classification of Triangle
1. On the Basis of Angles A
Perpendicular

Hy
(i) Right Angled Triangle A triangle in which one of the angle measures 90 ° po
ten
is called a right angled triangle. The side opposite to the right angle is called us
e
its hypotenuse and the remaining two sides are called as perpendicular and
base depending upon conditions. Here, ∆ABC is a right angled triangle in θ
which ∠B = 90 ° and AC is hypotenuse. B Base C
C
(ii) Acute Angled Triangle A triangle in which every angle measure more than
0° but less than 90° is called an acute angled triangle.
Here, ∆ABC is an acute angled triangle. A B
(iii) Obtuse Angled Triangle A triangle in which one of the angle measures A
more than 90° but less than 180° is called an obtuse angled triangle.
Here, ∆ ABC is an obtuse angled triangle and ∠ABC is the obtuse angle.
B C
MATHEMATICS Triangles 249

2. On the Basis of Sides • Centroid divides the median in the ratio 2 : 1.


(i) Equilateral Triangle A triangle having A • A median bisects the area of the triangle i.e.
all sides equal is called an equilateral 1
Ar( ∆ABD) = Ar( ∆ADC ) = Ar( ∆ABC ), etc.
triangle. Here ∆ABC, is an equilateral 2
triangle in which AB = BC = AC. Incentre and Angle Bisector The
B C A
All angles are equal and are of measure point of intersection of all the three
60°. angle bisectors of a triangle is called its
r
incentre. It is denoted by I.
(ii) Isosceles Triangle A triangle in which two sides
• The circle with centre I and touches all x r
are equal, is called an isosceles triangle. Here, ∆ABC B D C
is an isosceles triangle as AB = AC. the sides is called incircle and radius of
• Angle opposite to equal sides are equal. this circle is called inradius denoted by ‘r’.
i.e. ∠B = ∠C 1 Side
• Inradius = × Height = ,
A 3 2 3
ID → Inradius, AD → Height
Perpendicular Bisector and Circumcentre The point
B C of intersection of the perpendicular bisectors of the sides
of a triangle is called its
(iii) Scalene Triangle A triangle in which all the sides A
circumcentre. It is denoted by C.
are of different lengths is called a scalene triangle.
∆ ABC is a scalene triangle as AB ≠ BC ≠ AC. The circle with centre C and
A passing through vertices C
A , B and C is called circumcircle. R
Radius of circumcircle is called B O D
circumradius denoted by R .
2 Side
B C Circumradius (CO) = × Height =
3 3
Note The sum of the lengths of three sides of a triangle is called
its perimeter. EXAMPLE 1. In the diagram given below what is the
So, perimeter of ∆ABC = AB + BC + AC
sum of all the angles ∠A, ∠B, ∠C, ∠D, ∠E and ∠F?
Some Term Related to a Triangle A

Altitudes and Orthocentre The F B


altitude of a triangle is a line segment A
perpendicular drawn from vertex to E C
the side opposite to it. The side on F E
which the perpendicular is being drawn H D
is called its base. a. 120° b. 180°
Here, In ∆ABC, AD, BE and CF are B D C
c. 290° d. 360°
altitudes drawn on BC, AC and AB, respectively.
Sol. d. Since, sum of the angles of a triangle is 180°.
• Altitudes of a triangle are concurrent.
In ∆ AEC , ∠A + ∠C + ∠E = 180° …(i)
• The point of intersection of all the three altitudes of a
and In ∆ BDF , ∠B + ∠D + ∠F = 180° …(ii)
triangle is called its orthocentre.
On adding the Eqs. (i) and (ii), we get
Medians and Centroid A line ∠A + ∠B + ∠C + ∠D + ∠E + ∠F = 180° + 180° = 360°
A
segment joining the mid-point of a side
to the opposite vertex is called median. EXAMPLE 2. ABC is an isosceles triangle in which
AB = AC, CH = CB and HK is parallel to BC. If the
F E
Here, In ∆ABC, AD, BE and CF are exterior ∠CAX = 137 °, then what is the measure of
medians. G
∠HCK?
• The medians of a triangle are B D C 1°
concurrent. a. 68 b. 43°
2
• The point of intersection of all the three medians of a

triangle is called its centroid. It is denoted by G. c. 25 d. 137°
2
250 CDS Pathfinder

Sol. c. Q ∠CAX = 137° • In ∆ABC, if ∠B > 90 ° and AD is perpendicular dropped


X on BC, then AC 2 = AB 2 + BC 2 + 2BC ⋅ BD
A
A

K
H

D B C
B C


1
∠ABC =
1
(137° ) = 68 °
• In ∆ABC, if ∠B < 90 ° and AD ⊥ BC, then
2 2 AC 2 = AB 2 + BC 2 − 2BC ⋅ BD
1
∴ Again, BC = CH and ∠ABC = 68 ° A
2
1
Therefore, ∠CHB = 68 ° , Therefore, ∠HCB = 43°
2
1 1
Hence, ∠HCK = 68 ° − 43° = 25 °
2 2
B D C
Some Important Results of a Triangle • In any triangle, the sum of the A
• If two angles of a triangle are equal, then the sides square of any two sides is equal
opposite to them are also equal. to twice the square of half of the
• In a triangle, the side opposite to a greater angle is the third side together with twice
longest side. the square of the median which B D E C
bisect the third side.
• The sum of all the three interior angles of a triangle is
• Here, AD is median, so
180°.
2
• If a side of a triangle is produced, then the exterior 1 
AB 2 + AC 2 = 2AD 2 + 2  BC
angle so formed is equal to the sum of the two interior 2 
opposite angles. • In a ∆ ABC, three times the sum of the squares of the
• An exterior angle of a triangle is greater than either of sides of a triangle is equal to four times the sum of the
the interior opposite angles. squares of the medians of the triangle.
• A triangle must have atleast two acute angles. A

• If a perpendicular is drawn from the vertex of a


F E
right angled triangle to the hypotenuse, then the
triangles on both sides of the perpendicular are
B C
similar to the original triangle and also to each other. D
• In a right angle ∆ABC, ∠B = 90 ° and AC is hypotenuse. So, in triangle, if AD, BE, FC are the medians, then
The perpendicular BD is dropped on hypotenuse AC • 3( AB 2 + BC 2 + AC 2 ) = 4 ( AD 2 + BE 2 + CF 2 )
from right angle vertex B, then • In an equilateral ∆ ABC, the side BC is trisected at D.
AB × BC Then, 9 AD 2 = 7 AC 2
(i) BD = A
AC • When the bisector of one internal base angle and the
AB 2 bisector of external angle meet at a point then the
(ii) AD =
AC D formed angle is equal to one half of the vertical angle.
2
BC A
(iii) CD = E
AC
1 1 1 C
(iv) 2
= 2
+ 2
B
BD AB BC
• The area of the equilateral triangle described on the side B C D
of a square is half the area of the equilateral triangle 1
• Here, ∠BEC = ∠BAC
described on its diagonal. 2
MATHEMATICS Triangles 251

• The side BC of ∆ ABC is produced to D. The bisector • Sum of any two sides of a triangle is greater than its
of ∠A meets BC in L . Then third side.
∠ABC + ∠ACD = 2 ∠ALC. D
A
A

B C

• Here, in ∆ ABC, AB + AC > BC


B L C D • similarly, AB + BC > AC, BC + AC > AB
• In a ∆ ABC the bisector of ∠B and ∠C intersect each • The difference between any two sides of a triangle is
other at a point O. less than its third side.
A
A
O
D

B C B C

1 Here, in ∆ ABC, AC − AB < BC


• then, ∠BOC = 90 ° + ∠A •
2 • Similarly, BC − AC < AB, BC − AB < AC
• In a ∆ABC, the side AB and AC are produced to P and • If the bisector of the vertical angle of a triangle bisects
Q , respectively. The bisectors of ∠PBC and ∠QCB
the base, then that triangle is an isosceles.
intersect at a point O.
A
• If the altitude from one vertex of a triangle bisects the
opposite side, then the triangle is an isosceles.
• The perpendiculars drawn from the vertices of equal
B C
angles of an isosceles triangle to the opposite sides are
equal.
P O Q • Medians of equilateral triangle are equal.
1
Then, ∠BOC = 90 ° − ∠A • If D is the mid-point of the hypotenuse AC of a right
2 1
angled ∆ ABC. Then, BD = AC
• In ∆ ABC, ∠B > ∠C. If AN is the bisector of ∠BAC 2
and AM ⊥ BC, then
A
A
D

B C

B M N C • The sum of any two sides of a triangle is greater than


1 twice the median drawn to the third side.
∠MAN = ( ∠B − ∠C )
2 • Perimeter of a triangle is greater than the sum of its
• The bisectors of the base angles of a triangle can never three medians.
enclose a right angle.
A
• A triangle is an isosceles if and only if any two altitudes
are equal. D F E
• If the three sides of a triangle be
A
produced in order, then the sum of all C
B
the exterior angles so formed is 360°. D
B F
• So, ∠DAB + ∠EBC + ∠ACF = 360 ° C
• So, AB + BC + AC > AD + BE + CF .
E
252 CDS Pathfinder

EXAMPLE 3. In the given figure, I is the incentre of In congruent triangles, corresponding parts are equal
∆ABC. What is the measure of angle A . and we write in short ‘CPCT’, i.e. corresponding part of
A congruent triangles.

Criteria for Congruence of Triangles


I
108°
Theorem 1 Two triangles are congruent if two sides
B C
and the included angle of one triangle are equal to the
corresponding sides and the included angle of other
a. 54° b. 18° c. 36° d. None of these triangle. (SAS criteria)
Sol. c. Here, I is the incentre of the ∆ ABC. Theorem 2 Two triangles are congruent if two angles
∴ BI and CI are the bisectors of ∠B and ∠C, then
and the included side of one triangle are equal to the
1 corresponding two angles and the included side of the
we know that, ∠BIC = 90° + ∠A other triangles. (ASA criteria)
2
1 Theorem 3 Two triangles are congruent if three sides
⇒ 108° = 90° + ∠A
2 of one triangle are equal to the corresponding three
1 sides of the other triangle (SSS).
⇒ ∠A = 108° − 90° = 18°
2 Theorem 4 Two triangles are congruent if the
∴ ∠A = 36°
hypotenuse and other side of one triangle are equal to
EXAMPLE 4. In a triangle ABC, ∠BAC = 90 ° and AD the hypotenuse and the corresponding side of the other
is perpendicular to BC. If AD = 6 cm and BD = 4 cm, triangle (RHS).
then the length of BC is Theorem 5 Two triangles are congruent if two angles
a. 8 cm b. 10 cm c. 9 cm d. 13 cm and a side other than the included side of one triangle
are equal to the corresponding two angles and a side
Sol. d. In ∆ABC, ∠BAC = 90° other than the included side of other triangle.
∴ AB = AD 2 + BD 2 [By pythagoras theorem] (AAS Criteria)
= 36 + 16 = 52
We know that, in a right angled C EXAMPLE 5. In a ∆ABC, the altitudes BD and CE are
triangle the perpendicular is drawn on equal and ∠A = 36 °. What is the value of the ∠B?
hypotenuse from right angle vertex, a. 72° b. 84° c. 18° d. 36°
then
Sol. a. For the ∆BDC and ∆BEC,
AB 2 D
BD =
BC 6 4 A
AB 2 = BD × BC
⇒ ( 52) 2 = BC × 4 A B E D
52
⇒ BC = = 13 cm
4 B C

BD = EC, BC = BC and ∠BEC = ∠BDC = 90°


Congruent Figures Thus, ∆ BEC ~
= ∆ BDC [by RHS rule]
The geometrical figures having the same shape and size 180° − 36°
are known as congruent figures. ∴ ∠B = ∠C = = 72° each
2
e.g. Two circles of the same radii and two squares of the
same sides are congruent. EXAMPLE 6. In the given figure, D is the midpoint of
BC, DE ⊥ AB and DF ⊥ AC such that DE = DF , then
Congruent Triangles which of the following is correct?
A
Two triangles are said to be congruent, if both are
exactly of same size i.e. all angles and sides are equal to
corresponding angles and sides of other.
• Every triangle is congruent to itself ∆ ABC ~ = ∆ ABC E F

• If ∆ ABC ~ = ∆ DEF , then ∆DEF ~ = ∆ ABC


B D C
• If ∆ ABC ~ = ∆ DEF and a. AB = AC b. AC = BC
∆ DEF ~= ∆ PQR , then ∆ ABC ~ = ∆ PQR c. AB = BC d. None of these
MATHEMATICS Triangles 253

Sol. a. In right angled ∆BED and right angled ∆CFD Theorem 3 The line joining the
A
DE = DF (given) mid-points of any two sides of a
hypotenuse BD = hypotenuse CD triangle is parallel to the third side
[QD is the mid-point of BC ] and is half of the third side. P Q
∴∆BED ≅ ∆CFD [by RHS congruency]
Here, P and Q are mid-point of AB
⇒ ∠B = ∠C ⇒ AC = AB [sides opposite to equal angles] 1
and AC. So, PQ = BC. B C
2
Similar Figures
Theorem 4 If two triangles are equiangular, then
The geometrical figures having the same shape, but different the ratio of their corresponding sides is the same as
sizes are known as similar figures. the ratio of the corresponding altitudes.
• The congruent figures are always similar, but two similar A P
figures need not be congruent.
e.g. Any two circles are similar. Any two rectangles are
similar.

Similar Triangles B D C Q S R
Two triangles are said to be similar to each other, if Here, ∆ ABC ~ ∆ PQR and AD and PS are altitude
(i) their corresponding sides are proportional. on BC and QR, respectively, then
BC AD
(ii) their corresponding angles are equal. =
QR PS
A P
Theorem 5 If two triangles are equiangular, then
the ratio of the corresponding sides is the same as
B C Q R the ratio of the corresponding angle bisector
Here, ∆ABC and ∆PQR are similar triangles. segments.
∴ ∠A = ∠P, ∠B = ∠Q, ∠C = ∠R A R
AB BC AC
and = =
PQ QR PR
Then, ∆ABC ~ ∆PQR
where, symbol ~ is read as, ‘is similar to’. B D C S P T
Here, ∆ ABC and ∆ RST are equiangular/similar and
Some Results on Similar Triangles AD, RP are the angle bisectors of ∠A and ∠R, then
Theorem 1 If a line is drawn parallel to one side of a BC AD
=
triangle to intersect the other two sides in distinct point, ST RP
then it divides these sides in the same ratio. It is also called
Theorem 6 If two triangles are equiangular, then
basic proportionality theorem.
the ratio of the corresponding sides is the same as
AD AE
=
A
Here, DE | | BC, then the ratio of the corresponding medians.
DB EC A P
AD AE
or = D E
AB AC
AB AC
or = B C
DB EC
A
Theorem 2 The internal bisector of an B D C Q S R
angle of a triangle divides the opposite sides Here, ∆ ABC and ∆ PQR are equiangular and AD, PS
internally in the ratio of the sides containing are the medians, then
the angle. BC AD
=
Here, AD is internal bisector of ∠A, then B D C QR PS
AB BD
=
AC DC Note: If two triangles are similar, then ratio of their
corresponding sides is same as ratio of their perimeters.
254 CDS Pathfinder

Criteria for Similarity of Two Triangles Sol. d. In ∆ABC and ∆PQC.

1. AAA similarity If the corresponding angles of two B Q


triangles are equal then their corresponding sides are
x
proportional and hence the two triangles are similar. a
c
Corollary AA similarity If two angles of one C
A P b
triangle are equal to the corresponding two angles of
another triangle, then the two triangles are similar. AB || PQ
∴ ∆ABC ~ ∆PQC
2. SSS similarity If the corresponding sides of two
PC PQ b a
triangles are proportional, then their corresponding ∴ = ⇒ =
AC AB c+ b x
angles are equal, and hence the two triangles are
similar. a ( c + b) ac
∴ x= = +a
b b
3. SAS similarity If one angle of a triangle is equal to
the corresponding angle of the other triangle and the Areas of Similar Triangles
sides including these angles are proportional, then
Theorem 1 The ratio of areas of two similar triangles
the two triangles are similar.
is equal to the square of the ratio of their corresponding
4. RHS similarity If hypotenuse and one side of a right sides.
triangle are proportional to the hypotenuse and A P
corresponding side of other right triangle then the
two triangles are similar.

EXAMPLE 7. If AB = 4.7, BC = 8.9, CA = 11.5, then EA is B C Q R

A Here, ∆ ABC ~ ∆ PQR


Area ( ∆ ABC ) AB 2 BC 2 AC 2
4.7 = = =
Area ( ∆ PQR) PQ2 QR2 PR2
B E 11.5
Theorem 2 The areas of two similar triangles is equal
to the ratio of the squares of corresponding altitudes.
8.9
C Here, ∆ ABC ~ ∆ PQR
A P
a. 6.07 b. 3.97
c. 2.37 d. None of these
Sol. b. Since, BE is the bisector of ∠ABC.
AB AE 4 .7 AE
∴ = ⇒ = [Q EC = AC − AE ]
BC EC 8.9 115. − AE B D C Q S R

⇒ AE = 397
. Area ( ∆ ABC ) AD 2
Then , =
Area ( ∆ PQR) PS 2
EXAMPLE 8. In the given triangle, AB is parallel to
PQ. AP = c, PC = b, PQ = a, AB = x. What is the value of Theorem 3 The areas of two similar triangles is equal
x? to the ratio of the squares of the corresponding
medians.
B Q A P

C
A P b
s

ab bc B D C Q S R
a. a + b. a +
c a
ca ac Here, ∆ ABC ~ ∆ PQR ,
c. b + d. a + Area (∆ ABC ) AD 2
b b then =
Area (∆ PQR) PS 2
MATHEMATICS Triangles 255

Theorem 4 The areas of two similar triangles is equal EXAMPLE 10. In a given figure, QR is parallel to AB
to the ratio of squares of the corresponding angle and DR is parallel to QB. What is the number of distinct
bisector segments. pairs of similar triangles?
A P P
D
Q R

B X C Q Y R
A Bs

Here, ∆ ABC ~ ∆ PQR a. 1 b. 2 c. 3 d. 4


Area (∆ ABC ) AX 2 Sol. c. Since, QR is parallel to AB.
So, here =
Area (∆ PQR) PY 2 ∴ ∆PQR ~ ∆PAB
Theorem 5 If the areas of two similar triangles are Also, DR is parallel to QB., ∆PQB ~ ∆PDR
equal, then the triangles are congruent. Again, DR || QB and QR || AB,
or ∴ ∆DQR ~ ∆QAB
Equal and similar triangles are congruent.
Theorem 6 The line segments is joining the mid-points EXAMPLE 11. In a triangle, a line XY is drawn parallel
of the sides of a triangle form four triangles, each of to BC meeting AB in X and AC in Y. The area of the
which is similar to the original triangle. ∆ ABC is 2 times the area of the ∆ AXY. In what ratio X
A divides AB?
a. 1: 2 b. 2 : 1 c. ( 2 − 1) : 1 d. 1: ( 2 − 1)
F E Area ( ∆ ABC) AB 2
Sol. d. Q =
Area (∆ AXY) AX 2
B D C A
Here, D, E and F are mid-point of BC, AC and AB. Then,
∆ AFE, ∆ FBD, ∆ EDC and ∆ DEF is similar to ∆ ABC. X Y
Here, also
2
1 
 AB 
Area (∆ DEF ) DE 2 2  1
B C
= = = 2 Area (∆ AXY) 2
Area (∆ ABC ) AB 2
AB 2 4 AB 2 AB 2 AB
⇒ = 2 ⇒ = , = 2
Area (∆ AXY) AX 1 AX 2 AX
So, area ( ∆ DEF ) : area ( ∆ ABC ) = 1 : 4
⇒ AB = 2 AX ⇒ AX + BX = 2 AX
EXAMPLE 9. In the figure given below, BC is parallel ⇒ BX = A X ( 2 − 1)
to DE and DE : BC = 3 : 5. What is the ratio of area of AX 1
the ∆ABC to that of ∆ADE? ∴ =
BX 2 −1
A
So, X divides AB in 1 : ( 2 − 1).

D E
PYTHAGORAS THEOREM
B C In a right angled triangle, the square of the hypotenuse is
equal to the sum of the square of the other two sides.
a. 3 : 1 b. 5 : 3 c. 9 : 2 d. 25 : 9
i.e. In ∆ABC , if ∠B = 90 ° , then AB 2 + BC 2 = AC 2
Sol. d. Q Given, DE : BC = 3 : 5 A
Since, DE || BC ⇒ ∠ADE = ∠ABC
and ∠AED = ∠ACB
∴ ∆ABC ~ ∆ADE [by AA similarity]
Area of ∆ABC  BC  2 25
∴ =  = or, 25 : 9
Area of ∆ADE  DE  9 B C
256 CDS Pathfinder

Converse of Pythagoras theorem B

In a triangle, if the square of one side is equal to the


sum of square of the other two sides then angle D L
opposite to the first side is a right angle i.e. In ∆ABC, if 11 m
AC 2 = AB 2 + BC 2 , then ∠B = 90 °. 6m

EXAMPLE 12. Two poles of heights 6m and 11m


stand vertically on a plane ground at a distance of C 12 m A
12m from each other. The distance between their Draw DL ⊥ AB
tops is Then, BL = AB − AL = AB − CD = (11− 6) m = 5m
a. 9 m b. 10 m and DL = AC = 12m
c. 13 m d. 15 m
In right angled ∆BDL,
Sol. c. Let AB and CD be two poles such that AB = 11m, BD 2 = DL2 + BL2 = 52 + 122 = 169
CD = 6 m and AC = 12 m. ⇒ BD = 169 = 13m

PRACTICE EXERCISE
1. If the bisector of an angle of a triangle bisects 7. Let ABC be an isosceles triangle in which AB = AC
the opposite side, then the triangle is and BD ⊥ AC. Then, BD 2 − CD 2 is equal to
(a) equilateral (b) isosceles A
(c) scalene (d) right angled triangle
2. The line segments joining the mid-points of the D
sides of a triangle form four triangles each of
which is
(a) similar to the original triangle
B C
(b) congruent to the original triangle 1
(c) an equilateral triangle (d) an isosceles triangle (a) 2DC ⋅ AD (b) 2 AD ⋅ BC (c) 3DC ⋅ AD (d) AD ⋅ DC
2
3. In a ∆ ABC , BD and CE are perpendicular on AC 8. D and E are the points on the sides AB and
and AB, respectively. If BD = CE , then the AC respectively of a ∆ ABC and AD = 8 cm,
∆ ABC is DB = 12 cm, AE = 6 cm and EC = 9 cm, then BC is
(a) equilateral (b) isosceles (c) right angled (d) scalene equal to
2 5 3 2
4. If the length of hypotenuse of a right angled (a) DE (b) DE (c) DE (d) DE
2 5 2 2 3
triangle is 5 cm and its area is 6 cm , then the
length of the remaining sides are 9. A vertical stick 15m long casts a shadow 12m
(a) 1 cm and 3 cm (b) 3 cm and 2 cm long on the ground. At the same time, a tower
(c) 3 cm and 4 cm (d) 4 cm and 2 cm casts a shadow 50m long on the ground. The
height of the tower is
5. ∆ ABC is such that AB = 3 cm, BC = 2 cm and (a) 60 m (b) 62 m (c) 62.5 m (d) 63 m
AC = 2.5 cm. ∆ DEF is similar to ∆ ABC. If
EF = 4 cm, then the perimeter of ∆ DEF is 10. The areas of two similar triangles are 81 cm 2
and 49 cm 2, respectively. The ratio of their
(a) 5 cm (b) 7.5 cm (c) 15 cm (d) 18 cm
corresponding heights is
6. A soldier goes to a warfield and runs in the (a) 9 : 7 (b) 7 : 9 (c) 6 : 5 (d) 81 : 49
following manner. From the starting point, he
goes West 25 m, then due North 60 m, then due 11. If D and E are points on the sides AB and AC,
East 80 m, and finally due South 12 cm. The respectively of a ∆ ABC such that DE|| BC. If
distance between the starting point and the AD = x, DB = x − 2, AE = x + 2 and EC = x − 1.
finishing point is The value of x is
(a) 177 m (b) 103 m (c) 83 m (d) 73 m (a) 2.5 (b) 2 (c) 3 (d) 4
MATHEMATICS Triangles 257

12. In the adjoining figure, ABCD is A B 18. In ∆ ABC , ∠C = 90° and CD ⊥ AB, also ∠A = 65°,
a trapezium in which AB||CD O then ∠CBA is equal to
and its diagonals intersect at O. C
If AO = ( 3x − 1), OC = ( 5x − 3),
BO = ( 2x + 1) and OD = (6x − 5), D C
then x is equal to
65°
(a) 1 (b) 2 (c) 3 (d) 4 A D B
13. In the adjoining figure, AE is the bisector of (a) 25° (b) 35° (c) 65° (d) 40°
exterior ∠CAD meeting BC produced in E. If
AB = 10 cm, AC = 6 cm and BC = 12 cm, then CE 19. The angles of a triangle are in the ratio 2 : 3 : 4.
is equal to The angles of triangle are, respectively
D (a) 30°, 60°, 90° (b) 40°, 60°, 80°
A (c) 60°, 40°, 80° (d) 20°, 60°, 80°
20. In figure, D and E are points on sides AB, AC of
∆ ABC such that DE|| BC. If ∠B = 30° and
B C E
∠A = 40°, then x , y and z are, respectively
A
(a) 6 cm (b) 12 cm (c) 18 cm (d) 20 cm
40°
14. If D , E and F are respectively the mid-points of z°
sides BC , AC and AB of a ∆ ABC. If EF = 3 cm, D x°
E
FD = 4 cm and AB = 10 cm, then DE , BC and CA,
respectively will be equal to y°
10 30°
(a) 6, 8 and 20 cm (b) , 9 and 12 cm B C
3
(c) 4, 6 and 8 cm (d) 5, 6 and 8 cm (a) 30°, 110°, 110° (b) 30°, 105°, 105°
(c) 30°, 85°, 85° (d) 30°, 95°, 95°
15. In ∆ PQR , ∠Q = 3a, ∠P = a , ∠R = b and
21. In figure, AB, EF and CD are parallel lines. Given
3b − 5a = 30, then the triangle is
that GE = 5 cm, GC = 10 cm and DC = 18 cm, then
(a) scalene (b) isosceles
EF is equal to
(c) equilateral (d) right angled D
16. In ∆ ABC show in the figure B A
∠A = 90°. Let D be a point on
BC such that BD : DC = 1 : 3. If D E
M G
DM and DL, respectively are
perpendicular on AB and AC,
then DM and LC are in the A C B F C
L
ratio of (a) 11 cm (b) 5 cm (c) 6 cm (d) 9 cm

(a) 1 : 3 (b) 1 : 2 (c) 1 : 1 (d) 4 : 1 22. In the given figure, PQ > PR. QS and RS are the
bisectors of ∠Q and ∠R respectively, then which
17. In a right angled ∆ ABC, right angled at B, if P of the following is correct?
and Q are points on the sides AB and AC P
respectively, then
A

S
P

Q R
B Q C (a) SQ < SR (b) SQ = SR
(a) AQ 2 + CP 2 = 2 ( AC 2 + PQ 2 ) (c) SQ > SR (d) None of these

(b) 2 ( AQ 2 + CP 2 ) = AC 2 + PQ 2 23. In a ∆ABC, AB = AC and 60° < A < 90°. If AB = c,


AC = b and BC = a, then which of the following is
(c) AQ 2 + CP 2 = AC 2 + PQ 2
correct?
1 (a) b < a < 2 b (b) c / 3 < a < 3c
(d) AQ + CP = ( AC + PQ )
2 (c) b < a < b 3 (d) c < a < c 2
258 CDS Pathfinder

(a) 60°, 75°, 75°, 60° (b) 50°, 75°, 75°, 65°
24. In ∆ABC , AD ⊥ BC, then
(c) 60°, 70°, 60°, 70° (d) 60°, 60°, 70°, 70°
(a) AB2 − BD2 = AC 2 − CD2 (b) AB2 + BD2 = AC 2 − CD2
31. In the given figure, AB||CD , ∠PTB = 55° and
(c) AB2 + BD2 = AC 2 + CD2 (d) AB2 + AC 2 = BD2 + DC 2
∠DVS = 45°, then what is the sum of the
25. ∆ ABC is a right angled at C and P is the length measures of ∠CUQ and ∠RTP?
of the perpendicular from C to AB. If BC = a, R P
AC = b, AB = c, then 55°
A B
a p 1 1 1
(a) = (b) pc = ab (c) + = (d) None of these T
b c a b ab
26. ABC is a triangle and the perpendicular drawn C
U V 45°
D
from A meets BC in D. If AD 2 = BD ⋅ DC, then Q S
which one of the following is correct? (a) 180° (b) 135° (c) 110° (d) 100°
(a) ABC must be an obtuse angled triangle
(b) ABC must be an acute angled triangle 32. What is the value of x in the figure given below?
(c) Either ∠B ≥ 45° or ∠C ≥ 45° (d) BC 2 = AB2 + AC 2 A a c B

27. If ∆ is the area of a right angled triangle and b is b


one of the sides containing the right angle, then x
what is the length of the altitude on the hypotenuse? C
2 ∆b 2 ∆2 b (a) b − a − c (b) b − a + c (c) b + a − c (d) π − (a + b + c )
(a) (b)
b 4 + 4∆2 b 4 + 4∆2 33. In the given figure, what is the value of x?
2 ∆b 2
2 ∆2 b 2
(c) (d) B C
b 4 + 4∆2 b 4 + ∆2
D E
2x 120°
4
28. In figure, AB||CD. If x = y
B
y° z°
D
3 x° x
3
and y = z, then the value of C A
8 A (a) 30° (b) 40° (c) 45° (d) 60°
x is
(a) 48° (b) 96° (c) 36° (d) None of these 34. PQR is a triangle right angled at Q. If X and Y
are the mid-points of the sides PQ and QR
29. In the given figure, AB||CD , then the values of
respectively, then which one of the following is
x, y and z are, respectively not correct?
D 2
B (a) RX 2 + PY 2 = 5 XY 2 (b) RX 2 + PY = XY 2 + PR 2
z° (c) 4 (RX 2 + PY 2 ) = 5PR 2 (d) RX 2 + PY 2 = 3 (PQ 2 + QR 2 )

P
35. In the given figure, AB is parallel to CD. If
y° ∠DCE = x and ∠ABE = y, then what is ∠CEB
35
75° ° equal to?
C x D
A C
(a) 75°, 35°, 80° (b) 70°, 35°, 60° B
A
(c) 35°, 70°, 75° (d) 70°, 35°, 80° y E
30. In the given figure, AB||CD , and EF ||GH . The (a) y − x (b) (x + y) / 2 (c) x + y − ( π / 2 ) (d) x + y − π
values of x, y , z and t are respectively 36. In the figure given below, what is the sum of the
G angles formed around A, B, C except the angles of
E
the ∆ ABC?
P Q
A
110° z° B A


R t° S
C D
B C
60° H
F (a) 360° (b) 720° (c) 900° (d) 1000°
MATHEMATICS Triangles 259

37. In the given figure, ABC is A 43. In the figure, ∠B = 38°, A


an equilateral triangle of side AC = BC and AD = CD. What
length 30 cm. XY is parallel is the value ∠D?
to BC, XP is parallel to AC (a) 26° (b) 28° 38°
and YQ is parallel to AB. If X Y (c) 38° (d) 52° B C D
( XY + XP + YQ ) is 40 cm,
B C
then what is PQ equal to? P Q 44. Consider the following statements in respect of
any triangle.
(a) 5 cm (b) 12 cm
(c) 15 cm (d) None of these I. The three medians of a triangle divide it into six
triangles of equal area.
38. O is any point on the X
II. The perimeter of a triangle is greater than the
bisector of the acute sum of the lengths of its three medians.
angle ∠ XYZ. The line P
OP is parallel to ZY . O Which of the statement(s) given above is/are
Then, ∆ YPO is Y Z correct?
(a) Only I (b) Only II (c) Both I and II (d) Neither I nor II
(a) scalene
(b) isosceles but not right angled 45. Consider the following in respect of C
(c) equilateral the given figure :
(d) right angled and isosceles I. ∆ DAC ~ ∆ EBC E D
39. The median BD of the ∆ ABC meets AC at D. If II. CA /CB = CD /CE
III. AD /BE = CD /CE
A B
1
BD = AC, then which one of the following is
2 Which of the statement(s) given above is/are
correct? correct?
(a) ∠ACB = 1 right angle (b) ∠BAC = 1 right angle (a) II and III (b) I and II (c) I and III (d) All of these
(c) ∠ABC = 1 right angle (d) None of these 46. Consider the following statements :
40. ABC is a triangle, X is a point outside the I. Let PQR be a triangle in which PQ = 3 cm,
∆ ABC such that CD = CX , where D is the point QR = 4 cm and RP = 5 cm. If D is a point in the
of intersection of BC and AX and ∠BAX = ∠XAC. plane of the ∆ PQR such that D is either outside
Which one of the following is correct? it or inside it, then
(a) ∆ABD and ∆ACX are similar (b) ∠ABD < ∠ACD DP + DQ + DR > 6 cm
(c) AC = CX (d) ∠ADB > ∠DXC II. PQR is a right angled triangle.
π
41. In the figure given, ∠ABD = ∠PQD = ∠CDQ = . Which one of the following is correct in respect of
2 the above two statements?
If AB = x, PQ = z and CD = y, then which one of (a) Both statements I and II are individually true and
the following is correct? statement II is the correct explanation of statement I
A
1 1 1 C (b) Both statements I and II are individually true but
(a) + = statement II is not the correct explanation of statement I
x y z x P
y (c) Statement I is true and statement II is false
1 1 1
(b) + = z
(d) Statement I is false and statement II is true
x z y B Q D
1 1 1 47. I. Let LMN be a triangle. Let P , Q be the
(c) + =
z y x mid-points of the sides LM , LN , respectively. If
1 1 2 PQ 2 = MP 2 + NQ 2, then LMN is a right angled
(d) + =
x y z triangle at L.
42. ∆ PQR is right angled at Q, PR = 5 cm and II. If in a ∆ ABC , AB2 > BC 2 + CA 2, then ∠ ACB is
QR = 4 cm. If the lengths of sides of another obtuse.
∆ ABC are 3 cm, 4 cm and 5 cm, then which one Which of the following is correct in the light of
of the following is correct? the above statements?
(a) Area of ∆ PQR is double that of ∆ ABC (a) Both statements I and II are individually true and
statement II is the correct explanation of statement I
(b) Area of ∆ ABC is double that of ∆ PQR
∠Q (b) Both statements I and II are individually true but
(c) ∠B = statement II is not the correct explanation of statement I
2
(c) Statement I is true and statement II is false
(d) Both triangles are congruent (d) Statement I is false and statement II is true
260 CDS Pathfinder

PREVIOUS YEARS’ QUESTIONS 54. I. Let the side DE of a ∆DEF be divided at S, so


1 1 1 DS 1
48. In a ∆ ABC, ∠ A + ∠ C + ∠B = 80°, then that = ⋅ If a line through S parallel to EF
2 3 2 DE 2
what is the value of ∠C ? e 2012 II meets DF at T, then the area of ∆DEF is twice
(a) 35° (b) 40° (c) 60° (d) 70° the area of the ∆DST.
49. In a ∆ ABC, side AB is extended beyond B, side II. The areas of the similar triangles are proportional
to the squares of the corresponding sides.
BC beyond C and side CA beyond A. What is the
sum of the three exterior angles? e 2012 II Which one of the following is correct in respect of
(a) 270° (b) 305° (c) 360° (d) 540° the above statements? e 2013 II
(a) Both statements I and II are true and statement II is
50. In the given figure, ABC is a triangle, BC is the correct explanation of statement I
parallel to AE. If BC = AC, then what is the (b) Both statements I and II are true but statement II is
value of ∠ CAE ? e 2012 II not the correct explanation of statement I
(c) Statement I is true but statement II is false
C
B (d) Statement II is true but statement I is false
65°
D 95° 55. Let ABC be a triangle with AB = 3 cm and
AC = 5 cm. If AD is a median drawn from the
E vertex A to the side BC, then which one of the
A
following is correct? e 2013 II
(a) 20° (b) 30° (c) 40° (d) 50° (a) AD is always greater than 4 cm but less than 5 cm
51. In the given figure, AB is parallel to CD. (b) AD is always greater than 5 cm
∠ABC = 65° , ∠CDE = 15° and AB = AE. What is (c) AD is always less than 4 cm
the value of ∠AEF ? e 2012 II (d) None of the above
A F 56. In the figure given above, ∠PQR = 90° and QL is
a median, PQ = 5 cm and QR = 12 cm. Then, QL
is equal to e 2013 II
P

B C
E
L

D Q R
(a) 30° (b) 35° (c) 40° (d) 45° (a) 5 cm (b) 5.5 cm (c) 6 cm (d) 6.5 cm

52. The angles x° , a ° , c° and ( π − b)° are indicated in 57. In a ∆ABC, ∠BCA = 90° and CD is perendicular
the figure given below : e 2012 II to AB. If AD = 4 cm and BD = 9 cm, then the
E value of DC will be e 2013 II
D x° (a) 18 cm (b) 20 cm (c) 65 cm (d) 6 cm
58. ABC is a triangle, where BC = 2 AB , ∠C = 30°
(π−b)°
A
c° a°
T and ∠A = 90°. The magnitude of the side AC is
B C e 2013 II
Which one of the following is correct ? e 2012 II 2 BC 3 BC BC 3 BC
(a) (b) (c) (d)
(a) x° = a° + c ° − b ° (b) x° = b ° − a° − c ° 3 4 3 2
(c) x° = a° + b ° + c ° (d) x° = a° − b ° − c °
59. Let ABC be an equilateral triangle. If the side
53. Consider the following statements: BC is produced to the point D so that BC = 2 CD,
I. If G is the centroid of ∆ABC, then GA = GB = GC. then AD 2 is equal to e 2013 II

II. If H is the orthocentre of ∆ABC, then (a) 3 CD2 (b) 4 CD2 (c) 5 CD2 (d) 7 CD2
HA = HB = HC. 60. ABC is a right angled triangle such that
Which of the statement(s) given above is are AB = a − b, BC = a and CA = a + b . D is a point
correct? e 2013 II on BC such that BD = AB. The ratio of BD : DC
(a) Only I (b) Only II for any value of a and b is given by e 2013 II
(c) Both I and II (d) Neither I nor II (a) 3 : 2 (b) 4 : 3 (c) 5 : 4 (d) 3 : 1
MATHEMATICS Triangles 261

61. The side BC of a ∆ABC is produced to D, 71. If triangles ABC and DEF are similar such that
bisectors of the ∠ABC and ∠ACD meet at P. If 2AB = DE and BC = 8 cm, then what is EF equal
∠BPC = x ° and ∠BAC = y °, then which one of the to? e 2014 I
following option is correct? e 2013 II (a) 16 cm (b) 12 cm (c) 10 cm (d) 8 cm
(a) x° = y° (b) x° + y° = 90° 72. The sides of a right angled triangle are equal to
(c) x° + y° = 180° (d) 2x° = y° three consecutive numbers expressed in
62. The side AC of a ∆ABC is produced to D such centimeters. What can be the area of such a
that BC = CD. If ∠ACB is 70°, then what is triangle? e 2014 I
∠ADB equal to? e 2013 II (a) 6 cm 2 (b) 8 cm 2 (c) 10 cm 2 (d) 12 cm 2
(a) 35° (b) 45° (c) 70° (d) 110°
73. The three sides of a triangle are 15, 25, x units.
63. The heights of two trees are x and y, where x > y. Which one of the following is correct? e 2014 I
The tops of the trees are at a distance z a part. If (a) 10 < x < 40 (b) 10 ≤ x ≤ 40
s is the shortest distance between the trees, then (c) 10 ≤ x < 40 (d) 10 < x ≤ 40
what is s2 equal to? e 2013 II
74. In a ∆ABC, if ∠B = 2 , ∠C = 2∠A. Then, what is
(a) x2 + y2 − z2 − 2 x y (b) x2 + y2 − z2
the ratio of AC to AB ? e 2014 II
(c) x2 + y2 + z2 − 2 x y (d) z2 − x2 − y2 + 2 x y
(a) 2 : 1 (b) 3 : 1 (c) 1 : 1 (d) 1 : 2
64. In a ∆ABC, ∠B = 90° and ∠C = 2∠ A, then what 75. Three straight lines are drawn through the three
is AB 2 equal to? e 2013 II vertices of a ∆ABC, the line through each vertex
(a) 2 BC 2
(b) 3BC 2
(c) 4BC 2
(d) 5BC 2 being parallel to the opposite side. The ∆DEF is
bounded by these parallel lines. e 2014 II
65. PQR is an equilateral triangle. O is the point of Consider the following statements in respect of
intersection of altitudes PL, QM and RN . If the ∆DEF.
OP = 8 cm, then what is the perimeter of the
I. Each side of ∆DEF is double the side of ∆ABC to
∆PQR ? e 2013 II
which it is parallel.
(a) 8 3 cm (b) 12 3 cm (c) 16 3 cm (d) 24 3 cm
II. Area of ∆DEF is four times the area of ∆ABC.
66. ∆DEF is formed by joining the mid-points of Which of the statement(s) given above is/are correct?
the sides of ∆ABC. Similarly, a ∆PQR is formed (a) Only I (b) Only II
by joining the mid-points of the sides of the (c) Both I and II (d) Neither I nor II
∆DEF. If the sides of the ∆PQR are of lengths
1, 2 and 3 units, what is the perimeter of the 76. In a ∆ ABC, AD is the median through A and E
∆ABC ? e 2013 II is the mid-point of AD and BE produced meets
(a) 18 units (b) 24 units AC at F. Then, AF is equal to e 2014 II
(c) 48 units (d) Cannot be determined (a) AC/5 (b) AC/4 (c) AC/3 (d) AC/2

67. E is the mid-point of the median AD of a ∆ABC. 77. The point O is equidistant from the three sides
If BE produced meets the side AC at F, then CF of a ∆ABC.
is equal to e 2013 II Consider the following statements: e 2015 II

(a) AC /3 (b) 2 AC / 3 (c) AC /2 (d) None of these I. ∠OAC + ∠OCB + ∠OBA = 90°
II. ∠BOC = 2∠BAC
68. If AD is the internal angular of ∆ABC with III. The perpendiculars drawn from any point on OA
AB = 3 cm and AC = 1 cm, then what is BD : BC to AB and AC are always equal
equal to? e 2014 I
Which of the above statements are correct?
(a) 1 : 3 (b) 1 : 4 (c) 2 : 3 (d) 3 : 4
(a) I and II (b) II and III (c) I and III (d) All of these
69. In a ∆ABC, AD is perpendicular to BC and BE is 78. An equilateral ∆BOC is drawn inside a square
perpendicular to AC. Which of the following is ABCD. If ∠AOD = 2θ, what is tan θ equal to?
correct? e 2014 I e 2015 II
(a) CE × CB = CA × CD (b) CE × CA = CD × CB (a) 2 − 3 (b) 1 + 2 (c) 4 − 3 (d) 2 + 3
(c) AD × BD = AE × BE (d) AB × AC = AD × BE
79. In a ∆PQR, point X is on PQ and point Y is on
70. The sides of a triangle are in geometric PR such that XP = 1 . 5 units, XQ = 6 units,
progression with common ratio r < 1. If the PY = 2 units and YR = 8 units. Which of the
triangle is a right angled triangle, the square of following are correct?
common ratio is given by e 2014 I I. QR = 5XY II. QR is parallel to XY .
5+1 5 −1 3+1 3 −1 III. ∆PYX is similar to ∆PRQ.
(a) (b) (c) (d)
2 2 2 2
262 CDS Pathfinder

I. AD > A′ D′, BE > B′ E′ and CF > C′ F ′ are always


Select the correct answer using the codes given
true.
below. e 2016 I
(a) I and II (b) II and III AB2 + BC 2 + CA 2 A′ B′ 2 + B′ C′ 2 + C′ A′ 2
II. =
(c) I and III (d) All of these AD + BE + CF
2 2 2
A′ D′ 2 + B′ E′ 2 + C′ F ′ 2
B
80. ABC is a triangle right angled Which one of the following is correct in respect of
at C as shown below. Which the above statements?
one of the following is correct? (a) Both statement I and statement II are true and
e 2016 I Q statement II is the correct explanation of statement I
(a) AQ 2 + AB2 = BP 2 + PQ 2 (b) Both statement I and statement II are true but statement
(b) AQ 2 + PQ 2 = AB2 + BP 2 II is not the correct explanation of statement I
A (c) Statement I is true but statement II is false
(c) AQ 2 + BP 2 = AB2 + PQ 2 C
P (d) Statement I is false but statement II is true
(d) AQ + AP = BK + KQ
2 2 2 2
84. ABC and DEF are similar triangles. If the ratio
81. In the given figure, AD = CD = BC. What is the of side AB to side DE is ( 2 + 1) : 3, then the
value of ∠CDB? e 2016 I ratio of area of ∆ABC to that of ∆DEF is e 2016 I
C (a) (3 − 2 2 ) : 3 (b) (9 − 6 2 ) : 2
96°
(c) 1 : (9 − 6 2 ) (d) (3 + 2 2 ) : 2
85. Let ∆ABC and ∆DEF be such that ∠ABC = ∠DEF,
∠ACB = ∠DFE and ∠BAC = ∠EDF. Let L be the
A B mid-point of BC and M be the mid-point of EF.
D
(a) 32° (b) 64° Consider the following statements e 2016 I
(c) 78° I. ∆ABL and ∆DEM are similar.
(d) Cannot be determined due to insufficient data II. ∆ALC is congruent to ∆DMF even, if AC ≠ DF .
82. ABC is an equilateral triangle and X, Y and Z Which one of the following is correct in respect of
are the points on BC, CA and AB respectively, the above statements?
such that BX = CY = AZ . Which of the following (a) Both statement I and statement II are true and
is/are correct? e 2016 I statement II is the correct explanation of statement I
I. XYZ is an equilateral triangle. (b) Both statement I and statement II are true but
statement II is not the correct explanation of statement I
II. ∆XYZ is similar to ∆ABC.
(c) Statement I is true but statement II is false
Select the correct answer using the codes given (d) Statement I is false but statement II is true
below.
(a) Only I (b) Only II (c) Both I and II (d) Neither I nor II
86. ABC is a triangle in which D is the mid-point of
BC and E is the mid-point of AD.
83. Let ABC and A′ B′ C ′ be two triangles in which I. The area of ∆ABC is equal to four times the area
AB > A′ B′, BC > B′C ′ and CA > C ′ A′. Let D , E and of ∆BED.
F be the mid-points of the sides BC, CA and AB, II. The area of ∆ADC is twice the area of ∆BED.
respectively. Let D′, E′ and F ′ be the mid-points Select the correct answer using the codes given
of the sides B′ C ′, C ′ A′ and A′ B′, respectively. below. e 2016 I
Consider the following statements e 2016 I (a) Only I (b) Only II (c) Both I and II (d) Neither I nor II

ANSWERS
1 b 2 a 3 b 4 c 5 c 6 d 7 a 8 b 9 c 10 a
11 d 12 b 13 c 14 d 15 d 16 a 17 c 18 a 19 b 20 a
21 d 22 c 23 d 24 a 25 b 26 d 27 a 28 a 29 c 30 d
31 b 32 a 33 d 34 d 35 d 36 c 37 d 38 b 39 c 40 a
41 a 42 d 43 b 44 c 45 d 46 b 47 b 48 c 49 c 50 d
51 b 52 c 53 d 54 a 55 c 56 d 57 d 58 d 59 d 60 d
61 d 62 a 63 d 64 b 65 d 66 b 67 b 68 d 69 b 70 b
71 a 72 a 73 a 74 a 75 c 76 c 77 c 78 d 79 d 80 c
81 b 82 c 83 b 84 c 85 c 86 c
MATHEMATICS Triangles 263

HINTS AND SOLUTIONS 


2
1. (b) Let ABC is any triangle and AD is  12  7. (a) As ADB is a right angled triangle.
52 =   + p 2
the bisector of angle A.
 p So, AB 2 = AD 2 + BD 2
Also, BD = DC (given) ⇒ AC 2 = AD 2 + BD 2 [Q AB = AC ]
144
A ⇒ 25 = 2 + p 2 ⇒ ( AD + DC )2 = AD 2 + BD 2
p ⇒ AD 2 + DC 2 + 2 AD ⋅ DC
25 p 2 = 144 + p 4 = AD 2 + BD 2
⇒ p 4 − 25 p 2 + 144 = 0 ⇒ BD − CD = 2DC ⋅ AD
2 2

⇒ p − 16 p 2 − 9 p 2 + 144 = 0
4 8. (b) As in ∆ ADE and ∆ ABC ,
C AD 8 2 AE 6 2
= = and = =
B D
⇒ p ( p − 16) − 9 ( p − 16) = 0
2 2 2
Q AD is the internal bisector of ∠A. AB 20 5 AC 15 5
⇒ ( p 2 − 9) ( p 2 − 16) = 0
AB BD A
∴ = ⇒ p = 3 or p = 4
AC DC
AB BD Hence, other sides are 3 cm and 4 cm.
⇒ = = 1 [QBD = DC given] D E
AC DC 5. (c) As ∆ ABC ~ ∆ DEF
⇒ AB = AC AB AC BC
⇒ = = B C
Hence, triangle is an isosceles triangle. DE DF EF AD AE
A D ∴ =
2. (a) The line segments joining the AB AC
mid-points of the sides of a triangle 2.5 and ∠A = ∠A [common]
form four triangles each of which is 3
similar to the original triangle. ∴ ∆ ADE ~ ∆ ABC
[by SAS similarity]
A B 2 C E 4 F DE AD DE 2
BC 2 1 ∴ = ⇒ =
Q = = BC AB BC 5
EF 4 2 5
F E
⇒ BC = DE .
∴ DE = 2 AB = 2 × 3 = 6 cm 2
B D C and DF = 2 × AC = 2 × 2.5 = 5 cm 9. (c) Let AB be a vertical stick and AC be
Here, ∆ BDF ~ ∆ ABC ∴ Perimeter of ∆ DEF = ( 6 + 5 + 4) its shadow. Also, let PQ be a tower
having shadow PR .
Also, ∆ DEC , ∆ DEF = 15 cm
Sun Sun
∆ AFE ~ ∆ ABC Shortcut Method B Q
3. (b) As BD = EC , Perimeter of ∆ABC
15 m

Perimeter of ∆DEF
∠AEC = ∠BDA = 90°each x
= Ratio of corresponding sides
[Q BD⊥AC and CE⊥AB]
(3 + 2 + 2.5) 1
A ∴ = A C P R
Perimeter of ∆DEF 2 12 m 50 m
E ∴Perimeter of ∆DEF = 2 ( 75
. ) = 15 cm. ∠A = ∠P
D
∠B = ∠Q
6. (d) Let P be the starting point of his run, [Q Sun with tower and stick forms
then PT is the distance between the same angle]
B C starting and the finishing point. As, ∆ ABC ~ ∆ PQR
Also, ∠A = ∠A [common] 80 m [ ∴ By AA similarity]
∴ ~ ∆ AEC
∆BDA =
R S AB AC 15 12
[by AAS congruency] U
12 m N ∴ = ⇒ =
T PQ PR x 50
⇒ AB = AC [by cpct] 60 m 15 × 50
12 m W E ⇒ x= = 62.5 m
So, triangle is an isosceles triangle. 12
4. (c) Let the other side be b and p. Q 25 m P Hence, the height of the tower is 62.5 m.
S
1 10. (a) Let the ratio of their corresponding
∴ b × p = 6 ⇒ b × p = 12
2 ∴ PU = RQ − ST = 60 − 12 = 48 m height be h : h .
1 2
12 UT = RS − QP = 80 − 25 = 55 m But the ratio of the areas of two similar
⇒ b= and
p triangles is equal to the ratio of the
∴ In ∆ PUT, PT 2
= ( PU )2 + ( TU )2
squares of their corresponding heights.
Also, by pythagoras theorem ∴PT = ( 48)2 + (55)2 = 2304 + 3025 h 2 81
h2 = b 2 + p2 ∴ 1 = ⇒ h : h =9: 7
= 5329 = 73 m h 2 49 1 2
2
264 CDS Pathfinder

11. (d) As DE || BC , by basic proportionality A So, angles are 2x = 40°,


theorem, 3x = 60°, 4x = 80°.
A F E
20. (a) In ∆ABC , ∠A + ∠B + y = 180°
x x+2 y = 180° − ( 40 + 30)° = 110°
D E
B D C ∠ADE = ∠ABC [corresponding angle]
1 1
x–1 ∴ DE = AB = × 10 = 5 cm x = 30°
x–2 2 2
1 Similarly, y = z = 110°
EF = BC ⇒ BC = 2EF
B C 2 21. (d) In ∆GEF and ∆GCD, we have
AD AE x x+ 2 = 2 × 3 = 6 cm ∠EFG = ∠GDC [alternate angles]
= ⇒ = 1
DB EC x−2 x−1 DF = AC ⇒ AC = 2 × DF ∠EGF = ∠CGD
2
[vertically opposite angles]
⇒ x ( x − 1) = ( x + 2) ( x − 2) = 2 × 4 = 8 cm
∆GEF ~ ∆GCD [by AA similarity]
⇒ x 2 − x = x 2 − 2x + 2x − 4 15. (d) In ∆PQR, ∠ P + ∠ Q + ∠ R = 180° GE EF
⇒ a + 3a + b = 180° ∴ =
⇒ x=4 CG DC
Hence, the value of x is 4. ⇒ 4a + b = 180° …(i) 5 EF
Given, − 5a + 3b = 30° …(ii) ⇒ =
12. (b) As AB || CD and the diagonals of a 10 18
Solving Eqs.(i) and (ii), we get a = 30° 5 × 18
trapezium divide each other and b = 60° ⇒ EF = = 9 cm
proportionally. 10
∴ ∠ P = 30° , ∠ Q = 90° and ∠ R = 60°
AO BO 22. (c) Given, In ∆PQR, PQ > PR
So, = So, ∆ PQR is right angled triangle.
OC OD
16. (a) Consider ∆BMD and ∆DLC , ⇒ ∠PRQ > ∠PQR
3x − 1 2x + 1 [angle opp to larger side is greater]
⇒ = As ∠ BMD = ∠ DLC = 90° [each]
5x − 3 6x − 5 Also, ∠ BDM = ∠ DCL corresponding ⇒
1 1
∠PRQ > ∠PQR
angle 2 2
(3x − 1) ( 6x − 5) = (5x − 3) ( 2x + 1) ⇒ ∠SRQ > ∠SQR ⇒ SQ > SR
∴∆BMD ~ ∆DLC [by AA similarity]
⇒ 18x2 − 15x − 6x + 5 BD DM BM [side opp to greater angle is larger]
∴ = =
= 10x2 + 5x − 6x − 3 DC LC DL 23. (d) Given, AB = AC
BD DM 1
⇒ 8x − 20x + 8 = 0
2
⇒ = = A
DC LC 3
⇒ 4x2 − 10x + 4 = 0
∴ DM : LC = 1 : 3
⇒ 2( 2x2 − 5x + 2) = 0
17. (c) In ∆ABC , by pythagoras theorem, c b
⇒ 2( 2x2 − 4x − x + 2) = 0 A
⇒ 2[ 2x( x − 2) − 1( x − 2) = 0
⇒ 2( 2x − 1)( x − 2) = 0 P B a C
1 ⇒ ∠ABC = ∠ACB
But as x = will make OC negative.
2 If ∠A = 60°, then
B Q C
∴ x= 2 AC 2 = AB 2 + BC 2 …(i) ∆ABC is an equilateral triangle a = b = c
13. (c) Exterior angle bisector theorem The And in ∆PBQ, If ∠A = 90°, then
exterior bisector of an angle of a triangle PQ 2 = PB 2 + BQ 2 …(ii) ∆ABC is right angled triangle
divides the opposite side externally in Adding Eqs. (i) and (ii), a 2 = b 2 + c 2 = c 2 + c 2 = 2c 2 [Q b = c ]
the ratio of the sides containing the AC 2 + PQ 2 = ( AB 2 + BC 2 )
angle. ⇒a = 2c
+ PB 2 + BQ 2
BE AB ∴ If 60°< A < 90°, then c < a < c 2
Q = as AE is an exterior angle = ( AB + BQ ) + ( PB 2 + BC 2 )
2 2
CE AC AC + PQ 2 = AQ 2 + CP 2
2 24. (a) Here, in ∆ABC
bisector. AB 2 = AD 2 + BD 2 …(i)
18. (a) In ∆BCA , ∠CAB = 65° and
Let CE = x, BE = BC + EC = 12 + x In right angled ∆ADC , we have
∠ACB = 90° [Q ∠C = 90° given]
12 + x 10 AC 2 = AD 2 + CD 2 …(ii)
⇒ = ∠CBA = 180° − ( ∠BCA + ∠CAB )
x 6 Subtracting Eq. (ii) from Eq. (i),
[by angle sum property of a triangle]
⇒ ( 12 + x) 6 = 10x = 180° − ( 65° + 90° ) AB 2 − AC 2 = BD 2 − CD 2
A
⇒ 72 + 6x = 10x ⇒ 4x = 72 = 180° − 155° = 25°
⇒ x = 18 cm 19. (b) Let the angles of a triangle be
14. (d) As the line joining the mid-points of 2x, 3x, 4x, then
any two sides or a triangle is parallel to 2x + 3x + 4x = 180°
the third side and is half of the third [by angle sum property of a triangle] B D C
side. 9x = 180° ⇒ x = 20° AB 2 − BD 2 = AC 2 − CD 2
MATHEMATICS Triangles 265

25. (b) Since, c is the base and p is the 7 3  7 33. (d) Q ∠ABC = 180°−∠DBA = 180°−2x
 z  + z = 180° ⇒ z + z = 180°
altitude of ∆ABC . 3 8  8 B C
D E
C Q y = 3 z  2x 120°
 8 
b a
p 15z
= 180° ⇒ z = 96° x
8
3
A D c
B So, y = × 96° = 36° A
8 and ∠ACB = 180° − ∠ACE
1 4
Here, area of ∆ABC = pc …(i) and x = × 36° = 48° = 180° − 120° = 60°
2 3
In ∆ABC ,∠ABC + ∠ACB + ∠BAC = 180°
1
Also, area of ∆ABC = ab …(ii) 29. (c) We have, x = 35° [alternate angles] ⇒ 180°−2x + 60°+ x = 180° ⇒ x = 60°
2
z° = 75° [alternate angles]
From Eqs. (i) and (ii), we get 34. (d) In ∆PQY , by pythagoras theorem,
In ∆ABP, x + y + 75° = 180°
1 1
pc = ab ⇒ pc = ab PY 2 = PQ 2 + QY 2
2 2 y = 180°−( 75°+35° ) ⇒ y = 70°
P
26. (d) Hence, AD 2 = BD ⋅ DC Hence the value of
C x = 35° , y = 70° ,z = 75° X

D 30. (d) x° = 60° [vertically opposite angles]


Q R
y = 60° [corresponding angles] Y
90° 2
∠PRS = 110°
⇒ PY 2 = PQ 2 + 
A [alternate angles] QR 

B
AD DC ∠QRS + x° = 110° [Alternate angles]  2 
⇒ =
BD AD ∠QRS = 110° − 60° = 50° QY = YR = QR 
∴ ∆ADB ~ ∆CDA ⇒ ∠BAD = ∠ACD ∴ t = 180° − ( y + ∠QRS )  2  …(i)
Now, ∠CAB = ∠CAD + ∠BAD = 180° − ( 60°+50° ) and in ∆ XQR, RX 2 = QX 2 + QR2
= ∠CAD + ∠ACD = 90° t = 70° 2
⇒ RX 2 = 
PQ 
 + QR
2
So, ∆ABC must be right angled triangle. Also, t =z [alternate angles]  2 
Hence, BC 2 = AC 2 + AB 2 ∴ z = 70°
Q P X = XQ = PQ  …(ii)
27. (a) In ∆ABC , 31. (b) Since, ∠PTB = 55°
 2 
[given]
C Then, ∠TUV = 55°
[corresponding angle] On adding Eqs. (i) and (ii), we get
D
Also, ∠TUV = ∠CUQ = 55° 5PQ 2 5QR2
PY 2 + RX 2 = +
[vertically opposite angle] 4 4
A b
B Also given, ∠DVS = 45° ⇒ PY 2 + RX 2 = 5 / 4( PQ 2 + QR2 )
1 Then, ∠UVT = 45°
Area of ∆ABC = × Base × Altitude ⇒ 4 ( PY 2 + RX 2 ) = 5( PR2 )
[vertically opposite angle]
2
In ∆UTV , In ∆XQR, by pythagoras theorem,
1 2∆
∆ = b × AC , AC = ∠T = 180° − (55° + 45° ) = 80° XR2 = XQ 2 + QR2 …(iii)
2 b
[angle sum property of a triangle]
In ∆ABC , Using pythagoras theorem, and In ∆PQY , by pythagoras theorem,
⇒ ∠T = ∠PTR = 80°
AC 2 + AB 2 = BC 2 PY 2 = PQ 2 + QY 2 …(iv)
[vertically opposite angle]
4∆ 2
∴ ∠CUQ + ∠RTP = 55° + 80° = 135° On adding Eqs. (iii) and (iv),
⇒ BC = + b2
b2 32. (a) XR2 + PY 2 = XQ 2 + QR2
Again in ∆ ABC , area of ∆ABC A a 4
B
1
3c + PQ 2 + QY 2
∆ = × BC × AD 5
⇒ XR + PY = XQ + QY 2
C 1 2 2 2
2
2∆ 2 ∆b
b
⇒ AD = = 2 + QR2 + PQ 2
4∆ + b
2 4
4∆2 + b 4 6
2
x ⇒ XR + PY 2 2
= XY + PR2
2
b
D and again,
28. (a) As, AB||CD and BC cuts them.
∴ ∠BCD = ∠ABC = x [alternate ∠S] ∠1 = ∠3 + ∠4, ∠2 = ∠5 + ∠6 XR2 + PY 2 = XY 2 + PR2
∴ In ∆BCD, x + y + z = 180° ⇒ ∠1 + ∠2 = ∠3 + ∠4 + ∠5 + ∠6 = XY 2 + ( 2XY )2
4
y + y + z = 180°
[since, exterior angle is equal to sum XR + PY
2 2
= XY 2 + 4XY 2
3 of two opposite interior angles]
⇒ XR2 + PY 2 = 5XY 2
7y ⇒ b = c+ a+ x
⇒ + z = 180° Thus, option (d) is incorrect.
3 ∴ x= b−c−a
266 CDS Pathfinder

35. (d) We produced line AB to meet line Also, XY + XP + YQ = 40 [given] x BD


Then, = …(i)
CE at M. ⇒ AX + XB + YQ = 40 z QD
C x D [Q XY = AX , XP = XB] Since CD ||PQ
⇒ AB + YQ = 40 So, ∆ BCD ~ ∆ BPQ,
z BQ
B ⇒ YQ = 40 − 30 = 10 cm ∴ =
A M y BD
y E ∴ YQ = XP = 10 cm z BD − QD
∴ BP = CQ = 10 cm ⇒ =
Since, AM || CD. y BD
∴ PQ = 30 − BP − CQ z QD
∠DCM = ∠BMC = x ⇒ = 1−
= 30 − 10 − 10 = 10 cm y BD
[alternate angles] z z
38. (b) As, OP || YZ ⇒ = 1−
CME is a straight line [from Eq. (i)]
⇒∠POY = ∠OYZ [alternate angles] y x
∴ ∠BME = π − x ⇒ ∠PYO = ∠POY z z 1 1 1
⇒ + =1 ⇒ + =
Also, ABM is a straight line. [since OY is angle bisector of ∠Y ] x y x y z
∴ ∠EBM = π − y ∴ PY = PO
42. (d) In right angle ∆ PQR , by pythagoras
Now, in ∆BEM As ∠XYZ is an acute angle.
1 theorem,
∠EBM + ∠BME + ∠BEM = π ∴ ∠ XYZ < 45° R
2
⇒ π − y + π − x + ∠E = π ∴ ∠POY = ∠PYO < 45°
⇒ ∠E = x + y − π 4 cm 5 cm
∴ ∠YPO > 90°
36. (c) ∴ ∠A = 360° − Ext ∠A Hence, ∆PYO is an isosceles triangle but
not a right angled triangle. Q P
A
39. (c) Here, we see that C QP 2 = (5)2 − ( 4)2 = 9 ⇒ QP = 3 cm
CD = BD = DA In second ∆ABC whose sides are 3 cm,
This is possible D 4 cm and 5 cm. So, the sides of both
only when ABC is triangle are same, hence they are
right angled congruent.
B C triangle. B A
43. (b) Given, AC = BC
Similarly, ∠B = 360° − Ext ∠B and 40. (a) In ∆DCX ,CD = CX [given] ∴ ∠ABC = ∠BAC
∠C = 360° − Ext ∠C ∠3 = ∠ 4 [angles of an isosceles triangle]
[opposite angle of same sides]
We know that, the sum of all the angles ⇒ ∠BAC = ∠ABC = 38°
of a triangle is 180°. But ∠ 3 = ∠ 5, So, ∠ 4 = ∠ 5
In ∆ABC ,
In ∆ ABD and ∆ ACX ,
∴ ∠A + ∠B + ∠C = 180° ∠ACB = 180° − ( ∠BAC + ∠ABC )
∠1 = ∠2 [given]
⇒ 360° − Ext ∠A + 360° − Ext ∠B [by angle sum property of a triangle]
A
+ 360° − Ext ∠C = 180° = 180° − (38° + 38° )
⇒ Ext ∠A + Ext ∠B + Ext ∠C 12
= 180° − 76° = 104°
= 1080° − 180° = 900°
In ∆ACD, ∠ACD = 180° − 104° = 76°
37. (d) Since, XP || AC and YQ || AB [by linear pair]
A and ∠CAD = ∠ACD = 76°
5
B
D
C [Q CD = AD]
3
∴ ∠ADC = 180° − ( ∠CAD + ∠ACD )
4 = 180° − ( 76° + 76° ) = 28°
X Y X 44. (c)I. It is true that the three medians of a
∠4 = ∠5 triangle divide it into six triangles of
B C equal area.
P Q ∴ ∆ABD ~ ∆ACX
[by AA similarity] II. It is also true that, the perimeter of a
∴ ∠XBP = ∠YQC and ∠XPB = ∠YCQ

You might also like